Study Package Ntse (Mat+Sat)

You might also like

Download as pdf or txt
Download as pdf or txt
You are on page 1of 1024

STUDY PACKAGE

N S
National Talent Search Examination

5 PRACTICE SETS &


SOLVED PAPERS 2021- 2017

FOR
CLASS
10

ARIHANT PUBLICATIONS (INDIA) LIMITED


ARIHANT PUBLICATIONS (INDIA) LIMITED
All Rights Reserved

© Publisher
No part of this publication may be re-produced, stored in a retrieval system or by any means,
electronic, mechanical, photocopying, recording, scanning, web or otherwise without the written
permission of the publisher. Arihant has obtained all the information in this book from the sources
believed to be reliable and true. However, Arihant or its editors or authors or illustrators don’t take
any responsibility for the absolute accuracy of any information published and the damage or loss
suffered thereupon.
All disputes subject to Meerut (UP) jurisdiction only.

Administrative & Production Offices


Regd. Office
‘Ramchhaya’ 4577/15, Agarwal Road, Darya Ganj, New Delhi -110002
Tele: 011- 47630600, 43518550

Head Office
Kalindi, TP Nagar, Meerut (UP) - 250002
Tel: 0121-7156203, 7156204

Sales & Support Offices


Agra, Ahmedabad, Bengaluru, Bareilly, Chennai, Delhi, Guwahati,
Hyderabad, Jaipur, Jhansi, Kolkata, Lucknow, Nagpur & Pune.

ISBN 978-93-25791-94-7

PO No : TXT-XX-XXXXXXX-X-XX
Published by Arihant Publications (India) Ltd.
For further information about the books published by Arihant, log on to
www.arihantbooks.com or e-mail at info@arihantbooks.com
Follow us on
ABOUT NTSE
National Talent Search Examination Scheme is a flagship activity of the NCERT started in the year
1963. The purpose of the scheme is to identify talented students and nurture their talent. It honours
and helps talented students by providing financial assistance in the form of a monthly scholarship.

PRESENT SCHEME
The scholarship under the present scheme awarded to the candidates for pursuing courses
in Science and Social science up to doctoral level and in professional courses like medicine
and engineering up to second-degree level subject to the fulfillment of the conditions
provided in this brochure. From the year 2000, scholarships have been awarded in the
country with reservation of 15 per cent for SC, 7.5 per cent for ST and 27 per cent for Other
Backward Classes and 4 per cent for group of students with benchmark disabilities.

RATES OF SCHOLARSHIP
The rates of scholarship at different stages are given as under.

S.No Stage Rate of Scholarship

1. Higher Secondary Level ` 1250/- p.m.

2. Graduate and Post Graduate ` 2000/- p.m.

3. For Ph.D. degree (four years) As per UGC norms

ELIGIBILITY
The scheme is open to students of Indian Nationality only. Students studying in class X are eligible to
appear for the selection process.

SELECTION PROCESS
There is a two-stage selection process for the award of scholarship.
STAGE I
Examination at the State/Union Territory (UT) level will comprise
(a) Mental Ability Test (MAT) 100 questions covering Reasoning & Numerical ability.
(b) Scholastic Aptitude Test (SAT) 100 questions covering Social Science, (40 questions),
Science (40 questions) and Mathematics (20 questions).
For Stage I, selections are done by States/UTs through the written examination.
STAGE II
Examination at the National level will comprise
(a) Mental Ability Test (MAT) 100 questions covering Reasoning and Numerical ability.
(b) Scholastic Aptitude Test (SAT) 100 questions covering Social Science (40 questions),
Science (40 questions) and Mathematics (20 questions).
For Stage II examination, each State/Union Territory has been allotted a quota to
recommend a specific number of candidates. Students who qualify Stage I will be eligible to
appear for Stage II examination conducted by NCERT.

PATTERN OF NTSE
STAGE I (State/UT Level)
Sections Duration No. of Questions Max. Marks
Mental Ability Test (MAT) 120 min 100 100
Scholastic Aptitude Test (SAT) 120 min 100 100

STAGE II (National Level)


Sections Duration No. of Questions Max. Marks
Mental Ability Test (MAT) 120 min 100 100
Scholastic Aptitude Test (SAT) 120 min 100 100

MEDIUM OF EXAM
The tests will be available in the following languages : Asamiya, Bangla, English, Gujarati,
Hindi, Kannada, Marathi, Malayalam, Odia, Punjabi, Tamil, Telugu and Urdu. The candidate
has to mention his option regarding the language in which he/she want to take the test in
the application form. Accordingly, the question booklet in that language shall be made
available to the candidate at the centre. After exercising this option, no request for the
change of medium will be entertained.

Note
Ÿ Only candidates qualifying in both the papers (40% for General & OBC ; 32% for SC, STs, PWD)
separately will be considered for merit.
Ÿ There will be no negative marking for incorrect questions.
Ÿ After state level examination, candidate is selected for national level examination.
Ÿ Selection of the awardees will be made on the basis of total marks scored in MAT & SAT.
Subject-wise
STRATEGY OF NTSE
To achieve success in NTSE, a student must have a good idea of the level
of the examination and achieve mastery in all the topics required.

1 REASONING & NUMERICAL ABILITY


Ÿ This section contains 100 questions at both Stage I & Stage II.
Ÿ Important topics which are asked under this section are Analogy, Classification,
Number Series, Insert the Missing Character, Coding-Decoding, Cubes and Dices,
Completion of Figures, Venn-diagram, Mirror and Water images, Basic arithmetic
operation, Arithmetical reasoning, Data interpretation and sufficiency etc.
Ÿ Practice a good number of questions and try to clarify the basics of the topic, attempt the
question firstly which seems to be easy for a candidate as per his/her understanding.

2 GENERAL SCIENCE
Ÿ This section comprises 40 questions at Stage I & II under SAT. In this section three subjects Physics,
Chemistry and Biology are equally distributed.
Ÿ Important topics covered under this section are Light, Motion, Force & Pressure, Work Energy &
Power, Organic & Inorganic Chemistry, Periodic Table & Its Classification, Atomic Structure,
Fundamental Unit of Life, Heredity and Evolution, Life Processes etc.
Ÿ ‘The best strategy for this section is to understand the basic concept of all topics, Try to learn the
periodic table, formulae of different Organic-Inorganic Compounds and their uses, Structure of cells
& tissue, Scientific names of Vitamin & Minerals and disease caused by their deficiency.

3 SOCIAL SCIENCE
Ÿ This section contains 40 questions at both stage I and stage II from History, Geography, Civics
& Economy are asked in this section.
Ÿ Topics covered under this section are Ancient India, Medieval India and World Geography,
Constitution of India, Judiciary and Legislature and concepts related to Indian Economy etc.
Ÿ Read all the topics carefully from class IX and X and try to remember the important dates &
facts that can be the only special tip for this section.

4 MATHEMATICS
This section contains 20 questions at Stage I and Stage II both.
Topics covered under this section are Number System, HCF & LCM, Trigonometry,
Height & Distance, Commercial Mathematics and Co-ordinate Geometry, Algebra etc.
For this section one has to learn the basic formulae and concept of every topic. One can
save his/her time by using short tricks while attempting the question.
CONTENTS
Solved Paper 2021 (Stage - I) 1-28
Solved Paper 2020 (Stage - I) 1-26
Solved Paper 2019 (Stage - II) 1-40
Solved Paper 2018 (Stage - I) 1-27
Solved Paper 2018 (Stage - II) 1-36
Solved Paper 2017 (Stage - I) 1-22

PAPER I MAT (Mental Ability Test)


PART I Verbal Reasoning 18. Syllogism 119-130
19. Statement and Conclusion 131-133
1. Coding-Decoding 3-12
20. Data Sufficiency 134-140
2. Classification 13-17
3. Analogy 18-26
PART II Non-Verbal Reasoning
4. Alphabet Test 27-33 1. Series 141-148
5. Series 34-41 2. Analogy 149-157
6. Complex Sequence 42-45 3. Classification (Odd One Out) 158-161
7. Logical Sequence of Words 46-50 4. Matrix 162-167
8. Blood Relations 51-58 5. Completion of Figures 168-172
9. Direction Sense Test 59-67 6. Spotting Out the Embedded Figure 173-177
10. Inserting the Missing Character 68-75 7. Mirror and Water Images 178-181
11. Ranking Test 76-80 8. Paper Folding 182-185
12. Arithmetical Reasoning 81-88 9. Paper Cutting 186-190
13. Mathematical Operations 89-93 10. Counting of Figures 191-198
14. Sitting Arrangement and Puzzles 94-100 11. Cubes and Dice 199-205
15. Problems Based on Calendar 101-105 12. Dots Situation 206-207
16. Problems Based on Clock 106-111 13. Grouping of Identical Figures 208-211

17. Venn-Diagrams 112-118 14. Formation of Figures 212-216


PAPER II SAT (Scholastic Aptitude Test)
PART I Physics 5. How do Organism Reproduce 350-353
6. Heredity and Evolution 354-356
1. Motion 219-226
7. Why do We Fall ill 357-360
2. Force and Laws of Motion 227-231
8. Diversity in Living Organism 361-366
3. Work, Energy and Power 232-236
9. Our Environment 367-370
4. Gravitation 237-240
10. Natural Resources 371-374
5. Pressure and Archimede’s Principle 241-244
11. Management of Natural Resources 375-377
6. Sound 245-249
12. Improvement in Food Resources 378-381
7. Heat 250-255
8. Light : Reflection, Refraction and
Human Eye 256-266 PART IV Mathematics
9. Electricity 267-272 1. Number System 382-389
10. Magnetic Effects of Electric Current 273-278 2. Basic Operations on Arithmetic 390-404
11. Sources of Energy 279-284 3. Algebra 405-411
4. Linear Equations 412-417
PART II Chemistry 5. Quadratic Equation 418-422
1. Is Matter Around us Pure 285-289 6. Arithmetic Progression 423-427
2. Atomic Structure 290-294 7. Introduction to Euclid's Geometry 428-431
3. Chemical Reactions and Equations 295-298 8. Lines and Angles 432-439
4. Acids, Bases and Salts 299-303 9. Triangles 440-449
5. Metals and Non-Metals 304-309 10. Quadrilaterals 450-457
6. Carbon and its Compounds 310-316 11. Circles 458-468
7. Periodic Classification of Elements 317-322 12. Coordinate Geometry 469-477
8. Atoms and Molecules 323-326 13. Area and Perimeter 478-487
9. Matter in Our Surroundings 327-329 14. Volume and Surface Area 488-494
15. Trigonometry 495-501
PART III Biology 16. Height and Distance 502-509
1. Fundamental Unit of Life 330-335 17. Statistics 510-517
2. Tissues 336-340 18. Permutations and Combinations 518-522
3. Life Processes 341-345 19. Probability 523-527
4. Control and Coordination 346-349
PART V History PART VII Civics
1. Ancient India 528-537 1. Framing of Indian Constitution and
its Basic Features 648-654
2. Medieval India 538-545
2. Working of Institutions Organs
3. Events and Process 546-568 of Government 655-660
4. Livelihoods, Economics and 3. Democracy and Related Phenomena 661-665
Societies 569-583 4. Electoral Process in India and
5. Everyday Life, Culture and Politics 584-597 Political Parties 666-670
5. Power Sharing and Federalism 671-674
PART VI Geography
1. India : Size and Location 598-600
PART VIII Economics
1. Agriculture and Allied Activities 675-677
2. Physical Features of India 601-606
2. People as Resource 678-680
3. Climate 607-612 3. Poverty as a Challenge 681-684
4. Drainage 613-616 4. Food Security in India 685-687
5. Resources and Development 617-623 5. Development 688-690
6. Agriculture 624-629 6. Sectors of the Indian Economy 691-694
7. Manufacturing Industries 630-635 7. Money and Credit 695-697

8. Minerals and Energy Resources 636-640 8. Globalisation and the Indian Economy 698-700
9. Consumer Rights 701-704
9. Population 641-643
10. Transport—Lifelines of National General Knowledge 705-724
Economy 644-647 Practice Sets (1-5) 3-111
NTSE ~ SOLVED PAPER 2021 (Stage I) 1

NTSE Solved Paper


NATIONAL TALENT
SEARCH
EXAMINATION
2021 (Stage I)

INSTRUCTIONS
This solved paper consists of two papers. Paper I consists of Mental Ability Test (MAT) and Paper II consists of
Scholastic Aptitude Test (SAT).
MAT covering (Q. Nos. 1-50) of Maths and (Q. Nos. 51-100) of Reasoning.
SAT consists of (Science, Mathematics and Social Science) which comprises 100 questions (40 Science, 20 Mathematics
and 40 Social Science).
There will be no negative marking.
Each correct answer will be awarded one mark.

Time : 240 Minutes Max. Mark : 200

Paper I Mental Ability Test (MAT)


é(62 + 72 + 82 + 92 + 102 ) ù 12
1. ê ú is equal to Sum of 12 terms (S12 ) = [2(7) + (12 - 1)3]
ë (7 + 2 3 ) - (5 + 2 3 ) û 2
n
(a) 160 (b) 165 (c) 167 (d) 161 S12 = 6(14 + 33) [QSn = [2 a + (n - 1)d]
2
(62 + 72 + 82 + 92 + 102 ) 36 + 49 + 64 + 81 + 100 S12 = 282
l (b) =
(7 + 2 3 ) - (5 + 2 3 ) 7 + 2 3 -5 -2 3
330 4. Two goods train each 500 m long, are running in
= = 165 opposite directions on parallel tracks. Their speeds
2
are 45 km/h and 30 km/h respectively. Find the
2. If 20% of x = y , then y % of 20 is the same as time taken by the slower train to pass the driver of
(a) 4% of x (b) 6% of y the faster one.
(c) 8% of x (d) 10% of x (a) 12 s (b) 24 s
l (a) 20% of x = y (c) 48 s (d) 60 s
20 (c) Relative speed of both the trains
Þ ´x=y …(i) l
100 = 45 + 30 = 75 km/h
y 20 20
Now, y% of 20 = ´ 20 = ´ ´x [from (i)] Total distance covered = 500 m + 500 m = 1 km
100 100 100
We know that,
4x
= = 4% of x Distance
100 Time =
Speed
3. The sum of the first 12 terms of an AP, whose nth \ Time taken by slower train to pass the driver of faster
term is given by an = 3 n + 4, is 1 1
one = h= ´ 3600 = 48 s
(a) 262 (b) 272 (c) 282 (d) 292 75 75
l (c) General term of AP, an = 3 n + 4
5. The next term of the AP 18 , 50 , 98 ...... is
Now, if we put n = 1, 2, 3, …… we get
an AP which is 7, 10, 13, … (a) 146 (b) 128
a = 7, d = 3 and n = 12 (c) 162 (d) 200
2 NTSE ~ SOLVED PAPER 2021 (Stage I)

l (c) Given AP is 18 , 50 , 98 . l (b)


Þ 3 2, 5 2, 7 2 A
a = 3 2, d = 2 2
\ The next term = 7 2 + d
= 7 2 + 2 2 = 9 2 or 162
h
6. If ( p + q) th term of an AP is m and ( p - q) th term is
n, then pth term is
(a) mn (b) mn θ
(m - n) (m + n) B C
(c) (d) AB is a tower with height h m,
2 2
\ AB = BC = h m (given)
l (d) We know that, in an AP,
Now, let q be the angle of elevation
an = a + (n - 1)d
In right DABC,
Therefore, ap + q = a + (p + q - 1)d = m …(i) AB éQ tan q = Perpendicular ù
ap - q = a + (p - q - 1)d = n …(ii) tanq = êë úû
BC Base
Adding Eqs. (i) and (ii), we get h
m + n = 2 a + (2 p - 2)d tanq =
h
m + n = 2 [a + (p - 1)d] tanq = 1 [Qtan 45° = 1]
m+ n
= a + (p - 1)d \ q = 45°
2
m+ n 9. There are 1400 students in a school, 25% of those
= ap
2 2
wear spectacles and of those wearing spectacles
m+ n 7
\ pth term of AP is .
2 are boys. How many girls in the school wear
spectacles?
7. If a flight of 600 km, an aircraft was slowed down
due to bad weather. Its average speed for the trip (a) 300 (b) 100
was reduced by 200 km/h and the time of flight (c) 200 (d) 250
increased by 30 min. The original duration of the l (d) Total number of students = 1400
flight is Number of students wearing spectacles =
25
´ 1400 = 350
(a) 1 h (b) 2 h 100
Number of girls in the class = æç1 - ö÷ ´ 350
(c) 3 h (d) 4 h 2
è 7ø
l (a) Let the original duration of the flight is ‘t’ hour.
5
Total distance = 600 km = ´ 350 = 250
7
Now, we know that,
Distance 10. P, Q and R jointly thought of engaging themselves
Speed =
Time in a business venture. It was agreed that P would
Then, according to question, invest ` 6500 for 6 months, Q, ` 8400 for 5 months
600
-
600
= 200 Þ
1
-
2
=
1 and R, ` 10000 for 3 months. P wants to be the
t 1 t 2 t + 1 3 working member for which, he was to receive 5% of
t +
2 the profits. The profit earned was ` 7400. Calculate
2t + 1 - 2t 1 1 1 the share of Q in the profit.
= Þ =
t (2t + 1) 3 2t 2 + t 3 (a) ` 1900 (b) ` 2660
2t 2 + t - 3 = 0 Þ 2t 2 + 3t - 2t - 3 = 0 (c) ` 2800 (d) ` 2840
t (2t + 3) - 1(2t + 3) = 0 Þ (2t + 3)(t - 1) = 0 l (b) Ratio of investments of P, Q and R
3
t = - (not possible), t = 1 P : Q : R = 6500 ´ 6 : 8400 ´ 5 : 10000 ´ 3 = 13 : 14 : 10
2
Total profit = ` 7400
\ Original duration of flight = 1 h 5
P’s profit = ´ 7400 = ` 370
8. A tower is observed from a point on the horizontal 100
through the foot of the tower. The distance of this Remaining profit = ` (7400 - 370) = ` 7030
point from the foot of the tower is equal to the Now, Q’s share in profit
height of the tower. The angle of elevation on the 14
= ´ 7030
top of the tower is 13 + 14 + 10
(a) 60° (b) 45° 14
= ´ 7030 = ` 2660
(c) 40° (d) 30° 37
NTSE ~ SOLVED PAPER 2021 (Stage I) 3
1
11. John cycling at a constant speed of 10 km/h, x+ = 10 …(i)
reaches his school in time. If he cycles at a constant x
speed of 15 km/h, he reaches his school in 12 min Now, we know that
3

x3 + 3 = æç x + ö÷ - 3 æç x + ö÷ = 103 - 3(10)
early. How many kilometers he has to cycle for his 1 1 1
school is? x è xø è xø
(a) 4 (b) 6 [from Eq. (i)]
(c) 9 (d) 12 = 1000 - 30 = 970
l (b) Original speed of John = 10 km/h 14. The correct arrangement of alphabetical order of
Let original distance = d km the words is
We know that, 1. music 2. monk
Distance
Time = 3. minimum 4. maximum
Speed
(a) 4, 1, 3, 2 (b) 1, 3, 4, 2
and, according to question,
d d 12 d d 1 (c) 4, 3, 1, 2 (d) 4, 3, 2, 1
- = Þ - =
10 15 60 10 15 5 l (d) The correct alphabetical order of the words
3d - 2d 1 Maximum ® Minimum ® Monk ® Music
=
30 5 (4) (3) (2) (1)
d = 6 km Þ 4, 3, 2, 1
\ Distance he cycles to school = 6 km
15. At present ages of a father and son are in the ratio
p
12. If 7 sin a = 24 cos a : 0 < a < , then the value of of 7 : 3 and they will be in the present ratio 2 : 1
2 after 10 yr. What is the present age of father?
14 tan a - 75 cos a - 7 sec a is equal to
(a) 70 yr (b) 65 yr
(a) 3 (b) 4
(c) 60 yr (d) 50 yr
(c) 1 (d) 2
l (a) Let the ages of father and son be 7x yr and 3x yr
l (d) Given, 7 sin a = 24 cos a
sin a 24 24 According to the question,
= Þ tana = …(i) 7 x + 10 2
cos a 7 7 =
3 x + 10 1
We know that,
2 7 x + 10 = 6 x + 20
sec2 a = 1 + tan2 a = 1 + æç ö÷
24
[from (i)] x = 10
è7 ø
576 \ Present age of father = 7x
sec2 a = 1 + = 7 ´ 10 = 70 yr
49
625
sec a =
2
Directions (Q. Nos. 16-19) Study the following table
49
carefully and answer the questions.
625 25
sec a = = …(ii)
49 7 Number of students in different classes of XYZ Primary school
7 Classes Years
Þ cos a = …(iii)
25 2014 2015 2016 2017 2018
Now, 14 tan a - 75 cos a - 7sec a
I 22 28 26 32 18
24 7 25
= 14 ´ - 75 ´ -7 ´ [from Eqs. (i), (ii), (iii)] II 20 26 22 24 16
7 25 7
= 2 ´ 24 - 7 ´ 3 - 25 III 39 33 52 17 29
= 48 - 21 - 25 = 48 - 46 = 2 IV 19 26 38 24 11
1 1 V 16 33 37 23 21
13. If x + 2 = 98( x > 0), then the value of x 3 + 3 is
2

x x
16. The difference between the sum of all the students
(a) 970 (b) 1030 of all the classes in 2017 and 2018 is
(c) - 970 (d) - 1030 (a) 112 (b) 85
l (a) Given, (c) 25 (d) 35
1 l (c) Sum of students in 2017
x + 2 = 98
2

x = 32 + 24 + 17 + 24 + 23 = 120
Adding 2 on both sides, Sum of students in 2018
1 = 18 + 16 + 29 + 11 + 21
x2 + 2 + 2 = 100
x = 95
2
æ x + 1 ö = (10)2 [Q (a + b)2 = a2 + b2 + 2 ab] \ Required difference = 120 - 95
ç ÷
è xø = 25
4 NTSE ~ SOLVED PAPER 2021 (Stage I)

17. The sum of all the students in class III in all the l (c) Here, each letter is coded as the twice of its
years is what percent of the sum of all students of alphabetical position
class I in all the years approximately? A =1 ´ 2 = 2 [QAlphabetical position A = 1]
(a) 30% (b) 100% T = 20 ´ 2 = 40 [QAlphabetical position T = 20]
As, ACT = 2 + 6 + 40 = 48 [QC = 3 ´ 2 = 6]
(c) 60% (d) 135%
Similarly,
l (d) Sum of students in class III
TAKE = 40 + 2 + 22 + 10 [QK = 2 ´ 11 = 22, E = 2 ´ 5 = 10]
= 39 + 33 + 52 + 17 + 29
\ TAKE = 74
= 170
Sum of students in class I Directions (Q. Nos. 23-27) Identify the wrong number /
= 22 + 28 + 26 + 32 + 18 letters in the series.
= 126
170 23. 126, 98, 70, 41, 14
\ Required percentage = ´ 100
126 (a) 98 (b) 70 (c) 126 (d) 41
» 135% l (d) 42
126 98 70 41 14
18. The square of sum of all the students of all classes
in year 2018 is –28 –28 –28 –28
(a) 2095 (b) 9025 \ 41 is the wrong term.
(c) 6059 (d) 9216
24. 1, 3, 7, 15, 31, 65, 127
l (b) Sum of students in 2018
(a) 7 (b) 31 (c) 15 (d) 65
= 18 + 16 + 29 + 11 + 21
= 95 l (d) 63
\ Required square = (95)2 = 9025 1 3 7 15 31 65 127

19. By what percent the number of students of Class I ×2+1 ×2+1 ×2+1 ×2+1 ×2+1 ×2+1
in year 2017 is more than the number of students of \ 65 is the wrong term.
Class II for the same year?
(a) 60 (b) 75 25. 7, 28, 63, 124, 215, 344
(c) 35 (d) 33.33 (a) 28 (b) 63 (c) 124 (d) 344
l (d) Number of students in class I in 2017 = 32 l (c) 126
Number of students in class II in 2017 = 24 7 28 63 124 215 344
32 - 24
\ Required percentage = ´ 100
24
23–1 33+1 43–1 53+1 63–1 73+1
8
= ´ 100 = 33 .33% \ 124 is the wrong term.
24
26. AD, EG, IJ, MM, QP, UP
Directions (Q. Nos. 20 and 21) First two terms are
(a) EG (b) UP
connected by some relationship. The same relationship is
applicable for the next terms. Identify the suitable pair. (c) QP (d) MM
l (b)
20. Square : Cube
+3 +3 +3 +3 +3
(a) Rectangle : Cuboid (b) Triangle : Square
(c) Quadrilateral : Cuboid (d) Cuboid : Rectangle A D E G I J M M Q P U P
l (a) As, cube is the 3-D form of a square similarly, cuboid is U S
+4 +4 +4 +4 +4
the 3-D form of a rectangle.
\ UP is the wrong term.
21. 82 : 9
27. Z8, W5, T2, Q8, N5, K3
(a) 5 : 26 (b) 6 : 37
(a) K3 (b) T2
(c) 35 : 6 (d) 26 : 5
(c) Q8 (d) Z8
l (d) As in (82 : 9), 92 + 1 = 82 K2
l (a)
Similarly in, (26 : 5) Z8 W5 T2 Q8 N5 K3
52 + 1 = 26

22. If A = 2 , T = 40 and ACT = 48, then TAKE is Z = 26 W = 23 T = 20 Q = 17 N = 14 K = 11


2+6 = 8 2+3 = 5 2+0 =2 1+7 = 8 1+4 = 5 1+1 = 2 ≠ 3
(a) 68 (b) 58
(c) 74 (d) 76 \ K3 is the wrong term.
NTSE ~ SOLVED PAPER 2021 (Stage I) 5
Directions (Q. Nos. 28 and 29) In the following questions l (c) As, M O N K E Y
first set of words are related in a certain way. You have to –1 –1 –1 –1 –1

choose a word. So that another set of words also become –1


related in same manner.
28. Transition : change : : Immobility : ? X D J M N L
(a) Stillness (b) Liveliness
Similarly, T I G E R
(c) Action (d) Busyness –1 –1
–1 –1
l (a) As, transition and change are synonyms to each other,
similarly, immobility and stillness are synonyms to each
other.
Q D F H S
29. Grain : Stock : : Stick : ?
(a) String (b) Collection \ T I G E R ÞQ D F H S
(c) Bundle (d) Heep 35. In the following matrix, certain numbers are
l (c) As, collection of grain is called stock, similarly, arranged in a certain way. Choose the missing
collection of sticks is called bundle. number to complete the matrix.
30. A is B’s sister, C is B’s mother, D is C’s father, E is 1 7 9
D’s mother. Then how is A related to D? 2 14 ?
(a) Grand father (b) Grand mother 3 105 117
(c) Daughter (d) Grand daughter
(a) 16 (b) 12
l (d) (c) 13 (d) 14
E(–) + ⇒ Male
l (b) As, in I column,
– ⇒ Female 1´2 +1=3
Mother
In II column,
D(+) 7 ´ 14 + 7 = 105
\ In III column,
Father 9 ´ 12 + 9 = 117
Grand-daughter
\ ? = 12
C(–)

Mother
36. What value will replace ‘?’ in the figures given
below?
A(–) B 42 36 38
Sister
From the above relation diagram, it is clear that A is grand
3 2 15 3 1 9 10 ? 20
daughter of E.

Directions (Q. Nos. 31-33) Find the odd one out. 6 9 19


(a) 0 (b) 1
31. (a) Crusade (b) Expedition
(c) 2 (d) 3
(c) Campaign (d) Cruise
l (a) As, in first circle,
l (d) Crusade, campaign and expedition is related to land
whereas cruise is related to water. (42 ¸ 6) - (15 ¸ 3) Þ 7 - 5 = 2
In second circle,
32. (a) Clove (b) Apricot (36 ¸ 9) - (9 ¸ 3) Þ 4 - 3 = 1
(c) Cinnamon (d) Pepper Similarly, in third circle,
l (b) Apricot is a dry fruit, whereas all other are spices. (38 ¸ 19) - (20 ¸ 10) Þ 2 - 2 = 0

33. (a) Acre (b) Yard 37. Five plays K, L, M, N and O are to be staged from
(c) Mile (d) Meter Monday to Friday of a week. On each day only one
play will be staged. O should be immediately
l (a) Mile, Yard and Meter are units for measuring distance,
whereas Acre is used for measurement of area of land. followed by M. L should be staged immediately
after N. One play is staged between K and L. N or
34. In a certain code MONKEY is written as XDJMNL. O should not be the first or last play. Which is the
How is TIGER written in the same code? second play to be staged?
(a) SHFDQ (b) HFDSQ (a) M (b) O
(c) QDFHS (d) PQRST (c) N (d) K
6 NTSE ~ SOLVED PAPER 2021 (Stage I)

l (c) According to the information given in the question, the 40. Identify the diagram that best represents the
arrangement is as follows. relationship among the given animals - Reptiles,
Play Day Lizard, Lion.
K Monday
N Tuesday
L Wednesday
O Thursday (a) (b) (c) (d)
M Friday
l (c)
\ N will be the second play to be staged.
Lizard Lion
38. In a town of 500 people, 285 people read ‘The
Hindu’ and 212 people read ‘Indian Express’ and Reptile
127 people read ‘Times of India’. Only 20 people
read ‘The Hindu’ and ‘Times of India’, only 29 Lizard is a reptile, whereas, Lion is different.
people read ‘The Hindu’ and ‘Indian Express’ and 41. If - stands for ÷, + stands for ´, ÷ stands for - and ´
only 35 people read ‘Times of India’ and ‘Indian stands for +, which of the following equations is
Express’. 50 people do not read newspaper. Then correct?
how many people read only one newspaper?
(a) 40 - 10 + 5 ¸ 4 ´ 5 = 21 (b) 40 + 10 - 4 ´ 5 ¸ 3 = 80
(a) 123 (b) 213
(c) 40 ¸ 10 - 4 ´ 5 + 3 = 32 (d) 8 - 4 ´ 40 ¸ 2 + 15 = 30
(c) 312 (d) 321
l (a) From option (a),
l (d) Let the number of people who read Hindu, Times of 40 - 10 + 5 ¸ 4 ´ 5 = 21
India and Indian express all is x.
After interchanging signs according to question, we get
So, number of people who only reads Hindu
40 ¸ 10 ´ 5 - 4 + 5 = 21
= 285 - 20 - 29 - x
By using BODMAS
= 236 - x
4 ´ 5 - 4 + 5 = 21
Number of people who only reads Times of India
20 + 1 = 21
= 127 - 20 - 35 - x
21 = 21
= 72 - x
Number of people who only reads Indian Express 42. P, Q, R, S, T, U, V and W are sitting round the
= 212 - 35 - 29 - x circle and are facing the centre. P is second to the
= 148 - x right of T who is neighbour of R and V. S is not the
Now, 236 - x + 72 - x + 148 - x + 20 + 29 + 35 + x neighbour of P, V is neighbour of U. Q is not
+ 50 = 500 between S and W. W is not between U and S. Then
590 - 2 x = 500 who is sitting opposite to W?
Þ x = 45 (a) S (b) Q (c) U (d) T
So, number of people who read only one news paper is l (d) According to the information given in the question, the
= 236 - 45 + 72 - 45 + 148 - 45 arrangement is as follows
= 191 + 27 + 103 W
= 321
S Q
39. Which of the following venn diagram truly
represents the relationship between Truck, Ship
and Goods? U P

V R

T
(a) (b) (c) (d) Hence, T is sitting opposite to W.
l (a)
43. Find the ratio in which rice at ` 7.20 per kg be
mixed with rice at ` 5.70 per kg to produce a
Truck Goods Ship mixture worth ` 6.30 per kg?
(a) 2 : 3 (b) 1 : 3
(c) 3 : 4 (d) 4 : 5
Truck and ship, both are used to transport goods.
NTSE ~ SOLVED PAPER 2021 (Stage I) 7
l (a) According to the law of Mixture and Alligation 47. Two goods trains each 390 m long are running in
7 : 20 5 : 70 same direction on parallel tracks. Their speeds are
42 km/h and 36 km/h respectively. Find the time
6 : 30 taken by the faster train to cross the driver of the
slower one?
0.6 0.9 (a) 7.8 min (b) 12 min (c) 18 min (d) 4 min
The required ratio =0.6 : 0.9 = 2 : 3 l (a) Total distance = Sum of lengths of two train
44. A pupil’s marks were wrongly entered as 83 instead = (390 + 390) = 780 m
of 63. Due to which the average marks for the class Relative speed = (42 - 36) km/h
got increased by half. The number of pupils in the = 6 km/h
5 5
class is =6 ´ = m/s
18 3
(a) 45 (b) 40 (c) 39 (d) 37
Distance 780
Now, required time = = = 468 s
l (b) Let there are x pupils in the class, Speed æ5 ö
1 x ç ÷
Total increase in marks = x ´ = è3 ø
2 2
x = 7.8 min
According to question, = 83 - 63
2 Directions (Q. Nos. 48-50) Study the pie chart carefully
x
= 20 and answer the following questions.
2
Pie chart shows the percentage quantity of fruits of two shops A
\ x = 40
and B.
45. A watch which gains 5 s in 3 min was set right at
7 am. In the afternoon of the same day when the Guava Guava
watch indicated quarter past 4 O’clock, the true 12% Mango 16% Mango
time is 24% 24%
Others Others
(a) 9 pm (b) 7 pm
30% Apple 26% Apple
(c) 4 pm (d) 5 pm 14% 14%
l (c) Time from 7 am to 4 : 15 pm Orange Orange
37 20% 20%
= 9 h 15 min = h
4
Shop A Shop B
3 min 5 sec of this clock = 3 min of the correct clock
(Total quantity 1200 kg) (Total quantity 1000 kg)
37 1
h of this clock = h of the correct clock.
720 20 48. What is the difference between the quantity of
h of this clock = æç
37 1 720 37 ö Guava at Shop B and that of Shop A?
´ ´ ÷ h of the correct clock.
4 è 20 37 4 ø (a) 20 kg (b) 25 kg (c) 40 kg (d) 16 kg
= 9 h of the correct clock (d) Quantity of Guava at Shop B = 1000 ´
16
l = 160 kg
So, the correct time is 9 h after 7 am. i.e. 4 pm. 100
3 5 12
46. A student multiplied a number by instead of . Quantity of Guava at Shop A = 1200 ´ = 144 kg
5 3 100
What is the percentage error in the calculation? \ Required difference = 160 - 144 = 16 kg
(a) 54% (b) 64%
49. The quantity of Mango at Shop B is what
(c) 74% (d) 84% percentage of the quantity of Mango at Shop A?
l (b) Let the number is 100. (a) 20% (b) 120% (c) 84% (d) 150%
5 500
Number after actual multiplication = 100 ´ = l (c) Quantity of Mango at Shop A
3 3 24
Number after student’s multiplication = 1200 ´ = 288 kg
100
3 300
= 100 ´ = Quantity of Mango at Shop B = 1000 ´
24
= 240 kg
5 5 100
500 300 1600
Error = - = 240
3 5 15 So, required percentage = ´ 100 = 83.33% » 84%
288
æ1600 ö
ç ÷
è ø 50. If the price of Mango is ` 30 per kg, Apple ` 40 per
Percentage error = 15 ´ 100
æ 500 ö kg and Orange ` 20 per kg, then what is the ratio of
ç ÷
è 3 ø their cost at Shop A?
1600 ´ 100 ´ 3 (a) 4 : 5 : 6 (b) 9 : 7 : 5
= = 64%
15 ´ 500 (c) 4 : 5 : 1 (d) 2 : 5 : 7
8 NTSE ~ SOLVED PAPER 2021 (Stage I)

l (b) In shop A, Sol. (Q. Nos. 54 and 55) According to the question, the
24 arrangement according to the age is as follows :
Quantity of Mango = 1200 ´ = 288 kg
100 Geeta > Kusum > Arti > Ananya > Shruti
So, cost of Mango = ` (288 ´ 30) = ` 8640 l (b) Shruti is the youngest.
14
Quantity of Apple = 1200 ´ = 168 kg l (c) Arti is in the middle with respect to age.
100
So, cost of Apple = ` (168 ´ 40) = ` 6720 56. A and B are brothers, D is brother of E, C is father
20 of A, E is daughter of B, then who is uncle of D?
Quantity of Orange = 1200 ´ = 240 kg
100 (a) A (b) B (c) C (d) D
So, cost of Orange = ` 240 ´ 20 = ` 4800
l (a)
\ Required ratio = 8640 : 6720 : 4800
or 9 :7 :5 C(+)
+ ⇒ Male
51. In given question, select the correct alternative. father – ⇒ Female
Monotony : Variety : : Crudeness : ? (+) (+)
A B
(a) Sobriety (b) Simplicity Brother Son
(c) Raw (d) Refinement daughter
l (d) As, variety is the antonym of Monotony. Similarly, E(+) D(+)
refinement is the antonym of crudeness. brother
From the above relation diagram, it is clear that A is the
52. If B is 20 m South from A and C is 10 m North uncle of D.
from D and A is 10 m West from D, then where is B
from C? 57. Which digit will appear on the face opposite to the
face with number 4?
(a) North-East (b) North-West
(c) South-West (d) North 4 6
l (c) 1 3 4 3
C N (a) 5 (b) 2 or 5 (c) 2 (d) 2 or 3
10 m l (b) According to the given position of dices 1, 3 and 6 are
10 m
A W E adjacent to 4.
D
20 m Hence, 2 or 5 will be opposite to 4.
S-W
S 58. If four different positions of a die are shown, what
B
number is opposite of 3?
Hence, B is in South-West direction with respect to C.
2 5 6 3
53. Amit ranked 16th from the top and 29th from the
3 6 2 4 3 1 5 1
bottom among those who passed the examination. 6
students does not participate and 5 failed in it. How
(a) 6 (b) 5
many students were there in the class?
(c) 2 (d) 4
(a) 50 (b) 55 (c) 44 (d) 52
l (d) From the position of first and fourth dice 2, 6, 1 and 5
l (b) Number of students who passed the examination
are ad(d) From the position of first and fourth dice 2, 6, 1
= (16 + 29) - 1 =44 and 5 are adjacent to 3.
Hence, total number of students in the class Hence, 4 will be opposite to 3.
= 44 + 6 + 5 = 55
59. Study the figure given below and find those youth
Directions (Q. Nos. 54 and 55) Study the following who are in service but not literate?
information carefully and answer the questions that
follow. Literate
Youth 2 3 4
Ananya is older than Shruti, Arti is older than Ananya but
5
younger than Kusum. Kusum is older than Shruti, Shruti is 7 6
younger than Arti and Geeta is oldest. 1
54. Who is youngest? Servicemen
(a) Ananya (b) Shruti (c) Arti (d) Kusum (a) 3 or 4 (b) Only 7
55. Who is in the middle with respect to the age? (c) 2 or 7 (d) 2 or 5
(a) Shruti (b) Ananya l (b) Youth who are in service but not literate is represented
(c) Arti (d) Kusum by only 7.
NTSE ~ SOLVED PAPER 2021 (Stage I) 9
60. In the number 59164823, how many digits will be Sol. (Q. Nos. 62-65) According to the information the
as far away from the beginning of the number if arrangement is as follows
digits are arranged in descending order as they are
in the number?
(a) 3 (b) more than 3 K D H J F A E South
(c) 1 (d) 2
l 62. (a) J sits exactly in the middle of the row.
l (d) Original Number = 5 9 1 6 4 8 2 3
l 63. (b) Two persons sits between A and H.
After arranging in = 9 8 6 5 4 3 2 1 l 64. (d) Except FH, in all the other pairs, first person sits to
descending order the left of second person.
Hence, two digits 4 and 2 will remain at the same l 65. (c) F is second to the left of H.
position.
66. Find out the missing number.
61. How many such pairs of letters are there in the
word EDUCATION, each of which has as many 7
letters between them in the word, as they have in
348 18
English alphabet?
(a) more than 3 (b) 1
(c) 2 (d) 3 172 40
l (a) ?

E D U C A T I O N
(a) 72 (b) 84 (c) 68 (d) 66
l (b) As, 7 ´ 2 + 4 = 18
There are total 5 pairs EA, ED, DA, DI, ON which has as 18 ´ 2 + 4 = 40
many letters between them in the word, as they have in 40 ´ 2 + 4 = 84
english alphabet. 84 ´ 2 + 4 = 172
and 172 ´ 2 + 4 = 348
Directions (Q. Nos. 62-65) Study the following
\ ? = 84
information carefully and answer the questions given
below. 67. Find the missing number in given question.
A, D, E, F, H, J and K are sitting on straight table facing towards 9 8 7
South. D sits fourth to the right of A. E sits at the left end of the
6 7 8
table. Five persons sit between E and K. J sits third to the left of
K. and F does not sit immediate to D. 5 4 6
260 216 ?
62. Which of the following person sits exactly at the
middle of the row? (a) 191 (b) 326 (c) 330 (d) 336
(a) J (b) F l (c) In first column,
(c) H (d) A (9 ´ 6 ´ 5) - 10 Þ270 - 10 = 260
In second column,
63. How many persons sit between A and H? (8 ´ 7 ´ 4) - 8 Þ224 - 8 = 216
(a) 1 (b) 2 Hence, in third column
(c) 3 (d) 4 (7 ´ 8 ´ 6) - 6 Þ336 - 6 = 330
64. Three of the following four pairs are alike in a \ ? = 330
certain way based on their positions and so formed 68. Find the missing number.
a group. Which of the following does not belong to
the group? 4 9
(a) AF (b) JH
(c) EA (d) FH 3 0 26 6

65. What is the position of F with respect to H?


63 ? 10
(a) Second to the right 8
(b) Exactly right
12 12
(c) Second to the left
(d) Third to the right (a) 7 (b) 12 (c) 16 (d) 14
10 NTSE ~ SOLVED PAPER 2021 (Stage I)

l (a) As, (4 - 3)3 - 1 = 0 73. Question Figures


(9 - 6)3 - 1 = 26
and (12 - 8)3 - 1 = 63
Similarly, (12 - 10)3 - 1 = 7
\ ?=7
?
69. Complete the series from given options.
a_caa_bcc_aabbb_cc Answer Figures
(a) bbca (b) Abca
(c) bbac (d) Babc
l (c) The series is as follows
a b c/aa/ b b/cc/ a aa/ bbb/c cc / Þ b b a c
Directions (Q. Nos. 70 and 71) Study the following (a) (b) (c) (d)
information to answer the given questions. l (c) The figure given in option (c) will complete the series.
In a certain code ‘always create new ideas’ is written as ‘ba ri sh
gi’, ‘idea and new thoughts’ is air written as ‘fa gi ma ri’, ‘create 74. Question Figures
thoughts and insights’ is written as ‘ma jo ba fa’, ‘new and better
solution’ is written as ‘ki ri to fa’.
G A O I T Z
70. What is code for ‘ideas’ ? E C M K V X ?
(a) sha (b) gi
(c) ba (d) ma
Answer Figures
71. What does ‘fa’ stand for?
(a) thoughts (b) insights
M R N R O R L R
(c) new (d) and
O P O P P Q N P
Sol. (Q. Nos. 70 and 71)
(a) (b) (c) (d)
always create new ideas ⇒ ba ri sh gi ...(i)
l (d) As,
idea and new thoughts ⇒ fa gi ma ri ...(ii) +2

create thoughts and insights ⇒ ma jo ba fa ...(iii)


G A O I
+2 +2 +2 +2
new and better solution ⇒ ki ri to fa ...(iv) E C M K

l 70. (b) From Eqs. (i), (ii), (iii) and (iv), it is clear that the +2 +2
code for ideas is ‘gi’.
Similarly,
l 71. (d) ‘fa’ stands for ‘and’. −2

Directions (Q. Nos. 72-74) Choose the correct answer


T Z L R
figure from the given alternatives which will complete the −2 −2 −2 −2
given questions figure series. V X N P

72. Question Figures −2 −2

Directions (Q. Nos. 75 and 76) A set of four figures are


? given in which three bear close resemblance. Select the
odd one out.
Answer Figures 75.

(a) (b) (c) (d)


(a) (b) (c) (d)
l (c) All the figures except figure (c) are divided into parts of
l (a) The figure given in option (a) will complete the series. equal areas by the lines inside.
NTSE ~ SOLVED PAPER 2021 (Stage I) 11
76. Answer Figures

(a) (b) (c) (d)

l (a) Except figure (a), all the figures are divided into two (a) (b) (c) (d)
equal parts. l (d) The figure given in option (d), will complete the given
question figure.
77. If a mirror is kept on a line, then which of the
answer figure is the correct mirror image of Directions (Q. Nos. 80 and 81) All the surfaces of a solid
question figure? cube of side 4 cm have been coloured black and cut into
Question Figures equal sized cube of side 1 cm each. Now answer the
following questions.
80. How many cubes are there which do not have any
of their surfaces coloured?
(a) 0 (b) 24 (c) 16 (d) 8
Answer Figures l (d) Cubes which do not have any of their surfaces coloured
= (n - 2)3
when n = 4
= (4 - 2)3 = 23 = 8

81. How many cubes have two of their surfaces


(a) (b) (c) (d) coloured with black on the opposite surfaces?
(a) 4 (b) 2 (c) 0 (d) 8
l (c) The figure given in option (c) is the correct mirror
image of the question figure. l (c) There is no such cubes having two of their surfaces
coloured with black on the opposite surfaces.
78. Which of the four alternatives given below is the
correct water image of the question figure? 82. In a row of 40 children, A is 13th from the left end
and B is 9th from the right end. How many
Question Figures
children are there between A and C, if C is 4th to
the left of B?
(a) 12 (b) 13
(c) 14 (d) 15
l (c) If B is 9th from the right and 4th left of B is C, so, C will
be 13th from the right and 28th from the left.
Answer Figures So, there are (28 - 13 - 1) = 14 children between A and C.

Directions (Q. Nos. 83-85) Read the following information


carefully and answer th questions that follow.
In a family A is younger brother of B and D is the son of B. D is
brother of E but E is not son of B. F is grandson of B and C is
(a) (b) (c) (d) cousin of E.
l (b) Figure (b) is the correct mirror image of the given 83. Who is father of F?
figure.
(a) A (b) B
79. Which one of the given alternative figures will (c) C (d) Cannot be determined
complete the given figure pattern?
84. The relation of A with D is
Question Figure
(a) father (b) brother
(c) cousin (d) uncle

85. The relation of E with F is


(a) uncle (b) aunt
(c) grand father (d) cannot be determined
12 NTSE ~ SOLVED PAPER 2021 (Stage I)

Sol. (Q. Nos. 83-85)


Brother
(–) + ⇒ Male 89. 2 6 2
A B
– ⇒ Female
Son 4 28 6 14 66 4 2 ? 13
Brother
L E(–) D(+) 2 4 7
Cousin
(a) 40 (b) 32
l 83. (d) F can be the son of E or D. Hence, his father cannot
be determined on the basis of the given information. (c) 35 (d) 30
l 84. (d) A is the uncle of D. l (c) As, in first circle
6 ´ 4 + (2 + 2) Þ24 + 4 = 28
l 85. (d) E can be mother or aunt of F. So, relation between
E and F, cannot be definitely determined. and in second circle
14 ´ 4 + (6 + 4) Þ56 + 10 = 66
86. How many times are the hands of a clock at right Similarly, in third circle
angles in 12 h? (13 ´ 2) + (7 + 2) = 26 + 9 = 35
(a) 24 times (b) 48 times \ ? = 35
(c) 22 times (d) 44 times
90.
l (c) In 12 h, the hands of the clock are right angles for
22 times. 7 9 11
2 3 2
Directions (Q. Nos. 87-90) Find the missing character in
53 87 ?
the following figures.
(a) 125 (b) 100
87. 4 8 5 (c) 129 (d) 64
l (a) As, in first column,
72 + (2 ´ 2) Þ49 + 4 = 53
196

196

36 36 36 ?
64 64 16 In second column,
92 + (3 ´ 2) Þ81 + 6 = 87
10 18 14 22 11 15 In third column,
(a) 16 (b) 196 (c) 100 (d) 144 (11)2 + (2 ´ 2) Þ121 + 4 = 125
l (c) In first figure, \ ? = 125
(10 - 4)2 = 62 = 36
91. How many triangles are there in the given figure?
(18 - 10)2 = 82 = 64
(18 - 4)2 = 142 = 196
In second figure,
(14 - 8)2 = 62 = 36
(22 - 14)2 = 82 = 64
(22 - 8)2 = 142 = 196
Similarly, in third figure
(11 - 5)2 = 62 = 36 (a) 15 (b) 14 (c) 16 (d) 20
(15 - 11)2 = 42 = 16 l (a) A
(15 - 5)2 = 102 = 100
\ ? = 100 F

88. E
4 7 6 9 1 7

B C
D
38 55 ? There are total 15 triangles in the given figure, which
are DBED, DDEC, DBFE, DCEF, DBFA, DCFA, DBFC,
(a) 49 (b) 63 (c) 48 (d) 50 DCFD, DBAD, DCAD, DABD, DADC, DABC, DBEC, DBFC.
l (b) As, in first figure, 92. If 2 ´ 2 = 16, 2 ´ 3 = 36, 2 ´ 4 = 64, then 2 ´ 6 = ?
72 - 42 + 5 Þ 49 - 16 + 5 = 38 (a) 72 (b) 80 (c) 96 (d) 144
92 - 62 + 10 Þ 81 - 36 + 10 = 55
l (d) As, 2 ´ 2 = 4 Þ 42 = 16
Similarly, 72 - 12 + 15 Þ 49 - 1 + 15 = 63
2 ´ 3 = 6 Þ 62 = 36
NTSE ~ SOLVED PAPER 2021 (Stage I) 13
2 ´ 4 = 8 Þ82 = 64 Directions (Q. Nos. 96-100) Study the diagram and
2 ´ 6 = 12 Þ122 = 144 answer each of the following.
\ ? = 144

93. Find the value of 10 25

10 + 25 + 108 + 154 + 225 . 20 17 15 30 40


(a) 10 (b) 8 20
7
(c) 6 (d) 4 20

l (d) 10 + 25 + 108 + 154 + 225


Person who takes tea
= 10 + 25 + 108 + 154 + 15
Person who takes lassi
= 10 + 25 + 108 + 169
Person who takes coffee
= 10 + 25 + 108 + 13
96. How many persons who take tea and lassi but not
= 10 + 25 + 121
coffee?
= 10 + 25 + 11 (a) 20 (b) 17
= 10 + 36 (c) 25 (d) 15

= 10 + 6 l (b) There are 17 persons who take tea and lassi but not
coffee.
= 16 = 4
97. How many persons are there who take both tea and
94. Find the correct figure.
coffee but not lassi?
(a) 22 (b) 17
(c) 7 (d) 20

? l (c) There are 7 persons who take both tea and coffee but
not lassi.

98. How many persons take lassi?


(a) 100 (b) 82
(c) 92 (d) 122
l (d) Number of persons who take Lassi
(a) (b) (c) (d) = (20 + 17 + 15 + 30 + 40) = 122
l (c) Figure given in option (c) is the correct figure. 99. How many persons are there who takes only coffee?
95. (a) 90 (b) 45
2 3 4
(c) 25 (d) 20
3 5 3
1 1 2 l (b) The persons who take only coffee
= 20 + 25 = 45
Which number is opposite to 3?
(a) 1 (b) 6 (c) 5 (d) 4 100. How many persons take all the three?
l (b) From the three positions of dice, it is clear that 1, 2,5 (a) 20 (b) 17
and 4 are adjacent to 3. (c) 25 (d) 15
Hence, only 6 is opposite to 3. l (d) 15 persons take all the three.
14 NTSE ~ SOLVED PAPER 2021 (Stage I)

Paper II Scholastic Aptitude Test (SAT)


101. The sound of same pitch and loudness are 104. A particle of mass 0.3 kg is subjected to a force
distinguished from one another by their F = Kx with K = 15 N/m, what will be its
(a) Wavelengths (b) Velocity acceleration if it is released from a point x = 20 cm.
(c) Quality (d) Tones (a) 1 m/s2 (b) 10 m/s2
l (c) Quality of sound helps to distinguish between two (c) 100 m/s2 (d) 0.1 m/s2
sounds. Quality is referred to the frequency of the sound. l (b) F = kx = 15 ´ 0.2 = 3 N
F 3
102. A water pumps lifts water from a level 10 m below a= = = 10 m/s2
M 0.3
the ground. The water is pumped at the rate of
30 kg/min with negligible velocity. Calculate the 105. An object is moving in a straight line. The velocity
minimum power the pump should have to do this time graph is as shown below. Then
work. B
(a) 49 J/s (b) 490 J/s (c) 500 J/s (d) 48 J/s
A
W mgh velocity
l (a) Power (P) = =
t t
30 ´ 9.8 ´ 10
= [Q 1 min = 60 sec] O
60 time
= 49 J/s (a) in part OA acceleration is increasing.
(b) in part AB acceleration is increasing.
103. Six identical resistors connected between points A,
B and C as shown in diagram. The equivalent (c) in part OA acceleration is decreasing.
resistance would be maximum between (d) in part AB acceleration is decreasing.

C l (d) Slope of v-t graph gives the value of acceleration. As,


slope of v-t graph is decreasing between points A and B, so
acceleration is decreasing.
R R
R R R 106. A force of 100 N acts on a body so that the body
acquire a velocity of 10 m/s after some time. Now
the force of 100 N is replaced by another force F
R
A B which decelerates the body and body come to the
(a) A and B rest then.
(b) B and C (a) F > 100 N (b) F < 100 N
(c) A and C (c) F = 100 N (d) All options are possible
(d) Options (a), (b) and (c) are correct. l (d) A retarding force of any magnitude can stop the
moving body.
l (a) If resistance of arm AC is R1 , then
R´ R R 107. 2 points A and B are at electric potentials 10 V and
R1 = =
R+ R 2 100 V respectively. A charge q is taken from A to B
Resistance of arm BC is R2 , then and 18 Joule of work is done. The value of q is
1 1 1 1 3 (a) 2 Coulomb (b) 0.2 Coulomb
= + + =
R2 R R R R (c) 20 Coulomb (d) 0.02 Coulomb

Þ R2 =
R l (b) Work done = Potential difference ´ Charge
3 = (V2 - V1 ) ´ Q
5R 18 = (100 - 10) ´ Q

\ æ R Rö
RAB = R ||(R1 + R2 ) = R || ç + ÷ = 6 = 5R 18
è 2 3 ø R + 5 R 11 = Q Þ Q = 0.2 C
90
6
3R R 108. Which of the following is not correct for magnetic
´
RBC = æç R + ö÷ || R2 = 2 3 = 3R field lines?
R
è 2ø 3 R R 11
+ (a) The direction of magnetic field lines outside the
2 3 magnet is from North pole to South pole.
R 4R (b) The direction of magnetic field lines inside the
´
R æ Rö 2 3 = 4R magnet is from South pole to North pole.
RAC = R1 ||(R + R2 ) = || ç R + ÷ =
2 è 3 ø R
+
4 R 11 (c) The degree of closeness of magnetic field lines tells
2 3 the relatives strength of magnetic field.
Hence, RAB > RAC > RBC (d) Magnetic field lines never form closed loop.
NTSE ~ SOLVED PAPER 2021 (Stage I) 15
l (d) Magnetic field lines are closed curve because l (c) For maximum current, 30 W resistance should be
magnetic monopoles do not exist. neglected by connecting arrow extreme left of 30 W
1 resistance.
109. A car moving along straight line covers th of total 2.2 2.2
5 \ Imax = = = 0.33 A
distance with speed v1 and remaining part of (10 ||20) 10 ´ 20
10 + 20
distance with speed v2 . The average speed of car
For minimum current, 30 W resistance should be taken
over entire distance is completely by connecting arrow of rheostat extreme right.
5 v1 v2 4 v1 v2 2.2
(a) (b) \ Imin =
v2 + 4 v1 5 v1 + v2 (10 ||20) + 30
5 v1 v2 4 v1 v2 2.2
(c) (d) = = 0.06 A
20
4 v2 + v1 4 v1 + v2 + 30
3
1
l (a) Let t be the time taken for th of total distance and t2 be 112. Three particles A, B and C are thrown from top of a
5
the time taken for remaining distance. building with same speed. A is thrown upwards,
s 4s B is thrown downwards and C is thrown
t1 = , t2 =
5 v1 5 v2 horizontally, they hit the ground with speed v A , v B
and v C respectively, then
Total time, T = t1 + t2
(a) v A = v B = v C (b) v B > v C > v A
Total distance
Average speed = (c) v A = v B > v C (d) v A > v B = v C
Total time
s 5 v1 v2 l (c) The given situation is shown below
= =
s 4s v2 + 4 v1
+
5 v1 5 v2
Q uA
uA P
110. Light travels through a glass slab of thickness t and uC
having refractive index n. If c is the velocity of light uB
in vacuum, then the time taken by light to travel
this thickness of glass is
t nt n2 t t
(a) (b) (c) (d)
nc c c n2 c
l (b) Refractive index (n)
Velocity of light in air or vacuum (c) From figure it is clear that, velocity of particle A and B at
= points P and Q will be same in downward direction.
Velocity of light in glass slab (v)
Hence, uA = uB
c c
Þ n = Þv = ...(i) Velocity of particle C in downward direction at the point P
v n will be zero.
Thickness (t ) = Velocity of light (v) ´ Time \ uA = uB > uC
t t
Þ Time = = [from Eq. (i)] \ vA = vB > vC
v c
n 113. An object of height 2.0 cm is placed on the principal
=
nt axis of a concave mirror at a distance of 12 cm from
c the pole. If the image is inverted, real and 5 cm in
height, then location of the image and focal length
111. The resistance of rheostat shown in the figure is of the mirror respectively are
0-30 W, Neglecting the resistance of ammeter and
(a) (- 30 cm, + 8.6 cm) (b) (- 30 cm, - 8.6 cm)
connecting wire the minimum and maximum
currents through the ammeter will be (c) (+ 30 cm, + 8.6 cm) (d) (+ 30 cm, - 8.6 cm)

2.2v
l (b) u = - 12
- v -5
m= =
u 2
-v -5
A Þ =
-12 2
10Ω v = - 30 cm
1 1 1
+ =
30Ω v u f
20Ω 1 1 1
+ =
(a) (0.08 A, 0.33 A) (b) (0.06 A, 0.08 A) - 30 -12 f
(c) (0.06 A, 0.33 A) (d) (0.33 A, 0.09 A) Þ f = - 8.6 cm
16 NTSE ~ SOLVED PAPER 2021 (Stage I)

114. When lead nitrate is heated, a brown gas is evolved, l (c) Denatured alcohol is a mixture of C2 H5 OH and
the evolved gas is CH3 OH. It is a mixture of ethyl alcohol, methyl alcohol
and pyridine. It is adulterated with toxic additives (e.g.
(a) Dioxygen (b) Nitrogen dioxide
methanol) to make it unsuitable for human consumption.
(c) Nitrous oxide (d) Dinitrogen
l (b) When lead nitrate is heated it decomposes to form lead
120. For welding, a mixture of oxygen and …………… is
oxide, nitrogen dioxide and oxygen. The brown fumes burnt.
liberated is of nitrogen dioxide. (a) Benzene (b) Butane (c) Methane (d) Ethyne
D l (d) A mixture of oxygen and ethyne is burnt for welding.
2Pb(NO3 )2 ¾® 2PbO + 4NO2 (­) + O2 (­)
When ethyne is burnt in air, it gives a sooty flame. This is
115. When a solution of lead(II) nitrate and potassium due to incomplete combustion caused by limited supply of
oxygen. However, if ethyne is burnt with oxygen, it gives a
iodide are mixed, the yellow ppt is formed, the ppt
clean flame with temperature of 3000°C (which is suitable
is of …………… . for welding) because of complete combustion.
(a) KNO3 (b) KCl
(c) Pbl2 (d) Pbl4 121. The following metals are arranged in the increasing
order of their metallic character. Choose the correct
l (c) When lead nitrate get mixed with potassium iodide, a option.
precipitation reaction occurs that forms yellow precipitate
of lead iodide. (a) Be < Si < K < Al (b) Si < Be < Al < K
Pb(NO3 )2 + 2KI ¾® PbI2 (¯) + 2 KNO3 (c) K < Al < Si < Be (d) Be < Si < Al < K
l (b) From right to left, across a period, the metallic
116. Baking powder is a mixture of ……… . character decreases due to increase in ionisation enthalpy.
(a) Sodium hydrogen carbonate and oxalic acid Metallic character increases as we move down a group,
(b) Sodium carbonate and tartaric acid due to decrease in ionisation enthalpy.
(c) Sodium hydrogen carbonate and tartaric acid So, the correct order is Si < Be < Al < K.
(d) Sodium carbonate and oxalic acid.
122. Which one of the following oxide is insoluble in
l (c) Baking powder is a mixture of baking soda (sodium water?
hydrogen carbonate NaHCO3 ) and a mild edible acid such (a) Na2 O (b) CuO
as tartaric acid. When baking powder mixes with water,
(c) K2 O (d) CaO
sodium hydrogen carbonate reacts with tartaric acid to
evolve carbon dioxide gas which make cake or bread l (b) Metal oxides are formed when a metal reacts with
fluffy. oxygen. Some metal oxides (soluble) will dissolve in water
to form an alkaline solution e.g. Na2 O, K2 O and CaO
117. Aqua regia is a mixture of conc. Hydrochloric acid whereas insoluble metal oxides such as copper oxide
and conc. Nitric acid in ratio. (CuO) will not affect the pH of water.
(a) 3 : 1 (b) 2 : 1 123. Which of the following oxide turns red litmus into
(c) 1 : 5 (d) 2 : 3 blue?
l (a) 3 mole conc. HCl combine with 1 mole conc. HNO3 to (a) SO2 (b) CO2
give aquaregia solution. So, ratio is 3 : 1. (c) NO2 (d) KO2
conc. H 2 SO 4
118. CH3 CH2 OH ¾¾¾® Products l (d) Metallic oxides are basic in nature whereas
443 K
non-metallic oxides are acidic in nature. KO2 being basic
The products formed in the above reaction is/are in nature turns red litmus blue.
(a) Ethene and H2 O 124. Which one of the following is not a green house
(b) Ethyne and H2 O gas?
(c) Ethane and H2 O (a) CH4 (b) O3
(d) Methane and H2 O (c) CO2 (d) SO2
l (a) When ethyl alcohol, that is commonly known as l (d) SO2 is not a green house gas Sulfur dioxide is not
ethanol reacts with conc. H2 SO4 (sulphuric acid), then it considered as green house gas because it does not absorb
forms ethene along with water. Here, H2 SO4 is a catalyst and trap infrared radiation of Sun. It is responsible for acid
which works like a dehydrating agent. rain as it decreases the pH of rain by forming sulphuric
Conc. H 2 SO 4
CH3 CH2 OH ¾¾¾¾® H2 C ==CH2 (Ethene) + H2 O. acid by reacting with water vapours.
443K
Ethanol 125. Which of the following element does not show
119. Denatured alcohol is a mixture of allotropy?
(a) CH3 OH and HCHO (a) Phosphorus (b) Sulphur
(b) CH3 OH and CH3 COOH (c) Oxygen (d) Aluminium
(c) C2 H5 OH and CH3 OH l (d) Aluminium is a soft metal and does not exist in
different forms, therefore does not show allotropy.
(d) C2 H5 OH and CH3 COOH
NTSE ~ SOLVED PAPER 2021 (Stage I) 17
126. Which one of the following will have the largest l (a) Deepak made the wrong pyramid as his observations
number of atoms? and number of trophic levels are not correct. It does not
follows the natural energy and nutrient flow in a food
(a) 100 g of He (b) 100 g of Na
chain or food web. The correct description is as follows.
(c) 100 g of li (d) 100 g of Al
Given mass
l (a) Number of atoms = ´ NA
Atomic mass
Decomposers
As, given mass of all the given atoms are same, i.e. 100 g.
Therefore, number of atoms depend inversely with atomic Lion (carnivores)
mass of the atom.
He has the smallest atomic mass of 4u out of other atoms,
Omnivores (Monkey)
therefore 100 g of He contain more number of atoms out of
other given options.
100 Herbivores (Insect & Deer)
(a) 100 g He = = 25N A
4
Producers (Grass)
(b)100 g of Na = 4.3 N A
(c) 100 g of Li = 14.28 N A
(d) 100 g of Al = 3.70 N A
129. UV rays cause cancer but in stratosphere the same
UV rays are helping us, how?
127. Rearrange the following sentences and choose the (a) They divert harmful UV rays back to sun
correct option. (b) They convert oxygen present in stratosphere into
1. Breakdown of H2O into Hydrogen and Oxygen ozone.
and Conversion of light energy into chemical (c) UV rays are not present in stratosphere.
energy (d) UV rays reach the earth surface, then bounce back
carrying ozone to stratosphere.
2. Reduction of carbon dioxide to carbohydrates.
l (b) In stratosphere, the ozone is formed naturally
3. Absorption of light energy by chlorophyll. through the interaction of solar ultraviolet radiation with
(a) 1 ® 2 ® 3 (b) 3 ® 2 ® 1 molecular O2.
(c) 3 ® 1 ® 2 (d) 1 ® 3 ® 2
130. Match the following.
l (c) Plants prepare their own food by the process of
photosynthesis. The following events occurs during the Column I Column II
process of photosynthesis A. Regeneration 1. Shoot
l
Absorption of light energy by chlorophyll. B. Rhizopus 2. Pollen grain
l
Breakdown of H2 O into Hydrogen and Oxygen and
Conversion of light energy into chemical energy. C. Plumule 3. Vegetative Propagation
l
Reduction of carbon dioxide to carbohydrates. D. Rose 4. Planaria
E. Stigma 5. Spores
128. Deepak is trying to study flow of energy in an area
and he made the following diagram for the same. A B C D E
How will you interpret his observations? (a) 1 5 4 2 3
(b) 2 4 1 3 5
rock (c) 2 1 4 3 5
monkey (d) 4 5 1 3 2

deer l (d) Planaria reproduce by the process of regeneration


whereas Rhizopus reproduce by the formation of spores.
insects Plumule is a part of seed embryo that develops into shoot,
bearing the first true leaves of a plant. The asexual
grass reproduction in rose plant occur through the stem cutting
lion
is known as vegetative propagation. The stigma is a
specially adapted portion of pistil modified for the
reception of pollen grains.
(a) His observations and number of trophic levels are
wrong. Column I Column II
(b) His observations are correct but the number of A. Regeneration 4. Planaria
trophic levels can be more.
B. Rhizopus 5. Spores
(c) His observations are wrong but number of trophic
levels are correct. C. Plumule 1. Shoot
(d) His observations as well as number of trophic D. Rose 3. Vegetative Propagation
levels are correct.
E. Stigma 2. Pollen grain
18 NTSE ~ SOLVED PAPER 2021 (Stage I)

131. The opening and closing of the stomata depends 135. Assertion (A) No carbon dioxide is released during
upon : the day in plants.
(a) Oxygen (b) Temperature Reason (R) Only photosynthesis occurs during the
(c) Carbon dioxide (d) Water in guard cells day.
l (d) The opening and closing of stomata regulated by water (a) A is true and R is false
in guard cells. (b) A is false and R is true
(c) Both A and R are false
132. Sonu performed an experiment to study dihybrid
(d) Both A and R are true but R does not explain A
cross for round/wrinkled and yellow/green coloured
seeds. He obtained 2432 seeds in total. What will be l (c) Both statements A and R are false.
the number of seeds which are round and yellow? Respiration in plants occurs throughout the day and night
(a) 1367 (b) 1356 thereby carbon dioxide is formed during day and night in
plants. Process like photosynthesis, respiration and
(c) 1368 (d) 1438
transpiration take place in plants during day.
l (c) Total number of seeds = 2432
Dihybrid cross ratio = 9 : 3 : 3 : 1 136. Choose the non-biodegradable substance from the
Yellow round - 9 following:
Yellow wrinkled 3 1. Carrot 2. Glass bottle
Green round - 3 3. Perfume spray bottle 4. Rice bran
Green wrinkled - 1 5. Papaya 6. Thermocol
Number of round and yellow seeds 7. Wooden stick 8. Ball pen refill
2432 ´ 9 (a) 2, 3, 7, 8 (b) 2, 3, 6, 8
= = 152 ´ 9 = 1368 seeds.
16 (c), 3, 1, 5, 7 (d) 8, 5, 1, 3
133. The stakeholders of various forest products are : l (b) Glass bottle, Perfume spray bottle, Thermocol, Ball pen
(i) People living near forests refill are non-biodegradable substances as they cannot be
decompose by the action of micro-organism present in soil.
(ii) Government only
(iii) Nature lovers 137. Match the column I and column II and select
(iv) Wild life correct option.
(a) All options are correct Column-I Column-II
(b) (i), (ii) and (iii) are correct A. Ribosome 1. ATP formation
(c) Only (ii) is incorrect B. Mitochondria 2. Photosynthesis
(d) None of the option is correct.
C. Centriole 3. Protein synthesis
l (b) A forest stakeholder is someone who uses forest
D. Chloroplast 4. Cell division
resources in one way or the other.
The stakeholders of various forest products are A B C D A B C D
l
People living near forests
(a) 1 2 4 2 (b) 3 1 4 2
l
Government only
(c) 4 3 2 1 (d) 2 1 3 4
l
Nature lovers
l (b) Ribosome helps in protein synthesis, Mitochondria is a
134. Choose the correct sequence : power house of the cell and responsible for ATP formation.
(a) Pulmonary vein ® Pulmonary artery ® Left auricle Centriole play a vital role in cell division by formation of
® Right ventricle spindle fibres. Photosynthesis take place in chloroplasts
(b) Pulmonary artery ® Right auricle ® Left ventricle which contain the chlorophyll.
® Pulmonary vein
Column-I Column-II
(c) Right auricle ® Pulmonary artery ® Pulmonary
vein ® Left ventricle A. Ribosome 3. Protein synthesis
(d) Left ventricle ® Pulmonary vein ® Pulmonary B. Mitochondria 1. ATP formation
artery ® Right auricle
C. Centriole 4. Cell division
l (c) Blood enters the right atrium from all parts of the body D. Chloroplast 2. Photosynthesis
through vena cava. It passes through the right ventricle
which pumps the blood to the lungs via. pulmonary artery.
138. Which of the following is a barrier method of
The exchange of gases take place in lungs where it
contraception?
becomes oxygenated and brought back to heart by the
pulmonary veins which enter the left atrium and passes to (a) Diaphragm (b) Contraceptive pills
ventricle and pumped to all parts of body Aorta. (c) Tubectomy (d) All of these
Right auricle ® Pulmonary artery ® Pulmonary vein ® l (a) Diaphragm is a barrier method of contraception
Left ventricle whereas tubectomy and contraceptive pills are surgical
and chemical method of contraception, respectively.
NTSE ~ SOLVED PAPER 2021 (Stage I) 19
139. Sperms are produced in the 143. The sum of the first ‘p’ odd natural numbers is 100
(a) Seminiferous tubules (b) Interstitial cell and the sum of the first ‘q’ even natural numbers is
(c) Vas deferens (d) Prostate gland 90. Find the value of ( p + q).
l (a) Sperms are produced in the seminiferous tubules of (a) 18 (b) 19
testes. (c) 20 (d) 21
l (b) QSum of n odd natural number = n2
140. Blood pressure is measured by an instrument called
\ Sum of p odd natural number = p2 = 100
(a) Barometer (b) Sphygmomanometer
Þ p = 10
(c) Photometer (d) Manometer
Sum of first ‘q’ even natural number
l (b) Blood pressure is measured by Sphygmomanometer. Þ2 + 4 + 6 + ¼ upto ‘q’ terms
141. If DABC is an equilateral triangle such that A(2, 2) = 2 (1 + 2 + 3 + ¼ upto q term)
and centroid of the triangle is ( - 2, 2), then find the q ´ (q + 1)
=2 ´ = 90
length of its side. 2
(a) 4 units (b) 6 units (c) 4 3 units (d) 9 units Þ q2 + q - 90 = 0
Þ (q + 10)(q - 9) = 0
l (c) Let length of side of DABC be ‘a’.
Þ q=9
A
(2, 2) p + q = 10 + 9 = 19
1 1
144. If x + = 1 and y + = 1 , then what is the value
y z
æ 1 ö
a a of ç z + + 1÷ ?
è x ø
(a) 0 (b) 1 (c) 2 (d) 3
G 1
(–2 , 2) l (c) Given, x + =1
y
Þ xy + 1 = y Þ xy - y = - 1
1
B a/2 K a/2 C Þ y(x - 1) = - 1 Þ y = …(i)
1- x
AG = (2 + 2)2 + (2 - 2)2 = (2 + 2)2 = 4 1
also, y + = 1 Þ yz + 1 = z Þ yz - z = - 1
2 z
Q AG = ´ AK 1
3 Þ z(y - 1) = - 1 Þ z =
2 1- y
\ 4 = ´ AK Þ AK = 6
3 1 1- x
\ z= Þz= [From Eq. (i)]
3 3 1 1 - x -1
\ AK = ´a Þ 6= ´a 1-
2 2 1- x
12 12 ´ 3 - xz = 1 - x Þ1 + xz = x
Þ =a Þ
3 3 ´ 3 1
Þ + z =1
x
Þ a = 4 3 units
1
\ + z + 1 =1 + 1 = 2
142. The sum of the n consecutive odd natural numbers x
starting from 5 is 60. Find the value of ( n2 - n).
145. If 3 p + 3 q + 3 r = 0, then the value of ( p + q + r )3 is
(a) 20 (b) 30 (c) 42 (d) 56
l (b) 5 + 7 + 9 + ¼ + upto n terms = 60 (a) 3pqr (b) 9pqr
QSum of n terms of AP (c) 27pqr (d) 0
n l (c) Given, p1 /3 + q1 /3 + r1 /3 = 0 ...(i)
= [2 a + (n - 1)d]
2 If a + b + c = 0 then a3 + b3 + c3 = 3 abc
According to the question, \ Using Eq. (i),
n (p1 /3 )3 + (q1 /3 )3 + (r1 /3 )3 = 3 ´ p1 /3 ´ q1 /3 ´ r1 /3
Sum = [2 ´ 5 + (n - 1)2] = 60
2 Þ p + q + r = 3(pqr)1 /3
Þ n[5 + n - 1] = 60 \ (p + q + r)3 = 27 pqr
Þ n(4 + n) = 60
Þ n2 + 10 n - 6 n - 60 = 0 146. If a1, a2, a3 ¼ ¼ an and b1 , b2 , b3 , ¼ ¼ bn are two
Þ n(n + 10) - 6(n + 10) = 0 AP’s such that a1 b1 = 120, a2 b2 = 143, a3 b3 = 154,
Þ (n + 10) (n - 6) = 0 then a8 b8 =
Þ n=6 (a) 209 (b) 89
\ n2 - n = n(n - 1) = 6 ´ 5 = 30 (c) 195 (d) 29
20 NTSE ~ SOLVED PAPER 2021 (Stage I)

l (d) We have, a1 , a2 , a3 , ¼, an ® AP1 148. If ABCD and PQRS are two squares, such that
b1 , b2 , b3 , ¼, bn ® AP2 area of square PQRS is ‘A’ m2, then find the value
a1 b1 = 120, a2 b2 = 143, a3 b3 = 154 then a8 b8 = ? of 17A .
Let’s say three terms of 1st AP are A 3m 1m B
a - d, a, a + d,
1m Q
and first three terms of 2nd AP are B - D, B, B + D, P
Now, a1 b1 = 120
Þ (a - d) (B - D) = 120 3m
Þ aB - aD - Bd + dD = 120 …(i)
a2 b2 = 143
a × B = 143 …(ii) 3m R 1m
a3 b3 = 154 S
Þ (a + d)(B + D) = 154 D 1m 3m C
aB + aD + Bd + dD = 154 …(iii) (a) 12 (b) 144 (c) 16 (d) 9
By Eqs. (i) and (ii), l (a) We have,
23 = aD + Bd - dD …(iv) ABCD and PQRS are two squares.
By Eqs. (iii) and (ii), 3m X 1m
A B
11 = aD + Bd + dD …(v)
By Eqs. (iv) and (v), 1m P Q
12 = - 2 dD Þ dD = - 6 W
and aD + Bd = 17 3m
Now, a8 b8 = (a + 6 d) (B + 6 D)
= aB + 6 aD + 6 Bd + 36 dD Y
= 143 + 6(17) + 36 (-6) 3m R 1m
= 143 + 102 - 216 = 29 S
147. In a DABC, AX bisects BC and AX is also the angle D 1m Z 3m C
bisector of angle A. If AB = 12 cm, BX = 3 cm, then
what is the area of DABC? CX = 42 + 12 = 17 m [by Pythagoras theorem]
(a) 9 cm2 (b) 2 cm2 We have, BW = AZ = DY = CX
(c) 9 5 cm2 (d) 9 15 cm2 1 1
Area of triangle BXC = ´ 1 ´ 4 = ´ (BQ) ´ (XC)
l (d) We have, 2 2
4
In DABC, AX bisect, BC and AX is also the angle bisector Þ 4 = BQ ´ 17 Þ BQ = m
of ÐA. 17
4
A Also, BQ = CR = DS = AP =
17
By Pythagoras theorem
θ θ
16 1
XQ = BX 2 - BQ2 = 1 - = m
17 17
1
Þ XQ = WP = ZS = YR = m
17
B C Sides of square, PQRS = CX - CR - XQ
3 X
4 1 12
= 17 - - = m
AB = 12 (given) 17 17 17
2

Area of square PQRS = æç


BX = 3 cm (given) 12 ö 144 2
÷ = m
Q ÐBAX = ÐCAX è 17 ø 17
and AX bisects BC. 144
So, 17 A = 17 ´ = 12
\ DABC is isosceles [QDBAX @ DCAX ] 17
\ AX ^ BC
\ AB = AC = 12
149. If cos2 q + 2 sin2 q + 3 cos2 q + 4 sin2 q ¼ + 200 sin2 q
= 10050, where q is an acute angle, find the value of
AX = 122 - 32 (sin q + 3 cos q )2 .
= 135 = 3 15 (a) 8 (b) 4 (c) 2 (d) 1
1 (a) We have,
\ Area of DABC = ´ 6 ´ 3 15 l
2 (cos2 q + 3 cos2 q + 5 cos2 q + ¼ + 199 cos2 q)
= 9 15 cm2 + (2 sin2 q + 4 sin2 q + ¼ + 200 sin2 q) = 10050
NTSE ~ SOLVED PAPER 2021 (Stage I) 21
Þ cos2 q(1 + 3 + ¼ 199) ar [COD] = c m2
+ 2 sin2 q(1 + 2 + 3 + ¼ + 100) = 10050 ar [ AOD] = b m2
100 ´ 101 1 1
Þ (100)2 cos2 q + 2 sin2 q ´ = 10050 Now, ar [ AOB] + ar[COD] = ´ AB ´ h1 + ´ DC ´ h2
2 2 2
Þ 104 cos2 q + 10100 sin2 q = 10050 1
= ´ AB ´ (h1 + h2 ) [Q AB = DC]
Þ 10000 cos2 q + 10000 sin2 q + 100 sin2 q = 10050 2
Þ 10000(cos2 q + sin2 q) + 100 sin2 q = 10050 1
a + c = ´ AB ´ AD …(i)
Þ 100 sin2 q = 50 [Qsin2 q + cos2 q = 1] 2
1 1 [Q h1 + h2 + AD]
sin2 q = Þsinq = Þ q = 45° 1
2 2 Similarly, ar [ AOD] + ar [BOC] = ´ AD ´ AB
\ (sin q + 3 cos q)2 = (sin45° + 3 cos 45° )2 2
1
2
Þ b + ar [BOC] = ´ AB ´ AD …(ii)
= æç
1 3 ö æ 4 ö
+ ÷= ç ÷ =8 2
è 2 2ø è 2ø
From Eqs. (i) and (ii),
150. A ‘p’ m long wire is cut into two pieces one of which Þ a + c = b + ar [BOC]
is bent into a circle and the other into a square Þ ar [BOC] = a + c - b
enclosing the circle. What is the radius (in meter) of 152. In DABC, a semi-circle with DE as diameter
the circle? is drawn such that BC = 26 m, the radius R
p 2p p 2 pp
(a) (b) (c) (d) (in meter) is
p+4 p+4 2p + 8 p+4 A
l (c) We have,
D C
75°

D E

R
45°
B C
A B
(a) 3 + 3 (b) 9 - 3 (c) 9 + 3 (d) 3 - 3
p=x+ y
Where, x = 2pR l (b) We have, DABC
y = 4a A
Now, 2r = side of square
75°
Þ 2r = a
Þ p = 2 pr + 4 a
Þ 2 pr + 8 r Þ p = r(2 p + 8) O
p D E
Þ r= m 45°
2p + 8 45°
45° R
151. In the figure, ABCD is a rectangle such that area of 45° 45° 60°
B C
DAOB = a m2, area of DAOD = b m2 , area of P Q S
DCOD = c m2. Then the area of DBOC (in m2) = Draw DP ^ BC, ES ^ BC
A B \ ¨ DOQP, ¨ ESQO will be a square of side R unit.
O \ PQ = R
In DBPD, BP = R unit [QÐDBP = ÐBDP = 45° ]
Similarly, QS = R units
In DESC,
D C
R R
(a) a + b + c (b) a + b - c (c) b + c - a (d) a + c - b tan60° = Þ 3 =
SC SC
l (d) We have, ABCD is a rectangle. R
SC =
A B 3
h1
\ BC = BP + PQ + QS + SC
h3 h4 R R
O 26 = R + R + R + Þ 26 = 3 R +
3 3
3 3R + R 26 3
Þ 26 = Þ =R
h2 3 3 3 +1
D C 26 3 (3 3 - 1)
Þ R= ´
ar [ AOB] = a m2 (3 3 + 1) (3 3 - 1)
22 NTSE ~ SOLVED PAPER 2021 (Stage I)

26 3 (3 3 - 1) 2 1 1
Þ R= = 3 (3 3 - 1) Þ = +
27 - 1 b a c
\ R=9 - 3 Þ 2ac = bc + ab
b+ a b+ c
Now, +
153. If the values of x in the roots of the equation b- a b- c
p(sin2 x ) + q( sinx ) + r = 0 are complementary, then b2 - bc + ab - ac + b2 + bc - ab - ac
=
(a) p = q(q + 2 r)
2
(b) q = p(p + 2 r)
2
b2 - bc - ab + ac
(c) r2 = q(q + 2 p) (d) r2 = p(q + 2 p) 2 b2 - 2 ac 2(b2 - ac)
= = 2 =2
l (b) Given, p(sin2 x) + q(sin x) + r = 0 b - (bc + ab) + ac
2
b - ac
According to the question,
[Qbc + ab = 2 ac]
Roots are sinq and sin(90 - q)
\ Sum of roots 156. If ( x + k) is a common factor of ( x 2 + px + q) and
= sin q + sin(90 - q) = -
q ( x 2 + lx + m) , then the value of k is
p (a) l + p (b) m - q
q l-p m- q
Þ sin q + cos q = - …(i) (c) (d)
p m- q l-p
r (d) We have, x2 + px + q = 0 …(i)
Product of roots = sin q × cos q = …(ii) l
p and x2 + lx + m = 0 …(ii)
q2 Q(x + k) is common factor of both
Þ sin2 q + cos2 q + 2 sin q cos q = [from Eq. (i)]
p2 \ x = - k will satisfy both
2 r q2 Þ k2 - pk + q = 0 …(iii)
Þ 1+ = 2 [from Eq. (ii)]
p p and k2 - lk + m = 0 …(iv)
p + 2 r q2 From Eqs. (iii) - (iv),
Þ = 2 Þ p (p + 2 r) = q2 (- p + l)k = m - q
p p
m- q
k=
154. The average age of all the 100 employees in an l-p
2
office is 29 yr, where employees are ladies. The 157. As a result of 40% hike in the price of rice per kg, a
5
person is able to purchase 10 kg less rice for ` 1400.
ratio of average age of men to women is 5 : 7. The
What was the original price of rice per kg?
average age of female employees is
(a) ` 50 (b) ` 60
(a) 18 yr (b) 35 yr
(c) ` 40 (d) ` 30
(c) 25 yr (d) None of these
2 l (c) Let’s say price of rice per kg = x
l (b) Female employee = of 100 = 40
5 After 40% hike, new price = x + 40% of x = 14
. x
\ Male employee = 100 - 40 = 60 Now, according to the question,
Total age 1400 1400
Q Average age = 29 = - = 10
100 x 14
. x
\ 2900 = x + y ...(i) 1400 æ 10 ö
Þ ç1 - ÷ = 10
Where, x = Total age of boys x è 14 ø
y = Total average age of girls 1400 4
Þ ´ = 10
x x 14
60 5 Þ x = 40
According to the question, = Þ 14 x = 15 y
y 7
40 158. A man takes half time in rowing a certain distance
From Eqs. (i) and (ii), downstream than upstream. What is the ratio of the
29 y speed of boat in still water to the speed of current?
2900 = Þ y = 1400
14 (a) 1 : 2 (b) 2 : 1 (c) 1 : 3 (d) 3 : 1
y 1400 l (d) Let speed of boat = x km/h
\ = = 35 yr
40 40 and speed of current = y km/h
\ Downstream speed = x + y
1 1 1 b+ a b+ c
155. If , , are in AP, then + equals Upstream speed = x - y
a b c b- a b- c According to the question,
(a) 1 (b) 2 1
b- c ab Time taken in downstream = of time taken in upstream
(c) (d) 2
a- b c d 1 d
Þ = ´
1 1 1 x+ y 2 x-y
l (b) Given, , , are in AP.
a b c
NTSE ~ SOLVED PAPER 2021 (Stage I) 23
Þ 2(x - y) = x + y 161. “God Save Our Noble King” is the National
2x - 2y = x + y Anthem of
x 3
x = 3y Þ = = 3 :1 (a) Britain (b) Italy
y 1
(c) Germany (d) France
159. The mean of a group of 20 consecutive natural l (a) “God Save Our Noble King” is the National Anthem of
numbers is M. What will be the percentage change Britain. It was first line sung in London in 1745.
in the mean when last six consecutive natural 162. Who was the ruler of France during the French
numbers are left from the group? revolution?
3M 3
(a) % (b) % (a) Louis XV (b) Louis XIV
100 M
300 30 (c) Louis XX (d) Louis XVI
(c) % (d) %
M M l (d) Louis XVI was the ruler of France during the French
revolution. The French revolution was occurred in the year
l (c) According to the question
n + (n + 1) + (n + 2) + ¼ + (n + 19) 1789. The Third class or common people launched
M= revolution against the king and other high classes owing
20
to large scale exploitation by them. The revolution led the
20 n + (1 + 2 + 3 + ¼ + 19)
M= end of Monarchy in France.The principle of “Liberty,
20
Equality and Fraternity” were main outcomes of the
19 ´ 20
20 n + French revolution.
M= 2
20 163. Hitler was born in
20 n + 190 19 (a) France (b) Austria
M= =n+
20 2 (c) Germany (d) Poland
Now, last six consecutive natural numbers are removed.
n + (n + 1) + ¼ + (n + 13) l (b) Hitler was born at Braunau am Inn in Austria on
\ New mean, Þ 20th April, 1889. He later moved to Germany. Then he was
14
14 n + (1 + 2 + ¼ + 13) recruited in the German Army. Because of his good work
N= he was promoted at the higher post. After the First World
14
War he formed the German Worker’s Party (Nazi Party)
13 ´ 14
14 n + and entered into politics. He became chancellor in 1933
2 14 n + 91
= = and ruled till 1945 as a dictator.
14 14
13 164. Who published ‘The folklores of southern India’?
N=n+
2 (a) Rabindranath Tagore (b) Natesa Shastri
M- N
Percentage change = ´ 100 (c) Mahatma Gandhi (d) Pattabhi Srimalu
M
æ n + 19 ö - æ n + 13 ö l (b) Natesa Shastri published ‘The Folklores of Southern
ç ÷ ç ÷ India”. It was collection of Tamil Folk tales. It was
è 2ø è 2 ø ´ 100
= published in four volumes. He was noted exponent of
æ n + 19 ö
ç ÷ Harikatha.
è 2ø
3 300 Rabindranath Tagore was a famous poet. He is renowned
= ´ 100 = % for his work ‘Gitanjali’. He received Nobel Prize for it.
M M
Mahatma Gandhi was a freedom fighter and he also wrote
160. One box contains four cards numbered 1, 3, 5 and 7 many books such as ‘Hind Swaraj, Indian Opinion’ etc.
and another box contains four cards numbered as 2,
165. A tax levied by the church, comprising one-tenth of
4, 6 and 8. One card is drawn from each box at
the agricultural produce was
random. The probability that the product of the
numbers so drawn is more than 14 is: (a) Livre (b) Taille
1 7 (c) Tithe (d) None of these
(a) (b)
2 10 l (c) A tax levied by the church, comprising one-tenth of the
3 5 agricultural produce was Tithe. It was a kind of religious
(c) (d)
8 8 tax.
l (a) Favourable events Livre was the currency of France from 1781 and 1794.
= {(3 ´ 6), (3 ´ 8), (5 ´ 4), (5 ´ 6), (5 ´ 8), Taille was a kind of direct tax imposed on the French
(7 ´ 4), (7 ´ 6), (7 ´ 8)} = 8 peasantry and non-nobles class. The tax amount was
decided on the basis of land they hold.
Total events = 4 ´ 4 = 16
Favourable events 166. What was not the aim of Swaraj Party?
\ Required probability =
Total events (a) To participate in Provincial Council election.
8 1
= = (b) To oppose British policies within the councils.
16 2
24 NTSE ~ SOLVED PAPER 2021 (Stage I)

(c) To demonstrate that councils were not democratic Withdrawl of all prosecutions except those involving
(d) To make the Act of 1919 successful. violent crimes.
l (d) The Swaraj Party was formed in 1923 by the Chitranjan Release of the political activists who were arrested for
Das and Motilal Nehru with in the fold of the Congress. taking part in the Civil Disobedience Movement.
It was formed with the aim to achieve following targets Salt tax was to be abolished.
To participate in Provincial council election. Simon Commission was formed in the year 1927 and it
To oppose British policies within the councils. visited India in 1928. It consisted of 7 members, headed by
To demonstrate that council were not democratic. Mr. Simon. It’s objective was to study the constitutional
Hence, option (d) is incorrect. reforms in India. Hence, option (c) is correct.

167. Who formed the secret society ‘Young Italy’? 171. In Swaraj Flag (1921), Self Help was represented
(a) Otto Van Bismarck by
(b) General Wang Run (a) Flower (b) Crescent Moon
(c) Giuseppe Mazzini (c) Two Oxen (d) Spinning Wheel
(d) King Victor Emmanual II l (d) In Swaraj Flag (1921), Self Help was represented by
l (c) The secret society ‘Young Italy’ was formed by the Spinning Wheel. This flag was designed by Mahatma
Giuseppe Mazzini in 1831. It’s main objective was to Gandhi. It was a tricolour flag, consisting of red, green
promote nationalism among the Italian people and create and white colour.
a United Italian Republic. During this time, Italy was ruled 172. Coffee cultivation was first introduced in
by different kingdoms. Later with the effort of Count
(a) Himalayas (b) Aravali Hills
Cavour and Garibaldi Italy was united in the year 1781.
(c) Garo Hills (d) Baba Budan Hills
168. The meaning of symbol ‘Crown of Oak Leaves’ was l (d) In India, coffee cultivation was first introduced in Baba
(a) Readiness to fight Budan Hills in Karnataka. Baba Budan Hills lies in the
(b) Heroism Western Ghats. Karnataka, Kerala and Tamil Nadu are
(c) Being freed main coffee producing states of India. The two well-known
(d) Willingness to make peace species of coffee grown are Arabica and Robusta.
l (b) The meaning of symbol ‘Crown of Oak Leaves’ was 173. Which one of the following groups of state have
‘heroism’. It stands for courage, bravery and heroism. largest number of cotton textile centres?
169. Which was achieved by the ‘Enabling Act of (a) Gujarat and Maharashtra
3rd March, 1933’ in Germany? (b) Karnataka and Tamil Nadu
(a) Established dictatorship (c) Maharashtra and Madhya Pradesh
(b) Hitler was appointed chancellor (d) Uttar Pradesh and Gujarat
(c) Establishment of German Workers’ Party
l (a) Gujarat and Maharashtra groups of state have largest
(d) Establishment of Weimar Republic number of cotton textile centres. These states are main
l (a) The ‘Enabling Act of 3rd March, 1933’ in Germany led centres of cotton textile industry owing to the large scale
to the establishment of dictatorship. The Act was initiated cotton production in these states. The main cotton textile
by the Adolf Hitler’s Nazi Party. With the enaction of this centres in Gujarat are Surat, Ahmedabad, Gandhinagar
Act, Adolf Hitler became dictator and assumed dictorial etc. The main centres of Maharashtra are Mumbai, Pune,
power. He reigned from 1933 to 1945 as a dictator. Nagpur and Solapur.

170. Which of the following was not related to Gandhi 174. Which one of the following describes a system of
Irwin Pact 1931? agriculture where a single crop is grown on a large
(a) Government agreed to release the political area?
prisoners. (a) Shifting Agriculture (b) Plantation Agriculture
(b) Gandhiji consented to participate in Second Round (c) Horticulture (d) Intensive Agriculture
Table Conference
(c) Formation of Simon Commission l (b) In Plantation Agriculture, a single crop is grown on a
(d) Gandhiji called off Civil Disobedience Movement large area. It is a capital intensive agriculture system. The
main examples of this type of plantation are banana,
l (c) The Gandhi-Irwin pact was signed in the year 1931.
coconut etc.
Lord Irwin was the then viceroy of India. The main
provisions of this Act are as follows Shifting Agriculture is mainly done in the tribal or under
developed areas. In this type, a patch of forest land is
The Indian National Congress (INC) will stop the Civil cleared and agriculture is done on it until it loses its
Disobedience Movement. fertility. Then, tribals move to another area.
The INC agreed to take part in the Round Table Horticulture refers to cultivation of fruits, vegetables etc.
Conference (held in London). In intensive agriculture a lot of money and labour are used
Withdrawl of all ordinances that curbed the activities of to increase the yield that can be obtained per unit land.
the congress.
NTSE ~ SOLVED PAPER 2021 (Stage I) 25
175. Neyveli lignite mines are located in the state of 181. According to the main role, industry can be divided
(a) Kerala (b) Tamil Nadu into
(c) Karnataka (d) Andhra Pradesh (a) Agro-based and mineral based
l (b) Neyveli lignite mines are located in the state of Tamil (b) Key and consumer industries
Nadu. Neyveli lignite mines are operated by the Neyveli (c) Public, private and joint sectors
Lignite Corporation (NCL) India Limited. Neyveli has the (d) Heavy and light industries
largest reserve of lignite in the country. Lignite is a type of l (b) On the basis of the role of the industry, it can be
coal which has very low content of carbon between divided into key and consumer industries. Former
25-35%. produces material which are used as raw material in other
176. Which of the following regions is suitable for industries while latter produces goods for the consumption
of people.
constructing railway lines?
On the basis of raw material, industries can be divided
(a) Himalaya Mountains Range
into agro and mineral based.
(b) Garo, Khasi and Jaintia Hills
On the basis of ownership, industries can be divided into
(c) The Northern Plains
public, private and joint sector.
(d) Rajasthan Desert
On the basis of weight, industries can be divided into
l (c) The Northern plains are most suitable for constructing heavy and light industries.
railway lines in comparison to Himalayan mountain range,
Garo, Khasi and Jaintia hills and Rajasthan desert. There 182. Which of the following is not a property of Mica?
are two main reasons behind it, first it has a flat (a) Conducting properties
topography which does not put any challenges in lying of (b) Excellent di-electric strength
railway tracts and second it is densily populated. Thus it (c) Less power, loss factor
has high demand of transportation facility. (d) Resistance to high voltage
The North Indian plains are spread in the states of Punjab, l (a) Mica has not conducting properties. The main
Haryana, Uttar Pradesh, Bihar, West Bengal etc. properties of Mica include excellent di-electric strength,
less power, loss factor and resistance to high voltage. It is
177. Hirakund Dam is constructed on the river
mainly used in paints, fillers, in electronic wire etc.
(a) Ganga (b) Manjira
(c) Manas (d) Mahanadi 183. Who was the chairman of the drafting committee of
l (d) Hirakund is constructed on the river Mahanadi. It is Indian Constituent Assembly?
located in the Sambalpur district of Odisha in India. It is (a) Dr. Rajendra Prasad (b) Dr. B.R. Verma
multipurpose dam. It was commissioned in the year 1957. (c) Dr. B.R. Menon (d) Dr. B.R. Ambedkar
It has a height of 61 m. l (d) B. R. Ambedkar was the chairman of the drafting
committee of Indian Constituent Assembly. The drafting
178. When and where first cement plant was set-up? committee was entrusted with the task of preparation of
(a) Chennai, 1905 (b) Chennai, 1904 draft of Constitution for India. The members of drafting
(c) Kolkata, 1905 (d) Kolkata, 1904 committee included K. M. Munshi, Muhammad Saadulah
l (b) The first cement plant in India was set-up in 1904 at Alladi, Krishnaswamy lyer,Gopala Swami Ayyangar, N.
Chennai (Tamil Nadu). The name of the company was Madhava Rao and T.T. Krishnamachari. The constituent
South India Industrial Ltd. Assembly was established in November 1946 on the
recommendation of Cabinet Mission plan.
179. Who wrote the book ‘Small is Beautiful’?
(a) Gandhiji (b) Brundtland 184. Amnesty International is an international
(c) Schumacher (d) Annie Besant organisation which works for
(a) World Peace
l (c) E.F. Schumacher wrote the book ‘Small is Beautiful’. It
(b) Human Rights
is a collection of essays. It was first published in the year
(c) World Justice
1973. Schumacher was a British economist.
(d) Restoration of Democracy
180. In which city of Haryana is the automobile industry l (b) Amnesty International is an international organisation
situated? which works for Human Rights. It was founded in the year
(a) Faridabad (b) Gurgaon 1961 by Peter Benenson and Eric Baker. Its headquarter is
(c) Panipat (d) Sonipat located in London.
l (b) In Haryana, automobiles industry is situated in 185. The movement that seeks equality in the personal
Gurgoan (It is now known as Gurugram). It lies in the and family life of women is known as
National Capital Region. The automobile industry started (a) Narivadi Andolan
in 1970’s when Maruti Suzuki established its (b) Nari Sashaktikaran Andolan
manufacturing plant in this city. It is also hub of (c) Mahila Shakti Andolan
information technology and financial and banking.
(d) Mahila Adhikar Andolan
26 NTSE ~ SOLVED PAPER 2021 (Stage I)

l (a) The movement that seeks equality in the personal and l (c) In case of conflict between the Centre Government and
family life of women is known as Narivadi Andolan. In the State Government over subject in the Concurrent list,
India, both men and women participated in the Narivadi the Central Government should be obeyed. The both
Andolan. The prominent people who led the Narivadi Central and State Governments can make laws on the
Andolan were Ishwar Chandra Vidyasagar, Jyotiba Rao subject of the Concurrent list.
Phule, Pandita Ramabai etc. The Constitution has divided legislation power between
186. A person who is not a member of Parliament is Centre and State into three subject lists Union, State and
Concurrent Central has exclusive power to make laws on
appointed on minister, he/she has to get elected to
the Union list whereas states have exclusive power to make
one of the houses of Parliament within
laws on the State lists.
(a) a month
(b) three months 190. What is Mid-Term Election?
(c) six months (a) Election for the seat to be vocated due to some
(d) selected time is fixed by the President reason.
(b) Election in the event of death of one member.
l (c) A person who is not a member of Parliament is (c) Election to be held in whole country or state before
appointed as Minister (it can be also be Prime Minister) the scheduled time.
then he/she has to get elected to one of the house of (d) Completion of 5 years election.
Parliament within six months. If not able to elect in any
l (c) Mid-term election is that election which is held in
house of the Parliament then he/she has to resign from
whole country or state before the scheduled time. It is also
his/her post. This principle is also applicable in the state
known by the name of election.
legislatures.
191. In India, seats are reserved for women in which of
187. Who become the members of Gram Sabha?
the following bodies?
(a) Only elderly people
(a) In Lok Sabha
(b) Only elected members of Gram Panchayat
(b) In State Legislatures
(c) All the voters of village
(c) In Rajya Sabha
(d) Only the youth of village
(d) In Panchayati Raj
l (c) All the voters of the village become members of Gram l (d) In India, seats are reserved for women in the
Sabha. A resident of Gram Sabha becomes voter when Panchayati Raj System. The 73rd Constitutional
he/she attains the age of 18 years. It is a primary body of Amendment Act, 1992 has provided that 33% of the seats
the Panchayati Raj System. All the decisions of the in the panchayats would be reserved for the women under
Panchayat are taken through the Gram Sabha. The term Article 243(D). Many states have extended this limit to
Gram Sabha is defined Article 243(b) in the Constitution. 50% such as Bihar, Gujarat etc.
188. President declares emergency when 192. Which famous Revolution took place in the world
(a) Prime Minister advises him to do.
history in 1789?
(b) Parliament advises him to declare.
(c) The council of minister, in writing advises him to (a) Russian Revolution (b) American Revolution
declare. (c) German Revolution (d) French Revolution
(d) Home minister asks him to declare. l (d) French revolution took place in 1789. The Russian
l (c) The President of India can declare emergency only revolution took place in 1917. American revolution
when council of minister, in writing advises him to occurred in 1775. German revolution took place in 1918.
declare. National emergency is declared under Article 352 193. Main Recommendation of Mandal Commission
of the Constitution. was
It can be declared on the grounds of ‘war’ or ‘external (a) Reservation for Socially and Educationally
aggression’ and armed rebellion. Every declaration is Backward Classes
required to belaid before each house of Parliament and (b) Reservation for Schedule Caste
has to be passed by either house of Parliament by a (c) Reservation for Schedule Tribes
majority of not less than two-third members of the houses. (d) Reservation for Minorities
l (a) The main recommendation of Mandal Commission
189. In case of conflict between the Centre Government was for the reservation for socially and educationally
and the State Government over subject in the Backward Classes. This commission was appointed in
Concurrent list 1979 by the Janata Party government under Prime
(a) Supreme Court decides which of the two should be Minister Morarji Desai. It submitted report in the year
considered. 1983. On the basis of this recommendation of commission
(b) The State Government should be obeyed. 27% reservation was granted to the other Backward
(c) The Central Government should be obeyed.
Classes.
(d) Both governments should be obeyed.
NTSE ~ SOLVED PAPER 2021 (Stage I) 27
194. Capital requirement of “NABARD” (National Bank 198. When an able person is willingly unemployed while
of Agriculture and Rural Development) are met by here is opportunity to work is known as
which of the following institutions? (a) Disguised unemployment
(a) Reserve Bank of India (b) Voluntary unemployment
(b) International Development Association (IDA) (c) Seasonal unemployment
(c) World Bank (d) Educated unemployment
(d) All of the above l (b) When an able person is willingly unemployed while
l (a) The capital requirement of “NABARD” (National Bank there is opportunity to work is known as voluntary
of Agriculture and Rural Development) are met by Reserve unemployment. It constitutes very less percentage of the
Bank of India. NABARD was established in the year 1982 overall employment.
on the recommendation of Shivaraman Committee. It was Disguised unemployment occurs when more people are
entrusted with the responsibility of matter concerning with employed in any work then actually required. Seasonal
the policy, planning and operation in the field of credit for unemployment occurs when people are unemployed
agriculture and other economic activities in rural areas. during particular season in the year.
195. Golden Revolution in India is related Educated unemployment occurs when person is not able
(a) Jewellery export (b) Gold mines to get employment according to his/her qualification.
(c) Honey and Horticulture (d) Electronic Goods 199. The production unit producing shoe comes under
l (c) Golden Revolution in India is related with the (a) Primary sector (b) Secondary sector
production of Honey and Horticulture. Other major and (c) Tertiary sector (d) None of these
Revolutions are as follows
l (b) The production unit producing shoe comes under
White - Milk secondary sector. All the manufacturing unit are included
Yellow - Oilseeds under the secondary sector.
Blue - Fisheries Primary sector encompasses the extraction and production
Golden Fibre - Jute of raw materials such as farming, logging etc.
Grey - Fertilizer Tertiary sector (also known as service sector) helps in
development of the primary and secondary sector. It
196. Which of the following is not a renewable resource?
includes banking, finance insurance etc.
(a) Forest (b) Animals
(c) Water (d) Petroleum 200. Which of the following prepares Human
l (d) Petroleum is not a renewable resource. Forests, Development Report?
animals, wind etc. are renewable resources. These are (a) Planning Commission of India
resources which can be replenished and infinite. On the (b) International Monetary Fund
other hand, non-renewable resources are exhaustable. (c) World Health Organisation
(d) United Nation Development Programme
197. Which of the following is not related to agriculture
marketing? l (d) Human Development Report is prepared by the United
(a) Storage Nation Development Programme (UNDP). In this Report,
UNDP publishes many index to compare position of
(b) Use of chemical fertilizers
different countries. The indices which are part of this
(c) Processing
Report are Human Development Index, Gender
(d) Preservation Development Index, Multidimensional Poverty Index,
l (b) Use of chemical fertilizer is not related to agriculture Gender Inequality Index and Inequality adjusted Human
marketing. Storage processing and preservation are Development Index. In Human Development Report,
parts/components of the agricultural marketing. 2020, India has been ranked at 131 position out of 189
countries.
NTSE ~ SOLVED PAPER 2020 (Stage I) 1

NTSE Solved Paper


NATIONAL TALENT
SEARCH
EXAMINATION
2020 (Stage I)

INSTRUCTIONS
This solved paper consists of two papers. Paper I consists of Mental Ability Test (MAT) and Paper II consists of
Scholastic Aptitude Test (SAT).
MAT covering (Q. Nos. 1-50) of Maths and (Q. Nos. 51-100) of Reasoning.
SAT consists of (Science, Mathematics and Social Science) which comprises 100 questions (40 Science, 20 Mathematics
and 40 Social Science).
There will be no negative marking.
Each correct answer will be awarded one mark.

Time : 240 Minutes Max. Mark : 200

n 15
\ Sum (Sn ) = [a + l] = [5 + 19]
Paper I Mental Ability Test (MAT) 2
15
2
= ´ 24 = 180
1. What is the sum of all positive factors of 256? 2
(a) 526 (b) 511 (c) 625 (d) 562 1 2 3
l (b) All the positive factors of 256 are 1, 2, 4, 8, 16, 32, 64, 4. If three numbers are in ratio : : , difference
2 3 4
128 and 256.
between largest and smallest is 27, then numbers are
\ Required sum
(a) 54, 72, 81 (b) 24, 45, 51 (c) 64, 72, 91 (d) 54, 65, 81
= 1 + 2 + 4 + 8 + 16 + 32 + 64 + 128 + 256 = 511
l (a) Ratio of three numbers
X X +1 1 1 2 3
2. Value of + - will be? = : : = 6 :8 :9
X +1 X X( X + 1) 2 3 4
(a) X 2 (b) 1 (c) X (d) 2 Let the largest number be 9x,the smallest number be 6x
and middle number be 8x.
X X +1 1
l (d) + - According to the question,
X +1 X X (X + 1)
9 x - 6 x = 27
X 2 + (X + 1)2 1
= - 3 x = 27
X (X + 1) X (X + 1)
Þ x =9
X2 + X2 + 1 + 2X - 1 2X2 + 2X \ Numbers are 6 ´ 9 = 54,
= =
X (X + 1) X (X + 1) 8 ´ 9 = 72 and 9 ´ 9 = 81
2 X (X + 1)
= =2 5. Which of the following number will completely
X (X + 1)
divide the value of (325 + 326 + 327 + 328 )?
3. Sum of sequence 5 + 6 + 7 + 8 + K + 19 will be? (a) 35 (b) 40
(a) 180 (b) 175 (c) 185 (d) 190 (c) 50 (d) 45

l (a) 5 + 6 + 7 + 8 + .... 19 l (b) 325 + 326 + 327 + 328 = 325 (1 + 3 + 9 + 27)


Q Given series is an AP. = 325 ´ 40
\ a = 5, l = 19, n = 15 Hence, 40 will divide the given value completely.
2 NTSE ~ SOLVED PAPER 2020 (Stage I)

6. Rohan’s score on the mid term exam was 75, and his 11. Some equations are based on the basis of a certain
score on the final exam was 90. If the weight of the system. Using the same pattern solve the unsolved
final exam is twice that of mid term, what is Rohan’s equation. If 10 - 3 = 12, 12 - 4 = 13, 14 - 5 = 14, what
final score in the course? is 16 - 6 = ?
(a) 82.5 (b) 80 (c) 85.5 (d) 85 (a) 10 (b) 15 (c) 16 (d) 18
75 ´ 1 + 90 ´ 2 l (b) 10 - 3 = 12 Þ10 - 3 + 5 = 12
l (d) Rohan’s final score = = 85
3
12 - 4 = 13 Þ12 - 4 + 5 = 13
7. A grandmother, mother and daughter wish to 14 - 5 =14 Þ14 - 5 + 5 = 14
arrange themselves in a row in order to be Similarly, 16 - 6 Þ16 - 6 + 5 = 15
photographed. How many different ways can they
12. Excluding stoppages, the speed of a bus is 54 km/h
arrange themselves?
and including stoppages, it is 45 km/h. For how
(a) 6 (b) 3 (c) 18 (d) 9
many minutes does the bus stop per hour?
l (a) Total number of members = 3 (Mother, Grand mother (a) 9 (b) 10 (c) 12 (d) 20
and daughter)
l (b) Speed of the bus = 54 km/h
Number of different ways to arrange = 3! Speed of bus including stoppages = 45 km/h
= 3 ´ 2 = 6 ways 54 - 45 9
The bus will stop for = h= ´ 60 = 10 min
8. At the time of marriage a man was 6 yr older 54 54
than his wife, but 12 yr after the marriage his age 13. If 40% of 1620 + 30% of 960 = ?% of 5200, the the
6 value of ? is
was times the age of his wife. Their ages (in years)
5 (a) 12 (b) 24 (c) 5 (d) 18
at the time of the marriage were
l (d) 40% of1620 + 30% of 960 = ?% of 5200
(a) 26, 20 (b) 24, 18 (c) 27, 21 (d) 30, 24 40 30 ? ´ 5200
Þ ´ 1620 + ´ 960 =
l (b) Let Man’s age at the time of marriage = x yr 100 100 100
Man’s wife’s age at the time of marriage = y yr Þ 4 ´ 162 + 3 ´ 96 = ? ´ 52
936
According to the question, x=y+6 … (i) \ ?= = 18
6 52
and (x + 12) = (y + 12)
5 14. In a row, 25 trees are planted at equal distance from
Þ 5 x + 60 = 6 y + 72 each other. The distance between 1st and 25th tree is
Þ 5 x - 6 y - 12 = 0 … (ii) 30m. What is the distance between 3rd and 15th tree?
On solving Eqs. (i) and (ii), (a) 8 m (b) 15 m (c) 16 m (d) 18 m
x = 24, y = 18
\ Man’s age at the time of Marriage = 24 yr l (b) Let the gap between a consecutive pair of tree = x m
\ Woman’s age at the time of Marriage = 18 yr There will be total 24 gaps.
Þ 24 x = 30
9. If we throw a dice, what is the probability of x=
30
obtaining a result that is less than 4, if we know that 24
the result obtained was an even number? Between 3rd and 15th tree = 12 gaps
1 2 1 4 30
(a) (b) (c) (d) \ Distance between 3rd and 15th tree = 12x = 12 ´ = 15 m
2 3 3 5 24
l (c) Total outcome (less than 4) = 3 (3, 2, 1) 15. In a school, the bell is rung once after each half
Favourable outcome (even number) = 1 (2) an hour. The school starts at 8 : 00 AM and close at
1
\ Required probability = 1 : 30 PM. The bell is rung 3 times continuously, at
3
the time beginning, at the time of lunch break at
10. There are 10 balls in a box, 5 white and 5 black. Two 10 : 00 AM and 10 : 30 AM and at the end. How
balls are removed randomly from the box, one after many times is the bell rung every day?
another. The first ball that is removed is black and it is (a) 21 (b) 22 (c) 19 (d) 20
not returned to the box. What is the probability that l (d)
the second ball that is removed is also black?
5 4 3 1 Time in AM 8:00 8:30 9:00 9:30 10:00 10:30 11:00 11:30 12:00 12:30 1:00 1:30
(a) (b) (c) (d)
9 9 9 2 Bell rung 3 1 1 1 3 3 1 1 1 1 1 3
(frequency)
l (b) Total balls = 10 (5 white + 5 black)
After removing first black ball, \ Bell rung for total 20 times.
Total balls left = 9 16. If 80% of A = 50% of B and B = x % of A, then value of
Total black balls = 4
4 x will be?
\ Required probability =
9 (a) 145 (b) 170 (c) 150 (d) 160
NTSE ~ SOLVED PAPER 2020 (Stage I) 3
l (d) 80% of A = 50% of B l (c) From option (c),
80 50 1 1 1
Þ ´ A= ´B = =
1 1 1
100 100 3+ 3+ 3+
B 8 1 1 æç13 ö÷
Þ = … (i) 1+ 1+
1 æç 9 ö÷ è9 ø
A 5 2+
x 4 è4 ø
Now, B = x % of A Þ B = ´A
100 1 13
= =
B æç 48 ö÷ 48
Þ x= ´ 100
A è13 ø
8
Þ x = ´ 100 = 160 [from Eq. (i)]
5 21. If for any two numbers a and b, the operation $ is
defined as follow
17. The mean of five consecutive numbers is 7. Which is
a$ b = a ´ (a + b), then (2$0)$1?
the highest number?
(a) 12 (b) 10 (c) 20 (d) 4
(a) 10 (b) 7 (c) 9 (d) 8
l (c) a$ b = a ´ (a + b)
l (c) Let the five consecutive numbers be
(n - 2), (n - 1), n, (n + 1), (n + 2) \ (2 $ 0)$1 = [2 ´ (2 + 0)]$1
\ Mean = 7 = 4 $1 = 4 ´ (4 + 1) = 4 ´ 5 = 20
(n - 2) + (n -1) + n + (n + 1) + (n + 2) 22. The accompanying figure shows a right + trapezoid
=7
5 (AD || BC). Based on this information and the
Þ 5 n = 35 information in the figure, the area of the trapezoid
n=7 (in m2 ) is
\ Highest number = n + 2 = 7 + 2 = 9 A 12m D
18. Find the value of x 3 + y 3 + z3 - 3 xyz, if x + y + z = 15
and x 2 + y 2 + z2 = 51. 8m 10m
(a) 540 (b) - 540 (c) - 225 (d) 765
l (b) We know, (x + y + z)2 = x2 + y2 + z2 + 2 (xy + yz + zx)
B C
Þ (15)2 = 51 + 2(xy + yz + zx) (given)
15 ´ 15 - 51 (a) 150 (b) 120 (c) 108 (d) 96
Þ xy + yz + zx = = 87
2 l (b) A 12m D
Now, x3 + y3 + z3 - 3 xyz
= (x + y + z) (x2 + y2 + z2 - xy - yz - zx) 10m
8m 8m
= 15 (51 - 87) = 15 ´ (- 36) = - 540

19. If area of any triangle is 384 cm 2 and its sides are in


ratio 3 : 4 : 5, then perimeter of triangle will be? B E 6m C
(a) 60 cm (b) 48 cm (c) 64 cm (d) 96 cm Let DE ^ BC, then BE = 12 m, DE = 8 m
l (d) Let the sides of the triangle be 3 x, 4 x and 5x respectively. In D CED, CE = CD2 - DE2 = 100 - 64 = 6 m
3x + 4x + 5x \ BC = BE + CE = 12 + 6 = 18 cm
Semi-perimeter (s) = = 6x 1
2 \ Area of trapezium = (sum of parallel sides) ´ Height
Area of triangle = s(s - a)(s - b)(s - c) 2
1 1
Þ 384 = 6 x ´ 3 x ´ 2 x ´ x = (BC + AD) ´ AB = (12 + 18) ´ 8 = 120 m2
2 2
Þ 384 = 6 x2 Þ x2 = 64
x = 8 cm
\ Perimeter = 3 x + 4 x + 5 x = 12x = 12 ´ 8 = 96 cm Directions (Q.Nos. 23-25) Find the missing number in the
number series.
13
20. is equal to 23. 4, 8, 28, ?, 244
48
1 1 (a) 69 (b) 75
(a) (b)
1 1 (c) 80 (d) 90
3+ 3+
1 1 (c) 4
1+ 1+ l 8 28 80 244
16 1
1+
8 ×3–4 ×3+4 ×3–4 ×3+4
1 1 \ ? = 80
(c) (d)
1 1
3+ 3+
1 1 24. 4, 7, 12, 19, 28, 39, ?
1+ 1+
1 8 (a) 48 (b) 52
2+
4 (c) 55 (d) 58
4 NTSE ~ SOLVED PAPER 2020 (Stage I)

l (b) 4 7 12 19 28 39 52 l (c) Let the required run rate be x.


\ 15 ´ 6 + x ´ 5 = 7.2 ´ 20
+3 +5 +7 +9 +11 +13 90 + 5 x =144
\ ? = 52 5 x = 54
x = 10.8
25. 10080, 1680, ?, 84, 28, 14
(a) 840 (b) 168 (c) 108 (d) 33 30. If P and Q are HCF and LCM of two Algebraic
l (d) 10080 1680 336 84 28 14 expressions x and y respectively and P + Q = x + y ,
then what will be value of P3 + Q3 ?
÷6 ÷5 ÷4 ÷3 ÷2
(a) x3 + y3 (b) x3 - y3 (c) x + y (d) x - y
\ ? = 336
l (a) We know that,
26. The compound interest on ` 30000 at 7% per annum Product of two expressions
is ` 4347. The period (in year) is = (HCF ´ LCM) of expressions
(a) 1 (b) 2 (c) 3 (d) 3.5 Þ xy = PQ ...(i)
l (b) CI = A - P and P+Q=x+ y …(ii)
A = CI + P = 4347 + 30000 Q (P + Q)3 = P3 + Q3 + 3 PQ (P + Q)
A = 34347 Þ P3 + Q3 = (x + y)3 - 3 xy(x + y) [from Eqs. (i) and (ii)]
T = x 3 + y3
A = Pæç1 +
R ö
Q ÷ 31. Pipe A and B can fill a tank in 12 min and 16 min
è 100 ø
T respectively. Both pipes are kept open for x minutes
Þ 34347 = 30000 æç1 +
7 ö and then B is closed and A fills the rest of tank in
÷
è 100 ø 5 min. The value of x will be
T T
34347 æ107 ö 11449 æ107 ö (a) 4 min (b) 6 min (c) 5 min (d) 7 min
Þ =ç ÷ Þ =ç ÷
30000 è100 ø 10000 è100 ø l (a) Q Pipe A can fill the tank = 12 min
2 T
Þ æç107 ö÷ = æç107 ö÷ Þ T = 2 yr and Pipe B can fill the tank = 16 min
è100 ø è100 ø According to the question,
x+5 x
27. Among the numbers 2, 3 9 , 4 16 , 5 32 the greatest + =1
12 16
one is 4 x + 20 + 3 x
Þ =1
(a) 2 (b) 3 9 (c) 4 16 (d) 5 32 48
1 1 48 - 20 28
Þ 7 x + 20 = 48 Þ x = = =4
l (b) 2 = (2)1 /2 = (23 )6 = (8)6 7 7
1 1
3
9 = (3)2 /3 = (34 )6 = (81)6 32. The accompanying figure shows right triangle ABC
1 1 and isosceles triangle ABD ( AB = AD).
4
16 = 2 = (2 ) = (64)
6 6 6

1 1
A
5
32 = 2 = (26 )6 = (64)6 α
\ The largest number is 3 9.
1 1
28. If x + = 2 and x is real, then the value of x 17 + 19
x x
2β β
is C
(a) 1 (b) 0 (c) 2 (d) - 2 B D
1 Based on this information and the information in the
l (c) x + =2
x figure, the value of angle a is
x2 + 1 = 2 x (a) 60° (b) 45° (c) 30° (d) 25°
x2 - 2 x + 1 = 0 l (c) From given figure,
(x - 1)2 = 0 A
x =1
α
1 1
Þ x + 19 = (1)17 + 19 = 2
17
90°–α
x (1)

29. To win a 20 over match, the run rate is required 7.2. 180°–2α
2β 2β β
If in the end of 15th over, the run rate is 6, then to C
win the match the required run rate is B D
(a) 1.2 (b) 13.2 In DABD,
(c) 10.8 (d) 12 90° - a + 2b + 2b = 180°
NTSE ~ SOLVED PAPER 2020 (Stage I) 5
Þ 4b - a = 90° … (i) 35. An official meeting is attended by 130 department
In DABC, employees of them 66 drink tea, 56 drink coffee
2b + b + 90° = 180° Þ 3b = 90° and 63 drink juice, 27 can drink either tea or coffee,
Þ b = 30° 25 can drink coffee or juice and 23 can drink juice
From Eq. (i), a = 4b - 90° = 4 ´ 30° - 90° (Qb = 30°) and tea. 5 employees can drink any of the three.
= 30° How many drink only tea?
(a) 21 (b) 22
33. The accompanying figure shows a circle whose
(c) 18 (d) 20
centre is O and radius is 10 cm. The shaded sector
1 l (a) Coffee
equal of the area of the circle. Based on this Tea
6
information and the information in figure the length 21 22 9
(in cm) of the arc AQB is 5
18 20
Q
20
Juice

O From the Venn diagram, 21 people drink only tea.


B
60°
36. Of the three numbers, the sum of first two is 55, sum
A of second and third is 65, and sum of third with
thrice of the first is 110. The third number is
P (a) 25 (b) 30 (c) 35 (d) 28
40 20 l (c) Let the numbers are x, y and z.
(a) 30p (b) p (c) p (d) 20p
3 3 x + y = 55 … (i)
l (b) Q
y + z = 65 … (ii)
z + 3 x = 110 … (iii)
From Eq. (i) – Eq. (ii),
Þ x - z = - 10
x = z - 10
O Put the value of x in Eq. (iii),
B
60° z + 3(z - 10) = 110
4 z = 140
A Þ z = 35
P
Directions (Q.Nos. 37-40) Study the following table and
1 answer the questions given below.
Q Shaded sector = ´ area of circle
6
360° Employees Source of Income (`)
\ Angle of shaded region = = 60° K L M N O
6
Now, angle in arc AQB = 360° - 120° = 240° Salary 12000 6000 21000 9000 12000
240° Bonus 2400 1200 4500 2400 3000
Length of arc AQB = ´ 2pr
360° Overtime 5400 2100 6000 5100 6000
240 40 p Arrears 6000 5400 12000 4200 7500
= ´ 2 ´ p ´ 10 =
360° 3 Miscellaneous 1200 300 1500 300 1500
Total 27000 15000 45000 21000 30000
34. If length of a rectangle is increased by 25% and its
width decreased by 20%, then of the following 37. The employee who has minimum ratio of income
which change in the area of rectangle occur? from arrear to income from salary is
(a) 10% increase (b) 16% increase (a) K (b) L (c) M (d) N
(c) 5% decrease (d) No change
38. The employee who earns maximum bonus in
l (d) Let length = x cm
comparison to his total income?
Let width = y cm (a) M (b) N (c) L (d) K
Original area = xy cm2
125 x 80 y 39. The employee who has maximum percentage of his
\ New area = ´ = xy
100 100 salary out of the income?
\ There is no change in the area. (a) K (b) L
(c) M (d) O
6 NTSE ~ SOLVED PAPER 2020 (Stage I)

40. The income from overtime is what percentage of the l (d) Different combinations of three course business lunch
income from the arrears in case of employee in = 3 ´ 4 ´ 2 = 24
category O?
43. If the length of a rectangular plot of land is
(a) 80 (b) 75 (c) 25 (d) 20 1
Sol. (Q.Nos. 37-40) increased by 12 % and the breadth is decreased by
2
6000 1 10%, its area is
37. (d) Ratio for K = = = 0.5
12000 2 (a) Decreased by 1.25% (b) Decreased by 2.5%
5400 9 (c) Increased by 2.5% (d) Increased by 1.25%
Ratio for L = = = 0.9
6000 10 a ´ bö
l (d) Percentage change in area = æç ± a ± b ± ÷%
Ratio for M =
12000 4
= = 0.57 è 100 ø
21000 7 é ´ 10 ù
25
4200 21 ê 25 2 ú
Ratio for N = = = 0.46 =ê - 10 - % [Qa = 25, b = -10]
9000 45 2 100 ú
ê ú
Hence, N has minimum ratio. ë û
2400 8 æ 5 5ö 5
= ç - ÷ % = % = 1 .25% increased
38. (b) Bonus for K = = = .08 è2 4 ø 4
27000 90
1200 4 44. K is an even number and P is an odd number.
Bonus for L = = = 0.08
15000 50 Which of the following statement is not correct?
Bonus for M =
4500
=
1
= 0.1 (a) P - K - 1is an odd number
45000 10 (b) P + K + 1is an even number
2400 8 (c) P ´ K + P is an odd number
Bonus for N= = = 0.11
21000 70 (d) P 2 + K 2 + 1 is an even number
Hence, N has maximum bonus. l (a) K is even and P is odd.
12000 From option (a),
39. (c) Percentage for K = ´ 100 = 44.44 % P - K - 1 = P - (K + 1)
27000 = even number
6000 Hence, option (a) is not correct.
Percentage for L = ´ 100 = 40%
15000
21000 45. All of the liquid filling a cubodial container that
Percentage for M = ´ 100 = 46.66% measures 2 cm ´ 10 cm ´ 20 cm is poured into a
45000
cylindrical container with a base radius of 5 cm.
12000
Percentage for O = ´ 100 = 40% What height (in cm) will the surface of the liquid
30000 reach in the cylindrical container?
Hence, M has maximum percentage. 16 40
(a) (b) (c) 8p (d) 8
6000 p p
40. (a) Required percentage = ´ 100 = 80%
7500 l (a) According to the question,
Volume of cuboidal = Volume of cylinder
41. The ratio of the present ages of Mohan and Suresh
Þ 2 ´ 10 ´ 20 = p ´ (5)2 ´ h
is 4 : 5. Five year ago, the ratio of their ages was 7 : 9.
Their present ages (in year) are [ h = height of the cylinder]
2 ´ 10 ´ 20
(a) 40, 50 (b) 18, 25 (c) 40, 60 (d) 20, 25 h=
25 ´ p
l (a) Let the present ages of Mohan and Suresh is 4x yr and 16
5x yr. \ h=
4x - 5 7 p
According to the question, =
5x - 5 9 46. (0° < q < 90° )
36 x - 45 = 35 x - 35 Þ x = 10 If tan q + cot q = 2, then what will be the value of
\ Age of Mohan = 4 x = 4 ´ 10 = 40 yr tan100 q + cot100 q ?
and Age of Suresh = 5 x = 5 ´ 10 = 50 yr 1
(a) 2 (b) 2 3 (c) 1 (d)
3
42. For a business lunch in a certain restaurant, you
many choose one of 3 different first courses and one l (a) tan q + cot q = 2
of 4 different main courses. In addition to first Þ x+
1
=2 [Let tan q = x]
course and the main course, you have a choice of a x
soup or dessert. How many different combinations Þ x - 2x + 1 = 0
2

of three course business lunch does this restaurant Þ (x - 1)2 = 0 Þ x = 1


offer? tanq = 1
(a) 12 (b) 14 (c) 18 (d) 24 tan100 q + cot100 q = (1)100 + (1)100 = 1 + 1 = 2
NTSE ~ SOLVED PAPER 2020 (Stage I) 7
47. What is the coefficient of a2 b2 in the expansion of 52. If Amit’s father is Ketan’s father’s only son and
( a + b)4 ? Ketan has neither a brother nor a daughter. What is
(a) 1 (b) 6 (c) 2 (d) 3 the relation between Ketan and Amit?
l (b) (a + b) = C0 a b + C1 a b + C2 a b + C3 ab3
4 4 4 0 4 3 4 2 2 4 (a) Uncle-Nephew (b) Father-Daughter
(c) Father-Son (d) Cousin
+ C4 a b 4 0 4

4! 4 ´ 3 ´ 2 ´1 l (c) only
Þ Coefficient of a b = C2 = 2 2
= 4
=6 son
2 !2 ! 2 ´2 Amit Father Ketan's father
(Ketan)
48. In a class composed of x girls, y boys. What part of From the above diagram, it is clear that Ketan and Amit
the class is composed of girls? are father and son.
x x y
(a) y( x + y) (b) (c) (d) 53. In a certain code language ‘si po re’ means ‘book is
xy ( x + y) xy
thick’, ‘ti na re’ means ‘bag is heavy’, ‘ka si’ means
x ‘interesting book’ and ‘de ti’ means ‘that bag’. What
l (c) Part of class, composed of girls =
x+ y should stand for ‘that bag is interesting’ in that code
language?
49. The expression 26 n - 42 n , where n is a natural
(a) ka re na ti (b) de si re ka
number is always divisible by (c) ti po ka na (d) de ti re ka
(a) 15 (b) 18 (c) 36 (d) 48
l (d) si po re ⇒ Book is thick
l (d) 26 n - 42 n
Þ 64 n - 16 n ti na re ⇒ bag is heavy
Divisible by 64 - 16 = 48 ka si ⇒ interesting book
[Qx n - an is always divisible by x - a ", n Î N]
de ti ⇒ that bag
1 2

50. If x = 2 - 2 + 2 , then the value of


3 3
\ that bag is interesting Þ de ti re ka
x 3 - 6 x 2 + 18 x + 18 is
54. In a certain language ‘PRINCIPAL’ is written as
(a) 22 (b) 33 (c) 40 (d) 45
‘MBOQSOMVW’ and ‘TEACHER’ is written as
l (c) x = 2 - 21 /3 + 22 /3 ‘FDVSZDB’. Then how is ‘CAPITAL’ written in that
(x - 2)3 = (22 /3 - 21 /3 )3 code?
Þ x3 - 8 - 3 ´ 2 ´ x(x - 2) (a) SVMOFVW (b) SVMODVW (c) BVMODVM (d) SVMIDVW
= 22 - 2 - 3 ´ 22 /3 ´ 21 /3 (22 /3 - 21 /3 ) l (a) As, P R I N C I P A L
Þ x - 8 - 6 x2 + 12 x = 4 - 2 - 6(x - 2)
3

Þ x3 - 6 x2 + 18 x = 14 + 8
M B O Q S O M V W
Þ x3 - 6 x2 + 18 x = 22
\ x3 - 6 x2 + 18 x + 18 = 22 + 18 = 40 and T E A C H E R
51. In this given figure how many triangle are there?
F D V S Z D B
Then, C A P I T A L

S V M O F V W

55. In a certain language ROPE is written as %57$,


(a) 12 (b) 10 (c) 14 (d) 8 DOUBT is written as 35#8* and LIVE is written as
l (a)
@24$. How is TROUBLE is written in that code?
A B C
(a) *%5#8@$ (b) *%#58@$ (c) *%5#8@4 (d) *%58$@
l (a) As, R O P E , D O U B T
G H
% 5 7 $ 3 5 # 8 *
and L I V E
F E D
There are total 12 triangles in the figure
DAGB, DAFG, DFGE, DBCH, DCHD, DDEH, @ 2 4 $
DAEC, DFBD, DAFB, DAFE, DBCD, DECD.
8 NTSE ~ SOLVED PAPER 2020 (Stage I)

Then, T R O U B L E 60. Anupriya was born on 29th November, 1970, which


was Sunday. When her next birthday will fall on
Sunday?
* % 5 # 8 @ $
(a) 1975 (b) 1976 (c) 1981 (d) 1982
56. If $ means ‘plus (+)’, # means ‘minus (-)’, @ means l (c) Anupriya was born on 29th November 1970 which is
multiplied (´), and * means ‘divided (¸)’, then what Sunday.
is the value of 16$4@5#72*8? From 29th November 1970 to 29th November 1981,
(a) 29 (b) 25 (c) 27 (d) 36 total odd days = 8 ´ 1 + 3 ´ 2
l (c) $ Þ+ = 8 + 6 = 14 days = 0 days
# Þ- Hence, on 29th November 1981, there will be Sunday.
@ Þ´ 61. Which one will replace the question mark?
* Þ¸
1 4 6
\ 16 $ 4 @ 5 # 7 2 * 8 9 5 5 3 8 3
Þ 16 + 4 ´ 5 - 72 ¸ 8 Þ16 + 20 - 9 Þ27 651 246 ?
57. In the number ‘5321648’ how many digit will be as 3 4 7 9 2 1
far away from the beginning of the number if digit 6 8 4
arranged in ascending order as they are in the (a) 262 (b) 622 (c) 631 (d) 824
number?
l (b) As, in first figure, 9 - 3 = 6
(a) Three (b) One (c) Two (d) None
6 -1 = 5
l (b) Given digits ⇒ 5 3 2 1 6 4 8 5 - 4 =1
ascending order ⇒ 1 2 3 4 5 6 8 In second figure, 7 - 5 = 2
8 -4 = 4
\ Only one digit ‘8’ is the required digit which is as far 9 -3 =6
from the beginning as when the digits are arranged in
ascending order. \ In third figure, 8 - 2 = 6
6 - 4 =2
58. In a class of 35 students Kunal is placed seventh 3 -1 = 2
from the bottom whereas Sonali is placed ninth from \ ? = 622
top. Pulkit is placed exactly in between the two.
What is Kunal’s position from Pulkit? 62. If ‘+’ means ‘¸’, - means ‘´’, ‘´’ means ‘+’ and ‘¸’
(a) 9th (b) 10th (c) 11th (d) 12th means ‘-’, then, 4 + 6 ´ 9 ¸ 6 - 2 ´ 5
4 8 9
l (b) Kunal’s position from the top (a) (b) (c) 2 (d)
6 3 2
= (Total number of students
- Kunal’s position from bottom ) +1
l (b) + Þ ¸, - Þ ´, ´ Þ + and ¸ Þ -
\ 4 + 6 ´9 ¸6 -2 ´5
= (35 - 7) + 1 = 29th
Þ 4 ¸ 6 + 9 -6 ´ 2 + 5
Sonali’s position from top = 9th
2
\ Number of students between Kunal and Sonali Þ + 9 - 12 + 5
= (29 - 9) - 1 = 19 3
19 + 1 2 8
Now, Pulkit’s position from Sonali = = 10th Þ +2 Þ
2 3 3
\ Pulkit’s position from the top = 9 + 10 = 19th
Directions (Q.Nos. 63-66) In the questions given below a
Hence, Kunal is 10th from Pulkit.
piece of paper is folded and cut as shown below in
59. In a row of girls facing North, Reena is 10th to the question figures, from the given answer figures indicate
left of Pallavi, who is 21st from the right end. If how it will appear when opened.
Malini, who is 17th from the left end is fourth to the 63. Question Figures
right of Reena. How many girls are there in a row?
(a) 37 (b) 43
(c) 44 (d) Data Inadequate
l (b) Malini
17th
Reena Pallavi Answer Figures

31st 21st

27th
Malini’s position 17th from left and 27th from right. (a) (b) (c) (d)
\ Total number of girls = 17 + 27 - 1 = 43
NTSE ~ SOLVED PAPER 2020 (Stage I) 9
l (a) The folded and cut piece of paper when unfolded will l (c) The folded and punched piece of paper when unfolded
look like as answer figure (a). will look like as answer figure (c).

64. Question Figures 67. In the matrix below, the numbers in the cells follow
some rules. Identify the number which will be
substituted for ? maintaining the same rule.?
4 1 2
13 11 6
Answer Figures 153 120 ?

(a) 32 (b) 45 (c) 16 (d) 48


l (a) As, in 1st column, (13)2 - 42 = 153
And, in 2nd column, (11)2 - 12 = 120
(a) (b) (c) (d) Similarly, in 3rd column, 62 - 22 = 32

l (d) The folded and cut piece of paper when unfolded will Directions (Q.Nos. 68-72) The Venn diagram given below
look like as answer figure (d). is about a small circle is Marathi, triangle is Bihari and
square is Punjabi.
Bihari

65. Question Figures 1 7

2 3
5
6 Marathi
Punjabi 8
4
Answer Figures
68. What is the total number of Biharis?
(a) 5 (b) 6 (c) 7 (d) 8
69. What is the total number of Punjabis?
(a) (b) (c) (d) (a) 22 (b) 28 (c) 29 (d) 35
70. What is the total number of Marathis?
l (a) The folded and punched piece of paper when unfolded
will look like as answer figure (a). (a) 20 (b) 15 (c) 22 (d) 21
71. How many Bihari which are not Punjabi?
(a) 1 (b) 2 (c) 3 (d) 4
72. How many Punjabi which are not Marathi?
(a) 10 (b) 11 (c) 12 (d) 13
66. Question Figures Sol. (Q.Nos. 68-72)
68. (b) Total number of Bihari = 1 + 2 + 3 = 6
69. (a) Total number of Punjabi = 1 + 7 + 3 + 5 + 6 = 22
70. (d) Total number of Marathi = 3 + 6 + 8 + 4 = 21

Answer Figures 71. (b) Number of Bihari’s who are not Punjabi = 6 - 4 = 2
72. (d) Punjabi who are not Marathi = 22 - 9 = 13

73. India became a republic on 26th January, 1950.


Which day of the week was it?
(a) (b) (c) (d) (a) Monday (b) Tuesday (c) Thursday (d) Saturday
10 NTSE ~ SOLVED PAPER 2020 (Stage I)

l (c) Odd days for 1600 yrs = 0 77. How many cubes have three faces coloured?
Odd days for 300 yrs = 1 (a) 4 (b) 6 (c) 8 (d) 12
Odd days for 49 yrs = (12 ´ 2 + 37 ´ 1) = 61
78. How many cubes have two faces coloured?
(Since, there are 12 leap years and 37 ordinary years in
the period of 49 yrs) (a) 8 (b) 16 (c) 36 (d) 24
Odd days for 26 days of Jan 1950 = 5 79. How many cubes have only one face coloured?
\ Total odd days = 0 + 1 + 61 + 5 = 67 days (a) 16 (b) 24 (c) 32 (d) 36
= (9 ´ 7 + 4) days = 4 odd days
Sol. (Q.Nos. 77-79)
\ Number of odd days = 4
\ Required day = Thursday
74. At what angle (larger) are two hands of a clock
inclined at 48 min past 12?
(a) 264° (b) 263° (c) 265° (d) 266°
l (a) Time from 4 am of a particular day to 3 am on 4 day is
7 h. Now, the clock loses 20 min in 24 h or in other words, Edge of larger cube 8
n= , n= =4
we can 5 day that 23h 40 min of this clock is equal to 24 h Edge of smaller cube 2
of the correct clock.
77. (c) There will be 8 cubes having 3 faces coloured.
æç23 + 40 ö÷ = 71 h of this clock = 24 h of the correct clock
è 60 ø 3 78. (d) Number of smaller cubes having two faces coloured
24 ´ 3 = 12 (n - 2) = 12 (4 - 2) = 12 ´ 2 = 24
\ 71 h of this clock = æç ´ 71ö÷ h of the correct clock. \ There will be 24 cubes having 2 faces coloured.
è 71 ø
= 72 h of the correct clock. 79. (b) Number of smaller cubes having one face coloured
or 71 h of this clock = 72 h of the correct clock. = 6(n - 2)2 = 6(4 - 2)2 = 6 ´ 2 ´ 2 = 24
Therefor, it is clear that in 72 h this clock loses 1h \ 24 cubes will have only one face coloured.
and hence, the correct time is am when this clock show 80. Choose the correct option to complete the matrix?
3 am.
4C 2B 3A
75. A clock is set right at 4 am. The clock loses 20 min
in 24 h. What will be the time, when the clock 28A 10C 45B
indicate 3 am on 4th day? 7C ? 15B
(a) 5 am (b) 4 am (c) 3 am (d) 4 pm
l (b) Time from 4 am of a particular day to 3 am on 4th day (a) 15A (b) 12B (c) 5A (d) 8C
is 71 h. Now, the clock loses 20 min in 24 h or in other l (c) In each row, there are three alphabets one in each box
words, we can say that 23 h 40 min of this clock is equal to which are A, B and C. So, the missing alphabet is A.
24 h of the correct clock. In first column, 4 ´ 7 = 28
æç23 + 40 ö÷ = 71 h of this clock = 24 h of the correct clock In third column, 15 ´ 3 = 45
è 60 ø 3
\ In second column, x ´ 2 = 10
24 ´ 3
\ 71 h of this clock = æç ´ 71ö÷ h of the correct clock x =5
è 71 ø
\ ?=5A
= 72 h of the correct clock
or 71 h of this clock = 72 h of the correct clock 81. Which of following is the best represented in
Therefore, it is clear that in 72 h this clock loses 1 h and diagram?
hence, the correct time is 4 am when this clock shows 3 am. (a) Chair, Table, Furniture
(b) Doctor, Social Person, Honest Person
76. A dice has four different positions. Find the number (c) Family, Parents, Children
on the face opposite to 3.
(d) Gold Jewellary, Silver Jewellary,
5 2 3 4 Ornaments
6 4 5 5
4 1 6 1 l (b) Some doctors can be social persons,
some social persons can be honest persons, some honest
(a) 1 (b) 2 (c) 4 (d) 6 persons can be doctors. Also, some doctors can be both
social persons and honest persons.
l (c) In position (i) and (iii) 5 and 6 are common.
\ Number ‘4’ will be on the face opposite to face having
number ‘3’. Honest Doctor
person
Directions (Q.Nos. 77-79) are based on given information.
A solid cube is painted red on all faces. The side of the
cube is 8 cm. It is cut into smaller cubes of side 2 cm. Social
person
Answer the following questions.
NTSE ~ SOLVED PAPER 2020 (Stage I) 11
Directions (Q.Nos. 82-84) Study the letter series given 88. Choose the correct mirror image of figure (X) from
below and answer the questions that follows. given alternatives.
HDYSMWNBQPOCRTBLZVEGUF
82. Which two neighbours in the given arrangement are
farthest in the alphabetical order?
(a) B and Q (b) D and Y (c) U and F (d) V and E (X)
83. Which letter has the same neighbours as in the
alphabetical order through they have change places?
(a) M (b) N (c) O (d) F

84. Which three letters have the same distance as they (a) (b) (c) (d)
have in the alphabetical order through they have
changed places? l (b) The figure given in the option (b) is the correct mirror
image of the figure given in the question.
(a) HMP (b) NQZ (c) QOE (d) YLF
Sol. (Q.Nos. 82-84) 89. Choose the correct water image of figure (X) from
given alternatives.
82. (b) D and Y are farthest as they have a difference of 21
places between them and all other have lesser difference.
83. (c) In the given series ‘P’ is the neighbour of ‘O’.
84. (d) YLF
Y – L = 13 places (X)
L – F = 6 places

85. A and B are sisters. R and S are brothers. Daughter


of A is sister of R. Then which relation is between
B and S? (a) (b) (c) (d)
(a) Aunt (b) Grand Mother (c) Sister (d) Mother
l (a) The figure given in the option (a) is the correct water
l (a) er
Sister
B image of the figure given in the question.
u ght A
Da
O
Sis
ter Brother
R S
From above diagram it is clear that ‘B’ is the Aunt of ‘S’.
90. What is the minimum number of straight lines
86. Abhay is the husband of Neena and Sunita is the needed to construct the following figure?
mother of Abhay. Sohan is the paternal uncle of
Neeraj, who is the son of Abhay. What is the relation
between Sohan and Neena?
(a) Jeth (b) Devar (c) Bhatija (d) Jeth/Devar
l (d) Sunita
Mother
Sohan Abhay Husband
Neena
Brother
Son
Neeraj
(a) 13 (b) 15 (c) 16 (d) 17
Hence, Sohan is either Jeth or Devar of Neena.
l (c) B D
87. Which one will replace the question mark?
A K C M
4 8 5 E

O P
6 14 6 14 6 ?

8 8 4 J F
L H N
10 18 14 22 11 15
(a) 8 (b) 14 (c) 10 (d) 6 I G
l (c) As, in first figure,18 - 4 = 14 There will be minimum 16 lines required to construct the
In second figure, 22 - 8 = 14 figure, which are AJ, KL, CH, MN, EF, AE, JF, AB, JD, IE,
Similarly, in third figure,15 - 5 = 10 GF, BF, DE, AG, JI, OP.
12 NTSE ~ SOLVED PAPER 2020 (Stage I)

Directions (Q.Nos. 91-94) A cube is coloured red on all of 97. Which of the following means that R is the wife of P?
its faces. It is then cut into 64 smaller cube of equal size. (a) P ´ R - Q - T (b) P ¸ T + R - Q
The smaller cube so obtained are now separated. (c) P ¸ R - Q + T (d) P ´ T - Q + R
91. How many smaller cubes have no surface coloured? 98. If ‘P ´ T ¸ Q + R’, how is R related to P?
(a) 24 (b) 16 (c) 8 (d) 10 (a) Daughter (b) Husband
(c) Son-in-law (d) Daughter-in-law
92. How many smaller cubes will have atleast two
surfaces painted with red coloured? 99. If ‘P ¸ R – Q ´ T, how is P related to T?
(a) 4 (b) 18 (a) Grandmother (b) Mother-in-law
(c) 32 (d) 24 (c) Sister (d) Grandfather

93. How many smaller cubes have two surfaces painted 100. If P ¸ Q + R ´ T, how Q is related to T?
with red coloured? (a) Aunt (b) Sister
(a) 24 (b) 8 (c) Brother (d) None of these
(c) 12 (d) 20 Sol. (Q.Nos 96-100)
94. How many smaller cubes have only three surfaces 96. (d) P + Q - R ¸ T
painted with red coloured? P Q brother R mother T
daughter
(a) 0 (b) 12 (c) 24 (d) 8
Sol. (Q.Nos. 91-94)
cousins
Here, n = 3 64 = 4
Hence, T is the Cousin of P.
91. (c) Number of cubes with no surface coloured = (n - 2)3
97. (d) P ´ T - Q + R
= (4 - 2)3 = 23 = 8
P T Q daughter R
92. (c) Number of cubes with two surfaces painted father brother
= (n - 2) ´ 12 = (4 - 2) ´ 12 = 24
Three surfaces painted = 8 wife
Total = 24 + 8 = 32
98. (c) P ´ T ¸ Q + R
\ Number of cubes with atleast two surfaces painted = 32
wife
93. (a) Number of smaller cubes with two surfaces painted
with red colour P T Q daughter R
father mother
= (n - 2) ´ 12 = (4 - 2) ´ 12 = 24
94. (d) As, three surfaces painted cubes are always 8.
son-in-law
95. A 6 cm cube is cut into 2 cm smaller cube. How Hence, R is the son-in-law of P.
many smaller cubes can be obtained from these?
99. (a) P ¸ R - Q ´ T
(a) 108 (b) 156 (c) 27 (d) 64
mother brother father
3 P R Q T
(c) Total number of smaller cubes = æç ö÷ = 33 = 27
6
l
è2 ø

grand mother
Directions (Q.Nos. 96-100) Read the following
informations and answer the questions which follow. Hence, P is the grand mother of T.
‘A ´ B’ means ‘A’ is father of ‘B’ 100. (b) P ¸ Q + R ´ T
‘A + B’ means ‘A’ is daughter of ‘B’ mother daughter father
‘A ¸ B’ means ‘A’ is mother of ‘B’ P Q R T
‘A - B’ means ‘A’ is brother of ‘B’
sister
96. If P + Q - R ¸ T, how is T related to P?
(a) Aunt (b) Brother (c) Father (d) Cousin Hence, Q is the sister of T.
NTSE ~ SOLVED PAPER 2020 (Stage I)

Paper II Scholastic Aptitude Test (SAT)


101. A bomb of mass 30 kg at rest explodes into two l (a) Person cannot see clearly distance more than 40 cm,
pieces of masses 18 kg and 12 kg. The velocity of then
18 kg mass is 6 m/s. The kinetic energy of the other image distance, v = − 40 cm and
mass is object distance, u = ∞
(a) 324 J (b) 486 J (c) 256 J (d) 524 J Hence, using the lens formula,
1 1 1
l (b) Mass of bomb, M = 30 kg − =
v u f
After explosion of bomb, masses of two pieces are given as
1 1 1
m1 = 18 kg, m2 = 12 kg ⇒ − =
Velocity of 18 kg mass, v1 = 6 m/s − 40 ∞ f
If v2 be the velocity of 12 kg mass, then using law of ⇒ f = − 40 cm
conservation of linear momentum, 100 100
∴ Power, P = = = − 2 .5 D
m1 v1 + m2 v2 = Mv f (incm) − 40
⇒ 18 × 6 + 12 × v2 = 30 × 0 [Q v = 0]
⇒ 12 v2 = 108 104. Gravitational force is essentially required for
108 (a) stirring in liquid (b) convection
⇒ v2 =
12 (c) conduction (d) radiation
⇒ v2 = 9 m/s l (b) The driving force for natural convection is gravity. For
∴Kinetic energy of 12 kg mass, example, if there is a layer of cold dense air on top of
1 1 hotter less dense air, then gravity pulls more strongly on
K = m2 v22 = × 12 × (9)2 the denser layer on top, so it falls while the hotter less
2 2
dense air rises to take its place. This creates circulating
= 6 × 81 = 486 J flow i.e., convection.
102. A body initially at rest starts moving when a Hence, gravitational force is essentially required for convection.
constant external force F is applied on it. The force F 105. An observer moves towards a stationary plane
is applied for time t = 0 to time t = T . Which of the mirror at a speed of 4 m/s, the speed with which his
following graph represents the variation of the image moves towards him is
speed (v) of the body with time (t)? (a) 2 m/s (b) 4 m/s
(c) 8 m/s (d) Image will stay at rest
l (c) Speed of observer towards plane mirror,
(a) v (b) v u = 4 m/s
Speed of image = 2u
t T T t = 2 × 4 = 8 m/s

106. If the ammeter in the given circuit reads 2 A, then


what is the value of resistance R (the resistance of
v v
ammeter is negligible)?
(c) (d)
3Ω
R
T t T t
6Ω
l (c) When a constant external force is applied on the body,
then constant acceleration is produced according to 6V
Newton’s second law of motion.
A
F
i.e., Acceleration, a = = constant
m (a)1 Ω (b) 2 Ω (c) 3 Ω (d) 4 Ω
From first equation of motion,
l (a) Given, I =2A
v = u + at
Q v = 0 + at Equivalent resistance of the circuit,
⇒ v = at [Qu = 0] 3Ω
i.e., v ∝t
R
Since, v is proportional to time t (for 0 to T), hence graph
shown in option (c) is correct. 6Ω

103. A person cannot clearly see objects at a distance I


more than 40 cm. He is advised to use lens of power
(a) − 2.5 D (b) 2.5 D (c) −15
. D (d) 15
. D A
6V
14 NTSE ~ SOLVED PAPER 2020 (Stage I)

3 ×6
R′ = 3 ||6 + R = + R 109. A concave mirror of focal length 15 cm forms an
3+6 image. The position of the object when the image is
18 virtual and linear magnification is 2, is
= + R
9 (a) 22.5 cm (b) 7.5 cm (c) 30 cm (d) 45 cm
⇒ R′ = 2 + R
l (b) Given, focal length of concave mirror,
∴By Ohm’s law,
f = − 15 cm
V = IR′
Linear magnification, m = 2
V
⇒ I= −v
R′ Q As, m= =2
6 −u
⇒ 2=
2+ R ⇒ v = + 2u … (i)
⇒ 4 + 2R = 6 Using mirror formula,
⇒ 2R = 2 ⇒ R = 1 Ω 1 1 1
+ =
v u f
107. A particle starts its motion from rest under the 1 1 1 1 1
⇒ + = − =
action of a constant force. If the distance covered in + 2u (− u) − 15 2u − 15
first 10 s is s1 and that covered in next 10 s is s2 ,
⇒ 2u = 15 ⇒ u = 7.5 cm
then
(a) s2 = 6s1 (b) s2 = 2 s1 1
110. A body on an inclined plane slides down th of
(c) s2 = 8s1 (d) s2 = 3s1 4
l (d) Initial velocity of particle, u = 0 distance in 2 s. It will slide down the complete
Particle moves with constant acceleration a under the distance along the plane in (the inclined plane have
action of constant force. zero friction)
According to first condition, (a) 4 s (b) 5 s (c) 2 s (d) 3 s
distance travelled by particle in first10 s (t1 = 10 s ) = s1 l (c) The given situation is shown in the following figure
1 below :
i.e., s1 = ut1 + at12
2
1
= 0 × 10 + × a × (10)2 α A
in
2 gs
s1 = 50 a … (i) s B s/4 g g cos α
According to second condition, distance travelled by the /4
particle in next 10 s (t2 ) = s2 3s
1 1
i.e., s2 = vt2 + at22 = v × 10 + × a × 102 α
2 2 O
s2 = 10 v + 50 a … (ii) s
where, v is the velocity of particle after t1 = 10 s. AB =
4
Now, v = u + at1 = 0 + a × 10 3s
⇒ v = 10 a … (iii) BO =
4
From Eqs. (ii) and (iii), we get s
Time taken to cover distance,
s2 = 10 × 10 a + 50 a 4
s2 = 150 a t1 = 2 s
= 3 × 50a ∴From second equation of motion,
= 3 s1 [from Eq. (i)] s 1
= 0 × t1 + g sinα (t12 )
⇒ s2 = 3 s1 4 2
s 1
= 0 + g sinα(2)2
108. Two planets of radii r1 and r2 are made from the 4 2
same material having same density. The ratio of ⇒ s = 8 g sinα … (i)
acceleration due to gravity g1 / g2 at the surfaces of Velocity of body at point B is given as
the planets is v = 0 × t + g sinα ⋅ t1 = gsinα ⋅ 2
(a) r1 / r2 (b) r2 / r1 (c) (r1 / r2 )2 (d) (r2 / r1 )2 v = 2 g sinα … (ii)
3s
l (d) Acceleration due to gravity on the surface of planet is If t2 be the time to cover distance , then by second
4
given as
equation of motion,
GM
g= 2 3s 1
r = vt2 + g sinα (t2 )2
4 2
1 3 1
i.e., g∝ 2 ⇒ × 8 g sinα = 2 g sinα(t2 ) + g sinα (t2 )2
r 4 2
2
⇒ 12 = 4t2 + t2 2

g1  r2  ⇒ t22 + 4t2 − 12 = 0
⇒ = 
g2  r1  t22 + 6t2 − 2t2 − 12 = 0
NTSE ~ SOLVED PAPER 2020 (Stage I) 15
⇒ t2 (t2 + 6) − 2(t2 + 6) = 0 V100 = V = 5 V
⇒ (t2 + 6) (t2 − 2) = 0 V 5 1
∴ I1 = = = A
As, t2 ≠ − 6, 100 100 20
∴ t2 − 2 = 0 ⇒ t2 = 2 s 1
⇒ I1 = A … (i)
20
111. When four equal resistors are connected in series V 5
Similarly, I2 = = A … (ii)
with a battery, they dissipate power of R R
10W. The power dissipated through any of them, if where, R is the resistance of voltmeter.
connected across the same battery will be ∴Equivalent resistance of the circuit,
(a) 40 W (b) 10/3 W (c) 90 W (d) 10 W R′ = R ||100 + 50
l (a) Total dissipated power, P = 10 W R′ =
100 R
+ 50
If R be the resistance of each resistor, then equivalent 100 + R
resistance in series combination, 150 R + 5000
=
R′ = R + R + R + R 100 + R
⇒ R′ = 4 R 10
∴ I = I1 + I2 =
V2 V2 R′
∴ Power, P = ⇒10 =
R′ 4R ⇒
1 5
+ =
10
⇒ V 2 = 40 R … (i) 20 R 150 R + 5000
Power dissipated through one resistor, 100 + R
V 2 40R R + 100 10(100 + R)
P1 = = = 40 W =
R R 20 R 150 R + 5000
1 10
112. An electron is moving with velocity v in a uniform ⇒ =
20 R 150 R + 5000
magnetic field B. The magnetic force experienced by
the electron is ⇒ 150 R + 5000 = 200 R
(a) always zero ⇒ 50 R = 5000 ⇒ R = 100 Ω
(b) never zero 114. Which of the following contain seven molecules of
(c) zero, if v is perpendicular to B water of crystallisation?
(d) zero, if v is parallel to B (a) Epsom salt (b) Green vitriol
l (d) Magnetic force on electron (of charge e) moving with (c) Blue vitriol (d) White vitriol
velocity v in magnetic field B, l (a,b,d) Green vitriol epsom salt and white vitriol contain
F = Bev sinθ seven, molecules of water of crystallisation. The chemical
When θ = 00 i.e., v is parallel to B, then sin0 ° = 0 formula for green vitriol is FeSO4 ⋅ 7H2 O. Epsom salt is
∴ F=0 MgSO4 ⋅ 7H2 O and white vitriol is ZnSO4.7H2O Green
Hence, magnetic force experienced by the electron is zero vitriol is CuSO4 ⋅ 5H2 O and it contains five water of
if v is parallel to B i.e., θ = 0 ° . crystallisation.

113. In the given circuit, the voltmeter reads 5V. The 115. Which elements are used for galvanisation?
resistance of the voltmeter (in Ohm) is (a) Zn and Sn (b) Na and K
(c) Cu and Fe (d) Ca and Mg
V 50Ω l (a) Zinc(Zn) and tin (Sn) are used for galvanisation. It is a
process of applying a protective zinc or tin coating on iron
100Ω to prevent rusting.
10 V 116. Ramesh dropped a metal piece ‘A’ in the solution of
another metal ‘M’. After some time, a new colourless
compound ‘N’ is formed. A, M, N respectively can be
(a) 200 (b) 100 (c) 10 (d) 50
(a) Mg, NaCI, MgCl 2
l (b) Reading of voltmeter, V = 5 V (b) Fe, ZnSO 4 , FeSO 4
100 Ω resistor is connected in parallel with voltmeter. (c) Zn, CuSO 4 , ZnSO 4
I2
V (d) Cu, ZnSO 4 , CuSO 4
50 Ω
l (c) Metal piece ‘A’ is Zn and solution of metal ‘M’ is CuSO4
I1 100 Ω
and the new colourless compound ‘N’ formed is ZnSO4 .
Complete reaction is as follows:
Zn + CuSO4 → ZnSO4 + Cu
(A) ( M) ( N)

As Zn is more reactive than copper, displacement reaction


10 V will take place.
Hence, voltage across 100 Ω resistor,
16 NTSE ~ SOLVED PAPER 2020 (Stage I)

117. Which fuel has highest calorific value? 122. Benzene(C 6H 6) have
(a) LPG (b) Petrol (c) CNG (d) Hydrogen (a) 12 covalent bonds (b) 15 covalent bonds
l (d) Hydrogen has highest calorific value. It is defined as (c) 18 covalent bonds (d) 9 covalent bonds
the quantity of heat produced by its combustion conditions l (b) Benzene have three double bonds and nine single
at normal temperature. The calorific value of LPG is 55000 bonds. Therefore, in total there are 15 covalent bonds.
kJ/kg and CNG is 50,000 kJ/kg. For hydrogen, value is
H
150000 kJ/kg.
H H
118. The pH of acid rain is
(a) less than 5.6 (b) more than 5.6 H H
(c) equal to 5.6 (d) more than 6.6
H
l (a) The pH of acid rain is less than 5.6. It results when SO2
(sulphur dioxide) and NOX (nitrogen oxides) are emitted 123. 1.0 kg of iron(Fe), having atomic mass equal to
into the atmosphere and transported by wind and air 56 g mol −1 contains
currents. On reaction with water, SO2 and NOX forms (a) 2.88 × 1024 atoms (b) 6.93 × 1023 atoms
sulphuric and nitric acids that drops pH less than 5.6.
(c) 6.93 × 10 atoms
21
(d) 1075
. × 1025 atoms
119. IUPAC name of the following compound will be l (d) 1 mol = 6.022 × 1023 atoms
O Given mass 1000 g
 Number of moles 1 kg of Fe = = = 17.85
CH 3  C  CH 2  CH 2  CH 2  COOH Molar mass 56 g
(a) 2-keto hexan - 6-oic acid So, 1.0 kg of Fe contains = 17.85 × 6.022 × 1023
(b) 5 - keto hexanoic acid = 1075
. × 1025 atoms
(c) methyl ketone butanoic acid 124. Aqueous solution of CsO 2 is
(d) 5 - aldo hexanoic acid
(a) basic (b) neutral (c) acidic (d) amphoteric
l (b) The IUPAC name of given compound is
O l (a) Alkaline solution of CsO2 is basic. CsO2 contain more
 oxygen than the corresponding basic oxide. In aqueous
6 5 4 3 2 1 solution, it is basic in nature.
CH3 C CH2 CH2 CH2 COOH ← Principal chain
5-keto hexanoic acid. Here, keto group is a secondary 125. A student added a drop of universal indicator to
functional group and COOH is a primary functional 1.00 mL of given solution and found that a green
group. colour is produced. The pH value of the solution
will be
120. Products obtained on electrolysis of brine are
(a) 7-9 (b) 0-3 (c) 10-12 (d) 4-6
(a) NaHCO 3 , H2 , Cl 2 (b) H2 , NaOH, NaHCO 3
(c) Cl 2 , NaOH, Na 2O 2 (d) NaOH, H2 , Cl 2 l (a) The pH value of the solution will be 7-9. Colour
produced is green and hence, the range lies above 7 but
l (d) Products obtained on electrolysis of brine are NaOH, below 10. The pH in 7-9 range is a neutral solution.
H 2 and Cl 2 . Chemical reaction occur at anode and cathode
are as follows. 126. Elements present in any group have the same
At cathode NaCl → Na+ + Cl − number of
Na + e− → Na
+
(a) valence electrons (b) neutrons
2Na + 2H2 O → 2NaOH + H2 (g) (c) protons (d) None of these
At anode Cl − → Cl + e− l (a) Elements present in any group have the same number
Cl + Cl → Cl2 of valence electrons.
e.g. In group 1 of periodic table,
121. In balanced chemical equation
Li(3) = 2 ,1
aKMnO 4 + bH 2SO 4 → Na(11) = 2, 8, 1
cK 2SO 4 + dMnSO 4 + eH 2O + f[O]
127. Which of the following reactions takes place during
Which of the following alternative are correct? breakdown of molecules in the respiration in our
(a) a = 2, b = 3, c = 1, d = 2, e = 3, f = 5 body?
(b) a = 1, b = 2, c = 1, d = 3, e = 2, f = 3 (a) Oxidation (b) Reduction
(c) a = 2, b = 3, c = 2, d = 3, e = 2, f = 5 (c) Oxidation-reduction (d) Photo-oxidation
(d) a = 3, b = 1, c = 3, d = 3, e = 1, f = 3
l (a) Respiration is an oxidative process in which respiratory
l (a) In the given chemical equation, substrates are oxidised to liberate energy inside the living
a = 2, b = 3, c = 1, d = 2, e = 3, f = 5 cells. The common respiratory substrates are
Complete reaction is as follows : carbohydrates, proteins, fats and organic acids. The most
2KMnO4 + 3H 2 SO4 → K 2 SO4 + 2 MnSO4 + 3H 2 O + 5[O] common respiratory molecule/substrate is glucose.
NTSE ~ SOLVED PAPER 2020 (Stage I) 17
128. Lactic acid is produced when pyruvate is broken 132. If a pea plant with wrinkled seeds and heterozygous
down tall plants were self-pollinated, what will be the
(a) in the presence of oxygen in mitochondria phenotypes of plants of F2 generation?
(b) in the absence of oxygen in mitochondria (a) 75% plants will be tall and have wrinkled seeds and other
(c) in the presence of oxygen in muscle cells 25% will be dwarf with wrinkled seeds
(d) in the absence of oxygen in muscle cells (b) 50% plants will be tall and have wrinkled seeds and other
50% will be dwarf with wrinkled seeds
l (d) Lactic acid is produced when pyruvate is broken down (c) 50% plants will be tall and have wrinkled seeds and other
under anaerobic conditions i.e. in the absence of oxygen in 50% will be dwarf with round seeds
some fungi and muscle cells of our body.
(d) 25% plants will be tall and have wrinkled seeds and other
129. Separation of oxygenated and deoxygenated blood. 75% will be dwarf with wrinkled seeds
I. Fulfils energy requirement of the body l (a) Pea plant with wrinkled seeds and heterozygous for
II. Ensures the efficient transfer of oxygen in the body tallness is Ttww
Ttww X Ttww Selfing
(a) Both statements are true
(b) Statement I is true but statement II is false
Tw tw Tw tw Gametes
(c) Statement I is false but statement II is true
(d) Both the statements are false
l (a) Both statements are true. Separation of oxygenated + Tw tw
and deoxygenated blood, helps to ensure efficient transfer Tw TTww Ttww
of oxygen in the body cells via blood. It is thus help in tw Ttww ttww
fulfilling the energy requirement of organisms with higher
metabolic rate. TTww, Ttww — Tall with wrinkled seeds
ttww — Dwarf with wrinkled seeds
130. Root pressure is effective way of transporting water
Thus, phenotypes of plants of F2 -generation will give 75%
in xylem. This pressure is generated tall plants with wrinkled seeds and other 25% plants dwarf
(a) in bright sunlight (b) during night with wrinkled seeds.
(c) at very low temperature (d) in high trees
133. Two similar pea plants are growing in two different
l (b) Root pressure is a positive pressure that develops in islands separated by a vast ocean. The phenomenon
the xylem sap of the root of some plants and is a
of geographical isolation will
manifestation of active water absorption. It helps in
(a) not be seen as the plants get self-pollinated
pushing up water to small heights in the stem. It is absent
under conditions like low temperature, bright light, (b) be seen as the plants are growing in isolated regions
reduced oxygen in the atmosphere, etc. Effects of root (c) not be seen as the plants get pollinated by ocean water
pressure is observable during night and early morning currents
when evaporation is low and excess water collects in the (d) be seen as the plants do not get pollinated and
form of droplets around special openings of veins near the reproduces asexually
tip of grass blades and leaves of many herbaceous plants. l (b) Two similar pea plants are growing in two different
islands separated by a vast ocean. Here the phenomenon
131. Choose the correct option to complete ‘A’, ‘B’, ‘C’
of geographical isolation will be seen as the plants are
and ‘D’ in the following table. growing in isolated regions. The separation of two
Hormones Functions populations of the same species by a physical barrier such
as mountain or body of water ultimately lead to
A Stimulates growth in all organs
geographical isolation which lead to the population
B Stimulates pituitary to release Growth hormone becoming separate species by adaptive radiation.
C Controls blood sugar level
134. DDT is non-biodegradable chemical when it enters
D Regulates carbohydrate metabolism
food chain, it gets accumulated in each tropical
(a) A - Insulin, B - Thyroxine, C - Growth hormone, D - Growth level. The phenomenon is called as
hormone release factor (a) eutrophication (b) chemical amplification
(b) A - Growth hormone, B - Insulin, C - Thyroxine, D - Growth (c) biomagnification (d) chemical magnification
hormone release factor
l (c) Biomagnification is the phenomenon through which
(c) A - Thyroxine, B - Insulin, C - Growth hormone, D - Growth
certain pollutants, such as pesticide, runoff, gets
hormone release factor
accumulated in tissues in an increasing concentration along
(d) A - Growth hormone, B - Growth hormone release factor, the food chains and produce fatal effect. Non-biodegradable
C - Insulin, D - Thyroxine chemical like DDT enters the food chain and gets
l (d) The correct matches are as follows accumulated in each level via biomagnification.
A. Growth hormone Stimulates growth in all organs
B. Growth hormone Stimulates ipituitary to release
135. Presence of ………… is an indicator of pollution
release factor growth hormone level in water.
C. Insulin Controls blood sugar level (a) colour (b) coliform bacteria
D. Thyroxine Regulates carbohydrate metabolism (c) rhizo bacteria (d) spiral bacteria
18 NTSE ~ SOLVED PAPER 2020 (Stage I)

l (b) Presence of coliform bacteria is an indicator of l (a) Here, draw QP || RB


pollution level in water. ∴In ∆DPB;∠DPB = 90 °
136. Leaves of tendu are the source of income of large D C
number of people of India. These leaves are used to R
make 6c
m
(a) thatched roofs (b) bidis 8cm
Q 10
(c) leaf plates (d) teeth cleaning agent cm

l (b) Leaves of tendu are the source of income of large

10
cm
number of people in India. These leaves are used for
wrapping the tobacco and making ‘bidis’ or Indian cigar. B
A
137. Maximum number of trophic levels supported in
any ecosystem is P
(a) one (b) two (c) three (d) four
DB2 = DP2 + PB2 (by Pythagoras theorem)
l (d) Maximum number of trophic levels supported in any ⇒ DB2 = (16)2 + (8)2
ecosystem is four. This is because only 10% energy is
= 256 + 64 = 320
transferred to next trophic level in a food, therefore if
trophic levels are more, not much of the energy will be left ⇒ DB = 8 5 = Diagonal of square ABCD
for higher trophic level. 8 5
∴Side of square = cm
2
138. The correct sequence of reflex arc is 2
(a) Receptor → Motor neuron → Sensory neuron → Effector  8 5  64 × 5
∴Area of square =   = = 32 × 5 = 160 m2
organ → Relay neuron  2  2
(b) Receptor → Sensory neuron → Motor neuron → Effector
organ → Relay neuron 142. If (2 x − 4)3 + (4 x − 2)3 = (4 x + 2 x − 6)3 , then the sum of
(c) Receptor Sensory Neuron → Motor neuron → Relay neuron all real values of x is
→ Effector organ (a) 0.5 (b) 1.5 (c) 2.5 (d) 3.5
(d) Receptor → Sensory neuron → Relay neuron → Motor
neuron → Effector organ l (d) Let 2x − 4 = a

l (c) The correct sequence of reflex arc is and 4x − 2 = b


Receptor → Sensory neuron → Motor neuron → Relay We have,
neuron → Effector organ (2 x − 4)3 + (4 x − 2)3 = (4 x + 2 x − 6)3
⇒ a3 + b3 = (a + b)3
139. Tricuspid valve is present in ⇒ a3 + b3 = a3 + b3 + 3 ab(a + b)
(a) right atria and right ventricle ⇒ ab(a + b) = 0
(b) left atria and left ventricle ⇒ ab = 0 or a + b = 0
(c) wall of atrium ⇒ a = 0 or b = 0 or a + b = 0
(d) wall of ventricle ⇒ 2x − 4 = 0
l (a) Tricuspid valve is present in between right auricle and ⇒ 2x = 4
right ventricle. ⇒ x =2
140. BCG vaccine provides protection against ⇒ 4x − 2 = 0
1
(a) measles (b) TB ⇒ x=
2
(c) cholera (d) small pox
Also, 4 x + 2 x − 6 = 0
l (b) BCG or Bacille Calmette-Guerin is a vaccine for ⇒ x =1
tuberculosis (TB) disease. ∴ Sum of all real values of x
1 7
141. Find the area of the square ABCD. = 2 + + 1 = = 3 ⋅5
2 2
D C
143. If 2019 x + 2019 − x = 3, then the value of
6 cm
20196 x − 2019 −6 x
8 cm is
2019 x − 2019 − x
10 cm (a) 3 (b) 6 (c) 9 (d) 12
l (d) Let a = 2019 x

1
A B ⇒ = 2019 − x
(a) 160 m 2 (b) 140 m 2 a
1
(c) 125 m 2 (d) 120 m 2 ∴ a+ =3
a
NTSE ~ SOLVED PAPER 2020 (Stage I) 19
1 x y
⇒ a2 + + 2 =9 ⇒ + = −1
a2 y x
1 x − (x + y)
⇒ a2 + 2 =7 ⇒ =
a y x
1
⇒ a4 + 4 + 2 = 49 2
a x x2 + y2 + 2 xy
1 ∴   =
⇒ a4 + 4 + 1 = 48 … (i) y x2
a − xy + 2 xy
1 = [Qx2 + y2 = − xy]
−6x a6 − x2
2019 − 2019
6x
a6
∴ = xy y
2019 x − 2019 − x 1 = 2 =
a− x x
a 3
  x 2  x
3 3
x
3
  x 2 y 
 a2 − 1   a4 + 1 + 1 Now,     +   =   +   Q   =   
y
  
 a2   a4   y  y x y   y   x 
=   
a − 1
   y x   y2 x2 
 a =  +   2 + 2 − 1
 x y  x y 
[Q x3 − y3 = (x − y)(x2 + y2 + xy)]
[Qa3 + b3 = (a + b)(a2 + b2 − ab]
 a + 1   a − 1   a4 + 1 + 1 Q
      + = − 1 
x y
 a  a  a4  
= y x
a − 1 = (− 1) (− 1 − 1)  2 2

   ⇒ x + y + 2 = 1
 a  
 y2 x2 
=  a + 1   a4 + 1 + 1
    =2
 a  a4 
= 3 × 48 = 144 = 12 (using Eq. (i)) 146. Let a, b and c are the roots of the polynomial
144. Let ‘p’ be a root of the equation x − 5 x + 7 = 0, then
2 equation x 3 − 597 x − 5236 = 0, then the value of
the area of circle with centre at (p, p) and passing ( a3 + b3 + c3 ) is
through point (1, 4) is (a) 597 (b) 15708
(a) 3π sq units (b) 5π sq units (c) 5236 (d) 10472
(c) 7π sq units (d) None of these l (b) We have, x3 − 597 x − 5236 = 0
l (a) Given, Here, a = − 11, b = − 17, c = 28
x2 − 5 x + 7 = 0 Since, a + b + c = (− 11) + (− 17) + 28 = 0
⇒ p2 − 5 p + 7 = 0 (since p is the root of the given equation) ∴ a3 + b3 + c3 = 3 abc
Now, radius (r) = (p − 1)2 + (p − 4)2 = 3(− 11) (− 17) (28) = 15708
= p2 + 1 − 2 p + p2 + 16 − 8 p 147. If cosec x + cot x = a, then the value of cos x is
= 2 p − 10 p + 17
2 1 a2 + 1
(a) a2 + (b)
a2 a2 − 1
= 2(p2 − 5 p) + 17
a −1
2
a2 + 1
= 2(−7) + 17 [Qp2 − 5 p + 7 = 0] (c) (d)
a +1
2
2a
= − 14 + 17 = 3
∴Area of circle = π( 3 )2 = 3 π sq units l (c) We have, cosec x + cot x = a
6 3 ⇒ cosec x = a − cot x
1 1 1  x  x ⇒ cosec2 x = a2 + cot2 x − 2 a cot x
145. If = + , then the value of   +   is
x+y x y  y  y ⇒ 1 + cot2 x = a2 + cot2 x − 2 a cot x
1 ⇒ 1 = a2 − 2 a cot x
(a) 0 (b) (c) 1 (d) 2 ⇒ 2 a cot x = a2 − 1
2 a2+1
a2 − 1 2a
l (d) We have, ⇒ cot x =
1 1 1 2a
= + a2 − 1 x
x+ y x y ∴ cos x = 2 a2–1
1 x+ y a +1
⇒ =
x+ y xy 148. In an AP 2, 5, 8, 11, ... 452, the mean of 15th, 16th,
⇒ (x + y)2 = xy 136th and 137th terms is
⇒ x2 + y2 + 2 xy = xy (a) 120 (b) 227
⇒ x2 + y2 = − xy (c) 220 (d) 454
20 NTSE ~ SOLVED PAPER 2020 (Stage I)

l (b) Given, 152. The angles of a pentagon are in arithmetic


AP is 2, 5, 8, 11, … 452 progression. The sum of the smallest and largest
∴First term, a = 2 angle is
Common difference, d = 3 (a) 172° (b) 108° (c) 180° (d) 216°
∴ 15th term = a + 14 d = 2 + 14 × 3 = 44
16th term = a + 15 d = 47 l (d) Let angles of a pentagon which in AP are
136th = a + 135 d = 2 + 135 × 3 = 407 a − 2 d, a − d, a, a + d, a + 2 d
137th term = a + 136 d = 2 + 136 × 3 = 410 ∴ (a − 2 d) + (a − d) + a + (a + d) + (a + 2 d) = (5 − 2) × 180 °
∴ Mean of 15th, 16th, 136th and 137th terms ⇒ 5 a = 540 ° ⇒ a = 108 °
44 + 47 + 407 + 410 908 ∴Sum of the smallest and largest angles
= = = 227
4 4 = (a − 2 d) + (a + 2 d) = 2 a = 2 × 108 ° = 216 °
149. The minimum value of tan2 x + cot2 x is 153. If p − q = 20, then the maximum value of
(a) 1 (b) 0 (c) 2 (d) 3  p − 5 q
tan2 x + cot2 x   is
l (c) ≥ (tan2 x ⋅ cot2 x)1 / 2  100 
2
(a) 5 (b) 10 (c) 15 (d) 25
⇒ tan2 x + cot2 x ≥ 2
l (a) We have,
150. If f( x ) = x 4 + ax 3 + bx 2 + cx + d is a polynomial such p − q = 20
that f(1) = 5, f(2) = 10, f(3) = 15, f(4) = 20, find the value ⇒ p = 20 + q
f(12) + f( −8) ⇒ p = 400 + q + 40 q
of .
100 Now,
(a) 198 (b) 198.4 p − 5 q (400 + q + 40 q) − 5 q
=
(c) 198.6 (d) 199.2 100 100
l (*) f(1) = 1 + a + b + c + d = 5 400 − 4 q + 40 q 100 − q + 10 q
= =
⇒ a+ b+ c + d = 4 ...(i) 100 100
f(2) = 16 + 8 a + 4 b + 2 c + d = 10 100 − (q − 10 q + 25 − 25)
=
⇒ 8 a + 4 b + 2 c + d = −6 ...(ii) 100
f(3) = 81 + 27 a + 9 b + 3 c + d = 15 100 − [( q − 5)2 − 25)]
=
⇒ 27 a + 9 b + 3 c + d = − 66 ...(iii) 25
and f(4) = 256 + 64 a + 16 b + 4 c + d = 20 100 + 25 − ( q − 5)2
⇒ 64 a + 16 b + 4 c + d = −236 ...(iv) =
25
On solving Eqs. (i),(ii),(iii) and (iv), we get p − 5 q 125
∴Maximum value of is =5
⇒ a = −10, b = 35, c = −45 and d = 24 100 25
∴ f(x) = x4 − 10 x3 + 35 x2 − 45 x + 24
Now, f(12) = (12)4 − 10(12)3 + 35(12)2 − 45 (12) + 24 = 7980
154. The area of trapezium ABCD, where AB = 52 cm,
and f(−8) = (−8)4 − 10 (−8)3 + 35(−8) − 45 (−8) + 24 = 11840 BC = 12 cm, CD = 39 cm and DA = 5 cm and
f (12) + f(−8) 7980 + 11840 AB|| CD, is
∴ =
100 100 D 39 cm C
19820
= = 198.20
100 5 cm 12 cm
So, given options not matched.

151. The product of two 2-digits numbers is 2160 and


their HCF is 12. Then, sum of the numbers is A 52 cm B
(a) 72 (b) 84 (a) 210 sq cm (b) 234 sq cm (c) 260 sq cm (d) 280 sq cm
(c) 96 (d) 60
l (a) Let a = 39, b = 52, c = 5, d = 12
l (c) Given, Then, a + b = 91
HCF = 12 and k = b − a = 52 − 39 = 13
∴ LCM =
2160
= 180 k + c + d 13 + 5 + 12 30
s= = = = 15
12 2 2 2
Now, ∴Area of trapezium
HCF × LCM = 12 × 180 a+ b
= s(s − k) (s − c) (s − d)
= 12 × (15 × 12) k
= 12 × 3 × 5 × 12 91
= 15(15 − 13) (15 − 5) (15 − 12)
= 36 × 60 13
∴Required numbers are 36 and 60. = 7 15 × 2 × 10 × 3 = 7 3 × 5 × 2 × 5 × 2 × 3
∴Sum of the numbers = 36 + 60 = 96. = 7 × 3 × 5 × 2 = 210 sq cm
NTSE ~ SOLVED PAPER 2020 (Stage I) 21
155. The difference between area of a triangle of largest 158. Find the sum of all real values of x which satisfy
area inscribed in a circle of radius ‘r’ units and a 1 1 2
triangle of largest area inscribed in a semi-circle of + =
x 2 − 10 x − 45 x 2 − 10 x − 29 x 2 − 10 x − 69
radius ‘r’ units is
(a) 7 (b) 10 (c) 13 (d) − 3
 2 3 − 1 2  4 −2 3  2
(a)   r sq units (b)   r sq units
 4   4 
l (b) We have,
 3 3 + 4 2  3 3 − 4 2 1 1 2
(c)   r sq units (d)   r sq units + 2 = 2
 4   4  x − 10 x − 45 x − 10 x − 29 x − 10 x − 69
2

(d) Area of triangle of largest area inscribed in a 1 1


l ⇒ + 2
3 x − 10 x − 29 − 16 x − 10 x − 29
2

semi-circle with radius r is (3 r2 ). 2


4 = 2
And the area of the largest triangle inscribed in a x − 10 x − 29 − 40
semi-circle of radius r is r2 . Let x2 − 10 x − 29 = p
3 3 3  1 1 2
∴Difference of both area = (3 r2 ) − r2 = r2  − 1 ∴ + =
4  4  p − 16 p p − 40
3 3 − 4 p + p − 16 2
= r2  ⇒ =
 sq units. p(p − 16) p − 40
 4 
⇒ (p − 8) (p − 40) = p(p − 16)
156. If p, q, r and s are distinct prime numbers such that ⇒ p2 − 40 p − 8 p + 320 = p2 − 16 p
p + q + r = 72, p + r + s = 74, q + r + s = 89. The ⇒ − 48 p + 320 = − 16 p ⇒32 p = 320
largest of these p, q, r and s is ⇒ p = 10
(a) r = 53 (b) q = 53 (c) s = 53 (d) s = 49 ∴ x2 − 10 x − 29 = 10
l (a) Given, p, q, r and s are distinct prime numbers. ⇒ x2 − 10 x − 39 = 0 ⇒(x + 3) (x − 13) = 0
such that p + q + r = 72 ...(i) ⇒ x = − 3, 13
p + r + s = 74 ...(ii) ∴Required sum = − 3 + 13 = 10
q + r + s = 89 ...(iii) 1 3 3
In Eqs. (i) and (ii), p and r is common and sum in Eqs. (i) 159. If N = 3 4 + 3 2 + 1, then the value of 3
+ 2
+ is
and (ii) are even. N N N
∴atleast p or q is 2. (a) 2 (b) 4
(c) 7 (d) 1
Now, putting p = 2 in Eqs. (i) and (ii), we get
q + r = 70 ...(iv) l (d) We have,
and r + s = 72 ...(v) N = (4)1 / 3 + (2)1 / 3 + 1
On adding Eqs. (iv) and (v), we get ⇒ (N − 1) = (4)1 / 3 + (2)1 / 3
q + 2 r + s = 142 ⇒ (N − 1)3 = 4 + 2 + 3 ⋅ 41 / 3 ⋅ 21 / 3 (41 / 3 + 21 / 3 )
⇒ 2 r + (89 − r) = 142 [from (iii)] ⇒ N3 − 1 − 3 N(N − 1)
⇒ r = 142 − 89 = 53 = 6 + 3 ⋅ 22 / 3 ⋅ 21 / 3 (41 / 3 + 21 / 3 )
⇒ N3 − 1 − 3 N2 + 3 N = 6 + 6(41 / 3 + 21 / 3 )
157. In the given figure, the value of m is = 6(1 + 41 / 3 + 21 / 3 )
E(12, 25)
D A ⇒ N − 1 − 3N + 3N = 6N
3 2

⇒ N3 = 3 N2 + 3 N + 1 … (i)
15 1 3 3
Now, + 2 +
F N3 N N
m 1 + 3 N + 3 N2
C(12, 15) B(24, 15) ⇒
N3
N3
⇒ =1 [using Eq. (i)]
N3
(a) 5 (b) 10 (c) 7 (d) 12
160. In a class average height of all students is ‘p’ cm.
l (a) Here, ∆ CFE ~ ∆CED (by AA similarity criteria)
CF CE Among them, average height of 10 students is ‘q’ cm
⇒ = and the average height of the remaining students is
CE CD
⇒ CF × CD = (CE)2
‘r’ cm. The number of students in the class is
p(q − r ) q −r
⇒ m × (m + 15) = (10)2 (a) (b)
( p − r) p− r
⇒ m + 15 m − 100 = 0 ⇒ m2 + 20 m − 5 m − 100 = 0
2
q −r 10(q − r )
⇒ m(m + 20) − 5 (m + 20) = 0 ⇒ (m − 5)(m + 20) = 0 (c) (d)
10( p − r ) ( p − r)
⇒ m=5 (m ≠ −20)
22 NTSE ~ SOLVED PAPER 2020 (Stage I)

l (d) Let number of students = x l (d) The book ‘Hind Swaraj’ was written by the Mahatma
According to the question, Gandhi in the year 1909. It was written in Gujarati
Total height of class = px cm language. It is the foundational text of Gandhian world
view. In 1910, the book was banned by Britishers.
Total height of 10 students = 10q cm
Other Important Books
Total height of (x − 10) students = (x − 10)r cm
J.L. Nehru The Discovery of India, Glimpses of World
Now, total height of class = 10 q + (x − 10)r History
⇒ px = 10 q + (x − 10)r
Subhas Chandra Bose The Indian Struggle, An Indian
⇒ px = 10 q + xr − 10 r
Pilgrim
⇒ (p − r)x = 10 q − 10 r = 10(q − r)
Kamla Nehru was the wife of J.L. Nehru (First P.M. of India)
10(q − r)
⇒ x=
p−r 166. Which country was known as ‘Siam’?
(a) England (b) Thailand (c) Holland (d) Swaziland
161. What are the National Colours of France?
l (b) Thailand was known as ‘Siam’. The country was
(a) Blue, Green, Red (b) Green, White, Red
renamed on 23rd June, 1939. It is situated in South-East
(c) Green, Yellow, Red (d) Blue, White, Red Asia and its capital is Bangkok. Its neighbours are
l (d) The National Colours of France are Blue, White and Myanmar, Combodia, Laos and Malaysia.
Red, representing the ideas of French Revolution liberty, Some other important new and old names are
equality and fraternity. National colours are usually parts New Name Old Name
of a country’s set of National Symbol. National Flags are Kingdom of Eswatini Swaziland
best examples of using National colours. Sri Lanka Ceylon
Iran Persia
162. Which was not included in Lenin's April Theses?
France Gaul
(a) Formation of Duma
(b) Banks be Nationalised 167. Which of the following Prime Ministers constituted
(c) Land be transferred to peasants ‘Simon Commission’?
(d) The war(First Warld War) be brought to an end (a) Robert Walpole (b) Stanley Baldwin
l (a) Vladimir Lenin’s April Theses does not included (c) Ramsay MacDonald (d) Winston Churchil
formation of Duma. During Russian Revolution (1917), l (b) Simon Commission was constituted in 1927 by the
Bolshevik leader Vladimir Lenin issued series of British Prime Minister, Stanley Baldwin. It consisted of 7
10 directives to the government. These directives are members and not even a single member was Indian.
collectively known as April Theses. Its objectives were to analyse the working of the
It was given name AP April Theses because these Government of India Act, 1919 and to suggest further
demands were made in the month of April. The main reforms in the system of administration. The Indian
directives of this Theses were bank to be nationalised, war Statutory commission, Commonly referred to as the Simon
to be brought to a close, land transferred to peasant, etc. Commission after its Chairman Sir John Allsebrook Simon,
was sent to India in 1928 to study potential constitutional
163. Hitler assigned the responsibility of Economic reform.
Recovery to
(a) Herbert Spancer (b) Hjalmar Schacht 168. Dr. B. R. Ambedkar formed the ‘Depressed Classes
(c) W Shirer (d) Robert Lay Association’ in
l (b) Hitler assumed the office of Chancellor of Germany in (a) 1928 (b) 1929
1933. Then, he focussed on the development of the country. (c) 1930 (d) 1931
In this regard, he assigned the responsibility of Economic l (c) Dr. B. R. Ambedkar formed the ‘Depressed Classes
Recovery to Hjalmar Schacht. He was also asked to take Association’ in 1930. The objective of setting up of this
measure to make Germany a self-reliant country. association was to improve the condition of untouchables
and to end the caste system. He played prominent role in
164. Which of these had worked as indentured labourer? improving the living condition of untouchables.
(a) Shaukat Ali (b) Alluri Sita Ram Raju He played a key role in drafting of Indian Constitution
(c) Jawaharlal Nehru (d) Baba Ramchandra and because of which he is referred as ‘Father of Indian
l (d) Baba Ramchandra had worked as indentured labourer. Constitution’.
He was born in 1875 in Gwalior (Madhya Pradesh). In the 169. ‘Jeevitha Samaram’ is the autobiography of
1904, he was sent to Fiji as an indentured labourer. In Fiji,
(a) C. Kesavan (b) Saudamini (c) Mankojee (d) R. C. Dutt
he carried out movement to emancipate problems of
indentured labour. After returning to India, he started l (a) ‘Jeevitha Samaram’ is an autobiography of C. Kesavan.
movements for the welfare of farmers. He was a freedom fighter, social reformer and Chief
Under indenture system, Britishers transported labour in Minister of erstwhile Travancore-Cochin (in Kerala)
their overseas colonies. It was a kind of slavery system. province in 1950-52. Due to sudden death in 1969, he was
not able to complete third volume of his autobiography.
165. Who wrote the Book ‘Hind Swaraj’? He was the founder member of the Sree Narayana
(a) Subhash Chandra Bose (b) J. L. Nehru Dharma Paripalana Yogam (SNDP). He worked for
(c) Kamla Nehru (d) Mahatma Gandhi eradication of untouchability.
NTSE ~ SOLVED PAPER 2020 (Stage I) 23
170. Who established the Vietnamese Communist Party? 175. In which industry, Bauxite is used as a raw material?
(a) Phu So (b) Mao Zedong (a) Steel (b) Cement
(c) Ho Chi Minh (d) Phan Boi (c) Aluminium (d) Jute
l (c) Vietnamese Communist Party was established by l (c) Bauxite is used as a raw material in the Aluminium
Ho Chi Minh on 3rd, February, 1930. It had played a industry. It is an ore of Aluminium. It is primarily
significant role in integration of North and South Vietnam comprised of Aluminium Oxide, Silica, Iron Oxide and
during Cold War era. Its current General Secretary is Titanium Dioxide. It is mainly found in the laterite soil. It
Nguyen Phu Trong. is most commonly found in tropical or sub-tropical regions.

171. “When France sneezes, the rest of Europe catches 176. Rooftop rainwater harvesting is the most common
cold”, who remarked this? practice in which of the following cities?
(a) Mazzini (b) Metternich (a) Shillong (b) Imphal
(c) Gottfried (d) John Locke (c) Guwahati (d) Patna
l (b) Austrian Chancellor, Metternich had made famous l (a) Rooftop rainwater harvesting is the most common
remark “When France sneezes, the rest of Europe catches practice in Shillong, capital city of Meghalaya. Shillong
cold”. This remark means that the political development in receives sufficient rainfall but due to mountainous terrain,
France spread quickly or impact the other countries of water drains into streams. To ensure availability of water,
Europe. For example, the French Revolution and the ideas nearly every household in the city has a rooftop rainwater
of liberty, equality and fraternity. harvesting system.
Guisseppe Mazzinihas played an instrumental role in
unification of Italy. 177. Which of the following groups constitute the basic
Gottfried Wilhelm Leibniz (Germany)and John Locke rock form?
(England) are famous philosophers. (a) Sandy, Igneous, Metamorphic
(b) Igneous, Sedimentary, Metamorphic
172. Which one of the following is the main cause of land (c) Lignite, Volcanic, Sedimentary
degradation in Punjab? (d) Sandy, Volcanic, Igneous
(a) Intensive cultivation
(b) Deforestation
l (b) The basic rock form consists of igneous, sedimentary
and metamorphic rocks. A basic unit of Earth’s crust is
(c) Over irrigation referred as a rock.
(d) Over grazing
Igneous Rock These rocks are formed when lava (molten
l (c) The main cause of land degradation in Punjab is over rock matter) cools down and gets harden. Magma is
irrigation. Due to plenty of water availibility, the farmers erupted from volcano and it is known as lava on Earth
over irrigate their fields which results in water logging. surface.
This logging increases salinity and alkalinity in the soil. Sedimentary Rock These rocks are formed by
Land degradation reduces the fertility of the soil. accumulation of land, shells and other fragments of
Deforestation, overgrazing and intensive cultivation are material. These rocks are also known as secondary rocks.
also responsible for land degradation. Metamorphic Rock These rocks are formed under the
surface of Earth when rock faces intense heat and process.
173. Traditional rainwater harvesting is called in This process of change is called metamorphosis.
Rajasthan
178. Mango showers occur in which one of the following
(a) Tank (b) Taanka (c) Pond (d) Lake
group of two states?
l (b) In Rajasthan, traditional rainwater harvesting is called (a) Bihar and West Bengal
Taanka or Kunds. This method of water harvesting is
(b) Tamil Nadu and Andhra Pradesh
common in Thar desert region of the state. A taanka is
madeup of covered, underground and impermeable (c) Karnataka and Kerala
reservoir to collect rainwater. It is meant to provide (d) Maharashtra and Andhra Pradesh
drinking water for a family or a small group of Families. l (c) Pre-monsoon showers are also called Mango Showers in
the state of Karnataka, Kerala, Konkan and Goa.
174. Which of the following states has most Sugar Mills
Pre-monsoon showers are given this title because they help
in India? in the ripening of mangoes. This showers also prevent
(a) Haryana (b) Punjab mangoes from dropping pre-maturely (unriped) from trees.
(c) Maharashtra (d) Bihar
179. Tropic of Cancer does not pass through
l (c) Among the given options, Maharashtra has most Sugar
(a) Chhattisgarh (b) Odisha
Mills in India. Maharashtra is one of the leading
producers of sugar and sugarcane in the country. The (c) Rajasthan (d) Tripura
major centres in state are Nasik, Pune, Satara, Kolhapur l (b) Tropic of Cancer does not pass through Odisha. It is an
and Sengli. imaginary line at 23.50° North of the equator. It passes
Haryana, Punjab and Bihar also contribute in production through 8 Indian states which includes Rajasthan, Gujarat,
of sugar. Uttar Pradesh is the leading sugarcane and sugar Madhya Pradesh, Chhattisgarh, Jharkhand, West Bengal,
producing state in the country. Tripura and Mizoram.
24 NTSE ~ SOLVED PAPER 2020 (Stage I)

180. AMUL Milk Scheme is an example of which type of l (c) General Elections are called as on completing of five
industry? years. In India, members of House of the People (Lok
(a) Basic Industry (b) Agro-Based Industry Sabha) and members of State Legislative Assembly are
elected after every five years or even earlier if body is
(c) Joint Industry (d) Co-operative Industry
dissolved by the President and Governor, respectively.
l (d) AMUL Milk Scheme is an example of Co-operative These members are elected directly by voting from a set of
Industry. It was established in 1946 as a Co-operative candidates who stands in their respective constituencies.
industry in the Kaira district of Gujarat. Co-operative Every adult citizen of India can vote only in their
industries are owned and operated by the producers or constituency.
suppliers of raw materials, workers or both. The main Bye-Election also called as By Poll is held to fill a vacant
objective of such industries is to reduce exploitation by electorate seat which may arise due to the death,
middlemen and increase profit level. resignation or due to any other reasons such as
disqualification. The member elected by a bye-election
181. Which one of the figures represents the working age will have a term of office equal to the remaining tenure of
group of the population? preceding member.
(a) 15 - 65 years (b) 15 - 66 years
185. In 44th Amendment, which Fundamental Right has
(c) 15 - 59 years (d) 15 - 64 years
been removed from the list of Fundamental Rights?
l (d) ‘15-64 years’ represents the working age group of the (a) Freedom to Speech
population. People belonging to this age group owing to (b) Freedom to Make Groups
enhanced their neurological capacity and physical
(c) Right to Work
strength and are capable of doing many types of jobs. The
people younger than 15 or older than 64 are considered as (d) Right to Property
dependents. According to the World Bank Report, Age l (d) In 44th Amendment, Right to Property has been
Dependency Ratio (% of working age population) in India removed from the list of Fundamental Rights. Previously, it
stood at 46-25% in 2019. was part of Fundamental Right under Article 31 in Part
111 of the Constitution. After 44th Amendment Act, 1978,
182. Chemical Industries are usually located near it was abolished and made a Constitutional Right under
(a) Iron and Steel Industries (b) Thermal Power Plant Article 300A. This Article states that no person can be
(c) Oil Refineries (d) Automobile Industry deprived of his/her property except by authority of law.
l (c) Chemical industries are usually located near Oil Freedom of Speech and Expression is a Fundamental
Refineries. These chemical industries get their raw Right under Article 19 of the Constitution.
materials from by-products of mineral oil which is Freedom to Make Groups (Association) is also a
processed and refined at oil refineries. These industries Fundamental Right under Article 19 of the Constitution.
are further divided into those producing inorganic Right to Work is a directive given to the State under
chemicals such as sulphuric acid, nitric acid, soda ash, Article 41 in Part IV of the Constitution, which deals with
etc., and organic chemical which utilizes petro-chemicals Directive Principles of State Policies.
for manufacturing synthetic fibres, synthetic rubber,
plastic, dye-stuffs, etc.
186. Which of the following statements is correct?
(a) Union List – 66 Subjects; State List – 97 subjects;
183. BAMCEF means Concurrent List – 47 Subjects
(a) Backward and Minority Community Employees Federation (b) Union List – 47 Subjects; State List – 97 Subjects;
(b) Backward and Mining Community Employees Federation Concurrent List – 66 Subjects
(c) Backward and Majority Community Employees Federation (c) Union List – 97 Subjects; State List – 47 Subjects;
(d) Backward and Malabar Coastal Employees Federation Concurrent List – 66 Subjects
(d) Union List – 97 Subjects; State List – 66 Subjects;
l (a) BAMCEF is the abbreviation of an organisation known Concurrent List – 47 Subjects
as ‘The all India Backward and Minority community
Employees Federation.’ It was established in 1971 by l (d) Union List– 97 Subjects, State List–66 Subjects,
Kashi Ram, D. K. Khaparde and Dinabhana. The main Concurrent List–47 Subjects is correct. It is provided in
objective of BAMCEF is to fight the entrenched system of Seventh Schedule of the Constitution which defines and
inequality that divides Indian Society and to abolish the specifies allocation of powers and functions between
caste system. It works as a pressure group to further the Union and States. In case of Union List, Centre has
cause of social justice in the country. Waman Meshram is exclusive powers to make law. State Legislative under
the National President of BAMCEF (August, 2020). normal circumstances has exclusive power to make laws
on subject enumerated in State List. While in case of
184. General Elections are called as Concurrent List, both Central and State Governments can
(a) on death of any member make laws, however, in case of any conflict, the law made
(b) election before specific time in the whole country and states by the Central Government prevails.
(c) on completing of five years At present (August, 2020), Union List contains 100
Subjects, State List contains 61 Subjects and Concurrent
(d) empty seat due to any reason List contains 52 Subjects.
NTSE ~ SOLVED PAPER 2020 (Stage I) 25
187. A person who is not a member of any House of l (c) The main aim of Civil Rights Movement in America
Parliament, if he is appointed as Minister, he has to was to abolish social discrimination. The Civil Rights
get elected to the one of the House of Parliament Movement was launched in America between 1954 and
1968 to abolish social discriminations suffered by Afro
within
American Communities. They were subjected to racial
(a) a month discrimination both at public and private places. It
(b) six months resulted into the passage of Civil Rights Act, 1964 which
(c) three months banned social discrimination in employment and public
(d) stipulated time is fixedly the President accommodations.
l (b) A person who is not a member of any House of 191. President can declare emergency if
Parliament, if he is appointed as Minister, he has to get (a) Prime Minister advises him to do so
elected to one of the House of Parliament within six
(b) Parliament advises him to declare emergency
months. The Articles 75(5) and 64(d) for Union and States,
(c) the Council of Minister, in writing, advises him to do so
provide that any non-member of the House can remain
Minister for the period of six consecutive months. After (d) Home Minister asks him to do so
that he is disqualified to be a Minister as well as Member l (c) President can declare emergency when the Council of
of the House. Thus, in order to remain as Minister, he/she Minister, in writing, advises him to do so. This safeguard
has to get elected to one of the House of Parliament within was provided through 44th Constitutional Amendment
six months. Act, 1978. Earlier, President of India was empowered to
declare emergency on the advice of Prime Minister only.
188. Why is “Power Sharing” regarded as good? The Constitution stipulates three types of emergencies viz.
(a) Reduces poverty National Emergency (Article 352), State Emergency
(b) Provides employment (Article 356)and Financial Emergency (Article 360).
(c) Maximizes wealth
(d) Reduces social conflict
192. Amnesty International is an international
organisation which works for
l (d) Power sharing is regarded as good because it reduces (a) Work Peace (b) Justice
social conflict. The term ‘power sharing’ means any policy
(c) Restoration of democracy (d) Human Rights
agreed between political parties, voluntary organisations
and social groups to share responsibility for l (d) Amnesty International is an international organisation
decision-making and political action. It gives people the which works for Human Rights. Amnesty International
right to be consulted on how they should be governed and was founded in London in 1961. Amnesty International is
all the citizens tend to have the same political and legal a global movement of more than 7 million people who take
rights. It helps in reducing the possibility of conflict injustice personally. It works for six key areas, namely
between the social groups by giving equitable positions to abolishing torture and death penalty, promotes rights of
all of them. refugees and prisoners and protection of rights of women,
children, minorities and indigenous people besides
189. Main feature of ‘Pressure Groups’ is protection of human dignity.
(a) Direct control on political power
(b) Try to influence the politics of government 193. In which year, ‘Universal Adult Franchise’ was
(c) Lax organisation implemented in India?
(d) Direct participation in political powers (a) 1947 (b) 1950 (c) 1919 (d)1935

l (b) The main feature of ‘Pressure Groups’ is that they try to l (b) In 1950, ‘Universal Adult Franchise’ was implemented
influence the politics of government. A pressure group is a in India. After implementation of Constitution of India on
type of voluntary association of people who is organised 16th January, 2020, Universal Adult Franchise was
actively for promoting and defending its common interest. granted to all eligible citizens of India. Universal Adult
They try to influence the politics of government through Franchise means the right to vote to every adult of the
legal and legitimate means such as lobbying, country. Article 326 of the Constitution provides for
correspondence, publicity, petitioning, public debating, Universal Adult Franchise.
etc. Examples of pressure groups from India are 194. In which year, Consumer Protection Act was enacted?
Associated Chamber of Commerce (ASSOCHAM), All
(a) 1986 (b) 1988 (c) 1985 (d) 1987
India Trade Union Congress (AITUC), Bhartiya Kisan
Sangh, etc. l (a) The Consumer Protection Act was enacted in the year
1986. The basic objective of this act was to protect the
190. Among the following, which is/are the main aim(s) consumers from unfair trade practices, defects in goods
of starting Civil Rights Movement in America? and deficiencies in services. It provided six rights to
(a) Adult franchise consumers namely-Right to be Protected, Right to be
(b) Vote to right for women Informed, Right to be Assured, Right to Heard, Right to
(c) Abolishing social discrimination Seek Redressal and Right to Consumer Education.
(d) Fan direct election of Congress This act was replaced by Consumer Protection Act, 2019.
26 NTSE ~ SOLVED PAPER 2020 (Stage I)

195. Which among the following is considered to be most 198. Under National Rural Employment Guarantee Act.
liquid asset? (2005), how many days of work are guaranteed in a
(a) Gold (b) Demand deposits year?
(c) Land (d) Money (a) 80 days (b) 100 days
l (d) Money is considered to be most liquid asset. A liquid (c) 200 days (d) 300 days
asset is an asset that can easily be converted into cash in a l (b) The mandate of tha MGNREGA is to provide at least
short amount of time. Because of ease of transaction, 100 days of guaranteed wage employment in a financial
storage and interoperability among various financial year to every rural household whose adult members
products, money is considered as most liquid asset. volunteer to do unskilled manual work. This Act aims to
Demand deposit is also considered as a liquid asset enhance livelihood and security in rural areas by
because it can easily be redeemed for cash. providing guaranteed wage employment to every
Gold is also considered as liquid asset as it can be easily household whose adult members volunteer to do unskilled
exchanged for cash. manual work. It is implemented mainly by Gram
Land is considered as a non-liquid asset because it can Panchayat.
take months for a person or company to receive cash from
the sale. 199. Who is the founder of Grameen Bank of Bangladesh?
(a) Abdul Rehman
196. Food security is ensured in a country, only if (b) M. Yunus
(a) enough food is available for all the persons (c) Mujibur Rehman
(b) all persons have the capacity to buy food of acceptable (d) Amartya Sen
quality
(c) there is no barrier on access to food l (b) Muhammad Yunus is the founder of Grameen Bank of
(d) All of the above Bangladesh. He established first Grameen Bank in
Bangladesh in 1983. His objective was to help poor people
l (d) Food security is the state of having reliable assess to escape from poverty by providing loans on terms suitable
sufficient quantity of affordable and nutritious food. Food to them. He was awarded with Nobel Peace Prize in 2006
security is ensured in a country only if enough food is for founding Grameen Bank and pioneering the movement
available for all the persons. All persons have the capacity for micro-finance and micro-credit.
to buy food of acceptable quality and there is no barrier on
access to food. Hence, all the options are correct. 200. In which of the following states of India, the use of
chemical fertilizer is highest?
197. The headquarters of World Trade Organisation is
(a) Punjab
situated in
(b) Haryana
(a) New York (b) China (c) Japan (d) Geneva
(c) Rajasthan
l (d) The headquarters of World Trade Organisation is (d) Himachal Pradesh
situated in Geneva. World Trade Organisation (WTO) is an
inter-governmental organisation that is concerned with the l (a) In Punjab, the use of chemical fertilizer is highest. Due
regulation of international trade between nations. The to intensive agriculture and general prosperity of farmers
WTO is the only global international organisation dealing amid the backdrop of Green Revolution, the per capita
with the rules of trade between nations. It was formed in consumption of fertilizers is highest in the state. It is
1995. As of now (August, 2020), 164 countries including followed by Haryana and West Bengal. Among the types of
India are its members. Robert Azevedo is the current fertilizers, Urea is widely used by the Indian farmers.
Director General of the World Trade Organisation.
NTSE~ SOLVED PAPER 2019 (Stage II) 1

NTSE Solved Paper


NATIONAL TALENT
SEARCH
EXAMINATION
2019 (Stage II)

INSTRUCTIONS
This solved paper consists of two papers. Paper I consists of Mental Ability Test (MAT) and Paper II consists of
Scholastic Aptitude Test (SAT).
MAT covering (Q. Nos. 1-50) of Maths and (Q. Nos. 51-100) of Reasoning.
SAT consists of (Science, Mathematics and Social Science) which comprises 100 questions (40 Science, 20 Mathematics
and 40 Social Science).
There will be no negative marking.
Each correct answer will be awarded one mark.

Time : 240 Minutes Max. Mark : 200

Paper I Mental Ability Test (MAT)


1. If p l (b) A B
D E
 +  = 10 C G
F H
 + ~ + ~ = 10
 ´ ~ –δ =5 I J

Then, the value of D will be ……… . K L


M
(a) 1.5 (b) 2.5 (c) 5 (d) 7.5
l (a) According to the given information, N O P Q
 +  = 10 Total parallelograms are
2 = 10 Þ  =5 ABEC, CEHF, CDGF, DEHG, ABHF, FGLK, GHML,
 + ~+ ~ = 10 FHMK, IGJL, GHLK, FGML, GHPO, LMPO, CDML,
5 + 2 ~ = 10 KMQN = 15
2 ~ = 5 Þ ~ = 2.5 3. A newspaper has 6 sheets consisting of 24 page in
 ´ ~– D ´ =5 total. If page number 17 of that newspaper is
5 ´ 2.5 - D ´ 5 = 5 missing, then find the set of missing pages in that
5(2.5 - D) = 5 newspaper, from the alternatives given below
2.5 - D = 1 Þ D = 1.5 (a) 6, 7, 16, 17 (b) 7, 8, 17, 18 (c) 8, 9, 17, 18 (d) 9, 10, 16, 17
l (b) Newspaper has 6 sheets and 24 pages.
2. How many parallelograms are there in the given Þ Each sheet has 4 pages.
figure?
Þ Group of papers are (1,2, 23, 24)
(3, 4, 21, 22)
(5, 6, 19, 20)
(7, 8, 17, 18)
(9, 10, 15, 16)
(11, 12, 13, 14)
(a) 14 (b) 15 (c) 16 (d) 17 Hence, set of (7, 8, 17, 18) pages is missing.
2 NTSE~ SOLVED PAPER 2019 (Stage II)

4. The given figure in the question has five squares 7. The opposite faces of Dice X are [(5, 2), (6, 3), (4, 1)]
and four equilateral triangles. Two squares and two The opposite faces of Dice Y are [(3, 5), (4, 1), (6, 2)]
triangles are shaded. The figure is folded along the Which figure can represent both Dice X and Dice Y
dividing lines the squares by 90° and triangle by 45° with faces shown below?
so as to form a close three, dimensional object. The 4 4 4 4
object is then placed with its apex pointing towards 2 5 3 6
6 3 2 3
your left. Which one among the figures given in the
alternatives can be seen? A B C D
(a) A (b) B (c) C (d) D
l (c) In dice X 5 « 2, 6 « 3, 4 « 1
In dice Y 3 « 5, 4 « 1, 6 « 2
In figure (A), 6 and 2 cannot be adjacent to each other
according to dice Y.
(a) (b) (c) (d) In figure (B), 3 and 5 cannot be adjacent to each other,
. according to dice Y.
In figure (D), 3 and 6 cannot be adjacent to each other,
l (a) Given figure,
according to dice X.
Hence, figure (C) has correct configuration.

8. R S S T U
+ N R S T
+ R T S
After folding, it is shown as
3 7 8 4 9
then, find the code of T U R N S from the given
alternatives provided there is no carrying over in the
5. Complete the following series given addition using letter codes.
6, 24, 60, ?, 210 (a) 1 3 6 2 5 (b) 6 5 2 3 1 (c) 1 6 3 5 2 (d) 5 3 1 2 6
(a) 96 (b) 120 l (c) Given, R S S T U
(c) 140 (d) 160 + N R S T
l (b) The pattern is as follows + R T S
3 7 8 4 9
6 24 60 120 210 Þ R =3
Þ S+ N = 7 … (i)
+18 +36 +60 +90 Þ S+ R+ R = 8 … (ii)
Þ T+ S+ T = 4 … (iii)
+18 +24 +30 Þ U+ T+ S= 9 … (iv)
From Eq. (ii),
S + R +R = 8
+6 +6 S + 2R = 8
\ ? = 120 S= 8 - 2 ´ 3
6. By studying the figure and number relationship, S=2
find the missing number ‘?’. From Eq. (i),
S+ N = 7
26 9 32 6 41 2 33 4 N=7 -2
N=5
3 4 7 ? From Eq. (iii),
15 7 18 8 9 19 8 15 T + S+ T = 4
2T + 2 = 4
(a) 1.5 (b) 6
(c) 9 (d) 12 T =1
From Eq. (iv),
l (b) As, (26 + 7) - (15 + 9) = 33 - 24 = 9 = 3
U + T +S = 9
(32 + 8) - (18 + 6) = 40 - 24 = 16 = 4 U+ 1 + 2 = 9
(41 + 19) - (9 + 2) = 60 - 11 = 49 = 7 U =6
(33 + 15) - (4 + 8) = 48 - 12 = 36 = 6 Code for T U R N S
\ missing number ? = 6 ¯ ¯ ¯ ¯ ¯
1 6 3 5 2
NTSE~ SOLVED PAPER 2019 (Stage II) 3
9. A comparison of ages of A, B, C, D and E are as follows 11. Three dice are rolled simultaneously and the numbers
I. B’s age is half the age of A. shown on all the three dice are added, then the total
1 number of possible ways to have a sum of 7 is
II. B’s age is 1 times the age of C. (a) 12 (b) 13 (c) 15 (d) 16
2
III. D’s age is 12 years less than C. l (c) The total number of possible ways of having a sum of 7
1 (1, 1, 5) (2, 1, 4) (3, 1, 3) (4, 1, 2) (5, 1, 1)
IV. D’s age is 1 times the age of E. (1, 2, 4) (2, 2, 3) (3, 2, 2) (4, 2, 1)
2
(1, 3, 3) (2, 3, 2) (3, 3, 1), (1, 4, 2) (2, 4, 1)
V. The age of E is 12 years.
(1, 5, 1) = 15
With the given data what will be the difference in the
ages of A and C? 12. A comparison of marks scored by Gauri, Aaban,
(a) 64 (b) 60 (c) 40 (d) 36 Seerat and Alvina in an examination is as follows.
l (b) According to the given information, I. Gauri has scored 15 marks less than Aaban.
A II. Gauri has scored 20 marks more than Seerat.
B= Þ A = 2B … (i)
2 III. Alvina has scored 10 marks less than Seerat.
3
B = ´ C Þ 3C = 2B … (ii) To decide who has scored the highest marks, identify
2
the statement from those given in the alternatives in
D = C - 12 … (iii)
respect of sufficiency of data.
3
D= ´E … (iv) (a) Data given in I and II are sufficient
2
(b) Data given in I and III are sufficient
E = 12 yr
3 (c) Data given in II and III are sufficient
From Eq. (iv), D = ´ 12, D = 18 yr (d) Data given in I, II and III are sufficient
2
l (d) According to the given information,
From Eq. (iii), C = 18 + 12
Gauri = Aaban - 15 … (i)
C = 30
Gauri = Seerat + 20 … (ii)
From Eq. (ii), Alvina = Seerat - 10 … (iii)
2B = 3 ´ 30
From Eqs. (i), (ii) and (iii),
B = 45
Aaban > Gauri > Seerat > Alvina
From Eq. (i), A = 45 ´ 2 = 90
Hence, data given in I, II and III are sufficient.
Þ Difference of A and C = A - C = 90 - 30 = 60
13. The number in the place of ‘?’ should be ………
10. If CLOUD = 11, BURST = 16 and ACE = 3, then
4 7
MONSOON = ? 7 6 9 8
(a) 13 (b) 15 (c) 17 (d) 19
l (b) As, C L O U D = 11 13 33
Positional
values 5 3 4 4
3+ 12+ 15 +21+4 55 10 5
Number of letters = = 11
5 5
B U R S T =16 6 7
Positional 8 5 6 10
values
2 + 21 +18 +19 +20 80 35 ?
Number of letters = = 16
5 5
and A C E =3 4 3 5 3
Positional 2 4
values (a) 30 (b) 32 (c) 34 (d) 36
1 + 3 +5 9 l (b) As, (7 ´ 5) + (6 ´ 3) - (4 ´ 10)
Number of letters = =3
3 3 35 + 18 - 40 = 13
Similarly, M O N S O O N (9 ´ 4) + (8 ´ 4) - (7 ´ 5)
36 + 32 - 35 = 33
(8 ´ 4) + (5 ´ 3) - (6 ´ 2)
13 +15+14 +19 +15 +15 +14 105 32 + 15 - 12 = 35
Number of letters = = 15 Similarly, (6 ´ 5) + (10 ´ 3) - (7 ´ 4), 30 + 30 - 28 = 32
7 7
\ MONSOON = 15 \ ? = 32
4 NTSE~ SOLVED PAPER 2019 (Stage II)

14. Find out which of the following figures can be 17. Study the following information
formed from the pieces given in the figure ‘X’? If ‘A$B’ means A is brother of B,
‘A@B’ means A is wife of B,
‘A#B’ means A is daughter of B and
‘AÁB’ means A is father of B.
Based on the above information, which of the
X
following alternative represents the correct group of
symbols that indicates the relationship for ‘K’ is
father-in-law of H?
(a) H@$L#PÁK (b) H@$PÁL#K
A B C D
(c) H@J$L#KÁP (d) H@P$JÁSI#K
Brother
(a) A (b) B (c) C (d) D l (c) A $ B Þ A( + ) ¬ ¾ ¾¾ ¾¾ B
Wife
l (a) Answer figure (A) can be formed from the pieces given ¾¾ B( + )
A @ B Þ A( - ) ¬ ¾ ¾
in the figure X. B
1 A # B Þ ­ Daughter
1 2 A( - )
A(+)
2 3 3 A Á B Þ ­ Father
B
From option (c), H @ J $ L # K Á P
15. Find the missing number ‘?’ in the figure given below
-law K Fat
er-in her
3 4 5 6 Fath
Daughter
7 22 3 8 30 5 9 30 6 10 ? 7 P
H J L
5 Brother
2 2 4
Clearly, K is the father-in-law of H.
(a) 30 (b) 32 (c) 33 (d) 35
l (d) Directions (Q. Nos. 18-20) The following figure represent
16. If MOBILE is coded as DFBICE, then CHARGE is
students who can play, sing and dance.
coded as
(a) CHBXQE (b) CLARTE
(c) CHAIGE (d) CHIAEF A B D
Play
Dance C
l (c) As, M O B I L E
13 15 2 9 12 5 F G
Sing

1+3 1+5 2 9 1+2 5


18. Which part of the figure represents students who
can sing and dance?
(a) F (b) C (c) F and C (d) E and G
4 6 2 9 3 5
l (c)

Play
D F B I C E Dance C
Similarly, C H A R G E F
3 8 1 18 7 5 Sing

Students who can sing and dance are F and C.


3 8 1 1+8 7 5 19. The number of students who can play is more by ‘a’
than the number of students who can dance; and the
number of students who can do both playing and
3 8 1 9 7 5 singing is more by ‘b’ than the number of students
who can do both singing and dancing. Then, what is
the difference of the number of students who can
only dance and who can only play?
C H A I G E (a) a + b (b) (2a - b) or (b - 2a)
(c) (a - 2b) or (2b - a) (d) (a - b) or (b - a)
NTSE~ SOLVED PAPER 2019 (Stage II) 5
l (d) According to the question, 23. Which of the following diagram indicates the best
B+ C+ D+ G = A+ B+ C+ F+ a relationship among men, fathers and teachers?
D+ G= A+ F+ a …(i)
C+ G= b+ F+ C
G= b+ F … (ii)
From Eqs. (i) and (ii),
A B C D
D+ b+ F= A+ F+ a
(a) A (b) B (c) C (d) D
D= A+ a- b
l (a) Diagram A indicates the best relationship among men,
D- A= a- b
fathers and teachers. All fathers are men. Some fathers as
or A- D= b- a well as some men can be teachers.
Hence, the difference of the number of students who can Men
only dance and who can only play is either (a - b) or (b - a).
Teachers
20. It is given that the total numbers of students in all
the three disciplines are same. Also, sum of the number Fathers
of students who can only dance, and twice of the number
of students who can do both singing and dancing, equals
the sum of the students who can do both singing and
playing and the students who can do both dancing
24. Ishan wishes Irfan ‘Good Morning’ when the hour
and playing. Then, which among the alternative is a
hand of a (measured clockwise) clock is positioned
correct statement about the number of students who
between 9 and 10. The angle between the two hands
can only play and those who can only sing?
is 145°. The time shown by the clock is
(a) The number of students who can only sing is twice as
(a) 9 : 08 AM (b) 9 : 10 AM (c) 9 : 12 AM (d) 9 : 15 AM
many as the number of students who can only play
(b) The number of students who can only sing is equal to the l (b) Given, h = 9, q = 145º Þ360 - 145 = 215 º
11 11
sum of the number of student who can sing and dance q= m ~ 30 h Þ 215 º = m ~ 30 ´ 9
and the number of students who can only play and sing 2 2
11 11m
(c) The number of students who can only play and sing equals m = 270 - 215, = 55, m = 10 min
the number of students who can only dance and play 2 2
Hence, time is 9 : 10 AM.
(d) The number of students who can only dance equals to the
number of students who can only sing 25. If ‘15 + 10 means 5’ ; ‘6 ´ 3 means 9’; ‘8 ¸ 4 means
l (a) 32’; and ‘12 - 2 means 6’; then what will be the
value of 27 + 81 - 9 ´ 6?
21. Complete the following series. (a) 36 (b) 24 (c) 12 (d) 6
1, -8, 81, ?, 15625 l (b) Given, 15 + 10 = 5
(a) -1022 (b) -1024 (c) -4094 (d) -4096 Here, ‘+’ means ‘-’.
l (b) The pattern of the series is as follows, \ 15 - 10 = 5
Þ 6 ´3 =9
1 –8 81 –1024 15625 Here, ‘´’ means ‘+’
\ 6+3 =9
Þ 8 ¸ 4 = 32
Here, ‘¸’ means ‘´’
(12) –(23) (34) –(45) (56) \ 8 ´ 4 = 32
\ ? = - 1024 Þ 12 - 2 = 6
Here, ‘-’ means ‘¸’.
22. Yaibiren is standing 4 m East of Rajib, who is 1 m \ 12 ¸ 2 = 6
North of Achira. If Sahibah is standing 3 m South of Þ27 + 81 - 9 ´ 6 = 27 - 81 ¸ 9 + 6 Þ27 - 9 + 6 = 24
Achira, then in which direction of Yaibiren is Sahibah?
(a) North-East (b) North-West (c) South-East (d) South-West 26. Which number will replace the ‘?’ in the following
sequence?
l (d) Rajib
5, 7, 14, 24, 42, ?, 119
4m Yaibiren
(a) 71 (b) 67 (c) 65 (d) 63
1m N l (a) The pattern is as follows,
N-W N-E
Achira 5 7 14 24 42 71 119
W E 5 + 7 + 2 = 14
3m 7 + 14 + 3 = 24
S-W S-E 14 + 24 + 4 = 42
S 24 + 42 + 5 = 71
Sahibah 42 + 71 + 6 = 119
Clearly, Sahibah is in South-West direction of Yaibiren. \ ? = 71
6 NTSE~ SOLVED PAPER 2019 (Stage II)

27. What will be the missing term ‘?’ in the given 31. Find the missing value ‘?’ in the following series.
series? 13, 34, 74, ?, 290
AK, FP, ?, PZ, UE, ZJ (a) 168 (b) 170 (c) 172 (d) 174
(a) KU (b) JT (c) JU (d) KV l (b) The pattern is as follows,
l (a) The pattern is as follows, 13 34 74 170 290
+5 +5 +5 +5 +5
A F K P U Z
+5 +5 +5 +5 +5
K P U Z E J
\ ? = KU 22+32 32+52 52+72 72+112 112+132
\ ? = 170
28. In a family of four members there is father, mother,
son and daughter. When sorted according to 32. What number comes in place of question mark (?) in
decreasing order of their ages, the order is father, the given figure?
mother, son and daughter. The difference between 5 4 3
the age of father and mother is 5 yr. The difference
6 4 7 2 2
between total age of male members and female
members is 15 yr. Also the total age of children is 7 2 1 ? 3 4 1
20 yr, then the age of the son is ……… 6 3 9 5 4
(a) 10 yr (b) 15 yr (c) 20 yr (d) 25 yr 9 6 3
l (b) According to the question, (a) 9 (b) 8 (c) 7 (d) 6
father > mother > son > daughter 5+3 8 6 + 3 + 5 + 4 18
l (a) As, = =4, = =9
F- M = 5 … (i) 2 2 2 2
(F + S ) - (M + D) = 15 … (ii) 6 + 4 + 2 + 2 14 9 + 3 12
S + D = 20 … (iii) = = 7, = =6
2 2 2 2
From Eqs. (i) and (ii), 7 + 2 + 1 + 3 + 4 + 1 18
Similarly, = = 9
5 + (S - D) = 15 2 2
S - D = 10 … (iv)
33. The following figures represent information given
From Eqs. (iii) and (iv),
against them.
S + D = 20
Total number of students who applied for Board
S - D = 10
Examination.
2S = 30
D Total number of students who actually appeared at
Hence, S = 15 yr
Board Examination.
29. If the ninth day of a month is four day earlier than  Total number of urban students who appeared at
Thursday, then what day will it be on the twenty Board Examination.
third day of the month? Total number of students who qualified at Board
(a) Monday (b) Wednesday
Examination.
(c) Friday (d) Sunday
Based on the above information which of the
l (d) 9th day of a month = Thursday - 4 days = Sunday following figures represents the above facts?
Then, 23rd day of a month = Sunday.

30. Which number will replace the question mark (?) in (a) (b)
the given figure?

8 20 (c) (d)
14 2
l (c) Students applied for
6 12 board examination
? 10
Students appeared at
(a) 4 (b) 16 (c) 18 (d) 22 board examination
l (b) As, 14 + 8 = 22 Urban students appeared
20 + 2 = 22 at board examination
10 + 12 = 22 Students qualified at
board examination
Similarly, 6 + 16 = 22
Option (c) is correct choice.
\ ? = 16
NTSE~ SOLVED PAPER 2019 (Stage II) 7
34. Five friends P, Q, R, S and T read a newspaper. The 38. Which of the following figure(s) can not be drawn
one who reads first gives it to R. The one who reads without either lifting the pen or re-tracing any line?
last had taken it from P. T was neither the first nor
the last one to read, There were two readers
between Q and P. Who reads the newspaper last?
(a) P (b) Q (c) R (d) S
l (d) According to the question,
A B C D
Q R T P S (a) Only A (b) Both A and B
Hence, S reads the newspaper last. (c) Only C (d) Both C and D
35. A clock shows 05:45. A plane mirror is kept on the l (c) Only figure C can not be drawn without either lifting
right of the clock, with its plane perpendicular to the the pen or re-tracing any line.
face of the clock. What time will be shown by the 39. Find the missing values in place of the question
mirror image? marks in the given pattern.
(a) 06:45 (b) 05:15
(c) 06:15 (d) 07:15 1 X 5 ? 34
l (c)
1 3 8 21

Y 2 U ? R
(a) 1 (b) N (c) M (d) Z
13 10 13 18
l (c) 1 X 5 M 34
Þ 11 : 60
- 05 : 45
6 : 15 1 3 8 21
Hence, time be mirror image is 06 : 15.

36. In a certain code language ‘‘Kolkata is cultural hub Y 2 U 13 R


of India’’ is coded as ‘‘a2463b’’ and ‘‘Mumbai is As, 1+ 2 =3
financial hub of India’’ is coded as ‘‘g3472b’’. Then 2 + 3 =5
in the same language ‘‘India is hub of democracy’’ 5 + 8 = 13
may be coded as 13 + 21 = 34
(a) a 2 4 3 9 (b) 2 4 3 g 7 Now, 4 + 1 = 25 + 1 = 26
(c) b 3 2 4 9 (d) 3 2 b 4 7
2 + X = 2 + 24 = 26
l (c) “Kolkata is cultural ‘‘hub of India’’ “a 2 4 6 3 b” U + 5 = 21 + 5 = 26
“Mumbai is financial “hub of India” “g 3 4 7 2 b” Þ ? + 13 = 26
Code for “India is hub of” “2 3 4 b” Þ ? = 13 = M
Hence, according to the option (c) code for ‘‘India is hub R + 34 = 18 + 34 = 52
of democracy’’ is
Þ 52 ¸ 2 = 26
= b3249
[Q English alphabet contains 26 letters]
37. Which letter is midway between 13th letter from the \ ?, ? = M 13
left and the 4th letter from the right in the sequence
40. What will be the missing number in the given series?
given below?
1332, 732, 348, ………, 36, 12
USBEYFHKOPRAWCGJMQDIVLNTXZ
(a) 32 (b) 132
(a) O (b) Q (c) P (d) M
(c) 148 (d) 216
l (b) The given sequence,
l (b) The pattern is as follows
USBEYFHKOPRAWCGJMQDIVLNTXZ
1332 732 348 132 36 12

13th from 4th from


the left the right 113+1 93+3 73+5 53+7 33+9 13+11
\ Middle letter between W and N is Q . \ ? = 132
8 NTSE~ SOLVED PAPER 2019 (Stage II)

41. Find the missing term ‘?’ in the given figure. l (c) Given expression =
5 + 4 ´ 8 ¸ 12 - 3 = 3
I12
From option (c),
D6 5 + 3 ´ 8 ¸ 12 - 4 = 3 [by interchanged]
? 2
5 + 3 ´ -4 =3
3
A2 Y30
5 + 2 -4 =3
(a) N10 (b) P20 (c) O 24 (d) Q16 5 -2 =3
l (b) A +3 +5 +7 +9 3 =3
D I P Y
Hence, option (c) is correct choice.
+4 +6 +8 +10
2 6 12 20 30
\ ? = P20 .
46. If a, b, c, d and e are positive numbers, and it is
given that
42. If, a > b, a > 0, and b ¹ 0, then which of the following a + b = c + d, b + d = 2a
statements is always true? d + e > a + b and c + d > a + e
(a) a ´ b > 0 (b) a ´ b < 0
then, which of the following statement is true?
(c) a ´ b is undefined (d) a ´ b 2 > 0
(a) d > a > b > e > c (b) d > b > e > a > c
l (d) Given, a > b, a > 0 (c) a > b > c > d > e (d) a > d > b > e > c
Case - 1 a > 0, b > 0 and a > b
l (a) Given, a + b = c + d, … (i)
Case - 2 a > 0, b < 0 and a > b
b + d = 2a … (ii)
Hence, a ´ b2 > 0 always true.
d+ e> a+ b … (iii)
43. In certain coded language c+ d> a+ e … (iv)
‘way to win’ is written as AAaa aaaa AAAa, From Eqs. (i) and (iv),
‘Go to Walk’ is written as Aaaa aaaa AAAA, a+ b> a+ e
‘Get up early’ is written as AaAa AaaA aaAA. \ b> e
From Eqs. (i) and (iii),
Then how can ‘Always go to morning walk early’ be
d+ e> c+ d
written in that coded language?
\ e> c
(a) aaAA Aaaa aaaa aaaA AAaa aAAA
(b) aaAA Aaaa aaaa aaaA AAAA AaAA From Eq. (iv), c+ d> a+ e
(c) aaAA AaAa aaaa aaaA aaAA AAaas Q e> c
(d) aaaA AaAa aaaa aaAA AAAA aaAA \ d> a
l (b) Way to win AAaa aaaa AAAa …(i) From Eq. (ii), b + d = 2a
Go to walk Aaaa aaaa AAAA …(ii) Þ b+ d= a+ a
Get up early AaAa AaaA aaAA …(iii) Q d> a
From options Eqs. (ii) and (iii) \ b< a
Code for ‘Always go to morning walk early’ So, we have d> a> b> e> c
aaAA Aaaa aaaa aaaA AAAA AaAA
47. Kashvi facing towards rising sun turned to her left
44. If + means ´; ¸ means -; - means +; and ´ means ¸, and walks for 60m. She then turned to West and
then 2 + 12 ´ 4 - 6 ¸ 6 is equal to ……… . walked for 15m. Then she turned towards left at an
(a) 0 (b) 6 (c) 12 (d) 49 angle of 45° and reached 95 m from her original
l (b) Given, position. How much total distance did she travel?
(a) 95 m (b) 115 m
+ Þ´
(c) 155 m (d) 175 m
¸ Þ-
- Þ+
l (d) A 15m
N
´ Þ¸
2 + 12 ´ 4 - 6 ¸ 6
0m

60m 60m
10

By interchanging signs, W E
= 2 ´ 12 ¸ 4 + 6 - 6 = 2 ´ 3 = 6 80m
Starting
45. In the given equation, which two numbers in the B C O S
95m position
expression on the left hand side will be
interchanged to form a correct equation? AB2 = AC2 + BC2 = 602 + 802
= 3600 + 6400 = 10000
5 + 4 ´ 8 ¸ 12 - 3 = 3
AB = 100 m
(a) (3, 5) (b) (4, 12 )
Hence, total distance = 60 + 15 + 100
(c) (3, 4) (d) (8, 5)
= 175 m
NTSE~ SOLVED PAPER 2019 (Stage II) 9
48. A cube is coloured on all the six faces with six different l (b) Given, (A) x2 = 49
colours–black, brown, green, red, yellow and blue. (B) x3 = 343
Red face is opposite to the black face. From (A), x2 = 49
x = ±7
Green face is between red and black faces.
Þ x = 7, x = - 7
Blue face is adjacent to yellow face. From (B), x3 = 343
Brown face is adjacent to blue face. x =7
Red face is in the bottom. Hence, only B is sufficient to answer the question.
Which of the following are adjacent to green? \ Conclusion II follows.
(a) Black, yellow, brown, red (b) Blue, black, red, yellow
(c) Red, black, blue, yellow (d) Yellow, blue, black, red
51. Find the values of ‘x’ and ‘y’ from the figure given
below.
l (a) According to the question,
E F 20 42
Black 5 7
12 x
H G 4 8
Blue
Yellow 13 9
Brown Green y 72
11
B
110
Red C
(a) 65, 150
A D (b) 46, 125
ABCD = Red (c) 56, 156
EFGH = Black (d) 56, 165
HEBA = Brown l (c) As, 4 ´ (4 - 1) = 4 ´ 3 = 12
GFCD = Yellow 5 ´ (5 - 1) = 5 ´ 4 = 20
HGDA = Green
7 ´ (7 - 1) = 7 ´ 6 = 42
EFCB = Bule
9 ´ (9 - 1) = 9 ´ 8 = 72
Hence, adjacent to green are red, black, yellow, brown.
11 ´ (11 - 1) = 11 ´ 10 = 110
49. A watch gains 10 seconds in 3 min. It was set right
13 ´ (13 - 1) = 13 ´ 12 = 156
at 9 AM. In the evening of the same day, when the
watch indicates half past 6’O clock, the true time is 8 ´ (8 - 1) = 8 ´ 7 = 56
(a) 5:30:00 PM (b) 5:48:10 PM (c) 5:58:20 PM (d) 6:08:20 PM Þ (x, y) = (56, 156)
l (c) Total time from 9 : 00 AM to 6 : 30 PM = 9 : 30 h 52. In a certain code ‘COUNTRY’ is written as
= 9 ´ 60 + 30 = 570 min ‘ZSUOVPD’. How is ‘TEACHER’ written in the
Now, 3 min 10 sec i.e. 190 sec of this clock is equal to the same code?
3 min i.e. 180 sec of correct clock.
(a) SUTIFED
180
\ 570 min of incorrect clock = ´ 570 min of correct clock (b) REHCAET
190 (c) QDGBDS
= 540 min i.e. 9h of correct clock (d) SFIDBFU
\ Actual time = 9 : 00 AM + 9h = 6 :00 PM» 5 : 58 : 20 PM
l (d) As,
(approximate) COU N T R Y Z S U OV P D
+1
50. Given x is real and that +1
+1
(A) x2 = 49, (B) x3 = 343 +1
Examine the given alternatives in respect of arriving +1
+1
at the Conclusion; x = 7 and find which is valid +1
I. Only A is sufficient to answer the question Similarly,
II. Only B is sufficient to answer the question T E A C HE R S F I DBF U
III. Either A or B alone is sufficient to answer the +1
+1
question +1
IV. Both A and B together are sufficient to answer the +1
+1
question +1
(a) I (b) II (c) III (d) IV +1
10 NTSE~ SOLVED PAPER 2019 (Stage II)

53. What number should replace the question mark? 54. (b) E teaches Hindi language.
55. (d) B teaches on Sunday.
14
56. (a) F teaches Manipuri language.
? 12
57. (b) G teaches English language.
15 13
58. (c) D teaches on Wednesday.
16 2 2 13
4 3 59. One morning at 8 AM Navneet and Ravneet were
18 11
2 6 standing on a lawn with their back towards each
19 2 3 17
other at the distance of 100 m. Navneet’s shadow
16 12
fell exactly towards his left hand side. After 15 min,
15 12 Ravneet turns 135° anti-clockwise. Which direction
10 Ravneet is facing now?
(a) North-East (b) North-West
(a) 15 (b) 14 (c) East (d) South-East
(c) 13 (d) 10
l (a)
l (c) Left Right
Sun N
Navneet
Directions (Q. Nos. 54-58) Read the information carefully Navneet’s W E
Shadow 135° (N-E)
and answer the questions given below. S
A, B, C, D, E, F and G are seven teachers. Each one teaches only Ravneet
one and different language from among Konkani, Hindi, Right Left
Malayalam, English, Manipuri, Tamil and Kannada on different
days of a week. C teaches Malayalam on Friday. B teaches Navneet’s shadow falls on left side, so he is facing towards
Konkani on the next day of the day on which the concerned North and Ravneet faces opposite to Navneet i.e. facing
teacher teaches English. F teaches on Thursday but neither towards South. After moving 135º anti-clockwise, Ravneet
teaches Hindi nor English. D teaches Tamil on the previous day is facing North-East.
on which day F teaches. A teaches Kannada on Tuesday.
G teaches on the next day of the day on which the concerned 60. Find the missing number.
teacher teaches Malayalam. E does not teach English. 2, 3, 7, ……, 2112
(a) 36 (b) 45
54. Which subject does E teach?
(c) 46 (d) 49
(a) Tamil (b) Hindi
(c) Manipuri (d) Malayalam l (c) The pattern is as follows

55. On which day B teaches? 2 3 7 46 2112


(a) Monday (b) Friday (c) Wednesday (d) Sunday
22–1 32–2 72–3 462–4
56. Which language does F teach?
(a) Manipuri (b) Kannada \ ? = 46
(c) Tamil (d) English
61. In a code BH = 16, DO = 60 and TA = 20, then the
57. Which language does G teach? code for BAT = ?
(a) Hindi (b) English (a) 20 (b) 30
(c) Kannada (d) Konkani (c) 40 (d) 60
58. On which day D teaches? l (c) As, B H = 16
(a) Saturday (b) Tuesday Positional values
(c) Wednesday (d) Thursday
2 × 8 = 16
Sol. (Q. Nos. 54-58) According to the given information, D O = 60
Teachers Language Day Positional
values
A Kannada Tuesday 4 ×15 = 60
B Konkani Sunday and T A = 20
C Malayalam Friday Positional
values
D Tamil Wednesday 20 ×1 = 20
E Hindi Monday Similarly, B A T
Positional
F Manipuri Thursday
values
G English Saturday 2 ×1 ×20 = 40
NTSE~ SOLVED PAPER 2019 (Stage II) 11
62. The figure given below is prepared by some sticks Figure - 2
x + z - y = 130 …(ii)
and provides an equation that is incorrect.
From Eqs. (i) and (ii)
How many minimum numbers of sticks must be
x - y + z = 130
removed from the left hand side to make it a correct
- x + y + z = 110
equation?
2z = 240
z = 120
Hence, the height of the wall (z) = 120 cm
(a) 1 (b) 2
(c) 3 (d) 4 65. In a certain coding scheme, consonants and vowels
are coded differently as illustrated below:
l (c) Given,
C is coded as 6.
Z is coded as 52.
E is coded as 9.
Þ

O is coded as 29.
Then find the sum of numerals in the coded version of
FAITH.
(a) 84 (b) 85
Minimum 3 sticks must be removed. (c) 86 (d) 87
l (c) Consonant Þ Place value ´ 2
63. If 63578 is to 1415,
Vowel Þ Place value ´ 2 - 1
56732 is to 185,
F A I T H
and 34124 is to 86,
then, 72648 is to ?
(a) 1215 (b) 1415 (6×2) (1×2–1) (9×2–1) (20×2) (8×2)
(c) 1512 (d) 1514
Þ 12 + 1 + 17 + 40 + 16
l (c) As, 14 15
Þ 86
63578 (6+3+5) (7+8)
18 5 66. In a class 20% of students are below 14 yrs of age.
56732 (5+6+7) (3+2) Out of the remaining students 10% are of the age
14-15 yrs and ratio of students who are between
8 6 15-16 yrs of age to student above 16 yrs of age is
34124 (3+4+1) (2+4) 3 : 2. If the number of students who are above
Similarly, 72648 (7+2+6) (4+8) 16 yrs is 72, what is the total number of students in
15 12 the class?
\ ? = 1512 (a) 200 (b) 250 (c) 300 (d) 400
l (b) Let the total students = x
64. Two friends Mr. A and B stand according to figure x
students are below 14 yrs.
1. The two friends then interchange their positions 5
as given in figure 2. x 4x
Remaining = x - =
5 5
B A 4x 10 2x
´ = are of14 - 15 yrs.
110 cm Wall 130 cm Wall 5 100 25
4x 2x 18 x
Remaining = - =
5 25 25
A B Now, between (15 - 16) yrs : above 16 yrs = 3 : 2
Now, let the number of students above 16 yrs age = 2y
Figure-1 Figure-2
and between (15-16) yrs = 3y
The height of the wall from the ground is ……… . Above 16 yrs = 2 y = 72
(a) 115 cm (b) 120 cm Þ y = 36
(c) 127.5 cm (d) 130 cm and between (15 - 16) yrs = 3 y = 3 ´ 36 = 108
18 x
l (b) Let the height of wall is z. Now, = 108 + 72
Length of A = x 25
18 x
Length of B = y = 180
Figure - 1 25
y + z - x = 110 ... (i) Þ x = 250
The total number of students = 250
12 NTSE~ SOLVED PAPER 2019 (Stage II)

67. Study the figure given below representing a 69. Study the given figure and answer the following
particular number in coded manner. question.
0 13

11 5
8
6 2
12 9 7 6 4
3
2 3
10
5
1
For example, the number 6825 coded by the following
symbols
Let x denote sum of numbers present in at least 2
circles and y denote sum of numbers present in
exactly 3 circles. Then x - y = …… .
(a) 11 (b) 25
(c) 36 (d) 61
Based on the above information find the number l (c)
coded for the following symbols. 13

11
8 5

12 9 7 6 4

10 2
3
(a) 63205 (b) 11309
(c) 11523 (d) 65230 1
l (b)
0 3
The sum of numbers present in atleast 2 circles (x)
6 = 11 + 5 + 3 + 10 + 9 + 8 + 6 + 2 + 7
5
2 = 61
6 The sum of numbers present in exactly 3 circles (y)
3 0 6 3
5 =9 + 8 + 6 + 2
= 25
x - y = 61 - 25
6+5 3 0 6+3 = 36

70. Choose the correct mirror image of the following


11 3 0 9 figure. If the mirror is placed as shown.
Þ 11309

68. Five friends decided to play a game of badminton.


Each of the five plays against every other friend.
The winner gets two points for each game he or she
wins and the loser gets zero. Then, which of
the following cannot represent the scores of five
(a) (b)
friends?
(a) 4, 4, 4, 4, 4 (b) 6, 4, 4, 4, 2
(c) 8, 8, 2, 2, 0 (d) 6, 6, 4, 2, 2
l (c) There are five friends. If one won all 4 games; he/she (c) (d)
scores 8 points and no other person can win, 4 games.
So 8 points cannot be repeated.
l (a) Option (a) is the correct mirror image of the given
Hence, (8, 8, 2, 2, 0) cannot represent the score of the five figure.
friends.
NTSE~ SOLVED PAPER 2019 (Stage II) 13
71. Observe the figures given below: Directions (Q. Nos. 74-76) Read the information carefully
and answer the questions given below.
Five students Ujith, Mahi, Rizan, Sahir and Amelia appeared for
an examination in English and Mathematics.
1 2 3 4 5
I. Sahir scored more marks than Amelia in
Mathematics but scored less in English than
Ujith and Mahi.
6 7 8 9 II. In Mathematics Rizan scored more marks than
Amelia but less than what Mahi has scored.
Based on the above figures identify the correct group
of categorisation? III. Amelia scored more than Rizan in English and
(a) 1, 3, 6; 2, 4, 9; 5, 7, 8 (b) 1, 2, 3; 4, 5, 8; 6, 7 9 Rizan scored more than Mahi in English.
(c) 1, 6, 8; 3, 5, 9; 2, 4, 7 (d) 1, 3, 6; 2, 5, 7; 4, 8, 9 IV. Ujith scored more than Mahi in Mathematics but
l (c) Correct group is (1, 6, 8) (3, 5, 9) (2, 4, 7) less than Rizan in English.
As, figures 3, 5 and 9 are divided into two parts, figures 2, V. Sahir scored less than Mahi in Mathematics.
4 and 7 are divided into four parts. 74. The least scorer in Mathematics and top scorer in
72. Raju invited friend George for a dinner at his house. English are respectively
When George asked for the direction of Raju’s (a) Sahir and Ujith (b) Amelia and Amelia
house, Raju gave him the following instruction: (c) Ujith and Sahir (d) Ujith and Ujith
Proceed 140 m South from your house then walk 75. Which of the following cannot be determined?
200 m to East. Then turn to North and walk 100 m. (a) Amelia scored more than Mahi in English.
After that, walk 160 m to West. (b) Mahi scored more than Amelia in Mathematics.
What is the shortest distance between the two houses (c) Sahir scored less than Mahi both in Mathematics and
and the direction to Raju’s house from George’s English.
house? (d) Ujith scored less than Mahi in English.
(a) 40 2 m and North-West (b) 40 2 m and South-East 76. Which of the following is necessarily correct?
(c) 80 m and South-East (d) 80 m and North-West (a) Rizan scored more than Sahir in Mathematics
l (b) A (b) Ujith scored more than Sahir both in Mathematics and
(George's house)
English
N
40m N-W N-E (c) Sahir scored more than Ujith in Mathematics
(Raju's house) (d) Rizan scored more than Ujith both in English and
F W E Mathematics.
B E
40m 160m S-W S-E Sol. (Q. No. 74-76) According to the question,
100m 100m S Mathematics
Ujith > Mahi > Rizan / Sahir > Sahir / Rizan > Amelia
C D English
200m
Amelia > Rizan > Ujith/Mahi > Mahi/Ujith > Sahir
In DABF, AF2 = AB2 + BF2 = 402 + 402
74. (b) The least scorer in Mathematics is Amelia and top
= 1600 + 1600
scorer in English is Amelia.
AF = 2 ´ 1600 = 40 2m
Raju’s house is in South-East direction of George’s house.
75. (d) Option (d) cannot be determined.
76. (b) Option (b), Ujith scored more than Sahir both in
73. In a code language if ‘APPEAL’ is coded as ‘256572’ Mathematics and English is necessarily correct.
and ‘PLAY’ is coded as ‘7259’ then in the same
language ‘PEARL’ will be coded as (each number 77. The third day before 1st January 2019 was Saturday.
code stands for unique alphabet) ……… . Which day will the fourth day of March 2020 be?
(a) 25768 (b) 25387 (c) 67522 (d) 25679 (a) Friday (b) Saturday
(c) Wednesday (d) Thursday
l (a) As,
A P P E A L Þ 2 5 6 5 7 2 l (c) Day on 1st January 2019 = Tuesday.
Day on 1st January 2020 = 1 odd day
P L A YÞ 7 2 5 9
= Tuesday + 1
Here, A/P = 2/5
= Wednesday
E=6
Day on 4th of March 2020 = Jan + Feb + 4 days of March
L =7
= 2 + 1 + 4 = 7 odd day
Y =9
= 0 odd day
So, PEARL = 25768 (from options R = 8)
Þ 4th March 2020 = Wednesday
Hence, code for P E A R L is 2 5 7 6 8.
14 NTSE~ SOLVED PAPER 2019 (Stage II)

78. Observe the given figure below l (c) Figure (C) will complete the given pattern.
A B C
G

D E F
Based on the figure how many maximum numbers of
triangles can be formed with the seven points A, B, C,
D, E, F and G? 83. Find the correct water image for the following
(a) 21 (b) 24 (c) 33 (d) 36
problem figure choosing from the alternatives.
l (c)
79. Find the correct mirror image for the following
problem figure from the alternatives. Water

A B C D A B C D
(a) A (b) B
(c) C (d) D (a) A (b) B (c) C (d) D
l (b) Figure (B) is the correct mirror image of the given l (b) Figure (B) is the correct water image of the given
problem figure. problem figure.

80. A circular disc is cut into two parts. One of the part Directions (Q. Nos. 84-88) In the following questions,
is given as the question figure. Which is the other there are statements followed by conclusions. Choose the
part? Select from the options. conclusion(s) which must logically follow(s) from the
given statements.
84. Statements
A. Some grandmothers are mothers.
A B C D
B. Some mothers are daughters.
(a) A (b) B
(c) C (d) D C. All the daughters are married women.
l (c) Figure (C) will complete the given question figure. Conclusions
I. Some married women are mothers.
81. Two figures on transparent sheets are given on the
II. Some daughters are grandmothers.
left side. When the upper figure is exactly placed on
the lower figure, find from the option figures how III. No daughter is grandmother.
the resultant looks like. IV. Some mothers are grandmothers.
(a) I and II
(b) II and III
(c) II and IV
(d) I and IV
l (*)
A B C D
Grandmothers Mothers Grand
mothers Mothers
(a) A (b) B (c) C (d) D
l (d) When the upper figure is exactly placed on the lower Daughters Or
Daughters
figure, it is shown as in figure (D).
Married
Women Married Women
82. Find the missing part of the given figure from the
alternatives which completes the pattern.
Conclusions
(I) ü
(II) û
(III) û
A B C D
(IV) ü
(a) A (b) B (c) C (d) D Here, either II or III and I and IV follow.
NTSE~ SOLVED PAPER 2019 (Stage II) 15
85. Statements l (c) Men Teacher
A. Some students are smart-working.
Doctors
B. All intelligent are smart-working. Women
C. All the teachers are students.
Conclusions
I. Some students are intelligent. Conclusions
II. No teacher is smart-working. (I) û (II) ü (III) ü (IV) û
III. Some intelligent are students. Hence, Conclusions II and III follow.
(a) Either I or II (b) I and II 88. Statements
(c) None of I, II and III (d) I and III
A. Some candidates are students.
l (c) Students Smart-working B. All children are citizens.
C. All citizens are candidates.
Conclusions
Teacher Intelligent I. Some citizens are students.
II. Some candidates are children.
III. All children are candidates.
Conclusions IV. No child is student.
(I) û (a) I and II (b) II and III (c) III and IV (d) I, II and III
(II) û l (b) Candidates Students
(III) û
Hence, none of I, II and III follow. Citizens
86. Statements
A. Some students are orators.
B. All orators are goalkeepers. Children
C. Some goalkeepers are honest. Conclusions
Conclusions (I) û (II) ü (III) ü (V) û
Hence, Conclusions II and III follow.
I. Some students are honest.
II. Some goalkeepers are students. 89. Study the figure given below:
(a) Only Conclusion I follows
(b) Only Conclusion II follows
(c) Both Conclusions I and II follow
(d) Neither conclusion I nor II follows
A B C D E
l (b) Students Orators Goalkeepers
Find which figure is to be removed, starting from A,
Honest
so that all fit into a pattern.
(a) B (b) C
(c) D (d) E
l (c) In the given pattern, outermost rectangle is moving 90º
Conclusions clockwise and shaded rectangle is moving 90º
(I) û anti-clockwise.
(II) ü Hence, figure (D) should be removed.
Hence, only Conclusion II follows.
90. What is the minimum number of un-shaded boxes to
87. Statements be crossed for covering the shortest path from ‘A’ to
A. Some men are women. ‘B’ (both exclusive) without retracting the path and
without diagonal movements?
B. All women are teachers.
A
C. Some teachers are doctors.
Conclusions
I. Some doctors are women.
II. Some teachers are women.
III. Some teachers are men. B
IV. Some doctors are men. (a) 8 (b) 9
(a) I and II (b) I and IV (c) II and III (d) III and IV (c) 10 (d) 11
16 NTSE~ SOLVED PAPER 2019 (Stage II)

l (b) A 93. How many people read only one newspaper?


× (a) 570 (b) 642 (c) 914 (d) 968
× × 94. How many people read all the three newspaper?
× × × × × (a) 40 (b) 58
× (c) 70 (d) 90
B Sol. (Q. Nos. 93 and 94)
The minimum number of un-shaded boxes to be crossed
Hindi English
is 9.
570 a 58 b 424
91. Observe the figure given below: x
40 70

c 254

A B C D Punjabi
The odd one out from the given figure is …… . 570 = a + 58 + x + 40
(a) A (b) B (c) C (d) D 570 - 58 - 40 = a + x
l (b) Figure (B) is different from others, as both figures a + x = 472 … (i)
overlaps each other in the right side. 424 = b + 58 + x + 70
b + x = 296 … (ii)
92. A river flows along the East-West direction. On a 254 = c + 40 + x + 70
particular day in the morning Kisku was seen at a c + x = 144 … (iii)
place ‘A’ located on the northern side of the river
From Eqs. (i), (ii) and (iii),
and on the same evening he was seen at a place ‘B’
a + b + c + 3x = 912 … (iv)
located on the southern side of the river.
and a + b + c + 40 + 58 + 70 + x = 1000 - 100
Following are the comments made by four friends. a + b + c + x = 900 - 168
Paulomi, Mimee, Sabeena and Grayson.
a + b + c + x = 732 … (v)
I. Paulomi said, Kisku must have crossed the river From Eqs. iv and v, x = 90
only once.
II. Sabeena said, Kisku might have crossed the river 93. (b) People read only one newspaper =
four times. a + b + c + x = 732
III. Mimee said, he might have crossed it five times. Þ a + b + c + 90 = 732
IV. Grayson said, he might have crossed it any Þ a + b + c = 732 - 90
number of times. Þ a + b + c = 642
Choose the correct alternative from the following. 94. (d) People read all the three newspaper (x) = 90
(a) Only I is correct North (Kisku)
A 95. Complete the given letter analogy.
(b) Only II is correct LTFQIW : YGSJVD : : DOIYKV : ?
(c) I or III is correct (a) QBVIXL
(d) Both I and II are correct W E (b) WLRBCI
River
l (c) In the morning, Kisku (c) QLVBXE
was seen on northern side. (d) QBVLXJ
In the evening, Kisku was
B (Kisku) South l (a) As, L T F Q I W : Y G S J V D
seen on southern side. So,
Kisku must crossed the river once or might have crossed it
five times. Hence, either Statement I or III is correct. 12+20+6+17+9+23 25+7+19+10+22+4
Directions (Q. Nos. 93 and 94) Read the information = 87 = 87
carefully and answer the questions given below. Similarly,
D O I Y K V : Q B V I X L
In a town of 1000 people, 570 read Hindi newspaper, 424 read
English newspaper and 254 read Punjabi newspaper. 40 read
Hindi and Punjabi newspaper; 58 read Hindi and English 4+15+9+25+11+22 17+2+22+9+24+12
newspaper; and 70 read Punjabi and English newspaper. = 86 = 86
100 read no newspaper.
? = QBVIXL
NTSE~ SOLVED PAPER 2019 (Stage II) 17
96. The given pie-diagram shows the streams opted by l (d) When child starts from the first step of the ground level
students at senior-secondary level. and mother starts coming down from the fourth step from
the floor level of the first floor, they meet at 12th step from
Commerce the floor level of the first floor.
and Management
18% First floor

35% Arts 4th


Computer 12% 12th
Science
Ground floor
Ist
25%
10%
Applied Science
99. The following question consists of four problem
Basic Science figures marked as A, B, C and D. Select a figure in
If sum of the angles for the students who opted place of ‘?’ for E which will continue the series
different streams is 144°, then the streams are ……… . established by the four problem figures, A, B, C, D.
(a) Arts, Applied Science ÷ – ×
+ ÷ + ÷
÷
(b) Basic Science, Computer Science
(c) Basic Science, Commerce and Management ?
(d) Applied Science, Computer Science, Commerce and –
+ + –
× × – ×
Management A B C D E
144 º
l (d) Sum of angles 144º in percentage = ´ 100 = 40% ÷ – × ÷ × ÷ – ÷
360 (a) (b) (c) (d)
Now, Applied Science + Computer Science + Commerce
and Management = 10 + 12 + 18 = 40% + × – + + – × +
.
Hence, option (d) is correct choice.
l (d) In the given series, all symbols change their positions
97. Four relation have been given as alternatives (p), (q), as
(r), (s) out of which only one becomes acceptable if
the signs, + and ¸ and the numbers, 4 and 5 are
mutually interchanged. Identify that relation.
(p) 24 + 8 ´ 4 = 20 ¸ 5 (q) 20 ¸ 4 ´ 16 + 5 = 75
Hence, option (d) will complete the given series.
(r) 3 ´ 24 + 5 = 16 ¸ 4 (s) 20 ¸ 5 - 6 = 3 ´ 30 + 4
(a) (p) (b) (q) (c) (r) (d) (s) 100. Which one of the following Venn diagrams
l (d) From alternative (d), represents the relation among men, doctors and
20 ¸ 5 - 6 = 3 ´ 30 + 4 patients in a hospital?
By interchanging signs and numbers,
20 + 4 - 6 = 3 ´ 30 ¸ 5
18 = 3 ´ 6 Þ 18 = 18
Hence, option (d) is correct choice. A B C D
98. There are 20 steps to go to the first floor of a (a) A (b) B
building from the ground floor. (c) C (d) D
A child starts climbing up from the first step of the l (a) Venn diagram (A) best represent the relation among
ground level. Mother starts coming down from the men, doctors and patients.
fourth step from the floor level of the first floor. Doctors Men Patients
If both have started at the same time with same speed,
at which step would they meet counting from the first
step from the floor level of the first floor?
Some doctors can be men and some patients can be men.
(a) 9 (b) 10 (c) 11 (d) 12
18 NTSE SOLVED PAPER 2019 (Stage II)

Paper II Scholastic Aptitude Test (SAT)


101. A set of guidelines called a model code of conduct is 104. Which of the following statements exemplify the
enforced during Parliamentary election in India to Independence of judiciary in India?
avert corrupt practices. This is applied to ………… . I. Judiciary is not under the control of executive
(a) Political parties only and legislature.
(b) Political parties and voters
II. There is less scope of interference in the working
(c) Political parties and candidates
(d) Candidates contesting elections only
of judiciary by the political executive.
III. A judge of higher judiciary can be removed only
l (c) The Model Code of Conduct (MCC) is a set of 2
guidelines issued by the Election Commission to regulate through a resolution which requires rd majority
political parties and candidates prior to elections, to 3
ensure free and fair elections. Article 324 of the of both the houses of parliament.
Constitution gives the power to Election Commission to Choose the correct option.
supervise elections to the Parliament and state
(a) I and II (b) I and III (c) I, II and III (d) II and III
legislatures. The MCC becomes operational from the date
the election schedule is announced till the date the results l (c) Judiciary in India is kept independent i.e. it is not
are declared. under the control of both the legislature and executive.
This basically means that both the legislature and the
102. A country has the ultimate rights and power to make executive branches of the government cannot interfere
decisions on internal and external matters. Also it is with the functions and decisions of the judiciary. The
not dictated by any external power on its decisions Judges of the Supreme Court and High Courts are
relating to its relation with other country. Which appointed by the President.
feature is reflected in these statements? While appointing other judges in the Supreme Court, the
President consults the Chief Justice of India. This ensures
(a) Republic (b) Socialism (c) Sovereignty (d) Authoritarian that there is less scope of interference in the working of
l (c) ‘Sovereignty’ means the independent authority of a judiciary by the political executive. No judge can be
country. It has two aspects-internal and external. From the removed from the office except by a very difficult process
internal point it means that it has the power to legislate on of impeachment which requires 2/3rd majority of both
any subject, to promote the health, morals, education, houses of parliament.
subject only to the federal division of legislative powers
and other limitations imposed by the Constitution, such as 105. Rajya Sabha is also called the Upper House, elders
Fundamental rights. house and permanent house. Which of the
The external sovereignty is the right of a country to be free statement/s given below is/are true about it?
from any external interference that would disrupt or I. Rajya Sabha has more power related to financial
remove the right of that country to exist and govern its matters.
own territory.
II. Rajya Sabha members continue to be in the office
103. Anti-defection law is an important feature of Indian till the next general election.
party system. The final authority to decide on the III. Resolution for removing the Vice-President and
disqualification of a member of the parliament with the President originates in Rajya Sabha.
respect to anti-deflection lies with the ………… . IV. Number of seats allotted to a state in Rajya Sabha
(a) President is directly proportionate to its population.
(b) Prime Minister
(a) I, II and III (b) III and IV (c) I and IV (d) Only IV
(c) Chief Justice, the Supreme Court of India
(d) Speaker of the Lok Sabha or the Chairman of the Rajya l (d) The IVth Schedule to the Constitution provides for
Sabha allocation of seats to the States and Union Territories in
Rajya Sabha. The allocation of seats is made on the basis
l (d) 52nd Constitutional Amendment Act, 1985 provided of the population of each State. A Money Bill can be
provisions related to anti-defection in India. The 10th introduced only in Lok Sabha. After it is passed by that
Schedule of Indian Constitution is popularly known as the House, it is transmitted to Rajya Sabha for its concurrence
Anti-Defection Act. Originally, Indian Constitution had no or recommendation.
such provisions. The law sets the provisions for The power of Rajya Sabha in respect to financial matters or
disqualification of elected members on the grounds of such a Bill is limited. Rajya Sabha is a permanent body
defection to another political party. and is not subject to dissolution. However, one third of its
Speaker of Lok Sabha or Chairman of Rajya Sabha is the members retire every second year and are replaced by
authority to decide on defection cases. Speaker sits as a newly elected members. Each member is elected for a term
tribunal while deciding on defection cases. All of 6 years. The Vice-President can be removed from office
proceedings in relation to any question on disqualification by a resolution of the Council of States (Rajya Sabha),
of a member of a House under this Schedule are deemed passed by a majority of its members at that time and
to be proceedings in Parliament or in the Legislature of a agreed to by the House of the People (Lok Sabha).
state.
NTSE SOLVED PAPER 2019 (Stage II) 19
The President of India can be removed by the process of l (a) In general, Dumping is a situation of international
impeachment. The impeachment process can be initiated price discrimination, where the price of a product when
by either house of parliament. sold in the importing country is less than the price of that
product in the market of the exporting country.
106. In the context of Indian elections, the parties which
fail to gain majority in the parliament play the role 109. Shruti and Gautami were discussing about India’s
of opposition. Consider the following statements GDP and Kerala’s SDP. Some of the observation
and choose which statement/s is/are not true. made were :
I. Opposition parties in India play an important role I. Kerala’s per capita SDP is India’s GDP divided by
in building public opinion. Kerala’s population in a particular year.
II. Opposition parties are not constitutionally II. Since Kerala has best literacy rate and excellent
quality of life indices, it must have the highest
recognised.
SDP.
III. Opposition immediately assumes power of
III. In a federal structure if we know all the SDPs we
government, if the majority party loses its vote can have a fair idea of how big India’s GDP will
of confidence in the Parliament. be in that year.
IV. Opposition parties keep a close check on the IV. Kerala’s per capita SDP in a particular year is the
activities of the government. value of all final goods and services produced by
(a) I and II (b) II and III (c) Only III (d) Only IV the Kerala state in that year divided by Kerala’s
l (b) In order to get the designation as the Opposition party, population in that year.
the party must have secured at least 10% of the total Which of the above statements are correct ?
number of seats in the parliament as per the Leaders of (a) I and III (b) II and III
Opposition in Parliament Act, 1977. Opposition party in
(c) III and IV (d) I, III and IV
India play an important role in building public opinions.
The main role of the opposition party is to question the l (c) State Domestic Product (SDP) is defined as a measure
government on all matters and bills so that the in monetary terms of the volume of all goods and services
government remains accountable to the public. produced within the boundaries of the State during a
Mainly, the opposition party must ensure that the given period of time, accounted without duplication.
government does not resort to any drastic step that would SDP is estimated annually by the Department of
have harmful effects on the people of the nation. In the Economics and Statistics (DES)based on the guidelines
democratic system of India, the principal role of the and methodology prescribed by the Central Statistical
opposition is to keep a close check on the activities of the Office (CSO). While GDP gives a good estimate of the
government and also give their inputs in legislation. entire nation’s output, SDP provides more economic
details about subnational territories. So, a fair idea of a
107. Consider the following statements : country’s GDP can be estimated on the basis of all the
SDPs.
Statement I All countries that are democratic have
written Constitution. So, both Statements III and IV are correct.
Statement II All countries that have written 110. Shehnaaz joined a coaching institute for a
Constitution are not necessarily democratic. professional course. At the time of joining the
Which of the above statement/s is/are correct ? course, she paid a lump sum fee for the entire
(a) Only I (b) Only II (c) Both I and II (d) Neither I nor II course of two years. However, she did not find the
quality of teaching satisfactory and decided to quit
l (b) The Constitution is a written document that includes
after one year. When she asked for a refund of the
the set of political principles by which a country is
governed, especially in relation to the rights of the people fee for one year, she was refused. Which of the
it governs. It describes procedure of election, the following right/s of Shehnaaz was/were violated?
fundamental rights, the right of Legislature, the power of I. Right to choose
Executive, the power of Supreme Court and another court II. Right to represent
in the country. III. Right to be informed
Most democracies that follow the Westminster system, like IV. Right to seek redressal
New Zealand, likewise don’t have written Constitutions.
(a) Only I (b) I and IV
So, we can divide nations into these
categories-Democratic countries, which have a (c) III and IV (d) Only IV
Constitution (which may be written or not) and l (d) Right to seek redressal was violated in case of
Non-democratic countries which nevertheless have a Shehnaaz. The right to seek redressal against unfair trade
Constitution. So, only Statement II is correct. practices or restrictive trade practices or unscrupulous
exploitation of consumers’ is referred to as the right to
108. China exports a toy to India at ` 150, whereas the redressal according to the Consumer Protection Act, 1986.
same toy is manufactured and available in India for The Consumer courts like District Consumer Disputes
` 250. When China continues to export this toy to Redressal Forums at district level, State Consumer
India, this trade practice is known as ………… . Disputes Redressal Commissions and National Consumer
(a) Dumping (b) Export Promotion Disputes Redressal Commissions have been incorporated
(c) Import Substitution (d) Export Subsidisation with the help of the consumer protection act.
20 NTSE SOLVED PAPER 2019 (Stage II)

111. Which of the following reflects situation where a 113. Which of the following statements are true about
person is employed but do not contribute in adding food security ?
to the total product? I. Landless people always have food insecurity.
I. Open unemployment II. Those who do not have enough nutritious food
II. Disguised unemployment are food insecure.
III. Seasonal unemployment III. Those who have enough food but not the
IV. Frictional unemployment requisite nutrition are food secure.
(a) I and II (b) Only II IV. Those who do not have enough purchasing
(c) III and IV (d) Only IV power to buy sufficient food are food insecure.
l (b) Disguised unemployment refers to the work area (a) I and III (b) I and IV (c) II and III (d) II and IV
where surplus manpower is employed out of which some l (d) Food security is defined as the availability of food and
individuals have zero or almost zero marginal productivity one’s access to it. A household is considered food secure
such that if they are removed the total level of output when its occupants do not live in hunger or fear of
remains unchanged. The persons who are employed and starvation. Stages of food insecurity range from food
remains unproductive throughout the work is said to be secure situations to full-scale famine.
disguisedly unemployed. The World Food Summit of 1996 defined food security as
For example, a land can be harvested by ten persons existing when all people at all times have access to
effectively, but actually, 14 workers are engaged in similar sufficient, safe, nutritious food to maintain a healthy and
kind of a job, then the marginal productivity of additional active life.
four persons will be zero as they will contribute nothing to So, both Statement II and Statement IV are true about
the level of output. And, thus, the removal of these extra food security.
workers will lead to no change in the total productivity
even without any change in the method of production. 114. Siddhik issues a cheque of ` 19,000 in favour of
Hanush’s bank. What happens when the cheque is
112. There are 100 households in the village of
received and processed in Hanush’s bank?
Awangkhul, of which the loan taken by 20
household are from the State Bank of India, another I. There is no change in their bank accounts.
20 households from their friends and relatives, 5 II. Both their bank balances increase by ` 19,000.
households from Indian Bank, 10 households from a III. Siddhik’s bank balance decreases by ` 19,000
Regional Rural Bank, 15 households from and Hanush’s bank balance increases by the
businessmen, 10 households from village headmen same amount.
and 20 households from cooperative societies. IV. There is no change in Siddhik’s bank balance
Which of the following inference(s) is/are correct? although Hanush’s bank balance sees an
I. Formal sources of credit are lower than the others increase.
II. Institutional sources of credit are higher than Based on the above statements which option is
others correct?
III. Non-institutional sources of credit are higher (a) Only I (b) I and III (c) Only III (d) III and IV
than others. l (c) When the cheque is received and processed in
IV. Informal sources of credit are slightly higher than Hanush’s bank, then Siddhik’s bank balance decreases by
others. 19000 and Hanush’s bank balance report an increase by
(a) Only I same amount.
(b) I and II 115. The daily wage of a person in rural area is ` 180.
(c) Only II Arrange the following households in descending
(d) III and IV order of vulnerability to poverty.
l (c) In case of households in village of Awangkhul,
Institutional source of credit (for 55 households) are higher Name of the Person days Size of the Working
than others (for 45 households). Household of employment Household members of
the family
Institutional or Formal sources of credit includes
Ruldoo 14 4 2
co-operative societies, commercial banks, and regional
rural banks and State Governments also. Mulkha 15 7 2
Informal or Non-Institutional sources of credit includes Fakira 10 5 4
money lenders, traders, relatives, friends, village headmen Preeto 12 6 3
etc. (a) Preeto > Mulkha > Fakira > Ruldoo
As compare to formal sources, in the informal sources of (b) Mulkha > Preeto > Ruldoo > Fakira
credit there is no supervisory body and their main motive (c) Mulkha > Ruldoo > Preeto > Fakira
is profit making and they charge much higher rates of (d) Ruldoo > Fakira > Mulkha > Preeto
interest.
NTSE SOLVED PAPER 2019 (Stage II) 21
l (b) More is the income of a family less is the vulnerability l (c) Moderation influences of land and sea breezes can be
to poverty of that family. Minimum income is for the calculated by annually mean temperature.
family of Mulkha hence this family is highest vulnerable to In case of Station A
poverty.
14 ´ 2 ´ 180 14.4 + 16.7 + 29.3 + 30.0 + 33.3 + 33.3 + 30.0 + 29.4
Ruldoo Household wage = = 7 ´ 180
4 + 28.9 + 25.6 + 19.4 + 15.6
=
15 ´ 2 ´ 180 12
Mulkha Household wage = = 4.2 ´ 180
7 305.9
= = 25.4°C
10 ´ 4 ´ 180 12
Fakira Household wage = = 8 ´ 180
5 In case of Station B
12 ´ 3 ´ 180
Preeto = = 6 ´ 180 16.8 + 19.2 + 26.6 + 29.8 + 33.3 + 33.9 + 31.3 + 29.0
6
+ 20.1 + 27.0 + 20.1 + 14.9
Hence, order of vulnerability to poverty is =
12
Mulkha > Preeto > Ruldoo > Fakira. 302
= = 25.1°C
116. The following graph shows the distribution of mean 12
monthly temperature and average rainfall of a In case of Station C
particular city during the year.
24.5 + 25.7 + 27.7 + 30.4 + 33.0 + 32.5 + 31.0 + 30.2
40 40
Rainfall (Centimeters )

+ 29.8 + 28.0 + 25.9 + 24.7


35 35 =
30 30 12
Temperature (°C)

25 25
343.4
20 20 = = 28.6°C (maximum)
15 15 12
10 10 In case of Station D
5 5
0
Jan. Feb. Mar. Apr. May. Jun. Jul. Aug. Sep. Oct. Nov. Dec.
0
21.5 + 23.9 + 28.3 + 32.7 + 35.5 + 32.0 + 27.7 + 27.3
Months + 27.9 + 26.7 + 23.1 + 20.7
=
Which one of the following cities shows the climatic 12
conditions presented in the above graph? 327.3
= = 27.2°C
(a) Nagpur (b) Chennai (c) Jodhpur (d) Bengaluru 12
l (a) The climate of Nagpur city witnesses a very hot From the above calculation it is proved that Station C
weather during the months of summer as shown in above experiences maximum moderation influence of the land
graph. It reaches the pinnacle in the month May and and sea breezes.
reaches its lowest in the month of January and December. 118. Observe the data given in the following table.
Rainfall (Monsoons) take its charge in the month of June.
It showers maximum in the month of July and August as City Female Literacy Male Literacy Sex-Ratio
shown in graph. Rate (%) Rate (%)
So, the presented graph shows the climatic conditions of A 66.77 85.38 960
the Nagpur city.
B 71.16 82.67 980
117. The average mean monthly temperatures of four
C 73.78 77.17 989
stations are given in the following table. The
temperature is influenced by the movements of land D 59.26 79.24 972
and sea breezes.
Based on the above table, identify the city which has
MONTHS the extent of equality between male and female better
Temperature in Degree Celsius
than the rest in terms of the given parameters?
Stations JAN FEB MAR APR MAY JUN JUL AUG SEP OCT NOV DEC
(a) A (b) B
A 14.4 16.7 29.3 30.0 33.3 33.3 30.0 29.4 28.9 25.6 19.4 15.6 (c) C (d) D
B 16.8 19.2 26.6 29.8 33.3 33.9 31.3 29.0 20.1 27.0 20.1 14.9
l (c) City ‘C’ has the extent of equality between male and
C 24.5 25.7 27.7 30.4 33.0 32.5 31.0 30.2 29.8 28.0 25.9 24.7
female better than the rest in terms of given parameters. It
D 21.5 23.9 28.3 32.7 35.5 32.0 27.7 27.3 27.9 26.7 23.1 20.7 can be concluded from the given fact that City ‘C’ has the
highest sex ratio (989) and also the difference between the
Which one of these stations experiences maximum male and female literacy rate is minimal as compared to
moderation influence of the land and sea breezes? other given cities.
(a) A (b) B (c) C (d) D
22 NTSE SOLVED PAPER 2019 (Stage II)

119. Ruhani observes sequential change in relation to Shaded Area C represents cotton textile industries.
altitudinal zones and natural vegetation types Coimbatore, Erode, Salem, Tirupur and Dindigul are
dominated by oak-chestnut; pine-deodar and silver prominant cotton textile areas in Tamil Nadu.
fur-birch. Shaded Area D represents evergreen forest cover. This
area is near to equator and the climate is very favourable
Identify the proper sequence of vegetation type, she to evergeen forests.
has observed from the following.
(a) Alpine to Temperate to Subtropical 121. Which of the following geological sequence
(b) Subtropical to Temperate to Alpine properly matches the tectonic events from old to
(c) Subtropical to Alpine to Temperate recent time periods?
(d) Temperate to Alpine to Subtropical (a) Formation of Aravalli - Deccan volcanism - Formation of
l (b) The proper sequence of vegetation type as observed by Shiwalik - Upliftment of Himadri.
Ruhani is Subtropical to Temperate to Alpine. (b) Deccan volcanism - Formation of Aravalli - Upliftment of
Himadri - Formation of Shiwalik.
Subtropical forest occurs through the central and western
(c) Deccan volcanism - Formation of Shiwalik - Upliftment of
Himalaya. In subtropical forests, oak-chestnut are
common species. Himadri - Formation of Aravalli.
(d) Formation of Aravalli - Deccan volcanism - Upliftment of
Temperate forest is found in the inner dry ranges of the
western Himalayas. Pine-Deodar are common species in Himadri - Formation of Shiwalik
these forests. l (d) Geological sequence that properly matches the
Alphine forest occurs above 2700 m of altitude in the tectonic events from old to recent time periods is as follows
eastern Himalayas and 3000 m in the western Himalayas. Formation of Aravalli - Deccan volcanism-Upliftment of
It is a dense forest of silver fur-birch and junipur species. Himadri - Formation of Siwaliks.

120. Observe the map given below. 122. The given map shows location of different mountain
peaks in India.

B A

A
C
B D
D
C

Identify the shaded regions with their corresponding


A mountaineer wants to scale the mountain peaks in
geographical features and select the correct option
Peninsular India starting from North to South.
using the codes given below.
(a) A = Zone of laterite soil, B = Coffee producing area, Identify the correct sequence of peaks the
C = Cotton textile industries, D = Evergreen forest cover mountaineer will follows?
(b) A = Evergreen forest cover, B = Coffee producing areas, (a) A = Mahendragiri; B = Anaimudi ; C = Dodabetta;
C = Zone of laterite soil, D = Cotton textile industries. D = Mahabaleshwar
(c) A = Evergreen forest cover, B = Zone of laterite soil, (b) A = Dodabetta; B = Mahabaleshwar ; C = Mahendragiri;
C = Coffee producing area, D = Cotton textile industries D = Anaimudi
(d) A = Cotton textile industries, B = Coffee producing area, (c) A = Anaimudi ; B = Mahendragiri ; C = Dodabetta ;
C = Zone of laterite soil, D = Evergreen forest cover. D = Mahabaleshwar
(d) A = Mahendragiri ; B = Mahabaleshwar ; C = Dodabetta ;
l (a) Shaded Area A represents laterite soil. Laterite soil
D = Anaimudi
develops in higher areas of the penisular plateau. It is
found mainly in Kerala and Tamil Nadu. l (d) The mountain peaks in Peninsular India starting from
Shaded Area B represents coffee producing area. North to South are as follows
Karnataka is the largest coffee producing state in India. A = Mahendragiri; B = Mahabaleshwar; C = Dodabetta;
D = Anaimudi.
NTSE SOLVED PAPER 2019 (Stage II) 23
123. While teaching a topic on agriculture, geography (a) Sikkim - West Bengal - Assam - Meghalaya
teacher had made the following statement about a (b) Sikkim - Arunachal Pradesh - Assam - Manipur
particular crop in her class. (c) Arunachal Pradesh - Assam - Meghalaya - Manipur
(d) Arunachal Pradesh - West Bengal - Manipur - Meghalaya
‘‘Mean Monthly Temperature of about 27°C., high
relative humidity, rainfall of 150 cm in summer months l (c) Arunachal Pradesh - Assam - Meghalaya -Manipur is
and Khaddar soils are the ideal physical requirements the proper sequence of the States the tourist travelled.
during the period of its vegetative growth.’’ Tawang Monastery of Arunachal Pradesh is the largest
monastery of India and second largest of Asia. Rice is
Which one of the following crops was stated by the main crop of Assam and three crops of rice are grown in
teacher? the same agricultural year.
(a) Tea (b) Jute (c) Rubber (d) Sugarcane Various cement factories are found in Meghalaya. Loktak
l (b) Jute plant is called fertility-exhausting plant. lake of Manipur is famous as floating lake due to the
Therefore, jute needs new alluvial soils. Jute thrives well presence of floating circular swamps (phumdis in local
in a hot damp climate. Monthly average temperature to language).
the extent of 27°C is ideal. Average humidity of 80% to
127. Observe the following diagrams carefully.
90% is necessary. An annual average rainfall exceeding
150 cm is ideal for Jute cultivation.
124. Bibhuti was travelling to study the traditional
agricultural practices among various communities
in Meghalaya, Jharkhand, Odisha and Western
Ghats. Identify the correct sequence of forms of I II III IV
cultivation practised in these regions. Which one of the above population pyramids is an
(a) Jhumming - Kumari - Pama Dabi - Kuruwa ideal representation of India’s population?
(b) Kuruwa - Pama Dabi - Jhumming - Kumari
(a) I (b) II (c) III (d) IV
(c) Jhumming - Kuruwa - Pama Dabi - Kumari
(d) Pama Dabi - Kumari - Jhumming - Kuruwa l (c) Pyramid III is an ideal representation of India’s
population because India has about 34.4% children of its
l (c) The cultivation form that is practiced in the given population which represents base of pyramid, 58.7% adults
regions is Primitive Subsistence Farming. In India, this which represents middle part of pyramid and 6.9% aged
primitive form of cultivation is called Jhumming population which represents top of the pyramid.
(Meghalaya), Kuruwa (Jharkhand), Pama Dabi (Odisha),
Kumari (Western Ghat) and Bewar/Dahiya (Madhya 128. Which of the following statements regarding
Pradesh). printing in Medieval Europe are correct?
This type of farming is practiced on small patches of land.
Primitive tools and family/community labour are used in
I. Wood block printing reached Europe in the 13th
this type of farming. This is also called ‘slash and burn’ century.
agriculture. A patch of land is cleared by slashing the II. The aristocrats and monks criticised printed
vegetation and then the slashed plants are burnt. The ash; books as cheap vulgarities in the beginning.
thus obtained is mixed with the soil and crops are grown.
III. Printing did not entirely displace the art of
125. River Indus flows through Leh and Kargil districts producing books by hand.
in the state of Jammu and Kashmir. It has four IV. Martin Luther had reservations against printing
major tributaries in India. Which one of the of books.
following is the correct sequence of the tributaries (a) I, II and III (b) I, III and IV (c) I, II and IV (d) II, III and IV
arranged from East to West in terms of their
l (a) The correct statements regarding printing in medival
confluence with river Indus? Europe are as follows
(a) Zaskar - Dras - Hunza - Shyok
l Wood block printing reached Europe in the 13th century.
(b) Zaskar - Hunza - Dras - Shyok
l The aristocrats and monks criticised printed books as
(c) Hunza - Dras - Zaskar - Shyok
cheap vulgarities in the beginning.
(d) Zaskar - Dras - Shyok - Hunza
l Printing did not entirely displace the art of producing
l (d) River Indus flows through Leh and Kargil districts in books by hand.
the state of Jammu and Kashmir. It has four major
tributaries in India. Following is the correct sequence of 129. Which of the following statements related to
the tributaries arranged from East to West in terms of their Mahatma Gandhi’s view on Satyagraha are correct?
confluence with river Indus : Zaskar - Dras -
Shyok-Hunza.
I. The movement in South Africa was not passive
resistance.
126. A tourist was travelling Indian States and came II. It is the weapon of the people who are not weak.
across a famous Buddhist Monastery, farming of
three rice crops within the same agricultural year, a III. India could not militarily face Britain.
cement factory and floating gardens on a lake. IV. Truth is the supreme dharma.
Identify the proper sequence of the States the tourist (a) I, II and III (b) I, II and IV (c) II, III and IV (d) I, III and IV
travelled.
24 NTSE SOLVED PAPER 2019 (Stage II)

l (a) The correct statements related to Mahatma Gandhi’s 133. Why were Nghe An and Ha Tinh provinces called
view on Satyagraha are as follows ‘electrical fuses’ of Vietnam?
l The movement in South Africa was not passive resistance. (a) They were near to the capital city and were centers of power.
l It is the weapon of the people, who are not weak. (b) They were among the poorest provinces and had an old
l India could not militarily face Britain. radical tradition.
(c) They were very rich and had strong trade links with the
130. Which of the following statements relating to the outer world.
‘Scorched Earth Policy’ in Java are correct? (d) They were at the borders and were in conflicts with the
I. The Dutch destroyed the saw mills. neighbouring countries.
II. Teak logs were burnt by the Dutch. l (b) Nghe An and Ha Tinh provinces called ‘electrical
III. Trees were cut freely to meet war needs. fuses’ of Vietnam because they were among the poorest
provinces and had an old radical tradition whenever the
IV. The villagers were encouraged to expand colonial system came under pressure, these regions were
cultivation in the forest areas. the first to rise up in rebellion.
(a) I and II (b) I, II and III (c) I and IV (d) II, III and IV
134. Which of the following would be the part of the
l (b) The correct statements relating to the ‘Scorched Earth
Policy’ in Java are as follows surroundings in a Chawl in Bombay during the
l The Dutch destroyed the saw mills.
colonial period?
l Teak logs were burnt by the Dutch.
I. Large number of people living in shared rooms.
l Trees were cut freely to meet war needs.
II. A large population of people belonging to
depressed and lower classes.
131. Which of the following statements about opium III. Streets and neighbourhood being used for a variety
cultivation in India during the British period are of activities such as cooking, washing and sleeping.
correct? IV. Liquor shops and Akharas in any open spot.
I. The peasants could sell off the produce freely. (a) I, II and III (b) I, III and IV (c) II and III (d) II, III and IV
II. Local traders offered higher prices for opium. l (b) Following are the part of the surroundings in a chawl
III. Opium production was increasing in territories in Bombay during the colonial period
that were not under the British. l Large number of people living in shared rooms.

IV. Peasants were getting money advances from the l Streets and neighbourhood being used for a variety of

village headman to produce opium. activities such as cooking, washing and sleeping.
(a) I, II and III (b) I, II and IV (c) I, III and IV (d) II, III and IV l Liquor shops and Akharas in any open spot.

l (d) The correct statements about opium cultivation in 135. Which of the following statements are true in the
India during the British period are as follows
context of Cricket in Victorian England?
l Local traders offered higher prices for opium.
I. The rules of Cricket were made to favour those
l Opium production was increasing in territories that were
who were described as ‘Players’.
not under the British.
l Peasants were getting money advance from the village
II. The wages of professionals was paid by
headman to produce opium. patronage or subscription or gate money.
III. Cricket was viewed as a way of teaching English
132. Observe the given picture taken from New Orleans, boys discipline, importance of hierarchy and
Illustrated London news, 1851. leadership qualities.
IV. The rich who played were called amateurs.
(a) I, II and III (b) I, II and IV (c) I, III and IV (d) II, III and IV
l (d) Following statements are true in the context of Cricket
in Victorian England
l The wages of professionals were paid by patronage or
subscription or gate money.
l Cricket was viewed as a way of teaching English boys
discipline, importance of hierarchy and leadership
qualities.
l The rich who played were called amateurs.

136. Which of the following statements are true for


What does the picture represent?
eighteenth century France?
(a) Mourning
(b) Slave auction I. There was much criticism of slavery.
(c) Market place II. The National Assembly feared opposition from
(d) Roadside gathering businessmen who were dependant on slave trade.
l (b) In the above picture, a condition of Slave auction is III. Plantation owners understood their freedom as
represented. including the right to enslave Africans.
NTSE SOLVED PAPER 2019 (Stage II) 25
IV. The Convention of 1791 legislated to free all l (d) The potato is the world’s fourth largest food crop,
slaves in the French overseas possessions. following rice, wheat and maize.The Europeans in Peru
(a) I and II (b) I, II and IV (c) II and III (d) II, III and IV were the first to cultivate potatoes around 8,000 BC to 5,000
BC. By the early 1840s almost half the Irish population-but
l (c) Following statements are true for eighteenth primarily the rural poor - had come to depend almost
century France exclusively on the potato for their diet.
l The National Assembly feared opposition from
Hence, both statements are true but Statement II does not
businessmen who were dependant on slave trade. explains Statement I.
l Plantation owners understood their freedom as including
the right to enslave Africans. 140. Statement I The President of India cannot claim the
kind of direct mandate that the Prime Minister of
137. Which of the following statements are true in the India can.
context of Liberals in Modern Europe?
Statement II A candidate contesting for the post of
I. They opposed the uncontrolled power of dynastic President has to gain a majority of votes to be elected
rulers. as the President of India.
II. They wanted to safeguard the rights of l (d) Under the Constitution of India, there shall always be a
individuals against governments. President of India (Article 52). He holds the highest
III. They argued for Independent judiciary. elective office in the country and is elected in accordance
IV. They believed in universal adult franchise for all with the provisions of the Constitution.
The President is elected by an Electoral College, which
men and women with property. consists of the elected members of both Houses of
(a) I, II and III (b) I, II and IV (c) I, III and IV (d) II, III and IV Parliament and the elected members of the Legislative
l (a) Following statements are true in the context of Liberals Assemblies of all the States and also of NCT of Delhi and
in Modern Europe the Union Territory of Puducherry (Article 54 of the
l They opposed the uncontrolled power of dynastic rulers.
Constitution of India).
l They wanted to safeguard the rights of individuals Hence, both statements are true but Statement II does not
against governments. explains Statement I.
l They argued for Independent judiciary.
141. If m = n2 - n, where n is an integer, then m2 - 2 m is
Directions (Q. Nos. 138- 140) Read the statements and divisible by
select the correct answer from the options given below. (a) 20 (b) 24
(a) Statement I is true, Statement II is false. (c) 30 (d) 16
(b) Statement I is false, Statement II is true. l (b) Given, m = n2 - n
(c) Both Statements are true and Statement II provides 2
Then, m - 2 m = m(m - 2)
explanation to Statement I. = (n2 - n) (n2 - n - 2) [Q putting the value of ‘m’]
(d) Both Statements are true but Statement II does not
= n(n - 1) (n2 - 2 n + n - 2) = n(n - 1) {n(n - 2) + 1(n - 2)}
provides explanation to Statement I.
= n(n - 1) (n - 2) (n + 1) = (n - 2) (n - 1) (n) (n + 1)
138. Statement I The Bretton Woods system came up For positive value of m2 - 2 m, the minimum value of n = 3,
during the Post-World War Period.
then (n - 2) (n - 1) (n) (n + 1) = (3 - 2) (3 - 1) (3) (3 + 1)
Statement II The industrial nations had massive = 1 ´ 2 ´ 3 ´ 4 = 24
growth of trade and incomes.
Hence, m2 - 2 m is divisible by 24 (by the given options).
l (a) The Bretton Woods Conference, officially known as the
United Nations Monetary and Financial Conference, was 142. The value of 97 ´ 98 ´ 99 ´ 100 + 1 is equal to
a gathering of delegates from 44 nations that met in July (a) 9901 (b) 9891 (c) 9801 (d) 9701
1944 in Bretton Woods, New Hampshire.
The two major accomplishments of the conference were l (d) \ The value of 97 ´ 98 ´ 99 ´ 100 + 1
the creation of the International Monetary Fund (IMF) = x ´ (x + 1) ´ (x + 2) ´ (x + 3) + 1 [let, 97 = x]
and the International Bank for Reconstruction and
= x ´ (x + 2) ´ (x + 1) ´ (x + 3) + 1
Development (lBRD). Until World War I, most countries
were on the gold standard. But they went off so they could = x ´ (x + 3) ´ (x + 2) ´ (x + 1) + 1
print the currency needed to pay for their war costs. = (x2 + 3 x) ´ (x2 + x + 2 x + 2) + 1
It caused hyperinflation, as the supply of money
overwhelmed the demand. The value of money fell so = (x2 + 3 x) ´ (x2 + 3 x + 2) + 1
dramatically that after the war, countries returned to the
safety of the gold standard. = y ´ (y + 2) + 1 [(x2 + 3 x) = y {let}]
Hence, Statement I is true but Statement II is false. = y2 + 2 y + 1 = (y + 1)2 = y + 1
139. Statement I Potatoes had been discovered by the = x2 + 3 x + 1 [putting the value of y]
Europeans in the Americas. = x2 + 1 + 2 x + x = (x + 1)2 + x
Statement II Poor people in Ireland were dependent = (97 + 1)2 + 97 = 9701
on potatoes to escape starvation in the 19th century.
26 NTSE SOLVED PAPER 2019 (Stage II)

1 145. In village Madhubani 8 women and 12 girls can


143. Let P( x ) be a polynomial of degree 3 and P( n) = for
n paint a large mural in 10 h. 6 women and 8 girls can
n = 1, 2, 3, 4. Then, the value of P(5) is paint it in 14 h. The number of hours taken by
1 2 3 7 women and 14 girls to paint the mural is
(a) 0 (b) (c) - (d)
5 5 5 (a) 10 (b) 15
1 (c) 20 (d) 35
l (a) Given, P(n) =
n
l (a) According to the question,
Now, put the values of ‘n’ in given polynomial. (8 women + 12 girls) ´ 10 = (6 women + 8 girls)´ 14
1 1 1 1
P(1) = = 1, P(2) = , P(3) = and P(4) = Þ 80 women + 120 girls = 84 women + 112 girls
1 2 3 4
Þ 4 women = 8 girls
We know that,
\ 1 woman = 2 girls
P(n) ´ n = K(n - 1) (n - 2) (n - 3) (n - 4) + 1 … (i)
Now, 8 women and 12 girls = (8 ´ 2 + 12 = 28) girls can
[Q K = constant] paint a large mural in = 10 h.
Now, put the value of n = 0 Þ 1 girl can paint a large mural in = 10 ´ 28 h.
0 = K(0 - 1) (0 - 2) (0 - 3) (0 - 4) + 1 [Q indirect proportion]
Þ 24 K + 1 = 0 Hence, 7 women and 14 girls = (7 ´ 2 + 14 = 28) girls can
1 paint a large mural in
\ K=- 10 ´ 28
24 = = 10 h
1 28
Again, P(n) ´ n = - (n - 1) (n - 2) (n - 3)(n - 4) + 1
24 3+ 5
[Q putting the value of K in Eq. (i)] 146. If x = and y = x 3 , then y satisfies the
2
1
P(5) ´ 5 = - (5 - 1) (5 - 2) (5 - 3) (5 - 4) + 1 quadratic equation
24 (a) y2 - 18 y + 1 = 0 (b) y2 + 18 y + 1 = 0
1
Þ P(5) ´ 5 = - ´ 4 ´ 3 ´ 2 ´1 + 1 2
(c) y - 18 y - 1 = 0 (d) y2 + 18 y - 1 = 0
24
3+ 5
= -1 + 1 = 0 l (a) Given, x =
2
\ P(5) = 0 Now, put the value of ‘x’ in y = x3
3
144. If a and b are the roots of the equation æ3 + 5 ö (3 + 5 )3
y=ç ÷ =
3 x 2 - 5 x + 3 = 0, then the quadratic equation whose è 2 ø (2)3

roots are a 2b and ab2 is (3)3 + ( 5 )3 + 3 ´ 3 ´ 5 (3 + 5 )


=
(a) 3 x2 - 5 x + 3 = 0 (b) 3 x2 - 8 x + 5 = 0 8
(c) 3 x2 - 8 x + 3 = 0 (d) 3 x2 - 5 x - 3 = 0 [Q(a + b)3 = a3 + b3 + 3 ab(a + b)]
27 + 5 5 + 9 5 (3 + 5 )
l (a) Given quadratic equation, =
8
3 x2 - 5 x + 3 = 0
27 + 5 5 + 27 5 + 45
Coefficient of x =
Then, a+ b = - 8
Coefficient of x2
(- 5) 5 72 + 32 5
=- = = =9 + 4 5
3 3 8
Constant term Hence, one root is 9 + 4 5 , then second root is 9 - 4 5
and a´b = because irrational roots are conjugate of each other.
Coefficient of x2
3 So, sum of roots = 9 + 4 5 + 9 - 4 5 = 18
= =1
3 and product of roots = (9 + 4 5 ) (9 - 4 5 )
\ Required quadratic equation of given roots, = (9)2 - (4 5 )2 [Q (a + b) (a - b) = a2 - b2 ]
Þ x2 - (sum of roots) x + product of roots = 0 = 81 - 80 = 1
Þ x2 - (a2 b + ab2 ) x + (a2 b ´ ab2 ) = 0 \ Required quadratic equation,
Þ x2 - ab (a + b)x + (ab)3 = 0 y2 - (sum of roots) y + (product of roots) = 0
Þ y2 - 18 y + 1 = 0
x2 - 1æç ö÷ x + (1)3 = 0
5
Þ
è3 ø
147. If tan2 q = 1 - e2 , then the value of
5
Þ x2 - x + 1 = 0 sec q + tan3 q cosec q is equal to
3
(a) (1 - e 2 )1/ 2 (b) (2 - e 2 )1/ 2
\ 3 x2 - 5 x + 3 = 0 2 3/ 2
(c) (2 - e ) (d) (1 - e 2 )3 / 2
NTSE SOLVED PAPER 2019 (Stage II) 27
l (c) Given, tan2 q = 1 - e2 149. A solid metallic cylinder of height 10 cm and
Now, sec q + tan3 q cosec q diameter 14 cm is melted to make two cones in the
1 1 proportion of their volumes as 3 : 4, keeping the
= + tan2 q × tan q ´
cos q sin q height 10 cm, what would be the percentage
1 sin q 1 increase in the flat surface area?
= + tan2 q ´ ´
cos q cos q sin q (a) 9 (b) 16 (c) 50 (d) 200
1 tan2 q l (c) Volume of cylinder = pr2 h
= +
cos q cos q 2
= p ´ æç ö÷ ´ 10 éQ radius = diameter ù
14
1 + tan2 q è2 ø êë úû
= 2
cos q
= 490p
= secq (1 + tan2 q)
According to the question,
= ( 1 + tan2 q) ´ (1 + tan2 q) [Qsecq = 1 + tan2 q]
3
Volume of first cone = ´ 490p
= (1 + tan2 q)3 / 2 = (1 + 1 - e2 )3 / 2 7
= (2 - e2 )3 / 2 1 2
pr1 h = 3 ´ 70 p
148. Let the volume of a solid sphere be 288 p cm3 . A 3
1 2
horizontal plane cuts the sphere at a distance of r1 (10) = 3 ´ 70
3 cm from the centre so that the ratio of the curved 3
surface areas of the two parts of the sphere is 3:1. r1 = 3 ´ 3 ´ 7 = 3 7 cm
The total surface area of the bigger part of the 4
Volume of second cone = ´ 490p
sphere (in cm2 ) is 7
(a) 36 p (b) 108 p(c) 135 p (d) 144 p 1 2
4 p r2 h = 4 ´ 70 p
l (c) Volume of solid sphere = p(r)3 3
3 1 2
4 r2 (10) = 4 ´ 70
Þ 288 p = p(r)3 3
3 r2 = 2 ´ 2 ´ 21 = 2 21 cm
3 288 ´ 3
Þ r = = 216
4 Now, flat surface area of cylinder = 2 pr2
\ r = 6 cm = 2 ´ p ´ (7)2 = 98 p
D and flat surface area of cones = p(r1 )2 + p(r2 )2

E = p(3 7 )2 + p(2 21)2 = 147p


A C \ Required increase percentage
3 cm 147 p - 98 p 49
= ´ 100 % = ´ 100% = 50%
O
6 cm 98 p 98

150. Each vertical face of square based vertical pillar of


height 3 m has 7 equal, semi-cylindrical surfaces in
such a way that its horizontal cross-section is as
shown in the figure.
In DEOC, by Pythagoras theorem,

EC = OC2 - OE2 = (6)2 - (3)3

= 36 - 9 = 27 = 3 3
\ Curved surface area of sphere = 4 pr2
= 4 p(6)2 = 144p
3
Then, curved surface area of bigger part = ´ 144p If the radius of each semi-circle is 10 cm, the volume
3 +1
æ 22 ö
= 108p (in m3 ) of the pillar so designed ç taking p = ÷ is
\ Total surface area of bigger part è 7 ø
= 108 p + p (3 3 )2 = 135 p cm2 (a) 5.88 (b) 6.14 (c) 6.42 (d) 7.2
28 NTSE SOLVED PAPER 2019 (Stage II)

l (d) Diameter of cylinder = r ´ 2 Perimeter of triangle = 7p


20 7p
= 10 ´ 2 = 20 cm = m Then, semi-perimeter of triangle(s) =
100 2
140 We know that,
So, length of cuboid (l) = 7 ´ 20 = 140 cm = m
100 Area of triangle D
Radius of inner circle = =
breadth of cuboid (b) = 7 ´ 20 = 140 cm Semi- perimeter s
and height of cuboid(h) = 3 m 1
´ AB ´ BC
\ Required total volume = 14 ´ volume of cylinder AB ´ BC
\ r=2 =
+ volume of cuboid 7p 7p
= 14 ´ pr2 h + l ´ b ´ h 2
22 10 10 140 140 Circumference of circle
= 14 ´ ´ ´ ´3 + ´ ´3 Hence, required ratio =
7 100 100 100 100 Area of triangle
éQ1 cm = 1 mù AB ´ BC
2´p´
êë 100 úû 2 pr 7p
= =
1 1
= 132
. + 5.88 = 7.2 m3 ´ AB ´ BC ´ AB ´ BC
2 2
151. Let ABCD be a square of side 20 cm. The area of the 4
= or 4 : 7
square PQRS (in cm2 ) interior to ABCD, shown in 7
figure is
A 10 L 10 B 153. It is known that area of a cyclic quadrilateral is
Q ( s - a)( s - b)( s - c)( s - d) where a, b, c, d are the
10 10 a+ b+ c + d
P sides and s = . If a circle can also be
M O 2
R inscribed in the cyclic quadrilateral, then the area of
10 10 this quadrilateral is
S
D 10 N 10 C (a) (ab )2 + (cd )2 (b) abcd
(a) 60 (b) 80 (c) 100 (d) 400 2
(c) (ac ) + (ac ) 2
(d) (ad )2 + (bc )2
l (b) In given figure,
l (b) Let ABCD is a cyclic quadrilateral inscribed a circle
Let the area of PQRS = A cm2 and a circle inscribed in quadrilateral ABCD.
and Area of DLBQ = DSDN = DAMP = DORC = a A
Then, Area of ALPQ = MPDS = SRNC = QBOR = 3a
1 S R
\ Area of DADN = ´ AD ´ DN
2
1 B D
Þ a + 3 a + a = ´ 20 ´ 10 Þ 5 a = 100
2
\ a = 20 P Q
\ Area of square = (side)2
C
Þ a + 3 a + a + a + 3 a + a + 3 a + 3 a + A = (20)2
Þ 16 a + A = 400 Let AB = a, BC = b,CD = c, DA = d
a+ b+ c + d
Þ A = 400 - 16 ´ 20 Given, s=
2
Þ A = 400 - 320
2s = a + c + b + d
\ A = 80 cm2
Þ s+ s= a+ c + b+ d
152. A circle is inscribed in a right angled triangle of Þ s= a+ c = b+ d
perimeter 7p. Then, the ratio of numerical values of [Q AS = AR, BS = BP, CP = CQ, DR = DQ]
circumference of the circle to the area of the right
angled triangle is Area of cyclic quadrilateral = (s - a) (s - b) (s - c) (s - d)
(given)
(a) 4 : 7 (b) 3 : 7 (c) 2 : 7 (d) 1 : 7
a+ b+ c + d
where, s=
l (a) Given, A 2
Put s= a+ c = b+ d
Then, area of quadrilateral = (s - a) (s - b) (s - c) (s - d)
r = (a + c - a) (b + d - b) (a + c - c) (b + d - d)
O
= c ´ d ´ a´ b= abcd
B C
NTSE SOLVED PAPER 2019 (Stage II) 29
154. Two circles, both of radii a touch each other and Þ a = 14 ´ 3 ´ 3 - 1 / 4
1
each of them touches internally a circle of radius 2a. 1-
Then, the radius of the circle which touches all the Þ a = 14 ´ 3 4 [Q am ´ an = am + n]
3
three circles is
\ a = 14 ´ 34

156. Let ABC be a triangle with sides a, b, c. Then,


lengths of medians of the triangle formed by the
a Pa
O O1 medians of the triangle ABC are
a a 1 1 1 2 2 2
(a) a, b, c (b) a, b, c
2 2 2 3 3 3
r 3 3 3 5 5 5
O2 (c) a, b, c (d) a, b, c
4 4 4 6 6 6
1 2 3
(a) a (b) a (c) a (d) a l (c) According to the question,
2 3 4 A
l (b) In the given above figure,
Q PO2 = 2 a - r
In DOPO2 , by Pythagoras theorem, c d1 b
OO2 = PO2 + (PO2 )2
d3
Þ (a + r)2 = (a)2 + (2 a - r)2 d2
Þ a + r + 2 ar = a2 + 4 a2 + r2 - 4 ar
2 2
B C
Þ 2 ar = 4 a2 - 4 ar a
Þ 2r = 4a - 4r Here, d1 , d2 and d3 are the medians of DABC.
Þ 6r = 4a By Apollonius theorem,
4a 2a é a ù
2
\ r=
6
=
3 2 ê d12 + æç ö÷ ú = b2 + c2
êë è2 ø ú
û
155. Let D be a point on the side BC of a triangle ABC 2 a2
2 d1 + = b2 + c2
such that ÐADC = ÐBAC. If AC = 21 cm, then the 2
side of an equilateral triangle whose area is equal to b2 + c2 a2
d12 = - … (i)
the area of the rectangle with sides BC and DC is 2 4
(a) 14 ´ 31/ 2 (b) 42 ´ 3-1/ 2 (c) 14 ´ 33 / 4 (d) 42 ´ 31/ 2 a2 + c2 b2
Similarly, d22 = - … (ii)
l (c) Given, 2 4
A
a2 + b2 c2
and d32 = - … (iii)
2 4
21 cm Again, let M1 , M2 and M3 are the lengths of medians of
triangle, whose sides d1 , d2 and d3 .
B C
D

Q ÐBAC = ÐADC d1 d2
M1
Now, DACB ~ DDCA
AC BC AB M3
Q = =
DC CA AD M2
\ AC ´ CA = BC ´ DC d3
Þ 21 ´ 21 = BC ´ DC
é 2ù
2 ê M12 + æç 3 ö÷ ú = d12 + d22
Þ BC ´ DC = 441 cm 2 d
êë è2 ø ú
According to the question, û
Area of equilateral triangle = Area of rectangle with sides d32 d2 + d22
BC and DC M12 + = 1
4 2
3 2 d12 + d22 d2
Þ (a) = BC ´ DC 2
M1 = - 3
4 2 4
441 ´ 4
Þ a2 = On putting the values of d1 , d2 and d3 , we get
3
21 ´ 21 ´ 2 ´ 2 b2 + c2 a2 a2 + c2 b2 a2 + b2 c2
Þ a= - + - -
(3)1 / 2 M12 = 2 4 2 4 - 2 4
2 4
Þ a = 42 ´ 3 - 1 / 4
30 NTSE SOLVED PAPER 2019 (Stage II)

a2 + b2 + 2 c2 a2 + b2 159. If two dice are thrown together the probability that


-
2 4 2 a2 + 2 b2 - c2 the difference of the numbers appearing on them is
M12 = -
2 4 ´4 a prime number
2 4
2 a2 + 2 b2 + 4 c2 - a2 - b2 2 a2 + 2 b2 - c2 (a) (b)
M12 = - 9 9
4 ´2 16 5 17
(c) (d)
4 a2 + 4 b2 + 8 c2 - 2 a2 - 2 b2 - 2 a2 - 2 b2 + c2 12 36
M12 =
16 l (b) According to the question,
2
9 c 3 c Minimum difference = 0
M12 = Þ M1 =
16 4 Maximum difference = 5
3 3
Similarly, M2 = b and M3 = a Q Prime numbers between 0 to 5 = 2, 3 and 5
4 4
\ Total (favourable outcomes) cases = [(3, 1), (1, 3), (4, 2),
157. ( x + 1)4 is divided by ( x - 1)3 . Then, the value of the (2, 4), (5, 3), (3, 5), (6, 4), (4, 6), (4, 1), (1, 4), (5, 2), (2, 5),
(6, 3), (3, 6), (1, 6), (6, 1)] = 16
remainder at x = 1 is
Total possible outcomes = 6 ´ 6 = 36
(a) - 16 (b) 0 (c) 16 (d) 32
\ Required probability
l (c) According to the question, Number of favourable outcomes
(x + 1)4 [(x - 1) + 2]4 =
\ = Total number of possible outcomes
(x - 1)3 (x - 1)3 16 4
= =
(x - 1)4 + 4 (x - 1)3 ´ 2 + 6(x - 1)2 ´ 4 + 4(x - 1) ´ 8 + 16 36 9
=
(x - 1)3
160. Observe the following data.
[Q(a + b)4 = a4 + 4 a3 b + 6 a2 b2 + 4 ab3 + b4 ]
Class 0 - 20 20 - 40 40 - 60 60 - 80 80 - 100 Total
\ Remainder = 24(x - 1)2 + 32(x - 1) + 16
Now, x = 1 Frequency 17 f1 32 f2 19 120
= 24(1 - 1)2 + 32(1 - 1) + 16
= 0 + 0 + 16 = 16 If the above data has mean 50, then missing
frequencies f1 and f2 are respectively.
158. A circle passes through the vertices of a triangle (a) 28 and 24
ABC. If the vertices are A( -2, 5), B( -2, - 3), C(2, - 3), (b) 24 and 28
then the centre of the circle is (c) 28 and 30
(a) (0, 0) (b) (0, 1) (c) (-2, 1) (d) (0, - 3) (d) 30 and 28
l (b) Given, l (a)
A (–2,5)
Class interval Frequency ( fi ) Class mark fi xi
( xi )

O 0-20 17 10 170

(x,y) 20-40 f1 30 30f1


B (–2,–3) C (2,–3) 40-60 32 50 1600
60-80 f2 70 70 f2
For given circle,
OA = OB = OC = radius of circle 80-100 19 90 1710
Q OA = OB Total Sfi = 120 Sfi xi = 3480
Þ (x + 2)2 + (y - 5)2 = (x + 2)2 + (y + 3)2 + 30f1 +70f2

Þ (x + 2)2 + (y - 5)2 = (x + 2)2 + (y + 3)2 Q 120 = 17 + f1 + 32 + f2 + 19


Þ y2 + 25 - 10 y = y2 + 9 + 6 y Þ 120 = 68 + f1 + f2
Þ 16 y = 16 \ f1 + f2 = 52 … (i)
\ y =1 Sf x
\ Mean (x) = i i
Again, OB = OC Sfi
3480 + 30 f1 + 70 f2
Þ (x + 2)2 + (y + 3)2 = (x - 2)2 + (y + 3)2 Þ 50 =
120
Þ (x + 2)2 + (y + 3)2 = (x - 2)2 + (y + 3)2
Þ 6000 = 3480 + 30 f1 + 70 f2
Þ x2 + 4 + 4 x = x2 + 4 - 4 x
Þ 30 f1 + 70 f2 = 2520
Þ 4x + 4x = 0 Þ 8x = 0
\ x =0 Þ 3 f1 + 7 f2 = 252 … (ii)
Hence, required centre of circle = (x, y) = (0, 1) From Eqs. (i) and (ii), f1 = 28 and f2 = 24
NTSE SOLVED PAPER 2019 (Stage II) 31
Directions (Q.Nos. 161 and 162) Suppose that the 163. An object of mass 2 kg moving under the action of a
acceleration versus time graph of a particle that starts from force which varies with time as shown in the figure.
rest at t = 0 is as shown in the figure.
F
a
(N)
(in m/s2)
10

5
0 t(s)
0 t (in s) 5 10 15 20
10 20 30

–10 –10

161. At what instant does the particle come to rest for the
first time?
(a) 5 s (b) 10 s (c) 15 s Which one of the following statements is correct for
(d) The particle never comes to rest the interval from 0 to 20 s?
(a) The momentum of the object decreases by 75 kg-m/s.
l (c) Diagram as shown below,
a (in m/s2)
(b) The momentum of the object increases by 75 kg-m/s
(c) The momentum of the object increases by 125 kg-m/s
(d) The change in momentum cannot be found as initial speed
5
A1 A3 is unknown.
t (in s) l (b) Diagram as shown below,
10 20 30
A2 F
–10
(N)
10
Area of the acceleration versus time graph shows the
velocity of the particle. A1 A2 A3
O t (s)
Let particle comes to rest for the first time at time t, 5 10 15 A4 20
Now, A1 + A2 = 0 –10
where A1 = Area of first half cycle
A2 = Area of second half cycle Q Area of force versus time graph shows the changes in
Þ 5 ´ 10 + (t - 10) (- 10) = 0 momentum.
Þ 50 - 10(t - 10) = 0 Hence, changes in momentum of an object is given by
Þ 50 - 10t + 100 = 0 Þ50 = 10t Þ t = 15 s DP = A1 + A2 + A3 + A4
1 1
162. What is the total distance travelled by the particle DP = ´ 5 ´ 10 + (10 - 5) ´ 10 + ´ 10 ´ (15 - 10)
2 2
during 30 s? 1
(a) 0 m (b) 500 m (c) 750 m (d) 1000 m + ´ (-10) ´ (20 - 15)
2
l (c) Slope of velocity versus time graph shows the 1 1 1
DP = ´ 5 ´ 10 + 5 ´ 10 + ´ 5 ´ 10 + ´ 5 ´ (- 10)
acceleration, i.e. 2 2 2
change in velocity
Acceleration of an object = = 25 + 50 + 25 - 25 = 75 kg- m / s
time interval
Hence, momentum of object increases by 75 kg-m/s.
Hence, velocity versus time graph will be,
164. Two cars A and B of same mass start from the same
V(m/s)
location at the same time but on different straight
50 roads. Car A travels on a road that has greater angle
A1 of inclination with horizontal compared to the road
20 on which B travels.
t (s)
10 15 A2 30 At any instant both cars A and B have the same height
–50
above the starting point. If EA and EB are total
energies of cars A and B respectively, then
Magnitude of area of velocity versus time graph gives the (a) EA < EB
distance. (b) EA = EB
So, total distance = A1 + | A2 | (c) EA > EB
1 1 (d) Relation between EA and EB cannot be decided based on
= ´ 15 ´ 50 + ´ 15 ´ (- 50) = 750 m given information.
2 2
32 NTSE SOLVED PAPER 2019 (Stage II)

l (a) According to the question, we can draw the following l (d) According to the question, diagram as shown below,
diagram
A

A B
P H
h
θA h h
θB B

In the above figure, qA > qB


Let both objects collide at point P on the height h above the
At any instant, height is same, so potential energy will be ground.
same but car B covers more distance than car A at same
From second equation of motion for objects A and B,
time. Therefore, speed of car B should be more than car A.
Hence, kinetic energy of car B will be more than car A. For object A,
1 1
i.e. EB > EA Þ H - h = uA t + gt 2 = gt 2 … (i) (QuA = 0)
2 2
where EA = Total energy of car A
For object B,
EB = Total energy of car B
1
h = ut + gt 2 … (ii)
165. The gravitational potential energy difference per 2
unit mass between the surface of a planet and a Substitute the value of h from Eq. (ii) in the Eq. (i), we get
point 100 m above it is 1000 J/kg. How much work
H - æçut - gt 2 ö÷ = gt 2
1 1
Þ
is required to be done in moving a 5 kg object 100 m è 2 ø 2
on a slope at 30° to the horizontal on this planet? H
(a) 1250 J (b) 2500 J (c) 4350 J (d) 5000 J Þ H = ut or t =
u
l (b) According to question, we can draw the following Again, from Eq. (i), we get
diagram 2
1 æHö
P Þ H-h= gç ÷
2 èuø
2
1 æHö
Þ h=H- gç ÷
C 100 m 2 èuø
m Q u = 2 gH (given)
100 h= 50 m 2
1 æ H ö
Hence, h = H - gçç ÷
30° 2 è 2 gH ÷ø
A B
H 3H
Q Gravitational potential energy from surface of planet to =H- or h =
4 4
at a point 100 m above is PE = mgh
or gravitational potential energy per unit mass, 167. After they collide, they stick to each other. What is
= gh = 10 ´ 100 = 1000 J / kg (given) the loss in their total energy?
(b) æç ö÷ mgH
In the above diagram, in the DABC, 1
(a) 0
h 1 h è2 ø
Þ sin30 º = Þ = Þ h = 50 m
(c) æç ö÷ mgH
100 2 100 3
(d) 2 mgH
è2 ø
Hence, net work done in moving 100 m on slope at 30º is
W ¢ = mgh¢ = 5 ´ 10 ´ 50 = 2500 J l (b) In the diagram of solution of Q. 166
uA = the initial velocity of object A,
Directions (Q. Nos. 166 and 167) Two identical objects A v A = the velocity at point P of object A,
and B each of mass m start moving along the same vertical
u = the initial velocity of object B
line in opposite directions at the same instant. Object A is
and uB = the velocity at point P of object B.
dropped from rest from a height H above the ground and
object B is projected vertically upward from the ground From third equation of motion,
with speed u = 2 gH . For object A,
v2A = u2A + 2 g(H - h)
166. At what height above the ground do they collide?
= 2 gæç H -
3 H ö gH
(a) æç ö÷ H
1
(b) æç ö÷ H
1 ÷=
è 4 ø 2
è 4ø è2 ø
gH
(c) æç ö÷ H (d) æç ö÷ H
2 3 or vA =
è 3ø è 4ø 2
NTSE SOLVED PAPER 2019 (Stage II) 33
For object B, Fig. 1 gives the wavelength of wave, i.e.
3
l = 100 cm
v2B = u2 - 2 gh = ( 2 gH )2 - 2 gæç
3H ö
÷
è 4 ø Fig. 2 gives the time period of wave, i.e.
3 gH T = 2 ms
v2B = 2 gH - gH =
2 2 Hence, speed of the wave in the given medium,
gH l 100
Þ vB = v= = = 50 ´ 103 cm / s
2 T 2 ´ 10 - 3
Objects A and B collide at point P, they stick each-other, or v = 500 m / s
therefore collision will be perfected inelastic collision.
Hence, loss in kinetic energy after collision, 169. A convex lens and a concave lens, each of focal
length 10 cm, are kept separated by a distance of
1 æ mm ö
DKE = ç 1 2 ÷ u2 2 cm as shown in the figure. If the light is incident
2 è m1 + m2 ø from left, the combinations of lenses will be
Q m1 = m2 = m, u = 2 gH ………… .
1 æ m× m ö 2
Hence, D KE = ç ÷ ( 2 gH )
2 è m + mø

´ 2 gH = æç ö÷ mgH
1 m 1
= ´
2 2 è2 ø

168. Given below are two different graphs of variation of (a) converging
density (or pressure) of the medium with position (b) diverging
(fig. 1) and with time (fig. 2) as a wave passes (c) behaving like a glass slab
through the medium. (d) converging or diverging depending on whether the lenses
Density are arranged as shown in the figure or in the reverse order.
or
Pressure l (a) Given, focal length of convex lens, f1 = 10 cm,
Focal length of concave lens,
o
50 100 150 200 x(cm) f2 = - 10 cm
Distance between convex lens and concave lens, d = 2 cm.
Fig. (1) Equivalent focal length of combination,
1 1 1 d
Density = + -
or feq f1 f2 f1 f2
Pressure
1 1 2
= + -
o 10 (- 10) 10 ´ (- 10)
1.0 2.0 3.0 4.0 t (ms)
1 1 2
= - + or feq = 50 cm
10 10 100
Fig. (2) Q Equivalent focal length of combination is positive, hence
What will be the speed of the wave in the given this combination behaves as converging lens.
medium? 170. In the circuit given, the ratio of work done by the
(a) 25 m/s (b) 50 m/s battery to maintain the current between point A
(c) 250 m/s (d) 500 m/s and B to the work done for the whole circuit is
l (d) 1Ω
Density
or 2Ω 1Ω 2Ω
pressure A B
0 1Ω
50 100 150 200 x (cm)

I
Fig. (1)
Density
or
pressure
1.3V
o 1 1
1.0 2.0 3.0 4.0 t (ms) (a) (b)
117 13
1
(c) (d) 1
Fig. (2) 12
34 NTSE SOLVED PAPER 2019 (Stage II)

l (b) The circuit diagram as shown below, (a) At both O1 and O2 are zero
1Ω (b) At both O1 and O2 are non-zero
(c) is zero at O1 but non-zero at O2
RA=2Ω 1Ω RB=2Ω (d) is non-zero at O1 but zero at O2
A B
1Ω l (a) i1

O1
R
1.3V i 90° i
The equivalent resistance of the circuit, C D
Req = 2 + RAB + 2 = 4 + RAB
i2
1 1
Q Resistance, RAB = Þ RAB = W
æ1 + 1 + 1 ö 3 iæ q ö
ç ÷ Q Bµ ç ÷ (given)
è1 1 1 ø R è 360º ø
1 13 i 270 i 3
\ Req = 4 + = W From above diagram, B1 = K 1 =K 1 ´
3 3 R 360 R 4
V .
13 i 90 i 1
So, I= = = 0.3 A B2 = K 2 =K 2 ´
Req æ13 ö R 360 R 4
ç ÷
è3 ø So, net magnetic field at O1
Now, potential difference between A and B, K æ i2 3 ö
B = B2 - B1 = ç - i1 ÷
1
VAB = IRAB = 0.3 ´ = 01
. V R è4 4 ø
3 K
or B= [i2 - 3 i1 ] …(i)
W V I 0.1 ´ 0.3 0.03 1 4R
So, ratio of work done, AB = AB = = =
W VI 1.3 ´ 0.3 0.39 13 For parallel combination, V1 = V2
Þ i1 R1 = i2 R2
171. Magnetic field at the centre of a circular coil of
i Þ æ
i2 ç
R
´ 90 ö÷ = i2 æç
R
´ 270 ö÷
radius R carrying current i is B µ and its direction è 360 ø è 360 ø
R
is given by right-hand thumb rule. Magnetic field at i2 3
Þ = i1
the centre of a circular arc subtending an angle q 4 4
i æ q ö Þ i2 = 3 i1
(in degree) is B µ ç ÷ and its direction can be
R è 360° ø From Eq. (i), we get
K
found using right hand rule. So, B= [3 i1 - 3 i1 ] = 0
i 4R
i Similarly, i3

B∝ i θ
R R R B∝ i θ
O2
R 360°
R
i 120° i
P Q
Consider two circular coils made of uniform
conductors as shown in figure 3 and 4. In figure i4
3, points C and D are diametrically opposite to each
other, and in figure 4, Ð PO2Q = 120°. Then, magnetic Net magnetic field (B) at O2 ,
fields ………… . K é120 240 ù
B= i4 - i3
i1 i3 R êë 360 360 úû
B = éê 4 - i3 ùú =
K i 2 K
[i4 - 2 i3 ] …(ii)
R ë 3 3 û 3R
O1 O2
In parallel combination,
R R
i3 éê ´ 240 ùú = i4 æç ´ 120 ö÷
i 90° i 120° R R
i i Þ
C D P Q ë 360 û è 360 ø
i2 i4 2 i3 i4
Þ =
Fig 3 Fig 4 3 3
Þ i4 = 2 i3
NTSE SOLVED PAPER 2019 (Stage II) 35
From Eq. (ii), we get, Given circuit can be simplified as
K 1 kΩ
So, B= [2 i3 - 2 i3 ] = 0
3R A 1 kΩ 1 kΩ
So, net magnetic field at both O1 and O2 are zero. 1 kΩ C
B
172. A pin AB of length 2 cm is kept on the axis of a 1 kΩ
convex lens between 18 cm and 20 cm as shown in 1 kΩ 1 kΩ
figure. Focal length of convex lens is 10 cm. Find
magnification produced for the image of the pin. I

8V

A B Between the points A and C, all resistance are connected


in parallel combination. Hence, the equivalent resistance
20 cm

between point A and C is given by


1 1 1 1 1 1 1
= + + + + or RAC = kW
RAC 1 1 1 1 1 5
(a) 0.83 (b) 1.00 (c) 1.25 (d) 6.78
Similarly, equivalent resistance between point C and B,
l (c) From the given diagram in question, image of A is 1 1 1 1
formed at v1 = 20 cm and for point B, image is formed at = + +
u = - 18 cm RCB 1 1 1
\ Focal length of convex lens, f = 10 cm 1
or RCB = kW
Focal length of convex lens, 3
1 1 1 1 1 1 Q Resistance RAC and RCB are in series.
= - Þ = -
f v u 10 v (- 18) Hence, equivalent resistance of circuit,
1 18 - 10 1 1 8
Þ
1
=
1
- = =
8 RAB = RAC + RCB = + = kW
v 10 18 180 180 5 3 15
180 Hence, current supplied by battery in circuit,
or v= = 22.5 cm 8 8
8 I= = = 15 mA.
Image length = v - v1 = 22.5 - 20 = 2.5 cm RAB (8 / 15)
So, magnification produced for the image of the pin, 174. Read the following statements.
image length 2.5
m= = = 125
. Statement I Sodium metal reacts violently with water
object length 2
to produce heat and fire.
173. What is the current supplied by the battery in the Statement II Potassium metal reacts violently with
circuit shown below? Each resistance used in circuit water to form potassium hydroxide and hydrogen gas.
is of 1 kW and potential difference VAB = 8 V. Select the correct answer from the options given
below.
A (a) Statement I is true and Statement II is false
(b) Statement I is false and Statement II is true
(c) Both Statements are true and Statement II provides correct
explanation to Statement I
(d) Both Statements are true and Statement II does not
B provide correct explanation to Statement I
l (d) Sodium and potassium react violently with cold water
to produce metal hydroxide and hydrogen gas.
(a) 64 mA (b) 15 mA (c) 9.87 mA (d) 1 mA From Statement I
l (b) According to the question, Na + H2 O ¾® NaOH + H2 (g) + Energy
C From Statement II
K + H2 O ¾® KOH + H2 (g) + Energy.
1Ω A 1Ω So, both statements are true, but Statement II does not
C C provide correct explanation to Statement I.
1Ω 1Ω 175. You are provided with 18 g each of O2 , N2 , CH4 and
H 2O. Which of the following is the correct
1Ω B 1Ω decreasing order of number of atoms present in
C C these samples?
1Ω (a) CH4 > H2O > N2 > O 2 (b) O 2 > N2 > H2O > CH4
(c) CH4 > N2 > O 2 > H2O (d) N2 > H2O > O 2 > CH4
C
36 NTSE SOLVED PAPER 2019 (Stage II)

l (a) 177. An organic compound A on heating with


(i) For O2 -atoms concentrated H 2SO 4 gave product B and on warming
Given mass = 18 with alkaline KMnO4 gave compound C.
Molar mass of O2 = 32 g/mole Compound A on heating with compound C in
18 presence of concentrated H2SO4 formed compound
Number of O2 -atoms = ´ 2 ´ N A = 1125
. NA
32 D, which has fruity smell.
(ii) For N2 -atoms Identify the compounds A, B, C and D
Given mass = 18 (a) A = Alcohol, B = Carboxylic acid, C = Alkene, D = Ester
Molar mass of N2 = 28 g/mole (b) A = Carboxylic acid, B = Ester, C = Alkene, D = Alcohol
Number of N2 -atoms =
18
´ 2 ´ N A = 128
. NA (c) A = Alcohol, B = Alkene, C = Carboxylic acid, D = Ester
28 (d) A = Alkene, B = Alcohol, C = Ester, D = Carboxylic acid
(iii) For H2 O-atoms l (c) Correct answer is
Given mass = 18 g A = Alcohol, B = Alkene, C = Carboxylic acid, D = Ester
Molar mass of H2 O = 18 Conc. H2SO4
18 C2 H5 OH ¾ ¾¾¾¾¾® CH2 == CH2 + H2 O
Number of H2 O-atoms = ´ 3 ´ NA = 3NA Alcohol ( A ) D Alkene( B )
18
Alk. KMnO4
(iv) For CH4 -atoms C2 H5 OH ¾ ¾¾¾¾
¾® CH3 COOH
Carboxylic acid ( C )
Given mass = 18 g
Molar mass of CH4 = 16 Conc. H 2SO4
CH3 COOH + C2 H5 OH ¾¾¾¾® CH3 COOC2 H5 + H2 O
18
Number of CH4 -atoms = ´ 5 ´ N A = 5.6 N A ( C) (A) Ester ( D)
16
Alternate (shortcut) Method 178. Match list-I (mixture) and list-II (type) with the
N¢ w list-III (example) and select the correct answer from
Q =
NA M the combination given below.
Where, w and N A are given mass and Avogadro Number List-I (mixture) List-II (type) List-III (example)
respectively and are same for all the species. A. Liquid in gas 1. Emulsion I. Mist
N¢ = Number of particles and M = molar mass of species. B. Liquid in liquid 2. Aerosol II. Sponge
1
Thus, N¢ µ C. Gas in solid 3. Foam III. Face cream
M
4. Gel IV. Butter
Also, M for O2 = 32
N2 = 28 (a) A-3-II, B-2-III, C-4-IV (b) A-2-I, B-1-III, C-3-II
H2 O = 18 (c) A-1-III, B-2-II, C-3-I (d) A-1-II, B-4-I, C-2-III
CH4 = 16 l (b) The correct answer from the combination is,
So, the decreasing order of number of atoms is (A) Liquid in gas (2) Aerosol (I) Mist
CH4 > H2 O > N2 > O2 (B) Liquid in liquid (1) Emulsion (III) Face cream
(C) Gas in solid (3) Foam (II) Sponge
176. Manya, Kartik, Gurnoor and Sheena has arranged
the ions F- , Na + , O2 - and Mg + in decreasing orders 179. Which of the following sets of reactions will not
occur?
of their ionic radii.
I. MgSO4 ( aq) + Fe( s) ® FeSO4 ( aq) + Mg( s)
Manya — O2 - > Mg 2 + > F - > Na +
II. CuSO4 ( aq) + Fe( s) ® FeSO4 ( aq) + Cu( s)
Kartik — Mg 2 + > Na + > O2 - > F -
III. MgSO4 ( aq) + Cu( s) ® CuSO4 ( aq) + Mg( s)
Gurnoor — O2 - > F - > Na + > Mg 2 +
IV. CuSO4 ( aq) + Zn( s) ® ZnSO4 ( aq) + Cu( s)
Sheena — F - > Na + > O2 - > Mg 2 + (a) I and III (b) II and IV (c) I, II and III (d) II, III and IV
Who had provided the correct order of their l (a) More reactive metal can displace less reactive metal
decreasing ionic radii? from its salt solution.
(a) Manya (b) Kartik According to activity series, the reactivity order of metals
(c) Gurnoor (d) Sheena is : Mg > Zn > Fe>Cu
(I) MgSO4 (aq) + Fe(s) ¾® FeSO4 (aq) + Mg(s)
l (c) If the species having same number of total electrons
then, it is called ‘isoelectronic’. Here, Fe is less reactive than Mg.
F - , Na+ , O2 - and Mg2 + are isoelectronic. So, this reaction will not be occured.
(III) MgSO4 (aq) + Cu(s) ¾® CuSO 4 (aq) + Mg(s)
If number of protons increases, size of ion decreases.
Here, Cu is less reactive than Mg.
So, the correct order of their decreasing ionic radii is
So, this reaction will not be occured.
O2 - > F - > Na+ > Mg2 +
Hence, Gurnoor had provided the correct order.
NTSE SOLVED PAPER 2019 (Stage II) 37
180. Two organic compounds ‘A’ and ‘B’ react with Choose the correct decreasing order of reactivity of
sodium metal and both produce the same gas ‘X’, these metals amongst the following :
but with sodium hydrogen carbonate only (a) M > L > H > K (b) K > M > H > L
compound B reacts to give a gas ‘Y ’. Identify ‘A’, ‘B’, (c) M > K > L > H (d) L > H > K > M
‘X’ and ‘Y ’. l (c) Metal M reacts with cold water and metal K reacts with
(a) A = Ethylene, B = Ethyl alcohol, X = Carbon dioxide, warm water, it shows metal M is more reactive than K.
Y = Hydrogen Metal H and L does not react with dilute HCl and water
(b) A = Ethyl alcohol, B = Acetic acid, X = Hydrogen, respectively but metal L displaces metal H from its salt
Y = Carbon dioxide solution, it shows metal L is more reactive than H.
(c) A = Methyl alcohol, B = Ethyl alcohol, X = Hydrogen, So, the decreasing order of reactivity of these metals is :
Y = Carbon dioxide M > K > L > H.
(d) A = Acetic acid, B = Formic acid, X = Carbon dioxide,
Y = Hydrogen 183. Match chemical reactions given in the list-I with the
type of chemical reactions given in list-II and select
l (b) 2CH3 ¾ CH2 ¾ OH + 2 Na ¾® 2 C2 H5 ¾ ONa + H2 (g) the correct option using the combinations given below.
Ethyl alcohol ( A ) (X )
Hydrogen
List-I List II
2CH 3 COOH + Na ¾® 2 CH 3 COONa + H2 (g) (Chemical reactions) (Type of chemical
Acetic acid ( B ) (X )
reactions)
CH 3 COOH + NaHCO3 ¾® CH 3 COONa + H 2 O + CO2 (g) A. Formation of NH 3 from N 2 and H 2 I. Decomposition
Acetic acid( B ) (Y)
Carbon B. Calcination of zinc carbonate II. Double
dioxide displacement
The correct answer will be
A = Ethyl alcohol, B = Acetic acid, X = Hydrogen, C. Reaction of aqueous BaCl 2 III. Combination
solution with dilute H 2 SO 4
Y = Carbon dioxide
D. Rancidity of oils IV. Redox
181. Consider the elements A, B, C and D with atomic V. Displacement
numbers 11, 12, 16 and 17, respectively. Which
among the following statements regarding these (a) A-I, B-V, C-III, D-IV (b) A-III, B-IV, C-V, D-I
(c) A-IV, B-III, C-V, D-I (d) A-III, B-I, C-II, D-IV
elements are correct?
I. The element C will gain electron more easily than l (d) (A) Formation of NH3 from N2 and H2 ® Combination
reaction.
element D
N2 + 3H2 ¾® 2NH3
II. The element B tends to lose electron more readily
(B) Calcination of zinc carbonate ® Decomposition reaction
than C. D
ZnCO3 ¾¾ ® ZnO + CO2
III. The oxide of A will be least basic while that of D
will be most basic. (C) Reaction of aqueous BaCl2 solution with dil.H2 SO4 ®
Double displacement reaction
IV. The energy required to remove an electron from
outermost shell from A will be minimum while BaCl2 + H2 SO4 ¾® BaSO4 + 2HCl
that from D will be maximum. (D) Rancidity of oil ® Redox reaction
(a) I and III (b) I and IV (c) II and III (d) II and IV So, A-III, B-I, C-II, D-IV is the correct answer.
l (d) A B C D 184. You are provided with aqueous solutions of three
Atomic number - 11 12 16 17 salts — A, B and C, 2-3 drops of blue litmus solution,
Elements - Na Mg S Cl red litmus solution and phenolphthalene were
Sodium and magnesium will show metallic properties added to each of these solution in separate
while, sulphur and chlorine will show non-metallic experiments. The change in colours of different
properties. indicators were recorded in the following table
(II) Mg (metal) lose electron more readily than sulphur. Sample With blue With red With
(IV) Energy required to remove an e- from outermost shell litmus solution litmus solution phenolphthalene
from sodium will be minimum while that from chlorine solution
will be maximum. A No change No change No change
So, statement (II) and (IV) are correct.
B Turns red No change No change
182. The following obse+rvations are given for four C No change Turns blue Turns pink
metals :
I. Metal H does not react with dilute HCl. On the basis of above observations, identify A, B and
C from the following options :
II. Metal K reacts with warm water.
(a) A = NH4Cl, B = NaCl, C = CH3COONa
III. Metal L does not react with water but displaces (b) A = NH4Cl, B = CH3COONa, C = NaCl
metal H from its aqueous salt solution. (c) A = NaCl, B = NH4Cl, C = CH3COONa
IV. Metal M reacts with cold water. (d) A = CH3COONa, B = NH4Cl, C = NaCl
38 NTSE SOLVED PAPER 2019 (Stage II)

l (c) A = NaCl, B = NH4 Cl, C = CH3 COONa l (c) Given, atomic number of A element = 1
atomic number of B element = 17
Sample With blue With red With
litmus litmus phenolphthal So, electronic configuration of A = 1
solution solution -ene solution electronic configuration of B = 2, 8, 7
A. NaCl No change No change No change Both A and B require one electron to complete their
(neutral) (neutral) outermost shell.
B. NH 4Cl Turns red No change No change So, the formula of the compound is AB.
(acidic Its nature is covalent and its aqueous solution is acidic in
solution) nature.
C. CH 3COONa No change Turns blue Turns pink 188. Choose one of the following alternative statements
(basic (basic given below, which correctly explain the process of
solution) solution)
osmosis.
(a) Movement of water from regions of concentrated to dilute
185. Match list-I (mixture to be separated) with the list-II
solution
(method used) and select the correct answer using
(b) The passage of solute from weak solution to strong
the options given below: solution through a selectively-permeable membrane
List-I List II (c) A passive transport of a solvent through a
(Mixture to be separated) (Methods used) selectively-permeable membrane from a region of low
A. Liquid N 2 and liquid O 2 I. Chromatography solute concentration to a region of high solute
concentration
B. Red and blue inks II. Sublimation (d) An energy-dependent transport of a solvent through a
C. Solution of NaCl in water III. Fractional distillation selectively-permeable membrane from region of low solute
D. Nephthalene and NaCl IV. Evaporation concentration to a region of high solute concentration
V. Crystallisation l (c) A passive transport of a solvent through a
selectively-permeable membrane from a region of low
(a) A-I, B-II, C-IV, D-V (b) A-III, B-V, C-II, D-IV solute concentration to a region of high solute
(c) A-III, B-I, C-IV, D-II (d) A-III, B-IV, C-I, D-II concentration is known as osmosis. It permits the free
passage of water molecules, but restricts the passage of
l (c) (A) Liquid N2 and liquid O2 can be separated by dissolved solutes. The net direction and rate of osmosis
fractional distillation. depends on both pressure gradient and concentration
(B) Red and blue inks can be separated by gradient.
chromatography.
189. In meiosis, each of the four daughter cells has one
(C) NaCl in water can be separated by evaporation.
set of chromosomes. Due to randomness of process
(D) Nephthalene and NaCl can be separated by of chromosome separation in meiosis, large number
sublimation as nephthalene is sublime in nature.
of chromosome combinations can form gametes.
So, A-III, B-I, C-IV, D-II is the correct answer. How many such chromosome combinations in the
186. Select the correct set of statements regarding gametes are possible in case of humans, assuming
change in properties, as we move down the second there is no crossing-over taking place?
group in periodic table. (a) 2 22 (b) 2 23 (c) 2 46 (d) 2 34
I. Atomic size increases l (b) During meiosis due to random separation of
homologous chromosome 2 n number of gametes are formed.
II. Electronegativity increases.
In human, n = 23
III. Tendency to loose electrons increases
Where, n is the number of homologous chromosome pairs.
IV. Valency remains same So, 223 chromosome combinations in the gametes are
(a) I, II and III (b) II, III and IV (c) I, II and IV (d) I, III and IV possible in case of humans.
l (d) As we move down the second group in periodic table, then
190. Sclerenchyma in plants is an example of simple
Atomic size increases.
permanent tissue comprising of two types of
Due to increase in size, it can easily loose electrons. cells-sclereids and fibres. Why these cells are
Valency remains same i.e. 2. functionally important to the plants even after they
So, the correct set of statements are I, III and IV. die?
187. Which of the following options containing formula Choose the correct alternative from the options given
bonding and nature of aqueous solution, respectively below.
is correct for the compound formed by two elements (a) Both are thin-walled cells lacking intercellular spaces
(b) Walls in both the types of cells are thick and cutinised.
A and B having atomic numbers 1 and 17,
(c) Walls in both the cell types are thick and usually lignified.
respectively?
(d) Both the cells are used for conducting solutes and
(a) AB, ionic, acidic (b) AB 2 , ionic, basic
providing strength to the plant.
(c) AB, covalent, acidic (d) AB 2 , covalent, neutral
NTSE SOLVED PAPER 2019 (Stage II) 39
l (c) Sclerenchyma in plants is an example of simple 194. Glucose is the prime source of energy in our body.
permanent tissue comprising of two types of cells, sclereids However, it is stored in the form of glycogen in the
and fibres. These cells are functionally important to the plants muscle and liver of animals and in the form of
even after they die because walls in both cell types are thick
and usually lignified and provide strength to the plant.
starch in plants. As a result, every time a cell
requires glucose, it must hydrolyse glycogen which
191. Which one of the following organisms has a cellular is an energy consuming process. Why does the cell
respiratory pigment dissolved in plasma and is also store glycogen instead of glucose in free form?
a predaceous carnivore and shows matriphagy? (a) Glycogen is more compact and more hydrophilic
(a) Scorpion (b) Cockroach (b) Storage of glucose in free form will consume more ATP
(c) Earthworm (d) Sea cucumber (c) Glucose in the free form creates more osmotic pressure
l (a) Scorpion has a cellular respiratory pigment dissolved (d) Glucose is highly reactive molecule hence storing in the
in plasma and is also a predaceous carnivore. It’s offspring free form can result in unwanted reactions in the cells
consume mother within first few weeks of life which is l (c) Glucose in the free form creates more osmotic pressure,
called matriphagy. so to maintain osmolarity, the cell stores glycogen instead
of glucose in free form. Osmotic pressure is determined by
192. Lichens are sensitive to certain air pollutants and
molar concentration of the solute particles in the solution.
are often replaced by other plants. From the given
options choose the best combination of sensitivity 195. The figure given below is designed to show yeast
and replacement of lichens. respiration. In one of the tubes, there is yeast
(a) Sulphur dioxide and moss (b) Sulphur dioxide and algae suspension in glucose solution. This solution was
(c) Carbon dioxide and ferns (d) Sulphur dioxide and grass boiled before yeast was added to it. Which one of
l (a) Lichens are sensitive to sulphur dioxide and are the following is the possible reason for boiling of
replaced by mosses. Lichens are sensitive to SO2 because sugar solution?
their efficient absorption systems result in rapid
accumulation of sulphur when exposed to high level of
sulphur dioxide pollution. Lichen is a composite, which is
formed by an algae and a fungus.

193. A student was performing an experiment to


understand the enzyme-substrate reaction. The
student measured the formation of coloured product
using a calorimeter. The student plotted the graph
Oil
below which shows the reaction rate versus the Bicarbonate
substrate concentration. Glucose+yeast
suspension indicator
Following interpretations were
drawn by the student : (a) To ensure aerobic fermentation
Reaction rate

A. The higher concentration of (b) To provide the initial warmth for the yeast to become active
substrate acts as an enzyme (c) To remove the dissolved oxygen and carbon dioxide from
inhibitor. the solution
B. It is a sigmoidal curve with (d) To remove dissolved carbon dioxide and trap the oxygen
Substrate
sharp transition from low to from the atmosphere
concentration
high reaction rates over the l (c) The possible reason for boiling of sugar solution is to
increasing substrate concentration. remove the dissolved oxygen and carbon dioxide from the
solution. It is done to ensure anaerobic fermentation and
C. The curve reaches a plateau and does not further to verify CO2 production during fermentation.
increase with increasing substrate
concentrations due to saturation of enzyme with 196. A squirrel was eating a fruit on the ground.
the substrate. Suddenly, it was attacked by a dog. The squirrel
Choose which of the interpretations of the graph are rushed to the tree immediately and saved itself from
correct. the dangerous attack. What immediate changes are
(a) A and B (b) A and C (c) Only B (d) B and C most likely to have taken place in the body of the
squirrel?
l (b) According to the given graph, reaction rate is
increasing with substrate concentration but after a certain A. Blood flows to the stomach for rapid digestion.
point, higher concentration of substrate act as an enzyme B. Adrenaline was secreted in the blood by the
inhibitor. Thus, the curve reaches a plateau and does not adrenal glands.
further increase with increasing substrate concentration
C. Heartbeat becomes faster and heart pumps more
due to saturation of enzyme with the substrate. Further
increase in reaction velocity occurs only if the enzyme blood so that muscles get more oxygen
concentration is increased. Thus, enzyme and substrate D. Adrenocorticotropic hormone is secreted in the
concentration affects the rate of reaction. blood and blood flows more towards the vital organs.
40 NTSE SOLVED PAPER 2019 (Stage II)

Select the correct combination of options given below 199. It is generally observed that malaria is rampant in
(a) A and B (b) A and C (c) B and C (d) C and D areas where construction work and/or stagnant water
l (c) In an emergency situation, adrenaline hormone is are usually seen. Plasmodium species are known to
secreted in the blood by the adrenal gland that increases cause malaria. The parasite when injected by the
heartbeat and pumps more blood, so that muscles get more mosquito into the human bloodstream goes through
oxygen. Adrenaline is associated with potentiating proper specific life cycle stages. Select from below the
response in conditions of “flight or fight”. Adrenaline and correct sequence of stages.
nor-adrenaline are rapidly secreted in response to stress of
any kind and during emergency situations. Both these (a) Mosquito (sporozoites) ® Human liver (merozoites)
hormones increase heartbeat, strength of heart contraction ® Human RBC (gametes) ® Mosquito
and rate of respiration. (zygote-oocyst-sporozoites)
(b) Mosquito (merozoites) ® Human RBC (gametes) ®
197. Stimulus from the environment is detected by the Human liver (sporozoites) ® Mosquito (oocyst - zygote -
nerve cells. The stimulus acquired is transmitted in sporozoites)
the form of electrical impulse. From the options (c) Mosquito (merozoites) ® Human liver (sporozoites) ®
given below choose the correct scheme showing the Human RBC (gametes) ® Mosquito (oocyst - zygote -
direction in which the nerve impulse travels. sporozoites)
(Arrows show the direction of impulse flow). (d) Mosquito (sporozoites) ® Human liver (sporozoites) ®
Human RBC (merozoites) ® Mosquito (zygote - oocyst -
sporozoites)
(a)
l (a) The correct sequence of malaria parasite’s life cycle
stages in the human body is
(b)
Mosquito (sporozoites) ® Human liver (merozoites) ®
Human RBC (gametes) ® Mosquito (zygote-oocyst -
(c) sporozoites).
Plasmodium completes its life cycle in two hosts. Its
(d) infectious stage is sporozoite which multiplies in liver cells
and resultant merozoites enters in RBC to produce
gametocyte which are further taken by mosquito where
l (c) The nerve impulse flows in one direction. The nerve they fertilize in mosquito intestine.
impulse travels along a neuron or across a synapse
(junction), between nerve endings of the preceeding neuron 200. A plant with red coloured flowers is crossed with a
and dendrites of the succeeding neuron. The dendrites of a plant having white flowers. The red and white colours
neuron receive incoming nerve impulses and axon of the flower are controlled by a single gene. Red is
transmits the impulse to another neuron or receptor dominant over white. The F1 progeny is
through nerve endings. self-pollinated and the flower colour in F2 is
observed.
198. ‘‘Double fertilisation’’ is a complex mechanism of
flowering plants that is also unique to angiosperms. Given the above information, what is the expected
phenotypic ratio of plants with different flower
Choose the most appropriate statement from the
colours?
options listed below that explains this phenomenon.
(a) All plants with red flowers
(a) Fertilisation in two flowers of the same plant forming
(b) Red : White in the ratio of 3 : 1.
endosperms
(c) Pink : White in the ratio of 3 : 1.
(b) Two male gametes fertilise two eggs inside the ovule as a
(d) Red : Pink : White in the ratio of 1 : 2 : 1.
result the ovary gives rise to bigger fruits
(c) Two fertilisations occur in a flower-one fertilisation results in l (b) According to given information, expected phenotypic
the formation of a diploid zygote and the second ratio of plants with different flower colours will be
fertilisation results in the formation of a triploid endosperm Red : White in the ratio of 3 : 1.
(d) Two pollen grains sending two pollen tubes inside the Red flower White flower Parents
ovary, resulting in the formation of two seeds inside the RR x rr
fruit
l (c) In “Double fertilisation”, two fertilisations occur in a Red flower
flower-one fertilisation results in the formation of a diploid Rr F1-generation
zygote and the second fertilisation results in the formation R r
of a triploid endosperm.
RR Rr
In angiosperms, during double fertilisation, two male R (Red) (Red)
gametes reach to ovule via pollen tube. One male gamete
fuses with zygote to form diploid zygote and second male r Rr rr
F2 -generation
gamete fuses with secondary nucleus to form triploid (Red) (White)
endosperm.
NTSE~ SOLVED PAPER 2018 (Stage I) 1

NTSE Solved Paper


NATIONAL TALENT
SEARCH
EXAMINATION
2018 (Stage I)

INSTRUCTIONS
This solved paper consists of two papers. Paper I consists of Mental Ability Test (MAT) and Paper II consists of
Scholastic Aptitude Test (SAT).
MAT covering (Q. Nos. 1-50) of Maths and (Q. Nos. 51-100) of Reasoning.
SAT consists of (Science, Mathematics and Social Science) which comprises 100 questions (40 Science, 20 Mathematics
and 40 Social Science).
There will be no negative marking.
Each correct answer will be awarded one mark.

Time : 240 Minutes Max. Mark : 200

Paper I Mental Ability Test (MAT)


25 50 \ (x - y)2 = x2 + y2 - 2 xy
1. If x + = 10, then value of x 2 + 2 will be
x x = 15 - (- 6) [from Eqs. (ii) and (iii)]
(a) 29 (b) 25 (c) 24 (d) 27 = 15 + 6 = 21
(d) Given, a b c a+b+c
25 3. If = = , then value of will be
x+ = 10 3 5 7 b
x
(a) 7 (b) 3 (c) 10 (d) 5
Þ x2 + 25 = 10 x
(b) Given,
Þ x - 10 x + 25 = 0
2
a b c
Þ (x - 5)2 = 0 [ (a - b)2 = a2 - 2 ab + b2 ] = = = k Þ a = 3 k, b = 5 k, c = 7 k
3 5 7
Þ x =5 a + b + c 3 k + 5 k + 7 k 15 k 15
50 50 \ = = = =3
\ x2 + 2 = (5)2 + 2 b 5k 5k 5
x (5)
50 4. If sum of two numbers is 25 and sum of their square
= 25 + = 25 + 2 = 27
25 is 425, then what will be their product?
(a) 200 (b) 300 (c) 100 (d) 400
2. If x + y = 3 and x 2 + y 2 = 15, then value of ( x - y )2
(c) Let the numbers are x and y.
will be According to the question,
(a) 21 (b) 36 (c) 25 (d) 16
x + y = 25 … (i)
(a) Given, x2 + y2 = 425 … (ii)
x + y =3 … (i) Squaring the both sides of Eq. (i), we get
and x2 + y2 = 15 … (ii) x2 + y2 + 2 xy = 625
On squaring the both sides of Eq. (i), we get Þ 425 + 2 xy = 625 [from Eq. (ii)]
x2 + y2 + 2 xy = 9 200
Þ 2 xy = 625 - 425 Þ xy = = 100
Þ 15 + 2 xy = 9 [from Eq. (ii)] 2
Þ 2 xy = 9 - 15 = - 6 …(iii) Hence, the product of their numbers is 100.
2 NTSE~ SOLVED PAPER 2018 (Stage I)

5. If 0.64 ¸ a2 = 64, then positive value of ‘a’ will be 8. If 17 + x 11 = 11 + 6 then value of x 2 will be
(a) 0.1 (b) 0.01 (c) 1.0 (d) 10 (a) 11 (b) 23 (c) 6 (d) 24
(a) Given, (d) Given,
0.64 ¸ a2 = 64 17 + x 11 = 11 + 6
0.64
Þ = 64
a2 On squaring the both sides, we get
0.64 64 17 + x 11 = 11 + 6 + 2 11 ´ 6
Þ a2 = =
64 64 ´ 100 [ (a + b)2 = a2 + b2 + 2 ab]
1 1 Þ x 11 = 17 + 2 11 ´ 6 - 17
Þ a2 = Þa=
100 10 Þ x 11 = 2 11 ´ 6
(taking positive value) 2 11 ´ 6
Þ x= = 2 6 Þ x2 = 24
Þ a = 01
. 11

6. Divisor is 30 times of Quotient and 4 times of a


9. If 0.02 ´ 0.2 ´ a = 0.2 ´ 0.2 ´ b, then value of will
Remainder. If Quotient is 20 then Dividend will be b
(a) 1200 (b) 12150 (c) 10000 (d) 600 be
(b) According to the question, (a) 0.4 (b) 0.2 (c) 0.04 (d) 0.02
Divisor = 30 ´ Quotient (a) Given,
= 30 ´ 20 [ Quotient = 20 (given)] 0.02 ´ 0.2 ´ a = 0.2 ´ 0.2 ´ b
= 600 On squaring the both sides, we get
and Divisor = 4 ´ Remainder 0.02 ´ 0.2 ´ a = 0.2 ´ 0.2 ´ 0.2 ´ 0.2 ´ b
Þ 600 = 4 ´ Remainder a 0.2 ´ 0.2 ´ 0.2 ´ 0.2
Þ = = 0.4
600 b 0.02 ´ 0.2
Þ Remainder = = 150
4
\ Dividend = Divisor ´ Quotient + Remainder
10. If 7 - 3 and 7 + 3 are solution of a quadratic
= 600 ´ 20 + 150 equation, the quadratic equation will be
= 12000 + 150 (a) x2 - 14 x + 46 = 0 (b) x2 + 14 x - 46 = 0
= 12150 (c) x2 - 14 x - 46 = 0 (d) x2 + 14 x + 46 = 0
(a) Roots of quadratic equation are 7 - 3 and 7 + 3 .
a
7. If 3 a - 2 b = 27 and 9 a + b = 3, then value of - will be Sum of roots = 7 - 3 + 7 + 3 = 14
b Product of roots = (7 - 3 ) (7 + 3 ) = 49 - 3 = 46
-4 5 5 8
(a) (b) (c) (d) [ (a - b) (a + b) = a2 - b2 ]
3 8 6 5
\ Quadratic equation, x - (sum of roots) x
2

(d) Given, + (product of roots) = 0


3 a - 2 b = 27 Þ x2 - 14 x + 46 = 0
Þ 3 a - 2 b = (3)3
Þ a - 2b = 3 … (i)
11. In a triangle PQR if ÐQ = 3 ÐR = 2( ÐP + ÐR), then
(on comparing of indices) value of ÐQ will be
and 9a +
=3 b (a) 110° (b) 120° (c) 40° (d) 102°
Þ (3) 2 ( a + b)
= (3)1 Þ 2 (a + b) = 1 (b) Given, In DPQR,
(on comparing of indices) ÐQ = 3 ÐR = 2 (ÐP + ÐR)
1 ÐQ
Þ a+ b= … (ii) \ ÐQ = 3 ÐR Þ R = … (i)
2 3
On subtracting Eq. (ii) from Eq. (i), we get and 3 ÐR = 2(ÐP + ÐR)
1 6 -1 Þ 3 ÐR = 2 ÐP + 2 ÐR Þ 2ÐP = ÐR
- 3b = 3 - Þ - 3b =
2 2 ÐR ÐQ
Þ ÐP = Þ ÐP = … (ii)
-5 2 6
Þ b=
6 [from Eq. (i)]
On putting the value of ‘b’ in Eq. (i), we get In DPQR, ÐP + ÐQ + ÐR = 180°
- 5ö ÐQ ÐQ
a - 2 æç
5
÷ =3 Þa=3 - Þ + ÐQ + = 180° [from Eqs. (i) and (ii)]
è 6 ø 3 6 3
4 ÐQ + 6 ÐQ + 2 ÐQ
Þ a= Þ = 180°
3 6
4 Þ 9 ÐQ = 6 ´ 180°
a 4 ´6 8 6 ´ 180°
\ - =- 3 = = \ ÐQ =
b 5 3 ´5 5 9
-
6 = 6 ´ 20° = 120°
NTSE~ SOLVED PAPER 2018 (Stage I) 3
p x +3 p2 - q2 (c) Given, 2 x = 8 y -1
12. If = then value of 2 will be
q x -3 p + q2 Þ 2 x = (2)3( y -1 )
6x 6x 12 x 12 x Þ x = 3(y -1) [comparing of indices]
(a) (b) 2 (c) 2 (d)
x2 - 9 x +9 x +9 x2 - 9 Þ x - 3y = - 3 … (i)
p x+3 and 9y = 3x - 6
(b) = = k (Let) Þ (3)2 y = (3)x - 6
q x -3
Þ p = k(x + 3), q = k (x - 3) Þ 2y = x - 6 [comparing of indices]
p2 - q2 [k(x + 3)]2 - [k(x - 3)]2 Þ x - 2y = 6 … (ii)
\ =
p + q
2 2
[k(x + 3)]2 + [k(x - 3)]2 On subtracting Eq. (ii) from Eq. (i), we get
- y = -9 Þ y =9
k2 (x2 + 9 + 6 x) - k2 (x2 + 9 - 6 x)
= Putting the value of ‘y’ in Eq. (ii), we get
k2 (x2 + 9 + 6 x) + k2 (x2 + 9 - 6 x)
x - 18 = 6 Þ x = 6 + 8 = 24
k2 (x2 + 9 + 6 x - x2 - 9 + 6 x)
= \ x + y = 24 + 9 = 33
k2 (x2 + 9 + 6 x + x2 + 9 - 6 x)
12 x 6x 16. If two numbers are such that their difference, their
= =
2(x2 + 9) x2 + 9 sum and their product are in ratio 1 : 7 : 24, then
product of the two number is
13. If perimeter of a square is same as that of a (a) 48 (b) 44 (c) 54 (d) 38
rectangle whose length is 24m is double of its (a) Let the numbers are x and y.
breadth, then area of square will be According to the question,
(a) 324 m 2 (b) 342 m 2 (c) 224 m 2 (d) 330 m 2 x- y = k … (i)
(a) Let breadth of rectangle = x m x + y = 7k … (ii)
\ Length of rectangle = 2x m and xy = 24 k … (iii)
Þ 24 = 2 x (given, length of rectangle = 24 m) On adding Eq. (i) and Eq. (ii), we get
24 2x = 8k Þ x = 4k
Þ x= = 12 m
2 On putting the value of ‘x’ in Eq. (ii), we get
Hence, length and breadth of rectangle is 24 m and 12 m 4k + y = 7k Þ y = 3k
respectively.
According to the question, Now, putting the values of x and y in Eq. (iii), we get
24
Perimeter of square = Perimeter of rectangle 4 k ´ 3 k = 24 k Þ k = =2
12
Þ 4 ´ side = 2 (length + breadth)
\ xy = (4 k) (3 k) = 12 k2 = 12 ´ (2)2 = 48
Þ 4 ´ side = 2 (24 + 12)
2 ´ 36 17. The Mean of the Median, Mode and Range of the
Þ side = = 18
4 observations 7, 6, 7, 9, 14, 9, 7, 15 is
\ Area of square = (side)2 = (18)2 = 324 m2 (a) 8 (b) 9 (c) 10 (d) 7
14. If volumes of two cones are in ratio of 2 : 3 and their (a) Given observations, 7, 6, 7, 9, 14, 9, 7, 15
base radii are in ratio of 1 : 2, then what will be ratio To write ascending order of observations,
of their heights? 6, 7, 7, 7, 9, 9, 14, 15
(a) 8 : 3 (b) 3 : 2 (c) 4 : 3 (d) 2 : 3 Here, n = 8 (Even)
(a) Let radii of cones are r1 , r2 and heights h1 , h2 \ Median
Value of th observation + Value of æç + 1ö÷ th observation
respectively. n n
r1 1 2 è 2 ø
Then, = … (i) =
r2 2 2
Value of th observation+ Value of æç + 1ö÷ th observation
1 2 8 8
pr1 h1 è2 ø
= 2
=3
Volume of first cone
\
Volume of second cone 1 pr2 h 2
3
2 2 Value of 4th observation + Value of 5th observation
2
=
2 æ r1 ö æh ö 2
Þ =ç ÷ ´ç 1÷ 7 + 9 16
3 è r2 ø è h2 ø = = =8
2
2 2
2 æ1 ö h The frequency 3 of 7, is the greatest in given observations.
Þ =ç ÷ ´ 1 [from Eq. (i)]
3 è2 ø h2 \ Mode of observations = 7
h1 2 4 8 and Range of observations = Maximum value of
Þ = ´ =
h2 3 1 3 observations - Minimum value of observations = 15 - 6 = 9
\ h1 : h2 = 8 : 3 8+7+9
Now, Mean of Median, Mode and Range =
3
15. If 2 x = 8 y - 1 and 9 y = 3 x - 6 , then value of x + y 24
= =8
(a) 34 (b) 25 (c) 33 (d) 24 3
4 NTSE~ SOLVED PAPER 2018 (Stage I)

18. A person spends 80% of his income. With increase 1


20. If 5 = a + , then value of a will be
in the cost of living, his expenditure increased by 1
1 2 1+
37 % and his income increases by 16 % . His 1
6+
2 3 2
present per cent saving is 15 62 14 61
1 1 1 5 (a) (b) (c) (d)
(a) 10 % (b) 12 % (c) 5 % (d) 5 % 62 15 61 14
5 3 3 7 1 1
(b) 5 = a + Þ 5 = a+
(d) Let first income = x 1 2
1+ 1+
80 1 13
Expenditure = 80% of x = x 6+
100 2
æ100 + 16 2 ö 13 13
Þ 5 = a+ Þa=5 -
ç ÷ 15 15
Income to be increasing = x ´ ç 3 ÷ = x ´ 350 = 7 x
75 - 13 62
ç 100 ÷ 300 6 Þ a= =
è ø 15 15
Expenditure to be increasing 7 5
æ100 + 37 1 ö 21. If of a number is 5 more than its . Then, nine
8 7
80 x ç ÷
2 ÷ = 80 x ´ 275 = 11x
= ´ç times of number will be
100 ç 100 ÷ 100 200 10
è ø (a) 380 (b) 208 (c) 308 (d) 280
7 11 35 x - 33 x x (d) Let the number is x.
Present saving = x - x= =
6 10 30 15 According to the question,
x 7 5
x - x =5
8 7
\ Per cent saving = 15 ´ 100
7 (49 - 40)x
x Þ =5
6 56
6 40 5 5 ´ 56 280
= ´ 100 = =5 % Þ x= =
15 ´ 7 7 7 9 9
\ Nine times of numbers
19. The cost of five chairs and three tables is ` 3110. If 280
= 9x = 9 ´ = 280
cost of one chair is 210 less than cost of one table. 9
What is the cost of two tables and two chairs?
(a) ` 1760 (b) ` 1000 22. If a cone of height 24cm and base 6 cm melted and
(c) ` 1660 (d) ` 1800 reshape into a sphere. Then, what will be the total
(c) Let price of one chair and one table are ` x and ` y surface area of sphere?
respectively. (a) 36 p Sq cm (b) 16 p Sq cm
(c) 144 p Sq cm (d) 142 p Sq cm
According to the question,
5 x + 3 y = 3110 … (i) (c) Height of cone, h = 24 cm
and y - x = 210 … (ii) Radius of cone, r = 6 cm
From Eq. (ii) y = x + 210, putting in Eq. (i), we get Let radius of sphere is R.
5 x + 3 (x + 210) = 3110 According to the question,
Þ 5 x + 3 x + 630 = 3110 Volume of sphere = Volume of cone
4 3 1 2
Þ 8 x = 3110 - 630 pR = pr h
2480 3 3
Þ x= = 310
8 Þ 4 R3 = (6)2 ´ 24
6 ´ 6 ´ 24
On putting the value of x in Eq. (i), we get Þ R3 =
4
y - 310 = 210
Þ R3 = 6 ´ 6 ´ 6
Þ y = 310 + 210 = 520
Þ R = 6 cm
\ Price of two tables and two chairs
Total surface area of sphere = 4 pR2
= 2 ´ 520 + 2 ´ 310
= 4 p ´ (6)2
= 1040 + 620 = ` 1660
= 144p cm2
Adding Eqs. (i) and (ii), we get 23. P and Q can do a piece of work in 10 days, Q and R
4 x + 4 y = 3320 can do same work in 15 days, R and P can do the
3320 same work in 20 days. Then, in how many days R
Þ 4(x + y) = 3320 Þ 2(x + y) =
2 will complete it alone?
Þ 2(x + y) = 1660 (a) 115 days (b) 110 days
Hence, price of two tables and two chairs is ` 1660. (c) 130 days (d) 120 days
NTSE~ SOLVED PAPER 2018 (Stage I) 5
1
(d) 1 day work of P and Q’s = 26. A vessel contains 60 L of milk, 12 L of milk is taken
10
1 out of it and is replaced by water. Then again from
1 day work of Q and R’s = the mixture 12 L are taken out and replaced by
15
1 water.
1 day work of R and P’s =
20 Find the amount of milk left after the operation.
\ 1 day work of P, Q and R’s (a) 28.4 L (b) 21.6 L
(c) 36 L (d) 38.4 L
= æç +
1 1 1 1 ö
+ ÷
2 è10 15 20 ø (d) If taken out quantity of substance a from the quantity x
1 6 + 4 + 3 13 of a substance, to replace a quantity other substance and
= ´ = this process revised n times, then
2 60 120 n
The quantity of original substance in mixture = x æç1 - ö÷
13 1 13 - 12 1 a
1 day work of R’s = - = = è
120 10 120 120 xø
\ R will complete the work in Here, x = 60 L, a = 12 L , n = 2
2 2
1
\ Amount of milk = 60 æç1 -
= = 120 days. 12 ö æ 1ö
1 ÷ = 60 ç1 - ÷
è 60 ø è 5ø
120 2

= 60 ´ æç ö÷ = 60 ´
4 16
= 38.4 L
24. In the following which one is the smallest è5 ø 25
3 ,3 2 , 2 ,3 4 ? 27. Select the one which is different from the other
(a) 3 (b) 3 4 three responses.
(c) 3 2 (d) 2 (a) 15 : 46 (b) 12 : 37 (c) 9 : 28 (d) 8 : 33
(c) Given numbers, (d) 15 : 46 ® 15 : (15 ´ 3 + 1)
3, 3 2, 2, 3 4 12 : 37 ® 12 : (12 ´ 3 + 1)
LCM of powers of given surd numbers 2, 3, 2, 3 = 6 9 : 28 ® 9 : (9 ´ 3 + 1)
\ 3 = 6 (3)3 = 6 27 8 : 33 ® 8 : (8 ´ 4 + 1)
Þ 3
2 = 6 (2)2 = 6 4 Clearly, 8 : 33 is different from others three.

2 = 6 (2)3 = 6 8 28. In a row of boys A is 20th from left and B is 16th


from right, interchange their position, then A
Þ 3
4 = 6 (4) = 16
2 6
becomes 30th from left. How many boys are there in
3
Clearly, 2 is the smallest. the row?
25. If P denotes +, Q denotes -, R denotes ´ and S (a) 46 (b) 44 (c) 45 (d) 48
denotes ¸, which of the following statement is correct (c) First condition
(a) 36 R 4 S 8 Q 7 P 4 = 10 A (20th) B (16th)
(b) 16 R 12 P 49 S 7 Q 9 = 200
19 15
(c) 32 S 8 R 9 = 160 Q 12 R 12
(d) 8 R 8 P 8 S 8 Q 8 = 57 After interchange
their position
(d) Given, B A (30th)
P ® ‘+’
Q ® ‘-’; R ® ‘´’; S ® ‘¸’ \ Number of boys in row = 30 + 15 = 45
To put given mathematical sign on the place of English 29. A 15 cm coloured cube is cut into 3 cm small cubes,
alphabet,
then how many cubes are formed which have only
(a) 36 ´ 4 ¸ 8 - 7 + 4 = 10 one face painted?
36 ´ 4
Þ - 7 + 4 = 10 (a) 54 (b) 64 (c) 44 (d) 84
8
Þ 15 = 10 (False) (b) Side of cube = 15 cm
(b) 16 ´ 12 + 49 ¸ 7 - 9 = 200 Side of small cube to cut in equal cubes of 3 cm side
15
Þ 192 + 7 - 9 = 20 = = 5 cm
3
Þ 190 = 200 (False)
Number of cubes to be painted have only one face = (5 - 1)3
(c) 32 ¸ 8 ´ 9 = 160 - 12 ´ 12
32 = (4)3 = 64 cubes
Þ ´ 9 = 160 - 144
8 30. A father tells his son ‘‘I was three times of your
Þ 36 = 16 (False) present age when you were born’’ If the father’s
(d) 8 ´ 8 + 8 ¸ 8 - 8 = 57 present age is 48 yr, how old was the boy 4 yr ago?
Þ 64 + 1 - 8 = 57 (a) 24 yr (b) 8 yr
Þ 57 = 57 (True) (c) 12 yr (d) 16 yr
Hence, statement (d) is correct.
6 NTSE~ SOLVED PAPER 2018 (Stage I)

(b) Let present age of boy = x yr (d) +1 +1 +1

Present age of father = 48 yr


+2 +3 +4 +5
Father’s age at time of birth of boy = (48 - x) yr
According to the question, 48 - x = 3 x 2 4 7 11 16
Þ 48 = 3 x + x 3 7 13 21 21
Þ 4 x = 48
48 +4 +6 +8 +10
Þ x= = 12 yr
4
+2 +2 +2
\ Age of boy 4 yr ago = 12 - 4 = 8 yr
\ ? = 7 /13
Directions (Q. Nos. 31-35) Find the missing term in the
series given below. 36. There are twelve dozen of apple in a basket. Two
dozen are added later. Then, apples got spoil and
31. 2, 12, 30, ?, 90, 120 are removed. The remaining are transferred equally
(a) 48 (b) 56 (c) 63 (d) 72 into two baskets, how many are there in each?
(b) (a) 168 (b) 158 (c) 79 (d) 89
Ú
2 12 30 56 90 132 (c) In first, the number of apples in basket = 12 dozen
= 12 ´ 12 apples = 144 apples
+10 +18 +26 +34 +42
Added apples later = 2 dozen = 2 ´ 12 = 24 apples
+8 +8 +8 +8 Spoil apples = 10
\ Remaining apples = 144 + 24 - 10 = 158
\ ? = 56 158
ÚIncorrect data in the given question. Now, number of apples in one basket = = 79 apples
2
32. 10, 100, 200, 310, ? 37. At what time between 8 and 9 will the hands of a
(a) 400 (b) 410 (c) 420 (d) 430 clock be together
7
(d) (a) 40 min past 8 (b) 43 min past 8
10 100 200 310 430 11
8 10
(c) 43 min past 8 (d) 44 min past 8
+90 +100 +110 +120 11 11

+10 +10 +10


(b) Let at 8 O’clock t min both hands will be together.
Round the angle by hour hand at 8 O’clock t min.
\ ? = 430 = Round angle in t min by minute hand
1
33. 0.5, 2, 4.5, 8, 12.5, ? 8 ´ 30 + t ´ = t ´ 6
2
(a) 16 (b) 17 (c) 16.5 (d) 18
t
(d) Þ 240 = 6t -
0.5 2 4.5 8 12.5 18 2
11 240 ´ 2
Þ 240 = t Þ t =
+1.5 +2.5 +3.5 +4.5 +5.5 2 11
480 7
+1 +1 +1 +1
Þ t = = 43 min
11 11
\ ? = 18 7
Hence, at 8 O’clock 43 min both hands will be together.
11
34. 109, 74, 46, 25, 11, ?
(a) 3 (b) 0 38. What is the value of A, B and C in the given matrix?
(c) 11 (d) 4 9 A 12
(d) 109 74 46 25 11 4 B 10 7
8 C 11
–35 –28 –21 –14 –7

(a) A = 13, B = 11, C = 9 (b) A = 13, B = 9, C = 11


–7 –7 –7 –7
(c) A = 9, B = 11, C = 13 (d) A = 9, B = 13, C = 11
\? = 4 (d) Given matrix,
2 4 ? 11 16 9 A =9 12
35. , , , ,
3 7 ? 21 31 B =13 10 7
6 5
(a) (b)
11 9 8 C =11 11
9 7
(c) (d) Put A = 9, B = 13, C = 11, the sum is 30 of each row, column,
11 13 and diagonal.
NTSE~ SOLVED PAPER 2018 (Stage I) 7
7 n + 3 + 14 ´ 7 n + 4 1
Part empty in 1 min by pipe C =
39. Simplified value of is 10
7n + 3
\ Part filled in 1 min by A, B and C pipes
(a) 98 (b) 100 (c) 99 (d) 97
1 1 1 5 + 4 -6 3 1
n+3 n+4
= + - = = =
7 + 14 ´ 7 12 15 10 60 60 20
(c)
7n + 3 1
\ A, B and C pipes will fill the tank = min = 20 min
7 n × 73 + 14 ´ 7 n ´ 74 1 / 20
=
7 n × 73
43. A sum amounts to ` 800 at 3% per annum in a
7 n × 73 (1 + 98)
= = 1 + 98 = 99 certain time but amount to ` 1000 at 5% per annum
7 n × 73 in the same time total sum and time are
40. If tan x = 5 - 3 then 22 tan(90° - x ) is equal to (a) ` 500, 20 yr (b) ` 400, 20 yr
(c) ` 550, 20 yr (d) ` 600, 10 yr
5+ 3
(a) 5 + 3 (b) 2 - 3 (c) (d) 13 + 3 (a) Let principle amount is P.
22
Amount after t yr with 3%
(a) Given, tanx = 5 - 3 P´3 ´t
1 1 =P+
\ cot x = = 100
tan x 5 - 3
Þ æ
800 = P ç1 +
3t ö
… (i)
1 5+ 3 5+ 3 ÷
= ´ = è 100 ø
5- 3 5+ 3 (5)2 - ( 3 )2 Amount after t yr with 5%
5+ 3 5+ 3 P´5 ´t
= = =P+
25 - 3 22 100
1000 = P æç1 +
5t ö
\ 22 tan (90° - x) = 22 cot x [ tan (90° - x) = cot x] Þ ÷ … (ii)
è 100 ø
5+ 3
= 22 ´
22 Dividing Eq. (ii) from Eq. (i),
P æç1 +
5t ö
=5 + 3 ÷
1000 è 100 ø 5 100 + 5t
1 1 = Þ =
41. If a = and b = , then find the value of 800 æ
P ç1 +
3t ö 4 100 + 3t
÷
2- 3 2+ 3 è 100 ø
7 a2 + 11ab - 7 b2 Þ 400 + 20t = 500 + 15t
(a) 11 + 3 56 (b) 13 + 11 56 Þ 20t - 15t = 500 - 400
100
(c) 11 + 5 3 (d) 11 + 56 3 Þ 5t = 100 Þ t = Þ t = 20
1 5
(d) Given, a =
2- 3 On putting the value of ‘t’ in Eq. (i), we get
3 ´ 20 ö
800 = P æç1 + ÷
1 2+ 3 2+ 3 è 100 ø
= ´ = =2 + 3
2 - 3 2 + 3 4 -3
800 = P æç1 + ö÷ Þ 800 = P ´
3 8
Þ
1 è 5ø 5
and b =
2+ 3 800 ´ 5
Þ P= = ` 500
1 2- 3 2- 3 8
= ´ = =2- 3
2+ 3 2- 3 4 -3 Hence, the principle amount and time are ` 500 and 20 yr.
\ 7 a2 + 11ab - 7 b2 44. If a and b are the roots of x 2 - 2 x - 1 = 0, then value
= 7(2 + 3 ) + 11 (2 +
2
3 ) (2 - 3 ) of a2 b + ab2 is
- 7(2 - 3 )2 (a) -2 (b) 2 (c) 1/2 (d) 4
= 7(4 + 3 + 4 3 ) + 11 (4 - 3)
(a) a and b are the roots of quadratic equation
- 7 (4 + 3 - 4 3 )
x2 - 2 x - 1 = 0.
= 49 + 28 3 + 11 - 49 + 28 3 - Coeff. of x
= 11 + 56 3 \ Sum of roots = a + b =
Coeff. of x2
2
42. Two pipes A and B can fill a tank in 12 and 15 min Þ a+ b= Þa+ b=2 … (i)
respectively. A third pipe C can empty it in 10 min. 1
Constant term
How long will it take to fill the tank if all pipes are Product of roots = ab =
opened simultaneously Coeff. of x2
-1
(a) 20 min (b) 30 min
(c) 40 min (d) 25 min Þ ab = Þ ab = - 1 … (ii)
1 1
(a) Part filled in 1 min by pipe A = \ a2 b + ab2 = ab (a + b)
12
1 = (- 1) (2) [from Eqs. (i) and (ii)]
Part filled in 1 min by pipe B =
15 = -2
8 NTSE~ SOLVED PAPER 2018 (Stage I)

Directions (Q. Nos. 45-49) Read the information carefully 48. The ratio of visitors from West Bengal below 20 yr to
and answer the questions the given below. visitors from Maharashtra above 40 yr in

Maharashtra
(a) 1 : 3 (b) 12 : 1 (c) 3 : 4 (d) 3 : 1
(b) Total number of visitors to go from West Bengal

5%
20
Above 40 = 20% of 100000 = 100000 ´ = 20000
yr 20% 15% 100
Below 20 others Number of visitors of age group below 20 yr to go from
yr 60% Karnataka
20-40 yr West Bengal = 60% of 20000
West 60%
20%
Bengal 20% 60
= 20000 ´ = 12000
100
Total number of visitors to go from Maharashtra
5
= 5% of 100000 = 100000 ´ = 5000
The pie chart above describes the characteristics of 100
Indian visiting UK from various states during a given Number of visitors of age group 40 yr above to to from
year. Maharashtra = 20% of 5000
Answer the following questions given below. Assume 20
= 5000 ´ = 1000
that the age wise distribution data applies to all states 100
and that in the given year 100000 Indian visited UK. 12000
\ Required ratio = = 12 : 1
1000
45. Number of visitors from Karnataka in the age group
of 20 - 40 yr. 49. Find the difference between visitors from West
(a) 20000 (b) 18000 Bengal and Maharashtra in the age group of 20-40 yr.
(c) 12000 (d) None of these (a) 4000 (b) 6000 (c) 3000 (d) 8000
(c) Total number of visitors to go from Karnataka (c) From question No. 48,
= 60% of 100000 Total number of visitors to go from West Bengal = 20000
60 Number of visitors of age group 20-40 yr to go from West
= 100000 ´ = 60000
100 Bengal = 20% of 20000
Number of visitors of age group 20-40 to go from Karnataka 20
= 20000 ´ = 4000
= 20% of 60000 100
20 and total number of visitors to go from Maharashtra = 5000
= 60000 ´ = 12000
100 Number of visitors of age group 20-40 yr to go from
Maharashtra = 20% of 5000
46. Number of visitors from Maharashtra below the age 20
= 5000 ´ = 1000
of 20 yr. 100
(a) 3000 (b) 5000 \ Required difference = 4000 - 1000 = 3000
(c) 60000 (d) 8000
(a) Total number of visitors to go from Maharashtra
50. The number of ways in which 6 students can be
seated at a round table is
= 5% of 100000
5 (a) 720 (b) 120 (c) 410 (d) 350
= 100000 ´ = 5000
100 (b) The number of ways to seated of 6 students at a round
Number of visitors of age group 20 yr below to go from table = (6 - 1)!
Maharashtra = 60% of 5000 = 5 ! = 5 × 4 × 3 × 2 ×1 = 120
60
= 5000 ´ = 3000 51. What letter will come next in the following series?
100 ABCDEFGZYXWUVTBCDEFYXWVU
47. How many visitors were below 20 yr of age but were CDEXWVR
neither from Karnataka nor Maharashtra or West (a) A (b) V (c) B (d) Z
Bengal? (*) The given series consist of two different series.
(a) 7000 (b) 15000 The pattern is as follows
(c) 9000 (d) 6000
(c) Total number of visitors to go from other states A B C D E F G B C D E F C D E D
= 15% of 100000
15
= 100000 ´ = 15000
100 ZYXWVUT YXWVU XWV W
Number of visitors of age group 20 yr below to go from
other states = 60% of 15000
60 Here, in first series ‘R’ is given (in question) in place of D
= 15000 ´ = 9000
100 also in second series ‘UV’ is given (in question) in place of
VU. So, the given data is incorrect.
NTSE~ SOLVED PAPER 2018 (Stage I) 9
52. Among P, Q, R, S and T each secured different 56. Who performs Dance?
marks, Q scored higher than T only and P scored (a) C (b) F
higher than S but lower than R. Who among them (c) D (d) A
scored highest marks?
(a) P (b) S (c) R (d) T
57. Which item is performed by D and on what day?
(a) Mimicry - Monday (b) Music - Tuesday
(c) According to the question, (c) Play - Wednesday (d) Speech - Monday
Q > T, R > P > S
Þ R> P > S>Q >T 58. A performs on which day of the week ?
\ R scored highest marks. (a) Tuesday (b) Wednesday
(c) Friday (d) Saturday
Directions (Q. Nos. 53-55) Study the following series 59. G performs on which day of the week?
carefully and answer the questions given below. (a) Wednesday (b) Saturday
7 M 4 P % J V 1 K 3 @ E W 2 Q © 6 T A* 8 Z I 5 $ F U (c) Tuesday (d) Friday
# 9 H N
60. Who performs in debate?
53. Which of the following is the sixth to the left of (a) B (b) D
ninteenth from the left end of the above (c) F (d) C
arrangement?
(a) $ (b) T (c) W (d) 2 Sol. (Q. Nos. 56-60) The arrangement is as follows
(c) Sixth element to the left of ninteenth from left Friends Days Performance
= (19 – 6)th element from left = 13th element from left A Tuesday Monologue
=W. The series is
B Thursday Play
7 M 4 P % J V I K 3 @ E W
C Saturday Debate
13th from left D Monday Speech
2 Q © 6 T A * 8 Z 1 5 $ F U#9 HN E Sunday Music
54. How many such consonants are there in the F Wednesday Dance
arrangement, each of which is immediately G Friday Mimicry
preceeded by a symbol and immediately followed by
56. (b) F performs dance.
2 numbers?
(a) four (b) One (c) Two (d) Three 57. (d) D performs speech on Monday.

(*) There is no such consonant in the given series. Hence, 58. (a) A performs on Tuesday.
no option is correct. 59. (d) G performs on Friday.
60. (d) C performs in debate.
55. If all the symbols are dropped from the above
arrangement then which of the following will be Directions (Q. Nos. 61-63) Study the following
twelfth from the right end? information carefully to answer these questions.
(a) Q (b) 6 (c) 2 (d) T
The venn diagram given below is about a small town
(b) After dropping all the symbols, having population of 500 persons. The square represents
7 M 4 P J V 1 K 3 E W 2 Q 6 T A 8 Z I5 F U 9 H N persons from urban area, the circle represents working
12th from right end persons, the triangle represents women and the rectangle
represents educated persons. Number written are number
\ Required element = 6.
of persons.
Directions (Q. Nos. 56-60) Study the following ?
Urban
information carefully to answer these questions. Educated 1 9 10 40
Seven friends A, B, C, D, E, F and G perform in stage 50
shows on a different day from Monday to Sunday not 10 70 10
necessarily in the same order. Each one performs a 10 30
different items viz Music, Speech, Dance, Mimicry, Play, 60
10 Working
Debate and Monologue, not necessarily in the same order.
B performs play on Thursday and E performs Music on
Sunday. G performs mimicry but not on Tuesday or 7 Women
Saturday. C’s performance is on the next day of G’s
performance. D performs on Monday but not the next day 61. What is the number of non-working females ?
of G’s performance. D performs on Monday but not Dance (a) 167 (b) 57 (c) 17 (d) 80
or Debate. A performs monologue which is on the next (c) Non-working females are = 10 + 7 = 17
day of speech. Dance is not performed on Saturday.
10 NTSE~ SOLVED PAPER 2018 (Stage I)

62. If urban population is 350, what is the number of (B) P HO N E S KRQH


+3
non-educated, non-working urban women? +3
(a) 0 (b) 9 (c) 10 (d) 20 +3
+3
(a) There is no non-educated non-working urban women. +3

63. What is the number of urban male who are educated (C) T ROUP E G I L FKV
but not working? Opposite letter
Opposite letter
(a) 30 (b) 40 (c) 50 (d) 110
Opposite letter
(b) The number of educated but not working urban male Opposite letter
= 40 Opposite letter
Opposite letter
64. In the matrix below, the numbers in the cells follow
some rules. Identify the number which when (D) T OU R I S T W LX O L P W
+3
substituted for (?) maintains the same rule
–3
7 12 ? +3
–3
21 27 35 +3
–3
7 14 23
+3

(a) 18 (b) 19 (c) 17 (d) 16


+7 +9
65. (b) J UDGE M XG J H
(b) In 3rd row, 7 ¾ ¾¾® 14 ¾ ¾ ¾® 23 +3
+6 +8 +3
In 2nd row, 21 ¾ ¾ ¾® 27 ¾ ¾¾® 35 +3
+5 +7
In 1 st row, 7 ¾ ¾¾® 12 ¾ ¾¾® 19 +3
+3
\ ? = 19
The code followed pattern B. Hence, option (b) is
Directions (Q. Nos. 65-67) Study the following correct.
information carefully to answer these questions. 66. (a) E MP L O Y A I L HK U
–4
In the table given below, there are two columns, column I –4
and column II. Four words are written in column I. In –4
–4
column II, Equivalent codes are used for these words. For
–4
each of the four words, four different patterns are used. –4
Identify the pattern in the questions given below & choose
The code followed pattern A. Hence, option (a) is
the correct option. correct choice.
Column I Column II 67. (*) J O U R N A L Q L F I MZ Q
Opposite letter
Sr. No Word Code Equivalent Opposite letter
A CHAIR YDWEN Opposite letter
Opposite letter
B PHONE SKRQH
Opposite letter
C TROUPE GILFKV Opposite letter
D TOURIST WLXOLPW +5

The code does not follow any given pattern.


65. If ‘JUDGE’ is coded as ‘MXGJH’ the code pattern
followed is serial number Directions (Q. Nos. 68-70) Study the following
(a) A (b) B (c) C (d) D information carefully to answer these questions.
66. If ‘EMPLOY’ is coded as ‘AILHKU’ the code pattern Eight person A, B, C, D, E, F, G, H are sitting around a
followed is serial number circular table facing the centre. B is sitting second to the
(a) A (b) B (c) C (d) D left of G, who is sitting third to the right of F. Only E is
67. If ‘JOURNAL’ is coded as ‘QLFIMZQ’ the code sitting between A & C. C is sitting third to the left of B.
pattern followed is serial number Only one person is sitting between E & H. Now answer
(a) A (b) B the following questions.
(c) C (d) D 68. Which of the following is the correct order of seating
of persons to the right of A?
Sol. (Q. Nos.65-67) (a) ECHDGBF (b) ECHFBDG (c) EBHDCFG (d) CHBEDGF
(A) C HA I R Y DW E N
–4 69. Who is sitting third to A on its left side?
–4 (a) B (b) H (c) D (d) F
–4
–4 70. Who is sitting exactly in front of A?
–4 (a) B (b) C (c) H (d) F
NTSE~ SOLVED PAPER 2018 (Stage I) 11
Sol. (Q. Nos. 68-70) The sitting arrangement is as follows 74. If Z = 52 and ACT = 48 then BAT is equal to
E
(a) 39 (b) 44
C A (c) 46 (d) 50
(c) As, Z = 52 = 26 (positional value) ´ 2
H G ACT = (1 + 3 + 20) ´ 2
= 24 ´ 2 = 48
F D Similarly, BAT = (2 + 1 + 20) ´ 2
B = 23 ´ 2 = 46
68. (b) The correct order of seating of persons to the right 75. If 20 * 3 = 180 and 4 * 5 = 100 then the value of 7 * 7 is
of A, ECHFBDG.
(a) 21 (b) 49
69. (a) B sits third to the left of A. (c) 343 (d) 7
70. (d) F sits exactly in front of A.
(c) As, 20 * 3 = 20 ´ 32 = 180
71. If % means + @ means -; Ù means ´; Ú means ¸ 4 * 5 = 4 ´ 52 = 100
Then the value of 42 Ù 7 Ú 8 @ 25 % 63 Ù 9 is Similarly, 7 * 7 = 7 ´ 72 = 343
(a) -10 (b) 14
76. How many points will be on the face opposite to the
(c) -20 (d) 30
face which contains two points?
(*) % Þ +, @ Þ -; Ù Þ ´; Ú Þ ¸
42 Ù 7 Ú 8 @ 25 % 63 Ù 9
By replacing signs according to questions,
42 ´ 7 ¸ 8 - 25 + 63 ´ 9
7 (a) 1 (b) 4
= 42 ´ - 25 + 567
8 (c) 5 (d) 6
147 2315 (d) From first and second positions of the dice,
= + 542 = = 578.75
4 4

Common

Opposite

Opposite
First position 5 3 2
\ All options are incorrect.
Second position 5 4 6
72. Arrange the following words in the sequence in
Hence, face contains six points is opposite to face contains
which they occur in the dictionary, then choose the two points.
correct option.
(i) BHAGWAN (ii) BHAGWAT 77. How many minimum line segment required to draw
the given figure?
(iii) BHAGIRATH (iv) BHAGAT
(a) iv, i, iii, ii (b) iv, ii, i, iii (c) iv, iii, ii, i (d) iv, iii, i, ii
(d) As per the dictionary
(iv) BHAGAT ® (iii) BHAGIRATH ® (i) BHAGWAN ®
(ii) BHAGWAT

73. R is the brother of S and M is the father of R, J is


brother of P and P is daughter of S. What is the
relation of P with M? (a) 16 (b) 17
(a) Grand daughter (b) Niece (c) 18 (d) 19
(c) Aunty (d) Sister
(*)
(a) According to the question,
K J L
r
M

A F G
B
Father

E I H
r S
Gr

R Brother
an
dd

Daughter D C
au
gh

Brother
Jr
ter

s Total lines - KL, AB, DC, EH, AD, FE, IJ, GH, BC, FK, GL,
P HC, JC, ED and JD = 15
Hence, minimum 15 line segments are required to draw
\ P is the grand daughter of M. the given figure, which is not given in the options.
12 NTSE~ SOLVED PAPER 2018 (Stage I)

78. A piece of paper is folded as shown in the figure (a) Given, C


and then punched:
B D

A E
F
C
After, 1st move
B D

A E
(1) (2) (3) (4)
F
Choose the correct option from the answer figure C
After 2nd move,
which appears the same when unfolded.
B D
(a) 1 (b) 2 (c) 3 (d) 4
(a) When the paper is unfolded, it is shown as in the A E
option figure (1).
F

After 3rd move, C


B D

79. A mirror is placed vertically as shown in the figure. A E


Choose the correct option for mirror image.
F

S U P E R -609 After 4th move, C


9 B D
(a) 9 0 6 - R E P U S (b) 09 - R E P US
9 9 A E
(c) 9 0 -REP US (d) R E P U S - 0 9
(c) Option (c) is the correct mirror image. F
Hence, after 4th move both the stars will be together
SUPER-609 906-REPUS again. And again after 6th, 8th and 10th move they will be
together.
So, option (a) is correct choice.
80. Each vowel in the word KILOMETER is replaced by
the previous letter in the English alphabet & each 82. Which of the given net from the answer options
consonant is replaced by the next letter in the when folded will results in the given cube?
English alphabet, then the substituted letters are
arranged in alphabetical order, which will be the
fifth from the left end?
(a) D (b) L (c) M (d) N
(c) Given word
K I L O M E T E R
+1 –1 +1 –1 +1 –1 +1 –1 +1
(a) (b)
L H M N N D U D S
ÞL H M N N D U D S
After arranging as per the alphabetical order
D D H L M N N S U

5 th from left end


(c) (d)
81. The black star moves one position at a time
anti-clockwise. The white star moves two positions
at a time clockwise. In now many moves will they be
together again? (b) When option (b) is folded, it will results as in the given
C cube.
B D

A E
F
(a) 4th (b) 6th (c) 8th (d) 10th
NTSE~ SOLVED PAPER 2018 (Stage I) 13
83. Which of the alternatives will complete the figure Directions (Q. Nos. 86 and 87) A net is given which can
be folded into a figure. Choose the correct alternative
which can be made from the net.
86. Question Figure

(i) (ii) (iii) (iv)


Answer Figures
(a) iii (b) i (c) ii (d) iv
(a) Answer figure (iii) will complete the given figure.
Rotate

+ Þ
(a) (b) (c) (d)
(*) No answer figure can be formed by folding the net
given in question figure.
Directions (Q. Nos. 84 and 85) Count the number of
87. Question Figure
cubes in the given figure of each question and choose
correct answer out of four alternative.
84.

Answer Figures

(a) 64 (b) 68 (c) 66 (d) 70 (a) (b) (c) (d)


(b) Number of cubes in lower most cuboid (d) From the given figure,
= 3 ´ 2 ´ 2 + 4 ´ 2 = 12 + 8 = 20 Opposite
Number of cubes in middle cuboid
=4´2 ´2 + 3 ´4 Opposite

= 16 + 12 = 28
Opposite
Number of cubes in upper most cuboid
=3 ´2 ´2 + 2 ´4
= 12 + 8 Answer figure (d) can be formed by folding the net given
in question figure.
= 20
Total number of cubes Directions (Q. Nos. 88 and 89) In each of the following
= 20 + 28 + 20 = 68 questions figure (X) is embedded in any one of the four
alternative figures (a), (b), (c) and (d). Find the alternative
85. The number of squares on a chess board is which contains figure (X) as its part.
(a) 203 (b) 204 (c) 205 (d) 206
(b) The number of squares on a chess board 88.
=8 + 7 + 6 + 5 + 4 + 3 + 2 +1
2 2 2 2 2 2 2 2

Sum of square of numbers


n (2 n + 1) (n + 1)
= (X) (a) (b) (c) (d)
6
8 (2 ´ 8 + 1) (8 + 1) (d) The question figure (X) is embedded in the answer
=
6 figure (d).
8 ´ 17 ´ 9
=
6
= 204
14 NTSE~ SOLVED PAPER 2018 (Stage I)

89. Directions (Q. Nos. 92 and 93) In each of the following


questions, figures A and B are related. Find the figure from
figures (a), (b), (c) and (d). Which has same relationship
(X) (a) (b) (c) (d)
with figure C.

l (b) The question figure (X) is embedded in the answer 92. Question Figures
figure (b).

?
(A) (B) (C) (D)
90. How many rectangles does the following figure
have? . Answer Figures

(a) (b) (c) (d)

l (*)
90°
(a) 10 (b) 12 (c) 13 (d) 14 As, Similarly,
l (d)
B G

A H Here, correct answer figure is not available in the given


N alternatives.
I J

M O 93. Question Figures


Q
L K
F P C ?

E D
. (A) (B) (C) (D)
Total rectangles = FEPM, ABNM, GHON,
CDPO, FNOE, ABOP, GHPM, CDMN, Answer Figures
FEHG, ABCD, INPL, NJKP, IMOJ,
MOKL = 14

91. How many squares are there in the given figure?


(a) (b) (c) (d)

l (d) As, lower outer line is disappeared in figure A to get


figure B. Similarly, lower outer line will be disappeared in
figure C to get figure D. Hence, answer figure (d) is
correct choice.

Directions (Q. Nos. 94-96) In each of the following


(a) 11 (b) 17 (c) 13 (d 16) questions choose the correct water image of figure (X)
l (b) A G B
from the four alternatives (a), (b), (c) and (d).
94.
I N J
R V S
C U O D (X) (a) (b) (c) (d)
M Y w
Q X T l (*)
L P K Real image

E H F
Total squares in the given fgure Water image
~ YURV, ~ YVSW, ~ YWTX, ~ YXQU, ~ QRST, ~ MNOP,
~ YMIN, ~ YNJO, ~ YOKP, ~ YPLM, ~ IJKL, ~ CGDH, Hence, correct water image is not available in the given
~ YCAG, ~ YGBD, ~ YDFH, ~ YHEC, ~ ABFE = 17 options.
NTSE~ SOLVED PAPER 2018 (Stage I) 15
95. l (d) The question figure X is embedded in the answer
figure (d).

(X) (a) (b) (c) (d) +

l (c) Option (c) is the correct water image.

Real image
98.

(X)

Water image Y

Y
96.
(a) (b) (c) (d)

l (a) The question figure X is embedded in the answer


(X) figure (a).

(a) (b) (b) (d)


Directions (Q. Nos. 99 and 100) Select a figure from the
l (*) four alternatives, which placed in the blank space of figure
(X) would complete the pattern.
Real image
99.

Water image
(X) (a) (b) (c) (d)
Here, correct water image is not available in the given l (b) Answer figure (b) will complete the given pattern.
options.

Directions (Q. Nos. 97 and 98) In each of the following


questions, you have figure (X) followed by four alternative
figures (a), (b), (c) and (d) such that figure (X) is embedded
in one of them. Trace out the alternative figure, which
contains figure (X) as its part. 100.
97.
+

(X) (a) (b) (c) (d)


(X)
l (d) Answer figure (d) will complete the given pattern.

+ + +
+

(a) (b) (c) (d)


16 NTSE~ SOLVED PAPER 2018 (Stage I)

Paper II Scholastic Aptitude Test (SAT)


101. A body starts from rest is accelerated uniformly for l (b) Given, refractive index of medium 1 with respect to
30s. If x1 , x2 , x3 are the distances travelled in first medium 2,
10s, next 10s and last 10s, respectively, then 2
2 m1 =
x1 : x2 : x3 is 3
(a) 1 : 2 : 3 (b) 1 : 1 : 1 sin r sin 45°
Q 2 m1 = = (Q r = 45° )
(c) 1 : 3 : 5 (d) 1 : 3 : 9 sin x sin x
l (b) In uniformly accelerated motion, change in velocity of 2 1/ 2
a body in an equal time interval is same. Þ =
3 sin x
Dx
i.e. Dv = = k (constant) … (i) 3 3
Dt Þ sin x = Þ x = sin-1
According to the question, x1 , x2 and x3 are the distances 2 2
travelled by body in equal time interval i.e. 10 s. or x = 60°
Hence, from Eq. (i), we get 104. Which of the following statements is true?
x1 = x2 = x3 = 10 k (a) A convex lens with power +4D has a focal length -0.25 m.
\ x1 : x2 : x3 = 1 : 1 : 1 (b) A convex lens with power -4D has a focal length + 0.25 m
(c) A concave lens with power + 4D has a focal length -0.25 m
102. A bomb of mass 3m kg explodes into two pieces of
(d) A concave lens with power -4 D has a focal length -0.25 m
mass m kg and 2m kg. If the velocity of m kg mass 1
is 16 ms-1 , then the total kinetic energy released in l (d) Q Power of lens, P =
f (focal length) (in m)
the explosion is Focal length of convex lens is positive, hence its power is
(a) 192 mJ (b) 96 mJ always positive.
(c) 384 mJ (d) 768 mJ Focal length of concave lens is negative, hence its power
l (a) According to the question, a bomb of mass 3m kg is always negative.
explodes into two pieces of mass m kg and 2m kg. Hence, option (d) is true.
From momentum conservation law,
105. A constant current I flows in a horizontal wire in the
Þ m1 v1 + m2 v2 = 0
plane of the paper from West to East as shown in the
Where, m1 = m kg, v1 = 16 ms -1 ,
figure. The direction of magnetic field at a point will
m2 = 2 m kg
be South to North
Þ m (16) + 2 m (v2 ) = 0
N
Þ v2 = - 8 m/s
Where negative sign indicates that motion of 2m kg is
opposite to direction of motion of m kg.
W E
Hence, total kinetic energy released in the explosion is,
k = k1 + k2
1 1
Þ k = m1 v12 + m2 v22 S
2 2
I
1 1
= (m)(16) + (2 m) (-8)2
2
(a) directly above the wire
2 2 (b) directly below the wire
= 128 m + 64 m = 192 mJ (c) at a point located in the plane of the paper, on the North
side of the wire
103. Figure shows a ray of light as it travels from
(d) at a point located in the plane of the paper, on the South
medium 1 to medium 2. If reflactive index of
side of the wire
2
medium 1 with respect to medium 2 is , l (b) According to the Maxwell’s right hand rule, the
3 direction of magnetic field at a point will be South to
then the value of angle x is North is directly below the wire.
It states that, if you hold the current carrying straight wire
Medium 2 in the grip of your right hand in such a way that the
45° stretched thumb points in the direction of current, then the
45° direction of the curl of the fingers will give the direction of
the magnetic field. This rule is also called Maxwell’s cork
screw rule.
x=?
Medium 1 106. If the current through a resistor is increased by 50%,
the increase in power dissipated will be
(a) 30° (b) 60° (assume the temperature remains constant)
(c) 15° (d) 45° (a) 225% (b) 200% (c) 250% (d) 125%
NTSE~ SOLVED PAPER 2018 (Stage I) 17
l (d) Power dissipated in a resistor, P = I2 R. 110. For the wave shown in the figure, calculate the
According to the question, current through a resistor is frequency and wavelength of the wave if its speed is
increased by 50% i.e. 320 ms-1 .
50 I
I¢ = I + = 15. I Y
100
\ P¢ = (15. I)2 R
Hence, increase in power dissipated will be,
P¢ - P
= ´ 100
P
X
. )2 I2 R - I2 R
(15 O 20 cm
= ´ 100
I2 R (a) 80 cm, 4000 Hz (b) 8 cm, 400 Hz
= 125% (c) 80 cm, 400 Hz (d) 80 cm, 40 Hz
107. The velocity-time graph of a moving body is shown l (c) Given, velocity of wave, v = 320 ms -1
in the figure. Q Wave velocity, v = Frequency (f) ´ Wavelength (l)
= 320 ms -1
By checking each option,
(a) f = 4000 Hz, l = 80 ´ 10 -2 m
v = 4000 ´ 80 ´ 10 -2 = 3200 ms -1
v
(b) f = 400 Hz, l = 8 ´ 10 -2 m
v = 400 ´ 8 ´ 10 -2 = 32 ms -1
O t (c) f = 400 Hz, l = 80 ´ 10 -2 m
Which of the following statements is true? v = 400 ´ 80 ´ 10 -2 = 320 ms -1
(a) The acceleration is constant and positive (d) f = 40 Hz, l = 80 ´ 10 -2 m
(b) The acceleration is constant and negative v = 40 ´ 80 ´ 10 -2 = 32 ms -1
(c) The acceleration is increasing and positive
Hence, option (c) is true.
(d) The acceleration is decreasing and negative
l (c) Slope of velocity-time graph is positive and increasing 111. If x calories of heat are supplied to 15g of water, its
with time. temperature rises from 20°C to 24°C. If specific heat
Q Acceleration, a =
DV
= tanq
for water is 1 cal g -1 °C, then the value of x is
Dt (a) 30 (b) 120
Hence, acceleration is increasing and it will be positive. (c) 15 (d) 60

108. Which of the following eye defects can be rectified l (d) Given, m = 15g,
using cylindrical lens? DT = (24 - 20) = 4° C
(a) Myopia C = 1 Cal -1 g-1 ° C -1
(b) Presbyopia Heat supplied to water, DQ = x = mCDT
(c) Astigmatism or DQ = 15 ´ 1 ´ 4 = 60 Cal
(d) Hypermetropia
112. In a hydro-power plant
l (c) Astigmatism eye defects can be rectified using (a) kinetic energy possessed by the stored water is converted
cylindrical lens. into potential energy
Near sightedness defect can be corrected by using (b) potential energy possessed by the stored water is
concave lens. Farsightedness defect can be corrected by converted into electricity
using a convex lens. Presbyopia can be corrected by using
bifocal or varifocal lenses. (c) water is converted into steam to produce electricity
(d) heat is extracted from water to produce electricity
109. The linear distance between a consecutive l (b) In a hydro-power plant, potential energy possessed by
compression and a rerefaction in longitudinal wave the stored water is converted into electricity.
is
g 113. The mass of a planet is twice and its radius is three
(a) g (b)
2 times that of the earth. The weight of a body, which
g 3g has a mass of 5 kg. On that planet will be
(c) (d)
4 4 (a) 11.95 N (b) 10.88 N (c) 9.88 N (d) 20.99 N

l (b) The linear distance between a consecutive l (b) Given,


Mass of planet, Mp = 2 ´ Mass of earth (Me )
compression and a rarefaction in longitudinal wave is half
g Radius of planet, Rp = 3 ´ Radius of earth (Re )
of wavelength æç ö÷. While the linear distance between two
è2 ø Hence, gravitational acceleration of the planet,
consecutive compressions or rarefactions is called as GMp
gp =
wavelength of wave (g). Rp2
18 NTSE~ SOLVED PAPER 2018 (Stage I)

G(2 Me ) æ 2 ö GMe 120. Tick the arrangement of metals Fe, Cu, Zn, Ag in
or gp = =ç ÷
(3 Re )2 è 9 ø Re2 the order of decreasing reactivity.
2 é GMe ù (a) Fe > Cu > Zn > Ag (b) Cu > Fe > Zn > Ag
or gp = ge êQ ge = 2 ú (c) Ag > Zn > Fe > Cu (d) Zn > Fe > Cu > Ag
9 ë Re û
Where ge is gravitational acceleration of earth. l (d) The arrangement of metals Fe, Cu, Zn, Ag in the order
of decreasing reactivity is
Hence, weight of body of 5 kg mass,
Zn > Fe > Cu > Ag
Wp = mgp = 5 æç ö÷ ge = 5 æç ö÷ ´ 9.8
2 2
(Q ge = 9.8 ms-2)
è9 ø è9 ø The above order is based on the values of standard
reduction electrode potential series.
Wp = 10.88 N
121. Which metal does not corrode easily?
114. Which of these can be used as olfactory indicator?
(a) Gold (b) Silver (c) Platinum (d) All of these
(a) Vanila (b) Onion
(c) Clove (d) All of these l (d) Metals such as gold (Au), Silver (Ag) and Platinum
(Pt) present at the bottom of the activity series do not
l (d) Vanila, onion and clove, all are natural olfactory corrode easily.
indicators as they have specific smell, which can be used
as indicator. 122. pH is defined as
(a) - log[H3O + ] (b) -log[H2O]
115. What will be the products when acid reacts with
(c) - log[OH- ] (d) - log[H+ ] [OH- ]
metals?
(a) Water and hydrogen gas (b) Acid and hydrogen gas l (a) pH is defined as the negative logarithm of H3 O + ion
(c) Salt and hydrogen gas (d) Base and hydrogen gas concentration (in mole per litre) present in it.
Thus, pH = - log [H3 O + ]
l (c) When acid (HCl and H2 SO4 ) react with active metals It is used to express and compare the acidic and basic
like Na, Mg, Al, Zn, Fe, etc., the products are hydrogen strength of a solution.
gas and the soluble salt of the metal.
e.g. 2Na + H2 SO4 ® Na2 SO4 + H2 ­ 123. A solution turns methyl orange into yellow the
(Sodium) (Hydrogen sulphate) (Salt) (Gas) approximate pH of solution is
116. What happen, when methyl orange solution mixed (a) 1.2 - 2.8 (b) 3.1 - 4.4 (c) 6.0 - 7.6 (d) 8.3 - 10.0
with HCl?
(a) Solution becomes Yellow (b) Solution becomes Red l (c) In a solution becoming less acidic, when methyl
(c) Solution becomes Blue (d) Solution becomes Pink orange moves from red to orange and finally to yellow.
So, the approximate pH of solution is 6.0 - 7.6.
l (b) Methyl orange is an acid-base indicator. When methyl
orange solution mixed with HCl, then solution becomes 124. Zinc reacts with NaOH solution to produce
red colour. The entire colour change occurs in acidic (a) O 2 (b) H2 (c) NH3 (d) NO 2
conditions. In an acid, it is reddish and in alkali, it is
yellow. l (b) Zinc reacts with sodium hydroxide to produce sodium
zincate and hydrogen. This reaction takes place at a
117. Which of these salts will give acidic solution? temperature near 550°C.
(a) Na 2CO 3 (b) NaCl Heat
(c) NH4 Cl (d) COONa Zn + NaOH ¾¾¾® Na2 ZnO2 + H2 ­
(Zinc)(Sodium
hydroxide) (550° C)
(Sodium (Hydrogen)
Zincate)
l (c) A neutral salt can dissolve in water to produce a
neutral solution. NH4 Cl is the salt which will give an
acidic solution when dissolve in water. 125. Aqueous solution of SO2 is
(a) Acidic (b) Basic (c) Neutral (d) Amphoteric
118. Name the metal which offers higher resistance to
l (a) An amphoteric solution is a substance that can
the passage of electricity than copper.
chemically react as either acid or base. However, it is also
(a) Gold (b) Silver possible for an oxide to be neither acidic nor basic. Oxides
(c) Mercury (d) None of these are binary compounds of oxygen with another element,
l (d) The metal that offers higher resistance to the passage e.g. CO2 , SO2 , CaO, CO, ZnO, BaO2 , H2 O, etc.
of electricity as compared to copper is tungsten. Here, SO2 is an acidic solution; as its solution can give H +
So, the answer is none of these. ions.
119. Name two metals both of which are very ductile as 126. Ethane with the molecular formula C2H6 has
well as malleable (a) 6 covalent bond (b) 7 covalent bond
(a) Gold and copper (b) Gold and silver (c) 8 covalent bond (d) 9 covalent bond
(c) Silver and copper (d) None of these
l (b) There are 7 covalent bonds: six C ¾ H bonds and one
l (b) Two metals which both are malleable and ductile are C ¾ C bond present in the molecule of ethane.
gold and silver. Malleable can be defined as those metals H H
which can be extended or can be shaped into any other ½ ½
form. Ductile is known to be those metals which can easily H ¾C ¾C ¾ H
be drawn into wires and undergoes a change without ½ ½
breaking. H H
NTSE~ SOLVED PAPER 2018 (Stage I) 19
127. A flagellum is present at one end of a protozoan. It is l (d) Cytokinin promotes cell division in plants. Abscisic
(a) Planaria (b) Paramecium (c) Hydra (d) Leishmania acid is a growth inhibitor hormone.
l (d) Leishmania is a protozoan which has a flagellum at Gibberellin hormone increases the height of the plant
(stem elongation).
one end of its body. It is a hair like structure, used
primarily as an organelle for locomotion in bacteria and Ethylene gas phytohormone is associated with fruit
protozoa. ripening, senescence and abscission.
Planaria and Paramecium are ciliated organisms. 133. Loop of Henle is found in
(a) lungs (b) liver
128. DNA is not present in
(c) nephron (d) neuron
(a) chloroplast (b) mitochondria
(c) nucleus (d) ribosome l (c) Loop of Henle is found in Nephron. Each kidney is
composed of numerous complex tubular structure called
l (d) Ribosome does not contain DNA. Ribosomes are
nephrons, which are the functional units of kidneys.
mainly composed of ribonucleoproteins and are also
known as protein factories, as they are primarily involved 134. Flight and Fight hormone is
in the synthesis of proteins or polypeptides. (a) adrenaline (b) thyroxine
Chloroplast, mitochondria and nucleus contain their own (c) oxytocin (d) insulin
DNA.
l (a) ‘Flight and Fight’ hormone is adrenaline hormone,
Chloroplasts are responsible for trappping light energy,
essential for the photosynthesis. Mitochondria are because adrenaline hormone is secreted in response to any
essential for aerobic respiration of eukaryotic cells. kind of stress, danger, during emergency situations, from
the adrenal gland.
Nucleus contains the genetic information for controlling
all essential processes related to cell metabolism and 135. In the food chain given below, if the amount of
transmission of genetic material.
energy available at fourth trophic level is 5 kJ, what
129. The wings of house fly and the wings of a sparrow was the energy available at the producer level?
are an example of Grass ® Grasshopper ® Frog ® Snake ® Hawk
(a) analogous organs (b) vestigial organs
(c) respiratory organs (d) homologous organs (a) 5000 kJ (b) 500 kJ
(c) 50 kJ (d) 5 kJ
l (a) The wings of house fly and the wings of a sparrow are
an example of analogous organs, because wings of house l (a) In this food chain energy available at the producer
fly and a sparrow have different origin and structural plan level is 5000 kJ.
but show similar appearance and perform similar function Grass ® Grasshopper ® Frog ® Shake ® Hawk
of flight. The organs, which have different origin and 5000 kJ 500 kJ 50 kJ 5 kJ
structural plan, but show similar appearance and perform T1 T2 T3 T4
similar functions are called analogous organs.
Homologous organs are the opposite of analogous organs, According to Lindeman’s law of trophic efficiency, the
which have similar basic structure and origin but shows efficiency of energy transfer from one trophic level to next
different functions in different organisms. is about 10%.
Vestigial organs are non-functional organs. They are not 136. Jaya and Ratna are varieties of
functional in related animals but were functional in their
ancestors. (a) maize (b) rice
(c) wheat (d) bajra
130. Which of the following is not the purpose of l (b) Jaya and Ratna are varieties of Rice.
transpiration?
(a) Helps in absorption and transportation in plants 137. Which of the following is not an ancient water
(b) Prevents loss of water harvesting structure?
(c) Maintains the shape and structure of plants by keeping the (a) Kattas (b) Sargam
cell turgid (c) Kulhs (d) Surangam
(d) Supplies water for photosynthesis
l (b) Sargam is not an ancient water harvesting structure.
l (c) Maintaining the shape and structure of plants by Kattas (Karnataka), Kulhs (Himachal Pradesh) and
keeping the cell turgid is not the purpose of transpiration. Surangam (Kerala) are ancient water harvesting
Loss of water in the form of vapours, from the aerial parts structures.
of the plant is called transpiration.
138. ATP is formed in a photosynthesising plant cell by
131. Pulmonary vein carries (a) photophosphorylation
(a) deoxygenated blood (b) oxygenated blood (b) oxidative phosphorylation
(c) mixed blood (d) None of these (c) substrate level phosphorylation
l (b) Pulmonary vein carries oxygenated or pure blood from (d) All of the above
lungs to the heart. l (b) ATP is formed by photosynthesising plant cell by
oxidative phosphorylation. Oxidative phosphorylation in
132. Cell division in plants is promoted by
mitochondria in catalysed by a multi-component
(a) abscisic acid (b) gibberellin
enzymatic complex called ATP synthase.
(c) ethylene (d) cytokinin
20 NTSE~ SOLVED PAPER 2018 (Stage I)

139. Breathing rate in human is controlled by l (b) a cos q - b sin q = c


(a) thalamus (b) hypothalamus On squaring the both sides, we get
(c) cerebellum (d) medulla oblongata a2 cos2 q + b2 sin2 q - 2 ab sin q cos q = c2
l (d) Breathing rate in human is controlled by medulla Þ a2 (1 - sin2 q) + b2 (1 - cos2 q)
oblongata. The respiratory centre in medulla regulates the - 2 ab sin q cos q = c2
cycle of inspiration and expiration. It controls involuntary [Q cos2 q = 1 - sin2 q, sin2 q = 1 - cos2 q]
actions and regulates reflex responses like salivation and
Þ a - a sin2 q + b2 - b2 cos2 q
2 2
vomiting.
- 2 ab sin q cos q = c2
140. The number of pairs of nerves which arise from Þ a sin q + b2 cos2 q + 2 ab sin q cos q
2 2

spinal cord is = a2 + b2 - c2
(a) 21 (b) 31
Þ (a sin q + b cos q)2 = a2 + b2 - c2
(c) 41 (d) 51
Þ a sin q + b cos q = ± a2 + b2 - c2
l (b) Total 31 pairs of spinal nerves arise from the spinal
cord. 144. If x 2 - 3 x + 2 is a factor of x 4 - px 2 + q, then the
2 ax - 25 by values of p and q respectively are
141. If a : b = 2 : 3 and x : y = 3 : 4, then is
3 ay + 4 bx (a) -5, 4 (b) -5, - 5 (c) 5, 4 (d) 5, - 4
24 5 -24 12 l (c) x2 - 3 x + 2 = x2 - 2 x - x + 2
(a) (b) (c) (d)
5 24 5 13 = x(x - 2) - 1(x - 2) = (x - 1)(x - 2)
l (c) Given, a : b = 2 : 3 Þ a = 2 m, b = 3 m Polynomial x2 - 3 x + 2 is the factor of polynomial
x4 - px2 + q
and x : y = 3 : 4 Þ x = 3 n, y = 4 n
2 ax - 25 by 2(2 m) (3 n) -25 (3 m) (4 n) \ x2 - 3 x + 2 = 0
\ = (x - 1) (x - 2) = 0
3 ay + 4 bx 3(2 m) (4 n) + 4 (3 m) (3 n)
12 mn - 300 mn Þ x = 1, x = 2 will be zeroes of polynomial x4 - px2 + q.
= Remainder = 0
24 mn + 36 mn
- 288 mn - 24 Put x = 1, (1)4 - p(1)2 + q = 0 Þ1 - p + q = 0 … (i)
= = Put x = 2, (2)4 - p(2)2 + q = 0
60 mn 5
Þ 16 - 4 p + q = 0 … (ii)
142. A square is inscribed in a circle of radius a. Another On subtracting Eq. (ii) from Eq. (i), we get
circle is inscribed in that square and again a square - 15 + 3 p = 0
is inscribed in this circle. The side of this square is 15
a a Þ 3 p = 15 Þ p = =5
(a) 2a (b) (c) (d) a 3
2 2 On putting the value of p in Eq. (i), we get
l (d) Given, radius of external circle = a 1 - 5 + q = 0 Þq = 5 -1 = 4
Diagonal of first in square = Diameter of circle Hence, p = 5, q = 4
\ 2 ´ side of first insquare = 2a
2a 145. If x1 , x2 , x3 , ..., x n are in AP, then the value of
Þ side of first insquare = = 2a
2 1 1 1 1
Diameter of incircle = side of first insquare = 2a + + +K+ is
x1 x2 x2 x3 x3 x4 xn - 1xn
Diagonal of second insquare = Diameter of incircle
n-1 n-1 n n+1
(a) (b) (c) (d)
x1 xn x2 xn - 1 x1 xn x1 xn
l (a) Given, x1 , x2 , x3 , ... , x n are in AP.
\ x2 - x1 = x3 - x2 = K = x n - x n - 1 = d
(common difference)
1 1 1 1
\ + + + ... +
x1 x2 x2 x3 x3 x4 xn - 1 xn

1 é d d d d ù
Þ 2 ´ side = 2 a = ê + + + ... + ú
d ë x1 x2 x2 x3 x3 x4 x n -1 x n úû
2a
Þ side = =a
2 1 é x2 - x1 x - x2 x - x3 x - xn - 1 ù
= ê + 3 + 4 + ... + n ú
Hence, required side of insquare is a. d ë x1 x2 x2 x3 x3 x4 x n -1 x n úû
143. If a cos q - b sin q = c, then a sin q + b cos q = ? 1 é1 1 1 1 1 1 1 1 ù
= ê - + - + - + ... + - ú
(a) ± a2 + b 2 + c 2 (b) ± a2 + b 2 - c 2 d x
ë 1 x2 x2 x3 x3 x4 x n -1 x ú
n û

(c) ± a2 - b 2 + c 2 (d) ± a2 - b 2 - c 2
NTSE~ SOLVED PAPER 2018 (Stage I) 21
1 é1 1 ù 1 é x n - x1 ù and draw AM ^ BE in the figure.
= ê - ú= ê ú 1 1 2
d x
ë 1 x n û d ë x1 x n û \ ar (DABD) = ´ BD ´ AM = ´ BE ´ AM
2 2 3
é x1 + (n - 1)d - x1 ù
1 1
= ê ú [Q Tn = a + (n - 1)d] = ´ BE ´ AM … (i)
d
ë x1 x n û 3
(n - 1) d n - 1 1
= = ar (DAPC) = ´ CP ´ AM
d(x1 x n ) x1 x n 2
1 1 1 1
= ´ BE ´ AM = ´ BE ´ AM
1 1 2 6 4 3
146. If x 2 + y 2 + + = 4, then the value of x 2 + y 2 is 1
x2 y2 = (ar DABD) [from Eq. (i)]
4
(a) 2 (b) 4 (c) 8 (d) 16
1 1 148. If x , y and z are positive real numbers and a, b and c
l (a) x2 + y2 + + = 4
x2 y2 are rational numbers, then value of
1 1
Þ x 2 + 2 - 2 + y2 + 2 - 2 = 0
x y 1 1 1
+ + is
2 1 + x b- a + x c - a 1 + x a- b + x c - b 1 + x b- c + x a- c
æx - 1 ö + æy - 1 ö = 0
2

Þ ç ÷ ç ÷ (a) -1 (b) 1 (c) 0 (d) 2


è xø è yø
1 1 1
On comparing the both sides, we get l (b) + +
1 1 1 + x b - a + x c - a 1 + x a - b + x c - b 1 + x b - c +x a - c
x - = 0 Þ x = Þ x2 = 1
x x 1 1 1
1 1 = + +
and y - = 0 Þ y = Þ y2 = 1 xb xc xa xc xb xa
y y 1+ a + a 1+ b
+ b 1+ c + c
x x x x x x
\ x 2 + y2 = 1 + 1 = 2 xa xb xc
= a + b +
x + xb + xc x + xa + xc xc + xb + xa
147. In the figure, BC = CD = DE and P is mid-point of
xa + xb + xc
CD. The area of DAPC is = =1
xa + xb + xc
A
149. If the height of right circular cylinder is increased
by 10% while radius of base is decreased by 10%,
then curved surface area of cylinder
(a) Remains same (b) Decreases by 1%
(c) Increases by 1% (d) Increases by 0.1%
l (b) Let radius and height of cylinder are r and h
B C P D E respectively.
1 1 \ Curved surface area of cylinder
(a) ar(DABC ) (b) ar(DABD)
3 2 S = 2prh … (i)
1 1
(c) ar(DABC ) (d) ar(DABD) At 10% decreasing, new radius
6 4 100 - 10 9 r
=r´ =
l (d) Given BC = CD = DE 100 10
1 At 10% increasing, new height
\ BC = CD = DE =
BE
3 100 + 10 11h
=h´ =
2 100 10
BD = BC + CD = BE
3 New curved surface area
= 2 p æç ö÷ æç
and P is the mid-point of CD. 9 r 11h ö 99
÷= (2 p rh)
A è10 ø è 10 ø 100
99
= S [from Eq. (i)]
100
\ Decreases per cent in surface area
First area - New area
= ´ 100
First area
99
S- S
B C MP D E = 100 ´ 100
S
1 100 - 99
\ CP = CD = ´ 100 = 1%
2 100
1 1 1
= ´ BE = BE
2 3 6
22 NTSE~ SOLVED PAPER 2018 (Stage I)

150. If a1 , a2 , a3 , .... , an are in AP and a1 = 0, then the value 152. In the given figure, the centre of the circle is A and
æa a a ö æ1 1 1 ö ABCDEF is a regular hexagon of side 6cm. The
of ç 3 + 4 + .... + n ÷- a2 ç + + .... + ÷ is approximate area of segment BPF is (take p = 314 . )
è a2 a3 an - 1 ø è a2 a3 an - 2 ø
equal to
1 1
(a) n + (b) n +
n n-1
A
1 1
(c) (n - 1) + (d) (n - 2 ) +
(n - 1) (n - 2 )
B F
l (d) a1 , a2 , a3 , ... an are in AP and a1 = 0 P
then, an = 0 + (n - 1) d = (n - 1)d
C E
(where, d is common difference of AP)
æa ö æ ö D
\ ç 3 + a4 + ... + an ÷ - a2 Þ ç 1 + 1 + ... + 1 ÷
ça a3 ÷
an -1 ø ç an - 2 ÷ø 2
(b) 22 cm 2
è 2 è a2 a3 (a) 25 cm
(c) 32 cm 2 (d) 30 cm 2
æa a ö æa a ö
=ç 3 - 2÷+ ç 4 - 2÷ l (b) Each side of hexagon = 6 cm
è a2 a2 ø è a3 a3 ø
We know that, each angle of hexagon is120°.
æa a ö æ an - 1 a ö a
+ ç 5 - 2 ÷ + ... + ç - 2 ÷+ n \ ÐBAF = 120° (q)
è a4 a4 ø ça a ÷ a
è n -2 n -2 ø n -1 In DABF, AB = AF (radius of circle)
é d 2d 3d (n - 3)d ù (n -1) d \ Radius of circle = 6 cm (r)
=ê + + + ... + ú+
d 2 d 3 d (n - 3)d û (n - 2) d and ÐABF = ÐAFB = 30°
ë
(n – 1) Draw AM ^ BF.
= [1 + 1 + 1 ... (n - 3) terms] + A
In right angled triangle ABM,
n–2
AM 30° M
n -1 n -1 sin 30° = B F
= (n - 3) + = (n - 2) + -1 AB
n-2 n-2 P
1 AM
n -1 - n + 2 1 Þ =
= (n - 2) + = (n - 2) + 2 6 C E
n-2 n-2 6
Þ AM = = 3 cm
2 D
151. Three circles touch each other externally and all the
BM
three touch a line. If two of them are equal and and cos 30° =
AB
radius of third circle is 4cm, then radius of equal
3 BM
circles is Þ =
(a) 12cm (b) 8cm (c) 16cm (d) 20cm 2 6
Þ BM = 3 3 cm
l (c) Let centres of equal radii circles are O and O¢ and
\ BF = 2 ´ BM [Q AB = AF in D ABF]
centre of 4 cm radius circle is A.
= 2 ´ 3 3 = 6 3 cm
In figure O ¢ P = r, AP = 4 cm, O ¢T = r
\ Area of part BPF = Sector area of ABPF - Area of D ABF
\ O¢ A = r + 4
pr2 q 1
= - ´ BF ´ AM
360° 2
. ´ 36 ´ 120° 1
314
- ´6 3 ´3
O r r O′ 360° 2
r = 37.68 - 15.588 = 22108
. » 22 cm2
Q P 1 1 2
M
4 A 153. If + = , then what is the value of
S R T y +`z z + x x + y
Draw AM ^ O ¢T x2 + y2 ?
(a) 1 (b) -2 z2 (c) 2 z2 (d) y2 + z2
Then,
MT = AR = 4 cm 1 1 2 z+ x+y+ z 2
O ¢ M = O ¢T - MT = r - 4 and AM = r l (c) + = Þ =
y+ z z+ x x+ y (y + z) (z + x) x + y
Now, in DO ¢ AM, O ¢ A2 = O ¢ M2 + AM2
Þ (r + 4)2 = (r - 4)2 + r2 2z + x + y 2
Þ =
Þ r + 16 + 8 r = r2 + 16 - 8 r + r2
2
yz + xy + z2 + xz x + y
Þ r2 - 16 r = 0 Þ 2 yz + 2 xy + 2 z2 + 2 xz
Þ r (r - 16) = 0 Þ r - 16 = 0 = 2 xz + 2 yz + x2 + xy + xy + y2
Þ r = 16 cm Þ 2 z2 = x2 + y2 Þ x2 + y2 = 2 z2
Hence, radius of equal circle is 16 cm.
NTSE~ SOLVED PAPER 2018 (Stage I) 23
154. If x 2 = y + z, y 2 = z + x and z2 = x + y , then what is 157. The minimum value of the expression
1 1 1 3b + 4c 4c + a a + 3b
the value of + + ? + + (a, b, c are +ve)
x +1 y +1 z +1 a 3b 4c
(a) 1 (b) 4 (c) 6 (d) 8
(a) 1 (b) 0 (c) -1 (d) 2
3b + 4c 4c + a a + 3b
l (c) For minimum value of + +
l (a) Given, x2 = y + z, y2 = z + x a 3b 4c
Q a, b, c are positive number.
and z2 = x + y
\ The value of a, b and c are between in 0 and 1
\ x + x=x+ y+ z
2

or will be equal to 1.
Þ x(x + 1) = x + y + z
x+ y+ z i.e. 0 < a, b, c £ 1
Þ (x + 1) = 1 1
x If a = 1, b = , c = [Q find the value positive and integer]
3 4
y2 + y = x + y + z
3b + 4c 4c + a a + 3b
Þ y (y + 1) = x + y + z Then, + +
x+ y+ z a 3b 4c
Þ (y + 1) = 1 1 1 1
z 3´ +4´ 4 ´ +1 1+ 3 ´
= 3 4 + 4 + 3
and z2 + z = x + y + z 1 1 1
3´ 4´
Þ z(z + 1) = x + y + z 3 4
x+ y+ z 1+1 1+1 1+1
Þ (z + 1) = = + + =2 + 2 + 2 =6
z 1 1 1
1 1 1
Now, + +
x +1 y +1 z +1 158. The volume of a cube is numerically equal to sum of
x y z x+ y+ z the length of its edges. The total surface area of
= + + = =1 cube in square units is
x+ y+ z x+ y+ z x+ y+ z x+ y+ z
(a) 12 (b) 36 (c) 72 (d) 144
155. If a, b, g are the roots of the equation x 3 + 4 x + 1 = 0, l (c) Let length of side of cube is a unit.
then (a + b)-1 + (b + g )-1 + ( g + a )-1 is equal to According to the question,
(a) 2 (b) 4 (c) 3 (d) 5 Volume of cube (numerical value)
l (b) Given, a, b, g are the roots of equation x3 + 4 x + 1 = 0. = Sum of length of all sides of cube
Coefficient of x2 Þ a3 = 12 a [Q numbers of side of cube are 12]
\ a+ b + g=
Coefficient of x3 Þ a2 = 12 Þ a = 2 3 unit
Þ a+ b + g =0 … (i) \ Curved surface area of cube = 6 a2 = 6(2 3 )2
ab + bg + ga =
Coefficient of x = 72 square unit
Coefficient of x3
159. The expression 14 m - 6 m will always divisible by
Þ ab + bg + ga = 4 … (ii)
Constant term (a) 8 (b) 20 (c) 14 (d) 6
and abg =
Coefficient of x3 l (a) 14 m - 6 m = (2 ´ 7)m - (2 ´ 3)m (take prime factors)
Þ a,bg = -1 … (iii) = 2 m (7 m - 3 m)
1 1 1 = 21 (71 - 31 )
Now (a + b)-1 + (b + g)-1 + (g + a)-1 = + +
a+ b b + g g + a [put m = 1 or putting minimum positive integer]
1 1 1 =2 ´4 =8
= + + [from Eq. (i)]
- g - a -b Hence,14 m - 6 m, will be divisible by 8.
é ab + bg + ga ù 4 160. PQRS is a square of side 6cm each and T is
=-ê ú = - (- 1)
ë abg û mid-point of QR. What is the radius of circle
[from Eqs. (ii) and (iii)] inscribed in DTSR.
=4 P Q

156. If x , y , z are three positive numbers, then the 3cm


y +z z +x x +y
minimum value of + + is
x y z 6cm T
(a) 1 (b) 2 (c) 3 (d) 6 3cm
l (d) Given x, y, z are three positive numbers.
y+ z z+ x x+ y S R
\ Minimum value of + +
x y z 3 6
(a) (b)
1+1 1+1 1+1 3- 5 3+ 5
= + + =2 + 2 + 2 =6
1 1 1 2
(c) (d) 3 + 5
[Q 1 is the lowest positive number] 3+ 5
24 NTSE~ SOLVED PAPER 2018 (Stage I)

l (b) Given, PQ = QR = RS = PS = 6 cm l (a) Former Secretary General of UNO, Kofi A Annan said
and T is the mid point of QR. that ‘US war on Iraq was not legal’. Kofi A Annan was a
P Q Ghanaian diplomat who served as the 7th Secretary General
of the United Nations from January 1997 to December 2006.
In 2001, he was awarded by Nobel Peace Prize.

T 165. President can declare emergency when


(a) Prime Minister advises him to do so
(b) Parliament advises
(c) The council of ministers, in writing, advises him to do so
S R (d) Home Minister ask him to do so
6
\ QT = TR = = 3 cm l (c) Under Article 352 of the Indian Constitution, President
2 can declare National Emergency on the grounds of war,
Q PQRS is a square. external aggression or armed rebellion.
\ ÐQRS = 90° Such emergency shall be imposed by the President on the
In right angled triangle TSR, basis of written request by the Council of Ministers.
ST = TR2 + SR2 = 32 + 62 = 45 = 3 5 cm
166. KOSOVO was a province of ........ before the split
1 1
Area of D TSR (D) = ´ SR ´ TR = ´ 6 ´ 3 = 9 cm2 (a) Vietnam (b) Zimbabve (c) Sri Lanka (d) Yugoslavia
2 2
l (d) KOSOVO was a province of Yugoslavia before the
and semi-perimeter of split. It is a partially recognised state and disputed
1
DTSR (s) = (ST + RS + TR) territory in South-Eastern Europe.
2
1 1 167. Which of the following state was born out of culture,
= (3 5 + 6 + 3) = (9 + 3 5 ) cm
2 2 ethnicity and geography?
D 9 ´2 6 (a) Kerala (b) Nagaland (c) Mizoram (d) Assam
\ Radius of incirle of DTSR = = = cm
s 9+3 5 3+ 5 l (b) North-East State Nagaland was born out of culture,
ethnicity and geography. Nagaland was carved out from
161. When was the democracy restored in Chile?
the state of Assam by the state of Nagaland Act, 1962.
(a) 1973 (b) 1988 (c) 1957 (d) 1991
Creation of the states can be considered as the first step for
l (b) Democracy was restored in Chile in 1988 when creating a democratic political environment.
Military General Pinochet’s dictatorship came to an end
after a referendum. A democratically elected government 168. ‘End of Racial Discrimination’ is a part of which
was formed as the army’s role in Chilean government was fundamental right?
eradicated. (a) Right to freedom
Chile is a country of South America continent and its (b) Right to equality
capital is Santiago. (c) Right against exploitation
(d) Right to education and culture
162. Which of the following country is not an operational
member of Security Council? l (b) ‘End of Racial Discrimination’ is a part of Right to
equality fundamental right of citizen. Article 17 of the
(a) Russia (b) China (c) Germany (d) America
Indian Constitution abolishes untouchability and forbids
l (c) Germany is not an operational member of Security its practice in any form.
Council. Security Council comprises of five permanent
members namely China, UK, Russia, France and USA, and 169. The movement for the individual and family right of
10 non-permanent members, elected for a term of 2 years women is known as
by a two-third majority of the General Assembly. Germany (a) Mahila Adhikar Aandolan
is one of the non-permanent members of Security Council. (b) Mahila Shakti Aandolan
(c) Narivadi Aandolan
163. Who among the following was not a member of the
(d) Nari Sashaktikaran Aandolan
constituent assembly?
(a) Mahatma Gandhi (b) Jawaharlal Nehru l (c) The movement for the individual and family right of
(c) Dr Rajendra Prasad (d) Dr BR Ambedkar women is known as Narivadi Aandolan. It emphasis on
political dissemination of women’s discussion on
l (a) Mahatma Gandhi was not a member of the constituent reproductive rights, domestic violence, maternity leave,
assembly. Dr Rajendra Prasad was the President of equal pay related rights, sexual harassment,
constituent assembly, Dr BR Ambedkar was the Chairman discrimination and sexual violence.
of Drafting committee and Jawaharlal Nehru was the 170. What is the meaning of ‘Transparency’?
Chairman of several committees in Constitution making. (a) When decision is taken by the ruler
164. Which of the following Secretary General said that (b) When decision are made through leader’s conclusion
‘‘US war on Iraq was not legal’’? (c) When decision are made for individual greeds
(a) Kofi A Annan (b) B B Ghali (d) When decision are taken with honesty and proper follow of
(c) U Thant (d) Ban Ki Moon rules
NTSE~ SOLVED PAPER 2018 (Stage I) 25
l (d) In politics, transparency is used as a means of holding 177. When was Simon Commission arrived in India?
public officials accountable and fighting against (a) 1928 (b) 1930 (c) 1931 (d) 1932
corruption. Therefore, the meaning of transparency is
‘Taking the decisions with honesty and properly follow the l (a) Simon Commission was a group of seven British
rules’. Members of Parliament.
The commission under the chairmanship of Sir John
171. The International organisation that works for human Simon arrived in India in 1928 to study the constitution
rights is reform. Due to the absence of any Indian member in it, the
(a) Amety International (b) Amnesty International commission was boycotted by the Indian National
(c) Asnesty International (d) Afnesty International Congress and most other Indian political parties.
l (b) Amnesty International is the international organisation 178. ‘Rinderpest’ is a term used for
that works for human right. It was founded in 1961 in (a) a cattle disease
London. This organisation was awarded the Nobel Peace (b) missing of cattle
Prize in 1977 and United Nations Prize in the field of (c) indentured labourer
Human Rights in 1978. (d) mass production in a factory
172. What was Livre? l (a) ‘Rinderpest’ is a term used for cattle disease. This
(a) Currency of France (b) Newspaper of France disease is characterised by fever, oral erosions, diarrhoea,
(c) Magazine of France (d) Flag of France lymphoid necrosis and high mortality.
l (a) Livre was the currency of France from 781 to 1794. 179. Giuseppe Garibaldi was a famous freedom fighter of
Presently, Euro is the official currency of France. (a) Germany (b) Poland
(c) Ireland (d) Italy
173. Who granted sole right to trade with East to East
India company? l (d) Giuseppe Garibaldi was a famous freedom fighter of
(a) James I (b) James II Italy. He contributed to the Italian unification and the
(c) Elizabeth I (d) Elizabeth II creation of the kingdom of Italy.
l (c) Elizabeth I granted sole right to trade with East to East 180. Gudem Rebellion was led by
India Company by a Royal Charter on 31st December, (a) Baba Ramchandra (b) Jawaharlal Nehru
1600. Initially, the focus of the company was trade not the (c) Alluri Sitaram Raju (d) Mahatma Gandhi
building of an empire in India. l (c) Gudem Rebellion was led by Alluri Sitaram Raju.
174. In which Congress session, Non-cooperation Gudem Rebellion spread in the hills of Andhra Pradesh in
response to the Non-cooperation movement in 1921.
programme was adopted? The methods adopted by the tribals to attain Swaraj was by
(a) Ahmedabad 1921 (b) Kolkata 1917 the means of violence and force.
(c) Amritsar 1919 (d) Nagpur 1920
181. ‘The Social Contract’ book was written by
l (d) On 4th September, 1920 Congress met in Calcutta in a
(a) Dantey (b) Rousseau
special session in which Gandhi Ji proposed
Non-cooperation resolution which was opposed by CR (c) Petrarek (d) Napolean
Das. In December 1920, the Nagpur session of the l (b) ‘The Social Contract’ book was written by Rousseau.
Congress confirmed the resolution of starting Rousseau was a Genevan philospher, writer and composer.
Non-cooperation movement. This movement envisaged In this book, he theorised about the best way to establish a
boycott of school, colleges, law courts, foreign cloths etc. political community in the face of problems of commercial
society.
175. The first Modern Novel published in Malayalam in
the year 1889 was 182. The principle of the ‘Garden City’ was developed by
(a) Indulekha (b) Rajasekhara Caritamu (a) Raymond Unwin (b) Barry Parker
(c) Manju Ghose (d) Pariksha Guru (c) Ebenezar Howard (d) Herbert Baker
l (a) The first Modern Novel published in Malayalam in the l (c) The principle of the ‘Garden City’ was developed by
year 1889 was ‘Indulekha’. It was written by Oyyarathu architect Ebenezar Howard to solve the problem of
Chandu Menon. It is a depiction of Nair society in the late growing congestion and pollution of the city of London.
19th century with a love story at its core.
183. Which of the following organisation looks after the
176. The painting ‘Damayanti’ was made by credit needs of agriculture and rural development in
(a) Abindra Nath Tagore India?
(b) William Jones (a) FCI (b) IDBI
(c) Raja Ravi Verma (c) NABARD (d) SBI
(d) Ravindra Nath Tagore l (c) NABARD is an apex development financial institution
l (c) The painting ‘Damayanti’ was made by Raja Ravi in India that looks after the credit needs of agriculture and
Verma. He was a Malayali painter and artist. He was rural development in India. It was established on 12th
closely related to the royal family of Travancore in Kerala July, 1982 with its headquarter located at Mumbai.
state in India.
26 NTSE~ SOLVED PAPER 2018 (Stage I)

184. How many phases are there in circular flow of 190. What per cent of the total surface area of India is
income? covered by mountains?
(a) 2 (b) 3 (c) 6 (d) 5 (a) 33% (b) 35%
l (b) There are three different phases in circular flow of (c) 30% (d) 25%
income. These are l (c) The total surface area of India is covered by mountains
l Production Phase (Generation of Income) is 30%, rest 43% part is covered by plains and 27% part is
l Income Phase (Distribution of Income) covered by plateau.
l Expenditure Phase (Disposition of Income)
191. Which mineral has excellent dielectric strength,
185. Which of the following is considered as social insulating properties, low power loss factor and
infrastructure ? resistance to high voltage?
(a) Transport (b) Education (a) Aluminium (b) Lime stone
(c) Energy (d) Communication (c) Copper (d) Mica
l (b) Education is considered as social infrastructure. Social l (d) Mica has excellent dielectric strength, insulating
infrastructure is the interdependent mix of facilities, properties, low power loss factor and resistance to high
places, spaces, programs, projects, services and networks voltage.
that maintain and improve the standard of living and It is used in electricity tools like transformer. India is a
quality of life in a community. leading producer of mica.
186. Multiple cropping refers to 192. Which of the following is an example of joint sector
(a) Cultivation of wheat and rice industry?
(b) Cultivation of two crops in alternate rows (a) BHEL (b) OIL
(c) Cultivating more than one crop on the same field in a year
(c) SAIL (d) TISCO
(d) Cultivating crops and rearing animals simultaneously
l (b) Oil India Limited (OIL) is an example of joint sector
l (c) Multiple cropping refers to cultivating more than one industry. It is the second largest hydrocarbon exploration
crop on the same field in a year. With multiple cropping
and production Indian public sector company with its
the risk of total loss from drought, pests and diseases is
operational headquarters in Duliajan, Assam. It was
reduced.
founded on 18th February, 1959.
187. Infant mortality rate refers to the death of child 193. Which mode of transport reduces transshipment
under the age of losses and delays?
(a) 1 year (b) 2 year (a) Railways (b) Roadways
(c) 3 year (d) 4 year (c) Waterways (d) Pipelines
l (a) Infant Mortality Rate (IMR) refers to the death of child l (d) Pipeline is the mode of transport that helps in reducing
under the age of one year. IMR is the number of deaths transshipment losses and delays. The pipeline mode is
per 1000 live birth of children under one year of age. It is used for transporting liquids and gases.
an important indicator of the overall physical health of a
community. 194. Which of the following lake lies on the Equator?
(a) Lake Victoria (b) Lake Malavi
188. In which year was the Integrated Child (c) Lake Nasser (d) None of these
Development Service (ICDS) introduced?
l (a) Lake Victoria lies on the Equator. With a surface area
(a) 1965 (b) 1975
of 68,800 sq km, lake Victoria is the Africa’s largest lake.
(c) 1985 (d) 1995
l (b) Introduced in 1975, Integrated Child Development 195. The longitudinal valleys lying between Lesser
Service (ICDS) is a government programme which Himalayas and Shivaliks are known as
provides food, pre-school education, primary healthcare, (a) Valleys (b) Coast (c) Passes (d) Duns
immunisation refferal services to children under 6 years of l (d) The longitudinal valleys lying between lesser Himalayas
age and their mothers. and Shivaliks are known as Duns. Dehra Dun, Kotli Dun,
189. The first Chairman of Planning Commission was Patli Dun are the well known examples of Duns.
(a) Indira Gandhi 196. In winters, the western cyclonic disturbances
(b) Dr Rajendra Prashad originate from which sea?
(c) Jawaharlal Nehru (a) Caspian sea
(d) Vallabhbhai Patel (b) Black sea
l (c) The first Chairman of Planning Commission was (c) Mediterranean sea
Jawaharlal Nehru. Planning Commission was an extra (d) Baltic sea
constitutional body, charged with the responsibility of
l (c) In winters, the western cyclonic disturbances originate
making assessment of all resources of the country.
from mediterranean sea and enter India after crossing over
Now, it has been replaced by new institution named NITI
Iraq, Iran, Afghanistan and Pakistan under the influence
Aayog on 1st January 2015.
of westerly jet stream.
NTSE~ SOLVED PAPER 2018 (Stage I) 27
197. Balancing the need to use resources and also 199. According to the ‘Theory of Plate Tectonics’, when
conserve them for future is called some plates come towards each other which one of
(a) Resource development the following is formed?
(b) Resource conservation (a) Convergent boundary
(c) Sustainable development (b) Divergent boundary
(d) Human resource development (c) Transform boundary
l (c) Balancing the need to use resources and also conserve (d) None of the above
them for future is called sustainable development. l (a) According to the ‘Theory of Plate Tectonics’, when
some plates come towards each other, then convergent
198. Which among the following has the maximum boundary is formed. While, when the plates move apart
number of National Parks? with the upwelling of material from the mantle, divergent
(a) Andaman and Nicobar Island plate boundary is formed.
(b) Arunachal Pradesh
(c) Assam 200. The largest producer of cotton in the world is
(d) Meghalaya (a) India
(b) China
l (a) With a total of 9 National Parks, Andaman and Nicobar
(c) Brazil
Island has the maximum number of National Parks. These
are: Campbell National Park, Galathea National Park, (d) USA
Mahatma Gandhi Marine National Park, Middle Button l (b) The largest producer of cotton in the world in China
Island National Park, Mount Harriet National Park, North followed by India and United States of America.
Button Island National Park, Rani Jhansi Marine National China produced 6532 thousand metric tonnes while India
Park, Saddle Peak National Park and South Button Island produced 6423 thousand metric tonnes of the cotton in
National Park. 2018.
NTSE~ SOLVED PAPER 2018 (Stage II) 1

NTSE Solved Paper


NATIONAL TALENT
SEARCH
EXAMINATION
2018 (Stage II)*

INSTRUCTIONS
This solved paper consists of two papers. Paper I consists of Mental Ability Test (MAT) and Paper II consists of
Scholastic Aptitude Test (SAT).
MAT covering (Q. Nos. 1-50) of Maths and (Q. Nos. 51-100) of Reasoning.
SAT consists of (Science, Mathematics and Social Science) which comprises 100 questions (40 Science, 20 Mathematics
and 40 Social Science).
There will be no negative marking.
Each correct answer will be awarded one mark.

Time : 240 Minutes Max. Mark : 200

Paper I Mental Ability Test (MAT)


1. Which figure completes the statement? I. Mehar has a goat.
II. Ravneet has a goat.
III. Mehar is lying.
Codes
(a) Only II (b) I and II (c) I, II and III (d) II and III
is to
l (c) From fact (3) i.e. Ravneet always tells the truth, it is
clear that Ravneet and Mehar both have goats and Mehar
is lying.
?
3. Look at the patterns in the squares and understand
is to their relationship to one another so as to fill in the
square missing symbols.

(a) (b) (c) (d)

l (a) Option figure (a) will come in place of question mark.


?
2. Fact 1: Ravneet said, “Mehar and I both have
goats.’’
Fact 2: Mehar said, “I don’t have a goat.”
Fact 3: Ravneet always tells the truth, but Mehar
sometimes lies.
If the three statements are facts, which of the
following statements must also be facts? (a) (b) (c) (d)

*Exam held on 13th May 2018


2 NTSE~ SOLVED PAPER 2018 (Stage II)

l (a) In first row, the rectangle is moving 1 step in clockwise l (d) Here, the pattern is,
direction and the circle is moving 2 steps in anti-clockwise (3)2 + (4)2 + (2)2 + (2)2 - (3 + 4 + 2 + 2)
direction. The arrow is moving 1 step in anti-clockwise = (9 + 16 + 4 + 4) - (3 + 4 + 2 + 2)
direction and at the same time it is rotating 45° clock-wise. = 33 - 11
In second row, the arrow remains at its position, the = 22
rectangle is moving 1 step in clockwise direction and the
circle is moving 2 steps in clockwise direction. (4)2 + (3)2 + (2)2 + (5)2 - (4 + 3 + 2 + 5)
In third row, the figures are following the same pattern as = 16 + 9 + 4 + 25 - (4 + 3 + 2 + 5)
of row 1. = 54 - 14
Hence, option figure (a) will come in place of question = 40
mark. Similarly,
4. Danish starts walking straight towards East. After (5)2 + (4)2 + (3)2 + (6)2 - (5 + 4 + 3 + 6)
walking 75 m, he turns to the left and walks 25 m = (25 + 16 + 9 + 36) - (5 + 4 + 3 + 6)
straight. Again he turns to the left, walks a distance = 86 - 18 = 68
of 40 m straight, again he turns to the left and walks
a distance of 25 m. Directions (Q. Nos. 7-9) Answer these questions by using
the following diagram.
How far is he from the starting point?
C
(a) 30 m (b) 35 m (c) 40 m (d) 50 m D 1

l (b) According to the given information, 12 2


D 40 m C 3
N 4 5 8 9

25 m 25 m W E
Final 7 10 11
Position
A B
E B S
Starting Point
75 m
Required distance = AE = AB - BE = AB - DC
= 75 - 40 = 35 m A

5. ln the question given below, there are four Each square stands for different class.
statements which are to be taken as truth even if A. represents Indians
they do not seem to be so. There are conclusions B. represents Students
numbered I, II, III and IV. Decide which of these C. represents Talented individuals
logically follow from the given statements. D. represents Players
All students who like English also like Mathematics.
7. How many Indian non-player students who are
Some students like Hindi. All students who like
talented?
Hindi do not like Mathematics. Students who like
(a) 5 (b) 7 (c)10 (d) 12
Mathematics also like English.
I. Students who like Hindi also like English. 8. How many talented Indians are players?
II. Students who like Mathematics also like Hindi. (a) 13 (b) 17 (c) 19 (d) 22
Ill. Students who like Mathematics do not like Hindi. 9. How many talented Indians are there, who are
IV. Students who like English do not like Hindi. students?
Codes (a) 13 (b) 15 (c) 17 (d) 19
(a) I and II (b) I and III (c) I and IV (d) III and IV l Sol. (Q. Nos. 7-9)
l (d) From the given information, i.e. all students who like 1
C (Talented Individuals)
English also like Mathematics and all students who like D (Players)
Hindi do not like Mathematics, we can say that students 12 2
who like English do not like Hindi. 3
Hence, both conclusions III and IV are correct. 4 5 8 9

6. The number in the place of ‘?’ should be 7 10 11


2 3 4
B (Students)
A (Indians)
3 22 2 4 40 2 5 ? 3

7. (c) 10 Indian non-player students are talented.


4 5 6 8. (b) Talented Indians who are players = 8 + 9 =17
(a) 42 (b) 58 (c) 59 (d) 68 9. (d) Talented Indians who are students = 9 + 10 = 19
NTSE~ SOLVED PAPER 2018 (Stage II) 3
Directions (Q. Nos. 10-11) Study the following diagram. On putting values of Eq (i) in Eq (ii)
A B Þ (58 - y) + y = 2 y + 44
2 y = 58 - 44
2 y = 14 (from Eq (i))
15 y x y =7
On putting value of y in Eq (i)
5
10 11 x = 58 - 7 Þ x = 51

12. In the question given below, some arguments is/are


25 logical and others are not. Identify the logical
argument(s).
C I. Eating lots of vegetables and fruits increases
A : Representing people who read newspaper A immunity. I eat lots of vegetables and fruits, so
my immunity is high.
B : Representing people who read newspaper B
II. Eating lots of vegetables and fruits increases
C : Representing people who read newspaper C
immunity. I do not eat vegetables and fruits, so
10. If the number of people in B is 10 more than A, what my immunity is low.
is the total number of people in only B (i.e. in B but III. Eating lots of vegetables and fruits increases
not in A or C)? immunity. My immunity is low, which means I
(a) 14 (b) 24 (c) 30 (d) 36 don’t eat fruits and vegetables.
(a) Only I (b) I and II (c) I and III (d) II and III
11. If sum of the number of people in only B and the
number of people common in both A and B is 63 l (c) Here, arguments I and III are logical.
and the number of people in B is twice the number 13. Consider the following figure
of people in A, then the values of x and y are
respectively
(a) 15, 4 (b) 48, 5 (c) 51, 7 (d) 51,8
l Sol. (Q. Nos. 10-11)

A (People who B (People who


15 y x
read newspaper A) read newspaper B) ?
5
10 11
Which of the following alternatives should replace
the question marks?
25 C (People who
read newspaper C)

10. (b) Number of people who read newspaper A (a) (b)

= 15 + 10 + 5 + y = 30 + y
and number of people who read newspaper B
= y + x + 11 + 5
= 16 + x + y
According to the given information,
(c) (d)
Þ x + y + 16 = (30 + y) + 10
Þ x + y + 16 = 40 + y
Þ x = 40 - 16
\ x = 24 l (d) After studying figure, it can be concluded that option
(d) will replace the question mark because in all the rows,
or number of people who read only newspaper B = 24
there are four trees and five fruits.
11. (c) Number of people reading only newspaper B = x
Number of people reading both A and B newspaper 14. Find out the water image of
=5 + y AVPU7436
A V PU 7 4 3 6 A V PU 3 6
According to the question, (a) (b) 7 4
x + y + 5 = 63 A V 4 3 6 A V 4 3 6
(c) PU7 (d) PU 7
Þ x + y = 63 - 5
A V P U 7 4 3 6
Þ x = 58 - y …(i) l (a) Here, A V P U 7 4 3 6
Also, 16 + x + y = 2(30 + y)
Þ x + y = 2 y + 44 …(ii) Hence, option figure (a) is the correct answer.
4 NTSE~ SOLVED PAPER 2018 (Stage II)

15. A man goes on trek from the bottom to the top of a 19. Which of the following alternatives will fit in the
mountain. He starts at 6 am of 15th October, 2017 place of ‘?’ ?
from the bottom and reaches the top at 6 pm of the AZ, GT, MN,? , YB
same day. On 16th October, 2017 he starts from the (a) KF (b) RX
top at 6 am and goes back following exactly the (c) SH (d) TS
same route and reaches the bottom at 6 pm. Based l (c) The pattern of the series is
on the above situation, the following possibilities –6 –6 –6 –6
are to be analysed.
I. It is not possible to find a point on the route which AZ GT MN SH YB
he will cross at the same time each day.
+6 +6 +6 +6
II. It is possible to find a point on the route which he
will cross at the same time each day provided 20. Look at this series
only if he travels on each day with equal uniform J14, L16, _____, P20, R22
speed.
Which of the following alternatives will fit in the
III. It is always possible to find a point on the route blank space?
which he will cross at the same time each day (a) N18 (b) S24
irrespective of his speed of travel. (c) M18 (d) T24
(a) Only I is true (b) Only II is true
(c) Only III is true (d) Both I and II are true
l (a) The pattern of the series is
+2 +2 +2 +2
l (b) Here only possibility II is true.
L 16 P R
16. At noon and at midnight the long and short hands of J 14 N 18 20 22
a clock are together. Between noon and midnight, +2 +2 +2 +2
how many times the long hand overtakes the short
hand? 21. What will be the missing term in the given
(a) 9 (b) 10 sequence?
(c) 11 (d) 12 ACC, ____, CEO, DFX
l (c) The minute hand of a clock will overtake the hour hand (a) BDD (b) BDE
of the clock 11 times in 12 hours. (c) BDH (d) BED
17. If MENTAL: SMXFOB, then ABILITY: ______ l (c) The pattern of the series is
(a) GJSXWJQ (b) GSXWJJQ +1 +1 +1
A B C D
(b) SGXWJJQ (d) SJXQJWG
+1 +1 +1
C D E F
l (a) As,
M E N T A L +5 +7 +9
C H O X

+6 +8 +10 +12 +14 +16


22. Which number comes in place of ‘?’ ?
S M X F O B 64, 57, 66, 55, ? 52
Similarly, (a) 68 (b) 69
A B I L I T Y (c) 70 (d) 71

+6 +8 +10 +12 +14 +16 +18


l (b) The pattern of the series is
–2 –3
G J S X W J Q
64 57 66 55 69 52
18. As JAISALMER is to JAILSARME as HYDERABAD
is to +2 +3
(a) HYDAERDBA (b) HYDRBEDAA
23. Select the suitable figure from the given alternatives
(c) HYDBDREAA (d) HYDEADRAB
to complete the figure.
l (a) As, J A I S A L M E R

J A I L S A R M E

Similarly,
H Y D E R A B A D
?

H Y D A E R D B A
NTSE~ SOLVED PAPER 2018 (Stage II) 5
27. When a square shaped transparent sheet with the
pattern shown in the figure is folded along the
dotted line which pattern would appear?

(a) (b) (c) (d)

l (a) Here, in each row the number of lines are decreasing


and the pin head is rotating 90° in clockwise direction.
Hence, option figure (a) will come in place of question
mark.

24. Arrange the given words in a meaningful sequence


and find the correct sequence from the given
options: (a) (b) (c) (d)
A. Wall B. Clay
l (a) The piece of sheet will appear like the figure given in
C. House D. Room option (a).
E. Bricks
(a) E, B, A, D, C (b) B, E, D, A, C 28. Fact 1 : All monkeys like to jump.
(c) B, E, A, D, C (d) A, B, C, D, E Fact 2 : Some monkeys like to swim.
l (c) The correct sequence is, Fact 3 : Some monkeys look like their masters.
Clay ® Bricks ® Wall ® Room ® House If the first three statements are facts which of the
i.e. B, E, A, D, C following statements must also be a fact(s)? .
I. All monkeys who like to swim look like their
25. Identify the figure the complete the pattern.
masters.
II. Monkeys who like to swim also like to jump.
III. Monkeys who like to jump do not look like their
masters.
Codes
? (a) Only I (b) Only II
(c) II and lll (d) I and II
l (b) According to the given facts, all monkeys like to jump
and some monkeys like to swim.

(a) (b) (c) (d) Masters Monkey Swim

l (c) Option figure (c) will complete the pattern of the given
figure.
Jump
26. Replace ‘?’ by the appropriate figure from the given
options. So, it is clear that those monkeys who like to swim also
like to jump. Hence, only statement II can be a fact.

? 29. Given below is a statement followed by two


assumptions.
Statement The population below poverty line has
increased in urban area during the last year.
Assumptions
I. The population below poverty line has decreased
in rural area.
(a) (b) (c) (d)
II. The population below poverty line has not
l (c) Here, the position of part of a figure is changing in increased during the current year.
each step and a new figure comes after every two steps. Which of the assumptions is implicit in the statement?
So, following the same pattern, option figure (c) will
(a) Only I is implicit (b) Only II is implicit
replace the question mark.
(c) Either I or II is implicit (d) Neither I nor II is implicit
6 NTSE~ SOLVED PAPER 2018 (Stage II)

l (d) No information regarding the population below crossings is called a block. Starting from a crossing, if
poverty line in rural areas is given in the statement. I travel four blocks North, take left and then travel
Hence, statement I is not implicit. three blocks West, I reach another crossing. What is
Also, no information regarding the increase in population the distance between these two crossings?
during current year is given.
Hence, statement II is also not implicit.

30. Identify the conclusion(s) which logically follow(s)


from the given statements: 1 block
A. Some men are educated.
B. Educated men prefer small families.
Conclusions
I. All small families are educated. (a) 5 km (b) 7 km
II. Some men prefer small families. (c) 7/8 km (d) 5/8 km
1 1 1 1 4 1
Codes l (d) AB = 4 blocks = + + + = km = km,
(a) Only conclusion I follows (b) Only conclusion II follows 8 8 8 8 8 2
1 1 1 3
(c) Both I and II follow (d) Neither I nor ll follows BC = 3 blocks = + + = km
8 8 8 8
l (b) As, some men are educated and educated men prefer C B
small families. Hence, we can say that some men prefer 1/8 1/8 1/8
small families. While conclusion I, i.e. small families are
1/8
educated is doubtful.

31. A watch is showing right time at 9 pm. This watch 1/8


gains 10 minutes in every 24 hours. What will be the
time shown next day by the watch when the correct 1/8
time is 2 am?
(a) 02:00:24 am (b) 2:00:48 am 1/8
(c) 02:02.05 am (d) 02:02:30 am A
l (c) The watch shows the correct time at 9 pm. 2 2
Total time since 9 pm to 2 am = 5 hours \ Required distance = æ1 ö + æ3 ö
ç ÷ ç ÷
è2 ø è8 ø
Q The clock gains 10 min in 24 hours.
10 1 9
\ Time gained by clock in 5 hours = ´ 5 = 2 min 05 sec = +
24 4 64
Hence, the clock will show 02 : 02 : 05 am at 2 am. 25 5
= = km
64 8
32. In a school, students are offered subjects in such a
manner that they have to choose at least one subject 34. How many minimum right turns will you take to
from History and Geography. Accordingly: reach school from home?
All students who study History also study Geography Home
logically implies:
I. There are no students who study Geography and
do not study History.
II. There are no students who study History and do
not study Geography.
III. There are no students who do not study History
and do not study Geography. School
(a) 7 (b) 8 (c) 9 (d) 10
IV. All students who do not study Geography are
students who study History. l (c) Home
(a) I and II (b) I and III
(c) Il and III (d) II and IV
l (c) Statement II and III are true with respect to the
statements given in question.

33. In a city, all the roads are either parallel to the


East-West or North-South direction.
School
1
Every th of a kilometre from each road there is a Here, circle represents the point where I will take right
8 turn.
crossing and the square area covered between four Hence, minimum number of right turns = 9
NTSE~ SOLVED PAPER 2018 (Stage II) 7
35. The figure shows the Gender, Marital Status and 38. Which region contains exactly eight integers?
Profession (GMP) of a number of people. Each (a) P (b) Q
shape shows a different GMP. (c) R (d) S
Sol. (Q.Nos. 36-38)
Integers in I = {2 , 4, 6, 8, 10, 12, 14, 16, 18, 20, 22, 24, 26}
Integers in II = {1, 4, 9, 16, 25}
Integers in III = {2, 3, 5, 7, 11, 13,17, 19, 23}
Integers in W = {15, 21}
I I II = {4, 16}
I I III = {2, 3}
II I III = f
Circle indicates total population, trapezium is males, S + V = {4,16}
pentagon is married, rectangle is teachers, and
U + V = {2}
triangle is doctors.
V = f,
What do the shaded regions represent in the S = {4, 16}
diagram?
u = {2}
(a) married males who are teachers.
T=f
(b) unmarried males who are doctors.
(c) unmarried females who are doctors. Integers in P = {8, 10, 12, 14, 18, 20, 22, 24, 26}
(d) married males who are neither doctors nor teachers. Integers in Q = {1, 9, 25}
Integers in R = {3, 5, 7, 11, 13, 17, 19, 23}
l (a)
Total Population
36. (c) Region S and W contains exactly two integers.
Males 37. (a) Number of integers in S = 2
Teachers Number of integers in R = 8
Married and number of integers in P = 10
38. (c) Region R contains exactly 8 integers.

39. In a school, commerce and arts subjects were


Doctor offered. Some students opted only for commerce
Here, shaded region represent the married males who are and some only arts. There were science students
teachers. also who did not choose any of these subjects. The
rest of them accepted both commerce and arts.
Directions (Q.Nos. 36-38) Which one of the following Venn diagram correctly
l Each of integers 1 to 26 is represented in the Venn diagram in reflects this situation?
the appropriate regions P to W where region K represents
integers from 1 to 26.
l I represents even integers from 1 to 26.
l II represents perfect square integers from 1 to 26.
l III represents prime numbers from 1 to 26.
(a) (b) (c) (d)
l W represents numbers from 1 to 26 other than those in I, II
and III. l (a) According to the given information,
K Total students
II I

Q S P Commerce
students Arts students

V
T U
Science students

R 40. A person walked 100 m straight from the point ‘A’ in


W
III the North-East direction, walked 200 m in South-
West direction from there, 100 m in North-East
direction again, walked 100 m eastward, 200 m
36. Which region contains exactly two integers?
southward and 100 m westward to reach at the point
(a) P and W (b) R and U (c) S and W (d) U and W
‘B’. Choose the right answer from the following to
37. The total number of integers in S and R is equal to find out his/her distance and direction from ‘A’.
the number of integers in (a) 100 m, North (b) 100 m, South
(a) P (b) T (c) V (d) W (c) 200 m, North (d) 200 m, South
8 NTSE~ SOLVED PAPER 2018 (Stage II)

l (d) According to the given information, l (d) Let the cost of 1 pen be `x.
C and cost of 1 pencil be ` y.
From statement I,
100 m E
6 x + 5 y = 30 …(i)
A
Starting point
N From statement II,
N-W N-E
12 æç
60 ö æ 60 ö y = 36
200 m ÷ x + 10 ç ÷
W E è100 ø è100 ø
D S-W S-E Þ 36 x + 30 y = 180
Final point S
100 m F
Þ 6 x + 5 y = 30 …(ii)
B
From both the statements, we get the same equation
Required distance = AB = EF (Q AE = BF)
Hence, we cannot find the cost of pen using both
= 200 m statements.
Hence, the person is 200 m away towards South from his
starting point. 43. What is the ratio of savings of A and B?
Directions (Q.Nos. 41-44) In the following question, each I. The ratio of income of A and B is 5 : 6.
contains two statements I and II, giving certain data. Select II. The ratio of expenditure of A and B is 3 : 4.
the correct answer from (1) to (4) depending on the (a) (1) (b) (2)
sufficiency of data given in the statements to answer each (c) (3) (d) (4)
question. l (d) From statement I,
(1) If I alone is sufficient and II alone is not sufficient Income of A = 5x
to answer the question. Income of B = 6x
(2) If II alone is sufficient and I alone is not sufficient From statement II,
to answer the question. Expenditure of A =3y
(3) If both I and II together are sufficient but neither Expenditure of B = 4y
statement alone is sufficient to answer the Even from both the statements we cannot find the ratio of
question. savings of A and B.
(4) If both I and II together are not sufficient to 44. What is the ratio of the selling prices of two articles
answer the question and additional data specific A and B?
to the question are needed. I. The cost price of article A is equal to the selling
41. A, B and C have money with them in the ratio 5:3:1. price of B.
How much money does B have? II. The profit made by selling A is equal to 1/5 of its
I. A has ` 60 more than C. selling price.
II. The money with B is 40 % less than the money (a) (1) (b) (2)
with A. (c) (3) (d) (4)
(a) (1) (b) (2) (c) (3) (d) (4) l (c) Let the cost price of article A be ‘A’
l (a) From statement I, and selling price of article A be ‘a’.
Let A, B and C 5 x, 3 x and x, respectively. Also, cost price of article B be ‘B’
A has ` 60 more than C and selling price of article B be ‘b’.
i.e. 5 x - x = 60 (Q A : C = 5 : 1) from statement I
Þ x = 15 A=b …(i)
Hence, share of B = 3 ´ 15 = ` 45 from statement II
60 1
From statement II, B= A a- A= a
100 5
60 A a
or a- b= [from Eq. (i)]
100 = 3 5
\ (Q B : A = 3 : 5)
A 5 4
Þ a= b
3 3 5
Þ =
5 5 a 5
or =
Hence, we cannot find share of B from statement II. b 4
Hence, both the statement are required to find the ratio of
42. What is the cost of each pen? selling price of A and selling price of B.
I. The cost of 6 pens and 5 pencils is ` 30.
45. If in a code language STAR=50 and CIRUS=65
II. If the cost of each pen and each pencil is reduced
then PLANET will be
by 40%, then the cost of 12 pens and 10 pencils (a) 68 (b) 78
will be ` 36. (c) 84 (d) 94
(a) (1) (b) (2) (c) (3) (d) (4)
NTSE~ SOLVED PAPER 2018 (Stage II) 9
l (d) Here, each letter’s position is according to reverse l (a) The pattern of the series is,
English alphabetical order. (Prime
2 3 5 7 11 13 numbers)
S T A R = 8 + 7 + 26 + 9 = 50
and C I R U S = 24 + 18 + 9 + 6 + 8 = 65 Y V V T V T (Position of
(25=2+23) (22=3+19) (22=5+17) (20=7+13) (22=11+11) (20=13+7) alphabet)
Similarly,
23 19 17 13 11 7 (Prime
P L A N E T = 11 + 15 + 26 + 13 + 22 + 7 = 94 numbers)

46. Pankaja puts her alarm clock on the table in such a 50. Identify which number does not fit in the sequence?
way that at 6 pm the hour hand points to North. In
156, 182, 210, 240, 282, 306,
which direction will the minute hand point at 9:15
(a) 182 (b) 210 (c) 282 (d) 306
pm?
(a) South-East (b) South l (c) The pattern of the series is
(c) North (d) West 272

l (d) 6 156 182 210 240 282 306


N
N-W N-E (132–13) (142–14) (152–15) (162–16) (172–17) (182–18)
3 9 W E Hence, 282 is the wrong term.
S-W S-E
S 51. What is the number in place of ‘?’ ?
12 6, 15, 35,? 143, 221
At 9 : 15 pm the minute hand will point West. (a) 45 (b) 65 (c) 77 (d) 93
?
47. One evening before sunset two friends Rajni and l (c) 6, 15, 35, ¾® , 143, 221
Sanjiv were talking face to face. If Sanjiv shadow The series follows the pattern as
was exactly to his right side, to which direction 2 ´3 =6
Rajni was facing? 3 ´ 5 = 15
(a) North (b) North-East 5 ´ 7 = 35
(c) South (d) South-East 7 ´ 11 = 77
l (c) Rajni 11 ´ 13 = 143
13 ´ 17 = 221
N
This series is product of prime numbers.
W E 52. A pattern is being followed to derive numbers using
Sanjeev’s shadow
two out of the six numbers appearing on the faces of
Sanjeev S a dice having numbers from 4 to 9, both inclusive.
Clearly, Rajni is facing South direction. Two such pair yield 106 and 52. What will the third
pairs yield?
48. The square boxes in the figures below are to be
5 7 7
painted with different colours such that no two
adjacent boxes (even diagonally) have same colour. 4 4 5
8 5 6
How many minimum colours do you need' in each
case?
(a) 100 (b) 113 (c) 130 (d) 145
l (b) From the given dice
we obtained opposite faces as
Opposite Opposite Opposite
4 ¾¾® 6, 7 ¾¾® 8, 5 ¾¾® 9
(a) (3,4) (b) (4,4) This follows the pattern as
(c) (4,5) (d) (3,5)
42 + 62 = 16 + 36 = 52
l (b) 52 + 92 = 25 + 81 = 106
3 1 1 2 1
\ 72 + 82 = 49 + 64 = 113
2 4 2 4 3 4 3

1 3 1 2 1 53. Which group of letter given in the alternatives will


complete the sequence?
Hence, minimum four colours are needed in each case.
a_tta_ant_an_nt_an
49. What is the number in place of ‘?’ ? (a) a t n t t (b) n n t a t
2Y23, 3V19, 5V17, 7T13, 11V11, ? (c) n a n t t (d) t n t a t
(a) 13T7 (b) 13V9 l (b) This sequence will be completed as
(c) 13W9 (d) 13U7 anttan / anttan / anttan/
\ nntat is the correct answer.
10 NTSE~ SOLVED PAPER 2018 (Stage II)

Directions (Q. Nos. 54-57) A group of students is sitting 59. Find the missing number at the place of ‘M’?
in such a way that each occupies a corner of a hexagonal
table. Ninong is sitting opposite to Yaangba, Ribiya is 5 4 3 8 9 4
sitting next to Silva, Nazeli is sitting opposite to Silva, but
not next to Ninong, one person is sitting between Talyang
and Yaangba. 20 9 24 11 M 13

54. Who is sitting opposite to Ribiya?


(a) Yaangba (b) Silva (c) Talyang (d) Nazeli (a) 36 (b) 52 (c) 81 (d) 117

55. Who is sitting between Ribiya and Ninong? l (a) In the given question
(a) Yaangba (b) Nazeli (c) Talyang (d) Silva As, 5 ´ 4 = 20
5+4 =9
56. Who is sitting between Talyang and Yaangba? and 3 ´ 8 = 24
(a) Nazeli (b) Ribiya (c) Ninong (d) Silva 3 + 8 = 11
57. If TaIyang sits to the right of Ninong, who is on the Similarly, 9 ´ 4 = 36 = M
left of Ninong? 9 + 4 = 13
(a) Ribiya (b) Nazeli (c) Yaangba (d) Silva \ M = 36
Sol. (Q.Nos. 54-57) 60. Which letter replaces the question mark (?) ?
Ninong

Talyang Silva E M H

N O A
Nazeli Ribiya

Yaangba I ? D
54. (c) From the above sitting arrangement, Talyang is
sitting opposite to Ribiya.
(a) A (b) E (c) H (d) M
55. (d) Silva is sitting between Ribiya and Ninong.
l (d) In the given question, the sum of number position of
56. (a) Nazeli is sitting between Talyang and Yaangba. letter 1st and IIIrd is equal to number position of letter
57. (d) Silva is sitting on the left of Ninong. second.
58. A cylinder is painted in 6 colours - Violet, Red, Blue, E + H ® M
Green, Yellow and Orange. The three positions of 5 + 8 ® 13
the cylinder are as follows. Looking at these figures, N + A ® O
identify the correct colour in place of ‘?’. 14 + 1 ® 15
Orange \ I + D ® M
9 + 4 ® 13
\ M = 13 is the right answer.
Orange
Yellow

? 61. Which letter replaces the question mark (?)?


Violet Violet
Blue 6 6 4 7
Red Green
6 N L 1
(I) (II) (III)

(a) Red (b) Blue (c) Green (d) Violet 5 U ? 1

6 10 14 2
l (c) In the given cylinder
Opposite (a) M (b) O (c) P (d) Q
Orange ¾¾® Blue l (d) In the given question,
Violet is adjacent to Red and Green. 6 + 4 + 4 = 14 ® N
Green is adjacent to Violet and Yellow. 4 + 7 + 1 = 12 ® L
Yellow is adjacent to Green and Red. 5 + 6 + 10 = 21 ® U
\ Yellow is come with Green. \ 14 + 2 + 1 = 17 ® Q
NTSE~ SOLVED PAPER 2018 (Stage II) 11
62. What is the number that should come in place of 65. A family consists of six members P, Q, R, X Y, Z.
‘M’? Q is the son of R but R is not mother of Q.
5 6 M P and R are a married couple.
Y is the brother of R. X is the daughter of P.
95 36 68 Z is the brother of Q.
Which symbol represents all the children of P?
(a) QXYZ (b) QXZ (c) XZR (d) QZ
10 3 3 2 4 8
l (b) According to the question
(a) 2 (b) 3
Y+ R+ P–
(c) 4 (d) 6 Brother Couple
n
l (b) The pattern in the given question is So Son Daughter
5 ´ 10 + 10 ´ 3 + 5 ´ 3 = 50 + 30 + 15 = 95
6 ´ 3 + 3 ´ 2 + 6 ´ 2 = 18 + 6 + 12 = 36 Brother Brother/Sister
Z+ Q+ X–
\ M ´ 4 + 4 ´ 8 + 8 ´ M = 4 M + 8 M + 32 = 68
\ QZX represent the children of P.
12 M + 32 = 68
12 M = 68 - 32 1
66. I noticed that my watch goes min fast at dusk. but
12 M = 36 2
1
M=3 at dawn it loses min. On Ist March morning my
3
63. In the given figure which number should replace watch showed right time, then on which of the
‘M’? following dates the watch was 5 min fast?
18 (a) 28th March (b) 29th March
8 13 (c) 30th March (d) 31st March
11 8 1 1
l (a) As watch gain min at dusk and loses min at down
13 11 2 3
1 1 1
M 21 \ Total gain in one day = - = min
2 3 6
1
10 9 min gain is in 1 day
6
23 10 8 9 1 min gain = 6 day
9 9
(a) 4 (b) 11 min of gain = 6 ´ = 27 days
2 2
(c) 13 (d) 19
Last half minute is gained on next morning.
l (a) In the given pentagon the sum of number across one \ On 28th March the watch gaines 5 min.
side is 50
As, 18 + 8 + 11 + 13 = 50 67. What is the length ‘x’ of the line segment CD in the
18 + 19 + 8 + 11 = 50 triangle drawn below?
C
11 + 21 + 9 + 9 = 50
23 + 10 + 8 + 9 = 50
\ 13 + M + 10 + 23 = 50 6 8
Þ M + 46 = 50 x
Þ M=4

64. Manushi remembers that birthday of Chaitra is after A 5 D 5 B


July 10 but before July 17, but Vishakha remembers (a) 4 (b) 5 (c) 6 (d) 8
that it is between 15 and 27 July. If July 10 was a l (b) C
Thursday and if both of them remember correctly
then on which day was Chaitra’s birthday?
(a) Sunday (b) Monday 6 8
x
(c) Tuesday (d) Wednesday h
l (d) Manushi, remembers, Chaitra’s birthday is between
10th July and 17th July i.e. (11, 12, 13, 14, 15, 16) A M D B
and Vishakha, remember’s, Chaitra’s birthday is between 5 5
15th July to 27th July i.e. (16, 17, 18, 19, 20, 21, 22, 23, 24, Let h be the perpendicular drawn on AB from vertex C.
25, 26, 27)
\ According to the question,
\ Chaitra’s birthday is on 16th July 1 1
10th July ® Thursday ´ 6 ´ 8 = ´ 10 ´ h
2 2
16th July = 10 + 6 = Thursday + 6 = Wednesday. 48 24
h= Þ h=
10 5
12 NTSE~ SOLVED PAPER 2018 (Stage II)

Now by using Pythagoras theorem, In D AMC l (d) In the given fig,


AM2 = AC2 - CM2
576
AM2 = 36 -
25 Opposite
324 18
AM2 = Þ AM =
25 5 Opposite
18 7
\ MD = 5 - =
5 5 Opposite
Now, In DCMD
CD2 = CM2 + MD2
2 2

Þ CD2 = æç ö÷ + æç ö÷
24 7 625
Þ CD2 = = 25 By folding the cube the face opposite to face having 3 dots
è5 ø è5 ø 25 contains 6 dots.
Þ CD = 25 Þ CD = 5 cm
71. If ‘÷’ is ‘+’, ‘×’ is ‘-’, ‘-’ is ‘÷’ and ‘+’ is ‘×’ then
68. If m+n=o+p, what is the value of 20 ¸ 4 ´ 12 - 6 + 11 ?
m+q=p+n, (a) 2
2 p <m+q and 2 m>o+n, then (b) 5
(c) 56
(a) o>m>n>p>q (b) m>o>p>n>q
(c) n>o>p>m>q (d) o>p>n>q>m (d) 65

l (a) Given, l (a) The expression is


m+ n = o + p …(i) 20 ¸ 4 ´ 12 - 6 + 11
m+ q = p + n …(ii) After sign changes
2p < m + q …(iii) 20 + 4 - 12 ¸ 6 ´ 11 = 20 + 4 - 2 ´ 11
2m > o + n …(iv) = 20 + 4 - 22 = 2
Now, as 2p < m + q 72. Six dice are stacked as shown in the figure.On each
Þ 2p < p + n dice, the sum of number appearing on a face and on
Þ p<n …(v) the face opposite to it is 7.
Also, 2m > o + n > o + p [from Eq. (v)]
Þ 2m > o + p
Þ 2m > m + n
Þ m> n …(vi)
Using Eq. (ii) m + q = p + n
Þ m- n = p -q
since, m > n, therefore p > q
Finally using Eq. (i), m + n = o + p
Þ m- o = p - n
What is the maximum possible sum of the numbers on
Since, p < n, therefore n < 0.
the visible faces?
Hence, o > m > n > p > q.
(a) 88
69. If water image of OXIDE is OXIDE, then water (b) 89
image of METAL will be (c) 96
M T A L (d) 147
(a) L A T E M (b) E
(c) M E T A L (d) M E T A L l (b) 6 3
T A L 5
l (b) The water image of METAL is W E . 2 5
6 3
70. How many dots lie opposite the face having three
5
dots when the given figure is folded to form a cube? 6 4 4
3
6
5
6 5 4 4

Maximum possible sum of the numbers on visible faces


= 6 ´5 + 5 ´ 3 + 4 ´ 6 + 3 ´ 6 + 2
= 30 + 15 + 24 + 18 + 2 = 89
(a) 2 (b) 4 (c) 5 (d) 6
NTSE~ SOLVED PAPER 2018 (Stage II) 13
73. Observe the sequence given below and select the On multiply equation (ii) by 4 both side
appropriate alternative which will maintain the 3
Þ 4x > ´ 4z
series. 2.4
5 6 3 4 5 ? Þ 4 x > 5z
\ 4 x ? 5z

77. If Q means ‘addition sign’, J means ‘multiplication


6 4 5 3 6 4 2 6 4 2 ? ? sign', T means 'subtraction' sign' and K means
6 3 2 1 'division sign' then,
30 K 2 Q 3 J 6 T 5 = ?
Find the number in place of ‘?’.
4 1 6 4 4 5 4 5 (a) 18 (b) 28 (c) 31 (d) 103
(a) (b) (c) (d)
l (b) 30 K 2 Q 3 J 6 T 5 = ?
l (d) The question follows the pattern as After sign coded language, the expression becomes
6 + 5 + 4 = 15 30 ¸ 2 + 3 ´ 6 - 5 Þ 15 + 18 - 5 = 28
6 + 5 + 3 = 14 78. Which figure should come next among the options
3 + 6 + 4 = 13 given below?
4 + 2 + 6 = 12
5 + 4 + 2 = 11
\ 1 + 4 + 5 = 10
So, option (d) is the correct answer.

74. Two positions of 8 dice are shown. When number 3


is on the top, what number will be at the bottom?
2 2

6 1
3 6
(a) (b) (c) (d)

(a) 1 but not 4 (b) 4 but not 1 (c) 5 or 4 (d) 5 but not 4
l (d) Following the same pattern figure in option (d) will
complete the series pattern.
l (a) From the given dices 2 and 6 are adjacent to 1 and 3.
Therefore 3 and 1 is at opposite faces. 79. Observe the trends in figures given below and find
the missing character.
75. Which interchange in signs and number would
make the equation correct?
(96 ÷128) + 64 = 2
(a)+ and ÷, 64 and 96 ?
(b) +and ÷,64 and 128
(c) + and ÷, 96 and 128
(d)÷ and ÷,94 and 128
l (a) After changing (+) and (¸) and 64 and 96 we get the
required answer.
As, (64 + 128) ¸ 96 = 2
Þ 192 ¸ 96 = 2 (a) (b) (c) (d)
Þ 2 =2
l (a) In this given question the middle section is
76. Let ‘‘%’’ stands for ‘‘is equal to’’, ‘‘?’’ for ‘‘greater represented as the resultant of the difference between
than’’, ‘‘#’’ for ‘‘lesser than’’. If 6x% 5y and 2y?3z, number of circles and number of triangle in all four
section.
then'
\ Option (a) is the correct answer.
(a) 2x ? 3z (b) 4x ? 5z (c) 2x# z (d) 4x % 3z
l (b) 6 x % 5 y = 6 x = 5 y 80.
6 Equivalent + - ´ ¸ = ( )
Þ y= x signs
5 ® ¬ ­ ¯ ↵ ↵
2 y = 2.4 x …(i)
Now, 2 y ? 3z = 2 y > 3z 3 ­ 8 ¯ 4 ® 2 ¬ 5 ↵ 7 ® 12 ¬ 1 ?? 6
Þ 2.4 x > 3z [from Eq. (i)] What will come in place of ‘??’ ?
3 (a) ↵ ↓ (b) ↵ ↑
Now, x> z …(ii)
2.4 (c) ↵ ↑ (d) ↵ ↓
14 NTSE~ SOLVED PAPER 2018 (Stage II)

l (a) 3 ­ 8 ¯ 4 ® 2 ¬ 5 ↵ 7 ® 12 ¬ 1 ?? 6 l (b) In matrix,


3 ´ 8 ¸ 4 + 2 - 5 = 7 + 12 - 1 ?? 6 P = 0 #, 1# , 0$, 5*
3 ´ 2 - 3 = 19 - 1 ?? 6 E = 2$, 3@ , 4#
3 = 18 ?? 6 A = 0@, 4*
3 = (18) ¸ 6 C = 4@, 4$
3 =3 E = 2$, 3@, 4#
\ ?? Þ ) ¸ = ↵ ¯ PEACE ® 1# , 3@, 4*, 4@, 4#

81. Complete the missing pattern. 84. A square sheet is folded into half, the line of folding
being parallel to a side of the square. It is again
folded into half, the line of folding being parallel to
the shorter side. In this condition the front of the
paper always appears as it is given in the figure
below (the dotted lines represent the folded
portions).

(a) (b) (c) (d) From the alternatives choose the correct figure which
l (d) Option figure (d) will complete the missing pattern in represents the paper in to original unfolded form.
the question figure.

82. Find the number of rectangles in the following


figure.

(a) (b) (c) (d)

l (c) Answer figure (c) is represented original figure of given


question figure.

85. A, B, C, D, E, F, G and H are each to be assigned a


(a) 18 (b) 17 (c) 16 (d) 15 different number from 1 to 8. What should be values
of B, D, E, F and G so that no consecutive numbers
l (a) Number of single rectangle = 8 are in adjacent (even diagonally) squares?
Number of rectangles made by 2 rectangles = 2 + 3 = 5
Given: A = 5, C = 2, H = 4
Number of rectangles made by 3 rectangles = 2 + 2 = 4
Number of rectangle made by 4 rectangle = 1 A B
\ Total rectangle = 8 + 5 + 4 + 1 = 18 C D E F

83. In the given matrix, first row and the first column G H
consist of symbols and numbers respectively, the
(a) (6, 8, 1, 7, 3) (b) (3, 8, 7, 1, 6)
combination of those would be the code for specific
(c) (8, 6, 3, 7, 1) (d) (3, 8, 1, 7, 6)
alphabets given in other cells. For example, the
code for ‘G’ could be 1$ or 2@. In the same manner, l (d) According to the question
what from the given alternatives will be the correct A5 B3
code for ‘PEACE’?
C2 D8 E1 F7
@ # ∗ $
G6 H4
0 A P Q P
1 T P S G Thus, B = 3, D = 8, E = 1, F = 7, G = 6 Þ 3, 8, 1, 7, 6
2 G R N E
86. In a farmhouse there are 50 hens, 45 goats and
3 E M O T
8 camels which are maintained by a few
4 C E A C supervisors. If the total number of feet be 224 more
5 J R P V than the number of heads in the farmhouse then the
(a) 0*, 3#, 0$, 4@, 5$ (b) 1#, 3@, 4*, 4@, 4# total number of supervisors is
(c) 1*, 3*, 2#, 4$, 5* (d) 0$, 3#, 5*, 4@, 4$ (a) 5 (b) 8 (c) 10 (d) 15
NTSE~ SOLVED PAPER 2018 (Stage II) 15
l (d) Let total number of supervisors be x. 90. In certain code 678 means ‘study very hard’, 347
\ Total number of legs = 50 ´ 2 + 45 ´ 4 + 8 ´ 4 + x ´ 2 means ‘hard work pays’ and 246 means ‘study and
= 100 + 180 + 32 + 2x work’.
= 2 x + 312 Which of the following is the code for ‘very’?
And number of heads = 50 + 45 + 8 + x (a) 4 (b) 6
= x + 103 (c) 7 (d) 8
Now, according to the question, l (d) According to the question,
(2 x + 312) - (x + 103) = 224
6 7 8 = study very hard
Þ x + 209 = 224
\ x = 15 3 4 7 = hard work pays

87. If in a coded language. 2 4 6 = study and work



‘Busy bees’ are coded as ‘Cpu off ’,
\ 7 ® hard, 6 ® study, 4 ® work

‘Busy crows’ are coded as ‘cpu hup’,
8 ® very, 3 ® pays, 2 ® and

‘Bright Crows’ are coded as 'Csj Hup’,
Thus, ‘8’ is coded for ‘very’.
Then, ‘Busy crows are clever’' will be coded as
(a) Cpu Hup Bsf Dmf 91. In a certain code ‘TOME’ is written as ‘@ $ * ?’ and
(b) Cpu hup bsf dmf ‘ARE’ is written as !& ?:
(c) cpu Hup Baf Dmf How can ‘REMOTE’ be written in that code?
(d) cpu hup bsf Dmf (a) & ? ! $ @ ? (b) & ? * $ @ ?
l (b) According to the question, (c) @ ? * $ @ ? (d) * @ $ * ?!
Busy bees Cpu off l (b) Given,
T O M E
Busy crows cpu hup

@ $ * ?
Bright Crows Csj Hup
and
\ Busy ® Cpu, crows ® Hup A R E
bees ® off , Bright ® Csj
Thus, Busy crows are clever ® Cpu hup bsf dmf.
! & ?
88. What is the code used for ‘Blue’ derived from the Similarly,
given coded statements as per a code language? R E M O T E

‘Flower Blue Red White' is coded as
‘Sa Ra Ga Ma’ & ? * $ @ ?

‘Take Red Pink Flower ’ is coded as
‘Sa Ha Ma Pa’ 92. If in a certain code

‘Take Blue Red Buds’ is coded as ‘Pa Da Ma Ga‘ 23×26=42 and 11×15=19, then 32×16=?

‘Bring Red Take White’ is coded as ‘Ma Na Pa Ra’ (a) 40 (b) 41
(a) Sa (b) Ga (c) Pa (d) Ra (c) 44 (d) 48
l (b) According to the question l (b) The following pattern is
(23 ´ 26) = (23 + 26) - 7 = 49 - 7 = 42
Flower Blue Red White Sa Ra Ga Ma and (11 ´ 15) - 7 = (11 + 15) = 26 - 7 = 19
Take Red Pink Flower Sa Ha Ma Pa
Similarly,
(32 ´ 16) = (32 + 16) - 7 = 48 - 7 = 41
Take Blue Red Buds Pa Da Ma Ga
93. In a family of 6 (A, B, C, D, E and F) members, there
Bring Red Take White Ma Na Pa Ra is one married couple with equal number of male
and female members. Read the following relations
\ ‘Ga’ is the code which is used for ‘Blue’.
and find out the one from the alternatives, which is
89. What will be the number of digits used in not true for the given family.
numbering the pages of a book having 199 pages? Relations
(a) 39 (b) 489 (c) 495 (d) 532 ●
A and E are sons of F.
l (b) Number of digits used is numbering the pages of a ●
D is the mother of a boy and a girl.
book having 199 pages ●
B is the son of A.
= 100 ´ 3 + 90 ´ 2 + 9 (a) A, E, B are males (b) C is the granddaughter of F
= 300 + 180 + 9 (c) C is the daughter of E (d) D is the wife of A
= 489
16 NTSE~ SOLVED PAPER 2018 (Stage II)

l (c) F– Directions (Q.Nos. 97-98) Study the pie chart and


Son Son information given below and answer the following
questions.
Couple
D– A+ E+
Brothers
Book
Daughter So Son
n reading
(–) Women
9%
Brother (+) Men Drawing
C– B+ 21%
Sister
Thus, option (c) is wrong according to the question. Sports
Pottery 23%
94. If P+Q means P is husband of Q, P/Q means P is 5%
sister of Q, P*Q means P is the son of Q. How is D Photography
6%
related to A in D*B+C/A?
(a) Son (b) Nephew (c) Sister (d) Couple Nature club
Music 14%
l (b) D * B + C / A Þ 22%
r Couple s Sister
B C A

Son Son
There are 1150 students of a school opted sports as
Nephew
a hobby.
Dr
Thus, ‘D’ is nephew of ‘A’. 97. How many students have book reading as a hobby?
(a) 390 (b) 420
95. Afsana was walking in a desert. Anwar was passing (c) 440 (d) 450
by riding on a camel. Afsana requested for a lift.
Anwar said he will give lift only to those who are 98. What is the total number of students in the school?
(a) 4990 (b) 5000
related to him.
(c) 5050 (d) 5100
At this, Afsana told him that Anwar’s mother-in-law is
l Sol. (Q.Nos. 97-98) Number of students who opted sports
the mother of her mother-in-Iaw.
= 1150 [given]
How is Anwar related to Afsana?
Let total student in a class be x.
(a) Father (b) Maternal uncle 23
(c) Brother-in-law (d) Father-in-law \ x´ = 1150
100
l (d) C
s 1150 ´ 100
Þ x=
23
Mother
Mother-in-law Þ x = 5000
r 97. (d) Number of students who opted book reading
s A
B Couple
Anwar 9
= 5000 ´ = 450
Mother-in-law
Son 10
Son
98. (b) Total number of students in a school = 5000
s Couple
E
Afsana r 99. The following table shows the distribution of Boys
D Father-in-law
and Girls students of seven different schools.
Thus, ‘Anwar’ is father-in-law of ‘Afsana’.
School Boys (Total 27300) Girls (Total 24700)
96. A person travels from Mumbai to Ahmedabad by car A 17% 8%
in 5 h. The speed of the car during first hour of B 12% 15%
journey was 60 km/h. For the next two hours speed
C 12% 12%
was 80 km/h, Next hour it was 100 km/h. Finally,
during the last hour of his journey he drove at D 13% 13%
40 km/h. What is the average speed during his E 19% 14%
journey? F 14% 21%
(a) 56 km/h (b) 67.4 km/h G 15% 17%
(c) 70 km/h (d) 72 km/h
l (d) Total distance covered by car in 5 h What is the ratio between the number of Girls and
= 60 + 80 ´ 2 + 100 + 40 = 360 Boys students respectively from school F?
360 (a) 14 : 21 (b) 19 : 21
\ Average speed of whole journey = = 72 km/h
5 (c) 17 : 21 (d) 19 : 14
NTSE~ SOLVED PAPER 2018 (Stage II) 17
l (d) Total students (Girls) in school F Odisha. George does not mind as long as it is not
21 Rajasthan.
= 24700 ´ = 247 ´ 21
100 Which destination would be acceptable to all?
Total students (Boys) in school F (a) Goa (b) Odisha
14
= 27300 ´ = 273 ´ 14 (c) Kerala (d) Madhya Pradesh
100
247 ´ 21 19 l (b) According to the question
\ Required ratio = =
273 ´ 14 14 Name Rajasthan Goa Kerala Odisha Madhya North
Pradesh East
100. Ayush, Hina, Harbhajan and George are student
Ayush 3 3 3 3 3 7
friends studying in Delhi and plan to go on winter
holiday somewhere in India. They can go to Hina 7 3 7 3 7 7
Rajasthan, Goa, Kerala, Odisha, Madhya Pradesh or Harbhajan 3 7 7 3 3 3
any of the North-Eastern States.
George 7 3 3 3 3 3
Ayush is willing to go anywhere except
North-Eastern States. Harbhajan prefers not to go to Thus, ‘Odisha’ destination is acceptable to all.
Goa and Kerala. Hina wants to go either to Goa or

Paper II Scholastic Aptitude Test (SAT)


101. Under which condition stated below, the six-carbon l (a) In the given situation, the medulla of the brain would
glucose molecule is broken down into three-carbon be involved in the regulation of blood pressure. Medulla
molecules pyruvate and lactic acid? receives and integrates signal from spinal cord and sends
(a) Aerobic condition in muscle cells resulting signals to cerebellum and thalamus. It contains
(b) Anaerobic condition in yeast cells centres that regulate heart rate, blood pressure, breathing,
swallowing, sneezing, vomiting, coughing and some other
(c) Aerobic condition in mitochondria
involuntary movements.
(d) Anaerobic condition in muscle cells
l (d) In muscle cells, under anaerobic conditions, the six- 104. Edward Jenner’s contribution for the eradication of
carbon glucose molecule is broken down into three-carbon smallpox is
molecules (pyruvate and lactic acid). The pyruvic acid (a) his proposition that smallpox had possibly spread
produced in glycolysis is directly reduced by NADH to throughout the world from India and China
form lactic acid. (b) his discovery of transformation procedure
Lactic dehydrogenase (c) his finding that rubbing of the skin crust of smallpox
Pyruvic acid +NADH ¾¾¾¾¾¾¾¾¾®
victims on the arm of a healthy person, would develop
FMN +Zn2+
Lactic Acid+NAD+ resistance against smallpox
(d) his finding that the cowpox infection protects the person
Lactic acid produced in muscles is sent to liver to
regenerate glucose. from subsequent infection from smallpox
l (c) Edward Jenner found out that cowpox infection
102. Which among the following is the correct sequence protects the person from subsequent infection from
regarding the flow of impulse in a neuron? smallpox. The development of disease results in the
(a) Dendrite ® Axon ® Cell body immunisation of the body against the disease. Edward
(b) Axon ® Cell body ® Dendrite Jenner contributed to the idea of vaccination to protect the
(c) Axon ® Dendrite ® Cell body person from various diseases.
(d) Cell body ® Axon ® Nerve terminal
105. Four important events given below may have led to
l (d) The correct sequence regarding the flow of impulse in the origin of life on the Earth.
a neuron is as follows:
Cell body® Axon® Nerve terminal I. Formation of amino acids and nucleotides
Cell body or cyton is the spherical or oval part of neuron. A II. Availability of water
nerve impulse from cyton progresses towards axon. The III. Organisation of cells
axon conducts nerve impulse away from the cell body.
IV. Formation of complex molecules
It ends in a group of branches, called axon terminals or
dendrons. Select the correct sequence of events.
(a) I, II, III and IV (b) II, I, IV and III
103. In a hypertensive patient, the systolic pressure (c) I, IV, II and III (d) II, III, I and IV
increased to 150 mm of Hg. Which part of the brain
l (b) The Earth formed about 4.6 billion years ago. The
would be involved in the regulation of blood liquid component, known as hydrosphere, appeared on the
pressure? Earth when it cooled down to a temperature below 100ºC.
(a) Medulla (b) Cerebrum Free atoms present on the Earth, combined to form
(c) Cerebellum (d) Hypothalamus inorganic molecules such as H2 , N2 , H2 O, NH3, etc. Their
18 NTSE~ SOLVED PAPER 2018 (Stage II)

early inorganic molecules interacted and produced simple afternoon. Wilting is the loss of turgidity of leaves and
organic molecules such as amino acids, purines and other soft aerial parts of a plant causing their drooping,
pyrimidines, etc. These later on gave rise to complex folding and rolling which occur due to loss of water as
molecules like polysaccharides, fats, proteins, etc. Their vapours by higher transpiration.
complex molecules formed protocells and then
organisation of cells. 109. Observe the experimental setup [A] and [B]
(A) (B)
106. Read the following statements carefully.
Test tube
I. Energy transfer in the biotic world always
proceeds from the autotrophs. Water
II. Energy flow is unidirectional.
III. Energy availability is maximum at the tertiary Cotton
level. plug
IV. There is loss of energy from one trophic level to
Plant
the other.
Select the relevant statements for the forest
ecosystem. Day 1 Day 5
(a) I, II and IV
(b) I, II and III
(c) I, III and IV From the options given below, choose the
(d) II, III and IV
combination of responses of shoot and root that are
l (a) Among the given statements, statement III is incorrect observed in B.
because energy availability is minimum at the tertiary (a) Positive phototropism and positive geotropism
level. Nearly 90% energy is lost when it moves from one (b) Negative phototropism and positive geotropism
trophic level to the next. Therefore, the residual energy
(c) Positive phototropism and negative geotropism
decreases drastically within second and third.
(d) Only negative phototropism
Producer ¾® Herbivore ¾® Carnivore I ¾® Carnivore II
Biomass Biomass Biomass Biomass l (a) In the given experimental setup, shoot is showing
1000 kcal 100 kcal 10 kcal 1 kcal positive phototropism while root is showing positive
geotropism. Phototropism is a growth response to a light
107. In a highly pesticide polluted pond, which of the gradient. In the experimental setup, it is clear that after
following aquatic organisms will have the maximum 5 days shoot bends towards light. Geotropism is the
amount of pesticide per gram of body mass? growth movement induced by the stimulus of gravity. In
(a) Lotus (b) Fishes test-tube (B), it can be clearly seen that roots have bend
(c) Spirogyra (d) Zooplanktons towards the Earth.
l (b) In a highly pesticide polluted pond, fishes will have 110. Raw banana has bitter taste, while ripe banana has
the maximum amount of pesticide per gram of body mass.
Heavy metals and persistent pesticides pass into food
sweet taste. It happens because of the conversion of
chain and increase in amount per unit weight of (a) starch to sugar
organisms with the rise in trophic level. They get (b) sucrose to fructose
accumulated in the fat, which is known as (c) amino acids to sugar
biomagnification. Lotus and Spirogyra are producers, (d) amino acids to protein
while zooplanktons form the first trophic level and fishes l (a) Raw banana has bitter taste, while ripe banana has
form second trophic level. Therefore, fishes will show sweet taste. It happens because of the conversion of starch
maximum biomagnification. to sugar. Starch is a polysaccharide, while sugar is a
108. A farmer made an observation in a backwater paddy monosaccharide. It has low molecular weight and is sweet
field of coastal Kerala that the paddy plants wilt in taste.
during noon onwards everyday but appear normal 111. In the flowering plants, sexual reproduction
next morning. What would be the possible reason involves several events beginning with the bud and
for wilting? ending in a fruit. These events are arranged in four
(a) The rate of water absorption is less than the rate of different combinations. Select the combination that
transpiration in the afternoon has the correct sequence of events.
(b) The rate of water absorption is more than the rate of (a) Embryo, zygote, gametes, fertilisation
transpiration in the afternoon (b) Gametes, fertilisation, zygote, embryo
(c) The changes in the rate of water absorption and (c) Fertilisation, zygote, gametes, embryo
transpiration are not associated with wilting (d) Gametes, zygote, embryo, fertilisation
(d) The rate of water absorption is not related to the rate of
transpiration l (b) In flowering plants, sexual reproduction involves
several events. Male and female gametes are formed
l (a) The possible reason for wilting is that the rate of water through gametogenesis. After maturation and pollination,
absorption is less than the rate of transpiration in the one male gamete fuses with egg cell to form zygote. It is
NTSE~ SOLVED PAPER 2018 (Stage II) 19
known as true fertilisation. Zygote develops into embryo. food with the help of CO2 and H2 O in the presence of
Therefore, the correct sequence of events in sexual sunlight and chlorophyll. In this process, oxygen is
reproduction in flowering plants is released as a byproduct.
Gametes ® Fertilisation ® Zygote ® Embryo Sunlight
6CO2 + 12H2 O ¾¾¾¾¾® C6 H12 O6 + 6H2 O + 6O2 ­
112. In pea plants, Round (R) and Yellow (Y) features of Chlorophyll Glucose
seeds are dominant over wrinkled (r) and green (y)
features. In a cross between two plants having the This oxygen is used in respiration for the oxidation of food.
same genotypes (RrYy), the following genotypic As a result, energy (ATP) is released which is used for
combinations of offspring are noticed. various metabolic processes.
A. RrYY B. Rryy Enzymes
C6 H12 O6 + 6O2 ¾¾¾¾® 6CO2 + 6H2 O + Energy as ATP
C. rrYy D. rryy Glucose
The phenotypic features of A, B, C and D are given
below in an order in four combinations. Select the 115. 100 grams of oxygen (O 2 ) gas and 100 grams of
correct combination of characters that corresponds helium (He) gas are in separate containers of equal
to the genotypes volume at 100°C. Which one of the following
(a) Round and yellow; round and green; wrinkled and yellow; statement is correct?
wrinkled and green. (a) Both gases would have the same pressure.
(b) Round and green; wrinkled and yellow; wrinkled and (b) The average kinetic energy of O 2 molecules is greater than
green; round and yellow. that of He molecules.
(c) Wrinkled and green; round and yellow; wrinkled and (c) The pressure of He gas would be greater than that of the
yellow; round and green. O 2 gas.
(d) Wrinkled and yellow; round and green; wrinkled and (d) The average kinetic energy of He and O 2 molecules is
yellow; round and yellow. same.
l (a) In the given situation, the correct combination of l (c) Pressure exerted by a gas is directly proportional to
characters that correspond to the genotypes is as follows number of mole of the gas present
A. Rr YY Round and Yellow 100
Number of moles of O2 gas= = 3.125 mol
B. Rr yy Round and Green 32
C. rr Yy Wrinkled and Yellow 100
Number of moles of He gas= = 25 mol
D. rr yy Wrinkled and Green 4
Hence, the pressure of He gas would be larger than that of
113. Eukaryotic organisms have different levels of O2 gas.
organisation. Select the combination where the
levels are arranged in the descending order. 116. At 298 K and 1 atm pressure, a gas mixture contains
(a) DNA, chromosome, cell, nucleus, tissue equal masses of He, H2 , O2 and NH3 . Which of the
(b) Tissue, cell, nucleus, chromosome, DNA following is correct for their average molecular
(c) Nucleus, cell, DNA, chromosome, tissue velocities?
(d) Tissue, cell, chromosome, nucleus, DNA (a) He > H2 > NH3 > O 2 (b) He < H2 < O 2 < NH3
l (b) The correct combination in which levels are arranged (c) H2 < He < NH3 < O 2 (d) O 2 < NH3 < He < H2
in the descending order is as follows l (d) Average molecular velocity of a gas is inversely
Tissue ® Cell ® Nucleus ® Chromosome ® DNA proportional to the molar mass of the gas.
Tissue is a group of cells of same origin which perform Hence, the correct order of gases along with molar mass
the similar function. Cell is the basic and functional unit (written in brackets) are as follows
of life. It contains nucleus which controls all the activities O2 (32) < NH3 (17) < He (4) < H2 (2)
of cell. In the nucleus, chromosomes are found.
These chromosomes are formed by DNA which is 117. A test tube along with calcium carbonate in it
responsible for the transfer of genetic information from initially weighed 30.08 g. A heating experiment was
one generation to the next generation.
performed on this test tube till calcium carbonate
114. The gaseous byproduct of a process in plants is completely decomposed with evolution of a gas.
essential for another vital process that releases Loss of weight during this experiment was 4.40 g.
energy. Given below are four combinations of What is the weight of the empty test tube in this
processes and products. Choose the correct experiment?
combination. (a) 20.08 g (b) 21.00 g (c) 24.50 g (d) 2.008 g
(a) Photosynthesis and oxygen l (a) Calcium Carbonate (CaCO3 ) when heated, decomposes
(b) Respiration and carbon dioxide in calcium Oxide (CaO) and Carbon Dioxide (CO2 ) gas.
(c) Transpiration and water vapour
(d) Germination and carbon dioxide D
CaCO3 (s) ¾¾¾® CaO (s)+ CO2 (g)
( mass=100 ) ( mass=56 ) ( mass=44 )
l (a) The correct combination is photosynthesis and oxygen.
Photosynthesis is the process in which solar energy is
converted into chemical energy. Plants synthesise their
20 NTSE~ SOLVED PAPER 2018 (Stage II)

Since 100g (molar mass) of CaCO3 produces 44 g of CO2 l (a) Number of a-particles deflected in directly proportional
hence, in order to produce 4.4 g of CO2 we could require to the square of atomic number (Z). Hence, the correct
4.4 order of increasing number of alpha particles passing
100 ´ = 10 g of CaCO3 .
44 undeflected the foils of given metals in as follows
Therefore, the weight of empty test tube is Element Au < Ag < Cu < Al
30.08–10.00=20.08 g Z 79 47 29 13

118. Match List I (Mixture to be separated) with the 121. The correct order of increasing pH values of the
List II (Method used) and select the correct option aqueous solutions of baking soda, rock salt, washing
using the codes given below soda and slaked lime is
(a) baking soda < rock salt < washing soda < slaked lime
List I List II (b) rock salt < baking soda < washing soda < slaked lime
(Mixture to be separated) (Method used)
(c) slaked lime < washing soda < rock salt < baking soda
A. Petroleum products (i) Chromatography (d) washing soda < baking soda < rock salt < slaked lime
B. Camphor and rock salt (ii) Centrifugation l (b) Rock salt (NaCl) is the salt of strong acid and strong
C. Cream from milk (iii) Sublimation base, hence, its aqueous solution remains neutral
(i.e. pH = 7)
D. Coloured components in a dye (iv) Fractional distillation
Baking soda (NaHCO3 ) is the salt of strong base (NaOH)
and weak acid (H2 CO3 ). Hence, its aqueous solution is
A B C D A B C D
slightly basic.
(a) (i) (ii) (iii) (iv) (b) (ii) (iv) (iii) (i)
NaOH + H2 CO3 ® NaHCO3 + H2 O
(c) (iv) (iii) (i) (ii) (d) (iv) (iii) (ii) (i) Strong acid Weak base Alkaline
salt
l (d) Petroleum products (petrol, diesel, paraffin wax, etc)
Washing soda (Na2 CO3 ) is the salt of 2 mol of strong base
have different boiling points and distill out at different (NaOH) and 1 mol of weak acid (H2 CO3 ).
temperature. Hence, fractional distillation (which involves
heating a mixture at different temperature to distill 2NaOH+H2 CO3 —® Na2 CO3 + 2H2 O
different volatile components) is the suitable method. Hence, washing Soda is more alkaline than baking soda
i.e. pH(N2 CO3 ) > pH(NaHCO3 ) > pH =7
The mixture of camphor and rock salt can be suitably
separated by the method of sublimation as camphor Slaked lime [Ca(OH)2 ] is a base it self hence its aqueous
sublimates i.e. convert in vapour state without going into solution would be more alkaline than the other given salts.
liquid state) even at room temperature while rock salt do
not. 122. How many grams of oxygen gas will be needed for
Milk is a colloidal mixture and when subjected to complete combustion of 2 moles of 3rd member of
centrifugation (also called churning of milk) the lighter alkyne series?
particles of cream move outward and thus cream is
separated. (a) 186 g (b) 256 g (c) 352 g (d) 372 g
Coloured components of a dye (i.e.n pigments) propagate l (c) The third member of alkyne series is butyne (C4 H6 ).
is medium (say paper strip) at different rates. The chemical equation for combustion of 2 moles of
Chromatography exploits this property of pigments to butyne is as follows:
separate them. 2C4 H6 + 11 O2 ¾® 8 CO2 + 6H2 O
119. In the balanced chemical equation, Hence, it require 11 moles of oxygen for complete
combustion of 2 moles of butyne.
(a lead nitrate + b aluminium chloride Mass of 11 mol O2 = 11 ´ 32 = 352 g
® c aluminium nitrate + d lead chloride)
Which of the following alternatives is correct? 123. Match List I (Position of the metal in the activity
(a) a = 1, b = 2, c = 2, d = 1 (b) a = 4, b = 3, c = 3, d = 4
series) with the List II (Related reduction process)
(c) a = 2, b = 3, c = 2, d = 3 (d) a = 3, b = 2, c = 2, d = 3 and select the correct option using the codes given
below.
l (d) The balanced chemical equation for the reaction
between lead nitrate and aluminum chloride is as follows: List I (Position of the List II (Related
3Pb(NO3 )2 + 2AlCl3 ® 2Al (NO3 )3 + 3PbCl2 metal in the activity series) reduction process)
Lead nitrate Aluminium Aluminium Lead
chloride nitrate chloride A. The bottom of the series (i) Electrolysis
B. The top of the series (ii) Reduction by heat alone
120. The correct order of increasing number of alpha
C. The lower regions of the (iii) Found in native state
particles passing undeflected through the foils of
series
Au, Ag, Cu and Al of 1000 atoms thickness each in a
simulated alpha particle scattering experiment of D. The middle of the series (iv) Reduction using carbon
or some other reducing
Rutherford would be agent
(a) Au < Ag < Cu < Al
(b) Al < Cu < Ag < Au A B C D A B C D
(c) Au < Cu < Al < Ag (a) (ii) (iii) (iv) (i) (b) (ii) (i) (iv) (iii)
(d) Ag < Cu < Al < Au (c) (iii) (i) (ii) (iv) (d) (iii) (i) (iv) (ii)
NTSE~ SOLVED PAPER 2018 (Stage II) 21
l (c) Highly reactive metals (like, Na, Mg, Al, etc) are (ii) Substitution reaction
extracted by electrolysis. Moderately reactive metals (like C3 H7 Cl + KOH(aq) ¾¾®C3 H7 OH+KCl
iron, zinc etc) are extracted by using a reducing agents Propanol ( c)
like carbon. Less reactive metals (like Cu) can be extracted
(iii) Dehydration reaction
by only heating. Very less reactive metals (like Au, Pt) are Conc. H 2 SO4
found in native state. C3 H7 OH ¾ ¾¾¾¾ ®CH3 CH =CH2 +H2 O
Propane( D )

124. Which of the following statements can help a (iv) Addition reaction
chemistry student to predict chemical properties of CH3 C H = CH2 + H2 ¾ ¾¾¾ ¾®CH3 CH2 CH3
Ni / P+ /Pd
(D) Propane (A)
an element?
I. Position of element in the periodic table. 127. An organic liquid ‘A’ with acidified potassium
II. Atomic number of the element. dichromate gave product ‘B’. The compound ‘B’ on
heating with methanol in presence of concentrated
III. Number of shells in the atom.
sulphuric acid formed compound ‘C’ which on
IV. Number of electrons in the outer most shell. subsequent treatment with sodium hydroxide formed
Codes two products ‘D’ and ‘E’. The product ‘D’ is known to
(a) I, II and III (b) I, II and IV (c) I, III and IV (d) II, III and IV affect of ‘D’ in very small quantities can cause
l (b) Chemical properties of an element is defined by its death. What are compound ‘A’, ‘B’, ‘C’, ‘D’ and ‘E’?
valency that can be found out from position of element, (a) A = Ethanol, B = Ethanoic acid, C = Methanol,
atomic number and number of electrons in its valence D = Sodium acetate, E = Methyl ethanoate
(outermost) shell. Number of shell in atoms of elements (b) A = Ethanol, B = Ethanoic acid, C = Methyl ethanoate,
having different properties can be same, hence, it is not a D = Methanol, E = Sodium acetate
defining criteria. (c) A = Sodium acetate, B = Ethanoic acid,
125. Consider the elements A, B, C and D with atomic C = Methyl ethanoate, D = Methanol, E = Ethanol
numbers 6, 7, 14 and 15 respectively. Which of the (d) A = Ethanol, B = Ethanoic acid, C = Methyl ethanoate,
D = Sodium acetate, E = Methanol
following statements are correct concerning these
elements? l (b) The organic liquid ‘A’ is ethanol (C2 H5 OH). The
reactions involved are as follows :
I. D will lose electron easily than C. Acidified
C2 H5 OH ¾ ¾¾¾®CH3 COOH + CH3 OH
II. B will gain electron more easily than C. Ethanol (A) K 2 Cr2 O7 Ethanoic acid Methanol
(B)
III. The element with highest electronegativity is D. Conc. H SO NaOH
¾¾¾¾®
2 4
CH3 COOCH3 ¾¾® CH3 COONa + CH3 OH
IV. The element with largest atomic size is C. Methyl ethanonate Sodium Methanol ( D )
( C) acetate
Codes ( E)

(a) I and II (b) II and III (c) II and IV (d) III and IV
128. Two nichrome wires A and B, each of length 5 cm
l (c) The elements A, B, C and D are actually carbon (6),
and of radius 1 cm and 3 cm respectively are
nitrogen (7), silicon (14) and phosphorus (15)
connected to each other in series. If a current of 5A
Hence, statement II and IV are correct. And the incorrect
statements (I and III) in their corrected form are as follows flows through the combination of wires, the ratio of
I. Phosphorus (D) will tend to get electron more easily potential difference across wire A to that across wire
then silicon (C). B will be :
III. The element with highest electronegativity is (a) 1 : 3 (b) 3 : 1 (c) 9 : 1 (d) 1 : 9
nitrogen (B). l (c) Given, length of wire A (lA )= 5cm
126. A hydrocarbon ‘A’ (C3H8 ) on treatment with chlorine length of wire B (lB )= 5 cm
Radius of wire A (rA )=1cm
in presence of sunlight yielded compound ‘B’ as
Radius of wire B (rB )=3cm
major product. Reaction of ‘B’ with aqueous KOH
gave ‘C’ which one treatment with concentrated According to Ohm’s law
H2SO4 yielded ‘D’. Hydrogenation of ‘D’ gave back V = IR
I rlA rl
‘A’. The sequence of reactions involved in above VA = [Q for a given conductor = R = ]
conversion is p (rA )2 A
(a) substitution, substitution, addition, dehydration I rlB
and VB =
(b) substitution, substitution, dehydration, addition p (rB )2
(c) substitution, dehydration, addition, addition So, the ratio of potential difference across wire A to that of
(d) addition, substitution, dehydration, substitution wire B,
l (b) The hydrocarbon A (C3 H8 ) is propane. The reactions VA I rlA p(r )2
= ´ B [Q lA = lB ]
involved are as follows: VB p (rA )2
I rlb
(i) Substitution reaction (r )2 (3)2 9
Sunlight
C3 H8 +Cl2 ¾ ¾¾¾®C3 H7 Cl +HCl = B 2 = 2 = = 9 :1
Propane( A ) Chloropropane ( B ) (rA ) (1) 1
22 NTSE~ SOLVED PAPER 2018 (Stage II)

129. Two convex lenses A and B, each of focal length From c to d,


30cm are separated by 30 cm, as shown in the by conservation of momentum,
3
figure. An object O is placed at a distance of 40 cm Mu1 = 2 Mu2
to the left of lens A. 2
3 2
M ´ u = 2 Mu2 éQ u = 2 uù
A B êë 1 3 úû
2 3
u2 = u / 2
O Time c to d,
l 2L
40 cm 30 cm Tcd = = ...(iii)
æu ö u
ç ÷
What is the distance of the final image formed by this è 2ø
lens system? From d to e,
(a) 120 cm to right of lens A (b) 90 cm to right of lens A similarly,
(c) 22.5 cm to right of lens B (d) 45 cm to right of lens B 5M u 5M 2
2 Mu2 = u3 Þ2 M = u3 Þ u3 = u
l (c) Here f1 = f2 = f = 30 cm 2 2 2 5
For lens A, Time from d to e
1 1 1 1 1 æ 1 ö æ ö
= - Þ = - ç- ÷ ç L ÷ 5L
f v1 u1 30 u1 è 40 ø t de = ç ÷ = …(iv)
ç 2u ÷ 2u
1 1 1 1 1 1 è5 ø
= + Þ = -
30 v1 40 v1 30 40
From e to f,
1 40 - 30 10
= = by conservation of momentum,
v1 1200 1200 5M 5 M 2u éQ u = 2u ù
u3 = 3 Mu4 Þ ´ = 3 Mu4 êë 3
\ v1 = 120 cm 2 2 5 5 ûú
u2 = 120 - 30 = 90 u4 = u/3
This image will be treated as source for second lens, then Time taken from e to f,
again from lens formula. L/2 l éQ u = u ù
For lens B, t ef = = êë 4 3 úû
u4 2(u / 3)
u2 = 90 cm, f = 30 cm 3L
1 1 1 1 1 1 = …(v)
= - Þ = - 2u
f v2 u2 30 v2 90
Total time, T= t AB + t BC + t CD + t DE + t EF
1 1 1 3 +1
= + = L 3 L 2 L 5 L 3 L 17 L
v2 30 90 90 t = + + + + =
u 2u u 2u 2u 2u
\ v2 = 22.5 cm
which is positive, so that it is 22.5cm from lens B. 131. A ball is thrown vertically up from the point A (see
figure). A person, standing at a height H of the roof
130. A cart of mass M moves at a speed u on a frictionless
atop a building tries to catch it. He misses the catch,
surface. At regular intervals of length L, blocks of
M the ball overshoots and simultaneously the person
mass m = drops vertically into the cart. How starts a stop-watch. The ball reaches it highest point
2 and he manages to catch it upon its return. By this
9
much time is taken to cover a distance of L? time, a time interval T has elapsed as recorded by
2 the stop-watch. If g is the acceleration due to gravity
9L 5L 19L 17 L
(a) (b) (c) (d) at this place, the speed with which the ball was
2u 2u 2u 2u
thrown from point A will be
l (d)
a b c d e f
u 3m u1 u2 5m u3 u4
m 2m 3m
2 2
l l l l
2
From a to b, initial speed (u) and H
L
t ab = ...(i)
u
From b to c, by law of conservation of momentum, a
Mu = æç M + ö÷ u1
M 2
Þ u1 = u ( g 2T 2 + 4gH )
è 2ø 3 (a) gH + gT (b)
l 3L 2
So, time taken from b to c is t BC = = ...(ii) ( g 2T 2 + 8gH )
æ 2 uö 2u (c) (d) ( g T + 2 gH )
2 2
ç ÷
è3 ø 2
NTSE~ SOLVED PAPER 2018 (Stage II) 23
l (c) Let u, u1
\Velocity (v) = u - g (T / 2) [Q v = 0]
gT
u¢ =
2
\ v2 = u2 + 2 gH (from second equation of motion)
(u¢)2 = u2 - 2 gH d
g2 T 2
+ 2 gH = u2
4 Water
g2 T 2
u= + 2 gH
4
Speed with which the ball was thrown point A will be
g2 T 2 + 8 gH (a) F (b) F
u= (N) (N)
4
g2 T 2 + 8 gH
=
2
d (cm) d (cm)
132. A ray of light of pure single colour is incident on the
face of a prism having angle of the prism 30° at an
angle of incidence 45°. The refracted ray does not
(c) F (d) F
change its direction as it crosses the other face and (N) (N)
emerges out of the prism. The refractive index of the
material of the prism is
2
(a) (b) 2 (c) 2 (d) 3 d (cm) d (cm)
3
l (c) A
l (c) Depth (d) of the lower surface of water, at some instant.
30º
S Buoyance forces, F = Adrg.
90º R where, A, r and g are constants.
Q θ So, F µ d
45º θ Hence, buoyance force increases linearly with depth of the
120º
P n1 dipped surface of block. But when cubic solid block is
n2
O dipped completely, then F becomes constant.
B
Hence, this condition is shown by graph (c).
C
134. Three balls A, B and C of same size but of different
In DOQR,
masses, are thrown with the same speed from the
q + q + 120° = 180° roof of a building, as shown in figure. Let v A , v B and
2 q = 180°-120° v C be the respective speeds with which the balls A, B
60°
Þ q= = 30° and C hit the ground. Neglecting air resistance,
2 which one of the following relations is correct?
At line AB, by Snell’s formula A
sin i n1
=
sin r n2 B
n1 sin45° = n2 sin30°
1 1 C
n1 × = n2 ×
2 2
n2 2
Þ = [Q n1 = 1]
1 2
\ n2 = 2
2 (it does not change mean incident at 90º on another (a) v A > vC > v B (b) vC > v A > v B
surface soÐr = 30°). (c) v A > v B > vC (d) v A = v B = vC
l (d) Let u is the initial velocity of balls at height h.
133. A metallic cubical solid block of side L is slowly
By conservation of mechanical energy between height and
lowered continuously in a large vessel, filled with ground,
water. Let d be the depth of the lower surface of the 1 1
block, measured from the surface of the water, at For ball A, mu2 + mgH = mv2A
2 2
some instant. The graph which represents correctly Þ v A = u2 + 2 gH …(i)
the variation of the buoyant force F with depth d is
24 NTSE~ SOLVED PAPER 2018 (Stage II)

1 1 2
(c) In the above graph, slope of car A is constant and slope
For ball B, mu2 + mgH = mv B l
2 2 car B well be same between origin to t0 . Therefore,
Þ v B = u2 + 2 gH …(ii) acceleration of two cars are equal to each other at an
1 1 2
instant earlier than t0 .
For ball C , mu2 + mgH = mv C
2 2 137. Three electric bulbs of rating 40 W - 200 V;
Þ v C = u2 + 2 gH …(iii) 50 W-200 V and 100 W - 200 V are connected in
From Eqs (i), (ii) and (iii), we get series to a 600 V supply. What is likely to happen as
vA = vB = vC the supply is switched on?
(a) Only 50 W bulb will fuse
135. Four blocks of different masses (m1 = 1 kg, m2 = 2 kg, (b) Both 40 W and 50 W bulbs will fuse
m3 = 1 kg and m4 = 5 kg) are connected with light, (c) All the three bulbs will emit light with their rated powers
inextensible strings, as shown in figure. This system (d) 100 W bulb will emit light of maximum intensity
is pulled along a frictionless surface by a horizontal l (b)
force of 36 N. The force pulling the block of mass m1
will be
m4
m2
m1 m3
36 N

40W-200V 50W-200V 100W-200V


(a) 2 N (b) 4 N (c) 12 N (d) 36 N
(b) Given, f = 36 N, V 2
200 ´ 200
l R40 = =
m1 = 1kg, m2 = 2 kg, m3 = 1kg and m4 = 5 kg P40 40
We know that, = 1000 W
F = ma Similarly,
F F 36 200 ´ 200
a= = = = 4 m/s2 R50 = = 800 W,
m m1 + m2 + m3 + m4 1 + 2 + 1 + 5 50
For first block, 200 ´ 200
R100 = = 400 W
4 m/s 2 100
V
\ I=
m F R40 + R50 + R100
600 600
F = m ´ a, F =1 ´ 4, F = 4 N I= = = 0.2727 A
1000 + 800 + 400 2200
136. The velocity-time graph of motion of two cars A and P 40
I40 = 1 = = 0.2 A
B is shown in the figure V 200
Car B P 50 5
I50 = 2 = = = 0.25 A
V 200 20
Car A
P 100
I100 = 3 = = 0.5 A
V 200
0.27 Ampere current is flowing in the circuit but capacity
Velocity
of 40 W and 50 bulb is low. Hence, both 40 W and 50 W
(v)
bulbs will fuse.

138. A sound wave is sent simultaneously through a long


hollow pipe AB and a solid pipe CD of same length
and having same cross-sectional area. A person
standing at point P as shown in the figure will hear
the sound
t0
A B
Time (t)

Choose the correct statements.


P
(a) Accelerations of two cars are equal to each other at time
t = t 0.
(b) Acceleration of two cars are equal to each other at an C D
instant greater than t 0 . (a) at the same time from pipes AB and CD
(c) Acceleration of two cars are equal to each other at an
(b) first from pipe CD then from pipe AB
instant earlier than t 0 .
(c) first from pipe AB, then from pipe CD
(d) At no instant in the interval 0 £ t £ t 0 , the two accelerations
are equal. (d) from pipe AB only and not from pipe CD
NTSE~ SOLVED PAPER 2018 (Stage II) 25
l (b) As we know that , speed of sound is maximum in solid. (i) The key K1 is closed and the key K2 is open.
In case of hollow pipe AB the pipe occupy air.
(ii) The key K2 is closed and the key K1 is open.
Similarly, in pipe CD, the pipe in through solid. So, when
sound will travel faster in solid than that of air. Hence, The ratio of respective currents in these two case will
person standing at point P will hear the sound first from be
pipe CD and then from pipe AB. (a) 3 : 4 (b) 4 : 3 (c) 4 : 5 (d) 5 : 4
139. Two long current-carrying parallel wires are placed l (d) Case I: When K1 is closed and K2 is open.
as shown. R1 R2 3 ´ 12 36 12
Then, Req = = = =
R1 + R2 3 + 12 15 5
i 12
So, I1 = = 5A
2 12
3i 5
2 Case II When K2 is closed and K1 is open.
a 4 ´ 12 4 ´ 12
b Then, R eq = = = 3W
12 + 4 16
Which of the following figures will represent the So, I2 =
12
= 4A
magnitude and direction of the forces exerted on the 3
wires? \
I1 5
= = 5 :4
F F 3F F I2 4
i i
2 2 1
141. Given that = 0.142857, which is a repeating
(a) 3i (b) 3i 7
2 2 decimal having six different digits. If x is the sum of
such first three positive integers n such that
1
F 3F = 0. abcdef , where a, b, c, d, e and f are different
i F F i n
2 2 digits, then the value of x is
(c) 3i (d) 3i (a) 20 (b) 21 (c) 41 (d) 42
2 2 1
l (c) We have, = 0. abc def
n
(where a, b, c, d, e and f are different digits.)
l (a) From Ampere’s circuital law, 1
m0 I a \ = 0.142857
BA = 7
2 pd 1
The conductor carrying a current (I0 ) will experience a = 0.076923
sideways force due to the field (b). 13
1
The direction of this force is towards the conductor A. and = 0.047619
21
We can write this force as FBA . So, the magnitude of this
force is given by \ Required number are 7, 13 and 21.
F = iLB Sum of numbers = 7 + 13 + 21 = 41
= m0 I A iB [force segment L of B due to A] 142. Which of the following digits is ruled out in the
m0 I A I B units place of 12 n + 1 for every positive integer n?
= L
2 pd (a) 1 (b) 3 (c) 5 (d) 7
Similarly, we can find
l (a) For n = 1,121 = 12, on unit place digit = 2
FBA = - FAB
n = 2,122 = 144, on unit place digit = 4
When current flowing in the same direction in the
conductor, then conductor attract each other and n = 3,123 = 1728, on unit place digit = 8
oppositely directed current repel each other. n = 4,124 =20736, on unit place digit = 6
n = 5,125 = 248832, on unit place digit = 2 and so on.
140. An electrical circuit, shown below, consists of a
So,12 n + 1, for very positive integer, having unit place
battery, an ammeter, three resistors and two keys. 2 + 1, 4 + 1, 8 + 1, 6 + 1 i.e., 3, 5, 9, 7.

143. The rational roots of the cubic equation


3Ω 4Ω 12Ω x 3 + 14 kx 2 + 56 kx - 64 k3 = 0 are in the ratio 1:2:4.
The possible values of k are
12 V (a) Only 0 (b) Only 1
K1 K2
(c) 2, 0 (d) -2, - 1
l (b) Given, cubic equation,
x3 + 14 kx2 + 56 kx - 64 k3 = 0
Consider two cases.
26 NTSE~ SOLVED PAPER 2018 (Stage II)

Now, let roots of the given equation l, 2 l and 4l, But the remainder is given as x + a
then sum of roots, l + 2 l + 4 l = - 14k On comparing their co-efficients, we have
Þ 7 l = - 14 k 2k - 9 = 1
Þ l = - 2k Þ 2 k = 10 Þ k = 5
and multiplication of roots =l × 2 l + 2 l × 4 l + 4l × l = 56k and -(8 - k)k + 10 = a
Þ 14 l2 = 56 k So, a = - (8 - 5)5 + 10 = - 5
Þ l2 = 4 k
146. The value of k, so that the equations 2 x 2 + kx - 5 = 0
Þ (-2 k)2 = 4 k
Þ 4 k2 = 4 k
and x 2 - 3 x - 4 = 0 have one root in common, are
Þ k2 - k = 0 27 27 -27 4
(a) 3, (b) 9, (c) -3, (d) -3,
Þ k(k - 1) = 0 2 4 4 27
Þ k = 0 or 1 l (c) Given, equations 2 x2 + kx - 5 = 0 and x2 - 3 x - 4 = 0
Þ k =1 have one root common.
(since given equation having rational roots) We know that, if a1 x2 + b1 x + c1 = 0 and a2 x2 + b2 x + c2 = 0
have one common root, then
144. The odd natural numbers have been divided in (a1 b2 - a2 b1 )(b1 c2 - b2 c1 ) = (c1 a2 - c2 a1 )2 …(i)
groups as Here, we have,
(1, 3); (5, 7, 9, 11); (13, 15, 17, 19, 21, 23); … a1 = 2, b1 = k, c1 = - 5
Then the sum of numbers in the 10th group is a2 = 1, b2 = - 3,c2 = - 4
(a) 4000 (b) 4003 (c) 4007 (d) 4008 On putting values in Eq (i)
l (a) Number of terms in Ist group = 1 ´ 2 = 2 \ [(2 ´ (-3) - (1)(k)][(k)(-4) - (-3)(-5))] = [(-5)(1) - (-4)(2)]2
Number of terms in IInd group = 2 ´ 2 = 4 Þ (-6 - k)(-4 k - 15) = (-5 + 8)2
Number of terms in IIIrd group = 3 ´ 2 = 6 Þ (6 + k)(4 k + 15) = (3)2
Number of terms in IVth group = 4 ´ 2 = 8 Þ 24 k + 90 + 4 k2 + 15 k = 9
Number of terms in IXth group = 9 ´ 2 = 18 Þ 4 k2 + 39 k + 81 = 0
Number of terms in Ist, IInd......, IXth group -27
After solving above equation, we get x = - 3,
9 9 4
2 + 4 + 6 + ......+18 = (2 + 8) = ´ 20 = 90
2 2
147. The value of cos x ° - sin x ° (0 £ x < 45) is
\ First term of Xth group = 1 + (91 - 1)(2)
(a) 0 (b) positive
= 1 + 90 ´ 2
(c) negative
= 1 + 180 = 181 (d) sometimes negative and sometimes positive
And numbers of term in 10th group = 10 ´ 2 = 20
l (b) We know that cos q > sin q, if 0 £ q < 45
\ Sum of terms in the 10th group
20 \ cos x° - sin x° > 0 for 0 £ x < 45
= [2 ´ 181 + (20 - 1)(2)]
2 148. A vertical pole of height 10 m stands at one corner
= 10(362 + 19 ´ 2) of a rectangular field. The angle of elevation of its
= 10(362 + 38) top from the farthest corner is 30°, while that from
= 10(400) = 4000 another corner is 60°. The area (in m 2 ) of
rectangular field is
145. If the polynomial x 4 - 6 x 3 + 16 x 2 - 25 x + 10 is
200 2 400 200 2 400 2
divided by another polynomial x 2 - 2 x + k, the (a) (b) (c) (d)
remainder comes out to be x + a, then the value of a 3 3 3 3
is l (a) Given, ABCD is a rectangular field and PC is a vertical
(a) -1 (b) -5 (c) 1 (d) 5 pole of height 10 m.
P
l (b) Given, polynomial x4 - 6 x3 + 16 x2 - 25 x + 10 and
x - 2 x + k,
2

10 m
then,
x2 - 2 x + k x4 - 6 x3 + 16 x2 - 25 x + 10 x2 - 4 x + (8 - k)
C
x4 - 2 x3 + kx2
-4 x3 + (16 - k)x2 - 25 x + 10
60º
-4 x3 + 8 x2 - 4 kx B
(8 - k)x2 + (4 k - 25)x + 10 D
(8 - k)x2 - 2(8 - k)x + (8 - k)k
30º
- + -
(2 k - 9)x - (8 - k)k + 10
A
\ Remainder = (2 k - 9)x - (8 - k)k + 10
NTSE~ SOLVED PAPER 2018 (Stage II) 27
In DAPC, 150. The surface of water in a swimming pool, when it is
PC full of water, is rectangular with length and breadth
tan30° =
AC 36 m and 10.5 m respectively. The depth of water
1 10
Þ = increases uniformly from 1 m at one end to 1.75 m
3 AC at the other end. The water in the pool is emptied by
Þ AC = 10 3 …(i) a cylindrical pipe of radius 7 cm at the rate of 5
and In DPDC, km/h. The time (in hours) to empty water in the pool
PC æ 22 ö
tan60° = is ç taken p =
CD ÷
è 7 ø
10
Þ 3 = 1 1 3 4
CD (a) 6 (b) 6 (c) 6 (d) 6
10 4 2 4 5
Þ CD = …(ii)
3 l (c) According to question
Now, BC2 = AC2 - AB2 A B
Þ BC2 = AC2 - CD2 (QCD = AB)
2 Before increase
BC2 = (10 3 )2 - æç ö÷
10
Þ in depth 10.5 m
è 3ø
100
= 100 ´ 3 -
3 D 36 m C
900 - 100 800
= = A 36 m B
3 3
800 After increase
Þ BC =
3 in depth 1m
20 2
= 1.75 m
3
\ Area of rectangular field = BC ´ CD (length ´ breadth)
20 2 10 200 2 2
= ´ = m
3 3 3
\ Volume of water of in a swimming pool
149. A circle is inscribed in a square and the square is 1
circumscribed by another circle. What is the ratio of = ´ (1 + 175
. ) ´ 3.6 ´ 10.5
2
the areas of the inner circle to the outer circle? 1
(a) 1 : 2 (b) 1 : 2 = ´ 2.75 ´ 36 ´ 10.5 = 519.75 m3
2
(c) 2 : 4 (d) 1 : 3
Length of the cylindrical pipe = 5 km = 5000 m
l (a) Let side of square is x, then radius of inscribed circle is Now, volume of water flowing in 1hour = pr2 h
x 22 7 7
. = ´ ´ ´ 5000 = 77 m3
2 7 100 100
x/2 A 519.75 3
\ Required time = =6 h
77 4
x/2
151. There is a right circular cone of height h and
vertical angle 60°. A sphere when placed inside the
O
cone, it touches the curved surface and the base of
the cone. The volume of sphere is
4 3 4 3 4 4 3
(a) ph (b) ph (c) ph3 (d) ph
3 9 27 81
\ Radius of circumscribed circle = OA l (d) Given, height of cone = h
2 2 A
= æx ö + æx ö
ç ÷ ç ÷
è2 ø è2 ø
º

h
60

x2 x2 2 x2 x
= + = =
4 4 4 2
\ Ratio of areas of the inner circle to the outer circle is
2 O
p æç ö÷ x2
x
è2 ø 2
2
= 42 = = 1 : 2
x 4
p æç ö÷
x D C
B
è 2ø 2
28 NTSE~ SOLVED PAPER 2018 (Stage II)

2
O is the centre of the circle, then OA : OB = 2 : 1
Then, l2 = (h)2 + æç 1 ö÷
x
2h h è2 ø
Then, OA = and OB =
3 3 x12
3
h3 Þ h2 = l2 - …(i)
\ Volume of sphere = p æç ö÷ = p ´
4 h 4 4
= ph3 4
3 3è ø 3 27 81 And let base of isosceles triangle is x2 when length of each
equal side is doubled.
152. A sealed bottle containing some water is made up of
two cylinders A and B of radius 1.5 cm and 3 cm
respectively, as shown in the figure. When the bottle 2l
is placed right up on a table, the height of water in it h
is 15 cm, but when placed upside down, the height
of water is 24 cm. The height of the bottle is
x2 x2
2 2
2
x22
(2 l)2 = h2 + æç 2 ö÷
x
Then, Þ 4 l2 = h2 +
è2 ø 4
x22
Þ h2 = 4 l2 - ...(ii)
15 cm 24 cm 4
From Eqs. (i) and (ii), we get
x2 x2
l2 - 1 = 4 l2 - 2
4 4
x22 x12
Þ - = 3l2

(a) 25 cm (b) 26 cm (c) 27 cm (d) 28 cm 4 4


Þ x22 - x12 = 12 l2
l (c) Let heights of cylinder A and B are h1 and h2 .
When bottle is placed is right up on a table 154. In the adjoining figure, ABC is a triangle in which
then volume of bottle ÐB = 90° and its incircle C1 has radius 3. A circle C2
= p(3)2 h2 + p(1. 5)2 ´ (15 - h2 ) of radius 1 touches sides AC, BC and the circle C1 .
= p[15 ´ (1. 5)2 + h2 ((3)2 - (1. 5)2 ] …(i) Then length AB is equal to
and when bottle is placed in up side down, then volume is A
p(1. 5)2 ´ h1 + p(3)2 ´ (24 - h1 )
= p[24 ´ (3)2 + h1 ((1. 5)2 - (3)2 )] …(ii)
According to question, in both situation volume of bottle is
same.
\ From Eqs. (i) and (ii), we get C1
p[24 ´ (3)2 + h1 ((1. 5)2 - (3)2 )] = p[15 ´ (1. 5)2 + h2 ((3)2 - (1. 5)2 )]
3 C2
Þ 24 ´ (3)2 - 15 ´ (1. 5)2 = (h1 + h2 )[(3)2 - (1. 5)2 ]
1
3 ´ (1. 5)2 (32 - 5)
Þ h1 + h2 = = 27 B C
3 ´ (1. 5)2
(a) 3 + 6 3 (b) 10 + 3 2
153. Let l be the length of each equal side of an isosceles (c) 10 + 2 3 (d) 9 + 3 3
triangle. If the length of each equal side is doubled, l (d)
keeping its height unchanged, then the difference A
of the squares of bases of the new triangles and the
x
given triangle is
2 x Q
(a) 0 (b) 4l
(c) 9l 2 (d) 12 l 2 C1
l (d) A R
P
P C2
3 3
l l 1
h
B S T C

Here, ST = C2 P = C1 C - C1 P
2 2 2
2
2
B C
x1 x1
2 2 Þ ST = 42 - 22
Let base of isosceles triangle is x1 and height is h. Þ ST = 16 - 4 = 2 = 2 3
Þ ST = 2 3 = C2 P
NTSE~ SOLVED PAPER 2018 (Stage II) 29
Since, DC1 C2 P ~ DC1 CS l (c) A
C1 P C1 S
Þ = 60º
C2 P CS
2 3
Þ =
2 3 CS
Þ 2 ´ CS = 2 3 ´ 3
60º P 60º
Þ CS = 3 3 = CQ B C
3 cm
Now, DABC, 4 cm
AC2 = AB2 + BC2 60º 60º
Þ (x + 3 3 )2 = (x + 3)2 + (3 + 3 3 )2 Q
Þ x2 + 27 + 6 3 x = x2 + 6 x + 9 + 9 + 27 + 18 3
Here, ÐBQA = ÐBCA = 60° (angle in same segment)
Þ x =6 + 3 3
Also, ÐAQC = ÐABC = 60° (angle in same segment)
\ AB = 3 + 6 + 3 3
\ QP is angle bisector of BC.
Þ AB = 9 + 3 3
\ PB : PC = QB : QC = 4 : 3
155. In DABC, AB = AC, P and Q are points on AC and (by using angle bisector theorem)
AB respectively such that BC = BP = PQ = AQ. Then Now, DBPQ ~ DAPC (By AA similarly criterion)
ÐAQP is equal to (use p = 180°) PQ PC
=
2p 3p BQ AC
(a) (b)
7 7 3
BC
4p 5p PQ 7
(c) (d) Þ =
7 7 4 BC
3 12
PQ = ´ 4 =
l (d) 7 7
A
157. How many points ( x , y ) with integral coordinates are
θ
there whose distance from (1, 2) is two units?
(a) one (b) two
θ (c) three (d) four
Q P
2θ l (d) According to question, we have
(x - 1)2 + (y - 2)2 = 2
Þ (x - 1)2 + (y - 2)2 = 4
B C Þ (x - 1)2 + (y - 2)2 = 4 + 0 or 0 + 4
Let ÐBAC = q Now, (x - 1)2 = 4
Þ ÐAPQ = q (Q AQ = PQ) Þ x -1 = ± 2
ÐPQB = 2q (Q exterior angle properties) Þ x = 3, - 1
Þ ÐBPC = ÐBAC + ÐABP = 3q and (y - 2)2 = 0
Þ y -2 =0
(Q PQ = PB Þ ÐPQB = ÐQBP) Þ y =0
Þ ÐBCP = ÐBPC = 3q (Q BP = BC) Hence, possible coordinates (3, 2), (-1, 2).
Also, ÐABC = ÐACB = 3q (Q AB = AC) And also, if (x - 1)2 = 0
Þ ÐBAC + ÐABC + ÐACB = q + 3 q + 3 q Þ x =1
(angle sum properties of triangle) and (y - 2)2 = 4
Þ 7q = p Þ y -2 = ±2
p Þ y = 4, 0
Þ q=
7 Possible coordinates (1, 4) and (1, 0)
Þ ÐAQP = p - ÐBQP \ Total integral coordinates are four.
= p - 2q
p 5p 158. If the vertices of an equilateral triangle have
= p -2 ´ = integral coordinates, then
7 7
(a) such a triangle is not possible.
156. A line from one vertex A of an equilateral DABC (b) the area of the triangle is irrational.
meets the opposite side BC in P and the circumcircle (c) the area of the triangle is an integer.
of DABC in Q. If BQ = 4 cm and CQ = 3 cm, then PQ (d) the area of the triangle is rational but not an integer.
is equal to l (a) Let integral coordinates of equilateral triangle
4 12 A(x1 , y1 ), B(x2 , y2 ) and C(x3 , y3 )
(a) 7 cm (b) cm (c) cm (d) 2 cm
3 7
30 NTSE~ SOLVED PAPER 2018 (Stage II)

A (x1, y1)
Þ x+5=m
Þ x = (m - 5) …(i)
Also, next 6 consecutive natural numbers are (x + 11),
(x + 12), (x + 13), (x + 14), (x + 15) and (x + 16),
then its sum = 6 x + 81
d d Again, new mean of 17 numbers
(11x + 55) + (6 x + 81) 17 x + 136
= = =x+8
17 17
= (m - 5) + 8 [by using Eq. (i)]
B (x2, y2) d C (x3, y3) = m+ 3
1 Now, change in mean = (m + 3) - m = 3
Then, area of triangle = [x1 (y2 - y3 ) + x2 (y3 - y1 ) 3 300
2 \ Percentage change in mean = ´ 100% = %
+ x3 (y1 - y2 )] m m
= Rational numbers 161. The Swaraj flag designed by Mahatma Gandhi had
3 3 2 the spinning wheel in it. What did it symbolise?
Now, area of equilateral DABC = (side)2 = (d)
4 4 (a) Ideal of self-help
3 (b) Symbol of defiance to the British rule
= [(x1 - x2 )2 + (y1 - y2 )2 ]
4 (c) Greatness of India in pre-colonial time
We know that (x1 - x2 )2 + (y1 - y2 )2 is rational. (d) Ahimsa (non-violence) in contemporary world
3 l (a) The Swaraj flag designed by Mahatma Gandhi had the
\ (Side)2 is a irrational.
4 spinning wheel at the centre. It symbolised Gandhi’s goal
of making Indians self-reliant by fabricating their own
Þ Area of DABC = irrational numbers.
clothing.
Hence, it is a contradiction.
\ No such DABC is possible. 162. Which of the following statements regarding the
Silk Routes are correct?
159. A box contains four cards numbered as 1, 2, 3 and 4
I. They also meant cultural links.
another box contains four cards numbered as 1, 4, 9
and 16. One card is drawn at random from each box. II. They spread over land and by sea.
What is the probability of getting the product of the III. They connected Asia with Europe and Africa.
two numbers so obtained, more than 16? IV. Besides textile, gold and silver got exported from
5 1 3 1 Asia to Europe through these routes.
(a) (b) (c) (d)
8 2 8 4 Codes
l (c) Ist box contains numbers 1, 2, 3, 4 and IInd contains (a) I, II and III (b) I, II and IV
numbers1, 4, 9, 16 (c) II, III and IV (d) I, II, III and IV
Now, getting the product of two numbers so obtained l (a) The Silk Route was a network of both the terrestrial
number is more than 16 and the maritime routes connecting China and the Far
\ Required pairs of numbers East with the Middle East, Africa and Europe. It was
(2, 9), (2, 16), (3, 9), (3, 16), (4, 9) and (4, 16) established when the Han Dynasty in China officially
Therefore favourable outcomes = 6 opened trade with the West in 130 BC. Through this route
not just transmission of goods took place but also ideas
and total outcomes = 4 ´ 4 = 16
and culture spread. These ideas were notably in the area
6 3
\ Required probability = = of religions such as Buddhism.
16 8
163. Which of the following statements regarding the
160. The mean of a group of 11 consecutive natural impact of Depression of 1929 are correct?
numbers is m. What will be the percentage change
I. India’s exports increased but imports decreased.
in the mean when next 6 consecutive natural
numbers are included in the group? II. India’s export of gold increased.
(a) m% (b)
m
% (c)
3
% (d)
300
%
III. Urban India suffered more than the rural India.
3 300 m IV. Industrial investment grew in India.
l (d) Let 11 consecutive natural numbers Codes
x, x + 1, x + 2, x + 3, x + 4, x + 5, x + 6, (a) I, II and III (b) I, III and IV
x + 7, x + 8, x + 9, x + 10 (c) II, III and IV (d) II and IV
According to question, l (d) The Great Depression was the worst and prolonged
x + (x + 1) + (x + 2) + ¼ + (x + 10) economic downturn in the world history, which affected
=m
11 the whole world and United States particularly. It lasted
11x + 55 from 1929 to 1940s. During Colonial period, India’s
Þ =m
11 exports and imports were also affected due to Great
Depression between 1928 and 1934. As a result in India
Þ 11x + 55 = 11m
prices also fell down and many Indian farmers were under
NTSE~ SOLVED PAPER 2018 (Stage II) 31
debt. To pay back their loans Indians started to export gold The British wanted tribal groups to settle down and
which promoted global economic recovery. During this become peasant cultivators, so that they could control and
period, the government, was forced to extend tariff administer them easily. But they failed because when their
protection to industries. Thus, it led to increase in fields did not produce good yields, shifting cultivators who
took to plough cultivation often suffered a lot.
industrial investments.
After facing widespread protests, the British had to
164. Which of the following statements about the French ultimately allow the right to carry on shifting cultivation in
in Vietnam are correct? some parts of the forest.
I. The Vietnamese teachers generally twisted the 166. Which of the following statements about the
school curriculum given by the French. Non-Cooperation Movement are correct?
II. The students protested the undue dominance by I. The Justice Party participated in the elections in
the colons. Madras.
III. The Annanese students was a French journal for II. The nationalist lawyers did not join back the
enlisting the students support. courts.
IV. The French had to counter the Chinese influence. III. The taluqdars were targeted.
Codes IV. The import of foreign cloth declined and the
(a) I, II and III (b) I, II and IV export of Indian textiles increased manifold.
(c) I, III, and IV (d) II, III and IV
Codes
l (b) The colonisation of Vietnam by the French brought the (a) I and III (b) I and IV (c) I, II and III (d) II, III and IV
people of the country into conflict with the colonisers in all
areas of life namely economic, cultural as well as l (a) The Non-Cooperation Movement started with
education. French colonisation was based on the idea of a middle-class participation in the cities in India. Many
‘civilising mission’. students left government-controlled schools and colleges,
headmasters and teachers resigned, and lawyers gave up
For this they started to bring changes in all aspects of life their legal practices. The council elections were boycotted
of Vietnamese and the big change was started from
education system. French modified school syllabus of the in many places except Madras, where the Justice Party,
Vietnamese but the Vietnamese teachers twisted the (the party of the non-Brahmans), participated in the
curriculum given by the French in order to save their elections.
culture. The movement spread in many places from urban areas to
Many times the students were discriminated by the countryside. In Awadh, peasants were led by Baba
French Colons (French people in the colonies) and they Ramchandra, against the talukdars and landlords who
protested their undue dominance. By the 1920s, the demanded high rates and a variety of other cesses from
Vietnamese students formed various political parties, such peasants.
as the Party of Young Annan and publishing nationalist Indians boycotted foreign clothes and started to make
journals such as the Annanese student. Indian ones. But this movement gradually slowed down
The French also faced another problem in the sphere of due to various reasons. Khadi clothes were more
education i.e. the elites in Vietnam were influenced by expensive than mill clothes and poor people could not
Chinese culture. To consolidate their power, the French afford that. During the movement, many alternative Indian
had to counter this Chinese influence thus; they institutions had to be set up so that they could be used in
systematically replaced the traditional educational system place of the British ones. But these were slow to be
and established French schools for the Vietnamese. established, up, as a result lawyers and students went
back to Government Institutions.
165. Read the statements about the impact of forest rules
on tribal communities in the 19th century. Which of 167. Which of the following regarding the Constitution of
the following statements are incorrect? 1791 and the status of women in France are correct?
I. Jhum cultivators could carry out their activities in I. It made them active citizens.
village forests? II. Provisions were made for schools for both boys
II. Jhum cultivators took to plough cultivation with and girls.
ease. III. Divorce rules were made stringent.
III. Tribal people could collect wood and graze cattle IV. Provisions were made for training women for
in the forests. jobs.
IV. Tribal people had access to protected forests for Codes
collecting wood for fuel and house building. (a) I, II and III (b) II and IV (c) III and IV (d) II, III and IV
Codes l (d) During 1791, many important changes brought in
(a) I and II (b) I and III French society. Provisions were made for schools for both
(c) II and III (d) III and IV boys and girls. With the creation of state schools, girls now
l (a) The lives of tribal groups was severely affected during could also attend schools as schooling was made
British rule as the British brought about several changes in compulsory for all girls. Their fathers could no longer force
the forest laws. The British extended their control over all them into marriage against their will. With the new law
forests and declared that forests as state (government) divorce was made legal, and could be applied for by both
property. The tribal people were not allowed to collect women and men. Women could now train for jobs and
fruits/woods hunt freely in the forests. could opt any profession.
32 NTSE~ SOLVED PAPER 2018 (Stage II)

168. Arrange the following historical development in a who wanted that the government to encourage
chronological sequence. cooperatives and replace capitalist enterprises. These
cooperatives were communities of people who produced
I. Rowlatt Act II. Kheda Satyagraha
goods together and divided the profits according to the
III. Champaran Movement work done by members.
IV. Ahmedabad Mill Strike
Codes 172. Statement (I) Hand printing developed in China.
(a) I, II, III, IV (b) II, I, III, IV (c) III, I, IV, II (d) III, II, IV, I Statement (II) The Chinese state printed textbooks in
vast numbers.
l (d) Champaran Satyagraha took place in 1917.
Kheda Satyagraha took place in 1918. l (c) The earliest kind of print technology also known as
hand printing was developed in China, Japan and Korea.
Ahmedabad Mill Strike took place in 1918.
The imperial state in China was the major producer of
Rowlatt Act took place in 1919. printed material. The bureaucratic system of China used to
recruit its personnel, through Civil Service Examinations.
Direction (Q. Nos. 169-174) Read the statements and For its examination a vast number of books were printed in
select the correct answer from the options given below. China.
(a) Statement I is true, Statement II is false. 173. Statement (I) Rainfall is low in the Western parts of
(b) Statement I is false, Statement II is true. Deccan plateau and East of Sahyadris.
(c) Both Statements are true, and Statement II Statement (II) Western Ghats causes convectional
provides explanation to Statement I. rainfall.
(d) Both Statements are true, but Statement II does l (a) The South-west monsoon winds from the Arabian Sea
not provide explanation of Statement I. strike almost perpendicularly to the western slopes of the
Western Ghats as these are higher in elevation and more
169. Statement (I) In the 19th century, London was a continuous (1500 m), thus, cause Orographic rainfall. The
colossal city. mountain barrier forces the moisture laden air to rise along
Statement (II) London had grown as an industrial city. the slope. This ascended air becomes condensed and
causes heavy rainfall on the windward side or western side
l (a) There are various reasons which explain that London of the Western Ghats.
was a colossal city during 19th century. Some of them are: But after reaching the crest, the air descends along the
l
It was seen that by 1750, one out of every nine people of leeward side or eastern slopes of the Sahyadri or Western
England and Wales lived in London. It was a colossal Ghats and gets warmed. This moisture present in this
city with a population of about 6, 75,000. warm air is already precipitated on the western slope, so
l
Over the 19th century, London was continued to expand there’s little amount left for the rainfall to happen on
its population fourfold between 1810 and 1860, eastern slope. So, the eastern slopes and the Deccan
increasing from 1 million to about 4 million. plateau beyond that, falls in the Rain-shadow region,
l
The city of London was a powerful magnet for migrant which receives scanty rainfall.
population, even though it did not have large factories. 174. Statement (I) A large part of the Deccan plateau is
Thus, it was not industrially well-off.
occupied by black soil.
170. Statement (I) Indians not taking off their turban Statement (II) Black soil in this part was formed by
before Colonial officials was considered offending. denudation of basaltic rocks overtime.
Statement (II) Turban was a sign of respectability in l (c) The Deccan plateau is formed by cooling of the basaltic
India. lava. Thus, the soil found here is made after working of
l (c) The Indians during Colonial period often wore Turban weathering agents over this basalt rocks and as a result
and thus named as ‘Turban Wearer’. Turban was give black soils.
considered as a sign of respectability, and could not be 175. If the local time at Varanasi, located at 83°E
removed at will.
longitude is 23:00 hour then what will be the local
But this was considered as an offence when Indians did
not remove their turban before the British. time at Kibithu located at 97°E longitude (Arunachal
Pradesh) and Jodhpur, located at 73°E longitude?
171. Statement (I) Louis Blanc built a cooperative (a) 00:00 hour, 22:00 hour (b) 22:20 hour, 23:56 hour
community. (c) 23:56 hour, 22:20 hour (d) 22:56 hour, 23:20 hour
Statement (II) He believed the community could l (c) The time at the Kibithu located at 97° East longitude is
produce goods together and divide the profits among 23:56 (11:56 pm) hours.
the members. The time at the Jodhpur located at 73° East longitude is
22:20 (10:20 pm) hours as per the local time of the at the
l (b) The ideas of social change developed after the French Varanasi which is located on 83° East longitude is 23:00
Revolution. There were many socialists who had different (11:00 pm).
visions of the future as some believed in the idea of
Time at 80° East Longitude = 23 : 00
cooperative community. For example, Robert Owen
(1771-1858), a leading English manufacturer, built a Time at Kibithu (Arunachal Pradesh) = 97°-83° = 14°
cooperative community called New Harmony in Indiana Q Time taken to cross 1° longitude is = 4 min
(USA). Another sociologist was Louis Blanc (1813-1882) 14° Þ 4 ´ 14 = 56 minutes = 23 : 56
NTSE~ SOLVED PAPER 2018 (Stage II) 33
Time at Jodhpur = 83°-73° = 10° (a) Montane Forest - Tropical Deciduous Forests - Tropical
10° = 10 ´ 4 = 40 minutes Evergreen Forests
= 23 : 00 - 40 (b) Tropical Evergreen Forests - Tropical Thorn Forests -
= 22 : 20 Tropical Deciduous Forests
(c) Tropical Deciduous Forests - Tropical Evergreen Forests -
176. Which one of the following statements are true Mangrove Forests
about latitudes and longitudes? (d) Tropical Evergreen Forests - Tropical Deciduous Forests -
I. All latitudes are angular distances measured Mangrove Forests
towards the pole from the equator. l (d) The correct sequence of the vegetation type is-
II. All longitudes do not join at poles. Tropical Evergreen Forests-Tropical Deciduous
Forests-Mangrove Forests.
III. All Parallels and Meridians are imaginary lines.
IV. Latitudes are used to determine the time of a 180. Which of the following statement(s) is/are true with
place. respect to monsoons in India?
(a) I and II (b) I and III I. The South-western Monsoon takes longer
(c) I, II and III (d) II, III and IV duration as compared to retreating Monsoon in
l (b) Latitudes and longitudes are imaginary lines used to covering India.
determine the location of a place on Earth. Latitude is the II. The South-western Monsoon has a shorter
angular distance of a point on the Earth’s surface, duration as compared to retreating Monsoon in
measured in degrees from the centre of the Earth. covering India.
Longitude is an angular distance, measured in degrees
along the equator East or West of the Prime Meridian. III. Both the Monsoons take almost the same
These lines are a series of semi-circles that run from Pole duration in covering India.
to Pole passing through the equator. These determine IV. The South-western Monsoon is propelled by the
local time in relation to Greenwich Mean Time (GMT).
depressions while retreating Monsoon results
177. If the current climatic condition of Srinagar (J & K) from the movement of air masses.
with average annual temperature of 13.5°C and (a) I and IV (b) Only II (c) Only III (d) II and IV
annual average precipitation 710 mm get modified l (b) A major part of the South-western Monsoon rainfall is
and become similar to that of Ranchi (Jharkhand) generated by depressions originating in the Arabian Sea
with annual average temperature 23.7°C and and Bay of Bengal. Some depressions develop over land
precipitation 1430 mm. Which one of the following also. About 3-4 depressions are formed per month from
types of vegetation will become predominant in June to September, which causes heavy rainfall in India.
Srinagar? 181. Which one of the following regions marked on the
(a) Tropical Semi Evergreen (b) Tropical Moist Deciduous sketch is an ideal representation with the following
(c) Tropical Dry Deciduous (d) Tropical Dry Evergreen characteristics?
l (b) Tropical Moist Deciduous forests need the following I. The approximate date for arrival of the
climatic conditions South-western Monsoon is 15th June.
l
Annual rainfall 100 to 200 cm. II. Well-developed in Thermal and Nuclear energy
l
Mean annual temperature of about 27°C. production.
l
The average annual relative humidity of 60 to 75%. III. Rich in the production of oil and natural gas.
l
Spring (between winter and summer) and summer are
dry.
IV. Well-developed Textile Industry.
These forests are found in Chota Nagpur plateau
(Jharkhand), along the Western Ghats, most of Odisha
and, some parts of West Bengal.

178. On a school field trip, a student spotted tigers,


turtles, gharials and snakes in their natural habitats.
D
Name the ecological region (delta) where that
student had gone. C
(a) Cauvery (b) Mahanadi
(c) Godavari (d) Ganga-Brahmaputra
l (d) Ganga -Brahmaputra region has a rich variety of A
wildlife. For example, tigers, elephants, deer, monkeys,
turtles, gharials, snakes etc.
B
179. A person travelling by road (shortest distance) from
Mangaluru to Machilipatnam will be able to
observe natural vegetation types in which of the
following sequences?
(a) A (b) B (c) C (d) D
34 NTSE~ SOLVED PAPER 2018 (Stage II)

l (c) The area C denotes Gujarat state where all the l (c) Refugee is a person who has been forced to leave
mentioned conditions prevail. his/her country in order to escape war, persecution, ethnic
conflicts or natural disaster.
182. With increasing urbanisation the main activity
which leads to loss of biodiversity is 185. Consider the following statements about the United
I. rural-urban migration. Nations Security Council (UNSC).
II. rapid increase in built-up area. I. UNSC consists of 15 members.
III. increased vehicular pollution. II. US, Russia and Germany are among the
IV. development of big industrial complexes. permanent members.
(a) I and III (b) I and IV (c) II and IV (d) III and IV III. China is the only Asian nation among the
l (c) The major causes for the destruction of bio-diversity are permanent members.
various human activities. For example rapid growth in IV. All members of the UNSC have veto power.
build- up area, rapid development of big industrial
complexes, building dams and other major construction Which of the above statements are correct?
works. (a) I and II (b) I and III
(c) I and IV (d) III and IV
183. Which one of the following statements is NOT correct
l (b) There are 15 members in the United Nations Security
about the shaded part on the given outline of India? Council. It has given ‘veto power’ to its 5 permanent
members i.e. China, France, Russia, United Kingdom and
United States. Germany is not the member of UNSC.

186. Which of these statements about the Election


Commission of India are true?
I. It conducts and controls the election process in
the country.
II. It gets the voters list updated before the elections.
III. It also conducts the Panchayat elections in the
country.
IV. It approves the election manifestoes of political
parties.
(a) I and II (b) II and III
(c) II and IV (d) III and IV
l (a) The Election Commission conducts and controls
elections according to the prevalent laws in India. The
commission also updates the voter list before the elections.
(a) It has high potential for hydel-power generation.
The State Election Commissions are vested with the power
(b) It has the lowest degree of urbanisation.
to conduct elections to the Corporations, Muncipalities,
(c) Ragi is an important millet grown here. Zilla Parishads, District Panchayats, Panchayat Samitis,
(d) It is famous for religious tourism. Gram Panchayats and other local bodies. They are
l (b) The total population in Uttarakhand increased by 20% independent of the Election Commission of India.
during the 2001-2011 period, while the urban population
went up by 40%. During this period, the share of urban 187. Consider the following statements about the Indian
population in the total population went up from 25.67% to Parliament.
30.23%. I. It is the ultimate authority to make laws in India.
As per Census 2011, the share of urban population in the II. It consists of the President, the Lok Sabha and the
total state population is almost 40% higher in Uttarakhand
compared to Uttar Pradesh, Chhattisgarh and Jharkhand. Rajya Sabha.
III. It consists of only the Lok Sabha and the Rajya
184. Chandimal, Jaysurya and Umesh left their Sabha.
respective village in Sri Lanka for Chennai in India.
IV. Lok Sabha members are chosen by the people
Who among the following could be a refugee?
through elections.
I. Chandimal, who is an IT professional, could not
Which of the above statements is/are correct?
find a job in Sri Lanka.
(a) Only I (b) I and II
II. Jaysurya, who left his village due to ethnic (c) II and III (d) I, II and IV
conflicts.
l (d) The Parliament of India is the supreme legislative body
III. Umesh, whose land and house were destroyed of India. It consists of the President of India and the
due to Tsunami. houses (Lok Sabha and Rajya Sabha). Lok Sabha members
(a) Only Jaysurya (b) Only Chandimal are the elected members by the people through election.
(c) Jaysurya and Umesh (d) Chandimal and Jaysurya
NTSE~ SOLVED PAPER 2018 (Stage II) 35
188. Which of the following is the inspiring philosophy of 192. Read the following statements and select one of the
the Constitution of India? four options given below.
I. Secularism, Equality, Communism, Democratic Statement (I) Enjoyment of pollution-free water is a
Republic fulfilment of right to life.
II. Democratic Republic, Sovereignty, Fraternity Statement (II) Release from forced labour is a
III. Secularism, Equality, Justice fulfilment of right to life.
IV. Equality, Fraternity, Communalism, Secularism (a) Only I is correct (b) Only II is correct
(c) Both I and II are correct (d) Both I and II are incorrect
(a) I and II (b) I and III (c) II and III (d) II and IV
l (c) Both the statements are correct.
l (c) The Indian Constitution is based on all those values
and principles which unite its people. The preamble is 193. The daily wage of a person in urban areas is ` 300.
based on these Principles. For example Sovereignty, The poverty line for a person is fixed at ` 1000 per
Socialist, Secularism, Democratic Republic, Fraternity,
Equality and Justice.
month for the urban areas. The following table
shows the details of employment of four families
189. Which of the following features of the Indian living in Mumbai city.
Judiciary are true?
Family Total days of Members of the
I. Integrated judicial system. employment got in a family
II. The Supreme Court is the highest court of appeal. month by the family

III. Only the Supreme Court can interpret the Hari 12 3


Constitution. Tenzin 15 4
IV. Public Interest Litigation (PIL) can be filed only Bala 15 5
in the Supreme Court and the High Court. Phulia 20 5
(a) I, II and III (b) I, III and IV (c) I, II and IV (d) II, III and IV
l (c) The Constitution of India provides for a single Identify the family living below poverty line.
integrated judicial system with the Supreme Court at the (a) Hari (b) Tenzin (c) Bala (d) Phulia
apex (the highest court of appeal), High Court at the l (c) Daily wages of Hari family = 12 ´ 300
middle (state) level and District Court at the local level. 3600
The Public Interest Litigation (PIL) can be filed in any Per person wages of Hari family =
3
level of court mentioned above.
= `1200
190. Which of the following statements is NOT true Daily wages of Tenzin family = 15 ´ 300 = `4500
about Indian federalism? Per person wages of Tenzin family =
4500
(a) The Union Government is vested with more financial 4
powers than the State Government. = `1125
(b) Power to legislate on residuary subjects is vested in the Daily wages of Bala family = 15 ´ 300 =`4500
Union Government. 4500
(c) The name and boundaries of a state can be changed by Per person wages of Bala family =
5
the Union Government without the consent of the
concerned state. = ` 900
(d) The Union Legislature can amend any provision of the Daily wages of Phulia family = 20 ´ 300 = `6000
6000
Constitution without the consent of the State Government. Per person wages Phulia family = = `1200
5
l (d) The Union Legislature can amend any provision,
except basic features of the Constitution of India, with the So, it is clear that Bala family is living below poverty line.
consent of at least half of the State Legislatures. 194. In a particular year, the price of wheat in a market is `
191. Democracy promotes equality through the following: 15 per kg and a farmer produces 100 kgs of wheat. In
I. Universal Adult Franchise the next year the price of wheat has fallen to ` 10 per
II. Equality before law and equal protection of law kg and the farmer produces 120 kgs. If the
III. Reservation for Scheduled Castes, Scheduled government wishes to stabilise the income of the
Tribes and Women farmer, then what will be the minimum support price?
IV. Independent and impartial media (a) ` 12 per kg (b) ` 12.5 per kg
(c) ` 13 per kg (d) ` 13.5 per kg
(a) I and II (b) I, II and IV (c) I, III and IV (d) II and IV
l (b) Given,
l (b) Democracy gives equal opportunity to all people by
providing Universal Adult Franchise, equality before law Price of wheat = `15/kg
and equal protection of law, and by providing dignity to ` 1500
\ Price of 100 kg wheat =
life irrespective of any caste, class, religion or any 120
biasness. For the upliftment of depressed classes, ` 1500
Hence, required minimum support price =
Constitution in India provides reservation under Article 120
19(9). = ` 12.5/kg
36 NTSE~ SOLVED PAPER 2018 (Stage II)

195. A country has four groups of people. The table 198. Which of the following statements are correct?
below describes some social indicators of these I. Globalisation has led to increased flow of capital
groups. Identify the group that is the most across countries.
vulnerable. II. Increase in flows of labour across countries has
Groups Literacy rate Life Unemployment been larger than the increase in flows of capital.
(%) expectancy rate (%) III. MNCs spread their production and work with
(years)
local producers in various countries across the
A 74 82 5 globe.
B 93 80 10 Codes
C 63 78 15 (a) I and II (b) I and III
(c) II and III (d) I, II and III
D 65 78 10
l (b) Globalisation is the ongoing process that is linking
(a) A (b) B (c) C (d) D people, countries, cities and regions much more closely
together than ever before. It also increased the business of
l (c) From the above data, it is clear that group C is the most many countries by flow of capital across countries.
vulnerable. Through globalisation many MNCs have spread their
196. Which of the following statements are correct? production and work with local producers in various
countries across the globe.
I. Bank deposits share the essential features of
money. 199. In a village Puranpur, 200 families are living. 85
II. Any depositor may demand his deposit at any families work on their own piece of land, 60 families
point of time from a bank. work on the field of other farmers, 5 families run
their own shops and 50 families work in a nearby
III. Bank must retain all deposits by itself.
factory to earn their livelihood. What percentage of
(a) I and II are true, but III is false.
(b) I is true, but II and III are false.
Purapur village depends on the secondary sector?
(c) I and II are false, but III is true. (a) 20 (b) 25 (c) 35 (d) 55
(d) All statements I, II and III are true. l (d) Total dependent persons on secondary sector
l (a) Bank is the bases of financial system. Bank is an = 50 + 60 = 110
institution to deposit money and provides loans. Deposit Total families in Pooranpur = 200
made by people in the bank are in the form of currency. 110
Hence the required Percentage = ´ 100% = 55%
Depositors can demand their deposits anytime from the 200
bank. Bank does not keep its deposit with itself and give it
as the loan. In this way bank earns profit. 200. Identify the correct pairs from List-I (Rights) and
List-II (Violation of rights) and select the correct
197. Bira and his wife Sheena have two daughters aged 12 option using the codes given below.
and 16. Sheena’s mother and father, aged 65 and 72,
also live with them. Bira is currently looking for work, List I (Rights) List II (Violation of rights)
but can’t find any. His elder daughter completed A. Right to Choose (i) Raman buys a packet of milk on
class 10 and prefers to look for work. Sheena prefers which the company’s name,
manufacturing date, and expiry
to stay at home to look after house works. How many date were missing.
unemployed members does Bira’s family have?
B. Right to be (ii) Sakina wants a particular
(a) 1 (b) 2 (c) 3 (d) 4
Informed channel from her cable operator
l (b) According to question, but operator offers some other
channel as part of a complete
s Couple r package.
65 Year 72 year
C. Right to Safety (iii) Joseph bought a television from
Mother Father a shop. He suffered electric
shock while using it.
Husband Sheena D. Right to Seek (iv) Murli felt ill and was admitted in
r Bira (She likes to stay at home)
Wife Redressal the hospital because of stale
food served in the restaurant.
er
Daughter ht
ug
Da
r

(a) A-I and C-III (b) B-II and C-III


hte

Daughter
ug

(c) B-II and D-IV (d) C-III and D-IV


Da

s
l (d) The correct order is
– Sister
(10th Pass and Unemployed) A - II B-I C - III D - IV
Hence, two persons are unemployed in the family.
NTSE~ SOLVED PAPER 2017 (Stage I) 1

NTSE Solved Paper


NATIONAL TALENT
SEARCH
EXAMINATION
2017 (Stage I)*

INSTRUCTIONS
This solved paper consists of two papers. Paper I consists of Mental Ability Test (MAT) and Paper II consists of
Scholastic Aptitude Test (SAT).
MAT covering (Q. Nos. 1-50) of Maths and (Q. Nos. 51-100) of Reasoning.
SAT consists of (Science, Mathematics and Social Science) which comprises 100 questions (40 Science, 20 Mathematics
and 40 Social Science).
There will be no negative marking.
Each correct answer will be awarded one mark.

Time : 240 Minutes Max. Mark : 200

Paper I Mental Ability Test (MAT)


Directions (Q. Nos. 1-4) Letter series is given with one 4. KLE, IND, GPC, ? ,CTA.
term missing shown by question mark ( ? ). This term is (a) DRB (b) BSE (c) ERB (d) ECR
one of four alternatives given under it. Find the right l (c) The pattern of given series is
alternative. –2 –2 –2 –2
K I G E C
1. G, K, O, S, ? +2 +2 +2 +2
L N P R T
(a) U (b) W
(c) V (d) X E
–1
D
–1
C
–1
B
–1
A
l (b) The pattern of given series is
G K O S W Directions (Q. Nos. 5-8) A number series is given with
one term missing shown by question mark (?). This term is
+4 +4 +4 +4 one of the four alternatives given under it. Find the right
alternative.
2. DX, HT, KQ, OM, ?
(a) SJ (b) RK 5. 4, 9, 25, ?, 121, 169
(c) QJ (d) RJ (a) 36 (b) 49 (c) 64 (d) 81
l (d) The pattern of given series is l (b) The pattern of given series is
+4 +3 +4 +3
D H K O R 4 9 25 49 121 169

–4 –3 –4 –3
X T Q M J
22 32 52 72 112 132
3. H, D, A, Y, X,? All the numbers of given series are square of prime
(a) X (b) W numbers.
(c) T (d) V
6. 1, 3, 7, 13, 21, ?, 43, 57
l (a) The pattern of given series is
–4 –3 –2 –1 –0
(a) 31 (b) 29 (c) 30 (d) 32
H D A Y X X

*Exam held on 5th November 2017


2 NTSE~ SOLVED PAPER 2017 (Stage I)

l (a) The pattern of given series is Arguments I. Yes; it helps in all-round development
1 3 7 13 21 31 43 57 of the child.
II. No, it puts more burden on teachers.
+2 +4 +6 +8 +10 +12 +14
(a) Arguments I and II both are strong
7. 5, 3, 10, 8, 17, 15, ?, 24 (b) Arguments I and II both are weak
(a) 25 (b) 23 (c) 26 (d) 27 (c) Argument I is strong and II is weak
(d) Argument I is weak and II is strong
l (c) The pattern of given series is
+5 +7 +9
l (c) After observing the statement, we got that argument I
is strong and II is weak.
5 3 10 8 17 15 26 24 11. In the question given below, a statement is followed
by a reason. Choose correct option for them.
+5 +7 +9
Statement Narmada river flows to west.
8. 97, 77, 59, ?, 29, 17 Reason Narmada river falls in the Bay of Bengal.
(a) 34 (b) 39 (a) Statement and reason both are true
(c) 37 (d) 43 (b) Statement is true but reason is false
l (d) The pattern of given series is (c) Statement is false but reason is true
97 77 59 43 29 17 (d) Statement and reason both are false
l (b) After observing the statement we got that statement is
–20 –18 –16 –14 –12 true and reason is false.
9. In the given question there are two statements and 12. Which of the following Venn diagrams correctly
they have two conclusions I and II. You have to take represents Bus, Car and Vehicle?
the given statements to be true even if they seem to
vary to commonly known facts. Read the
conclusions and decide which of the given
conclusions logically follows from the two given
statements even disregarding commonly known
facts. (a) (b) (c) (d)
Statements
l (c) Venn, diagram (c) represents correctly bus, car and
1. All women are intelligent. vehicle.
2. Some women are educated.
Conclusions Vehicle

I. All educated women are intelligent.


II. All intelligent are women.
(a) Only conclusion I is true
(b) Only conclusion II is true
Bus Car
(c) Both conclusions I and II are true
(d) Neither conclusion I nor conclusion II is true 13. Which of the following Venn diagrams correctly
l (a) According to the question, represents white colour, clothes and natural
flowers?
Intelligent

Women Or

Intelligent Educated Women Educated


(a) (b) (c) (d)
I ® True
l (a) Venn diagram (a) represent correctly white colour,
II ® False
clothes and natural flowers.
Thus, only conclusion I is true.

10. In the given question, a statement is followed by Natural Flowers


two arguments I and II. You have to decide which of
the following arguments is ‘strong’ or ‘weak’.
Statement Continuous and comprehensive
evaluation system should be implemented at school
Clothes White colour
level.
NTSE~ SOLVED PAPER 2017 (Stage I) 3
Directions (Q. Nos. 14-15) Out of 500 students, the 19. P, Q, R, S and T are sitting around a circular table
following Venn diagram represents the number of students facing centre to the table. R is just the right to P and
who got distinction in Physics, Chemistry and Maths is second to the left of S. T is not between P and S.
subjects. Who is second to the left of R?
(a) Q (b) S
(c) T (d) P
47 17 l (a) According to the question,
Chemistry 42
Physics
S
11
18 15

Q
T
Maths
50

14. How many students got distinction in both Physics


and Maths subjects, when the students who got P R
distinction in Chemistry subject is not included? Thus, ‘Q’ is second to the left of R.
(a) 26 (b) 15 (c) 28 (d) 24
20. If ‘<’ means ‘–’, ‘>’ means ‘+’, ‘=’ means ‘´’ and ‘$’
l (b) Number of student who got dictinction in only Physics means ‘¸’, then what will be the value of 27 > 81 $ 9
and Maths 15.
< 6?
15. What is the percentage of students who got (a) 6 (b) 36
distinction in all the three subjects? (c) 30 (d) 54
(a) 28% (b) 35% l (c) According to the question,
(c) 38% (d) 40% 27 > 81 $ 9 < 6 = 27 + 81 ¸ 9 - 6
l (d) Number of student in all three subject = 27 + 9 - 6
= 50 + 47 + 42 + 18 + 15 + 17 + 11 = 200 = 36 - 6
Q Total student in a class = 500 = 30
200
\ Required percentage = ´ 100% 21. The four different positions of the dice are given
500
= 40%
below. Which number is on the face opposite to 6?

Directions (Q. Nos. 16-17) In the following questions,


6 6 5 1
three alternatives are alike in a certain way but the rest
one is different. Find out the odd one and choose the 3 2 2 4 6 4 4 2
correct answer.
16. (a) Afghanistan (b) Kabul (1) (2) (3) (4)
(c) Spain (d) Iraq (a) 1 (b) 2
l (b) All except ‘Kabul’ are countries while ‘Kabul’ is the (c) 3 (d) 4
capital of Afghanistan. l (a) After observing all the given dice and got, 6 is adjacent
to face 2, 3, 4 and 5. So face ‘6’ is opposite to face ‘1’.
17. (a) 1 (b) 729 (c) 144 (d) 64
l (c) All except ‘144’ are perfect cube while ‘144’ is perfect 22. All faces of a solid cube of edge 8 cm are coloured.
square. It is divided equally in the cubes of edge 2 cm. How
many cubes will have all faces coloured ?
18. ‘A + B’ means A is the son of B. ‘A - B’ means A is (a) 1
the wife of B. Then what does P + R - Q mean? (b) 0
(a) Q is the father of P (b) Q is the son of P (c) 8
(c) P is the father of Q (d) R is the son of Q (d) 4
l (a) According to the question, l (b) If we divide a coloured solid cube whose edge is 8 cm
P + R-Q equally in cube of edge 2 cm, then there will be no cube
Couple formed which have all faces coloured.
– +
R Q
Father
23. In a coded language the word 'SOLID' is written as
Son
'HLORW’, then in the same code language 'GAS'
will be written as
P+ (a) THZ (b) TYI
(c) TZH (d) ZHT
Thus, ‘Q’ is father of ‘P’.
4 NTSE~ SOLVED PAPER 2017 (Stage I)

l (c) As, 28. Arrange the following in a meaningful sequence


Opposite
S O L I D H L O R W A. Medicine
Alphabet
B. Diagnosis
C. Doctor
D. Fever
E. Recovery
(a) D C A B E (b) D E C A B
Similarly, (c) D C B A E (d) C D B A E
Opposite
G A S
Alphabet
T Z H l (c) The meaningful sequence of given words is DCBAE.
1 2 3 4 5
Fever Doctor Diagnosis Medicine Recovery

D C B A E

24. If in a certain code I = 9 and GIRL = 46, then BOY 29. Find the missing number (?) from the given
=? alternatives, when same rule is applied in all three
(a) 37 (b) 39 (c) 24 (d) 42 situations.
l (d) As, I = 9 5 21 51
GIRL = 7 + 9 + 18 + 12 = 46
16 109 2 22 53 19 17 ? 48
Similarly,
BOY = 2 + 15 + 25 = 42 6 15 13

25. If Ranjana is the sister of the son of Sohan’s son, (a) 7 (b) 25
how is Ranjana related to Sohan? (c) 49 (d) 129
(a) Daughter (b) Sister
l (b) As (16 - 6)2 + (5 - 2)2
(c) Granddaughter (d) Uncle
= 102 + 32
l (c) = 109
Sohan
r
and (22 - 15)2 + (21 - 19)2
= 72 + 22
Granddaughter
= 53
Sohan’s Sonr
Similarly (17 - 13)2 + (51 - 48)2
= 42 + 32
Ranjanas = 25
Sonr Sister

Thus, ‘Ranjana’ is granddaughter of ‘Sohan’. 30. As ‘part’ is related to ‘whole’, in the same way an
‘Arc’ is related to which of the following?
26. If North direction is called East and South direction (a) Rectangle
is called West, then what will be called North-East (b) Circle
direction? (c) Triangle
(a) North-East (b) East-South (d) Square
(c) West-South (d) North-West
l (b) As, ‘part’ is related to ‘whole’, similarly ‘arc’ will be
l (b) According to question, related to ‘circle’.
North East
North-East East-South Directions (Q. Nos. 31-34) In the following questions,
West East North
there are two sets of figures. One set contains problem
South
figures while the other has answer figures. There is a
sequence according to which the problem figures are
South West
arranged. You have to select an answer figure which can
Thus, ‘North-east’ direction will be called ‘East-South’ be added in sequence with the problem figures. Choose
direction.
the correct figure.
27. How many pairs of successive numbers have a 31. Problem-figures
difference of 2 in the following sequence?
6, 4, 1, 2, 2, 8, 7, 4, 2, 7, 5, 3, 8, 6, 2, 1, 7, 0, 4, 1, 3, 2, 8, 6
(a) 4 (b) 5
(c) 6 (d) 7
l (d) There are 7 pair in given series whose difference is 2.
6, 4, 1, 2, 2, 8, 7, 4, 2, 7, 5, 3, 8, 6, 2, 1, 7, 0, 4, 1, 3, 2, 8, 6 (A) (B) (C) (D)
NTSE~ SOLVED PAPER 2017 (Stage I) 5
Answer-figures l (d) Answer-figure (d) will complete the sequence of given
problem-figures.

Directions (Q. Nos. 35-38) There are four figures given in


each. One of these does not correlate with the rest of the
figures. Select that odd figure.
(a) (b) (c) (d) 35.
l (d) Answer-figure (d) will complete the sequence of given
problem-figures.

32. Problem-figures
C J S U
(a) (b) (c) (d)

l (d) ‘U’ is odd figure in given figures, because all except ‘U’
are different in mirror image which is same as original in
mirror image.

(A) (B) (C) (D) 36.


Answer-figures

(a) (b) (c) (d)

(a) (b) (c) (d)


l (c) All except figure (c) are formed from rotation.

l (c) Answer-figure (c) will complete the sequence of given 37.


problem-figures.

33. Problem-figures

(a) (b) (c) (d)

l (a) Figure (a) is different from others.


38.
(A) (B) (C) (D)

Answer-figures

(a) (b) (c) (d)

l (d) Figure (d) is different from others because all the


shaded portion in figure (a), (b) and (c) digonally opposite.
(a) (b) (c) (d)
Directions (Q. Nos. 39-40) Find the correct mirror image
l (a) Answer-figure (a) will complete the sequence of given of the given figure, when mirror is placed on right side of
problem-figures.
the figure.
34. Problem-figures
39. STOP
(a) S T O P (b) PO T S (c) P T O S (d) P O T S

l (b) The correct mirror image of ‘STOP’ is

S TOP PO T S
(A) (B) (C) (D)

Answer-figures
40. Question-image

(a) (b) (c) (d)


6 NTSE~ SOLVED PAPER 2017 (Stage I)

Answer-image 44. Transparent-sheet

(a) (b) (c) (d)

l (d) Answer-figure (d) is correct mirror image of given Answer-figures


question-figure.

Directions (Q. Nos. 41-42) In questions below, select the


correct water image of the given figure.
41. Question-figure
(a) (b) (c) (d)

l (b) When we folded transparent-sheet along the dotted


Answer-images line, its appear as answer-figure (b).

Directions (Q.Nos. 45-46) In the following figures there is


a question figure, which is embedded in one of the answer
figures. Trace out the correct figure.
45. Question-figure
(a) (b) (c) (d)

l (b) Answer-figure (b) is correct water image of given


question-figure.
42. Question-image
X7W4
Answer-image Answer-figures
7W4 W 7 4 7 W4
(a) X (b) X7 4 (c) X W (d) X
l (a) Answer-figure (a) is correct mirror image of given
question-figure.

Direction : (Q. Nos. 43-44) A square transparent sheet


with a pattern is folded along the dotted line. Which of the (a) (b) (c) (d)
following answer figures is formed after folding the
l (c) Question-figure is embedded in answer-figure (c).
transparent sheet?
43. Transparent-sheet 46. Question-figure

Answer-figures Answer-figures

(a) (b) (c) (d) (a) (b) (c) (d)

l (c) When we folded transparent-sheet along the dotted l (d) Question-figure is embedded / hidden in answer-figure
line, its appear as answer-figure (c). (d).
NTSE~ SOLVED PAPER 2017 (Stage I) 7
47. Which of the answer figures completes the given Directions (Q. No. 51- 54) Read the passage and answer
matrix figure? the questions based on it.
Of all the trees of Southern Asia, the banyan is unique, not
only for the manner of its growth, but for the area of shade it
provides from the burning sun. Its close relationship with
man has evolved over the years to make the banyan a
popular meeting place, a focal point of worship and a source
of practical materials for commerce.
Known as the ‘Strangler fig’ because of its unusual manner of
growth, the banyan is an epiphyte or air plant, that has its
birth in the branches of a host tree and lives on airborne
moisture and nutrients. Banyan seeds are deposited by birds,
bats or monkeys in the rich soil collected in the crevices of
host tree branches.
As the banyan grows, it sends aerial roots down the trunk of
the supporting tree. In time, the roots that reach the ground
choke the host tree by preventing its trunk from enlarging.
The two best known species of banyans are : the Indian, one
(a) (b) (c) (d) of the world’s largest tropical trees; and the Chinese, a
smaller species with fewer aerial roots.
l (a) In each row, both the elements interchange their
position and the element at the right side rotates by 180°. 51. The banyan tree is unique for
(a) it grows in Southern Asia
48. If 20 * 3 = 180 and 4 * 5 = 100, then what is the (b) it is a small tree
value of 7 * 7 ? (c) its growth is unusual and it gives shade for big area
(a) 21 (b) 49 (d) its fruit is very big
(c) 343 (d) 7
l (c) As mentioned in the paragraph, Banyan tree is unique
l (c) As because of its manner of growth and it gives shade for big
20 * 3 = 20 ´ 32 = 180 area.
4 * 5 = 4 ´ 52 = 100 52. The banyan tree is a focal point of worship for
Similarly, (a) birds (b) wild animals
7 * 7 = 7 ´ 72 = 343 (c) nature (d) man
49. Determine the number of squares in the following l (d) The banyan tree is a focal point of worship for man.
figure 53. Owing to its unusual manner of growth, the banyan
tree is called
(a) a place of worship (b) a place of rest
(c) a strangler fig (d) a tree of beauty
l (c) The tree is named thus because of the manner in which
its roots choke the roots of the host tree.

(a) 14 (b) 9 54. The two species of banyan trees are


(c) 10 (d) 16 (a) the Indian and the Chinese (b) the shady and the big
(c) the aerial and the land (d) the host and the guest.
l (a) Number of smallest square = 9
Number of squares formed by combine four smallest
l (a) The two species of banyan trees are the Indian Banyan
square = 4 and largest square = 1 and the Chinese Banyan.
\ Total number of square in given figure
Directions (Q. No. 55- 90) Choose the suitable option for
= 9 + 4 + 1 = 14 each of the given questions.
50. How many cubes are laid on a plane as shown in 55. The aerial roots ……… the trunk of the host tree.
the following figure? (a) support (b) grow (c) choke (d) enlarge
l (c) Choke
56. A doctor ……… the patients but God cures all.
(a) treating (b) treated (c) treats (d) had treated
l (c) ‘Treats’ is the correct answer as the sentence is in simple
present tense.

(a) 14 (b) 12 57. While ……… basketball Sunil was badly hit.
(c) 10 (d) 8 (a) play (b) playing (c) played (d) had played
l (c) Number of cubes are laid on a plane = 6 + 3 + 1 = 10 l (b) The word playing has been used as present participle.
8 NTSE~ SOLVED PAPER 2017 (Stage I)

58. ……… slow, this is school area. 67. The policeman said to my mother, “May I help you?”
(a) Drive (b) Is driving (c) Drove (d) Driven The policeman asked my mother
l (a) Drive is the correct answer. (a) if she need his help (b) may he help you
(c) that he might help her (d) if he might help her
59. ‘Applaud’ means
(a) remark (b) backbite l (d) ‘The policeman asked my mother if he might help her’
is the correct sentence. It is in Indirect speech as the
(c) consent (d) praise
principal verb is in past tense.
l (d) Praise is the synonym of ‘applaud’.
The cricketer was applauded/praised for his speedy 68. The landlord commanded, “You ……… not enter my
century. house again.”
(a) can (b) shall (c) may (d) could
60. Find the odd one out.
(a) advance (b) promote (c) forward (d) farther
l (b) ‘Shall’ is the correct word to be filled here. Here, ‘shall’
is used to give order.
l (d) ‘Farther’ word is odd one here as advance, promote or
forward means ahead in position or time whereas farther 69. One ……… be punctual.
means a long way or a great distance. (a) can (b) may (c) should (d) will
61. The new slogan ……… by our school teacher last l (c) ‘Should’ is the correct modal to be filled in the blank. It
year. is used to show ‘obligation’.
(a) is given (b) is being given 70. The synonym of ‘benevolent’ is
(c) was given (d) was being given (a) honest (b) holy (c) generous (d) cruel
l (c) Sentence is in passive voice (simple past) so we will l (c) ‘Generous’ is the synonym of benevolent. Both words
use ‘was given’. mean charitable.
62. The notorious band ……… today. For example, the ‘benevolent’ gentleman donated
(a) has been arrested (b) had been arrested regularly in the orphanage.
(c) was been arrested (d) will have arrested 71. The antonym of ‘realistic’ is
l (a) Use of today indicates present perfect tense. So other (a) imaginary (b) true
options do not match here except (a). (c) reliable (d) authentic
63. Let the picture l (a) ‘Imaginary’ is antonym of realistic, which means
(a) is hung (b) being hung natural or authentic whereas imaginary means which is
(c) be hung (d) been hung unreal or non-existent.
For example, We had a realistic chance of winning. He
l (c) ‘Be hung’ is the correct the answer as sentence is in lives in an imaginary world.
passive voice (imperative type).
72. Gold and silver ………... precious metals.
64. Those cars ……… by robots years ago. (a) is (b) are (c) was (d) am
(a) are being built (b) are built
(c) were built (d) had built l (b) ‘Are’ is correct answer as with plural numbers. We use
‘are’ in simple present tense.
l (c) ‘Were built’ it the correct answer as the sentence is in
passive voice (simple past tense). 73. The wages of sin ……… death.
(a) is (b) are (c) were (d) am
65. The sages said, “You will be winner”.
The sages said l (a) Wages appear to be plural in form but it is singular in
meaning. Wages means salary so use of ‘is’ is correct here.
(a) if he would be winner (b) that he would be winner
(c) that he will be winner (d) that they would be winner 74. A rupee has ………. hundred paise.
l (b) ‘The sages said that he would be winner’ is the correct (a) the (b) a (c) an (d) any
sentence. It is in Indirect speech. Principal verb ‘said’ is in l (b) We pronounce hundred with ‘H sound, therefore we
simple past so whole sentence will be in past tense only. will use ‘a’ instead of ‘an’.
66. Sarla said to me, “I need your help today”. 75. Please give me …………… more tea.
Sarla told me (a) few (b) many
(a) that she needed my help that day (c) some (d) any
(b) that I needed her help that day l (c) ‘Some’ is the correct answer as some shows quantity
(c) that she needed your help that day which is uncountable.
(d) that you needed my help now
76. How ……… money do you need? She asked the boy.
l (a) ‘Sarla told me that she needed my help that day’ is the
correct sentence. It is in Indirect Speech as the principal (a) many (b) some (c) any (d) much
verb is in past tense. l (d) Much is used in uncountable noun. Here money is
uncountable noun.
NTSE~ SOLVED PAPER 2017 (Stage I) 9
77. The correctly spelt word is 89. The king was happy ……… people gave him gifts.
(a) comepasion (b) compassion (a) where (b) when (c) whom (d) who
(c) comepation (d) compasion
l (b) ‘When’ is the correct adverb to be used in the sentence.
l (b) ‘Compassion’ is the correctly spelt word here which
means ‘pity’ or ‘sympathy’. 90. She broke the glass ……… was very expensive.
(a) what (b) how (c) who (d) that
78. The correctly spelt world is
(a) depandence (b) deppendance l (d) ‘That’ is correct option. It is used to elaborate details
(c) dependence (d) dependance about glass (adjective clause).
l (c) ‘Dependence’ is the correctly spelt word which means Directions (Q. Nos. 91-93) Reorder the words to make
‘state of relying on’ or ‘helplessness’.
sentences.
79. The Piper stepped ……… the street.
(a) on (b) in (c) into (d) for 91. is / my / democracy / ideal / political
l (c) ‘Into’ is the correct preposition to be used with verb
(1) (2) (3) (4) (5)
stepped. (a) 1 3 2 5 4 (b) 2 5 4 1 3 (c) 4 2 5 1 3 (d) 1 4 5 3 2

80. The sailors sailed ……… the river. l (b) My political ideal is Democracy.
(a) on (b) down (c) below (d) above 92. ideal/were/ as an/couple/ my parents/regarded
l (a) ‘On’ is the correct preposition as a ship sails on a river (1) (2) (3) (4) (5) (6)
meaning/showing its current location. (a) 5 2 6 3 1 4 (b) 3 1 4 6 5 2 (c) 2 4 1 5 6 3 (d) 5 4 6 3 1 2
81. The mail train is due ……… 3 p.m. l (a) My parents were regarded as an ideal couple.
(a) on (b) of (c) by (d) in
93. the model/we/the outlook/of/changed
l (c) ‘By’ is the correct preposition to be used to make the
sentence meaningful. (1) (2) (3) (4) (5)
(a) 2 1 5 4 3 (b) 4 1 5 3 2 (c) 2 5 3 4 1 (d) 1 5 3 2 4
82. The tiger is now confined …………… national parks.
l (c) We changed the outlook of the model.
(a) to (b) off (c) at (d) of
l (a) ‘To’ is used after verb confined other options are not 94. The correct suffix that goes with the word ‘charge’ is
suitable. For example, The dog is confined to the floor. (a) - ment (b) - ity (c) - ish (d) - able

83. He learnt to play physical games. ……… ? l (d) ‘Able’ is the correct suffix that goes with ‘charge’. The
(a) Did he (b) Didn’t you word becomes chargeable.
(c) Doesn’t he (d) Didn’t he 95. The correct prefix that goes with the word ‘legal’ is
l (d) ‘didn’t he’ is the correct question tag to be used with a (a) un- (b) il- (c) in- (d) im-
sentence which is simple past tense (affirmative). l (b) The correct prefix with the word is ‘Il’ which makes the
84. I ain’t interested in video games. ……… ? word ‘Illegal’.
(a) Is you (b) Aren’t you (c) Am I (d) Are you
Directions (Q. Nos. 96-97) Choose the correct meaning of
l (c) ‘Am I’ is the correct question tag to be used here with a the given phrasal verbs.
sentence in simple present tense (negative type).
96. ‘Call for’
85. Be just ……… fear not. (a) shout (b) allow (c) require (d) give
(a) or (b) but (c) and (d) if
l (c) ‘Require’ is the most appropriate answer. For example,
l (c) Use of conjunction ‘and’ is appropriate here. Other His plan will call for a lot of money.
options are not suitable.
97. The company ‘turned down’ his application.
86. Catch me ……… you can. (a) rejected (b) put on table (c) accepted (d) entered
(a) or (b) but (c) and (d) if
l (a) Rejected is the correct meaning of phrasal verb ‘turned
l (c) ‘If’ is correct option be used in the blank for making the down’.
sentence meaningful.
98. The correct example of homophones is
87. Nalini’s grandfather died ……… she was born. (a) care - cure (b) liar - layer
(a) but (b) before (c) and (d) so (c) except - accept (d) gold - glad
l (b) ‘Before’ preposition is appropriate here to show an action l (c) Except - Accept is correct example of Homophone.
which took place earlier than a certain point of time. Homophones are two or more words having the same
pronunciation but different meanings.
88. The police caught the thief ……… looted travellers
Except means — not including (Every one except Shivani
in the passenger trains. were happy).
(a) which (b) whom (c) who (d) whose
Accept means — receive with approval. (He accepted the
l (c) ‘Who’ is correct option which used for ‘thief’ mentioned invitation).
in the sentence.
10 NTSE~ SOLVED PAPER 2017 (Stage I)

99. A person who seeks to promote the welfare of 100. A poem consisting of fourteen iambic pentameter
others is lines is
(a) atheist (b) philanthropist (a) lyric (b) ballad
(c) fatalist (d) pessimist (c) ode (d) sonnet
l (b) Philanthropist l (d) Sonnet

Paper II Scholastic Aptitude Test (SAT)


101. Consider the following five graphs (note the axes 103. A ball is shot vertically upward with a given initial
carefully). Which of the following represents motion velocity. It reaches a maximum height of 100 m. If
at constant speed? on a second shot, the initial velocity is doubled, then
the ball will reach a maximum height of
(a) 70.7 m (b) 141.4 m (c) 200 m (d) 400 m
Distance

Speed

l (d) Given, maximum height (H) = 100 m


From question, if on a second shots, the initial velocity
(u) gets doubled,
Time Time So, H µ (u)2
(A) (B) 2
H æ u1 ö
=ç ÷ (Here u2 = 2u1 )
H ¢ è u2 ø
Acceleration

2
H æ u1 ö H u
=ç Þ = 1
Velocity

÷
H ¢ è 2u1 ø H ¢ 4u1
H 1
Þ =
Time Time H¢ 4
(C) (D) 100 1
Þ = = 400
H¢ 4
Hence, maximum height (H¢) achieved by ball at second
shot will be 400 m.
Acceleration

104. Let M denotes the mass of earth and let R denotes its
radius. The ratio g / G at earth’ s surface is
Time (a) R 2 / M (b) M / R 2 (c) M / R (d) R / M
(E)
GM
l (b) Acceleration due to gravity (g) =
(a) D only (b) D and E R2
(c) A, B and C (d) A and D where, G = universal gravitational constant,
l (d) If an object covers equal distances in equal intervals of R = radius of Earth
time, then it is said to be moving at constant or uniform and M = mass of Earth
speed.
Ratio of G/g at the surface of Earth,
Graph A represent distance-time constantly starting from GM
origin. g 2 M
Similarly, graph D represent velocity-time constantly. = R = 2
G G R
Hence, graphs A and D represent the motion at constant
speed. 1
\ M / R2
102. A bullet of mass 50 g is horizontally fired with a
velocity 100 ms-1 from a gun of mass 10 kg. What 105. The unit ‘hertz’ is same as
will be the recoil velocity of the gun ? (a) second (b) second –1 (c) metre (d) metre –1
-1 -1 -1
(a) 100 ms (b) 500 ms (c) 0.5 ms (d) Zero l (b) Hertz is the SI unit of frequency. It is defined as the
l (c) Given, mass of bullet (m2 ) = 50 g = 0.5 kg = 50 ´ 10 -3 kg number of complete waves (or oscillations) produced in
one second or the number of vibration per second. So, it
Mass of gun (m1 ) = 10kg
becomes inverse of second.
and final velocity (v2 ) = 100ms -1 1
According to law of conservation of momentum, The correct answer is , i.e. second-1 .
second
m1 v1 + m2 v2 = m1 v1 + m2 v2
- m1 v1 = m2 v2 106. A sound wave has a frequency of 10 kHz and
-10 ´ v1 = 50 ´ 10 -3 ´ 100 wavelength 3 mm. How much time will it take to
50 ´ 10 -3 ´ 100 travel 3m?
v1 = = 0.5 (a) 0.1 s (b) 1 s
10
(c) 10 s (d) 0.01 s
Therefore, recoil velocity of gun (v) = 0.5 ms -1 .
NTSE~ SOLVED PAPER 2017 (Stage I) 11
l (a) Given, frequency (n) = 10 kHz = 10 ´ 103 Hz l (b) For the given circuit, we find the resistance
Wavelength (l) = 3mm = 3 ´ 10-3 m 1Ω 2Ω
We know that,
Speed of wave (v) = nl 1Ω 3Ω 2Ω
A B
v = 10 ´ 103 ´ 3 ´ 10 -3
1Ω 2Ω
v = 30 m/s
Distance 3m 1
\ Time = = = s = 01. s Circuit 1 Circuit 2
Speed 30m/s 10 Ciruit 1 (in parallel combination),
1 1 1 1
107. The size of image formed by a concave mirror is = + +
R R1 R2 R3
same as the size of object. The position of the object
1 1 1 1
will be = + +
(a) at F (b) between F and C R 1 1 1
1
(c) at C (d) between C and infinity \ R=
3
l (c) At centre of curvature (C), it gives real, inverted and
equal to the object image. When we place an object at F Circuit 2 (in parallel combination),
1 1 1 1
(focus) in concave mirror, it gives real, inverted and = + +
extremely magnified image of object. R 2 2 2
1 1 +1 +1
When we place image in between focus (F) and centre of =
inverted curvature (C), it gives real and bigger than object R 2
image. Hence, it will form the image of same the size of 2
\ R=
object. 3
When we place image in between centre of curvature (C) Now, the equivalent resistance between points A and B
and infinity, it gives real, inverted and diminished image. 1 2 12
Req = + 3 + = = 4W
3 3 3
108. A convex has focal length 30 cm. If an object is
placed at a distance of 15 cm from it, then the 111. 4 bulls rated 100 W each, operate for 6 hours per
magnification produced by the lens is day. What is the cost of the energy consumed in
(a) 6.66 (b) 0.5 (c) 1 (d) 2 30 days at the rate of ` 5 / kWh ?
l (d) Given, focal length (f) = 30 cm (a) ` 360 (b) ` 90 (c) ` 120 (d) ` 400
Distance of object (u) = 15 cm l (a) Given,
Magnification (m) = ?
No. of Device Power (kW) Time Energy
By Lens formula, devices (Hour) (kWh)
1 1 1 1 1 1
- = Þ = + 100 12
v u f v f u 4 Bulb ´4 6
1 1 1 1000 5
Þ = + (Q it is convex lens, so u = -15 cm)
v 30 (-15) 12
Energy consumed in 1 day = kWh
Þ v = - 30 cm 5
v -30 12
Magnification (m) produced by lens = = =2 \ Energy consumed in 30 days = ´ 30 = 72 kWh
u -15 5
As, 1 kWh rate of ` 5
109. The electrical resistivity of a conducting wire is K. If
\ 72 kWh = 72 ´ 5 = ` 360
its length and area of cross-section are doubled,
then the new resistivity of the wire will be 112. An electric current is passed through a straight
(a) K (b) 2K (c) K / 2 (d) K / 4 wire. Magnetic compasses are placed at the points
l (a) Resistivity of a conductor is defined as the resistance of A and B. True statement is
a conductor of unit length and unit area of cross-section. A
The resistivity of a material does not depend on its length B
(l) and area of cross-section ( A), i.e. it is independent.
(a) their needles will not deflect
110. What is the equivalent resistance of the given circuit (b) only one of the needles will deflect
between points A and B ? (c) both the needles will deflect in the same direction
1Ω 2Ω (d) the needles will deflect in the opposite direction
l (d) An electric current through a metallic conductor
1Ω 3Ω 2Ω produces magnetic field around it. The direction of
A B magnetic field produced by the electric current is reversed.
1Ω 2Ω Hence, when a magnetic compass is placed in above figure,
the direction of magnetic field will be opposite as given.
(a) 10W (b) 4W A
14 17 B
(c) W (d) W
3 6
12 NTSE~ SOLVED PAPER 2017 (Stage I)

113. Which diagram below illustrates the path of a light 119. The polymer formed by condensation of adipic acid
ray as it travels from a given point X in air to and hexamethylene diamine is
another given point Y in glass? (a) isoprene (b) rayon (c) terylene (d) nylon-6, 6
Y
Glass
l (d) The polymer formed by condensation of adipic acid
(a) Glass (b) Y
and hexamethylene diamine is nylon-6,6. The numerals
6,6 represent number of carbon atoms in the monomers.
Air Air
120. The method for separation of mixture of common
X X
salt and ammonium chloride is
(a) fractional distiliation (b) sublimation
Glass Y Glass Y (c) chromatography (d) crystallisation
(c) (d)
l (b) Ammonium chloride sublimates at room temperature
Air Air while sodium chloride (table salt) does not. Hence,
sublimation can be applied to separate the mixture of
X these two.
X
l (d) When a light travels from a rarer medium to a denser 121. Number of molecules present in 14 g of N2 molecule is
one, it bends towards the normal (i>r) and when travels (a) 6.022 ´ 1023 (b) 3.011 ´ 1023
from a denser medium to a rarer medium, it bends away . ´ 1023
(c) 151 (d) 6.022 ´ 1022
from the normal (i< r).
l (b) 1 mole of N 2 (i.e. 28 g of N 2 ) contains 6.022 ´ 1023
114. Conjugate base of HCl in the following reaction is molecules (Avogadro number).
6.022 ´ 1023
HCl ( aq ) + H2O —® Cl -( aq) + H3O+ 14 g of N2 = ´ 14 = 3.011 ´ 1023
(a) H3O + (b) H2O 28
(c) Cl – (d) HCl Hence, 14 g of N2 (0.5 mol) will have 3.011 ´ 1023
molecules.
l (c) Conjugate base of any species is obtained by removing
H+ ion from that species. Hence, Cl - ion is the conjugate 122. Which of the following elements has an electronic
base of HCl. configuration 2, 8, 6 ?
(a) Sulphur (b) Oxygen
(c) Phosphorus (d) Chlorine
115. The chemical formula of Plaster of Paris is
(a) CaSO 4 (b) CaSO 4 .2H2O l (a) The electronic configuration 2,8,6 represents atomic
(c) CaSO 4 .1/2H2O (d) CaSO 4 .H2O number16 (2 + 8 + 6 = 16), which is the atomic number of
sulphur.
l (c) Chemical formula of Plaster of Paris is represented as
CaSO4 ×1 / 2 H2 O or (CaSO4 )2 × H2 O. 123. Which of the following elements shows variable
116. Which type of catalyst is ethanol in the following valency?
(a) Na (b) Mg (c) Fe (d) Zn
reaction?
C2 H 5 OH
CHCl 3 + O2 ¾ ¾ ¾ ¾® 2 COCl 2 + 2 HCl l (c) Iron (Fe) shows variable valency, while all the given
elements show only one type of valency.
(a) Positive catalyst (b) Negative catalyst
(c) Bio-catalyst (d) Autocatalyst Element Valency
l (b) For the given reaction, ethanol (C2 H 5 OH) acts as a Iron (Fe) 2, 3
negative catalyst as it decreases the rate of reaction. Sodium (Na) 1
Magnesium (Mg) 2
117. Metalloid among the following is
Zinc (Zn) 2
(a) lithium (b) sulphur
(c) sodium (d) silicon
124. Formula of aluminium carbonate is
l (d) Silicon is a metalloid. Lithium and sodium are metals, (a) Al 2 (CO 3 )3 (b) Al 2CO 3 (c) Al 2HCO 3 (d) AlCO 3
while sulphur is a non-metal.
l (a) The chemical formula of aluminium carbonate is as
CH3 follows:
118. The IUPAC name of C == CH2 is Al CO3
CH3 º Al2 (CO3 )3
(a) 1, 1 dimethyl-2- ethene 3 2
(b) 2-methyl-1 propene
(c) 2, 2-dimethyl ethene 125. Formula of freon-112 is
(d) 2-methyl prop-2-ene (a) C 2F2Cl 4 (b) CF2Cl 2 (c) CFCl 3 (d) CCl 3F

l (b) IUPAC name for the given structure is l (a) Freon 112 is a refrigerant which has following structure
Cl F
2-methyl-1-propene.
CH 3 ½ ½
Cl ¾ C ¾ C ¾Cl
½ ½ ½
CH 3 ¾ C == CH 2 F Cl
3 2 1
NTSE~ SOLVED PAPER 2017 (Stage I) 13
126. The element X reacting with chlorine forms a water 133. The substance not essential for photosynthesis is
soluble compound having high melting point. (a) sunlight (b) chlorophyll
Element X is (c) nitrogen (d) carbon dioxide
(a) magnesium (b) argon (c) carbon (d) neon l (c) Among the given options, the substance that is not
l (a) Magnesium reacts with chlorine to form magnesium essential for photosynthesis is nitrogen. Photosynthesis is
chloride (MgCl 2) which is water soluble and has high a process in which solar energy is converted into chemical
melting point. energy. In this process, carbon dioxide and water react in
the presence of sunlight and chlorophyll to form glucose
127. Which tissue is found in fibrous covering of cocount? and oxygen is released as a byproduct.
(a) Parenchyma (b) Collenchyma Sunlight
6CO2 +12 H 2 O ¾¾¾® C6 H12 O6 + 6H2 O + 6 O2 ­
(c) Sclerenchyma (d) Meristematic tissue Chlorophyll Glucose
l (c) Sclerenchyma is found in fibrous covering of coconut.
It is a simple, dead and protective tissue with highly-thick 134. The nature of nerve impulse is
walled cells alongwith little or no protoplasm. The (a) chemical (b) magnetic
thickening of the wall may be made up of cellulose or (c) electrochemical (d) electromagnetic
lignin or both. l (c) The nature of nerve impulse is electrochemical.
A nerve impulse may be defined as wave of depolarisation
128. Nucleus of the cell was discovered by of the membrane of the nerve cell.
(a) Robert Hooke (b) Leeuwenhoek
(c) Robert Brown (d) Virchow 135. The examples of uricotelic animals are
l (c) Nucleus of the cell was discovered by Robert Brown in (a) fishes (b) reptiles
1831. It is a specialised double membrane bound largest (c) amphibians (d) mammals
cell organelle. It contains nucleolus and all the genetic l (b) The examples of uricotelic animals are reptiles. They
information for controlling cellular metabolism as DNA or excrete uric acid as nitrogenous waste to conserve water in
chromatin. their bodies, e.g. lizards, snakes, etc.
129. Which of the following is a plant hormone? 136. According to Mendel, in monohybrid cross the
(a) Insulin (b) Thyroxine (c) Cytokinin (d) Oestrogen genotypic ratio of F2 -generation is
l (c) Among the given options, cytokinin is a plant hormone. (a) 3 : 1 (b) 9 : 3 : 3 : 1 (c) 1 : 1 (d) 1 : 2 : 1
It is basic in nature, which is either aminopurine or phenyl l (d) According to Mendel, in monohybrid cross the
urea derivative. It promotes cell division. It is essential for genotypic ratio of F2 - generation is 1 : 2 : 1. In
tissue culture and promotes morphogenesis. F1 -generation all offspring are hybrid. On self-pollination
of F1 offsprings, in F2 - generation, one homozygous
130. Plant group more sensitive to the levels of sulphur dominant, two heterozygous dominant and one
dioxide in air is homozygous recessive trait organisms are obtained.
(a) Thallophyta (b) Lichen TT tt
(c) Pteridophyta (d) Gymnosperm (Tall) (Dwarf) P-generation

l (b) The plant group more sensitive to the levels of sulphur (Hybrid Tall)
Tt
dioxide in air is lichen. They (lichens) are dual organisms F1 -generation
Gametes
which possess a permanent symbiotic association of a T t
fungus or mycobiont and an alga or phycobiont. Lichens On selfing
T t
are not found in SO2 or acid rain affected areas.
T TT Tt
(Pure Tall) (Hybrid Tall)
131. Examples of perennial evergreen and woody plants F2 -generation
t Tt tt
are (Hybrid Tall) (Pure Dwarf)
(a) Funaria, Marchantia (b) Marsilea, Horsetail
(c) Cycas, Pinus (d) Ulothrix, Spirogyra Phenotypic ratio - 3 : 1
Genotypic ratio - 1 : 2 : 1
l (c) Among the given options, the examples of perennial,
evergreen and woody plants are Cycas and Pinus. They are 137. Example of connective tissue is
gymnospermic plants. Their seeds do not occur inside a (a) cartilage (b) skeletal muscles
fruit, thus, they remain exposed over the surface of the (c) skin of animals (d) nerve cells
megasporophylls.
l (a) Example of connective tissue is cartilage. It is a
132. Turgidity of cell is maintained by specialised connective tissue. It is found more abundantly
(a) Vacuole (b) Lysosome (c) Plastid (d) Golgi body in vertebrate embryos, while it is only found in the few places
of body of adult vertebrates, e.g. ear pinna, trachea, etc.
l (a) The turgidity of cell is maintained by vacuole. They are
non-cytoplasmic area present inside the cytoplasm which 138. The example of egg laying mammal is
are separated from the latter by specific membranes (a) bat (b) kangaroo
(Tonoplast). They content very high level of water which (c) pigeon (d) echidna
helps in maintaining turgidity of cell by osmoregulation.
l (d) The example of egg laying mammal is echidna. It
Depending upon the content and functions, vacuoles are
belongs to the sub-class Prototheria of class Mammalia. Its
of four types; viz sap vacuoles, contractile vacuoles, food
body is covered with strong pointed spines.
vacuoles and air vacuoles.
14 NTSE~ SOLVED PAPER 2017 (Stage I)

139. Non-communicable disease is 143. If a , b are zeros of polynomial x 2 - p ( x + 1) - k such


(a) cancer (b) AIDS (c) amoebiasis (d) jaundice that (a + 1)(b + 1) = 6, then value of k is
l (a) Among the given options, cancer is a (a) 5 (b) –1
non-communicable disease. It involves an abnormal and (c) –3 (d) –5
uncontrolled division in some cells. Cancer cells are
l (d) Given, a, b are zeros of polynomial.
different from normal cells in some aspects as in some
cases they do not remain confined to one part of the body x2 - p(x + 1) - k or x2 - px - (p + k)
and show metastasis. \ Sum of roots, a + b = p and ab = - (p + k) ...(i)
Since, (a + 1)(b + 1) = 6
140. Animals of which phylum are pseudocoelomate?
Þ ab + ( a + b) + 1 = 6
(a) Porifera (b) Platyhelminthes
(c) Aschelminthes (d) Mollusca Þ -(p + k) + p + 1 = 6 [using Eq. (i)]
l (c) The animals of Aschelminthes are pseudocoelomate Þ -k + 1 = 6
because their mesoderm is found as scattered pouches in Þ -k = 5
between the ectoderm and endoderm, and originated from Þ k = -5
blastocoeld. Such a body cavity is called pseudocoelom
and animals possessing them are called 144. Which is unit digit of 618 - 510 ?
pseudocoelomates, e.g. Ascaris. (a) 5 (b) 8 (c) 1 (d) 9
l (c) We have, 618
-5 10
3+2 3
141. If = a + 3 b, then the value of a + b, where Now, 61 = 6 , 51 = 5
3- 3 62 = 36, 52 = 25
a and b are rational numbers is 63 = 216, 53 = 125
(a) 5 (b) 8 (c) 2 (d) 16 64 = 1296, 54 = 625
3+2 3
l (c) Given, = a + 3b For every positive integral power of 6 and 5 having unit
3- 3 digit 6 and 5 respectively.
3+2 3 3+ 3
Þ ´ = a + 3b \ Unit digit of 618 - 510 = 6 - 5 = 1
3- 3 3+ 3
9 + 6 3 + 3 3 + 2 ´3 145. In the following figure QT^ PR and QS = PS. If
Þ = a+ 3b
9 -3 ÐTQR = 40º and ÐRPS = 20º, then value of x is
15 + 9 3 P
Þ = a + 3b
6
5 3
Þ + 3 = a + 3b
2 2 T
On comparing both sides; we get
5 3
a= , b=
2 2 x
5 3 8
Þ\ a+ b = + = = 4 =2
2 2 2
Q S R
142. For which positive values of k and p, equations (a) 80º (b) 25º (c) 15º (d) 35º
2 x 2 + px + 8 = 0 and p ( x 2 + x ) + k = 0 have equal l (c) We have, in DPQS
roots?
QS = PS
(a) k = 1, p = 4 (b) k = 2, p = 8
(c) k = 4, p = 8 (d) k = 2, p = 4 Þ ÐQPS = ÐPQS = x + 40°
P
l (b) Given equations, 2 x2 + px + 8 = 0 and p(x2 + x) + k = 20°
or px + px + k = 0 have equal roots.
2

\ Both have discriminant equal to zero. x+40°


T
\ P2 - 4 ´ 2 ´ 8 = 0
Þ P2 = 64
Þ P = ±8
Þ P=8 x
[Q P > 0] 40° 50°
Again, P - 4 pk = 0
2
Q S R
when P=8
Now, ÐQPR = ÐQPS + ÐRPS
(8)2 - 4 ´ 8 k = 0
= (x + 40° ) + 20° = x + 60°
Þ 32 k = 64
And also, in DQTR
Þ k =2
ÐTRQ = 180°-(ÐQTR + ÐTQS)
\ k = 2, p = 8
= 180°- (90°+40° ) = 180° - 130° = 50°
NTSE~ SOLVED PAPER 2017 (Stage I) 15
In DQPR, 1 12
Þ =
ÐQPR + ÐPQR + ÐPRQ = 180° (angle sum properly of tanq x
triangle) 1 12
Þ =
(x + 60° ) + (x + 40° ) + 50° = 180° 3/4 x
Þ 2 x + 150° = 180° 3
Þ x = 12 ´ =9m
Þ 2 x = 30° 4
Þ x = 15° cos A cos A
148. If m = and n = , then ( m2 + n2 ) cos2 B is
1 1 cos B sin B
146. Which term of AP 20, 19 , 18 , ..... is first negative
4 2 equal to
term? (a) m2 (b) n2 (c) m2 + n2 (d) m + n
(a) 18th (b) 15th (c) 28th (d) 27th cos A cos A
l (b) Given, m = and n =
l (c) Given AP, cos B sin B
1 1 æ cos2
A cos2 A ö
20,19 ,18 , ……… Now, (m2 + n2 )cos2 B = ç + ÷ × cos B
2

4 2 è cos B
2
sin2 B ø
77 77 - 80 -3 æ sin2 B + cos2 B ö
Here, a = 20, d = - 20 = = = cos2 Aç ÷ × cos B
2
4 4 4 è cos2 B sin2 B ø
Here, Tn = a + (n - 1)d
= cos2 Aæç 2 ö÷
1
-3 (Q sin2 x + cos2 x = 1)
= 20 + (n - 1)æç ö÷ è sin B ø
è4 ø
cos2 A
= 20 -
3 n 3 83 3 n
+ = - = = n2
4 4 4 4 sin2 B
For first negative term, Tn < 0 149. If ratio of heights of two similar triangles is 4 : 9,
83 - 3 n then ratio between their areas is
Þ <0
4 (a) 2 : 3 (b) 3 : 2 (c) 81:16 (d) 16: 81
Þ 83 - 3 n < 0
l (d) Given ratio of heights of two similar triangle is 4 : 9 We
Þ 83 < 3 n know that ratio of areas of two similar triangle is equal to
83 the ratio of square of their corresponding altitude.
Þ n>
3 2
\ Required ratio = æç ö÷ = 16 : 81
4
2
Þ n > 27 è9 ø
3
\ n » 28 150. In a circle of 10 cm radius, two chords
AB = AC = 12cm, then the length of the chord BC is
147. The angles of elevation of the top of a 12 m high
(a) 12 cm (b) 9.6 cm (c) 19.2 cm (d) 7.2 cm
tower from two points in opposite directions with it
are complementary. If distance of one point from its l (c) Given, radius of circle = 10 cm
base is 16 m, then distance of second point from and chords AB = AC = 12 cm
tower’s base is B
(a) 24 m (b) 9 m A
(c) 12 m (d) 18 m P
l (b) Let PQ be a tower of height 12 m and A and B are two
points in opposite directions with tower and angles of O
elevation are complementary. C
P

Here, AO is perpendicular bisector of BC and passes


through the centre O.
Now, in DABP, ÐP = 90°
12 m
AB2 = AP2 + BP2
θ (90 – θ) Þ BP2 = AB2 - AP2
A B = 122 - AP2 …(i)
16 m Q x
In DOPB, BO2 = BP2 + OP2
PQ 12 3 Þ BP2 = BO2 - OP2
Now, in DAPQ, tanq = = =
AQ 16 4 Þ BP2 = 102 - OP2
PQ 12
and in DBQP, tan(90 - q) = = = 100 - ( AO - AP)2
BQ x
= 100 - (10 - AP)2
12
Þ cot q = [where, x be the distance of = 100 - (100 + AP2 - 20 AP)
x
second point from tower’s base] = - AP2 + 20 AP …(ii)
16 NTSE~ SOLVED PAPER 2017 (Stage I)

From Eqs. (i) and (ii), we get Now, area of shaded region
122 - AP2 = - AP2 + 20 AP = 2 ´ area of quadrant ABPD - area of square ABCD
Þ 144 = 20 AP Þ AP = 7.2 550
=2 ´ - (10)2
Again from Eq. (i), we get 7
BP2 = 122 - (7.2)2 = 144 - 5184 1100 - 100 ´ 7 1100 - 700 400
. = = = cm2 (sq. unit)
= 92.16 7 7 7
Þ BP = 92.16 = 9.6 153. Find the capacity of a glass which is in the shape of
\ BC = 2 BP = 2 ´ 9.6 = 19.2 cm frustum of height 14 cm and diameters of both
151. If mean of 10 consecutive odd numbers is 120, then circular ends are 4 cm and 2 cm.
the mean of first five odd numbers among them is 308 298 3
(a) cm3 (b) cm
(a) 113 (b) 115 (c) 114 (d) 116 3 21
298 2
l (b) Let 10 consecutive odd numbers are (c) 112 cm2 (d) cm
21
x + 1, x + 3, x + 5, x + 7, x + 9,
l (a) Given, height of frustum, h = 14 cm
x + 11, x + 13, x + 15, x + 17, x + 19
and diameters of both circular ends are 4 cm and 2 cm
According to question,
(x + 1) + (x + 3) + (x + 5) + (x + 7) + (x + 9) \ 2 r1 = 4 Þ r1 = 2 cm
and 2 r2 = 2 Þ r2 = 1 cm
+ (x + 11) + (x + 13) + (x + 15) + (x + 17) + (x + 19)
= 120 Now, volume of frustum
10 ph 2
Þ 10 x + (1 + 3 + 5 + 7 + 9 + 11 +13 + 15 + 17 + 19) = 1200 = (r1 + r22 + r1 r2 )
3
10 x + 100 = 1200 Þ 10 x = 1100 22
´ 14
Þ x = 110 = 7 (22 + 12 + 2 ´ 1)
Now, mean of first five odd numbers 3
(x + 1) + (x + 3) + (x + 5) + (x + 7) + (x + 9) 22 ´ 2 44
= = (4 + 1 + 2) = ´7
5 3 3
5 x + 25 =
308
cm3
= = x+5
5 3
= 110 + 5 (Q x = 110) æ 23 33 ö
= 115 154. If a point P ç , ÷, divides line AB joining two
è5 5 ø
152. Find the area of shaded region, where side of points A(3, 5) and B( x , y ) internally in ratio of 2 : 3,
square ABCD is 10 cm and two arcs drawn from two then the values of x and y will be
opposite vertices of the sequare. (a) x = 4, y = 7 (b) x = 5, y = 9
D C (c) x = 7, y = 9 (d) x = 7, y = 8
æ 23 , 33 ö
l (c) ç ÷
è5 5 ø
P
A (3, 5) m n B (x,y)
Given, m : n = 2 : 3
A B Now, by using section formula
10 cm 2 ´ x + 3 ´ 3 23
=
200 400 2+3 5
(a) sq unit (b) sq unit
7 7 2 x + 9 23
600 100 Þ =
(c) sq unit (d) sq unit 5 5
7 7 Þ 2 x + 9 = 23
l (b) Area of sector, ABPD (quadrant ) Þ 2 x = 14
22 Þ x =7
´ 100
pr2 22 / 7 ´ (10)2 22 ´ 25 550 2 ´ y + 3 ´ 5 33
= = = 7 = = cm2 and =
4 4 4 7 7 2+3 5
D C 2 y + 15 33
Þ = Þ 2 y + 15 = 33
90° 5 5
Þ 2 y = 18 Þ y =9
P
155. If a leap year is elected randomly, then what is the
probability of having 53 Mondays in this year ?
Q 1 2 53 52
90° (a) (b) (c) (d)
A B 7 7 366 365
10 cm
NTSE~ SOLVED PAPER 2017 (Stage I) 17
l (b) In a leap year, there are 366 days. It means 52 full Now, GD is the median of DGBC
weeks + 2 days. These two days can be (Mon, Tues), \ ar (DBGD) = ar(DCGD)
(Tues, Wed), (Wed, Thurs), (Thurs, Fri), (Fri, Sat), (Sat, Þ ar (DCGD) = 1 …(iii)
Sun), (Sun, Mon)
Similarly ar (DCGE) = ar(DAEG) = 1 …(iv)
2
So, required probability = as GE is the median of D AGC
7
Now,
156. If the length of circumference of a circle is 60 cm ar (DDCEG) = or(DCGD) + ar (DAEG)
more than its diameter, then length of its = 1 + 1 = 2 cm2 [using Eqs. (iii) and (iv)]
circumference is
(a) 14p cm (b) 28 p cm 158. What is the radian value of angle 60 º 30 ¢ ?
(c) 35p cm (d) 42 p cm pc 121 c 121pc 121 c
(a) (b) p (c) (d) p
l (b) Let radius of circle be r. 3 360 180 540
Then, according to question ° °
(b) 60°30 ¢ = æç60 +
30 ö æ 1ö
l ÷ = ç60 + ÷
2 pr = 2 r + 60 è 60 ø è 2ø
Þ pr = r + 30 °
p
= æç
121 ö 121
Þ r(p - 1) = 30 ÷ = ´ radian
è 2 ø 2 180
30
Þ r= 121 121 c
p -1 = p radian = p
360 360
Now, circumference = 2pr
30 30 159. The diameter of a sphere is decreased by 25%. By
= 2p ´ = 2p ´
p -1 22 what per cent does its curved surface area decrease?
-1
7 (a) 25% (b) 56.25%
30 ´ 7 30 ´ 7 (c) 4375
. % (d) 62.5%
= 2p ´ = 2p ´
22 - 7 15 l (c) Let radius of sphere = r, diameter = 2r
= 28p cm Then, surface area of sphere = 4 pr2
Given, diameter of a sphere is decreased by 25%.
157. In given DABC, AD and BE are medians of triangle 75 3 r
New diameter = (2 r) ´ =
which intersect each other at point G. If area of 100 2
DBDG is 1 cm2 , then what is the area of DCEG ? and radius =
3r
A 4
New surface area of sphere
2
9 r2 9 pr2
= 4 p æç ö÷ = 4 p ´
3r
=
E è4 ø 16 4
G
9 p r2
7 pr2
\ Decrease surface areas = 4 pr2 - =
4 4
Per cent decrease in surface area
B D C
7 pr2
(a) 2 cm2 (b) 3 cm2 (c) 4 cm2 (d) 1 cm2
= 4 2 ´ 100
l (a) Given, AD and BE are medians of D ABC. 4 pr
A 7
= ´ 100
16
2 = 43.75%
E
2 G 160. Value of ( x - y )3 + ( y - z)3 + ( z - x )3 is
1
(a) ( x - y)3 + ( y - z)3 ( z - x)3
2 (b) 3( x - y)( y - z)( z - x)
B D C (c) x3 + y3 + z3 - 3 xyz
(d) x3 + y3 + z3 - 2 x2 y - 2 y2 z - 2 z2 x
1
\ ar ( ABD) = ar (DABC) …(i) l (b) Let a = x - y,
2
1 b = y - z,
and ar (DABE) = ar (DABC) …(ii)
2 and c = z - x
On subtracting Eq. (ii) from Eq. (i), we get Here,
ar (DABD) - ar (DABE) = 0 a + b + c = (x - y) + (y - z) + (z - x) = 0
Þ ar (DABG) + ar (DBGD) - ar (DABG) - ar (DAGE) = 0 Þ (x - y)3 + (y - z)3 + (z - x)3 = 3(x - y)(y - z)(z - x)
Þ ar (DBGD) - ar (DAGE) = 0 (Q If a + b + c = 0, then a3 + b3 + c3 = 3 abc)
Þ ar (DBGD) = ar(DAGE) = 1
18 NTSE~ SOLVED PAPER 2017 (Stage I)

161. Match List-I with List-II correctly and choose the 165. The year of the partition of Bengal was
correct code from the following. (a) 1903 (b) 1905 (c) 1907 (d) 1909
List-I List-II l (b) The British Government announced the idea for
Partition of Bengal in January 1904 on the grounds of
A. Meeting of the Estates General (i) 20th June, 1789 administrative reasons since Bengal was geographically as
B. Bastille was destroyed on (ii) 4th August, 1789 large as France and had a significantly larger population.
The partition took place on 16th October, 1905, by Viceroy
C. Abolishment of feudal system (iii) 14th July, 1789
Curzon. Now the province of Bengal was divided into two
in France
new provinces-‘Bengal’ (comprising West Bengal, Bihar
D. Swore of Tennis Court (iv) 5th May, 1789 and Odisha) and ‘Eastern Bengal and Assam’. But it was
not supported by the people of Bengal as they considered it
Codes as the ‘Divide and Rule’ policy of the British Government.
A B C D A B C D 166. Which one of the following countries was not among
(a) i ii iii iv (b) iv iii ii i
the Allied Powers ?
(c) iv i ii iii (d) i iv iii ii
(a) England (b) France (c) Russia (d) Germany
l (b) The Estates General was made up of different groups
of people called ‘Estates’. It was divided into First Estate, l (d) In World War II, there were two powers namely and
Second Estate and Third Estate. On 5th May, 1789, the Axis Powers. Allied Powers were comprised of Great
King Louis XVI called a meeting of the Estates General Britain, France, the Soviet Union, the United States of
because the French government was having financial America and China. The Axis powers were comprised of
problems. Germany, Japan and Italy.
The Bastille formally known as ‘Saint-Antoine, the 167. When was the publication of Bengal Gazette
Bastille’, was a fortress in Paris which was used as a state initiated?
prison by the kings of France. It was destroyed by a crowd
on 14th July, 1789, in the French Revolution, and became (a) 1750 (b) 1780 (c) 1850 (d) 1880
a symbol for the French Republican movement and later l (b) The Bengal Gazette was started in 1780. It was the first
replaced by the Place de la Bastille. major newspaper in English which published from Kolkata
Abolition of feudalism was an important event of the (then Calcutta), India. It was a weekly newspaper. It was
French Revolution. The National Constituent Assembly published for two years and was ceased by the British
announced on 4th August, 1789 that, ‘The National government on 23rd March, 1782.
Assembly abolishes the feudal system entirely.’
The Tennis Court Oath was also an important event in the 168. Consider the following points
French Revolution. It took place on 20th June, 1789 by the A. Mahatma Gandhi started Salt March with his 78
members of the French Estates-General or the Third
Estate, who called themselves the National Assembly. confidential volunteers.
They took the oath, that ‘‘not to separate, and to B. Mahatma Gandhi violated the Salt law at Dandi
reassemble wherever circumstances require, until the on 20th April, 1930.
Constitution of the kingdom is established’’.
Choose the correct answer from the codes given
162. The state of India where the Jallianwalla Bagh is below
situated, is (a) Only A (b) Only B
(a) Haryana (b) Uttar Pradesh (c) both A and B (d) None of these
(c) Punjab (d) Rajasthan
l (d) The Salt March, also known as Dandi March, was
l (c) Jallianwalla Bagh is situated in Amritsar, Punjab. It is a started by Mahatma Gandhi with his 79 volunteers (stated
public garden which was established in 1951 by the in the Gazateer of India. It was a peaceful Civil
Government of India. It commemorates the massacre of Disobedience Movement against the British Raj and which
peaceful celebrators by Brigadier General R.E.H. Dyer gave impetus to the Indian Independence Movement. It
along with British forces during British India, on 13th was a 25 day march which lasted from 12th March, 1930 to
April, 1919. 6th April, 1930 in Dandi, Gujarat where Gandhiji broke the
‘Salt Law’ by making salt from sea water.
163. The German King in 1871 was
(a) William I (b) Napoleon III 169. After which war the British rule was founded in
(c) Frederik William IV (d) Emmanuel II India?
l (a) William I was the King of Germany in 1871. He was (a) Battle of Sabrao (b) Battle of Panipat
the king of Prussia from the Hohenzollern dynasty. (c) Battle of Plassey (d) Second Anglo Mysore war
l (c) The Battle of Plassey led foundations of British rule in
164. Who discovered the spinning Jenny? India. In 1756, Siraj-ud-Daula succeeded his grandfather
(a) John Ke (b) T.E. Nicholson Alivardi Khan as the Nawab of Bengal who was worried by
(c) Raphael Samuel (d) James Hargreaves the growing power of the British East India Company, as
l (d) The spinning jenny was invented by James Hargreaves they were extending fortification.’ Siraj-ud-Daula ordered
in Stanhill, Lancashire in England in 1764. It is a them to stop this extension but they disobeyed the Nawab.
multi-spindle spinning frame, which had that great This disobedience was changed into Battle of Plassey and
importance in the industrialisation of weaving during the after defeating the Nawab by a conspiracy, the British
early Industrial Revolution. established their power over India.
NTSE~ SOLVED PAPER 2017 (Stage I) 19
170. When was the Great Economic Depression between 175. In which Indian forest are silver, fir and pine trees
the two World Wars held? found?
(a) 1921 (b) 1929 (a) Tropical deciduous forest
(c) 1935 (d) 1939 (b) Montane forest
l (b) The Great Depression was the worst and prolonged (c) Mangrove forest
economic downturn in the world history which affected the (d) Tropical evergreen rain forest
whole world and United States particularly. It lasted from l (b) Montane forest are found in mountainous areas. These
1929 to 1940s. This was a significant event between two forests are divided into Wet Temperate, Temperate and
world wars. Alpine forests. The important tree species of these forests
are silver, fir, pine, birches, fir, junipers, etc.
There were many reasons behind this. Some of these are:
176. Match List-I and List-II and choose the correct code
l
Speculation on borrowed money, which led to the
collapse of American Share Market. from the following
l
Rapid selling created further fall in prices. List-I List-II
l
Failure of banks to provide credit to peasants and A. Northern end (i) 8º 4¢ N
industries.
B. Southern end (ii) 37º 6¢ N
l
Economic inactivity due to First World War.
Over the next several years, this event remained in worse C. Eastern end (iii) 68º 7 ¢ E
condition as investment dropped, steep declines in D. Western end (iv) 97º 25¢ E
industrial output and employment.
Codes
171. Who composed Anandamath? A B C D A B C D
(a) Rabindranath Tagore (a) ii iii iv i (b) i ii iv iii
(b) Munsi Premchand (c) ii i iv iii (d) iii ii i iv
(c) Mahatma Gandhi
(d) Bankim Chandra Chattopadhyay l (c) India is a South-Asian country with area of 32, 87, 263 sq
km. It is situated on the north of the equator between 8°4 to
l (d) Anandamath is a Bengali fiction which was written by 37°6’ North latitude and 68°7’ to 97°25’ East longitude. It is
Bankim Chandra Chattopadhyay. It was published in the seventh largest country in the world in terms of area and
1882. second largest country in terms of population.
172. ‘Khadar’ is found in 177. Which of the following is the major sugarcane
(a) the Northern mountain region producing state?
(b) Thar desert (a) Uttar Pradesh (b) Rajasthan
(c) the vast Northern plain (c) West Bengal (d) Madhya Pradesh
(d) the Peninsular plateau
l (a) As per 2015-16 estimates, Uttar Pradesh is the largest
l (c) The floodplains formed by younger alluvium in the producer of sugarcane as it produces about 41.28% of the
Northern plains are called Khadar. The soil in this region all India production. Maharashtra comes at the second
is highly fertile as this is renewed every year. place with the production of 20.52% followed by Karnataka
(11%) and Tamil Nadu (7.5%).
173. The rising place of the largest river of Peninsular
plateau is 178. Important deposits of which mineral are found in
(a) Betul (b) Nasik Koraput in Odisha ?
(c) Jabalpur (d) Cuddalore (a) Iron ore (b) Coal (c) Copper (d) Bauxite
l (b) Godavari is the second longest river of India and l (d) Koraput is a district of Odisha which lies on the
largest river of Peninsular plateau. Its source lies in Eastern Ghats. It has rich and diverse types of mineral
Triambakeshwar, Nasik, Maharashtra. deposits like limestone, bauxite, China clay, gold, etc.
174. The quantity of rainfall received on the Western 179. In which year was the first successful cotton textile
Ghats by South-West monsoon is mill established in India?
(a) 100-150 cm (a) 1853 (b) 1854 (c) 1855 (d) 1856
(b) 150-200 cm
(c) 200-250 cm l (b) Although the first Indian cotton cloth mill was
(d) Above 250 cm established in 1818 at Fort Gloaster near Kolkata, but this
mill was a failure. In 1854, the first successful modern
l (d) Western Ghats is a mountain range that lies parallel to cotton mill ‘Bombay Spinning and Weaving Company’ was
the Western coast of Indian Peninsula. During the established by KGN Daber in Bombay, India.
Monsoon season (June to September) it receives a good
amount of rainfall (400-500 cm), as the continuous 180. Indian Population Policy 2000 not includes
Western Ghats chain acts as a barrier to the moisture laden (a) free education
Clouds. The Western Ghats do not allow the wind to cross (b) free from diseases
over without shedding their moisture on the Western (c) reducing infant mortality rate below 30
slopes. (d) increase the employment opportunities
20 NTSE~ SOLVED PAPER 2017 (Stage I)

l (d) The National Population Policy 2000 was a policy 185. Which of the following institutions can make
framework of achieving the set goals and prioritising changes to an existing law of our country?
strategies during the next decade. It laid emphasis on (a) Supreme Court of India (b) International Court of Justice
some issues related to public welfare like to meet the (c) Prime Minister (d) Parliament
reproductive and child health needs and to achieve net
replacement levels of TFR (Total Fertility Rate) by 2010. l (d) The Parliament of India has the power to enact a law or
Some of its goals are: bring amendments to the Constitution of India and can
l
To make School Education upto the age of 14 free and change the existing law with retrospective effect.
compulsory, and reduce drop outs at primary and
secondary school levels to below 20% for both boys and
186. Which one of the following is considered as a
girls. Fundamental Right according to the Constitution of
l
To reduce infant mortality rate to below 30 per 1000 live India?
births. (a) Right to Work
l
To achieve universal immunisations of children against (b) Right to Adequate Livelihood
all vaccine preventable diseases. (c) Right to Protect One’s Culture
l
To promote delayed marriage for girls, not earlier than (d) Right to get Higher Education
age 18 and preferably after 20 years of age and so on. l (c) The Fundamental Rights are sections of the
181. Gas transportation pipeline which passes through Constitution of India which provide basic rights to its
Kota in Rajasthan is citizens. There are six Fundamental Rights such as :
(a) Guwahati– Barauni– Allahabad– Kanpur 1. Right to Equality (Articles 14-18)
(b) Barauni– Rajbandh– Haldia 2. Right to Freedom (Articles 19-22)
(c) Hazira– Vijaipur– Jagdishpur 3. Right Against Exploitation (Articles 23-24)
(d) Salaya– Viramgam– Mathura– Delhi 4. Right to Freedom of Religion (Articles 25-28)
l (c) Hazira-Vijaipur-Jagdishpur gas pipeline is 1,750 km 5. Cultural and Educational Rights (Articles 29-30)
long. It was constructed by Gas Authority of India Limited Article 29 deals with the protection of language, script,
(GAIL) to transport gas. It connects Hazira in Maharashtra and culture of minorities.
to Vijaipur in Madhya Pradesh and Jagdishpur in Uttar Article 30 deals with the right of minorities to establish
Pradesh. It passes through Kota in Rajasthan. and administer educational institutions
182. In which State of India is red and yellow soil found? 6. Right to Constitutional Remedies (Article 32)
(a) Chhattisgarh (b) Rajasthan 187. Match the following in reference to Constitution
(c) Jammu and Kashmir (d) None of these making process
l (a) Red soil develops a reddish colour due to a wide
List-I List-II
diffusion of iron in crystalline and metamorphic rocks. It
looks yellow when it occurs in a hydrated form. Often, A. B. N. Rao (i) President of the Constituent
their upper layer is red and the lower layer is yellow. Assembly
These soils are found mainly along the piedmont zone of B. B.R. Ambedkar (ii) Member of the Drafting
the Western Ghats, parts of Odisha and Chhattisgarh and Committee
in the Southern parts of the middle Ganga plain. C. Rajendra Prasad (iii) Chairman of the Drafting
Committee
183. In which house is the Finance Bill presented first?
(a) Rajya Sabha D. T.T. Krishnamachari (iv) Legal Advisor
(b) Lok Sabha
(c) Both Lok Sabha and Rajya Sabha anywhere Codes
(d) Reserve Bank of India A B C D A B C D
(a) iv iii i ii (b) iv ii i iii
l (b) Finance Bill is introduced each year in Lok Sabha just (c) i iii iv ii (d) iii iv iii ii
after the presentation of the Union Budget. It is presented
in the Lower House or Lok Sabha to give effect to the l (a) The Constitution of India is the supreme law of India
financial proposals of the Government of India for the which consists of fundamental political principles,
immediately following financial year. procedures, powers and duties of government institutions,
Fundamental Rights, Directive Principles and the duties
184. Who among the following is a part of the political of citizens. It is the longest written Constitution in the
executive ? world. It was adopted by the Constituent Assembly on
(a) District Collector 26th November, 1949, and came into effect on 26th
(b) Secretary of the Ministry of Home Affairs January, 1950. There were many members who were
(c) Home Minister engaged in the Constitution making process like :
(d) Director General of Police l
Dr. B.R. Ambedkar- the Chairman of the Drafting
Committee
l (c) The political executive branch of Indian Government l
B.N. Rao- Legal Advisor
consists of President, Vice-President, Prime Minister and
Council of Ministers. Council of Ministers consists of
l
Dr. Rajendra Prasad- President of the Constituent
Defence Minister, Home Minister, External Minister, Assembly
Finance Minister and so on. l
T.T. Krishnamachari- Member of the Drafting
Committee.
NTSE~ SOLVED PAPER 2017 (Stage I) 21
188. Choose the correct statement describing the word 192. The Vice-President of India is elected by
‘Code of Conduct’. (a) elected members of Lok Sabha
A. A set of norms and guidelines to be followed by (b) all members of Rajya Sabha
Political Parties. (c) elected members of Lok Sabha and Rajya Sabha
(d) all members of Lok Sabha, Rajya Sabha and all state
B. A set of norms and guidelines to be followed by Legislative Assemblies
candidates in Election
l (c) The Vice-President of India is elected by an electoral
C. Guidelines for Election Commission college consisting of members of both Houses of
D. Compulsory voting for voters Parliament namely the Lok Sabha and Rajya Sabha
(a) A, B, C (b) A, B (c) B, C (d) C, D through the Single Transferable Voting System.
l (b) A ‘Code of Conduct’ is a set of norms and guidelines 193. Match List-I and List-II and choose the correct code
which are set for political parties and the candidates who from given codes.
contest election. These rules are expected to be followed
by each political party to act properly and their supporters List-I List-II
during an election campaign.
A. Union List (i) Computer Software
189. According to the Constitution of India, how many B. State list (ii) Communications
maximum number of judges can be appointed in
C. Concurrent list (iii) Police
Supreme Court?
(a) 29 + 1 (b) 30 + 1 D. Residuary powers (iv) Forests
(c) 28 + 1 (d) 31 + 1
Codes
l (b) The Supreme Court of India is the highest court of A B C D A B C D
appeal under the Constitution of India with the power of (a) iii ii i iv (b) ii iii iv i
Constitutional review. It consists of the Chief Justice of
(c) ii iv i iii (d) iv iii ii i
India along with 30 sanctioned other judges. It has
extensive powers of original, appellate and advisory l (b) The division of powers between the Union (Centre)
jurisdictions. and states is provided by the Constitution of India. There
are three lists under which subjects are divided between
190. How many members will be nominated in both the powers like
Legislative Council? l
The Union List 97 subjects under the control of central
1 1 1 1 Government. e.g. Defence, Foreign Affairs, currency and
(a) (b) (c) (d)
3 2 6 4 coinage, war and peace, communications, etc.
l (c) In bicameral legislatures, the Legislative Council or
l
The State List 61 subjects under the jurisdiction of
states. e.g. public order, police, state court fees, prisons,
Vidhan Parishad is the Upper House in some states of
local government, public health and sanitation,
India. According to Article 171 of the Constitution of India, hospitals and dispensaries, police, etc.
the strength of any council shall not exceed 1/3rd of the l
The Concurrent List 52 subjects under the joint control
strength of the State Assembly. But for smaller states, the
of the Centre and States. e.g. criminal law, criminal
minimum strength of the council can be 40. The people of procedure, preventive detention for reasons concerned
the Council are elected through five different with the security of state, marriage and divorce, transfer
constituencies : of property other than agricultural land, forests , etc.
l
One third (1/3rd) of the members are elected by l
Residuary Power The subjects which do not fall in these
representatives of the local authorities such as lists i.e. residuary subjects have been given to the
Municipalities, District Boards, etc. Centre. e.g., computer technology, hardware, software,
l
1/3rd members are elected by the members of the internet regulation, etc.
Legislative Assembly of the State.
l
1/12th members are elected by an electorate of 194. The example of capital is
university graduates. (a) water (b) forest (c) climate (d) machine
l
1/12th members are elected by the electorate consisting l (d) Capital is more durable than money and is used to
of the secondary school teachers. generate wealth through investment like machine tools,
l
1/6th members are nominated by the Governor on the automobiles, software and brand names, etc.
basis of their special knowledge/practical experience in
literature, art, science, cooperative movement or social 195. The rabi crop is
service. (a) Jowar (b) Bajra (Millet)
(c) Maize (d) Wheat
191. By which article of the Constitution of India is the
Prime Minister appointed? l (d) There are three major cropping seasons in India. These
are
(a) 74th (b) 75th (c) 52nd (d) 61st
(i) Rabi (October to March) – Important crops are wheat,
l (b) The Prime Minister of India is the head of the Council oat, barley, etc.
of Ministers and leader of the executive of the Government (ii) Kharif (July to October) – Important crops are rice,
of India. The Prime Minister is also the chief advisor to the millet, maize (corn), cotton, etc.
President of India. Under Article 75 (1), it is stated that (iii) Zaid (March to June) – Important crops are
‘‘the Prime Minister shall be appointed by the President’’. watermelon, cucumber, muskmelon, sunflower, etc.
22 NTSE~ SOLVED PAPER 2017 (Stage I)

196. In India, the currency note is issued by (ii) Secondary Sector includes manufacturing,
(a) Reserve Bank of India (b) State Bank of India construction works, etc.
(c) NABARD (d) Bank of India (iii) Tertiary Sector or Service Sector includes Banking
services, mass media, education,
l (a) The Currency Department in Reserve Bank of India information technology, etc.
has the sole right to issue currency notes in India.
199. The Government of India enacted the law of ‘Right
197. The source of institutional credit is to Information’ Act in
(a) money lender (b) landlord (a) October, 2005 (b) November, 2006
(c) bank (d) relatives (c) December, 2007 (d) January, 2008
l (c) Institutional sources of credit are the cooperative l (a) Right to Information (RTI) is an Act of the Parliament
societies, commercial banks, regional rural banks and of India which enacted on October 2005. It provides
state governments. These are the best sources of credit as people the right to request information from a ‘public
compared to private credit sources which charge high rate authority’ or a body of Government which is required to
of interest. reply within thirty days.
198. The example of tertiary sector is 200. The Multinational Company of India is
(a) agriculture (a) Infosys (b) Asian Paints
(b) fisheries (c) Tata Motors (d) All of these
(c) making sugar from sugarcane
(d) banking services l (d) Multinational Companies (MNCs) are business units
which operate and establish their offices simultaneously in
l (d) There are three sectors of economy different countries of the world. In India, some MNCs are
(i) Primary Sector includes agriculture, fishing, mining, Infosys, Tata Motors, Asian Paints, Parle Agro, Dabur, etc.
etc.
Part-1 : Verbal Reasoning
Chapter

1
Coding-Decoding
Coding means a method to transmit a message between the sender and the receiver without a third person knowing
it. Decoding means the method of making out the actual message that is hidden in coding. In questions based on
coding-decoding a word is coded in a particular way and the candidate are asked to code other word in the sameway.
The questions based on coding-decoding are of the following types

Type I Letter Coding


In these type of questions, the real alphabet letters in a word are replaced by certain other alphabet letters
according to a specific rule to form its code. The students are required to detect the common rule and answer the
questions accordingly.
It is always easy to handle numbers than alphabet letters. So, if there is a set of questions on alphabet coding in the
exam, it is better to use this table to decode the code.
Left to Right
A B C D E F G H I J K L M
1 2 3 4 5 6 7 8 9 10 11 12 13
N O P Q R S T U V W X Y Z
14 15 16 17 18 19 20 21 22 23 24 25 26

To Remember ! E J O T Y C F I L O R U X
5 10 15 20 25 3 6 9 12 15 18 21 24

Right to Left
Z Y X W V U T S R Q P O N
1 2 3 4 5 6 7 8 9 10 11 12 13
M L K J I H G F E D C B A
14 15 16 17 18 19 20 21 22 23 24 25 26

To Remember ! B G L Q V C F I L O R U X
25 20 15 10 5 24 21 18 15 12 9 6 3

Series of Opposite Alphabets


A B C D E F G
Z Y X W V U T
H I J K L M
S R Q P O N
î Sum of the positional numbers of opposite letters in the alphabet is always 27.
4 Study Package for NTSE

Example 1 If TAP is coded as SZO, then how will be FREEZE coded?


(a) EQDFYG (b) ESDFYF (c) GQFDYF (d) EQDDYD
Solution (d) Clearly, each letter in the word TAP is moved one step backward to obtain the corresponding letter of the
code.
S Z O
–1↑↑ –1 ↑ –1
T A P
Thus, in FREEZE, F will be coded as E, R as Q , E as D and Z as Y.
So, the code of FREEZE becomes EQDDYD.

Type II Number-Alphabet Coding


In these type of questions, either numerical code values are assigned to a word or alphabetical code letters are
assigned to the numbers. The students are required to analyse the code as per the directions given.
Example 2 If in a certain language A is coded as 1, B is coded as 2, C is coded as 3 and so on, how is BIDDIC coded
in that language?
(a) 294493 (b) 284563 (c) 375582 (d) 394492
Solution (a) As given the letters are coded as
A B C D E F G H I
1 2 3 4 5 6 7 8 9
So, in BIDDIC, B is coded as 2, I as 9, D as 4 and C as 3. Thus, BIDDIC is coded as 294493.

Example 3 In a certain code, 15789 is written as XTZAL and 2346 is written as NPSU. How is 23549 written in that
code?
(a) NPTUL (b) PNTSL (c) NPTSL (d) NBTSL
Solution (c) Looking into the method of coding, it is clear that 2 is coded as N, 3 as P, 5 as T, 4 as S and 9 as L.
So, 23549 is coded as NPTSL.

Type III Substitution Coding


In these type of questions, some particular objects are assigned codes. Then, a question is asked that is to be
answered in that code language.
Example 4 If ‘cook’ is called ‘butler’, ‘butler’ is called ‘manager’, ‘manager’ is called ‘teacher’, ‘teacher’ is called
‘clerk’ and ‘clerk’ is called ‘principal’, who will teach in the class?
(a) Clerk (b) Butler (c) Manager (d) Teacher
Solution (a) Clearly, a teacher teaches in a class and as given teacher is called clerk.
So, a clerk will teach in the class.

Type IV Mixed Letter Coding


In these type of questions, three or four complete messages are given in the coded language and the code for a
particular word is asked. To analyse such codes, any two messages bearing a common word are picked up. The
common code word will mean that word. Proceeding similarly by picking up all possible combinations of two, the
entire message can be analysed.
Example 5 If ‘nso ptr kli chn’ stands for ‘Sharma gets marriage gift’, ‘ptr inm wop chn’ stands for ‘wife gives
marriage gift’ and ‘tti wop nhi’ stands for ‘he gives nothing’, what would mean ‘gives’?
(a) chn (b) nhi (c) ptr (d) wop
Solution (d) nso ptr kli chn Sharma gets marriage gift .....(i)

ptr inm wop chn wife gives marriage gift .....(ii)

tti wop nhi he gives nothing .....(iii)

In the second and third statements, the common word is ‘gives’ and the common code word is ‘wop’. So, ‘wop’ means
‘gives’.
Coding-Decoding 5
Example 6 If ‘tee see pee’ means ‘drink fruit juice’, ‘see kee lee’ means ‘juice is sweet’ and ‘lee ree mee’ means ‘he is
intelligent’, which word in that language means ‘sweet’?
(a) see (b) kee (c) lee (d) pee
Solution (b) tee see pee drink fruit juice .....(i)

see kee lee juice is sweet .....(ii)

lee ree mee he is intelligent .....(iii)

In the first and second statements, the common word is ‘Juice’ and the common code word is ‘see’. So, ‘see’ means ‘juice’.
In the second and third statements, the common word is ‘is’ and the common code is ‘lee’. So, ‘lee’ means ‘is’. Thus, in the
second statement, the remaining word ‘sweet’ is coded as ‘kee’.

Type V Mixed Number Coding


In these type of questions, a few groups of numbers each coding a certain short message, are given. Through a
comparison of the given coded messages, taking two at a time, the candidate is required to find the number code for
each word and then formulate the code for the message given.
Example 7 In a certain code, ‘786’ means ‘study very hard’, ‘958’ means ‘hard work pays’ and ‘645’ means ‘study and
work’. Which of the following is the code for ‘very’?
(a) 8 (b) 6 (c) 7 (d) 5
Solution (c) 7 8 6 study very hard .....(i)

9 5 8 hard work pays .....(ii)

6 4 5 study and work .....(iii)

In the first and second statements, the common word is ‘hard’ and the common code digit is ‘8’, so ‘8’ means ‘hard’. In the
first and third statements, the common word is ‘study’ and the common code digit is 6, so ‘6’ means ‘study’. Thus, in the first
statement ‘7’ means ‘very’.

Miscellaneous Solved Examples


Example 8 If D = 4 and COVER = 63, then BASIC is equal to
(a) 49 (b) 50 (c) 34 (d) 55
Solution (c) In the alphabets, D is the 4th.
So, A = 1, B = 2 and C = 3 ⇒ COVER = 3 + 15 + 22 + 5 + 18 = 63
∴ BASIC = 2 + 1 + 19 + 9 + 3 = 34

Example 9 In a certain code, ‘RATIONAL’ is written as ‘ARITNOLA’. How would ‘TRIBAL’ be written in that code?
(a) TRIALB (b) RBTLAI (c) RTBILA (d) TIRABLE
Solution (c) As, R A T I O N A L

A R I T N O LA
Sim ilarly, T R I B A L

RT B I LA

Example 10 In a certain code, SIKKIM is written as THLJJL. How is TRAINING written in that code?
(a) SQBHOHOH (b) UQBHOHOF
(c) UQBJOHHO (d) UQBJOHOH
Solution (b) Clearly, the letters in the word SIKKIM are moved alternately one step forward and one step backward to
obtain the letters of the code.
So, in TRAINING, T will be coded as U, R as Q, A as B, I as H, N as O and so on. Thus, the code is UQBHOHOF.
6 Study Package for NTSE

Example 11 If DELHI is coded as 73541 and CALCUTTA as 82589662, how can CALICUT be coded?
(a) 5279431 (b) 5978213 (c) 8251896 (d) 8543691
Solution (c) Letter D E L H I C A U T
Code 7 3 5 4 1 8 2 9 6
The code for CALICUT is 8251896.

Example 12 In a certain language, TRANSMISSION is written as RTANMSISISON. How will COMMUNICATIONS be


written in same language?
(a) OCMMUNCIATOINS (b) OCMMNUCTAIONS (c) OCMMNUICTAIONS (d) OCMMNUICTAIOSN
Solution (d) The word is divided into section of two letters and then the places of the letters of the odd numbered
sections are interchanged.
Similarly, by interchanging the letters
CO MM UN IC AT IO NS OC MM NU IC TA IO SN TR AN SM IS SI ON RT AN MS IS IS ON

Example 13 In a certain code, BRAIN is written as ⊕ % ÷ # × and TIER is written as $ # + %. How is RENT
written in that code?
(a) % × # $ (b) % # × $ (c) % + × $ (d) + × % $
Solution (c)
Letter B R A I N T E
Code ⊕ % ÷ # × $ +

The code for RENT is % + × $.

Target Exercise
Elementary Level Questions
1. If ‘TEAM’ is coded as ‘VGCO’, then ‘LIFE’ will be coded 5. In a certain code, ‘POLISH’ is written as ‘MLIFPE’,
as then ‘DIG’ is the code for which word?
(a) NJGE (b) NLIH (a) GLJ (b) CHI
(c) NKHH (d) NKHG (c) ECH (d) AFD

2. In a certain code language, ‘MAN’ is coded as ‘NBO’, 6. If ‘FRIEND’ is coded as ‘HUMJTK’, how is ‘CANDLE’
then in the same code language ‘TUL’ will be written written in that code?
as (a) EDRIRL (b) ESJFME
(a) UVP (b) UWN (c) UVM (d) UMW (c) DCQHQK (d) DEQJQM

3. If the word ‘CLERK’ is coded as ‘EOIWQ’, how would 7. In a certain language, ‘CHAMPION’ is coded as
you code the word ‘TABLE’? ‘HCMAIPNO’, how is ‘NEGATIVE’ coded in that code?
(a) VCDNG (b) VCDGIN (a) BGAETVIE (b) ENAGITEV
(c) VDFQK (d) VDFOK (c) EGAITEVN (d) NEAGVEIT

4. If the word ‘TRADE’ is coded as ‘XVEHI’, how the word 8. If ‘FULFNHW’ is the code for ‘CRICKET’, then
‘PUBLIC’ should be coded? ‘EULGH’ is the code for which word?
(a) TYFMPG (b) SXEOLF (a) PRIDE (b) BRIDE
(c) TYFPMG (d) SXLLOP (c) BLADE (d) BLIND
Coding-Decoding 7
9. In a certain code, ‘FORGET’ is written as ‘DPPHCU’. 20. If ‘air’ is called ‘green’, ‘green’ is called ‘blue’, ‘blue’ is
How would ‘DOCTOR’ be written in that code? called ‘sky’, ‘sky’ is called ‘yellow’, ‘yellow’ is called
(a) BPAUPS (b) EMDRPP ‘water’ and ‘water’ is called ‘pink’, then what is the
(c) BPAUMS (d) BRARPP colour of clear sky?
(a) Blue (b) Sky
10. In a certain code, ‘SIKKIM’ is written as ‘THLJJL’.
(c) Yellow (d) Water
How is ‘TRAIN’ written in that code?
(a) SQBHO (b) UQBHO 21. If ‘sky’ is called ‘sea’, ‘sea’ is called ‘water’, ‘water’ is
(c) UQBJO (d) UBQJO called ‘air’, ‘air’ is called ‘cloud’ and ‘cloud’ is called
‘river’, then what do we drink when thirsty?
Directions (Q. Nos. 11-15) The number in each question (a) Sky (b) Air
below is to be codified in the following code. (c) Water (d) Sea

Code digit 7 2 1 5 3 9 8 6 4 22. A ‘room’ is called ‘bed’, ‘bed’ is called ‘window’,


W L M S I N D J B ‘window’ is called ‘flower’ and ‘flower’ is called ‘cooler’,
Letter digit
on what would a man sleep?
11. 184632 (a) Cooler (b) Flower
(a) MDJBSL (b) MDJBIL (c) Window (d) Bed
(c) MDJBWL (d) MDBJIL 23. In a certain code, OVER is written as $ # % * and
12. 879341 VIST is written as # + × − , then how SORE will
(a) DWNIBS (b) DWNBIM written in that code?
(c) DWNIBM (d) NDWBIM (a) × $ * % (b) % × $ *
(c) × * $ % (d) None of these
13. 69428
(a) JBNLD (b) JNBLD 24. Certain letters are coded as TODAY-45738,
(c) JDNLB (d) JBLND WROTE-10542, DATE-7342 and DIRTH-79046.
What does the code number ‘5’ stand for?
14. 1234567
(a) D (b) R
(a) MILBSJW (b) MLISBJW
(c) O (d) T
(c) MLIBSJW (d) MLIJWBS
25. In certain code language, ‘UNIVERSITY’ is written
15. 39394929
as ‘VNJVERTIUY’. Following the same code
(a) ININBNLN (b) NINIBNLN
(c) ININNBNL (d) INBNINLN language, how would you write ‘SECRETARY’?
(a) TFDRFTBRZ (b) TFERFTBRY
(c) SFCSEUASY (d) TEDRFTBRZ
Directions (Q. Nos. 16-22) Choose the correct option for
following coding problems. 26. If Z = 52 and ACT = 48, then BAT will be equal to
(a) 41 (b) 39
16. If ‘diamond’ is called ‘gold’, ‘gold’ is called ‘silver’, (c) 44 (d) 46
‘silver’ is called ‘ruby’ and ‘ruby’ is called ‘emerald’,
which is ‘golden yellow metal’? 27. If E = 5 and PEN = 35, then PAGER is equal to
(a) Diamond (b) Silver (c) Gold (d) Ruby (a) 47 (b) 39
(c) 58 (d) 67
17. If ‘eye’ is called ‘hand’, ‘hand’ is called ‘mouth’, ‘mouth’
is called ‘ear’, ‘ear’ is called ‘nose’ and ‘nose’ is called 28. If ‘POND’ is written as ‘RSTL’. HEAR may be written
‘tongue’, with which of the following would a person as
hear? (a) JKLZ (b) GHIZ
(c) GHIJ (d) JIGZ
(a) Eye (b) Mouth (c) Nose (d) Ear

18. If ‘rat’ is called ‘dog’, ‘dog’ is called ‘lion’, ‘lion’ is called 29. In a certain code ‘COURT’ is written as 5%@38 and
‘snake’ and ‘snake’ is called an ‘elephant’, which is TILE is written as 8 $4 .How is CITE written in that
smallest in size? code?
(a) 5$84 (b) 5%84
(a) Rat (b) Dog (c) Mangoes (d) Lion
(c) 584 (d) 384
19. If ‘finger’ is called ‘toe’, ‘toe’ is called ‘foot’, ‘foot’ is
30. In a certain code 59346 is written as $ AD % F and
called ‘thumb’, ‘thumb’ is called ankle, ‘ankle’ is called
8173 is written as HB # D. How is 9865 written in
‘palm’ and ‘palm’ is called ‘knee’, which one finger has
that code?
different name?
(a) HAF$ (b) AHF$
(a) Thumb (b) Ankle (c) Knee (d) Palm
(c) ADF$ (d) BHF$
8 Study Package for NTSE

High Skill Questions


1. If ‘TEACHER’ is coded as ‘VGCEJGT’. What will be 15. If in a certain coding A is coded as 1, B is coded as 2
the code for ‘CHILDREN’? and so on, how is ‘FEBEDDI’ coded in that coding?
(a) EJKNFTGP (b) EJKNFHTP (a) 294493 (b) 6525449
(c) EJKNFGTO (d) EJKNEGTP (c) 6255449 (d) 6255494

2. In a certain code language, ‘PARENTS’ is written as 16. If D = 4 and FAN = 21, then TAPE is eqaul to
‘RCTGPVU’. How will ‘CHILDREN’ be written in same (a) 40 (b) 42 (c) 44 (d) 43
code language?
17. In a certain code language, ‘749’ means ‘fruit is
(a) EFJKCPCM (b) EJKNFTGP
sweet’; ‘248’ means ‘very sweet voice’ and ‘637’ means
(c) DGJKCQDO (d) MDGFKSNB
‘eat fruit daily’. Which digit stands for ‘is’ in that
3. In a certain code, ‘ROMANTIC’ is written as code?
‘MORNACIT’. How is ‘MARRIAGE’ written in that (a) 7 (b) 9
code? (c) 4 (d) Cannot be determined
(a) MARRIAGE (b) RAMRIEGA
18. In a certain code language, ‘851’ means ‘good sweet
(c) RAMIRAGE (d) RAMIREGA
fruit’; ‘783’ means ‘good red rose’ and ‘341’ means
4. If the word ‘PENCIL’ is coded as ‘LICNEP’, how the ‘rose and fruit’. Which of the following digits stand for
word ‘INKPOT’ should be coded? ‘sweet’ in that language?
(a) TOPINK (b) JOLQPU (a) 8 (b) 5 (c) 1 (d) 3
(c) HMKOPS (d) TOPKNI 19. In a certain code language, ‘Mink Young Pe’ means
5. In a certain code ‘IDIOT’ is written as ‘KFKQV’. How ‘Fruits are ripe’; ‘Pe Lao May Mink’ means ‘Oranges
can ‘GENIUS’ be written in that code? are not ripe’ and ‘May Pe Neu Mink’ means ‘Mangoes
(a) HFOJVT (b) IFOKWT (c) IGPKWU (d) HGOKVU
are not ripe’. Which word in that language means
‘Mangoes’?
6. In a code language, ‘TIGER’ is written as ‘RIGET’. (a) May (b) Lao (c) Neu (d) Mink
How ‘CROWN’ will be written in the same code? 20. If ‘gnr tag zog qmp’ stands for ‘seoul olympic
(a) NRWCO (b) NROWC (c) ROWRC (d) NOWCR
organising committee’; ‘hyto gnr emf’ stands for
7. In a code language, A is written as B, B is written as ‘summer olympic games’ and ‘emm sdr hyto’ stands
C, C is written as D and so on, then how will ‘SMART’ for ‘modern games history’. What would be the code
be written in that code language? for ‘summer’?
(a) TLBSU (b) SHBSU (c) TNBSU (d) SNBRU (a) hyto (b) gnr (c) emf (d) zog

21. In a certain code language, ‘3a, 2b, 7c’ means ‘Truth is


Directions (Q. Nos. 8-12) The number in each question
Eternal’, ‘7c, 9a, 8b, 3a’ means ‘Enmity is not Eternal’
below is to be codified in the following code.
and ‘9a, 4d, 2b, 8b’ means ‘Truth does not Perish.
Letter code A I D O P E U R S T Which of the following means ‘Enmity’ in that
Digit code 0 3 1 4 2 9 6 7 8 5 language?
(a) 3a (b) 7c (c) 8b (d) 9a
8. SPORT
(a) 84275 (b) 82475 (c) 82457 (d) 84257 Directions (Q. Nos. 22-24) In the table given below, there
are two column, Column I and Column II. Four words are
9. ARREST
written in Column I. In Column II, equivalent codes are used
(a) 077859 (b) 770985 (c) 077985 (d) 077589
for these words. For each of the four words, four different
10. DROPER patterns are used. Identify the pattern in the questions given
(a) 714297 (b) 172497 (c) 179742 (d) 174297 below and choose the correct option.
11. PASTA Column I Column II
(a) 20850 (b) 20580 (c) 20058 (d) 02058 S. No. Word Code Equivalent
12. RUST A. CAR XZI
(a) 5678 (b) 7685 (c) 7865 (d) 7568 B. BOOK CQRO
13. If ‘SHARP’ is coded as 58034 and ‘PUSH’ as 4568, the C. FRIEND KWNJSI
‘RUSH’ will be coded as D. TABLE RYZJC
(a) 3685 (b) 3658 (c) 3568 (d) 3583
22. If ‘POLISH’ is coded as ‘NMJGQF’, the code pattern
14. If ‘POWER’ stands for 97254, then ‘ROW’ stands for followed is serial number
(a) 472 (b) 425 (c) 497 (d) 427 (a) A (b) B (c) C (d) D
Coding-Decoding 9
23. If ‘TRADE’ is coded as ‘YWFIJ’, the code pattern 28. Which word will be code for word keep?
followed is serial number (a) joc (b) per (c) pod (d) tan
(a) A (b) B (c) C (d) D
29. Which word will be code for word the?
24. If ‘PEN’ is coded as ‘KVM’, the code pattern followed (a) nu (b) tan (c) per (d) sa
is serial number
30. If E is replaced by 2, R by 5, T by 7, D by 3, S by 4, I by
(a) A (b) B (c) C (d) D
6 and N by 9, then how is RESIDENT written if its
letters are reversed?
Directions (Q. Nos. 25-29) Some words are given in
(a) 79326425 (b) 52463279
Column I. These words are written in a code language in
(c) 52463297 (d) 54263927
Column II. The code equivalents of the words given in
Columns I and II are not necessarily in the corresponding 31. In a certain code language
order. Choose the correct code for the words from the given Try to understand is written as AaAa AAAA aaaa
alternatives. Find your problem is written as aAaA AaaA aAAa
Way to win is written as AAaa AaAa AAAa
S.No. Column I Column II
Then, how can, Please try to understand my problem,
(i) pod na joc very bright boy be written in that code language?
(ii) tan nu pod the boy comes (a) AAaa AAAA AAaa AaAa AAAa AaAa
(iii) nu per ton keep the doll (b) AaAa aaaA aaaa AAAA Aaaa aAAa
(iv) joc ton su very good doll (c) AaAa AAAA aAaA aAAa AAaa AAAa
(v) sa pod ton doll is boy (d) AAaa AaAa aaaa aAAa AAAa aAAa

32. In a certain coding scheme, consonants and vowels


25. Which word will be code for word doll? are coded differently as illustrated below :
(a) ton (b) na (c) joc (d) per
B is coded as 4 ,T is coded as 20
26. Which word will be code for boy? E is coded as 9, I is coded as 17
(a) nu (b) sa (c) pod (d) tan Then, find the sum of numerals in the coded version
27. Which word will be code for word bright? of TRUTH.
(a) 150 (b) 91 (c) 40 (d) 173
(a) pod (b) tan (c) nu (d) na

Hints and Solutions


Elementary Level Questions
4. (c) As, T R A D E X V E H I
1. (d) As, T E A M
+2 +2 +2 +2 +4
+4
V G C O +4
Similarly, L I F E +4
+4
+2 +2 +2 +2
N K H G Similarly, P U B L I C T Y F P M G
+4
2. (c) As, M A N Similarly, T U L +4
+1 +1 +1 +1 +1 +1 +4
+4
+4
N B O U V M +4

3. (c) As, C L E R K E O I W Q 5. (d) AS, P O L I S H M L I F P E


+2
+3 –3
+4 –3
+5 –3
+6 –3
–3
Similarly, T A B L E V D F Q K –3
+2 Similarly, D I G
+3 A F D
+4 –3
+5 –3
+6 –3
10 Study Package for NTSE

6. (a) As, F R I E N D +2
H U M J T K 16. (b) Golden yellow metal is gold, which is called as silver.
+3 17. (c) We can hear with ear, which is called as nose.
+4
+5 18. (b) Rat is smallest in size, which is called dog.
+6
+7 19. (b) Thumb is a finger having a different name. But ‘thumb’ is
called ‘ankle’.
Similarly,
20. (b) Colour of clear sky is blue, which is called sky.
C A N D L E E D R I R L
+2 21. (b) We drink water, which is called air.
+3
+4 22. (c) Man can sleep on bed, which is called window.
+5
+6 23. (a) O V E R V I S T
+7
↓ ↓ ↓ ↓ and ↓ ↓ ↓ ↓
7. (b) As, C H A M P I O N $ # % * # + × −
HCMAIPNO
So, by using the above two
Similarly, S O R E
N E G A T I V E ENAGITEV ↓ ↓ ↓ ↓
× $ * %
8. (b) As F U L F N H W C R I C K E T
–3 24. (c) The number ‘5’ is common in words TODAY and WROTE,
–3 and so it is clear that O and T are common letters but T cannot be
–3
–3 correct option because it is common in all the four words and so
–3 option (c) is correct answer.
–3
–3 25. (d) Given,
U N I V E R S I T Y
Similarly, EU L GH BR I D E
+1 +0 +1 +0 +1 +0 +1 +0 +1 +0
–3
–3 V N J V F R T I U Y
–3
–3 Similarly,
–3
S E C R E T A R Y

9. (c) As, F O R G E T D P P H C U +1 0 +1 0 +1 0 +1 0 +1

–2
+1 T E D R F T B R Z
–2
+1 26. (d) Z is 26th letter, so it is coded as 2 × 26 = 52
–2
+1 Similarly, A = 2 × 1, C = 2 × 3 and T = 2 × 20
∴ ACT = 2 + 6 + 40 = 48
Similarly, D O C T O R B P A U M S
Now, similarly, B = 2 × 2, A = 2 × 1 and T = 2 × 20
–2
+1 ∴ BAT = 4 + 2 + 40 = 46
–2
+1 27. (a) P = 16, E = 5, N = 14
–2
PEN = 16 + 5 + 14 = 35
+1
PAGER = 16 + 1 + 7 + 5 + 18 = 47
10. (b) As, S I K K I M T H L J J L
+2
+1 28. (d) P  → R ....... a gap of (1) letter forward
–1
+4
+1 O  → S ....... a gap of (3) letter forward
–1 +6
+1 N  → T ....... a gap of (5) letter forward
+8
–1 D  → L ...... a gap of (7) letter forward
Similarly, T R A I N UQ B HO Using the same trend,
+2
+1 H J
–1 +4
+1 E I
–1 +6
+1 A G
+8
11. (d) 184632 = MDBJIL R Z

12. (c) 879341 = DWNIBM 29. (c) C OUR T T I L E CITE



13. (b) 69428 = JNBLD 5 %@ 3 8 8$ 4 5©84

14. (c) 1234567 = MLIBSJW 30. (b) 5 934 6 8 173 9865



15. (a) 39394929 = ININBNLN $ A D% F H B#D AHF$
Coding-Decoding 11
High Skill Questions
1. (a) As, T E A C H E R +2
V G C E J G T 8. (b) SPORT = 82475
+2 9. (c) APREST = 077985
+2
+2 10. (d) DROPER = 174297
+2 11. (a) PASTA = 20850
+2
+2 12. (b) RUST = 7685
Similarly, C H I L D R E N E J K N F T C P 13. (c) SHAR P PUSH
+2 58 03 4 45 6 8
+2
+2 Q RUSH = 3568
+2
+2 14. (a) P O W E R
+2 9 7 2 5 4
+2
+2 ⇒ ROW = 472
2. (b) As, P A R E N T S R C T G P V U 15. (b) A → 1, B → 2, C → 3 and so on
+2 ⇒ FEBEDDI = 6525449
+2
+2 16. (b) A = 1, B = 2, C = 3, D = 4 etc.
+2
+2
FAN = 6 + 1 + 14 = 21
+2 TAPE = 20 + 1 + 16 + 5 = 42
+2
17. (b) 7 4 9 = fruit is sweet
Similarly, C H I L D R E N E J K N F T G P
+2 2 4 8 = very sweet voice
+2
+2 6 3 7 = eat fruit daily
+2
+2 Fruit correspond to 7 and is will correspond to 9.
+2 18. (b) ‘good sweet fruit’ and ‘good red rose’ have ‘good’ common
+2
+2 word which corresponds to ‘8’.
‘good sweet fruit’ and ‘rose and fruit’ have’ ‘fruit’ common word
3. (d) As, which corresponds to ‘1’.
ROM AN TIC MOR NA CIT
good sweet fruit
⇒ = 851
8 5 1
Similarly, 19. (c) Mink Yoing Pe → Fruits are ripe …(i)
MARRIAGE R AM I R E G A
Pe Lao May Mink → Oranges are not ripe …(ii)
May Pe Neu Mink → Mangoes are not ripe …(iii)
4. (d) PENCIL →
Reverse
LICNEP From Statements (i) and (ii). Pe mink means are ripe and from
Reverse Statements (ii) and (iii) may means not, so pe mink may means
Similarly, INKPOT  → TOPKNI
are not ripe and finally from Statement (iii) neu will be mangoes.
5. (c) As, I D I O T K F KQ V
20. (c)
+2
+2 gnr tag zog qmp seoul olympic organising committee.....(i)
+2
+2 hyto gnr emf summer olympic games .....(ii)
+2

Similarly, G E N I U S I G P K W U emm sdr hyto modern games history .....(iii)


+2 From Statements (i) and (ii), gnr means olympic and from
+2
+2 Statements (ii) and (iii), hyto means games. So, summer will be
+2 emf with the help of Statement (ii).
+2
+2 21. (c) 3a 2b 7c Truth is Eternal .....(i)
6. (b) TIGER RIGET
7c 9a 8b 3a Enmity is not Eternal .....(ii)
Similarly, by interchanging first and last letter
9a 4d 2b 8b Truth does not Perish .....(iii)
CROWN NROWC
In the Statements (ii) and (iii) we find that ‘9a’ means ‘not’. In
7. (c) A →
+1
B →
+1
C the Statements (i) and (ii) the common codes are ‘7c’ and ‘3a’,
so as the common words are ‘is’ and ‘Eternal’. Finally, in the
Similarly, SMART ⇒ TNBSU Statement (ii), ‘8b’ means ‘Enmity’.
12 Study Package for NTSE

Sol. (Q. Nos. 22-24) Sol. (Q. Nos. 25-29)


(A) 3 1 18
C A R pod na joc very bright boy
opposite
letter tan nu pod the boy comes
X Z I
nu per ton keep the doll
24 26 9
(B) B O O K joc ton su very good doll
+1 +2 +3 +4
sa pod ton doll is boy
C Q R O
(C) 25. (a) doll → ton
F R I E N D
26. (c) boy → pod
+5 +5 +5 +5 +5 +5
27. (d) bright → na
K W N J S I 28. (b) keep → per
(D) T A B L E 29. (a) the → nu
–2 –2 –2 –2 –2 30. (a) The following table depicts the code off the given letters

R Y Z J C Letter R E S I D E N T

22. (d) P O L I S H Codes 5 2 4 6 3 2 9 7


–2 –2 –2 –2 –2 –2 The code for RESIDENT is 52463297.
N M J G Q F when RESIDENT is reversed
The above pattern follows code pattern (D). ∴ TNEDISER = 79236425
23. (c) T R A D E 31. (b) From first statement, the required code must contain ‘AaAa
+5 +5 +5 +5 +5
AAAA aaaa’ for ‘try to understand’.
Now, Problem = aAaA/AaaA/aAAa
Y W F I J
The given sentence has two new word, so the required code
The above pattern follows code pattern (C). must have two new code.
24. (a) 16 5 14 Hence, option (b) is the most suitable option.
P E N
opposite 32. (d) Here, the numerical value of consonant gets double and the
letter numerical value of vowel is equal to (2 × numerical value −1)
K V M T R U T H
11 22 13 (20 × 2 ) + (18 × 2 ) + (21 × 2 − 1) + (20 × 2 ) + (8 × 2 )
The above pattern follows code pattern (A). 40 + 36 + 41 + 40 + 16 = 173
Chapter

2
Classification
Classification means to distinguish different items from a given group on the basis of certain common properties
like shape, size, category colour, trait etc. More clearly in such type of questions, alphabets, words, objects or things
are purely based on similarity or similar relationship and one to be picked out is that which does not belong to the
group.

Type I Word Classification


In this type, similar groups are found among the names, places, things, nouns, verbs, different sexes, or any other
matter. Candidate should identify the odd one out.
Example 1 Choose the word which is least like the other words in the group.
(a) Apple (b) Carrot (c) Orange (d) Guava
Solution (b) Here, all except carrot are fruits, while carrot is a vegetable.
Example 2 Choose the word which is least like the other words in the group.
(a) Copper (b) Zinc (c) Brass (d) Aluminium
Solution (c) Here, all except brass are metals, while brass is an alloy.
Example 3 Choose the word which is least like the other words in the group.
(a) Volume : Litre (b) Time : Second (c) Length : Metre (d) Pressure : Barometer
Solution (d) In all other pairs, except (d) second is the unit to measure the first. On the other hand, barometer is an
instrument to measure the atmospheric pressure.

Type II Letter Classification


In such questions, four or five groups of letters are given as options. Out of these groups, one is different which is
your correct answer.
Example 4 Which one of the following pair is different from the other groups?
(a) CFIL (b) PSVX (c) JMPS (d) ORUX
Solution (b) In all other groups, except PSVX, each letter moves 3 steps forward to obtain the next letter.
Example 5 Which one of the following pair is different from the other groups?
(a) ABC : CBA (b) JKL : KLJ (c) XYZ : ZYX (d) MNO : ONM
Solution (b) The letters in the second group are the letters of the first group written backward.
14 Study Package for NTSE

Type III Odd Numbers Classification


In such questions, certain numbers are given out of which all, except one are same in some manner while one is
quite different.
Example 6 Choose the one which is different from the rest of three.
(a) 27 (b) 63 (c) 56 (d) 49
Solution (a) 27 is not divisible by 7.
Example 7 Which of the following pair of numbers is different from the others?
(a) 14, 28 (b) 30, 60 (c) 25, 50 (d) 15, 35
Solution (d) In each pair, the second number is double the first number. In option (d), the number 35 should have been
30, so it is the odd one.

Type IV Odd Pair of Number Classification


In such questions, certain pairs of numbers are given. Out of which all except one are similar in some manner while
one is different.
Example 8 Choose the numerical pair which is different from others.
(a) 14-22 (b) 17-25 (c) 16-24 (d) 26-34
Solution (b) In all the pairs, the difference of two numbers is 8 and both are even number, but in option (b) both numbers
are odd (not divisible by 2).

Target Exercise
Elementary Level Questions
Directions (Q. Nos. 1-20) In the following questions, three terms are alike in some way and so form a group while one is
different than others. Identify the term that does not belong to that group.
1. (a) Pen (b) Pencil 11. (a) Ramayana (b) Mahabharata
(c) Ball pen (d) Eraser (c) Geeta (d) Godan
2. (a) Carrot (b) Tomato 12. (a) Silver (b) Gold
(c) Ginger (d) Potato (c) Copper (d) Mercury
3. (a) Delhi (b) Lucknow 13. (a) Ring (b) Bangle
(c) Hyderabad (d) Jaipur (c) Bracelet (d) Necklace
4. (a) Asia (b) Europe 14. (a) Jowar (b) Urad
(c) Australia (d) India (c) Moong (d) Gram
5. (a) Ear (b) Eye 15. (a) Dozen (b) Yard
(c) Nose (d) Throat (c) Metre (d) Inch
6. (a) Mother (b) Father 16. (a) Haldighati (b) Panipat
(c) Sister (d) Friend (c) Plassey (d) Shirdi
7. (a) Himachal Pradesh (b) Punjab 17. (a) Bharat Ratna (b) Padma Shri
(c) Haryana (d) Chandigarh (c) Padmini (d) Param Vir Chakra
8. (a) Hindi (b) English 18. (a) Green (b) Violet
(c) Sanskrit (d) Mathematics (c) Red (d) Orange
9. (a) Rose (b) Lotus 19. (a) Dollar (b) Pound
(c) Marigold (d) Lily (c) Franc (d) Cuba
10. (a) Pistol (b) Sword 20. (a) He-goat (b) He-buffalo
(c) Gun (d) Rifle (c) Cow (d) Bull
Classification 15
Directions (Q. Nos. 21-45) In the following questions, four 32. (a) PR (b) SU (c) VX (d) YZ
groups of letters/numbers are given, three of them are alike in 33. (a) Xa (b) Xb (c) Xc (d) XX
some way while one is different. Choose the odd one.
34. (a) ABC (b) DEF (c) PQS (d) GHI
21. (a) AB (b) CD (c) EF (d) GI
35. (a) AaB (b) BcC (c) DdE (d) FfG
22. (a) AC (b) QT (c) IK (d) LN
36. (a) 100 (b) 125 (c) 121 (d) 225
23. (a) xX (b) iI (c) aB (d) lL
37. (a) 37 (b) 49 (c) 131 (d) 154
24. (a) Bb (b) Cc (c) Dd (d) eE
38. (a) 27 (b) 400 (c) 125 (d) 1000
25. (a) AZ (b) BY (c) CQ (d) DW
39. (a) 18 (b) 27 (c) 36 (d) 26
26. (a) VX (b) FG (c) PR (d) DF
40. (a) 30 (b) 40 (c) 66 (d) 50
27. (a) ZX (b) RS (c) TU (d) XY
41. (a) 115 (b) 66 (c) 110 (d) 220
28. (a) AE (b) BD (c) CG (d) DH
42. (a) 1357 (b) 7841 (c) 3517 (d) 7935
29. (a) ac (b) pq (c) rt (d) df
43. (a) 1001 (b) 2004 (c) 2211 (d) 3004
30. (a) ap (b) qr (c) ST (d) pt
44. (a) 27 (b) 45 (c) 63 (d) 35
31. (a) az (b) bz (c) zc (d) dz
45. (a) 123 (b) 204 (c) 305 (d) 240

High Skill Questions


Directions (Q. Nos. 1-10) In the following questions three 13. (a) Stallion : Colt (b) Drake : Duck
are alike in some way and so form a group while in different (c) Dog : Bitch (d) Drone : Bee
one is different than others. Identify the term that does not
14. (a) Bees : Humm (b) Snake : Hiss
belong to that group.
(c) Frog : Bleat (d) Lion : Rar
1. (a) Numerus (b) Carpals
15. (a) Pascal : Pressure (b) Watt : Power
(c) Forearm (d) Ulna
(c) Ampere : Current (d) Radian : Degree
2. (a) Trigonometry (b) Calculus
(c) Algebra (d) Thermodynamics Directions (Q. Nos. 16-20) In each of the following
3. (a) Speedometer (b) Thermometer
questions, certain pairs of words are given out of which the
words in all pairs except one, bear a certain common
(c) Galvanometer (d) Compass-needle
relationship choose the pair in which the words are
4. (a) Fable (b) Elegy differently related.
(c) Epic (d) Ballad 16. (a) Tripitaka : Buddhist (b) Temple : Hindu
5. (a) Vitamin A (b) Vitamin B (c) Avesta : Parasi (d) Torah : Jew
(c) Vitamin C (d) Vitamin D
17. (a) Whale : Mammal (b) Frog : Amphibian
6. (a) Aryabhatta (b) Bhaskara (c) Salamander : Insect (d) Snake : Reptile
(c) Rohini (d) Apollo soyuz
18. (a) Donald : Comedy (b) Holmes : Suspense
7. (a) Nightingle (b) Skylark (c) Premchand : Novel (d) Robinson : Adventure
(c) Parrot (d) Canary
19. (a) Sahara : Africa (b) Thar : India
8. (a) Onion (b) Coriander (c) Gobi : Mangolia (d) Kalahari : America
(c) Beetroot (d) Ginger
20. (a) Mulder : Proteins (b) Curle : Radium
9. (a) Uranium (b) Jupiter (c) Becquerel : Radioactivity (d) Einstein : Television
(c) Mars (d) Earth
Directions (Q. Nos. 21-25) In the following questions three
10. (a) Diabetes (b) Chickenpox of the following groups are similar in connection with the
(c) Smallbox (d) Conjunctivitis position of these letters in English alphabet and hence form a
group. Which one does not belong to the group?
Directions (Q. Nos. 11-15) In the following questions, three
are alike in certain way and so form a group. Which is one 21. (a) ADFG (b) MPQR
that does not belong to that group? (c) EHJK (d) ILNO
11. (a) Lock–key (b) Motorcycle–fuel 22. (a) NSWX (b) HMQR
(c) Book–copy (d) River–water (c) KPSU (d) EJNO
12. (a) Food : Hunger (b) Water : Thirst 23. (a) HOL (b) JQM
(c) Air : Suffocation (d) Talent : Education (c) MTP (d) DKG
16 Study Package for NTSE

24. (a) SUV (b) KLN Directions (Q. Nos. 31-40) Choose the odd number pair
(c) BDE (d) GIJ in each of the following questions.
25. (a) HJG (b) PQN 31. (a) 14-23 (b) 36-63
(c) DEB (d) TUR (c) 45-54 (d) 27-72
32. (a) 80-9 (b) 64-8
Directions (Q. Nos. 26-30) In the following questions, three (c) 36-6 (d) 49-7
of the following have similar relationship and hence belong to
the group. Find the one which does not belong to the group.
33. (a) 13-31 (b) 45-54
(c) 16-61 (d) 71-88
26. (a) DISTASTE : AFSATSXI
(b) GASWORKS : DXROWSNV 34. (a) 2-8 (b) 3-12
(c) MARTYRED : JXRYTRIH (c) 4-15 (d) 5-20
(d) OUTHOUSE : LRUOHTWI 35. (a) 3-9 (b) 4-16
27. (a) BANKING : HPLOSGI (c) 5-25 (d) 6-35
(b) CUSTOMER : SGPSYYBK 36. (a) 99-86 (b) 70-57
(c) BUYERS : TTHCZH
(c) 56-43 (d) 49-35
(d) INTEREST : UUHVJYUQ
28. (a) WANT : NATW (b) WITH : TIHW
37. (a) 1-1 (b) 2-4
(c) 3-9 (d) 5-20
(c) HAVE : AVEH (d) JUST : SUTJ
29. (a) SAFETY : VYICWW (b) SMOKER : VKRIHP
38. (a) 81-9 (b) 36-6
(c) 69-8 (d) 121-11
(c) SERIES : VCUGHQ (d) HEALTH : KCYJWF
30. (a) PLENTY : LEPTYN 39. (a) 14-16 (b) 17-14
(c) 12-18 (d) 11-19
(b) COWARD : OWCRAD
(c) ALWINS : LWANIS 40. (a) 10-30 (b) 50-60
(d) EXAMIN : XAEIMN (c) 30-40 (d) 70-80

Hints and Solutions


Elementary Level Questions
1. (d) Except eraser, all are used for writing. 17. (c) Except Padmini, all are prestigious awards.
2. (b) Except tomato, all others grow under the ground or soil. 18. (d) Except Orange, Green, Violet and Red are primary colours.
3. (a) Except Delhi, all others are state capital. 19. (d) dollar, pound and franc are currencies of different countries.
4. (d) India is country while others are continents. 20. (c) Except cow, all are male characters.
5. (d) Throat is not a sense organ. 21. (d) 1
A B , C D , E F while G I
23 123 123 123
+ 1 + 1 + 1 + 2
6. (d) Friend is not a blood relation.
7. (d) Chandigarh is not a state of India. 22. (b) Q
123T , while others
+ 3
8. (d) Mathematics is not a language.
A C D F I K L N in each term.
123 123 123 123
9. (b) All except lotus are flowers that grow on land whereas lotus + 2 + 2 + 2 + 2
is a water-flower.
23. (c) aB
10. (b) All except sword are fire arms.
24. (d) Capital letter, then small letter but eE does not follow the
11. (d) Except Godan, all are religious books. order.
12. (d) Mercury is liquid while others are solids. 25. (c) In AZ, BY and DW first letter and second letter have similar
13. (d) Necklace is wear in neck while others are concerned with positions from left and right, respectively in English alphabets.
hand. 26. (b) 1
V23XP RD
123 123F F123
G
+2 +2 +2 +1
14. (a) All except Jowar are pulses while Jowar is a millet.
27. (a) R
123S 1
T 23
U1X 23
Y1Z23X
15. (a) All except dozen are units of measuring distances. +1 +1 +1 −2
16. (d) Shirdi is famous for Sai temple while others are famous for 28. (b) A
battle. 123EB DC
123 GD
123 H
123
+4 +2 +4 +4
Classification 17
29. (b) a123c p123q 1
r23t d123f 37. (d) Except 154, all are odd numbers.
+2 +1 +2 +2
38. (b) (3)3 = 27, (5)3 = 125, (10)3 = 1000 but 400 is not a cube of
30. (c) Except ST, all includes small English alphabets (a, b, c, … , z). same numeral value.
31. (c) Except zc, in all others second letter is z. 39. (d) Except 26, all are divisible by 3 and 9.
32. (d) P123
RS U1
123 V23X Y
123Z 40. (c) Except 66, all are divisible by 10.
+ 2 + 2 + 2 + 1
41. (a) Except 115, all are divisible by 11.
33. (d) Except XX, all contains first letter is X and second is other
than X. 42. (b) Only 7841 contains an even number, while others have odd
numbers only.
34. (c) PQS, all others follow the correct sequence (ABC, DEF, GHI,
FGH). 43. (c) Except 2211, all numbers have two zeroes between two
numerals.
35. (b) Except BcC, all others have second letter similar to first letter.
44. (d) Except 35, all are divisible by 9.
36. (b) (10)2 = 100, (11)2 = 121, (13)2 = 169 and (15)2 = 225 but 125 is
45. (c) Except 305, all are divisible by 3.
not a square of some numeral value.

High Skill Questions


1. (a) Except (a), rest are parts of human of human hand. 25. (a) In all other, the sequence is that of + 1and − 3, respectively
2. (d) Except (d), all others are branches of Mathematics. in English alphabet.
3. (b) Except (b), all others have needle. 26. (b) In all others, the first two letters move three places
backward while the last two move four places forward. The
4. (a) Except (a), all others are type of poems.
middle four letters reverse among themselves.
5. (c) Except (c), all others are fat soluble vitamins.
27. (d) All letters are reversed and then coded as one, two, three...
6. (d) Except (d), all others are satellites. letters forward as in English alphabet.
7. (c) Except (c), all others are singing birds. 28. (c) In each option, a word consists of four letters the loglc of
8. (b) Except (b), all others are obtained from ground. two relationship is related with the order of letter used in the word.

9. (a) Except (a), all others are celestial bodies. 29. (d) Except (d), the following rule is followed in other pairs the
first, third and fifth letters of the first word are being replaced
10. (a) Except (a), all others are infections diseases.
with three letters forward as in English alphabet and second,
11. (c) Except (c), all others forms a complementary pair. fourth and sixth letters of the first word are being replaced with
12. (d) Except (d) the lack of first causes the second. two letters backward as in English alphabet.
13. (a) Except (a), second is the female of the first. 30. (a) In each option, a word consist of six letters. The logic
of the relationship is related with order of letter used in the word.
14. (c) Except (c), second is the noise produced by the first.
31. (a) Except 14-23, in all others pairs of number numeral digits
15. (d) Except (d), first is the unit to measure the second. get exchange their position like, 27-72.
16. (b) Except (b), first is a religious book of the second. 32. (a) (8)2 = 64 (64- 8), (6)2 = 36 (36- 6), (7 )2 = 49 (49-7 ), but in
17. (c) Except (c), second is the class of animals to which the first (80-9) 80 ≠ (9)2
belongs.
33. (d) Except 71-88, be in all other pairs of number member al
18. (c) Except (c), first is the character of the type of movie denoted digits get exchange three position.
by the second.
34. (c) 2 - 8 (2 × 4 = 8), 3- 12 (3 × 4 = 12 ), 5-20 (5 × 4 = 20), but
19. (d) Except (d), first is the name of a desert which is situated in the 4-15 (4 × 4 = 16) is not following the trend.
country denoted by the second.
35. (d) Except 6-35, in all pairs second number of the pair is a
20. (d) Except (d), first is the name of the scientist who discovered perfect square of first.
the second.
36. (d) Except 49-35, in all other pair the difference of numbers is 13.
21. (b) In all others, the sequence is that of +3, + 2 and +1,
37. (d) Except 5-20, in all pairs second number is a perfect square
respectively as in English alphabet.
of first number.
22. (c) In all others, there is a gap of four letters between the first two 38. (c) Except 69-8, in all pairs second number is a perfect square
letters and no gap between the last two letters. of first number.
23. (a) In all other, the sequence is that of +7 and − 4, respectively in 39. (b) Except 17-14, because in all other pairs the difference
English alphabet. between number is even.
24. (b) In all other, the sequence is that of + 2 and + 1, respectively 40. (a) 10-30, all other pairs having a difference of 10 between the
as in English alphabet. numbers.
Chapter

3
Analogy
The word Analogy has been derived from two words taken together, Ana means relation and Logus means
knowledge.
The word ‘Analogy’ means
l a similar feature, condition, state etc., shared by two things that are compared and
l a process of reasoning based on similar feature of two things. Thus, we can conclude that ‘Analogy’ means ‘similar feature’,
‘a common feature’ or ‘correspondence’.
In questions based on analogy, a particular relationship is given and another similar relationship has to be
identified from the given alternatives. Analogy can be of types from which questions are generally asked in
competitive exams.

Word Analogy
In questions based on word analogy, two words are (b) Dog — Bitch
given related to each other in some way. The candidate (c) Horse — Mare
has to find that relationship and apply the same to (d) Man — Woman
choose the correct alternative which is either in the form (e) Lion — Lioness
of a word or a pair of words.
V. Singular and Plural
I. Synonymous Relationship (a) Man — Crowd
(a) Slim — Thin (b) Bees — Swarm
(b) Meet — Contact (c) Flowers — Bouquet
(c) Endless — Eternal (d) Sailors — Crew
(d) Ban — Prohibition
(e) Vacant — Empty VI. Animal and Young one
(a) Cow — Calf
II. Antonymous Relationship
(a) Scarcity — Abundance (b) Man — Child
(b) Deep — Shallow (c) Hen — Chicken
(c) Choos — Peace (d) Butterfly — Caterpillar
(d) Harsh — Gentle (e) Horse — Pony
(e) Blunt — Sharp VII. Physical Quantity and Unit
III. Worker and Tool (a) Force — Newton (N)
(a) Author — Pen (b) Area — Hectare (H)
(b) Soldier — Gun (c) Time — Second (s)
(c) Tailor — Needle (d) Mass — Gram (g)
(d) Doctor — Stethoscope (e) Volume — Litre (L)
(e) Chef — Knife (f) Pressure — Pascal (P)
IV. Gender (g) Work — Joule (J)
(a) Tiger — Tigress (h) Length — Metre (m)
Analogy 19
VIII. Worker and Product XI. Games and Place of Playing
(a) Cobbler — Shoes (a) Badminton/Tennis — Court
(b) Chef — Food (b) Cricket — Pitch
(c) Goldsmith — Ornament (c) Boxing — Ring
(d) Racing — Track
(d) Tailor — Clothes
(e) Wrestling — Arena
(e) Mason — Wall/Building
(f) Farmer — Crop
XII. Study and Topic
(a) Ornithology — Birds
IX. Country and Currency (b) Seismology — Earthquakes
(a) India — Rupee (c) Anthropology — Man
(b) China — Yuan (d) Pathology — Diseases
(c) Iraq — Dinar (e) Taxonomy — Classification
(d) Thailand — Baht (f) Botany — Plants
(e) Bangladesh — Taka (g) Entomology — Insects
(f) Japan — Yen (h) Cardiology — Heart
(g) UK — Pound (i) Nephrology — Kidney
X. Individual/Thing and Class XIII. Animal/Things and Sound
(a) Man — Mammal (a) Lion — Roar
(b) Frog — Amphibian (b) Donkey — Brey
(c) Butterfly — Insect (c) Snake — Hiss
(d) Snake — Reptile (d) Elephant — Trumpet
(e) Chair — Furniture (e) Goat — Bleat
(f) Cup — Crockery (f) Coins — Jingle
(g) Ostrich — Bird (g) Rain — Patter

Kinds of Relationship
Example 1 Find out the missing word from the given alternatives, which bears the same relationship to the third
word, as the first two bear. Cattle : Herd : : Sheep : ?
(a) Flock (b) Swarm (c) Shoal (d) Mob
Solution (a) Herd is a group of cattle. Similarly, flock is a collection of sheep.
Example 2 Cat is related to mouse in the same way as Bird is related to ………
(a) Worm (b) Snake (c) Animal (d) Crow
Solution (a) In this case, cat feeds on mouse. Similarly, bird feeds on worm.
Example 3 Find out the pair in which the words bear the same relationship as given pair bears.
Chalk : Blackboard : : ……… : ………
(a) Door : Handle (b) Table : Chair (c) Ink : Paper (d) Dog : Tail
Solution (c) As, we write with chalk on a blackboard. Similarly, we write with ink on a piece of paper.

Alphabet Analogy
It is based on letters of alphabet. These letters are parts i.e., third group will relate the fourth group in the
written in a particular pattern. All are divide in four pattern as first and second bears the relation.
Example 4 K c a C : C a c K : : X g m F : ?
(a) E m g F (b) E m g X (c) F m g X (d) G m e F
Solution (c) K c a C X g m F

As, Similarly,

C a c K F m g X
∴ ? = Fmgx
20 Study Package for NTSE

Points to Remember
In dealing with an analogy question, the following points should be followed
1. Analyse the relationship between the question pair.
2. Look for the pair of words (among the choices with a similar relationship).
3. If more than one pair of words seem to be fit, reexamine the relation in the question pair.
4. Eliminate answers that do not appear to be the best.

Number Analogy
In this type of analogy, a group of numbers is given in question, having a particular pattern. On the basis of that
pattern a candidate have to find out the same pattern in a particular option.
Example 5 Which set of numbers is like the given set (64 , 32, 8)?
(a) (125, 25, 5) (b) (81, 27, 3) (c) (56, 28, 7) (d) (112, 56, 16)
Solution (c) (64, 32, 8) First number is divided by 2 to get the second and second is divided by 4 to get the third number.
56 28
Option (c), = 28, = 7 is correct as per the same relationship.
2 4

Alpha-Numeric Analogy
In this type of analogy, letters and numbers are given following a certain relationship. The relationship may be
based on the position of alphabets or letters. Sum or product of the position of alphabet or letters etc. The candidate
is required to understand the relationship and complete the analogy.
Example 6 Choose the number which will replace the question mark(?).
FILM : 10 : : HOTEL : ?
(a) 12 (b) 16 (c) 18 (d) 30
Solution (a) As,
F I L M

6 + 9 + 12 + 13 = 40 = 40 ÷ 4 =10 [Q number of letters = 4]


Similarly,
H O T E L

8 + 15 + 20 + 5 + 12 = 60 = 60 ÷ 5 =12 [Q number of letters = 5]

Target Exercise
Elementary Level Questions
Directions (Q. Nos. 1-8) In the following questions, there (a) Pakistan (b) Japan
are three words. The first two words to the left of (: :) are (c) Bangladesh (d) Nepal
related in some way. The same relationship holds between the 4. Plough : Farmer : : Sword : ?
third words to the right of the sign (: :) and one of the responses (a) Soldier (b) Surgeon
(a), (b), (c) or (d). Identify the correct related word. (c) Tailor (d) Carpenter
1. Newspaper : Editor : : Film : ? 5. Wax : Candle : : Pulp : ?
(a) Actor (b) Producer (a) Fruits (b) Sand
(c) Director (d) Story writer (c) Paper (d) Transport
2. Calendar : Dates : : Dictionary : ? 6. Dog : Bark : : Goat : ?
(a) Language (b) Sentences (a) Bleat (b) Howl (c) Grunt (d) Bray
(c) Words (d) Vocabulary
7. Birds : Ornithology : : Animals : ?
3. Rupee : India : : Yen : ? (a) Botany (b) Philology
Analogy 21
(c) Zoology (d) Seismology 22. 37 : 63 : : 25 : ?
8. Patient : Doctor : : Student : ? (a) 76 (b) 75 (c) 60 (d) 90
(a) School (b) Teacher
(c) Mother (d) Friend
Directions (Q. Nos. 23-26) In this type of questions, a
group of three words is given, followed by four other words as
Directions (Q. Nos. 9-11) In the following given questions, alternatives. The candidate is required to choose the
two words are given, they have a certain relationship to each alternative which is similar to the given three words.
other, four alternatives each with one pair of words are given. 23. Jam, Jelly, Pickles
Select the pair which has same relationship as the original pair (a) Butter (b) Marmalde (c) Grapes (d) Preserve
of words has.
24. Wrestling, Karate, Boxing
9. Kapil Dev : Cricket (a) Swimming (b) Polo (c) Pole vault (d) Judo
(a) P T Usha : Hockey
25. Dozen, Score, Decade
(b) Vishwanathan Anand : Chess
(a) Century (b) Number
(c) Sania Mirza : Golf
(c) Measurement (d) Counting
(d) Sachin Tendulkar : Tennis
10. Platform : Train 26. Edge, Corner, Tip
(a) Snow (b) Hill (c) Brink (d) Cap
(a) Aeroplane : Aerodrome (b) Hotel : Tourist
(c) Footpath : Traveller (d) Quaf : Ship
Directions (Q. Nos. 27-30) For the following questions,
11. Light : Darkness choose the correct option.
(a) Education : Illiteracy (b) Medicine : Patient
(c) Anger : Friendship (d) None of these
27. Mirror is related to Reflection in the same way as
‘Water’ is related to …
Directions (Q. Nos. 12-22) In each of the following (a) Conduction (b) Dispersion
(c) Immerison (d) Refraction
questions, there are some relationship between the two terms
to the left of (: :) and the same relationship holds between the 28. Engineer is related to Machine in the same way as
two terms to the right to the sign (: :). You have to identify the ‘Doctor’ is related to …
related option which come in place of questions mark (?). (a) Hospital (b) Body
(c) Disease (d) Medicine
12. AF : KP : : FK : ?
(a) GL (b) OX (c) TY (d) PU 29. Sweet is related to Chocolate in the same way as
13. CIRCLE : RICELC : : SQUARE : ? ‘Book’ is related to …
(a) Dictionary (b) Library
(a) QSUERA (b) QUSERA
(c) Encyclopaedia (d) Atlas
(c) UQSAER (d) UQSERA
30. Umbrella is related to Rain in the same way as
14. Dda : aDD : : Rrb : ?
‘Goggles’ is related to …
(a) BBr (b) bRR (c) RRr (d) rBB
(a) Light (b) Sun rays (c) Star (d) Sight
15. PALE : LEAP : : POSH : ?
(a) HOSP (b) POHS (c) SHOP (d) POSH Directions (Q. Nos. 31-34) In each of these questions,
find out the missing term from the given alternatives.
16. DWH : WDS : : FUL : ?
(a) UFO (b) UFU 31. C : 16 : : F : ?
(c) FOU (d) ELV (a) 30 (b) 40 (c) 49 (d) 50
17. BUCKET : ACTVBDJLDFSU : : BONUS : ? 32. M × N : 13 × 14 : : F × R : ?
(a) ACMNMOTVRT (b) SUNOB (a) 7 × 19 (b) 5 × 17 (c) 14 × 15 (d) 6 × 18
(c) ACNPMOTVRT (d) ACMNMOTURT 169
33. MK : : : JH : ?
18. 63 : 21 : : 27 : ? 121
(a) 6 (b) 9 (c) 1 (d) 3 100 100 64 81
(a) (b) (c) (d)
64 81 120 100
19. 9 : 24 : : ? : 6
(a) 1 (b) 6 (c) 5 (d) 3 T X
34. :2:: :?
J H
20. 85 : 42 : : 139 : ? 23
(a) 68 (b) 69 (c) 70 (d) 67 (a) 2 (b) 3 (c) (d) 4
7
21. 2 : 8 : : 4 : ?
(a) 24 (b) 64 (c) 40 (d) 44
22 Study Package for NTSE

High Skill Questions


Directions (Q. Nos. 1-5) For the following questions, choose 11. Hunt : Pleasure : Panic
the correct option. (a) Death : Disease : Germs (b) Game : Match : Win
(c) Theft : Gain : Loss (d) Rain : Cloud : Flood
1. As COUNSEL is to BITIRAK, so also GUIDANCE is
to 12. The words in the bottom row are related in the same
(a) HOHYBJBA (b) FPHZZKAB way as the words in the top row. Find the word that
(c) FOHYZJBB (d) OHYZKBB completes the bottom row of words.
2. As BLOCKED is to YOLXPVW, so also OZFMXS is to Evaporation Cloud Rain
(a) LABOUR (b) LAUNCH Sneezing Cough ?
(c) NAUGHT (d) RESULT
(a) Cold (b) Injury
3. As THEREFORE is to TEEOERFRH, so also
(c) Flower (d) Pain
HELICOPTER is to
(a) RETPOCILNE (b) RETPOCILEH 13. Which of the following represent the similar
(c) HLCPERTOIE (d) HELICORETP relationship as ‘CASE’ has with ‘EASC’ ?
(a) PILE : EPLI (b) DOME : MOED
4. As RATIONAL is to RATNIOLA, so also TRIBAL is to (c) JAIL : IJLA (d) None of these
(a) TRILBA (b) TIRLBA
(c) TRIALB (d) TIRLAB 14. Which of the following represent the same
relationship as ‘HEART’ has with ‘THREA’?
5. As CIRCLE is related to RICELC, so also SQUARE is
(a) SWORN : NSOWR (b) FUNDS : FSDUN
to (c) GLAZE : EGZAL (d) None of these
(a) UQSERA (b) UQSAER (c) QUSERA (d) QSUERA
15. Which of the following represent the same
Directions (Q. Nos. 6 and 7) Choose the correct option on relationship as ‘OFTEN’ has with ‘FOTNE’?
the basis of similar pattern in pyramid given below.
(a) HEART : TRAHE (b) OPENS : SNEOP
1 (c) RISKY : IRSYK (d) FIRST : IFRST
2 3 4 16. Which of the following represent the same
9 8 7 6 5 relationship as ‘BELOW’ has with ‘OBWEL’?
10 11 12 13 14 15 16 (a) DRAFT : FDTRA (b) REPLY : LRYPE
(c) YOUNG : UYGON (d) YEARS : RESYA
25 24 23 22 21 20 19 18 17
26 27 28 29 30 31 32 33 34 35 36 17. Which of the following represent the same
49 48 47 46 45 44 43 42 41 40 39 38 37 relationship as ‘LOWER’ has with ‘WORLE’?
(a) ENTRY : RNYET (b) WORDS : ROSWD
6. 2812 : 765 : : 91123 : ? (c) AMONG : GNOMA (d) YEARS : RESYA
(a) 121110 (b) 121314 (c) 122132 (d) 303132
Directions (Q. Nos. 18-22) Choose the set of numbers
7. 129 : 145 : : 3811 : ? from the four alternatives which is similar to the given set.
(a) 3713 (b) 328 (c) 346 (d) 3615
18. Given set (9, 15, 21)
Directions (Q. Nos. 8-11) In each of the following (a) (10, 14, 16) (b) (7, 21, 28)
questions, some words are given which are related in some (c) (5, 10, 25) (d) (4, 8, 12)
way. The same relationship is obtained among the words in
one of the four alternatives given under it. Find the correct
19. Given set (12, 20, 60)
alternative. (a) (15, 12, 90) (b) (8, 12, 24)
(c) (7, 11, 75) (d) (6, 16, 90)
8. Researcher : Historian : Scholar 20. Given set (10, 30, 70)
(a) History : Story : Book (b) Teacher : Graduate : Poet
(a) (17, 37, 77) (b) (20, 30, 60)
(c) Epic : Novel : Book (d) Teacher : Professor : Lecturer
(c) (41, 55, 66) (d) (60, 70, 90)
9. France : Spain : Germany 21. Given set (9,15, 24)
(a) Canada : California : Mexico (a) (11, 14, 16)
(b) Iraq : Kuwait : Iran
(b) (6, 21, 28)
(c) Sri Lanka : Japan : India
(c) (5, 10, 25)
(d) Japan : China : Nigeria
(d) (4, 8, 15 )
10. Love : Adoration : Infatuation 22. Given set (256, 64, 16)
(a) Smile : Frown : Anger (b) Murder : Stab : Assassinate (a) (160, 40, 10) (b) (144, 36, 9)
(c) Hate : Dislike : Attract (d) None of these (c) (80, 20, 5) (d) (64, 32, 8 )
Analogy 23
Directions (Q. Nos. 23-25) What number will be come in 26. HNP : PBA :: DLP : ?
place of question mark? (a) PJG (b) CDP (c) CLP (d)PME
23. 2836 : 13; 9423 : 14; 7229 : ? 27. AFHO : GBDM :: CHFM : ?
(a) 16 (b) 18 (c) 20 (d) 12 (a) GBLD (b) GBJO
24. 4322 : 48; 4172 : 56; 7615 : ? (c) GPLD (d) GBDM
(a) 110 (b) 210 (c) 140 (d) 180 28. AKJO : IOHN :: ? : CLBK
25. 14 + 55 → 69, 28 + 23 → 60, 22 + 31 → 35, (a) LDME (b) EMGH
17 + 28 → 99, 41 + 27 → ? (c) GNFM (d) EHDL
(a) 118 (b) 115 (c) 116 (d) 113
Directions (Q. Nos. 29-32) In each of these questions find
Directions (Q. Nos. 26-28) The following questions are out the missing term from the given alternatives.
based on the alphabets written along the figure given below. E 22 B
29. : :: : ?
In each question, the relationship between the two terms H 19 I
written left of (::) is retained in the two terms written to the 24 25
(a) (b)
right of it. Out of these four terms, one term is missing. 17 18
Choose this term out of the given alternatives. 3 17
(c) (d)
A 7 19
C
30. MLO : 121114 :: GJL : ?
B
(a) 192022 (b) 060911
(c) 160813 (d) 2218175
K L 31. GREAT : 25 :: NUMBER : ?
(a) 36 (b) 38
J D (c) 27 (d) 24
P
O M 32. X M A E : 16 :: V T N G : ?
I (a) 21 (b) 17
H N F E
(c) 35 (d) 18

Hints and Solutions


Elementary Level Questions
1. (c) Editor and director have same functions in newspaper and 11. (a) The words in each pair are antonyms of each other.
film, respectively.
12. (d) +10 +10
2. (c) Calendar is a list of dates and dictionary is collection of Similarly,
AF KP FK PU
words. Hence, the correct option is (c).
+10 +10
3. (b) As, Rupee is the currency of India, in the same way, Yen is
the currency of Japan. 13. (d) As,
C I R C L E
4. (a) Plough is used by farmer, similarly sword is used by soldier.
5. (c) We get wax by candle in same manner pulp is get by
paper. R I C E L C

6. (a) Dog is bark, similarly goat is bleat. Similarly,


S Q U A R E
7. (c) Study of birds is ornithology same as study of animals is
called zoology.
8. (b) Patient : Doctor :: Student : Teacher U Q S E R A

9. (b) Kapil Dev is associated with cricket similarly, Vishwanathan 14. (b) Dda : aDD, similarly Rrb : bRR
Anand is associated with chess. 15. (c) As,
P A L E
10. (d) As, platform is the place, where the train stops during the
journey in the same way quaf is the place where a ship stay
during its journey.
L E A P
24 Study Package for NTSE
Similarly, 21. (b) (2 )3 = 8 ⇒2 : 8
P O S H
Similarly, (4)3 = 64 ⇒ 4 : 64
22. (b) 37 : 63 ⇒ 37 + 63 = 100
S H O P
Q 25 : ? ⇒ 25 + ? = 100
16. (a) DWH : WDS ⇒ ? = 75
First two letters are rearranged and the third letter occupies the 23. (b) The given three words are all different forms in which
same position from the beginning and the end. fruits/vegetables are preserved. So, marmalde is a similar one.
Similarly, FUL : UFO 24. (d) All the given words are martial arts and hence, Judo is
17. (c) Each letter of the first group is replaced by two letters–one similar to others.
that comes after it and one that comes before it, in the second
25. (a) The given words are all terms used for a definite number of
group.
items. Century is the answer.
B U C K E T
26. (c) All the given words and the brink are synonyms.
↓ ↓ ↓ ↓ ↓ ↓
AC TV BD JI DF SU 27. (d) Light rays following on mirror undergo reflection and those
falling on water undergo refraction.
Similarly,
B O N U S 28. (c) As engineer tackles the machine, similarly doctor tackles
the disease.
29. (c) As sweet is one of the material of chocolate, similarly book
AC NP MO TV RT material are part of encyclopaedia.
∴ ? = ACNPMOTVRT 30. (b) As umbrella protects from rain. Similarly, goggles protect
18. (b) As, 63 ÷ 3 = 21 from Sun rays.

Similarly, 27 ÷ 3 = 9 31. (c) C is the 3rd letter of the English alphabet and (3 + 1)2 = 16
19. (d) As, 9 × 3 − 3 = 27 − 3 = 24 Similarly, F is the 6th letter of the English alphabet and
(6 + 1)2 = 49
Similarly, ? ×3−3=6
⇒ ? ×3=6+ 3
32. (d) M and N are 13th and 14th letters of the English alphabets,
respectively. So, M × N corresponds to 13 × 14. Similarly, F and
9
⇒ ?= =3 R are 6th and 18th letters of the English alphabet, respectively.
3 So, F × R corresponds to 6 × 18.
⇒ ?= 3
33. (a) M and K are 13th and 11th letters of English alphabet,
20. (b) As, 85 42 132 169
respectively and = . Similarly, J and H are 10th 8th
112 121
1 1 102 100
×
2

2 letters of the English alphabet, respectively and 2 = .
8 64
Similarly, 139 69 34. (b) T and J are 20th and 10th letters of the English alphabet,
respectively and 20 ÷ 10 = 2.
1 1
× – Similarly, X and H are 24th and 8th letters of the English
2 2
alphabet, respectively and 24 ÷ 8 = 3.
∴ ? = 69

High Skill Questions


1. (c) As, 2. (b) As,
C O U N S E L B L O C K E D
Opposite
–1 –6 –1 –5 –1 –4 –1
letters
B I T I R A K Y O L X P V W
Similarly, Similarly,
G U I D A N C E O Z F M X S
–1 –6 –1 –5 –1 –4 –1 –3
Opposite
letters
F O H Y Z J B B L A U N C H
Analogy 25
3. (c) As, 13. (d) As,
1 2 3 4 5 6 7 8 9 1 3 5 7 9 8 6 4 2
T H E R E F O R E T E E O E R F R H 1 2 3 4 4 2 3 1
Similarly, C A S E E A S C
1 2 3 4 5 6 7 8 9 10 1 3 5 7 9 10 8 6 4 2
H E L I C O P T E R H L C P E R T O I E (a) 1 2 3 4 4 1 3 2
P I L E E P L I
4. (a) As,
R A T I O N A L (b) 1 2 3 4 3 2 4 1
D O M E M O E D
(c) 1 2 3 4 3 1 4 2
R A T N I O L A J A I L I J L A
Similarly, Clearly, no option shows the correct pattern.
T R I B A L
14. (d) As,
1 2 3 4 5 5 1 4 2 3
T R I L B A H E A R T T H R E A
5. (a) As, (a) 1 2 3 4 5 5 1 3 2 4
C I R C L E S W O R N N S O W R
(b) 1 2 3 4 5 1 5 4 2 3
R I C E L C
F U N D S F S D U N
(c) 1 2 3 4 5 5 1 4 3 2
Similarly,
S Q U A R E G L A Z E E G Z A L
Clearly, no option shows the correct pattern.

U Q S E R A 15. (c) As,


1 2 3 4 5 2 1 3 5 4
6. (b) The answer to the above question should be (b) 121314 to
O F T E N F O T N E
satisfy the same relation.
Clearly, this pattern is show in option (c).
1
2 3 4 1 2 3 4 5 2 1 3 5 4
9 8 7 6 5 R I S K Y I R S Y K
10 11 12 13 14 15 16
16. (a) As,
25 24 23 22 21 20 19 18 17
1 2 3 4 5 4 1 5 2 3
7. (d)
1 B E L O W O B W E L
2 3 4 Clearly, this pattern is show in option (a).
9 8 7 6 5 1 2 3 4 5 4 1 5 2 3
10 11 12 13 14 15 16 D R A F T F D T R A
25 24 23 22 21 20 19 18 17
17. (b) As,
So, correct similar pattern is followed in 3615. 1 2 3 4 5 3 2 5 1 4
8. (d) Third donates the class to which the first and second L O W E R W O R L E
belong. Clearly, this pattern is shown in option (b).
9. (c) As, all three are European countries and Sri Lanka, India 1 2 3 4 5 3 2 5 1 4
and Japan are Asian countries. W O R D S R O S W D
10. (b) Second and third are of higher intensity than the first and 9 + 21
18. (d) As, = 15.
second, respectively. 2
4 + 12
11. (c) As hunting gives pleasure to the hunter and creates panic for Similarly, =8
the haunted. Similarly, theft is a gain to the theft and a loss to the 2
person robbed. 19. (b) As, LCM of 12, 20 = 60,
12. (a) First is caused by the second and second leads to the Similarly, LCM of 8, 12 = 24
third. So, correct answer set is (8, 12, 24).
26 Study Package for NTSE
E 22
20. (a) As, 10
1
30 30 70,
424
3 1 424
3 As, =
H 19
+ 20 + 40
B 25
Similarly 17 37 37 77 Similarly, =
1 31
424 424
3 I 18
+ 20 + 40 −1
30. (b) As, 13M   → 12
21. (c) As, 9, 15 and 24 are divisible by 3. −1
12 L   → 11
Similarly, from option (c), 5, 10 and 25 are divisible by 5. −1
15O   → 14
22. (d) All the numbers in the each set can be expressed in terms
i.e. MLO : 121114
of power of 2.
−1
Similarly, 07 G   → 06
23. (a) As, 2 + 8 − 3 + 6 = 13 −1
10 J   → 09
9 + 4 − 2 + 3 = 14
−1
Similarly, 7 + 2 − 2 + 9 = 16 12 L   → 11

24. (b) As, 4322 = 48 ∴ GJL : 060911


⇒ 4 × 3 × 2 × 2 = 48 31. (a) As, GREAT has 5 letters, it is represented by square of 5
and 4172 = 56 i.e 25. Similarly, NUMBER has 6 letters. So, it is represented by
4 × 1 × 7 × 2 = 56, square of 6 i.e. 36.
Similarly, 7 × 6 × 1 × 5 = 210 32. (d) As,
25. (d) Reverse the digits to the right of the plus sign. As, X M A E

As, 14 + 55 → 69 ⇒ 14 + 55 = 69
24 13 1 5
and 22 + 31 → 35 ⇒ 22 + 13 = 35
and 28 + 23 → 60 ⇒ 28 + 32 = 60
2+4 1+3 1 5
and 17 + 28 → 99 ⇒ 17 + 82 = 99
Similarly, 41 + 27 → ? ⇒ 41 + 72 = 113 6 + 4 + 1 + 5 =16
26. (a) As HNP is corresponding to PDA, similarly DLP is Similarly,
corresponding to PJG. V T N G

27. (b) As AFHO is corresponding to GBDM, similarly CHFM is


corresponding to GBJO. 22 20 14 7

28. (a) As AKJO is corresponding to IOHN, similarly LDME is


corresponding to CLBK. 2+2 2+0 1+4 7

29. (b) Here, each letter is assigned its positional value in reverse
English alphabetical order. 6 + 2 + 5 + 7 =18
Chapter

4
Alphabet Test
Alphabet test is a test to solve the problems based on letters of English alphabet.
This requires the knowledge of English alphabets. Some important points related to English alphabet are given
below
1. Positional values of letters in English alphabet.
Forward 1 2 3 4 5 6 7 8 9 10 11 12 13 14 15 16 17 18 19 20 21 22 23 24 25 26
Alphabet A B C D E F G H I J K L M N O P Q R S T U V W X Y Z
Backward 26 25 24 23 22 21 20 19 18 17 16 15 14 13 12 11 10 9 8 7 6 5 4 3 2 1

2. Opposite letters of English alphabet


A ↔ Z, B ↔ Y, C ↔ X, D ↔ W, E ↔ V, F ↔ U, G ↔ T, H ↔ S, I ↔ R, J ↔ Q, K ↔ P, L ↔ O, M ↔ N
There are three types of questions based on English alphabet which are as follows

Type I Arranging the Words in Alphabetical Order


In these type of questions, some words are given. The candidate is required to arrange them as per dictionary in
order to answer the question. We know that in a dictionary, the words are arranged in alphabetical order.
Therefore, you must know the basics of the dictionary usage.
The words beginning with the same letter of English alphabet are again arranged in alphabetical order with
respect to the second letter of words and so on. To solve such questions, first of all observe the first letter of each
word. If the first letter is the same, observe the second letter and so on till you get the required arrangement of the
words.
Example 1 Which of the following will be the third word in the dictionary?
(a) Countdown (b) Cough
(c) Council (d) Couch
Solution (c) C o u n t d o w n
C o u g h
C o u n c i l
C o u c h
On the basis of the fourth letter, the arrangement of words would be as follows
(1) (2) (3) (4)
Couch → Cough → Council → Countdown
(d) (b) (c) (a)
On the basis of fifth letter in countdown and council, word ‘council’ will come third in the dictionary order.
28 Study Package for NTSE

Type II Meaningful Word Formation


In these type of questions, you are required to form meaningful word(s) using the specified letters of a keyword
using each letter only once. In such type of questions, a keyword is given and some letters of that word are singled
out with the help of those letters, you are required to form a new meaningful word and then point out the required
letter of the word so formed.
Example 2 Select the combination of numbers, so that letters arranged accordingly will form a meaningful word.
M T R E N I L A
1 2 3 4 5 6 7 8
(a) 2, 3, 4, 1, 6, 5, 8, 7 (b) 2, 4, 3, 6, 1, 5, 8, 7 (c) 2, 4, 3, 1, 6, 5, 8, 7 (d) 1, 3, 4, 2, 6, 8, 5, 7
Solution (c) Clearly, the given letters, when arranged in the order 2, 4, 3, 1, 6, 5, 8, 7 form the word ‘TERMINAL’.
Example 3 Choose one word out of the given alternatives, which cannot be formed from the letters of the word
‘INTERNATIONAL’.
(a) LATTER (b) TERMINAL (c) ORIENTAL (d) RATIONALE
Solution (b) Carefully, looking on the words, it is clear that ‘TERMINAL’ cannot be formed by given word because the key word
does not contain ‘M’, so correct answer is TERMINAL.

Example 4 If we make a meaningful word of the first, fourth, ninth and fourteenth letters of the word
‘ADMINISTRATION’, which of the following will be second letter of the word from the left end of that word?
(a) R (b) A (c) I (d) N
Solution (b) The first, fourth, ninth and fourteenth letters of the word ‘ADMINISTRATION’ are ‘A’, ‘I’, ‘R’ and ‘N’.
The meaningful word is ‘RAIN’.
The second letter of the word from the left end is ‘A’.

Type III Finding the Positions of Letters


In such type of questions, the position of letters in English alphabets are determined based on certain conditions, such
as which letter is in the middle, end or at the first position etc. The candidate has to analyse the questions based upon
those condition and then has to determine the answer.
Example 5 Which letter will be in the middle between 6th letter from the left and 14th letter from the left in the
English alphabet?
(a) K (b) L (c) J (d) O
Solution (c) To determine mth and nth letters from the left in English alphabet, following formula can be used. Letter in
m + n
the middle from left =   it letter
 2 
6 + 14
= = 10th letter which is J.
2
[Remember E J O T Y – 5 10 15 20 25]

Example 6 Which letter will be in the middle between 6th letter from the left and 13th letter from the right in the
English alphabet?
(a) K (b) L (c) O (d) J
Solution (d) To determine mth letter from left and nth letter from the right in English alphabet, following formula can be
{(m − n) + 27}
used. Letter in the middle =
2
(6 − 13) + 27 20
= = = 10th letter = J
2 2
− +
î = = =
Target Exercise
Elementary Level Questions
Directions (Q. Nos. 1-10) Arrange the given words in an 17. (1) Inward (2) Iron
alphabetical order and choose the one that comes first. (3) Iodine (4) Invite
1. (a) Xylem (b) Xylophone
(5) Iodoform
(c) Xylophagous (d) Xylograph (a) (4), (1), (2), (5), (3) (b) (4), (1), (3), (5), (2)
(c) (1), (3), (2), (4), (5) (d) (5), (4), (1), (2), (3)
2. (a) Artist (b) Artificial
(c) Astrology (d) Ascent 18. (1) Matter (2) Meal
(3) Maze (4) Maximum
3. (a) Opportunity (b) Opposite (5) Mean
(c) Optics (d) Option
(a) (2), (3), (5), (4), (1) (b) (1), (4), (2), (5), (3)
4. (a) Inspect (b) Install (c) (1), (4), (3), (2), (5) (d) (4), (1), (3), (2), (5)
(c) Insist (d) Insulin
19. (1) Overweight (2) Overtime
5. (a) Magical (b) Media (3) Overtax (4) Overzeal
(c) Machine (d) Magnet (5) Overwork
6. (a) Penalty (b) Pedestrian (a) (2), (1), (4), (5), (3) (b) (3), (1), (2), (5), (4)
(c) People (d) Pencil (c) (3), (2), (1), (5), (4) (d) (5), (4), (1), (2), (3)

7. (a) Demand (b) Demon 20. (1) Recreate (2) Recollect


(c) Democracy (d) Demerit (3) Reclaim (4) Recover
(5) Recognise
8. (a) Scenery (b) School
(c) Scholar (d) Scientist (a) (3), (2), (5), (4), (1) (b) (3), (5), (2), (4), (1)
(c) (3), (2), (4), (1), (5) (d) (3), (5), (1), (2), (4)
9. (a) Cover (b) Caught
(c) Collect (d) Career 21. (1) Satellite (2) Scholar
(3) Saturday (4) Scenery
10. (a) Literature (b) Little
(5) Satisfaction
(c) Literacy (d) Litchi
(a) (1), (4), (5), (3), (2) (b) (1), (3), (5), (4), (2)
Directions (Q. Nos. 11-15) Arrange the given words in an (c) (1), (5), (3), (4), (2) (d) (1), (2), (3), (4), (5)
alphabetical order and choose the one that comes in last.
22. (1) Mechanical (2) Mechanist
11. (a) Sunderlal (b) Soonderlal (3) Mechanism (4) Mechanician
(c) Sundaralal (d) Sunder (5) Mechanic
12. (a) Thermo (b) Thermodynamics (a) (5), (4), (3), (2), (1) (b) (5), (1), (4), (2), (3)
(c) Thermometer (d) Thermal (c) (5), (1), (4), (3), (2) (d) (1), (2), (3), (4), (5)

13. (a) Central (b) Census Directions (Q. Nos. 23-25) Find one word which cannot
(c) Center (d) Ceramics be made from the letters of the given word.
14. (a) Develop (b) Determination 23. ADULTERATION
(c) Determine (d) Detect (a) RELATION (b) RATIO
15. (a) Fluorescence (b) Florentine (c) RETURN (d) TOILET
(c) Floriculture (d) Flop 24. RECREATION
Directions (Q. Nos. 16-22) In each of the following (a) ACTION (b) REFER
questions, arrange the given words in the sequence in which (c) TORN (d) RATION
they occur in the dictionary and then choose the correct 25. QUESTIONNAIRE
sequence.
(a) QUINATE (b) QUEUE
16. (1) Page (2) Palisade (c) QUESTOR (d) QUERIES
(3) Pageant (4) Parrot 26. Which letter is midway between 8th letter from left
(5) Peacock and 7th letter from right in the English alphabet ?
(a) (1), (3), (2), (5), (4) (b) (1), (2), (3), (4), (5) (a) N (b) M
(c) (1), (3), (2), (4), (5) (d) (5), (4), (1), (2), (3) (c) P (d) O
High Skill Questions
Directions (Q. Nos. 1-7) In each of the following 9. What will come in place of question mark (?) in the
questions, arrange the given words in the sequence in which following series?
they occur in the dictionary and then choose the correct NDP, QWB, ZFR, ?
sequence. (a) SVI (b) AFS (c) IVS (d) SFA
1. (1) Cloth (2) Cinema 10. Which of the following is the fifth to the right of
(3) Chronic (4) Christmas thirteenth letter from your left?
(5) Create
(a) U (b) J (c) S (d) Z
(a) (4), (3), (2), (1), (5)
(b) (1), (2), (3), (4), (5) 11. If every alternate letter starting from O is replaced
(c) (4), (2), (3), (5), (1) with odd numbers starting from 1, which letter or
(d) (4), (2), (3), (5), (1) number will be third to the left of tenth letter from
2. (1) Dialogue (2) Diagram your right?
(3) Diameter (4) Diagnose (a) 15 (b) L (c) 13 (d) J
(5) Dial 12. If it is possible to make a meaningful word from the
(a) (4), (2), (5), (1), (3) (b) (2), (4), (5), (1), (3) eight, sixteenth, seventeenth and twenty second
(c) (1), (2), (4), (5), (3) (d) (1), (2), (3), (4), (5) letters from your left in the given series, which will be
3. (1) Enlock (2) Enrich the first letter of that word? If no such word can be
(3) Energetic (4) Encounter formed, your answer would be ‘X’ and if more than
(5) English one such words can be formed answer is P.
(a) (1), (2), (3), (4), (5) (b) (4), (3), (5), (1), (2) (a) M (b) T (c) X (d) P
(c) (1), (3), (2), (4), (5) (d) (5), (4), (1), (2), (3)
Directions (Q. Nos. 13-15) These following questions are
4. (1) Handkerchief (2) Handsome based on the alphabetical series given below.
(3) Handloom (4) Handglass
SLUAYJVEIONQGZBDRH
(5) Handicraft
(a) (1), (2), (4), (5), (3) (b) (5), (4), (3), (2), (1) 13. If ‘SU’ is related to ‘HD’ and ‘UY’ is related to ‘DZ’ in a
(c) (4), (5), (1), (3), (2) (d) (5), (4), (1), (3), (2) certain way, to which of the following is YV related to
the following in the same pattern?
5. (1) Navigate (2) National
(a) ZQ (b) IN (c) QO (d) DZ
(3) Naughty (4) Nation
(5) Narrow 14. What will come in place of question mark (?) in the
(a) (5), (4), (3), (2), (1) (b) (5), (4), (2), (3), (1) following series?
(c) (1), (2), (3), (4), (5) (d) (5), (2), (3), (4), (1) LA, UJ, YI, EG, ?
(a) IB (b) NR (c) QR (d) QH
6. (1) Peerless (2) Penal
(3) Petroleum (4) Pedestrian 15. In a certain code‘VERB’ is coded as ‘ YJBG’ and
(5) Pharmacy ‘QUIZ’ is coded as ‘OSVQ’. How will ‘JOHN’ be coded
(a) (4), (1), (2), (5), (3) (b) (1), (2), (5), (4), (3) in the language?
(c) (4), (1), (2), (3), (5) (d) (4), (1), (5), (3), (2) (a) EQDG (b) AEDI (c) YIRO (d) VNRQ
7. (1) Unstable (2) Unship 16. Choose the word that cannot be formed by using the
(3) Unsafe (4) Unseat letters of the given word PROMINENT.
(5) Unshared (a) PENT (b) TENT (c) RENT (d) MINT
(a) (1), (3), (2), (4), (5) (b) (3), (4), (5), (2), (1)
(c) (1), (4), (5), (2), (3) (d) (5), (4), (3), (2), (1) 17. Choose the word that cannot be formed by using the
letters of the given word RECOMMENDATION.
Directions (Q. Nos. 8-12) Answer these questions (a) MOTION (b) DANCE (c) NATIONAL (d) MAN
referring to the letter sequence given below.
NOPQYBZARSHIJKLMTUVGFEWXDC
Directions (Q. Nos. 18-22) Select the combination of
numbers so that letters arranged accordingly will form a
8. If the letter of the above given series are written in meaningful word.
reverse order, then which letter will be third to the
18. B L I P U S H
left of eighteenth letter from your right?
1 2 3 4 5 6 7
(a) Z (b) F
(a) 4, 5, 1, 2, 6, 3, 7 (b) 4, 5, 3, 2, 1, 6, 7
(c) I (d) L
(c) 1, 2, 3, 4, 5, 6, 7 (d) 4, 5, 1, 2, 3, 6, 7
Alphabet Test 31
19. H L R A O C S 24. If you pickup from the following alphabet, the sixth
1 2 3 4 5 6 7 and fourteenth letters from your right and then
(a) 1, 2, 3, 4, 5, 6, 7 pickup the fifth and twentieth letters from your left
(b) 7, 6, 5, 1, 4, 2, 3 and form a meaningful word, what is the first letter of
(c) 7, 6, 1, 5, 2, 4, 3 that word?
(d) 7, 6, 5, 1, 2, 4, 3
ABCDEFGHIJKLMNOPQRSTUVWXYZ
20. I K E S R T (a) M (b) E
1 2 3 4 5 6 (c) No word can be formed
(a) 6, 4, 5, 1, 2, 3 (b) 4, 6, 5, 1, 2, 3 (d) More than one word can be formed
(c) 6, 5, 4, 3, 2, 1 (d) 1, 2, 3, 4, 5, 6
25. If in the word EQUALITY, the position of first and
21. M B L A L R U E the fifth letters are interchanged. Similarly, the
1 2 3 4 5 6 7 8 positions of the second and the sixth letters are
(a) 7, 1, 2, 6, 8, 3, 5, 4 interchanged and so on, which letter will be third
(b) 1, 2, 3, 4, 5, 6, 7, 8 from the right end?
(c) 8, 7, 2, 1, 4, 5, 3, 6 (a) Q (b) U (c) I (d) T
(d) 1, 2, 3, 6, 7, 8, 4, 5
26. If each vowel in the word ‘GLADIOLUS’ is
22. R G O S I E A N substituted with the next letter of the English
1 2 3 4 5 6 7 8 alphabetical series and each consonant is substituted
(a) 3, 8, 7, 4, 5, 6, 1, 2 with the letter preceding it. How many vowels are
(b) 3, 2, 1, 5, 8, 7, 4, 6 present in the new arrangement?
(c) 3, 1, 2, 8, 7, 4, 5, 6 (a) None (b) One (c) Two (d) Three
(d) 3, 1, 2, 7, 8, 5, 4, 6
27. Each consonant in the word ‘TIRADES’ is replaced by
23. If it is possible to make a meaningful word with the the previous letter and each vowel is replaced by the
second, sixth, ninth and twelfth letters of the word next letter in the English alphabet and the new letters
‘CONTRIBUTION’, which of the following will be the are rearranged alphabetically. Which of the following
last letter of that word? will be the fourth from the right end?
(a) N (b) O (c) T (d) M (a) F (b) J (c) Q (d) C

Hints and Solutions


Elementary Level Questions
1. (a) Observe the words letter by letter 10. (d) On the basis of fourth letter, litchi will come first.
Xylem 11. (a) Arrangement of words as per dictionary.
Xyl ophone (1) (2) (3) (4)
Xyl ophagous Soonderlal Sundralal Sunder Sunderlal
(b) (c) (d) (a)
Xyl o graph
12. (c) Arrangement of words as per dictionary.
On the basis of fourth letter, ‘Xylem’ comes first.
(1) (2) (3) (4)
2. (b) In ‘artist’ and ‘artificial’, artificial will come first on the basis Thermal Thermo Thermodynamics Thermometer
(d) (a) (b) (c)
of fifth letter.
Hence, thermometer comes last.
3. (a) On the basis of fifth letter, opportunity comes first.
13. (d) Arrangement of words as per dictionary.
4. (c) On the basis of fourth letter, insist will come first.
(1) (2) (3) (4)
5. (c) Out of magical, machine and magnet, on the basis of third Census Center Central Ceramics
(b) (c) (a) (d)
letter, machine will come first.
Hence, ceramics comes last.
6. (b) On the basis of third letter, pedestrian will come first.
14. (a) Arrangement of words as per dictionary.
7. (a) On the basis of fourth letter, demand will come first.
Detect Determination Determine Develop
8. (a) On the basis of third letter, scenery will come first. (d) (b) (c) (a)

9. (d) On the basis of third letter, career will come first. Hence, develop comes last.
32 Study Package for NTSE

15. (a) Arrangement of words as per dictionary. 20. (b) Arrangement of words as per dictionary
(1) (2) (3) (4)
Reclaim → Recognise → Recollect → Recover → Recreate
Flop Florentine Floriculture Fluorescence (3) (2) (4) (1)
(d) (b) (c) (a) (5)
Correct sequence = 3, 5, 2, 4, 1
Hence, fluorescence comes last.
21. (c) Arrangement of words as per dictionary.
16. (c) Arrangement of words as per dictionary.
Satellite → Satisfaction → Saturday → Scenery → Scholar
Page → Pageant → Palisade → Parrot → Peacock (1) (5) (3) (4) (2)
(1) (3) (2) (4) (5) Correct sequence = 1, 5, 3, 4, 2
⇒ Correct sequence = 1, 3, 2, 4, 5 22. (c) Arrangement of words as per dictionary.
17. (b) Arrangement of word as per dictionary. Mechanic → Mechanical → Mechanician → Mechanism
(5) (1) (4) (3)
invite → inward → iodine → iodoform → iron Correct sequence = 5, 1, 4, 3, 2 → Mechanist
(4) (1) (3) (5) (2) (2)
Correct sequence = 4, 1, 3, 5, 2 23. (c) RETURN cannot be made from the given word. As, R does
not appears twice in the given word.
18. (c) Arrangement of words as per dictionary.
24. (b) REFER cannot be made from the given word. As, given
Matter → Maximum → Maze → Meal → Mean word do not contain letter F.
(1) (4) (3) (2) (5)

Correct sequence = 1, 4, 3, 2, 5 25. (b) QUEUE cannot be made from the given word. As, U does
not appears twice in the given word.
19. (c) Arrangement of words as per dictionary.
Overtax → Overtime → Overweight → Overwork → Overzeal 26. (a) 7th letter from right = (27 − 7 ) th from left
(3) (2) (1) (5) (4)
= 20th letter from left
Correct sequence = 3, 2, 1, 5, 4 8 + 20
∴ Required middle letter = = 14 th letter from left = N
2

High Skill Questions


1. (a) Arrangement of words as per dictionary. 7. (b) Arrangement of words as per dictionary.
Christmas → Chronic → Cinema → Cloth → Create Unsafe → Unseat → Unshared → Unship → Unstable
(4) (3) (2) (1) (5) (3) (4) (5) (2) (1)
Correct sequence = 4, 3, 2, 1, 5 Correct sequence = 3, 4, 5, 2, 1
2. (a) Arrangement of words as per dictionary. 8. (b) Third to the left of eighteenth letter from the right means
Diagnose → Diagram → Dial → Dialogue → Diameter (18 + 3) = 21st letter from the right in the reverse series or 21st
(4) (2) (5) (1) (3) letter from left in the original series. That is (26 − 21 + 1) = 6 th
Correct sequence = 4, 2, 5, 1, 3 letter from the right in the original series, i.e., F.
3. (b) Arrangement of words as per dictionary. 9. (a) N + 3 = Q Q+ 3= Z Z+ 3= S
Encounter → Energetic → English → Enclock → Enrich D− 2= W W− 2= F F− 2= V
(4) (1) (2)
(3) (5) P+ 3= B B+ 3= R R+ 3= I
Correct sequence = 4, 3, 5, 1, 2
Hence, ? = SVI
4. (d) Arrangement of words as per dictionary.
10. (a) Fifth to the right of the thirteenth from left means
Handicraft → Handglass → Handkerchief → Handloom →
(5) (4) (1) (3) (13 + 5) = 18 th letter from your left i.e., U.
Handsome
(2) 11. (c) Third to the left of tenth letter from right means (10 + 3)
Correct sequence = 5, 4, 1, 3, 2
= 13 th letter from the right i.e., K. K will be replaced by the
5. (b) Arrangement of words as per dictionary. number 13, if we number the letter starting from O.
Narrow → Nation → National → Naughty → Navigate 12. (d) The specified letters are A, M, T and E. Meaningful words
(5) (4) (2) (3) (1)
made with the help of these letters are as follows
Correct sequence = 5, 4, 2, 3, 1
1. MEAT 2. TEAM 3. MATE 4. TAME
6. (c) Arrangement of words as per dictionary.
13. (a) After find the corresponding letters from the right, we get
Pedestrian → Peerless → Penal → Petroleum → Pharmacy
(4) (1) (2) (3) (5) ZQ.
Correct sequence = 4, 1, 2, 3, 5
Alphabet Test 33
14. (d) The first letter follows +1, + 2, + 3, + 4, ... 22. (d) Meaningful word
3 1 2 7 8 5 4 6
The second letter follows + 2, + 3, + 4, + 5, ...
O R G A N I S E = ORGANISE
We get, QH.
23. (b) CO N T R I B U T I O N
15. (b) After moving two letters backward in the given alphabetical 2 6 9 12
series from each corresponding letter. So, letters are O, I, T and N and the meaningful word can be
As, V E R B and Q U I Z ‘INTO’, last letter of this word is ‘O’.
–2 –2 –2 –2 –2 –2 –2 –2
Y J B G O S V Q 24. (a) From right, sixth letter = U
Similarly, J O H N Fourteenth letter = M
–2 –2 –2 –2 From left, fifth letter = E
A E D I
Twentieth letter = T
16. (b) TENT can not be formed from the given word. By using these letters, the meaningful word is ‘MUTE’. First
As, T does not appear twice in the given word. letter of this word is ‘M’.
17. (c) National cannot be formed from the given word. 25. (a) Given word = EQUALITY
As, given word does not contain letter L.
After interchanging the letters, new word formed = LIUAEQTY
18. (d) Meaningful word So, Q is the third from the right.
4 512367
26. (a) According to the question,
P U B L I S H = PUBLISH
Given word G L A D I O L U S
19. (c) Meaningful word
↓ ↓ ↓ ↓ ↓ ↓ ↓ ↓ ↓
7 6 1 5 2 4 3 New word F K B C J P K V R
S C H O L A R = SCHOLAR
So, no vowel is present in the new arrangement.
20. (b) Meaningful word
27. (b) According to the question,
465123
S T R I K E = STRIKE Original word, T I R A D E S
i. Change, S J Q B C F R
21. (a) Meaningful word
ii. Change, B C F J Q R S
7 1 2 6 8 3 5 4
So, J will be the fourth from the right end.
U M B R E L L A = UMBRELLA
Chapter

5
Series
A series is a sequential arrangement of terms. All the terms in the sequence follow a certain pattern. The term can
be in the form of numbers, letters or both numbers and letters. We come across several types of questions based on
any given series. In order to solve such questions, you must recognise the pattern of the given series and then
complete the given series with the most suitable alternative or find a incorrect number in the series.

Type I Number Series


The series in which all the terms are number is called Solution (a) 11 23 48 99 ? 409
number series. ×2+1 ×2+2 ×2+3 ×2+4 ×2+5
Some suggestive relationship between the numbers may be Q ? = 99 × 2 + 4 , ? = 198 + 4 = 202
(i) Consecutive odd/even number
Example 4 Find the missing term in the following series.
(ii) Consecutive prime numbers set 1, 1, 3, 9, 6, 36, 10, 100, ? , 225
(iii) Squares/cubes of some numbers with/without variation (a) 12 (b) 14 (c) 13 (d) 15
of addition or subtraction of some numerals
(iv) Sum/product/difference of preceding number(s) Solution (d) There are two series
(v) Addition/subtraction/multiplication/division by some First series 1 3 6 10 ?

numbers +2 +3 +4 +5

? = 10 + 5 = 15
Example 1 Which number would replace question mark (?) in
the given series? 11, 22, 44, 77, 121, ? Second series is the squares of the correspondent
number of first series.
(a) 167 (b) 176 (c) 165 (d) 146
1, 1, 3, 9, 6, 36, 10, 100, ?, 225
Solution (b)
11 22 44 77 121 ? 12 = 1, 3 2 = 9 , 6 2 = 36 , 10 2 = 100, 152 = 225
+11 +22 +33 +44 +55
Example 5 Find the number in the given series, which is
121 + 55 = 176 incorrect. 1, 2, 4, 8, 16, 32, 64, 96
Example 2 Find out the missing number in the following (a) 16 (b) 96 (c) 64 (d) 32
series. 4, 10, 22, 46, 94, ?
Solution (b) 1 2 4 8 16 32 64 96
(a) 190 (b) 178 (c) 122 (d) 116
×2 ×2 ×2 ×2 ×2 ×2 ×2
Solution (a) The order of the given series is as follows
? = 64 × 2 = 128
4 10 22 46 94 ? Hence, 96 should be replaced by 128.
×2+2 ×2+2 ×2+2 ×2+2 ×2+2
î For solving this type of number series questions, we should check
? = 94 × 2 + 2 = 190 the following thing
(i) Each number of series may be multiply by a fix number.
Example 3 Find the missing term in the following series. (ii) The numbers may be divided to get the next number.
11, 23, 48, 99, ? , 409 (iii) The numbers may be squared to get the next one.
(a) 202 (b) 212 (c) 102 (d) 216 (iv) Sometimes, something is added to get the next number or
subtracted or divided or multiplied to get the next one.
Series 35
Type II Alphabet Series Solution (b) A B D G K

The series in which all the terms are alphabet letters +1 +2 +3 +4

called alphabet series. ∴ ?= K

Directions (Ex. 6 and 7) In each of the following Example 7 CE, GI, KM, OQ, ?
questions, various terms of a letter series are given with one (a) ST (b) TP (c) SU (d) SQ
term missing. Choose the missing term out of the given
Solution (c) +2 +2 +2 +2
alternatives.
C E G I K M O Q S U
Example 6 A, B, D, G, ?
(a) J (b) K +2 +2 +2 +2 +2

(c) L (d) M ∴ ? = SU

Type III Alpha-Numeric Series


The series in which both alphabets and numbers î Trick 9 + 7 = 16th term from right
constitute the series is called alpha-numeric series. Hence, seventh to the left to the ninth from right means
16th from right is 5.
Directions (Ex 8 and 9) Study the following
letter/number sequence and answer the questions given below. Example 10 What should come in place of the question
mark (?) in the following letter series with reference to the below
B3KG5PD79ERJIMT6ZHQ3W2A given sequence?
Example 8 Which of the following is exactly in the middle KBH, PGM, ULR, ?
between the ninth letter/number from the left end and seventh (a) ZQW (b) ZHW (c) Z82 (d) ZQ2
letter/number from the right end?
(a) I (b) R (c) M (d) J Solution (a) K
+5
P
+5
U
+5
Z
Solution (a) The ninth letter/ number from the left is 9 +5 +5 +5
and seventh letter/number from the right is Z. From 9 to B G L Q
Z, there are total nine letters and numbers. So, middle +5 +5 +5
letter between 9 and Z is I. H M R W
E R J I M T 6
Example 9 If the first twelve ∴ ? = ZQW
letters/numbers in the above sequence are Middle term
Example 11 a a b __ a a a __ b b a __
written in the reverse order, which of the following letter/number
will be seventh to the left to ninth letter/number from our right? (a) b a a (b) a b b (c) b a b (d) a a b
(a) K (b) P Solution (a) Divide it into triplet
(c) 5 (d) None of these a a b, __ a a, a __ b, b a __
Solution (c) 12th term from left is J. Therefore, we have Thus, the missing letters for completing the series are b,
to start the sequence, with J in the reverse order. a and a.
Thus, J R E 9 7 D P 5 G K 3 B I M T 6 Z H Q 3 W 2 A The answer b, a and a, then series will be
aab baa aab baa
7th from left to T 9th from right Hence, required answer is b a a.

Type IV Continuous Pattern Series


In continuous pattern series, a series of small/capital Example 12 In the following series, some of the letters are
letters is given which follows a particular pattern. missing. Select the correct alternatives.
However, some letters are missing from the series. In mc_m_a_ca_ca_c_mc
some of the questions, a set of letters is given four or five (a) acmmma (b) camcam (c) aaacmm (d) acmmca
times with blank spaces or question marks in between.
Solution (a) Here, the pattern is as follows
The series follows a specific pattern and candidates are
mca / mca / mca / mca / mca / mc
required to find the letters which should come in place of
⇒ Required answer is acmmma.
the blank spaces or question marks.
Target Exercise
Elementary Level Questions
Directions (Q. Nos. 1-15) Complete the given series. 17. 5, 11, 24, 52, 106
(a) 11 (b) 24
1. 5, 9, 17, 29, 45, ? (c) 52 (d) 106
(a) 60 (b) 65 (c) 70 (d) 68
18. 7, 9, 16, 25, 41, 66, 105
2. 15, 51, 105, ? (a) 41 (b) 66
(a) 501 (b) 51 (c) 15 (d) 150 (c) 16 (d) 105
3. 25, 20, 15, 10, ? 19. 8, 24, 72, 248, 648
(a) 50 (b) 75 (c) 100 (d) 5 (a) 72 (b) 248 (c) 24 (d) 648
4. 3, 7, 15, 31, ? 20. 10, 90, 170, 250, 340, 410
(a) 63 (b) 53 (c) 43 (d) 73 (a) 170 (b) 250 (c) 340 (d) 410
5. 1, 2, 6, 15, 31, ?
(a) 56 (b) 55 (c) 57 (d) 58 Directions (Q. Nos. 21-25) Complete the given series.
6. 4, 9, 19, 34, 54, ? 21. AZ, GT, MN, ?, YB
(a) 79 (b) 74 (c) 89 (d) 83 (a) KF (b) RX (c) SH (d) TS

7. 1, 9, 25, 49, ? 22. AZ, CX, FU, ?


(a) 64 (b) 56 (c) 71 (d) 81 (a) IR (b) IV (c) JQ (d) KP

8. 0, 3, 8, 15, ?, 35 23. OTE, PUF, QVG, RWH, ?


(a) 24 (b) 25 (c) 26 (d) 20 (a) SYJ (b) TXI (c) SXJ (d) SXI

9. 1, 8, 27, 64, ?, 216 24. BEH, KNQ, TWZ, ?


(a) 127 (b) 125 (c) 124 (d) 128 (a) IJL (b) CFI (c) BDF (d) ADG

10. 1, 3, 6, 10, 15, ?, 28, 36 25. MHZ, NIW, OKT, PNQ, ?


(a) 24 (b) 22 (a) RRN (b) QRN (c) QRM (d) QQN
(c) 64 (d) 21

11. 6, 36, 7, 49, 8, 64, ?, 81, 10, 100 Directions (Q.Nos. 26-28) Study the following
arrangement carefully and answer the questions given below.
(a) 9 (b) 8 (c) 69 (d) 66
F @ 3 9 H A D I % 4 E $MK2URP5Wδ81TJV7
12. 90, 84, 86, 80, ?, 76, 78
(a) 87 (b) 82 (c) 77 (d) 89 26. How many consonants are there in the given
13. 2, 5, 10, 13, 26, 29, ? arrangement, which are preceded by a number and
(a) 32 (b) 44 (c) 52 (d) 58 not followed by a letter?
(a) None (b) One
14. 7, 13, 25, 43, 67, ? (c) Two (d) Three
(a) 99 (b) 100 (c) 97 (d) 98
27. Four of the following five options are alike in a certain
15. 17, 25, 41, 65, 97, ? way, based on their positions in the above
(a) 147 (b) 100 (c) 137 (d) 98 arrangement and so form a group. Which is the one
that does not belong to that group?
Directions (Q. Nos.16-20) In each of the following (a) D %  (b) 5 δ R (c) T V 8 (d) E % $
questions, one term is wrong. Find out the wrong/incorrect
term. 28. If all the symbols from the above arrangement are
dropped, then which of the following will be the
16. 11, 22, 44, 77, 122 twelfth from the left end?
(a) 22 (b) 122 (c) 77 (d) 44 (a) 4 (b) K (c) 2 (d) U
Series 37
High Skill Questions
Directions (Q. Nos. 1-15) Find the missing term in the Directions (Q.Nos. 16-25) In each of the following
following questions of series. questions, a letter series is given in which some of the letters
are missing. The missing letters are given in that order as
1. 6, 9, 18, 21, 42, 45, ?, ?
one of the alternatives below it. Choose the correct
(a) 90 and 91 (b) 90 and 92
alternative.
(c) 90 and 93 (d) 90 and 94
16. a b_ _ _ b_ b b a a_
2. 9, 5, 14, 19, 33, ? (a) a b a a b (b) a b b a b
(a) 50 (b) 48 (c) b a a a b (d) b a b b a
(c) 62 (d) 52
17. _ _a b a_b a_a b
3. 7, 11, 13, 17, 19, 23, ? (a) a b b a b (b) a b b b
(a) 27 (b) 29 (c) b a a b b (d) b b a b a
(c) 26 (d) 24
18. _b a a_a a b_a_a
4. 5, 7, 10, 15, 22, ? (a) a a b b (b) a a b a
(a) 33 (b) 25 (c) a b a b (d) b a a b
(c) 27 (d) 29
19. a_b b c_a a b_c c a_b b c c
5. 0, 6, 24, 60, 120, 210, ? (a) b a c b (b) a c b a
(a) 240 (b) 290 (c) a b b a (d) c a b a
(c) 336 (d) 540 20. b c_ b_ c_ b_c c b
(a) c b c b (b) b b c b
6. 10, 15, 30, 45, 90, ?
(c) c b b c (d) b c b c
(a) 140 (b) 135
(c) 145 (d) 125 21. a b b_b a a_a_b a b_a b
(a) a b b a (b) a b a b
7. 1, 4, 2, 8, 6, 24, 22, 88, ? (c) c c a c (d) a a b b
(a) 86 (b) 90
(c) 154 (d) 352 22. a a b_a a_c a a_ c a a_ c
(a) b b c a (b) b c b c
8. 7, 26, 63, 124, 215, 342, ? (c) c a b a (d) c b b b
(a) 541 (b) 415
(c) 511 (d) 521 23. a c_c a b_b a c a_a b a_a c a c
(a) a a c b (b) a c b c
9. YEB, WFD, UHG, SKI, ? (c) b a b b (d) b c b b
(a) QOL (b) QGL
24. _ b c_ a b_c a a b c
(c) TOL (d) QNL
(a) a c b (b) b a b
10. A, CD, GHI, ?, UVWXY (c) a b a (d) a a c
(a) LMNO (b) MNOL 25. a b_ b c_ c_b a_c
(c) NOPQ (d) MNOP (a) b a a c (b) a a b b
11. 3 F, 6 G, 11 I, 18 L, ? (c) c a a b (d) a a a b
(a) 21 O (b) 25 N
(c) 27 P (d) 27 Q Directions (Q. Nos. 26-28) Find the missing term in the
following series.
12. CX, FU, IR, ?, OL, RI
(a) LO (b) MN 26. APZLT, CQYNR, ERXPP, GSWRN, ITVTL, ?
(c) NO (d) OP (a) KUUVJ (b) KVUUJ
(c) JUVUR (d) KVUVJ
13. BF, CH, ?, HO, LT
(a) DN (b) EL (c) EK (d) EM 27. ACEGI, JLNPR, SUWYA, ?, KMOQS
(a) LBGHI (b) ADIFH
14. C1L, F4O, I9R, L16U, ? (c) YWXAC (d) BDFHJ
(a) O25X (b) N25X (c) N25Y (d) N20Z
28. 3624, 4363, 3644, 4563, 3664 ?
15. KM5, IP8, GS11, EV14, ?
(a) 4263 (b) 4363
(a) BY17 (b) CY18 (c) CZ17 (d) CY17
(c) 4563 (d) 4763
38 Study Package for NTSE

Directions (Q. Nos. 29-31) A pyramid of letters is given 30. ZWOHL, BQFLM, GTRMQ, ?, ESJGZ
below. Study the pyramid and select the correct alternatives (a) OXCQG (b) HESFR
to fill in the missing term. (c) OCXCQ (d) XCJQG
P 31. PJXZM, LBQZG, ?
Z J M (a) DWZYS (b) AUCBO
(c) UACBO (d) DZWYS
L Y X U I
Z B G O E V H 32. What letter will come next in the following series?
A W Q T X S D E M
ABCDEFZYXWVUBCDEYXWV
(a) C (b) A
B U O F R C J W S I T
(c) U (d) B
N P C H L M Q G Z Y R F D
33. What letter will come next in the following series?
29. LBQF, UXYL, GTRM, ? ABCDABCDEABCDEFABCDEF
(a) HEVO (b) DSXT (a) E (b) G
(c) VEOG (d) IUXY (c) F (d) H

Hints and Solutions


Elementary Level Questions
1. (b) The pattern of the series is 8. (a) The pattern of the series is
0 3 8 15 24 35
5 9 17 29 45 65
+3 +5 +7 +9 +11
+4 +8 +12 +16 +20 ∴ ? = 24
+4 +4 +4 +4 9. (b) The pattern of the series is
? = 45 + 20 = 65 1 8 27 64 125 216
2. (a) 15 ←→ 51,105 ←→ 501 reverse order of numerals.
3. (d) The pattern of the series is 13 23 33 43 53 63
25 20 15 10 5 ∴ ? = 125
–5 –5 –5 –5 10. (d) The pattern of the series is
21 28 36
∴ ?=5
1 3 6 10 15

4. (a) The pattern of the series is +2 +3 +4 +5 +6 +7 +8


3 7 15 31 63 ? = 15 + 6 = 21

×2+1 ×2+1 ×2+1 ×2+1 11. (a) The pattern of the series is
6 36 7 49 8 64 9 81 10 100
∴ ? = 63
×6 ×7 ×8 ×9 ×10
5. (a) The pattern of the series is
1 2 6 15 31 56 ∴ ?=9

+12 +22 +32 +42 +52 12. (b) The pattern of the series is
∴ ? = 56 90 84 86 80 82 76 78

6. (a) The pattern of the series is –6 +2 –6 +2 –6 +2


4 9 19 34 54 79 Q ? = 82

+5 +10 +15 +20 +25 13. (d) The pattern is as follows


2 5 10 13 26 29 58
+5 +5 +5 +5
+3 ×2 +3 ×2 +3 ×2
∴ ? = 79
∴ ? = 58
7. (d) The pattern of the series is
1 9 25 49 81 14. (c) The pattern of the series is
7 13 25 43 67 97

+6 +12 +18 +24 +30


12 32 52 72 92
+6 +6 +6 +6
∴ ? = 81
? = 97
Series 39
15. (c) The pattern of the series is 22. (c) The pattern of the series is
17 25 41 65 97 37 A Z C X F U J Q
+8 +16 +24 +32 +40 +2
+3 +4
+8 +8 +8 +8 –2
–3
? = 137 –4
∴ ? = JQ
16. (b) The pattern of the series is
121 23. (d) The pattern of the series is
11 22 44 77 122 +1 +1 +1 +1
O P Q R S
+11 +22 +33 +44 +1 +1 +1 +1
T U V W X
Hence, term 122 is wrong, it should be replaced by 121. +1 +1 +1 +1
E F G H I
17. (c) The pattern of the series is
51 ∴ ? = SXI
5 11 24 52 106
24. (b) The pattern of the series is
×2+1 ×2+2 ×2+3 ×2+4 +9 +9 +9
B K T C
Hence, term 52 is wrong. It should be replaced by 51. +9 +9 +9
E N W F
18. (d) The pattern of the series is H
+9
Q
+9
Z
+9
I
7 + 9 = 16 ∴ ? = CFI
9 + 16 = 25
25. (b) The pattern of the series is
16 + 25 = 41 +1 +1 +1 +1
M N O P Q
25 + 41 = 66 +1 +2 +3 +4
H I K N R
41 + 66 = 105 –3 –3 –3 –3
Z W T Q N
Hence, term 105 is wrong, it should be replaced by 107.
∴ ? = QRN
19. (b) The pattern of the series is
216 26. (c) Number Consonant Number Symbol
8 24 72 248 648
Here, F @ 3 9 H  A D I % 4 E $MK2URP5Wδ 81TJV7
×3 ×3 ×3 ×3 In the given arrangement, 2 such consonants are there viz. 9 H 
Hence, term 248 is wrong. It should be replaced by 216. and 5 W δ which are preceded by a number and not followed by a
20. (c) The pattern of the series is letter.
330
27. (d) Here, F @ 3 9 H  A D I % 4 E $MK2URP5Wδ 81TJV
10 90 170 250 340 410
7
+80 +80 +80 +80 +80 In all the groups except ‘E % $’, second element is two positions
Hence, term 340 is wrong. It should be replaced by 330. ahead of first element and the third element is two positions
21. (c) The pattern of the series is behind to the first element.
+6 +6 +6 +6 28. (c) After dropping all the symbols, the resultant arrangement will
A G M S Y
be
Opposite Opposite Opposite Opposite Opposite
letter letter letter letter letter F39HADI4EMK 2 URP5W81TJV7
Z T N H B
12th from left
∴ ? = SH
So, twelfth element from the left is 2.
40 Study Package for NTSE

High Skill Questions


1. (c) The pattern of the series is 12. (a) The pattern of the series is
×2 ×2 ×2 C X F U I R L O O L R I
+3
6 9 18 21 42 45 90 93 +3 +3 +3
–3 –3 –3 –3
+3 +3 +3
∴ ? = LO
+3
∴ ? = 90, 93 13. (c) The pattern of the series is
+2 +3 +4 +5
2. (d) Each number is the total of its two preceeding numbers.
Hence, the missing number is 52. (19 + 33 = 52 ) BF CH EK HO LT

3. (b) A prime number series, next number in series will be 29. +1 +2 +3 +4

4. (a) The pattern of the series is Q ? = EK


14. (a) The pattern of the series is
5 7 10 15 22 33 +3 +3 +3 +3
C F I L O
+2 +3 +5 +7 +11 +3 +5 +7 +9
1444442444443 1 4 9 16 25
Prime number +3 +3 +3 +3
L O R U X
Q ? = 33
∴ ? = O 25 x
5. (c) Given series is 13 − 1, 2 3 − 2, 33 − 3, 43 − 4, 53 − 5, 63 − 6
15. (d) The pattern of the series is
etc., next term 7 3 − 7 = 336 –2 –2 –2 –2
K I G E C
6. (b) The pattern of the series is M
+3
P
+3
S
+3
V
+3
Y
×3 ×3 +3 +3 +3 +3
5 8 11 14 17
10 15 30 45 90 135 ∴ ? = CY17
×3 ×3 16. (c) The series is : a, b b; a a, b; a, b b; a a, b
∴ ? = 135 Required answer is b a a a b
17. (a) The series is : a b, a b, a b, a b, a b, a b
7. (a) The pattern of the series is
Required answer is a b b a b
1 4 2 8 6 24 22 88 86
18. (c) The series is : a b a, a b a, a b a, a b
×4 –2 ×4 –2 ×4 –2 ×4 –2 Required answer is a b a b
∴ ? = 86 19. (b) The series is : a a, b b, c c, a a, b b, c c, a a, b b, c c
Required answer is a c b a
8. (c) The pattern of the series is
20. (a) The series is : b c c, b b, c c, b b, c c, b
2 3 − 1, 33 − 1, 43 − 1, 53 − 1, 63 − 1,7 3 − 1, 83 − 1
Required answer is c b c b
Q = 8 × 8 × 8 − 1= 512 − 1 = 511
21. (a) The series is : a b b a, b a a b, a b b a, b a a b
9. (a) The pattern of the series is Required answer is a b b a
–2 –2 –2 –2
Y W U S Q 22. (d) The series is : a a b c; a a b c, a a b c, a a b c
+1 +2 +3 +4
E F H K O Requried answer is c b b b
+2 +3 +2 +3
B D G I L 23. (a) The series is : a c a c, a b a b, a c a c, a b a b, a c a c
∴ ? = QOL Required answer is a a c b

10. (d) The pattern of the series is 24. (a) The series is : a b c, c a b, b c a, a b c
Requried answer is a c b
A CD GH I MNOP UVWXY
25. (c) The series is : a b c, b c a, c a b, a b c
+2 +3 +4 +5
Required answer is c a a b
The number of letters in each term is increasing by one.
26. (a) The pattern of the series is
∴ ? = MNOP +2 +2 +2 +2 +2
A C E G I K
11. (c) The pattern of the series is +1 +1 +1 +1 +1
P Q R S T U
+3 +5 +7 +9 –1 –1 –1 –1 –1
3 6 11 18 27 Z Y X W V U
+1 +2 +3 +4 +2 +2 +2 +2 +2
F G I L P L N P R T V
–2 –2 –2 –2 –2
T R P N L J
∴ ? = 27 P
∴ ? = KUUVJ
Series 41
27. (d) The pattern of the series is 30. (a) P
+1 +1
A C E G I J L N P R S U W Y A Z J M
L Y X U I
+2 +2 +2 +2 +2 +2 +2 +2 +2 +2 +2 +2
Z B G O E V H
+1 A W Q T X S D E M
B D F H J K M O Q S
B U O F R C J W S I T
+2 +2 +2 +2 +2 +2 +2 +2
N P C H L M Q G Z Y R F D
28. (d) Reverse the previous number and add 1 to the same digit
each time. ? = OXCQG
⇒ 3624 → 4263 31. (b)
|
→ + 1 = 4363 P
Z J M
3644 → 4463
|→ + 1 4563 L Y X U I
=
Z B G O E V H
3664 → 4663
|→ + 1
= 4763
A W Q T X S D E M

∴ ? = 4763 B U O F R C J W S I T

29. (c) N P C H L M Q G Z Y R F D
P
Z J
? = AUCBO
M
L Y X U I 32. (a) The given series contain two different series,
Z B G O E V H I→ ABCDEF BCDE CD
A W Q T X S D E M
B U O F R C J W S I T II → Z Y X W V U YXWV

N P C H L M Q G Z Y R F D
33. (b) The series follows below pattern
? = VEOG ABCD/ABCDE/ABCDEF/ABCDEF G
Chapter

6
Complex
Sequence
In these type of questions, a long series of numbers with or without repetitions is given. The candidate must find
out how many times a number satisfying the conditions, specified in the questions.
While solving the questions based on complex sequence candidate is required to follow the given steps
Step 1. First of all, spot the number in the given sequence.
Step 2. Observe minutely to the left and right of such numbers to examine whether they satisfying the conditions or not
as given in the question.
Step 3. Finally, count the numbers which satisfying the required conditions.

Example 1 In the series given below, how many 8’s are there which are exactly divisible by its immediate preceding
as well as succeeding numbers?
2838248248682824838286
(a) One (b) Two (c) Three (d) Four
Solution (b) Given series,
28382 48248682824838286
↓ ↓
Therefore, there are only two such 8’s that fulfill the given condition.

Example 2 In the following series, how many such odd numbers are there which are divisible by 3 or 5, then
followed by odd number and again followed by even number?
12, 19, 21, 3, 25, 18, 35, 20, 22, 21, 45, 46, 47, 48, 9, 50, 52, 54, 55, 56
(a) Zero (b) One (c) Two (d) Three
Solution (c) Odd numbers in the given series 19, 21, 3, 25, 35, 21, 45, 47, 9, 55
Odd numbers divisible by 3 : 21, 3, 21, 45
Odd numbers divisible by 5 : 25, 35, 45, 55
Multiple of 3 or 5 (odd) Odd number Even number

12, 19, 21, 3, 25, 18, 35, 20, 22, 21, 45, 46, 47, 48, 9, 50, 52, 54, 55, 56

Satisfies conditions Satisfies conditions


Thus, there are two such odd numbers.
Target Exercise
Elementary Level Questions
1. How many such 4s are there in the following number 6. How many 8s are there in the following number
series which are immediately preceded by an odd sequence which are immediately followed by 5 but not
number and immediately followed by an even immediately preceded by 3?
number? 385685857385285638578532585
12342341344241231234132412341241 (a) Five (b) Six (c) Seven (d) Four
(a) Nil (b) One
7. How many 6s are there in the following number
(c) Two (d) Three
sequence which are immediately followed by an even
2. How many 5s are there in the following number number and immediately preceded by one odd
sequence which are immediately followed by 2 and number?
immediately preceded by 7? 3625632632627646586764268
6543752378525775275314752352 (a) One (b) Two (c) Three (d) Four
(a) Nil (b) One
(c) Two (d) Three 8. How many odd numbers are there in the following
sequence which are immediately followed by an odd
3. How many such 5s are there in the following number number?
sequence which are immediately preceded by 7 but
51473985726315863852243496
not immediately followed by 9?
(a) Five (b) Two (c) Three (d) Four
25975214759317528759475
(a) One (b) Two (c) Three (d) Four 9. How many numbers amongst the numbers 9 to 99 are
there which are exactly divisible by 9 but not by 3?
4. How many 7s are there in the following sequence (a) Eight (b) Ten (c) Twelve (d) None of these
which are preceded by 5 as well as followed by 3?
7753757377335737573 10. Nitin was counting down from 32. Sumit was
(a) One (b) Two (c) Three (d) Four counting upwards the numbers starting from 1 and
he was calling out only the odd numbers. What
5. How many 5s are there in the following number series common number will they all out at the same time,
which are immediately followed by 3 but not if they were calling out at the same speed?
immediately preceded by 7? (a) 19
2153253855687335775365325739 (b) 21
(a) One (b) Two (c) 22
(c) Three (d) Four (d) They will not call out the same number

High Skill Questions


1. How many such 9s are there in the following number 3. How many even numbers are there in the series
sequence which are either immediately preceded by or which are preceded as well as followed by an even
immediately followed by an even number in the series? number?
(a) One (b) Two (c) Three (d) Four
39549783921897315699516972
(a) Two (b) Three 4. In the following number series, how many 5s are
(c) Four (d) More than four there which are immediately preceded by an odd
number and immediately followed by an even
Directions (Q. Nos. 2-3) Read the following number series number?
carefully and answer the questions given below it. 249765543794583567984256752
(a) Two (b) Three (c) Four (d) Five
7828938523847819885
2. How many 8s are there in the series which are 5. How many pairs of successive numbers have a
preceded by an odd number but not followed by an difference of 2 each in the below series?
even number? 641228742153862171453286
(a) Three (b) Two (c) One (d) Five (a) Four (b) Three (c) Five (d) Six
44 Study Package for NTSE

6. How many 3s are there in the following number 8. In the following series of numbers, how many times 1,
sequence which are immediately preceded by 7 but not 3 and 7 have appeared together, 7 being in the middle
immediately followed by 4? and 1 and 3 on either side of 7?
73483736534735873437353 2973173771331738571377173906
(a) Four (a) Three (b) More than three
(b) Three (c) Two (d) More than five
(c) Five
(d) Six Directions (Q. Nos. 9 and 10) Study the number series
given below and answer the questions that follow.
7. In the following number sequence, how many such
even numbers are there which are exactly divisible by 7897653428972459297647
its immediate preceding number but not exactly 9. How many 7’s are preceded by 9 and followed by 6?
divisible by its immediate following number? (a) Two (b) Three (c) Four (d) Five
38415728348939421582
10. Which figures have equal frequency?
(a) Five (b) Two
(a) 253 (b) 245 (c) 375 (d) 865
(c) Three (d) Four

Hints and Solutions


Elementary Level Questions
1. (c) Odd number 4 Even number 2153253855687335775365335739
Thus, there are three such 5’s.
12 3 4 2 3 4 1 3 4 4 2 4 1 2 3 1 2 3 4 1 3 2 4 1 2 3 4 1 2 4 1
6. (a) 3 8 5 6 8 5 8 5 7 3 8 5 2 8 5 6 3 8 5 7 8 5 3 2 5 8 5
Thus, there are two such 4s.
↓ ↓ ↓ ↓ ↓
2. (d) 6 5 4 3 7 5 2 3 7 8 5 2 5 7 7 5 2 7 5 3 1 4 7 5 2 3 5 2 Satisfies conditions
↓ ↓ ↓ Thus, there are five such 8’s.
Satisfies Satisfies Satisfies 7. (c) Odd number 6 Even number
conditions conditions conditions
3625632632627646 586764268
Therefore, there are three such 5’s.
↓ ↓ ↓
3. (c) 2 5 9 7 5 2 1 4 7 5 9 3 1 7 5 2 8 7 5 9 4 7 5 Satisfies conditions Satisfies conditions
↓ ↓ ↓ Thus, there are three 6’s satisfy the condition.
Satisfies Satisfies Satisfies
conditions conditions conditions 8. (d) 5 1 4 7 3 9 8 5 7 2 6 3 1 5 8 6 3 8 5 2 2 4 3 4 9 6
Therefore, there are three such 5’s. ↓ ↓ ↓ ↓
4. (c) 7 7 5 3 7 5 7 3 7 7 3 3 5 7 3 7 5 7 3 Satisfies conditions
↓ ↓ ↓ There are four odd numbers satisfy the condition which are
Satisfies Satisfies Satisfies immediately followed by an odd number.
conditions conditions conditions
9. (d) 9, 18, 27, 36, 45, 54, 63, 72, 81, 90, 99
Therefore, there are three such 7’s. There is no such number because a number which is divisible by
5. (c) A number which comes after a given number is said to follow 9 is also divisible by 3.
it while one which comes before the given number preceded it. 10. (d) Nitin—32, 31, 30, 29, 28, 27, 26, 25, 24
The 5s satisfying the given conditions, can be shown in the Sumit—1, 3, 5, 7, 9, 11, 13, 15, 17
series as
Clearly, both will never call out the same number.
Complex Sequence 45
High Skill Questions 5. (d) In the series,
6412287 421 53 8 6 21714 53 286
1. (d) 2 or 4 or 6 or 8 9 or 9 2 or 4 or 6 or 8
There are six pairs having a difference of 2.
3954978392189 731 56 9 95 1 6 9 72
6. (b) In the series,
Thus, there are five such 9s.
2. (b) Odd number 8 Odd number 73483 7 3 6 534 7 3 5 87343 7 3 5 3
Thus, here only three such 3’s which preceded by 7 but not
782893852384 7819885
immediately followed by 4.
↓ ↓
Satisfies Satisfies 7. (b) 3 8 4 1 5 7 2 8 3 4 8 9 3 9 4 2 1 5 8 2
conditions conditions
Thus, there are two such 8s. So, there are two such sets which satisfy the conditions given in
the question.
3. (a) Even number Even number Even number
8. (a) 1 7 3 or 3 7 1
7 82838523847819885
2973 1 7 3 77133 1 7 3 8571377 1 7 3 906

Satisfies conditions Thus, three times 1, 3 and 7 have appeared together.
Thus, there is only one even number which is preceded as well
as followed by even numbers. 9. (a) 7 8 9 7 6 5 3 4 2 8 9 7 2 4 5 9 2 9 7 6 4 7
Thus, there are two times 7’s are preceded by 9 and followed
4. (b) Odd number 5 Even number
by 6.
24976 5 5 4 37945 8 3 5 6 7984256 7 5 2
↓ ↓ ↓
10. (d) In the given series, 2 occurs 3 times, 3 occurs once, 4 occurs
3 times, 5 occurs 2 times, 6 occurs 2 times, 7 occurs 5 times, 8
Satisfies Satisfies Satisfies
occurs 2 times and 9 occurs 4 times clearly. The frequency of
conditions conditions conditions
5, 6 and 8 is same i.e., 2.
Thus, there are three such 5s.
Chapter

7
Logical Sequence
of Words
In these type of questions, a group of words is given. The candidate is required to arrange these words in a
meaningful order such as the sequence of occurrence of events, sequence form a part to the whole, sequence of
increasing/decreasing size, value, intensity, etc., and then choose the correct sequence accordingly.
Example 1 Arrange the following in a meaningful sequence.
(1) Consultation (2) Illness
(3) Doctor (4) Treatment
(5) Recovery
(a) (2), (3), (1), (4), (5) (b) (2), (3), (4), (1), (5)
(c) (4), (3), (1), (2), (5) (d) (5), (1), (4), (3), (2)
Solution (a) We know that, illness occurs first. Then, patient goes to the doctor and after consultation with
him undergoes treatment to finally attain recovery.
Thus, the correct order is (2), (3), (1), (4) and (5).

Example 2 Arrange the following in a meaningful sequence.


(1) Cut (2) Stitch
(3) Mark (4) Measure
(5) Tailor
(a) (5), (4), (3), (1), (2) (b) (4), (5), (3), (2), (1)
(c) (5), (4), (3), (2), (1) (d) (5), (3), (4), (2), (1)
Solution (a) The meaningful sequence is
Tailor → Measure → Mark →Cut → Stitch i.e., (5), (4), (3), (1) and (2).

Example 3 Arrange the following in a meaningful sequence.


(1) Country (2) Furniture
(3) Forest (4) Wood
(5) Trees
(a) (1), (3), (5), (4), (2) (b) (1), (4), (3), (2), (5)
(c) (2), (4), (3), (1), (5) (d) (5), (2), (3), (1), (4)
Solution (a) From the above words, it is deduced that a country contains forests, a forest has trees. Trees give
wood that is used to make furniture.
Thus, the correct order is (1), (3), (5), (4), (2).
Target Exercise
Elementary Level Questions
Directions (Q. Nos. 1-20) In the following questions, arrange the given words in a meaningful sequence and then choose the
most appropriate sequence from the options given below in the each question.
1. (1) Maharashtra (2) Shirdi 10. (1) September (2) April
(3) India (4) Asia (3) December (4) August
(5) World (5) March
(a) (2), (1), (3), (4), (5) (b) (1), (2), (5), (4), (3) (a) (1), (2), (3), (4), (5) (b) (5), (2), (4), (1), (3)
(c) (5), (1), (2), (4), (3) (d) (3), (2), (1), (4), (5) (c) (5), (4), (3), (2), (1) (d) (5), (4), (2), (1), (3)

2. (1) Village (2) State 11. (1) Mother (2) Cry


(3) World (4) Country (3) Milk (4) Smile
(5) District (5) Child
(a) (1), (5), (3), (4), (2) (b) (1), (5), (2), (4), (3) (a) (5) , (2), (1), (3), (4) (b) (1), (5), (4), (3), (2)
(c) (1), (5), (3), (2), (4) (d) (1), (2), (5), (4), (3) (c) (1), (4), (3), (2),(5) (d) (4), (2), (3), (5), (1)

3. (1) Post-box (2) Delivery 12. (1) Lake (2) Sea


(3) Letter (4) Envelope (3) River (4) Pool
(5) Clearance (5) Pond
(a) (4), (3), (1), (5), (2) (b) (4), (1), (3), (2), (5) (a) (4), (1), (2), (3), (5) (b) (4), (3), (2), (1), (5)
(c) (4), (2), (3), (5), (1) (d) (4), (3), (5), (2), (1) (c) (4), (5), (1), (3), (2) (d) (4), (5), (2), (1), (3)

4. (1) Child (2) Play-way 13. (1) Seed (2) Wood


(3) College (4) School (3) Tree (4) Plant
(5) Job (5) Furniture
(a) (2), (1), (3), (4), (5) (b) (1), (5), (3), (2), (4) (a) (1), (3), (4), (2), (5) (b) (1), (4), (3), (2), (5)
(c) (1), (2), (4), (5), (3) (d) (1), (2), (4), (3), (5) (c) (1), (3), (4), (5), (2) (d) (1), (4), (3), (5), (2)

5. (1) Locality (2) Member 14. (1) Whale (2) Elephant


(3) Community (4) Country (3) Dog (4) Tiger
(5) Family (5) Mosquito
(a) (2), (5), (3), (1), (4) (b) (2), (3), (4), (1), (5) (a) (5), (3), (4), (2), (1) (b) (5), (4), (1), (2), (3)
(c) (2), (5), (1), (3), (4) (d) (2), (3), (5), (4), (1) (c) (5), (3), (4), (1), (2) (d) (5), (3), (2), (4), (1)

6. (1) Delhi (2) India 15. (1) Bungalow (2) Flat


(3) Universe (4) World (3) Cottage (4) House
(5) Asia (5) Palace
(a) (1), (5), (2), (3), (4) (b) (2), (1), (5), (3), (4) (a) (3), (5), (4), (2), (1) (b) (3), (2), (4), (1), (5)
(c) (1), (2), (4), (5), (3) (d) (1), (2), (5), (4), (3) (c) (5), (1), (2), (4), (3) (d) (3), (4), (2), (1), (5)
7. (1) Deepawali (2) Independence day 16. (1) Child (2) Job
(3) Christmas (4) Holi (3) School (4) College
(5) Republic day (5) Salary
(a) (5), (4), (2), (3), (1) (b) (5), (4), (3), (2), (1)
(a) (1), (3), (2), (4), (5) (b) (1), (4), (5), (2), (3)
(c) (5), (4), (2), (1), (3) (d) (4), (1), (2), (3), (5)
(c) (1), (2), (3), (4), (5) (d) (1), (3), (4), (2), (5)
8. (1) VI (2) IX 17. (1) Key (2) Door
(3) VIII (4) XI (3) Lock (4) Room
(5) V (5) Switch on
(a) (5), (1), (2), (4), (3) (b) (5), (2), (4), (3), (1)
(a) (3), (2), (4), (5), (1) (b) (1), (2), (3), (5), (4)
(c) (5), (3), (1), (2), (4) (d) (5), (1), (3), (2), (4)
(c) (1), (2), (3), (4), (5) (d) (2), (1), (3), (4), (5)
9. (1) Reading (2) Composing 18. (1) Rainbow (2) Rain
(3) Proofreading (4) Writing (3) Sun (4) Happy
(5) Printing (5) Child
(a) (4), (3), (2), (1), (5) (b) (4), (2), (3), (5), (1)
(a) (2), (4), (1), (5), (3) (b) (2), (3), (1), (5), (4)
(c) (4), (2), (3), (1), (5) (d) (5), (4), (3), (2), (1)
(c) (4), (3), (5), (1), (2) (d) (2), (4), (1), (3), (5)
48 Study Package for NTSE

19. (1) Morning (2) Afternoon 20. (1) Graduate education


(3) Night (4) Evening (2) Secondary education
(5) Midnight (3) Pre-primary education
(a) (1), (2), (5), (4), (2) (4) Middle education
(b) (1), (2), (3), (4), (5) (5) Primary education
(c) (1), (2), (4), (3), (5) (a) (5), (4), (3), (2), (1) (b) (4), (5), (3), (2), (1)
(d) (1), (2), (4),(5), (3) (c) (3), (5), (4), (2), (1) (d) (3), (5), (4), (1), (2)

High Skill Questions


Directions (Q. Nos.1-15) In the following questions, arrange the given words in a meaningful sequence and then choose
the most appropriate sequence from the options given below in the each question.
1. (1) Copper (2) Iron 8. (1) Yarn (2) Plant
(3) Silver (4) Gold (3) Saree (4) Cotton
(5) Platinum (5) Cloth
(a) (1), (2), (3),(5), (4) (b) (1), (2), (4),(5), (3) (a) (2), (5), (4), (3), (1) (b) (2), (1), (3), (4),(5)
(c) (2), (1), (3), (4), (5) (d) (5), (4), (2), (3), (1) (c) (2), (4), (1), (5), (3) (d) (2), (4), (3), (5), (1)
2. (1) July (2) August 9. (1) Index (2) Content
(3) May (4) April (3) Title (4) Chapter
(5) June (5) Introduction
(a) (4), (5), (3), (2), (1) (b) (4), (3), (5), (2), (1) (a) (2), (3), (4), (5), (1) (b) (3), (2), (5), (4), (1)
(c) (4), (3), (5), (1), (2) (d) (4), (5), (3), (1), (2) (c) (3), (2), (1), (4), (5) (d) (3), (4), (2), (1), (5)
3. (1) XII (2) M A 10. (1) Colonel (2) Major
(3) B A (4) X (3) General (4) Captain
(5) Ph. D (5) Lieutenant
(a) (5), (4), (3), (2), (1) (b) (4), (1), (3), (2), (5) (a) (5), (4), (2), (1), (3) (b) (3), (4), (5), (2), (1)
(c) (4), (2), (1), (3), (5) (d) (4), (3), (2), (1), (5) (c) (1), (2), (5), (3), (4) (d) (5), (4), (2), (3), (1)

4. (1) Birth (2) Death 11. (1) Table (2) Tree


(3) Funeral (4) Marriage (3) Wood (4) Plant
(5) Education (5) Seed
(a) (1), (3), (4), (2), (5) (b) (1), (4), (5), (2), (3) (a) (4), (5), (3), (2), (1) (b) (1), (2), (3), (4), (5)
(c) (1), (5), (4), (2), (3) (d) (1), (5), (4), (3), (2) (c) (5), (4), (2), (3), (1) (d) (5), (4), (1), (2), (3)

5. (1) Letter (2) Word 12. (1) Grass (2) Milk


(3) Paragraph (4) Story (3) Cow (4) Butter
(5) Sentence (5) Curd
(a) (1), (2), (4), (5), (3) (a) (2), (1), (5), (4), (3) (b) (1), (3), (2), (5), (4)
(b) (1), (2), (5), (4), (3) (c) (1), (2), (3), (4), (5) (d) (5), (4), (3), (2), (1)
(c) (1), (2), (4), (5), (3)
13. (1) Cutting (2) Dish
(d) (1), (2), (5), (3), (4)
(3) Vegetable (4) Market
6. (1) Foetus (2) Child (5) Cooking
(3) Baby (4) Adult (a) (4), (3), (1), (2), (5) (b) (5), (4), (1), (2), (3)
(5) Youth (c) (1), (2), (3), (5), (4) (d) (5), (1), (2), (4), (3)
(a) (1), (2), (4), (3), (5)
14. (1) Punishment (2) Prison
(b) (1), (3), (2), (5), (4)
(c) (2), (3), (5), (4), (1) (3) Arrest (4) Crime
(d) (5), (4), (2), (3), (1) (5) Judgement
(a) (1), (5), (4), (3), (2) (b) (1), (2), (3), (5), (4)
7. (1) Kanchanjunga (2) Nanda Devi (c) (1), (2), (5), (4), (3) (d) (4), (3), (5), (1), (2)
(3) Godwin Austin (4) Nanga Parbat
(5) Everest 15. (1) Window (2) Wall
(a) (5), (4), (3), (2), (1) (3) Foundation (4) Room
(b) (5), (3), (4), (1), (2) (5) Roof
(c) (5), (3), (1), (4), (2) (a) (1), (5), (4), (2), (3) (b) (1), (2), (3), (4), (5)
(d) (5), (4), (3), (1), (2) (c) (3), (2), (1), (5), (4) (d) (5), (4), (1), (2), (3)
Hints and Solutions
Elementary Level Questions
1. (a) The meaningful sequence is 12. (c) The meaningful sequence is
Shirdi Maharashtra India Asia World Pool Pond Lake River Sea
→ → → → → → → →
(2) (1) (3) (4) (5) (4) (5) (1) (3) (2)
i.e., 2, 1, 3, 4, 5 i.e., 4, 5, 1, 3, 2
2. (b) The meaningful sequence is 13. (b) The meaningful sequence is
Village District State Country World Seed Plant Tree Wood Furniture
→ → → → → → → →
(1) (5) (2) (4) (3) (1) (4) (3) (2) (5)
i.e., 1, 5, 2, 4, 3 [Development of furniture product]
3. (a) Firstly, we took an envelope and put a letter in it, which is i.e., 1, 4, 3, 2, 5
posted in a post-box and after clearance it will be delivered, so 14. (a) Animals must be arranged according to increasing body
the correct sequence is (4), (3), (1), (5) and (2). size.
4. (d) Child firstly go the play-way, then school, then college and Mosquito Dog Tiger Elephant Whale
→ → → →
finally as a result he/she will get a job. (5) (3) (4) (2) (1)
5. (a) The meaningful sequence is i.e., 5, 3, 4, 2, 1
Member Family Community Locality Country 15. (b) The meaningful sequence is
→ → → →
(2) (5) (3) (1) (4) Cottage Flat House Bungalow Palace
→ → → →
i.e., 2, 5, 3, 1, 4 (3) (2) (4) (1) (5)
6. (d) The meaningful sequence is i.e., 3, 2, 4, 1, 5
Delhi India Asia World Universe 16. (d) The meaningful sequence is
→ → → →
(1) (2) (5) (4) (3) Child School College Job Salary
→ → → →
i.e., 1, 2, 5, 4, 3 (1) (3) (4) (2) (5)
7. (c) Festivals must be arranged as per annual calendar. i.e., 1, 3, 4, 2, 5
Hence, 17. (d) Process to enter in a locked room.
Republic day Holi
→ Door

Key

Lock

Room

Switch on
(5) (4) (2) (1) (3) (4) (5)
Independence Day Deepawali Christmas
→ → →
(2) (1) (3) i.e., 2, 1, 3, 4, 5
i.e., 5, 4, 2, 1, 3 18. (b) The meaningful sequence is
Rain Sun Rainbow Child Happy
8. (d) The meaningful sequence is → → → →
(2) (3) (1) (5) (4)
V < VI < VIII < IX < XI (increasing order)
i.e., 5, 1, 3, 2, 4 i.e., 2, 3, 1, 5, 4

9. (b) The meaningful sequence is 19. (c) The meaningful sequence is


Morning Afternoon Evening Night Midnight
Writing

Composing

Proofreading

Printing

Reading → → → →
(4) (2) (3) (5) (1) (1) (2) (4) (3) (5)

i.e., 4, 2, 3, 5, 1 i.e., 1, 2, 4, 3, 5

10. (b) Months must be arranged as per annual calendar. 20. (c) Ascending order of different education levels
March April August September December Pre-primary education → Primary education
→ → → →
(5) (2) (4) (1) (3) (3) (5)
i.e., 5, 2, 4, 1, 3 → Middle education → Secondary education
(4) (2)
11. (a) The meaningful sequence is → Graduate education
Child Cry Mother Milk Smile
→ → → → (1)
(5) (2) (1) (3) (4) i.e., 3, 5, 4, 2, 1
i.e., 5, 2, 1, 3, 4
50 Study Package for NTSE

High Skill Questions


1. (c) Metals are arranged in the increasing order of their cost. 8. (c) The meaningful sequence is
Iron Copper Silver Gold Platinum Plant Cotton Yarn Cloth Saree
→ → → → → → → →
(2) (1) (3) (4) (5) (2) (4) (1) (5) (3)
i.e., 2, 1, 3, 4, 5 i.e., 2, 4, 1, 5, 3

2. (c) The meaningful sequence is 9. (b) The meaningful sequence is


Title Content Introduction Chapter Index
April May June July August → → → →
→ → → → (3) (2) (5) (4) (1)
(4) (3) (5) (1) (2)
i.e., 3, 2, 5, 4, 1
i.e., 4, 3, 5, 1, 2
10. (a) Army designations in ascending order
3. (b) Qualification in ascending order Lieutenant Captain Major Colonel General
→ → → →
X XII BA MA Ph.D (5) (4) (2) (1) (3)
→ → → →
(4) (1) (3) (2) (5) i.e., 5, 4, 2, 1, 3
i.e., 4, 1, 3, 2, 5 11. (c) The meaningful sequence is
4. ( c ) The meaningful sequence is Seed Plant Tree Wood Table
→ → → →
(5) (4) (2) (3) (1)
Birth Education Marriage Death Funeral
→ → → →
(1) (5) (4) (2) (3) i.e., 5, 4, 2, 3, 1

i.e., 1, 5, 4, 2, 3 12. (b) Process of obtaining butter follow the sequence


Grass Cow Milk Curd Butter
5. (d) Letters form word, words form sentence, sentence form → → → →
paragraph, paragraph form a story. (1) (3) (2) (5) (4)
i.e., 1, 3, 2, 5, 4
i.e., 1, 2, 5, 3, 4
6. (b) The meaningful sequence is 13. (a) The meaningful sequence is
Market Vegetable Cutting Dish Cooking
Foetus Baby Child Youth Adult → → → →
→ → → → (4) (3) (1) (2) (5)
(1) (3) (2) (5) (4)
i.e., 4, 3, 1, 2, 5
i.e., 1, 3, 2, 5, 4
14. (d) The meaningful sequence is
7. (c) Mountain peaks must arranged in descending order of Crime Arrest Judgement Punishment Prison
height. → → → →
(4) (3) (5) (1) (2)
Everest Godwin Austin Kanchanjunga
→ → i.e., 4, 3, 5, 1, 2
(5) (3) (1)
15. (c) The meaningful sequence is
Nanga Parbat Nanda Devi
→ → Foundation Wall Window Roof Room
(4) (2) → → → →
(3) (2) (1) (5) (4)
i.e., 5, 3, 1, 4, 2 i.e., 3, 2, 1, 5, 4
Chapter

8
Blood Relations
Blood relations means any relation between two or more persons. Which is acquired by them by the virtue of their
birth. In these questions, a chain of relationship is given in the form of information and on the basis of this
information relation between any two members of the chain is to determined. So, candidates are supposed to be
familiar with the knowledge of different relations in the family. Some of the blood relationships are listed below to
help in solving these problems.
(i) Mother’s or father’s son Brother
(ii) Mother’s or father’s daughter Sister
(iii) Mother’s or father’s brother Uncle
(iv) Mother’s or father’s sister Aunt
(v) Mother’s or father’s father Grandfather
(vi) Mother’s or father’s mother Grandmother
(vii) Son’s wife Daughter-in-law
(viii) Daughter’s husband Son-in-law
(ix) Husband’s or wife’s sister Sister-in-law
(x) Husband’s or wife’s brother Brother-in-law
(xi) Brother’s son Nephew
(xii) Brother’s daughter Niece
(xiii) Uncle or aunt’s son or daughter Cousin
(xiv) Sister’s husband Brother-in-law
(xv) Brother’s wife Sister-in-law
(xvi) Grandson’s or granddaughter’s, daughter/son Great granddaughter/son
(xvii) Son’s or daughter’s son Grandson
(xviii) Son’s or daughter’s daughter Granddaughter

Some other relationships are also listed below to help in Important Notations
solving the problem easily l + Male

l The only son of your father — Yourself l - Female l + - Husband-wife


The only son of grandfather or grandmother — Father + - Brother-sister Father-son
+

l l l

l Father-in-law of mother — Grandfather


+

l Son of the father of the sister — Brother


Father-daughter Mother-son
-
+

l l
l Wife of uncle — Aunt
l Husband of maternal aunt — Uncle -
+

Mother-daughter
-
l

-
52 Study Package for NTSE

There are three types of questions based on Blood Relations which are as follows

Type I Statement Based Questions


In this type of questions, some description is given in the form of certain small relationships and candidate is
required to analyse the whole chain of relations and then, find out the direct relationship between the concerned
people.
Example 1 Pointing to a photograph, a man said, “I have no brother or sister but that man’s father is my father’s
son”. Whose photograph was it?
(a) His own (b) His son’s (c) His father’s (d) His nephew’s
Solution (a) Since, the narrator has no brother, his father’s son is he himself. The photograph is his own
photograph.

Example 2 Introducing a woman, a man said, “Her husband is only son of my mother”. How is the woman related
to that man?
(a) Sister (b) Daughter (c) Wife (d) Mother
Solution (c) Only son of man’s mother means the man himself. Therefore, the Man Mother
man is the husband of that woman. Hence, the woman is the wife of that man.
Husband
Woman
Type II Relation Puzzle
In this type, mutual blood relations of more than two persons are mentioned. Candidate has to analyse the
information and find the required relation.
Example 3 A is B’s sister and C is B’s mother. D is C’s father. E is D’s mother. How is A related to D?
(a) Grandmother (b) Grandfather (c) Daughter (d) Granddaughter
Solution (d) According to the given information,
E(–)
(Mother)

D(+)

(Grand (Father)
daughter)
C(–)
(Mother)
(–)
A B
Sister
Clearly, A is granddaughter of D.

Example 4 Given that


1. P is the mother of Q. 2. R is the son of P. 3. S is the brother of T. 4. T is the daughter of Q.
Who is the grandmother of S?
(a) P (b) Q (c) R (d) S
Solution (a) From the given informations, S is the brother of T and T is the daughter of Q. This means that S is
the son of Q. Since, P is the mother of Q, so P is the grandmother of S.

Directions (Ex. 5 and 6) Read the information and answer the questions given below.
Ram and Sita are married couple having two daughters, Medha and Deepti. Deepti is married to Anurag, who is the son of Garima and
Tarun. Nidhi is the daughter of Anurag. Komal, who is Anurag’s sister, is married to Harshit and has two sons, Amar and Prem.
Example 5 What is the relationship between Amar and Nidhi?
(a) Cousins (b) Husband-Wife (c) Father-Daughter (d) Uncle-Niece

Example 6 How is Komal related to Deepti?


(a) Aunt (b) Sister-in-law (c) Sister (d) None of these
Blood Relations 53
Solution (5 and 6) According to the given information,
Ram == Sita Garima == Tarun

Daughter Daughter
Son
(–) (+)
(–) Medha Deepti == Anurag Komal == Harshit
Sister
(–) (+)
Son Son
(+) (+)
Amar Prem

(a) Nidhi is the daughter of Anurag and Amar is the son of Anurag’s sister. So, Amar and Nidhi are cousins.
(b) Deepti is Anurag’s wife and Komal is Anurag’s sister. So, Komal is Deepti’s sister-in-law.

Type III Questions Based on Use of Mathematical Sign


In this type of questions, relation is given between two persons in the form of algebraic equation. You have to
choose the equation from given options which is best related to given information.
Example 7
1. P + Q means ‘P is the brother of Q’.
2. P – Q means ‘P is the mother of Q’.
3. P × Q means ‘P is the sister of Q’.
Which of the following means P is the maternal uncle of Q?
(a) P + Q + R (b) P – Q + R (c) P + R – Q (d) P + R × Q
Solution (c) Option (a), P + Q + R implies that P is the brother of Q, who is the brother of R means that P is the
brother of Q. So, it is not correct.
Option (b) also does not correspond to required relationship.
Option (c), P + R – Q means P is the brother of R, who is the mother of Q means that P is the maternal uncle of Q.
Hence, it is correct.

Target Exercise
Elementary Level Questions
1. Pointing to Amit, Anita said, “His mother is the only 5. Pointing to a man in a photograph, a woman said,
daughter of my mother”. How is Anita related to the “His brother’s father is the only son of my
Amit? grandfather”. How is the woman related to the man
(a) Mother (b) Daughter in the photograph?
(c) Sister (d) Grandmother (a) Mother (b) Niece
(c) Sister (d) Daughter
2. P and Q are brothers, X and Y are sisters, son of P is
the brother of Y. How is Q related to X? Directions (Q. Nos. 6 and 7) Read the following
(a) Father (b) Brother (c) Daughter (d) Uncle information and answer the questions given below it.
3. If Mohan says that his mother is the only daughter of A is the father of C but C is not his son. E is the daughter of C.
Shyam’s mother. Then, how is Shyam related to F is the spouse of A. B is the brother of C. D is the son of B. G is
Mohan? the spouse of B. H is the father of G.
(a) Son (b) Father (c) Sister (d) Uncle 6. Who is the grandmother of D?
4. Ram is the brother of Deepak, Sunita is sister of (a) A (b) C
(c) F (d) H
Rajesh. Deepak is the son of Sunita. How is Ram
related to Sunita? 7. Who is the son of F?
(a) Son (b) Brother (c) Nephew (d) Father (a) B (b) C (c) D (d) E
54 Study Package for NTSE

Directions (Q. Nos. 8-12) Read the information given 19. Pointing towards a girl, a teacher said, “She is the
below and answer the following questions. only daughter of the only son of the wife of the
(i) A, B, C, D, E and F are six members of a family. father-in-law of my wife”. How is the girl related with
(ii) One couple has parents and their children in the family. the teacher?
(iii) A is the son of C and E is the daughter of A. (a) Daughter (b) Niece
(iv) D is the daughter of F, who is the mother of E.
(c) Sister (d) Daughter-in-law
8. Who are the male members in the family?
20. Pointing to Keshav, Priyanka said, “His mother’s
(a) A, C or B (b) C and F (c) A, B and D (d) D and F
brother is the father of my son Nikhil”. How is
9. Which of the following pairs is the parents of the Keshav related to Priyanka?
children? (a) Niece (b) Aunt (c) Nephew (d) Sister-in-law
(a) BC (b) CF (c) BF (d) None of these
21. Pointing to a woman in a photograph, a man says,
10. Which of the following pairs is the parents of the “She is the mother-in-law of the sister-in-law of the
couple? only sister of my son”. How is the woman related to
(a) AB (b) BC (c) AF (d) CF man?
11. How many female members are there in the family? (a) Sister (b) Sister-in-law
(a) Two (b) Three (c) Mother (d) Wife
(c) Four (d) Cannot be determined
22. Pointing to a man, Radhika said, ‘‘His sister is my
12. What relationship of D and E bear to each other? daughter’s brother’s mother’’. How is the man related
(a) Sister and brother to Radhika?
(b) Mother and son (a) Brother (b) Father
(c) Grandmother and granddaughter (c) Uncle (d) Grandfather
(d) Sisters
23. Pointing to a photograph, a woman says, “This man’s
13. P, Q, R, S, T, U and V are seven members of a family
son’s sister is my mother”. How is the woman related
consisting of four adults and three children, two of
to the man in the photograph?
whom, U and V are girls. P and S are brothers and Q
(a) Daughter (b) Mother
is a doctor. T is an engineer married to one of the
(c) Granddaughter (d) Mother-in-law
brothers and has two children. Q is married to S and
V is their child. Who is R? 24. Pointing to a woman in a photograph, a man says,
(a) P’s son (b) T’s daughter ‘‘She is the mother of my son’s only daughter’’. How is
(c) U’s father (d) Cannot be determined the woman related to the man in the photograph?
14. Radha is the niece of Alok. Alok’s mother is Priya. (a) Daughter (b) Wife
Seema is Priya’s mother. Seema’s husband is Hari. (c) Mother (d) Daughter-in-law
Rita is the mother-in-law of Hari. How is Alok related 25. Pointing to a man in a photograph, a woman says,
to Hari? ‘‘He is the father of my only daughter-in-law’s
(a) Daughter (b) Daughter’s son father-in-law’’. What is the man to the woman?
(c) Grandniece (d) Great grandson’s daughter
(a) Father (b) Brother
15. Pointing to a man, Sanjay said, “His son is my son’s (c) Husband (d) Father-in-law
uncle”. How is the man related to Sanjay? 26. Pointing to a woman in the photograph, a man says,
(a) Father (b) Uncle (c) Brother (d) Grandfather
‘‘This woman is the wife of the father of my
16. Ram is the brother of Shyam and Mahesh is the brother-in-law’’. How is the woman related to the
father of Ram. Jagat is the brother of Priya and Priya man?
is daughter of Shyam. Who is the uncle of Jagat? (a) Mother (b) Mother-in-law
(a) Shyam (b) Mahesh (c) Sister (d) Sister-in-law
(c) Ram (d) None of these
27. Pointing to a man in the photograph, a woman says,
17. A husband and wife had five married sons and each of “He is the son of my sister’s mother’s husband”. Who
them had four children. How many members are is the man related to the woman?
there in the family? (a) Brother (b) Father
(a) 50 (b) 40 (c) 32 (d) 36 (c) Uncle (d) Brother-in-law

18. Introducing a man, a woman says, “His wife is the 28. Pointing to a man in a photograph, a woman says, ‘‘He
only daughter of my father”. How is the man related is the only son of the only daughter-in-law of my only
to the woman? son’s father’’. How is the man related to the woman?
(a) Husband (b) Father-in-law (a) Son (b) Father
(c) Maternal uncle (d) Paternal uncle (c) Son-in-law (d) Grandson
Blood Relations 55
High Skill Questions
Directions (Q. Nos. 1-5) Read the following information 10. How is L related to O?
and answer the questions given below it. (a) Father (b) Mother-in-law
(c) Brother-in-law (d) Father-in-law
There are six persons in a family A, B, C, D, E and F.
(i) C is the sister of F. 11. How is K related to O?
(ii) A is the brother of the husband of E. (a) Sister (b) Mother
(iii) D is the father of A and D is the grandfather of F. (c) Mother-in-law (d) Brother-in-law
(iv) There are two fathers, three brothers and a mother in the
family. 12. How is N related to L?
(a) Son (b) Cousin
1. What is the relationship between E and F? (c) Brother (d) Brother-in-law
(a) Daughter-Father (b) Mother-Son
(c) Husband-Wife (d) Grandfather-Grandson 13. How is M related to L?
(a) Aunt (b) Niece
2. Who is the mother in the family? (c) Daughter (d) Daughter-in-law
(a) E (b) D (c) C (d) B
14. Q’s mother is sister of P and daughter of M. S is
3. How many male members are there in the family? daughter of P and sister of T. How is M related to T?
(a) 1 (b) 5 (c) 4 (d) 3 (a) Father (b) Grandfather
(c) Grandmother
4. Who is the husband of E?
(d) Either grandfather or grandmother
(a) F (b) D (c) B (d) C

5. How many persons are there in the category of Directions (Q. Nos. 15-17) Study the following
brothers? information carefully and answer the questions given below.
(a) 1 (b) 2 (c) 4 (d) 3 Five girls Madhu, Arti, Kamla, Rekha and Suman are married
to Mahesh, Yogesh, Ravi, Mukesh and Gyan, but not in the
6. If (i) A 5 B means ‘A is the husband of B’. same order.
(ii) A 4 B means ‘A is the sister of B’. (i) First letters of the names of couple are not the same.
(iii) A 3 B means ‘A is the son of B’. (ii) Arti and Kamla are not married to Yogesh. Suman is not
married to Mukesh.
Then, which of the following shows that A is the
(iii) Yogesh and Gyan are not Suman’s or Madhu’s husband.
daughter of B?
(iv) Kamla’s and Rekha’s husband’s names do not start with M.
(a) A 4 C 3 B (b) C 3 B 4 A (c) B 5 C 3 A (d) C 3 B 5 C 4 A
15. Who is Ravi’s wife?
7. (i) P + Q means ‘P is the father of Q’.
(a) Madhu (b) Arti (c) Kamla (d) Rekha
(ii) P × Q means ‘P is the brother of Q’.
(iii) P – Q means ‘P is the wife of Q’. 16. Who is Arti’s husband?
(iv) P ÷ Q means ‘P is the mother of Q’. (a) Mahesh (b) Yogesh (c) Mukesh (d) Gyan
If P– R + Q, which of the following statements is true? 17. Which of the following statements is wrong?
(a) P is the mother of Q (b) P is the aunt of Q (a) Gyan is married to Kamla (b) Suman is not Mukesh’s wife
(c) P is the sister of Q (d) P is the brother of Q (c) Arti is not Ravi’s wife (d) Rekha is not Yogesh’s wife
8. A is the father of C and D is the son of B. E is the
Directions (Q. Nos. 18-20) Read the following information
brother of A. If C is the sister of D, how is B related to
carefully and answer the questions which follow.
E?
(a) Daughter (b) Brother-in-law If ‘A × B’ means ‘A is the father of B’.
(c) Husband (d) Sister-in-law If ‘A + B’ means ‘A is the wife of B’.
If ‘A ÷ B’ means ‘A is the daughter of B’.
9. Pointing to a man, a woman said, “He is the only If ‘A – B’ means ‘A is the son of B’.
son-in-law of my mother’s father”. How is the woman
related to that man?
18. How is L related to Q in the expression
L ÷ M × O − P ÷ Q?
(a) Aunt (b) Daughter (c) Sister (d) Wife
(a) Granddaughter (b) Niece
(c) Daughter-in-law (d) Daughter
Directions (Q. Nos. 10-13) Study the following
information carefully and answer the questions given below. 19. What will come in the place of the question mark to
Five persons K, L, M, N and O are sitting around dinning establish that Q is the nephew of T in the expression
table. K is the mother of M, who is the wife of O. N is the Q(?) R ÷ S × T?
brother of K and L is the husband of K. (a) + (b) × (c) − (d) ÷
56 Study Package for NTSE

20. Which of the following relations is true based upon 21. How is Mrs. Mohan related to Sumit?
the relations given in the equation, ‘A – B × C + D – E’? (a) Aunt (b) Mother-in-law
(a) C is the mother of A (b) E is the wife of B (c) Mother (d) Sister-in-law
(c) D is the brother of A (d) None is true
22. What is the surname of Sohan?
Directions (Q. Nos. 21-24) Study the following information (a) Sharma (b) Mohan (c) Mishra (d) Raghu
carefully to answer these questions. 23. What is the surname of Leela?
Adhir Mishra has three children—Urmila, Raghu and Sumit. (a) Sharma (b) Mohan
Sumit married Roma, the eldest daughter of Mr. and Mrs. (c) Mishra (d) None of these
Mohan. The Mohan married their youngest daughter to the
eldest son of Mr. and Mrs. Sharma and they had two children 24. How is Shivendar related to Roma’s father?
Sandeep and Shaifali. Mohan has two more children, Roshan (a) Son-in-law (b) Cousin
and Bimla, both elder to Sheila. Sohan and Shivendar are sons (c) Son (d) Grandson
of Sumit and Roma. Leela is the daughter of Sandeep.

Hints and Solutions


Elementary Level Questions Sol. (Q. Nos. 8-12)
A, B, C, D, E and F are six family members.
1. (a) Anita’s
mother C B
Son Spouse
Only
daughter (+) Spouse (–)
A F

Amit’s Daughter r Daughter


Anita the
mother Mo D
(–)
(–) E

Amit 8. (a) So, the male members are A, C or B.


Anita is only daughter of her mother. So, she will be mother of 9. (d) AF is parents of the children.
Amit.
10. (b) BC is the parents of the couple.
2. (d) On the basis of statements, we conclude that Q is the brother
of P and P’s son is the brother of Y. So, Y is the daughter of P. 11. (c) Four (F, D and E, C or B)
Thus, Q is the uncle of X. 12. (d) D and E are sisters.
3. (d) Only daughter of Shyam’s mother means that Shyam is Married Married
13. (d)
brother of Mohan’s mother i .e., Shyam is uncle of Mohan.
(–) (+) (+) (–)
Sister T P S Q
4. (a) (–)
Sunita Rajesh

Son Daughter
Girl
(+) Deepak Ram (+) R (+)/(–)
U (–)
(–)
Girl V
So, Ram is also son of Sunita.
5. (c) Only son of woman’s grandfather means woman’s father, So, R is either son of P or daughter of P.
man’s brother. So, the woman is the sister of the man. 14. (b) (–)
Mother-in-law Rita
Sol. (Q. Nos. 6 and 7) Mother
(+)
(+) Hari Seema (–)

H (+) Spouse A F (–)


Not son Father
Mother
daughter
Father
Priya (–)
(+)
Brother
(–) G B C (–)
Spouse
Son Mother
Daughter Niece
(–) (+)
(+) D E (–) Radha Alok

6. (c) So, grandfather of D is F. So, Alok is Hari’s daughter’s son.


7. (a) Son of F is B.
Blood Relations 57
15. (a) From the given information, man’s son is brother of Sanjay. Keshav’s mother is Priyanka's husband’s sister.
Hence, man is father of Sanjay. Keshav is the son of Priyanka’s husband’s sister.
∴Keshav is teacher’s nephew.
16. (c) (+)
Mahesh 21. (d) Sister of son = Daughter of man
Sister-in-law of daughter = Daughter-in-law of man
Father Mother-in-law of daughter-in-law = Wife of man
∴Woman is wife of man.
(+) (+) 22. (a) Man’s sister = Radhika’s daughter’s brother’s mother =
Ram Shyam
Brother Radhika’s son’s mother = Radhika. Hence, man is Radhika’s
brother.
Uncle Daughter 23. (c) ‘This man’s sister is my mother’ means ‘‘This man’s daughter
is woman’s mother’’ ⇒ Man is grandfather. So, woman is the
(+) (–) granddaughter of man.
Jagat Priya
Brother 24. (d) Mother of son’s daughter = Son’s wife. Hence, the woman is
Clearly, Ram is the uncle of Jagat. daughter-in-law of a man.

17. (c) Total family member = Husband + Wife + Their five married 25. (d) Daughter-in-law’s father-in-law = Husband. Hence, man is
husband of the woman.
sons + Each son have four children
26. (b) Father of man’s brother-in-law means man’s father-in-law.
= 1 + 1 + 10 + 4 × 5 = 12 + 20 = 32
Wife of the man’s father-in-law means man’s mother-in-law.
18. (a) The only daughter of the woman’s father is the woman
27. (a) Mother’s husband = Father
herself. Therefore, the man is the husband of the woman.
Sister’s mother’s husband = Sister’s father son of father a man
19. (a) The wife of the father-in-law of the person’s wife ⇒ The is brother of man.
mother of the person and only son of the person’s mother is the 28. (b) My only son’s father = My husband
person himself. Hence, the girl is the teacher’s only daughter. Daughter-in-law of husband = Daughter-in-law
20. (c) His mother’s brother is the father of my son means Only son of daughter-in-law = Grandson = Father
his mother’s brother is Priyanka’s husband. Hence, a man is father of the woman.

High Skill Questions


Sol. (Q. Nos. 1-5) 7. (a) From option (a),
According to the given information. ⇒P − R + Q
(–) P R (+)
(+) D

Father Mother Father


(+) (–)
(+) A B E Q
Brother
Hence, P is the mother of Q is true.
Son 8. (d) According to the question,
Son
(+) (+) Brother (+)
(–) (B) A E
(–) C F
Sister
Father Son
1. (b) E and F are Mother-Son.
2. (a) E is the mother in the family. (–)
C D (+)
Sister
3. (c) There are 4 male members i.e., D, A, B and F. Clearly, B is sister-in-law of E.
4. (c) B is the husband of E. 9. (b) Father of
5. (b) Two persons i.e., A and F are in the category of brothers. woman's mother
Son-in-law
6. (a) From option (a)
⇒ A4C3B
B Mother of
Man
a woman
Daughter
Son

Daughter
(–) A C (+) Woman
Sister
Hence, A is the daughter of B.
Hence, a woman is daughter of a man.
58 Study Package for NTSE

Sol. (Q. Nos. 10-13) 18. (a) From the below figure, it is clear that L is granddaughter of Q.
(–) Q
Brother Wife (+)
N K L Daughter
(+)
Husband
Sister
Father-in-Law M(+) P(–)
Mother
Daughter Father Son
Wife (+)
(–) M O O(+)
L(–)
10. (d) L is father-in-law of O.
19. (c) On putting sign (–) in place of question mark (?),
11. (c) K is mother-in-law of O.
S(+)
12. (d) N is brother-in-law of L. Father
Daughter
13. (c) M is daughter of L.
T R(–)
Sister Son
14. (d) (+) M
Nephew
G

Q(+)
ra
nd

Daughter
m

From the above figure, it is clear that Q is the nephew of T.


ot
he
ro


Sister 20. (d) (+)
rG

(–) P B E
ra
nd

Father
fa

Mother Daughter Son Son


th
er

(–) (+)
(–) Q S T (+)(A) C D
Sister Sister (–)

Clearly, M is either grandfather or grandmother of T. Hence, none is true.

Sol. (Q. Nos. 15-17) Sol. (Q. Nos. 21-24)


On the basis of information provided.
Madhu Arti Kamla Rekha Suman Married
Adhir Mishra Mr and Mrs Mohan
(–)
Mukesh Gyan Ravi Yogesh Mahesh Urmila Raghu Sumit Roma Roshan Bimla Sheila
Sons
Husband Wife Eldest son
Married
of Mr Sharma
Yogesh → Rekha (+) Sohan Shivendar (+)

Ravi → Madhu Sundeep


Daughter
Mukesh → Arti Shaifali
Leela
Gyan → Kamla
21. (b) Mrs. Mohan is Sumit’s mother-in-law.
Mahesh → Suman 22. (c) Surname of Sohan is Mishra because Sohan is son of Sumit
15. (a) Madhu is the wife of Ravi. and Sumit is son of Mishra.
23. (a) Surname of Leela is Sharma because Leela is granddaughter
16. (c) Mukesh is the husband of Arti. of Mr. Sharma.
17. (d) Rekha is not Yogesh’s wife is wrong statement. 24. (d) Shivendar is grandson of Roma’s father (Mr Mohan).
Chapter

9
Direction Sense Test
In this chapter the questions consist of a sort of direction In this triangle, AC is hypotenuse, AB is perpendicular
puzzle. A successive follow up of directions is formulated and BC is base of triangle.
and the candidates are required to ascertain the final In the morning, sunrise in the East, the shadow of
direction or the distance between two points. The test is person or object falls in the West. In the evening, the
meant to judge the candidates ability to trace and follow sunset in the West, the shadow of a person or object falls
correctly and sense the direction correctly. in the East.
Some important points to be kept in mind while solving Movement in the direction of clock is called clockwise
the quesiton on direction sense test. movement and in the direction opposite to that of clock
Four major directions (clockwise) are North (N), is called anti-clockwise movement.
East (E), South (S) and West (W). 360º 360º
325º 45º 325º 45º
N

270º 90º 270º 90º


W E

225º 135º 225º 135º


S 180º 180º
Clockwise Anti-clockwise
Four sub-major directions (clockwise) North-East (NE),
South-East (SE), South-West (SW) and North-West The different types of questions which are asked in
(NW). exams related to direction sense test are as follows

Type I Finding the Direction


N
NW NE
This type of questions are based on the final direction of
45º 45º
an object with respect to its starting point or the
45º 45º
W
45º 45º
E direction facing by it in the end.
45º 45º
Example 1 Mohan starts from point A and walks 1 km
SW SE
towards South, turns left and walks 1 km. Then, he turns left
again and walks 1 km. Now, he is facing
S
(a) East (b) West (c) North (d) South
Solution (c)
Pythagoras Theorem N
A A Final
The theorem is applicable only in right
angled triangle. Right angled triangle W E 1km 1km
is one in which one angle is of 90°. a c

According to this theorem, S 1km


(AC) 2 = (AB) 2 + (BC) 2 ⇒ c2 = a 2 + b 2 B b C So, Mohan is facing in North direction.
60 Study Package for NTSE

Example 2 Ram is facing North-West. He turns 90° in the Type II Finding the Total Distance
clockwise direction and then turns 135° in the anti-clockwise
direction, which direction is he facing now? In this type of questions, we have to find out the final
(a) West (b) North (c) South (d) East distance between the starting and the final points or
N distance between two points/persons/things.
Solution (a) Ram faces in OA direction, NW A B
by first move of 90° clockwise, he faces Example 6 One day, Ravi left home and cycled 10 km
in the direction OB and in final move of W E Southwards, turned right and cycled 5 km and turned right and
O
135° anti-clockwise, he faces in the West cycled 10 km and turned left and cycled 10 km. How many
direction. S kilometres will he have to cycle to reach his home straight.
Example 3 If SE becomes West, NE becomes South and so (a) 10 km (b) 15 km
on. What will North become? (c) 20 km (d) 25 km
(a) North-East (b) North-West (c) South-East (d) South-West Solution (b) According to the given information,
Solution (c) The directions are as follows
10 km D N
N A (Home)
(SE) E
N-W N-E (end point)
(E) (S) 10 km 10 km W E
W E
(NE) (SW) C 5 km B S
S-W S-E
S
(N)
(NW)
(W) Thus, his distance from initial position A = AE = AD + DE
= BC + DE = (5 + 10 ) km = 15 km
Clearly, North will become South-East.

Example 4 Early morning after sunrise Sangeeta went for a Type III Finding Both Direction
walk. Her brother, Ramesh, who was coming towards, her from and Distance
opposite direction, saw that Sangeeta’s shadow had fallen to his
right. From which direction was Ramesh coming? In this type of questions, the candidate have to
determine both the distance and direction.
(a) South (b) North (c) East (d) West
Solution (b) The directions are as follows. Example 7 Kunal walks 10 km towards North. From there,
he walks 6 km towards South. Then, he walks 3 km towards East.
N Ramesh
How far and in which direction is he with reference to his
starting point?
W E
Sun (a) 5 km West
S (b) 7 km West
Sangeeta shadow Sangeeta (c) 7 km East
(d) 5 km North-East
Clearly, Ramesh was coming from North.
Solution (d) According to the given information,
Example 5 In a clock at 12 : 30, hour hand is in North B
direction while minute hand is in South direction. In which N
6 km N-W N-E
direction would be minute hand at 12 : 45?
(a) North-West (b) South-East (c) West (d) East W E
10 km
Solution (c) The hour hand of the clock is North 3 km
D S-W S-E
C S
direction and minute hand is in South direction (End point)
At 12 : 30 At 12 : 45
North North A
(Starting point)
12 1 12 1
11 11
10 2 10 2 Then, AC = (AB − BC)
Hour Hour
West 9 3 East West 9 3 East = (10 − 6 ) = 4 km
Minute
minute
8 4 8 4 Kunal’s distance from starting point A
7
6 5 7
6 5 AD = AC2 + CD2 (Pythagoras theorem)
South South
= 4 2 + 3 2 = 5 km
As, it is clear from the above diagram that minute hand is
in West direction. Also, D is to the North-East of A.
Target Exercise
Elementary Level Questions
1. Janki started from her house and walked 2 km 9. Ruby leaves her home and walks 30 m in North-West
towards North. Then, she took a right turn and direction and then 30 m in South-West direction.
covered 1 km, then she again took a right turn and Next, she walks 30 m in South-East direction.
walked for 2 km. In which direction is she going? Finally, she turns towards her house. In which
(a) North (b) East (c) South (d) West direction is she moving?
(a) North-East (b) North-West (c) South-East (d) South-West
2. A man starts from a point, walks 4 miles towards
North and turns left and walks 6 miles, turns right 10. A man is facing North-East. He turns 90° in the
and walks for 3 miles and again turns right and clockwise direction and then 135° in anti-clockwise
walks 4 miles and takes rest for 30 min. He gets up direction. Which direction is he facing now?
and walks straight 2 miles in the same direction and (a) East (b) West (c) North (d) South
turn right and walks 1 mile. What is the direction, he
11. Anuj started walking positioning his back towards
is facing?
the Sun. After sometime, he turned left, then turned
(a) North (b) South (c) South-East (d) West
right and then towards the left again. In which
3. A cat runs 20 m towards East and turns to right, runs direction is he going now?
10 m and again turns to right, runs 9 m and again (a) North or South (b) East or West
turns to left, runs 5 m and then turns to left, runs (c) North or West (d) South or West
12 m and finally, turns to left and runs 6 m. Which 12. If South-East becomes North, North-East becomes
direction is the cat facing now? West and so on, what will West become?
(a) East (b) West (c) North (d) South (a) North-East (b) North-West
4. Ram wants to go to the school. He starts from his (c) South-East (d) South-West
home which is in the East and comes to the crossing. 13. A watch reads 4:30. If the minute hand points East,
The road to the left ends in a fire station ahead is the in what direction will the hour hand point?
bookshop. In which direction is the school? (a) North (b) North-West (c) South-East (d) North-East
(a) South (b) North (c) East (d) West
14. A man travels 4 km due North, then travels 6 km due
5. From his house, Vikki went 15 km to North. Then, he East and further travels 4 km due North. How far he
turned West and covered 10 km. Then, he turned South is from the starting point?
and covered 5 km. Finally turning to East, he covered (a) 6 km (b) 14 km (c) 8 km (d) 10 km
10 km. In which direction is he from his house?
(a) East (b) West (c) North (d) South 15. Shikha went 10 km South from her house. She turned
right and goes 5 km, again turned right and cycled
6. After walking 6 km, I turned right and covered a 10 km. Now, she turned left and cycled 10 km. What
distance of 2 km, then turned left and covered a is the shortest distance of Shikha from her house?
distance of 10 km. In the end, I was moving towards the (a) 5 km (b) 15 km
North, from which direction did I start my journey? (c) 20 km (d) 25 km
(a) North (b) South (c) East (d) West
16. Sheela walks from A to B, which is 2 km away turns
7. Raj starts from his office facing West and walks right at 90° and walks for 3 km to point C, turns right at
100 m straight, then takes a right turn and walks 90° and walks to D, which is 8 km away, turns 90° right
100 m. Further, he takes a left turn and walks 50 m. and goes 3 km to point K. Then, once again she turns
In which direction is Raj now from the starting right 90° and walks 4 km to point F. How far is she
point? from A?
(a) North-East (b) South-West (c) North (d) North-West (a) 2 km (b) 4 km
(c) 6 km (d) 8 km
8. I am facing South. I turn right and walk 20 m. Then,
I turn right again and walk 10 m. Then, I turn left 17. Rina walks 20 m towards North. She then turns left
and walk 10 m and then turning right walk 20 m. and walks 40 m. She again turns left and walks 20 m.
Then, I turn right again and walk 60 m. In which Further, she moves 20 m after turning to the right.
direction am I from the starting point? How far is she from her starting point?
(a) North (b) North-West (a) 60 m (b) 40 m
(c) East (d) North-East (c) 20 m (d) 80 m
62 Study Package for NTSE

18. Sunita walks 14 m towards West, then turn to right again turns left and walks 40 m. How many metres is
and walks 14 m and then turn to her left and walks he from his original position?
10 m. Again turning to her left she walks 14 m. What (a) 0 (b) 10
is the shortest distance (in metres) between her (c) 20 (d) 40
starting point and her present position? 21. A man walks 1 km towards East and then he turns to
(a) 38 (b) 28 (c) 24 (d) 10 South and walks 5 km. Again, he turns to East and
19. Ashish walks 40 m towards North. He then turns left walks 2 km, after this he turns to North and walks
and walks 40 m. He again turns left and walks 40 m. 9 km. Now, how far is he from his starting point?
(a) 3 km (b) 4 km (c) 5 km (d) 7 km
Further, he moves 40 m after turning to the right.
How far is he from his original position? 22. A boy ride his bicycle Northwards, then turned left
(a) 20 m (b) 30 m (c) 50 m (d) 80 m and ride 1 km and again turned left and ride 2 km. He
found himself exactly 1 km West of his starting point.
20. Umesh goes 30 m South, then turns left and walks How far did he ride Northwards initially?
40 m, then again turns left and walks 20 m, then (a) 1 km (b) 2 km (c) 3 km (d) 5 km

High Skill Questions


1. Mohan starts from his house and walks 4 km straight. 6. Raj starts from his office facing West and walks
He then turns towards his right and walks 2 km. He 100 m straight, then takes a right turn and walks
turns again to his right and walks 2 km. If he is 100 m. Further, he takes a left turn and walks 50 m.
North-West from the starting position now, then In which direction is Raj now from the starting point?
in which direction did he start in the beginning? (a) North-East (b) South-West
(a) North (b) South (c) North (d) North-West
(c) East (d) West
7. If ‘South-East’ is called ‘East’, ‘North-West’ is called
2. Lalit walks 8 km East, turns South-West and walks ‘West’, ‘South-West’ is called ‘South’ and so on, what
another 8 km. He again takes a turn towards will ‘North’ be called?
North-West and walks another 8 km. In which (a) East (b) North-East
direction from his starting point, is he standing now? (c) North-West (d) South-West
(a) North-East (b) South-East
(c) West (d) East 8. One evening before sunset, two friends Ram and
Shyam were talking to each other face to face.
3. Mohit walks a distance of 5 km towards South, then
If Shyam’s shadow was exactly to his right side,
turns to his right and walks 3 km. He again turns
which direction was Ram facing?
right and walks 5 km. He then turns to his left and
(a) North-East
walks 5 km. How far is he from the starting point and
(b) South
in what direction? (c) South-West
(a) 5 km and West
(d) South-East
(b) 3 km and North
(c) 3 km and East 9. Indu walked 25 m towards South. Then, turning to
(d) 8 km and West his right he walks 30 m. Then, turning to his left, he
4. A dog is taken out every evening by the owner whose walks 20 m. Again, he turns to his left and walks
house faces East. They walk 200 m West, then 500 m 30 m. How far is she from his initial position?
in South direction. Which direction should they take (a) 20 m (b) 30 m
to reach home? (c) 45 m (d) 80 m
(a) North-East (b) North 10. Govind went 15 km from his house to the North.
(c) North-West (d) South-East
Then, he turned West and covered 10 km. Then, he
5. A man travels 2 km towards North, then he turns 45° turned South and covered 5 km. Finally, turning to
1 East, he covered 10 km. Then, he turned South and
South-East and travels 1 km. From there, he makes
2 covered 5 km. In which direction is he from his house?
a line towards his starting point. Which direction (a) East (b) West
(c) North (d) South
would he then be travelling?
(a) South 11. If X is to the South of Y and Z is to the East of Y. In
(b) West what direction is X with respect to Z?
(c) South-West (a) North-East (b) North-West
(d) South-East (c) South-East (d) South-West
Direction Sense Test 63
12. Raju from A moved North-West side 2 km to B from Directions (Q. Nos. 15 and 16) Study the information
there he turned 90° clockwise and moved 2 km to C. given below carefully and answer the questions that follow.
From there, he turned 90° clockwise and travelled From a same point K, Indu and Saumya walked in opposite
2 km to D. Which of the answer figures shows the directions. After walking 5 km, Indu turned right and
exact path he traced? walked 3 km, again turned right and walked 5 km, from the
C C starting point Saumya walked.
15. At this point, if Indu was facing East, Saumya would
(a) D (b) be facing which direction?
B B
(a) North
A (b) South
(c) East
D
(d) North-West
A
(c) D (d) 16. How far is Indu from the starting point?
C (a) 1 km (b) 2 km
(c) 3 km (d) 4 km
B
13. Mahesh starts from A and walks North to B, then he Directions (Q. Nos. 17 and 18) Study the information
turned to East and walks to C and then turns given below carefully and answer the questions that follow.
North-West and walks to D. Finally, he turns to East Anshu started from ‘A’ and walked 10 km Eastwards to
and walks to E. Which of the answer figures shows the reach ‘B’, then turned to North and walked 3 km to reach ‘C’
exact path he traced? and then turned West and walked 12 km to reach ‘D’. He
D then again turned South and walked 3 km to reach ‘E’.
C
B E
B
C 17. How far is Anshu from his starting point?
(a) (b) (a) 2 km (b) 3 km
E D A
(c) 1 km (d) 2.5 km

C
A 18. In which direction is point ‘E’ from point ‘A’?
D E
B (a) East
(b) West
(c) D (d) B C
(c) North
E
A (d) South
A
Directions (Q. Nos. 19 and 20) Read the given information
14. Ranjana starts from A and walks towards South-East carefully and answer the given questions.
to B. She turns West and walks to C. Then, she turns
Point N is 8 m to the West of point O. Point P is 4 m to the
North-West and walks to D. Finally, she turns East
South of point O. Point Q is 4 m to the East of point P. Point
and walks to E. Which of the answer figures exactly R is 6 m to the North of point Q. Point S is 8 m to the West
shows the path Ranjana traced? of point R. Point T is 2 m to the South of point S.
E A E D A
D 19. How far and in which direction is point T from
point N?
(a) (b) (a) 4 m to the East
C B C B (b) 8 m to the West
(c) 4 m to the West
E (d) 8 m to the East
D C B
(c) (d) 20. If point T is 4 m to the North of point E. Then, what is
the distance between E and Q?
B C D (a) 11m (b) 8 m
(c) 15 m (d) 5 m
Hints and Solutions
Elementary Level Questions
1. (c) C
1 km
B
N 6. (b) Clearly, the route is as shown in the diagram. Thus, the man
2 km started his journey from the South and moved Northwards.
W E
2 km
S
10 km

End point
2 km
Starting point
(House) 6 km

So, the final position is towards the F, i .e., South direction.


7. (d) From the figure given below, Raj is finally in the North-West
2. (b) 4 miles 2 miles E direction from the starting point.
D
1 mile 50 m
3 miles Final N N
End NW NE
6 miles
C B W E 100 m
W E
4 miles S
SW SE
A Start S
Starting point 100 m
He is finally in South facing state from point E. 8. (d) The movements of 60 m F
E
the person are from A to
3. (c) N 20 m
F, as shown in figure.
Clearly, the final position C
of F which is to the D 10 m 10 m
North-East of the starting B A
A 20 m
W E point A.
O 20 m 10 m
B 9. (a) N
5m 9m 6m
D 12 m E A
m
30
30

S
m

B
W E
From the figure, cat is facing North.
30

O
m
m

4. (b) Starting from his house which is Eastward and moves


30

C
Westwards comes at O, the crossing. To his left is fire station
(South) and straight, i .e., towards West, there is a bookshop. So,
S
obviously his school is towards North.
Final direction is CO which is in North-East.
North N
O School 10. (c)
Final Anti-clockwise
initial (N–E)
Y C
West East
O House
Bookshop W E
O
Clockwise
O
South
(S–E)
Fire station S
OC is initial direction and he moves 90° clockwise, then 135°
5. (c) Clearly, his final position is E which is to the North of his house anti-clockwise, it means as a result the change in position will be
at A. 45° anti-clockwise direction that is OY direction in North
C 10 m B direction.
11. (a) Clearly, there are two possible movements of Anuj as shown
5m
below.
North
15 m
D 10 m E

A South
Thus, Anuj is finally moving towards either North or South.
Direction Sense Test 65
12. (c) From the diagram, it is clear that West will be called 17. (a) EA = ED + DA = ED + CB 40 m
C B
South-East = 20 + 40 = 60m
S-W 20 m 20 m
E

W
S D C
20 m D 40 m A
S-E N-W
18. (c) 10 m
14 m 14 m
E
A
B 14 m
N
E

Final Starting
N-E point
13. (d) Clearly to show 4:30, the position of the minute and hour Shortest distance = AB + BE = 14 + 10 = 24 m
hands of a clock will be as shown. So, again as shown, if the N
minute hands points East, the hour hand will point in the 19. (d)
North-East. 40 m
12 1 W
11
40 m 40 m
W

N
-W
S-

10 2
W D E
9 3 40 m C O
S N
8 4
7 5
-E
S-

6
N
E

E
S
14. (d) End point
Q Distance from initial point OW = OC + CD
D
= 40 + 40 = 80 m
N
20. (b)
4 km
6 km
B C
W E
O 40 m
4 km 4 km D C
30 m 20 m
A E A
Starting 6 km 40 m B
point S
Distance between the starting point and end point is AD. Distance OD = OA – AD = 30 − 20 = 10 m
(Pythagoras theorem in ∆ ADE,) 21. (c) The movements of the man are shown in figure.
Now, AD2 = AE2 + DE2 E
AD2 = (6)2 + (8)2
4 km
∴ AD = 36 + 64 = 100 = 10 km
A F
15. (b) She starts from A. 1 km B
D 5 km
E A 5 km
10 cm
C D
10 cm 10 km 2 km
(A to B, B to C, C to D and D to E)
C B Clearly, DF = BC = 5 km
5k EF = (DE − DF) = (9 − 5) = 4 km
Distance = ED + DA = (10 + 5) BF = CD = 2 km
= 15 km AF = AB + BF = AB + CD = (1 + 2) = 3 km
16. (a) 4 km ? 2 km Man’s distance from starting point A = AE
K F A B
= (AF)2 + (EF)2 = 32 + 42 = 25 = 5 km
3 km 3 km
22. (b) Clearly, the boy rode from A to B, then B to C and finally upto
D C D. Since, D lies to the West of A, so required distance
8 km = AB = CD = 2 km
∴ Distance between A and F C B
= DC − (AB + FK) = 8 − (4 + 2 )
= 8 − 6 = 2 km 2 km

D A
1 km
66 Study Package for NTSE

High Skill Questions


N
1. (d) NW 8. (b) In the evening, Sun is in the West and so the shadows fall
2 km towards East. Now, since Shyam’s shadow fell towards right, so
2 km Shyam is facing North. Thus, Ram is standing face to face with
W
4 km O
E Shyam and is facing South.
9. (c) B
N
S
6 km N-W N-E
From the figure, we can see that Mohan starts from his home O
in West direction. W E
10 km
2. (c) Initial position
3 km S-W S-E
C D
W 8 km (End point) S
E
Final 45°
A
8

km

(Starting point)
km

NW
8

90°
SW Starting
25 m point
He is now in West from its starting point.
B
3. (d) 5 km
Start 30 m A
3 km
End 5 km 5 km
20 m 20 m
3 km
He is 18 km West from the starting point. C 30 m D
4. (a)
House
NE N Final distance = OA + AD
= 25 + 20
200 m
500 m W E
= 45m
S
10. (c) Clearly, he is in North direction from his house
Direction from final point is North-East. 10 km
5 km 5 km
5. (c) North
1.5 km 10 km 5 km 15 km
2 km A
45° B end point
West East 5 km
starting
point

South 11. (d) Y Z (East)


Final direction is along AO, i.e., South-West. N

6. (d) End
50 m W E

100 m S
X (South)

100 m Clearly, X is in South-West direction with respect to Z.


Starting point 12. (b) Raju’s sequence of the path is
So, the final direction is North-West.
C
m 2k
7. (c) South-East is called East means that the direction is turning 2k m N
NW NE
anti-clockwise through 45°. So, North will become North-West. B
N D
2k W E
N
W

m
N

A SW SE
W E S
SE
SW

S
Direction Sense Test 67
13. (d) Mahesh’s sequence of the path is Sol. (Q. Nos. 17 and 18)
N
N D 12 km C
D E NW NE

W E
W E 3 km
B C 3 km
SW SE S
S
A E A 10 km B
Obviously, Anshu is 2 km West from A, starting point.
14. (a) Ranjana’s sequence of the path is
N
17. (a) AE = EB − AB = 12 − 10 = 2 km
E A NW
D
18. (b) Clearly, Anshu is 2 km West from A, starting point.
W E
Sol. (Q. Nos. 19-20) According to the question, the direction diagram
SE is as given below.
C B S
S 8m R
Sol. (Q. Nos. 15 and 16) 2m N
5 km 4m 4m
A N N O
8m 6m W E
3 km T
3 km W E 4m 4m
S
K S E 4m P 4m Q
5 km 4 km
Indu Saumya 19. (a) Clearly, point T is 4 m to the East of point N.
15. (a) Saumya would be facing towards North. 20. (b) Distance between E and Q
16. (c) Indu’s distance = KA = 3 km = PQ + PE [QPE = OT = 4 m ]
= 4m+ 4m
= 8m
Chapter

10
Inserting the Missing
Character
In such type of questions, some figures are given which have certain characters, numbers or a combination of
number and characters. The numbers and letters are allotted according to some rules. They may follow a pattern
and arranged according to a rule [continuation, square root, multiplied, divided, cube etc]. The students are
required to find out the rule that has been applied and select the suitable answer from the given choices.
Some examples are given below to clear the concept

Type I Questions Based on Numbers


Example 1 Insert the missing number. Solution (d) As, 6 × 7 + 4 = 46 ,5 × 4 + 5 = 25.
1 1 1 Similarly, 7 × 5 + 12 = 47 .
2 4 16 ∴ The missing number is 47.
3 9 ?
Example 3 Insert the missing number.
(a) 81 (b) 18 (c) 27 (d) 17
2
Solution (a) In this set of problem the consequent ? 5
number in each line from left to right is the square of the
predecessor digit/number. 20 9
(i) line — 12 = 1, 12 = 1 14

(ii) line — 22 = 4 , 4 2 = 16
(a) 25 (b) 26
(iii) line — 3 2 = 9, 9 2 = 81
(c) 27 (d) 29
∴ The missing number is 81.
Solution (c) Starting with 2, move in the clockwise
Example 2 Find the missing character. direction for making the series.

2 + 3 = 5
5 + 4 = 9
7 6 7 5 5 4 9 + 5 = 14
46 ? 25 14 + 6 = 20
20 + 7 = 27
4 12 5
∴ The missing number is 27.
(a) 25 (b) 41 (c) 37 (d) 47
Inserting the Missing Character 69
Type II Questions Based on Letters
Example 4 Insert the missing character.
L G H

C W A

J ? O

(a) A (b) B (c) D (d) G


Solution (b) In each row add the numerical value of the left and right hand letters together and write the letter
with the reverse alphabetical value in the centre.
Hence, J + O = 10 + 15 = 25 Reverse alphabet on 25 = B.

Example 5 Insert the missing letter.


Z B X D ? F T

A Y C W E U G

(a) V (b) F (c) G (d) X


Solution (a) Every column has opposite letter. So, V is opposite to E or addition of colums letter is equal to 27.
Example 6 Which letter replaces the question mark?

A P Y C V C G Y K ?
U S E G B

(a) L (b) K (c) J (d) I


Solution (d) Working from left to right, letters in corresponding segments of the circles move through the
alphabet in steps of 2, 3, and 4, with their relative positions moving one place clockwise at each step.
∴ Missing term = G + 2 = 7 + 2 = 9 = I.

Target Exercise
Elementary Level Questions
Directions (Q. Nos. 1-26) Insert the missing letter or numerical value in following questions.

1. A F K 4. 15 25 30
C H M 12 14 16
F K ? 6 ? 16
(a) N (b) O (c) P (d) Q (a) 60 (b) 16 (c) 20 (d) 10

2. 6 8 10 5.
1 4 4 9 9 ?
5 110 10 15 305 10 10 ? 11

10 15 12
2 6 12
(a) 240 (b) 230 (c) 100 (d) 132
(a) 14 (b) 3 (c) 4 (d) 16
3.
6 3 12 07 09 08 6. 10 15 12

18 09 84 19 ? 17 4 48 8 9 78 8 4 ? 8

(a) 72 (b) 17 8 10 11
(c) 153 (d) 25 (a) 88 (b) 100 (c) 32 (d) 132
70 Study Package for NTSE

7. 15. 9 A 12
5
4 B 10 7
4 8 C 11
6

100 81
(a) A = 13, B = 11 and C = 9
2 ? 64 5
(b) A = 13, B = 9 and C = 11
4 3 (c) A = 9, B = 11 and C = 13
(d) A = 9, B = 13 and C = 11
(a) 64 (b) 36 (c) 16 (d) 72
16. 7
8.
13 11 56
12 14 4 28 ? 8
15 ? 60
10 16 15
(a) 33 (b) 145
(a) 17 (b) 8 (c) 9 (d) 18
(c) 120 (d) 18
9. 10
4 3
17.
4
3 64 5
9 7 27 125
? 216
8 343 6
13 12 7
?
(a) 9 (b) 7 (c) 8 (d) 6 (a) 640 (b) 512 (c) 16 (d) 24

10. 18. 0 2 4
4 3 5 6 7 8
2 6 3
25 61 ? 3 ? 5
3 4 6 5 8 7 35 225 216
(a) 0 (b) 2
(a) 64 (b) 113 (c) 49 (d) 15
(c) 1 (d) 4
11. 11 1 10 02 09 03
19. 6 1 15

120 96 ? 2 60 4 4 40 2 20 ? 03
(a) 81 (b) 72
(c) 90 (d) 27 3 3 09
(a) 235 (b) 141
12. (c) 144 (d) 188
84 81 88

14 12 18 09 ? 11 20.
?

(a) 12 (b) 16 (c) 32 (d) 20 27 25

13. 31 23 27
112 128 55
7 4 11 (a) 26 (b) 25 (c) 27 (d) 24
32 64 ? 3 7
21 52
5 11 126
(a) 5 (b) 10
(c) 12 (d) 13 ? 9 107

14. (a) 18 (b) 26 (c) 36 (d) 6


101 43 48 34

38 142 22. 5 11 55
7 ? 91
35 15 56 ? 6 16 96
(a) 198 (b) 142 (c) 158 (d) 184 (a) 11 (b) 12 (c) 13 (d) 14
Inserting the Missing Character 71
23. 4 20 25 3 8 10 2 ? 1
25.
27 81 9 6 56 90 2 20 0
11 44 ?
(a) 0 (b) 3
(a) 4 (b) 30 (c) 55 (d) 16 (c) 5 (d) 7

24. 3 5 4 7 3 5
26. B2 ?
39 51 ? Q48
F4
6 3 5 4 5 4 K12
A B C (a) W, 210 (b) X, 240
(a) 47 (b) 45 (c) 37 (d) 35 (c) X, 120 (d) Y, 220

High Skill Questions


Directions (Q. Nos. 1-22) Insert the missing number or letter from among the given alternatives.

1. 6 18 15 7. 2Z 10Y 5X
3 2 5 20X ? 4Y
4 3 ? 10Y 40X 4Z
8 27 9
(a) 8Z (b) 80Z (c) 80X (d) 8X
(a) 11 (b) 6 (c) 3 (d) 2 8. 18 60 ?
2. 1 2 1 3 1 5
2 3 4
4 5 6 2 3 4 5 6 7
4 5 6 6 9 ?
(a) 98 (b) 116 (c) 125 (d) 126
(a) 6 (b) 7 (c) 8 (d) 9
9. 1 6 5 2 2 0 5 3 3
3. 7
4
3
5

1 64 131 248 ?

11 27 ? 8
2 3 4 4 6 8 7 9 1
8 2
(a) 320 (b) 274 (c) 262 (d) 132

(a) 0 (b) 8 (c) 125 (d) 216 10. 15 2 9 7 13 16


4. 13
80 65 ?
1
? 4 2 5 5 6 4 6 11 8

3
(a) 48 (b) 35 (c) 72 (d) 120
11.
8 A C G I
(a) 10 (b) 11 (c) 12 (d) 13 E Y K E
5. 7 5 5 21 24 06
I U O A
6 13 ?
M Q S ?
(a) 12 (b) 14 (c) 15 (d) 13
(a) U (b) W (c) V (d) X
6. B K F
12. 4 8 0

G A
5 0 6 7 1 6 11 ? 2

L Q ?
7 4 2
(a) S (b) V (c) T (d) U (a) 0 (b) 2 (c) 11 (d) 12
72 Study Package for NTSE

13. 18. 1 4 6
9 5 5 3 8 3

425 416 ? 551 246 ?

2 5 2 4 3 5 3 4 7 9 2 1
6 8 4
(a) 140 (b) 280 (A) (B) (C)
(c) 875 (d) 925 (a) 262 (b) 622 (c) 631 (d) 824
14. 5 4 8 2 19. 12 18 30 16 32 40 36 18 27
4 7 2 8

7 4 2 9
5 4 4 ?
6 8 ?
(A) (B) (C)
(a) 5 (b) 4
(c) 14 (d) 5 (a) 18 (b) 12 (c) 9 (d) 6

15. 20. 3 GHT 5


1
15
?
2 17 5 MZO 4
16
4 6 3 JTI 9
3 18 5
19 3 MOH 6
3 OEL ?
(a) 13 (b) 14 (a) 2 (b) 4 (c) 3 (d) 1
(c) 20 (d) 21
21. 5 4 3
16. 16 25 49
2 4 10 7 7
9 25 16 36 36 ? 5 4 2 10 2 3 4
36 49 81 7 2 ? 1 2
4 4 4
(a) 64 (b) 144
(c) 169 (d) 25 (a) 6 (b) 5 (c) 2 (d) 3
17. 4 8 5 22.
2 V 9 ?
6 14 6 14 6 ? 2 5 10
8 8 4
X 4 Q ?
10 A 18 14 B 22 11 C 15
(a) 6 (b) 8 (a) G, 19 (b) H, 14
(c) 10 (d) 14 (c) I, 20 (d) E, 14

Hints and Solutions


Elementary Level Questions Similarly, 10 × 12 + 10 × 11 = 120 + 110 = 230
∴ ? = 230
1. (c) In first column, A C F

+2 +3 3. (a) As, 6+ 3=9 12 + 7 = 19


6 × 3 = 18 12 × 7 = 84
In second column, F H K
Similarly, 9 + 8 = 17
+2 +3
Q 9 × 8 = 72
∴ In third column, K M P
∴ ? = 72
∴ ?= P +2 +3
4. (c) As, (15 + 25) − 30 = 10
2. (b) As, 6 × 10 + 5 × 10 = 60 + 50 = 110
and 8 × 10 + 15 × 15 = 80 + 225 = 305 (12 + 14) − 16 = 10
Inserting the Missing Character 73
Similarly, (6 + ?) − 16 = 10, 32 × 7
13. (b) As, = 112
? = 26 − 6 = 20 2
64 × 4
5. (d) As, (1)2 = 1 (2 )2 = 4,1 × 2 = 2 and = 128
2
(2 )2 = 4, (3)2 = 9, 2 × 3 = 6 ? × 11
Similarly, = 55
2
Similarly, (3)2 = 9, (?) = ( x)2 ,
∴ ? = 10
but 3 × x = 12
14. (d) As, (101 + 15) − (43 + 35) = 38
⇒ x = 4 and ? = 42
Similarly, (48 + ?) − (34 + 56) = 142
⇒ ? = 16 ⇒ ? = 184
6. (b) As, 10 × 8 − 8 × 4 = 80 − 32 = 48 15. (d) As, row III, 12 + 7 + 11 = 30
15 × 10 − 9 × 8 = 150 − 72 = 78 row I, 9 + B + 8 = 30
Similarly, 12 × 11 − 4 × 8 = 132 − 32 = 100 B = 30 − 17
∴ ? = 100 B = 13
7. (b) As, 5 + 4 = 9 ⇒ 92 = 81 row II, A + 10 + C = 30
A + C = 30 − 10
5 + 3 = 8 ⇒ 82 = 64
A + C = 20
6 + 4 = 10 ⇒ 102 = 100 From option (d),
Similarly, (2 + 4) = 6 ⇒ 62 = 36 A = 9, B = 13, C = 11
∴ ? = 36 16. (c) As, 7 × 4 = 28, 4 × 15 = 60, 8 × 7 = 56
8. (c) Quarter parts of circle have total 25. Similarly, 15 × 8 = 120
As, 11 + 14 = 25 ∴ ? = 120
10 + 15 = 25 17. (b) As, 43 = 64, 53 = 125, 63 = 216.
12 + 13 = 25 7 3 = 343, 33 = 27
Similarly, ? + 16 = 25 Similarly, 83 = 512
⇒ ?=9 ∴ ? = 512
9. (d) Sum of numbers on line = 16 18. (c) As, (0)3 + (2 )3 + (3)3 = 35
As, 3 + 13 = 16 (4)3 + (3)3 + (5)3 = 216
7 + 9 = 16 Similarly, (2 )3 + (6)3 + (?)3 = 225
12 + 4 = 16 (?)3 = 1
Similarly, 10 + ? = 16 ⇒ ? =1
⇒ ?=6
19. (d) As, 4 × (6 + 2 + 3 + 4) = 4 × 15 = 60
10. (b) As, 4 × 4 + 3 × 3 = 16 + 9 = 25 4 × (1 + 4 + 3 + 2 ) = 4 × 10 = 40
and 5 × 5 + 6 × 6 = 25 + 36 = 61 Similarly, 4 (15 + 20 + 9 + 3) = 4 × 47 = 188
Similarly, ? =7 ×7 + 8× 8 ? = 188
= 49 + 64 31 + 23 54
20. (a) As, = = 27
∴ ? = 113 2 2
23 + 27 50
11. (b) As, (11 + 1) × (11 − 1) = 12 × 10 = 120 = = 25
2 2
(10 + 2 ) × (10 − 2 ) = 12 × 8 = 96 27 + 25 52
Similarly, = = 26
Similarly, ? = (9 + 3) × (9 − 3) = 12 × 6 2 2
? = 72 ∴ ? = 26

12. (b) As, 12 ×


14
= 12 × 7 = 84 21. (b) As, (7 ) = 49, 49 + 3 = 52
2

2 (11)2 = 121, 121 + 5 = 126


18
9× = 9 × 9 = 81 Similarly, (9)2 = 81, 81 + ? = 107
2
? ? = 107 − 81 = 26
Similarly, 11 × = 88, ? = 8 × 2 ,
2 22. (c) As, 5 × 11 = 55, 6 × 16 = 96
∴ ? = 16 Similarly, 7 × ? = 91
∴ ? = 13
74 Study Package for NTSE
20 As, 32 − 3 = 9 − 3 = 6
23. (d) As, = 5, (5)2 = 25
4
81 82 − 8 = 64 − 8 = 56
and = 3, (3)2 = 9
27 102 − 10 = 100 − 10 = 90
44
Similarly, = 4, (4)2 = ?
11 22 − 2 = 4 − 2 = 2
⇒ ? = 16 ⇒ 12 − 1 = 1 − 1 = 0
24. (c) As, In figure (A), (3 × 3) + (6 × 5) = 9 + 30 = 39 Similarly, 52 − 5 = 20
In figure (B), (4 × 4) + (5 × 7 ) = 16 + 35 = 51 So, the missing number is 5.
Similarly,
26. (b) Letter follow below pattern
∴In figure (C), missing number +4 +5 +6 +7
B 
→ F  
→ K  
→ Q   → X
= (3 × 4) + (5 × 5) = 12 + 25 = 37
Number follow below pattern
×2 ×3 ×4 ×5
25. (c) Suppose, X denotes the number in the first row and Y denotes 2 → 4   → 12  → 48  → 240
the number in the second row. Then , the pattern is X 2 − X = Y

High Skill Questions


6×4 8. (d) As, 2 × 3 = 6 × 3 = 18
1. (c) As, I column, =8
3 4 × 5 = 20 × 3 = 60
18 × 3
II column, = 27 6 × 7 = 42 × 3 = 126
2
15 × ? ∴ ? = 126
III column, = 9 ⇒? = 3
5 9. (c) As, (2 − 1) = 1, (6 − 3) = 3, (5 − 4) = 1 ⇒ 131
2. (b) The upper number at the cross is to be added to the right side (4 − 2 ) = 2, (6 − 2 ) = 4, (8 − 0) = 8 ⇒ 248
upper number to get the left side lower and cross is to be added
to the left side upper and number to get the right side lower Similarly, (7 − 5) = 2 , (9 − 3) = 6,
number. (3 − 1) = 2 ⇒ 262
As, 2 + 2 = 4, 1 + 4 = 5, 3 + 3 = 6, 10. (a) As, I. (15 − 5) × ( 2 + 6) = 80
1 + 5 = 6 and Similarly 1 + 6 = 7.
II. (9 − 4) × ( 7 + 6) = 65
3. (d) In each quarter (a − b)3 pattern is followed.
Similarly, III. (13 − 11) × (16 + 8) = 48
As, 7 − 3 = 4 = 43 = 64
+4 +4 +4
5 − 4 = 1 = 13 = 1 11. (b) As, column I, A → E → I → M
11 − 8 = 3 = 33 = 27 −4 −4 −4
column II, C → Y → U → Q
8 − 2 = 6 = 63 = 216
∴ ? = 216 +4 +4 +4
column III, G → K → O → S
4. (c) As, 5 + 3 = 8 , 8 + 4 = 12
−4
column IV, I → −4 −4
Similarly, 12 + 1 = 13 E → A → W
∴ ? = 12 ∴ ? =W
7+ 5 5 + 21
5. (c) As, = 6, = 13 12. (c) As, I. (5 + 6) − ( 7 + 4) = 0
2 2
24 + 6 II. (7 + 6) − (8 + 4) = 1
Similarly, = = 15
2
III. (11 + 2 ) − ( 2 + 0) = 11
∴ ? = 15
13. (d) As, (2 )2 = 4, (5)2 = 25 ⇒ 425
6. (b) Starting at the top left, and moving anti-clockwise around the
figure, letters advance through the alphabet 5 letters at a time. (2 )2 = 4, (4)2 = 16 ⇒ 416
Hence, missing term is V.
10Y Similarly, (3)2 = 9, (5)2 = 25 ⇒ 925
7. (b) As, first row, = 5X
2Z ∴ ? = 925
40X
Second row, = 4Z 14. (a) In each figure the addition of numbers is equal to 20.
10Y
Similarly, third row,
?
= 4Y ∴ 2 + 9 + 4 + ? = 20
20X ⇒ 15 + ? = 20
∴ ? = 80Z ⇒ ? = 20 − 15 = 5
Inserting the Missing Character 75
15. (b) As, outside numbers 19 + 1 = 0, 17 + 3 = 20, 15 + 5 = 20 J T I ⇒10 + 20 + 9 = 39

Inside numbers, 18 + 2 = 20, 16 + 4 = 20 (Positional values)

Similarly, 6 + ? = 20 ⇒ ? = 14 M O H ⇒13 + 15 + 8 = 36
(Positional values)
16. (a) In the Circle, the number of squares are given in ascending
order. O E L ⇒15 + 5 + 12 ⇒ 32

∴ In last circle start from side of circle (Positional values)


6 = 36
2 21. (a) In first row,
7 2 = 49 5+ 3=8
8 = 64
2 ⇒ 8 ÷ 2 = 4 (in middle circle)
9 = 81
2 In second row,
2 + 4 + 7 + 7 = 20
So, 64 will replace the question mark.
⇒ 20 ÷ 2 = 10 (in middle circle)
17. (c) As, In figure (A), 10 − 4 = 6
In third row,
18 − 10 = 8
5+ 4+2 +2 + 3+ 4
18 − 4 = 14
In figure (B), 14 − 8 = 6, ⇒ 20 ÷ 2 = 10 (in middle circle)
22 − 14 = 8 In fourth row,
22 − 8 = 14 7 + 2 + 1+ 2
In figure (C), 11 − 5 = 6, 15 − 11 = 4 ⇒ 12 ÷ 2 = 6 (in middle circle)
Similarly, missing number = 15 − 5 = 10 In fifth row,
∴ ? = 10 4+4=8
18. (b) As, In figure (A), (915 − 364) = 551 ⇒ 8 ÷2 = 4 (in middle circle)
In figure (B), (789 − 543) = 246 ∴ ?=6
Similarly, In figure (C), missing number 22. (a) Number follow below pattern
= (863 − 241) = 622 2 + 2 = 4, 4 + 5 = 9
19. (c) The above three numbers are multiples of the number at the 9 + 10 = 19
bottom.
Letter follow below pattern
Clearly, 36, 18 and 27 are all multiples of 9. So, the missing
2 + 24( X ) = 26 (Q positional value of X is 24)
number is 9.
4 + 22(V ) = 26 (Q positional value of V is 22)
20. (a) G H T ⇒7 + 8 + 20 = 35
(Positional values) 9 + 17(Q ) = 26 (Q positional value of Q is 17)
M Z O ⇒13 + 26 + 15 = 54 19 + ? = 26
(Positional values) ⇒? = 26 − 19 = 7 = 9 (Q positional value of G is 7)
Chapter

11
Ranking Test
The process of determining the position or place of a person or a thing on the basis of comparision or relative
position of other person or thing is called ranking. There are two types of questions based on ranking test which is
as follows.

Type I Statement Based Questions


In this type of questions, a set of information pertaining to persons, objects or some other entities along with their
qualities, which can be compared is provided. Candidates are required to arrange the given entities in either
ascending or descending order on the basis of relative quality. The candidate has to understand the given
information systematically candidate is to answer the question.
Sometimes candidate is required to ascertain only the rank of a person either from top or bottom in a class or a
group of persons. In determining the rank, the information about total number of persons in the class or a group is a
pre-requisite otherwise the candidate cannot determine the rank. Sometimes the rank of a person from either end
is given and the candidate is required to calculate the total number of persons.
Note Total number of persons = Position of person from top + Position of person from bottom −1
l If two persons are on a definite position from top and bottom and they interchange their ranks. Then, total number of
persons = (First position of first person + Second position of second person) − 1
e.g., Let there be n persons who qualified in a certain event. A particular girl Priya is one of them whose rank from the top
e.g., Top rank (Tr ) is 16th and his rank from the other side i. e., Bottom rank (Br ) is 2nd. Then, total number of girls (n) who
qualified, is
⇒ n = Tr + Br − 1 ⇒ n = 26 + 16 − 1 = 41
Example 1 In a row of boys, Ram is seventh from the left and Shyam is twelfth from the right. If they interchange
their positions, Ram becomes twenty-second from the left. How many boys are there in a row?
(a) 49 (b) 31 (c) 33 (d) Data inadequate
Solution (c) Ram’s new position is 22nd from left. But is the same as Shyam’s earlier position, which is 12th
from right.
⇒ The number of persons in a row = Tr + Br − 1 = ( 22 + 12 − 1) = 33

Example 2 In a class, Ajit ranks 11th from the top and 27th from the bottom. If in the same class Anuja ranks
14th from the top, how many children below Anuja, in the class?
(a) 23 (b) 30 (c) 24 (d) 29
11th
Solution (a) 10 students Ajit
→
26 students
27th ←
Total number of students in the class = 10 + 1 + 26 = 37
Anuja’s rank from top = 14th
Therefore, the number of students, who are below Anuja in the class = 37 − 14 = 23
Ranking Test 77
Type II Comparison Based Questions
In another type of questions on ranking, you are required to ascertain some other qualities such as height, weight,
marks, age etc., In this type of questions, the two extreme ends are occupied by the persons having the lowest and
the highest degrees of quality that has to be compared. While, solving this type of questions, you should employ
certain symbols and notations to organise the given information, so that you can use the data more conveniently
and quickly too.
Example 3 A is taller than E, B is taller than D, F is taller than C, D is taller than A and E is taller than F, then
who is the tallest among them?
(a) D (b) B (c) E (d) F
Solution (b) A>E …(i)
B>D …(ii)
F>C …(iii)
D>A …(iv)
E>F …(v)
From equations B > D > A > E > F > C. Clearly, B is the tallest among them.

Target Exercise
Elementary Level Questions
1. Vishnu ranks 16th from the top and 49th from the 7. Sixty students were there in a mental ability test.
bottom in a class. How many students are there in the Ten students did not qualify, out of remaining Yogesh
class? ranked 25th from the last. What was his rank from
(a) 66 (b) 65 top?
(c) 64 (d) Cannot be determined (a) 27th (b) 26th
(c) 29th (d) 28th
2. Mita is taller than Rita but not as tall as Soni. Rita is
taller than Sarita. Soni is not as tall as Rupa. Who 8. Rajesh ranked 25th in the class of forty-one students.
among them is the tallest? What is the rank of Rajesh from the end?
(a) Mita (b) Rupa (a) 15th (b) 17th
(c) Soni (d) Sarita (c) 16th (d) 18th

3. In a class of twenty students, X ranks above Y who 9. Rajni is 6th from either end of a row of girls. How
ranks below M. N ranks above X but below M. T ranks many girls are there in that row?
between X and Y. Y ranks between T and L. Who (a) 10 (b) 12 (c) 13 (d) 11
among them has secured the topmost ranks? 10. Ravi is 7th ranks ahead of Sumit in a class of 39. If
(a) M (b) N Sumit’s rank is 17th from the last, what is Ravi’s
(c) T (d) X rank from the top?
4. In a cricket match, Sachin scored more runs than (a) 14th (b) 12th
Rahul but not as many as Kiran. Vinod scored less (c) 16th (d) 11th
than Rahul but more than Gagan and Anand. Whose 11. In a row of children facing North, Seema is shifted to
score was the lowest in the match? her right by four places becomes eighteenth from the
(a) Rahul (b) Gagan right end of the row. Sudhakar, who is fifteenth from
(c) Anand (d) Either Gagan or Anand the left end of the row, is fifth to Seema. How many
children are there in the row?
5. Sudhir is shorter than Ram. Rajan is shorter than
(a) 42 (b) 38
Preetam,Vijay is shorter than Ram. Who among
(c) 41 (d) 39
them is the shortest?
(a) Vijay (b) Rajan 12. In a row of thirty boys, R is fourth from the right end
(c) Sudhir (d) Cannot be determined and W is tenth from the left end. How many boys are
6. In a row of plants, a plant is 16th from either end of there between R and W?
the row. How many trees are there in the row? (a) 15 (b) 16
(c) 17 (d) Cannot be determined
(a) 32 (b) 30 (c) 31 (d) 16
78 Study Package for NTSE

13. Ravi is sixteenth from the first in a column of boys. twenty-first from the left. Which of the following will be
There are twice as many behind him as there are in Purnima’s new position from the right?
first. How many boys are there between Ravi and (a) 8th (b) 17th
fifth boy from the end of the column? (c) 21st (d) Data inadequate
(a) 20
15. Three persons P, Q and R are standing in a queue.
(b) 25
There are eight persons between P and Q and twelve
(c) 30
persons between Q and R. If there be seven persons
(d) 28
ahead of R and twenty eight persons behind P, what
14. In a row of girls, Purnima is eight from the right and could be the minimum number of persons in the
Bhavana is twelfth from the left when Purnima and queue?
Bhavana interchange their positions, Bhavana becomes (a) 57 (b) 42
(c) 40 (d) Data inadequate

High Skill Questions


1. In a row of students Prabhu is 9th from left and the right end of the row, if Swati was 3 places to the
Padma is 12th from right. In the same row, Ram is right of Komal’s original position?
12th from left and Radha is 9th from right. How many (a) 22nd (b) 23rd (c) 25th (d) 24th
students are there between Prabhu and Radha?
8. Six friends A, B, C, D, E and F start climbing a hill
(a) 18 (b) 21 together. After sometime, it is observed that ‘D’ has
(c) 24 (d) Data inadequate
reached the position higher than ‘B’ but not as high as
2. In a group of six students, Nitin is heavier than ‘E’, ‘C’ is at a position not higher than ‘D’ and not lower
Mahesh but lighter than Nandu. Ketan is lighter than ‘A’, who is not lower than ‘F’ and not higher than
than Mahesh but he is not as light as Ramesh. If ‘B’. Who has climbed to the highest position?
Nandu is lighter than Amit, then who is the lightest? (a) D (b) E
(a) Mahesh (b) Ketan (c) B (d) Cannot be determined
(c) Ramesh (d) Cannot be determined 9. Five students participated in an examination and
3. Ravi ranked 14th from the top and 29th from the each scored different marks. Nidhi scored higher
bottom among those who passed an examination. Five than Mamta. Kavita scored lower than Prashant but
boys did not participate in examination and five get higher than Nidhi. Anil’s score was between Mamta
and Nidhi. Which of the following pairs represents
failed in it. How many boys were there in the class?
the highest and lowest scores respectively?
(a) 52 (b) 50 (c) 54 (d) 51
(a) Nidhi and Kavita (b) Kavita and Mamta
4. In a class, ‘A’ ranks 5th from the top while ‘B’ ranks (c) Anil and Kavita (d) Prashant and Mamta
8th from the bottom. If ‘C’ ranks 6th from A and is 10. Among six friends L, M, N, P, Q and S, each having a
immediately preceding B, then how many students different height, N is taller than Q and P but shorter
are there in the class? than M. P is taller than only Q while S is shorter than
(a) 15 (b) 16 (c) 18 (d) 19 only. Which of the following pairs represents the
tallest and shortest among the five friends?
5. Out of six towns, Dhulia is bigger than Amalner,
(a) M and P (b) L and Q
Shrirampur is bigger than Nasik, Jalgaon is not as
(c) P and Q (d) Cannot be determined
big as Shrirampur but is bigger than Amalner.
Amalner is smaller than Nasik but bigger than 11. In a unit test Suman’s marks in Mathematics is more
Manmad. Which is the smallest? than her marks in History but not more than
(a) Amalner (b) Nasik (c) Jalgaon (d) Manmad Geography. Her marks in Science is more than that
in History but not as much as in Mathematics and her
6. Five friends A, B, C, D and E read a newspaper. The marks in English is not as much as in Geography. In
one who reads first givens it to C. The one who reads which of the given subjects did she score the lowest?
last had taken it from A. E was neither the first nor (a) History (b) English (c) Science (d) Data inadequate
the last one to read. There were two readers between
B and A. Who reads the newspaper last? Directions (Q. Nos. 12 and 13) Read the following
(a) A (b) B (c) C (d) D information given below to answer the questions that follow.
7. In a row of 40 girls, when Komal was shifted to her Among A, B, C and D, B is heavier than A and C but C is taller
left by 4 places, her number from the left end of the than him. D is not as tall as C while A is the shortest. C is not
row became 10. What was the number of Swati from as heavy as A. D is heavier than B but shorter than him.
Ranking Test 79
12. Who is the heaviest? 15. Who is the eldest?
(a) B (b) A (c) D (d) C (a) Deepti (b) Lata
(c) Anita (d) Rajan
13. Who is the tallest?
(a) D (b) C 16. Which of the following pairs of persons are of the
(c) Either A or D (d) Cannot be determined same age?
(a) Mahima and Lata
Directions (Q. Nos. 14-18) Read the following information (b) Anita and Mahima
carefully to answer the questions that follow. (c) Mahima and Rajan
I. Anita, Mahima, Rajan, Lata and Deepti are five cousins. (d) There are no person
II. Anita is twice as old as Mahima. 17. Anita is younger than whom?
III. Rajan is half the age of Mahima.
(a) Rajan (b) Mahima
IV. Anita is half the age of Deepti.
(c) Deepti (d) Lata
V. Rajan is twice the age of Lata.
18. If Mahima is 16 yr old, then what is the age of Lata?
14. Who is the youngest?
(a) 4 yr (b) 5 yr
(a) Deepti (b) Rajan
(c) 7 yr (d) 14 yr
(c) Lata (d) Anita

Hints and Solutions


Elementary Level Questions
Top Bottom 6. (c) Numbers of plants = 16 + 16 − 1 = 31
16th
15 students 48 students
1. (c) → Vishnu ←
49th
7. (b) Out of 60 students, 10 of them did not qualify the test. So,
only 50 could qualify.
Total number of students in the class
= (16 + 49) − 1= 65 − 1 = 64 Or = 15 + 48 + 1 = 64 n = 50,B r = 25th
Tr = n + 1 − B r = (50 + 1) − 25 = 26th
2. (b) Soni > Mita > Rita …(i)
8. (b) n = 41, Tr = 25th
Rita > Sarita …(ii)
Rupa > Soni …(iii) ⇒ n = Tr + B r − 1
From Statements (i), (ii) and (iii), B r = 41 + 1 − 25 = 17th
Rupa > Soni > Mita > Rita > Sarita
9. (d) Rajni is on 6th place.
So, it is clear that Rupa is the tallest. So, n = 6 + 6 − 1= 11
3. (a) According to the question, 10. (c) Sumit is 17th from the last and Ravi is 7th rank ahead means
M>X>Y Ravi is 24th rank from the last.
M>N>X So, Ravi’s rank from the top
X>T>Y = 39 − 24 + 1= 16th
T>Y>L 11. (c) From the given information, we get
For all the statements, M > N > X > T > Y > L 14 children → Sudhakar → 4 children
Clearly, M secured the topmost rank. → Seema → 21 children
∴ Total number of children in the row
4. (d) Kiran > Sachin > Rahul = 14 + 1 + 4 + 1 + 21 = 41
Rahul > Vinod > Gagan or Anand
12. (b) W = 10th from the left, R = 4 th from the right
From both the statements, Kiran > Sachin > Rahul > Vinod >
Gagan or Anand. Thus, it is clear that either Gagan or Anand = (30 − 4 + 1) = 27 th from left
scored the lowest runs in the match. ∴ Number of boys between R and W
5. (b) Ram > Sudhir, = (27 − 10) − 1 = 16
Preetam > Rajan 13. (b) 15 students → Ravi → 30 students
Ram > Vijay ∴ Total boys = 15 + 1 + 30 = 46
From all three statements,
∴ Total boys between Ravi and the fifth boy from the end
Ram > Vijay, Sudhir > Rajan and Preetam > Rajan
= 46 − 16 − 5 = 25
Thus, it is clear that Rajan is the shortest among them.
80 Study Package for NTSE

14. (b) Purnima is eighth from the right and Bhavana is twelth from Purnima Bhavana
the left, we have left 11 girls 1 2 3 4 5 6 7 8 7 girls right
Bhavana Purnima [11 + 1 + 8 + 1] = 21
left 11 girls 7 girls right Now, Purnima’s new position from the right = 7 + 1 + 8 + 1 = 17
Again, when Purnima and Bhavana interchange their positions 15. (c) 7 persons → R → 3 persons → P → 8 persons → Q.
Bhavana becomes twenty first from the left, we get the following Hence, required number of persons
= 7 + 1 + 3 + 1 + 28 = 40

High Skill Questions


1. (d) 9th 12th 12th 9th Combining all the informations, we obtained the following
8 Students sequence.
P 12 Padma Radha
8 Students Prashant > Kavita > Nidhi > Anil > Mamta

Number of students unknown ∴ The pairs represent the highest and lowest scores are
Prashant and Mamta.
Since, the total number of students in the row is not given, we
cannot find the number of students between Prabhu and Radha. 10. (b) If P is taller than only Q, we can infer that Q is the shortest.
Similarly, if S is shorter than only L, we get to know that L is the
2. (c) Nandu > Nitin > Mahesh …(i)
tallest.
Mahesh > Ketan > Ramesh …(ii)
Amit > Nandu …(iii) 11. (d) Geography > Mathematics > History ...(i)
From all the three statements, Mathematics > Science > History ...(ii)
Amit > Nandu > Nitin > Mahesh > Ketan > Ramesh Geography > English …(iii)
Hence, Ramesh is the highest Thus, the lowest score was either in History or English.
3. (a) Number of boys who passed = 14 + 29 − 1 = 42 Sol. (Q. Nos. 12 and 13) Here, H and T in the subcript denote heavier
∴ Total number of boys in class = 42 + 5 + 5 = 52 and taller.
4. (d) Total number of students = 5 + 6 + 8 = 19 BH > A H and CH CT > B T
5. (d) Dhulia > Amalner A H > CH C T > D T, D T> AT
Shrirampur > Nasik DH > BH BT > DT
Jalgaon < Shrirampur 12. (c) DH > B H > A H > C H
Jalgaon > Amalner
Here, D is the heaviest.
Amalner < Nasik
Amalner > Manmad 13. (b) CT > BT > DT > AT

6. (d) According to the question, Here, C is the tallest.

B, C, E, A, D Sol. (Q. Nos. 14-18)


Hence, D reads the newspaper last. Let age of Mahima = x
7. (d) On shifting 4 places left, Komal is 10th from the left end of the Then, age of Anita = 2 x
row. Thus, Komal’s original position was 14th from the left end. x
Age of Rajan =
Swati is 3 places to the right of Komal’s original position. 2
Clearly, Swati is 17th from the left end. Age of Deepti = 4 x
Number of girls to the right of Swati = (40 − 17 ) = 23 Age of Lata =
x
4
Thus, Swati is 24th from the right end of the row.
Now, we have the following arrangement in descending
8. (b) E > D> B ...(i) order Deepti > Anita > Mahima > Rajan > Lata
D>C>A>F ...(ii)
14. (c) Lata is youngest.
B>A ...(iii)
15. (a) Deepti is eldest.
Combining all three, we get
E>D>B>C>A>F 16. (d) There are no two persons of the same age as per the given
∴ E has climbed the highest position. information.
17. (c) Anita is younger than Deepti.
9. (d) According to the question,
18. (a) Mahima’s age = 16 yr
Nidhi > Mamta
16
Prashant > Kavita > Nidhi ∴ Lata’s age = = 4 yr
4
Nidhi > Anil > Mamta
Chapter

12
Arithmetical
Reasoning
In arithmetical reasoning, simple unitary method based Example 1 Mani is double the age of Prabhu. Ramona is
questions and intelligent puzzle type questions are asked half the age of Prabhu. If Mani is sixty, then find out the age
in exams. of Ramona.
Some important formulae which can be used for solving (a) 20 yr (b) 15 yr
such problems are as follow (c) 10 yr (d) 24 yr
1. HCF and LCM of fractions Solution (b) According to the question,
HCF of numerators age of Mani = 2 × age of Prabhu and age of Ramona
HCF of fraction = 1
LCM of denominators = × age of Prabhu
2
LCM of numerators
and LCM of fraction = Now, age of Mani = 60 yr
HCF of denominators
⇒ 60 = 2 × age of Prabhu
2. In rule of BODMAS for simplification, we follow the order
⇒ Age of Prabhu = 30 yr
Bracket → Of → Division → Multiplication → Addition →
1
Subtraction ∴ Age of Ramona = × 30 = 15 yr
m 2
3. To find m% of a given number A = ×A
100 Example 2 Four numbers whose average is 60, the first is
4. To convert a decimal into a per cent, we shift decimal 1
th of the sum of last three. The first number is
point by two places top its right and put symbol of per 4
cent. (a) 15 (b) 45
Total profit
5. Profit percentage = × 100, (c) 42 (d) 48
Total cost
Total loss Solution (d) Let the four numbers be a, b , c and d.
Loss percentage = × 100
Total cost Then, a + b + c + d = 4 × 60
Principal × Rate × Time
6. Simple interest = a + b + c + d = 240
100

Sum of observations
7. Mean or average = a=
1
( b + c + d) (given)
Number of observations 4
Value of all units
8. Value of one unit = Suppose, b+ c+ d = y
Number of given units
1
Distance Now, y + y = 240
9. Speed = 4
Time
⇒ y + 4 y = 240 × 4
10. Proportion Two ratios are said to be in proportion, if 960
they are equal. The ratio 3/4 and 6/8 are equal. ∴ y=
5
3 6
The statement 3 : 4 = 6 : 8 or = is called a proportion. = 192
4 8
a c ∴ a = 240 − 192
If = ⇒ a × d =b×c
b d = 48
Target Exercise
Elementary Level Questions
1. In a fraction, if the denominator is increased by 1, it 11. In a row of girls, if Seeta who is 7th from the left and
becomes 1/2. If the numerator is increased by 9, it Leena who is 9th from the right interchange their
becomes 1. Find the denominator. seats, Seeta becomes 11th from the left. How many
(a) 18 (b) 19 (c) 20 (d) 21 girls are there in the row?
(a) 16 (b) 18 (c) 19 (d) 22
2. The number of students in each section of a modern
school is 36. After admitting some new students, four 12. In a row of children, Kamal is 6th from the left and
new sections were started. If the total number of Appu is 4th from the right. When Kamal and Appu
students in each section now is 30, then what was the exchange their positions, Appu becomes 17th from
number of sections initially? the right. Which will be Kamal’s position from the
(a) 3 (b) 4 left?
(c) 6 (d) Cannot be determined (a) 12th (b) 19th (c) 21st (d) 7th
3. Two persons x and y together have total ` 50 with 13. Mohan is 15th from both ends of a row of boys. How
them. If x loses half of his money and y loses 1/3rd of many boys are there in the row?
his money, they will have only ` 20. How much (a) 15 (b) 16 (c) 30 (d) 29
amount does y has?
(a) ` 13 (b) ` 20 (c) ` 30 (d) ` 35
14. If a clock strikes once at 1 O’ clock, twice at 2 O’ clock
etc., How many times will it strike in 24 h?
4. Age of Rajesh is 1/5th of his father’s age. The father’s (a) 300 (b) 156 (c) 78 (d) 136
age will be twice of Rekha’s age after 10 yr. If 3 yr ago,
15. Manisha is 11th from the either end of the row of
Rekha’s age was 12 yr, find the present age of Rajesh.
girls, how many girls are there in the row?
(a) 3 yr (b) 12 yr (c) 10 yr (d) 8 yr (a) 19 (b) 20
5. One-fourth of Nikhil’s money is equal to 1/6th of (c) 21 (d) 22
Yogesh’s money. If they both have ` 600, then the 16. 69 is divided into 3 parts such that the first number is
difference between their amounts is the arithmetic mean of the other two numbers. The
(a) ` 125 (b) ` 110 (c) ` 100 (d) ` 120 second number is 4 more than the third number. The
6. Sum of three numbers is 432. First number is three numbers are
3/4 times the second and the third number is half of (a) 23, 27 and 19 (b) 21, 27 and 23
the second number. The smallest number is (c) 23, 25 and 21 (d) 19, 29 and 21
(a) 96 (b) 97 (c) 85 (d) 98 17. A is 3 yr older than B and 3 yr younger then C, while
7. The sum of the digits of a two-digit number is 10. If 18 B and D are twins. How many years older is C
is subtracted from the numbers, the place values of than D?
the digit get interchanged. Find the number. (a) 3 yr (b) 6 yr
(a) 96 (b) 99 (c) 90 (d) 64 (c) 2 yr (d) 10 yr

8. The middle digit of a number between 100 and 1000 is 18. The cost of four pens, eight sharpeners and nine
zero and sum of the digits of the number is 12. If the erasers is ` 136, the cost of three pens, six sharpeners
digits in unit’s place and hundred’s place are and seven erasers together is ` 104. Find the cost of
interchanged, the number increases by 396. Find the an eraser.
(a) ` 7 (b) ` 8 (c) ` 4 (d) ` 5
number.
(a) 402 (b) 408 (c) 412 (d) 420 19. The cost of 5 mangoes is equal to the cost of
9. Five burgers, six pizzas and seven cokes cost is ` 178. 20 oranges. If the total cost of 2 mangoes and
Six burgers, four pizzas and two cokes cost is ` 124. 10 oranges is ` 22.50, find the cost of one orange.
(a) ` 1.20 (b) ` 1.30
Find the cost of one burger, one pizza and one coke.
(c) ` 1.25 (d) ` 1.35
(a) ` 28 (b) ` 30 (c) ` 32 (d) ` 40
20. In a school, half of the boys and a one-fourth of the
10. In a team, the captain took 1/3rd of a cake and the
girls enrolled for a course. If 7/8th of the boys and
remaining was divided equally among the others. The
3/4th of the girls actually joined, then what fraction of
captain had twice the share of any others member.
the total number of students did not join the course?
How many members are there in the team? 1 1 1 1
(a) (b) (c) (d)
(a) 4 (b) 5 (c) 6 (d) 7 15 16 14 13
Arithmetical Reasoning 83
High Skill Questions
1. A group of 1200 persons consisting of captains and 12. In a group of 15 people, 7 read French, 8 read English
soldiers are travelling in a train. For every 15 soldiers while 3 of them read none of these two. How many of
there is one captain. The number of captains in the them read French and English both?
group is (a) 0 (b) 3 (c) 4 (d) 5
(a) 85 (b) 80 (c) 75 (d) 70
13. Father’s age is 6 yr more than three times his son’s
2. At the end of a business confrence, ten people present age. If after 4 yr, the father’s age would be 12 yr more
all shakehands with each other once. How many than two times the son’s age, find the present age of
hands will there be altogether? the father.
(a) 20 (b) 45 (c) 55 (d) 90 (a) 30 yr (b) 35 yr
(c) 36 yr (d) 37 yr
3. A students got twice as many sums as he got right. If
he attempted 48 sums in all. How many did he solved 14. Divide ` 320 among A, B and C, so that A may have
correctly? `30 more than B and C may have ` 20 more than A.
(a) 12 (b) 16 (c) 18 (d) 24 (a) ` 110, ` 80 and ` 130 (b) ` 110, ` 150 and ` 70
(c) ` 80, ` 120 and ` 120 (d) ` 100, ` 100 and ` 120
4. A farmer built a fence around his square plot. He
used 27 fence poles on each sides of the square. How 15. The length of a rectangle is 10 cm more than its
many poles did he need altogether? breadth. If its perimeter is 84 cm, then what is the
(a) 100 (b) 104 length of the rectangle?
(c) 108 (d) None of these (a) 20 cm (b) 24 cm
(c) 28 cm (d) 26 cm
5. If every 2 out of 3 readymade shirts need alterations
in sleeves and every 4 out of 5 need it in the body, how 16. In 5h Pawan can walk 4 miles more than what Sachin
many alterations will be required for 60 shirts? can in 8h. In 7h Sachin can walk 2 miles more than
(a) 24 (b) 123 (c) 133 (d) 143 Pawan can walk in 3h. What is Pawan’s speed?
(a) 4 m/h (b) 5 m/h
6. If a clock takes seven seconds to strike seven, how (c) 6 m/h (d) 7 m/h
long will it take to strike ten?
(a) 7 s (b) 9 s 17. Atul’s age is 8 yr more than thrice the sum of the ages
(c) 10 s (d) None of these of the two grandsons who are twins. After 8 yr, Atul’s
age will be 10 yr more than twice the sum of the ages
7. On children’s day, sweets were to be equally of his grandsons. What was the age of Atul when his
distribute among 175 children in a school. Actually on grandsons were born?
the children’s day, 35 children were absent and (a) 70 yr (b) 72 yr (c) 73 yr (d) 75 yr
therefore each child got 4 sweet extra. Total how
many sweets were available for distribution? 18. There are some monkeys on two trees T1 and T2 . If
(a) 2400 (b) 2480 (c) 2600 (d) 2800 two monkeys jump from T1 to T2 the number of
monkeys on each tree will be equal. If two monkeys
8. Between two book ends in your study are displayed jump from T2 to T1, number of monkeys on T1 would
your five favourite puzzle books. If you decide to be doubled the number of monkeys on T2 . Find the
arrange the five books in every possible combination initial number of monkeys on T1 and T2 respectively.
and moved just one book every minute, how long (a) 15 and 12 (b) 14 and 10
would it take you? (c) 12 and 10 (d) 14 and 12
(a) 1 h (b) 2 h (c) 3 h (d) 4 h
19. A sum of ` 1420 is distributed among 7 men, 11 women
9. If 100 cats kill 100 mice in 100 days, then 4 cats would and 10 boys, so that each woman gets thrice the
kill 4 mice in how many days? amount that a boy gets, and the share of one man
(a) 4 days (b) 10 days (c) 40 days (d) 100 days
equals the sum of the shares of a woman and a boy.
10. Mr. Johnson was to earn ` 300 and a free holiday for Find the sum of the shares of the women and the boys
seven weeks work. He worked for only 4 weeks and put together.
earned ` 30 and a free holiday. What was the value of (a) 1220 (b) 660 (c) 860 (d) 760
the holiday? 20. The sum of the heights of three persons is 5.4 m.
(a) ` 300 (b) ` 330 When the shortest person stands on a box of height
(c) ` 360 (d) ` 420 30 cm, he is taller than the middle person by the same
11. A father is now three times as old his son. Five years amount by which he is shorter than the tallest
back, he was four times as old as his son. The present person. Find the height of the shortest person.
age of the son (in years) is (a) 380 cm (b) 160 cm
(a) 12 (b) 15 (c) 18 (d) 20 (c) 540 cm (d) Cannot be determined
84 Study Package for NTSE

21. If ‘L’ is less than ‘M’ which is less than ‘N’, then it 29. Given that LOT ÷ TO = 8. Find out which of the
follows that following does not stand for TOL to fulfill the above
(a) ‘L’ is not equal to ‘M’ which is less than ‘N’ equation.
(b) ‘L’ is equal to ‘M’ which is greater than ‘N’ (a) 3, 2 and 8 (b) 5, 7 and 8 (c) 8, 5 and 7 (d) 7, 3 and 8
(c) ‘L’ is less than ‘M’ which is greater than ‘N’
(d) ‘L’ is equal to ‘M’ which is less than ‘N’ 30. Given the following subtraction problem, find out which
of the following numbers does not stand for LENS?
22. The teacher gave the following puzzle to students :
L E N
‘A container holds the equivalent of 30 glasses of
water. If the tap on it is turned with a uniform speed, –E N S
it takes 10 s to fill a glass of water. How long will the 2 2 2
container take to be emptied, if the tap is kept open?’
(a) 9753 (b) 8420 (c) 6420 (d) 7531
Four different answers came from the students. They
were 31. In a examination, a student scores 3 marks for every
(a) 4 min (b) 5 min correct answer and loses 1 mark for every wrong
(c) more than 4 min (d) more than 5 min answer. If he attempts all 60 questions and secures
23. Visiting the historical monuments, a tourist asked, 160 marks, the number of questions he attempted
‘‘How old is this monument’’? The guide replied, ‘‘The correctly is
monument’s exact age was a square number two years (a) 50 (b) 55 (c) 45 (d) 48
ago, and will be a cube number after two years’’. What 32. A monkey climbs 4m at the beginning of each
is the age of this historical monument?
(a) 343 (b) 216 (c) 66 (d) 123
hour and rest for a while when he slips back 2m
before he again starts climbing in the beginning
24. Today, the age of mother is thrice as that of her next hour. If he starts climbing as 6 am. At what
daughter. After 12 yr, the age of the mother will be time will he first touch the flag at 10m from the
twice that of her daughter. The present age of the ground?
daughter is (a) 10 am (b) 11 am
(a) 18 yr (b) 16 yr (c) 14 yr (d) 12 yr (c) 9 am (d) 9 : 30 am
25. I have a few sweets to be distributed. If I keep 2, 3 or 33. A newspaper has 6 sheets consisting of 24 pages in
4 in a pack, I am left with one sweet. If I keep 5 in a total. If page number 10 of that newspaper is missing
pack, I am left with none. What is the minimum then find the set of missing pages in that newspaper,
number of sweets I can pack and distribute? from the alternatives given below
(a) 54 (b) 25 (c) 37 (d) 65 (a) 8, 9, 16, 17 (b) 9, 10, 15, 16
(c) 11, 12, 13, 14 (d) 7, 8, 17, 18
26. A train running at 60 km/h or a car speeding at 100 m
per 6 s. Which runs faster? 34. If p, q, r, s and t are positive numbers, and it is given
(a) Train that
(b) Car
(c) Both will progress in the same speed p + q = r + s, q + s = 2p
(d) It is not possible to say s + t > p + q and r + s > p + t
27. In the following multiplication problem, find the then which of the following statement is true?
numerical values of T, F and H in that order (a) s > p > q > t > r (b) s > q > t > p > r
L T B D F (c) p > q > r > s > t (d) p > s > q > t > r

4 6 35. Study the figure given below representing a


3 4 T G F T particular number in coded manner.
2 3 D 4 H D
7 2 1
D 6 L F 6 L 8
(a) 2, 8 and 9 (b) 8, 3 and 9 0 4
(c) 6, 3 and 5 (d) 8, 4 and 7
for example, the number 7 6 1 coded by the following
28. Given the following addition problem, find the symbols
number NET in digits.
N O T
Based on the above information find the number
+ T O N E
coded for the following symbols.
9 0 0 0
(a) 6, 2 and 8 (b) 2, 6 and 8
(c) 8, 5 and 3 (d) 2, 1 and 6 (a) 4256 (b) 7210 (c) 4078 (d) 8406
Hints and Solutions
Elementary Level Questions
x 3x + 4x + 2 x
1. (b) Let the given fraction be . ∴ = 432
y 4
If the denominator is increased by 1, it becomes 1/2. 9 x = 432 × 4 ⇒ x = 192
x 1 ∴ Smallest number is x/2.
⇒ = ⇒ 2x − y − 1= 0 …(i)
y+1 2 192
⇒ = 96
2
If the numerator is increased by 9, it becomes 1.
x+ 9 7. (d) Let the unit’s place number be y and ten’s place number be x.
⇒ =1
y Then, number = 10 x + y
⇒ x− y+ 9=0 …(ii) ∴ 10 x + y − 18 = 10 y + x
On multiplying Eq. (ii) by 2 and then subtract from Eq. (i), we ⇒ x − y =2 …(i)
get Also, given that, x + y =10 …(ii)
∴ y = 19 On adding Eqs. (i) and (ii), we get
2. (d)Let the number of sections of the school be x. 2 x = 12
⇒ x = 6 and y = 4
Number of students in each section = 36
∴ The required number = 64
Total number of students in the school = 36 x
8. (b) Let the number be xyz, i.e., 100 x + 10 y + z
If y new students were added, then total number of students
Given, y = 0 and x + z = 12 …(i)
= 36 x + y
Also, (100 z+ 10 z+ x) − (100 x + 10 y + z) = 396
Number of section, if four new section were started = x + 4 z− x = 4 …(ii)
Number of students in each section = 30 On solving Eqs. (i) and (ii), we get
Now, total number of students = 30 ( x + 4 ) z = 8 and x = 4
∴ 36 x + y = 30 ( x + 4 ) ∴ The required number is 408.
6 x + y = 120 9. (b) Let the cost of each burger, pizza and a coke be b, p and c
Since, the question involves two unknowns and only one respectively.
equation, So, x cannot be determined uniquely. Given, 5b + 6 p + 7c = 178 …(i)
3. (c) Given, x + y = 50 …(i) 6b + 4 p + 2c = 124 …(ii)
x y On multiplying Eq. (i) by 2 and then adding, we get
and + = 20 = 3 x + 2 y = 120 …(ii)
2 3 ⇒ 16b + 16 p + 16 c = 480
On multiplying Eq. (i) by 2 and then subtracting from Eq. (ii), ⇒ b + p + c = ` 30
we get the values of x = ` 20 and y = ` 30. 10. (b) Let there be x members in the team excluding the captain.
1
4. (d) Rekha’s age, 3 yr ago = 12 yr Captain’s share = of the cake
3
∴ Rekha’s present age = 15 yr 2
Rekha’s age, after 10 yr = 15 + 10 = 25 Other’s share = of the cake
3
∴ Father’s age, after 10 yr = 2 (25) = 50 2
The share of each member of the team =
∴ Father’s present age = 50 − 10 = 40 yr 3x
40
∴ Rajesh’s present age = = 8 yr
2   =
2 1
5 Given,
 3x 3
5. (d) Let Nikhil and Yogesh’s money be N and Y, respectively. ⇒ x=4
N Y
Given, = ⇒ 3N = 2Y
4 6 ∴ Total number of members = 4 members + captain
2Y = 5 members
Also, N + Y = 600 ⇒ + Y = 600
3 11. (c) 7th 9th
⇒ 5Y = 1800 Seeta
⇒ Y = ` 360 and N = ` 240 Leena
∴The difference in the amounts is ` 120.
11th
6. (a) Given, the sum of the three numbers is 432.
Seeta
Let the second number be x, then
3x x Leena is 9th from the right.
+ x + = 432
4 2 ∴Total number of girls = 10 + 9 = 19
86 Study Package for NTSE

12. (b) After exchanging position with Kamal, Appu becomes 6th 17. (b) Since, B and D are twins, therefore, D is also 3 yr younger to A,
from left and 7th from right, which means there are 22 children in and A is 3 yr younger to C. Therefore, C is older than D by 3 + 3, i.e.,
the row. After exchanging positions with Appu, Kamal will be 4th 6 yr.
from right and 19th from left. 18. (b) Let the cost of each pen, sharpener and an eraser be p, s and
14 boys Mohan 14 boys e respectively.
13. (d) → 15th ←
4p + 8s + 9 e = ` 136 …(i)
Total number of boys in the row = 14 + 1 + 14 = 29 and 3 p + 6s + 7e = ` 104 …(ii)
14. (b) In 12 h, it will strike = 12 × 13/ 2 = 78 On multiplying Eq. (i) by 3 and Eq. (ii) by 4, we get
In 24 h, it will strike = 78 × 2 = 156 times 12 p + 24 s + 27e = ` 408
10 girls 10 girls 12 p + 24 s + 28 e = ` 416
15. (c) ← Manisha → On solving both the equations, we get
←11th → e =` 8
10 + 1 + 10 = 21 19. (c) Let the cost of each mango be x and cost of each orange be y.
16. (c) Let the third number be x. Given that, 5 x = 20 y ⇒ x = 4 y
∴Second number = x + 4 1 45
Also, 2 x + 10 y = 22 ⇒18 y =
Arithmetic mean of second and third numbers = (x + x + 4) / 2 2 2
= (2x + 4) / 2 = x + 2 45 1
∴ y= × = ` 125
.
∴ First number = x + 2 2 18
So, x + 2 + x + 4 + x = 69 or 3 x + 6 = 69 20. (b) Let the number of boys be b.
3 x = 69 − 6 Number of girls be g.
x = 63/ 3 ⇒ x = 21 Number of students who did not join the course
=  1 −  +
∴ First number = 21 + 2 = 23 b 7 g  3 1
 1 −  = (b + g ) ×
Second number = 21 + 4 = 25 2  8 4  4 16

Third number = 21 So, the fraction of total number of students who didn’t join the
1
course =
16

High Skill Questions


1. (c) Clearly, out of every 15 persons including caption, which is 7. (d) Let total number of sweets be x.
equal to 16. x x
Then, − =4
1200 140 175
∴Number of captains = = 75
16 = 5 x − 4 x = 4 × 700
2. (b) Clearly, total number of shakehands ∴ x = 2800
= (9 + 8 + 7 + 6 + 5 + 4 + 3 + 2 + 1) = 45 8. (b) Clearly, number of ways of arranging 5 books
3. (b) Suppose the boys got x sums right and 2 x sums wrong. = 5! = 5 × 4 × 3 × 2 × 1 = 120
Then, x + 2 x = 48 So, total time taken = 120 min = 2 h
⇒ 3 x = 48 ⇒ x = 16 Cats Mice Days
9. (d)
4. (b) Since, each pole at the corner of the plot is comman to its two 100 100 100
4 4 x
sides, so we have
100 : 4 
Total number of poles needed  x : 100
4 : 100
= 27 × 4 − 4
⇒ 100 × 4 × 100 = 4 × 100 × x
= 108 − 4 = 104
100 × 4 × 100
5. (c) Number of alterations required in 1 shirt ⇒ x= = 100
4 × 100
=  + +  =
2 3 4 133
∴ x = 100 days
 3 4 5 60
10. (b) Let the value of the holiday be x.
∴Numbers of alterations required in 60 shirts
133 Then, pay for seven weeks work = ` 300 + x
= × 60 = 133 ` 300 + x
60 So, × 4 = ` 30 + x
7
6. (d) Clearly, seven strikes of clock have 6 intervals while 10 strikes
have 9 intervals. ⇒ ` 1200 + 4 x = ` 210 + 7 x
7 1 ⇒ 3 x = ` 990
∴ Required time = × 9 = 10 s
6 2 ∴ x = ` 330
Arithmetical Reasoning 87
11. (b) Let son’s present age be x yr. a + 8 = 2 (2 s + 16) + 10 …(ii)
Then, father’s age = 3x ⇒ 6s + 8 = 4 s + 34
5 yr ago, father’s age = (3 x − 5) yr ⇒ 2 s = 26 ⇒ s = 13
and son’s age = ( x − 5) yr and a = 86
So, 3 x − 5 = 4( x − 5) ⇒ 3 x − 5 = 4 x − 20 ∴When the twins were born Atul’s age was 86 − 13 = 73 yr.
∴ x = 15 18. (b) Let the number of monkeys on T1 be x and the number of
12. (b) Let circles F and E represent people who read French and monkeys on T2 be y.
English respectively. Now, x − 2 = y +2
F E ⇒ x− y = 4 …(i)
Further, x +2 =2 ( y − 2)
P Q R ⇒ x = 2y − 6 …(ii)
On solving Eqs. (i) and (ii), we get
3
x = 14 and y = 10
(P + Q + R) + 3 = 15 19. (c) 1 woman = 3 boys
or P + Q + R = 12 ...(i)
1 man = 1woman + 1boy
Also, P+Q =7 …(ii)
1 man = 3 boys + 1boy
and Q+R = 8 …(iii)
1 man = 4 boys
On adding, Eqs. (ii) and (iii), we get
7 men : 11 women : 10 boys
P + 2Q + R = 15 …(iv)
⇒ (7 × 4) boys : (11 × 3) boys : 10 boys
Subtracting Eq. (i) from Eq. (iv), we get
⇒ 28 : 33 : 10
Q=3 1420
Hence, each part of the ratio = = 20
∴Number of people who read French and English both = 3 71
13. (c) Let the son’s age be x. Share of 11 women = 33 × 20 = ` 660
Father’s age = 3 x + 6 Share of 10 boys = 10 × 20 = ` 200
Son’s age, after 4 yr = x + 4 Share of the shares of 11 women and 20 boys
Father’s age, after 4 yr = 3 x + 6 + 4 = 3 x + 10 = 660 + 200 = 860
Given that, after 4 yr, the father’s age would be 12 more than
20. (b) Let heights of the shortest, middle and the tallest persons be
2 times the son’s age.
x, y and z cm.
⇒ 3 x + 10 − 2 ( x − 4) = 12
x + y + z = 540 cm …(i)
x = 12 − 2 = 10
Also, ( z + 30) − y = x − ( z + 30)
⇒ Father’s present age = 3 x + 6 = 3 (10) + 6 = 36 yr
⇒ 2 z − ( y + x) = − 60 …(ii)
14. (a) Let the amount with B is ` b. On adding Eqs. (i) and (ii), we get
So, the amount with A is 30 + b and the amount with C is 50 + b. ⇒ 3 z = 480
According to the question, ∴ z = 160 cm
b + 30 + b + 50 + b = 320, b = 80 So, height of the shortest person is 160 cm.
So, ` 110, ` 80 and ` 130 are distributed among A, B and C,
21. (a) L<M<N
respectively.
⇒ L ≠ M and M < N
15. (d) Let the width of rectangle be x and its length be ( x + 10).
22. (b) A plain mathematical calculation gives us the answer– five
Perimeter = 2 (l + b ) minutes.
⇒ 2 ( x + 10 + x) = 84 Time required = 30 × 10 = 300 s
2 x + 20 + 2 x = 84 ⇒ x = 16, x + 10 = 26 Time in minutes =
300
= 5 min
∴ Length of the rectangle = 26 cm 60

16. (a) Let the speed of Pawan and Sachin be P and S respectively. 23. (d) Take exact age as 66 yr.
66 − 2 = 64,which is square of 8 but 66 + 2 = 68. It is not cube of
5P = 4 + 8S …(i)
any number.
7S = 2 + 3 P …(ii) Then, take 343, 216 for exact age but these figures do not satisfy
On solving Eqs. (i) and (ii), we get the two criteria. Then, take 123 as exact age, it will satisfy the two
criteria.
S = 2 and P = 4
∴ Pawan walks 4 m/h. 24. (d) Let the age of daughter be x.
∴ Age of mother = 3x
17. (c) Let the age of Atul be a and that of each of the twins be s.
12 yr later, Daughter = x + 12
a = 3 × (2 s ) + 8 …(i)
88 Study Package for NTSE

Mother = 3 x + 12 31. (b) Let the number of correct questions = x


2 ( x + 12 ) = 3 x + 12 and the number of incorrect questions = y
2 x + 24 = 3 x + 12 Now, according to the question,
2 x − 3 x = − 24 + 12 x + y = 60 …(i)
− x = − 12, x = 12 3 x − y = 160 …(ii)
On adding Eqs (i) and (ii), we get
25. (b) LCM of 2, 3 and 4 = 24, 24 + 1 = 25
4 x = 220
26. (c) 60 km/h = 60000 m/h ∴ x = 55
= 60000 m/3600 s So, the student attempted 55 question correctly.
= 600 m/36 s = 100 m/6 s 32. (a) The monkey actually climbs in 1h
27. (b) 5 8 1 2 3 = 4 − 2 = 2m
4 6 So, in 3 h he climbs 3 × 2 = 6 m
3 4 8 7 3 8 Now, in next hour he climbs 4 m and he will reach at the top of
2 3 2 4 9 2 the flag.
2 6 7 3 6 5 8 So, the required time = 6 + 4 = 10 am
∴T= 8, F = 3 and H = 9 33. (b) Group of pages are —
6 3 8 (1, 2, 23, 24)
28. (a)
+ 8 3 6 2 (3, 4, 21, 22)
9 0 0 0 (5, 6, 19, 20)
(7, 8, 17, 18)
∴B = 6, E = 2 and T = 8
3×2 × 8 (9, 10, 15, 16)
29. (c) From option (a), =8 (11, 12, 13, 14)
3×2
5×7 × 8 So, the missing pages are (9, 10, 15, 16).
From option (b), =8
5×7 34. (a) Given, p+ q = r + s …(i)
8× 5×7 q + s = 2P …(ii)
From option (c), =7
8×5 s + t > p+ q …(iii)
7 ×3×8 r + s> p+ t …(iv)
From option (d), =8
7×3 From Eqs. (i) and (iv), we get
p+ q < p+ t
Hence, option (c) does not stand for fulfill the equation.
∴ q>t
30. (b) From option (a),
From Eqs. (i) and (iii), we get
9 7 5 s+t>r+ s
– 7 5 3 t>r
2 2 2
From Eqs. (iv), we get
From option (b), r + s> p+ t
8 4 2 Q t>r
– 4 2 0 ∴ s> p
4 2 2 From Eqs. (ii), we get
From option (c), q + s = p+ p
6 4 2 Q s> p
– 4 2 0 ∴ q < p or p > q
2 2 2 So, we get s > p > q > t > r
From option (d), 35. (d) Given,
7 5 3
– 5 3 1 (7 + 1) 4 0 (2 + 4)
2 2 2 ∴ 8406
Hence, option (b) does not stand for LENS. ∴ =8406
Chapter

13
Mathematical
Operations
In this section, questions are related to simple mathematical operations.
There are four fundamental operations
(i) Addition ( + ), (ii) Subtraction (–), (iii) Multiplication ( ×) and (iv) Division ( ÷ ).
There are some additional symbols such as; less than (<), greater than (>), equal to (=) and not equal to (≠) etc., also
used in problems.
In this type of questions, the candidate has to make a substitution of real sign and solve the equation accordingly.
For solving mathematical expressions, proceed according to the rule of ‘BODMAS’ i.e. Brackets, Of, Division,
Multiplication, Addition and Subtraction.

Type I Questions Based on Substitution of Mathematical Sign


In this type of questions, you are provided with substitutes for various mathematical symbols or numbers. Followed
by a question involving calculations of an expression or choosing the correct/incorrect equations. The candidate is
required to put in the real signs or numerals in the given equation and then solve the questions.

Example 1 If L stands for +, M stands for –, N stands for × Example 3 Find the value of (10 α 8) γ (7 β 5).
and P stands for ÷, then 14 N 10 L 42 P 2 M 8 is equal to 5 7
(a) (b)
(a) 153 (b) 216 (c) 248 (d) 251 4 4
Solution (a) Using the proper signs, we get 9 3
(c) (d)
= 14 × 10 + 42 ÷ 2 − 8 = 14 × 10 + 21 − 8 4 4
= 140 + 21 − 8 = 161 − 8 = 153 Solution (a) (10 α 8 ) γ (7 β 5) =  10 + 8  γ  7 − 5  = 9 γ 1
 2   2 
Example 2 If + means ÷, – means ×, ÷ means + and
 9 + 1 5
× means –, then the value of 48 × 12 + 6 ÷ 9 + 3 − 3 = =
when simplified, is  9 − 1 4
(a) 55 (b) 42 (c) 18 (d) 24 Example 4 Find the value of [1 β (4 γ 5)] θ (8 α 2).
Solution (a) Using proper signs, we get (a) 0 (b) 2 (c) 10 (d) 40
= 48 − 12 ÷ 6 + 9 ÷ 3 × 3 = 48 − 2 + 3 × 3 = 48 − 2 + 9 = 55
Solution (a) [1 β (4 γ 5)] θ ( 8 α 2)
Directions (Ex. 3 and 4)   4 + 5   8 + 2
= 1 β   θ
A+B A−B   4 − 5    2 
If A α B = , Aβ B = ,
2 2 1 − ( − 9 )  5−5
A+B A−B =  θ5= 5θ5= 5 + 5 = 0
AγB= and A θ B =  2 
A−B A+B
90 Study Package for NTSE

Type II Based on Correction of Expression by Interchange of Sign


In this type of questions, an equation becomes incorrect due to the interchange of two signs. One of the four
alternatives under specifies the interchange of sign in the equations which when made will make the equation
correct, find the correct alternative.
Example 5 16 − 8 ÷ 4 + 5 × 3 = 13
(a) ÷ and × (b) – and ÷ (c) ÷ and + (d) – and ÷
Solution (b) On interchanging – and ÷ , we get
= 16 ÷ 8 − 4 + 5 × 3 = 13

Type III Based on Some Specific Rule


In this type of questions, three statements of numbers following same rules are given. Find the rule and according,
find the value of the number.
Example 6 If 84 ⊕ 72 = 45 and 63 ⊕ 41 = 33 , then 94 ⊕ 82 is equal to
(a) 45 (b) 59 (c) 56 (d) 65
Solution (c) The rule is difference of the digits of the number
84 ⊕ 72 = ( 8 − 4 ) (7 − 2) = 45
63 ⊕ 41 = ( 6 − 3 ) (4 − 1) = 33
∴ 94 ⊕ 82 = ( 9 − 4 ) ( 8 − 2) = 56

Target Exercise
Elementary Level Questions
1. If + means ×, – means ÷, × means + and ÷ means –, 7. In the problem below instead of conventional +, –, ×
then 10 × 18 − 9 + 3 ÷ 1 is equal to and ÷ symbols, some others are used. However, all
(a) 16 (b) 15 (c) 17 (d) 18 lead to the same final answer. Correct final answer
after assessing the new symbols.
2. If + means ×, – means ÷, × means – and ÷ means +,
then the value of 9 + 8 ÷ 8 − 4 × 9 is (i) 10 * 6 * 7 = x
(a) 56 (b) 65 (c) 66 (d) 55 (ii) 3 * 10 * 10 = x
(iii) 8 * 5 * 10 = x
3. If – means ×, × means +, + means ÷ and ÷ means –, (iv) 4 * 8 * 11 = x
then the value of 14 − 10 × 4 ÷ 16 + 8 is (a) 352 (b) – 15 (c) 420 (d) 23
(a) 241 (b) 124 (c) 142 (d) 412

4. If + means ×, – means ÷, × means – and ÷ means +, Directions (Q. Nos. 8-10) In each of the following
questions an equation becomes incorrect due to the
then the value of 32 ÷ 8 − 4 × 12 + 4 is
interchange of two signs. One of the four alternatives under it
(a) – 14 (b) – 41
(c) – 40 (d) – 12 specifies the interchange of signs in the equation, which when
made will make the equation correct. Find the correct
5. If A means ‘+’, B means ‘–’, C means ‘×’ and D means alternative.
‘÷’, then 18 C 14 A 6 B 16 D 4 is equal to
8. 12 ÷ 2 − 6 × 3 + 8 = 16
(a) 63 (b) 288
(a) ÷ and + (b) – and +
(c) 254 (d) 1208
(c) × and + (d) ÷ and ×
6. If A denotes ÷, B denotes +, C denotes – and D
9. 9 + 5 ÷ 4 × 3 − 6 = 12
denotes ×, then which of the following is not true?
(a) + and × (b) ÷ and ×
(a) 24 A 6 B 10 = 5 D 6 C 16
(c) ÷ and – (d) + and –
(b) 60 − 4 D 6 = 4 B 2 D 6
(c) 30 D 4 A 12 = 30 A 12 D 4 10. 16 − 8 ÷ 4 + 5 × 2 = 8
(d) 108 C 72 = 78 C 42 (a) ÷ and × (b) – and ÷ (c) ÷ and + (d) – and ×
Mathematical Operations 91
Directions (Q. Nos. 11-14) In the questions given below, 16. If sign ‘–’ means ‘+’, sign ‘+’ means ‘–’, sign ‘×’
which one of the following four interchanges in signs and/or means ‘÷’ and sign ‘÷’ means ‘×’, then simplify
numbers would make the given equation correct. 7 − 8 × 5 ÷ 6 + 4.
11. 3 + 5 − 2 = 4 (a) 3 (b) 12
3 48
(a) + and –, 2 and 3 (b) + and –, 2 and 5 (c) 12 (d)
(c) + and –, 3 and 5 (d) None of these 5 5

12. 6 × 4 + 2 = 16 17. If A denotes +, B denotes ×, C denotes – and D denotes


(a) + and ×, 2 and 4 (b) + and ×, 2 and 6 ÷, then what is the value of
(c) + and ×, 4 and 6 (d) None of these 5 B 9 A 7 C 9 D 3 A 6?
13. 4 × 6 − 2 = 14 (a) 55 (b) 54 (c) 45 (d) 50
(a) × to ÷, 2 and 4 (b) – to ÷, 2 and 6 18. Which of the following meanings of the arithmatical
(c) – to +, 2 and 6 (d) × to +, 4 and 6 signs will yield value ‘Zero’ for the expression given
14. (6 ÷ 2) × 3 = 0 below?
(a) ÷ and ×, 2 and 3 (b) × to –, 2 and 6 200 × 100 + 300 × 200 − 10 ÷ 2 + 40
(c) ÷ and ×, 2 and 6 (d) × to –, 2 and 3 (a) + means –, – means ×, × means ÷ and ÷ means +
(b) + means –, – means ÷, × means + and ÷ means ×
15. If × stands for +, < stands for –, > stands for ×,
(c) + means ×, – means –, × means ÷ and ÷ means +
+ stands for ÷, – stands for =, ÷ stands for >, and =
stands for <, then which of the given equations is (d) + means ÷, – means +, × means – and ÷ means ×
correct? 19. If 20 – 10 means 200, 8 ÷ 4 means 12, 6 × 2 means 4
(a) 8 < 4 × 3 − 3 × 2 × 1 and 12 + 3 means 4, then 100 – 10 × 1000 ÷ 100 + 100
(b) 8 > 4 < 3 − 3 > 2 < 1 × 10 is equal to
(c) 8 + 4 < 3 ÷ 3 < 2 < 1 (a) 1090 (b) 0 (c) 1900 (d) 20
(d) 8 + 4 × 3 = 3 > 2 × 1

High Skill Questions


1. If ( a * b) stands for ( a + b)2 and ( a ⊕ b) stands for 6. Find θ [ λ (1, 3), θ (1, 3)].
( a − b)2 , then the value of ( a * b) + ( a ⊕ b) is (a) 5 (b) 2
(a) 2 a2 + b 2 (b) a2 + 2 b 2 (c) 4 (d) 0
(c) 2 (a2 + b 2 ) (d) None of these  λ { − ( − 3 + 2), ( − 2 + 3)}
7. What is the value of  ?
2. If x * y = x − y
2
when x ≥ y and ( y − x) when y > x,
2 2
 − ( − 2 + 1) 
then find the value of 6 * ( 4 * 3). (a) 1 (b) 0
(a) 2 (b) 1 (c) 2 (d) 3
(c) 3 (d) 0
8. Find θ [0, λ {0, θ (0, 1)}].
3. If @ is –, * is +, × is –, + is × and % is ÷, then the value (a)
1
(b)
1
(c)
1
(d) 2
of [{(5 ⊕ 2) @ 1} % 3 ] * 2 is 2 3 4
(a) 30 (b) 28
(c) 32 (d) 5 Directions (Q. Nos. 9-12) Read the following information
carefully and answer the questions that follow.
4. If 16 − 2 = 2, 9 − 3 = 0, 81 − 1 = 8, then what is
64 − 4 = ? For two numbers a and b, P( a, b) = a − b,
(a) 4 (b) 2 Q ( a, b) = a × b, R ( a, b) = a ÷ b, S ( a, b) = a + b, if a
(c) 6 (d) 8 = 1, b = 2 and c = 3.

5. If 7 * 2 = 81, then what is the value of 8 * 5? 9. The value of P [a, Q {b, R (b, c)}], is
(a) 144 (b) 89 (a) > 0 (b) < 0
(c) 169 (d) 121 (c) 1 (d) 0

10. Solve P [Q {R ((c, a), S (a, b)), S {Q (a, c), P (c, a)}]
Directions (Q. Nos. 6-8) Read the following information
(a) 2 (b) – 4
carefully and answer the questions that follow.
(c) 4 (d) 3
λ ( a, b ) = ( a − b )2 , if a≥b
=a + b , if b >a
11. Solve R [Q {S (x, y), P (x, y), P (Q {3, S (x, y), S (x, y)}]
a+b (a) 0 (b) xy
θ ( a, b ) = , if a + b ≥ 0 x+ y x− y
2 (c) (d)
= − ( a + b ), if a + b > 0 2 2
92 Study Package for NTSE

12. Which value is the greatest? (a) 7 ← 43 ↑ 6 ↓ 1 4


(b) 3 ↓ 6 ↑ 2 → 3 ← 6 5
I. S (P (a, b), P (a, b)))
(c) 5 → 7 ← 3 ↑ 2 5
II. Q (S (a, b), R (a, b))) (d) 2 ↓ 5 ← 6 → 2 6
III. P (S (a, b), Q (a, b)))
(a) I (b) II 16. Figure below represents a balance, which symbol
(c) III (d) Cannot be determined replaces (?).

Directions (Q. Nos.13 and 14) Read the following


information carefully and answer the questions that follow.
If a * b = 1 when a × b is even and a * b = a × b otherwise,
then
13. What is the value of 1 * (2 * (1 * (2 * ((1 * 2))))
(a) 1 (b) 2 (c) 3 (d) 4 ?
14. If a and b are natural numbers, then the value of
(3 * ( a * (2 * ( b * 4)))) is (a) oO (b) o
(a) 3a, if a is odd (b) 3, if a is even (c) ∆ (d) O
(c) 3a, if a is even (d) Cannot be determined
17. If ∆ + ∆ = 10, ∆ + O + O = 10, ∆ + O − o × ∆ = 5
15. If → stands for ‘addition’, ← stands for ‘subtraction’,
↑ stands for ‘division’, ↓ stands for ‘multiplication’ and Then, the value of o will be
(a) 0.5 (b) 5
 stands for ‘equal to’, then which of the following
(c) 2.5 (d) 7.5
alternatives is correct?

Hints and Solutions


Elementary Level Questions
1. (b) Using proper symbols, we get 11. (c) By interchanging (+ and –, 3 and 5)
= 10 + 18 ÷ 9 × 3 − 1 5−3+2 =4
1
= 10 + 18 × × 3 − 1= 10 + 6 − 1 = 15 12. (c) By interchanging (+ and ×, 4 and 6)
9
4 + 6 × 2 = 16
2. (b) Using proper symbols, we get
= 9 × 8 + 8 ÷ 4 − 9 = 72 + 2 − 9 = 65
13. (c) By interchanging (– to +, 2 and 6)
4 × 2 + 6 = 14
3. (c) Using proper symbols, we get
= 14 × 10 + 4 − 16 ÷ 8 = 140 + 4 − 2 = 142 14. (d) By interchanging ( × to –, 2 and 3)
4. (a) Using proper operations, we get 6 ÷2 − 3 = 0
= 32 + 8 ÷ 4 − 12 × 4 = 32 + 2 − 12 × 4 15. (d) 8 + 4 × 3 = 3 > 2 × 1
= 34 + (− 48) = − 14 By changing the signs 8 ÷ 4 + 3 < 3 × 2 + 1
5. (c) Using proper signs, we get = 18 × 14 + 6 − 16 ÷ 4 = 2 + 3< 6 + 1= 5< 7
= 252 + 6 − 4 = 254 16. (c) The given expression becomes after changing signs
6. (b) 60 − 4 × 6 = 36, while 4 + 2 × 6 = 16 48 3 3
= 7 + 8 ÷ 5 × 6 − 4= 7 + − 4 = 3 + 9 = 12
5 5 5
7. (d) If we replace all the stars by the sign of addition ‘+’, we get a
common figure.
17. (a) After replacing the symbols, we get
(i) 10 + 6 + 7 = 23 (ii) 3 + 10 + 10 = 23 = 5 × 9 + 7 − 9 ÷ 3 + 6 = 45 + 7 − 3 + 6 = 55
(iii) 8 + 5 + 10 = 23 (iv) 4 + 8 + 11 = 23 18. (b) Using the information given in option (b), we get
8. (b) By interchanging (– and +) 200 + 100 − 300 + 200 ÷ 10 × 2 − 40
12 ÷ 2 + 6 × 3 − 8 = 16 = 300 − 300 + (20 × 2 ) − 40 = 0(Zero)
9. (c) By interchanging (÷ and –) 19. (b) Here, ‘–’ ⇒ ‘×’, ‘÷’ ⇒ ‘+’, ‘×’ ⇒ ‘–’ and ‘+’ ⇒ ‘÷’
9 + 5 − 4 × 3 ÷ 6 = 12 Hence, 100 − 10 × 1000 ÷ 1000 + 100 × 10 can be written as
10. (b) By interchanging (– and ÷ ) 100 × 10 − 1000 + 1000 ÷ 100 − 10
16 ÷ 8 − 4 + 5 × 2 = 8 = 1000 − 1000 + 10 − 10 = 0
Mathematical Operations 93
High Skill Questions
1. (c) (a * b) + (a ⊕ b) = (a + b)2 + (a − b)2 III. P (S (a, b), Q (a, b)) = P (a + b, a ⋅ b)
= a + b + 2 ab + a + b − 2 ab = 2 (a + b )
2 2 2 2 2 2 = a + b – ab
= 1 + 2 − 1⋅ 2 = 1
2. (b) 6 * (4 * 3) = 6 * (42 − 32 ) = 6 * (7 ) = (7 − 6)2 = 1
13. (a) 1 * (2 * (1 * (2 * ((1 * 2 ))))
3. (d) [{(5 × 2 ) − 1} ÷ 3] + 2 = (10 − 1) ÷ 3 + 2 = 1 * (2 * (1 * (2 * 1)[1 * 2 = 1 as, 1 × 2 is even.
= (9 ÷ 3) + 2 = 3 + 2 = 5 = 1 * (2 * (1 * 1))
4. (a) As, 16 − 2 = 4 − 2 = 2 = 1 * (2 * 1) (as 1 × 1 * is odd, so 1 * 2 1× = 1)
9−3=3−3=0 = 1* 1 = 1
and 81 − 1 = 9 − 1 = 8 14. (a) (3 * (a * (2 * (b * 4)))) = (3 * (a * (2 * 1)))
Similarly, 64 − 4 = 8 − 4 = 4
= (3 * (a * 1))
5. (c) Rule is a * b = (a + b)2 Case When a is odd.
So, 8 * 5 = (8 + 5)2 = 169 = 3* a ( a * 1 = a × 1 = a)
4 + (−4) 4 − 4 0 = 3a (Q a is odd)
6. (d) θ ((1 + 3), − (1 + 3 )) = θ (4, − 4) = = = =0
2 2 2 15. (d) Using the proper notations in (d), we get the statement as
 λ {− (− 3 + 2 ), (− 2 + 3)}  λ (1, 1) (1 − 1) 2 2 × 5 − 6 + 2 = 6 or 10 − 6 + 2 = 6 or 6 = 6, which is true.
7. (b) 
− (− 2 + 1) = 1 = 1
=0
  16. (d) Given,
 0 + 1 
8. (c) θ  0, λ 0, 
=
... (i)
 
   2  
=
1 ... (ii)
0+
  0, 1   = θ  0, 1  = 2 = 1
= θ 0, λ  
  2    2  2 4
= ? ... (iii)

9. (b) P [a, Q {b, R (b, c)}] = P  a, Q b, 


2 From Eqs. (i) and (ii), we get
 3
=
= P  a, 2 ×  = P  1,
2 4 4 1
 = 1− = − < 0
 3  3 3 3 =
10. (c) P [Q {R ((c, a), S (a, b)}, S{Q (a, c), P (c, a)}]
∴ =
= P (Q (R (3, 1), S (1, 2), S (Q (1, 3), P (3, 1)))
= P (Q (3, 3), S (3, 2)))= P (9, 5) = 4 Now, from Eq. (iii), we get
11. (d) R [Q (x + y, x – y)], P (Q (3, S (x, y)), S (x, y)))) =
R (Q (x + y, x – y), P (Q (3, x + y), S (x + y)))
R (Q (x + y, x – y)), P (3x + 3y), (x + y) 17. (a) Given, 2 ∆ = 10
x2 − y2 ( x − y) ( x + y) x − y
R (x 2 – y 2 , 2x + y) = = ∴ ∆=5
2x + 2y 2 ( x + y) 2
Now, ∆ + O + O = 10
12. (b) I. S [P (a, b), P (a, b)] = S (a – b, a – b) 5 + O + O = 10, 5 + 2 O = 10∴ O = 2.5
= a – b + a – b = 2a – 2b = – 2 Now, 5 + 2.5 − o × 5 = 5
7.5 − 5 o = 5
Q (S (a, b), R (a, b)) = Q  a + b, 
a
II.
 b 5 o = 2.5
(a + b) ⋅ a 3 × 1 3 ∴ o = 0.5
= = =
b 2 2
Chapter

14
Sitting Arrangement
and Puzzles
Sitting Arrangement
Sitting arrangement is a sequential arrangement of objects/persons on the basis of predefined conditions.
In this type of questions, some conditions are given, on the basis of which candidates are required to arrange
objects/persons, either in a row or in a circular order or in any other geometrical shape. While making
arrangements, it should be noted that all the conditions given are complied with.

Puzzles
Puzzles are the mind boggler and head hunters, which checks the mental ability of candidate to comprehend and
analyse the given information presented in twisted and unsegragated manner and transform them into sequential
and logical manner to know the hidden ideas and information behind it.
The puzzles can generally be classified into the following
l Simple problem of categorisation l Comparison problems l Arrangement problems
l Blood relations and professions l Conditional selection l Miscellaneous problems

Example 1 Study the following information carefully and answer the questions given below.
Six friends A, B, C, D, E and F are sitting in a row facing towards North. C is sitting between A and E. D is not at the
extreme end. B is sitting at immediate right of E. F is not at the right end but D is sitting at 3rd left of E.
1. How many persons are there to the right of D?
(a) One (b) Two
(c) Three (d) Four
2. Which of the following is sitting to the left of D?
(a) F (b) C
(c) E (d) A
3. Who is at the immediate left of C?
(a) A (b) E
(c) Either E or A (d) Cannot be determined
4. Who is at the right end?
(a) A (b) B
(c) E (d) Cannot be determined
Sitting Arrangement and Puzzles 95
Solutions According to the given information, the sitting arrangement of six friends is as given below
N

F D A C E B
W E

Facing North S

1. (d) It is clear from the diagram that there are four persons to the right of D—A, C, E and B.
2. (a) It is clear from the diagram that F is sitting to the left of D.
3. (a) It is clear from the diagram that A is at the immediate left of C.
4. (b) It is clear from the diagram that B is at the right end.

Example 2 Read the following information carefully and answer the questions that follow.
There are six cities A, B, C, D, E and F.
A is not a hill station. B and E are not historical places. D is not industrial city.
A and D are not historical places. A and B are not alike.
1. Which two cities are industrial centers?
(a) A and B (b) E and F (c) C and D (d) B and F
2. Which two cities are historical places?
(a) A and C (b) B and F (c) C and F (d) B and E
3. Which two cities are hill stations?
(a) A and B (b) C and A (c) B and D (d) A and F
4. Which city is a hill station and an industrial center but not historical place?
(a) E (b) F (c) A (d) B
5. Which two cities are neither historical places nor the industrial centers?
(a) A and B (b) D and E (c) F and C (d) B and D
Solution The given information can be summarised in tabular form as follow

A B C D E F
Historical × × ✓ × × ✓
Industrial ✓ × ✓ × ✓ ✓
Hill station × ✓ ✓ ✓ ✓ ✓

1. (b) 2. (c) 3. (c) 4. (a) 5. (d)

Target Exercise
Elementary Level Questions
1. There are 30 plants of Chiku, Guava, Sitafal and 2. Four girls (G1, G2, G3, G4) and three boys (B1, B2, B3)
Mango in a row. There is one pair of Mango plants after are to sit for dinner such that not two boys should sit
Chiku and Guava and Mango plants are followed by together nor two girls. If they are successively sitting,
one Chiku and one Sitafal plant and so on. If the row what is the position of B2 and G3?
begins with a plant of Chiku, then which of the (a) 5th and 6th
following will be last in the row? (b) 4th and 5th
(a) Guava (b) Mango (c) 3rd and 4th
(c) Chiku (d) Sitafal (d) 2nd and 3rd
96 Study Package for NTSE

3. Four friends were playing a game of cards sitting in a Directions (Q. Nos. 9-14) Read the following information
circle. Shankar was on the right of Ram and Gopal carefully and answer the questions given below.
was on the left of Arvind. Which of the following pairs Asha and Charu are good in Mathematics and Athletics.
were partners? Deepa and Asha are good in Athletics and Studies, Charu and
(a) Ram and Shankar Beena are good in General Knowledge and Mathematics.
(b) Gopal and Shankar Deepa, Beena and Ela are good in Studies and General
Knowledge. Ela and Deepa are good in Studies and Art.
(c) Ram and Arvind
(d) Gopal and Ram 9. Who is good in Studies, General Knowledge, Athletics
4. A, B, C, D, E and F are sitting around a round table. A and Art?
is between E and F, E is opposite to D and C is not in (a) Asha (b) Beena (c) Charu (d) Deepa
either of the neighbouring seats of E. Who is opposite 10. Who is good in Studies, General Knowledge and
to B? Mathematics?
(a) F (b) C (a) Asha (b) Beena (c) Charu (d) Ela
(c) D (d) None of these
11. Who is good in Studies, Mathematics and Athletics?
5. Seven men A, B, C, D, E, F and G are standing (a) Asha (b) Beena (c) Charu (d) Deepa
in a queue. Each one is wearing a cap of different
12. Who is good in Athletics, General Knowledge and
colours–Violet, Indigo, Blue, Green, Yellow, Orange
Mathematics?
and Red. D is able to see infront of him Green and
(a) Asha (b) Beena (c) Charu (d) Deepa
Blue but not Violet. F can see Violet and Yellow but
not Red. G can see caps of all the colours other than 13. Who is good in Studies, General Knowledge and Art
Orange. If E is wearing an Indigo coloured cap, then but not in Athletics?
the colour of the cap worn by F is (a) Asha (b) Beena (c) Charu (d) Ela
(a) Blue (b) Violet 14. Who is not good in only one area?
(c) Red (d) Orange (a) Deepa (b) Asha (c) Charu (d) Beena
6. Thirty six vehicles are parked in a parking lot in a
single row. After the first car, there is one scooters. Directions (Q. Nos. 15-19) Study the following information
After the second car, there are two scooters. After the carefully and answer the questions given below.
third car, there are three scooters and so on. Work out P, Q, R, S, T, V, W and Z are sitting around a circle facing at
the number of scooters in the second half of the row. the centre. R is fourth to the left of P, who is second to the
(a) 10 (b) 12 right of S. V is fourth to the right of S. Q is fourth to the left of
(c) 15 (d) 17 W, who is not an immediate neighbour of P or S. Z is not an
immediate neighbour of R.
7. A, B, C, D, E, F and G are sitting in a line facing the
East. C is immediate right of D. B is at one of the 15. Who is to the immediate right of V?
extreme ends and has E as his neighbour. G is (a) R (b) W
between E and F. D is sitting third from the South (c) Z (d) Data inadequate
end. Who are the persons sitting at the extreme ends? 16. Who is to the immediate right of R?
(a) A and E (b) A and B (a) T (b) S
(c) F and B (d) C and D (c) W (d) Data inadequate
8. Asha, Babita, Chinni, Deepa, Evita and Fatima are 17. In which of the following pairs is the first person
standing in a row. Babita is between, Fatima and sitting to the immediate right of the second person?
Deepa. Evita is between Asha and Chinni. Asha does (a) VW (b) RT (c) WR (d) ZP
not stand next to either Fatima or Deepa. Chinni does
not stand next to Deepa. Fatima is between which of 18. Who is third to the right of R?
the following pair of persons? (a) P (b) S
(c) Q (d) Data inadequate
(a) Chinni and Babita
(b) Babita and Deepa 19. Who is second to the left of Z?
(c) Evita and Chinni (a) Q (b) V
(d) Asha and Evita (c) S (d) W
Sitting Arrangement and Puzzles 97
High Skill Questions
Directions (Q. Nos. 1-5) Read the following information Directions (Q.Nos. 10-13) Read the following information
carefully and answer the questions given below. carefully and answer the given questions given below.
At a party A, B, C, D and E are sitting in a circle. The group P, Q, R, S, T, V and W are the seven members of a family.
comprises a professor, an industrialist and a businessman. There are three female members. Each of them has a different
The businessman is sitting in between the industrialist and his profession—Lawyer, Chartered Accountant (CA), Engineer,
wife D. A, the professor is married to E, who is the sister of B. Teacher, Doctor, Architect and Pharmacist. No lady is either
The industrialist is seated to the right of C. Both the ladies are Pharmacist or Chartered Accountant. Each of them has a
unemployed. different monthly income. The Chartered Accountant earns
1. What is A to B? the most. S, the Engineer earns less than V, the Doctor. R, the
Teacher, earns more than P and less than S. W’s wife earns the
(a) Brother (b) Uncle
least. T is an unmarried lady Lawyer and she earns less than
(c) Brother-in-law (d) Cannot be determined
P and more than only Q. The Pharmacist’s income is not the
2. A is sitting to the right of lowest.
(a) the industrialist (b) his wife
(c) D (d) Cannot be determined
10. Who earns the least?
(a) P (b) Q
3. Who is the industrialist? (c) P or Q (d) R
(a) D (b) A
(c) B (d) Cannot be determined
11. What is P’s profession?
(a) Pharmacist (b) Lawyer
4. Who is unmarried in the group? (c) Teacher (d) Cannot be determined
(a) Professor (b) Industrialist
12. How many members earn less than the Doctor?
(c) Businessman (d) Cannot be determined
(a) 2 (b) 3
5. Who among them must be graduate? (c) 4 (d) 5
(a) B (b) A 13. Which of the following represents the three female
(c) C (d) None of these
members of the family?
Directions (Q.Nos. 6-8) Study the following information (a) PTQ
carefully and answer the questions given below. (b) TRQ
(c) VTQ
There are eight persons namely S, T, U, V, W, X, Y and Z lives (d) Cannot be determined
on eight different floors from one to eight. Ground floor is
number one and one above that is number two and so on till Directions (Q.Nos. 14 and 15) Study the following
the topmost floor is number eight. X lives on odd number floor information carefully to answer the given questions.
but does not live on 3rd floor. Z lives immediate below X. More Seven people J, K, L, M, N, O and P have to attend a seminar
than two person lives between Z and Y. There are six person on seven different days of the same week starting from
lives between S and Y. V lives immediate above W, but live Monday and ending on Sunday, but not necessarily in the
below T. U does not live above X. W does not live immediate same order.
above Y. K has to attend a seminar on Wednesday. Only one person has
6. Who lives on floor number five? to attend a seminar between K and P. J attends a seminar
(a) U (b) S (c) Z (d) T immediately after P. The number of people who have to attend
a seminar before J is same as who have to attend a seminar
7. How many persons live between W and X?
after L. Only one person has to attend a seminar between L
(a) One (b) Three (c) Five (d) Two and M. O has to attend a seminar immediately after M.
8. Who lives immediately above V? 14. On which of the following days does N have to attend
(a) Z (b) T (c) Y (d) W a seminar?
9. P, Q, R, S, T and U are 6 members of family in which (a) Sunday (b) Thursday
there are two married couples. T, a teacher, is married (c) Saturday (d) Wednesday
to a doctor who is mother of R and U. Q, the lawyer, is 15. As per the given arrangement, four of the following
married to P. P has one son and one grandson. Of the five are alike in a certain way and so form a group.
two married ladies one is a housewife. There is also Which one of the following does not belong to the
one student and one male engineer in the family. group?
Which of the following is true about the (a) M-Friday
granddaughter of the family? (b) M-Wednesday
(a) She is a lawyer (b) She is an engineer (c) O-Sunday
(c) She is a student (d) She is a doctor (d) O-Thursday
98 Study Package for NTSE

Directions (Q. Nos. 16-20) Read the following information 20. Which of the following is the correct sequence of
and answer the questions that follow. organising plays?
(i) Six plays A, B, C, D, E and F are to be organised from Monday (a) A E C F B D (b) D F E C B A
to Saturday. i.e. From 5th to 10th one play each day. (c) B D E F C A (d) None of these
(ii) There are two plays between C and D and one play between
A and C. Directions (Q. Nos. 21-24) Read the following
(iii) There is one play between F and E is to be organised before information carefully and answer these questions.
F. A Blacksmith has five iron articles A, B, C, D and E each
(iv) B is to be organised before A, not necessarily immediately. having a different weight.
(v) The organisation does not start with B.
(i) A weights twice as much as B.
16. The organisation would start from which play? (ii) B weights four and half times as much as C.
(a) A (iii) C weights half as much as D.
(b) F (iv) D weights half as much as E.
(c) D (v) E weights less than A but more than C.
(d) Cannot be determined
21. Which of the following is the highest in weight?
17. On which date, the play E is to be organised? (a) C (b) B (c) A (d) D
(a) 5th
(b) 7th
22. E is lighter in weight than which of the other two
(c) 6th articles?
(d) Cannot be determined (a) A and C (b) D and C
(c) A and B (d) D and B
18. The organisation would end with which of the
following plays? 23. E is heavier than which of the following two articles?
(a) A (b) F (a) A and C (b) D and C
(c) D (d) Cannot be determined (c) D and B (d) A and B

19. On which day, the play B is organised? 24. Which of the following articles is the lightest in
(a) Tuesday (b) Friday weight?
(c) Thursday (d) None of these (a) C (b) B (c) A (d) D

Hints and Solutions


Elementary Level Questions
1. (d) Chiku - Guava - Mango - Mango - Chiku - Sitafal. This sequence 5. (c) On the basis of the information given in the question, the
is followed throughout, so the required 30th plant will be Sitafal. colours of cap of all the seven persons is given as under.
2. (b) G1 B1 G2 B2 G3 B3 G4 A – Green or Blue
1 2 3 4 5 6 7 B – Green or Blue
Arvind C – Yellow
3. (d)
D – Violet
E – Indigo
Ram Gopal
F – Red
G – Orange
Shankar 6. (c) In the second half of the row, there are 18 vehicles. Thus, the
Clearly, two partners are Gopal and Ram. number of scooters is 15.
4. (a) C 7. (b) The order in which they sit is given below
B D B E G F D C A
Hence, A and B are sitting at the extreme ends.
E F 8. (a) The order in which all the six women stands is given below
A Asha, Evita, Chinni, Fatima, Babita, Deepa.
F is opposite to B. Hence, Fatima is between Babita and Chinni.
Sitting Arrangement and Puzzles 99
Sol. (Q. Nos. 9-14) The given information can be summarised as Sol. (Q. Nos. 15-19) According to the given information, the sitting
follows arrangement as shown below
T
Math Athletics Studies GK Art
S R
Asha P P P O O
Charu P P O P O Q W
Deepa O P P P P
Beena P O P P O P V
Ela O O P P P Z

9. (d) Clearly, Deepa is good in Studies, GK, Athletics and Art. 15. (b) W sits to the immediate right of V.
10. (b) Beena is good in Studies, GK and Mathematics. 16. (a) T is to the immediate right of R.
11. (a) Asha is good in Studies, Mathematics and Athletics. 17. (d) In pair ZP, Z is sitting to the immediate right of P.
12. (c) Charu is good in Athletics, GK and Mathematics.
18. (c) Q is third to the right of R.
13. (d) Ela is good in Studies, GK and Art but not in Athletics.
19. (a) Q is sitting second to the left of Z.
14. (a) Deepa is not good in only one area i.e. Mathematics.

High Skill Questions


Sol. (Q. Nos. 1-5) 9. (c) According to the given information, arrangement is as
Professor A is married to E and E is the sister of B. Industrialist is follows
(–) Couple
married to D. Since, industrialist is seated to the right of C. (Lawyer) Q P(+)(Housewife)
Hence, industrialist cannot be A or C. Hence, B is an industrialist
and C is a businessman. A\E Son
A - Professor A\E
B - Industrialist
(Teacher) T(+) Couple (–)
(Doctor)
C - Businessman
D B S
D - Unemployed
Mother Mother
E - Unemployed C
1. (c) A is the husband of E and E is the sister of B. Hence, A is the R
brother-in-law of B. (Student/Engineer) U
(Engineer/Student)
2. (d) No information about its position is given.
So, the granddaughter of the family is a student and either R or
3. (c) B is the industrialist. U is a student.
4. (c) Businessman is unmarried in the group. Sol. (Q. Nos. 10-13) According to the given information,
5. (b) Since, A is professor, hence he must be graduate. arrangement is as follows
Sol. (Q.Nos. 6-8) According to the given information, arrangement is Persons Professions Sex
as follows
P Pharmacist M
Floor Person Q Architect F
8 S R Teacher M/F
7 X S Engineer M/F
6 Z T Lawyer F
5 T V Doctor M/F
4 V W Chartered Accountant M
3 W Q ← W
Wife
2 U In terms of income, Q < T < P < R < S < V < W
1 Y
10. (b) Q earns the least.
6. (d) T lives on floor number five. 11. (a) P is Pharmacist.
7. (b) Three persons live between W and X. 12. (d) V is Doctor and Q, T, P, R and S earn less than V.
8. (b) T lives immediately above V. 13. (d) The females are Q, T and one from R, S and V.
100 Study Package for NTSE

Sol. (Q. Nos. 14 and 15) According to the given information, 18. (a) The organisation would end with A.
arrangement is as follows
19. (a) B is to be organisation on Tuesday.
Person Day
20. (d) None of these
P Monday
J Tuesday Sol. (Q. Nos. 21-24) We have,
A:B=2:1
K Wednesday
B:C=9:2
M Thursday
C:D=1:2
O Friday
D:E=1:2
L Saturday
A:B:C:D:E
N Sunday 2:1
14. (a) N has to attend a seminar on Sunday. 9:2
1:2
15. (c) Except option (c), in all other options, the given day is either
1:2
the previous day or the next day of the day on which the given
person has to attend a seminar. 2 × 9 × 1× 1 : 1× 9 × 1× 1 : 1× 2 × 1× 1 :
Sol. (Q. Nos. 16-20) 18 : 9: 2 :
On the basis of information given in the question the following 1× 2 × 2 × 1 : 1× 2 × 2 × 2
arrangement can be made. 4 : 8
5 6 7 8 9 10 A>B>E>D>C
Monday Tuesday Wednesday Thursday Friday Saturday 21. (c) A has highest weight.
D B E C F A
22. (c) E is lighter then A and B.
16. (c) The organisation would start from D. 23. (b) E is heavier then D and C.
17. (b) E is to be organised on 7th. 24. (a) C is lighter in the weight.
Chapter

15
Problems Based
on Calendar
The record of all the days of the year is given in the calendar. A calendar shows us the days, weeks, months in a
year.
1 yr → 12 months
1 week → 7 days
1 yr → 52 weeks or 365 or 366 days.
î Year with 29 days in February has 366 days. It is called leap year. It is divisible by 4.
The number of days in a month of a year are as follows
January 31 days May 31 days September 30 days
February 28 days or 29 days June 30 days October 31 days
March 31 days July 31 days November 30 days
April 30 days August 31 days December 31 days

Same day is always repeated after 7 days.

Odd Days
In a given period, the number of days more than the complete weeks are called odd days. When total number of days
is divided by 7, then the remainder is called odd Days.

Counting of Odd Days


l Number of days in an ordinary year = 365 days l Number of days in a leap year = 366 days
7 365 52 7 366 52
35 35
15 16
14 14
1 odd days 2 odd days

or 365 = 52 × 7 + 1 = 1 odd days. or 366 = 52 × 7 + 2 = 2 odd days.


102 Study Package for NTSE

Important Points
l Days of the week related to odd days
Number of days 0/7 1 2 3 4 5 6
Days Sunday Monday Tuesday Wednesday Thursday Friday Saturday

l A century (100 yr) contain = 5 odd days


l Number of odd days in 200 yr = 5 × 2 = 10 = 3 odd days
l Number of odd days in 300 yr = 5 × 3 = 15 = 1 odd days
l Number of odd days in 400 yr = 5 × 4 + 1 = 20 + 1 = 21 = 0 odd days
l To determine the number of leap year, divide the numbers of year by 4. The remaining will be ordinary year.

Example 1 How many months are there in 5 yr 7 months?


(a) 60 (b) 65 (c) 67 (d) 70
Solution (c) 5 yr = 5 × 12 = 60 months
Total number of months = 60 + 7 = 67 months
Example 2 If 24th day of January 1999 is Saturday. What will be the day of 20th day of September of the same
year?
(a) Sunday (b) Monday (c) Friday (d) Saturday
Solution (a) Let us count days from 24th January to 20th day of September.
January 31 − 24 = 7 days June 30 days
February 28 days July 31 days
March 31 days August 31 days
April 30 days September 20 days
May 31 days 239 days

Divide 239 by 7, we get remainder as 1. Now, 24th day of January is Saturday. So, 20th September will be Sunday.

Example 3 Find the day of the week on 26 January 1950?


(a) Monday (b) Wednesday (c) Thursday (d) Friday
Solution (c) 26 January 1950 means 1949 yr + 26 days
Number of odd days in 1949 yr 1600 yr give 0 odd day.
300 yr give 1 odd day.
49 yr have 12 leap years and 37 ordinary years.
So, there are 12 × 2 + 37 × 1 = 24 + 37 = 61 odd days or 5 odd days.
Number of days between 1st January to 26th January 1950 = 26
odd day = 5
Total number of odd days = 0 + 1 + 5 + 5 = 11 = 4 odd days
So, 26 January 1950 was Thursday.

Target Exercise
Elementary Level Questions
1. If 3rd January of a year was Thursday, what will be the 3. If 15th October of a year was Saturday, what will be
day on 25th January of the same year? the day on 27th October of the same month?
(a) Thursday (b) Friday (a) Monday (b) Tuesday
(c) Saturday (d) Sunday (c) Wednesday (d) Thursday

2. If 7th February of a year was Monday, what will be the 4. If 29th April of a year was Friday, what will be the day
day on 24th February of the same year? on 18th September of the same year?
(a) Thursday (b) Friday (a) Sunday (b) Monday
(c) Saturday (d) Sunday (c) Tuesday (d) Wednesday
Problems Based on Calendar 103
5. If 17th September of a year was Monday, what will be 8. If day before yesterday it was Monday, what day will
the day on 29th October of the same year? fall on day after tomorrow?
(a) Saturday (b) Sunday (a) Saturday
(c) Monday (d) Tuesday (b) Friday
6. If today is Thursday, then what will be the day on (c) Sunday
363rd day? (d) Thursday
(a) Sunday (b) Saturday
(c) Thursday (d) None of these
9. How many months are there in 7 yr 2 months?
(a) 72 (b) 86
7. If Saturday falls on 4th January 1997, what day of (c) 80 (d) 94
the week will fall on 4th January 1998?
10. How many weeks are there in 3 yr 3 months?
(a) Monday (b) Friday
(a) 150 (b) 160
(c) Wednesday (d) Sunday
(c) 170 (d) 169

High Skill Questions


1. If 17th January of a year was Monday, what will be 7. If 21st August of 1989 was Friday, what would be the
the day on 19th June of the same year? (February has day on 13th July of 1988?
28 days.) (a) Friday (b) Saturday
(a) Sunday (c) Monday (d) Sunday
(b) Monday
(c) Tuesday 8. If 16th July of 1995 was Sunday, what would be the
(d) Wednesday day on 30th March of 1994?
(a) Sunday (b) Monday
2. If it was Monday on 4th February of 2004, what will (c) Tuesday (d) Wednesday
be the day on 1st January of the same year?
(a) Saturday 9. If 9th June of 2003 was Friday, what will be the day
(b) Sunday on 9th June 2006?
(c) Monday (a) Sunday
(d) Tuesday (b) Monday
(c) Tuesday
3. If it was Thursday on 15th July of 2001, what will be
(d) Friday
the day on 17th December of the same year?
(a) Thursday 10. If 25th December of 2008 is Thursday, what will be
(b) Friday the day on 1st January of 2010?
(c) Wednesday (a) Friday (b) Saturday
(d) Saturday (c) Sunday (d) Monday
4. If 23rd February of 2008 was Saturday, what will be 11. Film actor-director Raj Kapoor died on 2nd June
the day on 3rd November of the same year? 1988. What day of week was it?
(a) Sunday (a) Monday (b) Wednesday
(b) Tuesday (c) Thursday (d) Saturday
(c) Wednesday
(d) Monday 12. Find the day of the week on August 15, 1947?
(a) Tuesday (b) Wednesday
5. If 13th March of 2003 was Tuesday, what will be the (c) Thursday (d) Friday
day on 11th May of 2004?
(a) Friday
13. My Brother’s date of birth is May 3, 1999 but I do not
(b) Saturday remember the day, can you tell me which day it was?
(c) Sunday (a) Monday (b) Tuesday
(c) Wednesday (d) Thursday
(d) Monday

6. If 15th November of 2002 was Wednesday, what will 14. On what dates of January 1998 did we have Friday?
(a) 2, 9, 16, 23, and 30 (b) 4, 11, 18, and 25
be the day on 27th October of 2003?
(c) 3, 10, 17, 24, and 31 (d) 1, 8, 15, 22, and 29
(a) Wednesday
(b) Thursday 15. How many days were there from January 2, 1995 to
(c) Friday March15 , 1995?
(d) Saturday (a) 73 (b) 74 (c) 71 (d) 35
Hints and Solutions
Elementary Level Questions
1. (b) As we know, the days repeat after every 7 days. Number of 5. (c) Number of days from 18 September to 30th September
days between 25th January and 3rd January = 22 days. = 13 days
The day after 21 days will be same means Thursday. Number of days in October = 29 days
So, on 25th January, it will be Friday. Total number of days = 42 days
42
2. (a) As we know, the days repeat after every 7 days. Number of Number of weeks = = 6 weeks
7
days between 24th February and 7th February = 17 days.
Days after 6 weeks will again be Monday.
The day after 14 days will be also Monday.
So, on 24th February, it will be Thursday. It will be Monday on 29th October again.
363
3. (d) Number of days between 27th October and 15th October 6. (d ) Here, = 51 weeks + 6 odd days
7
= 12 days. ∴ Thursday + 6 days = Wednesday
The day after 7 days will be also Saturday. The day after 5 days
7. (d) Total number of days from 4th January 1997 to 4th January
will be Thursday. On 27th October, it will be Thursday.
1998 = 365
4. (a) Number of days in April = 30 April = 1day 365
Q Number of odd days = = 52 + 1
Number of days in May = 31days 7
Number of days in June = 30 days After one day, it will be Sunday.
Number of days in July = 31days 8. (b) Day before yesterday = Monday
Number of days in August = 31days So, today is Wednesday. Hence, day after tomorrow = Friday.
Number of days in September = 18 days 9. (b) 7 yr = 7 × 12 = 84 months
Total number of days = 1 + 31 + 30 + 31 + 31 + 18 = 142 days ∴ Total number of months = 84 + 2 = 86 months
142 2
Number of weeks = = 20 weeks 10. (d) 3 yr = 52 × 3 = 156 weeks
7 7
1
Day after 140 days will be Friday. 3 months = 52 × = 13 weeks
4
Day after 2 days will be Sunday.
∴ Total weeks in 3 yr and 3 months = 156 + 13 = 169 weeks
On 18th September, it will be Sunday.

High Skill Questions


1. (a) Number of days between 17th January and 19th June Days after 22 weeks will be Thursday again.
= 14 days in January + 28 days in February + 31 days in March Day after 1 day will be Friday.
+ 30days in April + 31days in May + 19days in June = 153days. Day on 17th December will be Friday.
153 6
Number of weeks = = 21 days 4. (d) Number of days between 23rd February and 3rd November
7 7
= 6 days in February + 31 days in March + 30 days in April
Day after 21 weeks will be Monday again. Day after 6 days will be
+ 31days in May + 30 days in June + 31days in July + 31days in
Sunday. Day on 19th June will be Sunday.
August + 30 days in September + 31days in October + 3 days in
2. (d) Number of days between 1st January and 4th February November = 254 days.
= 3 days in February + 31days in January = 34 days 254 2
6 Number of weeks = = 36 weeks
Number of weeks = 4 weeks 7 7
7
Day after 36 weeks will be Saturday. Day after 2 days will be
Day before 4 weeks will be Monday again. Day before 6 days will Monday. Day on 3rd November will be Monday.
be Tuesday.
5. (c) Number of days between March 13, 2003 to May 11, 2004
3. (b) Number of days between 15th July and 17th December = 366 days from 2003 to 13th March 2004 + 18 days in March,
= 16 days in July + 31 days in August + 30 days in September
2004 + 30 days in April + 11days in May, 2008 = 425 days.
+ 31 days in October + 30 days in November + 17 days in
December = 155 days 425 5
Number of weeks = = 60 weeks
155 1 7 7
Number of weeks = = 22 weeks
7 7 Day after 60 weeks will be Tuesday. Day after 5 days will be
Sunday. Day on 11th May 2004 will be Sunday.
Problems Based on Calendar 105
6. (d) Number of days between 15th November 2002 and 27th Jan Feb March April May June
October 2003 = 365 days from 15th November 2002 to 15th 31 + 29 + 31 + 30 + 31 + 2 = 154 days
November 2003 – (15 days of November, 2003 + 4 days in or 22 weeks and 0 odd day
October 2003) = 346 days. ∴ Total number of odd days = 0 + 1 + 3 + 0 = 4
346 3 So, the day was Thursday.
Number of weeks = = 49 weeks
7 7 12. (d) 15th August 1947 means
Day after 49 weeks will be Wednesday. 1946 year + 7 months + 15 days
Day after 3 days will be Saturday. Number of odd days in 1946 yr
Day on 27th October 2003 will be Saturday. 1600 year give 0 odd day.
7. (d) Number of days between 13th July 1988 to 21st August 1989 300 year give 1 odd day.
= 365 days from 13th July, 1988 to 13th July, 1989 + (18 + 21) 46 year have 11 leap years and 35 ordinary years.
days = 404 days. So, there are 11 × 2 + 35 × 1 = 57 odd days or 1 odd day.
404 5 Number of days between 1st January 1947 to 15th August
Number of weeks = = 57 weeks
7 7 1947.
Day before 57 weeks will be Friday. Day before 5 days will be Jan Feb March April May June July Aug
Sunday. 31 + 28 + 31 + 30 + 31 + 30 + 31 + 15
8. (d) Number of days between 30th March 1994 and 16th July, = 227 days
1995 = 365 days from 30th March 1994 to 30th March 1995 = + 1 i.e., 32 weeks and 3 odd days.
day in March, 1995 + 30 days in April, 1995 + 31 days in May, ∴ Total number of odd days = 0 + 1 + 1 + 3 = 5 odd days
1995 + 30 days in June, 1995 + 16days in July, 1995 = 473days. So, August 15, 1947 was Friday.
473 4
Number of weeks = = 67 weeks 13. (a) May 3, 1999 means 1998 complete years + first 4 months
7 7
upto April of 1999 + 3 days of May.
Day before 67 weeks will be Sunday.
1600 year have 0 day.
Day before 4 days will be Wednesday.
300 year have 1 day.
Day on 30th March 1994 will be Wednesday. 98 year have 24 leap years + 74 ordinary years.
9. (c) Number of days between 9th June 2003 to 9th June 2006 = (24 × 2 ) + (74 × 1)
= 366 days from 9th June 2003 to 9th June 2004. =122 days = 17 weeks + 3 odd days
From 9th June 2004 to 9th June 2005 = 365 days 1998 year have (0 + 1+ 3) = 4 odd days.
From 9th June 2005 to 9th June 2006 = 365 days January has 31 days i.e., 3 odd days.
1999 is an ordinary year.
Total days = 366 + 365 + 365 = 1096
1096 4 Hence, February has 28 days i.e., 0 odd day.
Number of weeks = = 156 weeks
7 7 March has 31 days i.e., 3 odd days.
Day after 156 weeks will be Friday. Day after 4 days will be April has 30 days i.e., 2 odd days.
Tuesday. Day on 9th June 2006 will be Tuesday. May 3 gives 3 more odd days.
10. (a) Number of days between 25th December 2008 to 1st Total number of odd days on May 3, 1999.
January of 2010 = 365 days between 25th December 2008 to = (4 + 3 + 0 + 3 + 2 + 3) odd days.
25th December 2009 + 7 days from 25th December 2009 to 1st = 15 odd days i.e., 1 odd day.
January 2010 = 372 days. Hence, May 3, 1999 was Monday.
372 1
Number of weeks = = 53 weeks + 1 odd day 14. (a) Let us first find the day on 1st January 1998.
7 7
Period Number of odd days
Day after 53 weeks will be Thursday again.
1600 0
Day after 1 day will be Friday.
300 1
Day on 1st January 2010 will be Friday.
97 (97 + 24 = 121)
11. (c) 2nd June 1998 means January 1, 1998 1 (Q 24 leap years)
1987 yr + 5 months + 2 days. = 23 days which is equivalent to 4 odd days.
Number of odd days in 1987 yr. ∴ January 1, 1998 was Thursday.
1600 yr give 0 odd day. Therefore, first friday in January 1998 was on January 2, 1998.
300 yr give 5 × 3 = 15 ∴ We had Friday on 2nd, 9th, 16th, 23rd and 30th January 1998.
or 1 odd day.
15. (a) The year 1995 was not a leap year.
87 yr have 21 leap years and 66 ordinary years.
∴Number of days from January 2, 1995 to March 15, 1995.
So, there are 21 × 2 + 66 × 1 = 108 odd days on 3 odd days.
January + Febaruary + March
Number of days between 1st January 1988 to 2nd June 1988.
30 + 28 + 15 = 73 days
Chapter

16
Problems Based
on Clock
A clock is a circular/square shape instrument used to Minute hand The longer hand of the clock is known
view/measure time in hours, minutes and seconds. as minute hand. The minute hand moves from one small
Measurement of time is an important aspect of our division to the next small division in 1 min.
daily life. It helps us to prepare our schedule and work The minute hand goes once round the dial in 1 h.
in an orderly manner. There are, mainly, four Thus, the minute hand covers 60 small divisions in 1 h.
important parts of a clock—Dial, Hour hand, Minute
î 60 min = An hour
hand and Second hand.
30 min = Half an hour
12 15 min = Quarter of an hour
11 1
l Angle traced by minute hand in 60 min = 360°
10 2
Dial Minute hand l Angle traced by minute hand in 1 min = 6°
9 3
Hour hand Hour hand The shorter hand of the clock is known as
8 4
Second hand hour hand. The hour hand moves from one number to
7 5
6 the next number in 1 h. The hour hand takes 12 h to
Clock complete one round.
Also, it takes 2 full rounds of the clock in a day. Its
The Dial movement is also in a clockwise direction.
The dial of a clock is circular one, whose circumference 1 day = 24 h
is divided into 12 parts (12 equal big divisions) labelled l Angle traced by hour hand in 12 h = 360°
as 1, 2, 3, 4, ... and 12. The space between every pair of
two numbers is divided in 5 equal small divisions. l Angle traced by hour hand in 1 h = 30°

Thus, the dial is divided into 60 small divisions. The There are some important points that would help the
clock has two hands. One is longer (minute hand) and candidate while solving the problems.
another is shorter (hour hand).

Points to Remember
● In 60 min, the minute hand gains 55 min on the hour hand.
● In every hour, both the hands coincide once.
● The hands are in the same straight line when they are opposite to each other.
● When the two hands are at right angles (90°) they are 15 min space apart.
● When the hands are in opposite directions, they are 30 min space apart.
● Both hands of a clock occurs right angle twice in one hour, 22 times in 12 h and 44 times in 24 h.
● Both hands of a clock occurs straight or opposite once in one hour, 11 times in 12 h and 22 times in 24 h.

● When minute hand covers distance of 1 min, in this time, hour hand covers distance of
2
Problems Based on Clock 107
Example 1 An accurate clock shows 8 O’clock in the morning. Through how may degrees will the hour hand rotate when the clock
shows 2 O’clock in the afternoon?
(a) 144 ° (b) 150° (c) 168° (d) 180°
°
Angle traced by the hour hand in 6 h =  × 6  = 180 °
360
Solution (d)
 12 

Example 2 A clock is started at noon. By 10 min past 5, the hour hand has turned through
(a) 145° (b) 150° (c) 155° (d) 160°
Solution (c) Angle traced by hour hand in 12 h = 360 °
°
h ⇒ 
31 360 31 
Angle traced by hour hand in 5 h 10 min i.e., ×  ⇒155°
6  12 6

Example 3 The angle between the minute hand and the hour hand of a clock when the time is 8.30, is
(a) 80° (b) 75° (c) 60° (d) 105°
°
Solution (b) Angle traced by hour hand in 17 h =  360 × 17  ⇒ 255°
2  12 2
°
Angle traced by minute hand in 30 min =  × 30  = 180 °
360
 60 
Hence, required angle = 255° − 180 ° = 75°

Target Exercise
Elementary Level Questions
1. A man seeing a watch in the mirror which is hanging 7. A monkey climbing up a greased pole; ascends 5 m
on the wall. In the mirror, the condition of watch is and slips down 2 m in alternate minutes. If the pole is
hour hand on 9 and minute hand on 12. Find out the 35 m high, then the monkey will reach the top in
exact time in watch. (a) 11 min (b) 11 min, 24 s
(a) 2 pm (b) 8 pm (c) 7 pm (d) 3 pm (c) 11 min, 36 s (d) None of these
2. A watch shows 1:30 pm, if minute hand is towards 8. A bus for Delhi leaves every thirty minutes from a
South, find the direction of hour hand. bus stand. An enquiry clerk told a passenger that the
(a) North (b) South-East bus had already left ten minutes ago and next bus
(c) North-West (d) North-East will leave at 9:35 am. At what time did the enquiry
3. Reaching the place of meeting on Tuesday 15 min clerk give this information to the passenger?
before 8:30 h, Anuj found himself half an hour earlier (a) 9:10 am (b) 8:55 am (c) 9:08 am (d) 9:15 am
than the man who was 40 min late. What was the 9. What will be the angle between hour hand and
scheduled time of the meeting? minute hand, if clock shows 5:15 pm?
(a) 8:00 h (b) 8:05 h (c) 8:15 h (d) 8:45 h (a) 90° (b) 70° (c) 67.5° (d) 85°
4. A person is to go up a tree 60 ft high. In every second, 10. How many times are the hands of a clock is at right
he climbs 5 ft but slips down 4 ft. After how many angle in a day?
seconds, will he be able to reach the top of the tree? (a) 22 (b) 24 (c) 44 (d) 48
(a) 60 (b) 59 (c) 56 (d) 58 11. At what time, (in minutes) between 3 O’clock and
5. A clock buzzes 1 time at 1 O’clock, 2 times at 4 O’clock, both the needles will coincide with each
2 O’clock, 3 times at 3 O’clock and so on. What will be other?
the total number of buzzes in a day? 1 4
(a) 5 min past 3 (b) 12 min past 3
(a) 150 (b) 156 11 11
4 4
(c) 100 (d) None of these (c) 13 min past 3 (d) 16 min past 3
11 11
6. A tortoise walks 1 km in 4 h. He takes rest of 20 min
12. At what time between 5:30 and 6 O’clock, will the
after every kilometre. So, you have to find out, how
hands of a clock be at right angle?
much time would be taken by tortoise to complete 3.5 3 4
km journey? (a) 43 min past 5 (b) 46 min past 5
11 11
(a) 14 h (b) 13 h (c) 15 h (d) 12 h (c) 40 min past 5 (d) 45 min past 5
108 Study Package for NTSE

13. How many times do the hands of a clock coincide in a 17. How many times do the hands of a clock points
day? towards each other in a day?
(a) 24 (b) 22 (a) 24 (b) 20 (c) 12 (d) 22
(c) 21 (d) 20
18. A clock is set right at 8 am. The clock gains 10 min in
14. A clock is set right at 5 am. The clock loss 16 min in 24 h. What will be the right time when the clock
24 h. What will be the right time when the clock indicates 1 pm on the following day?
indicates 10 pm on the third day? (a) 11:40 pm (b) 12:48 pm
(a) 11:15 pm (b) 11:00 p (c) 12:00 pm (d) 10:00 pm
(c) 12:00 pm (d) 12:30 pm 19. At what time between 4 and 5 O’clock, will the hands
15. At what time between 5 and 6, are the hands of a of a watch point in opposite directions?
clock coincides? (a) 45 min past 4 (b) 40 min past 4
(a) 22 min past 5 (b) 30 min past 5 4 6
(c) 50 min past 4 (d) 54 min past 4
8 3 11 11
(c) 22 min past 5 (d) 27 min past 5
11 11 20. Find at what time between 8 and 9 O’clock will the
hands of a clock be in the same straight line but not
16. At what time between 9 and 10 O’clock, will the
together.
hands of a watch be together? 10 10
(a) 45 min past 9 (b) 50 min past 9 (a) 10 min past 8 (b) 50 min past 8
1 2 11 11
(c) 49 min past 9 (d) 48 min past 9 12
11 11 (c) 10 min past 8 (4) 10 min past 8
11

High Skill Questions


1. The reflex angle between the hands of a clock at 6. The angle between the minute hand and the hour
10 : 25 is hand of a clock when time is 4 : 20, is
1° (a) 0° (b) 10°
(a) 180° (b) 192
2 (c) 5° (d) 20°

(c) 195° (d) 197 7. At what angle the hands of a clock are inclined at
2
15 min past 3?
2. A watch which gains 5 seconds in 3 min was set right 1°
(a) 10 (b) 64°
at 7 a.m. In the afternoon of the same day, when the 2
watch indicated quarter past 4 O’clock, the true time 1° 1°
(c) 7 (d) 72
is 2 2
7 8. At 3 : 40 the hour hand and the minute hand of a clock
(a) 59 min past 3 (b) 4 pm
12 from an angle of
7 3
(c) 58 min past 3 (d) 2 min past 4 (a) 120° (b) 125°
11 11
(c) 130° (d) 135°
3. How much does a watch lose per day, if its hand
9. In Ravi’s clock shop, two clocks were brought for
coincide every 64 min?
8 5
repairs. One clock has the cuckoo coming out every 16
(a) 32 min (b) 36 min min, while the other one has the cuckoo coming out
11 11
every 18 min. Both cuckoos come out at 12:00 noon.
(c) 90 min (d) 96 min
When will they both come out together again?
4. At what time between 7 and 8 O’clock will the hands (a) 2 : 06 PM (b) 2 : 08 PM
of a clock be in the same straight line but, not (c) 2 : 24 PM (d) 2 : 32 PM
together? 10. At what time, in minutes, between 3 O’clock and
2
(a) 5 min past 7 (b) 5 min past 7 4 O’clock both the needles will coincide each other?
11 1” 4” 4” 4”
3 5 (a) 5 (b) 12 (c) 13 (d) 16
(c) 5 min past 7 (d) 5 min past 7 11 11 11 11
11 11
5. A clock gains five min every hour. What will be the 11. A watch which gains uniformly is 2 min low at noon
on Monday and is 4 min. 48 sec fast at 2 pm on the
angle traversed by the second hand in 1 min?
following Monday. When was it correct?
(a) 360° (b) 360.5°
(c) 390° (d) 380° (a) 2 pm on Tuesday (b) 2 pm on Wednesday
(c) 3 pm on Thursday (d) 1 pm on Friday
Problems Based on Clock 109
12. (b)
Elementary Level Questions 11 12 1 11 12 1 11 12 1
10 2 10 2 10 2
9 3 9 3 9 3
1. (d) 12 12 8 4 8 4 8 4
7 6 5 7 6 5 7 6 5
9 3 9 3

6 6 Fig. (i) Fig. (ii) Fig. (iii)

At 5, both the hands are 25 min apart and to be at right angle


2. (d) N
NE both the hands have to be 15 min apart as shown in Fig. (ii) and
W

Fig. (iii). Since, we have to take the position of clock between


N

N 11
12 1
10 2 5:30 and 6 O’clock, therefore the positions of hands of clock as
W E W 9 3 E above Fig. (iii) is our answer.
8 4
Now, it is clear from Fig. (ii) and Fig. (iii) that minute hand needs
7 5
S
SW
6
SE to travel 15 min space in order to form a right angle with the hour
S hand. 55 min space is gained in 60 min. Therefore, 15 min

spaces will be gained in  × 15 min or


Hence, hour hand is towards North-East. 60 180
min.
 55  11
3. (b) Anuj reached the place of meeting at 8:15 h, he reached
Past  + 30 min = 46 min
30 min earlier than the man who was 40 min late, i .e., he 180 4
reached 10 min late. Hence, the scheduled time of the  11  11
4
meeting was 8:05 h. Therefore, the hands are at right angle at 46 min past 5.
11
4. (c) Effective distance covered in one second = (5 − 4) = 1ft
13. (b) From the properties of the clock, we know that hands of a
Hence, required time = 60 − [(1 + 1) + (1 + 1)] = 60 − 4 = 56 clock coincide once in every hour but between 11 O’clock and
12 (12 + 1) 1 O’clock, they coincide only once. Therefore, the hands of a
5. (b) Number of buzzes in a day = × 2 = 156
2 clock coincide 11 times in every 12 h. Hence, they will coincide
6. (c) Total time taken by tortoise (11 × 2 ) = 22 times in 24 h.

= (4 + 4 + 4 + 2 ) h and (20 + 20 + 20) min 14. (b) Time from 5 am of a particular day to 10 pm on the 4th day is
89 h. Now, the clock loses 16 min in 24 h or in other words, we
= 14 h + 60 min = 15 h
can say that 23 h 44 min of this clock is equal to 24 h of the
7. (a) Effective length covered in 1 min = 5 − 2 =3m correct clock.
First 30 m = 3 m × 10 min  23 + 44 
Second 5 m = 5 m × 1 min or  
 60 
∴Overall time taken by monkey is 11 min. 356
⇒ h of this clock = 24 h of the correct clock
8. (d) Next bus will leave at 9:35 am. So, last bus left at 15
24 × 15
(9:35 − 0 :30) = 9:05 am ∴ 89 h of this clock =  × 89 h of correct clock
 356 
So, enquiry clerk gave the information to the passenger at
= (9 : 05 + 0 : 10) = 9 : 15 am = 90 h of the correct clock
1 360 21 315 ⇒ 89 h of this clock = 90 h of the correct clock
9. (c) Angle traced by hour hand in 5 = × =
4 12 4 2 Therefore, it is clear that in 89 h this clock loses 1 h and hence,
360 the correct time is 11:00 pm when this clock shows 10:00 pm.
Angle traced by minute hand in 15 min = × 15 = 90°
60 15. (d)
315 11 12 1
Hence, required angle = − 90° = 67 . 5° 10 2
2 9 3
10. (c) In 12 h, they are at right angles, 22 times. 8 4
7 6 5
So, in 24 h, they are at right angles, 44 times.
11. (d) At 3 O’clock, the minute hand is 15 min away from the hour From the figure, we find that minute hand is 25 min spaces apart
hand. To coincide, it must gain 15 min as 55 min are gained in from hour hand. In order to coincide, it has to gain 25 min
60 min. spaces. Now, 55 min are gained by minute hand in 60 min.
60 3
15 min are gained in  × 15 = 16 min
60 4 25 min will be gained in × 25 = 27 min
 55  11 55 11
4 3
Hence, the hands are coincide at 16 min past 3. So, the hands will coincide at 27 min past 5.
11 11
110 Study Package for NTSE

16. (c) Both the hands are 15 min space apart at 9 O’clock. To be 19. (d)
11 12 1 11 12 1
together between 9 and 10, minute hand has to gain 45 min. 10 2 10 2
9 3 9 3
11 12 1 8 4 8 4
10 2 7 6 5 7 6 5
9 3
8 4 Fig. (i) Fig. (ii)
7 6 5
At 4 O’clock, the hands are 20 min apart from each other and
for having in the opposite direction they needs to be 30 min
Now, minute hand gains 55 min in 60 min. apart.
60 1 From Fig. (i) and Fig. (ii), it is clear that minute hand has to travel.
It will gain 45 min in × 45 = 49 min
55 11 (20 + 30) min space in order to be in opposite direction of each
1 other. Now, 55 min space is gained in 60 min.
Therefore, the hands are together at 49 min past 9.
Therefore, 50 min space will be gained in  × 50 min or 54 min.
60 6
11
 55  11
17. (d) The hands of a clock point towards each other 11 times in Hence, the hands of the clock will be in opposite direction at
every 12 h (because between 5 and 7, they point towards each
6
other only once at 6 O’clock). 54 min past 4.
11
Therefore, in a day, the hands points 22 times in all, towards
each other. 20. (a)
11 12 1 11 12 1
18. (b) Time from 8 am of a particular day to 1 pm of the following day 10 2 10 2
= 29 h 9 3 9 3
8 4 8 4
Now, the clock gains 10 min in 24 h, it means that 24 h 10 min of 7 6 5 7 6 5
this clock is equal to 24 h of the correct clock.
145 Fig. (i) Fig. (ii)
h of this clock = 24 h of the correct clock
6 Fig. (i) shows the positions of the hands of the clock and it is
24 clear that they are 20 min apart. To be in the straight line, they
29 h of this clock = × 6 × 29 = 29 h 48 min of the correct
145 have to be 30 min apart. So, the minute hand will have to move
clock 29 h of this clock = 28 h 48 min of the correct clock 10 min space in order to be 30 min apart from hour hand. 55 min
It means that the clock in question is 12 min faster than the are gained in 60 min.
60 12
correct clock. 10 min will be gained in × 10 = × 10 min
55 11
Therefore, when this clock indicates 1 pm the correct time will be
Therefore, the hands will be at right angle but not together at
12 min to 1 pm. 10
10 min past 8.
11

High Skill Questions


1. (d) Angle traced by hour hand in 3. (a) 55 min spaces are covered in 60 min.
125
h=  360 × 125  ° = 312 1° 60 min spaces are covered in 
60
× 60 min = 65 min
5
 
12  12 12  2  55  11

Loss in 64 min =  65 − 64 =


Angle traced by minute hand in 25 min 5 16
min
°  11  11
=  × 25 = 150°
360
 60 
Loss in 24 h =  × 24 × 60 min
16 1
° ×
 11 64 
∴ Reflex angle = 360° −  312 − 150
1
 2  8
1° 1° = 32 min
= 360° − 162 = 197 11
2 2
37 4. (d) When the hands of the clock are in the the same straight line
2. (b) Time from 7 am to 4 : 15 pm= 9 h 15 min = h but not together, they are 30 min spaces apart. At 7 O’clock, they
4
are 25 min space apart.
3 min 5 sec of this clock = 3 min of the correct clock.
37 1 ∴Minute hand will have gain only 5 min spaces. 55 min space are
⇒ h of this clock = h of the correct clock. gained in 60 min.
720 20
5 min spaces are gained  × 5 min = 5 min
60 5
hof this clock = 
37 1 720 37 
⇒ × ×  h of the correct clock.  55  11
4  20 37 4
5
= 9 h of the correct clock. ∴Required time = 5 min past 7
11
So, the correct time 9 h after 7 am i.e., 4 pm.
Problems Based on Clock 111
5. (c) Each seconds space equals 6°. 9. (c) Both the cuckoos will come out together after intervals which
A clock gains 5 min every hour. are common multiples of 16 min and 18 min.
Q Clock gains 5 min in 60 min. LCM of 16 and 18 = 2 4 × 9
5 = 144
It means the clock gains min in 1 min.
60 144
5 144 min = h
∴Angle traversed × 360 = 30° 60
60
= 2h 24 min
The second hand will traverse
360°+30° = 390° in 1 min. ∴Both the cuckoos will come out together
360 13  ° again at 12 : 00 PM + 2h 24 min = 2 : 24 PM
6. (b) Angle traced by hour hand in h = 
13
×  = 130°
3  12 3 10. (d) At 3 O’clock, the minute hand is 15 min spaces apart from the
° hour hand.To be coincident, it must gain 15 min spaces.
Angle traced by min hand in 20 min =  × 20 = 120°
360
 60  Since, 55 min are gained in 60 min.

15 min are gained in  × 15 min = 16 min


60 4
∴ Required angle = (130 − 120)° = 10°
 55  11
360 13  °
h = 
13 1
7. (c) Angle traced by hour hand in ×  = 97 ° 4
4  12 4 2 So, the hands are coincident at 16 min past 3.
11
°
Angle traced by min hand in 15 min =  × 15 = 90°
360
 60  11. (b) Time from 12 pm on Monday to 2 pm on the following
Monday = 7 days 2 h = 170 h.
1 ° 1°
∴ Required angle =  97  − 90° = 7
The watch gains  2 + 4  min or
 2 4 34
2 min in 170 h
 5 5
8. (c) Angle traced by hour hand in 12 h = 360° 34
Now, min are gained in 170 h.
360 11 °
Angle traced by it in h = 
11 5
×  = 110°
3  12 3
∴2 min are gained in  170 × × 2  h = 50 h
5
Angle traced by minute hand in 60 min. = 360°  34 
°
Angle traced by it in 40 min =  × 40 = 240°
360 So, watch is correct 2 days 2 h after 12 pm on Monday i.e., it will
 60  be correct at 2 pm on Wednesday.
∴Required angle (240 − 110)° = 130°
Chapter

17
Venn-Diagrams
This section deals with questions, which aim to analyse the ability of candidates to relate a certain given group of
items and illustrate it diagrammatically by Venn-diagram.

Type I Questions of Diagrams Based on Words Relation


In this type of questions, few different types of sets of Venn-diagrams are given to clear with their implications.
1. If the items belong to three different groups. They can 4. If two separate items belong to the class of the third,
be represented by three separate figures. they are represented by two disjoint circles inside a
e.g., Doctors, Engineers and Lawyers. bigger circle as shown in figure given below.
Doctors Engineers
e.g., Table, Chair and Furniture.

Furniture

Table Chair

Lawyers
2. Three items that are partly related to each other are
represented by Venn-diagram is as follows Clearly, table and chair are separate items but both
e.g., Authors, Teachers and Men. are furniture.
5. If one item belongs to the class of second while third
Authors
is entirely different from the two. It can be
Teachers
represented by Venn-diagram given below.
e.g., Thief, Criminal and Judge.
Men

3. If one item belongs to the class of Hours


Thief Judge
the second and the second belongs Minutes
to the class of third, then the
Seconds
representation by Venn-diagram is Criminal
as follows
e.g., Seconds, Minutes and Hours. Thief is a criminal and judge is not directly related
Clearly, seconds are a part of with thief or criminal.
minutes and minutes are a part of hours.
Venn-Diagrams 113
6. If one item is related to second while second is Example 1 Choose from the four diagrams given below, the one
related to third but first and third are not related that illustrates the relationship among Languages, Hindi and English.
to each other, then it must be represented as
e.g. Travellers, Train, Bus. Clearly,some
travellers travel by train and some travellers (a) (b)
travel by bus.
Travellers

(c) (d)
Train Bus

Solution (c) Both Hindi and English are languages but both of
7. If two items are partly related to one another but them are different from each other.
both items belong to a single group, then it can
represented by Venn-diagram given below. Example 2 Select from the four alternative diagrams, the one
Teachers, Graduates and Human beings that best illustrates the relationship among the three
classes—Pigeons, Birds and Cats.
Human being
(a) (b)

Teachers Graduates
(c) (d)
Clearly, all graduates are human being and
some graduates can be teachers also all teachers
are human beings and some teachers are Solution (a) All pigeons are birds but cats are entirely different.
graduates.

Type II Questions Based on Geometrical Figure


In some problems, a Venn-diagram is given in the Solution (c) Married persons living in joint families are
form of a geometrical figure that represents represented by the region common to the square and the circle,
different classes. The candidate is required to study i.e., D and B but according to the given conditions, the persons
should not be teachers. So, B is to be excluded. Hence, the
and analyse the figure carefully and then answer
required condition is denoted by region D.
certain questions as per given informations.
Example 4 Persons who live in joint families, are unmarried and
Directions (Ex. Nos. 3-5) In the following diagram, who do not work as teachers are represented by
three classes of population are represented by three (a) C (b) B (c) E (d) D
geometrical figures. The triangle represents the teachers, Solution (c) According to the given conditions, the persons
the square represents the married persons and the circle should be unmarried and not working as school teachers. So, the
represents the persons living in joint families. region should not be a part of either the square or the triangle.
Thus, the given conditions are satisfied by the region E.
F
Example 5 Married teachers living in joint families are
E B C represented by
D (a) C (b) B (c) D (d) A
A Solution (b) Married teachers are represented by the region
common to the square and the triangle, i.e., B and C. But,
Example 3 Married persons living in joint families but according to the given conditions, the persons should be living in
not working as teachers are represented by joint families. So, the required region should be a part of the
(a) C (b) F circle. Since, B lies inside the circle, so the given conditions are
(c) D (d) A satisfied by the persons denoted by the region B.
Target Exercise
Elementary Level Questions
Directions (Q. Nos. 1-10) You are to choose from the five Directions (Q. Nos. 15-18) These questions are based on
Venn-diagrams that best illustrates the relationship among the following Venn-diagram and answer the questions given
three given classes or groups in the questions. below.
The rectangle represents artists, the circle represents players
and the triangle represents doctors.

6 28
25
(a) (b) (c) 20 4
10
40

15. How many players are neither artists nor doctors?


(a) 20 (b) 10
(c) 25 (d) 45
(d) (e)
16. How many doctors are both players and artists?
1. Language, Hindi, Science
(a) 6 (b) 4 (c) 10 (d) 20
2. Boy, Girl, Player
3. Copper, Silver, Zinc 17. How many doctors are neither players nor artists?
(a) 28 (b) 20 (c) 25 (d) 10
4. Deer, Rabbit, Goat
5. Sea, Island, Mountain 18. How many artists are players but not doctors?
6. Atoms, Protons, Electrons (a) 20 (b) 25 (c) 28 (d) 10
7. Liquid, Milk, Water Directions (Q. Nos. 19-28) The four alternatives in each of
8. Engineers, Lawyers, Doctors the following questions, three alternatives are such that the
9. Jaipur, Rajasthan, Haryana three words are related among themselves in one of the four
ways represented by (a), (b), (c), and (d) below while none of
10. World, Asia, India
these relationship is applicable to the remaining alternative.
Directions (Q. Nos. 11-14) These questions are based on That is your answer.
the following diagram. Answer the questions given below.
A B
1 2 5
Cricket Hockey
3
4 6 (a) (b) (c) (d)
7
19. Authors, Teachers, Women
Football
C
20. Elements, Metals, Non-metals
21. Cats, Dogs, Pets
11. Which numeral represents the set of persons, who
22. Games, Cricket, Hockey
play all three games?
(a) 3 (b) 4 (c) 6 (d) 7 23. Elephants, Tigers, Animals
24. Country, State, District
12. Which numeral represents the set of persons, who
play football and hockey but not cricket? 25. Rose, Flower, Lotus
(a) 7 (b) 2 (c) 3 (d) 6 26. Father, Mother, Child
13. Which numeral represents the set of persons, who 27. Biology, Botany, Zoology
play cricket and hockey but not football? 28. Flowers, Clothes, White
(a) 2 (b) 3 (c) 1 (d) 4 29. In a dinner party, both fish and meat were served.
14. Which numeral represents the set of persons, who Some took only fish and some took only meat. There
play hockey but neither cricket nor football? were some vegetarians who did not accept either. The
(a) 5 (b) 4 (c) 6 (d) 7 rest accepted both fish and meat. Which of the
Venn-Diagrams 115
following logic diagrams correctly reflects this cocacola. There were some teetotaller who did not
situations? accept either. The rest accepted both thums up and
cocacola. Which of the following logical diagrams
correctly effects this situation?

(a) (b) (c) (d)

30. In a cocktail party, both thums up and cocacola were (a) (b) (c) (d)
served. Some took only thums up and some took only

High Skill Questions


Directions (Q. Nos. 1-5) Each of the questions given below sections of the diagram. On the basis of these numbers,
is based on the given diagrams. The diagram comprises three answer the following.
circles showing people from three different countries who
4 Female
have travelled to either, both or none of the countries
graduates
mentioned in the diagrams. 1 7
9
5 8
R
us

China
si

6
a

A SE females 2 3
E C D F
bank loan facility
Self-employed female
B
India 6. How many female graduates are self-employed?
(a) 12 (b) 13 (c) 15 (d) 20
1. Which of the following is true about A?
(a) It represents Russians who have travelled only to China but 7. How many female graduates are not self-employed?
not to Indians. (a) 4 (b) 8 (c) 10 (d) 29
(b) It represents Indians who have travelled only to Russia but
not to China 8. How many non-graduate females are self-employed?
(c) It represents Chinese who have travelled only to Russia but (a) 7 (b) 9 (c) 21 (d) 15
not to India
9. How many self-employed female graduates are with
(d) It represents all those Indians who have travelled only to
Russia as well as to the China
bank loan facility?
(a) 5 (b) 12 (c) 7 (d) 15
2. Which of the following represents Indians who have
10. How many non-graduate self-employed females are
travelled only to China but not to Russia?
with bank loan facility?
(a) Only C (b) Only B
(a) 3 (b) 8 (c) 9 (d) 12
(c) Only A (d) Only E

3. Which of the following represents choose Chinese Directions (Q. Nos. 11-13) Study the following diagram
who have travelled only to the Russia but not to consisting of a circle, a rectangle and a triangle and answer
India? the questions given below it.
(a) Only A (b) Only C
(c) Only D (d) Not shown in diagram 10

4. Which of the following represents such Indians who 07 18 20


have travelled only to Russia but not to China? 15 17
40
(a) Only F (b) Only D
(c) A and D (d) C and F 27 18
12
5. Which of the following represents Russians who have 15 16
travelled neither to China nor to India?
(a) Only D (b) C and D (c) A and D (d) Only F 11. Which is the largest number among those which are
there in the rectangle but not in the circle?
Directions (Q. Nos. 6-10) The following five questions are (a) 27 (b) 40 (c) 15 (d) 17
based on the following diagram in which the triangle 12. Which is the smallest number among those which are
represents female graduates, square represents self-employed there in the triangle but not in the rectangle and
females and circle represents self-employed females with circle?
bank loan facility. Numbers are shown in the different (a) 10 (b) 15 (c) 16 (d) 12
116 Study Package for NTSE

13. Which of the following numbers is common in all the 19. Educated hardworking and honest urban people are
three diagrams? indicated by
(a) 40 (b) 18 (c) 12 (d) 15 (a) 1 (b) 2 (c) 3 (d) 4

20. Urban people who are hardworking and educated but


Directions (Q. Nos. 14-15) In the following figure,
rectangle, square, circle and triangle represent the regions of not honest are indicated by
wheat, gram, maize and rice cultivation respectively. On the (a) 1 (b) 2 (c) 3 (d) 4
basis of the figure, answer the following questions. 21. Non-urban educated people who are neither
hardworking nor honest are indicated by
3
(a) 5 (b) 7 (c) 10 (d) 11
2 22. Urban educated people who are neither honest nor
4 8 hardworking are indicated by
7 9 1
6 (a) 2 (b) 4 (c) 6 (d) 9
5 10

11 Directions (Q. Nos. 23-26) The following questions are


based on the diagram given below.
14. Which of the area is cultivated for wheat and maize
9
only? 4
(a) 8 (b) 6 (c) 5 (d) 4 7 13
14 16
15. Which of the area is cultivated for maize only ? 10 6
(a) 10 (b) 2 (c) 3 (d) 4 15
05
Directions (Q. Nos. 16-18) In the following diagram, the
square represents girls, the circle represents tall persons, the (i) Rectangle represents females.
triangle is for tennis players and the rectangle stands for the (ii) Triangle represents uneducated persons.
swimmers. On the basis of the given diagram, answer the (iii) Circle represents urban.
following questions. (iv) Square represents Government servants.
In different regions in the diagram some numbers are
indicated which corresponds to the persons.
E B A 23. How many persons are uneducated female?
C (a) 6 (b) 5 (c) 15 (d) 21
D
G
I 24. How many persons are Government servants but not
L F H females as well as uneducated?
K J (a) 13 (b) 7 (c) 9 (d) 20

25. How many persons are urban females but not


16. Which letter represents tall girls who are swimmers uneducated and Government servants?
but don’t play tennis? (a) 14 (b) 4 (c) 7 (d) 6
(a) C (b) D (c) G (d) H 26. How many total females which are Government
17. Which letter represents tall girl who do not play servants but not urban?
tennis and are not swimmers? (a) 26 (b) 20
(a) C (b) D (c) E (d) G (c) 13 (d) 7

18. Which letter represents tall persons who are male 27. 4
and swimmers but do not play tennis?
4 2
(a) I (b) J (c) K (d) L 1
7 7 8 2 11
Directions (Q. Nos. 19-22) These 9 6
questions are based on the following 5 15
diagram in which the circle stands for
3 2 6 7
educated, the square stands for
hard-working, the triangle stands for 4
1 8 Let X denote sum of numbers present in only on circle
9
urban and the rectangle stands for honest. 11 10
and Y denote sum of number present in any two
Different regions in the diagram are 12
circle. Then X − Y = ……… .
numbers from 1 to 12. Study the diagram (a) 8 (b) 16
carefully and answer the each question. (c) 12 (d) 19
Hints and Solutions
Elementary Level Questions
1. (b) Hindi is a language but it is not related with science. 17. (a) 28 persons are doctors but they are neither players nor artists.
2. (a) Player can be a girl or a boy. 18. (a) Numeral which do not corresponds to doctors are 20, 25 and
40 but for both artists as well as players numeral 20 is suitable,
3. (d) All are different metals.
so 20 persons are artists as well as players but not doctors.
4. (d) All are different animals. 19. (d) Authors, teachers and women are partly related to each other.
5. (d) All are different and not related to one another. 20. (c) Metals and non-metals both are elements.
6. (a) Atom contains protons as well as electrons. 21. (c) Both cats and dogs are pets.
7. (a) Milk and water both are liquids. 22. (c) Cricket and hockey both are games.
8. (d) All are different professions. 23. (c) Elephant and tiger both are animals.
9. (b) Jaipur is the capital of Rajasthan but Haryana is different 24. (a) Country contains states and state contains number of
state. districts.
10. (e) India is part of Asia and Asia is part of World. 25. (c) Rose and lotus both are flowers.
11. (a) 26. (d) Child is equally related with father and mother.
Hockey 27. (c) Zoology and Botany are branches of Biology.
Cricket
28. (d) Flowers and clothes both can be white in colour.
29. (a) The given situation can be represented as under
Football Both fish and meat

Fish Meat
So, three persons will play all games.
12. (d) 6 persons will play football as well as hockey but not cricket.
13. (a) Three numerals 1, 2 and 5 are not related with football and Vegetarian
numeral 2 is common. So, 2 persons play cricket and hockey but
not football. 30. (a) The given situation can be represented as under
Both thums up
14. (a) Only numeral 5 is related with hockey and not with cricket and and cocacola
football.
Thums Cocacola
15. (c) Part of circle not common in triangle as well as rectangle, i.e., up
numeral 25, so 25 persons are neither doctors nor artists.
16. (b) Common region of all three figures, i.e., numeral 4 containing Teetotallers
part which corresponds to doctors which are both players as
well as artists.

High Skill Questions


1. (d) Region A is common to all the three circles. Hence, it 6. (d) The region common to the triangle, square and circle
represents all these Indians who have travelled to Russia as well represents the number of self-employed female graduates. It is
as China. 8 + 5 + 7 = 20.
2. (a) Region C is common to the circles representing India and 7. (a) The region lying inside the triangle but outside the circle and
China. Hence, it represents Indians who have travelled only to
square represents the number of female graduates who are not
China but not to Russia.
self-employed. It is represented by 4.
3. (d) There is no common region between only China and Russia
in the given diagram. 8. (c) The region lying outside the triangle but inside square and
4. (b) The region D is common to India and USA. Hence, it circle represents number of non-graduates self-employed
represents those Indians who have travelled only to Russia but females. It is 3 + 6 + 2 + 1 + 9 = 21.
not to China. 9. (b) The region common to triangle and circle represents the
5. (d) The region F represents Russians who have travelled neither number of self-employed female graduates with bank loan
to China nor to India. facility. It is 7 + 5 = 12.
118 Study Package for NTSE

10. (c) The region lying outside the triangle but inside the circle 18. (c) A, I, J, K and L are not girls.
represents the number of non-graduate self-employed females Out of A, I, J, K only K and J are tall persons.
with bank loan facility. It is 6 + 2 + 1 = 9. J is not swimmer.
11. (a) Two numerals 27 and 18 are in rectangle but not in the circle, K is swimmer but do not play tennis.
so largest number will be 27. 19. (a) Common region of all four figures, i .e., 1 corresponds to
12. (a) Three numerals 10, 15 and 16 are in the triangle but not in the educated hardworking and honest urban people.
rectangle and circle. Out of these smallest number is 10. 20. (b) Urban hardworking and educated can be represented by
common region of square, triangle and circle but not related with
13. (a)
rectangle. So, number 2 will be correct answer.
40 21. (b) Non-urban educated, who are neither hard working nor
honest can be represented by region of circle which is not part of
other figures. So, number 7 will be correct answer.

Three figures common part is shaded region, which contains 22. (c) Urban educated people who are neither honest nor hard
numeral 40. working can be indicated by 6 because it must be common part
of circle and triangle.
14. (d) The required region is the one which is common only to the
rectangle and circle and is not a part of either the triangle or square, 23. (a) Uneducated female = Common region of rectangle and
i .e., 4. triangle that corresponds 6 persons.
24. (c) Government servants not uneducated and not females
15. (c) The required region is the one which lies inside the circle but
= Non-common part of square that corresponds to 9 persons.
outside the rectangle, square and triangle, i .e., 3.
25. (a) 14 persons.
16. (c) Tall girls can be represented by common region of square
26. (c) Female Government servants = 20 but 7 are urban females.
and circle. (D, C, G and H). For swimmers, G, H part is common
among the above mentioned common region. According to the ∴ Total females which are Government servant but not urban
question, if they are not tennis players, then ‘G’ region fulfill all = 20 − 7 = 13.
the conditions. 27. (b) X = 7 + 4 + 11 + 15 = 37
17. (b) Tall girls region is D, C, G, H part, out of G, H and C are either Y = 4 + 2 + 6 + 9 = 21
swimmer or tennis. So, D region fulfill all the conditions ∴ X − Y = 37 − 21 = 16
according to question.
Chapter

18
Syllogism
Venn Diagram Representation of Four Propositions
Syllogism Types of Propositions Venn Diagram
Syllogism is a greek word which means inference or A : All S is P
deduction. It is deductive argument in which conclusion S Always
has to be drawn from the given statements. P

In this type of questions, a set of statements along with E : No S is P S p Always


two or more conclusions is given on the basis of these
statements. Candidates are required to check the Either Some S are P
S p [All S are P]
correctness of conclusions whether the conclusions
follows from statement or not. or S Some S are P
I : Some S are P P [All S are P]

Propositions
or P Some S are not P
Proposition is a statement giving a relation between two S [All P are S]

terms. It is also known as premise.


Either Some S are not P
P S [Some P are not S]
Classification of Proposition
or P Some S are not P
This classification is based upon the quantity and O : Some S are not P
S [All P are S]
quality of proposition. Here, quantity denotes whether
the proposition is universal or particular and quality or S p Some S are not P
[No S are P]
denotes the proposition is affirmative or negative.
There are four types of propositions which are as Example 1 Statements All terrorists are
follows: human. All humans are bad.
Propositions Quantity Quality
Conclusions I. All terrorists are bad.
II. No human can be a terrorist.
A All A are B Universal Affirmative (a) Only Conclusion I follows
E No A are B Universal Negative (b) Only Conclusion II follows
I Some A are B Particular Affirmative (c) If neither I nor II follows
O Some A are not B Particular Negative (d) If both I and II follow
120 Study Package for NTSE

Solution (a)
Bad
Human
Figure I Validity
Terrorist Conclusion I 3 3
Conclusion II × ×

From above, only Conclusion I follows.

Example 2 Statements Some teachers are professors.


Some professors are readers.
All readers are rectors.
Conclusions I. Some readers are teachers. II. Some rectors are teachers.
III. All rectors are readers.
(a) None follows (b) Only I follows (c) Only II follows (d) All follow
Solution (a)
Teachers
Readers Figure I Figure II Validity
Teachers
Conclusion I × 3 ×
Readers
Conclusion II × 3 ×
or
Rectors Conclusion III × × ×

Hence, none of the conclusion follows.


Professors Professors
Rectors
Fig. (I) Fig. (II)

Example 3 Statements All calculators are computers.


No computer is television.
No television is radio.
No radio is transistor.
Conclusions I. No calculator is television. II. No calculator is radio.
III. No television is transistor. IV. No computer is radio.
(a) None follows except I (b) None follows except II (c) None follows except III (d) All follow

Solution (a)
Radio
r
R

ute ter
ad

p u
om mp
io

Computer C Co
Calculator Television Calculator
Calculator Television Radio Transistor or or
Television
to r
n sis
Tra
Transistor
Fig. (I) Fig. (II) Fig. (III)

Figure I Figure II Figure III Validity


Conclusion I 3 3 3 3
Conclusion II 3 × × ×
Conclusion III 3 3 × ×
Conclusion IV 3 × × ×

Hence, none follows except I.


Target Exercise
Elementary Level Questions 5. Statements
Directions (Q. Nos. 1-4) In these questions, two statements Alcoholic drinks are injurious to health.
are followed by two conclusions numbered I and II. Assume All old women drink whisky.
the given statements to be true, even, if they seem to be at Conclusions
variance with commonly known facts. Read both the I. All old women have poor health.
conclusions and then decide which of the given conclusions II. All young women are in good health.
logically follows from the given statements disregarding
commonly known facts. 6. Statements
Give Answer All ants are hardworking.
(a) If only Conclusion I follows Some ants are lazy.
(b) If only Conclusion II follows Conclusions
(c) If both the Conclusions follow I. All lazy are hardworking.
(d) If neither I nor II follows II. All hardworking are lazy.
1. Statements 7. Statements
All players are smokers. All rivers are mountains.
Some smokers are wine-addicts. Some rivers are deserts.
Conclusions Conclusions
I. All smokers are players. I. Some mountains are deserts.
II. Some wine-addicts are smokers. II. Some deserts are not mountains.
2. Statements Directions (Q. Nos. 8-12) In these questions, two
All women are ministers. statements followed by two conclusions numbered I and II.
All ministers are simpleton. Assume the given statements to be true, even, if they seem to
be at variance with commonly known facts. Read both the
Conclusions
conclusions and then decide which of the given conclusions
I. All women are simpleton. logically follows from the given statements disregarding
II. All ministers are simpleton. commonly known facts.
3. Statements Give Answer
All cars are not trains. (a) If only Conclusion I follows
(b) If only Conclusion II follows
All cars are four-wheeled vehicles.
(c) If both I and II follow
Conclusions (d) If neither I nor II follows
I. All trains are not four-wheeled vehicles. 8. Statements
II. Some trains are four-wheeled vehicles.
All books are pencils,
4. Statements All pencils are flowers.
All jails are guest houses. Conclusions
All guest houses are comfortable. I. All books are flowers.
Conclusions II. Some pencils are not books.
I. All jails are comfortable. 9. Statements
II. No jail is comfortable.
Some books are pencils.
Directions (Q. Nos. 5-7) In these questions, there are two Some pencils are pens.
statements followed by two conclusions numbered I and II. Conclusions
Decide which of the two given conclusions logically follows I. Some books are pens.
from the two given statements, disregarding commonly II. Some pens are books.
known facts. Read both the conclusions and then decide
which of the given conclusions logically follows from the 10. Statements
given statements disregarding commonly known facts. All horses are bullocks.
Give Answer All bullocks are parrots.
(a) If only Conclusion I follows Conclusions
(b) If only Conclusion II follows I. All horses are parrots.
(c) If neither I nor II follows
II. All parrots are horses.
(d) If both I and II follow
122 Study Package for NTSE

11. Statements 16. Statements


All flowers are leaves. Some years are decades.
Some leaves are birds. All centuries are decades.

Conclusions Conclusions
I. Some birds are flowers. I. Some centuries are years.
II. Some decades are years.
II. Some leaves are flowers.
III. No century is a year.
12. Statements (a) Either I or III and II follow
All windows are doors. (b) II and III follow
No door is wall. (c) Only I follows
(d) Only II follows
Conclusions
I. No window is wall. 17. Statements
II. No wall is door. All Shares are debentures.
No debenture is an equity.
Directions (Q. Nos. 13-15) In these questions, two
statements are followed by two conclusions numbered I and
Conclusions
II. Assume the given statements to be true, even, if they seem I. No equity is a share.
to be at variance with commonly known facts. Read both the II. Some debentures are shares.
conclusions and then decide which of the given conclusions III. No share is an equity.
logically follows from the given statements disregarding (a) Only I follows
commonly known facts. (b) Only II follows
Give Answer (c) All follow
(a) If only Conclusion I follows (d) Only III follows
(b) If only Conclusion II follows 18. Statements
(c) If neither I nor II follows All nets are jets.
(d) If both I and II follow All jets are sets.
13. Statements Conclusions
All markers are flude. I. All jets are nets.
All fludes are copies. II. Some sets are neither nets nor jets.
Conclusions III. No set is a net.
I. All markers are copies. (a) Only I follows
II. Some copies are markers. (b) Only III follows
(c) Only II follows
14. Statements (d) None follows
Some pens are tables. 19. Statements
No table is blue.
All cities are towns.
Conclusions Some cities are villages.
I. No pen is blue. Conclusions
II. No blue thing is pen.
I. All villages are towns.
15. Statements II. No village is a town.
Some trees are horses. III. Some villages are towns.
Some ships are trees. (a) Only III follows (b) Only I follows
(c) Only II follows (d) None of the above
Conclusions
I. Some horses are ships. 20. Statements
II. Some trees are not ships. Some tables are chairs.
Some chairs are beds.
Directions (Q. Nos. 16-20) In these questions, there are two Conclusions
statements followed by three conclusions numbered I, II and
I. Some tables are beds.
III. You have to take the given statements to be true even, if
II. No bed is either a chair or a table.
they seem to be at variance from commonly known facts and
then decide, which of the given conclusions logically follows
III. All beds are chairs.
from the given statements. (a) Only either I or II follows (b) Only III follows
(c) Only II follows (d) None follows
Syllogism 123
High Skill Questions
Directions (Q. Nos. 1-20) In these questions, there are Conclusions
statements followed by three conclusions. You have to take I. Some trucks are pencils.
the given statements to be true even, if they seem to be at II. Some buses are pencils.
variance from commonly known facts and then decide, which III. No truck is pencil.
of the given conclusions logically follows from the given
(a) Only I follows
statements. (b) Only II follows
1. Statements (c) Either I or III and II follows
(d) Either I or III follows
Some benches are beads.
All beads are flowers. 6. Statements
No tree is a flower. All stamps are packets.
Conclusions Some packets are buckets.
I. Some trees are benches. All buckets are tubes.
II. Some trees are beads. Conclusions
III. No tree is a bead. I. Some tubes are stamps.
(a) Only I follows (b) Either I or II follows II. Some buckets are stamps.
(c) Either II or III follows (d) Only III follows
III. Some tubes are packets.
2. Statements (a) None follows (b) Only I follows
All breads are tables. (c) Only II follows (d) Only III follows
Some tables are brushes. 7. Statements
All brushes are paints.
Some doors are windows.
Conclusions Some windows are lamps.
I. Some paints are breads. All lamps are candles.
II. Some brushes are breads. Conclusions
III. Some paints are tables.
I. Some candles are doors.
(a) None follows (b) Only II follows
II. Some candles are windows.
(c) Only III follows (d) I and II follow
III. Some lamps are doors.
3. Statements (a) Only I follows (b) Only II follows
Some pictures are frames. (c) Only III follows (d) I and II follow
Some frames are idols. 8. Statements
All idols are curtains.
All furnitures are jungles.
Conclusions No jungle is road.
I. Some curtains are pictures. Some roads are hills.
II. Some curtains are frames. Conclusions
III. Some idols are frames.
I. Some roads are furnitures.
(a) I and II follow (b) II and III follow
II. Some jungles are furnitures.
(c) I and III follow (d) All follow
III. Some hills are jungles.
4. Statements (a) Only I follows (b) Only II follows
Some blades are papers. (c) Only III follows (d) I and II follow
Some papers are books. 9. Statements
Some books are pens.
All bricks are stones.
Conclusions Some stones are rocks.
I. Some pens are papers. All rocks are mountains.
II. Some books are blades. Conclusions
III. Some pens are blades.
I. Some mountains are stones.
(a) Only I follows (b) Only II follows
II. Some mountains are bricks.
(c) Only III follows (d) None follows
III. Some stones are bricks.
5. Statements (a) Only I follows
Some pencils are marbles. (b) Only III follows
All marbles are buses. (c) I and III follow
Some buses are trucks. (d) All follow
124 Study Package for NTSE

10. Statements III. Some sticks are guns.


Some bags are plates. IV. Some balls are guns.
Some plates are chairs. (a) I, II and IV follow (b) II and IV follow
All chairs are tables. (c) II, III and IV follow (d) I, II and III follow

Conclusions 15. Statements


I. Some tables are plates. Some books are notebooks.
II. Some chairs are bags. Some notebooks are dictionaries.
III. No chair is bag. Some dictionaries are files.
(a) Only I follows (b) Either II or III follows All files are envelopes.
(c) I and either II or III follow (d) Only III follows Conclusions
11. Statements I. Some envelopes are notebooks.
All pens are bags. All bags are glasses. II. Some files are books.
No glass is a spoon. All spoons are books. III. Some books are dictionaries.
IV. No book is an envelope.
Conclusions
(a) None follows (b) I, II and III follow
I. Some glasses are pens. (c) II and III follow (d) III and IV follow
II. Some books are bags.
III. No spoon is a pen. 16. Statements
IV. No bag is a book. Some keys are chains.
(a) II and III follow (b) I, III and either II or IV follow All chains are locks.
(c) Either II or IV follows (d) All follow All locks are numbers.
12. Statements No number is a digit.
All petals are flowers. Conclusions
All thorns are flowers. I. Some keys are numbers.
Some leaves are thorns. II. All chains are numbers.
Some stems are flowers. III. Some locks are keys.
IV. No digit is a chain.
Conclusions
(a) I, II and III follow (b) II and III follow
I. Some petals are leaves.
(c) II, III and IV follow (d) All follow
II. All leaves are flowers.
III. Some stems are petals. 17. Statements
IV. No petal is a leaves. All graduates are advocates.
(a) None follows (b) Only II follows Some advocates are judges.
(c) II and either I or IV follow (d) Either I or IV follows All judges are lawyers.
13. Statements Some lawyers are doctors.
All buses are cars. Conclusions
Some buses are not motorcycles. I. Some doctors are advocates.
No motorcycle is scooter. II. All graduates are judges.
All scooters are tempos. III. Some doctors are graduates.
Conclusions IV. Some lawyers are advocates.
I. Some tempos are not cars. (a) Only I follows (b) Only II follows
II. Some tempos are not motorcycles. (c) Either III or IV follows (d) None of the above
III. No car is tempo. 18. Statements
IV. Some cars are not motorcycles. Some roses are flowers.
(a) Only IV follows (b) II and IV follow Some flowers are buds.
(c) I and II follow (d) None of these All buds are leaves.
14. Statements All leaves are plants.
Some umbrellas are sticks. Conclusions
Some sticks are balls. I. Some plants are flowers.
Some balls are bats. II. Some roses are buds.
All bats are guns. III. No leave is rose.
Conclusions IV. No rose is bud.
I. Some balls are umbrellas. (a) Only I follows (b) I and II follow
II. Some guns are bats. (c) I and either II or IV follow (d) Either II or IV follows
Syllogism 125
19. Statements 20. Select the conclusion which logically follows from the
All snakes are eagles. given Statements
Some eagles are rabbits. All doors are windows.
All rabbits are birds. No window is chair.
Some birds are animals. All chairs are tables.
Conclusions No table is bottle.
Conclusions
I. Some animals are snakes.
II. Some birds are snakes. I. All windows are doors.
II. Some tables are chairs.
III. Some birds are eagles.
III. Some tables are not doors.
IV. All birds are rabbits.
IV. No door is bottle.
(a) Only I follows (b) Only II follows
(a) Only I follows (b) Only II follows
(c) Only III follows (d) None follows (c) Only III follows (d) None follows

Hints and Solutions


Elementary Level Questions 4. (a)
Comfortable
1. (b)
Guest house
Wine
Smokers Smokers addicts Jail

Players Wine or
addicts Players

I
Figure Validity
II
Conclusion I 3 3
Figure I Figure II Validity Conclusion II × ×
Conclusion I × × × Only Conclusion I follows.
Conclusion II 3 3 3 5. (c) None of the conclusions follow, because it cannot be said
Only Conclusion II follows. that all old women have poor health and also we cannot say
definitely. On the basis of statements, that all young women
2. (c) Simpleton are in good health.
Ministers 6. (c)
Hardworking
Woman
Ants Lazy

Figure Validity Figure Validity


Conclusion I 3 3 Conclusion I × ×
Conclusion II 3 3 Conclusion II × ×
Both the conclusions follow. Neither Conclusion I nor II follows.
3. (d) 7. (a)
FW V Mountains

Cars Trains or Cars Trains Rivers Deserts

FW V I II

Figure I Figure II Validity Figure Validity


Conclusion I 3 × × Conclusion I 3 3
Conclusion II × 3 × Conclusion II × ×
None of the conclusion follows. Only Conclusion I follows.
126 Study Package for NTSE

8. (a) 12. (c) Doors

Windows Wall
Books

Pencils
Flowers
Figure Validity
Figure Validity Conclusion I 3 3
Conclusion II 3 3
Conclusion I 3 3
Conclusion II × × Both Conclusions I and II follow.
13. (d)
Only Conclusion I follows. Copies

9. (d) Flude
Pencils
Books Pens I Markers

or

Books Pencils Figure Validity


II Conclusion I 3 3
Conclusion II 3 3
Pens
Both Conclusions I and II follow.
14. (c)
Figure I Figure II Validity Pens Tables Blue I
Conclusion I × 3 ×
Conclusion II × 3 ×

Neither Conclusion I nor II follows. or Blue Pens Tables II


10. (a)

Figure I Figure II Validity


Horses Conclusion I 3 × ×
Conclusion II 3 × ×
Bullocks
Neither I nor II follows.
Parrots
15. (c)
Ships Trees
Figure Validity
Conclusion I 3 3 Ships Trees Horses or
Conclusion II × ×
Horses
Only Conclusion I follows. I
11. (b) II
Leaves Leaves
Figure I Figure II Validity
Flowers Birds or Flowers Birds Conclusion I × 3 ×
Conclusion II 3 × ×
Neither I nor II follows.
I II
16. (a) Decades
Figure I Figure II Validity
Conclusion I × 3 × Years Centuries I
Conclusion II 3 3 3

Only Conclusion II follows.


Syllogism 127
Decades
Centuries 20. (d)

or Tables Chairs
II
Chairs
or
Tables Beds
Years
Beds
Figure I Figure II Validity
I
Conclusion I × 3 Doubtful
II
Conclusion II 3 3 3
Conclusion III 3 × Doubtful Figure I Figure II Validity
Conclusion I × 3 ×
Here, Conclusions I and III are complementary pair and
Conclusion II is valid. Conclusion II × × ×
Conclusion III × × ×
17. (c) Debentures
None follows.
Shares Equity
High Skill Questions
1. (d)
Flowers
Figure Validity
Conclusion I 3 3 Benches Beads Tree I
Conclusion II 3 3
Conclusion III 3 3
Flowers
All follow.
18. (d) or Benches Beads II

Nets Tree

Jets
Sets Figure I Figure II Validity

Figure Validity Conclusion I × 3 ×


Conclusion II × × ×
Conclusion I × ×
Conclusion III 3 3 3
Conclusion II × ×
Conclusion III × × Only Conclusion III follows.
None follows.
2. (c) Tables Paints

19. (a) Towns Villages


Breads Brushes
Cities I
I

or
Paints
Towns

Cities Villages II or Breads Brushes II

Tables

Figure I Figure II Validity


Figure I Figure II Validity
Conclusion I × 3 ×
Conclusion I × 3 ×
Conclusion II × × ×
Conclusion II × 3 ×
Conclusion III 3 3 3
Conclusion III 3 3 3
Only III follows. Only Conclusion III follows.
128 Study Package for NTSE

3. (b) Curtains
6. (d)
Frames
Figure I Figure II Validity
Pictures Idols I
Conclusion I × 3 ×
Conclusion II × 3 ×
or Conclusion III 3 3 3
Curtains Only Conclusion III follows.

Frames 7. (b)
Idols II Windows

Doors Lamps I

Pictures
Candles
or

Figure I Figure II Validity Candles

Conclusion I × 3 ×
Windows Lamps
Conclusion II 3 3 3 II
Conclusion III 3 3 3

Conclusions II and III follow.


Doors
4. (d) Books
Blades Pens I

Papers
Figure I Figure II Validity
or Conclusion I × 3 ×
Conclusion II 3 3 3
Blades Papers
Conclusion III × 3 ×
II Only Conclusion II follows.
Books Pens 8. (b)

Figure I Figure II Validity Furnitures Road Hills I

Conclusion I × 3 ×
Conclusion II × 3 × Jungles
Conclusion III × 3 ×
None follows.
5. (c) II
or Furnitures Hills Road

Pencils Marbles Trucks I


Jungles
Buses
or Figure I Figure II Validity
Buses
Conclusion I × × ×
Pencils Marbles II Conclusion II 3 3 3
Conclusion III × 3 ×
Only Conclusion II follows.
Trucks
9. (c) Stones Mountains
Figure I Figure II Validity
Conclusion I × 3 Doubtful Bricks Rocks I
Conclusion II 3 3 3
Conclusion III 3 × Doubtful
Either I or III and Conclusion II follows.
Syllogism 129
Stones Flowers
Mountains 12. (d)

or Bricks Rocks II
Thorns Petals

Leaves

Figure I Figure II Validity


Conclusion I 3 3 3 Stems
Conclusion II × 3 ×
Figure I Validity
Conclusion III 3 3 3
Conclusion I Doubtful ×
Conclusions I and III follow. Conclusion II × ×
10. (c) Conclusion III × ×
Plates
Conclusion IV Doubtful ×
Bags Chairs I Either I or IV follows (complementary pair).
13. (d) Cars
Tables

Buses Motorcycles I
Tables Scooters

Tempos
or Bags
Chairs II
or

Cars
Motorcycles
s
Plates se
Bu

Scooters II
Figure I Figure II Validity
Conclusion I 3 3 3 Tempos
Conclusion II × 3 Doubtful
Conclusion III 3 × Doubtful Figure I Figure II Validity

Conclusion I and Either II or III follow (complementary pair). Conclusion I × × ×


Conclusion II × × ×
11. (b)
Glasses Conclusion III × 3 ×
Bags Books Conclusion IV × × ×
None follows.
Pens Spoons I
14. (b) Guns

Umbrellas Sticks Balls Bats I


or

Glasses or
Bags Guns
Books

Pens Spoons Balls Bats


II
II

Sticks Umbrella

Figure I Figure II Validity


Figure I Figure II Validity
Conclusion I 3 3 3
Conclusion II × Doubtful Conclusion I × 3 ×
3
Conclusion II 3 3 3
Conclusion III 3 3 3
Conclusion III × 3 ×
Conclusion IV 3 × Doubtful
Conclusion IV 3 3 3
I, III and either II or IV follow. Conclusions II and IV follow.
130 Study Package for NTSE

15. (a) 18. (c)


Plants

Notebooks

Dictionaries
I Leaves
Books Files

Flowers
Flowers
or Buds
Envelopes Roses Buds
or
Envelopes Leaves
Plants
Books Roses
Files I
II
II
Notebooks Dictionaries
Figure I Figure II Validity
Conclusion I 3 3 3
Figure I Figure II Validity Conclusion II × 3 Doubtful
Conclusion I × 3 × Conclusion III × × ×
Conclusion II × 3 × Conclusion IV 3 × Doubtful
Conclusion III × × × Conclusion I and either II or IV follow.
Conclusion IV 3 × × 19. (c)
Eagles Birds
None follows.
16. (d)
Snakes Rabbits Animals I

or Keys Chains
Keys Chains Digits

Locks Eagles Birds


Locks
Numbers Digits Numbers
or Snakes Rabbits II
II
I

Figure I Figure II Validity


Conclusion I 3 3 3 Animals

Conclusion II 3 3 3
Conclusion III 3 3 3 Figure I Figure II Validity
Conclusion IV 3 3 3 Conclusion I × 3 ×
All follow. Conclusion II × 3 ×
Conclusion III 3 3 3
17. (d)
Conclusion IV × × ×

I
Only Conclusion (III) follows.
Graduates Judges Doctors
20. (b)
Advocates Windows Tables
Lawyers
Windows Tables
or
Lawyers Chairs
es

Doors
at

Doors Chairs
c
vo
Ad

Judges II
Graduates
Bottles
Bottles

Doctors
I II

Figure I Figure II Validity Figure I Figure II Validity

Conclusion I × 3 × Conclusion I × × ×
Conclusion II × × × Conclusion II 3 3 3
Conclusion III × × Conclusion III × × ×
3
Conclusion IV 3 × ×
Conclusion IV 3 3 3
Conclusion II follows.
None follows.
Chapter

19
Statement and
Conclusion
The ‘conclusion’ means the facts or relevant thought that can be clearly and truly inferred from the content of the
given sentence or passage. It checks the mental ability of the candidate to analyse the statement understand their
direct and indirect implications and then decide which conclusion would logically follow from the given statement.
In this type of questions, the statement is followed by two conclusions. The candidate is required to find out which of
these conclusions would definitely follow from the given statement and state their answer in form of the given
conclusion.

Example In the following question, a statement is given Conclusions


followed by two conclusions numbered I and II choose I. Tenders are invited only from experienced contractors.
answer. II. It is difficult to find competent tenders in construction jobs.
(a) If only Conclusion I follows Solution (c) According to the given statement, tenders are
(b) If only Conclusion II follows invited from contractors experienced in executing
(c) If neither Conclusion I nor Conclusion II follows construction jobs but it is not mentioned in the statement
that tenders are invited only from experienced contractors.
(d) If both Conclusions I and II follow
So, Conclusion I does not follows. The availability of
Statement Sealed tenders are invited from competent competent tenderers in construction is not mentioned, so
contractors experienced in executing construction jobs. Conclusion II does not follow.

Target Exercise
Elementary Level Questions
Directions (Q. Nos. 1-10) In these questions, two Conclusions
statements are given followed by two Conclusions I and I. It is not raining.
II. You have to consider both the statements to be true
II. X has some urgent business to transact.
even, if they seem to be at variance from commonly known
facts. You have to decide which of the given Conclusions 2. Statements In a golf club, all the members are not active
is/are definitely drawn from the given statements. players of the game but all of them are rich. Mrs X is a
member.
Give Answer
(a) If only I follows (b) If only II follows Conclusions
(c) If neither I nor II follows (d) If both I and II follow I. Mrs X is a golfer. II. Mrs X is rich.
1. Statements When it rains, usually X does not 3. Statements All employees of company A have identity
go out. X has gone out. cards. Ram is an employee of company A.
132 Study Package for NTSE

Conclusions 7. Statements All persons who own a house or a car


I. Ram has an identity card. should file income tax return. Sheela files her income
II. Ram is the General Manager of the company. tax return.
Conclusions
4. Statements If there is shortage in the production
I. Sheela owns a house or a car.
of onions, the price of onions will go up.
II. Sheela neither owns a house nor a car.
Price of onions has gone up.
8. Statements In order to be selected for the local
Conclusions cricket team, a person should be either a good
I. There is shortage in the production of onions. batsman or a bowler. Shyam is selected for the team.
II. Onions were exported. Conclusions
I. Shyam is a good batsman.
5. Statements If all players play to their full
II. Shyam is not a good bowler.
potential, we will win the match.
9. Statements Without rains the crops will not be
We have won the match.
good. The crops were good.
Conclusions
Conclusions
I. All players played to their full potential. I. There were rains.
II. Some players did not play to their full potential. II. Crops were good due to good fertilizers.
6. Statements Some businessmen are rich. Soman is 10. Statements According to the evolution theory, man
rich. evolved from a monkey.
Conclusions X is a monkey.
I. Soman is a businessman. Conclusions
I. X can become a man.
II. Soman has a big farm.
II. Man can become a monkey.

High Skill Questions


Directions (Q. Nos. 1-4) In each of these questions a 3. Statement The manager humiliated Sachin in the
statement is given followed by two Conclusions I and II. presence of his colleagues.
Mark your answer as
Conclusions
Give Answer I. The manager did not like Sachin.
(a) If only Conclusion I follows II. Was not Sachin popular with his colleagues?
(b) If only Conclusion II follows
(c) If neither I nor II follows 4. Statement Any young man who makes dowry as a
(d) If both I and II follow condition for marriage discredits himself and
1. Statement The Prime Minister emphatically stated dishonours womanhood.
that this government will make every possible effort Conclusions
for the upliftment of poor farmers and farmhands. I. Those who take dowry in marriages should be
Conclusions condemned by the society.
I. Except poor farmers and farmhands, all others II. Those who do not take dowry in marriages respect
have got benefits of fruits of development. womanhood.
II. No serious efforts have been made in the past for
upliftment of farmers. Directions (Q. Nos. 5-10) One or two statement (s) are
given followed by two Conclusions I and II. Take the
2. Statements The distance of 900km by road
statement to be true and then decide which of the conclusions
between Bombay and Jafra will be reduced to 280 km
logically follows.
by sea. This will lead to a saving of ` 7.92 crore
per annum on fuel. Give Answer
(a) If only Conclusion I follows
Conclusions (b) If only Conclusion II follows
I. Transportation by sea is cheaper than by road. (c) If either Conclusion I or II follows
II. Fuel must be saved to the greatest extent. (d) If neither Conclusion I nor II follows
Statement and Conclusion 133
5. Statements Of the ten fisherman caught in a storm, to marked rise in prices. Large financial outlays year
nine managed to return to the shore. Praveen has not after year had little impact on level of living.
yet returned after four days. Conclusions
Conclusions
I. A boom in revenues leads to rise in prices.
I. Praveen got killed in the storm. II. Large financial outlays should be avoided.
II. Praveen has survived the storm.
9. Statements The average number of students per
6. Statement Now you don’t need import licence to teacher is 50 in the urban area whereas it is 60 in rural
own a VCR. areas. The national average is 55.
Conclusions
Conclusions
I. VCRs are now manufactured indigenously.
I. The student teacher ratio in the rural areas is
II. VCRs are now freely permitted to be imported. higher than in the urban areas.
7. Statements Just about everyone in Germany has II. More students study with the same teacher in the
been on a diet at one time or the other and millions of rural areas as compared to those in the urban areas.
them have learned that the weight they lose is all too 10. Statement The Government run company has
easily regained. Still despite their frustration, few asked its employees to declare their income and
question the wisdom of dieting. assets but it has been strongly resisted by employees
Conclusions union and no employee is going to declare his income.
I. Germans should stop dieting. Conclusions
II. Germans do not learn from experience. I. The employees of this company do not seem to
8. Statements A study of planning commission have any additional undisclosed income besides
reveals boom in revenues. However, this has been of their salary.
little avail owing to soaring expenditure. In the event, II. The employees union wants all senior officers to
there has been a high dose of deficit financing, leading declare their income first.

Hints and Solutions


Elementary Level Questions High Skill Questions
1. (c) It is given that when it rains, X does not go out, but reverse of it 1. (c) Given, both conclusions are not related to the statement,
may or may not be true. Hence, none of the conclusions is a valid which gives the right conclusions.
conclusion in the light of the statement. 2. (a) It is given in the statement that reducing distance will lead to a
saving of ` 7.92 Cr per annum on fuel. Hence, Conclusion I is
2. (b) It is clear that all the members are necessarily rich, hence
implicit, Conclusion II is a fact but not related with statement.
Mrs X, being a member, will definitely be rich.
Hence, Conclusion I follows.
3. (a) Ram being the employee of company A, has an identity card. 3. (a) The manager disliking led to humiliation of Sachin. Hence,
4. (a) Clearly, there is shortage in the production of onions. Conclusion I follows. Nothing has been given about the
popularity of Sachin. Hence, Conclusion II does not follow.
5. (a) Given, the match can be won only if all players play to their full
4. (b) We can easily conclude from the statement that those who do
potential. Since, match has been won this implies that all players
not take dowry in marriages respect womanhood.
played to their full potential.
5. (c) Either Conclusion I or II follows, since there one of two
6. (c) It is given that some businessman are rich which implies that possibilities can be possible either Praveen got killed in the
some rich are businessman. Hence, man may or may not be a storm and not come back or Praveen has survived but he is not
businessman. getting any route to come back.
7. (c) Besides owning a car or house there may be other 6. (b) Statement implies that VCR can be imported but no tax is
requirements for filling an Income Tax. Hence, nothing can be applicable, it does not mean that VCR are manufactured in the
concluded on the basis of statement given. country. Hence, only Conclusion II follows.

8. (c) No conclusion is logically correct as Shyam is selected, it 7. (d) Both the conclusions are invalid in the light of the statement
given.
implies that he is a good batsman or a bowler.
8. (d) None of the conclusions follows.
9. (a) Clearly, Conclusion I follows.
9. (b) From the given information, it is easily concluded that more
10. (c) Both the conclusions are invalid as nothing can be said in students study with the same teacher in the rural areas as
view of the statement. compared to those in the urban areas.
10. (d)
Chapter

20
Data Sufficiency
Data sufficiency is a method to analyse the given set of Now, from both statements,
information and decide whether the given information Eit
he
is sufficient to answer the question. Data sufficiency R rs
on
questions include problems based on Coding-Decoding, or
Son da
Blood-Relations, Direction Sense Test, Ranking and ug
hte
Time Sequence Test, Arithmetical Reasoning, Brother r
Q P
Mathematical Operations and Puzzle Test etc.
Here, P is either the son or daughter of R. So, we cannot
The problems based on this topic consist of two or more find the exact relation between P and R.
than two statements containing the information related Hence, Statements I and II together are not sufficient to
to it. You have to decide whether the problem can be answer the question.
solved by using the information from the given Illustration 2. Who is older Ritu or Situ?
statements combined or individually.
I. Situ was born in the year 1980 and she is 5 yr younger
In these questions, it is also possible that either of the than Ritu’s brother.
given statements is sufficient to answer the question. II. Ritu is twice as old as her brother.
Solution (c) From Statement I, it is clear that Situ is
Directions (Illustrations 1-3) The questions given below younger than Ritu's brother.
consist of a question followed by two statements labelled as I
and II. We have to decide whether these statements give From Statement II, it is clear that Ritu is older than her
brother.
enough information required to answer the question or not.
So, from both the statements, we get
Give Answer Ritu > Ritu’s brother > Situ
(a) If Statement I alone is sufficient to answer the question
Hence, Ritu is older than Situ.
(b) If Statement II alone is sufficient to answer the question
(c) If Statements I and II together are needed to answer the Illustration 3. Six persons A, B, C, D, E and F are sitting in
question a row facing North. A and F are sitting at two extreme ends of
(d) If Statements I and II together are not sufficient to the row. B is to the immediate right of A and D is 2nd left of F.
answer the question What is the position of E with respect to A?
Illustration 1. How is P related to R? I. E is to the right of A.
I.Q is the son of R. II. Q is the brother of P. II. E is to the left of C.

Solution (d) Statement I Solution (b) On the basis of the statements given in the
question, we can arrange the persons in a row in the
R
following way
Son A B C or E D C or E F

Q 1 2 3 4 5 6
Positions of C and E can be either 3rd or 5th.
Statement II
From the Statement I, we get that E is to the right of A
Q Brother P and hence, position of E can either be 3rd or 5th.
Data Sufficiency 135
Therefore, Statement I alone is not sufficient to answer II. Suresh went to Chennai the next day of the day his
the question. Statement II fixed the positions of C at 5th mother came to his house.
and E at 3rd place, respectively. Therefore, we get our
III. Suresh’s mother came to his house neither on Monday
answer from Statement II.
nor Friday.
Direction (Illustration 4) In the question given below, (a) II and III
three Statements I, II and III are given. You are required to (b) I and II
find out which of the given statements is/are sufficient to (c) I and III
answer the question. (d) Even I, II and III together are not sufficient
Illustration 4. On which day Suresh went to Chennai, if week Solution (d) Statements I, II and III together are not
starts on Monday? sufficient as the required day can be anyone of
I. Suresh took leave on Wednesday. Wednesday, Thursday, Friday and Sunday.

Target Exercise
Elementary Level Questions
Directions (Q. Nos. 1-5) Each of the following question I. P is the mother of Q. Q is the son of R. R is the
below consists of a question and two statement numbered I son of T.
and II. You have to decide whether the data provided in the II. L is the father of N and N is the daughter of T.
statements are sufficient to answer the question.
5. Are all the five friends viz. Leena, Amit, Arun, Ali and
Give Answer Ken who are seated around a circular table facing the
(a) If the data in Statement II alone is sufficient to answer the centre?
question, while the data in Statement I alone is not sufficient to
answer the question Statements
(b) If the data either in Statement I alone or in Statement II alone I. Leena sits second to the left of Amit. Amit faces
is sufficient to answer the question the centre. Arun sits second to the right of
(c) If the data in both Statements I and II together are necessary Leena.
to answer the question
(d) If the data in both the Statements I and II together are not II. Ali sits third to the left of Ken. Ken faces the
sufficient to answer the question centre. Amit sits to the immediate left of Ali but
Ken is not an immediate neighbour of Amit.
1. Who amongst L, M, N, O and P is the shortest?
Statements Directions (Q. Nos. 6-12) Each question consists of a
question and two statements numbered I and II given below
I. O is shorter than P, but taller than N.
it. You have to decide, whether the data provided in the
II. M is not as tall as L. statements are sufficient to answer the question. Read both
2. Point A is towards which direction from Point B? the statements and mark the appropriate answer.
Statements Give Answer
I. If a person walks 4 m towards the North from (a) If the data in Statement I alone is sufficient to answer the
Point A and takes two consecutive right turns, question while the data in Statement II alone is not sufficient
to answer the question
each after walking 4 m, he would reach Point C,
(b) If the data either in Statement I alone or in Statement II alone
which is 8 m away from Point B.
is sufficient to answer the question
II. Point D is 2 m towards the East of point A and
(c) If the data in both statements together are necessary to
4 m towards the West of point B. answer the question
3. How many brothers does Bharat have? (d) If the data in Statement II alone is sufficient to answer the
question while the data in Statement I is not sufficient to
Statements
answer the question
I. Shiela, the mother of Bharat, has only three
children.
II. Meena, the grandmother of Bharat, has only one 6. How many sisters does Madhu have?
granddaughter.
Statements
I. Madhu’s parents have four children.
4. Is T the grandmother of Q?
II. Madhu has three brothers.
Statements
136 Study Package for NTSE

7. Is R the granddaughter of C? II. B sits second to right of A. E is not an


Statements immediate neighbour of D.
I. The only sister of A is the mother of R’s 11. How many people are standing in a straight line
brother, B. (Note: All are facing North)?
II. C, the mother of A has only one grandson, B. Statements
8. Four friends A, B, C and D are seated in circle facing I. U stands third from the left end of the line. U is
the centre but not necessarily in the same order. Is an immediate neighbour of P and W. Only one
anyone seated exactly between C and D, when counted person stands between W and T. Only two
from the left of C? people stand to the right of W.
Statements II. S stands at extreme left end of the line.
I. B is seated to the immediate right of C. T stands to the extreme right end of the line.
Only one person stands between S and U.
II. B is seated to the immediate left of A. D is not an
Only one person stands between T and W.
immediate neighbour of B.
12. How far is point M from point K?
9. Among five friends M, N, O, P and Q (each earning a
different amount), who earns the least? Statements
Statements I. Point D is 5m to the South of Point P. Point M
is 8m to the West of point D. Point S is 2.5m to
I. M earns more than O, P and N.
the North of point M. Point O is 10m to the east
II. P earns more than only O. of Point S. Point K is 2.5m to the South of point
10. Among A, B, C, D and E, seated in a straight line, but O.
not necessarily in the same order, facing North, who II. Point K is 10m to the East of point M. Point U
sits exactly in the middle of the line? is 8m to the West of point M. Point D is to the
East of M. Point M is the midpoint of the lines
Statements formed by joining points U and D.
I. A sits third to left of D. B sits to the immediate
right of C.

High Skill Questions


Directions (Q. Nos. 1-7) Each of the following question below Statements
consists of a question and two statements numbered I and II I. Point P is in East of Point L. Point L is in
given below it. You have to decide whether the data provided in North of Point M. Point M is in West of Point
the statements are sufficient to answer the question. Read both N. Point N is in South of Point O. Point O is
the statements and give answer. in West of Point Q.
Give Answer II. Point L is in West of Point P and North of Point
(a) If the data in Statement II alone is sufficient to answer the M. Point M is in East of Point N which is South
question, while the data in Statement I alone is not sufficient to
of Point O. Point Q is in East of Point O and
answer the question
(b) If the data either in Statement I alone or in Statement II alone is North of Point L.
sufficient to answer the question 3. Six persons viz. A, B, C, D E and F are sitting on a
(c) If the data in both Statements I and II together are necessary to circular table for lunch, who among them sits
answer the question immediate left of F (If all the persons are facing
(d) If the data in both the Statements I and II together are not
towards the centre)?
sufficient to answer the question
Statements
1. How is ‘also’ written in a code language?
I. A sits third of right of F. One person sits
Statements between A and C. E sits third to left of C.
I. ‘he also show data’ is written as ‘sx fa mn ca’ and II. Only one person sits between E and D. F sits
‘now many person also’ is written as ‘zb ct sx ya’
second to right of D. A sits third to left of F.
in that code language.
II. ‘she visit the also’ is written as ‘sx lm nc ty’ and 4. Sharvan’s birthday is in which of the following
‘she visit the always’ is written as ‘lm kc ty nc’ in month in a year?
that code language. Statements
2. Point Q is in which direction with respect to Point P?
Data Sufficiency 137
I. Sharvan’s mother correctly remembers that (d) If all the statements are sufficient enough to answer the
Sharvan’s birthday will come after September question
but not in the month which has 30 days. 8. Which of the following represents ‘come’ in a code
II. Sharvan’s father correctly remember that language?
Sharvan’s birthday will not come in first and last Statements
month of the year.
I. ‘pit na ja od’ means ‘you may come here’ in that
5. How far and in which direction is Point B with respect language.
to Point A?
II. ‘ja ta ter’ means ‘come and go’ in that code
Statements language.
I. Point G is 6 m to the East of Point A. Point C is III. ‘od na pit ter’ means ‘you may go home’ in that
9 m to the North of Point G. Point F is 3 m to code language.
the West of Point C. Point B is 6 m away from
Point F. 9. How many sons does D have?
II. Point M is 8 m to the West of Point B. Point R is 8 Statements
m to the South of Point M. Point A is 11 m to the I. B and F are brothers of A.
East of Point R. Point C is to the North-East of II. C is the sister of A and F.
Point A. III. C and E are daughters of D.
6. Amongst the people viz. A, B, C, D, E and F sitting
around a circular table facing the centre, who sits Directions (Q. Nos. 10-12) Each of the question given
second to the right of A? below consists of a question and three statements numbered I,
Statements II and III. You have to decide whether the data provided in
the statements are sufficient to answer the question.
I. A sits second to the right of F. Only two people
sit between A and D. B is neither an Give Answer
(a) If the data in Statements I and II are sufficient to answer the
immediate neighbour of D nor F.
question while the data in Statement III are not required to
II. Only one person sits between A and F (either answer the question
from left or right). Only two people sit between F (b) If the data in Statements I and III are sufficient to answer the
and B. C sits to the immediate left of D. D is not question, while the data in Statement II are not required to
an immediate neighbour of B. answer the question
(c) If the data in Statements II and III are sufficient to answer the
7. Among six people viz. C, D, E, F, G and H, sitting in a question, while the data in Statement I are not required to
straight line with equal distance, between each other answer the question
and facing North, who sits second to the left of G? (d) If the data in all the Statements I, II and III together are
Statements necessary to answer the question

I. C sits third from the left end of the line Only 10. Among six people viz. P, Q, R, S, T and V each lives on
one person sits between C and H. Only two a different floor of a six storey building having six
people sit between E and G. floors numbered one to six (the ground floor is
G sits at one of the position to the right of E. numbered1, the floor above it, number 2 and so on
II. E sits third to the left of G. G does not sit at any and the topmost floor is numbered 6). Who lives on
extreme end of the line. More than three people the topmost floor?
sit between H and F. F sits at one of the positions Statements
to the right of H.
I. There is only one floor between the floors on
Directions (Q. Nos 8 and 9) Each of the question below which R and Q live, P lives on an even numbered
consists of a question and three statements numbered I, II floor.
and III. You have to decide whether the data provided in the II. T does not live on an even numbered floor. Q lives
statements are sufficient to answer the questions. on an even numbered floor. Q does not live on the
topmost floor.
Read all the three statements and give answer
(a) If the Statements II and III are sufficient to answer the III. S lives on an odd numbered floor. There are two
question, but Statement I alone is not sufficient to answer the floors between the floors on which S and P live.
question T lives on a floor immediately above R’s floor.
(b) If all the Statement taken together are not sufficient to 11. There are six letters W, A, R, S, N and E. Is
answer the question
‘ANSWER’ the word formed after performing the
(c) If the Statements I and II are sufficient to answer the
question, but Statement III is not sufficient to answer the following operations using these six letters only?
question. Statements
138 Study Package for NTSE

I. E is placed fourth to the right of A. S is not Statements


placed immediately next to either A or E. I. Point A is to the West of point B. Point C is to the
II. R is placed immediately next (either left or right) North of point B. Point D is to the South of point
to E. W is placed immediately next (either left or C.
right) to S. II. Point G is to the South of point D. Point G is 4 m
III. N and W are placed immediately before and from point B. Point D is 9 m from point B.
after S. The word does not begin with R. A is not III. Point A is to the West of Point B. Point B is
placed immediately next to W. exactly midway between points A and E. Point F
12. Point D is in which direction with respect to point B? is to the South of Point E. Point D is to the West
of point F.

Hints and Solutions


Elementary Level Questions
1. (d) From Statement I, P > O > N 5. (b) From Statement I, Arun
From Statement II, L > M
Amit
Here, we cannot find the shortest and tallest by both the
statements. Hence, the data in both the statements are not
sufficient to answer the question.
Leena
2. (a) From Statement II,
N Hence, Leena faces outside.
From Statement II,
W E Ali
A 2m D 4m B
S
Amit
Now, it is clear form the diagram that, Point A is in West direction
from Point B.
Hence, Statement II alone is sufficient. Ken

3. (c) From both Statements it is clear that Meena, the grandmother


of Bharat has only one granddaughter and Shiela the mother of Hence, Ali faces outside.
Bharat has only three children. It means one is Bharat, second It is clear from diagrams that all the five friends are not facing
his sister and third one definitely his brother because the centre of circular table.
grandmother has only one granddaughter, So, Bharat has one Hence, either Statement I or II alone is sufficient to answer the
brother. Hence, both the statements together are necessary. question.
4. (c) From Statement I, 6. (c) From Statement I, Madhu has three siblings.
T
Son From Statement II, Madhu has three brothers.
Couple So, from both statements, we can say that Madhu has no sister.
P R
s r
7. (c) From Statements I and II,
Mother C–
n

Mother
So

Q
r

A
From Statement II, Sister
Married
Couple Mother
T L

Da B+
Father

R
u gh Brother
te
r Hence, R is the granddaughter of C.
N
8. (d) From Statement I,
From Statement I, it is clear that T is either the grandmother or
grandfather of Q. B
And, from Statement II, it is clear that T is female. So, T is the
grandmother of Q. Hence, both the statements together are C
necessary.
Data Sufficiency 139
From Statement II, From Statements I and II,
D
E A C B D
C A
Hence, C sits exactly in the middle of the line.
B 11. (a) From Statement I
P U W T
So, from Statement II, we can say that there is no person
between C and D. From, Statement II,
S U ....... W T
9. (d) From Statement I,
M > O, P, N Hence, data in Statement I alone is sufficient to answer the
Nothing is given about Q. question, while data in Statement II is not.
From Statement II, 12. (b) From Statement I,
...... P > O 10 m P N
S O
Here, P earns more than only O. So, O earns least. Hence, only
2.5 m 5 m 2.5 m W E
Statement II is sufficient to answer the question.
M D K
8m
10. (c) From Statement I, S
or
E A C B D A C B D E Clearly, Point M, is 10m away from Point K.
From Statement II, Point K is 10m to the East of Point M.
From Statement II, E is not an immediate neighbour of D and B
Hence, data in either of the two statements is sufficient to answer
sits second to the right of A.
the question.

High Skill Questions


1. (b) From Statement I, A A

he also show data sx fa mn ca


E C or C E
now many person also zb ct sx ya

∴ also = sx F F
From Statement II, From Statement II,
F
she visit the also sx lm nc ty
she visit the always lm kc ty nc

∴ also = sx D
E
Hence, the data in either Statement I or II in Statement II alone is A
sufficient to answer the question. Now, from Statements I and II,
2. (a) From Statement II, F
O Q B C
N-W N N-E
W E D
L P E
S-W S-E A
S
N M Clearly, C sits to the immediate left of F.
Hence, data in both the statements are required to answer the
Clearly, Q is in North-West direction with respect to P.
question.
Hence, data in Statement II alone is sufficient.
4. (c) From Statement I,
3. (c) From Statement I,
Months according to Sharvan’s mother = October, December
From Statement II,
140 Study Package for NTSE

Months according to Sharvan’s father = February to November Hence, C sits second the left of G. The data in both Statements
∴From Statements I and II, I and II are together necessary to answer the question.
Sharvan’s birthday = October. 8. (c) From Statement I,
Hence, the data in both the statements are necessary to answer You may come here → pit na ja od
the question. From Statement II,
5. (a) From Statement I, come and go → ja ta ter
B 6m F 3m C 3m B From Statements I and II together, come → ja
Hence, it is clear that Statements I and II together are sufficient to
N answer the question.
6m 9m
9. (b) From all the three statements, the gender of A is not clear. So,
W E we cannot determine how many sons D have.
B
S 10. (d) From the given statements, we get
A 6m G Floors People
6 P
We cannot determine the position of B.
5 T
From Statement II, 4 R
M 8m B 3 S
2 Q
1 V
8m C Therefore, the data in Statements I, II and III together are
necessary to answer the question.

R 11 m A 11. (b) From Statement I,


A _ S _ E _ or _ A_ S_ E
So, B is in the North-West direction with respect to A.
From Statement II,
Thus, Statement II alone is sufficient to answer the question. E R _ or R E and S W or W S
6. (a) From Statement I, From Statement III, N S W or W S N
B A is not placed next to W and the word does not begin with R.
C/E From Statements I and III together,
ANSWER
D A Hence, data only in Statements I and III are sufficient to answer the
question, while the data in Statement II is not required to answer
the question.
C/E
F 12. (b) From Statement I,
C C
C or E is second to right of A. ↑ ↓
A ← B, D
From Statement II,
From Statement III,
D
A ← B → E
F ↓
D ← F
E C By combining Statements I and III, we get
C

B A B E
A
D F
C is sitting second to the right of A.
Hence, Statement II alone is sufficient to answer the question. So, the data in Statements I and III are sufficient to answer the
question, while the data in Statement II is not required to answer
7. (c) From Statements I and II, the question.

H E C D G F
Part-II : Non-Verbal Reasoning
Chapter

1
Series
This is the most important section of non-verbal reasoning. The word series is defined as anything that follows to
form a specific pattern or as continuation of a given pattern or sequence. In simple terms, after observing the trend
of pattern involved in three or four items (figures/designs), you have to decide which will be the next figure to
complete the given series.
In these questions a series of figures is given as problem figures and the candidates are asked to select one of the
figure from the set of answer figures which will continue the given series.
For solving such series problems, some important points must be kept in mind to solve such problems correctly and
rapidly.
l In series questions, figures may move in clockwise or anti-clockwise direction by 45°, 90° or 135°. So, observe the question
figure carefully.
W CW
AC 45° 45°
90° 90° 45° 45°
90° 90°

Clockwise Anti-clockwise
(CW) (ACW)

l In series questions, the design can be took place by the addition or deletion of lines or some design, so observe the change
carefully and answer the question correctly.
l In some problems of such series, problem figure (first) is same as the problem figure (third), then the answer figure must
be similar to (second) figure.
l In some cases, one element of the figure is moved by clockwise and other is moved by anti-clockwise, answer must be
drawn by following both the moves.
l In some questions, mirror images are used to frame a series of figures. This can be understood by careful watching of the
following figure.

l In some cases, mirror images of digits or English alphabets are also used in series.

Example 1
Problem Figures Answer Figures

(a) (b) (c) (d)

Solution (d) Answer figure (d), is correct in the series because in each subsequent step one arm as well as one
dot is increasing, so answer figure must have six arms and four dots.
142 Study Package for NTSE

Example 2
Problem Figures Answer Figures

(1) (2) (3) (4) (5) (a) (b) (c) (d)

Solution (d) From problem figure (1) to (2), two inner line segments disappear similar from problem figure (2)
to (3) two line segments disappear similarly. Again from problem figure (4) to (5) two line segments will
disappear. Therefore, in answer figure, there should be only four line segments.

Target Exercise
Elementary Level Questions
Directions (Q. Nos. 1-30) In each of the question, four problem figures are given. Select a figure from amongst the
answer figures which will continue the same series as given in the problem figures.
Problem Figures Answer Figures

1. ?

(a) (b) (c) (d)

2. ?

(a) (b) (c) (d)

3. ?

(a) (b) (c) (d)

4. ?

(a) (b) (c) (d)

5. ?

(a) (b) (c) (d)

6. ?

(a) (b) (c) (d)

7. ?

(a) (b) (c) (d)


Series 143
Problem Figures Answer Figures

8. ?

(a) (b) (c) (d)

9. ?

(a) (b) (c) (d)

10. ?

(a) (b) (c) (d)

+
11. ?
+

+
+
+

(a) (b) (c) (d)

12. ?

(a) (b) (c) (d)

13. ?

(a) (b) (c) (d)

14. ?

(a) (b) (c) (d)

15. ?

(a) (b) (c) (d)

16. ?

(a) (b) (c) (d)

X
X X
17. X ? X
X
(a) (b) (c) (d)

18. ?

(a) (b) (c) (d)


144 Study Package for NTSE

Problem Figures Answer Figures

19. ?

(a) (b) (c) (d)

20. ?

(a) (b) (c) (d)

21. ?

(a) (b) (c) (d)

?
22.

(a) (b) (c) (d)

?
23.

(a) (b) (c) (d)

?
24.

(a) (b) (c) (d)

25. ?

(a) (b) (c) (d)

26. ?

(a) (b) (c) (d)

27. ?

(a) (b) (c) (d)

28. ?

(a) (b) (c) (d)


Series 145
Problem Figures Answer Figures

29. ?

(a) (b) (c) (d)

30. ?

(a) (b) (c) (d)

High Skill Questions


Directions (Q. Nos. 1-20) In each of the questions, five problem figures are given. Select a figure from the answer
figures which will continue the same series as given in the problems figure.
Problem Figures Answer Figures

×
×

×
1.

×
× ?
×
×

× ×

(a) (b) (c) (d)

2. ?

(a) (b) (c) (d)

3. ?

(a) (b) (c) (d)

4. ?

(a) (b) (c) (d)

5. ?

(a) (b) (c) (d)

6. ?

(a) (b) (c) (d)

7. ?

(a) (b) (c) (d)

R
8. ?
(a) (b) (c) (d)

9. ?

(a) (b) (c) (d)


146 Study Package for NTSE

Problem Figures Answer Figures

10. ?
(a) (b) (c) (d)

11. ?

(a) (b) (c) (d)

12. ?

(a) (b) (c) (d)

13. ?

(a) (b) (c) (d)

14. ?

(a) (b) (c) (d)


S C C S
15. C S C S ? C S
S S C S C C
(a) (b) (c) (d)

16. ?

(a) (b) (c) (d)

17. ?

(a) (b) (c) (d)

18. ?

(a) (b) (c) (d)

S
19. C S ? S S S S
S
(a) (b) (c) (d)

20. ?

(a) (b) (c) (d)

Directions (Q. Nos. 21 and 22) In the following questions, find which figure is to be removed, starting from A.
So, that all fit into a pattern.
21. 22.

A B C D E A B C D E
(a) B (b) C (a) B (b) C
(c) E (d) D (c) D (d) E
Hints and Solutions
Elementary Level Questions
1. (a) In each subsequent step, one quarter part of square as 17. (b) × moves in clockwise while shaded part moves in
well as number of dots increases. anti-clockwise direction.
2. (b) In each subsequent step, one inner circle is removed in 18. (b) In each figure, one line is removed in the design
clockwise direction. subsequent.
3. (b) Figure is moving 90° in clockwise. 19. (c) In each figure, one arrow is added in the design subsequent.
4. (b) As, figure (ii) and (iii) is similar in the same way. Answer 20. (b) Figure moves in 90° anti-clockwise direction.
figure (b) is similar to figure (i).
21. (a) In each successive problem figure, a new design is added
5. (c) One flower lob is removing in clockwise direction and it get and circle at the top of the design comes at the bottom.
converted into triangle in front of the removing lob.
22. (b) In each successive problem figure, a new arrow is added.
6. (c) Black dot moves two places in anti-clockwise direction. 23. (c) In each successive problem figure, bigger circle inside the
7. (a) Figure moves 90° in clockwise direction. smaller circle.
8. (d) Figure moves 90° in clockwise direction. 24. (c) In each successive problem figure, cross mark gets
accompanied with black circle followed by removal and addition
9. (d) In each subsequent step, two small lines are added to a
in subsequent stage.
horizontal line.
10. (a) Shapes in figure moves right to left. 25. (a) In each successive problem figure, the flags moves one,
two, three and four steps in clockwise direction.
11. (a) + moves in anti-clockwise while shaded part in clockwise
direction.
26. (c) In each successive problem figure, a line is added which get
transformed into rectangle in subsequent step.
12. (a) In each subsequent step, black shaded square get
converted into white.
27. (b) One dash is added in opposite direction in each step while
the main figure takes a similar shape every three steps.
13. (c) In each subsequent step, shaded part increases.
28. (a) Similar figure appears alternately while the number of
14. (b) Figure moves in clockwise direction. dashes decreases by one in each step.
15. (b) Figure moves 90° in anti-clockwise direction. 29. (a) In each step one line segment is reduced from the upper
16. (a) Figure (a) is correct answer, because one by one each figure and one line segment is added to the lower figure.
arrow rotate by 180° in each subsequent step. 30. (a) In each step a plus is disappeared and a small circle is
added inside the circle.

High Skill Questions


1. (b) Square rotates 45° anti-clockwise on its axis while arrow 7. (c) One of the line segments from the lower and upper bars
rotates 45° anti-clockwise in each subsequent figure. reverse its direction alternately and it changes into T-shaped
design on the lower bar and into pin-head on the upper bar.
2. (d) The shape design moves anti-clockwise alternatively while
the triangles rotates 180° and 90° anti-clockwise alternatively in 8. (d) From problem figure (i) to (ii), the designs interchange
each subsequent figure. position and the larger design becomes smaller and the smaller
design becomes larger after being reversed.
3. (c) In each subsequent figure, one arrow is added and the
pre-existing arrow(s) is/are reversed. 9. (b) From problem figure (1) to (2), the main design rotates
45° clockwise. Similar, problem figure (5) would rotates to give
4. (b) In one step, the number of arrows increases by one and in
answer figure. Again, both dots interchange sides and remain
the second step, the number of circle decrease by one.
inside the main design in the first step and in the second step
These two processes are repeated alternately while in each
both dots come out of the main design.
step the entire figure rotates 90° clockwise.
10. (c) In the first step, one arc starting from upper left gets inverted
5. (d) In each subsequent figure, an arrow is inverted starting and in the second step two arcs get inverted. These two
from left. processes are repeated alternately and the process of inversion
6. (b) In every alternate figure, the outer design is circle. Thus, in takes place in clockwise direction.
the answer figure, the outer design would be circle. Therefore, 11. (c) In the first step, the lower triangle is reversed, in the second
answer figure (c) and (d) can be ruled out. Again, in each step, the middle triangle and in the third step, the upper triangle
subsequent figure, the outer design of the previous figure is is reversed. The same procedure is repeated to get the answer.
reduced and becomes the inner figure.
148 Study Package for NTSE

12. (b) In each step, one more part of the square gets blackened. In the 17. (d) In each successive problem figure, second design get
first step, the previous shaded part of the square moves two steps enlarged and is placed on LHS while larger figure gets
and in the next step, it moves three steps in clockwise direction. reduced and is placed on RHS.
These two steps are repeated alternately and the other parts of the
18. (b) In each successive problem figure, larger part reduces in
square just behind the previous one get shaded.
size while smaller part enlarges in size and whole figure gets
13. (a) In each subsequent figure, the entire design rotates laterally inverted.
90° clockwise and three and four arrow heads are reversed at
19. (d) In each successive problem figure, the designs move one
their position respectively.
step clockwise. The designs on the clockwise side is
14. (d) In each successive problem figure, the two designs of replaced by a new one in first step and in the next step, the
upper row and lower interchange their positions alternately and designs on the clockwise side is lost and new design
the remaining four designs move one step anti-clockwise. So, appears on the anti-lockwise side. The lost or replaced
in the next step the elements of the upper row will interchange. design does not reappear. So, the possibility of (c) is ruled
out.
15. (d) In each successive problem figure, alternate designs (namely
C, O and S) move one step clockwise while the remaining three 20. (b) In each successive problem figure, a new design
move one step anti-clockwise. appears at the left most position on the lower part and the
lower left design moves to upper left, upper left to lower
16. (c)
middle, lower middle to upper right and upper right to right
to lower right and lower right design is lost.
21. (d) In each successive figure the black circle moves one step
clockwise and the white circle moves one step
From I to II From II to III anti-clockwise. So, in the given series figure D must be
From III to IV From IV to V removed to make the sequence correct.
From V to VI 22. (b) In each successive figure a design is added in
The changes in the position of designs are taking place in the clockwise direction. So, in this sequence figure C have to
following manners. be removed to make the sequence correct.
Chapter

2
Analogy
Analogy literally means ‘similarity’ i.e., having similar features or characteristic. This section of non-verbal
reasoning has been designed to test the ability of a candidate to understand the relationship between two figures
which follow a certain rule and apply the same rule to select the figure which establishes the same relationship
with the figures asked in the questions.
In these types of questions, two sets of figures namely problem figures and answer figures are given. The set of
problem figures consists of two parts. The first part comprises of two figures which have same relationship between
them on the basis of certain rule. The second part comprises one figure and a sign of question mark (?). Candidates
are asked to select one figure from the set of answer figures which replaces the sign of questions mark (?). In such a
way that it bears same relationship with the other figure as first figure of the first part bears with second figure of
the same part.
Examples given below, will give you a better idea about the types of questions generally asked.

Directions (Examples 1 and 2) Study the relationship between the first and second figure. Following the same relationship,
change the third figure to get fourth figure. Find the answer figure.

Example 1 Problem Figures Answer Figures

(a) (b) (c) (d)

Solution (c) In the above example, first figure is moved 135°, in anti-clockwise direction to get second figure.
Applying the same rule on third figure, we get the answer figure i . e., figure (c) of answer figures, which is the
correct answer.

Example 2 Problem Figures Answer Figures

(a) (b) (c) (d)

Solution (c) In the second unit, the figure rotates 90° in clockwise direction and its shaded inverted image is
placed over it. This rule is also observed in third figure (c) of answer figures. So, option (c) is correct.
150 Study Package for NTSE

Target Exercise
Elementary Level Questions
Directions (Q. Nos. 1-30) Each of the following questions consists of two sets of figures. Figures (A), (B), (C) and (D)
constitute the problem set while (a), (b), (c), and (d) constitute answer sets. There is a definite relationship between figures (A)
and (B), establish a similar relationship between figures (C) and (D) by choosing a suitable figure from answer figures.

Problem Figures Answer Figures

1.
?

(A) (B) (C) (D) (a) (b) (c) (d)

2.
?

(A) (B) (C) (D) (a) (b) (c) (d)


×
3. ×

× ? ×
× ×
(A) (B) (C) (D) (a) (b) (c) (d)

4.
?

(A) (B) (C) (D) (a) (b) (c) (d)

5.
?

(A) (B) (C) (D) (a) (b) (c) (d)

6.
?

(A) (B) (C) (D) (a) (b) (c) (d)

7.
?

(A) (B) (C) (D) (a) (b) (c) (d)

8.
?

(A) (B) (C) (D) (a) (b) (c) (d)


Analogy 151
Problem Figures Answer Figures
9.
?

(A) (B) (C) (D) (a) (b) (c) (d)

10.
?

(A) (B) (C) (D) (a) (b) (c) (d)

11.
?

(A) (B) (C) (D) (a) (b) (c) (d)

12.
?

(A) (B) (C) (D) (a) (b) (c) (d)

13.
? × × × ×
×
(A) (B) (C) (D) (a) (b) (c) (d)

14.
?

(A) (B) (C) (D) (a) (b) (c) (d)

15.
?

(A) (B) (C) (D) (a) (b) (c) (d)

16.
?

(A) (B) (C) (D) (a) (b) (c) (d)

17.
?

(A) (B) (C) (D) (a) (b) (c) (d)

18.
?

(A) (B) (C) (D) (a) (b) (c) (d)


152 Study Package for NTSE

Problem Figures Answer Figures


19.
?

(A) (B) (C) (D) (a) (b) (c) (d)


20.
?

(A) (B) (C) (D) (a) (b) (c) (d)

21.
?

(A) (B) (C) (D) (a) (b) (c) (d)

22.
?

(A) (B) (C) (D) (a) (b) (c) (d)

23.
?

(A) (B) (C) (D) (a) (b) (c) (d)


P M K
24. P M K N K N K N K N
?
R B
R S S L B B B B
(A) (B) (C) (D) (a) (b) (c) (d)

25.
?

(A) (B) (C) (D) (a) (b) (c) (d)

+
26.
?
+ +
(A) (B) (C) (D) (a) (b) (c) (d)

27.
?

(A) (B) (C) (D) (a) (b) (c) (d)

28.
?

(A) (B) (C) (D) (a) (b) (c) (d)


Analogy 153
Problem Figures Answer Figures
29.
?

(A) (B) (C) (D) (a) (b) (c) (d)

30.
?

(A) (B) (C) (D) (a) (b) (c) (d)

High Skill Questions


Directions (Q. Nos. 1-23) Each of the following questions consists of two sets of figures. Figures (A), (B), (C) and (D)
constitute the problem set while (a), (b), (c) and (d) and constitute answer sets. There is a definite relationship between figures
(A) and (B), establish a similar relationship between figures (C) and (D) by choosing a suitable figure from answer set.
Problem Figures Answer Figures

1. ?

(a) (b) (c) (d)


(A) (B) (C) (D)

2.
?

(a) (b) (c) (d)


(A) (B) (C) (D)

3.
? –

(A) (B) (C) (D) (a) (b) (c) (d)

4. ?

(A) (B) (C) (D) (a) (b) (c) (d)

5. ?

(a) (b) (c) (d)


(A) (B) (C) (D)
=

6. +
?
=

+
=

+
+
+

=
=

+
+

(A) (B) (C) (D) (a) (b) (c) (d)

7. ?

(a) (b) (c) (d)


(A) (B) (C) (D)
154 Study Package for NTSE

Problem Figures Answer Figures


+
8. ?
+ (a) (b) (c) (d)
(A) (B) (C) (D)

9.
?
(a) (b) (c) (d)
(A) (B) (C) (D)

10.
?
(a) (b) (c) (d)
(A) (B) (C) (D)

11.
?

(A) (B) (C) (D) (a) (b) (c) (d)

12.
?

(A) (B) (C) (D) (a) (b) (c) (d)

13.
?

(A) (B) (C) (D) (a) (b) (c) (d)

14.
?

(A) (B) (C) (D) (a) (b) (c) (d)


S

S S C S C S C
15.
?
C C
(A) (B) (C) (D) (a) (b) (c) (d)

16. ?

(A) (B) (C) (D) (a) (b) (c) (d)

17.
?

(A) (B) (C) (D) (a) (b) (c) (d)


Analogy 155
Problem Figures Answer Figures

18.
?

(A) (B) (C) (D) (a) (b) (c) (d)

19.
?

(A) (B) (C) (D) (a) (b) (c) (d)

20.
?

(A) (B) (C) (D) (a) (b) (c) (d)

21.
?

(A) (B) (C) (D) (a) (b) (c) (d)

22.
?

(A) (B) (C) (D) (a) (b) (c) (d)

23.
?

(A) (B) (C) (D) (a) (b) (c) (d)

Directions (Q. Nos. 24 and 25) In each of the following questions, choose the picture that would go in the empty box so that
the two bottom pictures are released in the same way as the top two are related.

Problem Figures Answer Figures

24.
?

(A) (B) (C) (D) (a) (b) (c) (d)

25.
ERASER ?

(A) (B) (C) (D) (a) (b) (c) (d)


Hints and Solutions
Elementary Level Questions
1. (c) Cross diagonals must be placed in rectangle. 18. (b) From figures A to B, the number of circles increases from 1 to 2
and one of the circle is inverted and small dark circle become
2. (d) Figure moves 90° anti-clockwise.
unshadded. An additional line appears in each circle.
3. (b) Dot (•) moves one place forward clockwise and cross (× )
19. (b) Figure (A) has two four arms while figure (B) have total eight arm
moves one place forward anti-clockwise.
figures. Similarly, answer figures must have six arms.
4. (d) Figure moves 90° clockwise or anti-clockwise.
20. (a) Figure rotated by 45° clockwise while the inner part get reverted
5. (d) From figure A to B, the central figure becomes the outer outside.
one, the inner arrow rotates 135° anti-clockwise and outer
21. (d) The whole figure rotates through 135° clockwise. The arc
figure diminishes in size and gets attached to the end of the
attached to it gets reversed while the bar with ‘equal to’ sign rotates
arrow.
45° anti-clockwise.
6. (c) From figure A to B, the figure is rotated by 180° and the
22. (c) The shaded portion moves one side anti-clockwise. The
arrowhead is replaced on the ‘U’ shaped figure.
abjacent line segments shift one side clockwise and rotates
7. (a) Change the position of darkened square accordingly. through 45° clockwise.
8. (d) Corner figure must be placed in the central figure 23. (b) Figure is rotated 45° clockwise and two half leaves are added
accordingly. on clockwise side.
9. (b) One arm increases in figure as observed in A and B. 24. (d) The second is the water image of the first.
10. (b) Inner figure moves 180° in clockwise direction. 25. (a) The upper left figure gets enlarged and encircles all the
elements. The lower right elements gets reduced and becomes the
11. (c) From figure A to B, the figure is rotated by 180° and one line
innermost.
and one circle are added. The circle is added on the same
side as in the previous figure. 26. (b) The number of lines in the main figure is reduced by one. One
of the outer elements goes inside the main figure and gets
12. (a) Line remains at same place but the figure get reverted by
shaded while the inner design moves out and encircles the outer
180°.
design, which moves one sides in clockwise direction.
13. (a) One other sign is added to the initial sign.
27. (b) Lines equal to the number of sides are added inside the main
14. (a) From figures A to B, the outer figure gets replaced by a figure. The outer design interchange their position and half of these
figure with one lesser number of sides. The circles present are shaded.
inside it comes out and new black circle is introduced inside.
28. (b) The shaded portion gets unshaded and vice-versa.
15. (a) Inner items and outer item(s) get exchanged and figure is
29. (a) The innermost figure comes in the middle with one more side
reverted by 180°.
while the middle one becomes innermost with one lesser side.
16. (c) From figures A to B, the inner circle changes to square and The number of sides of the outermost figure increase by two.
vice-versa. The arrow rotates by 180°. A dot appears inside.
30. (b) The main figure get inverted and inner lines join to form a new
17. (b) Figure get reverted diagonally. figure in right corner.

High Skill Questions


1. (b) Figure rotate 180° in clockwise direction. 8. (b) From figures A to B, the complete figure gets inverted.
2. (c) As, triangle in the figure rotate 180° in clockwise direction 9. (d) From figures A to B, the second figure is the inverted image of
and rectangle rotate 90° in clockwise direction. the first figure and the inner bulge becomes the outer and
3. (b) Internal figure get interchanged their position. vice-versa.

4. (a) Option (a) is correct. 10. (b) From figures A to B, the first left hand figure becomes enlarge
and others are inserted inside the enlarged figure, similarly from III
5. (d) Figure rotate 90° in anti-clockwise direction.
to IV, we get answer figure.
6. (c) From figures A to B, the main design/arrows rotates
through 90° clockwise while inner design move one step in 11. (c) The first and third design from the top rotate 90° clockwise and
anti-clockwise direction. interchange their positions. The second from the top and first from
the bottom rotate by 90° anti-clockwise and interchange positions.
7. (c) From figures A to B, the arrow sign is replaced by circle and
The line of orientation rotates by 90° anti-clockwise.
vice-versa, similarly square and arc sign interchanged.
Analogy 157
12. (d) The two designs rotates 180° clockwise and shift two 18. (b) The design of the upper left portion moves one are anti-clockwise
arm clockwise while third rotates by 90° clockwise and and design on the right moves one arm clockwise. The lower portion
shift one arm clockwise. remains as it is.
13. (a) The innermost figure becomes the outermost and 19. (c) The number and sides of the upper figures reduces by one while the
vice-versa. The two middle figures also interchange number of the lower figure increases by one.
themselves. 20. (d) Except the upper left design other three designs exchange their
14. (a) The main figure gets vertically inverted while the base positions in anti-clockwise direction and designs insides rotate by 45°.
arrow gets laterally inverted. 21. (a) The left design becomes the main one while the right one goes inside.
15. (a) The upper left design gets inverted vertically at its The main design gets reduced and shaded and shifts to lower side.
position while other three designs interchange their 22. (d) The whole figure rotates by 180°. The small circle outside and the
positions. The lower right design gets inverted vertically line inside to main figure move one step anti-clockwise.
while the lower left and upper right design gets shaded 23. (c) The inner figures gets divided into two parts which get separated
after shifting. and inverted. The outer figure gets reduced in size and is placed in the
16. (a) The number of lines in each design reduces by one. gap between two portions.
The design on the right column interchange their position. 24. (d) As, a car has four wheels, similarly, a cycle has two wheels.
17. (d) The number of sides in the main figure increases by 25. (a) As, shirt and pants both are cloths. Similarly, eraser and pencil both
one and the other design interchange places diagonally. are stationary items.
Chapter

3
Classification (Odd One Out)
In non-verbal classification questions, you will be given a group of four or five items (diagrams). All but one of these
items will be similar in a certain way or some other way. You will be asked to choose the one that is not similar to
the other figures given in the questions.
In other words we can say that the questions based on classification are designed to test the candidate’s ability to
classify or segregate a group of objects from the given objects, on the basis of their common property i. e., it is a
process of marking out a homogeneous group from a heterogeneous larger group.
In these types of questions, four figures, labelled as a, b, c and d are given , three of these four figures have common
feature and hence are similar in a certain way. One of the figures does not share the common feature and hence does
not fit with other figures. You have to select that figure which does not belong to the group and this figure is your
answer.
While solving the problems the candidate has to observe all the figures minutely and try to correlate them.
Example 1 Choose the figure which is different from the other.

(a) (b) (c) (d)


Solution (d) The figures form a series. The complete figure rotates 90° clockwise in each step. Figure (d) does
not fit in the series.

Example 2 Choose the figure which is different from the other.

(a) (b) (c) (d)


Solution (d) In this case, all the figures except figure (d), can be rotated into each other.
Target Exercise
Elementary Level Questions
Directions (Q. Nos. 1-25) In the following exercises, group of four figures are given in questions. Out of these four figures
one figure is different, while three figures are similar in some way. Find out the different figure (odd one out).

1. 9. R R R 17.
R

(a) (b) (c) (d) (a) (b) (c) (d) (a) (b) (c) (d)

18.
2. 10.

T L V R (a) (b) (c) (d)


(a) (b) (c) (d) (a) (b) (c) (d)
19.
3. 11.

(a) (b) (c) (d)

(a) (b) (c) (d) (a) (b) (c) (d) 20.

4. 12. × × × – + × + × – × – +
+ + + – + ×+ × – × – + (a) (b) (c) (d)
– – – – + ×+ × – × – +
21.
(a) (b) (c) (d) (a) (b) (c) (d)

5. 13.
(a) (b) (c) (d)

22.
(a) (b) (c) (d) (a) (b) (c) (d)

6. 14. (a) (b) (c) (d)

23. O × ×
O

×
×O O
×
O ×

O O
O
× O× ×O×
×

×
×
O O

×
×

O××
×

×OO
O

(a) (b) (c) (d) (a) (b) (c) (d)


×

(a) (b) (c) (d)


7. 15.
24.

(a) (b) (c) (d) (a) (b) (c) (d)


(a) (b) (c) (d)

8. 16. 25.

(a) (b) (c) (d) (a) (b) (c) (d) (a) (b) (c) (d)
160 Study Package for NTSE

High Skill Questions


Directions (Q. Nos.1-24) Out of the four figures (a), (b), (c) and (d), given in each problem, three are similar in a certain
way. Choose the figure which is different from other figures.

1. 9. 17.

(a) (b) (c) (d) (a) (b) (c) (d) (a) (b) (c) (d)
10. × × × × × G
QS
2. 18.

(a) (b) (c) (d)


(a)
× ×
(b) (c) (d)
×
R (a) (b) (c) (d)
11. 19.
3.

(a) (b) (c) (d) (a) (b) (c) (d)


(a) (b) (c) (d)
12. 20. =
××
= +
===== ++
= =
+
+
4. =
= = +
+
×××

(a) (b) (c) (d) (a) (b) (c) (d)


(a) (b) (c) (d)
13. 21.
5.

(a) (b) (c) (d) (a) (b) (c) (d)


(a) (b) (c) (d)
14. 22.
6.

(a) (b) (c) (d) (a) (b) (c) (d)


(a) (b) (c) (d)
15. 23.
7.
T V X I
(a) (b) (c) (d) (a) (b) (c) (d)
(a) (b) (c) (d)
16. 24.
8.
121 231 301 341

(a) (b) (c) (d) (a) (b) (c) (d) (a) (b) (c) (d)
Hints and Solutions
Elementary Level Questions
1. (d) In figure (d), outer and inner figure is different. 13. (b) In figure (b), along diagonal of square figures are different.
2. (d) Except figure (d), all other have only straight line, there is no 14. (c) In figure (c), outer shapes are different.
curved part.
15. (c) In figure (c), there are two same straight lines.
3. (d) In figure (d), inner and outer figure is not connected by small 16. (d) In figure (d), parts are in same direction.
lines.
17. (c) In figure (c), arrow direction is different from the terminal dots.
4. (a) In figure (a), inner and outer shaded part concide to one other
part. 18. (c) Both the arrow heads are in the same direction in figure (c). In
all other figures, they are in the opposite direction. Hence, (c) is
5. (d) In figure (d), three lines are headed by arrows on the both the answer.
sides, while other figure has four arrows.
19. (b) In figure (b), centre have black dot, which makes it different
6. (a) In figure (a), inner circle is not in touch with other outer circles. from others.
7. (c) In figure (c), and are not on a single line. 20. (d) In figure (d), both niddles are at the angle of 180°.
8. (c) In figure (c), directions of arrows is not alternative direction. 21. (d) In figure (d), direction of small hooks is different from others.
9. (c) In figure (c), cyclic ring contains only two small lines. 22. (a) In figure (a), small end line is opposite to shaded triangle.
10. (c) Figure (c) is different in direction. 23. (d) Figure (d), is different from other figures.
11. (a) Direction of outer figures are same in figure (a). 24. (a) Figure (a) has three black dots.
12. (a) In figure (a), columns have different signs (+ , ×, −, etc). 25. (c) Figure (c) has identical arrows.

High Skill Questions


1. (d) In figure (d), shaded part have different distance or angle 15. (d) Figure (d), is different from others. As, figure (d) is made of
from arrow. only one line.
2. (d) In figure (d), there are four dots, two black circles and two 16. (c) In figure (c), black dot is not present in small rectangle region.
small circles.
17. (d) In figure (d), parts are not mirror images of each other.
3. (d) Figure (d) is different from others.
18. (b) Except figure (b), all are mirror images while it is a water
4. (c) In figure (c), arrow makes 90° angle. image of G.
5. (c) In figure (c), directions of alternative arrows are different. 19. (d) In figure (d), there is no part with three arms while other
6. (c) In figure (c), one line cannot passes through the centre of figures have 1, 2, 3 and 4 arms parts.
circle. 20. (a) In figure (a), outer sign marks are not in increasing number of
7. (d) In figure (d), small line (—) and —o have same direction. signs is, i.e., 2, 3, 4, 5, 6.
8. (c) 301 is not divisible by 11. 21. (d) Except figure (d), In all the remaining figure, there are two
9. (d) In figure (d), black dot is placed on the bottom of arrow. curves are attached outside of the square.

10. (c) Except figure (c), in all other figures the black and white circles 22. (d) Except figure (d), in rest of the figure, the number of lines on
occur on opposite corners of the square and so do the symbol x. the neck of the pot are same as the number of lines on the
middle of the pot.
11. (d) In figure (d), figure is not divided into two equal parts.
23. (d) Except figure (a), all other figures have two arrows.
12. (d) In figure (d), direction of inner part is different, then outer figure.
24. (a) Except figure (a), in rest of the figure, the number of
13. (d) In figure (d), all three angles are mentioned while other projected part on the circumference of circle are even.
figures have two angles marked.
Or in rest of the figure, there are projected tooth on the
14. (d) In figure (d), figure does not contains any in side figure but circumference of circle in opposite direction.
figure have four lines.
Chapter

4
Matrix
Matrix is also one of the most important chapter of non-verbal reasoning. There are two types of matrix-one of them
is figure matrix and remaining one is word or letter based matrix.

Type I Figure Based Matrix


Figure matrix is one kind of magical block. It is divided into n number of small squares. In each small square,
different symbols are given. In this matrix one small square is left blank in which you have to fill in a square from
the answer choices such that each row. Vertically, horizontally and diagonally follow the same rule.
Candidates are required to analyse the given matrix, find out the common rule and then basis on the common rule,
choose the missing figure from the given answer figures.
Example 1 Select one alternative figure out of (a), (b), (c) and (d), which completes the given matrix.

Problem Figure

Answer Figures

? (a) (b) (c) (d)

Solution (d) Clearly, in the first and second rows, the second figure is the inner part of the first figure and the
third figure is the inner part of the second figure.
Thus, the missing figure should be the inner part of the second figure in third row, i.e., a small darkened circle.

Type II Word or Letter Based Matrix


Basically, this type of matrix is based on coding-decoding. In this matrix, the codes of letters and numbers are given
in two matrix. To solve the question based on it, first of all the possible values of letters must be write and after that
choose the correct answer from given alternatives.
Matrix I Matrix II
0 1 2 3 4 5 6 7 8 9
0 F O M S R 5 A T D I P
1 S R F O M 6 I P A T D
2 O M S R F 7 T D I P A
3 R F O M S 8 P A T D I
4 M S R F O 9 D I P A T
Matrix 163
Example 2 MOST
(a) 40, 44, 22, 89 (b) 33, 20, 11, 79 (c) 21, 00, 03, 88 (d) 02, 13, 34, 56
Solution (d) From matrix I, M can be coded as 02, 14, 21, 33 or 40.
From matrix I, O can be coded as 01, 13, 20, 32 or 44.
From matrix I, S can be coded as 03, 10, 22, 34 or 41.
From matrix II, T can be coded as 56, 68, 75, 87 or 99.
Clearly, (d) is the only set of correct codes.

Example 3 ROAD
(a) 42, 32, 79, 58 (b) 23, 32, 98 , 99 (c) 11, 13, 67, 69 (d) 04, 20, 55, 78
Solution (c) From matrix I, R can be coded as 04, 11, 23, 30 or 42.
From matrix I, O can be coded as 01, 13, 20, 32 or 44.
From matrix II, A can be coded as 55, 67, 79, 86 or 98.
From matrix II, D can be coded as 57, 69, 76, 88 or 95.
Clearly, only (c) contains the correct codes.

Target Exercise
Elementary Level Questions
Directions (Q. Nos. 1-5) In each of the following questions, find out which of the answer figures (a), (b), (c) and (d)
completes the figure matrix.
1. 4. Matrix I
0 1 2 3 4
0 A E S T H
?
1 T H A E S
? 2 E S T H A
3 H A E S T
4 S T H A E
Matrix II
(a) (b) (c) (d) (a) (b) (c) (d)
5 6 7 8 9
2. 5. 5 P O R K L
6 K L P O R
7 O R K L P
8 L P O R K
? ? 9 R K L P O
6. EAST
(a) 44, 32, 21, 03 (b) 32, 31, 02, 04
(a) (b) (c) (d) (a) (b) (c) (d) (c) 20, 43, 33, 11 (d) 13, 12, 14, 10

3. 7. ROSE
Directions (Q. Nos. 6-10) In each of (a) 95, 75, 02, 32 (b) 88, 76, 31, 32
the following questions, a word is (c) 86, 67, 33, 44 (d) 57, 87, 32, 33
represented by only one set of numbers as
given in any one of the alternatives. The 8. SOLE
? sets of numbers given in the alternatives (a) 41, 57, 87, 31 (b) 33, 99, 66, 44
are represented by two classes of (c) 21, 75, 44, 02 (d) 02, 78, 87, 13
alphabets as in the two given matrices.
The columns and rows of Matrix I are 9. LAKE
(a) (b) (c) (d) numbered from 0 to 4 and those of
Matrix II from 5 to 9. A letter from these (a) 97, 00, 77, 12 (b) 66, 12, 58, 40
matrices can be represented first by its (c) 85, 31, 77, 44 (d) 77, 43, 76, 31
row and then the column number.
164 Study Package for NTSE

10. LEST 14. Matrix II


(a) 97, 32, 21, 34 (b) 87, 32, 21, 31
(c) 85, 02, 04, 22 (d) 66, 00, 20, 34 5 6 7 8 9
5 W N R M L
Directions (Q. Nos. 11-15) In each 6 N R M L W
?
of the following questions, find out 7 R M L W N
which of the anwer figures (a), (b), (c) 8 M L W N R
and (d) completes the figure matrix. 9 L W N R M
(a) (b) (c) (d)
11. 16. DRAW
15.
(a) 41, 66, 23, 55
(b) 32, 75, 44, 76
(c) 23, 57, 30, 68
? (d) 14, 89, 12, 78
?
17. BAND
(a) 43, 21, 97, 33
(b) 11, 21, 79, 41
(a) (b) (c) (d) (a) (b) (c) (d) (c) 34, 44, 66, 14
12. (d) 20, 30, 89, 23
Directions (Q. Nos. 16-20) In each of the
following questions, a word is represented 18. BLOW
by only one set of numbers as given in any (a) 11, 68, 42, 69
one of the alternatives. The sets of numbers (b) 21, 95, 33, 97
? given in the alternatives are represented by (c) 34, 68, 10, 88
two classes of alphabets as in the two given (d) 34, 86, 44, 78
matrices. The columns and rows of Matrix I
are numbered from 0 to 4 and those of 19. RAIN
Matrix II from 5 to 9. A letter from these (a) 57, 12, 31, 56
(a) (b) (c) (d)
matrices can be represented first by its row (b) 57, 21, 23, 79
13. and then the column number. (c) 66, 44, 42, 96
(d) 75, 30, 31, 87
Matrix I
20. LAMB
0 1 2 3 4
(a) 68, 21, 58, 34
0 D O B A I
? (b) 77, 44, 76, 33
1 O B A I D
(c) 86, 21, 67, 12
2 B A I D O
(d) 95, 30, 80, 20
3 A I D O B
(a) (b) (c) (d) 4 I D O B A

High Skill Questions


Directions (Q. Nos. 1-5) In each of the following questions, find out which of the answer figures (a), (b), (c) and (d)
completes the figure matrix.

1. 2. AA BB CC 3. a a a
b b b

Aa Bb Cc b b b

a a
? aa bb ? ?
b b

b a a a
C CC cc c b b b b
(a) (b) (c) (d) (a) (b) (c) (d) (a) (b) (c) (d)
Matrix 165
4. 8. FINE 15.
?
(a) 31, 32, 33, 82 (b) 24, 19, 21, 78
(c) 12, 10, 13, 67 (d) 00, 04, 02, 56

(a) (b) (c) (d) 9. HEAT


(a) 79, 53, 20, 87 (b) 65, 56, 13, 57 ?
5.
(c) 57, 56, 01, 59 (d) 29, 85, 34, 93

10. BOTH
? (a) 88, 30, 85, 86 (b) 75, 22, 76, 79
(a) (b) (c) (d)
(c) 69, 67, 68, 59 (d) 58, 02, 68, 65
16. + ÷
(a) (b) (c) (d) Directions (Q. Nos. 11-19) In each +
of the following questions, find out ÷
÷ –
which of the answer figures (a), (b), (c) –
Directions (Q. Nos. 6-10) In each of – –
the following questions, a word is and (d) completes the figure matrix. + ÷ ?
+
represented by only one set of numbers 11. –
as given in any one of the alternatives. + ÷ + ÷
÷
+
The sets of numbers given in the ÷ +

alternatives are represented by two (a) (b) (c) (d)


classes of alphabets as in the two given ?
17.
matrices. The columns and rows of
Matrix I are numbered from 0 to 4 and
those of Matrix II from 5 to 9. A letter (a) (b) (c) (d)
from these matrices can be represented
first by its row and then the column 12.
?
number.
Matrix I
0 1 2 3 4 ? (a) (b) (c) (d)
0 F A N O I
1 I O F A N 18.
2 A N O I F
(a) (b) (c) (d)
3 O F I N A
4 N I A F O 13.
Matrix II ?
5 6 7 8 9
5 S E H B T ?
6 H S E T B
(a) (b) (c) (d)
7 B T S E H
8 E H T B S 19.
(a) (b) (c) (d) + +
+

9 T S E H B
+
+
+ +

6. NEST 14.
+

(a) 02, 56, 55, 59


(b) 14, 67, 66, 67
+

(c) 21, 76, 77, 76 θ ψ


(d) 33, 85, 88, 86 ψ φ φ θ
?
?
7. FAITH φ θ θ φ φ ψ φ
(a) 43, 42, 41, 78, 89
ψ ψ θ ψ θ
(b) 31, 34, 23, 76, 79
(c) 24, 31, 10, 59, 57 (a) (b) (c) (d) (a) (b) (c) (d)
(d) 12, 20, 40, 68, 65
Hints and Solutions
Elementary Level Questions
1. (b) Considering row wise, both the figures rotate 180° and joined 11. (b) The first figure in each row is completely unshaded, the second
together. The common line in the middle of the figure gets one has one-fourth part shaded and the third-one is half shaded.
disappear. 12. (b) In each figure, the circles are towards the longer line.
2. (a) Each row of the matrix contains one shaded circle with bar The number of the lines in the first and third figures are identical
lines in each subsequent step of a column is being removed. and number directions of circles are increased by one in each
3. (b) The shaded part inside the square moves from one corner to step.
another in clockwise direction, as we move from left to right in a row. 13. (b) The third figure in each row comprises of parts, which are not
4. (b) The third column contains the line which is common to the common to the first two figures.
design in the first two columns. 14. (a) There are three triangles, in which one has number of lines
5. (c) There are 3 outer figures (circle, triangle and square), 3 inner and other is shaded. The figures has different number of legs.
figures (circle, triangle and square) and 3 types of 15. (d) Each row and column contains three black and four white
shading–plane, line and dark and inner figure becomes outer stars.
figure in each step.
16. (d) From matrix I, D can be coded as 00, 14, 23, 32 or 41.
6. (d) From matrix I, E can be coded as 01, 13, 20, 32 or 44. From matrix II, R can be coded as 57, 66, 75, 89 or 98.
From matrix I, A can be coded as 00, 12, 24, 31 or 43. From matrix I, A can be coded as 03, 12, 21, 30 or 44.
From matrix I, S can be coded as 02, 14, 21, 33 or 40. From matrix II, W can be coded as 55, 69, 78, 87 or 96.
From matrix I, T can be coded as 03, 10, 22, 34 or 41. Required answer – 14, 89, 12, 78.
Required answer – 13, 12, 14, 10.
17. (b) From matrix I, B can be coded as 02, 11, 20, 34 or 43.
7. (a) From matrix II, R can be coded as 57, 69, 76, 88 or 95. From matrix I, A can be coded as 03, 12, 21, 30 or 44.
From matrix II, O can be coded as 56, 68, 75, 87 or 99. From matrix II, N can be coded as 56, 65, 79, 88 or 97.
From matrix I, S can be coded as 02, 14, 21, 33 or 40. From matrix I, D can be coded as 00, 14, 23, 32 or 41.
From matrix I, E can be coded as 01, 13, 20, 32 or 44. Required answer – 11, 21, 79, 41.
Required answer – 95, 75, 02, 32.
18. (a) From matrix I, B can be coded as 02, 11, 20, 34 or 43.
8. (b) From matrix I, S can be coded as 02, 14, 21, 33 or 40.
From matrix II, L can be coded as 59, 68, 77, 86 or 95.
From matrix II, O can be coded as 56, 68, 75, 87 or 99.
From matrix I, O can be coded as 01, 10, 24, 33 or 42.
From matrix II, L can be coded as 59, 66, 78, 85 or 97.
From matrix II, W can be coded as 55, 69, 78, 87 or 96.
From matrix I, E can be coded as 01, 13, 20, 32 or 44.
Required answer – 11, 68, 42, 69.
Required answer – 33, 99, 66, 44.
19. (a) From matrix II, R can be coded as 57, 66, 75, 89 or 98.
9. (c) From matrix II, L can be coded as 59, 66, 78, 85 or 97.
From matrix I, A can be coded as 03, 12, 21, 30 or 44.
From matrix I, A can be coded as 00, 12, 24, 31 or 43.
From matrix I, I can be coded as 04, 13, 22, 31 or 40.
From matrix II, K can be coded as 58, 65, 77, 89 or 96.
From matrix II, N can be coded as 56, 65, 79, 88 or 97.
From matrix I, E can be coded as 01, 13, 20, 32 or 44.
Required answer – 57, 12, 31, 56.
Required answer – 85, 31, 77, 44
20. (a) From matrix II, L can be coded as 59, 68, 77, 86 or 95.
10. (a) From matrix II, L can be coded as 59, 66, 78, 85 or 97.
From matrix I, A can be coded as 03, 12, 21, 30 or 44.
From matrix I, E can be coded as 01, 13, 20, 32 or 44.
From matrix II, M can be coded as 58, 67, 76, 85 or 99.
From matrix I, S can be coded as 02, 14, 21, 33 or 40.
From matrix I, B can be coded as 02, 11, 20, 34 or 43.
From matrix I, T can be coded as 03, 10, 22, 34 or 41.
Required answer – 68, 21, 58, 34.
Required answer – 97, 32, 21, 34.

High Skill Questions


1. (a) In each column, second arrow rotate 90° anti-clockwise, so 6. (a) From matrix I, N can be coded as 02, 14, 21, 33 or 40.
answer (a) is correct for third column. From matrix II, E can be coded as 56, 67, 78, 85 or 97.
From matrix II, S can be coded as 55, 66, 77, 89 or 96.
2. (c) As per order in column, both will be small English alphabets.
From matrix II, T can be coded as 59, 68, 76, 87 or 95.
3. (c) In each column, third figure contains to half parts of Requried answer – 02, 56, 55, 59.
first figure, placed first on one another, so answer (c) is
correct.
7. (b) From matrix I, F can be coded as 00, 12, 24, 31 or 43.
From matrix I, A can be coded as 01, 13, 20, 34 or 42.
4. (b) Half figures are joined in such a way to produce a complete From matrix I, I can be coded as 04, 10, 23, 32 or 41.
figure, so figure (b) will be correct. From matrix II, T can be coded as 59, 68, 76, 87 or 95.
5. (b) As per the pattern in first and second column, answer figure From matrix II, H can be coded as 57, 65, 79, 86 or 98.
must have three rectangles as mentioned in figure (b). Required answer – 31, 34, 23, 76, 79
Matrix 167
8. (d) From matrix I, F can be coded as 00, 12, 24, 31 or 43. 12. (d) In each row, one figure is unshaded, the other has its upper part
From matrix I, I can be coded as 04, 14, 23, 32 or 41. shaded and the third one has its RHS part shaded. There are three
specified positions of the two triangles each of which is used only
From matrix I, N can be coded as 02, 14, 21, 33 or 40.
once in a row. Also of the figures in each row have one triangle
From matrix II, E can be coded as 56, 67, 78, 85 or 97. shaded.
Required answer – 00, 04, 02, 56. 13. (c) Clearly, triangle figure in third row forms the outermost and
9. (c) From matrix II, H can be coded as 57, 65, 79, 86 or 98. rectangle becomes inner most part of the third figure.
From matrix II, E can be coded as 56, 67, 78, 85 or 97. 14. (d) There are 3 types of faces i.e. triangle, circle and rectangle, 3 types
From matrix I, A can be coded as 01, 13, 20, 34 or 42. of body, 3 types of hands and 3 types of legs, each of which is used
only once in a single row. So, the features which have not been used
From matrix II, T can be coded as 59, 68, 76, 87 or 95. in the first two figures of the third row would combine to produce the
Required answer – 57, 56, 01, 59. missing figure.
10. (b) From matrix II, B can be coded as 58, 69, 75, 88 or 99. 15. (a) The third figure in each row having one lesser figure than
second figure.
From matrix I, O can be coded as 03, 11, 22, 30 or 44.
From matrix II, T can be coded as 59, 68, 76, 87 or 95. 16. (b) As per similar pattern in first and second row, option (b) will
correct answer in the sequence.
From matrix II, H can be coded as 57, 65, 79, 86 or 98.
17. (d) Each line across and down contains one each of the three
Required answer – 75, 22, 76, 79.
symbols. In each line one symbol is black and one is upside down.
11. (d) The second figure in each row forms the outermost and 18. (b) Figure (b) will satisfy the similar trend in third row.
innermost part of the third figure, and the first one forms the
middle part. 19. (c) Signs in square are moving 90° clockwise, option (c) is correct.
Chapter

5
Completion of Figures
Completion of figures is also one of the most important chapter of non-verbal reasoning. In these types of questions,
a segment in a figure, generally a quarter is left blank. This incomplete figure is followed by few choices showing
the missing segment which if fitted in the incomplete figure, completes the figure.
Candidates are expected to be vigilant and be extra careful while selecting the correct option as sometimes, the
alternatives have very minute differences among them.
Example 1
Problem Figure Answer Figures

? (a) (b) (c) (d)

Solution (d) Figure (d) will complete the pattern. For solving such problems, firstly complete
the figure by using pencil with following the trends in the rest of the figure. Then, compare this
imagined figure drawn in the blank space with the answer figure. The answer figure that
resembles with this imagined figure is the correct answer. For above example, completion by
imagination will give the following figure.

Now, we can say is the imagined figure, which resembles with answer figure (d). Therefore, figure (d) is
correct answer figure. Now, rub the figure drawn in blank space by pencil.
Example 2
Problem Figure Answer Figures

?
(a) (b) (c) (d)
Solution (c) If option (c) is placed in the missing portion of the original figure, it completes the original figure
and the figure will look like the figure given below
Target Exercise
Elementary Level Questions
Directions (Q. Nos. 1-18) In these questions, problem figure is given on the left side of the line, which is incomplete. One, out
of the four answer figures (a), (b), (c) and (d) can complete the same. You have to locate the answer figure which if inserted in the
problem figure without changing the direction completes the same.
Problem Problem
Figure Figure
1. 7. Answer Figures
Answer Figures

?
(a) (b) (c) (d) (a) (b) (c) (d)

8.
2.

? ?

(a) (b) (c) (d) (a) (b) (c) (d)

9.
3.

? ?

(a) (b) (c) (d) (a) (b) (c) (d)

4. 10.

? ?
(a) (b) (c) (d) (a) (b) (c) (d)
5. 11.

? ?
(a) (b) (c) (d) (a) (b) (c) (d)

6. 12.

? ?
(a) (b) (c) (d) (a) (b) (c) (d)
170 Study Package for NTSE

Problem Problem
Figure Figure
13. 16.
Answer Figures
Answer Figures

? ?
(a) (b) (c) (d) (a) (b) (c) (d)

14. 17.
?
× ×
× × ×

× ?
(a) (b) (c) (d) (a) (b) (c) (d)
15. 18.

? ?
(a) (b) (c) (d) (a) (b) (c) (d)

High Skill Questions


Directions (Q. Nos. 1-14) In these questions, a problem figure is given on the left side of the line, which is incomplete. One
out of the four answer figures (a), (b), (c) and (d) can complete the same. You have to locate the answer figure which if inserted
in the problem figure, without changing the direction, completes the same.
Problem Problem
Figure Figure
1. 5.
Answer Figures Answer Figures

? ?
(a) (b) (c) (d) (a) (b) (c) (d)

2. 6.

? ?

(a) (b) (c) (d) (a) (b) (c) (d)

3. × × 7.
?
× ×
×
?
×
(a) (b) (c) (d) (a) (b) (c) (d)

4. 8.

? ?

(a) (b) (c) (d) (a) (b) (c) (d)


Completion of Figures 171
Problem Problem
Figure Figure
9. 12.
Answer Figures Answer Figures

? ?
(a) (b) (c) (d) (a) (b) (c) (d)

10. 13.

? ?
(a) (b) (c) (d) (a) (b) (c) (d)

11. 14.

(a) (b) (c) (d) (a) (b) (c) (d)

Hints and Solutions


Elementary Level Questions
In questions, the problem figures have been shown as a complete figure. The new portion, which is completed is shown
dark. Students can compare this portion to the answer figures. Only that identical answer figure is the required
answer to the given question.

1. (c) 5. (c) 9. (d)

(c) (c) (d)

2. (d) 6. (a) 10. (b)

(d) (a) (b)

3. (d) 11. (d)


7. (d)

(d) (d)
(d)

12. (c)
4. (a) 8. (c)

(a) (c) (c)


172 Study Package for NTSE

13. (c) 15. (c) 17. (a)

(a)
(c) (c)

18. (b)
14. (d) 16. (b)
×
(b)
(d)
(b)

High Skill Questions


1. (c) 6. (b) 11. (b)

(c) (b) (b)

2. (c) 7. (c) 12. (a)

(c) (c) (a)

13. (b)
3. (a) × × × 8. (c)

× ×
× ×
(a) (c)
× × (b)

4. (d) 9. (b) 14. (a)

(d) (b)
(a)

5. (c) 10. (c)

(c) (c)

Answers
Elementary Level Questions
1. (c) 2. (d) 3. (d) 4. (a) 5. (c) 6. (a) 7. (d) 8. (c) 9. (d) 10. (b)
11. (d) 12. (c) 13. (c) 14. (d) 15. (c) 16. (b) 17. (a) 18. (b)

High Skill Questions


1. (c) 2. (c) 3. (a) 4. (d) 5. (c) 6. (b) 7. (c) 8. (c) 9. (b) 10. (c)
11. (b) 12. (a) 13. (b) 14. (a)
Chapter

6
Spotting Out the
Embedded Figure
In this chapter, a figure is given as an original figure followed by four answer figures. One of the answer figures is
embedded or hidden in the original figure. These non-verbal tests are designed to judge the candidate’s sense of
observation and analytical ability. Candidates are required to select the alternative that carries the correct figure
which clearly shows the embedded portion in the original figures.
Example 1 Figure (X) is embedded in any one of the four alternative figures. Find the alternative which contains
figure (X).

(X) (a) (b) (c) (d)

Solution (b) Figure (b) contains the figure (X). It can become more clear from the figure given below.

Example 2 Figure (X) is embedded in any one of the four alternative figures. Find the alternative
which contains figure (X).

(X) (a) (b) (c) (d)


Solution (b) The problem Figure (X) is embedded in the answer figure (b).
Target Exercise
Elementary Level Questions
Directions (Q. Nos. 1-15) In each of the questions given below, you are given a figure (X) followed by four figures (a), (b),
(c) and (d) such that (X) is embedded in one of them. Find out the correct alternative.
Problem Figure
Problem Figure
1. Answer Figures 9. Answer Figures

(X) (a) (b) (c) (d)


(X) (a) (b) (c) (d)
2.
10.

(X) (a) (b) (c) (d)


3. (X) (a) (b) (c) (d)

11.

(X) (a) (b) (c) (d)


4.
(X) (a) (b) (c) (d)

12.

(X) (a) (b) (c) (d)

5.
(X) (a) (b) (c) (d)

13.
(X) (a) (b) (c) (d)

6.
(X) (a) (b) (c) (d)

14.
(X) (a) (b) (c) (d)

7.

(X) (a) (b) (c) (d)

15.
(X) (a) (b) (c) (d)

8.

(X) (a) (b) (c) (d)

(X) (a) (b) (c) (d)


Spotting Out the Embedded Figure 175
High Skill Questions
Directions (Q. Nos. 1-14) In each of the questions given below, you are given a figure (X) followed by four figures (a), (b),
(c) and (d) such that (X) is embedded in one of them. Find out the correct alternative.

Problem Problem
Figure Figure
1. 8.
Answer Figures Answer Figures
Y

(X) (a) (b) (c) (d) (X) (a) (b) (c) (d)

2. 9.

(X) (a) (b) (c) (d) (X) (a) (b) (c) (d)

3. 10.

(X) (a) (b) (c) (d) (X) (a) (b) (c) (d)

4. 11.

(X) (a) (b) (c) (d) (X) (a) (b) (c) (d)

5. 12.

(X) (a) (b) (c) (d) (X) (a) (b) (c) (d)
6.
13.

(X) (a) (b) (c) (d) (X) (a) (b) (c) (d)
7.
14.

(X) (a) (b) (c) (d)


(X) (a) (b) (c) (d)
Hints and Solutions
Elementary Level Questions
1. (d) Problem figure (X) is embedded in 6. (c) Problem figure (X) is embedded in 11. (b) Problem figure (X) is embedded in
the answer figure (d). the answer figure (c). the answer figure (b).

2. (d) Problem figure (X) is embedded in 7. (b) Problem figure (X) is embedded in 12. (a) Problem figure (X) is embedded in
the answer figure (d). the answer figure (b).
the answer figure (a).

8. (b) Problem figure (X) is embedded in


3. (d) Problem figure (X) is embedded in the answer figure (b). 13. (d) Problem figure (X) is embedded in
the answer figure (d). the answer figure (d).

9. (a) Problem figure (X) is embedded in


4. (b) Problem figure (X) is embedded in the answer figure (a).
14. (b) Problem figure (X) is embedded in
the answer figure (b). the answer figure (b).

10. (c) Problem figure (X) is embedded in


the answer figure (c).
5. (c) Problem figure (X) is embedded in 15. (d) Problem figure (X) is embedded in
the answer figure (c). the answer figure (d).

High Skill Questions


1. (b) Problem figure (X) is embedded in 3. (b) Problem figure (X) is embedded in 5. (c) Problem figure (X) is embedded in
the answer figure (b). the answer figure (b). the answer figure (c).

2. (b) Problem figure (X) is embedded in 4. (d) Problem figure (X) is embedded in 6. (d) Problem figure (X) is embedded in
the answer figure (b). the answer figure (d). the answer figure (d).
Spotting Out the Embedded Figure 177
7. (b) Problem figure (X) is embedded in 10. (b) Problem figure (X) is embedded in 13. (d) Problem figure (X) is embedded in
the answer figure (b). the answer figure (b). the answer figure (d).

8. (a) Problem figure (X) is embedded in 11. (b) Problem figure (X) is embedded in 14. (d) Problem figure (X) is embedded in
the answer figure (b). the answer figure (b). the answer figure (d).

9. (d) Problem figure (X) is embedded in 12. (d) Problem figure (X) is embedded in
the answer figure (d). the answer figure (d).
Chapter

7
Mirror and Water Images

Mirror Images
Reflection of an object into the mirror is called its mirror image. It is obtained by inverting an object laterally i. e.,
towards the sides.
In these types of questions, a key/main figure is given under which four alternatives are given. Candidates are
required to identify the figure that is the mirror reflection of the main figure. For this identification, remember that
the mirror image is always opposite to the original and of the same size.

Variable
LHS RHS
Constant
UP Down

For English alphabets and number, mirror images are as follow

English Alphabets
Letter A B C D E F G H I J K L M
Mirror A B C D E F G H I J K L M
Letter N O P Q R S T U V W X Y Z
Mirror N O P Q R S T U V W X Y Z

Mirror Images of Small Letters


Letter Mirror Image Letter Mirror Image Letter Mirror Image
a j j s s
b d k k t t
c c I I u u
d d m v v
e e n n w w
f f o o x x
g g p p y y
h h p p z z
i i r r - -

Digit
Mirror 1 2 3 4 5 6 7 8 9 10
Image 1 2 3 4 5 6 7 8 9 01
Mirror and Water Images 179
Water Images
The reflection of an object into the water is called the water image of that object. It is obtained by inverting an object
vertically.
Examples of formation of water images of alphabets and numbers are given below.

Water Images of Capital Letters


Letters Water Image Letters Water Image Letters Water Image
A J S
A J S
B K T
B K T U
L
C C L U
M V
D D M V W
N
E E N W
F
F G O O X X
Y
P
G P Y
Q Z
H H Q Z
R
I I R - -

Water Images of Small Letters


Letter Water Image Letter Water Image Letter Water Image
a j s
a j s
b k t
b k t
c u
c I I u
d m v
d m n v
e w
e n w
f
f o o x x
g p y y
g p
h q z
h q z
r
I I r - -

Water Images of Numbers


Number 0 1 2 3 4 5 6 7 8 9
1 2 3 4 5 6 7 8 9
Water Image 0

Here, some examples will give you a better idea about the type of questions generally asked in NTSE
examination.
Example 1 In the following question, choose Example 2 In the following question, choose
the correct mirror image of the figure (X) from the correct water image of the figure (X) from
amongst the four alternatives (a), (b), (c) and (d) amongst the four alternatives (a), (b), (c) and (d)
given along with it. given along with it.

(X) (a) (b) (c) (d) (X) (a) (b) (c) (d)
Solution (a) Option figure (a) is the correct Solution (c) Option figure (c) is the correct
mirror image of the given figure ( X ). water image of the given figure ( X ).
Target Exercise
Elementary Level Questions
Directions (Q. Nos. 1-14) In each one of the following questions, choose the correct mirror image of the figure (X) from
amongst the four alternatives (a), (b), (c) and (d) given along with it.
1. 12.

(X) (a) (b) (c) (d) (X) (a) (b) (c) (d)
2. 13.

(X) (a) (b) (c) (d) (X) (a) (b) (c) (d)
3.
14.

B
B
B
R R R R R
(X) (a) (b) (c) (d)
(X) (a) (b) (c) (d)
4.
Directions (Q. Nos. 15-20) In each one of the following
questions, choose the correct water image of the figure (Z) from
(X) (a) (b) (c) (d) amongst the four alternatives (a), (b), (c) and (d) given along
with it.
5.
15.

(X) (a) (b) (c) (d)


6. (Z) (a) (b) (c) (d)

16. ×
(X) (a) (b) (c) (d)
× × × +
7.
(Z) (a) (b) (c) (d)

17.
(X) (a) (b) (c) (d)
8.
(Z) (a) (b) (c) (d)

(X) (a) (b) (c) (d) 18.


9.

(Z) (a) (b) (c) (d)


(X) (a) (b) (c) (d)
19.
10.

(Z) (a) (b) (c) (d)


(X) (a) (b) (c) (d)
20.
11.

(Z) (a) (b) (c) (d)


(X) (a) (b) (c) (d)
Mirror and Water Images 181
High Skill Questions
Directions (Q. Nos. 1-15) In each of the following questions, you are given a combination of letters and numbers followed
by four alternatives (a), (b), (c) and (d). Choose the alternative which most closely resembles the mirror image of the given
combination.
1. STROKE 9. panipat
(a) STROKE (b) EKORTS (a) tapinap (b) qaniqat
T
(c) S ROKE (d) E KORTS (c) taqinaq (d) patpani

10. GANDHI1869
2. JUDGEMENT A
(a) GANDHI1899 (b) 6981IHDN G
A
(a) TNEMEGDUJ (b) TNEMEGDUJ (c) 9681IHDNAG (d) G NDHI1896
(c) TNEME GDUJ (d) JUD GE ME NT
11. DOCTOR 7829
3. QUANTITATIVE (a) ROTCOD 2687S
UANTITATIVE T T (b) DOCTOR 2687
(a) (b) EVI ATI NAUQ
Q
6
(c) QUANTITATIVE (d) EVITATITNAUQ (c) 287 ROTCOD
T 6
(d) RO COD 87
2
4. REASONING
A
(a) RE SONING (b) GNINOS AE R 12. Disturb
A
(c) GNINOS ER (d) REASONING (a) brut siD (b) brutsiD
(c) brutsiD (d) bisturD
5. WESTERN
(a) NRETSEW (b) RE SEW
N T
13. LATERAL
L L
(c) NRE SEW
T
(d) NRETSE
W
(a) ABETA (b) LARETAL
(c) LARETA
L
(d) LARETAL
6. DBV8478
V
(a) 8 48 BD (b) 8748VBD 14. REPUBLIC
7
(c) 8748DBV (d) 8748VBD (a) CILB UPE R (b) CILB UPE R
(c) CILBUPE R (d) CILBUPE R
7. 15UP5062 U
(a) 5062UP15 (b) 2605P 51 15. INDEPENDENCE
(c) 5905P U5 (d) None of these
1
(a) EC EDNEPEDNI
N

8. 247593 (b) ECNEDNEPEDNI


(a) 395742 (b) 395742 (c) ECNEDNEPEDNI
(c) 3 57 2 (d) 247593 (d) ECNEDNE PEDNI
9 4

Directions (Q. Nos. 16-20) In each of the following questions, you are given a combination of letters or/and numbers
followed by four alternatives (a), (b), (c) and (d). Choose the alternative which most closely resembles the water image of the
given combination.
16. DISC-6 18. UP 15847
6 UP15 47 UP 47 UP1 47 UP 5 4
(a) CSID-6 (b) CSID- (a) 8 (b) 8 (c) 8 (d) 8 7
51 5 1
6
(c) DISC- (d) DISC-6
19. FRUIT
F FR URF FRU
17. TRAY (a)
TIUR
(b)
T U
I (c) TI (d) IT
R
YA T
(a) (b) YART
RA T 20. FAMILY
(c) Y
TRAY
(d) F
YL MA FAMI Y L FAMILY
(a)
MAF Y
IL (b) I (c) (d)

Answers
Elementary Level Questions
1. (a) 2. (d) 3. (d) 4. (d) 5. (b) 6. (c) 7. (c) 8. (d) 9. (a) 10. (a)
11. (b) 12. (a) 13. (b) 14. (b) 15. (a) 16. (c) 17. (b) 18. (c) 19. (c) 20. (b)

High Skill Questions


1. (d) 2. (c) 3. (d) 4. (b) 5. (a) 6. (d) 7. (d) 8. (b) 9. (c) 10. (c)
11. (c) 12. (a) 13. (b) 14. (a) 15. (b) 16. (c) 17. (a) 18. (a) 19. (b) 20. (d)
Chapter

8
Paper Folding
In this section of non-verbal reasoning, a figure is obtained by folding the piece of paper containing some design
along the dotted line. The resultant figure is to be selected out of a set of four figures given as alternatives
(a), (b), (c) and (d). This test is designed to judge the candidate’s understanding of forming a design by folding a
sheet of paper.
Here, some examples will give you a better idea about the type of questions generally asked in NTSE examination.

Directions (Ex. 1 and 2) In each of the following questions, find one correct answer figures from four alternatives which
resembles the pattern formed when the transparent sheet, having a design folded along the dotted line.

Example 1
Problem Figure Answer Figures

(a) (b) (c) (d)

Solution (a) In this question, lower half of the square sheet has been folded over the upper half, hence the
answer is (a).

Example 2
Problem Figure Answer Figures

(a) (b) (c) (d)

Solution (d) Similarly, on folding the transparent answer sheet along the dotted line the answer figure must
be figure (d).
Target Exercise
Elementary Level Questions
Directions (Q. Nos. 1-13) In each of the following questions, find one correct answer figure from four alternatives which
resembles the pattern formed when the transparent sheet, having a design folded along the dotted line.
Problem 8. Problem
Figure Figure
1. Answer Figures Answer Figures

(a) (b) (c) (d) (a) (b) (c) (d)


2.
9.

(a) (b) (c) (d)


(a) (b) (c) (d)
3.
10.

(a) (b) (c) (d)

s
(a) (b) (c) (d)
4.
11.

(a) (b) (c) (d)

5. (a) (b) (c) (d)

12.
(a) (b) (c) (d)

6.
(a) (b) (c) (d)

13.
(a) (b) (c) (d)

7.
(a) (b) (c) (d)

(a) (b) (c) (d)


184 Study Package for NTSE

High Skill Questions


Directions (Q. Nos. 1-18) In each of the following questions, find one correct answer figures from your alternatives which
resembles the pattern formed when the transparent sheet, having a design folded along the dotted line.
Transparent Transparent
Sheet Sheet
1. 8.
Answer Figures Answer Figures

(a) (b) (c) (d) (a) (b) (c) (d)

2. 9.

(a) (b) (c) (d)


(a) (b) (c) (d)
3.
10.

(a) (b) (c) (d)


(a) (b) (c) (d)
11.
4.

(a) (b) (c) (d)


(a) (b) (c) (d)

5. 12.

(a) (b) (c) (d) (a) (b) (c) (d)

6. 13.

(a) (b) (c) (d) (a) (b) (c) (d)

7. 14.

(a) (b) (c) (d) (a) (b) (c) (d)


Paper Folding 185
Transparent 17. Transparent
Sheet Sheet
15.
Answer Figures Answer Figures

(a) (b) (c) (d) (a) (b) (c) (d)


16. 18.

(a) (b) (c) (d) (a) (b) (c) (d)

Directions (Q. Nos. 19 and 20) In each of the following questions, two figure on transparent sheets are given. When the left
figure is exactly placed on the right figure, Find from the option figures how the resultant looks like.
19. 20.

(a) (b) (c) (d)


(a) (b) (c) (d)

Answers
Elementary Level Questions
1. (a) 2. (d) 3. (a) 4. (d) 5. (b) 6. (d) 7. (c) 8. (c) 9. (c) 10. (b)
11. (d) 12. (d) 13. (d)

High Skill Questions


1. (a) 2. (a) 3. (b) 4. (a) 5. (d) 6. (d) 7. (b) 8. (c) 9. (b) 10. (c)
11. (d) 12. (d) 13. (a) 14. (b) 15. (c) 16. (c) 17. (a) 18. (a) 19. (b) 20. (d)
Chapter

9
Paper Cutting
This chapter deals with the questions related with paper cutting. In the questions based on paper cutting, few
figures are given showing the manner in which a piece of paper is folded and then cut from a particular section. A
cut may be of varying designs. The design on the paper, after the cut which appears when the paper is unfolded is
shown by four answer figures. One of the answer figures correctly represents the design that the paper have after it
is unfolded.
The candidate has to select the one answer figure from the answer figures (a), (b), (c) and (d) which would most
closely resemble the pattern when the paper is unfolded.
Example 1 Consider the following three figures, marked (x), (y), (z) showing one fold in (x) another in y and cut in
(z). The candidate has to select one figure from the answer figure (a), (b), (c) and (d), when showing the unfold position
of (z).

Problem Figures Answer Figures

(x) (y) (z) (a) (b) (c) (d)


Solution (c)
In figure (x),
square is folded diagonally from upper part.

In figure (y),
paper is refolded to a quarter triangle of square.

In figure (z),
a square is punched in the folded quarter triangle of square like.

So, each quarter triangle will have a square.


So, triangle answer figure can be drawn.
Target Exercise
Elementary Level Questions
Directions (Q. Nos. 1-15) In following questions, three figures (x), (y) and (z) showing a sequence of folding a paper are
given. The figure depicts the cut made on the folded paper. Select the answer from the alternatives, which would most closely
resemble the third figure, when it is unfolded.
Problem Figures Answer Figures
1.

(x) (y) (z) (a) (b) (c) (d)

2.

(x) (y) (z) (a) (b) (c) (d)

3..

(x) (y) (z) (a) (b) (c) (d)

4.

(x) (y) (z) (a) (b) (c) (d)

5.

(x) (y) (z) (a) (b) (c) (d)

6.

(x) (y) (z) (a) (b) (c) (d)

7.

(x) (y) (z) (a) (b) (c) (d)

8.

(x) (y) (z) (a) (b) (c) (d)

9.

(x) (y) (z) (a) (b) (c) (d)


188 Study Package for NTSE

Problem Figures Answer Figures


10.

(x) (y) (z) (a) (b) (c) (d)

11.

(x) (y) (z) (a) (b) (c) (d)

12.

(x) (y) (z) (a) (b) (c) (d)

13.

(x) (y) (z) (a) (b) (c) (d)

14.

(x) (y) (z) (a) (b) (c) (d)

15.

(x) (y) (z) (a) (b) (c) (d)

High Skill Questions


Directions (Q. Nos. 1-15) In following questions, three figures (x), (y) and (z) showing a sequence of folding a paper are
given. The figure depicts the cut made on the folded paper. Select the answer from the alternatives which would most closely
resemble the third figure when it is unfolded.

Problem Figures Answer Figures


1.

(x) (y) (z) (a) (b) (c) (d)

2.

(x) (y) (z) (a) (b) (c) (d)

3.

(x) (y) (z) (a) (b) (c) (d)


Paper Cutting 189
Problem Figures Answer Figures
4.

(x) (y) (z) (a) (b) (c) (d)

5.

(x) (y) (z) (a) (b) (c) (d)

6.

(x) (y) (z) (a) (b) (c) (d)

7.

(x) (y) (z) (a) (b) (c) (d)

8.

(x) (y) (z) (a) (b) (c) (d)

9.

(x) (y) (z) (a) (b) (c) (d)

10.

(x) (y) (z) (a) (b) (c) (d)

11.

(x) (y) (z) (a) (b) (c) (d)

12.

(x) (y) (z) (a) (b) (c) (d)

13.

(x) (y) (z) (a) (b) (c) (d)


190 Study Package for NTSE

Problem Figures Answer Figures


14.

(x) (y) (z) (a) (b) (c) (d)

15.

(x) (y) (z) (a) (b) (c) (d)

Answers
Elementary Level Questions
1. (c) 2. (b) 3. (d) 4. (c) 5. (d) 6. (b) 7. (c) 8. (d) 9. (c) 10. (c)
11. (b) 12. (b) 13. (c) 14. (d) 15. (b)

High Skill Questions


1. (c) 2. (b) 3. (b) 4. (c) 5. (c) 6. (d) 7. (a) 8. (d) 9. (c) 10. (d)
11. (b) 12. (c) 13. (c) 14. (b) 15. (c)
Chapter

10
Counting of Figures
In these types of problems, a complex figure is given which includes different kinds of geometrical figures and
candidates are asked to find the number of lines, triangles, squares, rectangles or any other geometrical figure
present in such complex figures.

For solving such problems, students must observe the figure carefully and mark the desired geometrical figures in
complex figure and then counted and rechecked. This test are designed to check the analytical disposition of the
candidates. Examples given below, will give you a better idea about the types of questions generally asked in NTSE
examination.
Example 1 Count the number of triangles in the following figure.

(a) 8 (b) 10 (c) 12 (d) 14


Solution (c) We must label the entire figure as shown below. A B C
Count the number of simplest triangles. These are ABH, BHO, BOD, BCD, DEF, DFO, FGH and
FHO. Thus, there are 8 such triangles. Now, count the number of bigger triangles, they are
HBD, BDF, DFH and FHB. Thus, there are 4 such triangles. H D
O

∴ The total number of triangles in the given figure = 8 + 4 = 12


G F E
Example 2 How many triangles are present in the following figure?
A

B E

C D

(a) 14 (b) 10 (c) 12 (d) 16


Solution (c) This figure includes 12 triangles.
∆ACD, ∆ABF, ∆AFE, ∆BCF, ∆CFD, ∆DEF
∆ACF, ∆AFD, ∆CAE, ∆ABD, ∆BCD, ∆CED
192 Study Package for NTSE

Example 3 How many parallelograms are there in the figure given below?
A B C D

F G
E H

I J K L
(a) 18 (b) 16 (c) 19 (d) 17

Solution (a) There are 18 parallelograms.


ABFE, BCGF, CDHG, EIJF, FJKG, GKLH = 6 parallelograms ACGE, BDHF, EGKI, FHLJ = 4 parallelograms
ABJI, BCKJ, CDLK = 3 parallelograms ADHE, EHLI = 2 parallelograms
ADLI = 1 parallelogram ACKI, BDLJ = 2 parallelograms

Target Exercise
Elementary Level Questions
1. How many squares are present in the given figure? 4. How many triangles are contained in the given
figure?

(a) 9 (b) 10
(a) 9 (b) 12
(c) 11 (d) 14
(c) 13 (d) 20
2. How many triangles are there in the figure given
5. Count the number of rectangles in the following
below?
figure.

(a) 6 (b) 7
(c) 10 (d) 8 (a) 21 (b) 18
(c) 19 (d) 20
3. Find the number of rectangles contained in the figure
given below. 6. How many triangles are there in the following figure?
F E D

H J
G C

A K B

(a) 9 (b) 8 (a) 8 (b) 9


(c) 14 (d) 11 (c) 16 (d) 11
Counting of Figures 193
Directions (Q. Nos. 7 and 8) These questions are based on 12. What is the number of hexagon(s) in the following
the figure given below. figure?

7. How many rectangles are there in the figure?


(a) 8 (b) 9 (c) 10 (d) 21 (a) 5 (b) 6 (c) 8 (d) 1

8. How many triangles are there in the figure? 13. How many triangles are there in the following figure?
(a) 10 (b) 14 (c) 18 (d) 12

9. Count the number of triangles in the


figure given below.
(a) 17 (b) 16
(c) 15 (d) 14 (a) 10 (b) 14 (c) 24 (d) 20

10. Count the number of triangles in the figure given 14. How many triangles are there in the following figure?
below.

(a) 16 (b) 24 (c) 28 (d) 32


(a) 8 (b) 9 (c) 10 (d) 11
15. How many squares are there in the figure given
11. What is the number of rectangles in the following below?
figure?

(a) 12 (b) 13
(c) 16 (d) 17
(a) 6 (b) 7 (c) 8 (d) 9

High Skill Questions


1. How many squares does the following figure contain? 2. Count the number of triangles in the figure.
(a) 20 (b) 18 (c) 17 (d) 16

3. How many straight lines are needed to draw the


figure?
(a) 16 (b) 18 (c) 17 (d) 12

4. How many squares are there in the figure?


(a) 20 (b) 19 (c) 11 (d) 12
(a) 19 (b) 20 (c) 25 (d) 27
5. How many triangles are there in the following figure?
Directions (Q. Nos. 2-4) These questions are based on the
figure given below.

(a) 27 (b) 23 (c) 21 (d) 25


194 Study Package for NTSE

6. How many cubes are present in the figure? 11. How many squares are there in the following figure?

(a) 5 (b) 4 (c) 7 (d) 6

(a) 15 (b) 12
12. How many circles are there in the figure given below?
(c) 16 (d) 18

7. How many straight lines are present in the figure?

(a) 11 (b) 12 (c) 16 (d) 14


(a) 11 (b) 12
13. How many squares are there in the following figure?
(c) 14 (d) 13

8. How many squares are there in the following figure?

(a) 14 (b) 16 (c) 22 (d) 12


(a) 24 (b) 25
(c) 26 (d) 27 14. How many quadrilaterals are there in the figure
given below?
9. Count the number of triangles and squares in the
figure given below.

(a) 12 (b) 20
(c) 29 (d) 30

15. How many quadrilaterals are there in the figure


given below?

(a) 26 triangles and 8 squares


(b) 32 triangles and 8 squares
(c) 27 triangles and 9 squares
(d) 30 triangles and 10 squares
(a) 6 (b) 7
10. How many triangles are there in the following figure? (c) 9 (d) 10

16. How many parallelograms are there in the figure


given below?

(a) 22 (b) 18 (a) 14 (b) 15


(c) 20 (d) 24 (c) 18 (d) 16
Hints and Solutions
Elementary Level Questions
A B C D Rectangles are — ABED, DEGF, ABGF, ACHF, BCHG, IJML,
1. (d)
LMON, IJON, IKPN, OPKJ, PQST, STVU, PQUV, OQRV,
OPRU, BCOP, BCJK, JKRU, GHRU, BCRU, and GHJK = 21.
E H
F G A
6. (c)
I J K L
F O E
M N O P
Squares are — AMPD, ABFE, BFCG, CGHD, EIJF, FJKG, C
GKLH, IMNJ, JNOK, KOPL, AIKC, BJLD, EGOM, and B D
FNPH = 14. ABC, ABD, ADC, AEB, AFC, EBC, FBC, FOB, BOD, COD, COE,
2. (c)
A AOE, AOF, AOB, AOC and BOC. Clearly, there are 16 triangles.
Q
7. (d)
A L B
G H
C D
J I E H
I J K
F T R

E F M N O G
B

Triangles are — AGH, HID, IFB, FEJ, JCG, FCD, EIA, JBA, D P C
CHB, and GED = 10.
S
3. (a) FE D
ABCD, EFGH, ADPL, BLPC, ALIJ, IJMN, MNDP, LBJK, JNOK,
NOPC, EIMF, HGOK, HJNG, HIMG, EKOF, EJNF, EIMF, AMNL,
G
H J
C DIJP, LNOB and PJKC.
There are 21 rectangles.

B
8. (d) Triangles are — ∆AQB, ∆BKH, ∆HRG, ∆GOC, ∆CSD, ∆DMF,
A K ∆FTE, ∆EIA, ∆QEH, ∆RBC, ∆SGF, ∆TDA . Clearly, there are 12
Rectangles are — ABDF, KBDE, AKEF, JCDE, JKBC, FHCD, triangles
HCAB, HJEF, and HJAK = 9. N
9. (d)
4. (d) A E
B I E J
A B

I J
M
H F
H F
O
L K
D C
G
D G C
L K
AIH, AIE, EJB, BJF, AHE, EBF, FCK, KCG, EHG, HEF, EFG, FGH, O
FGC, LDG, DLH, GHD, EHO, HGO, GFO and EFO = 20. HNF, LMK, AHI, HNM, BFJ, FMN, DGM, HDM, CGM, FMC, LOM,
Clearly, there are 20 triangles contained in given figure. MOK, IEN and ENJ.
Clearly, there are 14 triangles.
5. (a) B C
A 10. (d) A
D E D

F H E
G F
J C
I K G J B
L M
P Q H
N O
S T I
R U V Triangles are— ACF, CFG, AEF, AGJ, CGH, CGI, GHI,
ACI, ACG, ACJ and BCD. Clearly, there are 11 triangles.
196 Study Package for NTSE

11. (d) A F L B 14. (d) A G


B
M
I J
E J
K G O
I
E F
P Q
D H C K
L N
AELJ, AFKE, FKJL, HLBC, LIGB, HIGC, EJHD, ALHD and ABCD. D H C
Clearly, there are 9 rectangles. There are sixteen triangles in the main figure
12. (d) B
ABCD —AOG, GOB, BOF, FOC, CON, HOD, DOE, EOA, AOB,
BOC, COD, DOA, ABC, BCD, CDA and DAB.
C D
A E Similarly, there are 16 more triangles in section I J K L of the figure.
Hence, there are total 32 triangles.
F L M 15. (d) A F
B

G J K M J
I I
Q V R
H
CDMJIF ⇒ Number of Hexagon = 1 E N W U
G
X O
13. (c) A
T Y S
E O F K P L
C
B

D C D
H
BEA, EOA, AOF, AFC, BED, EDO, DOF, DFC, BOA, AOC, COD, ABDC, EFGH, IJLK, MNPO, QRST, VUXR, VUQW, UYTW, UXSY,
DOB, BAD, DCA, BAC, BDC, AEF, EDF, DEA, AFD, BAF, EAC, MUNI, MUOJ, UPLO, NUPK, AFUE, FUGB, UGDH and ECHU.
BDF and DEC. Clearly, there are 24 triangles. = There are 17 squares.

High Skill Questions ABM, MA1A 2 , CDR, RA 3 A 4 , BMO, MOA 2 , CPR, PRA 3 , MBA 2 ,
CRA 3 , MEW, RHZ, BEF, CGH, WXA 2 , YZA 3 , EMN, MNW, HQR
1. (d) A
I J
B and QRZ. Clearly, there are 20 triangles.
E K L
F
3. (c) AD, AA1, A1A 4 , DA 4
M
N O P Q
W BA 2 , CA 3 , CR, RA 3
S T U V EH, IL, SV, WZ, MR
R X
EW, HZ
H MB, MA 2 . These 17 lines are needed.
G Y Z
C D 4. (a) ADA1 A 4 , EHWZ, ABMU, MOA1A 2 ,
A1 A2
RA3A4, CDPR, BCOP, OPA2A3, EFIJ, FGJK, GHKL, IJNO, JROP,
ABCD, EFGH, IJKL, KLOP, OPTU, TUYZ, YZA1A 2 , MNRS, KLPQ, NOST, OPTV, PQUV, STWX, TUXY and UVYZ. Clearly,
NSOT, AVPU, QVWX, EKNO, LFPQ, STGY, UVZH, UXA 2 D, there are 20 squares.
RTCA1, AIMO, JPBW, ELSU, KFTV, QOYH, NPGZ, AJRU, 5. (c) A F B
OWDA1, IBTX and MPCA 2 . Clearly, there are 27 squares.
G M N H
2. (a) B C
S K R
A D
L J
E F G H
J K L
I E P I Q D
M N O P Q R
S T U V
C
X Y
W Z
The triangles are as follows.
ABC, AFC, FBC, GHI, GKI, KHI, FED, FEI, FDI, FMN, FMK, FNK,
A1 A2 A3 A4
PQC, PIC, IQC, ALF, GSM, NHR, JDQ, EPL and FBJ. Clearly,
there are 21 triangles.
Counting of Figures 197
6. (a) Number of cubes in first lower layer = 9 11. (c) A G B

I J

E O F

Number of cubes in second layer = 5


L K
Number of cubes in third layer =1 C
∴ Total number of cubes = 9+5+1 = 15 D H

7. (b) A
F H Clearly, there are total 7 squares in the figure namely, ABCD,
B
EGFH, IJKL, EIOL, IGJO, OJFK and KHLO.
J
E 12. (c)

G L 1 3
2 7
D C
I K 4 6
Straight lines — AD, DC, BC, AB, EK, FL, AC, FE, HG, BD, 8 5 12
JI and LK. 9 11

There are 12 straight lines in the figure. 13 10 16


A E G I K B
8. (c) 14 15

M N
Q R S T There are 16 circles in the given figure.
O U V W X
P 13. (a) A B
row 4
D L C C
F H J D G
There are five squares in upper row of the main figure namely. H
row 3
E I
AEQM, EGRQ, GISR, IKTS and KBNT. F row 2
Q
Hence, there are fifteen squares, five in each row of the main P S J
R
figure. Similarly, there are total eight squares, (AOVG, EUWI, row 1
GVXK, IWPB, MDHR, RHLT, QFJS, SJCN) and finally there are
O
three squares (AIJD, EKLF and GBCH) from main figure. N M L K
∴ Total squares = 26 Clearly, there are four, three, two and one squares in the first,
9. (b)
A second, third and fourth row (counting from bottom),
C D
respectively. Besides there are four more squares namely, :
B
DGQR, EHMO, FILN and RFQN = 14 squares.
F
E G H I 14. (d) A B C D E
J L
K M N O
L F
M
K G
N O P J I H

ABDG, GJLM, BDJL, JLNP, BCFG, CDGH, FGJK, and GHKL. Smallest quadrilaterals = ABML, BCNM, CDON,
MNO, MOP, KML, KJM, JMN, LMP, MNP, JCM, GJK , GKL, GJL, DOFE, LKJM, MNIJ, NIHO, OFGH = 8
FGJ, GHL, BGJ, DGL, BFG, DGI, BGE, DGH, DHI, BEF, BPG, Quadrilaterals formed with two quadrilaterals
ABC, ACD, ABD, AEI, BDI, DJL, JNP, LNP, GLM and GJM. = ABJK, BCIJ, CDHI, DEGH, ACNL,
There are 8 squares and 32 triangles in this figure. CEFN, LNIK, NFGI, BDOM, MOHJ = 10
A G I K Quadrilaterals formed with three quadrilaterals
10. (a) B
P Q = ADOL, BEFM, LOHK, MFGJ = 4
F
Quadrilaterals formed with four quadrilaterals
E
M N O = ACIK, CEGI, BDHJ, AEFL, LFGK= 5
R S Quadrilaterals formed with six quadrilaterals
D H J L C = ADHK, BEGJ = 2
ABFE → PEM, PGI, AEI, EIN, IKQ, QOF, IBF, INE, IEF Largest quadrilateral = AEGK = 1
EFCD → has another 9 triangles and 4 more triangles. ∴Total number of quadrilaterals = 8 + 10 + 4 + 5 + 2 + 1
EPR, EIJ, IFJ and QFS. = 30
Hence, total number of triangles = 9 + 9 + 4 = 22
198 Study Package for NTSE

15. (c) B C From the above diagram parallelograms are as follow

G (i) Parallelograms from the outer figure


A D = ABCD, AIJD, GBCH, ADHG,
H
GHJI, IJCB = 6
F E (ii) Parallelograms from the upper half of the figure
9 quadrilaterals are as follow = AGKE, GILK, IBFL, AILE,
~ AGDH, ~ BAFD, ~ CAED, ~ AGDF, ~ AGDE, ~ ABDH, GBFK, ABFE = 6
~ ACDH, ~ ACDF, and ~ ABDE (iii) Parallelograms from the lower half of the figure
16. (c)The figure in question has been labelled at different points as = EKHD, KLJH, LFCJ, KFCH,
shown in the following figure ELJD, EFCD = 6
∴ Total number of Parallelograms = 6 + 6 + 6 = 18
A G I B

K L F
E

D H J C
Chapter

11
Cubes and Dice
Cube A cube is a three dimensional block, having 8 corners, 6 surface and 12 edges.
Dice Dice is a three dimensional figure with 6 surfaces. It may be in the form of a cube or a cuboid.
In this chapter, we will study about the problems based on cubes and dice which is as follow
F E
H
G Corner
Edge Face
(surface)
D C
A a B

Type I Problems Based on Cubes


(i) Volume of cube = a 3unit, if a unit is the edge length of cube.
3
(ii) The number of small cubes of edge ‘b’ unit to be cut from the large cube of edge ‘a’ unit is   .
a
 b
(iii) If a cube is painted on all its outer surface with a colour and further divided into small cubes, then different kind of small
cubes can be computed as given below
2
(a) Number of smaller cubes with three painted surfaces = 8
3 2 3
2 1 1 2
2 1 1 2 3
(b) Number of smaller cubes with two painted surfaces = ( N − 2) × 12
3 2 2 3 2
2
3 2 2 3 3
(c) Number of smaller cubes with one painted surface = ( N − 2)2 × 6
2
1
1
(d) Number of smaller cubes with no painted surface = ( N − 2) 3 2 1 1 2 2 2
1
(where, N is the number of divisions on the surfaces of the bigger cube) 2
1
3
2 1 1 2
2
Edge length of big cube
N=
2
3 2 2 3 3
Edge length of smaller cube
(iv) Counting the number of cubes in the figure.
To count the number of total cubes, firstly each column and then add column wise sum to find the total number of cubes
in the figure.
Example 1 A cube of side 6 cm is painted white on all of its surfaces and then divided into various smaller cubes of
side 1 cm each. Then, determine
(i) number of smaller cubes.
(a) 218 (b) 108 (c) 216 (d) 105
(ii) number of smaller cubes with three painted surfaces.
(a) 7 (b) 9 (c) 5 (d) 8
(iii) number of smaller cubes with two painted surfaces.
(a) 56 (b) 46 (c) 49 (d) 48
200 Study Package for NTSE
(iv) number of smaller cubes with one painted surfaces.
(a) 96 (b) 108 (c) 56 (d) 110
(v) number of smaller cubes with no painted surfaces.
(a) 54 (b) 64 (c) 61 (d) 69
Solution (i) (c) Total number of small cubes = 6 × 6 × 6 = 216
1×1×1
(ii) (d) Number of smaller cubes with three painted surfaces = 8
(iii) (d) Number of smaller cubes with two painted surfaces = (N – 2) × 12 = ( 6 − 2) × 12 = 48
(iv) (a) Number of smaller cubes with one painted surface = ( N − 2) 2 × 6 = ( 6 − 2) 2 × 6 = (4 ) 2 × 6 = 96
(v) (b) Number of smaller cubes with no painted surface = ( N − 2) 3 = ( 6 − 2) 3 = (4 ) 3 = 64

Example 2 Count the number of cubes in the given figure.

(a) 80 (b) 85 (c) 88 (d) 90


Solution (c) There are 9 columns have 5 cubes and 7 columns have 4 cubes each, 5 columns have 3 cubes each.
So, total number of cubes = ( 9 × 5) + (7 × 4 ) + (5 × 3 ) = 88

Type II Construction of Boxes


If cube or cuboid is unfolded, it may appear in any of the following forms.

1 a

1 1 2 1 2 1

2 3 4 3 2 3 3 4 d 2 3 4 b

5 4 4 5 5
a
6 5 6 5 6 6 c d b
c
(A) (B) (C) (D) (E)

Opposite faces Opposite faces Opposite faces Opposite faces


 1 ↔ 5  1 ↔ 6 1 ↔ 4  1 ↔ 3
       
2 ↔ 4 2 ↔ 4  2 ↔ 6  2 ↔ 5
 3 ↔ 6  3 ↔ 5  3 ↔ 5  4 ↔ 6
       
Example 3 Select from the alternatives, the box that can be formed by folding the sheet shown in figure (X).

(X) (A) (B) (C) (D)

(a) Only A (b) A, B and C (c) B and C (d) A, B, C and D


Solution (d) Sheet in figure (X) is folded, then the face bearing a ∆ appear opposite to a blank face, the face
bearing a sign appears opposite to another blank face and the face bearing a black dot appears opposite to the
third blank face. Clearly, all the four cubes shown in figures (A), (B), (C) and (D) can be formed.
Cubes and Dice 201
Type III Problems Based on Dice
A dice is a three-dimensional figure with six surfaces. It may be in the form of a cube or a cuboid. After observing
these figures, we have to find the opposite side of the dice.
E
H
A
D

G
F
B
C

Front surface → ABCD


opposite
Back surface → EFGH
Top surface → ADHE
opposite
Bottom surface → BCGF
Left surface → ABFE
opposite
Right surface → DCGH
Types of Dice
There are two types of dice.

1. Ordinary Dice
In this type of dice, the sum of opposite sides is not 7. But the sum of two adjacent sides are 7.
2. Standard Dice
In such type of dice, the sum of opposite sides is 7.
3 1
7=+ 4 5 5 4

(i) Ordinary (ii) Standard


dice dice

Here, In dice (ii), 1 + 4 = 5, 4 + 5 = 9, 1 + 5 = 6


Opposite of 1 → 6, Opposite of 2 → 5, Opposite of 3 → 4
Example 4 A dice is thrown four times and its four different positions are given below. Find the number on the face
opposite the face showing 6.
6 6 5 1
3 2 2 4 6 4 4 2

(A) (B) (C) (D)

(a) 1 (b) 2 (c) 3 (d) 4


Solution (a) From figures (A), (B) and (C), it is clear that the numbers 3, 2, 4 and 5 lie adjacent to the number 6,
so 1 lies opposite 6.
Target Exercise
Elementary Level Questions
Directions (Q. Nos. 1-5) In the figures below marked (X) at left hand side is folded in such a way that it could form the
shape of a box. You have to choose one of the choices given as (A), (B), (C) and (D) that is similar as the box formed.
1. 5.

(X) (A) (B) (C) (D)


(X) (A) (B) (C) (D)
(a) ‘A’ and ‘B’ (b) ‘B’ and ‘D’
(a) ‘A’ and ‘B’ (c) ‘B’ and ‘C’ (d) ‘A’ and ‘D’
(b) ‘B’ and ‘C’
(c) ‘B’ and ‘D’ 6. Which of the given net from the answer options when
(d) ‘A’, ‘B’ and ‘C’ folded will results in the given cube?
2. +
@
÷

÷ +
(a) + @ (b) ÷ @
#

(X) (A) (B) (C) (D) ÷


(c) + (d) ÷ @ +
(a) ‘A’ and ‘D’
(b) ‘C’ and ‘D’ @
(c) ‘A’ and ‘B’
(d) ‘B’ and ‘C’
7. Which of the following figures marks (A), (B), (C) and
(D) can be fitted suitably in the figure marked ( X)?
3.

(X) (A) (B) (C) (D)


(a) Only A (b) Only B
(X) (A) (B) (C) (D) (c) Only C (d) Only D
(a) ‘C’ and ‘D’ 8. Which number is opposite to 3?
(b) ‘A’ and ‘D’
(c) ‘B’ and ‘C’ 6 5 3 2
(d) ‘B’ and ‘D’ 3 1 2 4 5 1 3 6
4. (I) (II) (III) (IV)
(a) 2 (b) 3 (c) 4 (d) 6

9. If the total number of dots on opposite faces of a dice


is 7. Find the correct figure among the following.

(X) (A) (B) (C) (D)


(a) Only ‘A’ (b) ‘B’ and ‘C’ (a) (b) (c) (d)
(c) ‘A’ and ‘C’ (d) ‘A’, ‘B’ and ‘D’
Cubes and Dice 203
10. Which one of the following figures is correct, if total 13. If number 1 is marked on the bottom, which number
number of dots on opposite faces of a dice always will be on the top?
remains 7?

(I) (II)
(a) (b) (c) (d) (a) 1 (b) 2 (c) 3 (d) 6

11. Which number is opposite to face 3? 14. If number 4 is placed on the bottom, which number
will be on the top?
2 3 4
1 3 1 5 2 3

(I) (II) (III)


(I) (II)
(a) 1 (b) 6 (a) 3 (b) 4 (c) 5 (d) 6
(c) 5 (d) 4
15. All the sides of the dice are painted with the paints
12. If the bottom face is marked as number 1, green, blue, red, yellow, orange and white. Some of
which number will be on the top among the following the faces of the dice are visible. Which colour will be
two figures? on the top when bottom is Blue?

White n ge
Ora
Blue Red
Green Yellow
(I) (II)
(I) (II)
(a) 2 (b) 3 (c) 4 (d) 5 (a) Orange (b) Red (c) White (d) Yellow

High Skill Questions


Directions (Q. Nos. 1 and 2) Read the following 4. Which of the following does not form correct pair on
information carefully and answer the questions that follow. opposite faces?
(a) Circle-rectangle
An electronic dealer selling colour TVs, VCD players and (b) Pentagon-cross mark
computer (PCs), is offering stereo radio, movable trolley and (c) Rectangle-cross mark
stainless steel rack free with these. The item, which goes (d) Triangle-question mark
free with a particular item is written on the face opposite it
on a cube. Two views of the cube are shown below. Directions (Q. Nos. 5-7) Read the following information
CTV VCD carefully and answer the questions that follow.
D
VC PC
Rack Trolley Ram paint a cubical box with three different colours,
green, white and yellow with instructions that two
1. Stereo radio is being offered free with opposite faces must have the same colour. He was asked
(a) colour TV (b) computer
to cut this box into twenty seven small cubes of equal
(c) VCD (d) VCD or TV
size.
2. The item offered free with the computer is
(a) stainless steel rack (b) trolley
5. How many small cubes will be there with no face
(c) radio (d) radio or trolley painted at all?
(a) 2 (b) 8
(c) 4 (d) 1
Directions (Q. Nos. 3 and 4) Study carefully the three
views of the same cube shown below and then answer the 6. How many cubes will be there with all the three
questions. colours on them?
(a) 3 (b) 5
(c) 8 (d) 4

7. How many cubes will be there with only one face


3. The figure on the face opposite the triangle is painted in blue?
(a) pentagon (b) circle (a) 4 (b) 6
(c) question mark (d) rectangle (c) 2 (d) 3
204 Study Package for NTSE

Directions (Q. Nos. 8-10) Count the number of cubes in Directions (Q. Nos. 12-14) Read the following
the given figures. information carefully and answer the questions that follow.
8. The six faces of a cube are coloured black, brown, green,
red, white and blue.
(i) Red is opposite to black.
(ii) Green is between red and black.
(iii) Blue is adjacent to white.
(iv) Pink is adjacent to blue.
(v) Red is at the bottom.
12. Which colour is opposite to pink colour?
(a) White (b) Red (c) Green (d) Blue
(a) 68 (b) 69
(c) 70 (d) 71 13. The four colours adjacent to one another are
(a) black, blue, pink and red (b) black, blue, pink and white
9. (c) black, blue, red and white (d) black, pink, red and white

14. Which colour of the following, can be deduced from (i)


and (v)?
(a) Black is on the top (b) Blue is on the top
(c) Pink is on the top (d) Red is on the top
(a) 25 (b) 35 15. A cube has six different symbols drawn over its six
(c) 32 (d) 34 faces. The symbols are •, , ∆, ×, # and >→. Three
10. different positions of the cube are shown in figures (I),
(II) and (III) below.

(I) (II) (III)


Which symbol is opposite to triangle?
(a) >→ (b) (c) × (d) •
(a) 45 (b) 46
16. The opposite faces of Dice X are :
(c) 48 (d) 49
[(1, 3), (4, 5), (2, 6)]
11. From a 10 × 10 × 10 cube, which is formed by
combinations of 1 × 1 × 1 cubes, a layer of the smaller The opposite faces of Dice Y are :
cubes is removed, at each corner. What will be the [(2, 1), (5, 4), (3, 6)]
number of 1 × 1 × 1 cubes present in this new cube? Which figure can represent both Dice X and Dice Y
(a) 900 with faces shown below ?
(b) 488
3 6 4 2
(c) 512 5 2 6 5
1 5 2 3
(d) 729
(a) (b) (c) (d)

Hints and Solutions


Elementary Level Questions
1. (d) The half shaded faces are opposite to each other. On careful 5. (c) The figure (X) is similar to form E. So, the four triangular
observation of figure (X), we can find that cube ‘A’, ‘B’ and ‘C’ are positions combine to form a single face of the cube. Therefore,
possible. cubes ‘A’ and ‘D’ cannot be formed. As per the pattern cubes ‘B’
and ‘C’ can be formed.
2. (a) On careful study of the figure (X), we can observe that on
folding the figure (X), cubes ‘A’ and ‘D’ can be formed. 6. (b) From the given figure, +, @ and ÷ are adjacent to each other
and they cannot be opposite to each other. In net (a), + and @, in
3. (a) As per the given question, two dots can not be adjacent.
net (c) ÷ and @, in net (d), ÷ and + are opposite to each other, so
Hence, figure A, C and D are similar to the figure (X).
these nets are incorrect.
4. (c) The shaded face and the face bearing the square will lie
Hence, only net (b) is correct.
opposite to each other. So, the cubes ‘B’ and ‘D’ cannot be
formed. Cubes ‘A’ and ‘C’ can be formed by folding the figure (X). 7. (a) Only figure ‘A’ can be formed by folding the figure (X).
Cubes and Dice 205
8. (c) From figures I, III and IV, it is clear that numbers 6, 1, 5 and 2 12. (b) It is clear from the figures that 4, 5, 2 and 6 dots cannot
are adjacent to 3. So, number 4 is opposite to 3. appear opposite to 3 dots. So, 1 dot appears opposite to 3 dots.
9. (c) Since, the sum of the number of dots on opposite faces of the Therefore, if 1 dot is at the bottom, then 3 dots will be on the top.
block is always 7, we cannot get 1 dot adjacent to 6 dots, 2 dots 13. (d) It is clear from the two positions of a dice, that 1 is opposite
adjacent to 5 dots and 3 dots adjacent to 4 dots. So, the figures to 6.
‘a’, ‘b’ and ‘d’ cannot be correct.
14. (a) It is clear from the figures that 2, 4, 3 and 5 dots cannot appear
10. (a) Since, the sum of the number of dots on opposite faces of the opposite to 1 dot. So, 6 dots appear opposite to 1 dot. Assume
block is always 7, we cannot get 1 dot adjacent to 6 dots, that 6 dots are on the face opposite to 1 dot in the two figures
2 dots adjacent to 5 dots and 3 dots adjacent to 4 dots. So, the and rotating figure I in the position of figure II, we get that 3 dots
figures ‘b’, ‘c’ and ‘d’ cannot be correct. are opposite to 4 dots. Therefore, if 4 dots will be on the bottom,
11. (b) It is clear from the given figures that the numbers 1, 2, 4 and 6 then 3 dots will be on the top.
appear adjacent to number 3. Therefore, none of these can 15. (b) By rotating first dice in the position of second dice, we get the
appear opposite to number 3. Therefore, number 6 appears blue colour opposite to red colour. Therefore, if blue is on the top,
opposite to number 3. then red will be at the bottom.

High Skill Questions


1. (c) Since, neighbouring faces of VCD have CTV, rack, trolley and 11. (c) A 10 × 10 × 10 cubes have 10 cubes of 1 × 1× 1 cubes along its
PC, it follows that stereo radio is written opposite the VCD and length, breadth and width each. If we remove a layer of 1 × 1 × 1
hence, is offered free with it. cubes (the smaller cubes), then the new cubes will have
following dimensions
2. (a) Considering the neighbouring of rack (figure I), these are VCD,
Length = 10 − 1 − 1 = 8
CTV, radio (which is opposite to VCD) and trolley (PC) cannot be its
Breadth = 10 − 1 − 1 = 8
neighbour, because then, it goes free with CTV, (which is not true).
Hence, rack must be free with a Personal computer. Width = 10 − 1 − 1 = 8
So, the number of 1 × 1 × 1 cubes in this 8 × 8 × 8 cube
3. (c) The neighbouring faces of the face having a triangle on it have = 8 × 8 × 8 = 64 × 8 = 512
circle, cross mark, rectangle and pentagon marked on these as
can be seen from figures (I) and (III). Therefore, the question mark Sol. (Q. Nos. 12-14)
must be opposite the triangle. On the basis of the given informations, the given cube looks like
having colours on the faces as given below.
4. (b) Pentagon is opposite the cross mark.
Top Black
5. (d) Number of smaller cubes without painted (no surface Side 4
painted) = (N − 2 )3
[Q 27 smaller cubes, it means N = 3] Pink
= (3 − 2 )3 = 1 Side 2
White Side 1 Side 3
6. (c) Numbers of cubes have all three colours = 8, because all Blue Green
corner cubes have all three colours.
7. (c) Total number of cubes having colour on one face Bottom Red
= (N − 2 )2 × 6
12. (a) Side 4 is pink coloured and the opposite will be side 2, which
= (3 − 2 )2 × 6 = 6 is coloured white.
But only two faces have blue colour, so number of cubes will be 13. (d) Black, pink, red and white are adjacent colours, which lie
6
there with only one face painted blue = × 2 = 2 respectively on top, side 4, bottom and side 2.
6
14. (a) Red is opposite to black and red is at the bottom implies
8. (b) In the figure, there are 11 columns containing 4 cubes each, 7 ‘black is on the top’.
columns containing 3 cubes each and 2 columns containing 15. (a) Arrow is opposite to triangle, because •, , # and × are
2 cubes each.
adjacent figures.
∴ Total number of cubes = (11 × 4) + (7 × 3) + (2 × 2 )
= 44 + 21 + 4 = 69 16. (d) In figure (a), 3 and 1 are adjacent to each other, so this dice is
incorrect for Dice X.
9. (b)∴ Total number of cubes = 5 × 3 + 3 + 6 + 6 + 2 + 3 = 35 In figure (b) and (c), 6 and 2 are adjacent to each other, so this
10. (b) There are 21 columns containing 2 cubes each and dice is incorrect for Dice X.
4 columns of 1 cube each. So, the number of cubes in this figure In figure (d), no opposite faces of Dice X and Y are adjacent to
= (21 × 2 ) + (4 × 1) = 46 each other. So, option (d) is the correct figure.
Chapter

12
Dots Situation
In such problems based on dot situation involve selection of a trace of similar condition in the alternative figures as
indicated in the problem figure.
The problem figure contains dots placed in the spaces enclosed between the combinations of geometrical figures
like triangle, square, circle etc. Select in one of these dots, we observe the region in which the dot is enclosed, i.e., to
which of the four figures is this region common. The candidates, now has to observe for such a region in the four
alternatives given. After having found it the same procedure has to be repeated to find the other dot if any.
Examples given below will give you a better idea about the type of questions generally asked in NTSE examination.

Example 1 From amongst the figures marked Example 2 From amongst the figure marked
(a), (b), (c) and (d) select the figure, which satisfies (a), (b), (c) and (d) select the figure, which satisfies
the same conditions of placement of dots as in the name conditions of placement of dots as in
figure. (X). figure ( X ).

(X) (a) (b) (c ) (d) (X) (a) (b) (c) (d)

Solution (c) In figure (X), the dot lies in the Solution (b) In figure (X), the dot lies in the
region common to the circle and triangle. Such a region common to the circle, triangle, rectangle.
region is present in figure (c). Such a region is present in figure (b).

Target Exercise
Elementary Level Questions
Directions (Q. Nos. 1-13) In each of the following question, there is a diagram marked (X), with one or more dots placed
in it. This diagram is followed by four other figures, marked (a), (b), (c) and (d) only one of which is such as to make possible
the placement of the dot (s) satisfying the same conditions as in the original diagram. Find the correct alternative in each
case.

1. 3.

(X) (a) (b) (c) (d) (X) (a) (b) (c) (d)

2. 4.

(X) (a) (b) (c) (d) (X) (a) (b) (c) (d)
Dots Situation 207
5. 9.

(X) (a) (b) (c) (d) (X) (a) (b) (c) (d)
10.
6.

(X) (a) (b) (c) (d)


(X) (a) (b) (c) (d) 11.
7.
(X) (a) (b) (c) (d)
12.
(X) (a) (b) (c) (d)

8.
(X) (a) (b) (c) (d)
13.
(X) (a) (b) (c) (d)

(X) (a) (b) (c) (d)

High Skill Questions


Directions (Q. Nos. 1-10) In each of the following questions, there is a diagram marked (X), with one or more dots placed in it.
This diagram is followed by four other figures, marked (a), (b), (c) and (d) only one of which is such as to make possible the
placement of the dot (s) satisfying the same conditions as in the original diagram. Find the correct alternative in each case.
1. 6.

(X) (a) (b) (c) (d) (X) (a) (b) (c) (d)

2. 7.

(X) (a) (b) (c) (d) (X) (a) (b) (c) (d)

3. 8.

(X) (a) (b) (c) (d) (X) (a) (b) (c) (d)

4. . 9.
.

(X) (a) (b) (c) (d) (X) (a) (b) (c) (d)
5. 10.

(X) (a) (b) (c) (d) (X) (a) (b) (c) (d)

Elementary Level Questions


Answers
1. (b) 2. (a) 3. (d) 4. (b) 5. (c) 6. (c) 7. (a) 8. (c) 9. (c) 10. (c)
11. (c) 12. (d) 13. (a)
High Skill Questions
1. (b) 2. (d) 3. (d) 4. (d) 5. (c) 6. (c) 7. (a) 8. (b) 9. (d) 10. (c)
Chapter

13
Grouping of
Identical Figures
Grouping of different figures on the basis of certain common properties/qualities, is called grouping of identical
figures.
In this chapter, we have a set of few figures or geometrical shapes which are numbered as 1, 2, 3, 4 and so on. We are
required to analyse these figures and classify them into groups consisting of figures having same properties.
Grouping of figures can be done on the basis of their shape, orientation, the number of elements and other such
characteristics. The best answer is to be selected from a given set of fairly close alternatives.
Examples given below, will give you a better idea about the type of questions generally asked in NTSE examinations.
Direction (Ex. 1) In the following question, group the given figures into three classes using each figure only once .

(1) (2) (3) (4) (5) (6) (7) (8) (9)

(a) (7, 8, 9); (2, 4, 3); (1, 5, 6) (b) (1, 3, 2); (4, 5, 7); (6, 8, 9)
(c) (1, 6, 8); (3, 4, 7); (2, 5, 9) (d) (1, 6, 9); (3, 4, 7); (2, 5, 8)
Solution (d) By observing the given figures, we found that, figures (1), (6) and (9) can be grouped together, as
they all are triangles. Figures (3), (4) and (7) can be grouped together, as they all are quadrilaterals and figures
(2), (5) and (8) can be grouped together, as they all are pentagons.

Direction (Ex. 2) There are two classes of three figures each. Class ‘A’ figures differ in certain way from the figures in class
‘B’. Which two of the four answer figures belong to class ‘A’?
Answer Figures

Class A Class B (1) (2) (3) (4)

(a) Both 1 and 3 (b) Both 1 and 2


(c) Both 2 and 4 (d) Both 2 and 3
Solution (d) By observing the given figures, we found that, each figure in class ‘A’ consists of two similar closed
figures, which are placed one inside the other.
Target Exercise
Elementary Level Questions
Directions (Q. Nos. 1-10) In each of the following question, group the given figures into three classes using each figure only
once.
1.

(1) (2) (3) (4) (5) (6)

(a) (1, 4); (2, 3); (5, 6) (b) (1, 5); (2, 6); (4, 3) (c) (1, 6); (2, 3); (4, 5) (d) (1, 2); (3, 6); (4, 5)

2.

(1) (2) (3) (4) (5) (6) (7) (8) (9)

(a) (1, 3, 4); (6, 7, 8); (2, 5, 9) (b) (1, 2, 3); (4, 5, 6); (7, 8, 9) (c) (1, 5, 9); (2, 4, 7); (3, 6, 8) (d) (3 7, 8); (1, 6, 5); (4, 2, 9)

3.

(1) (2) (3) (4) (5) (6) (7) (8) (9)


(a) (1, 5, 7); (2, 4, 6); (3, 9, 8) (b) (1, 5, 7); (2, 4, 8); (3, 6, 9) (c) (1, 5, 7); (4, 8, 9); (2, 3, 6) (d) (1, 5, 7); (3, 8, 9); (2, 4, 6)

4.

(1) (2) (3) (4) (5) (6) (7) (8) (9)

(a) (1, 4, 9); (2, 6, 8); (3, 5, 7) (b) (1, 4, 5); (3, 6, 8); (2, 7, 9) (c) (1, 4, 6); (2, 7, 8); (3, 5, 9) (d) (1, 3, 6); (2, 4, 7); (5, 8, 9)

5.

(1) (2) (3) (4) (5) (6) (7) (8) (9)

(a) (1, 2, 4); (3, 5, 6); (8, 7, 9) (b) (9, 7, 6); (5, 3, 1); (4, 8, 2) (c) (2, 3, 4); (7, 6, 5); (9, 8, 1) (d) (1, 4, 7); (2, 6, 9); (3, 5, 8)

6.

(1) (2) (3) (4) (5) (6) (7) (8) (9)


(a) (1, 5, 7); (2, 3, 9); (4, 6, 8) (b) (1, 3, 9); (2, 4, 6); (5, 7, 8) (c) (3, 4, 7); (2, 4, 9); (1, 6, 8) (d) (6, 7, 9); (1, 3, 4); (2, 4, 8)

7.

(1) (2) (3) (4) (5) (6) (7) (8) (9)

(a) (1, 2, 3); (4, 8, 9); (5, 7, 6) (b) (4, 5, 7); (3, 1, 2); (7, 8, 9) (c) (1, 3, 7); (4, 8, 9); (2, 5, 6) (d) (3, 5, 6); (8, 7, 4); (9, 1, 2)

8.

(1) (2) (3) (4) (5) (6) (7) (8) (9)

(a) (1, 4, 8); (2, 5, 7); (3, 9, 6) (b) (1, 4, 6); (2, 5, 8); (3, 7, 9) (c) (1, 4, 6); (2, 5, 7); (3, 8, 9) (d) (1, 2, 3); (4, 5, 6); (7, 8, 9)

9.

(1) (2) (3) (4) (5) (6) (7) (8) (9)

(a) (1, 5, 6); (2, 3, 4); (7, 8, 9) (b) (1, 2, 4); (3, 5, 8); (6, 7, 9) (c) (5, 6, 7); (1, 2, 4); (3, 8, 9) (d) (1, 2, 4); (3, 5, 7); (6, 8, 9)
210 Study Package for NTSE

10.

(1) (2) (3) (4) (5) (6) (7) (8) (9)


(a) (1, 5, 7), (2, 3, 8), (4, 6, 9) (b) (3, 6, 7), (1, 4, 8), (2, 5, 9) (c) (1, 4, 9), (2, 5, 6), (3, 7, 8) (d) (1, 2, 6), (3, 4, 5), (7, 8, 9)

High Skill Questions


Directions (Q. Nos. 1-7) In each of the following questions, group the given figures into three classes using each figure only
once.
1.

(1) (2) (3) (4) (5) (6) (7) (8)

(a) (1, 4, 6); (3, 5); (2, 7, 8) (b) (2, 5, 8); (1, 3, 8); (4, 6) (c) (3, 7); (14, 5); (2, 5, 8) (d) (2, 5, 8); (1, 6); (4, 7)

2.

(1) (2) (3) (4) (5) (6) (7) (8) (9)


(a) (1, 5, 8); (3, 4, 7); (2, 6, 9) (b) (1, 3, 6); (4, 5, 9); (2, 7, 8) (c) (1, 3, 6); (2, 5, 7); (4, 8, 9) (d) (6, 7, 8); (1, 3, 7); (2, 4, 9)

3.

(1) (2) (3) (4) (5) (6) (7) (8) (9)


(a) (1, 5, 8); (2, 6, 7); (3, 4, 9) (b) (1, 5, 7); (2, 6, 8); (3, 4, 9) (c) (1, 7, 8); (2, 6, 9); (3, 4, 5) (d) (1, 5, 8); (2, 4, 7); (3, 6, 9)

4.
A M B H W D E N U
(1) (2) (3) (4) (5) (6) (7) (8) (9)
(a) (1, 3, 6); (2, 8, 9); (4, 7, 5) (b) (1, 4, 8); (2, 5, 7); (3, 6, 9) (c) (1, 4, 7); (2, 5, 8); (3, 6, 9) (d) (1, 4, 7); (3, 5, 8); (2, 6, 9)

5.

(1) (2) (3) (4) (5) (6) (7) (8) (9)

(a) (1, 4, 7); (3, 6, 9); (2, 5, 8) (b) (1, 4, 5); (2, 6, 8); (3, 7, 9) (c) (1, 7, 9); (3, 6, 8); (2, 4, 6) (d) (1, 6, 9); (2, 5, 8); (3, 4, 7)

6.

(1) (2) (3) (4) (5) (6) (7) (8) (9)

(a) (1, 4, 7); (3, 6, 9); (2, 5, 8) (b) (1, 4, 7); (2, 6, 9); (3, 5, 8) (c) (1, 6, 9); (2, 4, 7); (3, 5, 8) (d) (1, 5, 7); (2, 6, 9); (3, 4, 8)

7.

(1) (2) (3) (4) (5) (6) (7) (8) (9)


(a) (1, 4, 7); (2, 5, 9); (3, 8, 6) (b) (2, 6, 9); (1, 4, 7); (5, 8, 3) (c) (1, 4, 7); (2, 3, 6); (5, 8, 9) (d) (3, 5, 1); (4, 7, 8); (6, 2, 9)

8. There are two classes of three figures each. Class ‘A’ figures differ in certain way from the figures in class B. Which
two of the four answer figures belong to class ‘A’?

Class A Class B (1) (2) (3) (4)


(a) Both 1 and 3 (b) Both 1 and 2 (c) Both 2 and 4 (d) Both 2 and 3
Hints and Solutions
Elementary Level Questions
1. (a) By observing the given figures, we found that, figures 7. (c) By observing the given figures, we found that, figures
(1 and 4) are similar. (1, 3 and 7) have three petals, three rectangles and three
Figures (2 and 3) are similar, as they all are triangles. triangles, respectively. So, they should be grouped in one
Figures (5 and 6) are similar. group. Figures (4, 8 and 9) have one triangle, one rectangle
and one oval respectively. So, they should be grouped in
2. (a) By observing the given figures, we found that, one group.
figures made by three lines = (1, 3, 4) Figures (2, 5 and 6) have five petals, five triangles and five
Figures made by four lines = (6, 7, 8) squares, respectively. So, they should be grouped in one
Figures made by five lines = (2, 5, 9) group.
3. (b) By observing the given figures, we found that, figures 8. (b) By observing the given figures, we found that, figures
(1, 5 and 7) are composed of two similar figures, one inside the other. (1, 4 and 6) have two triangles, two circles and two
Figures (2, 4 and 8) contain a figure placed inside a different hexagons, respectively.
figure. Figures (3, 6 and 9) are figures with thick boundaries. Figures (2, 5 and 8) are similar in shape.
4. (a) By observing the given figures, we found that, figures In figures (3, 7 and 9), there are eight small circles around
(1, 4 and 9) are the similar. a big circle.
Figures (2, 6 and 8) are the similar. 9. (b) By observing the given figures, we found that, figures
Figures (3, 5 and 7) are the similar. (1, 2 and 4) are made of three lines.
Figures (3, 5 and 8) are made of four lines.
5. (d) By observing the given figures, we found that, figures of four sides
= (1, 4, 7) Figure (6, 7 and 9) are made of five lines.
Figures of one big and small circle = (2, 6, 9) 10. (b) By observing the given figures, we found that, figures
Figures (3, 5 and 8) are wide from one side and narrow from other. (3, 6 and 7) form a group of figures which are divided into
three unequal parts.
6. (a) By observing the given figures, we found that, figures
Figures (1, 4 and 8) form a group of figures which are
(1, 5 and 7) contain two similar figures, one inside the other. Figures
divided into three equal parts.
(2, 3 and 9) form a group of figures in which half portion of the figure is
enclosed inside that figure. Figures (2, 5 and 9) form a group of figures in which the
figures are divided into three parts in such a way that one
Figures (4, 6 and 8) contain two straight lines perpendicular to each
part is half and the other two parts are one-fourth of the
other dividing the figure into four parts.
original figure.

High Skill Questions


1. (a) By observing the given figures, we found that, grouping the figure 4. (b) By observing the given figures, we found that, figures
on the basis of their use, figures (1, 4, 6) are used for lighting. Figures (1, 4 and 8) are made of three lines. Figures (2, 5 and 7) are
(3 and 5) are used for playing and figures (2,7 and 8) are used for made of four sides. Figures (3, 6 and 9) have curved
learning. portions.
2. (c) By observing the given figures, we found that, figures 5. (a) By observing the given figures, we found that, figures
(1, 3 and 6) contain one complete circle each. Figures (2, 5 and 7) (1, 4, 7); (3, 6, 9) and (2, 5, 8) are similar.
contain a semi-circle each. Figures (4, 8 and 9) contain a triangle each.
6. (b) By observing the given figures, we found that, figures
3. (d) By observing the given figures, we found that, figures (1, 4, 7); (2, 6, 9) and (3, 5, 8) are similar.
(1, 5 and 8) form a group of figure having a circle and a triangle. The
7. (c) By observing the given figures, we found that, figures
triangle is divided into two equal parts. Figures (2, 4, and 7) contain
(1, 4, 7); (2, 3, 6) and (5, 8, 9) are similar.
figures which have curved portions. Figures (3, 6 and 9) form a group
of figures formed by straight lines only. 8. (c) By observing the given figures, we found that, each
figure in class A consists of two similar intersected figures.
Chapter

14
Formation of Figures
Formation of figures refers to the use of pieces of different designs to construct a desired figure. It requires a high
spatial visualisation skill, as different fragmented parts of a figure are combined to form the desired figure.
In this chapter, two sets of figures are provided namely problem figure and answer figures. A candidate is asked to
arranged the different components of the problem figure, so as to form one of the answer figure.
Besides this, there are questions in which one problem figure is given and the candidate is asked to find out the
correct answer figure in which all the pieces which are required to form the question figure are present.
Examples given below, will give you a better idea about the types of questions generally asked in NTSE examination.
Example 1. In the question given below, find out which of the figures can be formed from the pieces given in the
problem figure.
Problem Figure Answer Figures

(a) (b) (c) (d)

Solution (a) Figure given in option (a) can be formed by joining the pieces given in question figure, as shown
below.

Example 2. In the question given below, find out which of the figures can be formed from the pieces given in the
problem figure.
Problem Figure Answer Figures

(a) (b) (c) (d)

Solution (b) Figure given in option (b) can be formed by joining the pieces given in question figure, as shown below.
Formation of Figures 213
Example 3. In the question given below, find the figure given in answer figures, that can form the problem figure.
Problem Figure Answer Figures

(a) (b) (c) (d)

Solution (a) The problem figure can be formed by joining the pieces given in option (a), as shown below.

Example 4. Which of the following answer figure can be combined with the question figure, so as to form a square.
Problem Figure Answer Figures

(a) (b) (c) (d)


Solution (a) Figure given in option (a) will make a square by joining with the given problem figure, as shown
below.

Target Exercise
Elementary Level Questions
Directions (Q. Nos. 1-9) In each of the following question, find the figure given in the answer choices, that can form the problem
figure.
1. Problem Figure Answer Figures 5. Problem Figure Answer Figures

(a) (b) (c) (d) (a) (b) (c) (d)

6.
2.
(a) (b) (c) (d)

(a) (b) (c) (d) 7.


3.
(a) (b) (c) (d)

(a) (b) (c) (d) 8.

4.
(a) (b) (c) (d)

(a) (b) (c) (d) 9.

(a) (b) (c) (d)


214 Study Package for NTSE

10. Which of the following answer figure can be combined 11. Which of the answer figure can be combined with the
with the problem figure, so as to form a square. problem figure, so as to form a circle.
Problem Figure Answer Figures Problem Figure Answer Figures

(a) (b) (c) (d) (a) (b) (c) (d)

High Skill Questions


Directions (Q. Nos. 1-11) In each of the following question, find out which of the figures (a), (b), (c) and (d) can be formed
from the pieces given in problem figure.
1. Problem Figure Answer Figures
9. Problem Figure Answer Figures

(a) (b) (c) (d)


(a) (b) (c) (d)
2.
10.

(a) (b) (c) (d)


(a) (b) (c) (d)
3.
11. Problem Figure

(a) (b) (c) (d)

4.
Answer Figures

(a) (b) (c) (d)

5.
(a) (b) (c) (d)

12. Which of the following answer figure can be combined


(a) (b) (c) (d) with the problem figure, so as to form a square.

6. Problem Figure Answer Figures

(a) (b) (c) (d)

7. (a) (b) (c) (d)


13. Which of the following answer figure can be combined
with the problem figure, so as to form a circle
(a) (b) (c) (d)
Problem Figure Answer Figures

8.
(a) (b) (c) (d)

(a) (b) (c) (d)


Hints and Solutions
Elementary Level Questions
1. (c) The problem figure can be formed by joining the pieces given
in option (c), as shown below.

7. (c) The problem figure can be formed by joining the pieces given
in option (c), as shown below.
2. (c) The problem figure can be formed by joining the pieces given
in option (c), as shown below.

8. (b) The problem figure can be formed by joining the pieces given
in option (b), as shown below.
3. (d) The problem figure can be formed by joining the pieces given
in option (d), as shown below.

9. (d) The problem figure can be formed by joining the pieces given
in option (d), as shown below.
4. (c) The problem figure can be formed by joining the pieces given
in option (c), as shown below.

10. (a) Figure given in option (a) will make a sqaure by joining with
the given problem figure, as shown below.

5. (c) The problem figure can be formed by joining the pieces given
in option (c), as shown below.

11. (b) Figure given in option (b) will make a circle by joining with the
given problem figure, as shown below.

6. (c) The problem figure can be formed by joining the pieces given
in option (c), as shown below.

High Skill Questions


1. (b) Figure given in option (b) can be formed by joining the
pieces given in question figure, as shown below.

4. (b) Figure given in option (b) can be formed by joining the


pieces given in question figure, as shown below.

2. (b) Figure given in option (b) can be formed by joining the


pieces given in question figure, as shown below.

5. (b) Figure given in option (b) can be formed by joining the


pieces given in question figure, as shown below.

3. (a) Figure given in option (a) can be formed by joining the


pieces given in question figure, as shown below.
216 Study Package for NTSE

6. (c)Figure given in option (c) can be formed by joining the pieces 10. (b) Figure given in option (b) can be formed by joining the
given in question figure, as shown below. pieces given in question figure, as shown below.

7. (b) Figure given in option (b) can be formed by joining the 11. (c)Figure given in option (c) can be formed by joining the pieces
pieces given in question figure, as shown below. given in question figure, as shown below.

8. (b) Figure given in option (b) can be formed by joining the


pieces given in question figure, as shown below. 12. (c) Figure given in option (c) will make a square by joining with
the given problem figure, as shown belolw.

9. (d) Figure given in option (d) can be formed by joining the


pieces given in question figure, as shown below.
13. (a) Figure given in option (a) will make a circle by joining with the
given problem figure, as shown below.
Part-I : Physics
Chapter

1
Motion
Scalar and Vector Quantities Displacement When a body moves from one position to
another, the shortest (straight line) distance between
Scalar Quantities Those physical quantities which
the initial position and final position of the body, along
have only magnitude are considered as scalar
with direction is known as its displacement. It is a
quantities, e.g., time, speed, density, volume, work,
vector quantity.
distance etc. are the examples of scalar quantities.
Vector Quantities Those physical quantities which Comparison between Distance and
have magnitude as well as direction. Acceleration, force, Displacement
linear momentum, angular velocity, torque, current, l Distance is always greater than or equal to the
density etc. are examples of vector quantities. displacement i. e., distance ≥ displacement.
l The distance travelled by a moving body cannot be zero
Mechanics but the final displacement of a moving body can be zero.
The branch of physics which deals with the study of l When a body travels along a straight line in a positive
motion of material-objects is called mechanics. direction, its distance and displacement are equal and
have same sign.
Mechanics can be broadly classified into following
branches l Zero displacement means that body after motion has
came back to initial position.
Kinematics î Whenever a body travels along a zig-zag path, the final
It is a branch of mechanics which deals with the study of displacement is obtained by joining the starting point and
finishing point of the body by a straight line.
the motion of the objects without taking into account the
cause of their motion.
Rest and Motion
Statics l If the position of a body changes with time, then the body
It is a branch of mechanics which deals with the study of is said to be in motion but if the position of the body
objects at rest. An object can be at rest, even when a does not change with time then it is said to be in rest.
number of forces acting on it are in equilibrium. l The motion of an object is said to be one-dimensional,
when one of the three coordinates specifying the position
Distance and Displacement of the object changes with time. The motion of a car on
the road, the motion of train along a railway track and
Distance The total length of the path travelled by a an object falling freely under gravity are examples of
body in any given time interval is called distance. This one-dimensional motions.
is a scalar quantity and is never negative. Its SI unit is l The motion of an object is said to be two-dimensional,
metre. when two of the three coordinates specifying the position
220 Study Package for NTSE
Initial velocity + Final velocity
of the object change with time. The motion of a planet Average velocity =
around the sun, a revolving in circle and an insect 2
crawling on a floor are examples of motion in two (ii) Uniform Velocity A body has a uniform velocity if it
travels in a specified direction in a straight line and move
dimensions.
over equal distances in equal intervals of time.
l The motion of an object is said to be three- (iii) Non-uniform Velocity When a body does not cover
dimensional, when all the three coordinates specifying equal distances in equal interval of time, the velocity is
the position of the object change with time. The motion said to be non-uniform velocity.
of gas molecules, the motion of bird in the sky and flying l The magnitude of speed and velocity of a moving body
kite are examples of motion in three dimensions. is equal only if the body moves in a single straight
line.
Speed l The average speed of a moving body can never be zero,
but the average velocity of a moving body can be zero.
Speed of a body is the distance travelled by it per unit
time. It is a scalar quantity. Its SI unit is m/s.
Distance travelled Acceleration
Speed =
Time taken Acceleration of a body is defined as the rate of change of
Speed is measured by speedometer, while distance is its velocity with time. It is a vector quantity. Its SI unit
is m /s 2 .
measured by odometer. Change in velocity
Acceleration =
Average Speed The average speed of a body is the Time taken for change
total distance travelled divided by the total time taken Final velocity − Initial velocity
to cover this distance. =
Total distance travelled Time taken
Average speed =
Total time taken Types of Acceleration
The average speed of some of the moving objects are given
Acceleration can be further divided into two types
below
(i) Uniform Acceleration A body has a uniform
S. No. Moving object Average speed acceleration if it travels in a straight line and its velocity
1. Light (in vacuum) 3 × 108 m/s or 1.08 × 109 km/h increases by equal amounts in equal interval of time.
2. Sound (in air) 332 m/s or 1195.2 km/h (ii) Non-uniform Acceleration A body has a non-uniform
3. Jet aeroplane 500 m/s or 1800 km/h acceleration if its velocity increases by unequal
4. Cheetah 27 m/s or 97.2 km/h amounts in equal intervals of time.
î Retardation A body is said to retarded if its velocity is
Uniform Speed A body has a uniform speed if it decreasing. When the velocity of a body increases with then
travels equal distances in equal intervals of time. its acceleration is positive, if velocity decreases with time
then its acceleration is negative and if a body moves with
uniform velocity, its acceleration is zero.
Velocity
Velocity of a body is the distance travelled by it per unit Uniform Motion and
time in a given direction. It is a vector quantity.
Its SI unit is m/s. Non-Uniform Motion
Distance travelled in a given direction
Velocity = Uniform Motion
Time taken
l A body has a uniform motion, if it travels equal distances
Displacement
= in equal intervals of time.
Time taken
l The distance-time graph for body having uniform
Types of Velocity motion is a straight line.
n
io

Velocity further divided into three types


ot
m
Distance

(i) Average Velocity If the velocity of body is always


r
fo
ni

changing, but changing at a uniform rate (the


U

acceleration is uniform), then the average velocity is


given by the arithmetic mean of the initial velocity and
final velocity for a given period of time, Time
Motion 221
Non-Uniform Motion Graphical Representation
l A body has a non-uniform motion, if it travels unequal
distances in equal intervals of time.
Distance-Time Graphs
l The distance-time graph for body having non-uniform l The distance-time graph of a body moving at uniform
motion is a curved line. speed is always a straight line.
Y A

d
ee
Distance

sp
Distance

rm
rm

fo
fo
ni n

ni
u

U
n- otio
No m
Y
O Time
Time
l The speed of a body is non-uniform, then the graph
between distance travelled and time is a curved line
Equations of Motion (called a parabola).
Let a body starts to move with velocity u, after time t, X A
and after covering a distance s, velocity becomes v and
acceleration of the body is a, then there are three relation

Distance
rm
among these quantities. These relations are called fo
u ni d
n- ee
equations of motion. No sp
(i) First equation of motion v = u + at Y
O Time
(ii) Second equation of motion v2 = u 2 + 2as
1 Speed-Time Graphs (or Velocity-Time Graphs)
(iii) Third equation of motion s = u t + at 2
2
l Speed-time graph for a body moving with constant speed
Distance travelled by the body in nth second is
a is straight line parallel to the time axis.
Sn = u + (2n − 1)
2
Speed

Constant speed
Motion Under Gravity
If a body is thrown upwards or falling downwards, then,
its motion is called motion under gravity.
Time
l If a body is projected vertically downward with some
l The speed-time graph for a uniformly changing speed
initial velocity, then equation of motion
will be a straight line.
(i) v = u + gt
n
tio

1
(ii) s = ut + gt 2
ra
le

2
ce
Speed

ac

(iii) v2 = u 2 + 2 gh
mr
fo
ni

If a body is projected vertically upward, then equations


U

(i) v = u − gt
Time
1
(ii) s = ut − gt 2 l Speed-time graph of a body is a straight line sloping
2
downwards indicates uniform retardation.
(iii) v = u − 2 gh
2 2

g = acceleration due to gravity


Un
where, ifo
v = initial velocity rm
Speed

re
u = final velocity ta
rda
h = height tio
n
t = time
Time
222 Study Package for NTSE

l Speed-time graph of a body when its initial speed is not to be uniform circular motion. In uniform circular
zero. motion velocity is not constant.
(ii) Non-uniform circular motion If a particle is moving

n
tio
ra
on a circular track such that its speed at each point on

le
ce
the circular path is not equal, its motion in
ac
Speed

non-uniform circular motion.


rm
fo
ni
U

Centripetal Acceleration
When a body executes uniform circular motion, then its
Time speed remains same but direction goes on changing i.e.,
l Speed-time graph of the body when speed changes at its velocity is changes, its means that there is an
non-uniform rate is curved line. acceleration in uniform circular motion, this
C
acceleration is called centripetal acceleration.
A v2
a= or a = rw2
ra rm
Speed

r
n
le ifo
tio
ce un

Direction of centripetal acceleration is always towards


ac o n -

l
N

centre.
l Centripetal acceleration is always same in magnitude
B
Time but its direction changes continuously.

l When a body moves with a constant acceleration its


acceleration time graph will as straight line as follows Centripetal Force
Acceleration The force which is needed to make an object travel in a
circular path is called centripetal force.
mv2
a F=
r
t1 t2 t3 F = mrω 2
o Time
Examples of Centripetal Force
Angular Velocity The angle turned by the body
l Centripetal forces acts on the earth moving round the
executing circular motion in 1s is called its angular
sun.
velocity.
∆θ l Electrons moving round the nucleus, require centripetal
Angular velocity ω = force.
∆t
l Cyclist bends his body towards the centre on a turn
If a body is moving on a circular path of radius r with while turning.
angular velocity ω then its linear velocity v = rω.
l Banking of road is to provide the sufficient centripetal
force to the vehicles.
Circular Motion Speed in Uniform Circular Motion The speed of body
When a particle moves along a circle of certain radius r, 2πr
moving along a circular path is given by v = .
its motion is known as circular motion. t
or
When a body moves in a circle, its motion is called Centrifugal Force
circular motion. l The virtual force that balances the centripetal force in
There are two types of circular motion uniform circular motion is called centrifugal force.
(i) Uniform circular motion If the magnitude of linear l Centripetal force is not the real force but it arises due to
speed of a body at each point on the circular path is acceleration of rotating frame.
equal (however its directions are different), it is said Its SI unit N-m.
Target Exercise
Elementary Level
1. A body has an acceleration of −4 ms −2 . What will be 6. A car travelling at 20 km/h speeds up to 60 km/h in
value of its retardation? 6 s. Its acceleration will be
(a) −4 ms −2 (a) 1.85 m / s 2 (b) 1.00 m / s 2
(b) 4 ms −2 (c) 1.58 m / s 2 (d) 2.85 m / s 2
(c) Zero 7. The area under a speed-time graph represents a
(d) Nothing can be decided physical which has the unit of
2. If the time-displacement graph of a particle is parallel (a) m (b) m2 (c) m/s (d) ms −2
to the time-axis, the velocity of the particle is 8. If the displacement of an object is proportional to the
square of time, then the object is moving with
(a) uniform velocity
Displacement

A B
(b) uniform acceleration
(c) increasing acceleration
(d) decreasing acceleration
9. A sprinter is running along the circumference of a big
O
Time sports stadium with constant speed. Which of the
(a) firstly infinity and then comes to zero following do you think is changing in this case?
(b) constant (a) Magnitude of acceleration being produced
(c) equal to acceleration of the body (b) Distance covered by the sprinter per second
(d) zero (c) Direction in which the sprinter is running
3. When body is moving along a circular path then its has (d) Centripetal force acting on the sprinter
(a) a constant speed (b) a constant velocity 10. The speedometer of a car measures
(c) no tangential velocity (d) None of the above (a) average speed (b) instantaneous speed
(c) acceleration (d) None of these
4. Velocity-time graph shown below shows that the body
has 11. Which of the graphs represents the motion with
A
uniform speed?
v
Distance

Distance

(a) (b)

B Time Time
O t
Speed
Speed

(a) a uniform acceleration (c) (d)


(b) a non-uniform retardation
(c) uniform velocity
(d) initial velocity OA and is moving with uniform retardation Time Time
5. In figure as shown below, the velocity
12. A body whose speed is constant
(a) has a constant velocity
Displacement

(b) must be accelerating


A (c) might be accelerating
(d) cannot be accelerating
13. A player completes a circular path of radius r in 40s.
O Time B At the end of 2 min 20 s. The displacement will be
(a) 2r (b) 2 πr (c) 9πr (d) zero
(a) decreases between point O and A
(b) increases between points A and B 14. Which of the following is a scalar quantity?
(c) decreases between points A and B (a) Energy (b) Acceleration
(d) is zero throughout (c) Velocity (d) Force
224 Study Package for NTSE

15. Four car A, B, C and D are moving on levelled straight R


road. Their distance, time graphs are shown in the B
given figure. Which of the following is the correct
S T
statement regarding the motion of these case?

Speed
C
D P Q
A
Distance (m)

B
A
Time

(a) ST (b) QR (c) RS (d) PQ


Time (s) 17. Velocity-time graph as shown below shows that the
body has
(a) Car A is faster than car D
(a) uniform acceleration
(b) Car B is the slowest
(b) uniform retardation
(c) Car D is faster than the car C
(c) uniform velocity throughout its
(d) Car C is the slowest v
motion and has zero initial
16. In the speed-time graph for a moving object shown velocity
here, the part which indicates uniform deceleration of (d) None of the above
the object is? t

High Skill Questions


1. A cyclist moves from a certain point X and goes round 4. A driver is driving his car
Car
a circle of radius r and reaches Y , exactly at the other along a road. The driver
side of the point X. The displacement of the cyclist makes sure that speedometer
would be reads exactly 40 km/h. What A B
happens to the speed of the
car when it travels from A to B as shown in figure?
r (a) Speed remains constant
(b) Speed first increases then decreases
X O Y (c) Speed first decreases then increases
(d) Nothing can be decided
5. In 1.0 s, a particle goes from A to point B moving in a
(a) πr (b) 2 πr (c) 2r (d) 2 π/r semicircle of radius 1.0 m. The magnitude of the
2. A man walks 8 m towards east and then 6 m towards average velocity is
north. His magnitude of displacement which he (a) 3.14 m/s (b) 2.0 m/s
covers is (c) 10 m/s (d) zero
(a) 10 m (b) 14 m (c) 2 m (d) zero 6. The numerical rational of average velocity to average
3. A body is moving, as shown in the graph given below, speed is
represents the motion as (a) always less than 1
(b) always equal to 1
(c) always more than 1
(d) equal to or less than 1
Displacement

7. The displacement-time graphs A and B as shown


below for a body moving in straight path drawn on
the same scales. Then
Displacement
Displacement

O Time
(a) backward with uniform velocity
(b) forward with uniform velocity
(c) backward with non-uniform velocity
(d) forward with non-uniform velocity Time Time
A B
Motion 225
(a) slope of line in A with time axis is greater than the slope of 13. Which of the following is a one-dimensional?
line in B (a) Landing of an aircraft
(b) slope of line in B with time axis is greater than the slope of (b) Earth revolving around the sun
line in A (c) Motion of wheels of a moving trains
(c) slope of line in A with time axis is equal to the slope of line (d) Train tunning on a straight track
in B 14. The correct statement from the following.
(d) nothing can be said about the slopes
(a) A body having zero velocity will not necessarily have zero
8. A body under the action of several forces will have acceleration
zero acceleration. (b) A body having zero velocity will necessarily have zero
(a) When the body is very light acceleraition
(b) When the body is very heavy (c) A body having uniform speed can have only uniform
(c) When the body is a point body acceleration
(d) When the vector sum of all the forces acting on it is zero (d) A body having non-uniform velocity will have zero
acceleration
9. The displacement-time graph shown in figure
represents 15. Match the following columns.
Column I Column II
Displacement

A. One-dimensional 1. Motion of freely falling body


motion
t1
B. Three-dimensional 2. Motion of billiards ball
motion
Time C. Two-dimensional 3. Motion of flying insect
(a) constant velocity motion
(b) velocity of the body is continuously changing D. Uniform circular 4. Artificial satellites moves
(c) instantaneous velocity motion around the earth
(d) the body travels with constant speed upto time t 1 and then
stops Codes
A B C D
10. The second’s hand of a watch is 2 cm long. The speed
(a) 2 1 3 4
of the tip of second’s hand is (b) 1 3 2 4
(a) 0.21 cm/s (c) 4 3 2 1
(b) 2.1 cm/s (d) 3 4 2 1
(c) 21.0 cm/s
(d) None of the above 16. Assertion If the displacement of the body is zero, the
distance covered by it may be zero.
11. The angular velocity of a scooter wheel is 40 rad/s. If
the diameter of the scooter wheel is 80 cm. Calculate Reason Displacement is a vector quantity and
the speed of the scooter in m/s? distance is a scalar quantity.
(a) 18 (b) 9 (a) If both assertion and reason are true and the reason is the
(c) 16 (d) 18 correct explanation of the assertion
(b) If both assertion and reason are true but reason is not the
12. Which of the following velocity-time graph represent correct explanation of the assertion
uniform motion? (c) If assertion is true but reason is false
(d) If assertion is false but reason is true
17. Assertion A body falling freely may do so with
(a) v (b) v
constant speed.
Reason The body falls freely, when acceleration of a
t t body is equal to acceleration due to gravity.
(a) If both assertion and reason are true and the reason is the
correct explanation of the assertion
(b) If both assertion and reason are true but reason is not the
(c) v (d) v correct explanation of the assertion
(c) If assertion is true, but reason is false
(d) If assertion is false, but reason is true
t t
Hints and Solutions
Elementary Level
1. (b) 2. (b) 3. (a) 4. (d) 5. (d) 7. (c) 8. (b)
5 100
6. (a) Initial speed = 20 km/h = 20 × = m/s 9. (c) If an object is travelling with uniform speed along a circular
18 18 path, then its velocity is continuously changing because its
5 300 direction is continuously changing.
Final speed = 60 km/h = 60 × = m/s
18 18 10. (a) 11. (c) 12. (c)
v ⋅u
We have v = u + at , a = 13. (d) Total time of motion is 2 min 20 s = 140 s. Its time period of
t
circular motion is 40 s so in 140 s athlete will complete 3.5
300 100
− revolution i .e., the circle be at diametrically opposite points
Acceleration of car, a = 18 18 = 200
6 18 × 6 i .e., Displacement = 2 r
14. (a) 15. (b) 16. (c) 17. (d)
= 1.85m/s

High Skill Questions


1. (c) 2. (a) 3. (a) 4. (a) 9. (d) Upto time t 1 slope of the graph is constant and after the t 1 slope
5. (b) Displacement covered by the particle is zero i .e., the body travel with constant speed upto time t 1 and
then stops.
10. (a) 11. (c)
12. (a) Slope of velocity time-graph measures acceleration.
For graph a slope is zero.
A B
1.0m Hence a = 0 i .e., motion is uniform.

Diameter of circle = 2r = 2 × 1 = 2 m 13. (d) 14. (a) 15. (b) 16. (a)

Average velocity =
Displacement 2
= = 2 m/s 17. (d) When a body falling freely, only gravitational force acts on it
Time 1 in vertically downward directions. Due to this downward
acceleration the velocity of a body increases and will be
6. (d) 7. (a) 8. (d) maximum when the body touches the ground.
Chapter

2
Force and Laws of Motion
Force Newton’s Laws of Motion
Force is that cause which produces acceleration in the Sir Isaac Newton, who was a great physicist,
body on which it acts. The direction in which a body is astronomer mathematician, propounded the law of
pushed or pulled is called the direction of force. Its unit motion in his book Principia.
is Newton and it is a vector quantity.
There are three laws of motion
Effects of Force
First Law
A force can produce the following effects
A body at rest will remain at rest and a body in motion
l A force can move a stationary body.
l A force can stop a moving body.
will continue in motion in a straight line with a uniform
l A force can change the speed of a moving body. speed, unless it is compelled by an external force to
l A force can change the direction of a moving body. change its state of rest or of uniform motion.
l A force can change the shape and size of a body First law defines inertia and is rightly called the law of
Balanced and Unbalanced Forces inertia.

Balanced Forces Inertia


If the resultant of all the forces acting on a body is zero, The tendency of a body to remain at rest or, if moving to
the forces are called balanced forces. continue moving in a straight line, is called inertia.
Though balanced forces cannot produce motion in a Inertia is of two types
stationary body or stop a moving body, they can, (i) Inertia of rest
however, change the shape of the body. (ii) Inertia of motion
Unbalanced Forces l Mass is a measure of the inertia of a body.
If the resultant of all the forces acting on a body is not l Heavy objects have more inertia than lighter objects as a
zero, the forces are called unbalanced forces. cricket ball has more inertia than a rubber ball of the
Unbalanced forces can move a stationary body or they same size.
can stop a moving body.
Examples of Inertia
î 1. To move a stationary object, we have to push it with a force
greater than the opposing force of friction. l When a car or bus starts suddenly, the passengers fall
2. When we talk of a force acting on a body, it usually means backward.
an unbalanced force. l When a running car or bus stops suddenly, the
passengers are jerked forward.
Torque or Moment of Force l When a hanging carpet is beaten with a stick, the dust
The turning effect of a force on a body is known as the particle start coming out of it.
moment of the force or torque. î 1. Newton’s first law of motion is called Galileo’s law of
Moment of force or torque τ = F. d inertia.
2. Newton’s first law of motion gives us a definition of force.
l Moment or torque is a vector quantity having SI unit N-m.
228 Study Package for NTSE

Second Law Examples of Impulse


The rate of change of linear momentum of a body is l A cricketer moves his hands backward while catching a
proportional to the force applied and it takes place in the ball.
direction of the force l Vehicles like cars, buses and scooters are provided with
Change in momentum
i. e., Force ∝ shockers.
Time taken l Bogies of trains are provided with Buffers.
mv
F=k , F = k ma
t
Q a = v (where, k is a constant)
Friction
 
 t If two bodies are in contact with each other, the property
F = ma, Force = mass × acceleration by virtue of which a force is exerted by them at their
point of contact to prevent one body from slipping over
The SI unit of force is newton (N).
the other, is called the friction and the force thus
exerted is called the force of friction.
Momentum Force of friction always opposes the relative motion
The momentum of a body is defined as the product of its between two surfaces in contact for a body moving on
mass and velocity. Its unit is kg-m/s. another it will always act in a direction opposite to that
Momentum = Mass × Velocity of the motion.
p = mv
Types of Friction
l Momentum is a vector quantity.
l If a body is at rest, its momentum is zero. Types of friction are
(i) Static Friction The opposing force that comes into play
Third Law when one body tends to move over the surface of another
Whenever one body exerts a force on another body, the body but the actual actual motion has yet not started is
second body exerts on equal and opposite force on the called static friction.
first body. Static friction is a self adjusting force and it adjusts itself
so that it becomes equal to the applied force.
Examples of Third Law
(ii) Limiting Friction The maximum static friction force
l During firing of a bullet the gun recoils back with a great
which comes into play, when one body is just at the verge
force.
of moving over the surface of the other body.
l To drive water boat forward the bamboo stick is pressed
into the land of water. Limiting friction ( f s ) = µs
l Jet aeroplanes utilise the principle of action and reaction. R = µmg
(iii) Kinetic Friction The opposing force that comes into
Law of Conservation of Momentum play when one body actually moves over the surface of
When two (or more) bodies act upon one another. Their another body, is called kinetic friction.
total momentum remains constant provided no external (iv) Sliding Friction It comes into play when one body slides
forces are acting. over the surface of the another body.
i. e., Total momentum before collision = Total momentum (v) Rolling Friction It comes into play when one body rolls
after collision over the surface of the another body.
Application of the law of conservation of momentum
(i) Motion of rocket (ii) Motion of jet aeroplane Coefficient of Friction
(iii) Recoil of a Gun (iv) Jumping from boat to the shore
Ratio of force of friction to the normal reaction. It is
represented by µ. Since it is a number and it has no
Impulse unit.
If a very large quantity of force is applied for a very short µ = F/R
interval of time, then the product of force and time is î Rolling friction is lesser than sliding friction. The limiting
called impulse. frictional force is independent of the area of contact but
Impulse = Force × Time depends on the nature of the material of the surfaces in contact
and their roughness or smoothness.
Its unit is kg-m/s.
Force and Laws of Motion 229
Methods of Reducing Friction l Two body sticks together due to friction.
We can reduce friction l Brake works on the basis of friction.
(i) By polishing l The transfer of motion from one part of a machine to other
(ii) By lubrication part through belts is possible by friction.
(iii) By proper selection of material
(iv) By streamlining the shape of the body
Disadvantages of Friction
(v) By using ball bearing l Friction causes wear and tear of the parts of machinery in
contact. Thus their life times reduces.
Advantages of Friction l Frictional force result in the production of heat which
l Walking is possible due to friction. causes damage to the machinery.

Target Exercise
Elementary Level
1. The action and reaction forces involved in Newton’s 7. A metallic ball strikes a wall and falls down whereas a
third law tennis ball having the same mass and velocity bounces
(a) must act on the same objects back. The reason for this is
(b) must act on the different objects (a) both suffer equal change in momentum
(c) may act on different objects (b) the tennis ball suffers a greater change in momentum
(d) None of the above (c) metallic ball suffers a greater change in momentum
(d) the momentum of the tennis ball is less than that of the
2. A long jumper runs before jumping to take long jump metallic ball
because
(a) he covers a greater distance 8. Which one of the following is wrong?
(b) he maintains momentum conservation (a) 1 kg-wt = 9.8 N
(c) he gains energy by running (b) Momentum is a vector quantity
(d) he gains momentum (c) Force is always conserved
(d) Momentum is conserved in the absence of an external force
3. Which one is a measure of inertia of a body?
(a) Momentum
9. A stone is tied to the middle of a string and suspended
(b) Mass from one end as shown in the figure. Here, S is the
(c) Velocity stone and O is the point of suspension. At what point a
(d) None of the above sharp jerk will break the string?

4. When an unbalanced force is applied to a body, then O


what quantity does not change?
(a) Its velocity
S
(b) Its momentum
(c) Its mass
(d) None of these
(a) Below the stone
5. A force acts on an object which is free to move. If we (b) At the point it self
know the magnitude of the force and the mass of the (c) Form above the stone
object, Newton’s second law of motion enables us to (d) Nothing can be decided
determine the object’s 10. A and B are two objects with masses 8 kg and 32 kg
(a) weight respectively. Then,
(b) speed
(a) A has more inertia than B
(c) acceleration
(b) B has more inertia than A
(d) position
(c) A and B have the same inertia
6. If action and reaction were to act on the same body or (d) Neither A nor B has any inerita
object then 11. Friction between any two objects is due to
(a) the resultant force would be zero (a) attraction between them
(b) the body would not move at all (b) repulsion between them
(c) Both a and b are correct (c) some adhesive forces between them
(d) Neither (a) nor (b) is correct (d) irregularities on the surfaces
230 Study Package for NTSE

12. External forces 14. It is difficult to walk on surface because of


(a) are always balanced (a) absence of inertia
(b) never balanced (b) absence of friction
(c) may or may not be balanced (c) more friction
(d) None of the above (d) more inertia

13. Rate of changes of momentum gives 15. When a toothpaste tube is squeezed, its shape
(a) impulse changes, the force responsible for this is an example of
(b) force (a) balanced force (b) centripetal force
(c) momentum (c) unbalanced force (d) centrifugal force
(d) acceleration

High Skill Questions


1. When a bicycle is in motion, (without pendelling) the (a) 0.6 m/s 2 (b) 6 m/s 2 (c) 0.06 m/s 2 (d) 60 m/s 2
force of friction exerted by the ground on the two
wheels act
7. A ball of mass 150 g starts moving with an
acceleration of 20 m/s2 . When hit by a force which
(a) in the backward direction, on the front wheel and in the
forward direction on the rear wheel acts on it for 0.1 s, the impulsive force, is
(b) in the forward direction on the front wheel and in the (a) 0.5 N-s (b) 0.1 N-s (c) 0.3 N-s (d) 1.2 N-s
backward direction on the rear wheel
8. A driver accelerates his car first at the rate of 1.8 m/s2
(c) in the backward direction on both the front and the rear
wheels and then at the rate of 1.2 m/s2 . The ratio of the force
(d) in the forward direction on both the front and the rear wheels exerted at by the engines will be respectively equal to
(a) 2 : 3 (b) 1 : 2 (c) 4 : 2 (d) 3 : 2
2. A cylinder rolls up without slipping on an inclined plane
and, after reaching a certain height, rolls down without 9. A body of mass m collides against a wall with velocity
slipping then, the direction of the frictional forces on the v and rebounds with the same speed.The change in
cylinder acting parallel to the inclined plane are momentum is
(a) upwards at the time of rolling up and downwards at the time (a) zero (b) mv (c) 2 mv (d) − mv
of rolling down 10. You are marooned on a frictionless horizontal surface
(b) upwards at the time of rolling up as well as at rolling down
cannot exert any horizontal force by pushing against
(c) downwards at rolling up and upwards at rolling down
(d) downwards at rolling up as well as at rolling down the surface. How can you get out of it?
(a) By jumping
3. Which of the following is correct when a person walks (b) By spitting or sneezing
on a rough surface? (c) By rolling your body on the surface
(a) The frictional force exerted by the surface keep him moving (d) By running on the plane
(b) The force which the man exerts on the floor keep him moving
11. Certain observation and certain causes are listed in
(c) The reaction of the force which the man exerts on floor keeps
columns I and II respectively in this following.
him moving
(d) None of the above Column I Column II
4. A particle is moving with a constant speed along a A. First law of Motion 1. Change in linear momentum
straight line path. A force is not required to B. Second law of Motion 2. Action and reaction
(a) increase its speed C. Third law of Motion 3. Law of inertia
(b) decrease the momentum D. Impulse 4. F = ma
(c) change the direction
(d) keep it moving with uniform velocity Codes
A B C D A B C D
5. An object will continue moving uniformly untill (a) 2 4 3 1 (b) 4 3 1 2
(a) the resultant force acting on it begins to decrease (c) 2 3 4 1 (d) 4 1 2 3
(b) the resultant force on it is zero
12. The masses of 10 kg and 20 kg respectively are
(c) The resultant force is at right angle to its rotation
connected by a massless spring. A force of 200 N acts on
(d) the resultant force on it is increased continuously
the 20 kg mass. At the instant shown, the 10 kg mass
6. A force of 5 N gives a mass m1 an acceleration of 8 m/s2 has an acceleration of 12 ms −2 , what is the acceleration
and a mass m2 an acceleration of 24 m/s2 . What of 20 kg mass?
acceleration would it give if both the masses are tied
together?
Force and Laws of Motion 231
mass M2 . If the surfaces are frictionless, the force
between the blocks is
10 kg 20 kg
M1
−2 −2
(a) 12 ms (b) 4 ms (c) 20 ms −2 (d) None of these M2

13. A rocket can go vertically upwards in earth’s F


atmosphere because
(a) it is lighter than air M1 F M1 M 2 g
(b) of gravitational pull of the sun (a) (b)
(c) it has a fan which displaces more air per unit time than the M2 M1 + M 2
weight of the rocket M2 F
(c) (d) None
(d) of the force exerted on the rocket by gases ejected by it M1 + M 2 B C
of these E
14. In which of the following cases forces may not be
required to keep the 16. Figure shows the displacement of a A D
(a) particle going in a circle particle going along the x-axis as a
(b) particle going along a straight line function of time. The force acting on the particle is
(c) the momentum of the particle constant zero, in the region
(d) acceleration of the particle constant
(i) A B (ii) BC (iii) C D (iv) DE
15. A single horizontal force F is applied to a block of
mass M1 which is in contact with another block of (a) i, iv (b) iii, ii (c) i, iii (d) iv, iii

Hints and Solutions


Elementary Level
1. (b) 2. (d) 3. (b) 4. (c) 5. (c) 6. (c) 13. (a) 14. (b) 15. (a)
7. (b) 8. (c) 9. (a) 10. (b) 11. (d) 12. (c)

High Skill Questions


1. (c) 2. (b) 3. (c) 4. (d) 5. (b)
12. (b)
5
6. (b) We have, F = m1 a 1 5 = m 1 × 8; m1 =
8 10 kg 2 kg
5
and F = m 2 a 2 ; 5 = m 2 × 24; m 2 =
24 10 kg mass has an acceleration of 12 m/s 2
5 5 15 + 5 20 F = m1a; F = 10 × 12; F = 120 N
Total mass of system = m1 + m 2 = + = = kg
8 24 24 24 Force of 120 N is applied on the mass of 20 kg
Again, F = ma Remaining force = 200 − 120, F′ = 80 N
20 24 × 5 80
5= × a; a = ; a = 6 m/s 2 Again, F ′ = m 2 a ; 80 = 20 × a ; a =
24 20 / 4 20
150
7. (c) Impulsive I = F × ∆ t = ma × ∆ t = × 20 × 0.2 = 0.6 a = 4 m/s 2
1000
F
N-s 15. (c) In this case, acceleration a =
M1 + M 2
8. (d) We have, F = ma
M2 F
∴ F1 = m a × 1.8 and F2 = m 2 × 1.2 The force between the blocks F ′ = M 2 a, F ′ =
M1 + M 2
F1 m × 1.8
∴ = 1 16. (c) In region AB and CD slope of the graph is consult i .e.,
F2 m 2 × 1.2
velocity is constant. It means no force acting on the particle in
F 1.8 F1 3 this region.
Because m1 = m 2 ∴ 1 = , =
F2 1.2 F2 2
Chapter

3
Work, Energy and Power
Work l Work done by a man to carry mass m to a height h above
ground is given by mgh.
Work is said to be done only when a body acted upon by a
Negative Work
force moves actually through some distance in the
direction of the force. In other words, work is the Negative work means that force opposite to displacement
product of force, F and the distance, s travelled in the (i.e., 90° < θ < 180°).
direction of force. It is represented by W. Examples of Negative Work
∴ W = F⋅s l When a person lifts a body from the ground, the work
If there is an angle θ between the direction of force and done by the force of gravity is negative.
direction of displacement, the work done l When a body is made to slide over a rough surface, the
work done by the frictional force is negative.
W = Fs cos θ
F
l When a positive charge is moved to towards another
positive charge. The work done by electrostatic force
between them is negative.
θ Motion
s
Zero Work
The amount of work done depends upon Under three conditions, work done becomes zero.
(i) the force applied
(i) If the force is perpendicular to the displacement
(ii) the displacement of a body
F⊥s
(iii) angle between the direction of force and displacement.
(ii) If there is no displacement
The SI unit of work is joule and work is a scalar quantity. s =0
1 Joule = 10 7 erg (iii) If there is no force acting on the body
For work to be done, the following conditions should be F =0
satisfied Examples of Zero Work
(i) Force must act on the body l When a coolie travels on a horizontal platform with a
(ii) The point of application of the force should move in load on his head, work done against gravity by the coolie
the direction of the force or opposite to it. is zero.
l When a body moves in a circle the work done by the
Nature of Work centripetal force is always zero.
Positive Work l When a person tries to displace a wall or heavy stone by
applying a force and it does not move, then work done
Positive work means that force is parallel to
is zero.
displacement (i.e., 0° ≤ θ ≤ 90°).
l A weightlifter does work in lifting the weight off the
Examples of Positive Work ground but does not work in holding it up.
l A body falls freely under gravity, the work done is positive. l Motion of an isolated body in free space.
l When a lawn roller is pulled by applying a force along l A man standing still at a bus stop with heavy suitcases
the handle at an acute angle, the work done by the force in his hands may get tired soon but he does not no work
is positive. in this situation.
Work, Energy and Power 233
Energy Transformation of Energy
The change of one form of energy into another form of
The energy of a body is defined as its capacity for doing.
energy is known as transformation of energy. For
It is a scalar quantity. The SI unit of energy is Joule.
example
Relation between Different Units l A motor converts electrical energy into mechanical energy.
1 Joule =10 7 erg, 1 eV = 1.6 × 10 −19 Joule l A generator converts mechanical energy into electrical
energy.
1 kWh = 3.6 × 10 Joule, 1 calorie = 4.18 Joule
16
l An electric iron converts electrical energy into heat energy.
l Energy has various forms such as mechanical energy, l An electric bulb converts electrical energy into light
heat energy, light energy, magnetic energy, sound energy.
energy, chemical energy and nuclear energy etc.
l A radio converts electrical energy into sound energy.
Mechanical energy is in two forms
l A steam engine converts heat energy into kinetic energy.
(i) Kinetic energy
l A cell converts chemical energy into electrical energy.
(ii) Potential energy
l A solar water heater converts lights energy into heat
energy.
Kinetic Energy
l A solar cell converts light energy into electrical energy.
The energy of a body due to its motion is called kinetic
energy. If a body of mass m is moving with velocity v, the l Electric motor is used in electric fans, washing machine,
kinetic energy of the body is given by refrigerators, mixer and grinder, hair dryer etc.
1
K = mv2 = F ⋅ s Law of Conservation of Energy
2
l Since the kinetic energy of a body depends on its mass l Energy can neither be created nor can be destroyed, but
and velocity, therefore, heavy bodies moving with high can only be converted from one form to another, the total
velocities have more kinetic energy than slow moving amount of energy of the universe remaining constant.
bodies of small mass. l The mechanical energy of a freely-falling body remains
l If the mass of body is doubled, its kinetic energy also gets constant at all positions. As the body falls, its potential
doubled and if the velocity of a body is doubled, its energy decreases, but at the same time, kinetic energy
kinetic energy becomes four times. increases by an equal amount.

Relation between Kinetic Energy and Momentum Example of Conservation of Energy


2 2
1 1 m v Simple Pendulum A simple pendulum consists of a
Kinetic energy K = mv2 =
2 2 m small metal ball (called bob) suspended by a long thread
p2 from a rigid support, such that the bob is free to swing
K = back and forth when displaced.
2m
where, p is the momentum. A swinging simple pendulum is an example of
conservation of energy.
Therefore, a body cannot have KE without having linear
momentum and vice-versa.

Potential Energy C (PE = maximum)


B KE = 0
The energy of a body due to its position or change in PE = maximum
KE = 0
A
shape is known as potential energy. If a body of mass KE = maximum, PE = 0
m is raised to a height h from the Earth’s surface, the
(i) When the pendulum bob is at position B, it has only
potential energy of the body U = mgh
potential energy.
(ii) As the bob starts moving down from position B to
Mass-Energy-Equivalence position A, its potential energy goes on decreasing
Einstein for the first time established that mass and but its kinetic energy goes on increasing.
energy are two different forms of the same entity. (iii) When the bob reaches the centre position A, it has
i.e., E = mc2 only kinetic energy.
where, E is the energy obtained when a mass is (iv) As the bob goes from position A toward position C, its
completely converted into energy and c is the velocity of kinetic energy goes on decreasing but its potential
light in free space (= 3 × 10 8 m /s ). energy goes on increasing. So at position C, the bob
has only potential energy.
234 Study Package for NTSE

Power
Power is measured as the rate of doing work. It is a scalar quantity having SI unit J/s or watt(W).
Work done ( W)
Power (P) =
Time taken ( t)
The power of object is the product of the force applied to a body and the velocity produced in the body P = F ⋅ v.

Other Units of Power


1 Kilowatt = 1000 W
1 Megawatt = 10 6 W
1 Horse power = 746 W

Target Exercise
Elementary Level
1. A body moves a distance of 6 m in the following 5. A 6 kg weight is attached to a spring. It is pulled down
different ways. In which case is the maximum work and then released. It oscillates up and down. Its KE
done? will be
(a) When pushed over an inclined plane (a) maximum in the middle of the movement
(b) When lifted vertically upward (b) maximum at the bottom
(c) When pushed over smooth rollers (c) maximum just before it is released
(d) When pushed on a plane horizontal surface (d) constant
2. Ashok lifts a doll from the floor and places it on a 6. A photocell converts light energy into
table. If the weight of the doll is known, what else (a) chemical energy (b) electrical energy
need not be known in order to calculate the work, (c) heat energy (d) mechanical energy
Ashok has done on the doll?
(a) The time taken
7. A body is dropped from a certain height from the
(b) Height of the table ground. When, it is halfway down, it possesses
(c) Mass of the ball (a) KE
(d) Cost of the doll or the table (b) Both KE and PE
(c) PE
3. In which of the following cases will the work done be (d) Zero energy
maximum? The body is moved through a distance S
on the ground? 8. An wound watch spring has the energy of type
(a) mechanical
F (b) kinetic
F
(c) potential
(a) (b) (d) kinetic and potential
30° 45°
9. A car is moving along a straight levelled road having
F constant speed. Then,
(a) the work done on the car is infinite
(c) (d) (b) the work done on the car is zero
60° F (c) the work done on the car is a measure of the gravitational
potential energy
(d) the work done on the car cannot be found
4. Two springs A and B are identical but A is harder
than B. On which spring’s more work will be done, if 10. Which of the following does not possess the ability to
they are stretched by the same force? do work not because of motion?
(a) In spring A (a) A sparrow flying in the sky
(b) In spring B (b) A sparrow moving slowly on the ground
(c) In spring A as well as in spring B (c) A sparrow in the nest on a tree
(d) Work done is zero for both springs (d) A squirrel going up a tree
Work, Energy and Power 235
11. A stone is thrown upwards as shown in the diagram. 13. How much work should be done on a bicycle of mass
When it reaches P, which of the following has the 20 kg to increase its speed from 2 m/s to 5 m/s?
greatest value for the stone?
(a) 210 J (b) 100 J
P
(c) 2100 J (d) 21 J

14. A bullet of mass 0.1 kg is fired with a speed of 100 m/s,


the mass of gun is 50 kg. The velocity of recoil is
(a) 0.2 m/s (b) 0.1 m/s
(c) 0.5 m/s (d) 0.05 m/s
(a) Its acceleration (b) Its kinetic energy
(c) Its potential energy (d) Its weight 15. A bird flying in the sky has
(a) KE only
12. A 12 HP motor has to be operated 8 hour/day. How (b) PE only
much will it cost at the rate of 50 paisa/kWh in 10 days? (c) Neither KE nor PE
(a) ` 350 (b) ` 358 (c) ` 375 (d) ` 397 (d) Both KE and PE

High Skill Questions


1. The kinetic energy possessed by a body of mass m
1
moving with a velocity v is equal to mv2 , provided p p
2 (a) (b)
(a) the body moves with velocities comparable to that of light
(b) the body moves with velocities compared to the speed of light
(c) the body moves with velocities greater than that of light
O K O K
(d) None of the above statement is correct

2. You lift a heavy book from the floor of the room and p p
keep it in the book-shelf having a height 2 m. In this (c) (d)
process, you take 5 s. The work done by you will
depends upon
(a) mass of the book and time taken
O K O K
(b) weight of the book and height of the book shelf
(c) height of the book-shelf and time taken 8. Which of the following graph shows the graphical
(d) mass of the book, height of the book-shelf and time taken relation between momentum p and kinetic energy K
for a body in motion?
3. A lorry and a car moving with the same KE are brought
to rest by applying the same retarding force, then 9. A body of mass 3 kg is acted on by a force which varies
(a) lorry will come to rest in a shorter distance as shown in the graph below. The momentum
(b) car will come to rest in a shorter distance acquired is given by
(c) both come to rest in a same distance F (N)
(d) None of the above

4. A man starts walking from a point on the surface of 10


earth (assumed smooth) and reaches diagonally
opposite point. What is the work done by him?
(a) Zero (b) Positive
2 4 6 t (s)
(c) Negativ (d) Nothing can be said
(a) Zero (b) 5 N-s (c) 30 N-s (d) 50 N-s
5. A total of 490 J energy was used in lifting a 5 kg mass. v
The mass was raised to
B C
(a) 10 m (b) 9.8 m (c) 960 m (d) 1000 m

6. When the momentum of a body decreased by 10%, its E


D
KE decreases by
(a) 20% (b) 40% (c) 36% (d) None of these
A t
7. A force 20 N displaces an object 10 cm and does work
of 1 J in the process. Find the angle between the force 10. The diagram shown the velocity verus time plot for a
and the displacement particle, the work done by the force on the particle is
(a) 60° (b) 30° positive from
(c) 120° (d) None of these (a) A to B (b) B to C (c) C to D (d) D to E
Hints and Solutions
Elementary Level
25 − 4
1. (b) 2. (b) 3. (b) 4. (c) 5. (c) as =
2
6. (b) 7. (b) 8. (c) 9. (b) 10. (c) 21 2 2
as = m /s
11. (a) 2
12. (b) Power P = 12 HP = 12 × 746 W Work done on a bicycle
Watt × Hours × Days 21
Unit = W = F ⋅ s = ma ⋅ s = 20 ×
1000 2
12 × 746 × 8 × 20 W = 210 J
= = 716.16 kWh
1000 14. (a) From conservation of linear momentum,
Cost = 716.16 × 0.50 = ` 358 mv1 = mv 2
13. (a) We have, M v1 = 0.1 × 100
v 2 − u 2 = 2 as 5 v1 = 0.2 × 200
0.1 × 100
∴ 2 as = v 2 − a2 v1 =
5
2 as = 52 − 2 2 v1 = 0.2 m/s
2 as = 25 − 4 15. (d)

High Skill Questions


1. (b) 81p2
p2
2. (b) Work done = Force × Displacement Kinetic energy decreases K % = 100 / × 100
2m 2m
= Weight of the books × Height of the book shelf
= 81%
3. (c) 4. (a)
7. (a)
5. (a) The potential energy = 490 J, mgh = 490
8. (d)
490 490
h= = = 9.8 m 9. (d) Momentum = Area of F-t graph
m × g 5 × 10
1
6. (d) Suppose the momentum of a body is P when the momentum m= × 2 × 10 + 4 × 10
2
of a body decrease by 10%, then
p × 20 9p = 10 + 40
p′ = p − p′ =
100 10 = 50 N-s
We have, 10. (a) From the graph, it is clear that force is acting on the particle
2 in the region ABand due to this force kinetic energy (velocity) of
 9p  81p2
  the particle increases. So, the work done by the force is
p′ 2
 2 m
Kinetic energy K = , K= , K = 100 positive.
2m 2m 2m
Chapter

4
Gravitation
Gravity l The value of g is maximum on the surface of the earth, it
decreases on going above the surface of earth or on going
The earth attracts (or pulls) all the objects towards its inside the surface of the earth.
centre. The force with which the earth pulls the objects
l At the centre of earth, the value of g is zero.
towards it is called the gravitational force of the earth or
gravity.
l The value of g on the moon is about one-sixth of the value
of g on the earth.
Universal Law of Gravitation
The gravitational force acting between two points Mass
objects is directly proportional to the product of their The mass of a body is the quantity of matter contained
masses and inversely proportional to the square of the it. Mass is a scalar quantity. Its unit is kg.
distance between them. l The mass of a body is constant and does not change from
G m1 m2
Gravitational force ( F) = place to place.
r2 l The mass of a body cannot be zero.
where, G is universal gravitational constant. Its value is
6.67 × 10 −11 N-m 2 kg −2 and remain same throughout
the universe. Weight
l The earth exerts a gravitational force of 9.8 N on a ball of The weight of a body is the force with which it is
mass 1 kg. attracted towards the centre of the earth.
Weight ( w) = mg
Acceleration due to Gravity (g)
SI unit of weight is Newton.
The uniform acceleration produced in a freely falling
l The weight of a body also changes from place to place.
body due to the gravitational force of the earth is known
as acceleration due to gravity. Its value is 9.8 m/s 2 . l The weight of body can be zero.
GM l The weight at the centre of earth is zero (because the
Relation between g and G is given by g = 2 where, M value of g is zero) at the centre of earth.
R
is mass of the earth, and R is the radius of the earth.
l Weight of any object on moon will be about one-sixth of
what it is on the earth.
î The acceleration produced in the freely falling bodies is the
same for all the bodies. Example If the weight of a body on the earth is 6 N, what
will it be on the moon?
Variation of Acceleration due to Gravity (g) Solution Weight of a body on the earth = 6 N
l The radius of the earth at the poles is minimum, the 1
Weight of a body on the moon = × Weight of a body on the
values of g is maximum at the poles. 6
The radius of the earth is maximum at the equator so, the 6
l
earth = =1N
value of g is minimum at the equator. 6
238 Study Package for NTSE

Kepler’s Laws of Planetary Motion


1. The planets move in elliptical orbits around the sun, with the sun at one of the two foci of the elliptical orbit.
2. Each planet revolves around the sun in such a way that the line joining the planet to the sun sweeps over
equal areas in equal intervals of time. It means that a planet moves faster when it is closer to the sun and
moves slowly when it is farther from the sun.
3. The cube of the mean distance of a planet from the sun is directly proportional to the square of time it takes
to move around the sun i.e.,
T2 ∝ r3
T2
or = constant
r3

Target Exercise
Elementary level
1. The mass of moon is about 0.012 times that of earth 6. Force of gravity is least at
and its diameter is about 0.25 times that of earth. The (a) the equator
value of G on the moon will be (b) the poles
(a) less than that on the earth (c) a point in between equator and any pole
(b) more than that on the earth (d) None of the above
(c) same as that on the earth 7. The weight of a body at the centre of the earth is
(d) about one-sixth of that on the earth
(a) zero
2. Two particles are placed at same distance from each (b) infinite
other. If keeping the distance between them unchanged, (c) same as on the surface of earth.
(d) None of the above
the mass of each of the two particles is doubled, the value
of gravitation force between them will become 8. The gravitational force between two point masses m1
1 1 m1m2
(a) times (b) times and m2 at separation r is given by F = k .
4 2 r2
(c) 4 times (d) 2 times The constant k
(a) depends on system of unit only
3. An apple of mass 100 g falls from a tree because of (b) depends on medium between masses only
gravitational attraction between the earth and the (c) depends on both a and b
apple. If the magnitude of force exerted by the earth (d) is independent of both a and b
on the apple be F1 and the magnitude of force exerted
by the apple on the earth be F2 , then 9. The correct answers to above question is based on
(a) F1 is very much greater than F2 (a) acceleration due to gravity in vacuum is same irrespective
(b) F2 is very much greater than F1 of size and mass of the body
(c) F1 is only a little greater than F2 (b) acceleration due to gravity in vaccum depends on the
(d) F1 and F2 are exactly equal mass of the body
(c) there is no acceleration due to gravity in vacuum
4. The force of attraction between two unit point masses (d) in vacuum, there is resistance offered to the motion of the
separated by a unit distance is called body and this resistance depends on the mass of the
(a) gravitational potential body
(b) acceleration due to gravity
(c) gravitational field strength 10. When a body is taken from the equator to the poles,
(d) universal gravitation constant its weight
(a) remains constant
5. As we go from the equator to the poles, the value of g (b) increases
(a) remains the same (b) decreases (c) decreases
(c) increases (d) decreases upto a latitude of 45° (d) increases at N pole and decreases at S pole
Gravitation 239

High Skill Questions


m
1. Spot the wrong statement 6. In the relation, F = G × M × , the quantity G
The acceleration due to gravity g decreases if d2
(a) depends on the value of g at the place of observation
(a) we go down from the surface of the earth towards its centre
(b) is used only when the earth is one of the two masses
(b) we go up from the surface of the earth
(c) is the greatest on the surface of the earth
(c) we go from the equator towards the poles on the surface of
(d) is of the same value irrespective of the place of
the earth
observation.
(d) the rotational velocity of the earth is increased of the earth
2. Which of the following statement is wrong? 7. A The ratio of inertial mass of gravitational mass is
equal to one.
(a) g is less at the earth’s surface than at a height above it or
a depth below it. B The inertial mass and gravitational mass of a body
(b) g is same at all places on the surface of the earth. are equivalent
(c) g has its maximum value at the equator (a) Both A and B are true (b) Both A and B are false
(d) g is greater at the poles than at the equator (c) A is true but B is false (d) A is false but B is true
3. At the surface of a certain planet, acceleration due to 8. A A force act upon the earth revolving in a circular
gravity is one quarter of that on earth. If a brass is orbit about the sun. Hence, work should be done on
transported to this planet, then which one of the the Earth.
following statement is not correct?
B The necessary centripetal force for circular motion
(a) The mass of the brass ball on this planet is a quarter of its
mass as measured on earth. of earth comes from the gravitational force between
(b) The weight of the brass ball on this planet is quarter of the earth the sun.
weight as measured on earth. (a) Both A and B are true
(c) The brass ball has the same mass on the other planet as (b) Both A and B are false
on earth. (c) A is true but B is false
(d) The brass ball has the same value on the other planet as (d) A is false but B is true
on earth. 9. Match the columns
4. Weight of a body is maximum at
(a) moon (b) poles of Earth Column I Column II
(c) equator of Earth (d) centre of Earth 1
A. Universal law of gravitation 1. w= mg
5. Which of the following graph represents the motion of 6
a planet moving about the sun? B. Kepler’s law of planetary motion 2. T 2 ∝ r 3
m1m2
C. Acceleration due to gravity 3. F =G
(a)
(b) r2
T2 T2 GM
D. Weight on the moon 4. g=
R2
3 3
R R
Codes
A B C D
T2 (d) T 2 (a) 4 2 1 3
(c) (b) 2 3 1 4
(c) 4 1 2 3
R3 R3 (d) 3 2 4 1
Hints and Solutions
Elementary Level
1. (c) 4. (d)
m1 m 2 5. (c) As we go from the equator to the poles, the value of g
2. (c) Force F = G
r2 increases.
The mass of each particle is doubled, then 6. (a) Since g is minimum at the equator. So the force of gravity is
least at the equator.
(2 m1 )(2 m 2 ) m1m 2
F′ = G = 4G 7. (a)
r2 r2
F′ = 4 F 8. (a)
3. (d) From Newton’s Third Law that every action have equal 9. (a)
and opposite reaction. Action and reaction always act on the 10. (b)
different bodies.

High Skill Questions


1. (c) 6. (d)
2. (a,b,c) 7. (a) Inertial mass and gravitational mass are equivalent. Both are
scalar quantities and measured in the same unit. They are quite
3. (a)
different in the method of their measurement. Also gravitational
4. (b) weight w = mg mass of a body is affected by the presence of other bodies near
since g is maximum at the poles. So the weight of the body will it whereas internal mass remain unaffected.
be at the poles. 8. (a) Earth revolves around the sun in circular path and required
5. (c) According to Kepler’s law centripetal force is provided by gravitational force between Earth
T ∝R
2 3
or T ∝R and sun but the work done by this centripetal force is zero.

The graph will be straight line. 9. (d)


Chapter

5
Pressure and
Archimedes’ Principle
Pressure Density
Magnitude of normal force per unit area is known as The density of a substance is defined as mass of the
pressure. It is a scalar quantity. Its unit is Nm −2 . substance per unit volume
Mass M
Thrust Density = or ρ =
Pressure = Volume V
Area
Its SI unit is kg / m 3 .
Force
=
Area Relative Density
The relative density of a substance is the ratio of its
î The force acting on a body perpendicular to its surface is density to that of water.
called thrust. Its unit is Newton.
Density of the substance
î The same force produces less pressure if it acts on a large area Relative density =
but it can produce high pressure if it acts on small area. Density of water

Densities of Some Common Substances


Pressure in Fluids
Substance Density
At the depth h below the surface of a liquid of density ρ,
pressure p is given by 1. Wood 800 kg/m3
2. Ice 920 kg/m3
p = ρgh 3. Glycerine 1260 kg/m3
where, g is an acceleration due to gravity. 4. Iron 7800 kg/m3
5. Mercury 13600 kg/m3
Atmospheric Pressure 6. Gold 19300 kg/m3

The pressure exerted by the atmosphere is called


atmospheric pressure. The value of atmospheric Buoyancy
pressure on earth at sea level is nearly 1.013 × 105 Nm −2
The upward force exerted by a fluid on the immersed
or pascal.
body is called buoyant force or upthrust.
l Aneroid barometer is used to measure atmospheric
l Every liquid exerts an upward force on the object
pressure and height of a place.
immersed in it.
l The pressure exerted by a liquid increases with depth l The maximum upward buoyant force acts on an object
and acts in all directions. when it is completely immersed in the liquid.
242 Study Package for NTSE

l As the volume of solid object immersed inside the liquid 2. When weight of solid is less than force of buoyancy
increases, the upward buoyant force also increases (upthrust) of liquie, then solid will float.
l The magnitude of buoyant force acting on a solid object
does not depend on the nature of the solid object. Floatation of Bodies
When a body of density and volume V is immersed in a
Archimedes’ Principle liquid of density, the forces acting on the body are
When a solid body i.s immersed wholly or partially in a Weight of body w = mg = Vρg
liquid, then there is some apparent loss in its weight. This (acting vertically downward)
loss in weight is equal to the weight of the liquid displaced Upthrust force = Vρg (acting vertically upward)
by the body. The observed weight of a body immersed in a
There are three possible cases
liquid is less than its true weight by an amount equal to
(i) An object will float in a liquid if its density is less than of
the weight of the liquid displaced by the body.
the liquid (i. e., weight is less than upthrust)
Applications of Archimedes’ Principle
l Archimedes’ principle is used in determining the
relative density of a substance.
l The hydrometer used for determining the density of
liquids are based on Archimedes’ principle.
(ii) An object will also in a liquid if its density is equal to
l The lactometer used for determining the variety of that of the liquid (i. e., weight is equal to upthrust)
milk are based on Archimedes’ principle.
l Archimedes’ principle is used in designing ship and
submarines.

Laws of Floatation
(iii) An object will sink in a liquid if its density is more
Whenever a solid body dipped into a fluid, the fluid
than that of the liquid (i. e., weight is more than
exerts force of buoyancy on it, if the force of buoyancy upthrust)
equals to the weight of the solid i.e., F = W , the solid will
remain in equilibrium. This is called floatation.
1. When weight of solid is greater than the force of
buoyancy (upthrust) of liquid, in this condition solid
will be sink. i.e., F < W .

Target Exercise
Elementary Level
1. An object is put in three liquids having different 3. A solid of density 900 kg / m 3 floats in oil as shown in
1 2 the given diagram. The oil floats on water of density
densities one by one. The object floats with , , and
9 11 1000 kg / m 3 as shown. The density of oil in kg/m 3
3
parts of its volume outside the surface of liquids of could be
7
densities d1, d2 and d3 respectively. Which of the
following is the correct order of the densities of the Oil
three liquids?
(a) d1 > d 2 > d 3 (b) d 2 > d 3 > d1
(c) d1 < d 2 < d 3 (d) d 3 > d 2 > d1

2. Four balls A, B, C and D displace 10 ml, 24 ml, 15 ml


Oil Water
and 12 ml of a liquid respectively, when immersed
completely. The ball which will undergo the (a) 850
maximum apparent loss in weight will be (b) 900
(a) A (b) B (c) C (d) D (c) 980
(d) 1050
Pressure and Archimedes’ Principle 243
4. The diagram represent four measuring cylinders 10. The buoyant force depends on the
containing liquids. The mass and volume of the liquid (a) depth of a liquid (b) density of a liquid
in each cylinder are stated. Which two measuring (c) colour of a liquid (d) None of these
cylinders could contain an identical liquid?
11. The buoyant force will be exerted on the body when
(a) the body is immersed partially and displaces same amount
of liquid
80 g 100 g 100 g 80 g (b) the body is not immersed in liquid
(c) the body is immersed fully in liquid
(d) the body is immersed partially
100 cm3 100 cm3 80 cm3 80 cm3
12. The pressure in the bottle filled with water, is
maximum
W X Y Z (a) in the middle of bottle
(b) at the top of the bottle
(a) W and X (b) W and Y (c) first at the top then in the middle
(c) X and Y (d) X and Z (d) in the bottom of bottle
5. Which of the following physical quantity has no unit? 13. From the adjacent figure, the correct observation is
(a) Relative density (b) Mass
(c) Pressure (d) None of these
Water Water
6. An ice cube is floating in glass of water. What
happens to the water level when the ice melts
completely? A B
(a) Rises (b) Falls (a) the pressure on the bottom of tank A is greater than that the
(c) Remains same (d) First rises and then falls bottom of B
7. A wooden piece is completely immersed in water. The (b) the pressure on the bottom of the tank A is smaller than that
the bottom of B
upthrust of water on wooden piece in comparison to
(c) the pressure depend on the shape of the container
the weight of the piece will be (d) the pressure on the bottom of A and B is the same
(a) more (b) equal (c) less
(d) Zero 14. A sample of metal weighs 210 g in air, 180 g in water
and 120 g in liquid, then relative density of
8. A piece of wood floats in water. What happens to the
(a) metal is 3 (b) metal is 7
piece of wood when it floats in alcohol? 1
(a) Floats higher (b) Stays as before sinks (c) liquid is 3 (d) liquid is
5
(c) Sinks (d) Sinks and rises
15. An ice-berg of density 900 kg / m 3 is floating in water
9. An object of wood has a weight w and volume V. The
of density 1000 kg / m 3. The percentage of volume of
apparent weight of object after the object is made to
ice-cube outside the water is
float on water, will be
(a) 20% (b) 35% (c) 10% (d) 25%
w
(a) (b) w − V (c) Zero (d) w
V

High Skill Questions


1. The piece of wood of 28 cm −3 volume and the piece of
glass of 1 cm 3 volume are tied to each other. The
density of wood is 0.6 g / cm 3 and the density of
glass 11.3 g / cm 3. According to the calculation, the
both pieces will
(a) swim in water (b) sink in the water
(c) sink the middle (d) None of these

2. A body floats in a liquid contained in a beaker. If the


whole system as shown in figure falls freely under 3. An aircraft is moving with a velocity of 300 ms −1. If
gravity, then the upthrust on the body due to liquid is all forces acting on it are balanced, then
(a) zero (a) it still moves with the same velocity
(b) equal to the weight of liquid displaced (b) it will be just floating at the same point in space
(c) equal to the weight of the body in air (c) it will fall down instantaneans
(d) None of the above (d) it will lose its velocity gradually
244 Study Package for NTSE

4. A cold soft drink is kept on the balance. When the cap 8. A beaker containing a liquid is kept inside a big
is open, then the weight closed jar. If the air inside the jar is continuously
(a) increases pumped out, the pressure in the liquid near the
(b) decreases bottom of the liquid will
(c) first increases then decreases
(a) increases
(d) remain same
(b) decreases
5. A box of mass 10 kg has as base area of 1 m 2. Find the (c) remain constant
pressure exerted by it on the ground. (d) first decreases and then increases
(a) 100 Pa (b) 50 Pa (c) 150 Pa (d) 200 Pa 9. The relative densities of four liquids P, Q, R and S are
6. A metallic block of density 5 g / cm and having 3 1.26, 1.0, 0.84 and 13.6 respectively. An object is
dimensions 5 cm × 5 cm × 5 cm is weighed in water. floated in all these liquids, one by one. In which
Its apparent weight will be liquid, the object will float with its maximum volume
(a) 5 × 5 × 5 × 5 × 5 g (b) 4 × 4 × 4 × 4 g submerged under the liquid?
(c) 5 × 4 × 4 × 4 g (d) 4 × 5 × 5 × 5 g (a) P (b) Q (c) R (d) S

7. A closed rectangular tank is completely filled with 10. A A piece of ice floats in water, the level of water
water and is accelerated horizontally with an remains unchanged when the ice melts completely.
acceleration in towards right. B According to Archimedes’ principle the loss in
Pressure is (i) Maximum at and (ii) Minimum at weight of the body in the liquid is equal to the weight
(a) Maximum at B, minimum at D of the liquid displaced by the immersed part of the
(b) Maximum at C, minimum at D body.
A D
(c) Maximum at B, minimum at C (a) Both A and B are true (b) Both A and B are false
(d) Maximum at B, minimum at A C (c) A is true but B is false (d) A is false but B is ture
B

Hints and Solutions


Elementary Level
1. (c) 2. (b) 3. (c) 4. (d) 5. (a) 6. (c) By solving these Eqs. (i), (ii) and (iii), we get
7. (a) 8. (c) 9. (c) 10. (b) 11. (c) 12. (a) ρ = 2 and σ = 3

13. (d) Pressure = hρg , i .e., pressure at the bottom is independent 15. (c) Let the total volume of ice-berg floats is V and its density is ρ. If
at the bottom is independent of the area of the bottom of the this ice-barg floats in water with volume Vin inside it,
 ρ
tank. It depends on the height of water upto which the tank is Then Vin σg = Vρg Vin =   g
filled with water. As in both the tanks, the levels of the water are σ
the same, pressure at the bottom is also the same.  σ − ρ
or Vout = V − Vin Vout =  g
14. (c) Density of metal = ρ, Density of liquid = σ  σ 
Vout  σ − ρ  1000 − 900
If V is the valume of sample, then according to problem = =
Vin  ρ  1000
210 = Vρg ...(i)
1
180 = V (ρ − 1) g ...(ii) = Vout = 10%of V
10
120 = V (ρ − σ ) g ...(iii)

High Skill Questions


1. (a) 7. (a ) Due to acceleration towards right there will be a pseudo
2. (a) Upthrust = Vρ liquid = g − a force in left direction so the pressure will be more on rear side
(points A and B) in comparison with front side (points D
where, a = downward acceleration and C)
V = volume of liquid displaced
Also due to height of liquid column pressure will be more at the
But for free fall a = 2g, ∴ Upthrust = 0
bottom (points Band C) in comparison with top (points A and D)
3. (a) 4. (c) 5. (a) so overall maximum pressure will be at point B and minimum
6. (d) Apparent weight = V (ρ − σ )g , = l × b × h × (5 − 1)g pressure will be at point D.
= 5 × 5 × 5 × 4 × g = 4 × 5 × 5 × 5g 8. (b) 9. (c) 10. (a)
Chapter

6
Sound
Wave Motion Longitudinal Waves
Sound is a wave motion, produce by a vibrating source. A In this, the particles of the medium vibrate back and
medium is necessary for the propagation of sound waves. forth along the directions of the propagation of wave.
A wave is a vibratory disturbance in a medium which Longitudinal waves can propagate freely through solids,
carries energy from one point to another without being liquids and gas. These waves propagates in the form of
in direct contact with the two points. alternative compressions and rarefaction.

Mechanical or Elastic Waves


The waves which can be produced in a material medium
are called elastic waves or mechanical waves. In
other words, mechanical waves require material Compression Rarefaction
medium for their propagation.
On the basis of the vibrations of the particles of the For example,
medium, the waves are classified into two types l The waves which travel along a spring (or slinky) when it
is pushed and pulled at one end, are longitudinal waves.
(i) Transverse waves (ii) Longitudinal waves
l Sound wave in air are longitudinal waves.
Transverse Waves
In this, the particles of the medium vibrate Graphical Representation
perpendicular to the direction of the propagation of
waves. These waves can propagate through solids and Density-Distance Graph
on the surface of liquids but not through gas. Transverse A longitudinal wave is represented by a density-distance
waves propagates in the form of crests and troughs as graph
shown in the figure.
Density greater

Crest
than normal

Compression Compression
A C
Distance
Trough Normal
density
l The waves produced by moving one end of a long spring
Density less

B
than normal

(or slinky) up and down rapidly, whose other end is D


fixed, are transvere waves. Rarefaction Rarefaction
l The water waves (or ripples) formed on the surface of
water in a pond are also transverse waves.
246 Study Package for NTSE

Displacement-Distance Graph Sound Waves


A transverse wave is represented by a displacement- Sound wave is a longitudinal mechanical wave which is
distance graph. generated or produced by the transmission of the
displacement

distrubances in the form of compressions and


rarefactions.
Positive

Crest Crest
A C
Audible Waves
Line of zero
disturbance Sound waves having frequency between 20 Hz to 20,000
Hz are audible to human ear. This frequency range is
displacement

B D
called audible frequency range.
Negative

Trough Trough
Infrasonic Waves
Sound waves of frequency less than 20 Hz are called
Terms Related to Wave Motion infrasonic waves.
Wavelength (λ)
Ultrasonic Waves or Ultrasound
The minimum distance in which a wave repeats ifself is
Sound waves of frequency more than 20000 Hz are
called its wavelength. The unit for measuring
known as ultrasonic waves. Ultrasound wave is
wavelength is metre (m).
produced by dolphins, bats and porpoises. Rats also play
Amplitude (A) games by producing ultrasound.
The maximum displacement of the particles of the Applications of Ultrasound
medium from their original undistrubed positions, l Ultrasounds are used extensively in industries and for
when a wave passes through the medium, is called medical purposes.
amplitude of the wave. l Ultrasound is generally used to clean parts located in
hard to reach places. For example, spiral tube, odd
Time Period (T) shaped parts, electronic components etc.
The time required to produce one complete wave (or l Ultrasound can be used to detect cracks and flaws in
cycle) is called time period of the wave. metal blocks.
l Ultrasonic waves are made to reflect from various parts
Frequency of the heart and form the image of the heart. This
The number of complete wave (or cycle) produced in one technique is called echocardiography.
second is called frequency of the wave. The SI unit of l Ultrasound scanner is an instrument which uses
ultrasonic waves for getting images of internal organs of
frequency is hertz.
the human body such as liver gall bladder, pancreas,
The frequency of a wave is the reciprocal of its kidney, uterus and heart etc.
time-period. l Ultrasound may be employed to break small stones
1
Frequency = formed in the kidneys into fine grains.
Time period l Ultrasound scans are used to monitor the development
of fetus (unborn baby) inside the mother’s uterus.
Velocity of Wave ( v) l Ultrasound is used in sonar apparatus to measure the
The distance travelled by a wave in one second is called depth of sea (or ocean) and to locate under sea objects
velocity of the wave (or speed of the wave). The SI unit like shoal of fish, shipwrecks submarines, sea-rocks and
for measuring the velocity of a wave is metre per hidden ice-bergs in the sea.
second (m/s). l The use of ultrasound waves to investigate the action of
the heart is called echocardiography.
Relationship between Velocity, Frequency
SONAR
and Wavelength of a Wave
The word SONAR stands for Sound Navigation And
The velocity of a wave in medium is equal to the product
Ranging. SONAR is an apparatus (or device which is
of its frequency and wavelength.
used to find the depth of a sea or to locate the
Velocity of a wave = Frequency × Wavelength under-water things like shoals of fish, shipwrecks and
⇒ v = nλ enemy submarines.
Sound 247
Speed of Sound (ii) The ceiling of concert halls, conference hall and
cinema halls are curved so that sound after reflection
l The speed of sound is different in different mediums.
reaches all corners of the half and curved sound
The speed of sound is dependent on elasticity, density
and temperature of medium. Sound travels slowest in board may be placed behind the stange so that the
gases, faster in liquids and fastest in solids. sound, after reflecting from the sound board, spreads
evenly across width of the hall.
l 1° C increase in temperature, the speed of the sound is
increased by 0.61 m/s.
Echo
l The density of dry air is more than of moist air. Thus in
moist air the speed of sound is more than dry air. Echo is the repetition of sound of short duration. To get
l If the medium has also the speed other the speed of an echo, the minimum distance of the source of sound
sound increases in the same direction and decrease in from the body should be 17 m.
opposite direction. Echo is heard sooner on a hot day (than a cold day)
Speed of Sound in Various Materials
Reverberation
S.No. Medium Speed of sound
The peristence of sound in a big hall due to repeated
1. Dry air (0° C) 332 m/s reflection from the walls, ceiling and floor of the hall is
2. Dry air (at 20° C) 344 m/s called reverberation.
3. Hydrogen (at 0° C) 1284 m/s
î The soft and porous materials are bad reflectors of sound. The
4. Water (at 20° C) 1498 m/s soft and porous materials are actually good absorbers of
5. Copper (at 20° C) 3750 m/s sound. In addition to certains carpets and sofa-sets in our
6. Iron (at 20° C) 5130 m/s rooms also reduce the formation of echos by absorbing sound
7. Glass (at 20° C) 5170 m/s waves.

Characteristics of Sound
Shock Waves
A sound has three characteristic loudness, pitch and
When an object moves with a velocity greater than that
quality
of sound it is termed as supersonic, when such a
supersonic body or plane travels in air, it produces Loudness
energetic disturbance which moves in backward l The loudness of sound is a measure of the sound energy
direction and diverge in the form alone such distrubance reaching the ear per second.
are called the shock waves. l The loudness of sound depends on the amplitudes of
sound waves.
Sonic Boom l The loudness of sound is measured in decibel.
Sonic Boom is an explosive noise caused by the shock l Loudness depends on intensity of sound. Unit of loudness
waves from an aircraft (or any other object) which is is bel.
travelling faster than the speed of sound.
Pitch
l Pitch is that characteristic of sound by which we can
Reflection of Sound distinguish between different sounds of the same
The bouncing back of sound when it strikes a hard loudness.
surface is called reflection of sound. l The pitch of a sound is directly proportional to its
frequency.
Law of Reflection
l Greater the frequency of a sound, the higher will be its
l The incident sound wave, the reflected sound wave, and pitch.
the normal at the point of incidence, all lie in the same
plane. Quality (or Timbre)
l The angle of reflection of sound is always equal to the l Quality is that characteristic of musical sound which
angle of incidence of sound. enables us to distinguish between the sound of same pitch
(i) The reflection of sound is utilised in the hearing of and loudness produced by different musical instruments.
devices such as Megaphone, Bulb horn, stethoscope, l The quality of a musical sound depends on the shape of
and sound board. sound wave produced by it.
Target Exercise
Elementary Level
1. A sound wave 6. Speed of sound at constant temperature depends on
(a) requires a material medium for its propagation (a) pressure (b) density of gas
(b) does not require a material medium for its propagation (c) above both (d) None of the above
(c) in some situations requires a material and in some other
cases does not 7. Sound travels in rocks in the form of
(d) None of the above (a) longitudinal elastic waves only
2. Sound waves cannot pass through (b) transverse elastic waves only
(a) a solid liquid mixture (b) a liquid gas mixture (c) both longitudinal and transverse elastic waves
(c) an ideal gas (d) a perfect vacuum (d) Non-elastic waves
3. A pulse is a wave 8. Water waves are
(a) of high duration
(b) of short duration (a) longitudinal
(c) which travels in vacuum only (b) transverse
(d) which travels in solids only (c) both longitudinal and transverse
4. A wave completes 48 cycle in 1.6 s, its frequency is (d) Neither longitudinal nor transverse
(a) 48 Hz (b) 1.6 Hz
9. The intensity of sound increases at height due to
(c) 30 Hz (d) 76.8 Hz
(a) increase in density of air
5. In transverse waves, particles of the medium vibrate (b) decrease in density of air
about their mean position (c) low temperature
(a) along the direction of propagation of wave (d) None of the above
(b) perpendicular to the direction of propagation of wave
(c) sometimes along and sometimes normal to the 10. Sonar emits which of the following waves?
propagation of wave
(a) Radio waves (b) Ultrasonic waves
(d) None of the above
(c) Light waves (d) Magnetic waves

High Skill Questions


1. Velocity of sound in air 3. Which of the following device does not work on the
I. increase with temperature multiple reflection of sound waves?
II. decrease with temperature (a) Stethoscope
III. increase with pressure (b) Hydrophone
IV. is independent of pressure (c) Soundboard
V. is independent of temperature (d) Megaphoner
Choose the correct answers 4. In the sound wave produced by a vibrating turning
(a) Only I and II are true
fork shown in the diagram, half the wavelength is
(b) Only I and III are true
(c) Only II and III are true represented by
(d) Only I and IV are true
2. A key of mechanical piano is first struch gently and
B C D E
then struck again but much harder this time. In the
A
second case
(a) Sound will be louder but pitch will not be drifferent
(b) Sound will be louder and the pitch will also be higher
(c) Sound will be louder but pitch will be lower (a) A B (b) B D
(d) Both loudness and pitch will remain unaffectd (c) D E (d) A E
Sound 249
5. When the pitch of note produced by a harmonium is 9. A rock under water is 1595 m deep. Calculate the
lowered, then the wavelength of the note time in which an ultrasonic signal return after
(a) decrease reflection from rock. Given velocity of ultrasonic
(b) first decrease and then increase waves in water is 1450 m/s
(c) increases (a) 1.4 s (b) 1.8 s (c) 2 s (d) 2.2 s
(d) remain the same
10. A The velocity of sound increases with increase in
6. When an aeroplane attains a speed higher than
humidity.
velocity of sound in air, a loud bang is heard. This is
because B Velocity of sound does not depend upon the
(a) It explodes medium.
(b) It produces a shock wave which is receives as the bang (a) Both A and B are true (b) Both A and B are false
(c) Its wings vibrate so violently that the bang is heard (c) A is ture and B is false (d) A is false and B is true
(d) the normal engine noises undergo a Doppler shift to
generate the bang 11. Which of the following frequency of sound can be
generated by a vibrating simple pendulum as well as
7. A body vibrating with a periodic time of 2 milli second
produces a wave travelling in a medium with a by the vibrating vocal cords of a rhinoceros?
(a) 8 Hz (b) 25 Hz
velocity of 1250 m/s. What is its wavelength?
(c) 10 Hz (d) 15000 Hz
(a) 1.5 m
(b) 1.5 cm 12. A Sound wave travel faster on a hot summer day
(c) 2.5 cm than on a cold winter day.
(d) 2.5 m
B Velocity of sound is directly proportional to square
8. A water wave is travelling with a speed of 1.5 m/s. If
of its absolute temperature.
the distance between two consecutive crests is 2.5 cm. (a) Both A and B are true
Calculate the frequency of the wave. (b) Both A and B are false
(a) 50 Hz (b) 60 Hz (c) A is true but B is false
(c) 80 Hz (d) 100 Hz (d) A is false but B is true

Hints and Solutions


Elementary Level
1. (a) 2. (d) 3. (b) 4. (c) 5. (b) 6. (d) 7. (c) 8. (c) 9. (a) 10. (b)

High Skill Questions


1 1
1. (d) 2. (a) 3. (b) 4. (b) Frequency = =
Time period 5 × 10−2
5. (c) 6. (b) 3
7. (d) Given, T = 2 × 10−3 s, u = 1250 m/s =
3
= 60 Hz
v ∴ n = 1 5 × 10−2
v = nλ , λ =
n  t 
9. (d) Time =
2d 2 × 1595
= = 2 .2 s
λ = vt = 1250 × 2 × 10 −3
= 2.5 m v 1450
10. (c) 11. (c)
8. (b) Speed of water wave v =15
. m/s
2.5 12. (c) The velocity of sound in a gas is directly proportional to the
d = 2.5 cm, T = × 10−2 square root of it absolute temperature. Since, temperature of a
.
15
hot day is more than cold winter day, therefore sound would
5
T= × 10−2s travel faster on a hot summer day than on a cold winter day.
3
Chapter

7
Heat
Temperature Thermal Equilibrium
Temperature of a body is defined as the degree of When two bodies attain the same temperature, they are
hotness or coldness of that body. The temperature of the said to be in thermal equilibrium with each other.
body is a measure of average kinetic energy of the Formula for calculating the quantity of heat absorbed or
molecules of the body. Kinetic energy of the molecules of given out by a body
a body increases as its temperature rises. Q = mc∆ t
where, Q = heat absorbed (or given out) by a body,
Thermometer m = mass of the body,
The device which measures the temperature of the body c = specific heat of the body,
is called thermometer. Clinical thermometer measures ∆ t = change in temperature of the body.
temperature in degree fahrenheit (F). In thermometer,
mercury is commonly used because it is opaque; shiny Thermal Expansion
and undergoes uniform thermal expansion through a
wide range from – 30°C to 300°C. When a body is heated then there is some increase in its
size, this expansion is called thermal expansion.
Temperature Scale
Expansion of Solids
The different types of temperature scales are
All the solids expand on heating. This expansion of
(i) Celsius scale (ii) Fahrenheit scale
(iii) Kelvin scale (iv) Reaumur scale
solids is called thermal expansion of solids.

Relation between the readings of temperature of these There are three types of expansion of solids
scales (i) Coefficient of linear expansion The increase in per
C F − 32 R K − 273 unit length, per degree celsius rise in temperature is
= = = called the coefficient of linear expansion.
100 180 80 100
∆L
5 Coefficient of linear expansionα =
Also, C= (F − 32) L × ∆t
9
Its unit is per °C.
Heat (ii) Coefficient of superficial expansion The increase in
per unit area per degree celsius size in temperature is
The form of energy which is exchanged among various called the coefficient of superficial expansion.
bodies or system an account of temperature difference is ∆A
Coefficient of superficial expansion β =
defined as heat. Its SI unit is joule and other unit A × ∆t
calorie is also usidely used. Its unit is per °C.
Heat 251
(iii) Coefficient of volume expansion The increase in per Melting Point (MP) The constant temperature at which a solid
unit volume per degree celsius rise in temperature is substance is converted into its liquid state is called melting point.
called the coefficient of volume expansion.
Boiling Point (BP) The constant temperature at which a liquid is
∆V
Coefficient of volume expansion γ = converted into its gaseous state is called its boiling point.
V × ∆t
Its unit is per °C. Calorie
î Relation between α , β and γ is α : β : γ = 1: 2 : 3
The amount of heat required to raise the temperature
of 1 g water by 1°C is called calorie.
Expansion of Liquid
1 cal = 4.18 J
Liquids have only cubical expansion because they have
no shape and size. When liquids are heated, then the Specific Heat
vessel in which liquids is kept also heated, so first vessel
The specific heat of a substance is the amount of heat
expands and then liquid.
which is required to raise the temperature of unit mass
Hence expansion of liquid is of two types
of the substance by 1°C.
1. Apparent Expansion The expansion of liquids
SI unit of specific heat is joule/kg/°C or J/kg/°C.
neglecting expansion of vessel is called
Q
apparent expansion. Specific heat c =
m × ∆t
Apparent expansion coefficient
Apparent increase in volume where, Q = amount of heat given to the substance
γα = m = mass of the substance
intial volume × rise in temperature
∆v α ∆t = rise in temperature
γα =
V × ∆t Principle of Calorimetry
2. Real Expansion It is the actual increase in When a hot body is mixed with a cold body, the heat lost
volume of liquid. Real expansion coefficient.
by hot body is equal to the heat gained by the cold body
Real expansion in volume provided no heat is allowed to escape to the surroundings.
γr =
Initial volume × rise in temperature Heat gained = Heat lost
∆V
γr =
V ×∆t Latent Heat
Latent heat of a substance is defined as the quantity of
Expansion of Gases heat which is supplied to a substance to bring about a
In expansion of gases, there are two coefficients change in its state without changing the temperature.
1. Volume Coefficients ( γ V ) The change in volume gas SI unit of latent heat is J/kg.
per unit volume per unit degree celsius constant pressure Formula for the heat absorbed or given out during
is known as coefficient of volume expansion. change of state of a substance
2. Pressure coefficient ( γ p ) The change in pressure of gas Q = mL
per unit degree celsius at constant volume is known as
where, Q = heat absorbed (or given out) during the
pressure coefficient.
change of state,
Thermal Capacity m = mass of the substance,
L = latent heat of the substance.
The thermal capacity of a body is the amount of heat
required to raise the temperature of the whole body Latent Heat of Fusion
by 1°C.
Latent heat of fusion of a solid is defined as the amount
Thermal capacity Q=m×c of heat required to convert a unit mass of the substance
where, m = mass of the body, from the solid state to the liquid state without
c = specific heat of the body. changing the temperature. The latent heat of fusion is
SI unit of thermal capacity is joule per degree centigrade 80 cal/g.
or J/°C.
252 Study Package for NTSE
î How does a water cooler cool the air? The Specific Heat Capacity of Some Materials
In the water cooler grass pads are used in all the three sides
and water is poured over them from the top. The atmospheric Specific Heat Capacity
air sucked through the wet pads evaporates water which Substance (103 J /g°C)
causes cooling.
Water 4.18
Aluminium 0.920
Latent Heat of Vaporization Glass 0.669
Latent heat of vaporization of a substance is the amount of Iron 0.419
heat required to change a unit mass of the substance from Copper 0.389
the liquid state to vapour state without changing the Brass 0.385
Silver 0.234
temperature. The latent heat of vaporization is 540 cal/g.
Mercury 0.138
Latent heat of vaporization is more than the latent heat
of fusion. The Values of Coefficient of
Linear Expansion (α)
Vaporization Coefficient of Linear Expansion
Solid
(10−5 °C −1 )
The process of converting a liquid into its gaseous state is
called vaporization. When water is heated Aluminium 2.5
continuously even after it attains the boiling point, it Brass 1.8
receives the latent heat due to which it changes its state Iron 1.2
Copper 1.7
from liquid to gaseous state.
Silver 1.9
Gold 1.4
Evaporation Glass 0.3
Lead 0.31
The process in which a liquid changes into vapours
called evaporation. Evaporation takes place below its The Values of Coefficient of Cubical (Volume)
boiling point. Expansion (γ)
The rate of evaporation depends upon
Coefficient of Coefficient of
l temperature of liquid Volume Expansion Volume Expansion
Solid Liquid
area of exposed surface
(10−5 per ° C −1 ) (10−5 per ° C −1 )
l

l nature of liquid
l open space available Iron 3.55 Mercury 18.2
Lead 0.84 Water 20.7
Paraffin 58.4 Petroleum 89.9
Relative Humidity Glass 2.53 Alcohol 122.0
Relative humidity is defined as the ratio of mass of
water vapour actually present in the certain volume of Difference between Fahrenheit Thermometer
air to the mass of water vapours required to saturate the and Celsius Thermometer
same volume of air at the same temperature. It is Fahrenheit Thermometer Celsius Thermometer
always expressed in percentage.
Lower fixed point is 32°F. Lower fixed point is 0°C.
m
Relative humidity = × 100 Upper fixed point is 212°F. Upper fixed point is 100°C.
M It is divided into 180 equal It is divided into 100 equal
where, m = mass of the water vapour actually present in divisions. divisions.
a given volume of air at the certain temperature, It is used to find out the It is used to find out the
temperature of human body. temperature of any substance
M = mass of the water vapour required to saturate the and atmosphere too.
same volume of air at the same temperature.
Heat 253
Difference between Heat and Temperature The Relationship between Heat
Heat Temperature and Work
Heat is form of energy. Temperature is the degree of When mechanical work is done, heat ( H) is produced.
hotness or coldness of body. The amount of heat produced is directly proportional to
The SI unit for measuring heat The SI unit for temperature is mechanical work done (W ).
is Joules. kelvin. W
i.e., W = JQ or J =
Heat is the sum of potential Temperature is the average Q
and kinetic energy of all the kinetic energy of all the
where, J is called the mechanical equivalent of heat.
molecules of a substance. molecules of a substance.

Difference between Specific Heat Air


and Thermal Capacity of a Body It is of three types
Specific Heat Thermal Heat (i) Dry air If the amount of water vapour present in the air
is very small. The air is said to be dry.
It is the amount of heat required It is the amount of heat
to raise the temperature of a unit required to raise the (ii) Humid air If the amount of water vapour present in the
mass of a substance through temperature of the whole body air is very large. The air is said to be humid.
1°C. by 1°C.
(iii) Saturated air The air containing maximum amount of
SI unit is J/kg/°C. SI unit is J°/C. water vapour at some particular temperature is called
saturated air at that temperature.

Target Exercise
Elementary Level
1. During change of state, the temperature of a 9. When vapour condenses into a liquid, then
substance (a) it absorbs heat
(a) increases (b) remains same (b) it evolves heat
(c) decreases (d) None of these (c) its temperature rises
(d) its temperature drops
2. At what temperature will the celsius and fahrenheit
scales indicate the same reading? 10. The quantity of heat required to change the temperature
(a) 40°C (b) – 40°C (c) 100°C (d) 0°C of 1 kg of substance by 1°C is called its
(a) specific heat (b) the total energy
3. One calorie is the heat required to raise the temperature (c) the latent heat (d) heat of fusion
of 1 g of water by
(a) 1°C (b) l K 11. Two blocks of lead, one twice as heavy as the other,
(c) 1°F (d) None of these are at 50°C. The ratio of the heat content of the
heavier block to that of the lighter block is
4. The unit of the coefficient of linear expansion (a) 0.5 (b) 1 (c) 2 (d) 4
(a) m (b) / °C (c) m/ °C (d) °C
12. A and B are two objects. The temperature of A is
5. The absolute zero on celsius scale is greater than that of B. This means that
(a) 80°C (b) – 12°C
(a) the molecules of A are moving faster on the average than
(c) 100°C (d) – 273°C
the molecules of B
6. Earliest thermometer was developed by (b) the total energy of A is greater than the total energy of the
(a) celsius (b) fahrenheit molecules of B
(c) kelvin (d) galileo (c) the average potential energy of A is greater than the average
potential energy of B
7. The unit of latent heat is (d) the heat content of A will always be greater than that of B
(a) cal/g (b) kcal/g/°C
(c) kcal/kg/°C (d) J/kg/°C 13. The temperature of a body is an indicator of
(a) the total energy of the molecules of the object
8. SI unit of temperature is (b) the average energy of the molecules of the object
(a) kelvin (b) celsius (c) the total velocity of the molecules of the object
(c) fahrenheit (d) joule (d) the average velocity of the molecules of the object
254 Study Package for NTSE

14. Heat energy of an object is 15. When an object is heated, the molecules of that object
(a) the average energy of the molecules of the object (a) begin to move faster
(b) the total energy of the molecules of the object (b) lose energy
(c) the average velocity of the molecules of the object (c) become heavier
(d) the average potential energy of the molecules of the object (d) become lighter

High Skill Questions


1. The temperature of the sun is measured with 8. What will be the thermal capacity of a vessel of mass
(a) platinum thermometer 20 kg and specific heat 450 J/kg/°C?
(b) gas thermometer (a) 2000 J/°C
(c) pyrometer (b) 4000 J/°C
(d) vapour pressure thermometer (c) 9000 J/°C
2. A litre of alcohol weighs (d) None of the above
(a) less in winter than in summer 9. When 500 g of boiling water is poured into a mug
(b) less in summer than in winter which weighs 0.15 kg, the temperature of water falls
(c) same both in summer and winter to 70°C, if the specific heat capacity of the substance
(d) None of the above of the mug is 800 J/kg° C, then the amount of heat lost
3. Water is used to coal radiators of engines, because by water will be
(a) of its lower density (b) it is easily available (a) 48000 J (b) 50000 J
(c) it is cheap (d) it has high specific heat (c) 58000 J (d) 63000 J
4. A metallic ball and highly stretched spring are made 10. The density of mercury at 0°C is 13.596 g/cm 3. Its
of the same material and have the same mass. They density at 100°C will be
are heated so that they melt, the latent heat required (a) 13.849 g/cm3
(a) are the same for both (b) 13.349 g/cm3
(b) is greater for the ball
(c) 18.849 g/cm3
(c) is greater for the spring
(d) 13.349 g/cm3
(d) for the two may or may not be the same depending upon
the metal 11. When hot tea is mixed with cold tea, the temperature
5. 540 g of ice at 0°C is mixed with 540 g of water at 80°C. of the mixture.
The final temperature of the mixture is (a) first decreases then becomes constant
(a) 0°C (b) 40°C (c) 80°C (d) less than 0°C (b) first increases then becomes constant
(c) continuously increasers
6. At what temperature will the fahrenheit scale have (d) is undefined for sometimes and then becomes nearly
the double reading as that of celsius? constant
(a) 260°C (b) 240°C
(c) 160°C (d) 370°C
12. A A beaker is completely filled with water at 4°C. It
will overflow, both when heated or cooled.
7. How much heat is required to convert 1 kg of ice at
B There is expansion of water below and above 4°C.
0°C to water at 0°C?
(a) Both A and B are true
(Latent heat of ice = 3.34 × 103 J / kg) (b) Both A and B are false
(a) 8.94 × 108 J (b) 3.34 × 103 J (c) A is true and B is false
(c) 3.34 × 108 J (d) 3.34 × 102 J (d) A is false and B is true
Hints and Solutions
Elementary Level
1. (b) 2. (b) 3. (a) 4. (b) 5. (d) 6. (d) 7. (a) 8. (a) 9. (b) 10. (a)
11. (d) 12. (a) 13. (b) 14. (b) 15. (a)

High Skill Questions


1. (c) 2. (b) 3. (d) 4. (a) 8. (c) Themal capacity W = m s
5. (a) Heat taken by ice to melt at 0°C is W = 20 × 450 = 9000 J/ ° C
Q1 = mL = 540 × 80 = 43200 cal
Heat given by water to cool upto °C is 9. (d)
Q2 = ms∆θ = 540 × 1 × (80 − 0) 10. (b)
= 540 × 80 = 43200 cal
Hence, heat given by water is just sufficient to melt the whole 11. (c) From Calorimetry
ice and final temperature of mixture is °C. m1 × s ∆t 1 ≡ m2 × s × ∆t 2
C F − 32
6. (c) = 2 × ∆t 1 = 1 × ∆t 2
5 7
C 2 C − 32 ∆t 1 1
= =
5 9 ∆t 2
2
7C = 10C − 32 × 5 ∆t 1
= 0.5
C = 32 × 5 = 160° C ∆t 2

7. (b) Q = mL 12. (a) Water has maximum density at 4°C on heating above 4°C or
cooling below, 4°C. Density of water decreases and its volume
= 1 × 3.34 × 103 = 3.34 × 103 J increases and therefore, water overflow in both the cases.
Chapter

8
Light
Reflection, Refraction
and Human Eye
l The light ray which falls on the mirror’s surface is
Light known as incident ray.

Light may be defined as the radiant energy which l The light ray which is sent back by the mirror is called
reflected ray. The normal is a line at right angle at the
causes sensation of sight. In other words, light itself
point of incidence.
is invisible but makes other objects visible. Light
l Angle between incident ray and the normal to the
waves are electromagnetic waves and do not require
surface is known as angle of incidence and the angle
any medium to propagate. The speed of light is between reflected ray and the normal to the surface is
different in different medium e.g., the speed of light called angle of reflection.
in vacuum is 3 × 10 8 ms −1 . When light travels in a
straight line, then this is known as rectilinear Laws of Reflection
propagation of light. There are two laws of reflection
Luminous object Those objects which emit their own (i) The incident ray, reflected ray and the normal, all lie
in the same plane.
light, stars, sun, fire, lamp candle are some examples of Normal
luminous objects. Reflected
Incident
Non-luminous object Those objects which do not emit ray i r ray
light energy itself are considered as non-luminous Mirror
objects.
(ii) Angle of incidence is always equal to the angle of
Planets, moon, buildings, chair are some examples of reflection.
non-luminous objects.
∠i = ∠r
Optical Medium A substance through which light can
travel without much loss in its intensity is known as Mirror
optical medium. For example air, water, glass etc.
A smooth and well polished surface which reflects most
Reflection of Light of the light falling on it regularly is known as mirror.
Phenomenon of change in the path of light without any Those images which can be obtained on the screen are
change in medium is known as reflection of light. In called real images.
other words, the process of sending back, the rays which Those images which cannot be formed on the screen are
falls on the surface of an object. Silver is the best called virtual images.
reflector of light.
Light Reflection, Refraction and Human Eye 257
Plane Mirror Centre of Curvature
l One surface of the mirror is plane and another surface It is the centre of the sphere of which the spherical
has a sharp metallic polish which is pasted. mirror is a part.
l For showing the full image of an object, the size of the Radius of Curvature
plane mirror should be at least half of the object.
Radius of curvature of spherical mirror is the radius of
l Image formed by a plane mirror is always virtual and
erect. The size of the image is equal to that of the object.
the sphere of which the mirror forms a part.
The image is laterally inverted. Mirror Formula
1 1 1
Spherical Mirror = +
f v u
A spherical mirror is a part of a hollow sphere, whose
one side is reflecting and other side is opaque. where, v is image distance, u is object distance and f is
focal length.
There are two types of spherical mirror R
Relation between f and R, f =
l Concave Mirror Whose reflecting surface is towards 2
the centre of sphere of which the mirror is a part. A
i.e., focal length is always equal to half the radius of
concave mirror is known as a converging mirror.
curvature of mirror or lens.
l Convex Mirror Whose reflecting surface is away from
the centre of sphere of which the mirror is a part. A Magnification
convex mirror is known as a diverging mirror. The linear magnification (m produced by a mirror is
Important terms Related to Spherical defined as the ratio of the size of the image (h′ to the size
Mirrors of the object h).
Size of image
Magnification =
Pole Size of object
The mid point or centre of the reflecting surface of h′
spherical mirror is called pole of the mirror. m=
h
Principal Axis v Image distance
and m= − =
The line joining the pole and the centre of curvature is u Object distance
called the principal axis of the mirror. It extends on both î If m is positive, then the image is erect and m is negative then
side of the mirrors. image is inverted.

Position and Nature of Image for Concave


S.
No. Position of object Ray diagram Position of image Nature and size of image
1. At infinity At focus or in the focal plane Real, inverted, extremely diminished in
size
C
P
F
m

2. Beyond the centre of Between focus and centre of Real, inverted and diminished
curvature A D curvature
C B
F P
B
A′
E

3. At the centre of At the centre of curvature Real, inverted and equal to object
curvature A D
B F
P
B' C
A' E
258 Study Package for NTSE

S.
Position of object Ray diagram Position of image Nature and size of image
No.
4. Between focus and centre Beyond centre of curvature Real, inverted and bigger than
of curvature E object
A
B' C D
B F P

A'

5. At the focus At infinity Extremely magnified


E
A
B' C D
B F P

A'

6. Between the pole and focus A' Behind the mirror Virtual, erect and magnified

A
C
B P B'
F

Position and Nature of Image for Convex Mirror


Image formed by convex mirror is always virtual, erect and smaller in size.
S.
No. Position of object Ray diagram Position of image Nature and size of image
1. At infinity Appears at the principal Virtual, erect and extremely
focus diminished

F C

2. Between infinity and the Appears between the Virtual, erect and diminished
pole A' principal focus and the pole
A
C
B' B F

Uses of Concave Mirrors


l Concave mirrors are commonly used in torches, search-lights and vehicles headlights to get powerful parallel beams
of light.
l They are used as shaving mirror.
l The dentists use concave mirrors to see large images of the teeth of patients.
l Large concave mirror are used to concentrate sunlight to produce heat in solar furnaces.

Uses of Convex Mirrors


Convex mirrors are commonly used as rear-view mirrors in vehicles.
Light Reflection, Refraction and Human Eye 259
Refraction of Light Refractive Index of Some Material Media
The phenomenon of change in path of the light when it Material medium Refractive index
goes from one medium to another is known as refraction Air 1.0003
of light. The basic cause of refraction is change in the Water 1.0003
velocity of light while going from one medium to the Crown glass 1.31
Carbon 1.52
other. Disulphide 1.63
N'
Incident Ruby 1.72
i
ray Diamond 2.42
O
r
M'
Effects of Refraction
Glass N
(i) A stick partly immersed in water appears to be bent
Emergent at the water surface.
Air O ray
(ii) An invisible coin placed in a vessel become visible
θ
C when water is poured in the vessel.
M
(iii) Rising and setting of sun appears visible even it
or below the horizon.
The phenomenon of bending of light when it passes from (vi) Twinkling of stars.
one medium to another is called refraction. î The refractive index depends upon the nature of the material
of the medium and on the wavelength of the light used. It
Law of Refraction does not depend on angle of incidence.

There are two laws of refraction


(i) The incident ray, the refracted ray and the normal at Lens
the point of incidence all three lie in the same plane.
(ii) The ratio of sine of angle of incidence to the sine of
Lens is a portion of a transparent refracting medium
angle of refraction remains constant for a pair of bounded by two spherical surfaces.
media These are of two types
sin i
i.e., = constant
sin r (i) Convex Lens
The constant value is called the refractive index of A convex lens is thicker in the middle and thinner at the
the second medium with respect to the first.
edges. This convex lens is also known as a converging
This law is also called Snell’s law.
lens.
Refractive Index
(ii) Concave Lens
Ratio of speed of light in two different mediums is
A concave lens is thinner at the middle and thicker at
known as refractive index. It is represented by µ.
the edges. This concave lens is also known as a
sin i
µ= diverging lens.
sin r
v Lens Formula
= 1
v2 1 1 1
= −
f v u
where, sin i = sine of angle of incidence (in air) and
sin r = sine of angle of refraction (in medium). where, f = focal length of lens
v = distance of image from lens
The refractive index of medium 1 with respect to
u = distance of object from lens
medium 2 ( 2 n1 ) is equal to the reciprocal of the refractive
index of medium 2 with respect to medium ( 1 n2 ).
1 Magnification Produced by Lenses
2 n1 = Linear magnification produced by a lens is the ratio of
1 n2
l

the size of the image to the size of the object.


The ratio of speed of light in vacuum c to the speed of h′
Linear magnification m =
light in any medium v is called absolute refractive index h
of the medium. l The linear magnification is equal to the ratio of image
c
Refractive index of a medium = distance to the object distance.
v v
m=
u
260 Study Package for NTSE

Power of Lens
Reciprocal of focal length is known as power of lens. Its unit is dioptre.
1
Power, P =
Focal length ( f )
If a number of thin lenses having power P1 , P2 and P3 etc., are placed in close contact with one another then their
resultant power P = P1 + P2 + P3 + ...
A convex lens has positive power and a concave lens has negative power.
Formation of Image by a Convex Lens

S. No. Position of object Ray diagram Position of image Nature and size of image
1. At infinity At focus Real, inverted, extremely small

F1 2F2
2F1 F2

2. Beyond 2 F1 Between F2 and 2 F2 Real, inverted, smaller than object


A
F2 B'
B 2F1 F1
A'

3. At 2 F1 At 2 F2 Real, inverted and same size as that of


A object
2F1 F2 B'
B F1 2F2
A'

4. Between F1 and 2 F1 Beyond 2 F2 Real, inverted and magnified


A
2F1 F2 2F2 B'
B F1 2F2
A'

5. At F1 At infinity Real, inverted and extremely magnified


A
F1 F2
B

6. Between pole and F1 A' On the same side as the Virtual, erect and magnified
object

B′ F1 B F2
Light Reflection, Refraction and Human Eye 261
Formation of Image by a Concave Lens
S.
Position of object Ray diagram Position of image Nature and size of image
No.
1. At infinity At F1 (on the same side as the object) Virtual, erect and extremely diminished

2. Between infinity Between the lens and F Virtual, erect and diminished
A
and the lens
A'
F2 2F2
2F1 B F1 B'

Optical Instruments 3. Telescope Telescope consists of two convex lenses


1. Simple Microscope This is simply a convex lens placed coaxially in a hollow tube. The lens facing the
of small focal length. The object to be enlarged is object is called objective and the lens toward the eye
placed within the focus of lens. is called eye piece.
B Objective
B
B

B2 Eye piece

A1 A u
A β
v α A2 A1
Fe Fe Fo

Magnification’s when final image is formed at D


Ve + L 2
and infinity B1
 D B2
mD = 1 + 
 f L1

 D fo  fe  fo
and m∞ =   Magnification mD = 1 +  and m∞ =
f fe  D fe
where, D is least distance of distinct vision. Length LD = f o + ue and L∞ = f o + f e
2. Compound Microscope It consists of two convex
lenses coaxially fitted in a hollow tube. The lens
facing the object is called objective and the lens
Human Eye
towards the eye is called eye piece. The front transparent part of the hard outer surface of
v  D the eye ball is called cornea. It protects the inner
Magnification mD = − o 1 + 
uo  fo  delicate parts of the eye. The image of an external object
Length of the microscope tube between two lenses is formed by the crystalline lens on the retina. The
L = vo + ue retina is a film of nerve fibres converting the curved
Ciliary
uo vo uo muscels

A Iris containing
fluid called Lens
1 Ratina
humour
B2 B
Cornea

A1

ve Optic
A2 nerves
262 Study Package for NTSE

back surface of eye. The most sensitive point on the 2. Hypermetropia or Long Sightedness
retina is called the yellow spot. The retina contains In this defect, we can see objects at far clearly but
rods and cones, which senses light intensity and colour cannot see which are near objects, clearly. This defect
respectively. The line joining the centre of cornea and the can be corrected by using convex lens.
centre of lens is called the optical axis of eye. The
anterior chamber contains a water fluid called aqueous Nearby
object
humour and posterior chamber contains a transparent
O
jelly called vitreous humour. The part from where the Image is formed
optic nerve starts from retina and enters the brain is behind the retina
insensitive to light and is called the blind spot. The ability Hypermetropic eye
of the eye to observe distinct objects situated at widely
different distances from it is known as accommodation. Its correction
Retina
Near Point and Far Point
Rays coming
l The point nearest to the eye at which an object is visible from distance object
distinctly is called the near point of the eye.
N R
l The maximum distance up to which the normal eye can
see the things clearly is called the far point of the eye. Convex lens
l The distance of near point from the eye is known as least
distance of distinct vision. For a normal eye, its value is Correction for
hypermetropic eye
about 25 cm.
l The distance between the near point and the far point is
called the range of vision. 3. Presbyopia
î Sometimes the crystalline lens of people at old age become It is that defect by which old man cannot read
milky and cloudy. This condition is called cataract. comfortably and clearly without spectacles. It can be
corrected by applying convex lenses.
Persistence of Vision
4. Astigmatism
The effect of vision due to which the impression of an
image rests on the retina even after the removal of the It is that defect due to which a person cannot focus on
image on it is called the persistence of vision. horizontal and vertical line simultaneously. It can be
corrected by superimposing cylindrical lenses.
Defects of Vision
Colour Blindness
1. Myopia or Short Sightedness
It is a defect in which we can see near objects clearly but Colour blindness is that defect of the eye due to which a
cannot see objects at far clearly. This defect can be person is unable to distinguish between certain colours.
corrected by using a concave lens. The colour blind person do not posses some cone cells
Retina that respond to certain colours. For example, red-green
Parallel rays
from distant object colour blind person does not have red and green colour
Image is
formed
pigment containing cells in the retina of his eye.
in front of
Eye lens has more
converging power or
retina
Prism
eye ball is too long
Myopic eye A prism is a transparent refracting medium bounded by
Its correction two plane surface inclined at same angle.
Parallel rays Retina
coming from infinity

Red
F R Orange
Yellow
Green
Concave lens Blue
White Indigo
Correction Light Violet
for Myopia Glass Prism
Light Reflection, Refraction and Human Eye 263
Dispersion of light l The brilliant red colour of rising and setting sun is due to
scattering of light.
The process of splitting up of a white beam into
constituent colours as it passes through a refracting î In the visible spectrum, the wavelength increases from violet
to red. Therefore blue or violet light is scattered much more
medium is known as dispersion of light. than red light.
î Higher the refractive index of the light rays, higher is the
angle of deviation caused by a prism. Colour
Rainbow Red, green and blue are primary or basic colours of light.
Dispersion and internal reflection results in production The colours obtained by mixing of any two primary colours
of colours are which appear hanging in the sky and of light are known as secondary colours.
known as rainbow. (i) Red + blue = magenta
(ii) Blue + green = peacock blue
Scattering of Light (iii) Red + green = yellow
l When light falls on various types of suspended particles, A mixture of all the three primary colours of light red,
it is deviated from its path in random directions. This green and blue produces white light.
phenomenon is called scattering.
(i) Red + green + blue = white
l The phenomenon of scattering of light by the suspended
⇒ Yellow + blue = red + peacock blue = green + magenta
particles (e.g., tiny water droplets) gives rise to Tyndall
= white
effect.
l The colour of the scattered light depends on the size of Any two colours on mixing, produces white light are
the scattering particles. known as complementary colours e.g.,
(i) Blue and yellow
l Scattering of light is maximum in case of violet colour and
minimum in case of red colour of light. (ii) Red and peacock blue
(iii) Green and magenta.
l Blue colour of sky is due to scattering of blue light.

Target Exercise
Elementary Level
1. Which one of the following materials cannot be used 6. A tank of water appears to be less deep than it
to make a lens? actually is due to
(a) Water (b) Glass (c) Plastic (d) Clay (a) reflection of light
(b) refraction of light
2. Where should an object be placed in front of a convex (c) high refractive index of water
lens to get a real image of the size of the object? (d) low refractive index of air
(a) At the principal focus of the lens
(b) At twice the focal length
7. Unit of power of lens is
(c) At infinity (a) metre (b) dioptre
(d) Between the optical centre of the lens and its principal focus (c) kg-m (d) dioptre-cm 2
8. The focal length of the thick lens having greater
3. The middle vascular coat that darkens the eye chamber
radius of curvature in comparison to the thin lens will
and prevents refraction by absorbing the light rays is be
(a) choroid (b) vitreous humour (a) more (b) less
(c) retina (d) aqueous humour (c) equal (d) None of these
4. Image of an object is formed by the human eye at? 9. Which one is not a primary colour?
(a) Pupil (b) Retina (c) Iris (d) Cornea (a) Red
(b) Blue
5. What should be the position of an object so that a real (c) Yellow
and inverted image of same size is obtained using a (d) All are primary colour
convex lens?
(a) Between the lens and its focus 10. The image formed by an object due to long
(b) At the focus sightedness
(c) At twice the focal length (a) behind the retina (b) on retina
(d) At infinity (c) before retina (d) None of these
264 Study Package for NTSE

11. Astigmatism occurs when the cornea does not have a 14. When an object approaches a convex lens, the size of
truly spherical shape. This defect can be rectified by the image formed will
the use of a (a) decrease
(a) concave lens (b) cylindrical lens (b) increase
(c) convex lens (d) plano-convex lens (c) first decreases then increases
(d) remains the same
12. Which one represents the unit of refractive index?
(a) Metre (b) Degree 15. Which one of the following denotes the unit of power
(c) Diopter (d) It has no units of a lens?
13. Planets do not twinkle like stars, because (a) Metre
(a) they do not have light of their own (b) Dyne
(b) they are nearer to earth as compared to stars (c) Diopter
(c) they are rotating all the time (d) None of the above
(d) they are smaller than stars

High Skill Questions


1. The true statement is
(a) The order of colours in the primary and secondary rainbow is
the same
(b) The intensity of colours in the primary and the secondary
(a) P
rainbows is the same Principal axis F
(c) The intensity of light in the primary rainbow is greater and the
order of colours is the same than the secondary rainbow
(d) The intensity of light for different colours in primary rainbow
is greater and the order of colours is reverse than the 4. Which one of the following correctly defines the
secondary rainbow reflection of light ray on a spherical mirror?
2. Match the List I with the List II from the combination
shown.
List I List II
(b) P
A. Presbiopia 1. Sphere-cylindrical lens Principal axis F
B. Hypermetropi 2. Convex lens of proper power may
a be used to close to the eye
C. Astigmatism 3. Concave lens of suitable focal
length
D. Myopia 4. Bifocal lens of suitable focal length

Codes (c) P
Principal axis F
A B C D
(a) 1 3 2 4
(b) 4 1 3 2
(c) 4 2 1 3
(d) 2 4 3 1

3. Which one of the following is correct for concave


mirror? (d) P
Principal axis F
Position Position Nature of
Image size
of object of image image
(a) At C At C Equal to Real and
object inverted 5. An object of size 4 cm placed perpendicular to the
(b) Beyond C Between F Diminished Virtual and principal axis of a concave mirror. The distance of the
and C erect object from the mirror equals radius of curvature. The
(c) Between F At infinity Enlarged Real and size of the image will be
and C inverted (a) 1 m (b) 2 m
(d) At F At infinity Highly Virtual and (c) 3.5 cm (d) 4 cm
diminished erect
Light Reflection, Refraction and Human Eye 265
6. The path of a refracted ray of light in a prism is 11. Which of the following would form a highly diminished
parallel to the base of the prism only when the image at focus?
(a) light is of a particular wavelength
(b) ray is incident normally at one face
(c) ray under goes minimum deviation
(d) prism is made of a particular types of glass
2F1 F 2F1 F
7. The distance v of the real image formed by a convex
lens is measured for various objects distance u.
(I) (II)
A graph is plotted between v and u, which one of the
following graph is correct? (a) I (b) II
(c) I and II (d) None of these

(a) v (b) v 12. An object is placed at a distance 30 cm from a convex


mirror of focal length 15 cm. The image formed is
(a) 20 cm from the pole
(b) real and magnified − 10 cm from the pole
u u
(c) + 10 cm from the pole
(d) + 60 cm from the pole

(c) v (d) v 13. A 10 mm long alpin is placed vertically in front of a


concave mirror. A 5 mm long image of the alpin is
formed at a distance of 30 cm in front of the mirror.
u u
The focal length of this mirror is
(a) − 30 cm (b) − 20 cm
8. A The air bubble shines in water. (c) − 40 cm (d) − 60 cm
B Air bubbles in water shines due to refraction of light.
(a) Both A and B are true (b) Both A and B are false 14. Figure shows a ray of light as it travel from
(c) A is true and B is false (d) A is false and B is true medium A to medium B. The refractive index of
medium B relative to medium A is
9. A The cloud in sky generally appear to be whitish.
B Diffraction due to cloud is efficient in equal measure at all
wavelengths. Medium B
(a) Both A and B are true (b) Both A and B are false 45°
(c) A is true and B is false (d) A is false and B is true 45°
10. How will the image formed by a convex lens be affected 30°
60°
if the upper half of the lens is covered with a black
paper? Medium A
one-half
Black paper (a) 3 / 2 (b) 2 / 3
(c) 1/ 2 (d) 2 /2

15. A light bulb is placed between two plane mirrors


inclined at angle of 60°. The number of images formed
(a) The size of the image is reduced to per half of the image will
are
be absent (a) 6
(b) The up (b) 2
(c) The brightness of the image is reduced (c) 5
(d) There will be no effect (d) 4
Hints and Solutions
Elementary Level
1. (d ) 2. (b) 3. (a) 4. (b) 5. (c) 6. (b) 7. (b) 8. (b) 9. (c) 10. (a)
11. (b) 12. (d) 13. (b) 14. (b) 15. (c)

High Skill Questions


1. (d) 2. (c) 3. (a) 4. (c) 13. (b) Magnification
5. (d) 6. (c) v h2
m= − =
u h1
7. (d) For lens
1 1 1
= − v h2
− =
f v u u h1
If u = ∞, v = f and if u = f, v = ∞ 30 5
− =
So, option d is correct. u 10
8. (c) Shining of air bubble in water is an account of total internal u = − 60 cm
reflection. we have u = − 60 cm
9. (c) The clouds consists of dust particles and water droplets. v = − 30 cm
Their size is very large as compared to the wavelength of the 1 1 1
incident light from the sun. So, there is very little scattering of = +
f v u
light. Hence the light which we receive through the clouds has all
1 1
the colours of light. As a result of this, we receive almost while = −
− 30 60
light, therefore the cloud are generally white.
−2 −1
10. (c) 11. (a) =
60
12. (c) Here u = − 30 cm, f = 15 cm 1 −3
=
1 1 1 f 60
We have = +
f v u 60
f= = − 20 cm
1 1 1 30
= −
15 v 30 sin i
14. (b) Refractive index µ =
1 1 1 sin r
= +
v 15 30 sin 45°
µ=
1 2+2 sin 60°
=
v 30
1/ 2 2 2
1 3 µ= = =
= 3 3× 2 3
v 30
2
⇒ v = 10 cm
360° 360°
15. (c) Number of image n =  − 1 =  − 1 = 5
Hence, the image formed is 10 cm from the pole.  0   60° 
Chapter

9
Electricity
Charge where, k is electrostatic force constant. The value of k in
SI unit is 9.0 × 10 9 Nm 2 / C2 , q1 and q2 are two point
Charge is a fundamental property of matter. There are charges and r is the distance between them. The force
two types of charges acts always along the line joining the two charges.
(i) Positive Charge Charge obtained by a glass rod
when rubbed with a silk cloth. Conductors and Insulators
(ii) Negative Charge Charge obtained by ebonite rod
when rubbed with woolen cloth.
Conductors The substances through which charges
can flow easily are called conductors. The examples of
Like charges repel each other while unlike charges conductors are all metals like copper, silver, gold,
attract each other. Electric charge is conserved, additive aluminium and iron etc. Carbon in the form of graphite is
and quantised. Charge carried by a body does not also a conductor. The human body is a very good
depend upon velocity of the body. Force between charges conductor.
can be attractive or repulsive, according as charges are
opposite or similar charges. Insulators The substances through which electricity
cannot flow are called insulators. The examples of
î Proton carries positive charge, neutron is electrically neutral
and electron carries negative charge.
insulators are glass, ebonite, paper, plastic, cotton, dry
air, mica and bakellite etc.
The magnitude of charge on an electron is called the
fundamental charge. Its value is 1.6 × 10 −19 coulomb
and is denoted by e. Electric Current
Conservation of Charge The charge can neither be Electric current is the rate of flow of electric charges. If a
created nor destroyed but it may simply be transferred net charge q flows across any cross-section in time t, the
from one body to another. This is called the principle of current I through the cross-section is
conservation of charge. q ne
I = =
t t
Coulomb’s Law
The unit of electric current is ampere.
According to this law, the force of attraction or repulsion
between any two point charges is directly proportional to 1 milli ampere (mA) = 10 −3 A
the product of the magnitude of charges and inversely 1 micro ampere ( µA ) = 10 −6 A
proportional to the square of the distance between them. Electric current is a scalar quantity.
qq
F ∝ 12 2 î The conventional direction of electric current is taken as
r opposite to the direction of the flow of electrons that is, from
k q1 q2 the positive terminal of the battery to the negative, through
F = the external circuit.
r2
268 Study Package for NTSE
y
Electrostatic Potential where, R is proportionality
constant called resistance of a
The electrostatic potential difference or potential
conductor.
difference is defined as the work done in bringing a unit
This graph shows that if we plot l
of positive charge from one point to another in an
electric field. It is denoted by V. a graph between current flowing
SI unit of potential difference is volt. and potential difference, we get x
V
a straight line passing through
Electric Potential and Potential Difference origin.
The work done on the electric charge in the process of Resistance The resistance R of a conductor is defined
charging gets stored as potential energy of charges. The as the ratio of the potential difference V across it to the
potential energy per unit charge is called electric current I flowing through it.
potential. V
Resistance R =
Electric potential energy I
i.e., Electric potential =
Charge The SI unit of resistance is the Ohm.
Work done
= 1 kΩ = 10 3 Ω
Charge
W Resistivity
or V =
q l Resistance of a conductor is directly proportional to its
length.
Electric potential difference between two points in an
electric circuit carrying some current as the work done R ∝ l
to move a unit charge from one point to the other. l Resistance of a conductor is inversely proportional to the
area of cross-section.
The SI unit of electric potential is volt.
1
R ∝
Voltmeter A
l The device which is used to measure the potential l Resistance of a conductor also depends upon the nature
difference between the two ends of the conductor is of material and temperature of the conductor.
called voltmeter. From all above discussion, we conclude that
l The voltmeter is connected parallel across the two ends l ρl
of the conductor where potential difference is to be R∝ or R =
A A
determined.
where, ρ is constant and is called resistivity of the
l A voltmeter must have high resistance.
conductor. The resistivity of a material does not depend
Ammeter on length and area of cross-section of wire. So resistivity
An ammeter is a device which is used to measure of wire remains unchanged.
electric current. An ammeter is always connected in
series with the circuit. Combination of Resistances
The resistances can be combined in two ways
Ohm’s Law l In series When we have to increase the resistance of a
conductor then individual resistors are connected in
This law states that the current flowing through a series. These resistors are connected end to end
conductor is directly proportional to the potential consecutively. The resistors are said to be connected in
difference across the ends of the conductor, provided series if same current is flowing through each resistor
physical condition such as temperature, mechanical when some potential difference applied across the
strain etc., are kept constant. conductor.
I ∝V R1 R2 R3

or V ∝I
or V = RI R = R1 + R2 + R3
V Maximum current can be drawn when the resistors
or =R are connected in series.
I
Electricity 269
l In parallel When we have to decrease the resistance of The SI unit of electric power is watt.
a conductor then individual resistors are connected in
1 kilowatt = 1000 W
parallel. These resistors are connected between the
same two points. In this combination, potential 1 Megawatt (MW) = 10 6 W
difference remains same
1
=
1
+
1
+
1 1 Horse power = 746 W
R R1 R2 R3
Kilowatt Hour Kilowatt hour is the commercial unit of
R1
electrical energy.
R2 Electrical energy = Power × Time
1 kWh = 1 kW × 1 hour
R3
= 1000 W × 60 × 60 s
= 3600000 J
Heating Effect of Electric Current 1 kWh = 3.6 × 10 6 J
l When an electric current flow through a metallic conductor, it gets
heated up. This is called heating effect of current. This effect utilised
in devices such as electric heater, electric iron etc. Household Electrical Appliances
l More amount of current passing through the conductor, large is the
rate of collision and consequently larger is the amount of heat Electric Bulb
produced. It is based on the principle of heating effect of electrical
current and used for lighting purposes. It consists of a
coiled filament fitted on a glass stem inside a glass bulb,
Electrical Energy the filament is made up of pure tungsten because it has
The amount of heat produced (or the electrical energy a high resistivity and a high melting point (3000°C).
spent) is equal to the amount of work done sending the The bulb is evacuated and is filled with an inert gas like
current through the conductor. argon and neon. This prevents oxidation and
V2T
H = I 2 RT = VIT = evaporation of the filament and thus increases its life.
R
The SI unit of electrical energy is Joule. Electric Iron

This relation was recognised and experimentally In the electric iron, nichrome wire is used because
verified by joule and is so called joule’s law of heating nichrome has a high resistivity and high melting point.
effect. When an electric current is passed through the
filament coil, it gets heated to about 800°C and
Electric Power becomes red hot.

The rate at which work is done by an electric current is Electric Fuse


known as electric power. A fuse is a device for the safety of appliances and electric
W H circuits against excessive heating during a short circuit
P= =
t t or overloading. Generally, the fuse is a wire of pure tin
2
I Rt or an alloy of tin and lead (63% tin + 37% lead). The fuse
= = I2R
t wire has high resistance and low melting point. It is
V2 used in series with electric installations.
P=
R
Target Exercise
Elementary Level
1. The resistivity does not change if 8. What is one kilowatt hour equal to?
(a) the material is changed (a) 1 J (b) 3.6 × 106 J
(b) the temperature is changed (c) 1000 J (d) None of these
(c) the shape of the resistor is changed 9. The resistance of germanium ……… with increase in
(d) both material and temperature are changed temperature.
2. The commonly used safety fuse wire is made of (a) decrease (b) increase
(a) copper (c) remains same (d) Cannot say
(b) lead 10. Which one is the correct graph between current and
(c) nickel potential difference?
(d) an alloy of tin and copper
3. An electric iron draws a current 4 A when connected

Current
Current
to a 220 V mains. Its resistance must be (a) (b)
(a) 1000 Ω (b) 55 Ω
(c) 44 Ω (d) None of these
Potential difference Potential difference
4. Out of three bulbs 25 W, 60 W and 100 W, which bulb
has least resistance?
(a) 25 W bulb

Current
Current

(b) 60 W bulb (c) (d)


(c) 100 W bulb
(d) All bulbs have equal resistance
Potential difference Potential difference
5. The unit of electric charge is
(a) watt (b) coulomb
11. The element used almost exclusively for filaments of
(c) joule (d) ampere
incandescent lamps
6. The force between two parallel wires carrying current (a) copper (b) gold
has been used to define (c) silver (d) tungsten
(a) ampere (b) pressure 12. Voltmeter is used to measure
(c) volt (d) force (a) potential difference (b) current
7. The resistance of carbon ……… with rise in the (c) power (d) resistance
temperature. 13. With which wire is the fuse wire connected?
(a) decrease (a) Live wire (b) Earth wire
(b) increase (c) Neutral wire (d) Any wire
(c) remains constant
(d) firstly decrease then increase 14. Fixed resistance is called
(a) rheostat (b) resistor (c) key (d) switch
Electricity 271

High Skill Questions


1. How many electrons will contribute in causing one 7. Four cells of resistance 1/4 Ω are connected in series
microampere of current? with one cell has its terminal reversed. The current
(a) 6.25 × 1015 (b) 6.25 × 109 flowing will be, if each cell has emf 1.5 V and 1 Ω is
(c) 6.25 × 1012 (d) 6.25 × 106 the external resistance
(a) 1.5 A
2. Two wires that are made up of two different
(b) 3.5 A
materials whose specific resistance are in the ratio (c) 2.8 A
2 : 3, length 3 : 4 and area 4 : 5. The ratio of their (d) None of the above
resistance is
(a) 6 : 5 (b) 6 : 8 (c) 5 : 8 (d) 1 : 2 8. If a wire of resistance 1 Ω is stretched to double its
length, then the resistance will become
3. The current flowing through a conductor when a 1
charge of 40 C flows through for 2 min is (a) Ω
2
(a) 20 A (b) 80 A
(b) 2 Ω
(c) 0.48 A (d) 0.33 A 1
(c) Ω
4. The equivalent resistance between the points A and B 4
in the circuit as shown in the figure given below is (d) 4 Ω
9. Calculate current flowing in the circuit given below
2Ω 2Ω 2Ω
A B
2Ω
2Ω
(a) 1 Ω (b) less than 1 Ω
(c) more than 1 Ω (d) 8 Ω
24V
5. In the circuit shown here, what is the value of the (a) 0.4 A
unknown resistor R so that the total resistance of the (b) 10 A
circuit between points P and Q is also equal to R (c) 24 A
10 Ω (d) 16 A

3Ω 10. Certain observation and certain causes are listed in


P Q
columns I and II respectively in the following table.
3Ω R
Column I Column II
(a) 3 Ω (b) 39 Ω
A. Electric charge 1. watt
(c) 69 Ω (d) 10 Ω
B. Electric potential 2. ampere
6. If the ammeter in the given circuit reads 2 A the C. Electric current 3. Coulomb
resistor R is D. Electric power 4. volt
3Ω
R Codes
A B C D
6Ω
(a) 3 4 2 1
(b) 2 4 3 1
6V
A (c) 1 2 4 3
(a) 1 Ω (b) 2 Ω (c) 3 Ω (d) 4 Ω (d) 3 1 2 4
Hints and Solutions
Elementary Level
1. (c) 2. (d) 5. (b) 6. (a) 7. (a) 8. (b) 9. (a)
q 40
3. (d) Current i = , i = = 0.33 A 10. (b) From Ohm’s law,
t 2 × 60
V=i R
V2 V ∝i
4. (c) We have, P= or
R
where, R is constant. Therefore, graph between an potential
1
i.e., P∝ difference V current will be straight line.
R
11. (d) 12. (a)
Therefore, as resistance decreases power increases.
13. (a) 14. (b)
So, bulb of 100 W has least resistance.

High Skill Questions


10 × (3 + R ) 30 + 10 R
1. (b) R=3+ =3+
10 + (3 + R ) 13 + R
2. (c) We have, ρ1 : ρ2 = 2 : 3 , l1 : l 2 = 3 : 4
39 + 3 R + 30 + 10 R 69 + 13 R
R= =
and A1 : A 2 = 4: 5 13 + R 13 + R
We have,
13 R + R 2 = 69 + 13 R
l
Resistance R= ρ
A R= 69 Ω
R1 ρ1 l A 2 3 5 R 5 6. (a) Resultant resistance
= × 1 × 2= × × , 1 =
R 2 ρ2 l 2 A1 3 4 4 R2 8 6×3
R’ = + R
i.e., R1 : R 2 = 5 : 8 6+ 3

3. (c) R’ = 2 + R
V
4. (c) Current I =
2Ω 2Ω R
6
A B 2=
2+ R
2Ω R = 1Ω

2 Ω and 2 Ω are in series, then 7. (b)


R = 2 + 2 = 4Ω l
8. (d) Resistance R = ρ
A
4 Ω and 2 Ω are in parallel, then R1  r
2
After stretching = M 
1 1 1 1+ 2 1 3 4 R2  l2 
= + = , . Ω
= R’ = = 133
R' 4 2 4 R’ 4 3 2
1 l 
Therefore, the equivalent resistance between the points A and B =  1
in the circuit is more than 1Ω . R2  l 2 
2
5. (c) The given circuit can be simplified as follows 1  l 
= 1
10 Ω R2  2 l 2 
1 1
3Ω =
R2 4
P Q
R2 = 4 Ω
(3 + R) R1 R 2 2 ×2
9. (c) Equivalent resistance Req = = = 1Ω
10 Ω R1 + R 2 2 + 2
V 24
3Ω Hence, current flowing in the circuit I = = = 24
Req 1
P Q
3Ω R 10. (a)
Chapter

10
Magnetic Effects of
Electric Current
Magnets Uniform and Non-uniform Field If the magnetic field
lines parallel and equidistant, the field is uniform and if
An object which shows the property of attracting small they are unequally spaced the field is non-uniform.
pieces of iron towards it and resting in north-south
direction, when it suspended freely is called a magnet. Magnetic Field due to Current Carrying Conductor When
This property of magnet is called magnetism. an electric current is passed through a conductor, a
magnetic field is set up around it, which exists so long as
The substances which have magnetism and are found
the current flows in the conductor. This is called magnetic
naturally are known as natural magnets, whereas the
effect of electric current.
materials which are made magnets artificially by
different methods are known as artificial magnets. A Magnetic Field due to a Current through a Straight
piece of lodestone (Fe 3O4 ) is a natural magnet. Conductor The magnetic field produced at a given
Magnetic Field The space surrounding a magnet point is directly proportional to the current flowing in
where a magnetic force is experienced is called a the wire.
magnetic field. The magnetic field produced is inversely proportional to
distance of that point from the wire
Magnetic Field Lines A magnetic fields line is a
1
continuous curve in a magnetic field. i. e., B∝
r
Properties of Magnetic Field Lines Thus greater is the current in the wire, stronger is the
magnetic field produced and greater is the distance of a
l They travel from north pole to south pole outside the
magnet and south pole to north pole inside the magnet. point from the wire, weaker is the magnetic field.
l They are always in the form of closed and continuous
curve.
Force on a Current Carrying Conductor
l The number of field lines per unit area is the measure of
Placed in Magnetic Field
the strength of magnetic field. l If a straight conductor of length l carrying current I of
ampere is placed in a magnetic field B, perpendicular to
l Magnetic field lines are closed near the pole of a
the field, thus it will be acted upon by a force
magnet and become wider as we move away from the
pole. F = BIl
l Two magnetic lines of forces never intersect each other. If l Force on a charge q, moving in magnetic field in a
the lines intersect then at the point of intersection there direction perpendicular to the field with a velocity v, is
would be two directions for the same magnetic field, given by the formula
which is not possible. F = qvB
274 Study Package for NTSE
ordinary steel. Such types of strong magnets are used in
Right Hand’s Thumb’s Rule microphones, loudspeakers, ammeters, voltmeters and
If a current carrying conductor is imagined to be held in speedometers etc.
your right hand such that the thumb points along the Force on a Current Carrying Conductor in a Magnetic
direction of current then the direction of magnetic field Field The magnetic force on a current carrying
lines. conductor
(i) is directly proportional to the strength of current
– Direction of i. e., F∝i
current (ii) is directly proportional to the length ( l ) of the
conductor
Direction of i. e., F ∝ l
magnetic field
(iii) is directly proportional to the strength of external
magnetic field ( B )
i. e., F ∝ B
+
(iv) is directly proportional to the sine of angle between
direction of current in conductor and the magnetic
Accordingly if current in wire is vertically upward, the field F ∝ sin θ
magnetic field lines are anticlockwise while if current in ∴ Force F = Bil sin θ
wire is vertically downward, the magnetic field lines are
clockwise. Fleming’s Left Hand Rule
Magnetic Field Due to a Circular Coil The magnitude If fore finger, middle finger and thumb of our left hand
of magnetic field at the centre of current carrying are stretched mutually perpendicular in such a way that
circular loop is directly proportional to the current fore finger points along the direction of magnetic field,
passing through the circular loop and is inversely middle finger points along the direction of current then
proportional to the radius of circular loop. thumb will indicates the direction of force.
µ I
i. e., B= 0 Thumb
2r (Motion)

Magnetic Field due to a Solenoid The magnetic First finger


(Field)
induction B at the centre of such solenoid is given by
µ NI
B = 0 where, N is the number of turns of length l
l Second finger
of solenoid, I is the current in ampere and µ 0 is the (Current)
permeability of material used to make magnet. Motion
Field
Electromagnet
l If an electric current is passed round a straight bar of soft Current
iron, it becomes a very strong magnet so long as the
current passes through it. As soon as the current is Electric Motor
stopped it loses its all magnetism. Such a magnet is called l An electric motor is a device which converts electrical
an electromagnet. energy into mechanical energy.
l The electromagnets are used in electric bell, telephone, l The electric motor works on the magnetic effect of current.
electric motor, etc. l Electric motor is used as an important component in electric
l The permanent magnets are made of steel while the fans, refrigerators, mixers, washing machines etc.
electromagnets are made of soft iron. Materials like
carbon steel, chromium steel, cobalt and tungsten steel Magnetic Flux
and some of the alloys which have trade names are
usually used for making permanent magnets.
l The total number of magnetic field lines passing through
a given area normally is called magnetic flux.
l Nipermag is an alloy of iron, nickel aluminium and
l Magnetic flux = Magnetic field × Normal area
titanium while alnico is an alloy of aluminium of these
alloys are many times stronger than those made up of φ = B⋅ A
Magnetic Effects of Electric Current 275
Electromagnetic Induction
Whenever there is a change in magnetic flux linked with

Voltage
a coil (or circuit) an emf is induced in the coil. This
phenomenon is called electromagnetic induction and Time
emf produced in the coil is called the induced emf.
Galvanometer A galvanometer is an instrument that
can detect the pressure of a current in a circuit. The
l The frequency of AC supply in india is 50 cycles/second (or
pointer remains at zero (the centre of the scale) for zero hertz).
current flowing through it. It can deflect either to the
left or to the right of the zero mark depending on the
direction of current. Generator/Dynamo
An electrical machine used to convert mechanical
Faraday’s Law energy into electrical energy is known as generator.
l Whenever there is a change in magentic flux linked with a
conductor an emf is induced. AC Dynamo
l The magnitude of the emf induced is directly proportional to It is a device which produces alternating current or in
the rate of change of magnetic flux by the conductor.
other words, in which the direction of the induced
The direction of current induced in a closed circuit is always such as
current changes alternatively after equal intervals of
to oppose the cause that produces it. This is called Lenz’s law. Lenz’s
law is based on conservation of energy. time. In this case we use slip rings due to which the
direction of the current changes. It works on the
principle of electromagnetic induction.
Fleming’s Right Hand Rule
If we stretch the forefinger, middle finger and thumb of DC Dynamo
our right hand in such a way that thumb points along It is a device which produces direct current or in other
the direction of motion of conductor, forefinger along the words, the direction of the current remains the same. In
direction of magnetic field, then the middle finger points case of DC dynamo, split ring commutator is used,
along the direction of induced current. which gives current only in one direction.
Thumb
(Motion) î An important advantage of AC over DC is that electric power
can be transmitted over long distances without much loss of
First finger energy.
(Field)
Motion
Field Points to Remember
Second finger
l
Neon testor is used to indicate the presence of electricity (AC
(Current) Current or DC ) in a circuit.
l
A switch is a device used to ON and OFF an electric circuit or
appliance.
l
A fuse is a small piece of wire made of a materials of high
Direct and Alternating Current resistance and low melting points. The maximum safe current
in an electric fuse I ∝ D3/2
Direct Current where I is current and D is diameter of wire.
l A current that always flows in one direction l
To avoid the risk of fatal electric shock, the metal body of
is called Direct Current (DC). electric appliance is earthed.
Current

l It is graphically represented by a straight l


The fuse in the lightning and fans circuit of a small house is of15
line parallel to the time axis. A rating which means that the fuse will melt if the current
Time exceeds 5A value.
l
Electric bell and electric fan are application based on magnetic
Alternating Current effect of current.
l An electric current which flows first in one direction in a
l
When a current carrying coil is suspended freely in earth’s
circuit called the positive direction, then in the reverse or magnetic field. It’s plane stays in east-west direction.
negative direction at a regular interval of time is called
Alternating Current (AC).
Target Exercise
Elementary Level
1. Magnetic lines of force 8. Which is immaterial for an electric fuse?
(a) have more directions (a) its radius (b) its material
(b) have no physical reality (c) its length (d) All of these
(c) can be used to indicate the directions of the magnetic field 9. According to international convention of colour
of a point coding in a wire
(d) All of the above (a) live is red, neutral is black and earth is green
2. The direction of the force on a current carrying wire (b) live is brown, neutral is blue and earth is green
(c) live is brown, neutral is green and earth is black
placed in a magnetic field depends on
(d) live is red, neutral is yellow and earth is blue
(a) the direction of the current but not on the direction of the field
(b) the direction of the field but not on the direction of the current 10. The magnetic field inside a long solenoid carrying
(c) the direction of the current as well as the direction of the field current
(a) is zero
(d) neither the direction of the current nor the direction of the
(b) decreases as we move towards its ends
field
(c) increases as we move towards its ends
3. An electric current can be produced in a closed loop (d) is the same at all points
(a) by connecting it to a battery, but not by moving a magnet 11. Which of the following is not associated with
near it Fleming’s right-hand rule?
(b) by moving a magnet near it, but not by connecting it to a (a) Magnetic field (b) Movement
battery (c) Resistance (d) Induced current
(c) by connecting it to a battery, as well as by moving a magnet 12. For a current in a long straight solenoid N and S
near it poles are created at the two ends. Among the
(d) neither by connecting it to a battery nor by moving a magnet following statements, the incorrect statement is
near it
(a) The field lines inside the solenoids are in the form of straight
4. Which of the following involves electromagnetic lines which indicate that the magnetic field is same at all
induction? points in the solenoid
(a) A rod is charged with electricity (b) The strong magnetic field produced inside the solenoid can
(b) An electric current produced a magnetic field be used to magnetise a piece of magnetic material, when
(c) A magnetic field exerts a force on a current carrying wire soft iron is placed inside the coil
(d) The relative motion between a magnet and a coil produces (c) The pattern of magnetic field associated with the solenoid is
an electric current different from the pattern of the magnetic field around a
magnet
5. You have a coil and a bar magnet. You can produce an (d) The N and S poles exchange positions when the direction of
electric current by moving current through the solenoid is reversed
(a) the coil, but not the magnet
13. In the arrangement shown in figure, there are two
(b) the magnet, but not the coil
coils wound on a non-conducting cylindrical rod.
(c) either the magnet or the coil
Initially the key is not inserted, then the key is
(d) neither the magnet nor the coil
inserted and later removed. Then
6. Which of the following measures will decrease the
strength of the magnetic field of a current carrying
solenoid?
(a) Increase the temperature of the solenoid
(b) Increase the current
(c) Increase the number of turns of wire G
K
(d) All of the above
(a) the deflection in the galvanometer remains zero throughout
7. When is the back emf of an electric motor the
(b) there is a momentary deflection in the galvanometer but it
greatest? dies out shortly and there is no effect when the key is removed
(a) When the motor is first switched on
(c) there are momentary deflections in the galvanometer that
(b) When the motor is speeding up
die out shortly, the deflections are in the same direction
(c) When the motor is at maximum speed
(d) there are momentary deflections in the galvanometer that
(d) When the motor is slowing down
die out shortly, the deflections are in opposite directions
Magnetic Effects of Electric Current 277
14. The most important safety method used for 15. The magnetic lines of force inside a bar magnet
protecting home appliances from short circuiting or (a) are from south-pole to north-pole of the magnet
overloading is (b) are from north-pole to south-pole of the magnet
(a) earthing (b) use of fuse
(c) do not exist
(c) use of stablizers (d) use of electric meter
(d) depend upon the area of cross-section of the bar magnet

High Skill Questions


1. If the key K in the arrangement (figure) is taken out 4. Dynamo core is laminated because
(i.e., circuit is made open) and the magnetic field (a) magnetic field increases
lines are drawn over the horizontal plane ABCD. The (b) magnetic saturation level in case increase
lines are (c) residual magnetism in core decrease
Variable resistance
(d) loss of energy in core due to eddy currents decrease
– +
A 5. A Magnetic flux can produce induced emf.
A Long strength B Faraday established induced emf experimentally.
conductor
(a) Both A and B are correct
B (b) Both A and B are false
D (c) A is true and B is false
(d) A is false and B is true
C
6. A A spark occur between the poles of a switch when
K the switch is opened.
(a) concentric circles B Current flowing in the conductor produces
(b) elliptical in shapes magnetic field.
(c) straight lines parallel to each other (a) Both A and B are correct
(d) concentric circles near the point O, but of elliptical shapes as (b) Both A and B are false
we go away from it (c) A is true and B is false
2. A circular loop placed in a perpendicular to the plane (d) A is false and B is true
of paper carries a current when the key (K , is ON the 7. Match the following columns.
current as seen from point A and B (in the plane of
paper and on the axis of the coil) is anticlockwise and Column I Column II
clockwise respectively. The magnetic field lines A. Electric motor 1. Indicate the presence of
points from B to A. The north pole of the resultant electricity
magnet is on the close to B. Generator 2. Electrical energy into
mechanical energy
A B C. Neon tester 3. Mechanical energy into
electrical energy
+ –
A B C
Variable
(a) 2 3 1
resistance (b) 1 2 3
(c) 2 1 3
(d) 1 3 2
A
8. A conducting wire is moving towards right in a
(a) A magnetic field B. The direction of induced current in
(b) B the wire is shown in the figure. The direction of
(c) A if current is small and B if current is large magnetic field will be
(d) B if current is small and A if current is large
(a) in the plane of paper pointing
3. According to Faraday’s law of electromagnetic towards right
induction (b) in the plane of paper pointing
(a) the direction of induced current is such that it opposes the towards left Bi
cause producing it (c) perpendicular to the plane of v
(b) the magnitude of induced emf produced in a coil is directly paper and downwards
proportional to the rate of change of magnetic flux (d) perpendicular to the plane of
(c) the direction of induced emf is such that if opposes paper and upwards
(d) None of the above
278 Study Package for NTSE
9. A helium nucleus makes a full rotation in a circle of 10. Due to 10 A of current flowing the circular coil of
radius 0.8 m in 2s. The values of the magnetic field B 10 cm radius, the magnetic field produced at its
at the centre of the circle will be centre is 3.14 × 10−3 Wb/m2 . The number of turn in
10−9 the coil will be
(a) (b) 10−19 µ 0
µ0 (a) 5000 (b) 50
−10 (c) 50 (d) 25
2 × 10
(c) 2 × 10−10 µ 0 (d)
µ0

Hints and Solutions


Elementary Level
1. (a) 2. (c) 3. (c) 4. (d) 5. (c) 6. (a) 14. (d) AC generator consist of construction, armature, strong field
magnet. Slip rings and brushes while DC generator consist of
7. (c) 8. (c) 9. (b) 10. (d) 11. (c) 12. (c)
armature coil, magnet, commutator and brushes.
13. (d) In arrangement, the key is inserted and later removed, then
15. (a) Magnetic field lines travel from north pole to south pole
there are momentary deflections in the galvonometer are in
outside the magnet and south pole to north pole inside the
opposite directions.
magnet.

High Skill Questions


1. (a) Magnitude of the magnetic field produced at a given point is spark as the current bridges the air gap between the poles of the
directly proportional to the current flowing in the wire. switch. The spark is more of likely in circuits wire of large
The magnetic field produced is inversly proportional to distance inductances.
of the point from the wire 7. (a)
1
B ∝
r 8. (c) According to Fleming right hand rule, the direction of B will be
perpendicular to the plane of paper and act downward.
If the current in wire i vartically downward the magnetic field lines
are clockwise that is the magnetic field lines are concentric . × 10−19
q 2 × 16
9. (c) Current i = = . × 10−19 A
= 16
circles. t 2
2. (a) 3. (b) 4. (d) . × 10−19
µ 0 i µ 0 × 16
∴ B= = = µ 0 × 10−19
2r 2 × 0.8
5. (d) Electromagnetic force induces, when there is no change in
magnetic flux. Faraday did experiments in which, there is relative 10. (c) The magnetic field
motion between the coil changes and emf induces. µ 0 π Ni
B= ⋅
6. (a) According to Lenz’s law, induced emf are in a direction such 4π r
as to attempt to maintain the original magnetic flux when a 10−7 × 3.14 × N × 10
3.14 × 10− 3 =
change occurs. When the switch is opened, the sudden drop in (10 × 10−2 )
the magnetic field in the circuit induces an emf in a direction that
N = 50
attempts to keep the original current flowing. This can cause a
Chapter

11
Sources of Energy
Energy Coal
Ability or capacity to do work is called energy. The Coal is a complex mixture of compounds of carbon,
energy associated with a hot body is called heat energy hydrogen and oxygen and some free carbon. It is found
or thermal energy. The energy produced by nuclear in deep coal mines under earth’s surface. Some small
reactions is called nuclear energy. The energy amounts of nitrogen and oxygen are also present.
produced by chemical reactions is called chemical Uses of Coal
energy. The energy associated with an electric current (i) It is used as fuel for heating purposes in homes and in
is known as electrical energy. These are some forms of industries.
energy. SI unit of all the forms of energy is joule and (ii) Coal after destructive distillation formed coke which is
denoted by J. Energy of a body is a scalar quantity. used as a reducing agent in the extraction of metals.
(iii) It is also used as a fuel in thermal power plants for
generating electricity.
Sources of Energy (iv) It is used in the manufacture of fuels gases like coal gas.
The main source of energy on this earth is the solar
energy. Sources of energy can be classified as renewable Petroleum
sources or non-renewable sources Petroleum (from Latin petra means rock and oleum
means oil) is a dark viscous oily liquid and is considered
Renewable Sources to be of biological origin. Usually, it is formed by the
Those sources which are being produced continuously in decomposition of marine animals under high pressure
nature and are inexhaustible and are also known as and temperature. It is well known that the crude
petroleum oil is not a single compound but it is a mixture
non-conventional sources of energy. Wood, falling
of compounds. It is lighter than water and insoluble in it.
water, sun, winds, plants are some examples of
In the crude petroleum oil, there is a complex mixture of
renewable sources. several solid, liquid and gaseous hydrocarbons mixed
with water, salt and earth particles.
Non-renewable Sources
Those sources which are not produced continuously and Natural Gas
it takes a long time to replenish and are exhaustible. It is another important fossil fuel which is found with
These sources are also known as conventional petroleum in oil wells. It is by product of petroleum
sources of energy. Coal, natural gas, petroleum are mining. There are also some oil wells, which yield only
some examples of non-renewable sources. natural gas. Natural gas mainly consists of methane
(CH4 ) with a small quantities of ethane and propane.
Fossil Fuels Uses of Natural Gas
A natural fuel formed deep under the earth from the
Following are the uses of Natural Gas
prehistoric remains of plants and animals is called
(i) As we know, natural gas mainly consists of CH4.
fossil fuel. Examples of the fossil fuel are: natural gas, When it is heated strongly, it produces carbon and
coal, petroleum etc. hydrogen. When this hydrogen is heated with
280 Study Package for NTSE
nitrogen, it combines to form ammonia (NH3 ) . NH3 is (ii) For producing electricity, light energy of the sun
reacted with acid to form ammonium salts, which are radiation can be used to produce electricity by using
used as fertilisers. solar cells made of silicon (these are called
(ii) As natural gas is heated strongly, methane photovoltaic cells). Photovoltaic cells are also used for
decomposes to form carbon and hydrogen. The carbon running the equipments in satellites.
thus formed is called carbon black which is used as (iii) Solar energy is used for making salt from the sea
filler in the tyre industry. water.
(iii) When natural gas is compressed highly due to high (iv) Solar energy is used for cooking food by solar cooker.
pressure, it is changed into CNG (compressed (v) Solar energy has been used for the preservation of
natural gas), it is used in vehicles (cars, trucks, buses fruits, vegetables and fish etc., by the process of sun
etc.). It is a good alternative of petrol and diesel drying.
because it does not cause much air pollution.
(vi) Solar energy is used to run watches and calculators.
(iv) It is also used as a fuel in thermal power plants for
(vii) Solar energy is used by the plants for photosynthesis.
generating electricity.
(viii) Solar energy keeps the atmosphere warm. We can sit
(v) It is used as a domestic and industrial fuel. Natural
in sunlight during the winter for making the body
gas burns readily to produce a lot of heat.
warm.

Advantages of Natural Gas Advantages of Solar Energy


Natural gas is a complete fuel in itself (when heated, (i) It is renewable (non-conventional) form of energy
produces a lot of heat without pollution) so it can be used which is produced coutinuous in nature and therefore
both in homes and industries directly. It does not it is inexhaustible.
produce pollutants, so it is termed as eco-friendly fuel. (ii) It is free and available in adequate quantities in
Natural gas can be directly supplied from the gas wells almost all the parts of world.
to homes and factories for burning purposes through a (iii) It is very clean in use and causes no pollution.
network of underground pipelines and which eliminates
the need for additional storage and transport. Solar Panel
l A single solar cell produces very small current at a small
Thermal Power Plant potential difference. So for practical use, a large number
A power plant in which heat energy is used to generate of such solar cells are connected together.
electricity is called thermal power plant. Heat energy is l A combination of a large number of solar cells is called a
obtained by burning fossil fuels like coal, oil or gas. solar cell panel. A solar cell panel can provide stronger
currents under high potential difference.
Solar Energy l Solar cell panels are used for producing electricity for use
in space stationary artificial satellites.
The sun is one of the main sources of renewable sources
of energy. The energy obtained from the sun is called Solar
irrandlance
solar energy. The light and heat energies of sunlight are Module

collectively called solar energy.


One square metre of the Earth’s upper atmosphere
receives 1.4 kJ of energy per second from the sun.
Then, solar energy Battery
Amount of energy by Earth’s surface
=
charger
Time (second) × Area controller

Thus, if E is total energy received by area A of earth Battery

surface in time t.
E
Then, solar constant S =
A×t

DC Loads
Uses of Solar Energy
Battery system
The main uses of solar energy are
Solar Panel
(i) Solar energy is used for drying clothes.
Sources of Energy 281
Use There are some conditions necessary for combustion,
these are
Solar cells are being used
l Attainment to ignition temperature
(i) in wrist watches, toys and calculators. l Presence of air
(ii) in artificial satellites and space probes as main l Presence of combustible material
source of energy.
(iii) in radio or wireless transmission systems or TV relay Calorific Value of Fuel
stations in remote locations.
The amount of heat produced by the unit mass of a fuel
(iv) in traffic signals.
is called the calorific value of a fuel. Heigher the
(v) in street lighting in rural areas.
calorific value, better the fuel.
(vi) for operating water pumps for domestic and
agricultural purposes. Calorific value of fuel
(vii) for operating radio and TV sets in remote/rural areas. Heat produced on complete combustion
=
(viii) for providing electricity to lighthouses, situated in Amount of fuel
the sea and off-shore oil drilling platforms.

Advantages of Solar Cells Nuclear Reaction


l A nuclear reaction may be defined as the reaction in
(i) Solar cells directly utilise solar energy.
which nucleus or nuclei of atom or atoms undergoes a
(ii) They can work satisfactorily even in the presence of change and new elements are formed.
diffused radiations.
(iii) They do not produce any pollution.
l We can consider when uranium-235 atoms are
bombarded with neutrons, nucleus of each uranium atom
(iv) They do not require maintenance.
breaks to form nuclei of barium and krypton with the
emission of neutrons.
Conventional Non- 235
92 U + 0 n → 56 Ba
1 139
+ 94
36 Kr + 3 10 n + Energy
Conventional Sources of Energy Uranium Neutron Barium Krypton Neutron

Nuclear reaction are mainly of two types


Conventional Energy l Nuclear fission
The traditional sources of energy which are being used l Nuclear fusion
for countries and are known to common people are called
the conventional sources of energy. Some Nuclear Fission
conventional sources of energy like wood, flowing water The process in which nucleus of a heavy element splits
are renewable while like coal, petroleum are non into two constituent nuclei is called nuclear fission.
renewable.
For example, when isotope of uranium U-235 is
bombarded with slow moving neutron, its nucleus splits
Non-Conventional Energy
into the nuclei of barium, krypton and 3 neutrons with
The sources of energy which are available in nature and release of enormous amount of energy.
can be used by technology of recent times are called the 235
92 U + 0 n → 56 Ba
1 139
+ 94
36 Kr + 3 10 n + Energy
non-conventional or alternate sources of energy.
Uranium Neutron Barium Krypton Neutron
The example of non conventional energy sources are
solar energy, wind energy, bio-mass energy, ocean Nuclear Fusion
energy, geothermal energy and nuclear energy etc.
The process in which two nuclei of light atoms combine
Bio-Energy together to form a heavy nucleus is called nuclear
fusion. In nuclear fusion also a large amount of energy
Green plants convert solar energy into chemical energy
is released, but to begin a nuclear fusion reaction, a very
by photosynthesis process, this energy is known as
high temperature is required and it is about 10 6 °C.
bio-energy.
We can see it, in sun, at temperature 4 × 10 6 ° C, 4 nuclei
Combustion of hydrogen combine to give the nuclear fusion as shown
l Process of burning of fuel is known as combustion.
below.
During the process of combustion, heat and light are 4 11 H → 4
2 He + 210 e + Energy
produced by the fuels. The mixture of methane and other Hydrogen Helium Positron
gases produced by an aerobic decomposition of slurry of
plants and animals wastes is called bio-gas.
282 Study Package for NTSE

92 U + → 139
56 Ba+ + 3 10 n + Energy
235 1 94
Difference between Nuclear Fission and 0n 36 Kr
Nuclear Fusion Uranium Neutron Barium Krypton Neutron

S.No. Nuclear Fission Nuclear Fusion From above reaction, we can see that uranium is the
radioactive element.
1. In nuclear fission, a heavy In a fusion reaction, two light
nucleus splits into two nuclei combine to form a
lighter nuclei. heavy nucleus. Chain Reaction
2. High temperature is not Very high temperature Chain reaction is that reaction in which the particle that
required to initiate the 4,000,000°C is required to begins a reaction is also produced in larger number
reaction. initiate the reaction. during the time of reaction and the reaction continues
3. It is chain reaction. It is not a chain reaction. on its own till one of the reactants is consumed
4. It is carried by bombarding It is carred by heating light completely.
the heavy nucleus with nuclei at very high.
neutrons. temperature, bombarding e.g., Fission of U-235 is a chain reaction.
of neutrons is not required. l Suppose a neutron is incident on a nucleus of U-235, it
5. Controlled fission reaction is Nuclear fusion reactions splits into two parts and 3 neutrons. The 3 neutrons will
used to generate electrcity in have not been controlled till strike the three nuclei and they will produce 9 neutrons
nuclear reactors. today. and then 27 neutrons, and so on.
6. A large amount of energy is In fusion reactions energy l The amount of energy released is very-very large and
released. released is several times result is a big explosion. A chain reaction is the basis of an
greater than that of fission Atom bomb.
reactions.
7. Splitting of uranium shows Combinatins of hydrogen Nuclear Reactor
nuclear fission. nuclei to form helium nuclei
in sun shows nuclear fusion. The whole arrangement where controlled fission
8. The products formed as The products formed as a reaction is carried out and' nuclear energy is converted
result of nuclear fission are result of nucelar fusion are into heat energy is called nuclear reactor.
radioactive and harmful. No not radioactive and not It was Italian scientist Enrico Fermi who carried out
proper method of disposal harmful. a controlled nuclear fission in the United States on
of the waste products has
2 December 1942 and mankind entered the nuclear
been invented so far.
age.

Radioactivity Principle of Working of Nuclear Reactor


l The phenomenon in which the unstable nuclei tend to
Disintegration of nuclear fuel is carried out and heat
disintegrate into smaller atomic nuclei to get stability
and radiates some radiations is known as radioactivity energy, so generated is used to heat water which then
and elements are called radio-active elements or turns turbines and electric generators. In this way,
radioisotopes. nuclear energy is used for peaceful work.
l In some atomic nuclei, there is a balance between the l Brown rods absorb excess neutrons in the nuclear
nuclear force which is attractive force and electrostatic reactor.
force which is repulsive force and a slight disturbance l A substance that slows down the speed of neutron in the
makes the nucleus of atom unstable and nucleus nuclear reactor, is called moderator.
disintegrates into smaller nuclei. We can see it when l Graphite and Heavy water act as a moderator in the
U-235 bombarded with slow moving neutrons, it starts to
nuclear reactor.
split into new elements barium and krypton with the
emission of large amount of heat as given below l Cadmium rod also absorbs neutrons in the nuclear
reactor.
Target Exercise
Elementary Level
1. At the present rate of consumption, our total oil 10. In a nuclear reaction the particles that take part, are
reserve would last for (a) electrons and protons
(a) 10-12 years (b) 12-15 years (b) protons and neutrons
(c) 20-30 years (d) 50-60 years (c) neutrons only
(d) All of the above
2. At the present rate of consumption, our total coal
reserves would last for 11. In a chemical reaction only ...... take part.
(a) 200-300 years (b) 300-400 years (a) electrons in outermost orbitals
(c) 50-100 years (d) 100-200 years (b) electrons in innermost orbitals
(c) protons of nucleus
3. India obtains … of energy from nuclear reactors. (d) neutrons of nucleus
(a) 9% (b) 17% (c) 8% (d) 3%
12. When all the hydrogen in sun’s crust change into
4. Enrichment of uranium is carried out at nuclear fuel helium
complex (a) stop extinguishing the nuclear fire
(a) Kota (b) Narora
(b) sun would, begin to collapse and shrink under its own
(c) Hyderabad (d) New Delhi
gravitational attraction
5. In India, uranium mainly comes from (c) the temperature in the interior core would rise again and
(a) Khetri mines (b) Jadugada mines helium nuclei will change to carbon nuclei
(c) Nangalam (d) Hyderabad (d) All the above processes may take place
6. The first nuclear reactor set up in India was at 13. A neutrino is
(a) Rana Pratap Sagar (b) Kalpakkam (a) massless and positively charged
(c) Tarapur (d) Trombay (b) equal to mass of electron and negatively charged
7. To absorb excess neutrons from the nuclear fuel we use (c) massless and negatively charged
(a) sodium or potassium rods (d) massless and having nil charge
(b) magnesium or beryllium rods
14. A positron is
(c) cadmium or boron rods
(d) sodium or lithium rods (a) equal to proton in mass and positively charged
(b) equal to electron in mass and positively charged
8. The scientist who first performed the fission reaction (c) equal to neutron in mass and negatively charged
was? (d) equal to electron in mass and negatively charged
(a) Hans Bethe (b) Enrico Fermi
(c) Rutherford (d) Alfred Noble 15. When two protons moving at high speed collide they
change into
9. In a nuclear fission reaction
(a) one neutron, one positron, one neutrino
(a) only electrons are redistributed, protons remain as such
(b) one positron, one neutrino
(b) only protons are redistributed, neutrons remain as such
(c) one proton, one positron, one neutrino
(c) only neutrons are redistributed, protons remain as such
(d) one proton, one neutron, one positron, one neutrino
(d) protons and neutrons both are redistributed
284 Study Package for NTSE

High Skill Questions


1. Complete the following reaction 21 H + 21 H → 5. The nuclear fusion reaction takes place at the
(a) 10
(b) 3 condition of
4 Be 2 He
4 4 (a) extremely high temperature
(c) 2 He (d) 2 Be (b) 378°C
2. Certain observation and certain are listed in (c) 205°C
(d) low pressure
Columns I and II respectively in the following table.
6. There are three elements A, B, C having mass
Column I Column II number 3, 180 and 235 respectively. Which one of
A. Electron 1. James Chadwick them is suitable for making an atom bomb?
(a) A (b) B
B. Proton 2. Rutherford
(c) C (d) All of these
C. Neutron 3. J.J. Thomson
D. Positron 4. Carl D. Anderson 7. Which two are isotopes of each other
88 Z 230 , 90 Z
230
, 88 Z 228 , 89 Z
230
A B C D
230 228 230 230
(a) 3 2 1 4 (a) 88 Z and 88 Z (b) 88 Z and 89 Z
(b) 4 3 2 1 (c) 230
and 230
(d) 228
and 230
88 Z 90 Z 88 Z 90 Z
(c) 1 4 3 2
(d) 2 1 4 3 8. Total coal reserves in our country will last for …
3. In addition to visible light, which other type of years only.
(a) 50-100 (b) 100-200
radiations are emitted by a 100 W bulb when it is
(c) 200-300 (d) 500-1000
glowing?
(a) Red light 9. In which kind of reaction only electron can take part?
(b) Violet light (a) Fusion (b) Fission
(c) Infra-red rays (c) Physical (d) Chemical
(d) None of the above 10. Which of the nuclear reaction are useful in the
4. Atom bombs based on which nuclear reaction? generation of electricity?
(a) Chemical (b) Physical (a) Fission (b) Fusion
(c) Fusion (d) Fission (c) Both a and b (d) None of these

Hints and Solutions


Elementary Level
1. (b) 2. (a) 3. (d) 4. (c) 5. (b) 6. (d) 7. (c) 8. (b) 9. (d) 10. (b)
11. (a) 12. (d) 13. (d) 14. (b) 15. (d)

High Skill Questions


1. (c) 2. (a) 3. (c) 4. (d) 7. (a) The atoms of element having same atomic number but different
mass number are called isotopes. If isotopes have same chemical
5. (a) The precess in which lighter nucleus fuse together to form a
properties.
heavier nucleus is called nuclear fusion. At very high temperature
230 230
or very high speed is required by two particles for fusion. i.e., Z 88 and 90 Z are isotopes
At high temperature, the material consisting of moving charged 8. (c) 9. (d)
particle with equal number of positive and negative charges are
called plasma. 10. (a) Nuclear reactor is used for generating electricity. A nuclear
reactor is a device in which nuclear fission can be controlled
6. (c) chain reaction.
Part-II : Chemistry
Chapter

1
Is Matter Around
us Pure
Matter
Anything which occupies space and has mass is called matter.
Matter

Classification on the basis Classification on the basis


of physical state of chemical constitution

Element Compound Mixture

Solid Liquid Gas

Solid is rigid, having Liquid is fluid, Gas having


definite shape having definite indefinite shape
and volume volume and volume

Mixture
A mixture is a chemical substance which consists of two or more elements or compounds not chemically combined
together. Mixture contains the properties of constituent elements or compounds. e.g., Air is a mixture of gases
like oxygen, nitrogen, argon, carbon dioxide and water vapour etc.

Types of Mixtures
Mixtures are of two types
1. Homogeneous Mixture Mixture having uniform composition throughout, is known as homogeneous mixture. A
mixture of sugar in water is homogeneous mixture.
2. Heterogeneous Mixture Mixture which doesn’t have a uniform composition throughout its mass, is known as
heterogeneous mixture. A mixture of petrol and water is a heterogeneous mixture.
286 Study Package for NTSE

Solution
Homogeneous mixture of two or more non-reacting substances, is known as solution. In a binary solution, the
substance exists in excess is known as solvent, while the substance in minority is known as solute.
e.g., In sugar solution, sugar is solute and water is solvent.
Types of Solutions
Solute Solvent Example Solute Solvent Example
Gas Gas Air (mixture of many gases). Liquid Solid Mercury in an amalgam, hydrated salts.
Gas Liquid Aerated drinks. Solid Gas Iodine vapours in air.
Gas Solid Gases adsorbed over metal surface Solid Liquid Salts dissolved in water.
(occlusion). Gases dissolved in minerals.
Liquid Gas Humidity in air. Solid Solid Alloys of metals e.g., Interstitial solid solution.
Liquid Liquid Mixture of miscible liquids (alcohol in water).

Saturated, Unsaturated Supersaturated Solutions


Saturated solution is a solution, which has the maximum amount of the solute dissolved in it at a particular
temperature. If such a solution is heated, it becomes unsaturated in the sense that it can dissolve more of the same
solute. Similarly, if a saturated solution is cooled, the solubility of the solute will decrease and it will come out of the
solution and will appear on the surface, such a solution is known as supersaturated solution.
Suspension, Colloidal Solution and True Solution
Colloidal Colloidal
Property Suspension True solution Property Suspension True solution
solution solution
Particle size > 10−7 m 10−7 − 10−9 m < 10−9 m Appearance Opaque Generally Transparent
transparent
Separation by Tyndall effect Shows Shows Doesn’t show
(a) Ordinary Possible Not possible Not possible Diffusion of Doesn’t diffuse Diffuse slowly Diffuse rapidly
filtration particles
(b) Ultrafiltration Possible Possible Not possible Nature Heterogeneous Heterogeneous Homogeneous
Setting of Settle under Settle only on Doesn’t settle Example Chalk-water Milk, blood, Sugar solution,
particles gravity centrifugation mixture, muddy soap solution salt solution
water

Separation of Mixtures
It can be done by the following methods
1. Evaporation To separate the volatile component from its non-volatile solute, we use evaporation e.g., Dye from ink.
2. Centrifugation The method of separating finely suspended particles in a liquid when it is rotated at a very high
speed in a used container is called centrifugation, e.g., Separating cream from milk.
3. Separating Funnel This method is used for immiscible liquids that separate out in layers depending on their
densities.
4. Sublimation The changing of a solid directly into vapour on heating is called sublimation e.g., Separation of salt and
ammonium chloride because ammonium chloride sublimes.
5. Chromatography It is used to separate those solutes that dissolve in the same solvent e.g., To separate colours in a
dye.
6. Distillation The process of separation of a liquid from a soluble solution by converting a liquid into its vapour and
then condensing vapour to its pure form in the liquid state is another container is called distillation.
7. Fractional Distillation It is used to separate the components of air, which is a homogeneous mixture.
8. Crystallisation To obtain pure copper sulphate from an impure sample we use crystallisation. It is generally used to
purify solids.
Is Matter Around us Pure 287
Physical and Chemical Changes
A physical change is one in which some of the physical properties colour, state etc., of the substances undergoing
change are altered temporarily, but the actual composition and mass remain unchanged and no new substances are
produced. The substance is restored to its original state as soon as the carve of change is withdrawn. Evaporation of
water, dissolution of salt in water etc., are the examples of physical change.
A chemical change is one in which the substances undergoing change are transformed into new substances,
chemical properties of which are quite different from the original substances. e.g., Burning of coal, souring of milk,
digestion of food in our body, rusting of iron etc. Chemical changes are irreversible, i. e., The obtained substance can
be changed back to the original substances.

Element
Elements are building blocks of matter and it can neither be broken nor be built up from two or more simple
substances by any known physical and chemical method.
There are 118 elements known at present, out of which 92 elements occurs in nature and rest are man-made.
Solid elements Sodium (Na), Carbon (C), Aluminium (Al) etc.
Liquid elements Mercury (Hg), Bromine (Br) etc.
Gas elements Helium (He), Neon (Ne) etc.

Compound
Compound is a pure substance made up of two or more elements chemically combined in a fixed ratio by weight. It is
always homogeneous.
A compound cannot be separated into its components by physical methods. The properties of a compound are
different from the constituent elements. e.g., Water (H2 O) is a compound made up of two elements, hydrogen and
oxygen, chemically combined in a fixed ratio of 1 : 8 by weight.

Points to Remember
● A mixture can be classified as homogeneous or heterogeneous.
● Mixtures can be separated by using various separation techniques, into pure substances.
● The concentration of a solution is the amount of solute present per unit volume or per unit mass of the solution/solvent.
Mass of solute
(i) Mass by mass percentage of a solution = × 100
Mass of solution
Mass of solute
(ii) Mass by volume percentage of a solution = × 100
Volume of solution
● Colloids are useful in industry and daily life. The particles are called dispersed phase and the medium in which they are distributed is called
dispersion medium.

Target Exercise
Elementary Level
1. Which of the following will show Tyndall effect? 3. A mixture of naphthalene and sodium chloride can be
(a) Salt solution (b) Milk separated by
(c) Copper sulphate solution (d) Potassium nitrate solution (a) steam distillation (b) fractional distillation
(c) filteration (d) sublimation
2. Which of the following is not a chemical change?
(a) Heating of wax 4. The formation of mixture
(b) Heating of hydrated CuSO 4 (a) releases energy
(c) Rusting of iron (b) absorb energy
(d) Burning of candle (c) does not involve any energy change
(d) None of the above
288 Study Package for NTSE

5. Which out of the following given options is not an 11. Solubility of a solute in a solvent to form a solution
element? (a) decreases with temperature
(a) Gold (b) Graphite (b) increases with temperature
(c) Silicon (d) Silica (c) shows no change
(d) None of the above
6. Which of the following is a compound?
(a) N2 (b) CO 2 (c) O 2 (d) H2 12. Chemical formula for hydrated copper sulphate is
(a) CuSO 4 ⋅ 5H2O (b) FeSO 4 ⋅ 7H2O
7. A liquid with soluble impurities is purified by (c) CuSO 4 ⋅ 7H2O (d) ZnSO 4 ⋅ 7H2O
(a) sublimation (b) crystallisation
(c) distillation (d) filteration 13. No new substance is formed in
(a) a physical change (b) a chemical change
8. Two immiscible liquids that have different density
(c) chemical reaction (d) None of the above
values can be separated by
(a) using separating funnel (b) distillation 14. Melting of ice is a ……… change.
(c) filteration (d) fractional distillation (a) chemical (b) physical
(c) Both (a) and (b) (d) None of these
9. NH4Cl shows the property of
(a) freezing (b) boiling 15. Which of the following is not a physical change?
(c) cooling (d) sublimation (a) Melting of wax
10. Air, a homogeneous mixture can be separated by (b) Stretching of a rubber band
(c) Digestion of food
(a) filteration (b) fractional distillation
(d) Grinding of wheat
(c) evaporation (d) distillation

High Skill Questions


1. Which of the following substances is essential to 9. Consider the following processes.
solute in saline water? 1. Rusting of iron
(a) Salicylic acid (b) Acetic acid 2. Electrolysis of water
(c) Sodium chloride (d) Ammonium chloride
3. Condensation of water
2. Which of these methods is used to extract alcohol 4. Heating of iron powder with water
from a mixture of alcohol and water?
Which of the above can be considered as chemical
(a) Distillation (b) Oxidation change?
(c) Crystallisation (d) Bleaching
(a) 1, 2 and 3 (b) 1, 2 and 4
3. What is the missing reactant? (c) 2, 3 and 4 (d) All of these
H2 O + → H 2 CO 3 10. When hydrogen and oxygen combined chemically in a
(a) CO (b) CO 2 (c) C (d) HCOOH fixed ratio of 1 : 8 by weight, X is obtained. X

4. An element which is not found in nature is 1. can be separated into its constituent by physical
methods.
(a) Pt (b) K (c) Zn (d) Pm
2. has properties quite different from hydrogen and
5. Which of the following is a compound? oxygen.
(a) Milk (b) Diamond 3. has fixed melting point.
(c) Marble (d) Air 4. is homogeneous.
6. Which of the following is not a homogeneous mixture?
Which of the above is/are considered as the
(a) A mixture of O 2 and N2 (b) Solution of salt in water characteristics of X?
(c) Brass (d) Milk
(a) 1 and 2 (b) 1, 2 and 3
7. Which substance is not a colloidal solution? (c) 1 and 4 (d) 2, 3 and 4
(a) Milk (b) Blood 11. Match the columns
(c) Muddy water (d) Soap solution
Column I Column II
8. Which type of solution doesn’t exhibit Tyndall effect?
(a) True solution A. Salt 1. Mixture
(b) Heterogeneous suspension B. Air 2. Contains only one type of atoms
(c) Colloidal solution C. Diamond 3. Compound
(d) None of the above D. Sodium 4. Lowest melting point
Is Matter Around us Pure 289
Select the correct answer for the given alternatives. 14. The composition of each new substance is always
Codes fixed for a
A B C D (a) mixture (b) compound
(a) 3 1 2 4 (c) element (d) None of these
(b) 1 2 3 4
(c) 2 1 3 4
15. There are ……… elements that are naturally
(d) 4 1 3 2 occurring.
(a) 72 (b) 92 (c) 100 (d) 112
12. Some solutions are given below
16. The element(s) that are liquid at room temperature
1. Milk churned with water
(a) Mercury (b) Bromine
2. Water containing mud (c) Both (a) and (b) (d) Gold
3. Blood
4. Starch solution
17. How many elements are in gaseous state at room
temperature?
Which of these show Tyndall’s effect? (a) 12 (b) 5 (c) 4 (d) 11
(a) Only 1 (b) 2 and 3
18. Malleability with respect to metals is the property
(c) 1 and 4 (d) All of these
(a) to be drawn into wires (b) to be drawn into sheets
13. Some processes are given below (c) to be drawn into circle (d) None of the above
1. Use of biogas as fuel 19. Purification of salt that we get from sea water is done
2. Burning of candle by
3. Mixing of iron and sulphur (a) filteration (b) crystallisation
4. Distillation of water (c) coding (d) decantation
Which of these are considered as only physical 20. The technique used to separate pigments from
change? natural colours is known as
(a) 1 and 2 (b) 2, 3 and 4 (a) sublimation (b) chromatography
(c) 2 and 4 (d) 3 and 4 (c) crystallisation (d) filteration

Hints and Solutions


Elementary Level
1. (b) Milk is a colloidal solution, that’s why it exhibits Tyndall effect, 4. (c) 5. (d)
i.e., scatter light passing through it.
6. (b) Compounds contains two or more elements in fixed
2. (a) When wax is heated, it melts but its chemical composition proportions. Thus, among the given only CO 2 is a compound.
remains unchanged. So, heating of wax is an example of
7. (c) 8. (a) 9. (d) 10. (b)
physical change.
11. (b) 12. (a) 13. (a) 14. (b)
3. (d) Naphthalene, being a sublimate, is separated from the non-
sublimate substance like sodium chloride by a process called 15. (c) Digestion of food is a chemical change because the
sublimation. chemical composition of food gets changed.

High Skill Questions


1. (c) Saline water is aqueous solution of sodium chloride. 13. (b) 14. (b) 15. (b)
2. (a) 16. (c) Mercury and bromine both are liquid elements one is a metal
(Hg) and the other is a non-metal (Br).
3. (b) H2O + CO 2 → H2CO 3
17. (d)
4. (d) 5. (c) 6. (d) 7. (c)
18. (b) Malleability is a property of metals by virtue of which metals
8. (a) 9. (b) 10. (d) 11. (a) can be drawn into sheets.
12. (d) All the solutions are examples of colloidal solutions. So, they 19. (b)
all show Tyndall effect which is the scattering of the light by 20. (b) Chromatography is the technique for separating pigments
colloidal particles. from natural colours.
Chapter

2
Atomic Structure
Atoms and molecules are the building block of matter. (i) Mass of proton = 1 u = 1.66 × 10−24 g
For a long time it was thought that, the atoms are (ii) Charge of a proton = 1.6 × 10−19C
indivisible but now these days atoms are divisible and (iii) Relative charge of a proton = + 1
they do have an inner structure.
l Indian philosopher Maharishi Kanad was the first man Neutron
to state that the matter is made up by Anu and
Paramanu. After the discovery of protons and electrons, it was noticed
l John Dalton gave the atomic theory and states that that all the mass of an atom cannot be accounted for, on
atoms are indivisible, which takes part in chemical the basis of only protons and electrons present in it, so
reaction. Chadwick (1932) suggested an another subatomic particle
l According to modern attempt, atoms are made up of three called neutron. The neutron is a neutral particle found in
subatomic particles : Electrons, Protons and Neutrons. the nucleus of an atom.
(i) Protons and neutrons are present in a small nucleus (i) Mass of a neutron = 1 u = 1.66 × 10−24 g
at the centre of the atom.
(ii) Charge of a neutron = 0
(ii) Electrons are outside the nucleus.
(iii) The atoms of different elements differ in the number Comparison between Proton, Neutron and Electron
of electrons, protons and neutrons.
Subatomic Relative Relative Location in the
S.No.
Electron particle mass charge atom

The existence of fundamental subatomic particle 1. Proton 1u +1 In the nucleus


electron was shown by JJ Thomson in 1897, by using a 2. Neutron 1u 0 In the nucleus
discharge tube and demonstrated the cathode ray 1
3. Electron u –1 Outside nucleus
experiment. Cathode rays consist of small, negatively 1840
charged particles called electrons.
1
(i) Mass of an electron = u = 9.11 × 10−28 g
1840 X-ray
(ii) Charge of an electron = −1.6 × 10−19 C
(iii) Relative charge of an electron = − 1 X-rays were discovered by WK Roentgen. X-rays are
(iv) Electron was named by Stoney produced when cathode rays strike with hard metals
like copper, tungsten etc. X-rays having a great
Proton penetrating power, so they can directly create shadows
The existence of protons in atoms was shown by of bones and doctors can easily trace the fractured part
E Goldstein by demonstrating anode ray experiment in of bone if any.
which a perforated cathode is used.
Atomic Structure 291
Radioactivity have positive charge whereas neutrons have no charge,
they are neutral. Due to the presence of equal number of
The phenomenon of spontaneous emission of invisible negative electrons and positive protons, the atom on the
radiations by certain substance is called radioactivity whole is electrically neutral.
and this substance is known as radioactive substance. l The protons and neutrons are located in a small nucleus
l Radioactivity was discovered by Henri Becquerel. at the centre of the atom. Due to the presence of protons,
l Po and Ra was discovered by Madam Curie. nucleus is positively charged.
l There are three types of radioactive radiations. These l The electrons revolve rapidly round the nucleus in fixed
are α-rays, β-rays and γ-rays. circular paths called energy levels or shells. The energy
l α-ray consist of positively charged He 2+ particles. Each levels or shells are represented in two ways; either by
particle has a mass of 4u. the numbers 1, 2, 3, 4, 5 and 6 or by the letters
K , L, M , N, O and P. The energy levels are counted
l β-rays are negatively charged and γ-rays are high energy
from the centre onwards.
electromagnetic radiations having no charge and
negligible mass. l There is a limit to the number of electrons which each
energy level (or shell) can hold. e.g., The first energy
level (or K shell) can hold a maximum of 2 electrons;
Thomson’s Model of Atom second energy level (or L shell) can hold a maximum of 8
electrons; third energy level (or M shell) can hold a
l An atom consists of a sphere (or ball) of positive charge
maximum of 18 electrons and fourth energy level (or N
with negatively charged electrons uniformly distributed
shell) can hold a maximum of 32 electrons.
in the entire atom.
l The positive and negative charges in an atom are equal l Each energy level (or shell) is associated with a fixed
in magnitude, due to which an atom is electrically amount of energy, the shell nearest to the nucleus
neutral. having minimum energy and the shell farthest from the
nucleus having the maximum energy.
Although this model of atom explained the neutrality of
the atom, but it could not explain the Rutherford’s l There is no change in the energy of electrons as long
experiment. as they keep revolving in the same energy level and
the atom remains stable. The change in the energy of
an electron takes place only when it jumps from a
Rutherford’s Experiment lower energy level to a higher energy level or when
it comes down from a higher energy level to a
On the basis of this experiment, nucleus was discovered by lower energy level.
Rutherford Rutherford’s model of an atom (or structure of
atom) can be described as follows Atomic Number The number of protons in one atom of
l An atom consists of a positively charged, dense and very an element is known as atomic number of that element.
small, nucleus containing all the protons and neutrons. Atomic number was discovered by Moseley.
Almost the entire mass of an atom is concentrated in the
nucleus. Mass Number The total number of protons and
l The nucleus is surrounded by negatively charged neutrons present in one atom of an element is known as
electrons. The electrons are revolving round the nucleus its mass number.
in circular paths called orbits. Mass number = Number of protons + Number of neutrons.
l Atom is electrically neutral due to same number of Mass number = Atomic number + Number of neutrons
protons and electrons in an atom. An atom may be represented as follows
l Most of the atom is empty space.
A major drawback of this model of the atom is that it Symbol
Mass number A
doesn’t explain the stability of the atom. Actually, of or X
according to the electromagnetic theory, if a charged Atomic number Z
Element
particles moves around positively charged nucleus then it
will lose energy continuously and finally drop into the
nucleus. Electronic Configuration of Elements The arrange-
ment of electrons in the various shells of an atom of the
Bohr’s Atomic Model element is known as electronic configuration of the
The present concept of an atom was given by Neils Bohr, element. Electronic configuration of an element can be
which can be described as follows written by Bohr-Bury scheme.
l An atom is made up of three particles : electrons, protons l The maximum number of electrons which can be
and neutrons. Electrons have negative charge, protons accommodated in any orbit is given by 2n 2.
292 Study Package for NTSE

l Electrons in an atom doesn’t occupy a new shell unless all the inner shells are completely filled with electrons.
Orbit Maximum Capacity Orbit Maximum Capacity
K (n = 1) 2 electrons M (n = 3) 18 electrons
L (n = 2 ) 8 electrons N(n = 4) 32 electrons

Electronic Configurations of First Twenty Elements


Electronic Electronic
Atomic Atomic
S.No. Element Symbol Configuration S.No. Element Symbol Configuration
Number Number
in K, L, M, N shells in K, L, M, N shells
1. Hydrogen H 1 1 11. Sodium Na 11 2, 8, 1
2. Helium He 2 2 12. Magnesium Mg 12 2, 8, 2
3. Lithium Li 3 2, 1 13. Aluminium Al 13 2, 8, 3
4. Beryllium Be 4 2, 2 14. Silicon Si 14 2, 8, 4
5. Boron B 5 2, 3 15. Phosphorus P 15 2, 8, 5
6. Carbon C 6 2, 4 16. Sulphur S 16 2, 8, 6
7. Nitrogen N 7 2, 5 17. Chlorine Cl 17 2, 8, 7
8. Oxygen O 8 2, 6 18. Argon Ar 18 2, 8, 8
9. Fluorine F 9 2, 7 19. Potassium K 19 2, 8, 8, 1
10. Neon Ne 10 2, 8 20. Calcium Ca 20 2, 8, 8, 2

Valence Electrons The electrons present in the outermost shell of an atom are known as valence electrons
because they decide the valency of an atom. Only valence electrons take part in the chemical reaction.
Isotopes Two or more form of an element having the same atomic number but different mass number. e.g.,
1 2 3
l 1 H (protium), 1 H (deuterium), 1 H (tritium)
35 37
l 17 Cl, 17 Cl Isotopes having different number of neutrons.

Points to Remember
● Isobars are atoms with same mass number, but different atomic number e.g., 14
6 C and
14
7 N.
● Isotones are atoms of different elements which contain the same number of neutrons. e.g., 15
7 N and
16
8 O.
2− − + 2+
● Isoelectronic species contain the same number of electrons, e.g., O , F , Na and Mg .
● e/m value is measured by Millikan in its oil drop experiment.
● Maximum number of electrons in an orbit can be calculated by using 2 n2 formula.

Target Exercise
Elementary Level
1. Number of electrons in sodium ion (Na+) and oxide 4. Who measured the e/m value of electron?
ion, (O 2− ) are (a) Dalton (b) Rutherford
(a) 8 (b) 9 (c) 10 (d) 11 (c) Goldstein (d) Millikan

2. Neutrons are present in all elements except 5. What is the charge on α particle?
(a) H (b) He (c) C (d) Ne (a) +2 (b) –2 (c) +1 (d) –1

3. An atom of an element has 15 electrons and has a 6. C-14 has a half-life period …… .
mass number 31. The nucleus of this atom (a) 5770 yr (b) 11520 yr (c) 2880 yr (d) 28750 yr
contains…… neutrons. 7. Which species of phosphorus is most stable?
(a) 31 (b) 16 (c) 46 (d) 15
(a) P (b) P 1 − (c) P2− (d) P3−
Atomic Structure 293
8. How many electrons are present in the outermost 15. Select the pair of atoms having the same number of
shell of aluminium ? electrons in their outermost shell.
(a) 0 (b) 1 (c) 2 (d) 3 (a) Na, Mg (b) N, S (c) Be, Al (d) C, Si

9. The shells or orbits are in atom denoted by 16. Which subatomic particle has negligible mass?
(a) K, L, M, N . . . (b) a, b, c, d , . . . (a) Electron (b) Proton (c) Neutron (d) Hydrogen
(c) A, B, C, D . . . (d) P, Q, R, S . . .
17. When alpha particles are bombarded on thin gold foil,
10. Electron revolve around the nucleus in definite most of the alpha particles pass through straight
circular paths called orbits. This was suggested by because
(a) Thomson (b) Bohr (a) α-particles are positively charged
(c) Dalton (d) Rutherford (b) α-particles have more mass than electron
11. In general, the nth shell can accommodate electrons, (c) nucleus is at the centre of the atom
where ‘n’ stands for the number of the shell. (d) most of the part of atom is empty
(a) 3n2 (b) 2 n2 (c) n2 (d) 2n 18. The relative atomic masses of some elements are not
whole numbers because of
12. Which of the following is the correct electronic
configuration of sodium? (a) accurate determination of atomic mass with respect to C-12
is difficult
(a) 2, 8, 1 (b) 8, 2, 1 (c) 2, 1, 8 (d) 12, 8, 2
(b) existence of isotopes of elements
13. Which rays can be used to detect the fractured bones (c) unstable nature
in our body ? (d) atoms may get ionised during atomic mass determination
(a) Canal rays (b) Cathode rays 19. The mass of neutron is nearly the same as that of
(c) X-rays (d) Cosmic rays
(a) electron (b) proton (c) atom (d) nucleus
14. The radioactive isotope of hydrogen is
20. Lithium has 3 electrons, 4 neutrons, 3 protons. Its
(a) protium (b) deuterium mass number is
(c) tritium (d) heavy water
(a) 3 (b) 4 (c) 7 (d) 10

High Skill Questions


13
1. 6 Cand 12
6 C differ from each other in respect to 2. Its valence electrons lies in N-shell
number of 3. 24 Cr 54 is isotonic with 25 Mn 60
(a) electrons (b) protons
(c) neutrons (d) None of the above The correct alternative/s is
(a) only 1 (b) 1 and 2
2. Match the columns (c) 2 and 3 (d) All of these
Column I Column II 5. Isoelectronic species are
A. Radioactivity 1. Chadwick (a) N3− , O 2 − (b) Na + , Ca 2 +
B. Electron 2. Roentgen (c) O 2− , C 2 − (d) K + , Na +
C. X-rays 3. Thomson
D. Neutron 4. Bacquerel 6. How many neutrons are present in tritium nucleus?
(a) 2 (b) 3
Codes (c) 1 (d) 0
A B C D
(a) 1 2 3 4 7. An atom contains 2 electrons in K shell, 8 electrons in
(b) 4 3 2 1 L shell and 4 electrons in M shell. The element is
(c) 3 4 2 1 (a) calcium (b) potassium
(d) 4 3 1 2 (c) silicon (d) magnesium
76
is isotonic with 8. How many valence electrons does calcium (atomic
3. 32Ge
(a) 77
(b) 77 number = 20) has?
32 Ge 33 C
(a) 3 (b) 1 (c) 10 (d) 2
77 78
(c) 34 C (d) 33 C
9. Which pair shows isobars?
4. The true statement(s) about chromium (atomic (a) 1H2 and 1H3 (b) 14 Si
30
and 15 P31
number = 24) is/are 40
(c) 18 Ar and 19 K 40 (d) 92 U
235
and 90 Th
231
1. It contains 24 neutrons
294 Study Package for NTSE

10. The number of electrons and neutrons in an element 13. The electronic configuration of phosphorus is
are 18 and 20 respectively. Its mass number is (a) 2, 4 (b) 2, 8, 4
(a) 22 (b) 32 (c) 38 (d) 42 (c) 2, 8, 5 (d) 2, 8, 1
11. The number of electrons in the nucleus of chlorine is 14. K + ion has ……… number of electrons.
(a) 17 (b) 11 (c) 9 (d) 0 (a) 18 (b) 17
(c) 20 (d) 19
12. Which is not true with respect to cathode rays?
(a) A stream of electrons 15. Which particle contains 2 neutrons and 1 proton?
4
(b) Charged particles beam (a) H2 (b) 2 He
(c) Moved with speed same as that of light (c) 1 T 3
(d) 2
1D
(d) Can be deflected by electrical field

Hints and Solutions


Elementary Level
1. (c) Na + = 11 − 1 = 10 electrons 9. (a) 10. (b) 11. (b) 12. (a)
O 2−
= 8 + 2 = 10 electrons 13. (c) 14. (c)
2. (a) 15. (d) C ans Si belongs to 14 group of periodic table so, having
same number of valence electrons i.e., 4.
3. (b) Number of neutrons = 31 − 15 = 16
16. (a) 17. (d) 18. (b) 19. (b)
4. (d) 5. (a) 6. (a) 7. (d)
20. (c) Mass number = 3 + 4 = 7
8. (d) Al13 = 2, 8, 3 Number of valence electrons = 3

High Skill Questions


1. (c) 7. (c) Silicon has atomic number 14
2. (b) 13
6 C atomic number = 6,. mass number = 13 So, electronic configuration

∴ Number of neutrons = 13 − 6 = 7 = 2, 8, 4
K, L, M
12
6 C atomic number = 6, mass number = 12
8. (d) Electronic configuration of calcium is 2, 8, 8, 2. So, 2 valence
∴ Number of neutrons = 12 − 6 = 6 electrons.
3. (b) Isotones have same neutron number, so they both have 44 9. (c)
neutrons 10. (c) Number of electrons = 18 = number of protons
76
32 Ge → 76 − 32 = 44 Number of neutrons = 20
So, mass number = number of protons + number of neutrons
77
33 C → 77 − 33 = 44
= 18 + 20 = 38
4. (a)
11. (a) 12. (c)
5. (a) N3− has 7 + 3 = 10 electrons
13. (c) Phosphorus has atomic number 15. So, the electronic
O 2− has 8 + 2 = 10 electrons configuration is 2, 8, 5.

So, they are isoelectronic because number of electrons is same. 14. (a) Atomic number of potassium (K) is 19. So, the number of
electrons for K + is 19 − 1 = 18.
6. (a)
15. (c)
Chapter

3
Chemical Reactions
and Equations
e.g.,
Chemical Equations l 2FeSO4( s ) → Fe 2O3( s ) + SO2( g ) + SO3( g )
The method of representing a chemical reaction with (Ferrous (Ferric oxide)
sulphate)
the help of symbols and formulae of the substances
involved in it, is known as a chemical equation and
l Silver chloride turns grey in sunlight. This is due to the
decomposition of silver chloride into silver and chlorine
unbalanced chemical equation are called skeleton by light.
equation. Sunlight
2AgCl( s ) → 2Ag( s ) + Cl2( g )
Balanced Chemical Equation 3. Displacement Reaction The reaction in which
The process of making the number of atoms of each a metal displaces a less reactive metal from its salt
element equal on both the sides of an equation is called solution, is called single displacement reaction.
balancing of an equation. Fe( s) + CuSO4 ( aq) → FeSO4 ( aq) + Cu( s)
e.g., 2Mg + O2 → 2MgO 4. Double Displacement Reaction A chemical
reaction in which two compounds in their aqueous
Types of Chemical Reactions solution react by exchanging their radicals is called
1. Combination Reaction A chemical reaction in a double displacement reaction.
which two or more substances combine to form a The reaction in which two compounds in their
single substance is called a combination reaction. aqueous solution react to form an insoluble salt (a
precipitate) as one of the products is known as the
CaO( s ) + H2O( l) → Ca(OH)2 ( aq)
(Quicklime)
precipitation reaction.
(Slaked lime)
Reactions in which heat is released along with the e.g., AgNO3 + HCl → AgCl ↓ + HNO3
Silver Silver chloride
formation of products are called exothermic nitrate (white precipitate)
chemical reactions.
5. Oxidation and Reduction Reaction which
CH4 ( g ) +2O2( g ) → CO2( g ) + 2H2O( g ) involves the addition of oxygen or the removal of
hydrogen is termed as oxidation.
2. Decomposition Reaction A reaction in which a
Reaction that involves the addition of hydrogen or the
substance breaks down into two or more substances
is called decomposition reaction. removal of oxygen is termed as reduction reaction.
296 Study Package for NTSE

The substance that can bring about oxidation is called Points to Remember
an oxidising agent while the substance which brings l A chemical reaction is a process in which substance (s) undergo a
about reduction is called reducing agent. change to produce a new substance (s) having different properties.
Reduction l Substances that take part in a chemical reaction are called reactants
Heat while substances that are formed in a chemical reaction is known as
e.g., CuO + H2 → Cu + H2 O products.
l Chemical equation is the method of representing chemical reaction
Oxidation in terms of symbols and formulae of substances.
Rancidity When fats and oils are oxidised, they
l Balancing of equation is the process of making the number of atoms
of each element equal on both the sides of an equation.
become rancid and their small and taste change, l Combination reaction, decomposition reaction, displacement
keeping food in air tight containers and adding reaction and double displacement reaction are the common types of
antioxidants to foods help to slow down oxidation. chemical reaction.

Target Exercise
Elementary Level
1. The substance that takes part in a chemical reaction is Codes
known as A B C D
(a) reactant (b) product (a) 3 4 1 2
(c) element (d) None of these (b) 1 2 3 4
(c) 2 1 4 3
2. In the chemical reaction iron powder + sulphur → iron (d) 4 3 2 1
sulphide, the product is
6. The colour of copper oxide is
(a) iron powder (b) sulphur
(a) green (b) black
(c) iron sulphide (d) All of these
(c) blue (d) white
3. Chemical change is generally
7. In exothermic reaction, heat is
(a) reversible (b) irreversible
(a) released (b) required
(c) Both (a) and (b) (d) None of these
(c) absorbed (d) None of these
4. If the number of atoms of each element is equal on both
sides of an equation then the equation is known as 8. The precipitate formed when copper sulphate reacts
(a) skeletel equation
with hydrogen sulphide
(b) balanced equation (a) H2SO 4 (b) H2O
(c) unbalanced equation (c) CuS (d) H2SO 3
(d) Both (a) and (b) 9. Carbon burns with oxygen to form
5. Match the column I and column II. (a) carbon dioxide
(b) carbon trioxide
Column I Column II (c) carbon salt
(d) None of the above
A. Product(s) 1. A + B → AB
B. Reactant(s) 2. AB → A + B 10. Calcium oxide and carbon dioxide is formed due to the
C. Combination 3. Right side of a chemical equation decomposition of
reaction (a) calcium sulphide
D. Decomposition 4. Left side of a chemical equation (b) calcium chloride
reaction (c) calcium carbonate
(d) calcium bicarbonate
Chemical Reactions and Equations 297

High Skill Questions


1. Fe2O3 + 2Al → Al2O3 + 2Fe 8. When a copper vessel is exposed to moist air for long
The above reaction is an example of a time, it acquires a dull green coating. This is due to
(a) combination reaction the
(b) decomposition reaction (a) formation of copper oxide
(c) displacement reaction (b) formation of copper sulphide
(d) double displacement reaction (c) formation of copper carbonate and copper hydroxide
(d) formation of copper nitrate
2. Which of the statements about the reaction below are
incorrect? 9. Calcium hydroxide reacts slowly with the carbon
dioxide in air to form a thin layer of
2PbO( s) + C( s) → 2Pb( s) + CO2 ( g) (a) calcium bicarbonate (b) calcium oxide
1. Lead is getting reduced. (c) calcium carbonate (d) None of these
2. Carbon dioxide is getting oxidised.
10. Lead nitrate on thermal decomposition gives lead
3. Carbon is getting oxidised.
oxide, ……… and ……… .
4. Lead oxide is getting reduced.
(a) NO 2 , H2O (b) NO, NO 2
(a) 1 and 2 (b) 1 and 3
(c) NO 2 , O 2 (d) None of these
(c) 1, 2 and 3 (d) All of these
11. Foods containing fats and oil usually contain ……… to
3. On burning magnesium in air we get
prevent oxidation.
(a) MgO (b) MgCO 3
(a) moisture (b) autooxidants
(c) MgSO 4 (d) Mg(NO 3 )2
(c) antacids (d) medicines
4. The metal that can displace Fe from its salt is 12. Which of the following is not a chemical change?
(a) Hg (b) Au
(a) Heating of wax
(c) Ag (d) Zn
(b) Rusting of iron
5. In the reaction,Mg + 2HCl → MgCl2 + H2 magnesium (c) Heating of limestone
acts as (d) Burning of candle
(a) an oxidising agent 13. Which of the following is a reversible reaction?
(b) a reducing agent (a) CaCO 3 + 2HCl → CaCl 2 + H2O + CO 2
(c) oxidising as well as reducing agent
(b) 2 Mg + O 2 → 2MgO
(d) None of the above
(c) N2 + 3H2 → 2NH3
6. When iron reacts with copper sulphate to form ferrous (d) Fe + CuSO 4 → FeSO 4 + Cu
sulphate and copper in this reaction change of colour
occurs from 14. Calcium oxide is commonly named as
(a) white to blue (b) blue to white (a) limestone (b) quicklime
(c) blue to pale green (d) white to black (c) slaked lime (d) None of these

7. Redox reactions are useful in 15. Silver chloride turns grey in sunlight because of its
(a) extraction of metals decomposition into
(b) bleaching industry (a) silver oxide
(c) dyeing industry (b) silver and chlorine
(d) All of the above (c) chlorine
(d) None of the above
Hints and Solutions
Elementary Level
1. (a) 8. (c) CuSO 4 + H2S → CuS + H2SO 4
Copper Hydrogen Copper Sulphuric
2. (c) The substance which is obtained by the reaction of reactants sulphate sulphide sulphide acid
and is written on right side of arrow is called product. Thus, iron
sulphide is the product of the reaction. 9. (a) C + O2 → CO 2
Carbon Oxygen Carbon dioxide
3. (b) 4. (b) 5. (a)
10. (c) CaCO 3 → CaO + CO 2
6. (b) 7. (a) Calcium Calcium Carbon
carbonate oxide dioxide

High Skill Questions


1. (c) This is displacement reaction because aluminium is 9. (c) Ca(OH)2 + CO 2 → CaCO 3 + H2O
displacing Fe from Fe 2O 3 . Calcium
carbonate
2. (b)
Heating
3. (a) 2Mg + O 2 → 2MgO 10. (b) 2Pb(NO 3 )2 → 2 PbO + 4 NO 2 + O 2
air Lead nitrate
4. (d) Zn, because it is above Fe in the reactivity series and Au, Ag 11. (b)
and Hg are below Fe in the series. 12. (a) Heating of wax is a physical change because only the state is
5. (b) It is acting as a reducing agent. changing from solid to liquid.
6. (c) 13. (c) N2 + 3H2 j 2NH 3 is a reversible reaction.
7. (d) 14. (b)
8. (c) CuCO 3 ⋅ Cu(OH)2 , this is green in colour. hν
15. (b) 2AgCl  
→ 2Ag + Cl 2
Chapter

4
Acids, Bases and Salts
Electrolytes
Chemical substances decompose into the constituent l Acids react with metal to form hydrogen gas.
ions in solutions or in molten state and allow flow of Metal + Acid → Salt + Hydrogen gas
current. They include salts, acids and bases. Zn( s ) + H2 SO4 ( aq ) → ZnSO4 ( aq ) + H2 ↑
All electrolytes can be divided into two groups l Acids react bases or alkalis to form salt and water.
1. Strong Electrolytes Electrolyte which decomposes Acid + Base → Salt + Water
completely almost 100% into the constituent ions in HCl + NaOH → NaCl + H2 O
solutions or in molten state are called strong Acid Base Salt Water
electrolytes. Virtually all salts, strong acids and l Acids have corrosive nature because they causes severe
strong bases are strong electrolytes, e.g., NaCl, burns on the skin and eat up materials.
KCl, HCl, H2SO4, HNO3, KOH, Ba(OH)2 etc. l Acids are never stored in metal containers because they
2. Weak Electrolytes Electrolyte which decomposes gradually corrode and eat up the metal container.
partially into constituent ions are known as weak
Name of acid Place where found
electrolytes. Weak electrolytes are organic acids or
weak bases. e.g., CH3COOH, H2CO3, NH4OH, H2S Acetic acid (ethanoic acid) Vinegar
Formic acid (methanoic acid) Red ant’s
Citric acid Citrus fruits such as lemons
Acids Hydrochloric acid Human stomach
Lactic acid Sour milk
l Acids are those chemical substances which have a sour
taste. Acids are those chemical substances which change
l Acids react with metal carbonates and metal hydrogen
the colour of blue litmus to red.
carbonates to liberate carbon dioxide.
l The acids present in plant materials and animals are
Na 2 CO 3 + 2HCl → 2NaCl + H 2 O + CO2 ↑
called organic acids.
NaHCO 3 + HCl → NaCl + H2 O + CO2 ↑
l The acids prepared from minerals of the Earth are called
mineral acids. Mineral acids are man made acids. e.g., On passing the carbon dioxide gas through lime
HCl, H2SO4, HNO3 etc. water, a white precipitate is formed (lime water
turns milky).
l All the acids can be divided into two groups.
Strong acids and weak acids Ca(OH)2( aq) + CO2( g ) → CaCO3( s ) + H2O( l)
(Lime water) (White precipitate)
(a) HCl, H2SO4, HNO3 are strong acids.
On passing excess CO2 the following reaction takes
(b) H2CO3, CH3COOH, HCOOH and citric acid are place
weak acids.
CaCO3 + H2O + CO2 → Ca(HCO3 )2( aq)
l Concentrated and Dilute acids A concentrated (Soluble in water)
acid is one which contains the minimum possible
l Acids react with metallic oxides to form salt and water
amount of water in it, while a dilute acid is one which
because metallic oxides are basic oxides.
contains much more of water in it.
300 Study Package for NTSE

Bases
l Bases are those chemical substances which have a bitter taste. Bases changes the colour of red litmus to blue.
l A base which is soluble in water is called an alkali e.g., KOH, NH4OH.
l Strong bases NaOH, KOH, Weak bases NH4OH, Ca(OH)2 and Mg(OH)2.
l Bases feel soapy on touching it, Bases react with acids to form salts and water.
l Bases react with non-metal oxides to form salt and water, e.g.,
Ca(OH)2 + CO2 → CaCO3 + H2O
l Insoluble bases in water are CuO, ZnO, FeO etc.

Arrhenius Theory
Arrhenius acids are the chemical substances that produces hydrogen ions or hydronium ion (H 3O + ), when mixed
with water and bases are substances that produces OH − , i.e., hydroxyl ion when dissolved in water.
H O
e.g., HNO 3 →
2
H + ( aq) + NO 3− ( aq)
or HNO 3 + H2 O → H 3O + + NO 3−
HNO 3 is an acid.
Similarly, NaOH is a base.
NaOH( aq) → Na + ( aq) + OH − ( aq)

pH Scale
l In 1909, Sorenson devised a scale known as pH scale, which represent the strength of acid or base.
l pH of a solution is inversely proportional to the concentration of hydrogen ion concentration.
 1   1 
pH = − log [H+ ] = + log or similarly pOH = log
 H  OH 
+ −

pH + pOH = 14
Neutral
These pH values are for acids These pH values are for bases (alkalis)

pH 0 1 2 3 4 5 6 7 8 9 10 11 12 13 14
Weak base Strong base
Strong acid Weak acid or or
Weak alkali Strong alkali
Increasing basic nature
Increasing acidic nature (or Increasing alkaline nature)

Increasing H+ ion concentration Increasing OH+ ion concentration


(Decreasing OH– ion concentration) (Decreasing H+ ion concentration)
pH scale

The pH values of some of the common substances from our everyday life are given below
pH Values of Some Common Substances

S.No. Solution pH S.No. Solution pH


1. Gastric juices (digestive juices in stomach) 1.4 6. Soft drinks 6.0
2. Lemon juice 2.5 7. Milk 6.5
3. Vinegar 4.0 8. Pure distilled water 7.0
4. Tomato juice 4.1 9. Blood 7.4
5. Coffee 5.0 10. Toothpaste 8.0
Acids, Bases and Salts 301
Sodium Hydroxide (NaOH) is prepared by chloro alkali
Salts process.
A salt is a compound made from an acid, when a metal 2NaCl( aq) + 2H2 O( l) →
takes the place of hydrogen in the acid. Salts can be
formed by neutralisation of an acid by a base. Salts are 2NaOH( aq) + Cl 2 ( g) + H2 ( g)
usually soluble in water. They are solids with high Bleaching Powder ( CaOCl2 ) Bleaching powder is
melting point and high boiling point. Aqueous solutions produced by the action of chlorine on dry slaked lime
of salts conduct electricity. Ca(OH) 2 + Cl 2 → CaOCl 2 + H2 O
Neutral Salts, e.g., NaCl are formed by neutralisation Bleaching powder

of a strong acid and a strong base. On hydrolysis, the It is used for bleaching purposes, as an oxidising agent
solution of NaCl remains neutral. and as disinfectant for drinking water.
Acidic Salts, e.g., NH4 Cl are formed by neutralisation of Baking Soda (NaHCO 3 ) The chemical name is sodium
a weak base by a strong acid. On hydrolysis, NH4 Cl gives
an acidic solution. hydrogen carbonate (NaHCO 3 ).

Basic Salt, e.g., CH 3COONa (salt of NaOH and NaCl + H2 O + CO2 + NH 3 → NH4 Cl + NaHCO 3
CH 3COOH) on hydrolysis gives a basic solution. When it is heated during cooking the following reaction
Sodium chloride is used as table salt, for tanning of takes place
leather, manufacture of washing soda, baking soda and Heat
2NaHCO 3 → Na 2 CO 3 + H2 O + CO2
sodium hydroxide.
Baking soda is used for making baking powder, antacids
Hydrated Salts Hydrated salts are salts with water in
and in fire extinguishers (soda-acid type).
them. Their general formulae are
(i) Blue vitriol CuSO4 ⋅ 5H2O Washing Soda (Na 2 CO 3 ⋅ 10 H2 O) Washing soda is
(ii) Green vitriol FeSO4 ⋅ 7H2O used in glass, soaps and paper industries, to make
(iii) Gypsum CaSO4 ⋅ 2H2O
borax, as a cleansing agent and for remaining
1 permanent hardness of water.
(iv) Plaster of Paris CaSO4 ⋅ H2O
2 Na 2 CO 3 + 10H2 O → Na 2 CO 3 ⋅ 10H2 O
Plaster of Paris CaSO4 ⋅ H2 O It is used by doctors
(v) Epsom MgSO4 ⋅ 7H2O 1
 2 
Chemicals from Common Salt The common salt is an
important raw material for various materials of daily as plaster for supporting fractured bones in the right
use like NaOH, baking soda, washing soda, bleaching position.
powder and many more. 373 K 1 1
CaSO4 ⋅ 2H2 O → CaSO4 ⋅ H2 O + 1 H2 O
Gypsum 2 2
Plaster of Paris

Points to Remember
● At 25ºC acid cannot have pH > 7 in any diluted state.
● Basicity corresponds to number of replaceable hydrogen atoms in acid.
● Acidity corresponds to number of replaceable hydroxyl ions in a base.
● Water of crystallisation is the fixed number of water molecules chemically attached to each formula unit of a salt in its crystalline form.
Target Exercise
Elementary Level
1. Which of the following acid is present in sour milk? 11. Bases turn red litmus into …… .
(a) Tartaric acid (b) Citric acid (a) black (b) blue
(c) Acetic acid (d) Lactic acid (c) yellow (d) orange
2. Which of the following is not an organic acid? 12. Which of the following is a dibasic acid?
(a) H2CO 3 (b) CH3COOH (a) H3PO 4 (b) HCl
(c) HCOOH (d) HCl (c) H2SO 4 (d) HNO 3
3. Which of the following acid is used in inverter 13. Which gas will be evolved when metal is reacted with
batteries? some mineral acid?
(a) Nitric acid (HNO 3 ) (a) H2 (b) O 2
(b) Hydrochloric acid (HCl) (c) CO 2 (d) N2
(c) Sulphuric acid (H2SO 4 )
(d) Acetic acid (CH3COOH) 14. Which acid is present in red ant?
(a) Formic acid (b) HCl
4. Acids are always stored in containers made of
(c) Citric acid (d) CH3COOH
(a) plastic (b) glass (c) metals (d) clay
15. Which type of substances feel soapy on touching it?
5. Acids turn blue litmus into …… .
(a) Acids (b) Bases
(a) green (b) red (c) Salts (d) None of these
(c) yellow (d) orange
16. Which of the following acid is present in vinegar?
6. Which of the following is a weak acid?
(a) Lactic acid (b) Malic acid
(a) CH3COOH (b) HCl (c) Acetic acid (d) Tartaric acid
(c) HNO 3 (d) H2SO 4
17. Which of the following acid is used in fire
7. Which of the following is a chemical formula of
extinguishers?
gypsum?
1 (a) Hydrochloric acid (b) Sulphuric acid
(a) CaSO 4 ⋅ 2H2O (b) CaSO 4 ⋅ H2O (c) Nitric acid (d) Tartaric acid
2
(c) FeSO 4 ⋅ 7H2O (d) CuSO 4 ⋅ 5H2O 18. When SO2 dissolves in water the acid formed is
8. Who proposed pH scale? (a) sulphuric acid (b) sulphurous acid
(c) sulphonic acid (d) None of these
(a) Ostwald (b) Sorenson
(c) Mendeleef (d) Bohr 19. Soft drinks like pepsi/cola contain
9. A water soluble base is known as …… . (a) CH3COOH (b) H2CO 3
(c) HNO 3 (d) H2SO 4
(a) basic (b) alkali (c) acidic (d) salty
20. Which of the following is a hydrated salt?
10. Which base is insoluble in water?
(a) Ca(OH)2 (b) MgCO 3
(a) CuO (b) NH4OH
(c) MgSO 4 ⋅ 7H2O (d) BaCO 3
(c) NaOH (d) KOH

High Skill Questions


1. Which of the following types of medicines is used for 3. The taste of an acid is
treating indigestion? (a) salty (b) sour (c) sweet (d) bitter
(a) Antibiotics (b) Analgesics 4. Which of the following is not a natural acid?
(c) Antacids (d) Antiseptic
(a) Citric acid (b) Acetic acid
2. A solution reacts with caused egg shells to give a gas (c) Sulphuric acid (d) Lactic acid
that turns lime water milky the solution contains
5. The acid used in batteries of cars and trucks is
(a) NaCl (b) LiCl
(c) HCl (d) KCl (a) nitric acid (b) sulphuric acid
(c) hydrochloric acid (d) tartaric acid
Acids, Bases and Salts 303
6. If the pH of a solution is 10 the given solution is 12. The chemical compound used by doctors as plaster for
(a) acidic (b) neutral supporting fractured bones, so as to put them in right
(c) basic (d) None of these place is
1
7. By dissolving metal oxide in water we get (a) MgSO 4 ⋅ 2H2O (b) MgSO 4 ⋅ H2O
2
(a) acid (b) salt 1
(c) CaSO 4 ⋅ 2H2O (d) CaSO 4 ⋅ H2O
(c) base (d) None of these 2
8. Which of the following is a strong base? 13. When hydrated CuSO4 is heated
(a) it becomes colourless (b) it becomes hydrous
(a) Mg(OH)2 (b) Ca(OH)2
(c) it gets melted into liquid (d) None of these
(c) KOH (d) NH4OH
14. Some statements related to bases are given below
9. Which of the following is a neutralisation reaction? 1. Bases change the colour of blue litmus red.
(a) NaOH + HCl → NaCl + H2O 2. Base may be soluble or insoluble.
(b) Na 2O + H2O → 2NaOH 3. All bases are alkali, but all alkalies are not base.
(c) SO 3 + H2O → H2SO 4 The correct statements are
(d) 2Mg + O 2 → 2MgO
(a) 1 and 2 (b) Only 2
10. Which of the following salt is used as an antacid? (c) Only 3 (d) 1, 2 and 3
(a) Potassium nitrate (b) Copper sulphate 15. Match the following.
(c) Sodium bicarbonate (d) Potash alum A. Aqueous solution of NaHCO3
11. Match the column I with column I. B. Aqueous solution of common salt
C. Hydrogen chloride gas
Column I Column II D. Aqueous solution of ammonium chloride
A. Acid 1. NaOH 1. does not affect dry litmus
B. Base 2. Pb(OH)Cl 2. turns blue litmus red
C. Neutral salt 3. HCl 3. aqueous solution is neutral
D. Basic salt 4. Na 2SO 4 4. turns red litmus blue

Codes Which of the following show correct matching?


A B C D A B C D A B C D A B C D
(a) 1 2 3 4 (b) 4 3 2 1 (a) 4 3 1 2 (b) 3 4 2 1
(c) 3 1 4 2 (d) 2 4 1 3 (c) 4 3 2 1 (d) 1 2 3 4

Hints and Solutions


Elementary Level
1. (d) 2. (d) 3. (c) 4. (a) 5. (b) 14. (a) Formic acid (HCOOH)
− +
6. (a) CH3COOH is partially ionized into CH3COO and H . 15. (b) 16. (c) 17. (b) 18. (b)
7. (a) 8. (b) 9. (b) 10. (a) 11. (b) 19. (b) H2O + CO 2 → H2CO 3
(Carbonic acid)
12. (c) H2SO 4 → 2H+ + SO 24 − (Dibasic)
13. (a) Zn + HCl → ZnCl 2 + H2 ↑ 20. (c) MgSO 4 ⋅ 7H2O having seven water molecules as hydrate.

High Skill Questions


1. (b) 7. (c) CaO + H2O → Ca(OH)2
(Metal oxide) (Base)
2. (c) Crushed egg shells have CaCO 3
8. (c)
(calcium carbonate)
9. (a) Neutralisation reaction is when acid and base react to give
CaCO 3 + 2HCl → CaCl 2 + H2O + CO 2 salt and water.
So, CO 2 gas is given out and it turns lime water milky. NaOH + HCl → NaCl + H2O
3. (b) 10. (c) 11. (c)
1
4. (c) H2SO 4 (sulphuric acid) is a mineral acid. 12. (d) Plaster of Paris → CaSO 4 ⋅ H2O
2
5. (b) 6. (c) 13. (a) 14. (b) 15. (a)
Chapter

5
Metals and Non-Metals
All the materials found in nature are made up of chemical elements. There are 118 elements known so far, 92
elements are naturally occurring elements and rest are man made or synthetic. 92 elements are classified into 72
metals and 20 non- metals by Lavosier.
Classification of Metals

Types of metals Symbols of metals


Heavy metals Hg, Pb, Ag, Pt, Cu
Light metals K, Mg, Na, Al
Noble metals Au, Ag, Pt
Rare metals Li, Be, Sr
Radioactive metals U, Th, Ra
Protective metals Cr, Ni, Zn, Sn

Differences between Metals and Non-Metals


Property Metals Non-metals
Electronic configuration Have 1, 2 or 3 electrons in their Have 4 to 8 electrons in their outermost orbit.
outermost orbit.
Physical state Generally solid (except mercury which Solids or gases (exception to bromine in liquid state).
exist in liquid state).
Lustre Lustrous and can be polished. Non lustrous except iodine.
Hardness Very hard (due to strong inter molecular Brittle (weak forces of attraction).
attraction forces).
Density High density. Low density
Malleability High. Low
Ductility High. Low
Conductivity Good conductor of heat and electricity. Poor conductor of heat and electricity.
Nature of oxide Basic. Acidic
Electropositive or Electronegative These are electropositive. These are electronegative.
character
Displacement of hydrogen from Displaces hydrogen. Doesn’t displace.
weak acids
Reaction with chlorine Reacts to form ionic chlorides. Reacts to form covalent chlorides.
Reaction with hydrogen Form electrovalent hydride. Stable covalent hydride.
Oxidising and reducing properties Reducing agent. Oxidising agent.
Metals and Non-Metals 305
Strategic Metals Metals and their alloys are Reaction of Metals and
important for the atomic energy, space science project
and in jet engines. Examples are titanium (Ti), Non-Metals
zirconium (Zr). Reaction of metals and non-metals can be explained by
taking the example of formation of sodium chloride.
Important Facts about Metals and Non-Metals Sodium loses one electron and forms sodium cation
l Most abundant element on Earth Oxygen (Na + ). On the other hand chlorine has seven electrons in
l Most abundant metal Aluminium the outer shell, it accepts one electron to complete its
l Liquid metal Mercury octet and hence forms chloride anion (Cl − ).
Lustrous non-metal Iodine
Opposite charges attract each other, so Na + and Cl −
l

l Hard non-metal Diamond


l Soft metals Sodium, Potassium ions are attracted to each other and held by strong
l Best ductile metals Gold, Silver electrostatic forces called as anion bond.
l Best conductor metal Silver Na → Na + + e − Na Cl
Noble metal Platinum 2, 8, 1 2, 8
− −
l

l Metal kept in paraffin wax Lithium


Cl + e → Cl Formation of NaCl
2, 8, 7 2, 8, 8
l Non-metal conductor Graphite Thus, ionic compounds are formed by the transfer of
l Coinage metals Ag, Cu, Au, Al electrons from metals to non-metals. The ionic
l Metal kept under kerosene Sodium
compounds are solids, have high melting and boiling
l Heaviest metal Osmium (Os)
points, are generally soluble in water and conduct
l Solid non-metals Carbon, Iodine, Sulphur
electricity in the molten or aqueous state.
l Liquid non-metal Bromine
l
Most electropositive metal Caesium (Cs)
Reactivity Series of Metals
The arrangement of metals in vertical column in order
Chemical Reactions of Metals of decreasing reactivities is called reactivity series.
l Reaction with oxygen Reactivity Series of Metals
4Na + O2 → 2Na 2O Potassium K (Most reactive)
2Mg + O2 → 2MgO Sodium Na
Barium Ba
4Al + 3O2 → 2Al2O3
Calcium Ca
l Reaction with water Magnesium Mg

Decreasing chemical reactivity


2Na + 2H2O → 2NaOH + H2 ↑ Aluminium Al
Zinc Zn Metals more reactive
Mg + H2O → MgO + H2 ↑ than hydrogen
Iron Fe
l Reaction with dilute acids Cobalt Co
2Na + 2HCl → 2NaCl + H2 ↑ Nickel Ni
Mg + 2HCl → MgCl2 + H2 ↑ Tin Sn
Lead Pb
l Reaction with salt solutions Hydrogen H
CuSO4 + Fe → FeSO4 + Cu Copper Cu
2AgNO3 + Cu → Cu(NO3 )2 + 2Ag Mercury Hg
Silver Ag Metals less reactive
l Reaction with chlorine than hydrogen
Gold Au
2Na + Cl2 → 2NaCl Platinum Pt
(Least reactive)
Ca + Cl2 → CaCl2
2Fe + 3Cl2 → 2FeCl3
Applications of Reactivity Series
l Reaction with hydrogen
l Metals at the top of reactivity table doesn’t become
2Na + H2 → 2NaH available in nature in free state.
Mg + H2 → MgH2 l A metal placed higher in the series is more metallic than
that below in the series.
306 Study Package for NTSE

l All metals placed above hydrogen can displace hydrogen


from its compounds such as water and acids to form
Minerals and Ores
hydrogen gas. The various compounds of metals which occur in the
l A metal placed in the above reactivity series can displace Earth’s crust and are obtained by mining are called
the less reactive metal from its salt solution. minerals.
Those minerals from which the metal can be extracted
Alloys profitably and conveniently are called ores. All ores are
minerals but all minerals are not ores.
An alloy is a homogeneous mixture of two or more
metals or a metal with a non-metal.
Metallurgy
Importance of making alloys
(i) To increase resistance to corrosion. Branch of science, which deals with the extraction of
(ii) To increase hardness. metals from their ores, is known as metallurgy.
(iii) To increase tensile strength. Some Important Metals and their Ores
(iv) To modify chemical reactivity.
S. No. Metal Ore Chemical formula
S.No. Name of alloy % Composition Applications 1. Copper Copper pyrites CuFeS2
1. Brass Cu—60-80% and Utensils, pipes, nut 2. Silver Argentite Ag 2S
Zn—40-20% and bolts 3. Zinc Zinc blend ZnS
4. Lead Galena PbS
2. Bronze Cu—70-90% and Statues, coins
Sn—30-10% 5. Aluminium Bauxite Al 2O 3 ⋅ 2H2O
6. Iron Haematite Fe 2O 3
3. Stainless steel Fe—80.6%, Utensils,
Cr—18%, Ni—1%, automobile, parts,
C—0.4% knives of blades
Based on the reactivity metals are grouped into three
categories
4. Duralumin Al—95%, Cu—4%, Air craft bodies,
Mg—0.5%, rockets l Metals of low reactivity
Mn—0.5% l Metals of medium reactivity
l Metals of high reactivity
Amalgam During metallurgy purification of ore is the first step
It is a special class of alloy in which one of the carried.
component of alloy is mercury (Hg). Metals low in the activity series e. g., Mercury can be
e. g., Sodium amalgam, (Na-Hg), Zinc amalgam (Zn-Hg). extracted by heating its oxide.
Heat
2HgS + 3O2 → 2HgO + 2SO2
Corrosion Heat
It can be defined as the gradual eating away of a metal 2HgO → 2Hg + O2 ↑
by chemical reaction with its environment consisting of Metals in the middle of activity series like zinc, iron,
oxygen, water vapour, acids and other chemicals. lead, copper etc., are extracted as follows.
The following two conditions are necessary for corrosion. Heat
Roasting 2ZnS + 3O2 → 2ZnO + 2SO2 ↑
l Presence of air.
Heat
l Presence of water or moisture. Calcination ZnCO 3 → ZnO + CO2 ↑
(i) Corrosion of iron ZnO + C → Zn + CO ↑
4Fe + 3O2 + 3H2O → Fe 2O3 + 2 Fe(OH)3
Rust
Metals towards the top of the activity series like sodium,
(ii) Corrosion of aluminium magnesium, calcium, etc., are obtained by electrolytic
4Al + 3O2 → 2Al2O3
reduction.
Methods for Prevention of Corrosion e. g., Sodium is obtained by the electrolysis of molten
(i) Painting of surface of metal. sodium chloride.
(ii) By using oil or greases.
At cathode Na + + e − → Na
(iii) Galvanisation.
At anode 2Cl − → Cl 2 + 2e −
(iv) Electroplating.
(v) Alloying. The last step in metallurgy is purification of metals
known as refining of metals.
Metals and Non-Metals 307
Metallurgy Process (the chart for metallurgy process) Points to Remember
Ore l Reactivity or activity series of metals can be reported as
Potassium > Sodium > Barium > Calcium > Magnesium > Aluminiu
Powdered ore m > Zinc > Iron > Copper > Mercury > Silver > Gold.
Froth floatation process l Aluminium foils are used to wrap chocolates, cigarettes, medicines
Liquation
Magnetic separation method and food stuff.
Leaching l The black material inside a lead pencil contains graphite, a non-metal
Concentrated and not the metal lead.
ore l Displacement reactions may be useful in determining the relative
Calcination
Roasting reactivities of metals. A more reactive metal displaces a less reactive
metal from its solution.
Conversion into
metal oxide Reduction by coke l Of the 21 non-metals, 11 non-metals are in the gaseous state, 1 is in
Reduction by metal the liquid state and the rest are in the solid state, at room
Electrolytic reduction temperature.
Crude metal Zone refining l The reaction of iron(III) oxide (Fe2O3) with aluminium is used to join
Liquation railway tracks and is known as thermite reaction.
van Arkel method
Pure metal Electrolytic refining Fe2O3(s) + 2Al(s) → 2Fe(l) + Al2O3(s) + Heat

Target Exercise
Elementary Level
1. Name of the first synthetic element is ……… 9. Who classified the elements in metals and
(a) U (b) Zn (c) Tc (d) Am non-metals?
(a) Lavoisier (b) Mendeleef
2. The most impure form of iron is
(c) Rutherford (d) Dalton
(a) steel (b) stainless steel
(c) cast iron (d) wrought iron 10. Which metal can be cut by a knife?
(a) Silver (b) Sodium
3. Coinage metals are
(c) Zinc (d) Aluminium
(a) Ca, Ag, Ni (b) Cu, Ag, Au
(c) Zn, Na, K (d) Cu, Zn, Cs 11. Which of the following element is a metalloid ?
(a) Lead (b) Tin
4. An essential constituent of amalgam is
(c) Antimony (d) Barium
(a) mercury (b) silver
(c) zinc (d) aluminium 12. The non-metal which is used to make electrodes of
dry cells
5. Metal M occurs in the Earth’s crust as its oxide M2O 3.
(a) carbon (b) silicon
An alloy of this metal is used in making aircrafts. The
(c) chlorine (d) oxygen
ore of the metal M is
(a) magnetite (b) haematite 13. Which metal reflects about 90% of light falling on it?
(c) bauxite (d) pyrolusite (a) Gold (b) Silver
(c) Sodium (d) Aluminium
6. Which is the purest form of gold?
(a) 24 carat (b) 14 carat 14. Which metal is the best conductor of heat and
(c) 20 carat (d) 10 carat electricity?
(a) Silver (b) Gold
7. Stainless steel is an alloy of iron, carbon and
(c) Sodium (d) Copper
(a) chromium
(b) nickel 15. Non-metal used for making the tip of match stick
(c) chromium and nickel (a) phosphorus (b) carbon
(d) copper and silver (c) silicon (d) nitrogen
8. Which was the first metal discovered by primitive 16. Which metal is used in thermometer?
man? (a) Na (b) Cs
(a) Fe (b) Cu (c) Pb (d) Ag (c) Hg (d) Ca
308 Study Package for NTSE

17. The metal present in haemoglobin is 19. Non-metals which occur in a solid state at room
(a) Na (b) Mg temperature are
(c) Fe (d) Ca (a) carbon and sulphur (b) bromine and iodine
(c) iodine and helium (d) nitrogen and phosphorus
18. The thin foils of which metal are used to decorate
sweets? 20. Complete the chemical reaction
(a) Aluminium (b) Silver 2Al + 6H2O → 2 . . . + 3H2
(c) Zinc (d) Copper (a) Al(OH)3 (b) Al 2O 3
(c) Al(OH)3 ⋅ H2O (d) Al 2O 3 ⋅ 2H2O

High Skill Questions


1. An element reacts with oxygen to give a compound 10. Which of the following is correct reactivity series?
with a high melting point. This compound is also (a) K, Na, Mg, Ca (b) K, Ca, Mg, Na
soluble in water. The element is likely to be (c) K, Ca, Na, Mg (d) K, Na, Ca, Mg
(a) calcium (b) carbon
11. Non-metal that exist in liquid state is
(c) silicon (d) iron
(a) mercury (b) bromine
2. Which of the following pairs will give displacement (c) sodium (d) chlorine
reactions? 12. Oxides of non-metals turn
(a) NaCl solution and Cu metal (a) red litmus blue (b) blue litmus red
(b) MgCl 2 solution and Al metal (c) no effect on litmus (d) None of these
(c) FeSO 4 solution and Ag metal
(d) AgNO 3 solution and Cu metal 13. Metals are malleable and ductile because
(a) metals are lustrous
3. In electrolytic refining impure copper is made of (b) metals are sonorous
(a) anode (b) electrolyte (c) layers of metal atoms can slip over each other
(c) cathode (d) None of these (d) attains form close clusters
4. Element used in making barometer is 14. Iron is galvanised when it is dipped in
(a) copper (b) iron (c) mercury (d) tin (a) molten zinc (b) molten copper
5. Oxides of potassium are reduced by (c) molten carbon (d) molten silver
(a) heat alone (b) carbon ∆
(c) aluminium (d) electrolytic reduction
15. PbO + C → Pb + CO

6. Cinnabar is an ore of ZnO + C → Zn + CO
(a) magnesium (b) mercury In the above reactions
(c) lead (d) tin (a) carbon is reduced
7. Rusting can be prevented by (b) carbon monoxide is oxidised
(a) painting (b) galvanisation (c) metal oxide is reduced
(c) greasing (d) All of these (d) metal oxide is oxidised

8. Which among the following is a noble metal? 16. Metal reacts with chlorine to form
(a) Copper (b) Zinc (a) covalent chloride (b) ionic chloride
(c) Platinum (d) Aluminium (c) coordinate compound (d) None of these

9. Match the column I and column II 17. Copper corrodes in moist air containing CO2 and O2
and changes to
Column I Column II (a) red Cu2O (b) green CuCO 3 ⋅ Cu(OH)2
A. Calcination 1. Froth floatation (c) green CuCl 2 (d) black CuO
B. Roasting 2. Smetling 18. Silver articles turn black after a period of time due to
C. Reduction with carbon 3. Absence of air the formation of
D. Sulphide ores 4. Presence of air (a) silver nitrate (b) silver oxide
(c) silver chloride (d) silver sulphide
A B C D 19. A homogeneous mixture of two or more metals is
(a) 3 4 2 1
(a) allotrope (b) alloy
(b) 1 2 3 4
(c) isotope (d) None of these
(c) 2 1 4 3
(d) 4 3 1 2
Metals and Non-Metals 309
20. In which test-tubes does the rusting occur?

Stopper
Olive
oil
Tap
Calcium
Iron water Boiled
chloride
nail water
(drying
agent)
1 2 3 4

(a) 1 and 4 (b) 2 and 3 (c) Only 2 (d) 1 and 2

Hints and Solutions


Elementary Level
1. (c) 7. (c) 8. (b) 9. (a) 10. (b)
2. (c) Cast iron contains 1.8 to 4.5% of carbon. 11. (c) 12. (b) 13. (b) 14. (a)
3. (b) 4. (a) 15. (a) 16. (c) 17. (c) 18. (b)
5. (c) Aluminium forms M2O 3 type oxide and it is used in making 19. (a) Carbon and sulphur are non-metals which occur in a solid
aircrafts, the ore of metal is bauxite. state at room temperature.

6. (a) 24 carat = 100% gold. 20. (a) 2Al + 6H2O → 2 Al(OH)3 + 3H2

High Skill Questions


1. (a) The element is calcium and the reactions are 13. (c)
2Ca + O 2 → 2CO 14. (a) Galvanisation is coating with zinc and this is mainly done to
(High melting point) prevent corrosion.
CaO + H2O → Ca(OH)2
15. (c)
2. (d) Cu + 2 AgNO 3 → Cu(NO 3 )2 + 2 Ag
16. (b)
Because copper is above than silver in the reactivity series. 17. (b)
3. (a) 4. (c) 5. (d) 18. (d) SAg + H2S → Ag 2S
6. (b) Cinnabar is an ore of mercury. From air Silver sulphide
(black colour)
7. (d) 8. (c) 9. (a) 10. (c) 11. (b) 19. (b)
12. (b) Oxides of non-metals are acidic in nature because they 20. (c) Because rusting process to take place requires the presence
dissolve in water to form acids, so they turn blue litmus red. of air and moisture both.
Chapter

6
Carbon and Its
Compounds
The compounds derived from living organisms are 3. Fullerenes Fullerenes are the most recently
called organic compounds and the chemistry dealing known allotropes of carbon, which was discovered in
1996. The C60 structure has named as Buckminster
with them was named organic chemistry.
Fullerene in honour of American architect
Organic compounds are now defined as the compounds Buckminster Fuller. It resembles with a soccer ball.
of carbon and carbon has a unique nature. This gives
rise to the formation of a large number of compounds. Amorphous Form of Carbon
Carbon has four valence electrons. It forms covalent 1. Coal It is the most impure form of carbon. It is
bonds by sharing its four electrons with other atoms. formed from vegetable matter burried under the
This property by virtue of which it forms for covalent Earth due to pressure and temperature conditions.
Coal is extracted from coal mines. On the basis of
bonds is called tetravalency of carbon. carbon percentage coals are classified in various
The property of self linking of atoms to form long chains types-peat (contains 60% C and lowest grade of
is also possessed by carbon atoms and is termed as a coal), lignite (67% carbon, brown in colour),
bituminous (88% carbon, soft coal) and anthracite
catenation. This is due to the greater strength of
(96% carbon lustrous hard coal).
carbon-carbon bond and hence, it forms straight, branch,
2. Coke It is obtained by the destructive distillation
cyclic chain of atoms and can involved single, double or of coal. It contains almost 85-95% of carbon. It is used
triple covalent bonds. as smokeless fuel and as a reducing agent in
metallurgy.
Different Forms of Carbon 3. Charcoal It is obtained by burning a carbon
compound in the absence of air. The different types of
Crystalline Form of Carbon charcoal are wood charcoal, animal charcoal and
sugar charcoal.
1. Diamond It is a very hard crystalline solid.
Diamond has a rigid structure. All four valence 4. Lamp black It can be obtained by burning carbon
electrons of carbon are used up in bonding. It is used rich oils in an insufficient amount of air. It is used in
as precious stone for jewellery and for cutting glass the production of printer ink and carbon papers.
etc.
Diamonds are found in Panna mines in Madhya Coal Tar
Pradesh and Golkunda mines of Karnataka. It is an
insulator. It is a black, thick liquid with an unpleasant smell. It is
2. Graphite It is a dark grey substance with layer a mixture of about 200 substances like benzene, phenol,
structure and having a greasy feel. It is a good
conductor of electricity due to presence of free aniline etc. Which are used in dyes, drugs and polymer
electron. It is used as a lubricant in making lead industry. Now these days bitumen, a petroleum product
pencils and as an electrode in a dry cell. is used in place of coaltar for metalling the roads.
Carbon and Its Compounds 311
Coal Gas
It is a mixture of CO, CH4 and H2 . It is used as a fuel in many industries situated near the coal processing plants.
Coal gas was used for street lighting, first time in London in 1810.

Classification of Hydrocarbons
Hydrocarbons are classified as
Hydrocarbons

Aliphatic or open chain compounds Cyclic or closed chain compounds

Saturated alkanes Unsaturated


(paraffins)
CnH2n+2
CH3 —CH3 Alkenes Alkynes
CnH2n CnH2n–2
H H
C—
—C — C—H
H—C —
H H

Chains, Branches and Rings To explain branching we can take example of C4 H10
The name and structure of six compounds of carbon in H H H H
| | | |
chain form are H — C— C— C— C— H
| | | |
No. of carbon H H H H
Name Formula Structure
atoms
H H


H
— —

| —H
C—
CH4 H —C— H H—C—C — H
1. Methane

| C——H
H H H H —
H H H
| |
H — C— C— H In addition to straight and branched carbon chains
2. Ethane C 2H6 | | some compounds have carbon atoms arranged in the
H H
form of ring also. e. g., Cyclohexane
H H H H H
| | |
3. Propane C 3H8 H —C—C—C— H
| | | C
H H
H H H C C
H H
H H H H
| | | | H H
4. Butane C 4H10 H —C—C—C—C— H C C
| | | | H H
H H H H C
H H H H H H H
| | | | |
5. Pentane C 5H12 H —C—C—C—C—C— H
| | | | | Homologous Series
H H H H H
H H H H H H A homologous series is a group of organic compounds
| | | | | | having a similar structure and similar chemical
6. Hexane C 6H14 H —C—C—C—C—C—C— H
| | | | | | properties in which the successive compounds differ by
H H H H H H a CH 2 group.
312 Study Package for NTSE

Nomenclature of Carbon Compounds l Ignition Temperature

The following simple rules are as follows The lowest temperature up to which a substance
must be heated before it catches fire, is called ignition
l Selection of longest carbon chain known as parent chain. temperature. Every combustible substance has its
l The remaining carbon atoms that are not the part of own ignition temperature.
parent chain are taken as substituents. (i) The ignition temperature of petrol is lower than
l The carbon chain is numbered in such a way that the that of kerosene, so petrol can easily vapourise
alkyl groups (substituents) get the smallest possible and catches fire.
number. (ii) Coal doesn’t catch fire just by putting in air
l If any functional group is present then it is gets the because it has high ignition temperature.
smallest possible number. The ‘e’ of alkane is replaced (iii) Alcohol, LPG, ether, benzene have low ignition
by suffix of functional group indicating the position. temperature, so they are kept and stored very
l In case different types of substituents are attached to carefully.
the chain, they are arranged and named alphabetically.
l If the compound has unsaturation then ‘ene’ is used for 2. Oxidation
double bond and ‘yne’ is used for triple bond. Carbon compounds can be easily oxidised on
combustion. We also have reactions where alcohols are
Chemical Properties of Carbon converted to carboxylic acids.
Compounds Alkaline KMnO + heat
CH 3CH2 OH       
4
 → CH 3COOH
Acidified K 2Cr2O 7
Chemical properties of carbon compounds are classified as
3. Addition Reactions
1. Combustion
When alkenes and hydrogen are passed over finely
Chemical process in which a substance (fuel) burns in
divided catalysts like platinum, palladium at ordinary
the presence of air (oxygen) with the release of heat and
temperature or nickel at 200°C, the two atoms of
light. It is an exothermic process. In this process, sound hydrogen molecule are added to the unsaturated
produced may be a bissing sound, crackling sound or an molecule which thus becomes a saturated one.
explosion. e. g.,
H H Ni/
(i) Charcoal (C) + Oxygen → CO2 + heat + light C====C + H2 CH3 CH3
H H 200°C
(ii) CH4 + 2O2 → CO2 + 2H2O + energy
4. Substitution Reactions
Types of Combustion
Alkanes react with chlorine, bromine or iodine in the
l Incomplete Combustion presence of sunlight to give halogen substituted
2C + O2 → 2CO products known as alkyl halides e. g.,
l Complete Combustion CH4 + Cl 2  Diffused
 → CH 3Cl + HCl
sunlight
C + O2 → CO2
l Rapid Combustion LPG gas, petrol and camphor go to Some Important Carbon
flames even with a spark from a gas lighter. These
materials are highly inflammable. Compounds
l Slow Combustion Burning of wood, candle, paper etc.
l Explosive Combustion When a cracker is ignited, it Ethanol
explodes with a big-bang producing heat and light. Ethanol also known as ethyl alcohol is a liquid at room
l Spontaneous Combustion Strike the head of a temperature. It is prepared by the addition of H2 SO4 on
matchstick on the side of the match box. The head of ethene at 80°C followed by hydrolysis with boiling
matchstick catches fire instantaneously.
water. It is a good solvent for gums and resins. It is used
Conditions Necessary for Combustion in the manufacture of chemicals and synthetic products.
l Combustible material (the fuel)
It is used as a beverage in alcoholic drinks intake of
small amount of absolute alcohol can be lethal.
l Air
Carbon and Its Compounds 313
Properties of Ethanol Soaps and Detergents
1. Reaction with sodium
2CH3CH2OH + 2Na → 2C2H5O− Na + + H2 Soaps
Sodium ethoxide
2. Dehydration with concentrated sulphuric acid These are made by reacting fats or oils with sodium
Conc. H2SO4 hydroxide. Soap has hydrophobic as well as
CH3CH2OH      → CH2 == CH2 + H2O
170 °C Ethene
hydrophilic parts. Hydrophilic part have affinity
towards water. Whereas hydrophobic part repel water
Ethanoic Acid molecules.
It is also known as acetic acid. Vinegar commonly The hydrophobic long chain part of molecule attaches
called ‘Sirka’ is a dilute (4-5%) solution of acetic acid. It itself to the grease and dirt particles and the hydrophilic
is widely used as a preservative. Acetic acid is used as a group makes the particles soluble in water so that by
solvent, in medicines, for coagulating rubber and for the agitation they are loosened from the fabric and
manufacture of cellulose acetate which is used for dispersed during the process of cleaning.
varnishes. Acetic acid in the pure state is also called
Hard water doesn’t produce lather with soap. Hard
glacial acetic acid.
water contains calcium and magnesium ions which
Properties of Ethanoic Acid react with soap and produce insoluble carbonates. This
prevents the formation of lather. This can be overcome
1. Esterification reaction
by adding sodium carbonate to precipitate the insoluble
H2SO4 carbonates.
CH3COOH + HOC2H5   → CH3COOC2H5 + H2O
Acetic acid Ethanol Ethyl acetate (ester)
Esters are sweet smelling compounds and on reaction with Detergents
alkalines like NaOH convert ester back to alcohol and These doesn’t form scum in hard water. Detergents are
sodium salt of carboxylic acid and the reaction is called
made chemically from petroleum hydrocarbons. Like
saponification.
soaps, detergents also possess two headed molecules,
CH3COOC2H5  NaOH
 → C2H5OH + CH3COONa i. e., hydrophilic and hydrophobic.
2. Action of a base They can wash the dirt in soft water as well as in hard
CH3COOH + NaOH → CH2COONa + H2O water, as they don’t produce insoluble compounds of
Sodium acetate calcium and magnesium salts. Now-a-days certain
3. Reaction with carbonates and bicarbonates enzymes are also added to detergents to improve their
2CH3COOH + Na 2CO3 → 2CH3COONa + H2O + CO2 ↑ efficacy. All detergents are of household use, are
CH3COOH + NaHCO3 → CH3COONa + H2 O + CO2 ↑ alkaline in nature and turn red litmus blue.

Points to Remember
l Carbon is a versatile element that forms the basis for all living organisms and many of the things we use.
l Carbon forms a variety of compounds due to its tetravalent nature and the property of catenation shown by it.
l Carbon forms both saturated and unsaturated compounds. The carbon chains may be in the form of straight chains, branched chains or rings.
l A functional group is an atom or a group of atoms which largely determines the chemical properties of an organic compound e.g. , Alcohol (  OH),
aldehyde (—CHO), ketone ( C==O) and carboxylic acid (—COOH).
l Carbon and its compounds are some of our major sources of fuels.
l Saponification process is used in the preparation of soaps.
l The action of soaps and detergents is based on the presence of hydrophobic and hydrophilic groups in the molecule and this helps to emulsify the oily
dirt and hence its removal.
Target Exercise
Elementary Level
1. The variety of coal having highest carbon content is 13. The variety of coal used as old house coal is
(a) peat (b) anthracite (a) anthracite (b) lignite
(c) bituminous (d) lignite (c) peat (d) bituminous

2. The main constituent of LPG is 14. When petroleum is heated the vapours contain
(a) methane (b) ethane mainly
(c) butane (d) propane (a) kerosene
(b) petroleum ether
3. Thermodynamically the most stable allotrope of (c) diesel
carbon is (d) machine oil
(a) diamond (b) graphite
(c) carbon black (d) charcoal 15. Which of these substances is made from coal?
(a) Paper (b) Coke
4. Graphite is used as a lubricant in machines because (c) Iron (d) Lactose
(a) it is a good conductor of electricity
(b) it has a high melting point and slippery layers 16. The soft brown coal mined in Germany and Australia
(c) its density ranges from 1.9 to 2.3 g/cm 3 is called …… coal.
(d) it is strong and soft (a) lignite (b) peat
(c) bituminous (d) anthracite
5. The allotrope of carbon which is a good conductor of
heat and electricity is 17. Which of these fules is smokeless?
(a) diamond (b) graphite (a) Coal (b) Wood
(c) charcoal (d) None of these (c) Paraffin (d) Coke

6. Which of the following is not considered as crystalline 18. When petrol burns, chemical energy changes into
allotrope of carbon? …… energy.
(a) Coal (b) Diamond (a) electrical
(c) Graphite (d) Fullerene (b) heat
(c) wind
7. Which form of carbon is found in Golkunda mines of (d) kinetic energy
Karnataka?
(a) Diamond (b) Graphite 19. The process in which a substance combines chemically
(c) Coal (d) Coke with oxygen above its ignition temperature with the
evolution of large amount of heat and light in a very
8. Lead pencil contains short time is
(a) graphite (b) diamond
(a) slow combustion
(c) lead (d) lead sulphate
(b) complete combustion
9. The major component of CNG is (c) spontaneous combustion
(a) CH4 (b) C 3H8 (d) rapid combustion
(c) C 4H10 (d) C 2H6 20. Match the column I with column II.
10. Which form of coal is the preferred domestic fuel? Column I Column II
(a) Anthracite (b) Peat
(c) Lignite (d) Bituminous A. Solid fuel 1. LPG
B. Liquid fuel 2. CNG
11. Petroleum is refined by
(a) simple distillation C. Gaseous fuel used in cooking 3. Paraffin wax
(b) fractional distillation Gaseous fuel used in
D. transportation 4. Kerosene
(c) destructive distillation
(d) None of the above Codes
12. An allotropic form of carbon used for cutting and A B C D
drilling is (a) 1 2 4 3
(b) 4 3 2 1
(a) diamond (b) charcoal
(c) fullerenes (d) graphite (c) 3 4 1 2
(d) 4 2 1 3
Carbon and Its Compounds 315
High Skill Questions
1. A dilute solution of ethanoic acid in water is called 9. Which of the following is an aldehyde?
(a) Tollen’s reagent (b) vinegar (a) CH3  C CH3 (b) H  C H
(c) acetone (d) acetaldehyde || ||
O O
2. The gas evolved when ethanoic acid reacts with (c) CH3  C CH2CH3 (d) H  C OH
sodium carbonate is || ||
(a) carbon dioxide O O
(b) oxygen 10. Which of the following functional group represents a
(c) hydrogen
ketone?
(d) carbon monoxide
(a)  C H (b)  C 
3. The ability of carbon to form long chain compounds is || ||
called O O
(a) isomerisation (b) tetravalency (c)  C  OH (d) —OH
(c) polymerisation (d) catenation ||
O
4. The type of reaction shown by unsaturated 11. Ethene with the molecular formula C2H 6 has
hydrocarbons is (a) 6 covalent bonds (b) 7 covalent bonds
(a) displacement (b) substitution (c) 8 covalent bonds (d) 9 covalent bonds
(c) addition (d) None of these
12. The number of carbon atoms in Buckminister
5. Pure ethanol is also given the name of fullerenes are
(a) spirit (b) acetone (a) 60 (b) 75 (c) 40 (d) 100
(c) absolute alcohol (d) vinegar
13. Crystalline form of carbon is
6. The IUPAC name of (CH 3) 3C CH == CH2 is (a) charcoal (b) coke
(a) hex-1-ene (c) cool (d) diamond
(b) 3, 3-dimethyl but-1-ene
(c) 2, 2-dimethyl 3-butene 14. Which amongst the given pair of compounds forms a
(d) pent-1-ene homologous series?
(a) CH3OH and CH3CH2CH2OH
7. The acid present in vinegar is known as
(b) CH3CHO and CH3CH2CHO
(a) butanoic acid (b) ethanoic acid
(c) methanoic acid (d) propanoic acid (c) CH3COOH and C 3H7COOH
(d) CH3CH2 OH and CH3CHO
8. The number of single and double bonds present in
benzene are 15. Which out of the following is a unsaturated
(a) 12 single and 3 double bonds hydrocarbon?
(b) 6 single and 6 double bonds (a) Cyclobutane (b) Propyne
(c) 15 double bonds (c) Hexanol (d) Ethane
(d) 6 single and 6 double bonds

Hints and Solutions


Elementary Level
1. (b) Anthracite is the purest variety of coal. It contains about 98% 10. (a) 11. ( b)
carbon.
12. (a) 13. ( d)
2. (c) 3. ( a) 4. (b) 5. (b)
14. (b) 15. ( b)
6. (a) 7. (a)
16. (a) 17. ( d)
8. (a) Lead pencil are made up of graphite, not of lead.
18. (b) 19. ( d)
9. (a) Methane, CH4 is the major component of compressed
20. (c)
natural gas (CNG).
316 Study Package for NTSE

High Skill Questions


1. (b) 9. (b)
− +
2. (a) 2 CH3COOH + Na 2CO 3 → 2CH3COO Na + H2O + CO 2 ↑ 10. (b)
3. (d) 11. (b) Ethane is C 2H6 .
4. (c) e. g., CH2 == CH2 + H2 Ni 
→ CH3 — CH3 H H
| |
(Addition reactions) H —C —C — H
| |
5. (a) H H
7 covalent bonds
CH 3
4 | 2 1 12. (a)
6. (b) CH3 —C — CH == CH2
| 13. (d)
CH 3 14. (b)
3, 3 - dimethyl but-1-ene
15. (b) Cyclo butane is
7. (b)
H 2C— CH 2
8. (a) Benzene is C 6H6 . | |
H H2C — CH2

C— Hexanol is CH3CH2CH2CH2CH2CH2OH

H—C C—H
Ethane is CH3 CH3



H—C— C—H Propyne is CH3 C ≡≡ CH
C—

Hence, it is the unsaturated hydrocarbon an example of an


H alkyne.
= 12 single and 3 double bonds.
Chapter

7
Periodic Classification
of Elements
Need for the Classifation of Elements At present, as Structural Features of the Mendeleef’s
many as 118 elements are known , it become difficult to Periodic Table
study these elements individually, so classification of l Mendeleef’s original periodic table consisted of eight
elements was done, to make the study easier. vertical columns called groups I-VIII and
Dobereiner’s Triads Dobereiner arranged certain seven horizontal rows called periods 1-7. Modified
elements with similar properties in groups of three Mendeleef’s periodic table contains nine groups, i.e.,
I-VIII and zero group (of noble gases).
(triads) in such a way that the atomic mass of the middle
element was nearly the same as the average atomic l All groups except VIII and zero have been further
divided into two sub-groups called A and B. A groups of
masses of the first and the third elements. A few triads
left hand side consist of normal elements while groups
proposed by him are listed e.g., B of right hand side consist of transition elements.
(i) Li, Na, K (ii) Cl, Br, I l Elements of group I A are called alkali metals while
(iii) Ca, Sr, Ba (iv) S, Se, Te those of group IB (i.e., Cu, Ag, Au) are called coinage
Newland’s law of Octaves Elements are arranged in metals.
increasing order of atomic masses and it was observed
that every eighth elements had similar properties to Modern Periodic Table
those of the first element. Moseley forms the basis of the modern periodic law. He
However, law of octaves also could not be applied discovered that the square root of the frequency of the
beyond calcium. Moreover, with the discovery of noble more prominent X-rays emitted by a metal was
gases, the eighth element no longer remains the similar proportional to the atomic number and not the atomic
properties with first element. mass of the atom of the metal. Hence, atomic number
should be the basis of classification of the elements.
Mendeleef’s Periodic Table Modern periodic law states that “the physical and
chemical properties of the elements are a periodic
It is based upon Mendeleef’s periodic law, which states function of their atomic number.”
that the physical and chemical properties of the
elements are a periodic function of their atomic masses. It was observed that the elements with similar
properties reoccurred at regular intervals of 2, 8, 8, 18,
Mendeleef named Gallium (Ga) and Germanium (Ge) or 32. These numbers (2, 8, 8, 18 and 32) are called
elements as eka-aluminium and eka-silicon magic numbers, and cause of periodicity in properties.
respectively.
318 Study Package for NTSE

Structural Features of the Long form of The absolute value of atomic radius cannot be
the Periodic Table determined because
l it is not possible to locate the exact position of electrons
1. Groups in an atom as an orbital has no sharp boundaries.
l The 18 vertical columns of the periodic table, are called l it is not possible to isolate an individual atom.
groups. l in a group of atoms, the probability distribution of
l Elements of groups 1, 2, 13-18 are called normal or electrons is influenced by the presence of neighbouring
representative elements. atoms.
l Elements of groups 3-12 are called transition elements. Since, absolute value of atomic size cannot be
l The elements belonging to a particular group is known determined, it is expressed in terms of the operational
as a family and is usually named after the first element. definitions such as covalent radius, Van der Waals’
e. g., Boron family (group 13). radius, ionic radius and metallic radius.
In addition to this, some groups have typical names. Periodic variation of atomic radii
e. g., (i) On moving down the group due to addition of new
shell the valence shells become far away from the
(i) Elements of group 1 are called alkali metals
nucleus and thus the atomic radius increases.
(ii) Elements of group 2 are called alkaline Earth
metals (ii) On moving along the period, the effective nuclear
charge increases and thus the electron cloud is
(iii) Elements of group 3 are called pnicogens
attracted more strongly towards the nucleus
(iv) Elements of group 16 are called chalcogens
resulting contraction in the atomic radius.
(v) Elements of group 17 are called halogens
(vi) Elements of group 18 are called noble gases Isoelectronic ions or species The ions of the different
elements which have the same number of electrons but
2. Periods different magnitude of the nuclear charge. The size of
The 7 horizontal columns (of rows) are called periods. isoelectronic ions decreases with the increase in the
The seven periods of periodic table are nuclear charge.

Shortest Period 1st period ( 1 H → 2 He) contains e.g., O2– , F − , Na + , Mg 2+ , Al 3+


2 elements.
Short Periods 2nd period ( 3 Li → 10 Ne) and 3rd period
Ionisation Energy (IE)
( 11 Na → 18 Ar) contains 8 elements each. The amount of energy required to remove an electron
Longest Period 6th period ( 55 Cs → 86 Rn) contains 32 from the last orbit of an isolated (free) atom in gaseous
elements and is the longest period. state is known as ionisation potential or energy or
better first ionisation energy of the element, i.e.,
Long Periods 4th period ( 19 K → 36 Kr) and 5th period
( 37 Rb → 54 Xe) contain 18 elements each. M ( g) + IE → M + ( g) + e −
Incomplete Period 7th period ( 87 Fr) is however, l The amount of energies required to remove the
incomplete and contains at present only subsequent electron (2nd, 3rd, …) from the monovalent
26 elements. gaseous cation of the element one after the other are
collectively called successive ionisation energies. These
are designated as IE2, IE3, and so on. It may be noted
Periodic Properties
that. IE3 > IE2 > IE1 (for a particular element). IE is
Properties which show a regular gradation when we expressed in eV/atom or kcal mol −1 or kJ mol −1 Note
move from left to right in a period or from top to bottom that eV atom −1 = 23.06 kcal mol −1 = 96.3 kJ mol −1.
in a group are called periodic properties. These l In general, the first IE increases along the period from
properties are atomic size, ionisation energy, electron left to right. However, there are some exceptions to the
affinity etc. general trend.
(i) IE decreases from elements of group 2 → 3.
Atomic Radii (ii) IE decreases from elements of group 15 → 16.
l In a group of the Periodic Table, the ionisation energy
It refers to the distance between the centre of the decreases from top to bottom.
nucleus of atom to its outermost shell of electrons.
Periodic Classification of Elements 319
Electron Affinity (EA) l Non-metallic character decreases down the group and
increases across a period.
It is the amount of energy released when a gaseous atom l Both metallic and non-metallic character depends upon
accepts the electron to form gaseous anion. the atomic size and nuclear charge.
X ( g) + e − → X − ( g) + EA
l EA values are expressed in eV/atom or kcal/mol or Electronegativity (EN)
kJ/mol. It is the tendency of an atom in a molecule to attract the
l The energy changes accompanying the addition of first, shared pair of electrons towards itself.
second, third etc electrons to neutral isolated gaseous
l In a period EN increases from left to right due to
atoms are called successive electron affinities and are
decrease in size and increase in nuclear charge of atoms.
designated as EA1, EA 2, EA 3 etc.
l In a group, EN decreases from top to bottom due to
l The first EA is always taken as positive. However, the
increase in atomic size.
addition of second electron to the negatively charged ion
is opposed by coulombic repulsion and hence required Valency It is the number of electrons gained or lost or
(absorbed) energy for the addition of second electron. shared with other atoms in the formation of compounds.
Thus, second Electron Affinity (EA 2) of an element is
taken as negative.
Valency of group 1 and 2 elements is equal to the
number of electrons in the outermost shell, while that
e. g.,
O ( g ) + e − → O− ( g ); EA1 = +141 k J mol−1 for groups 13 to 14 is group number-minus 10 and that
(Exothermic) for group 15-18 is 8-minus the number of electrons in
O − ( g) + e − → O2 − ( g); EA 2 = −780 kJ mol −1 the outermost shell.
(Endothermic)
l Electron affinity increases in moving across the period Points to Remember
from left to right due to increase in charge, but the l Moseley discovered atomic number.
values are unexpectedly low in elements of group 2, 15 l Smallest period of Modern periodic table contains 2 elements.
and 18 due to stable electronic configurations. (H and He)
l Within a group of the Periodic Table the electron l Variation in periodic properties
affinities decreases from top to bottom.
Period Atomic size ↓
l In general, electron affinity follows the following trend
Halogens > oxygen family > nitrogen Ionisation enthalpy or energy ↑
family > metals of groups 1 and 13 and non-metals of Electron affinity ↑
group 14 > metals of group 2. Electronegativity ↑
Metallic character ↓
Metallic and Non-metallic Character Those elements Group Non-metallic character ↓
which have a tendency to lose their valence electrons
and form a positive ion are considered as metals. Atomic size ↑
l The metallic character increases as one moves down a Ionisation enthalpy or energy ↓
group and decreases across a period, moving from left to Electron affinity ↓
right. Electronegativity ↓
Metallic character ↑
l Those elements which have a tendency to gain electrons
in order to obtain octet in their outermost orbit are Non-metallic character ↓
considered as non-metals.
Target Exercise
Elementary Level
1. Who is known as the father of chemistry? 15. Mendeleef’s periodic table is based on
(a) Lavoisier (b) van’t Hoff (a) atomic number (b) atomic mass
(c) Rutherford (d) Mendeleef (c) atomic value (d) atomic radii

2. Who was awarded the first nobel prize in chemistry? 16. On going from left to right in a period in the periodic
(a) Rutherford (b) Einstein table the electronegativity of the elements
(c) van’t Hoff (d) Mendeleef (a) increases
(b) decreases
3. ‘Law of octaves’ was proposed by (c) remains unchanged
(a) Newland (b) Dobereiner
(d) decreases first than increased
(c) Mendeleef (d) Lother Meyer
17. Which of the following is isoelectronic with carbon
4. The elements of group 18 are
atom?
(a) halogens (b) noble gases
(a) Na + (b) Al 3+ (c) O 2− (d) N+
(c) chalcogens (d) alkaline Earth metals

5. According to the Modern Periodic Law of elements 18. Which of the following have same number of electrons
the variation in properties of elements is related to in the outermost orbit?
their (a) As, Bi (b) N, O
(a) atomic number (b) atomic mass (c) Pb, Sb (d) P, Ca
(c) mass number (d) nuclear mass 19. Which of the following remains unchanged on
6. Dobereiner’s triad of elements is illustrated by descending a group in the periodic table?
(a) Fe, Co, Ni (b) Na, K, Cs (a) Valence electrons (b) Atomic size
(c) F, Cl, I (d) Li, Na, K (c) Density (d) Metallic character

7. Which of the element has maximum ionisation 20. Which pair of elements belongs to the same group?
energy ? (a) Elements with atomic number 17 and 38
(a) C (b) N (c) B (d) O (b) Elements with atomic number 20 and 40
(c) Elements with atomic number 17 and 35
8. Which of the following has the largest atomic size? (d) Elements with atomic number 11 and 33
(a) Al (b) Al+ (c) Al 2+ (d) Al 3+
21. Elements of group-2 are called
9. Element ‘A’ has eleven protons in its nucleus. To (a) transition elements (b) alkali metals
which group of the periodic table it will belong? (c) alkali earth metals (d) halogens
(a) 1 (b) 2 (c) 6 (d) 8
22. Eka-aluminium and Eka-silicon are known as
10. The family of elements with the highest ionisation (a) gallium and germanium (b) aluminium and silicon
enthalpy is (c) iron and sulphur (d) neutron and proton
(a) alkaline Earth metals (b) halogens 23. Which of the following is the most electronegative
(c) noble gases (d) alkaline metals
element?
11. Which of the following is not a transition metal? (a) F (b) Cl
(a) Ag (b) Pb (c) W (d) Mn (c) Br (d) I

12. The element which exists in liquid state at room 24. The correct decreasing order of the size of the
temperature is following ions is
(a) Na (b) Br (c) Hg (d) Ga (a) Li+ > H+ > H− (b) H+ > H− > Li+
(c) H− > Li+ > H+ (d) H− > H+ > Li+
13. Which amongst the following is not an alkali metal?
(a) Na (b) Li (c) Mg (d) K 25. What is the atomic number of next halogen after
85 At?
14. Who developed long form of the periodic table?
(a) 85 (b) 117
(a) Lother Meyer (b) Neils Bohr
(c) 104 (d) 167
(c) Mendeleef (d) Moseley
Periodic Classification of Elements 321
High Skill Questions
1. Which group of the periodic table contains no metal? 13. Arrange in the increasing order of ionic radii the
(a) I A (b) III A elements O, C, F, Cl, Br
(c) VII A (d) VIII (a) F, O, C, Cl, Br (b) F, C, O, Cl, Br
(c) F, Cl, Br, O, C (d) C, O, F, Cl, Br
2. The atomic radius decreases in a period due to
(a) increase in nuclear attraction 14. Which are the three elements of Dobereiner’s triad?
(b) decrease in nuclear attraction (a) H, Li, Be (b) Be, N, O
(c) increase in number of electrons (c) Ti, Mn, Fe (d) Li, Na, K
(d) decrease in number of electrons
15. Which of the following set has the strongest tendency
3. Match the column A with column B to form anions?
Column A Column B (a) V, Cr, Mn (b) Ga, In, Tl
(c) Na, Mg, Al (d) N, O, F
A. Alkaline Earth metal 1. Rb
B. Radioactive element 2. As 16. According to the periodic law of elements, the
C. Alkali metal 3. Mg variation in properties of elements is related to their
D. Metalloid 4. Fr (a) atomic masses
Codes (b) nuclear masses
A B C D A B C D (c) atomic numbers
(a) 1 2 3 4 (b) 3 4 1 2 (d) nuclear neutron proton number ratios
(c) 2 1 4 3 (d) 4 3 2 1 17. Which element has the maximum density?
4. How many elements are found in nature? (a) Os (b) Li (c) H (d) Bi
(a) 92 (b) 107 (c) 105 (d) 104 18. The number of elements in 5th period of periodic table
5. The element with atomic number 9, 17, 35, 53 and 85 (a) 8 (b) 18
are all (c) 10 (d) 32
(a) noble gases (b) heavy metals 19. One element M is present in the 13th group, the
(c) light metals (d) halogens
formula of its chloride is
6. The neutral particle of an atom is (a) MCl 3 (b) M 2Cl 3
(a) electron (b) proton (c) M 3 Cl (d) MCl 2
(c) neutron (d) None of these
20. The law of octaves was only applicable up to which
7. An atom is electrically neutral because element?
(a) protons = neutrons (b) electrons = neutrons (a) Sodium (b) Calcium
(c) protons = electrons (d) None of these (c) Fluorine (d) Bromine
8. The third energy level is 21. Eka-aluminium was replaced by which element later
(a) K (b) M (c) L (d) N on
(a) Silicon (b) Indium
9. The atomic number of an atom with electronic
(c) Gallium (d) Scandium
configuration 2, 8, 1 is
(a) 2 (b) 1 22. The element with atomic number 8 resembles with the
(c) 8 (d) 11 element having atomic number
(a) 16 (b) 17 (c) 26 (d) 27
10. The number of electrons present in the outer most
shell of all noble gases (except helium) is 23. Which element is not a noble gas?
(a) 2 (b) 8 (a) Helium (b) Neon
(c) 6 (d) 10 (c) Radium (d) Argon
11. The most active step in the development of Periodic 24. Which of the following periodic property generally
Table was taken by decreases along a period?
(a) Mendeleef (b) Dalton (a) Metallic character (b) Electron affinity
(c) Avogadro (d) Cavendish (c) Ionisation energy (d) Valency
12. In the periodic table going down in fluorine group 25. Chloride and potassium ions are isoelectronic. Then
(a) reactivity will increases (a) their atomic sizes are same
(b) electronegativity will increase (b) Cl − ion is bigger than K + ion
(c) ionic radius will increase (c) K + ion is relatively bigger
(d) ionisation potential will increase (d) None of the above
Hints and Solutions
Elementary Level
1. (a) 2. (c) 3. (a) 4. (b) 5. (a) 15. (b) 16. (a)
6. (a) 7. (b) 17. (d) Carbon and N+ have 6 electrons, so they are isoelectronic.
8. (a) Neutral atom have bigger sizes from the consitituent positive 18. (a) 19. (a) 20. (c) 21. (c) 22. (a)
ions.
23. (a)
9. (a) Eleven protons means valence shell contains 1 electron, so it
will belong to group 1. 24. (c) H− > Li+ > H+

10. (c) 11. (b) 12. (c) 13. (c) 14. (b) 25. (b) Atomic number of halogen = 85 + 32 = 117

High Skill Questions


1. (c) Group VIIA contains F, Cl, Br, I, At, all are halogens and 10. (b) 11. (a)
non-metals.
12. (c) In the periodic table for all the group ionic radius increases on
2. (a) Since, the number of electrons increases the attraction by the going down the group.
nucleus increases thereby increasing the effective number
13. (a) 14. (d) 15. (d) 16. (c) 17. (a)
charge.
18. (b) There are 18 elements in the 5th period starting from
3. (b) 4. (a)
Rubidium to Xenon.
5. (d) Atomic number
9 → fluorine 19. (a) M is belonging to group 13, so the cation of
17 → chlorine M → M 3 + → MCl 3 .
35 → Bromine
20. (b) 21. (b)
53 → Iodine
85 → Astatine 22. (a) Atomic number 8 is of oxygen → 2, 6 is the electronic
They are all called halogens. configuration. So at number 16 is of sulphur → 2, 8, 6 is the
electronic configuration. Since valence electrons are same so
6. (c) 7. (c) they both belong to the same group and show similar properties.
8. (b) n = 1, 2, 3 K (energy levels) and alphabetically → K, L, M 23. (c) Radium is not a noble gas it s radioactive element belonging
.... So third energy level is M shell. to group 2.
9. (d) The atomic number = number of electrons 24. (a) 25. (b)
∴ 2 + 8 + 1 = 11
Chapter

8
Atoms and Molecules
Law of Chemical Combination Symbols Derived from Latin Names of the Elements

There are two laws of chemical combination. English name of Latin name of the
S. No. Symbols
the elements elements
Law of Conservation of Mass It states that mass can 1. Sodium Na Natrium
neither be created nor destroyed in a chemical
2. Potassium K Kalium
reaction. 3. Iron Fe Ferrum
Law of Constant Proportion It states that in a 4. Copper Cu Cuprum
chemical substance the elements are always present in 5. Silver Ag Argentum
definite proportions by mass. 6. Gold Au Aurum
7. Mercury Hg Hydragyrum
Chemical Symbols 8. Lead Pb Plumbum
9. Tin Sn Stannum
The symbol of an element is the ‘first letter’ or the ‘first
letter and another letter’ of the English name or Latin
Atomic Masses of a Few Elements
name of the element.
Element Atomic mass ( µ)
Symbols Derived from English Names
of the Elements Hydrogen 1
Carbon 12
English name English name
S. S. Nitrogen 14
of the Symbols of the Symbols
No. No.
elements elements Oxygen 16
1. Hydrogen H 10. Zinc Zn Sodium 23
2. Helium He 11. Iodine I Magnesium 24
3. Lithium Li 12. Sulphur S Sulphur 32
4. Carbon C 13. Calcium Ca Chlorine 35.5
5. Nitrogen N 14. Phosphorus P Calcium 40
6. Oxygen O 15. Silicon Si
7. Fluorine F 16. Chlorine Cl
Atomicity
8. Neon Ne 17. Bromine Br
The number of atoms present in one molecule of an
9. Aluminium Al
element or compound is called its atomicity.
324 Study Package for NTSE
Atomicity of Some Elements 1st step Mg O → MgO
Type of element Name Atomicity 2nd step Mg 2
O 2 → Mg 2 O2
Non-metal Argon Monoatomic 3rd step Mg 2 O2
Helium Monoatomic
Common factor is 2, so divide both by 2 to get 1.
Oxygen Diatomic
Hydrogen Diatomic Hence, MgO is the formula.
Nitrogen Diatomic
Phosphorus Tetraatomic
Sulphur Octaatomic
Atoms and Molecules
An atom is the smallest particle of an element that can
take part in a chemical reaction.
Ion and Radical
An atom carrying a unit positive or negative charge is A molecule is the smallest particle of a substance
called as an ion. (element or compound) which has the properties of that
substance and can exist in free state. e.g.,
A group of atoms of different elements that carries
a charge and behaves as single unit are called (i) Molecule of an element H2, Cl2, O2
radicals. (ii) Molecule of a compound H2O, HCl, NH3

Names and Symbols of Some Ions Molecular Mass


Names Symbols Names Symbols Molecular mass of a substance is the relative mass of its
Sodium Na +
Oxide O 2− molecule as compared with the mass of a 12 C atom
Potassium K+ Sulphide S 2− taken as 12 units. Molecular mass can be computed by
Silver Ag + Nitride N 3− taking an algebric sum of atomic masses of different
Copper (I) Cu + Ammonium NH+4 atoms present in the molecule.
Magnesium Mg 2+ Hydroxide OH − Molecular mass of H2O = Mass of H atoms + Mass of O
Calcium Ca 2+ Nitrate NO −3 atom = 2 + 16 = 18
Zinc Zn 2+ Bicarbonate HCO −3
The molecular mass expressed in grams is called gram
Iron (II) Fe 2+ Carbonate CO 2−
3 molecular mass of substance.
Copper (II) Cu 2+ Sulphite SO 2−
Aluminium Al 3+
3 Gram molecular mass of H2O = 18 g
Sulphate SO 2−
Iron (III) Fe 3+ 4

Chloride Cl −
Phosphate PO 3−
4 Mole Concept
One mole is the quantity of a substance that contains
Writing Chemical Formulae of a the number of entities equal to the number of atoms
Compound contained in 12 g of carbon.

The chemical formula of a compound is the short hand 1 mole of atoms = 6.022 × 1023 atoms
notation to represent the name of the compound.
1st Step Write the symbols of the elements in such a
Points to Remember
way that positive ion is written on the left hand side and Amount of substance (in grams)
l
Number of moles =
negative ion on the right hand side. Gram atomic mass
Amount of substance (in grams)
2nd Step Write the valency of each above the symbol. l
Number of moles =
Gram molecular mass
3rd Step Now criss-cross the valencies to get the Gram molecular mass
l
Mass of a molecule =
chemical formula. Avogadro number NA
e. g., Let us write the chemical formula of magnesium l
For a gas at STP (0°C temperature and 760 mm Hg pressure)
oxide. volume is 22.4 L for one mole gas.
Target Exercise
Elementary Level
1. Atomicity of phosphorus is 8. Which of the following is not represented by chemical
(a) 1 (b) 2 (c) 4 (d) 6 symbol?
2. The chemical symbol of silver is (a) Size of the atom/element
(a) Ag (b) Si (c) Cu (d) S (b) Name of the element
3. How many atoms of oxygen are contained in (c) Definite mass of the element
10 molecules of sulphur trioxide? (d) None of the above
(a) 20 (b) 10 (c) 30 (d) 40 9. The chemical symbols for elements were stated by
4. What is the gram molecular mass of H2SO4 ? (a) Berzelius (b) Dalton
(a) 49 g (b) 98 g (c) Ostwald (d) Mendeleef
(c) 196 g (d) 92 g 10. A compound is formed by the combination of atoms of
5. Match Column I and Column II at least ……… elements.
(a) four (b) three
Column I Column II (c) five (d) two
A. Thallium 1. Co
11. Atomicity of helium is
B. Titanium 2. Zn
(a) 1 (b) 2
C. Zinc 3. Ti
D. Cobalt 4. Tl (c) 3 (d) 4
12. The particle in an atom which has negative charge
Codes
(a) electron (b) proton
A B C D
(a) 3 4 1 2 (c) neutron (d) nucleus
(b) 2 1 3 4 13. The symbol of magnesium element is
(c) 4 3 2 1 (a) Na (b) Mo
(d) 4 1 3 2 (c) Mg (d) Mn
6. What is the Latin name of mercury (Hg)? 14. Hydrogen is
(a) Hydragyrum (b) Argentum (a) an element (b) a compound
(c) Aurum (d) Ferrum (c) a mixture (d) None of these
7. The number of atoms present in a molecule of an
element is called 15. Which among the following element has valency of 1?
(a) Magnesium (b) Calcium
(a) allotropy (b) atomicity
(c) Aluminium (d) Sodium
(c) aromaticity (d) None of these

High Skill Questions


1. How many moles are represented by 90 g of glucose 5. If 60g of sugar is dissolved in 180 g of water, the
(C6H12O6 )? concentration of solution is
(a) 1 mol (b) 0.5 mol (a) 60% (b) 40%
(c) 1.5 mol (d) 2 mol (c) 25% (d) 75%
2. Calculate the ratio of molecules present in 4.4 g of 6. How many molecules of H2O2 are present in 68 g of
CO2 and 1.6 g of SO2 . H2O2 ?
(a) 4 : 1 (b) 1 : 4 (c) 8 : 1 (d) 1 : 8 (a) 6.022 × 1023 (b) 12.044 × 1023
3. What is the exact weight of 3 moles of Na? (c) 3.011 × 10 23
(d) 24.088 × 1023
(a) 23 g (b) 46 g (c) 69 g (d) 92 g
7. The cost of sucrose (sugar) C12H22O11 is ` 20 per kg. Its
4. The mass of one atom of an element X, is 2 × 10−23 g. cost per mole is
What could element X be? (a) ` 3.60 (b) ` 7.20
(a) Carbon (b) Nitrogen
(c) ` 4.00 (d) ` 10
(c) Oxygen (d) Sulphur
326 Study Package for NTSE
8. If the mass of 1 mole of water is 18 g, what is the mass 12. Which of the following has largest number of atoms?
of 1 molecule of H2O ? (a) 0.5 g atom of Cu (b) 0.635 g of Cu
(a) 3.99 × 10−23 g (b) 2.99 × 10−23 g (c) 1 g of Cu (d) 0.25 moles of Cu atom
(c) 6.022 × 1023 g (d) 18 g 13. One mole of oxygen gas is the volume of
9. What is the numerical value of Avogadro number? (a) 1 L of oxygen at STP
(a) 6.022 × 10 23
(b) 6.022 × 1024 (b) 32 L of oxygen at STP
(c) 6.022 × 1022 (d) 6.022 × 1025 (c) 22.4 L of oxygen at STP
(d) 6.02 × 1023 molecules at any temperature and pressure
10. Total number of atoms in 44 g of CO2 is
(a) 6.02 × 1023 (b) 6.02 × 1024 14. The volume of 4.4 g of CO2 at STP is
(c) 1.806 × 1024 (d) 18.06 × 1022 (a) 22.4 L (b) 0.224 L
(c) 2.24 L (d) 0.01 L
11. The number of grams of H2SO4 present in 0.25 mole of
H2SO4 is 15. The largest number of molecules is in
(a) 0.245 (b) 2.45 (a) 34 g of H2O (b) 28 g of CO 2
(c) 24.5 (d) 49 (c) 46 g of CH3OH (d) 54 g of N2O 5

Hints and Solutions


Elementary Level
1. (c) Phosphorus exists as P4 molecule. 5. (c) 6. (a) 7. (b) 8. (a) 9. (a)
Thus, its atomicity is 4. 10. (d) 11. (a) 12. (a) 13. (c) 14. (a)
2. (a) 15. (d) Mg (magnesium), 12 = 2, 8, 2
3. (c) In one molecule of SO 3 , Ca (calcium), 20 = 2, 8, 8, 2
Number of O atoms = 3 Al (aluminium), 13 = 2, 8, 3
∴ in 10 molecule of SO 3 , Na (sodium) 11 = 2, 8, 1
Number of O atoms = 10 × 3 = 30 Thus, the valency of Na is 1 (as valency shows the number of
4. (b) Molecular mass of atomic H2SO 4 = atomic mass of H × 2 + electrons present in outermost shell.)
mass atomic of S + atomic mass of O × 4
= 1 × 2 + 32 + 16 × 4 = 2 + 32 + 64 = 98 g

High Skill Questions


ω 90 8. (b) 6.023 × 1023 molecules of water weigh 18 g
1. (b) Number of moles = = = 0.5 mol
m 180 18
1 molecule = = 2.99 × 10−23 g
2. (a) 6.023 × 1023
ω
3. (c) n = , ω = 3 × 23 = 69 g 9. (a) 10. (c)
m
ω 0.25
11. (c) n = , ω = n × m = × 98 = 24.5 g
4. (a) m 100
60
5. (c) Concentration of solution = × 100 12. (c) 13. (c)
180 + 60
14. (c) 44 g of CO 2 occupies 22.4 L at STP
60 100
= × 100 = = 25%
22.4 4.4
240 4 4.4 g of CO 2 occupies × = 2.24 L
44 100
6. (b)
15. (a)
7. (a)
Chapter

9
Matter in Our
Surroundings
Physical Nature of Matter
A substance from which an object is made is called a Properties of Gases and Molecular Arrangement in
material. Anything that has mass and occupies space is Gases In gases, the molecules are much farther apart
called matter. from one another. The force of attraction between the
molecules of a gas is negligible. The spaces between the
Classification of Matter molecules of a gas are very large.
Matter can be classified on the basis of its physical state l Gases neither have a definite shape nor a definite volume.
as solid, liquid or gas. l Gases are highly compressible.
Properties of Solids and Molecular Arrangement in l Gases flow in all directions.
Solids In solids, the molecules are closely packed
together, there is a strong force of attraction between Interconversion of States of Matter
the molecules of a solid which holds them together The change of matter from one state to another state
in fixed position. The space between the molecules is and back to the original state is known as
very small. interconversion of states of matter.
l Solids have definite shape and volume.
Heating Heating
l Solids are rigid (cannot be compressed). Ice s Water s Steam
Cooling Cooling
l Solids do not flow. (Solid form) (Liquid form) (Gaseous form)
Properties of Liquids and Molecular Arrangement in
Concept of Molecules
liquids In liquids the molecules are not very closely
packed like solids. They do not have strong Matter whether solid, liquid or gas is made up of very
intermolecular force of attraction which can hold the small particles called molecules. Some forces of
molecules of liquid together in fixed position. Therefore, attraction exists between the molecules of matter. This
the molecules of a liquid can move freely from one is called intermolecular force of attraction.
position to another. The intermolecular spaces between
The intermolecular forces of attraction tend to hold the
the molecules is larger.
molecules together the molecules of matter are in
l Liquids have definite volume, but no definite shape.
continuous motion. The space between the molecules is
l Liquid is slightly compressible. called intermolecular space.
l Liquids can flow easily.
328 Study Package for NTSE

Evaporation
This method is used to obtain a solid substance that has dissolved in a liquid common salt dissolved in water can be
separated by the process of evaporation. The process of changing a liquid into its vapour by heating is called
evaporation.
The rate of evaporation depends upon the surface area exposed to the atmosphere, the temperature, the humidity and
the wind speed. Evaporation causes cooling.

Points to Remember
● Matter is made up of small particles. Matter around us is present in three phases i.e., solid, liquid and gas.
● The forces of attraction between the particles are maximum in solids, intermediate in liquids and minimum in gases.
● In gases the particles move about randomly.
● The states of matter can undergo interconversion on changing the pressure or temperature.
● Boiling is a bulk phenomenon.
● During sublimation the solid state changes to the gaseous state without going through liquid state.
● Evaporation is a surface phenomenon.
● Latent heat of vaporisation is the heat energy required to change 1 kg of a liquid to gas at atmospheric pressure at its boiling point.
● Latent heat of fusion is the amount of heat energy required to change 1 kg of solid into liquid at its melting point.

Target Exercise
Elementary Level
1. Which of the following has maximum intermolecular 7. On converting the following i.e., 25°C from °celsius to
force? °kelvin, we get
(a) Honey (b) Water (a) 298 K (b) 248 K
(c) Ice (d) Water vapour (c) 246 K (d) 500 K
2. Which of the following is the property of matter? 8. 470 K temperature can be written in the celsius scale
(a) It has mass as
(b) It occupies space (a) 440°C (b) 197°C
(c) Both (a) and (b) (c) 177°C (d) 187°C
(d) None of the above
9. Particles of matter are continuously moving that is,
3. The form of matter which has a definite shape and a they possess what we call
definite volume is called (a) potential energy
(a) solid (b) liquid (b) nuclear energy
(c) gas (d) All of these (c) kinetic energy
4. A liquid has (d) atomic energy
(a) fixed shape and fixed volume 10. The ……… of a solid is an indication of the strength of
(b) no fixed shape but fixed volume the force of attraction between its particles
(c) fixed shape but no fixed volume
(a) boiling point (b) freezing point
(d) neither fixed shape nor fixed volume
(c) transition point (d) melting point
5. The form of matter which neither has a definite shape
11. The physical state of water at 100°C is
nor a fixed volume is called
(a) liquid (b) gaseous
(a) solid (b) liquid
(c) semi solid (d) solid
(c) gas (d) All of these
12. Cotton is a good absorber of water hence it makes
6. Which of the following is the smallest unit of an
……… easier.
element?
(a) Molecule (b) Compound (a) boiling (b) evaporation
(c) Atom (d) Matter (c) vaporisation (d) condensation
Matter in Our Surroundings 329

High Skill Questions


1. What type of clothes should we wear in summer? 6. The intermixing of particles of two different types of
(a) Cotton (b) Nylon matter on their own is called
(c) Woollen (d) Polyester (a) evaporation (b) diffusion
(c) condensation (d) effusion
2. ……… is change from solid to gaseous state without
going through a liquid state. 7. The kinetic energy of the particles increases on
(a) Melting (b) Boiling increasing the ………
(c) Evaporation (d) Sublimation (a) pressure (b) humidity
(c) temperature (d) None of these
3. Why does a palm full cold when we put some acetone
on it? 8. Full form of CNG is
(a) Evaporation takes place causing cooling (a) Combined Natural Gas
(b) Boiling takes place (b) Common Natural Gas
(c) Condensation takes place (c) Compressed Neutral Gas
(d) None of the above (d) Compressed Natural Gas
4. The amount of water vapour present in the air is 9. In homes for cooking the gas used is
known as (a) LPG (b) methane
(a) condensation (b) sublimation (c) CNG (d) ethane
(c) humidity (d) None of these 10. The states of matter are
5. Evaporation increases with the ……… of surface area (a) not known (b) now interconvertible
(a) contraction (b) decrease (c) interconvertible (d) None of these
(c) no effect (d) increase

Hints and Solutions


Elementary Level
1. (c) Among the given intermolecular forces are maximum in ice. 9. (c)
2. (c) 3. (a) 4. (b) 10. (d) Higher the melting point of a solid, stronger is the force of
attraction. Thus, melting point is an indicator of strength of forces
5. (c) 6. (c)
of attraction between its particles.
7. (a) Temperature at Kelvin scale = ° C + 273 = 25 + 273 = 298 K
11. (b)
8. (b) Temperature at celcius scale = K − 273= 470 − 273 = 197 °C 12. (b)

High Skill Questions


1. (a) 8. (d) CNG → Compressed Natural Gas, its major component is
2. (d) Sublimation is the change from solid to gaseous state. methane.

3. (a) 9. (a) LPG → Liquified petroleum gas is used in homes for


cooking and its major component is butane.
4. (c)
10. (c) States of matter are interconvertible.
5. (d) Evaporation is a surface phenomenon and it increases with
the increase of surface area. Heat Heat
6. (b)
Solid state s
Cool
Liquid state s
Cool
Gaseous state

7. (c)
Part-III : Biology
Chapter

1
Fundamental Unit
of Life
Cell
Cell is the structural and functional unit of life. In 1665, Robert Hooke made a chance observation, while
examining a thin slice of cork. It resembled the structure of honey comb consisting of many little compartments.
Free living cells were first discovered by Leeuwenhoek in 1674. In 1831, Robert Brown discovered nucleus in
cell. Purkinje coined the term ‘protoplasm’. Schleiden and Schwann put forward cell theory according to which
all plants and animals are made-up of cell. Virchow furthur expanded this theory by suggesting cells arises from
pre-existing cells.

Types of Cells
Despite the complexity and variety, all living cells can be classifed into two groups namely
1. Prokaryotic cell (Gr Pro = Primitive or pre be before; karyon = Nucleus) These are primitive cells lacking a true
nucleus and most of other cell organelles, e. g., bacterial cell, blue-green algae, mycoplasma etc.
2. Eukaryotic cell (Eu = true; karyon = Nucleus) These have the nucleus and membrane bound cell organelles. These
are present in unicellular and multicellular plant and animal.

Differences between Prokaryotic and Eukaryotic Cell


S.No. Feature Prokaryotic Cell Eukaryotic Cell
1. Size Generally, small in size 1-10 µm. Generally, large in size 10-100 µm.
2. Nuclear Nuclear region is not well defined. Nuclear membrane Nuclear region is well defined and enclosed in a nuclear
region is absent. membrane.
3. Histones Histone proteins are absent. Histone like proteins are Histone proteins are present, which forms an octamer with
attached to nucleic acid. nucleic acid.
4. Membrane Absent Present
bound cell
organelles
5. Nucleolus Absent Present
6. Cell division Fission or budding Mitosis or meiosis
Fundamental Unit of Life 331
Differences between Plant and Animal Cell barrier for internal components and surroundings. The
S.No. Plant Cell Animal Cell cell membrane is also called selectively permeable
membrane.
1. Cell wall is present. Cell wall is absent.
l CO2 and O2 move across the cell membrane by a process
2. Plastids are present. Plastids are absent.
called diffusion (spontaneous movement of a substance
3. Vacuoles are larger in size Vacuoles are smaller in size
and less in number. and more in number. from region of high concentration to a region, where
4. Nucleus lies on one side Nucleus lies in the centre. concentration of solute is low).
in the peripheral cytoplasm. l The movement of H2O across the plasma membrane
5. Plant cell contains several Prominent and highly complex occurs by the process called osmosis. Osmosis is
sub-units of Golgi Golgi apparatus is present movement of solvent (H2O) from region of high
apparatus called near nucleus. concentration through a semi-permeable membrane to a
dictyosomes. region of low water concentration. Movement of water
6. Plant cells are larger in size. Animal cells are comparatively across the cell membrane is affected by the amount of
smaller in size.
solutes present in the cell.
Rough endoplasmic Three types of solution depending upon solute
reticulum Nucleolus
concentration are
Smooth
endoplasmic
Hypertonic Solution When the solute concentration
Chloroplast with the surrounding medium is higher than the solute
reticulum
Nucleus concentration in cell. When cell is put in hypertonic
Golgi
apparatus
solution, the cell lose water by osmosis, e.g., when grape
Nuclear
is put into sugar solution, it becomes flaccid. This
envelope phenomenon is known as exo-osmosis.
Plasma
membrane
Hypotonic Solution If the medium surrounding the
Peroxisome cell has high H2 O concentration than the cell, i.e., very
Middle lamella
Cell wall diluted solution. Water enter the cell through osmosis
Mitochondrion and cell swell up. This process is known as
Cytoplasm Ribosomes
endo-osmosis.
Chloroplast
Structure of plant cell When RBCs are put into hypotonic solution. Water
enters the cell through endo-osmosis and cells bursts.
Microvilli
Isotonic Solution If the solute concentration is same
as that of the cell, there will be no net movement of H2 O.
Golgi Plasma membrane
apparatus Lysosome
There will be no change in the cell size.

Smooth Centriole Cell Wall


endoplasmic
reticulum Ribosomes In plant cells, cell membrane is surrounded by another
Nuclear Mitochondrion wall called cell wall. It is made up of ‘cellulose’ whose
envelope function is to give structural strength to plants. Also
Rough
Nucleus endoplasmic maintains its shape and size and provides rigidity to
reticulum
plant cells.
Nucleolus
When a living plant cell loses H2 O through exosmosis
Cytoplasm
there is shrinkage or contraction of the contents of the
Structure of animal cell
cell away from the cell wall. This phenomenon is known
as plasmolysis.
Cell Structure
Nucleus
Cell contains the following parts
The nucleus is spherical and located in the centre of the
Cell Membrane or Plasma Membrane cell. It is separated from cytoplasm by a membrane
It is the boundary for each cell, present in both animals called nuclear membrane. In nucleus, two components
and plants. Its components are molecules of lipids and are present chromatin material and nucleolus. Nucleolus
proteins. The main function of cell membranes are allows contains RNA and DNA is present as the part of
selective molecules through it, acts as a mechanical chromatin material.
332 Study Package for NTSE

During cell division chromatin material gets organised Secretory vesicles


into chromosomes. Chromosomes contain information trans region

for inheritance of features from parents to next


generation in form of DNA (Deoxyribonucleic Acid) Medial region
molecules.
Golgi sac
Chromosome consist of both DNA and histone proteins
and functional segments of DNA are called genes. In cis region
cis
bacteria, nuclear membrane is absent and nuclear Structure of Golgi apparatus
region is undefined containing only nucleic acid. This
undefined region is called nucleoid.
Group of cisternae is called dictyosome. It is various
Cytoplasm enzymes are present in Golgi apparatus like
It is a jelly-like substance present between the cell (i) Glycosyl transferases
(ii) Sulpho and glucotransferases
membrane and nucleus. Various other organelles of
(iii) Oxidoreductases
cells are present in cytoplasm, viz., mitochondria, Golgi
(iv) Phosphatases
bodies, ribosome, endoplasmic reticulum etc. In (v) Phospholipases
eukaryotes, cell organelles are enclosed in a well defined (vi) Kinases
membrane. In prokaryotes, membrane bound cell
organelles are absent. Lysosomes
These are small vacuoles, which contains strong
Cell Organelles digestive enzymes are also called as digestive bags or
suicidal bags of the cell.
Cell organelles are following types
Mitochondria
Endoplasmic Reticulum (ER) These are rod or spherical-shaped organelles having a
It consists of double membrane. One of its end is double membrane. The inner membrane is folded into
connected to the plasma membrane and another to finger-like structures called cristae but the outer
outer nuclear membrane. Proteins and lipids are membrane is smooth. It contains enzymes for cellular
synthesised by endoplasmic reticulum. The fluid filled respiration, also called as ‘power house of the cell’.
between the membranes are called as cisternae. Mitochondria have their own DNA and it is semi-
There are two types of endoplasmic reticulum autonomous in nature.
(i) Rough Endoplasmic Reticulum (RER) It is Inner membrane
connected to plasma membrane. It looks rough due to the (cristae)
presence of ribosomes attached to its surface. Ribosomes
are the site of protein synthesis. The manufactured
proteins are transported to different parts of the cell
depending upon need.
(ii) Smooth Endoplasmic Reticulum (SER) It is the site
of fat lipid synthesis, protein and lipid together help in
building the cell membrane called membrane biogenesis. Matrix
It also help in detoxification of the xenobiotic compounds. Outer membrane
Ultra structure of mitochondria
Golgi Apparatus
First time observed by Camillo Golgi in 1898. It Plastids
consists of semilunar-shaped cisternae and flat These are present in plant cells only. These are of three
membranes appears in stacked manner. It synthesises types
the complex sugars from simple sugars. It is involved in Chromoplasts Coloured due to presence of pigments.
cell trafficking material synthesised near the
endoplasmic reticulum is packaged and dispatched to Chloroplasts Green-coloured plastids, the green
various targets. colour is due to the presence of chlorophyll.
Fundamental Unit of Life 333
Leucoplasts Colourless plastids, stores fat and starch. Ribosome
These also possess their own DNA. Plastid consist of It synthesise proteins that are used in cytoplasm and
numerous membrane layer thylakoid embedded in a found free in cytoplasm. Ribosomes are formed in
material called the stroma. nucleolus from ribosomal RNAs and proteins.
Smallest cell organelle which is not surrounded by any
Vacuoles cell membrane. In prokaryote, 70 S ribosome is present,
These are small sac-like structures surrounded by a while in eukaryotes 80 S ribosomes is present. Both 70 S
single membrane called tonoplast. In plant cells, large and 80 S ribosome is composed of two subunits.
central vacuole is present and they are permanent but 70 S 80 S
in animal cells they are small and uniformaly
distributed throughout the cytoplasm.
30 S 50 S 40 S 60 S
Centrosome Small sub-unit Large sub-unit Small sub-unit Large sub-unit

It is mainly found in animal cells only. It helps in the ‘S’ stands for Svedberg’s unit, which is a sedimentation
formation of spindle during cell division. coefficient. It gives indirect measurement of density
and size.

Target Exercise
Elementary Level
1. Which of the following is called as ‘digestive bags’ of 8. Lysosomes are called as ‘digestive bags’ because of
the cell? the presence of
(a) Plastids (b) Ribosomes (a) parasitic activity (b) catalytic activity
(c) Lysosomes (d) Peroxisomes (c) hydrolytic activity (d) saprophytic activity
2. Which of the following is called as ‘power house’ of the 9. Smooth endoplasmic reticulum synthesises
cell? (a) proteins (b) steroids and lipids
(a) Cell wall (b) Chromosomes (c) carbohydrates (d) vitamins
(c) Mitochondria (d) Lysosomes
10. Starch grain is developed in
3. Golgi apparatus is absent in (a) mitochondria (b) lysosomes
(a) plants (b) yeast (c) Golgi apparatus (d) chloroplast
(c) blue-green algae (d) cells
11. Which of the following is a hereditary unit?
4. Largest animal cell is (a) Nucleolus (b) Gene
(a) liver cell (b) ostrich egg (c) Chromosome (d) Centrosome
(c) nerve cell (d) chicken egg
12. Plasma membrane is
5. Mitochondria are found in (a) permeable (b) selectively permeable
(a) eukaryotic cell (c) semi-permeable (d) impermeable
(b) prokaryotic cell
13. Undefined region containing only nucleic acid in
(c) plant cells
bacteria is
(d) All of these
(a) nucleoid (b) nucleolus
6. Largest cell in human body is (c) nucleus (d) cytoplasm
(a) DNA (b) kidney 14. In prokaryotes which type of ribosome are present?
(c) nerve cell (d) muscle cell (a) 70 S (b) 80 S
7. Orange-yellow colours of flowers is due to the (c) Both (a) and (b) (d) None of these
presence of 15. In 70 S ribosome what ‘S’ stands for
(a) chlorophyll (b) chloroplasts (a) Svedberg unit (b) sedimentation
(c) leucoplast (d) chromoplast (c) size (d) shape
334 Study Package for NTSE

16. In animal cell, centrosome are involved in the 21. Who suggested that cells arises from pre-existing
(a) protein synthesis cells?
(b) formation of spindle fibres during cell division (a) Robert Brown (b) Virchow
(c) respiration (c) Schleiden and Schwann (d) Robert Hooke
(d) transport of proteins
22. Who discovered nucleus?
17. Membrane which encloses vacuole in plant cells is (a) Robert Brown (b) Virchow
known as (c) Purkinje (d) Robert Hooke
(a) tonoplast (b) chloroplast
23. Who discovered free living cell?
(c) protoplast (d) plasma membrane
(a) Robert Hooke (b) Leeuwenhoek
18. Cell organelle involved in the detoxification of (c) Purkinje (d) Virchow
xenobiotic compounds 24. Who coined the term ‘protoplasm’ for the fluid
(a) RER (b) SER substance of cell?
(c) lysosome (d) centrosome (a) Purkinje
19. Rough endoplasmic reticulum looks rough due to the (b) Schleiden and Schwann
presence of (c) Robert Brown
(a) ribosomes (b) cristae (d) Virchow
(c) gene (d) vacuoles 25. Cell looses water when put in hypertonic solution by
20. Cell theory was put forwarded by a process known as
(a) Robert Brown (b) Schleiden and Schwann (a) hypotonic solution (b) end-osmosis
(c) Purkinje (d) Virchow (c) exo-osmosis (d) reverse osmosis

High Skill Questions


1. Bacteria are not present in highly salted pickles due to 7. What supports the presence of DNA in mitochondria
(a) pickles doesn’t have necessary nutrients and chloroplasts?
(b) bacteria killed by plasmolysis (a) Chloroplasts and mitochondria undergoes meiosis and
(c) photosynthesis doesn’t occurs mitosis independent of nucleus
(d) salt inhibits reproduction (b) Glycolysis occurs in both mitochondria and chloroplasts
(c) Chloroplast and mitochondria both originated as
2. Major role of Golgi complex is independent free-living organisms
(a) in trapping light quanta
(d) ATP is produced in chloroplasts as well as in mitochondria
(b) as energy transferring organelles
(c) in glycosidation of lipids and proteins to produce glycolipids 8. Which of the following shows the selective digestion
and glycoproteins of cytoplasmic organelles by the lysosomes?
(d) in digestive system (a) Autolysis
3. Electron transport system is present in which of the (b) Autophagy
(c) Heterophagy
following cell organelle?
(d) Osmotrophy
(a) Endoplasmic reticulum (b) Centriole
(c) Nucleolus (d) Mitochondria 9. Identify the correctly matched pair
4. Enzyme used for Kreb’s cycle in mitochondria are I. Nucleolus — RNA synthesis
located II. Spherosome — Transport of macromolecules
(a) on inner membrane (b) on outer membrane III. Glyorysome — Glyoxylate cycle
(c) in mitochondrial matrix (d) in perimitochondrial space
IV. Plasmodesmata — Lipid storage
5. In which organelles, chemical energy changes into (a) I, II and III (b) I and II
utilisable energy? (c) II and IV (d) I and III
(a) Microsomes (b) Chromosomes
10. I. Mitochondria contain DNA.
(c) Lysosomes (d) Mitochondria
II. 70 S ribosomes occur in prokaryotes.
6. What happened when green tomatoes turn red? III. Ribosomes are made-up of phospholipids and
(a) Chromoplasts changed into chloroplasts oligosaccharides.
(b) New chloroplasts are made IV. Ribosomes are not found in Protista and Monera.
(c) Chloroplasts are disintegrated and get converted into
chromoplasts
The correct statements are
(d) All of the above (a) I, II and III (b) I and II
(c) II and IV (d) I, II and III
Fundamental Unit of Life 335
11. Unit cell membrane is having average thickness of 18. Fungal cell wall is composed of
(a) 5 Å (b) 250 Å (a) chitin (b) cellulose
(c) 75 Å (d) 25 Å (c) pectin (d) peptidoglycan

12. When dried raisins or apricots are put in plain water, 19. Bacterial cell wall is composed of
what you observe (a) cellulose (b) chitin
(a) cells gains, water and swell up (c) peptidoglycan (d) fructose
(b) it loses H2O and consequently shrinks 20. Much of the DNA is localised in chromosome of
(c) no change occur in cells nucleus. The other major constituent of the
(d) None of the above chromosome is
13. When RBCs are put in hypotonic solution cell are (a) RNA (b) histones
(c) acid protein (d) None of these
likely to get
(a) burst (b) shrink 21. Smooth endoplamsic reticulum is specialised for
(c) swell up and then burst (d) flaccid synthesis of lipids and steroids. These organelles are
14. Cells of plants, fungus and bacteria exist in hypotonic found predominantly in
(a) pancreas (b) ovary
solution without bursting due to the presence of
(c) reticular cells (d) blood
(a) cell wall (b) cell membrane
(c) vacuole (d) tonoplast 22. In a living cell, the site of anaerobic respiration is
(a) cytoplasm (b) mitochondria
15. Which among the following is an organalle within an
(c) cell membrane (d) cell wall
organelle?
(a) Ribosomes (b) Endoplasmic reticulum 23. Chemically mitochondria is consist of
(c) Mitochondria (d) Chloroplast (a) proteins (b) lipids
(c) phospholipids (d) All of these
16. Mitochondria and chloroplast are also known as self
autonomous in nature because 24. Type of ribosomes present in mitochondria and
(a) they can make their own food chloroplast
(b) they have their own DNA and ribosome (a) 70 S (b) 80 S
(c) 60 S (d) 40 S
(c) can exist independently
(d) None of the above 25. In which of the following phase of replication of DNA
take place?
17. In plant cell wall plasmodesmata is made-up of (a) G1-phase (b) G 2 -phase
(a) calcium pectate (b) chitin
(c) S-phase (d) All of these
(c) magnecium pectate (d) silica

Answers
Elementary Level
1. (c) 2. (c) 3. (c) 4. (c) 5. (a) 6. (c) 7. (d) 8. (c) 9. (b) 10. (d)
11. (b) 12. (b) 13. (a) 14. (a) 15. (a) 16. (b) 17. (a) 18. (b) 19. (a) 20. (b)
21. (b) 22. (a) 23. (b) 24. (a) 25. (c)

High Skill Questions


1. (b) 2. (c) 3. (d) 4. (c) 5. (d) 6. (c) 7. (c) 8. (b) 9. (d) 10. (b)
11. (c) 12. (a) 13. (c) 14. (a) 15. (a) 16. (a) 17. (a) 18. (a) 19. (c) 20. (b)
21. (b) 22. (a) 23. (d) 24. (a) 25. (c)
Chapter

2
Tissues
Group of cells having a common origin and performing similar functions are called ‘tissues’. Tissues are a group of
cells with similar embryonic origin and specific functions. The branch of zoology in which tissue are studied is
called Histology.
Tissues are divided into two types
1. Plant Tissues 2. Animal Tissues

Plant Tissues
Plant tissues are the following types

Meristematic Tissues Apical meristem


Group of similar cells, which are in a continuous state of division are called as
meristems or meristematic tissues. They have dense cytoplasm, very less
vacuoles and have large nuclei. It produces new cells for maintenance and Intercalary meristem
repairing of the plants. Depending on the region they are divided into three
apical, lateral and intercalary meristem.
Apical Meristem It is present at the growing tips of stem and roots. It helps in
increasing the length of stem and root.
Lateral Meristem It is responsible for the increase in girth of the stem and
root. Lateral meristem

Intercalary Meristem It is present at the base of leaves or internodes.

Permanent Tissues Location of meristematic tissue


in plant body
The meristematic cells when lose their power of division, become permanent
tissues. These are classified into two types—simple permanent tissue and complex permanent tissue.

Simple Tissues
Group of similar type of cells
(i) Parenchyma The cells of parenchyma are thin-walled living cells found everywhere in plant like, in leaves, roots,
stems, flower and fruits. Parenchyma is oval, round, polygonal or iso-diametric shapes. It stores food and metabolic
waste products like gum. They are loosely packed and contain large intercellular spaces. If it contains chlorophyll and
perform photosynthesis they are known as chlorenchyma. In case of aquatic plants, large air cavities are present
which provide buoyancy to the plant such cells are called arenchyma.
Tissues 337
(ii) Collenchyma These are living cells generally Epithelial Tissues
polygonal in shape and located in stems and roots. It
helps the plants to carry out photosynthesis. It is called as covering tissues and forms a continuous
layer on the entire body. Depending on shape, following
(iii) Sclerenchyma It occurs in the old roots, stems and
are the types of epithelial tissues—squamous
branches. These cells are dead with no protoplasm. Its
main function is to give mechanical support. Walls are
(flattened), cuboidal (cubical), columnar, ciliated and
thickened due to presence of lignin. It is present in glandular epithelial tissues.
husk of coconut. (i) Simple Squamous Epithelium These are
extremely thin and flat. Oesophagus and lining of
Complex Tissues mouth is covered by such cells.
(ii) Stratified Squamous Epithelium These are
Those tissues which are made-up of more than one type
simple squamous epithelium arranged in many layers
of cells. to prevent wear and tear.
(i) Xylem It is responsible for conducting water and (iii) Columnar Epithelium It is present in inner lining
mineral salts. It is made-up of four parts–tracheids, of intestine. In respiratory tract, columnar epithelial
vessels, xylem parenchyma and xylem sclerenchyma. consist of cilia having hair-like projections forming
ciliated columnar epithelium.
(a) Trachieds are elongated or tube-like cells with
thick and lignified walls and tapering ends. (iv) Cuboidal Epithelium Corms the lining of kidney
tubules and ducts of salivary glands.
(b) Vessels are long cylindrical tube-like structure
made-up of many cells. Protoplasm is absent.
Vessels are interconnected through perforations.
Connective Tissues
Vessels are absent in gymnosperm, its a chief Its function is to bind and pack different tissues and
characteristic feature of angiosperms. organs together and support.
(c) Xylem parenchyma cells are made-up of Connective Tissues
cellulose and usually stores food.
(d) Xylem sclerenchyma provides support to the
Proper Supporting Liquid connective
tissue. connective tissues connective tissue tissue
(ii) Pholem It is responsible for conducting sugar from Areolar Bone Blood
Adipose
the leaf. It is made-up of four parts namely–phloem Cartilage Lymph
fibres, pholem parenchyma, sieve tubes and Tendons
companion cells. Gymnosperm lack sieve tube and Ligaments
companion cells.
(a) Phloem fibres are made-up of sclerenchy-
Proper Connective Tissues
matous cells. This types of tissues are defined as
(b) Sieve tube elements are long tube-like (i) Areolar tissue is present in internal organs
structures associated with companion cell. helps in repair of the tissue, present between skin
and muscles, around blood vessels and nerves and
(c) Companion cells help in maintaining in the in the bone marrow.
pressure gradient in the sieve tubes.
(ii) Adipose tissue are found below the skin and
(d) Gymnosperm have albuminous cells in place between internal organs. Cells of this tissue is
of companion cells. filled with fat globules.
(e) Phloem parenchyma is absent in most of the (iii) Tendons connects muscle to the bones having
monocotyledons. great strength but limited flexibility.
(iv) Ligament connects bone to bone. It is very elastic
with considerable strength.
Animal Tissues
Animal tissues are the following types Supporting Connective Tissues
Animal Tissues (i) Bone are porous tissues, it contains blood vessels
and nerves. Its function is to form the skeleton and
help in locomotion.
Epithelial Connective Muscular Nervous (ii) Cartilage is composed of proteins and sugars. It
tissues tissues tissues tissues smoothens bone surfaces at joint and is also
present in nose, ear, trachea and larynx.
338 Study Package for NTSE

Liquid Connective Tissue


(i) Blood It is a bright red colour fluid, a connective tissue. It has three types of cells.
(a) Red Blood Cells (RBCs) or Erythrocytes.
(b) White Blood Cells (WBCs) or Leucocytes.
(c) Blood platelets. Its function is to transport oxygen and carbon dioxide and regulate temperature of the
body. Haemoglobin is a complex molecule, includes iron, proteins called globin.
(ii) Lymph does not contain RBC. It is a colourless fluid. It has lower protein contents. It contain very small amount
of nutrient and oxygen but contains abundant CO2 and other metabolic wastes.

Muscular Tissues
Muscles of the body are made-up of muscle cells. Following are the types of muscular tissues
(i) Striated Muscle These are also known as skeletal muscle. These are uninucleated cells.
(ii) Cardiac Muscle This muscle is present in the heart, therefore, also called heart muscles. Cells are in long
and faint striations are present.
(iii) Unstriated Cells These cells are called as smooth muscles. These are found in ureter, bronchi, stomach, etc.
These cells are long with pointed ends.

Nervous Tissues
The cells of nervous tissues are known as neurons. It is responsible for transmission of nerve message, which is
nerve impulse. Neuron consist of a cell body with a nucleus and cytoplasm from which long thin hair parts arises.
Each neuron has single long part called axon and many short branched parts called dendrites.

Target Exercise
Elementary Level
1. The difference between collenchyma and parenchyma 6. Which of the following blood pigments contains copper?
is (a) Haemoglobin (b) Chlorocruorin
(a) cellulose walls (c) Haemocyanin (d) Haemoerythrin
(b) vacuoles 7. Bones formed from the cells called
(c) pectin deposits at corners
(a) chondroclasts (b) osteoclasts
(d) living protoplasm
(c) chondroblasts (d) osteoblasts
2. From the following, best methods of determining the
8. Tendon is connected with
age of tree is to
(a) bones with muscles (b) ligaments with muscles
(a) measure its height
(c) bones with bones (d) cartilage with muscles
(b) count the number of leaves
(c) measure its diameter 9. Ligaments contain
(d) count the number of annual rings in the main stem at the base (a) yellow fibres only
3. Lateral meristem is due to growth (b) white fibres only
(c) white and some yellow elastic fibres
(a) length (b) cortex
(d) yellow fibres and muscle fibres
(c) thickness (d) collenchyma
10. Blood is ............ in nature.
4. Commercial cork is obtained from which of the
(a) alkaline (b) acidic
following species?
(c) neutral (d) variable
(a) Berberis (b) Betula (c) Salix (d) Quercus
11. Which of the following is a tissue?
5. In haemoglobin, iron is present as
(a) Lung (b) Kidney
(a) ferric ions only
(c) Pancreas (d) Blood
(b) unionised iron atom
(c) ferrous ions only 12. Life span of RBC is
(d) ferric or ferrous ions depending upon the oxygenated state (a) 75 days (b) 50 days
of the haeme moiety (c) 120 days (d) 100 days
Tissues 339
13. Tissue forming glands are 19. Wrinkles in old age, appears due to
(a) epithelial (b) muscle (a) keratin (b) actin
(c) nervous (d) connective (c) myosin (d) collagen
14. A tissue is called as 20. Camel’s hump is made-up of a tissue, which provides
(a) layer of cells surrounding an organ water when oxidised
(b) sheet of cells, one layer thick (a) adipose (b) areolar
(c) group of similar cells functions in specialised manner (c) skeletal (d) muscular
(d) group of separate organs 21. Body is warm due to
15. Tissue forming inner lining of blood vessels are (a) hairs (b) sweat glands
(c) adipose tissue (d) connective tissue
(a) nervous (b) connective
(c) epithelial (d) muscle 22. Cork tissue is also named as
16. In man, thickest skin part is (a) periderm (b) phelloderm
(c) phellem (d) phellogen
(a) thumb (b) sole
(c) thigh (d) palm 23. Plant cells without nucleus is
17. Horns of mammals are made-up of (a) root hair (b) xylem vessels
(c) companion cells (d) cambium cells
(a) chitin (b) connective tissues
(c) cartilage (d) keratin 24. Companion cells is the characteristic feature of
18. Tissue forming external ear’s framework is (a) albunious cell (b) gymnosperms
(c) angiosperms (d) monocots
(a) muscle
(b) nervous 25. Vessels are absent in
(c) epithelial (a) gymnosperms (b) angiosperms
(d) connective (c) dicots (d) monocots

High Skill Questions


1. Which of the following are smooth muscle fibres? 7. Due to which of the following reasons, grafting is not
(a) Cylindrical, striated, unbranched and multinucleate possible in monocots?
(b) Spindle-shaped, unbranched, unstriated, uninucleate and (a) Have parallel venation
involuntary (b) Lack cambium
(c) Spindle-shaped, unbranched, non-striated, multinucleate (c) Herbaceous
and involuntary (d) Having scattered vascular bundles
(d) None of the above
8. Sieve tubes are mostly used for translocation because
2. Autonomic genome system is present in these
(a) Golgi body and mitochondria (a) possess bordered pits
(b) ribosomes and chloroplast (b) possess no end walls
(c) mitochondria and chloroplast (c) are broader than long
(d) mitochondria and ribosome (d) possess a broader lumen and perforated cross walls
3. How many nucleus are present in striped muscle 9. Connective tissue
fibre? (a) transport various materials in body
(a) One nuclei (b) Two nuclei (b) provides the body a supporting skeletal framework
(c) Many nuclei (d) No nuclei (c) act as packing material in various organs
4. Triceps and biceps are the examples of which of the (d) allow the absorption of nutrients from the gut
following? 10. Consider the following statements
(a) Involuntary muscles (b) Antagonistic muscles I. Aerenchyma is present in Potamogeton.
(c) Smooth muscles (d) Sphincter muscles
II. Aerenchyma is also found in Neptunia.
5. The surface of myelin sheath bears narrow areas III. Air containing tissue is found in Vallisneria.
called IV. Aerenchyma is present in Bryophyllum.
(a) Schwann nodes (b) Schwann cells
(c) nodes of Ranvier (d) Nissl’s granules Which of the statements given above is/are correct?
6. Striated muscles are found in which of the following (a) I, II and III
parts? (b) I and II
(c) II and IV
(a) Lungs (b) Walls of bronchi
(d) I and III
(c) Leg muscles (d) Gall bladder
340 Study Package for NTSE
11. The exchange of gas takes place due to which type of (c) dicot stem
tissue? (d) upper epidermis of dicot leaves
(a) Muscle (b) Connective 14. Sebaceous glands are found in
(c) Nervous (d) Epithelial (a) dermis of skin of mammals
12. Sieve tubes are having (b) epithelium of intestine of frog
(c) epithelium of stomach of frog
(a) apical and oblique plates
(d) epidermis of skin of mammals
(b) perforated and oblique septa
(c) simple oblique wall 15. Cardiac muscle contains branched fibres which are
(d) perforated and longitudinal plates (a) striated and under voluntary control
13. Bulliform cells are present in (b) striated and not under voluntary control
(c) non-striated and not under voluntary control
(a) lower epidermis of monocot leaves
(d) non-striated and under voluntary control
(b) upper epidermis of monocot leaves

Answers
Elementary Level
1. (b) 2. (d) 3. (c) 4. (d) 5. (d) 6. (c) 7. (d) 8. (a) 9. (a) 10. (b)
11. (d) 12. (c) 13. (d) 14. (c) 15. (c) 16. (b) 17. (a) 18. (d) 19. (d) 20. (a)
21. (c) 22. (b) 23. (b) 24. (c) 25. (b)

High Skill Questions


1. (b) 2. (c) 3. (a) 4. (b) 5. (c) 6. (c) 7. (b) 8. (d) 9. (a) 10. (d)
11. (b) 12. (b) 13. (b) 14. (a) 15. (c)
Chapter

3
Life Processes
Various metabolic processes, which together perform in an order to maintain life is called life processes.
Maintenance of life requires processes like nutrition, respiration, transport of material within the body and
excretion of waste products.

Nutrition l The energy generated during light reaction is utilised to


fix CO2 and converting it into starch. This set of reaction
In order to maintain a state of order in our body, energy occurs in stroma and known as dark reaction.
is needed. This demand of energy is fulfilled by food. However, this reaction is dependent on light reaction to
Some organisms can make their own food by utilising get assimilatory powers (ATP and NADPH).
inorganic sources in form of CO2 and H2 O and
converting them into stored forms of energy. Such Heterotrophic Nutrition
organisms are called autotrophs (like green plants and l Heterotrophs are dependent on autotrophs to get food,
some bacteria). directly or indirectly. Some organism directly eats
plants. Such organism are known as herbivores.
There are some organism are dependent on autotrophs
l There are some organism are dependent on herbivores
to meet their energy demands. They consume complex
to get food. Such organism are known as carnivores.
food material and breakdown into simpler ones. Such
There is wide range of strategies by which food is taken
organisms are known as heterotrophs.
in and utilised by organism. In unicellular organism, the
food is taken in by the entire surface.
Types of Nutrition
l Amoeba takes in food using temporary finger-like
Nutrition are divided into following two categories projections called psuedopodia. In case of Paramecium,
food is taken at a specific spot.
Autotrophic Nutrition
l Autotrophs make their own food through a process
known as photosynthesis. They convert CO2 and H2O Nutrition in Human
into carbohydrates and O2 in presence of sunlight and
The human digestive system consist of an alimentary
chlorophyll. General reaction which occur during this
process is canal and the associated glands.
Chlorophyll
6CO2 + 12H2O → C6H12O6 + 6O2 + 6H2O Alimentary Canal
Sunlight
It begins with anterior opening, called mouth and it
l This reaction take place in chloroplast and stromal
opens out posterior through the anus. Firstly, food is
lamella and grana constitutes the membranous system.
This is responsible for trapping the light energy and for taken in mouth. Food is crushed with our teeth. To make
the synthesis of ATP and NADPH. This set of reactions its passage smooth food is wetted by saliva secreted by
is known as light reaction. salivary gland. It contains an enzyme called salivary
amylase.
342 Study Package for NTSE

That breakdown starch which is a complex molecule structure called alveoli. Exchange of gases take place at
into simpler sugar molecule. From mouth food is taken alveoli. Blood brings CO2 from rest of the body and takes
to stomach Muscular walls of stomach expand when up O2to transport it to other parts of the body. In blood,
RBC respiratory pigment haemoglobin is present,
food enters it.
which has high affinity for O2. It takes up O2 from lung
(i) Gastric glands present in the wall of the and carry it to tissues which are O2 deficient.
stomach releases HCl (Hydrochloric Acid) and
Glucose In cytoplasm 2 Pyruvate
protein digesting enzyme called pepsin. HCl (6 C) 3 (C)
provides an acidic medium to facilitate the (glycolysis)
+
action of pepsin. Mucus secreted by this gland (Breakdown Energy
protects the inner lining of stomach from acidic of glucose) (ATP)
medium. From stomach food enters small (In cytoplasm)
intestine, it is the longest part of alimentary
In absence of O2
canal. Herbivores have longer small intestine in
(2C) Ethanol
comparison to carnivores. Complete digestion of Anaerobic + CO2 + ATP
carbohydrates, proteins and fats occurs in small respiration
intestine.
Lack of O2
(ii) Pancreas secretes pancreatic juice, which (3C) Lactic acid
contain trypsin and lipase. Trypsin digest (in muscle
cells)
+ energy
proteins while lipase digest fats.
(iii) Liver also secretes bile juice into small In presence
intestine, which is used to emulsify fats. of O2
CO2 + H2O
Glands present in small intestine also produce (in mitochondria) + energy
intestinal juice. It converts carbohydrate into (Krebs’ cycle)
sugar protein into amino acids and fats into Breakdown of glucose by various pathways
fatty acid and glycerol.
The inner lining of small intestine contain
microscopic projections called villi, which Transportation in Human
absorbs digested food. Undigested food is sent to
large intestine, where food and water is In humans, blood is the chief connective tissue, which
absorbed by villi. The undigested food material transports food, carbon dioxide (CO2 ) and waste
is removed from the body via anus. material. O2 is carried by RBC and plasma transports
food, CO2 and nitrogenous waste. For proper circulation
of blood throughout the body. A pumping organ is
Respiration needed. This pumping organ is our heart. Humans have
Respiration is breakdown or oxidation of food into CO2 four-chambered heart with two atria and two ventricle.
and H2 O. Some organism utilise O2 to breakdown The deoxygenated blood comes from body to the upper
glucose into CO2 and H2 O. Such process is called right atrium and from right atrium it reaches to right
aerobic respiration. There are some organism in ventricle. Right ventricle pumps deoxygenated blood to
absence of O2 convert pyruvate into CO2 and ethanol. lungs through pulmonary artery. From lungs
Such process is called anaerobic respiration. Common oxygenated blood enters left atrium. Then it moves to
step in both the reaction is breakdown of glucose (6 C) left vertricle. From left ventricle oxygen rich blood is
into 3C pyruvate. pumped to various parts of the body through aorta.
l ATP acts as energy currency and utilised in various l Pulmonary circulation is done by right atrium and
endothermic processess. O2 is absorbed by different right ventricle which move blood from right ventricle to
organs in different animals. Aquatic organisms take O2 lungs. Left atrium and left ventricle. Powers systemic
through gills. In case of humans, O2 is taken into body circulation, which moves blood from left ventricle to the
through nostrils. Hair projection present inside nostrils tissues of the body. Tricuspid valve is present between
contain mucous, which helps in filtering the air. From right atrium and right ventricle.
nose air passess through the throat and finally into
lungs. l Bicuspid or mitral valve is present between left atrium
and left ventricle. When blood fill the two atria, atria
l In throat, air passes through trachea, which divides into
and ventricles are relaxed called artrial diastole.
right and left primary bronchi. Bronchi further
When artria contracts simultaneously causes artrial
undergoes repetitive division and divided into
bronchioles. Bronchioles give rise to bag-like small systole.
Life Processes 343
l When ventricles contracts and AV value is closed. A cross section of a kidney shows two distinct regions–an
Ventricular systole occurs. At this points first heart outer dark granular cortex and an inner lighter medulla.
sound lub arises. When ventricles relax ventricles
diastole occurs. The normal systolic pressure is about The region where ureter leaves the kidney is a hollow
120 mm Hg and distolic pressure is 80 mm Hg. Blood space, which is called pelvis. Each kidney is made up of
pressure is measured by an instrument called numerous (about one million) coiled tubules or
sphygmomanometer. nephrons which are known as the structural and
l Arteries carries the oxygenated blood and are thick, functional unit of the kidney.
while veins carries deoxygentated blood. Smaller vessels Kidneys have two types of nephrons–cortical (80-85%)
which are one cell thick are called capillaries. Apart
and juxta medullary nephrons (15-20%). Each nephron
from blood another types of fluid involved in
transportation is lymph. From walls of the capillaries has two parts–Malpighian corpuscle and renal tubule.
some amount of plasma, protein and blood cell reaches Malpighian corpuscle consists of glomerulus and
the interstitial space to form colourless fluid called Bowman’s capsule. Renal tubule is long tube
lymph. consisting of PCT (Proximal Convoluted Tubule),
Henle’s loop, DCT (Distal Convoluted Tubule) and
Transportation in Plants collecting tubule.
In case of plants, water and food is transported to the
different parts of plant. Water is transported through Ureters
xylem tissues, vessels and trachieds. These are These are two in number and each emerges from their
interconnected to form continuous system of H2 O respective kidneys. Each ureter opens their respective
conducting channels reaching all the parts of the body. kidneys to urinary bladder and thus, helps in transfer of
l At roots cells, which are close to the soil particles urine from kidney to urinary bladder.
actively takes up ions. So solute concentration within
these cells is high. H2O from soil enters to the cell. H2O is Urinary Bladder
also lost by stomata of the leaves, which creates a
suction which pulls water from xylem cells of the roots. It is one in number sac-like bean-shaped muscular
l This process of evaporation of H2O through stomata is structure. It has three openings, two openings due to two
known as transpiration. Transport of food material ureters coming from kidneys and one opening through
occurs through phloem and known as translocation. which the urethra leaves the bladder. The main function
l It take place in sieve tubes with the help of adjacent of urinary bladder is to store urine temporarily.
companion cells both in upward and downward
direction. Food is transported in form of sucrose by Urethra
utilising energy.
It is a canal-like structure present in both male and
female, longer in male than female. Urethra extends
Excretion in Human from neck of urinary bladder to the exterior and helps in
excretion of urine.
Excretion is the process of elimination of metabolic
wastes from the body.
Organism eliminate N2 in form of Excretion of Urine
Ammonia → Ammonotelism
Kidneys get oxygenated blood and nitrogenous waste
Urea → Ureotelism materials by renal artery. Blood is filtered from the
Uric acid → Uricotelism blood capillaries into Bowman’s capsule. After
Excretory system of human beings include pair of filtration, the filtrate passes through the tubular parts
kidneys, pair of ureters a urinary bladder and urethra. of the nephron.
Useful products such as glucose, amino acids, water,
Kidneys sodium ions, etc., are reabsorbed while the waste
Kidneys are solid, bean-shaped, reddish brown, paired materials (urine) drain by nephrons into a space inside
structures lying in the abdominal cavity one on either kidney leading to the ureter. From the ureters, urine
side of the vertebral column. Each kidney is enclosed in passes into the urinary bladder, where it remains
a thin, fibrous covering or capsule. Oxygenated blood temporarily stored till micturition (expulsion of urine
and nitrogenous waste materials are supplied to the from the urinary bladder).
kidney by renal artery.
344 Study Package for NTSE

Excretion in Plants
Plants use a variety of techniques to excrete (i.e., get rid of waste materials). Oxygen generated during
photosynthesis can be thought as a waste product. Plants can get rid of excess water by transpiration. Some other
waste materials may be stored in the cell-vacuoles or as gums and resins in old xylem, removed in the falling leaves
or excreted into the surrounding soil.

Target Exercise
Elementary Level
1. Autotrophic mode of nutrition requires 12. Lactic acid accumulation occurs in muscle due
(a) CO 2 and H2O (b) chlorophyll (a) aerobic respiration (b) anaerobic respiration
(c) sunlight (d) All of these (c) fermentation (d) None of these
2. The breakdown of pyruvate to give CO2 , H2O and 13. In higher mammals, in the transport of oxygen,
energy take place in haemoglobin plays an important role. The metal
(a) cytoplasm (b) mitochondria associated with haemoglobin is
(c) chloroplast (d) nucleus (a) magnesium (b) iron
(c) manganese (d) copper
3. The xylem in plants is responsible for
(a) transport of H2O 14. Which one of the following binds with haemoglobin
(b) transport of food irreversibly?
(c) transport of amino acids (a) Carbon dioxide (b) Carbon monoxide
(d) transport of oxygen (c) Ethane (d) Nitrogen
4. The phloem in plants is responsible for 15. The tricuspid value occurs between the
(a) translocation of food (b) transport of H2O (a) right auricle and right ventricle
(c) transport of O 2 (d) transport of amino acids (b) pulmonary aorta
(c) cortico-systemic aorta and left ventricle
5. The kidneys in human being are the part of (d) left ventricle
(a) excretory system (b) respiratory system
(c) circulatory system (d) nutrition 16. Indicate correct statement for man
(a) Arteries always carry deoxygenated blood, while veins
6. Dark reaction in photosynthesis always carry deoxygenated blood.
(a) is dependent on light reaction (b) Arteries are provided with valves, while veins are devoid of
(b) is independent of light reaction
valves.
(c) occurs only in dark
(d) None of the above (c) Arteries always carry blood away from the heart, while veins
always carry blood towards the heart.
7. Site of light reaction in photosynthesis is
(d) Venous blood is returned to left auricle.
(a) stroma (b) stroma lamellae
(c) grana (d) Both (b) and (c) 17. The smallest blood vessel in the body is a
8. Which gland releases HCl and pepsin in stomach? (a) capillary (b) artery (c) vena cava (d) vein
(a) Pancreas (b) Gastric 18. The carbohydrate synthesised in the leaves are
(c) Liver (d) Salivary gland transported through sieve tubes most commonly in
9. Bile juice is secreted by the form of
(a) liver (b) gall bladder (a) glucose (b) triose sugar
(c) pancreas (d) stomach (c) sucorse (d) soluble starch
10. Basic functional unit of kidney is 19. Which of the following is used for determining the
(a) neuron (b) nephron rate of transpiration in plants?
(c) capillaries (d) None of these (a) Tensiometer (b) Auxanometer
(c) Porometer (d) Potometer
11. Blood pressure is measured with an instrument
called 20. Which one of the following is the most common type of
(a) spirometer (b) tensiometer transpiration?
(c) sphygmomanometer (d) sthescope (a) Stomatal (b) Lenticular
(c) Foliar (d) Cuticular
Life Processes 345

High Skill Questions


1. Transpiration rate is dependent upon 5. Which of the following processes keeps plant cool?
I. stomatal frequency (a) Transpiration (b) Guttation
II. position of stomata (c) Photosynthesis (d) Translocation
III. state of stomata 6. The process of the escape of liquid from the tip of
IV. epidermal cells uninjured leaf or through hydathodes is called
The correct statement is (a) transpiration (b) guttation
(a) I, II and III (b) I and II (c) evapo-transpiration (d) evaporation
(c) II and IV (d) I and III
7. Body tissues obtain oxygen from haemoglobin
2. Match the different type of instrument with their use because of its dissociation in tissues caused by
(a) low oxygen concentration and high carbon dioxide
List I List II concentration
(b) low oxygen concentration
A. Auxanometer 1. Measurement of imbibition
(c) low carbon dioxide concentration
pressure
(d) high carbon dioxide concentration
B. Potometer 2. Measurement of blood
pressure 8. The principal pathway by which water is
C. Dilatometer 3. Measurement of transpiration translocated in angiosperms is
rate (a) xylem vessel system
D. Sphygmomanometer 4. Measurement of diffusion (b) xylem and phloem
(c) sieve tubes members of phloem
5. Measurement of growth in
(d) sieve cells of phloem
plants
9. Phenylmercuric acetate
Codes (a) reduces transpiration rate (b) reduces photosynthesis
A B C D A B C D (c) kills the plant (d) reduces respiration
(a) 5 1 3 2 (b) 4 3 2 1
(c) 5 3 2 1 (d) 5 3 1 2 10. Rate of transpiration in a dorsi-ventral leaf is
(a) greater at the upper surface
3. The mechanism of uric acid excretion in nephron is (b) greater at the lower surface
(a) diffusion (b) ultrafiltration (c) equal at both the surfaces
(c) osmosis (d) secretion (d) None of the above
4. The conversion of protein waste and the ammonia 11. Which one of the following is responsible for
into urea occurs mainly in guttation?
(a) kidney (b) lungs (a) Root pressure (b) Transpiration
(c) liver (d) intestine (c) Photosynthesis (d) Osmosis

Answers
Elementary Level
1. (d) 2. (b) 3. (a) 4. (a) 5. (a) 6. (a) 7. (d) 8. (b) 9. (a) 10. (b)
11. (c) 12. (b) 13. (b) 14. (b) 15. (a) 16. (a) 17. (a) 18. (c) 19. (a) 20. (a)

High Skill Questions


1. (a) 2. (d) 3. (b) 4. (c) 5. (a) 6. (b) 7. (a) 8. (a) 9. (a) 10. (a)
11. (a)
346 Study Package for NTSE

Chapter

4
Control and Coordination
Nervous System At the end of the axon it releases neurotrans mitters,
which cross the gap of synapse and starts same impulse
In animals, control and coordination is provided by in the consecutive next neuron.
nervous and muscular tissues. All information from our
environment is detected by specialised tips of nerve
Reflex Action
cells. Receptors are usually located in our sense organs It is the process of detecting the signal or the input and
such as the inner ear, the nose, the tongue to receive the responding to it by an out put action, which might be
stimulus. completed quickly.
l Structural and functional unit of nervous system is The connection between input nerve, which detects the
neuron. Neuron is largest cell in the human body. signal and output nerve, which responds towards the
Neuron has two main components—axon and cyton.
signal is known as reflex arc.
Cyton consist of dendrites and nucleus. Axon consist of
long extension of cyton having medullary sheath and Nervous system divided into three parts
nodes of Ranvier. (i) Central Nervous System (CNS)
(ii) Peripheral Nervous System (PNS)
Nucleus
Dendron (iii) Autonomic Nervous System (ANS)
Dendrites Central nervous system consists of brain and spinal
Cyton
Axon
Nissl’s granules cord. Peripheral nervous system consists of cranial
Nucleus and spinal nerves. Autonomic nervous system is of
Medullary sheath two types, i.e., sympathetic and parasympathetic. Brain
Node of Ranvier is covered by protective layer called meninges.
Cerebrum


Cranium (skull)

Axon endings


Midbrain
Forebrain
Synapse
Neuron Hypothalamus
Pituitary gland
Spinal cord
l Stimulus is acquired at the end of the dendritic tip of  Pons
nerve cell, it sets off a chemical reaction, which creates an Hindbrain  Medulla
Cerebellum
electric impulse. This electric impulse travel from
dendrite to the cell body and then along the axon to its Brain
end.
Control and Coordination 347
Human brain is divided into three parts Hormones
(i) Forebrain
These are chemical substances secreted by certain
Forebrain is classified as
glands directly into the blood stream. These are called
(a) Cerebrum — Muscular activities
endocrine glands or ductless glands.
(b) Parietal lobe — Touch, smell and temperature
(c) Temporal lobe — Hearing Endocrine Glands
(d) Occipital lobe — Seeing
It is found in human body in form of hypothalamus,
(e) Olfactory lobe — Smell
pituitary gland, pineal gland, thyroid gland,
(ii) Midbrain parathyroid gland, thymus gland, pancreatic islets,
In mid brain thalamus and hypothalamus makes a respiratory adrenal gland, ovaries and testes.
centre controls that pituitary gland (i) Pituitary gland It secrete a growth hormone.
(iii) Hindbrain (ii) Thyroid gland (thyroxine hormone) It regulate
carbohydrate, protein and fat metabolism and
It can be classified as
provide a balance of growth.
(a) Cerebellum Movements responsible for voluntary
(iii) Pancreas (Insulin) It helps in regulating the blood
actions and maintaining posture and balance of the
sugar.
body.
(iv) Adrenal gland It secretes adrenaline. It increases
(b) Pons Regulate respiration
heart beat due to contraction of muscles around small
(c) Medulla oblongata Swallowing, coughing, sneezing arteries.
and vomiting central involuntary action.

Pineal body
Pituitary
Parathyroid
Thyroid
Thymus
Stomach
Adrenal
Pancreas
Kidneys
Intestine

Testes
Ovaries

Endocrine glands in male and female

Movement in Plants (i) Phototropism movement in response to light.


(ii) Geotropism movement in response to gravity.
The situations, which initiate a response are called
(iii) Chemotropism movement in response to
stimuli. The reaction to situation is called response. chemical stimulus.
Plants do not bear a nervous system, their response (iv) Hydropism growth of roots towards soil moisture.
mechanism is controlled by hormones. This is called (v) Thigmotropism growth response to touch.
chemical coordination and the chemical substance are (vi) Seismonasty nastic response resulting from
called as phytohormones. Movement of a plant in contact or mechanical disturbance.
response to a stimulus is called tropism, e.g.,
348 Study Package for NTSE

Nastic Movement
This movements is neither towards nor away from the stimulus, e.g., leaves of ‘touch me not’ (Mimosa pudica)
plant. The length of the day during which sunlight is available to plants is photoperiod and the effect of photoperiod
on development of plants is called photoperiodism.

Phytochrome
Plant pigment that is a photoreceptor of red light and is involved in a number of developmental process. Such as
flowering, dormancy and leaf formation.

Target Exercise
Elementary Level
1. Which of the following is a plant hormone? 12. Which of the following acts as both exocrine and
(a) Insulin (b) Thyroxin endocrine glands?
(c) Oestrogen (d) Cytokinin (a) Liver (b) Pancreas
(c) Spleen (d) Adrenal
2. Gap between two neurons is called a
(a) dendrite (b) synapse 13. Phototropic movements of roots and stems are due to
(c) axon (d) impulse (a) action of gravity
(b) effect of light
3. Brain is responsible for (c) differential hormonal effect
(a) thinking (b) regulating the heart beat
(d) epinasty and hyponasty
(c) balancing the body (d) All of these
14. Bending of growing shoot towards sunlight is called
4. The longest cell in the body of an animal is (a) heliotropism (b) hydrotropism
(a) osteocyte (b) neuron (c) photonasty (d) phototropism
(c) chromatophore (d) blood corpuscle
15. Nastic movements are
5. Which cell stops dividing after birth? (a) towards the stimulus
(a) Glial cells (b) Epithelium (b) away from stimulus
(c) Liver (d) Neuron (c) independent of direction of stimulus
6. The largest number of cell bodies of neurons in our (d) None of the above
body is found in 16. Endocrine glands put their secretion directly into
(a) brain (b) spinal cord (a) ducts (b) blood
(c) tongue (d) retina (c) Both (a) and (b) (d) None of these
7. Learning is related to 17. Nastic response resulting from contact or mechanical
(a) cerebrum (b) cerebellum disturbance is known as
(c) pons (d) medulla oblongata (a) phetotropism (b) geotropism
8. Voluntary movements in human beings are (c) chemotropism (d) seismonasty
controlled by 18. Adrenaline causes
(a) cerebrum (b) cerebellum (a) increase in heart beat (b) decrease in heart beat
(c) pituitary (d) spinal cord (c) no change in heart beat (d) None of these
9. The brain is protected by 19. Brain is covered by a protective layer
(a) spinal fluid (b) cerebrospinal fluid (a) meninges (b) cranial nerves
(c) peptoneal fluid (d) peritoneal fluid
(c) myelin sheath (d) All of these
10. Which is also called ‘emergency gland’? 20. Which of the following acts as a photoreceptor of red
(a) Pituitary (b) Thyroid
light and involved in flowering?
(c) Pancreas (d) Adrenal
(a) Phytochrome
11. Hormone insulin is secreted by (b) Cryptochrome
(a) liver (b) pancreas (c) Chlorophyll
(c) spleen (d) adrenal (d) Haemoglobin
Control and Coordination 349
High Skill Questions
1. The information pairing through neurons is in the 10. Which of the following statements are correct about
form of body coordination
(a) chemical and electrical signal I. Controlled by nervous system.
(b) chemical and electronic signal II. Controlled by circulatory system.
(c) thermal and electrical signal III. Maintained by endocrine system.
(d) chemical signal IV. Maintained by excretory system.

2. The neurons that transmit impulses from the The correct statements are
central nervous system towards the muscle cells are (a) I, II and III (b) I and II
called (c) II and IV (d) I and III
(a) sensory neurons (b) motor neurons 11. Anterior lobe of pituitary gland secrete by
(c) relay neurons (d) synapse
I. ACTH, TSH and oxytocin II. STH, GH and TSH
3. The characteristic feature of axon is III. TSH, ADH and prolactin IV. FSH, GH and LH
(a) receiving impulse (b) conducting impulse
Which of the following is correct?
(c) hormone secretion (d) electrolyte balance
(a) I, II and III (b) II and IV (c) III and IV (d) I and III
4. Centre of thirsty and hunger are located in
(a) cerebellum (b) cerebrum
12. Photoperiodism is
(a) recurrence of day and night
(c) medulla oblongata (d) hypothalamus
(b) effect of day length on flowering of a plant
5. Flowering and seed germination are regulated by (c) flowering plant
(a) geotropism (b) phototropism (d) growth curvature in response to light
(c) photoperiodism (d) photosynthesis 13. Pneumatophores are
(a) positive geotropic (b) negative phototropic
6. Which coordination system is present in plants?
(c) thigmotropic (d) ageotropic
(a) Nervous coordination (b) Chemical coordination
(c) Both (a) and (b) (d) None of these 14. Melatonin and serotonin, the recently reported
hormones is secreted by
7. The nerves which carry message from brain to (a) thymus (b) pineal (c) adrenal (d) pituitary
effector organ are called
(a) sensory nerves (b) motor nerves 15. Which of the following compounds is involved in
(c) sympathetic nerves (d) parasympathetic nerves decreasing blood glucose levels?
(a) Insulin (b) Glycagon
8. The region where the nerve endings of one neuron
(c) Glucogen (d) Parathyroid hormone
come into contact with another neuron is called as
(a) cyton (b) receptor 16. Which of the following pairs is/are correctly matched?
(c) effector (d) synapse
9. Increase in heart beat after seeing an accident is due S.No. Gland Hormone
to
I. Pituitary Follicle stimulating hormone
(a) sympathetic response
(b) parasympathetic response II. Thyroid Somatotrophic hormone
(c) hormonal response III. Parathyroid Thyroxine
(d) All of the above (a) Only I (b) II and III (c) Only III (d) I, IV and III

Answers
Elementary Level
1. (d) 2. (b) 3. (d) 4. (b) 5. (d) 6. (a) 7. (a) 8. (a) 9. (b) 10. (d)
11. (b) 12. (b) 13. (c) 14. (a) 15. (c) 16. (b) 17. (d) 18. (a) 19. (a) 20. (a)

High Skill Questions


1. (a) 2. (b) 3. (b) 4. (d) 5. (c) 6. (b) 7. (b) 8. (d) 9. (a) 10. (d)
11. (b) 12. (b) 13. (d) 14. (b) 15. (a) 16. (a)
Chapter

5
How do Organism
Reproduce
Reproduction
It is a biological process in which an organism give rise Regeneration
to young ones (offspring) similar to itself. This event is Simple animals like Hydra and Planaria cut into
important for the continuity of life species.Two basic number of pieces and each piece grows into complete
types of reproduction are asexual and sexual. organism. Specialised cells are responsible for
regeneration.
Asexual Reproduction
When the offspring is produced from a single parent
without the involvement of gamete formation. In this
case all the individual are genetically identical and clone
of each other. It is more common among unicellular
organism and plants.
Asexual reproduction includes

Fission
Many bacteria split into two equal halves during cell
division, e.g., in case of Amoeba, splitting of two cells
occur in any plane.
Regeneration in Planaria

Budding
Organism like Hydra develops an outgrowth at a
specific site. Bud develops into a new individual.
Tentacles
Binary fission in Amoeba

Plasmodium shows multiple fission.

Fragmentation
Bud
In case of Spirogyra. It simply breaks up into smaller
pieces. These pieces or fragments later grow into new
individual. Budding in Hydra
How do Organism Reproduce 351
Vegetative Reproduction by Natural Method
Part Type Description Example
Stem (a) Bulb Modified shoot, short stem having apical and axillary bud. Onion, tulipa, etc.
(b) Runner Creeping step, produce roots at nodes. Doob grass, etc.
(c) Rhizomes Underground horizontally brewing stem. Ginger
(d) Corm Condensed rhizome growing vertically. Saffron and Colocasia
(e) Tuber Modification of underground stem tip. Potato
(f) Offset Prostate branch with leaves at apex. Water lettuce and water hyacinth
Root Root tubers Root develop buds and each forms a plant. Sweet potato, Asparagus, dahlia, etc.
Leaves Plantlets develop along the leaf margin, which an Bryophyllum pinnatum
detachment produces independent plant.

Vegetative Propagation transferred to the stigma of the same flower it is called


self-pollination.
It is the regeneration or formation of a new individual
from vegetative part of the plant body. l When pollen grains are transferred to the stigma of
another flower by an agent is known as
Spore Formation cross-pollination. These agents are wind (anemophily),
During unfavourable conditions, some unicellular algae birds (ornithophily), bats (chiropterophily). Water
(hydrophily) and insects (entomophily).
forms unicellular, uninuclear thick walled spores.
l When pollen sticks to the stigma, it reach the female
Sexual Reproduction germ cell which are in ovary. Pollen tube grows out of
the pollen grain and travels through the style to reach
Sexual reproduction involves involvement of two the ovary. Pollen tube carries two male gamete. One
parents. Sexual reproduction causes variation in a male gamete fuses with the egg cell and another male
individual by process of combining DNA from two gamete fuses with the polar nuclei. This event is known
different individuals during reproduction. as double fertilization.
Sexual Reproduction in Flowering Plants One male gamete + egg cells → embryo (syngamy)
Reproductive part of a plant is flower. It the flower male (n) (n) (2n)
part is stamen known as microsporophyll or androecium Second male gamete + polar nuclei → endosperm
and female part is carpel known as megasporophyll or (n) (2n) (3n)
gynoecium. Stamen includes filament and anther.
Endosperm is a nutritive tissue. It is the product of
Anther consist of four microsporangia in which pollen
triple fusion.
grain matures.
l Carpel has three parts, swollen part is ovary. Slender Reproduction in Human
part is style and sticky head is stigma. Ovary contain
ovules or megasporangia. It laters develops into seed, Humans uses sexual made of reproduction.
after fertilisation. Transfer of pollen grains from stamen
Male Reproductive System
to carpel is called pollination. It could be
self-pollination or cross pollination. When pollen grain is Functions and components of male reproductive system.

Organs and their Description


Organs Description Function
Scrotum Pouch of pigmented skin arising from the lower abdominal wall. It maintain testes at 2°C to 2.5°C and
causes thermoregulation.
Testes Primary sex organ suspended in scrotal sac. Sperm production in seminiferous tubules.
Epididymis Greatly coiled long tube attached to hind surface of testes. Site of sperm maturation and storage.
Vas deferens Continuation of cauda epididymis. Duct of sperm maturation storage and transport.
Ejaculatory duct It is 2 cm long thin walled tubes. passes through prostrate gland Transport of sperm.
and opens into urethra.
Penis External intromettent organ through which urethra runs. Erectile organ used during copulation
352 Study Package for NTSE

Female Reproductive System


Function of components of female reproductive system
Organs and their Description
Organs Description Function
Ovaries Primary sex organ, have ovarian follicles. Produces ova secretion of sex hormones oestrogen and progesterone.
Fallopian tubes About 10-12 cm long. Transports oocycte from ovary to the uterus, also the site of fertilisation.
Uterus Hollow pear-shaped muscular organ. Site for embryo development.
Vagina Flexible muscular tube extend forward and Organ for sexual intercourse.
downward opens out at its lower end into vestibule.

l Through spermatogenesis one spermatogonia produces four spermatocytes in seminiferous tubules which are haploid in
number. Oogenesis one oogonia produces, one large ovum and three secondary polar body.
l Spermatids enters through vaginal passage during sexual intercourse. It travels and reach the oviduct, where it
fertilises with egg. After fertilisation it forms zygote and gets implanted in the lining of uterus. Zygote develops into an
embryo, which gets nourishment from placenta. Gestation period in case of human being is 9 months.

Target Exercise
Elementary Level
1. Oocyte is fertilised in the 10. The process of transfer of pollen grain from anther to
(a) vagina (b) Fallopian tube stigmatic surface with the help of water is called
(c) uterus (d) ovary (a) anemophily (b) zoophily
2. Ovule integument gets transformed into (c) hydrophily (d) ornithophily
(a) seed (b) fruit wall 11. Anemophily is pollination by
(c) seed coat (d) cotyledons (a) wind (b) air (c) insects (d) birds
3. In humans mature sperms are stored in the 12. Wind pollination is common in
(a) epididymis (b) seminiferous tubules (a) lilies (b) grasses (c) orchids (d) legumes
(c) vas deferens (d) seminal vesicles
13. Transfer of pollen grain from one flower to another
4. Each diploid cell during oogenesis produces flower of same plant is
(a) two functional eggs and two polar bodies
(a) geitonogamy (b) autogamy
(b) one functional egg and three polar bodies (c) allogamy (d) pollen kit
(c) four functional eggs
14. How many sperms are formed by four primary
(d) None of the above
spermatocytes?
5. Spermatogenesis requires the availability of (a) 1 (b) 4
(a) oestrogen (b) progesterone (c) 16 (d) 32
(c) thyroxine (d) testosterone 15. Endometrium is the lining of
6. Stem cuttings are commonly used for the propagation of (a) bladder (b) vagina
(a) banana (b) rose (c) mango (d) cotton (c) uterus (d) oviduct
7. What is pollen grain? 16. Which of the following process ensures the continuity
(a) Microspore mother cell (b) Male gamete of life on Earth?
(c) Male gametophyte (d) Partially developed embryo (a) Reproduction (b) Respiration
8. Triploid tissue in angiosperm is (c) Digestion (d) Growth and development
(a) nucellus (b) endosperm 17. Binary fission is a mode of
(c) endothelium (d) tapetum (a) micropropagation (b) vegetative propagation
9. Double fertilisation is fusion of (c) macropropagation (d) asexual reproduction
(a) two eggs 18. Fertilisation of egg takes place inside
(b) two eggs and polar nuclei (a) anther (b) stigma
(c) one male gamete with egg and other with synergids (c) pollen tube (d) embryo sac
(d) one male gamete with egg and other with polar nuclei
How do Organism Reproduce 353
19. Match the items in column I with column II and Codes
choose the correct option. A B C D E
(a) 1 4 5 3 2
Column I Column II (b) 2 1 4 3 5
(c) 2 4 3 5 1
A. Binary fission 1. Algae
(d) 1 4 3 2 5
B. Zoospore 2. Amoeba
20. The ovary after fertilisation is converted into
C. Conidium 3. Hydra
(a) embryo
D. Budding 4. Penicillium
(b) endosperm
E. Gemmules 5. Sponge (c) fruit
(d) stem

High Skill Questions


1. Pollen kit material is secreted by 8. Parthenogenesis is a type of
(a) tapetum (b) endothecium (a) sexual reproduction (b) asexual reproduction
(c) epidermis (d) endodermis (c) budding (d) regeneration
2. The embryo sac n, 2n, 3n conditions are found 9. Which of the following provides nutrition to sperm?
respectively in (a) Leydig cell (b) Scrotum (c) Sertoli cell (d) Epididymis
(a) egg, antipodal and endosperm 10. In mammals, corpus luteum is found in which organ?
(b) nucleus, endosperm and egg (a) Brain (b) Ovary (c) Liver (d) Eyes
(c) antipodal, zygote and endosperm
(d) endosperm, nucleus and egg 11. Primary spermatocyte differs from spermatogonium in
(a) number of chromosomes (b) size of volume
3. Tapetum is found in (c) DNA content (d) size of chromosomes
(a) anther (b) microspore
(c) male gametophyte (d) female gametophyte 12. Which of the following hormone is not the secretory
product of human placenta?
4. Double fertilisation was discovered by
(a) Prolactin
(a) Nawaschin (b) Strasburger
(b) Human gonadotropin hormones
(c) Emerson (d) None of these
(c) Progesterone
5. Which of the following is not true for double (d) Oestrogen
fertilisation?
13. Natural parthenogenesis is found in
(a) Discovered by Nawaschin (a) housefly
(b) Male gamete and secondary nucleus fused to form (b) honey bee
endosperm nucleus (c) Drosophila
(c) Endosperm nucleus is diploid (d) All of the above
(d) Endosperm provides nutrition to embryo
14. Which of the following is viviparous?
6. The wall of pollen tube is made of (a) Running bird (b) Whales
(a) cellulose (b) pectin (c) Bats (d) Both (a) and (b)
(c) Both (a) and (b) (d) None of these
15. Sexual reproduction leads to
7. One of the most resistant known biological material is (a) genetic recombination (b) polypoidy
(a) lignin (b) hemicellulose (c) aneuploidy (d) euploidy
(c) sporopollenin (d) lignocellulose

Answers
Elementary Level
1. (b) 2. (c) 3. (a) 4. (b) 5. (d) 6. (b) 7. (c) 8. (b) 9. (d) 10. (c)
11. (b) 12. (b) 13. (b) 14. (c) 15. (c) 16. (a) 17. (d) 18. (b) 19. (d) 20. (c)

High Skill Questions


1. (b) 2. (c) 3. (a) 4. (a) 5. (c) 6. (c) 7. (c) 8. (b) 9. (c) 10. (b)
11. (a) 12. (a) 13. (d) 14. (d) 15. (a)
Chapter

6
Heredity and Evolution
Heredity
Heredity is defined as the transmission of features from l In a dissimilar pair of factor one member of a pair
one generation to another. Genetics is the study of dominate the other which is recessive. Dominant factor
heredity and variations. will mask the affect of recessive factor.
(Male) (Female)
Inherited Traits TT
tall
tt
dwarf
Tt × Tt

These are the features/traits, which a child gets from TT Tt Tt tt


parents. Father and mother contribute equal amount of
Tt F1 Tall Dwarf
genetic material (DNA) to the child. Each trait is
influenced by maternal and paternal DNA. l Another law proposed by him was law of segregation.
l Variation is defined as the differences, which exist According to which alleles do not show any blending
between individuals belonging to same species. and both the characters are recovered in the F2
generation.
l Theory of inheritance was given by Gregor Johann
Mendel in 1866. He started his experiment on
controlled hybridization in garden pea
Sex Determination
( Pisum sativum). He explained that each cell have two Different species uses different strategies in order to
copies of each chromosome. determine sex. Some uses environmental signals like
l One from male parent and other from female parent. temperature.
Germ cells carries one of the chromosome, which may l In animals like snails sex is not genetically determined.
be paternal or maternal in origin. When two germ cells l In human being sex is largely genetically determined.
combines they restores the original number of Humans have 22 set of chromosome called autosome
chromosome with different features. and one pair of chromosome called sex chromosomes.
l Mendel undertook seven contrasting visible Woman have XX pair of sex chromosome while males
features/characters of garden pea. have XY pair of sex chromosome. A child who inherits
X-chromosome from her father will be a girl and other
These seven characters were
who inherits Y-chromosome will be boy.
(i) round/wrinkled seeds
(ii) tall/short plants
(iii) white/violet flowers
(iv) green/yellow seeds
Organic Evolution
(v) constricted inflated pod shape The branch of science, which deals with fossils is known
(vi) green/yellow pod colour
as Palaeontology. The first organisms which first
(vii) axial/terminal flower position
appeared on the Earth at about 3.5 billion (350 crore)
l Based on his observation he proposed. Law of years ago. Age of mammals, birds, fishes, insects and
dominance, according to which characters are
angiosperms is called Cenozoic era.
controlled by pair of factors.
Heredity and Evolution 355
Theory of Special Creation Homologous Organs
According to Father Suarez, God created life about These are the organs, whose basic structure is same but
4004 BC. Life was created by some super natural power. function vare like four limbs of birds, reptiles and
amphibians.
Theory of Catastrophism Analogous Organs
Sudden creation of life from inorganic material. It is
These are the organs whose basic structure is very
suported by Cuvier (Father of Modern Palaeontology).
different but performs same function. e.g., wings of birds
Analogous organs are also known as, homoplastic and bat.
organs. They differs in basic structure and origin but
similar in functions, e.g., wings of insects, bats and Fossils
birds. These are the preserved traces of a living organism in
form of impression skeleton imprints, etc.
Theory of Evolution
Molecular Phylogeny
The main requirements of this theory is progression and
variation. The main and ultimate source of organic When evolutionary relationships are set up by studying
variation is known as mutation. the variation in DNA among individuals human.

Human Evolution
Origin of Life on Earth l Study of human evolution is known as anthropology.
The arrangement of fossils in evolutionary sequence
In year 1953 Stanley and Miller and Harold C Urey
includes
in 1953 conducted an experiment. They took four gases,
Tree shrews → Prosimi early arthropoids →
which were thought to exist on Early earth
Dryopithecus → Ramapithecus → Australopithecus →
(NH 3, CH4, H2 S, H2 O vapour but no O2 ). It was Homo erectus → Cro-magnon → Homo sapiens.
maintained just below 100° C and spark was passed four
l Evolution of man take place in Pleistocene period
amino acids were formed.
l Dryopithecus was the common ancestor of humans and
(i) alanine (ii) aspartic acid (iii) glutamic acid (iv) glycine apes that lived an arboreal life in Asia as well as in Africa.
l Australopithecus was the first man ape. Modern man
Speciation Homo sapiens sapiens appeared after last glacial period
It is the change/variation in two sub-population when about 10000 years ago.
subjected to geographical isolation.

Target Exercise
Elementary Level
1. The wings of birds and bats are called as 5. Wings of birds and insects are called as
(a) vestigial organs (b) homologous structures (a) homologous structures
(c) analogous structures (d) None of these (b) vestigial organs
(c) analogous structures
2. Which of the following is similar to analogous organs?
(d) inactive structures
(a) Origin (b) Structure
(c) Function (d) None of these 6. Archaeopteryx is related to
(a) Aves and reptiles (b) mammals
3. The study of fossil plants is known as (c) Pisces (d) reptiles
(a) Botany (b) Palaeontology
7. Evolution refers to
(c) Zoology (d) None of these
(a) life in sea
4. Dinosaurs are called as (b) fossils
(a) extinct birds (b) extinct animals (c) living things constantly changes
(c) extinct reptiles (d) extinct amphibians (d) None of the above
356 Study Package for NTSE
8. Theory of evolution that was supported by Lamarck, 12. Dead remains of plants and animals is called as
named as (a) fossils (b) coal
(a) mutation theory (c) charcoal (d) All of these
(b) inheritance of acquired characters 13. Which of the following is the correct term for
(c) natural selection
homologous organs?
(d) None of the above
(a) Origin (b) Function
9. Theory of spontaneous generation was given by (c) Structure (d) Both (a) and (c)
(a) Von Helmont (b) Pasteur
(c) Redi (d) None of these 14. Which of the following ‘analogous organs’ contains?
10. An organism having two same types of alleles of a (a) Different embryonic origin and perform different functions
(b) Different embryonic origin but perform similar functions
gene in a cell, is known as
(c) Common embryonic origin and perform dissimilar functions
(a) hybrid (b) dihybrid (d) None of the above
(c) homozygous (d) heterozygous
15. Which one is found in male only?
11. Source of organic variations is
(a) X-chromosome (b) Y-chromosome
(a) mutations (b) sexual reproduction (c) 2 X-chromosome (d) X + Y-chromosome
(c) asexual reproduction (d) None of these

High Skill Questions


1. Which one of the following statements is correct? 5. Which of the following symbols are used for
(a) Cro-magnon man’s fossil has been found in Ethiopia representing chromosomes of birds?
(b) Homo erectus is the ancestor of man (a) ZZ-ZW (b) XX-XY (c) XO-XX (d) ZZ-WW
(c) Neanderthal man is the direct ancestor of Homo sapeins
6. Universe originated about 15 billion year ago by
(d) Australopithecus is the real ancestor of modern man
(a) thermonuclear explosion (b) fusion
2. Which one of the following pair has homologous (c) gravity (d) destruction of atmosphere
organs?
7. Age of Earth is about
(a) Pectoral fins of a fish and forelimbs of a horse
(b) Wings of a bat and wings of cockroach (a) 4.6 billion years (b) 10 billion years
(c) Air sac of fish and lungs of frog (c) 3.0 billion years (d) 20 billion years
(d) Wings of a bird an wings of a butterfly 8. Which of the following is known as oldest living
3. A child receives genes from his father to the extent of fossil?
(a) 25% (b) 50% (a) Eubacteria (b) Archaebacteria
(c) 75% (d) 100% (c) Protozoa (d) Eukaryotes
4. Transfer of genes from one gene pool to another is 9. Atmosphere of earth at the time of origin of life was
called (a) reducing (b) oxidizing
(a) speciation (c) Neither (a) or (b) (d) aerobic
(b) genetic drift 10. There is no life on moon because there is no
(c) gene flow
(a) H2O (b) N2 (c) carbon (d) silicate
(d) mutation

Answers
Elementary Level
1. (c) 2. (c) 3. (b) 4. (c) 5. (c) 6. (a) 7. (c) 8. (b) 9. (b) 10. (c)
11. (b) 12. (a) 13. (c) 14. (b) 15. (b)

High Skill Questions


1. (d) 2. (a) 3. (b) 4. (c) 5. (a) 6. (a) 7. (a) 8. (b) 9. (a) 10. (a)
Chapter

7
Why do we Fall Ill
Health
Health is defined as the state of physical, mental and Communicable diseases which could be easily
social well being which depends upon their surroundings transmitted from a diseased person to a healthy person
and environments. via various agents like air, food, water, vectors
Health depends upon social environment. Social (mosquitoes and insects), physical contact, etc., e.g.,
equality and harmony among individuals is necessary small pox, measeles, cold, etc.
for individual health.
Infectious Agents
Physical Environment Microorganisms which causes diseases comes under
Health of a person depends upon his surroundings. wide range of classification. These include virus,
Public cleanliness and hygiene is important for an bacteria, some fungi and some are protozoans. Before
individual health. giving treatment, it is necessary to diagnose the type of
microorganism causing the infection.
Disease
Common Methods of Transmission
Disease free person may not be healthy. Disease literally of Diseases
means being uncomfortable. For a diseased person we
Air Pneumonia, common cold, tuberculosis spread
can find a specific and particular cause of discomfort.
through air.
Some types of disease are as follows
Water Excrete of diseased person carries the
Acute disease which last for few days or short time, infections agent and may mix up with the drinking
e.g., common cold. water, e.g., incase of cholera.
Chronic disease which last for longer period of time or Vectors Common vector is mosquitoes they carry the
life time. The symptoms of such diseases appear at later infecting agent from sick person to another potential
stages, e.g., elephantiasis. Chronic diseases causes more host. Anopheles mosquito carries Plasmodium vivax,
harmful effect on body. which causes malaria.
Infectious diseases which are caused due to infection Physical Contact AIDS, syphillis and gonorrhoea
of microorganism. These may be virus, bacteria or fungi. are sexually transmitted diseases, which are
Non-infectious diseases which are internal in origin. transmitted by sexual contact from one partner to
For example, some cancers arises due to presence of another. Although these are not spread by casual
genetic abnormalities, high blood pressure, diabetes. physical contact including hand shake or hugs.
358 Study Package for NTSE

Some common effects of diseases include inflammation. To kill disease causing microorganism many cells of our
immune system are recruited at the site of infection in order to kill that microbe such situation causes inflammation.
Treatment
Disease could be cured by killing the microorganism or by providing treatment that will reduce the symptoms.
Biochemical processess could be targeted to kill the microorganism. Antibiotics are the chemical substance
released by fungi which kills the bacteria.

l Penicillin is an antibiotic, which attacks the cell wall of Gram positive bacteria. Human do not have such cell wall so,
bacterial cells are killed selectively.
l Viruses live inside host cell and do not have their own biochemical pathways, which could be targeted to control them. Due
to this limitation antiviral drugs are difficult to make.

Some Important Diseases Caused by Microorganism

Microorganism Disease Symptoms

Viruses ● Influenzas Sudden fever with headache


● Common cold (rhinovirus) Nasal and bronchial irritation
● Small pox (variola DNA virus) High fever bodyache rashes on body
● Measeles (paramyxo RNA) virus Sore threat, cough, fever, small white spots
● Rabies (rabies virus) Hydrophobia
● Mumps paramyxo virus (RNA virus) Fever followed by swelling of the parotid or salivary gland.
● Poliomyelitis (polio virus) picornovirus Nerve cells to muscles are destroyed causing paralysis
● AIDS (caused by HIV virus) Breakdown of immune system
● Hepatitis-B Infected liver, fever, jaundice
Bacteria ● Diphtheria (Cornebacterium diphtheriae) Fever, severe damage to heart
● Tuberculosis (Mycobacterium tuberculosis) Infects lungs and bones
● Whooping cough (pertusis) Bordetella pertusis Severe coughing bouts
● Tetanus (Clostridium tetani ) Toxins causes muscular spasm
Protozoa ● Malaria (Plasmodium vivax) High fever
● Ameabiasis (Entamoeba histolytica) Diarrhoea loss of blood in stools fever vomiting
Fungi ● Athletes foot (Tinea pedis) Presense of sudden peding and cracked skin between toes
Helminths ● Taeniasis (Taenia solium) tapeworm Infects intestine
● Ascariasis (Ascaris lumbricoides) Infects intestine
● Filiariasis (Wuchereria bancrofti) also known as Legs become heavy stout
elephantiasis

Prevention
General ways of preventing an infection is to present exposure to infectious microbes by providing safe drinking
water, clean environment, better public hygiene, etc. Another way of preventing an infection is boosting our
immune system.
l Human body has ability to resist among types of organism or toxins. When the immune system first sees the infectious
agent, it responds against it and remembers it specifically. When a particular microbe again attacks the person,immune
system recognises it and shows greater response eliminating the infection. Highly effective method of immunisation is
using vaccines.
l Vaccines are the live or inactivated pathogen purified antigen or recombinant peptide, which could elicit immune
response in body against particular pathogen. Edward Jenner first time developed vaccine for small pox.
Target Exercise
Elementary Level
1. BCG vaccine is used for 14. Which of the following is a viral disease?
(a) polio (b) malaria (a) Influenza (b) Cholera
(c) cholera (d) tuberculosis (c) Dengue fever (d) Plague
2. AIDS virus contains 15. Plague is caused by
(a) single stranded DNA (b) double stranded RNA (a) Yersinia pestis (b) Monocystis
(c) single stranded RNA (d) double stranded DNA (c) Shigella (d) Streptococcus
3. Mumps is caused by which type of gland? 16. Match the disease with the causative organisms
(a) Sublingual gland (b) Thyroid gland
A. Pneumonia 1. Vibrio cholera
(c) Parotid gland (d) None of these
B. Citrus canker 2. Mycobacterium leprae
4. Which of the following drug is used for the treatment C. Cholera 3. Xanthomonas citri
of typhoid? D. Leprosy 4. Diplococcus penumoniae
(a) Chloramphenicol (b) Tetracyclin
(c) Penicillin (d) Chloroquinine Codes
A B C D
5. DPT vaccine is used to immunise infants against (a) 4 2 1 3
(a) diphtheria, whooping cough and tetanus (b) 4 3 1 2
(b) cholera and typhoid (c) 2 1 3 4
(c) influenza (d) 3 4 2 1
(d) pneumonia
6. Gonorrhoea is caused by 17. A non-communicable disease is
(a) bacteria (b) virus (a) maseles (b) rabies
(c) TMV (d) Giardia (c) diphtheria (d) diabetes

7. Who discovered small pox vaccine? 18. A chemical substance produced by microorganism for
inhibiting of growth of another is
(a) D Jenner (b) Salk
(c) Charles Laveran (d) Edward Jenner (a) antibody
(b) antibiotic
8. Mumps is caused due to the inflammation of (c) aflatoxin
(a) thyroid gland (d) antiallergic
(b) parotid gland
(c) eyes
19. Bacterium which is concerned with pertusis is
(d) sneezing (a) Bordetella pertusis
(b) Bacillus
9. Salk vaccine fights against (c) Diplococcus
(a) polio (b) cholera (d) Mycobacterium tuberculosis
(c) typhoid (d) rabies
20. Which of the following is a protozoan disease?
10. Typhoid is caused by (a) Malaria
(a) Salmonella (b) Virus (b) Amoebiasis
(c) Streptococcus (d) Mycobacterium (c) Sleeping sickness
(d) All of the above
11. Which of the following is a bacterial disease?
21. Which of the following is a bacterial disease?
(a) Tuberculosis (b) Malaria
(a) Typhoid (b) Dengue fever
(c) Plague (d) Mumps
(c) Malaria (d) AIDS
12. Jaundice affects which part of the body?
22. A sexually transmitted disease, symptomised by the
(a) Kidney (b) Liver
development of ulcers on the genitals is caused by
(c) Stomach (d) Brain
infection of
13. Leprosy is caused by (a) Treponema pallidum
(a) Mycobacterium (b) Neisseria gonorrhoeae
(b) Pneumococcus (c) Human Immunodeficiency Virus
(c) Shigella (d) hepatitis-B virus
(d) Giardia
360 Study Package for NTSE

High Skill Questions


1. Which of the following disease is communicable? 9. AIDS virus contain
(a) Rickets (b) Amoebiasis (a) RNA with protein (b) DNA with protein
(c) Diabetes (d) Cancer (c) DNA without protein (d) single stranded DNA
2. Resistance against pathogen is due to 10. Which of the following organs is not involved in the
(a) antibodies (b) RBCs eliutation of immune response?
(c) haemoglobin (d) pertusis (a) Brain (b) Lymph node
3. First antibiotic was discovered by (c) Spleen (d) Thymus
(a) A Flemming (b) W Flemming 11. Disease which last for longer period of time are
(c) R Koch (d) Louis Pasteur known as
4. Who among the following established the scientific (a) acute diseases (b) chronic diseases
basis of vaccination? (c) infections diseases (d) communicable diseases
(a) Edward Jenner (b) George Kohler 12. Which of the following is transmitted through air?
(c) Louis Pasteur (d) Von Behring (a) Pneummia (b) Common cold
5. The primary host of Plasmodium is (c) Tuberculosis (d) All of these
(a) man (b) male Culex 13. Aedes aegypti is a vector for
(c) sheep (d) female Anopheles (a) dengue fever
6. Which of the following is a pair of viral diseases? (b) yellow fever
(a) Ringworm, AIDS (c) Both dengue and yellow fever
(b) Common cold, AIDS (d) Japanese encephalitis
(c) Dysentery, common cold 14. Use of anti-histamines and steroids give quick relief
(d) Typhoid, tuberculosis from
7. The organ related with immunity is (a) allergy (b) nausea
(a) liver (b) parathyroid (c) cough (d) headache
(c) spleen (d) thymus 15. Which cell secretes antibody?
8. Which one of the following acts as a physiological (a) Lymphocytes
barrier to the entry of microorganism in human body? (b) Monocytes
(c) Eosinophils
(a) Tears (b) Monocytes
(d) Neutrophils
(c) Skin (d) Epithelium of urogenital tract

Answers
Elementary Level
1. (d) 2. (c) 3. (c) 4. (d) 5. (a) 6. (d) 7. (d) 8. (b) 9. (a) 10. (a)
11. (a) 12. (b) 13. (a) 14. (a) 15. (a) 16. (b) 17. (d) 18. (b) 19. (a) 20. (d)
21. (b) 22. (b)

High Skill Questions


1. (b) 2. (a) 3. (a) 4. (a) 5. (d) 6. (b) 7. (d) 8. (b) 9. (a) 10. (c)
11. (b) 12. (b) 13. (a) 14. (a) 15. (a)
Chapter

8
Diversity in Living
Organism
Biodiversity
It is the existence of wide variety of species (species Classification is done under following scheme
diversity) in a specified region. Region between tropic l Kingdom l Phylum l Division
of cancer and tropic of capricon is megadiversity region. l Class l Order l Family
l Genus l Species
Basis of Classification RH Whittaker in year 1959 give five kingdom
Classification is the arrangement of life forms into classification on the basis of cell structure, mode and
various groups on the basis of common characteristic, source of nutrition and body organisation. Five kingdoms
affinities or relationships. were Monera, Protista, Fungi, Plantae and Animalia.
Classification of organism is based on Monera
(i) Nuclear organisation presence or absence of These are unicellular organism. It includes all
clearly defined nucleus.
prokaryotes like bacteria, mycoplasma, Actinomycetes
(ii) Mode of nutrition whether an organism is able to
and blue-green algae. Some are autotrophs and
produce its own food or dependent of others.
synthesise their own food like cyanobacteria. These are
(iii) Unicellularity/Multicellularity.
sub-divided into archea having thermophiles and
(iv) Organisation of different body parts.
archaebacteria and eubacteria.
(v) Presence of specialised organs and their functions.
Classification of life forms is linked to evolution. Protista
Evolution is the gradual change over a period of time to These include unicellular eukaryotic organism. Some
allow the organism to survive better. species are autotrophs some are heterotrophs.
Possesses hair like appendages and whip like flagella,
Hierarchy of Classification e.g., protozoans, dinoflagellates, diatoms, slime mould
Ernst Haeckel (1894), Robert Whittaker (1959) and and algae.
Carl Woese (1977) have classified into broad categories Fungi
called kingdom.
These are eukaryotic heterotrophic organisms. These
These are defined as may be saprophyte (feeds on decaying organic matter)
Later Woese modified this classification by dividing or parasitic in nature. Their cell wall is made up of
Monera into archaebacteria and eubacteria. chitin, e.g., yeast and mushrooms.
362 Study Package for NTSE

Plants

Water vacuole
Cilia Do not have Have differentiated
differentiated plant body
Macronucleus plant body

Thallophyta Without specialised With vascular


vascular tissue tissue
Micronucleus

Oral groove Bryophyta


Food vacuole
Cytosome Do not produce seeds Produce seeds-phanerogams

Cytopyge Pteridophyta
Bear naked seeds Bear seeds
inside fruits
Waste Gymnosperms
Angiosperms
Paramecium
Have seeds with Have seeds with
Endoplasm two cotyledons one cotyledon

Ectoplasm Dicots Monocots

Mitochondria
Nucleus Thallophyta
Crystals These do not have well differentiated body design.
Plants in this group are known as algae. These are
photosynthetic autotrophs like red algae, brown algae,
green algae and mosses.

Bryophyta
These are also known as amphibians of plant kingdom.
Food vacuole Contractile vacuole
Advancing They are differentiated into stem and leaf-like
pseudopodia structure. Do not have specialised tissue for conduction
Amoeba
of H2 O and other substance, e.g., moss (Funaria and
Marchantia).
Flagellum (long)
Eye spot
Pteridophyta
Plant body is differentiated into root, stem and leaves. It
Nucleolus
Photoreceptor has specialised tissue for conduction of H2 O and other
Flagellum
substances, e.g., in Marsilea ferns.
(short) All the above three thallophytes, bryophytes and
Contractile pteridophytes are also known as cryptogams.
vacuole Reproductive organs of these groups are not well
Chloroplast
Nucleus defined and have naked embryos called spores.
Euglena
Gymnosperms
It is made up of two greek words gymno means naked
Plantae and sperma means seed.
Plants are classified on the basis of their ability bear Plant of these group bear naked seeds vascular system
seeds type of tissues for the transport of H2 O and is well developed. They are perennial, evergreen and
substance. These are classified as woody.
Diversity in Living Organism 363
Angiosperms
l This word is made up of two greek words angio means covered and sperma means seed. Reproductive organ is well defined
and known as flower hence, these are also known as flowering plants.
l Ovules inside ovary modifies into seed after fertilisation. Ovary later modified to became fruit and encloses seed. Embryo
in seeds have structure called cotyledons also known as seed leaves.
l On the basis of number of cotyledons plants are divided into monocots/ monocotyledonous or dicots/ dicotyledonous plants,
e.g., Paphiopedilum (monocot) and Ipomoea (dicot).

Animalia
These include multicellular eukaryotic and heterotrophic animals. They include both predator and parasite. Their
classification is based on the extent and type of body design.

Animals

Cellular level of Tissue level of


organisation organisation

Porifera

No body cavity between Pseudocoelom Coelomate


epidermis and gastrodermis

Coelenterata and Platyhelminthes Nematoda

Mesodermal cells from Coelom formed from


a single cell during pouches pinched off
growth of the embryo from the endoderm

Annelida, Mollusca and


Arthropoda Notochord absent Notochord present

Echinodermata Chordata

Notochord present
in at least larval forms, Notochord replaced by vertebral
but very rudimentary column in adults

Protochordata Vertebrata

Exoskeleton of Gills in larva, Exoskeleton Exoskeleton Exoskeleton


scales, endoskeleton lungs in most of scales, laying of feathers, lay of hair, external
of bone/cartilage adults, slimy eggs outside eggs outside ears, mostly
breathing through skin water water, flight giving birth
gills possible to live young

Pisces Amphibia Reptilia Aves Mammalia


364 Study Package for NTSE

Porifera Nematods
l These are non-motile animals attached to a solid support. Their body is bilateral symmetrical and triplobastic body
They have holes all over the body leading to a canal is cylindrical. They show hydrostatic support through
system helps in circulating H2O throughout the body.
psuedocoel, have tough cuticle. They have complete gut
l Contractile cells change openings at body surfaces and gas exchange system across the body surface, e.g.,
commonly called sponges. Animals are covered with
parasitic worms causing elephantiasis (filarial worms)
skeleton, e.g., Sycon, Euplectelea and Spongilla.
and worms in intestine (roundworm or pinworm).
Cnidaria (Coelenterata)
Annelida
l These are mostly marine attached creeping swimming
or floating carnivore. There is cavity in the body showing These are also bilaterally, symmetrical and triploblastic.
hydrostatic support (by fluid in gut). They have well developed musculature in body wall.
l Body is made up of two layers of cell. One makes up Showing extensive organ differentiation have brain,
outside of the body and other makes up inner living of double ventral nerve cord. Differentiation occurs in
the body, e.g., Hydra, sea anemone, jellyfish. segmental fashion, e.g., leeches, earthworm, etc.

Platyhelminthes Arthropoda
l These are marine fresh water and some terrestial l Largest group of animals. These are bilaterally
organism. They have brain and shows hydrostatic symmetrical as well as segmented. They Shows open
support. Body is bilaterally symmetrical (both the circulatory system, blood do not flow in defined vessels.
halves of the body have equal design). They have brain, double ventral nerve cord has
l They possess 3 layers of cells and also known as exoskeleton jointed appendages (arthropod means
triploblastic. These may be parasitic or free living, e.g., jointed legs).
planarians (free living) and parasitic like liver flukes l Coelomic cavity is blood filled, e.g., prawn, scorpion,
also known as flatworms. spider, butterfly, house fly, centipede, cockroach, etc.

Eyes
Mollusca
Branched Acetabulum They show bilateral symmetry little segmentation.
gastrovascular They shows hydrostatic skeleton, well-developed
cavity
musculature in foot mantle other structure. They form
Pharynx haemocoel. These have open circulatory system.
Mouth and
anus Echinodermata
l Echinos means hedgehog and derma means skin. Has
endoskeleton and have spines. Uses tube feet for
locomotion. They are strictly marine, mostly carnivores,
have hard calcium carbonate structures that they use as
Planaria Liverfluke skeleton.
l They are triploblastic and have a coelomic cavity, e.g.,
Scolex Sucker
Neck
Asterias (star fish), Echinus (sea urchin), Antedon
(feather star), Holothuria (sea cucumber).

Protochordata
Bilaterally symmetrical, triploblastic and have a
coelom. Possess long red like support structure called
notochord it separates the nervous system from gut.
These are marine animals, e.g., Balanoglossus,
Hardmania and Amphioxus.

Tapeworm Vertebrata
Some Platyhelminthes animal They have true vertebral column and an internal
skeleton. These are bilaterally symmetrical
triploblastic, coelomic and segmented.
Diversity in Living Organism 365
These possess Reptilia
l notochord l dorsal nerve cord These have scales all over the body and breathe through
l triploblastic l hair paired gill pouch lungs. They have two-chambered heart but in crocodile
l coelom.
four-chambered heart is present. Lay eggs with tough
coverings and these are cold-blooded animals.
Pisces
These includes fish which are exclusively aquatic Aves
animals and skin is covered with plates/scales. They are These are warm-blooded animals. They possess
cold-blooded animals and have two-chambered heart. four-chambered heart. Forelimbs are modified into
In some fishes skeleton is made up of cartilage, in some feathers for flight. They breath through lungs, e.g., all
it is made up of both bones and cartilage. They lay eggs, birds.
e.g., angler fish, lion fish, dog fish and string ray.
Mammalia
Amphibia They have four-chambered heart. These are
These include frogs, toad, salamander. They can live in warm-blooded. They give birth to young ones and
H2 O as well as land due to the presence of both gill and possess mammary glands for milk production, e.g.,
lungs. They lay eggs. They have mucous secretion in the human beings, dog, cat, etc.
skin and possess three-chambered heart.

Target Exercise
Elementary Level
1. Which is the only phylum in the animal kingdom 7. A chordate character is
without any nerve cell? (a) gills (b) spiracles
(a) Porifera (b) Coelenterata (c) post-anal tail (d) chitinous exoskeleton
(c) Annelida (d) Nematoda
8. The canal system is a characteristic feature of
2. Which of the following animal belongs to the (a) echinoderms (b) helminthes
phylum–Mollusca? (c) coelenterates (d) sponges
(a) Devil fish (b) Dog fish
(c) Silver fish (d) Jelly fish
9. The plant group that produces spores and embryo but
lacks vascular tissues and seeds is
3. The Hierarchial arrangement of taxonic categories in (a) Pteridophyta (b) Rhodophyta
descending order is (c) Bryophyta (d) Phaeophyta
(a) kingdom, phylum, class, order, family, genus, species
(b) kingdom, division, phylum, order, family, genus, species
10. A plant having seeds but lacking flowers and fruits
(c) kingdom, class, phylum, order, family, genus belongs to
(d) division, kingdom, class, phylum, order, family, genus, (a) pteridophyte (b) mosses
species (c) ferns (d) gymnosperms
4. The shape of the cocci bacteria is 11. The basic unit of classification is
(a) rod shaped (b) spherical shaped (a) species (b) genus (c) family (d) phylum
(c) comma shaped (d) spiral shaped
12. Scientific study of diversity of organisms and their
5. A common characteristic of all vertebrates is evolutionary relationship is
(a) presence of skull (a) morphology (b) anatomy
(b) division fo body into head, neck, trunk and tail (c) taxonomy (d) systematics
(c) presence of two pairs of functional appendages
(d) body is covered with an exoskeleton
13. Chitin is present in cell wall of
(a) fungi (b) bacteria (c) yeast (d) algae
6. All chordates possess
14. The kingdom of prokaryotes is
(a) exoskeleton (b) limbs
(a) Protista (b) Monera
(c) skull (d) axial skeletal rod of notochord
(c) Fungi (d) Plantae
366 Study Package for NTSE
15. Single celled eukaryote are included in 18. Canal system is present in phylum
(a) Protista (b) Fungi (c) Archaea (d) Monera (a) Annelida (d) Porifera
(c) Acanthocephala (d) Echinodermata
16. Which of the following are called vascular
cryptograms? 19. Budding is found in
(a) Pteridophyte (b) Bryophytes (a) Sycon (b) Hydra
(c) Gymnosperms (d) Algae (c) Fasciola (d) Obelia
17. Gametophytic and sporophyte phases are 20. Which of the following phyla does not possess a
independent in coelom?
(a) pteridophytes (b) bryophytes (a) Platyhelminthes (b) Annelida
(c) gymnosperms (d) pheophytes (c) Mollusca (d) Echinodermata

High Skill Questions


1. Female reproductive part of bryophyte is 8. Dry skin with scales or scutes without gland is a
(a) anthredium (b) oogonium characteristic of
(c) archaegonium (d) sporangium (a) mammals (b) birds
(c) snake (d) crocodile
2. Which has vascular tissue, produces spores but does
not has seeds? 9. Mosses occur in moist place because
(a) Bryophyta (b) Pteridophyta (a) they cannot grow on land
(c) Gymnosperms (d) Angiosperms (b) their gamete fuses in water
(c) they lack vascular tissue
3. Fruits are not found in gymnosperm because (d) they lack root and stomata
(a) they are not seedless
(b) they are not pollinated 10. Which of the following is a gymnosperm?
(c) they have no ovary (a) Mango (b) Walnut (c) Funaria (d) Chilgoza
(d) fertilisation does not take place 11. Pseudocoelom is not found in
4. Which of the following plants produces seed but not (a) Ascaris (b) Ancylstoma
flower? (c) Fasciola (d) None of these
(a) Maize (b) Mint 12. Nephridia of earthworm are performing same
(c) Peepal (d) Pinus functions as
5. Classification of sponges is primarily based on the (a) gills of prawn (b) flame cells of planaria
(a) body organisation (b) body plan (c) trachea of insects (d) nematoblast of Hydra
(c) skeleton (d) canal system 13. The number of heart chambers found in cockroach is
6. Which of the following have highest number of (a) 4 (b) 7 (c) 5 (d) 13
species in nature? 14. Pearls are produced by the animals of phylum
(a) Insects (b) Birds
(a) Annelid (b) Arthropoda
(c) Angiosperms (d) Fungi
(c) Mollusca (d) Echinodermata
7. Tube-feet is the locomotory organ of
15. Tube feet is the locomotory organ in
(a) Platyhelminthes (b) Echinodermata (a) star fish (b) jelly fish
(c) Mollusca (d) Arthropoda (c) silver fish (d) Scoliodon

Answers
Elementary Level
1. (a) 2. (a) 3. (a) 4. (b) 5. (b) 6. (d) 7. (c) 8. (b) 9. (c) 10. (d)
11. (a) 12. (d) 13. (a) 14. (b) 15. (a) 16. (a) 17. (a) 18. (d) 19. (b) 20. (a)

High Skill Questions


1. (c) 2. (b) 3. (c) 4. (d) 5. (d) 6. (a) 7. (b) 8. (c) 9. (b) 10. (c)
11. (c) 12. (b) 13. (d) 14. (c) 15. (a)
Chapter

9
Our Environment
Ecosystem Food Chain
Term ‘Ecosystem’ was proposed by British ecologist It is the simpler and linear relationship between the
AG Tansley (1935). He defined the ecosystem as ‘The producer and consumer of different trophic levels. It
system’ resulting from the integration of all the living shows the flow of energy from one trophic level to
(biotic) and non-living (abiotic) factors of environment. another in linear direction.
An ecosystem consist of biotic components like living Types of Food Chain
organism, (autotrophs, heterotrophs, macroconsumers, Grazing Food Chain It starts from producers (green
microconsumers) and abiotic components (air, wind, plants) goes to herbivores and finally to carnivores.
water, etc.), which interact with each other and
e.g., green plants → rabbit → jackal → tiger.
maintain a balance in nature.
Prasitic Food Chain It starts from host and reaches
Components of Ecosystem to parasite, e.g., human beings → helminthes → bacteria.
Components of ecosystem are classified as Saprophytic Chain It starts from organic material
and goes to microorganism, e.g., leaves→ primary colonizer
Abiotic Components → secondary colonizer. It is also known as detritus food
l Basic abiotic components are inorganic substances like chain.
C, N, P, H2O, etc. These are involved in mineral cycle.
Organic compound like carbohydrate, fat, protein,
which links abiotic with biotic components. Flow of Energy
l Physical factors which includes air, temperature, solar Flow of energy is unidirectional. Energy captured by
radiation, etc.
autotrophs does not revert back or taken by
Biotic Components heterotrophs. Green plants capture 1% of the solar
l Autotrophs/Produces These are mostly green energy which fall on them and covert it into food energy.
plants including photosynthetic bacteria, which can Some amount of energy is lost through decay or
synthesise their own food. It also includes chemosynthetic respiration from one trophic level to another. Only 10%
bacteria. of the food eaten is turned into its own body and made
l Heterotrophs or Consumers These are the available for the next consumers.
organism, which are dependent on autotrophs to obtain
their food energy.
l Decomposers These include mainly bacteria, Ecological Pyramids
actinomycetes, fungi. These breakdown complex It represents the trophic structure and function of the
compound into simpler compounds.
ecosystem. Producers forms the base and successive
trophic level make up the apex.
368 Study Package for NTSE

It includes pyramid of number. It is the relationship


between the number producers, primary consumers, Biomagnification
etc. It is the gradual accumulation of chemicals/pesticides
Pyramid of number of grassland ecosystem is upright, from one trophic level to another. Soil contain several
while that of tree ecosystem is inverted. pesticides and chemicals, which is absorbed by plants.
These plants are taken up by animals and pesticides
enters the food chain. These chemicals are
TC TC
non-biodegradable and gets accumulated from one
SC SC trophic level another trophic level.
Humans occupies top most position in the food chain
PC PC
hence maximum concentration of chemicals get
P P accumulated in humans.

Upright pyramid Inverted pyramid Ozone Depletion and Ozone Hole


l Ozone is present in the stratosphere. It is formed when
P — Producers
O2 combine with free (O) to give O3.
PC — Primary Consumers (herbivores)
SC — Secondary Consumers (carnivores) O2  UV
→ O + O
TC — Tertiary Consumers (carnivores) O + O2 → O3 (ozone)
l Presence of ozone is an essential necessity of life on
Pyramid of Biomass Earth. It absorbs dangerous UV β-rays of sun and
protects the earth surface from high energy radiation.
It represents the biomass of members of each trophic
l In recent years, amount of ozone began to decrease
level. It could be inverted or upright. sharply. It is due to man-made pollutants, which
catalyze. The dissociation of O3 at very fast rate. Major
Pyramid of Energy pollutants which deplete ozone layer are
chlorofluorocarbons (CFCs), N2O (nitrogen oxide).
It shows the flow of energy from one trophic level to
Hydrocarbons and oxides of chlorine and bromine.
another trophic level. It is always upright and
l In 1987, the United Nations Environment Programme
unidirectional.
(UNEP) succeeded to freeze CFC production.

Target Exercise
Elementary Level
1. Energy flow in ecosystem is 5. Extinction of species in a food chain is compensated
(a) bidirectional (b) unidirectional by
(c) all around (d) None of these (a) food chain (b) ecological pyramid
2. Pyramid that is never inverted (c) food web (d) None of these
(a) energy (b) mass 6. Which of the following is an example man-made
(c) number (d) size ecosystem?
(a) Herbarium (b) Aquarium
3. Pond is defined as a
(c) Tissue culture (d) Forest
(a) biome (b) agroecosystem
(c) natural ecosystem (d) community 7. In a pyramid of numbers in a grassland ecosystem,
the largest population is that of
4. Which of the following shows detritus food chain? (a) producers
(a) Organic waste → bacteria→molluscs (b) tertiary consumers
(b) Grass → insects → snakes (c) secondary consumers
(c) Plankton → small fishes → large fishes (d) primary consumers
(d) All of the above
Our Environment 369
8. The ultimate energy source of ecosystem is 15. These belong to the category of primary consumers
(a) solar energy (b) biomass (a) snakes and frogs
(c) producer (d) carbohydrates (b) water insects
(c) eagle and snakes
9. The term ‘ecosystem’ was given by
(d) insects and cattle
(a) Odum (b) Koestler
(c) Tansley (d) Mobius and Forbes 16. If a water body is contaminated with a toxicant, its
10. In food chain, maximum energy is stored in biomagnification will be more marked in
(a) producer (b) primary consumer (a) water (b) planktons
(c) small fishes (d) birds
(c) secondary consumer (d) decomposer
11. In food chain, lion is a 17. Biological equilibrium is found among the
(a) tertiary consumer (b) secondary consumer (a) producers, consumers and decomposers
(b) producers and consumers
(c) primary consumer (d) None of these
(c) producers and decomposers
12. The correct sequence of food chain is (d) None of the above
(a) grass → insect → snake → bird
18. In terrestrial ecosystem such as forest, maximum
(b) grass → bird → insect → snake
(c) snake → bird → insect → grass
energy is found in which trophic level?
(d) grass → snake → bird → insect (a) T1 (b) T2
(c) T3 (d) T4
13. Ecosystem consist of
(a) producers (b) consumers
19. Most stable ecosystem is
(c) decomposer (d) All of these (a) desert (b) marine
(c) mountain (d) forest
14. In a food chain total amount of living material is
depicted by 20. Most diverse organisms of an ecosystem are
(a) pyramid of biomass (b) pyramid of energy (a) producers (b) consumers
(c) carnivores (d) decomposers
(c) pyramid of number (d) trophic levels

High Skill Questions


1. The ecological niche of population is a 6. The 10% energy transfer law of food was given by
(a) geographical area that it covers (a) Lederberg (b) Lindemann
(b) place where it lives (c) Weismann (d) Lindley
(c) set of conditions and resource it uses
(d) None of the above
7. Overlapping region between two ecosystem is called
(a) biome (b) ocotone
2. The great barrier reef along the East coast of (c) niche (d) photic zone
Australia can be categorised as
8. In ecotone, some species become abundant called
(a) population (b) community
(c) ecosystem (d) biome (a) sibling species (b) endemic species
(c) rare species (d) edge species
3. An ecosystem which cab be easily damaged but can
recover after some time if damaging effect stops, will
9. Energy flow and energy transformation in living
system strictly conform to the
be having
(a) law of limiting factors
(a) low stability and high resilience
(b) Liebigs law of minimum
(b) high stability and low resilience
(c) law of thermodynamics
(c) low stability and low resilience
(d) high stability and high resilience (d) Shelford’s law of tolerance

4. The amount of living matter present in an ecosystem 10. The lentic ecosystem includes
at a given time is called (a) gravitational H2O (b) standing H2O
(c) rain water (d) running water
(a) biomass (b) standing crop
(c) standing state (d) productivity 11. Mr X is eating curd/yoghurt. For this food take in a
5. Stratification occurs in food chain, he should be considered as occupying
(a) desert (a) first trophic level
(b) tropical forest (b) second trophic level
(c) deciduous forest (c) third trophic level
(d) fourth trophic level
(d) tundra
370 Study Package for NTSE

12. In a comparative study of grassland ecosystem and 14. Detritus food chain law accounts for more energy flow
pond ecosystem. It may be observed that than grazing food chain because
(a) biotic components are almost similar (a) most organisms die without having being eaten
(b) the abiotic components are almost similar (b) most organisms do not die
(c) primary and secondary consumers are similar (c) most organism having being eaten
(d) both biotic and abiotic components are different (d) None of the above

13. Energy storage at consumers level is called 15. Niche is defined as the
(a) gross primary productivity (a) position of species in a community in relation to other
(b) secondary productivity species
(c) net primary productivity (b) place where organism lives
(c) place where organism live and perform its duty
(d) net productivity
(d) place where population perform their duties

Answers
Elementary Level
1. (b) 2. (a) 3. (c) 4. (a) 5. (c) 6. (b) 7. (a) 8. (a) 9. (c) 10. (a)
11. (a) 12. (a) 13. (d) 14. (a) 15. (d) 16. (d) 17. (a) 18. (a) 19. (b) 20. (a)

High Skill Questions


1. (b) 2. (c) 3. (a) 4. (b) 5. (b) 6. (b) 7. (b) 8. (d) 9. (c) 10. (b)
11. (a) 12. (d) 13. (b) 14. (a) 15. (c)
Chapter

10
Natural Resources
Natural resources of earth constitutes land, water, air, Rain
mineral, etc. 75% of the earth’s surface is covered by During the day time water bodies get heated up, large
water, which constitutes hydrosphere. Outer layer of amount of water evaporates goes up into the air. It
the earth crust is called lithosphere and constitutes 25% expands and condenses into tiny droplets which fall
of earth’s surface. Air which covers the earth like a down in form of rain.
blanket and is known as atmosphere. Hydrosphere,
lithosphere and atmosphere together with living Air Pollution
components constitute biosphere. Abiotic components of l It could be defined as unfavourable alteration in physical,
biosphere is soil, H2 O and air. chemical or biological characteristics of air.
l Air pollutants could be gaseous in nature like CO2,
NO2, SO2, SO3 or particulate (substances which may be
Air solid or liquid, e.g., aerosols, smoke. Unburnt carbon
It is the mixture of many gases like N2 , O2 , CO2 and particles) in nature. In winters, this suspended
particulate matter combines with fog to produce smog
water vapour. Air constitutes 21% of O2 , 78% N2 , 0 .03%
which lowers the visibility.
of CO2 . Percentage of CO2 in air is less since, it is fixed by
green plants to convert CO2 into glucose in presence of Bioindicators
sunlight. Marine animals also uses calcium carbonate to Some organisms are very sensitive towards the level of
make their shells. contaminants like sulphur dioxide. These are indicators
of pollution for example, lichens which are commonly
Role of Atmosphere in Climate Control found on barks of trees as thin greenish white crust.
Atmosphere plays a very important role in
thermoregulation. It maintains the temperature during Water
the day and night. During the day time it does not allow
It occupies 75% of the earth’s surface and also found
harmful UV rays to reach Earth. In night, it slows down underground. It is essential for animals and plants in
the escape of heat into the outer space. order to survive. All the cellular processes need water as
Movement of Air medium. It is one of the major factor which determines
the sustainability and survival of life in a particular
l Due to uneven heating of the atmosphere in different
region.
regions , convection currects are set up which causes
movement of air.
Water Pollution
l During day time water vapour is formed due to
l It could be defined as the addition of undesirable
evaporation. Excessive heating causes air over the plain substances to the water bodies which deteiorates
to rise up creating low pressure above the land. Air over physical, chemical and biological properties of water.
the sea is heavy and it start moving towards the low air
Water pollutants could be organic or inorganic in
pressure such movements causes wind.
nature.
372 Study Package for NTSE
Organic water pollutants include detergents, food
l
processing waste, insecticides and herbicides, Nitrogen Cycle
petroleum hydrocarbons, volatile organic compounds, Ultimate source of N2 in living organism is atmospheric N2 .
etc. Aerial N2 could be fixed by nitrogen fixing bacteria in
l Inorganic water pollutant includes acidity caused by form of ammonia. This unstable form is converted into
industrial discharge. Ammonia from food processing nitrites and finally into nitrates. Plant take up nitrogen
waste. Chemical waste as industrial byproduct. in nitrate form and convert nitrate into amino group
which is available for animals.
Soil Nitrogen is return to H2 O and soil through excretion and
death of nitrogenous compounds in living matter.
It is the outmost layer of Earth called crust and the Denitrifying bacteria also converts nitrate and nitrites
minerals found in this layer capable of supporting life. into elemental nitrogen.
Process of formation of soil from huge rocks, is known
N2 in
as weathering. It occurs with the aid of biological, atmosphere
Nit
physical and chemical agents. Denitrification
fixaroge
tio n
n
Physical Agents Nitrates
It includes sun, water and wind. Sun heats up the rocks Protoplasm
(green plants)
during the day. So that they expand, when these rocks Protoplasm
(animals)
cool down they contract and develops cracks.
Ultimately huge rocks break up into smaller pieces.
Nitrification Nitrites
l Water moves into these cracks, it freezes and causes
cracks to widen. Fast flowing water can also bring Ammonification
down big and small pieces of rock. The rocks
sub-against each other causing abrasions, which Ammonia
Nitrification
causes the rocks to wear down into smaller and smaller
particles.
Atmos n
pheric and industrial fixatio
l Winds also acts in same way.
Nitrogen cycle in nature
Biological Agents
Lichen grows on surface of the rocks. They release Carbon Cycle
certain chemical substances which degrades the rock Carbon is found in various forms. In elemental form it
surface. occurs as granite and diamond. It is find as CO2 in
atmosphere. Carbonates and hydrogen carbonates in
Humus various minerals.
CO2 in atmosphere
It is the top layer of soil, which contain bits of decayed
Respiration and decomposition

living organism and organic matter. It causes soil to


become more porous and allows water and air to
penetrate deep underground.
Organic compounds
(plants)
Biogeochemical Cycle
It is the representation which shows the interaction
between biotic and abiotic component of ecosystem. It
shows the circulation of chemicals from environment to Organic compounds
organism and back to environment in a cyclic manner. (animals)
Carbonates
Inorganic in water
Water Cycle carbonates Coal
(shells)
Water from oceans evaporates from water bodies and Petroleum
Limestone
condense to form clouds. It fall down on earth as rain. Carbon cycle in nature
Some water flow back into sea, some form the
Photosynthesis incorporates CO2 into organic substances.
underground water and some is utilized by plants and
It replenished through respiration, burning of industrial
animal as freshwater.
fuel combustion.
Natural Resources 373
Oxygen Cycle
It is found in its elemental form O2 . It is also found as oxides of most metals and silicon. It is also an important
component of most biological molecules. Atmospheric molecular O2 enters the organism through respiration and
back to atmosphere through photosynthesis.

Greenhouse Effect
The increased amount of CO2 in atmosphere is found to increase temperature of Earth. CO2 has higher capacity to
trap infra red radiation and does not allow long wave radiation from Earth to escape into outer space. This causes
increase in temperature and called greenhouse effect.

Target Exercise
Elementary Level
1. One of the following greenhouse gas contribute about 9. Common indicator organism of water pollution is
20% of the global warming (a) Lemna pancicostata
(a) CH4 (b) CFCs (b) Eichhornia crassipes
(c) CO 2 (d) N2O (c) Escherichia coli
2. Section through different layers of soil is called (d) Entamoeba histolytica
(a) soil section (b) soil horizon 10. Which of the following are the indicators of pollution?
(c) soil profile (d) All of these (a) Licher (b) Fungi
3. Organic matter present in soil is called (c) Algae (d) None of these
(a) fertiliser (b) humus 11. Which of the following nutrient cycle shows
(c) living matter (d) non-living matter sedimentary cycle?
4. Which layer of soil is richest in humus? (a) Sulphur cycle (b) Phosphorous cycle
(c) Oxygen cycle (d) Both (a) and (b)
(a) B-Horizon (b) A-Horizon
(c) C-Horizon (d) Bed rock 12. The solid component of Earth is called
(a) lithosphere (b) stratosphere
5. The best soil for the growth of wheat crop is
(c) atmosphere (d) troposphere
(a) loamy soil (b) sandy soil
(c) clayey soil (d) All of these 13. The multilayered gaseous envelope surrounding
planet Earth is called
6. Loamy soil is most suitable for
(a) lithosphere (b) hydrosphere
(a) paddy (b) pulses (c) atmosphere (d) ionosphere
(c) wheat (d) None of these
14. The main factor responsible for water pollution is
7. Which of the following pair of gases are the major (a) smoke
cause of ‘greenhouse effect’? (b) NH3
(a) CO 2 and CO (b) CFCs and SO 2 (c) industrial waste
(c) CO 2 and N2O (d) CO 2 and O 3 (d) detergent
8. CO2 is called greenhouse gas because it is 15. Melting of glaciers may result from
(a) used in greenhouse to increase plant growth (a) depletion of ozone layer
(b) transparent to heat but traps sunlight (b) increase in UV rays
(c) transparent to sunlight but traps heat (c) increase in O 2 in the atmosphere
(d) transparent to both sunlight and heat (d) increase in CO 2 in the atmosphere
374 Study Package for NTSE

High Skill Questions


1. Which of the following atoms typically cycles within 6. Mercury pollution causes
the most localised area? (a) black-foot disease
(a) Carbon (b) Water (b) itai-itai disease
(c) Nitrogen (d) Phosphorus (c) blue-baby syndrome
(d) minamata disease
2. Smog is caused by the
(a) depletion of ozone layer 7. CO is a pollutant because it
(b) presence of high humidity in the atmosphere (a) reacts with O 2
(c) rapid accumulation of CO 2 in the atmosphere (b) inhibits glycolysis
(d) rapid build-up of products of photochemical oxidation of (c) reacts with haemoglobin
pollutant in the atmosphere (d) makes nervous system inactive

3. Water pollution is best assessed by determining 8. Acid rain is due to


(a) DO and BOD (a) CO 2 and H2O (b) CO and NO 2
(b) BOD and turbidity (c) SO 2 and NO 2 (d) SO 2 and N2O
(c) DO and acidity
9. At present the concentration of CO2 in atmosphere is
(d) Hardness and alkalinity
about
4. The protective ozone shield is to be found in the (a) 100 ppm (b) 240 ppm
(a) hydrosphere (b) ionosphere (c) 380 ppm (d) 520 ppm
(c) stratosphere (d) troposphere
10. Eutrophication is often seen in
5. Acid rain is mainly caused due to increase in the (a) freshwater lakes
levels of the gases (b) ocean
(a) SO 2 (b) CO 2 (c) mountains
(c) SO 2 and CO 2 (d) NO 2 and SO 2 (d) deserts

Answers
Elementary Level
1. (c) 2. (c) 3. (b) 4. (b) 5. (a) 6. (c) 7. (a) 8. (c) 9. (c) 10. (a)
11. (b) 12. (a) 13. (c) 14. (c) 15. (d)

High Skill Questions


1. (c) 2. (d) 3. (b) 4. (c) 5. (d) 6. (d) 7. (c) 8. (c) 9. (c) 10. (a)
Chapter

11
Management of Natural
Resources
It includes sustainable use and conservation of our recently instituted an ‘Amrita Devi Bishnoi National
natural resources (like forest, water bodies, wildlife, Award’ for wildlife conservation in memory of Amrita
coal, petroleum, etc.). It includes protection uplift and Devi.
scientific management of our resources to maintain
their optimum level, and derive sustainable benefits for Sustainable Management
present as well as future generation. Forest resources are often made available for industrial
l There are number of national laws and acts for use of rates for below the market value while these are
environmental protection. denied to the local people.The Chipko Movement was
l There is general 3R rule to save our resources. the result of a grassroot level effort to end the alienation
l This implies Reduce, Recycle and Reuse. of people from their forests.
Reduce Reduction is the usuage of resources, e.g., not
Chipko Movement (Hug the Trees Movement)
wasting water, switching off unnecessary lights and fans.
Originated in remote village called Reni in Garhwal
Recycle Recycle of non-biodegradable products like high up in Himalayas during 1970. Local villagers
recycling of plastic paper, glass or metal. actively participated in protect the forest. There was
Reuse It is using the things again and again, e.g., dispute in between. Local villagers and contractor.
plastic bottles could be used to store jam or pickle. Contractor was not able to cut down the forest trees
because women of the village reached the forest and
clasped the tree trunk preventing the workers from
Forest and Wildlife felling the trees.
Forest are ‘biodiversity hot spots’ and loss of Peoples Participation in the
biodiversity leads to ecological instability’ We are
Management of Forest
dependent on forest for Fibre, fodder, food, furniture,
Due to the lack of traditional practice sal forest in southern western. District
fuel. The forest department of the Government owns the of West Bengal was degraded. West Bengal forest department was not able
land and control the resources of forest. to revive this forest. In years 1972, forest officer AK Banarjee involve local
India has 581 protected areas, covering 4.7% of the land villagers to protect 1,272 hectares of Arbari forest range of Midnapore district
due to active and willing participation of local people. Sal forest underwent a
surface. Protected areas are the part of land or sea.
remarkable recovary.
Specially dedicated to protection and maintenance of
biological diversity. There are 89 National Parks and
492 Wildlife Sanctuaries in India. Water Resource Management
l In 1731 Amrita Devi Bishnoi died with 363 others for Water availability depends upon the rain fall pattern.
protection of ‘khojri’ trees in Khojrali village near Rains in India is largely dependent on monsoons.
Jodhpur in Rajasthan. Government of India has
Despite of heavy rainfall. We are not able to sustain
376 Study Package for NTSE

the ground water level. It is due to the neglact local Water Harvesting
irrigation methods and conservation strategies.
It aims at developing a primary resource for
Irrigation methods like dams, tanks and canals have
conservation of water. Which could be used in a manner
been used in various parts of India since ancient times.
which will not disturb ecological imbalance. Ancient
Kulhs in Himachal Pradesh system of water harvesting is being promoted to
conserve water.
It is the local system of canal irrigation water flowing in stream is diverted
into man-made channels, which took this water this numerous villages Some ancient practices for water harvesting is
down hill.
(i) Khadins and Nadis — Rajasthan
(ii) Bandharas and Tal — Maharashtra
Dams (iii) Bundhis — Madhya Pradesh
(iv) Ahors and Pynos — Bihar
l Dams are the large reservoires of water. They ensure
the storage of adequate water for irrigation as well as (v) Kulhs — Himachal Pradesh
generating electric it. People close to the H2O source can (vi) Surangams — Kerala
be benefitted by, growing H2O intensive crops like (vii) Kaltas — Karnataka
sugarcane and rice, e.g., Indira Gandhi canal has
brought greenary to considerable areas of Rajasthan.
l Apart from benefits there arises opposition to the Coal and Petroleum
construction of large Dam, e.g., Tehri dam on river Ganga. These are essential and non-renewable resource of
Narmada Bachao Andolan etc.
energy. These are formed by the degradation of biomass
l Criticism of construction of large dam is due to following million years ago and hence can be exhausted in near
reason. Inadequate compensation or rehabilitation to
future.
large number of local tribes.
The management of coal and petroleum also addresses
l Construction of dam involves huge amount of public
money, which would be masted if benefit are not the efficiency of our machines. Fuel is most commonly
generated. used in internal combustion engines for transportation
and recent research in the field of ensuring complete
l It involves enormous deforestation, which would lead to
loss of biodiversity. Apart form these there is also threat combustion in these engines in order to increase
from Natural calamities like earthquake, etc. efficiency and also reduce air pollution.

Target Exercise
Elementary Level
1. ‘Chipko Movement’ is concerned with 4. Which soil has the maximum water holding capacity?
(a) plant conservation (b) project tiger (a) Sandy soil (b) Clayey soil
(c) animal breeding (d) plant breeding (c) Loamy soil (d) Both (a) and (c)
2. In the recent years, there has been an increasing 5. Most fertile farm lands have
incidence of floods in the plains of northern India (a) acidic soil (b) alkaline soil
because (c) neutral soil (d) None of these
(a) there has been an increase in annual rainfall
6. In your opinion, which is the most effective way to
(b) the rate of silting of dams has done up
(c) there has been increased deforestation in the catchment
conserve the plant diversity of an area?
areas (a) By tissue culture method
(d) increased areas of land is being self cultivated (b) By creating biosphere reserve
(c) By creating botanical garden
3. Which agricultural practice help in maintaining soil (d) By developing seed bank
fertility? 7. India has only 2.4% of the world’s land area, its share
(a) Crop rotation cropping of the global species diversity is
(b) Inter cropping
(a) 1.8% (b) 3.1%
(c) Multiple cropping
(d) All of the above (c) 5.1% (d) 8.1%
Management of Natural Resources 377
8. Total number of identified biodiversity hot spots in 10. Which of the following is the use of lichens in case of
the world are pollution?
(a) 25 (b) 24 (c) 40 (d) 34 (a) Lichens are not related with pollution
9. Kanha National Park is located in .....and is famous (b) They act as bioindicators of pollution
(c) They treat the polluted water
for ..... .
(d) They promote pollution
(a) Madhya Pradesh, elephant (b) Madya Pradesh, tiger
(c) Odisha, tiger (d) Asom, elephant

High Skill Questions


1. The general rule to preserve of our resource is 6. The wild life protection Act was introduced in
(a) Reduce (b) Recycle (a) 1972 (b) 1981 (c) 1986 (d) 1991
(c) Reuse (d) All of these
7. Prolonged liberal irrigation of agricultural field is
2. ‘Amrita Dui Bishnoi’ is associated with likely to create problem of
(a) Protection of Tigers (a) acidity (b) aridity
(b) Protection of ‘Khejn Trees’ in Kejrali (c) metal toxicity (d) salinity
(c) Protection of trees in Reni in Garhwal
8. Minerals, metals and fossil fuel are which type of
(d) Protection of Arbari forest range
resources of energy?
3. Local system of canal irrigation in Himachal Pradesh (a) Renewable
is (b) Non-renewable
(a) Khadins (b) Bandharas (c) Biodegradable
(c) Kulhs (d) Ahars and Pynos (d) Degradable
4. Rajaji National Park is situated in 9. The greatest problem of water conservation is to
(a) Tamil Nadu (b) Karnataka reduce the amount of
(c) Uttarakhand (d) Rajasthan (a) precipitation (b) run-off water
5. Biodiversity Act of India was passed by the (c) ground H2O (d) evaporation
parliament in year 10. Which of the following is an in exhausitable resource?
(a) 1996 (b) 1992 (a) Fossil fuel (b) Solar energy
(c) 2002 (d) 2000 (c) Coal (d) Petroleum

Answers
Elementary Level
1. (a) 2. (c) 3. (d) 4. (c) 5. (c) 6. (b) 7. (c) 8. (d) 9. (b) 10. (b)

High Skill Questions


1. (d) 2. (b) 3. (c) 4. (c) 5. (c) 6. (a) 7. (d) 8. (b) 9. (b) 10. (b)
Chapter

12
Improvement in Food
Resources
As the population is growing, there is an increase in food l High Yield to increase the productivity.
demand. Although several efforts have been done to l Improved Quality Quality of seeds, protein content,
meet this demand by increasing food production. There type of fatty acids, preserving quality in fruits and
is an urgent need to increase our food production vegetables.
without degrading our environment and disturbing l Biotic and Abiotic Resistance Biotic stress includes
ecological balance. disease caused by various post and abiotic stress includes,
drought, salinity water logging, heat, cold and frost.
l Change in Maturity Duration Uniform maturity is
Improvement in Crop Yield helpful in harvesting shorter duration of crop from
sowing to harvesting enables the farmers to grow
There are mainly two types of crops which are grown,
multiple rounds of crops in a year.
e.g., Kharif crops and Rabi crops.
l Wider Adaptability Crop should have capability to
Kharif crops are grown in rainy season, e.g., paddy, be grown under different climatic conditions.
soyabean, pigeon pea, etc. l Desirable Agronomic Traits This includes
Rabi crops are grown in winter season, e.g., wheat, dwarfness in case of cereals. Tallness and profuse
gram, peas, mustard, linseed. branching is desirable characteristics for fodder crops.

Crop yield could be improved by


Crop Production Management
l Crop variety improvement.
l Yield of a crop depends upon the farming practices and
l Crop production improvement. agricultural technologies employed during its cultivation.
l Crop protection management. l There is correlation in between higher inputs and yield.
Better financial condition of farmers boosts farmers
Crop Variety Improvement purchasing capacity for inputs and decides the type of
The basis of crop variety improvement programme is cropping system to be followed.
breeding of superior quality variety.
l Breeding of superior quality could be done on the basis of
Nutrient Management
disease resistance, high yield, product quality, post l In total 16 nutrient are required for the proper growth of
resistance, abiotic stress tolerance. plant. Some of them are macronutrients which are
required in large quantities, e.g., N, P, K, C, O,
l Hybridisation is a common technique, which is used to
Ca, S and Mg. Some of them are micronutrients
incorporate desired characteristics in crop.
which are required in small quantities, e.g., Zn, Fe, Cu,
Various traits for which variety improvement is done Mn, Cl, Mo and B.
are
Improvement in Food Resouces 379
Manure
It is the decomposed organic matter which enriches the soil with nutrients and increases soil fertility. Increases the
water holding capacity of soil.
Manure

Vermicompost Green Manure


Compost consist of decomposed organic matter Before sowing some plants like sunnhemp
which include animal waste, vegetable waste, and guar are grown and later mulched by
sewage waste, etc. ploughing. These act as a green manure
and enriches the soil with N2 and O2.

When earthworms are used for decomposition of this waste process is known as vermicomposting

Fertilisers Cropping Pattern


These are the chemical compounds which provide It includes different way of growing crops
specific type of nutrient to the plant. These should be
applied in propordose, time and with precautions. Mixed Cropping
Continuous use of fertilisers destroys soil fertility since, Growing two or more crops together on same field, e.g.,
microorganisms in soil are harmed by fertilisers and wheat, gram or ground nut and sunflower.
organic matter cannot be replenished.

Organic Farming
Inter Cropping
Growing of two crops simultaneously in a definate
Farming system with maximum use of organic manures
pattern, e.g., few rows of one crop and few rows of
and minimal or no use of chemical fertilisers.
another crop, e.g., soyabean + maize and millet + cowpea.
Biopesticides and biofertilisers are used in this type of
cropping system.
Crop Rotation
Irrigation Growing of different crop alternatively on same piece of
Farming system in India is largely dependent on land. It helps in maintaining the fertility of soil.
monsoons. Sometimes crop failure occurs due to poor
monsoon. Hence, proper irrigation is needed to ensure Crop Protection Management
that the crop get water at right stage of plant
development. Field crops are damaged by large number of weeds and
pest.
Different type of irrigation system are
Weeds are the unwanted plants in cultivated field,
Well e.g., Parthenium (gajar ghas), Xanthium (gokhroo),
Cyperinus rotundrus (motha).
Pests Various insects attacks the plant, through root,
Dug well Tube well
stem and leaf. They suck cell sap or bore into stem and
Water is collected from H2O Tube well can trap energy fruits.
bearing a strata. from deeper strata by using pumps.
Pathogens Various pathogens like bacteria, fungi and
viruses attack crops. Fungicides, insecticides and
Canals It receive H 2O from main river. Canal is
herbicides are used to control this problem.
divided into many distributories to irrigate the field.
River Lift System Water is directly drawn from the
river.
Animal Husbandry
It is the scientific management of animal livestock,
Tanks Small storage reservoire which store the run
off rain water and later could be used for irrigation. which includes feeding, breeding and disease control. As
the population is increasing there is an increase in
Rain water harvesting and small check dams are also demand for milk, eggs and meat. Hence, livestock
used for irrigation. production need to be improved and managed.
380 Study Package for NTSE

Cattle Farming Egg and Broiler Production


It is done to get milk as drought labour for agricultural Broiler chickens are fed with vitamin-A and vitamin-K
work like tilling, irrigation, carting, etc. Indian cattle rich diet. Maintenance of temperature and hygienic
belongs to Bos indicus (cows) and Bos bubalis (buffaloes). conditions in housing and poultry feed.

Milch Fish Production


These are milk producing females. Fish production is done through marine and freshwater
Drought Animals ecosystem. Popular marine fish varieties are pomphret,
mackerel, tuna, sardines, Bombay duck, finned fishes
l Use to farm labour. Milk production depends upon
like mullet, bhetki, pearl spots, shell fish like prawns.
lactation period some foreign breed with increased
Oysters are also cultivated for the pearl they make.
lactation period are Jersey, Brown swiss, while, local
Some freshwater fishes includes catlas, rohus, silver
breeds like red Sindhi, Sahiwal show resistance to
carp, grass carp, mrigal and common carp.
diseases.
l Proper cleaning and shelter facilities are required for
cows and buffaloes. Maintenance of proper hygiene is
Bee Keeping
required. Food requirement of dairy animals includes Bee keeping is also known as apiculture. Local varieties
roughage and concentrates. are used for commercially honey production like Apis
corana indica known as Indian bee. A. dorsata (rock
Poultry Farming bee) and A. florae (little bee). An Italian bee variety A.
It is undertaken to raise the domestic fowl after egg Mellifera has also brought an increase in commercial
production. honey production.

Target Exercise
Elementary Level
1. Crosses between the plants of same variety are called 8. BGA is chiefly used as biofertiliser in the crop of
(a) interspecific (b) intervarietal (a) wheat (b) gram
(c) intravarietal (d) inter generic (c) paddy (d) mustard
2. Plants having similar genotypes produced by plant 9. Which one of the following statements is incorrect?
breeding are called (a) Green manure added to the soil
(a) clone (b) haploid (b) Fertilisers causes soil and water pollution
(c) auto polyploid (d) genome (c) Biofertilisers improve soil fertility
(d) Berseem is an ideal biofertiliser
3. Dwarf wheat was developed by
10. A manure supply makes which nutrients available to
(a) MS Swaminathan (b) Vavilov
(c) Dr NE Borloug (d) BD Singh the plants
(a) nitrogen (b) phosphorus
4. Quickest Method for plant breeding is (c) potassium (d) All of these
(a) introduction (b) selection
11. Crop rotation is used by farmers to increase
(c) hybridization (d) mutation breeding
(a) soil fertility
5. Objective of plant breeding is (b) community area
(a) better yield (b) better quality (c) organic content of the soil
(c) disease/stress resistance (d) All of these (d) nitrogenous content in the soil
6. A man made allopolyploid cereal crop is 12. Bee keeping is known as
(a) Hordeum vulgare (b) Triticale (a) pearl culture (b) pisciculture
(c) Raphanus brassica (d) Zea mays (c) sericulture (d) apiculture
7. The name Norman Borlaug is associated with 13. Apis dorsata refers to
(a) green revolution (b) yellow revolution (a) rock bee (b) little bee
(c) white revolution (d) blue revolution (c) Indian bee (d) European bee
Improvement in Food Resouces 381
14. Which of the following is called marine fish 15. Pearl is get from
(a) Catla catla (b) Labio rohita (a) fish (b) whale
(c) Bombay duck (d) Wallago attu (c) oyster (d) Sepia

High Skill Questions


1. Uprooting of weeds is a method of controlling them by 5. Humus is
……… means. (a) partially decomposed organic matter
(a) biological (b) completely decomposed organic matter
(b) mechanical (c) partially decomposed inorganic matter
(c) cultural (d) completely decomposed organic matter of pond
(d) chemical 6. The scientist who are concerned with the
2. Integrated pest management discourages the improvement of plants is known as
excessive use of (a) plant breeders (b) pathologist
(a) biological methods (c) microbiologist (d) economist
(b) chemical pesticides 7. Disease resistance crop is obtained by
(c) mechanical methods (a) crossing with new varieties
(d) variable cultural control (b) crossing with wild varieties
3. Which of the following is not used as Biopesticide? (c) injecting with organic compounds
(d) None of the above
(a) Bacillus thuringiensis
(b) Trichoderma harzianum 8. Heterosis is
(c) Nuclear Polyhedrosis Virus (NPV) (a) hybrid sterility (b) hybrid vigour
(d) Xanthomonas campestris (c) hybrid incompatibility (d) hybrid in viability
4. Cow dung is appropriatly used as 9. Pureline breeds refers to
(a) manure (a) Homozygosity and independent assortment
(b) fuel (b) Homozygosity only
(c) medicine (c) Heterozygosity
(d) building material (d) Heterozygosity and linkage

Answers
Elementary Level
1. (a) 2. (a) 3. (a) 4. (c) 5. (d) 6. (b) 7. (a) 8. (c) 9. (d) 10. (d)
11. (a) 12. (d) 13. (a) 14. (c) 15. (c)

High Skill Questions


1. (b) 2. (b) 3. (c) 4. (a) 5. (a) 6. (a) 7. (b) 8. (b) 9. (a)
382 Study Package for NTSE

Part-IV : Mathematics
Chapter

1
Number System
A system in which we study to write and read of the Even and Odd Numbers
numbers, is called number system. In the Hindu Arabic Numbers which are divisible by 2 are called even and
system we use the symbols 0, 1, 2, 3, 4, 5, 6, 7, 8 and 9.
which are not divisible by 2 are called odd numbers.
These symbols are called digits.
To write a number, we put digits from right to left the In general, even numbers can be represented by 2n
places as the following table (e.g. the number 16 9 75 2 and odd numbers can be represented by 2n ± 1, where
384 write). n is an integer.
Ten Crore’s Crore’s Ten Lakh’s Lakh’s Ten Thousand’s Note ● The sum and product of any number of even numbers is
1 6 9 7 5 an even number.
● The difference of two even numbers is an even number.
Thousand’s Hundred’s Ten’s Unit’s
2 3 8 4 ● Prime number A number greater than 1 having
It is read Sixteen crore ninty seven lakh fifty two exactly two factors, 1 and itself is called a prime
thousand three hundred eighty four. number. e.g., 2, 3, 5, 7, ...
Face and place values of the Digits in a Note 2 is the only even prime number.
Number ● Composite number A number which has factors
In a numeral the face value of a digit is the value of the besides other than 1 and itself is called composite
digit itself and place value of digit changes according to number. e.g., 6, 8, 12, 15, ...
the change of its place. e.g., Face value and place value of Note 1 is neither prime nor composite.
3 in the numeral 4 8 3 7 2 9, are 3 and 3000. ● Coprime or relative prime Two numbers are
called coprime or relative primes, if they have only one
Types of Numbers common factor which is 1. e.g., (7, 9) is coprime but 15
There are following some types of numbers and 21 is not coprime.

Natural Numbers Integers


Numbers which are used for counting i.e., 1, 2, 3, 4, ... Whole numbers and negative natural numbers
are called natural numbers. Smallest natural number is constitute the set of integers.
1 but we cannot find the largest natural number as i.e., Z = ..., - 4, - 3, - 2, -1, 0, 1, 2, 3,...
successor of every natural number is again a natural
number. Here, positive numbers are called positive integer and
negative numbers are called negative integers.
Whole Numbers
Rational and Irrational Numbers
When zero is included in natural numbers, then it is called
a set of whole numbers i.e., 0, 1, 2, 3, 4, ... . Zero is the Any number which can be expressed in the form of
smallest whole number but there is no largest whole p/q, where p and q are integers and q ¹ 0 is called a
number. Every natural number is also a whole number. rational number. e.g., 4 / 5, -1/ 2, 0 etc. Every natural
Number System 383
number, whole number and integer is also a rational (iv) Test for a number divisible by 5 If the digit at
number. Any number which cannot be expressed in unit’s place is 5 or 0, then the number is divisible by
p 5. e.g., 895, 700 etc.
the form of , is called irrational numbers.
q (v) Test for a number divisible by 6 If a number is
x+ y divisible by 2 and 3, then it is also divisible by 6.
● If x and y are any two rational numbers, then is a
2 e.g., 759312: Here, as the last digit or unit’s digit is
rational number between them. 2. So, number is divisible by 2.
● If x and y are any two distinct irrational numbers, then Also, the sum of digits 7 + 5 + 9 + 3 + 1 + 2 = 27 ¸ 3 = 9
xy is an irrational number between them. is divisible by 3. Thus, the number is divisible by 6.
Real Numbers (vi) Test for a number divisible by 7 If double of
unit’s place digit of given number is subtracted from
All these numbers which have been discussed above
rest of digits and if the remainder is divisible by 7,
are called real numbers i.e., when natural numbers,
then that number is divisible by 7.
whole numbers, integers, rational and irrational
numbers are collected together, they are called real e.g., 875 ® 87 - (2 ´ 5) = 87 - 10 = 77 ¸ 7 = 11
numbers. Hence, 875 is divisible by 7.
Example 1 The product of any number and the 1st whole î This trick is applicable for number greater than 99.
number is
(a) 0 (b) 2 (c) 1 (d) -1 (vii) Test for a number divisible by 8 If the last three
digits of a number are divisible by 8 or are 000, then
Solution (a) 1st whole number = 0 the number is divisible by 8.
Clearly, when any number is multiplied with 0 (the 1st whole
number), then the result is 0. e.g., 96432 ® 432 ¸ 8 = 54 is divisible by 8 and 16000
is divisible by 8.
p
Example 2 Express 0.235 in the form, where p and q are (viii) Test for a number divisible by 9 If the sum of all
q
the digits of a number is completely divisible by 9,
integers and q ¹ 0.
then the number is divisible by 9.
235 1 233 235
(a) (b) (c) (d) e.g., 317349 ® 3 + 1 + 7 + 3 + 4 + 9 = 27 ¸ 9 = 3
990 99 990 999
Solution (c) In the numerator, take the difference between So, 317349 is divisible by 9.
the number formed by all the digits after decimal point (ix) Test for a number divisible by 10 If zero exists at
(repeated digits will be taken only once) and the number the unit’s place, then the number is divisible by 10.
formed by non-repeating digits. In the denominator, place as
e.g., 130, 15680 etc.
many nines as there are repeating digits and after nine, put
as many zeroes as the number of non-repeating digits. (x) Test for a number divisible by 11 If the difference
235 - 2 233 between the sum of digits at even places and sum of
0.235 = =
990 990 digits at odd places is 0 or 11, then the number is
divisible by 11.
Divisibility Test e.g., 123432166
Sum of digits at odd places = 1 + 3 + 3 + 1 + 6 = 14
(i) Test for a number divisible by 2 If the number is an
even number or has ‘0’ in its unit place. Sum of digits at even places = 2 + 4 + 2 + 6 = 14
e.g., The numbers like 17980, 314782, 2854, 316, Again, 14 - 14 = 0, so number is divisible by 11.
6148, as all are even numbers, so are divisible by 2.
(ii) Test for a number divisible by 3 If the sum of the
Division on Numbers
digits of the given number is divisible by 3. (Division Algorithm)
e.g., 23457 : Here, 2 + 3 + 4 + 5 + 7 = 21 ¸ 3 = 7 Let ‘a’ and ‘b’ be two integers such that b ¹ 0 on
Hence, 23457 is divisible by 3. dividing ‘ a’ by ‘b’.
(iii) Test for a number divisible by 4 If the number Let ‘q’ be the quotient and ‘r’ the remainder, then the
formed by the ten’s place and unit’s place digits is relationship between a, b, q and r is a = bq + r.
divisible by 4 or last two digits are zero or divisible
by 4. e.g., 589372 . Here, 72 ÷ 4 = 18 In general, we have
So, it is divisible by 4. Dividend = Divisor ´ Quotient + Remainder
384 Study Package for NTSE

Example 3 Find the dividend when divisor is 13, quotient is O — Of (Multiplication); D — Division
30 and remainder is 12. M — Multiplication; A — Addition
(a) 402 (b) 543 (c) 436 (d) 455 and S — Subtraction
Solution (a) Given, divisor = 13, quotient = 30,
remainder = 12 and dividend = ?
Formulae to Remember
We know that,
● + b)22 = a 22 -+ 22ab
(a - ab ++ bb22

Dividend = Divisor ´ Quotient + Remainder ● ( a + b + c )2 = a 2 + b2 + c 2 + 2ab + 2bc + 2ca


= 13 ´ 30 + 12 = 390 + 12 = 402 ● ( a + b)3 = a 3 + b3 + 3 ab ( a + b) = a 3 + 3 a 2b + 3 ab2 + b3
● ( a - b)3 = a 3 - b3 - 3 ab ( a - b) = a 3 - 3 a 2b + 3 ab2 - b3
To Find the Unit’s Place Digit of a Given ● a 2 - b2 = ( a + b) ( a - b)
Exponential ● a 3 + b3 = ( a + b) ( a 2 - ab + b2 )
● In case of 0, 1, 5, 6—the unit’s place digit is 0, 1, 5 and ● a 3 - b3 = ( a - b) ( a 2 + ab + b2 )
6, respectively. ● a 3 + b3 + c 3 - 3 abc
● In case of 4 and 9 = ( a + b + c ) ( a 2 + b2 + c 2 - ab - bc - ca )
(i) If power is odd, then unit’s place digit is 4 and 9, ● a 3 + b3 + c 3 = 3 abc, if a + b + c = 0
respectively.
Example 4 Find the value of (625 - 425 + 25) + 50 ¸ 10 ´ 3?
(ii) If power is even, then unit’s place digit is 6 and 1,
respectively. (a) 240 (b) 250 (c) 350 (d) 154
● In case of 2, 3, 7, 8-see the following examples Solution (a) ( 625 - 425 + 25) + 50 ¸ 10 ´ 3
e.g., = ( 200 + 25) + 50 ¸ 10 ´ 3 = 225 + 50 ¸ 10 ´ 3
= 225 + 5 ´ 3 = 225 + 15 = 240
To Find the Unit’s Place Digit of (134647) 553

Step I 553 ¸ 4 gives 1 as remainder, this remainder is Simple Fraction


taken as new power.
553 1 1 The fraction, which has denominator other than power
Step II (134647) º (134647) º 7 = 7
of 10, is called simple fraction.
So, the unit’s place digit is 7. 3 5 7
or e.g., , , , etc.
7 11 9
Step III If on dividing the remainder obtained is zero,
● Simple fraction is also known as vulgar fraction.
take 4 as new power instead of zero.
e.g., (134647)552 º (134647)0 º ( 7)0 º ( 7)4 = 2401 Types of Simple Fractions
So, the unit’s place digit is 1. There are following types of fractions
Note ● an - bn is always divisible by ( a - b), if a, b and n are natural
1. Proper Fraction When the numerator of a fraction
numbers and divisible by ( a + b), if n = Even numbers.
● an + bn is divisible by ( a + b), if n = Odd numbers. is less than its denominator, then fraction is called
● The total number of factors of a number proper fraction.
N = a p ´ b q ´ c r , where a , b , c are different prime 1 15 21
e.g., , , , etc.
numbers, is ( p + 1) (q + 1) ( r + 1.
) 2 17 43
2. Improper Fraction When the numerator of a
To Find the Number of Prime factors of the fraction is greater than its denominator, then
Expression fraction is called improper fraction.
The number of the prime factors the expression 17 18 45
a p b q c r d s = p + q + r + s, where a, b, c and d are prime e.g., , , , etc.
13 14 19
numbers.
3. Like and Unlike Fractions The fractions whose
denominators are same, is called like fractions,
Simplification whereas fractions whose denominators are different,
The word ‘simplification’ refers to a procedure useful is called unlike fractions.
for converting long fractions or expressions in one 2 3 6
e.g., , , etc., are like fractions
fraction or numbers. 11 11 11
For solving questions on simplification, we should use the 2 8 6
‘VBODMAS’ rule. The letters of word means
and , , etc., are unlike fractions.
3 7 4
V — Vinculum, i.e., bar; B — Brackets;
Number System 385
4. Equivalent Fractions The fractions whose values (ii) Non-terminating and non-repeated
are same, is called equivalent fractions. decimals The number which neither terminates nor
2 4 6 repeats in process of division. e.g., 1.030030003........
e.g., = = .
3 6 9 3 etc. These numbers are also called irrational
numbers.
5. Compound Fraction A fraction, in which
numerator or denominator or both are in fraction,
is called compound fraction. Indices
1 11 / 9 1 / 4 Any number, multiplied by itself m times is written as
e.g., , , , etc.
7 / 9 13 7 / 13 a m , where a is called base and m is called power or
index or exponent.
6. Inverse Fraction If we inverse the numerator
and the denominator of a fraction, then the
● Any non-zero real number when raised to the power zero
is equal to 1.
resultant fraction will be the inverse fraction of the
original fraction. ● When m is a positive integer, a m = a ´ a ´ a ´ a ´ K m
3 8 times.
e.g., If fraction = , then its inverse fraction = .
8 3 Laws of Exponents
7. Mixed Fraction The fraction, which is the
combination of integer and fraction, is called mixed S.No. Laws Examples
m+ n
fraction. 1. m
a ´a =a n
2 ´ 23 = 22 +
2 3
= 25
2 1 2. m
a ¸a = a n m- n 6
2 ¸2 = 2 2 6-2
= 24
e.g., 3 , 7 , etc.
5 9 3. a- m = 1/ am 2 -4 = 1/ 2 4
q
8. Continuous Fraction It has no certain 4. a p /q
= a p 3
24/ 3 = 24
definition but only say that a fraction contains 5. (am ) n = amn (2 3 )2 = 2 6
additional fractions in its denominators, is called
6. (a b )m = am × b m (2 ´ 3)6 = 2 6 ´ 36
continuous fraction. m m 6 6
7. æ aö a æ2 ö = 2
1 1 ç ÷ = ç ÷
e.g., (i) 2 + (ii) 6 + èbø bm è 3ø 36
2 1
2+ 1+ 8. a0 = 1
2 1 1
5+ + 9. a means a1
3 2 2
l
To simplify a continuous fraction, start from bottom and work
upwards. Surds
9. Decimal Fraction It the fraction has nth root of a i.e., n a is called a surd, if a is a rational
denominator in the powers of 10, then fraction is number, n is a positive integer and n a is irrational.
6
called decimal fraction. e.g., = 06
..
10 e.g., 2, 3, 5 etc., p, 9 are not surds as p is not
There are two types of decimal fraction. rational and value of 9 is not irrational.
Terminating Decimals Laws of Surds
The rational number which terminates after a finite
S.No. Laws Examples
number of step in process of division is called a
terminating decimals. 1. (n P )n = (P1/ n )n = P (3 4 )3 = (41/ 3 )3 = 4
e.g., 1.25, 3.14 etc. 2. n PQ = n
P ´nQ 3
2´3 = 32 ´3 3
n
3. P P 6 36
Non-Terminating Decimals n = 3 =
Q nQ 7 37
● The number which doesn’t terminate in the process of
4. (n P )m = (P1/ n )m = P m / n (3 5 )4 = (51/ 3 )4 = 54 / 3
division is called non-terminating decimals.
It has two types 5. mn
P = mn
P 3 4
12 = 3´4
12
(i) Non-terminating repeated decimals The
number which doesn’t terminate but repeats the Types of Surds
same numbers in process of divisions. ● Order of a surd Order of a surd n a is n.
e.g., 1/ 3 = 0.3333....... = 0. 3 e.g., Order of 4 7 is 4.
386 Study Package for NTSE

● Pure surds A surd which has 1 (unity), the only ● Comparison of surds Suppose the given surds a P ,
rational factor. It may have other factors but only bq, c r . First we take LCM of power of surds a, b, c.
irrational.
Then, we can change of all surds in same order and
4 3
e.g., 2, 9 and 15 comparing them. e.g., In 3 2, 4 5 is the greatest.
● Mixed surds A surd which has rational factors other LCM of 3 and 4 is 12. So 12 24 = 12 16, 12 53 = 12 125. so 4 5
than 1.
is the greatest.
e.g., 2 7, 5 3 7 etc.
9 1
● Like surds Two or more surds having same Example 5 If 27 x = , then find the value of ?.
irrational factor is called like surds.
3 x
x4
(a) 14 (b) 16 (c) 18 (d) 12
e.g., 2 5 3 and 7 5 3
Solution (b) 27 = 9x Þ(3 3 ) x ´ 3 x = 3 2
x

Operations of Surds 3
● Addition and subtraction of like Þ 3 3x + x = 3 2 [
surds Addition and subtraction is defined only for Q am ´ an = am + n ]
like surds. They can be added or subtracted by taking 1
By comparing, 4 x = 2 Þ x=
the irrational factor common. 2
e.g., 53 17 + 19 3 17 = 3 17(5 + 19) = 24 3
17 Now,
1
=
1
=
1
= 16
x4 4 1
5 17 - 17 17
3 3 æ1ö
ç ÷ 16
è 2ø
= 3 17 (5 - 17)
= -12 3 17
HCF and LCM of Numbers
● Multiplication and division of surds
● Factor If y is divisible by x leaving remainder zero,
Multiplication and division is defined only for the surds
n then x is a factor of y.
of same order as n a ´ n b = n ab and a = n a .
n
b b ● Prime factorisation When factorisation of a number
expressed completely involving only prime factors.
● Rationalising factor If product of two surds is a
rational number, then they are rationalising factors of ● Highest common factor The greatest number which
each other. divides two or more numbers is called their HCF or
e.g., 3 16 ´ 3 4 = 4. So, RF of 3 16 is 3 4 GCD (Greatest Common Divisors).
and RF of 3 4 is 3 16. e.g., HCF of 2 and 4 is 2.
● Rationalisation of surds Process of converting ● Least common multiple The least number which is
surd into rational number is called it’s rationalisation. It divisible by two or more given numbers is called their
is carried out by multiplying surd with an appropriate LCM.
rationalising factor. e.g., LCM of 3 is 6 is 6.
(i) The conjugate surd of a + b is ± ( a - b ).
Some formulae related to HCF and LCM
1 ( a - b)
(ii) To rationalise , multiply it by or (i) (LCM ´ HCF) of two numbers = Product of two
a + b ( a - b)
numbers
( b - a)
. LCM of numerators
( b - a) (ii) LCM of fractions =
HCF of denominators
(iii) a a a ... ¥ = a HCF of numerators
(iii) HCF of fractions =
1-
1 LCM of denominators
(iv) a a a a ... n times = a 2n
Example 6 Two numbers have an HCF of 18 and a product
of 3240. Find their LCM.
(v) If a + a + a + ... ¥ = p, then p ( p - 1) = a.
(a) 150 (b) 160 (c) 180 (d) 164
(vi) The rationalising factor of a + b is a - b. Solution (c) According to the question,
(vii) The rationalising factor of a - b is a + b. Product of numbers = HCF ´ LCM Þ 3240 = 18 ´ LCM
(viii) The rationalising factor of a + b is a - b. 3240
\ LCM = = 180
18
Target Exercise
Elementary Level Questions
1. The rational form of - 71.625 is 10. The 6th power of an even number will always end with
71 573 (a) 4 or 2 (b) 2 or 6
(a) - (b)
625 8 (c) 6 or 0 (d) 0, 4 or 6
71
(c) (d) None of these 11. If 32 x is divisible by 9, the value of x is
625
(a) 6 (b) 4 (c) 2 (d) 8
2. The sum of the face value and place value of 7 in
12. The unit’s digit of 3 82 is
343742.
(a) 3 (b) 9 (c) 1 (d) 7
(a) 693 (b) 700
(c) 770 (d) 707 13. The total number of factors of 40 is
(a) 4 (b) 6 (c) 8 (d) 10
3. 0.6 + 0.8 + 0. 92 + 0.3 is equal to
31 31 32 32 14. 1015 + 1215 is divisible by
(a) (b) (c) (d)
10 11 11 12 (a) 15 (b) 22
4. The number of prime factors in the expression (c) 27 (d) None of these
618 ´ 711 ´ 913 is 15. According to the fundamental theorem of arithmetic,
(a) 70 (b) 42 each number can be expressed as product of
(c) 73 (d) None of these
(a) prime numbers (b) composite numbers
5. Maximum number of students, in which 175 bananas (c) even numbers (d) odd numbers
and 105 oranges can be equally divided, is 16. If the sum of two numbers is 15 and their HCF is 5, the
(a) 35 (b) 7 (c) 5 (d) 105 numbers can be
6. Which of the following is the greatest? (a) 5, 10 (b) 10, 15
(c) 5, 15 (d) None of these
(a) 8+ 22 (b) 1+ 29
(c) 12 + 18 (d) 10 + 20 17. The expression 14 n - 6n is always divisible by
(a) 8 (b) 20 (c) 14 (d) 6
7. The value of 42 + 42 + 42 + ... ¥ is
18. If HCF of ( p, q) = 1, then p and q are
(a) 7 (b) 6 (c) 8 (d) 9 (a) even (b) coprime
1 1 (c) odd (d) composite
8. If = 02437
. , then the value of is
4.103 00004103
. 19. If LCM ( p, q) = p ´ q, then their HCF is
(a) 2437 (b) 24370 (a) p q (b) p + q
(c) 2.437 (d) 243700 (c) 1 (d) None of these
9. The last digit of the number (1076) 976 is 20. If 3214 a4 is divisible by 4, then the number of possible
(a) 4 (b) 5 values of a is
(c) 6 (d) 7
(a) 3 (b) 4 (c) 5 (d) 6

High Skill Questions


1. The unit’s digit of (65) 73 ´ (19)45 is 4. The LCM of two numbers is 45 and their HCF is 5, how
(a) 9 (b) 5 (c) 0 (d) 1 many such pairs of numbers are possible?
2n 2n (a) 1 (b) 2
2. The expression 6 -4 is divisible by (c) 3 (d) None of these
(a) 20 (b) 17
(c) 8 (d) None of these 5. The unit’s digit of 12 222 + 23 333 + 34444 is
(a) 3 (b) 2 (c) 4 (d) 5
3. The total number of factors of 120 is
3 3 3
(a) 8 (b) 10 6. 15 - 8 - 7 is divisible by which of the following?
(c) 12 (d) 16 (a) 32 (b) 49 (c) 56 (d) 25
388 Study Package for NTSE

7. The least number which should be multiplied with 12. A number having odd number of factors is
525, so that it becomes a perfect square, is (a) odd number (b) even number
(a) 3 (b) 7 (c) 5 (d) 21 (c) perfect square (d) None of these
1 13. The total number of factors of 12 4 ´ 20 5 is
8. If x + = 2, then what is x equal to?
1 (a) 30 (b) 420 (c) 570 (d) 430
1+
1
2+ 2 4 5
3 14. The HCF of , , is
7 13 11 17 3 9 6
(a) (b) (c) (d) 5 4 1 1
10 10 10 10 (a) (b) (c) (d)
9 27 18 9
9. The area of smallest square which can be formed with
rectangles of dimension 6 cm ´ 4 cm , is
15. The value of 0.2 72 + 0.1363 is
2 2 2 2 3 41 9 1
(a) 16 cm (b) 24 cm (c) 36 cm (d) 144 cm (a) (b) (c) (d)
22 100 22 2
10. The remainder when 13 6 - 3 6 is divided by 2170, is 16. ( x b+ c ) b- c ( x c + a ) c - a ( x a + b) a - b is equal to
(a) 0 (b) 1 2 + b2 + c 2
(c) 16 (d) None of these (a) 0 (b) 1 (c) x (d) xa
. ) 3 - (225
(275 . )3
11. The number of zeroes at the end of 1530 ´ 1012 is 17. The values of 2
(a) 30 (b) 12 . ) + 275
(275 . ´ 225 . )2
. + (225
(c) 42 (d) 6 (a) 0.3 (b) 0.5 (c) 3 (d) 5

Hints and Solutions


Elementary Level Questions
71625 -573 ( 10 + 20 )2 = ( 10 )2 + ( 20 )2 + 2 ´ 10 ´ 20
1. (d) -71625
. =- =
1000 8
= 10 + 20 + 2 200 = 30 + 2 200
2. (d) Required sum = 700 + 7 = 707 Thus, 12 + 18 is greatest.
6 2 8 92 1
3. (b) 0.6 = = , 0.8 = , 0.92 = , 0.3 = 7. (a) Let 42 + 42 + ... ¥ = x
9 3 9 99 3
2 8 92 1 31 i.e., 42 + x = x
\ 0. 6 + 0. 8 + 0. 92 + 0. 3 = + + + =
3 9 99 3 11 Þ 42 + x = x2 [Q by squaring both sides]
4. (c) 6 ´ 7 ´ 9 Þ 2 ´ 3´7 ´ 3´ 3 Þ x2 - x - 42 = 0 Þ ( x - 7 ) ( x + 6) = 0
Þ 2, 3 and 7 are three prime numbers.
\ x=7
\ 618 ´ 711 ´ 913 = (2 ´ 3)18 ´ 711 ´ (32 )13
= 0.2437 ´ 10000 éQ = 0.2437 ù
1 10000 1
8. (a) =
= 218 ´ 318 ´ 711 ´ 326 0.0004103 4.103 êë 4103
. úû
= 218 ´ 344 ´ 711 = 2437
\ Prime factors = 18 + 44 + 11 = 73
9. (c) Last digit of (1076)976 = 6
5. (a) HCF of (175, 105)
5 175, 105 Because, here 6 at the unit place.
7 35, 21 10. (d) Even6 = Even4 + 2 ® 2 6 = 2 4 + 2 = 2 4 ´ 2 2
5, 3
= 5 ´ 7 (taken only common factor for HCF) = 35 Þ 22 = 4
Number of students = 35
46 Þ 4 and 66 ends with 6 and 86 ends with 82 = 4 and 106 ends
6. (c) ( 8 + 22 )2 = ( 8 )2 + ( 22 )2 + 2 ´ 8 ´ 22
with 102 = 0
= 8 + 22 + 2 176 = 30 + 2 176 Thus, 0 or 4 or 6.
( 1+ 29 )2 = ( 1)2 + ( 29 )2 + 2 ´ 1 ´ 29
11. (b) A number is divisible by 9, if the digit’s sum is divisible by
= 1 + 29 + 2 29 = 30 + 2 29 9 Þ 3 + 2 + x = 5 + x be divisible by 9. So, x = 4.
( 12 + 18 )2 = ( 12 )2 + ( 18 )2 + 2 ´ 12 ´ 18 12. (b) 382 = 380 + 2 , so the unit’s digit of 34 K + 2
= 380 + 2
form is
= 12 + 18 + 2 216 = 30 + 2 216 same as for 3 = 9.2
Number System 389
13. (c) 40 = 2 3 ´ 51 17. (a) an - b n is always divisible by (a - b), so 14n - 6n is divisible
So, total number of factors = (3 + 1) (1 + 1) = 8 by 14 - 6 = 8.
n n
14. (b) a + b is divisible by (a + b), if n is odd, so 10 15 15
+ 12 is 18. (b) If HCF ( p, q ) = 1
divisible by 10 + 12 = 22. Then, ( p, q ) both are coprime.

15. (a) We get factors of some numbers like 6, 8 and 14. 19. (c)Q First number ´ Second number
6 = 2 ´ 3, 8 = 2 ´ 2 ´ 2, 14 = 2 ´ 7 = Their LCM ´ Their HCF
Here, we say that each number can be expressed as product of Þ p ´ q = LCM ( p, q ) ´ HCF ( p, q )
prime numbers. p´q p´q
Þ HCF( p, q ) = =
16. (a) Let the numbers be bx and by, b = HCF LCM ( p, q ) p´q
So, numbers are 5x and 5y, where ( x, y) = Coprime Þ HCF( p, q ) = 1
5 x + 5 y = 5 ( x + y) = 15
20. (c) 3214 a4 is divisible by 4, if the last 2 digits are divisible by 4
Þ x + y = 3 Þ ( x, y) = (1, 2 )
i.e., a4, so a can be 0/2/4/6/8 i.e., 5 values.
So, numbers are 5 x, 5 y = 5 and 10

High Skill Questions


1. (b) Unit’s digit of (65)73 = 5 9. (d) LCM of (6, 4) = 12 = Smallest possible side of square
and 45
(19) = 9 (9 odd
= 9 and 5 any number
= 5) So, area of square = 12 2 = 144 cm 2
So, 5´9=5
10. (a) 136 - 36 = (133 )2 - (33 )2 = (2197 )2 - 27 2 is divisible by
2. (a) 62 n - 42 n = (62 )n - (42 )n = 36n - 16n is divisible by 2197 - 27 = 2170, so the remainder is zero.
a - b = 36 - 16 = 20, for all natural number values of n.
11. (b) 1530 ´ 1012 , number of zeroes in any product depends upon
3. (d) 120 = 2 3 ´ 31 ´ 51, the presence of number of 5’s and number of 2’s in it.
So, total number of factors 1530 ´ 1012 = 330 ´ 530 ´ 212 ´ 512 = 212 ´ 330 ´ 542
= (3 + 1) (1 + 1) (1 + 1) = 16 The product has12, 2’s and 42, 5’s, so the number of zeroes is 12.
4. (a) LCM = hxy, where h = HCF and ( x and y) coprime 5 xy = 45 (since, 5’s alone cannot give 10, same
number of 2’s are required)
Þ xy = 9 Þ xy = 1 ´ 9is the every coprime combination possible.
Thus, option (a) is correct. 12. (c) A perfect square number always has an odd number of factors.

5. (a) 12 222
+ 23 333
+ 34 444 13. (c) 12 4 ´ 205 = (2 2 ´ 31 )4 ´ (2 2 ´ 51 )5
= 2 8 ´ 34 ´ 210 ´ 55 = 218 ´ 34 ´ 55
= 12 4 ´ 55 + 2
+ 234 ´ 83 + 1 + 34even number
So, the total number of factors = (18 + 1) (4 + 1) (5 + 1) = 570
= 12 2 + 231 + 34even number 2 4 5 HCF of 2, 4, 5 1
14. (c) HCF of , , = =
=4+ 3+ 6=3 3 9 6 LCM of 3, 9, 6 18
6. (c) We know, a3 + b 3 + c 3 = 3abc 272 1363 - 13 272 1350 9
15. (c) 0.272 + 0.1363 = + = + =
If a+ b+c=0 999 9900 999 9900 22
Here, 15 - 8 - 7 = 0 16. (b) ( xb + c )b -c ( xc + a )c - a ( xa + b )a - b
3 3 3
So, 15 - 8 - 7 = 3 ´ 15 ´ 8 ´ 7 2 -c 2 2 - a2 2 - b2
= ( x)b ( x)c ( x)a
which is divisible by 56 = 8 ´ 7. 2 -c 2 + c 2 - a 2 + a 2 - b 2
2 1 1
= ( x)b = ( x)0 = 1
7. (d) 525 = 5 ´ 3 ´ 7 is a perfect square number should have
even powers as prime numbers. (2.75)3 - (2.25)3
17. (b)
(2.75) + 2.75 ´ 2.25 + (2.25)2
2
So, 3 ´ 7 = 21 is to be multiplied with 525 to make it a perfect
square. Let 2.75 = a, 2.25 = b
1 1 7 a3 - b 3
8. (b) Given, x + =2Þx+ =2 Þx+ =2 Then given expression =
1 3 10
1+ 1+ a2 + ab + b 2
1 7
2+ 2
(a - b )(a + ab + b ) 2
3 =
7 20 - 7 13 (a2 + ab + b 2 )
Þ x=2- Þ x= =
10 10 10 = (a - b ) = 2.75 - 2.25 = 0.5
Chapter

2
Basic Operations on
Arithmetic
Ratio 5. Inverse Ratio If two numbers are in ratio and their
antecedent and consequent are interchanged, then
Ratio of two quantities is the number of times one
the ratio obtained is called inverse ratio.
quantity contains another quantity of same kind.
If x and y are two numbers and their ratio is x : y,
The ratio between x and y can be represented as x : y,
then its inverse ratio will be y : x.
where x is called antecedent and y is called consequent.
e.g., Inverse ratio of 4 : 5 is 5 : 4.
x
i.e. or x : y 6. Compound Ratio If two or more ratios are given
y
and the antecedent of one is multiplied with
Types of Ratios antecedent of other and respective consequents are
also multiplied, then the ratio obtained is called
There are various types of ratios, which are as follow
compound ratio. If a : b, c : d and e : f are three ratios,
1. Duplicate Ratio If two numbers are in ratio, then then their compound ratio will be ace : bdf .
the ratio of their squares is called duplicate ratio. If e.g., Compound ratio of 2 : 5, 6 : 7 and 9 : 13
x and y are two numbers, then the duplicate ratio of 2´6´9 108
x and y would be x2 : y2 . = =
5 ´ 7 ´ 13 455
e.g., Duplicate ratio of 3 : 4 = 32 : 42 = 9 : 16
2. Sub-duplicate Ratio If two numbers are in ratio, Proportion
then the ratio of their square roots is called An equality of two ratios is called the proportion. If
sub-duplicate ratio. If x and y are two numbers, then a c
= or a : b = c : d, then we can say that a, b, c and d
the sub-duplicate ratio of x and y would be x : y. b d
e.g., Sub-duplicate ratio of 2 : 3 is 2 : 3 . are in proportion and can be written as a : b :: c : d,
3. Triplicate Ratio If two numbers are in ratio, then
where symbol ‘::’ represents proportion and it is read
the ratio of their cubes is called triplicate ratio. If x
as ‘ a is to b ¢ as ‘c is to d¢.
and y are two numbers, then the triplicate ratio of x Here, a and d are called ‘extremes’ and b and c are
and y would be x 3 : y 3. called as ‘means’.
e.g., Triplicate ratio of 2 : 3 = 23 : 3 3 = 8 : 27 Example 1 The ratio of A : B = 1 : 3, B : C = 2 : 5 and C : D =
4. Sub-triplicate Ratio If two numbers are in ratio, 2 : 3. Find the value of A : B : C : D.
then the ratio of their cube roots is called (a) 4 : 12 : 30 : 45
sub-triplicate ratio. If x and y are two numbers, then (b) 12 : 4 : 30 : 55
the sub-triplicate ratio of x and y would be 3 x : 3 y . (c) 1 : 2 : 3 : 4
e.g., Sub-triplicate ratio of 1 : 125 = 3 1 : 3 125 = 1 : 5 (d) 4 : 3 : 2 : 1
Basic Operations on Arithmetic 391
Solution (a) Given, A : B = 1 : 3, B : C = 2 : 5, C : D = 2 : 3 ● The number of men (M1 ) is multiplied by the number of
\ A : B : C : D = (1 ´ 2 ´ 2) : (3 ´ 2 ´ 2) : (3 ´ 5 ´ 2) : (3 ´ 5 ´ 3 ) days ( D1 ) that is taken to complete a given work. The
= 4 : 12 : 30 : 45 total number of man-days required to complete the work
remains same i.e., M1D1 = M 2D2.
Mixture ● If two persons can do a certain work in a and b days,
respectively, then they together can do the same work
The new product obtained by mixing two or more
ingredients in a certain ratio is called a mixture. in ab days.
a+b
or ● M1D1 = M 2D2 or M1D1H1 = M 2D2H2
Combination of two or more quantities is known as M1D1H1 M 2D2H2
mixture. or =
W1 W2
Mean Price
The cost price of a unit quantity of the mixture is where, M ® Number of Men, D ® Number of Days,
called the mean price. It will always be higher than H ® Number of Hours, W ® Work done by men
cost price of cheaper quantity and lower than cost ● If a pipe can fill/empty a tank in ‘m’ h, then the part of
price of dearer quantity. 1
tank filled/emptied in 1h = .
m
Rule of Mixture or Alligation ● Time taken to fill a tank is taken positive (+ve) and time
It is the rule that enables us to find the ratio in which taken to empty a tank is taken negative (-ve).
two or more ingredients at the given price must be ● If a pipe fills a tank in mh and another pipe fills in nh.
mixed to produce a mixture of a desired price. 1 1
n1 A 2 - A w Then, part filled by both pipes in 1h = + .
According to this rule, = m n
n2 A w - A 1
Example 2 A and B together can do a piece of work in 12
where, n 1 /n 2 is the ratio, in which two quantities days and A alone can do it in 18 days. In how many days can B
should be mixed, while A1 , A2 and A w are the cheaper alone do it?
price, dearer price and mean price, respectively. (a) 15 (b) 36 (c) 40 (d) 20
The above rule can be represented pictorially as shown Solution (b) ( A + B)'s 1 day work = 1
below 12
Dearer price 1
Cheaper price and A’s 1 days work =
A1 A2 18
Mean price \ B’s 1 day’s work = ( A + B)'s 1 day’s work -A’s 1 day’s work
Aw 1 1 3-2 1
= - = =
12 18 36 36
A2 – Aw Aw – A1
\ Time taken by B to complete the work alone = 36 days
[remember, A1 < A w < A2 ]
Amount of cheaper : Amount of dearer Speed, Time and Distance
A - Aw Time, distance and speed are always related to each
( n1 : n2 ) = 2
A w - A1 other for a body in motion.
Problems involving these quantities can be solved using
Time and Work the following formulae
For doing any work certain time is required. Hence, d 1 s t
● s= , when distance is constant, s µ , Þ 1 = 1
time and work are related to each other. t t s2 t2
The number of units of work (W) done in a unit of time where, s = Speed, d = Distance, t = Time
is the rate of work (R) per unit of time (T). d d
● When speed is constant, d µ t Þ 1 = 2
W t1 t2
Hence, W = RT, T =
R d1 d2
● When time is constant, d µ s Þ =
i.e., R and T are inversely related. s1 s2
A very useful approach to solve problems is the concept ● Speed in metres per second = Speed in kilometres
of man-days. 5
per hour ´
18
392 Study Package for NTSE

Total distance covered


● Average speed =
Total time taken
Percentage
● Relative speeds of two bodies moving with speeds of u1 Per cent means per hundred. To make a comparison
and u2 easier, the concept of percentage is used 16%
(i) in opposite directions = u1 + u2 (read as sixteen per cent which means 16 for every
(ii) in same direction = u1 - u2 16
100) is same as or 0.16 or 16 : 100.
(iii) upstream speed of a boat = u - w 100
(iv) downstream speed of a boat = u + w Any percentage value can be expressed as
(where, u = speed of boat, w = speed of stream) (i) An ordinary fraction (ii) A decimal fraction
● If equal distance (i.e., m : n is 1 : 1) are covered at speeds (iii) As a ratio
of u and v units, then
2 uv To convert a given fraction or decimal or ratio into a
Average speed = percentage, we multiply it by 100.
(u + v)
To convert a given percentage into a fraction, decimal
● If a train of length l, moving at a speed s, or ratio, we divide it by 100.
(i) passes a tree in time t, then l = s ´ t Actual increase
● Percentage increase = ´ 100
(ii) passes a bridge/platform/stationary train of Original quantity
length L in time t, then d = l + L = s ´ t Actual decrease
● Percentage decrease = ´ 100
● If a train of length l, moving with a speed v, crosses a Original quantity
car/moving with a speed u, (v > u ), in time t, then
● If percentage increase is p%, then new value
(i) if they are moving in same directions, then
l = (v - u ) t
p ö
(ii) if they are moving in opposite directions, then = æç + ÷ ´ Initial value
l = (v + u ) t è 100 ø
● If a train of length l1, moving with a speed v, crosses ● If the value of an item goes up/down by x%, the
another train of length l2 moving with speed u(v > u ) percentage reduction/increment to be now made to bring
time t, then
it back to the original level is 100x %.
(i) if they are moving in same directions, then (100 ± x )
( l1 + l2 ) = (v - u ) ´ t
100x
(ii) if they are moving in opposite directions, then ● If A is x% more/less than B, then B is %
(100 ± x )
( l1 + l2 ) = (v + u ) ´ t
less/more than A.
Example 3 A train of length 250 m, passes a platform of 350
m length in 50 s. What time will this train take to pass the
● If the price of an item goes up/down by x%, then the
platform of 230 m length? quantity consumed should be reduced/increased by
100x
(a) 40s (b) 30 s (c) 50s (d) 25 s %, so that the total expenditure remains the
(100 ± x )
Solution (a) Given, length of train = 250 m ; length of
platform = 350 m and time taken to cover the distance = 50 s same.
So, to cross the platform, the train has to cover a distance ● If p1, p2, p3, ... are successive percentages of change, then
= Length of train + Length of platform Final value = [(1 ± p 1%)(1 ± p2%)(1 ± p3%)... ]
= ( 250 + 350 ) m = 600 m ´ (Initial value)
Distance covered 600 If a certain quantity is first increased by x% and then
\ Speed of train = = ●

Time of train 50 increased by another y% , then the effective percentage


= 12 m/s increase in the quantity will be æç x + y + xy ö÷% .
Now, the time taken to cross the platform of length 230 m è 100 ø
Total distance covered
=
Speed of train Growth in Population
Length of train + Length of platform Problems of growth or decay in population can be
=
Speed of train solved by using the formula of compound interest.
r ön
=
250 + 230 480
= = 40 s A = P æç1 + ÷
12 12 è 100 ø
Basic Operations on Arithmetic 393
where, A = Increased value ● If the selling price of m articles is equal to cost price of n
P = Original value articles, where
m-n
r = Rate of growth (i) m > n, then profit percentage = ´ 100
m
n = Time in number of years n-m
(ii) m < n, then loss percentage = ´ 100
m
Description in Vehicle or Machine ● If d1, d2, d3, ... are percentages of successive discounts
r ön
A = P æç1 - ÷ offered on a marked price MP, then
è 100 ø
SP = MP (1 - d1 / 100)(1 - d2 / 100)(1 - d3 / 100)
where, A = Depriciated value ● If d1, d2, d3, ... are the percentages of successive
P = Original value discounts offered, then the effective discount is
r = Rate of depriciation d% = 100 [1 - (1 - d1/100)(1 - d2/100) (1 - d3/100)... ]
n = Number of years ● If x and y are two successive discount percentages, then
Example 4 If the price of a commodity is raised by 40%, by it is equivalent to a single discount percentage of
how much per cent must a householder reduce his consumption x + y - xy/100.
of that commodity, so as not to increase his expenditure? Example 5 Ravish lost 20% by selling a radio set for ` 3072.
4 2 What per cent will he gain by selling it for ` 4080?
(a) 28 % (b) 31 %
7 5 (a) 7.25% (b) 6.25%
3 (c) 5.45% (d) 7.15%
(c) 29 % (d) None of these
7 Solution (b) Given, SP = ` 3072 and loss = 20%
Solution (a) Here, x = 40% 100 ö ´ SP = 100 ´ 3072 = ` 3840
\ CP = æç ÷
According to the formula, è 100 - Loss% ø 80
40
Reduction in consumption = æç ´ 100 ö÷ % Now, CP = ` 3840 and SP = ` 4080
è 100 + 40 ø
\ Gain = SP – CP
40 200 4
= æç ´ 100 ö÷ % = % = 28 % = 4080 - 3840 = ` 240
è 140 ø 7 7
Gain
\ Gain% = ´ 100%
CP
Profit and Loss =
240
´ 100% = 6 .25%
In our daily life, we come across so many financial 3840
transactions. We purchase and sale of some items.
Shopkeepers and businessmen has different Simple Interest and
situations of profit and loss. This is very much Compound Interest
applied aspect of Mathematics in our daily life. Formulae to calculate interest (I) on principal (P) at
Following are some important formulae based on profit the rate of r% of per annum after n yr have been
and loss given below
Important Formulae Amount A= P+I
Pnr
● Profit = Sale Price - Cost Price (i) Simple interest, SI =
100
● Loss = Cost Price - Sale Price
(ii) Compound interest,
● Discount = Marked Price - Sale Price éæ r ö
n ù
CI = P ê ç1 + ÷ - 1ú
● Profit % =
Sale Price - Cost Price
´ 100 êë è 100 ø úû
Cost Price (iii) Compound Amount,n
Cost Price - Sale Price æ r ö
● Loss % = ´ 100 A = P ç1 + ÷
Cost Price è 100 ø
Marked Price - Sale Price
● Discount % = ´ 100 ● Difference between SI and CI
Marked Price
(compounded annually) for
● If two articles are sold at the same price (i.e., the selling
(i) one year is zero.
prices are equal) and the magnitude of percentage of
r ö2
profit x on one article is the same as the magnitude of (ii) two years is P æç ÷.
è 100 ø
percentage of loss x on the second article, then there is
an overall loss and the percentage of loss is x 2/100. r ö2 æ r
(iii) three years is P æç ÷ ç + 3 ö÷ .
è 100 ø è 100 ø
394 Study Package for NTSE

Equal annual instalment under Solution (d) Difference between CI and SI for 3 yr = ` 152
2
r ö æ r
(i) Simple interest, x =
2P(100 + nr) \ P æç ÷ ç + 3 ö÷ = 152
n [200 + ( n - 1)r] è 100 ø è 100 ø
2
20 ö æ 20
P æç + 3 ö÷ = 152
Pr
(ii) Compound interest, x = Þ ÷ ç
é è 100 ø è 100 ø
100 ö ù
n
100 ê1 - æç ÷ ú 1 16
êë è 100 + r ø úû Þ P æç ö÷ æç ö÷ = 152
è 25 ø è 5 ø
Example 6 The difference between CI and SI for 3 yr at the 152 ´ 25 ´ 5
Þ P=
rate of 20% per annum is ` 152. What is the principal lent? 16
(a) ` 1180 (b) ` 1080 Þ P = 9.5 ´ 25 ´ 5 = ` 1187.5
(c) ` 1190 (d) ` 1187.5

Target Exercise
Elementary Level
a2 + b2 8. A job has to be completed by 12 boys in 15 days. If
1. If a : b = c : d, then the value of is
c2 + d2 three boys are absent from the first day, then by what
1 a+ b a-b ab percentage should the remaining boys increase their
(a) (b) (c) (d)
2 c+d c -d cd rate of working to complete the job?
1 1 2 1
(a) 33 % (b) 22 % (c) 40 % (d) 30 %
2. What should be subtracted from each of the numbers 3 2 3 3
54, 71, 75 and 99, so that the remainder are in
continued proportion ? 9. Ravi and Prince contract for a work for ` 3000. Ravi
(a) 9 (b) 7 (c) 4 (d) 3 alone can do it in 12 days. Prince alone can do it in
24 days. With the help of Reena, they complete the
3. In three vessels of 10L capacity, mixture of milk and work in 4 days. The wages of Reena is
water is filled. The ratio of milk and water are 2 : 1, (a) ` 1400 (b) ` 1600
3 : 1, and 3 : 2 in the respective vessels. If all the three (c) ` 1500 (d) ` 1700
vesseles are emptied into a single large vessel, find
10. Ram can count 100 notes of ` 100 in 1 min. Counting
the ratio of milk and water in the resultant mixture
machine can do it in 20 s. If they work together, they
(a) 121 : 41 (b) 117 : 22 (c) 121 : 59 (d) 127 : 41
will count 100 notes in
4. If y varies directly with x and the constant of (a) 12 s (b) 14 s (c) 13 s (d) 15 s
variation is 20, then value of y for x = 6 is
20 11. A train, 130 m long crosses a platform in 30 s with a
(a) 120 (b) 26 (c) (d) 72 speed of 45 km/h, length of platform is
6
(a) 270 m (b) 245 m
5. If 6 men working 8 h per day earn ` 1680 per week, (c) 280 m (d) 296 m
then amount earned by 9 men working 6 h per day
per week is 12. Moving at 140% more than his usual speed. Ravi
(a) ` 1890 (b) ` 1950 reaches his office 6 min early. Time taken with usual
(c) ` 1980 (d) ` 2000 speed is
(a) 21 min (b) 25 min
6. Two pipes can fill a tank in 3 h and 4 h separately and (c) 18 min (d) 15 min
a drain pipe can empty it in 6 h. If tank is empty and
all pipes are opened simultaneously, then number of 13. A and B start at the same time and move with a speed
12 9
hours to fill the whole tank is of km/h and km/h, respectively. B reaches 5 min
(a) 2 h 12 min (b) 2 h 48 min (c) 2 h 36 min (d) 2 h 24 min
5 4
before than A. Distance they have moved is
7. Sita and Gita can do a piece of work in 15 days and (a) 2 km (b) 3 km (c) 4 km (d) 5 km
30 days, separately. They both work for 3 days, then
14. Average speed of train without stops is 40 km/h and
Gita withdraws. Number of days in which Gita
with stops is 26 km/h. How many minutes per hour
completes the remaining work is
does the train stops?
(a) 20 (b) 21 (c) 22 (d) 23
(a) 20 min (b) 21 min (c) 22 min (d) 23 min
Basic Operations on Arithmetic 395
15. Vinod covers a distance of 75 km in 40 min with a 24. Which of the following is 10% of the year?
x (a) 36 days (b) 37 days
speed of x km/h for 8 min, km/h for next 12 min,
2 (c) 36.5 days (d) 36.5 days
x
km/h for next 20 min, the value of x is
2 25. Rahim gets 73% marks and got 135 marks more than
(a) 180 km/h (b) 185 km/h Rita who got 58% marks. Then, which of the following
(c) 187.5 km/h (d) 172.5 km/h is total maximum marks?
(a) 900 (b) 1000
16. Dinesh can row at 18 km/h in still water. If speed of (c) 1100 (d) None of these
stream is 6 km/h, then how long will Dinesh take to
row upto a distance of 6 km and return to the starting 26. A man’s salary is increased by 10%. In order to have
point? his salary back to the original amount, it must be
(a) 40 min (b) 45 min reduced by x%, then value of x is
1 11
(c) 50 min (d) 52 min (a) 11 % (b) 1 %
9 9
17. Hasan rows 36 km with the stream in 6 h and 40 km 9 1
(c) 1 % (d) 9 %
against the stream in 8 h. Speed of boat in still water 11 11
is
27. A gets ` 5000. He spends 15% on study, 28% on house,
(a) 5 km/h (b) 6 km/h
10% as tax. His saving will be
(c) 5.5 km/h (d) 6.5 km/h
(a) ` 2500 (b) ` 2400 (c) ` 2350 (d) ` 2450
18. In a river C is the mid-point between A and B. Hari
28. The salaries of Rohan and Raj together amount to
can row from A to C and return in 8 h and from A to B
` 3500, if they spend 60% and 80% of their respective
in 4 h. How long would he take to row from B to A?
salary and ratio of their savings is 1 : 3, then their
(a) 10 h (b) 11 h
(c) 12 h (d) 13 h respective salaries should be
(a) ` 500, ` 3000 (b) ` 1500, ` 2000
19. A train leaves Delhi at 6 : 00 am and reaches (c) ` 2500, ` 100 (d) None of these
Haridwar 10 : 00 am. Another train leaves Haridwar
at 8 : 00 am and reaches Delhi at 11 : 30 am. At what 29. The salaries of A and B together amount to ` 26000, if
time the trains meet? they spend 75% and 60% of their respective salaries
(a) 8 : 50 am (b) 8 : 52 am
and their savings are equal, then their respective
(c) 8 : 56 am (d) 8 : 45 am
salaries should be
(a) ` 15000, ` 10000 (b) ` 18000, ` 8000
20. X , Y and Z move along a circular path of length (c) ` 20000, ` 6000 (d) ` 16000, ` 10000
1.2 km with speeds of 6 km/h, 8 km/h and 9 km/h,
30. A candidate gets 71% of votes and wins the election
respectively. X and Y move in the same direction but
by 756 votes. If there are two candidates, then the
Z move in opposite directions. If they all start at the
total number of votes is
same time and from same place, how many times will
(a) 1800 (b) 1850 (c) 1860 (d) 1812
X and Z meet any where on the path by the time X
and Y for the first time anywhere on the path? 31. The present value of a vehicle is ` 7290. Its value
(a) 6 times (b) 7 times decreases every year by 10%, then its value before
(c) 8 times (d) 9 times 3 yr should be
21. In a survey, 34% of population was found to know (a) ` 10000 (b) ` 12000
Hindi, 46% to know English and 23% to know both (c) ` 13000 (d) None of these
languages. The percentage of those knowing neither 32. A student fails by 50 marks when he gets 30% marks.
English nor Hindi, is
Another student pass by 30 marks when he gets
(a) 42% (b) 43%
230 marks. Then, which of the following is the
(c) 44% (d) 45%
passing marks?
22. If 40% of a number exceeds the 25% of it by 54. The (a) 500 (b) 600
number is (c) 550 (d) None of these
(a) 350 (b) 240
(c) 360 (d) 260
33. How many shares can be purchased for ` 113625,
if the market value is ` 112.5 and brokerage paid
23. What per cent of an hour is 2 min? is 1%?
(a) 3.2% (b) 3.3% (a) 1200 (b) 1000
(c) 3.6% (d) None of these (c) 1100 (d) 800
396 Study Package for NTSE

34. A person sold two chairs for ` 990 each. On one he 40. The difference between the simple and compound
gained 10% while on other he lost 10%, which of the interest on a certain sum for 3 yr at 10% per annum is
following is correct? ` 46.5. Then, which of the following is the sum?
(a) He gained 1% (b) He lost 1% (a) ` 1500 (b) ` 1600
(c) Neither profit nor loss (d) None of these (c) ` 1700 (d) ` 1800

35. A person purchases 15 L of milk at ` 18.6 per L and 41. The compound interest on a sum of ` 6000 at 16% per
dilute it with 3 L of water. In order to earn the profit of annum for 9 months when interest is compounded
30%, he should sold it at the rate of quarterly, is
(a) ` 20.05 (b) ` 20.10 (a) ` 762.80 (b) ` 854.10
(c) ` 20.15 (d) None of these (c) ` 749.20 (d) ` 924.10
36. By selling a cooler for ` 630, a man gains 5%, at what 42. A sum of ` 8000, invested at compound interest,
price he must sell it to gain 20%? doubles in 5 yr. After 20 yr, it becomes
(a) ` 670 (b) ` 720 (a) ` 964000 (b) ` 128000
(c) ` 780 (d) None of these (c) ` 78000 (d) None of these

37. A person purchased 18 dozen eggs at the rate of ` 8 43. Ajay lent a certain sum of money to Bhanu at 12% per
per dozen. On checking, he found that 16 eggs were annum under simple interest for 3 yr. The interest
broken during transport. In order to earn a profit of received by Ajay is less than the sum by ` 5760. The
12.5%, he should sold each egg for how much whole sum is
amount? (a) ` 8000 (b) ` 10000
(a) 80 paise (b) 82 paise (c) ` 8500 (d) ` 9000
(c) 81 paise (d) None of these
1 44. A certain amount x was deposited for 4 yr and
38. The simple interest on a certain sum of money at 5 % 3 months at 3% simple interest and amounts to
4 2
per annum for 2 yr is ` 1100, Then which of the ` 5412. Then, x is equal to
23 (a) ` 4800 (b) ` 4900
following is the sum? (c) ` 5000 (d) ` 5100
(a) ` 9000 (b) ` 9100 (c) ` 9150 (d) ` 9200
45. Sanjay needs to earn a monthly interest of ` 80.
39. Raj borrowed ` 5000 at 10% per annum on simple The interest rate is 15% per annum but it is paid
interest and lent the same amount at 15% per annum every month. Which of the following is his
on compound interest. At the end of 2 yr, he would investment?
(a) gain ` 612 (b) gain ` 615 (a) ` 6000 (b) ` 6200
(c) gain ` 612.5 (d) loss ` 612.5 (c) ` 6400 (d) ` 6600

High Skill Questions


1. Ajay says to Bharat, ‘‘I am 4 times as old as you were 4. When the distance between a policeman and a thief is
when I was as old as you are’’. The difference of their 150 m, the policeman starts chasing the thief. The
present ages is 15 yr, then the age of Bharat at policeman runs at a speed of 18 m/s and speed of thief
present, is is 12 m/s. Policeman catches the thief in
(a) 30 yr (b) 25 yr (c) 35 yr (d) 40 yr (a) 20 s (b) 25 s
(c) 30 s (d) Cannot catch him
2. A train running at a speed of 54 km/h takes 12 s to
pass a certain point. Then, the length of the train is 5. A bus travels from A to B at a speed of 65 km/h in one
(a) 160 m (b) 170 m (c) 180 m (d) 190 m hour. If the speed is reduced by 15 km/h, then the
extra time taken to cover the same distance is
3. An employee goes to his office from his home at a (a) 18 min (b) 12 min
speed of 33 km/h. If he takes 2 h 10 min for the entire (c) 16 min (d) None of these
journey. Then, the distance between the house and
office is 6. Two trains of equal length are running on parallel
(a) 71.5 km lines in the same direction at the rate of 46 km/h and
(b) 70.4 km 36 km/h. The faster train passes the slower train in
(c) 211 km 36 s, the length of each train is
(d) 89.6 km (a) 50 m (b) 80 m
(c) 72 m (d) None of these
Basic Operations on Arithmetic 397
7. Certain distance is covered at a certain speed. If half 17. A man incurred a loss by selling a pen for ` 80. Had he
their distance is covered in doubled the time. The sold it for ` 180, his gain would have been four times
ratio of the two speeds is the earlier loss. The cost price of the pen will be
(a) 4 : 1 (b) 1: 4 (a) ` 100 (b) ` 110
(c) 2 : 1 (d) 1 : 2 (c) ` 120 (d) None of these

8. If 4 men reap 40 acres in 30 days. How many acres 18. A shopkeeper mixes two varieties of rice in the ratio
will 18 men reap in 12 days? 3 : 4 and sells the mixture at ` 18 per kg. If the cost of
(a) 70 (b) 72
two varieties respectively is ` 16 and ` 13 per kg, then
(c) 60 (d) 68
his profit is
(a) 22% (b) 24%
9. A and B can do a work in 72 days. B and C in 120 days, (c) 25% (d) 26%
A and C in 90 days. In how many days can A alone do
19. 4/11 of a commodity was sold at a loss of 20% and the
the work? rest at a profit of 20%. The net gain is ` 120. The cost
(a) 80 (b) 100 (c) 110 (d) 120 of the commodity is
(a) ` 220 (b) ` 2220
10. Two cars A and B starts at the same time from two
(c) ` 2020 (d) ` 2200
points P and Q, respectively. A starts from P to Q
while B starts from Q to P. They arrive their 20. Ram sold his bag to Amir at a gain of 8% and Amir
1 1 sold it to Raj at a gain of 10%. If Raj paid ` 297 for it,
destination in 3 h and 7 h, respectively. At what the price paid by Ram is
5 5
speed does car A travels, if car B travels with a speed (a) ` 250 (b) ` 260 (c) ` 270 (d) ` 280

of 36 km/h? 21. Sohan sell a plot of land valued at ` 500000 to Mohan


at a loss of 10%. After some time, Mohan sells it back
(a) 24 km/h (b) 54 km/h
to Sohan at a gain of 10%. Then, Sohan had a
(c) 81 km/h (d) 72 km/h
(a) loss of ` 43000 (b) loss of ` 45000
5 (c) gain of ` 5000 (d) None of these
11. Walking of his usual speed, Ram reaches his school
7
6 min late. His usual time to reach school is 22. Raj bought a cycle for ` 1500 and spent ` 100 for
transport and sold it to Jai at gain of 10%. Jai spent
(a) 25 min (b) 30 min
(c) 15 min (d) 20 min
` 180 on it and sold it to Rama at 15% gain. The cost of
cycle for Rama will be
12. A boy walks 1/3 of a journey at 6 km/h, 1/4 of a journey (a) ` 2200 (b) ` 2210
at 2 km/h and the rest at 4 km/h. The average speed of (c) ` 2221 (d) ` 2231
a boy for the total journey is
23. The cost price of 25 articles is equal to the selling
(a) 3.5 km/h (b) 3 km/h
price of 20 articles. Find the gain%.
(c) 2.5 km/h (d) 2 km/h
(a) 20% (b) 25%
13. A boy goes to school with the speed of 3 km/h and (c) 30% (d) 40%
returns with a speed of 2 km/h. If he takes 5 h in all
24. A trader mixes 220 kg of tea purchased at ` 110 per kg
the distance between the village and the school is
with 80 kg of tea purchased at ` 90 per kg. At what
(a) 6 km (b) 7 km
price per kg should he sell the mixture to make a profit
(c) 8 km (d) 9 km
of 18%?
14. If the weight of a 13 m long iron rod be 23.4 kg. The (a) ` 123 (b) ` 124
weight of 6 m long of such rod will be (c) ` 123.51 (d) None of these
(a) 7.2 kg (b) 12.4 kg
(c) 10.8 kg (d) 18 kg 25. A man buys an old car for ` 55000 and sells it at a gain
of 3%. If his overhead expenses are 1%, he sold the car
15. The milk and water ratio in 16 L mixture is 5 : 3, then for
the amount of milk to be added to make this ratio 2 : 1 (a) ` 57216.50 (b) ` 56000
is (c) ` 56111 (d) None of these
(a) 5 L (b) 2 L
(c) 4 L (d) 8 L 26. Ram buys a scooter at 13/15 of its value and sells it for
10% more than its value. His gain percentage is
16. 17% of [18% of {19% of 1000000} ] is 13 13
(a) 29 % (b) 3 %
(a) 5812 (b) 6812 3 23
(c) 5816 (d) 5814 3
(c) 29 % (d) None of these
13
398 Study Package for NTSE

27. When 40% of a number is added to 75, then 34. 30.8% of 745 is same as which of the following?
multiplied by 4, we get 1200. Which one is the value of (a) 3.08% of 74.5
number? (b) 3.08% of 7.45
(a) 668.5 (b) 550 (c) 578 (d) 562.5 (c) 3.08% of 745
(d) 3.08% of 7450
28. Two candidates contested for a post 85 votes were
declared invalid. The winner secured 60% of votes 35. (78% of 1000) + (69% of 500) = (x% of 1000) + (78% of 500).
and win by 78 votes. Total number of votes is polled, Which of the following is the value of x ?
is (a) 69 (b) 73.5
(c) 6.9 (d) 7.35
(a) 562 (b) 482 (c) 475 (d) 745

29. If x is z% of y. Then, y will be how much per cent of x? 36. A merchant buys 200 kg of rice at ` 7.25 per kg,
400 kg of rice at ` 5.75 per kg. He mixes them and
1000 100
(a) % (b) % sells one-third of mixture at ` 6 per kg. At what rate
z z2 should he sell the remaining mixture, so that he may
1000 10000
(c) % (d) % earn a profit of 20% on the whole outlay?
z2 z
(a) ` 28.25 per kg
30. Mohan purchased oranges at the rate of 8 oranges for (b) ` 13.25 per kg
(c) ` 8.25 per kg
` 6 and sold the whole at the rate of 3 oranges for ` 5.
(d) None of the above
The net gain is ` 286. The number of purchased
oranges is 37. By selling an article for ` 450 a man loses 25%. At
(a) 302 (b) 320 (c) 312 (d) 316 what price he should sale in order to gain 25%?
(a) ` 750 (b) ` 700
31. Suresh purchases 20 L of milk at ` 5 per L and dilutes (c) ` 800 (d) ` 650
it with water to increase the volume to 24 L. In order
to earn a profit of 32%, he should sell the milk at the 38. If a shopkeeper sells a bat marked ` 150 at 15%
rate of discount and gives a ball costing 7.50 free with each
(a) ` 5.4 per L (b) ` 5.5 per L bat, then he makes a profit of 20%. His cost price per
(c) ` 5.6 per L (d) None of these bat will be
(a) ` 90 (b) ` 100
32. Deepak purchased a certain quantity of rice at the (c) ` 110 (d) ` 120
rate of ` 1200 per quintal. 20% of the rice was spoilt.
He mixes the impurities to make it again that much 39. A merchant purchase 11 toys for ` 10 and sold 10 toys
quantity. At what price should he sale the rice to gain for ` 11. His profit percentage is
10%? (a) 15% (b) 20%
(a) ` 13 per kg (b) ` 14 per kg (c) 21% (d) 25%
(c) ` 13.4 per kg (d) ` 13.2 per kg
40. A shopkeeper sold the article at a loss of 12%. If he
33. If the commission of 10% is given on marked price, had purchased it for 15% lesser price and sold it for
the gain is 60%. If the commission of 20% is given, the ` 49 more, he would have gained 20%. His original
gain would be cost price is
2 2 2 (a) ` 330 (b) ` 350
(a) 44 % (b) 42 % (c) 43 % (d) None of these
9 9 9 (c) ` 370 (d) None of these
Hints and Solutions
Elementary Level Questions
a c b ´c 10. (d) When Ram and machine will work together counting of 100
1. (d) = or a =
b d d 60 ´ 20
notes will completed in = 15 s
c2 60 + 20
b ´ 2 + b2
2
a2 + b 2 d b 2 (c 2 + d 2 ) b 2
Hence, = = = 11. (b) Length of platform = l1
c2 + d 2 c2 + d 2 d 2 (c 2 + d 2 ) d 2
b 2
b ´ b a b ab and length of train = l2
Now, = = ´ = 5
d 2
d ´ d c d cd Þ l1 + l2 = Speed ´ Time = 45 ´ ´ 30 = 375
18
2. (d) Let number x be subtracted from each of the numbers, then \ l1 = 375 - 130 = 245 m
54 - x 75 - x 7
= . For x = 3, this relation is correct. 12. (a) New speed = 140% of usual speed = of usual speed
71 - x 99 - x 5
3. (c)\ Required Ratio = Quantity of milk : Quantity of water 1
Time µ
Speed
= æç
2 3 3 ö æ 1 1 2 ö
+ + ÷:ç + + ÷
è2 + 1 3 + 1 3 + 2 ø è2 + 1 3 + 1 3 + 2 ø 5
New time = of usual time
7
= æç + + ö÷ : æç + + ö÷ = 121 : 59
2 3 3 1 1 2
è 3 4 5ø è 3 4 5ø 5 2
Time saved = 1 - = of usual time
7 7
4. (a) According to the question, 6´7
= = 21min [Q 6min=usual time]
y= kx [Q k = constant] 2
= 20 ´ 6 = 120 13. (b) Time taken by the person walking slowly
5. (a) By formula, d é Distance ù
= h êQ Time = Speed ú
M1H1 M 2 H2 6´8 9´6 9/ 4 ë û
= Þ =
W1 W2 1680 W2 d
and that of the other = h
\ W2 = ` 1890 12 / 5
6. (d) If all pipes are opened, the part of tank filled in 1 h According to the question,
1 1 1
= + - =
5 d
-
d
=
5 éQ 1min = 1 hù
3 4 6 12 9 / 4 12 / 5 60 êë 60 úû
12 2 4d 5d 1
\ Required time = = 2 h = 2 h 24 min Þ - =
5 5 9 12 12
16d - 15d 1 d 1
7. (b) Let Gita finish the remaining job in x days. Þ = Þ =
36 12 36 12
Þ Sita’s 3 days work + Gita’s ( x + 3) days work = 1
3 x+ 3 6+ x+ 3 \ d = 3 km
Þ + = 1Þ = 1 Þ 9 + x = 30
15 30 30 14. (b) 40 km covered in 60 min without stoppage, 26 km covered in
\ x = 21 60 min with stoppage and difference 14 km cannot be covered
M1D1R1 M 2 D2 R 2 due to stops 40 km takes 60 min.
8. (a) By formula, = 14
W1 W2 So, 14 km takes = ´ 60 = 21min/h
40
4R1
Þ 12 ´ 15 ´ R1 = 9 ´ 15 ´ R 2 Þ R 2 =
3 15. (c) Total distance covered in 40 min
R - R1 8 12 x 20 x
Þ Percentage increase is = 2 ´ 100 = x+ × + × = 75 [Q distance = speed ´ time]
R1 60 60 2 60 2
8x x x 24 x 75 ´ 5
4R1 Þ + + = 75 Þ = 75 Þ x =
- R1 60 10 6 60 2
(R / 3) 1
= 3 ´ 100 = 1 ´ 100 = 33 %
R1 R1 3 \ x = 187.5
16. (b) Speed upstream = 18 - 6 = 12 km/h
9. (c) Ravi worked for 4 days.
4 Speed downstream = 18 + 6 = 24 km/h
He will get = ´ 3000 = ` 1000 Distance
12 Total time taken to row up 6 km and return =
Speed
Prince worked for 4 days.
4 6 6 3
He will get = ´ 3000 = ` 500 = + = h = 45 min
24 12 24 4
So, Reena will get = 3000 - 1000 - 500 = ` 1500
400 Study Package for NTSE

17. (d) Let speed of boat in still water be x km/h and speed of stream 25. (a) Let total marks be x.
is y km/h. Then , 73% of x - 58% of x = 135
36 é distance ù 73 x 58 x
=6 êQ time = speed ú Þ - = 135
x+ y ë û 100 100
Þ x+ y=6 …(i) \ x = 900
40
Þ =8
x- y 26. (d) Let original salary = x
10 x 11x
Þ x- y=5 …(ii) New salary = x + =
100 10
On solving Eqs. (i) and (ii), we get
11x x
2 x = 13 Salary to be reduced = - x=
10 10
\ x = 6.5 km/h x
1
18. (c) Time taken from A to B = 4 h Percentage reduction in salary = 10 ´ 100 = 9 %
11x 11
1
Time taken from A to C = ´ 4=2h 10
2
Alternate Method
Time taken from C to A = Time taken from A to C and back x
\ Required percentage = ´ 100
– Time taken from A to C (100 + x)
= 8 - 2 = 6h 10 1000 1
= ´ 100 = =9 %
(100 + 10) 110 11
Time taken from B to A = 2 ´ Time taken from C to A
27. (c) Saving = 5500 - æç ´ 5000ö÷
15 28 10
= 2 ´ 6 = 12 h ´ 5000 + ´ 5000 +
è 100 100 100 ø
19. (c) Ratio of time of travel of two trains
= 5000 - 2650 = ` 2350
= 4:
7
= 8:7 éQ speed µ 1 ù
2 êë time úû 28. (a) Let the salaries be ` x and ` y.
Ratio of speed of two trains = 7 : 8 According to the question,
Ratio of distance of two before they meet is 7 : 8. x ´ (100 - 60)
100 1
Before the second train started the first travelled for 2 h. =
y ´ (100 - 80) 3
The two trains will meet after 100
7
´ 2 = 56 min after 8 : 00 am æ 40 ´ xö
15 ç ÷
è 100 ø 1 x 1
= Þ =
i.e., 8 : 56 am 20 3 y 6
´ y
20. (b) Time taken by A and B to meet Ist time anywhere on the path 100
1
Distance 1.2 1.2 Hence, Rohan’s salary = 3500 ´ = ` 500
= = = = 0.6h (1 + 6)
Relative speed 8 - 6 2
6
and Raj’s salary = 3500 ´ = ` 3000
Time taken by A and C to meet anywhere out the path (1 + 6)
Distance 1.2 1.2
= = = = 0.8 h 29. (d) Let salaries be ` x and ` y.
Relative speed 9 + 6 15
The number of times A and C meets anywhere on the path by the Then, x + y = 2600
1 \ 25% of x = 40% of y
time A and B meets each other for the first time = 36 ¸ 4.8 = 7
2 x 2y 8y
Þ = Þx=
i.e., 7 times. 4 5 5
8y
21. (b) Percentage of knowing neither Hindi nor English Þ + y = 26000
= 100% - (34% + 46% - 23%) = 43% 5
54 Þ 13 y = 26000 ´ 5
22. (c)\ Required number = 26000 ´ 5
(40 - 25)% \ y= = ` 10000
13
54
= ´ 100 = 360 and x = ` 16000
40 - 25
30. (a) Let total votes be x.
x 71x 29 x
23. (b) Let it is x%, then ´ 60 min = 2 min [Q 1 h = 60 min] 71% of x - 29% of x = 756 Þ - = 756
100 100 100
200
\ x= = 3. 3 % Þ 42 x = 75600
60
10 75600
24. (c) 10% of 365 = ´ 365 = 36.5 days \ x= = 1800
100 42
[Q 1 normal year = 365 days]
Basic Operations on Arithmetic 401
n 3
A = Pæç1 -
r ö æ 10 ö 40. (a) Let x be the sum of amount.
31. (a) ÷ Þ 7290 = P ç1 - ÷ P ´ n ´ r x ´ 3 ´ 10
è 100 ø è 100 ø Simple interest = =
729 100 100
Þ 7290 = P ´
1000 3x
=
\ P = ` 10000 10
3 3
CI = x æç1 +
10 ö æ 11ö - x
32. (a) Let total marks be x. ÷ - x Þx´ ç ÷
è 100 ø è 10 ø
30 ´ x
Then, + 50 = 230 - 30 According to the question,
100
CI–SI = 46.5
3x 150 ´ 10
Þ = 200 - 50 Þ x = é 11 3 ù 3x
10 3 Þ x ê æç ö÷ - 1ú - = 46.5
êë è 10 ø úû 10
\ x = 500
3x
Þ x [1. 331 - 1] - = 46. 5
33. (b) Let the shares are x. 10
1 x [0 . 331 - 0 . 3] = 46.5
Total price for shares, 112.5 x + ´ 112.5 x = 113625
100 46 . 5 46500
\ x= = = ` 1500
Þ112.5 xé1 +
1 ù 0 . 031 31
= 113625
êë 100 úû
41. (c) Here, P = ` 6000 and r = 16%,
113625
\ x= = 1000 é 12 ù
113 .625 éæ r ö
n ù æ 16 1ö 4
Now, CI = P ê ç1 + ÷ - 1ú = 6000 ç1 +ê ´ ÷ - 1ú
34. (b) In this type of transaction always loss accured. êë è 100 ø úû êè 100 4 ø ú
2 2
ë û
Loss % = æç
Common loss or gain ö æ 10 ö éæ 3 ù é 3 ù
\ ÷ % = ç ÷ = 1% 4 ö æ 26 ö
è 10 ø è 10 ø = 6000 ê ç1 + ÷ - 1ú = 6000 ê ç ÷ - 1ú
êë è 100 ø úû êë è 25 ø úû
35. (c) Total cost price = 15 ´ 18.6 = `279
= 6000 æç - 1ö÷ = 6000 ´
17576 1951
15 ´ 18.6 ´ 30 è 15625 ø 15625
Total selling price = 15 ´ 18.6 + = ` 362.7
100 11706000
= = 749.184
362.7 15625
Hence, new selling price = = `20.15
18
= ` 749.20
36. (b) Let cost of cooler be ` x.
42. (b) According to the question,
Then, CP + profit = SP 5
16000 = 8000 æç1 +
r ö
5x ÷
Þ x+ = 630 è 100 ø
100
\ x = ` 600
20 é æ Rate ö
Time ù
SP for gain of 20% = 600 + ´ 600 = ` 720 êQ Compound amount = Principle ç1+ ÷ ú
100 êë è 100 ø úû
5
37. (c) Total CP = 18 ´ 8 = ` 144
2 = æç1 +
r ö
Þ ÷ …(i)
Total SP = 144 + 144 ´
12.5
= ` 162 è 100 ø
100 4
20 é r ö ù
5
After 20 yr, A = 8000 æç1 +
r ö
Total saleable eggs = 18 ´ 12 - 16 = 200 ÷ = 8000 ê æç1 + ÷ ú
è 100 ø êë è 100 ø úû
162
SP of each egg = = 81paise
200 = 8000 ´ 2 4 = ` 128000 [from Eq. (i)]
38. (d) Let the sum be ` x. 43. (d) Let amount is ` x.
x ´ 11 ´ 50 éQ SI = P ´ n ´ r ù
Then, = 1100 Then,Amount – SI = 5760
2 ´ 23 ëê 100 ûú
x ´ 3 ´ 12 64 x
2 ´ 23 ´ 1100 Þ x- = 5760 Þ = 5760
Þ x= = ` 9200 100 100
11 ´ 50 576000
n 2
\ x= = ` 9000
64
39. (c) Earning = Pæç1 +
r ö æ 15 ö
÷ = 5000 ç1 + ÷
è 100 ø è 100 ø 44. (a) SI = Amount – Principal
P ´ n ´ r 5000 ´ 2 ´ 10 x ´ 51 ´ 3
Borrowing = = = ` 1000 = 5412 - x Þ x = ` 4800
100 100 100 ´ 12
5000 ´ 115 ´ 115 P´r´n x ´ 1 ´ 15
\ Gain = - (5000 + 1000) = ` 612.50 45. (c) SI = Þ 80 =
100 ´ 100 100 12 ´ 100
\ x = ` 6400
402 Study Package for NTSE

High Skill Questions


1. (b) Let their present ages are x (Ajay) and y (Bharat), respectively. 1
9. (d) Work done by ( A + B) in 1 day =
According to the question, 72
x = 4[ y - ( x - y)] 1
Work done by (B + C ) in 1 day =
120
Þ x = 8y - 4x
1
Þ 5x = 8y Work done by (C + A) in 1 day =
90
and x - y = 15 (given)
Work done by ( A + B + C ) in 1 day
8y
So, - y = 15 1 ö 1 æ 5 + 3 + 4ö 1
= æç
1 1 1
5 + + ÷= ç ÷=
Þ 3 y = 15 ´ 5 2 è 72 120 90 ø 2 è 360 ø 60
\ y = 25 yr 1 1 1
Work done by A in 1 day = - =
60 120 120
2. (c) Length of train = Distance covered in 12 s
So, A alone will do in 120 days.
= Speed ´ Time
54 ´ 1000 10. (c) P A → ←B Q
= ´ 12 = 180 m
3600
Speed of B = 36 km/h
3. (a) Distance = Speed ´ Time 1 36
1 13 429 Time taken = 7 = h
= 33 ´ 2 = 33 ´ = = 71.5 km 5 5
6 6 6
Distance = Speed ´ Time = æç 36 ´
36 ö 1296
÷= km
4. (b) Difference in speed = 18 - 12 = 6 m/s è 5ø 5
Distance 150 Distance between P and Q = 1296/ 5 km
\ Time taken = = = 25 s
Speed 6 1 16
Time taken by A = 3 = h
5 5
5. (a) Let time taken is x h.
1296 5
So, 65 ´ 1 = (65 - 15) ´ x [Q Distance = Speed ´ Time] Speed = Distance/ Time = ´ = 81km/h
5 16
65 13
\ x= = h 5 7
50 10 11. (c) Since, he walks at of usual speed, so he will take of usual
7 5
Extra time taken = æç - 1ö÷
13
time.
è 10 ø
Extra time = æç - 1ö÷ of usual time = 6 min
7
3 è5 ø
= h = 18 min
10 2
Þ ´ usual time = 6
6. (a) Speed of faster train relative to slower one (relative speed) 5
\ usual time = 15 min
= (46 - 36) km/h = æç10 ´ ö÷ m/s
5
è 18 ø 12. (a) Let the total distance = x km
25 Distance x / 3 x
= m/s Time taken for 1st part = = = h
9 Speed 6 18
Distance covered in 36 s = speed×time = æç ´ 36ö÷ = 100 m
25 x/ 4 x
è9 ø Time for 2nd part = = h
2 8
\ 2 (length of each train) = 100 m Distance 5x 5x
Time for 3rd part = = = h
or length of each train = 50 m Speed 12 ´ 4 48
7. (a) Let x km be covered in y h. Total distance x
Average speed = =
Total time x x 5x
Distance æ x ö + +
Then, 1st speed = = ç ÷ km/h 18 8 48
Time è yø 144 x 144
= = » 3.5 km/h
1 41x 41
x
x
2nd speed = 2 = km/h 13. (a) Let distance between home and school = y km
2 y 4y
Time taken from home to school = x h
x x 1
\ Required ratio = : = 1: = 4 : 1 Time taken from school to home = (5 - x) h
y 4y 4
For journey from home to School
M1 ´ D1 M 2 ´ D2
8. (b) By formula, = y = 3 ( x)
W1 W2
y = 3 x km …(i)
4 ´ 30 18 ´ 12
Þ = For journey from school to Home
40 W2
y = 2 ( 5 - x)
\ W2 = 72 acres y = (10 - 2 x) km …(ii)
Basic Operations on Arithmetic 403
21. (d) He sold at æç 500000 - 500000 ´
From Eqs. (i) and (ii), we get 10 ö
÷ = ` 490000
x=2h è 100 ø
Distance between home to School 10
Bought at 450000 + 450000 ´ = `495000
Þ y = 3 (2 ) = 6 km 100

14. (c) Weight of 13 m long rod = 23.4 kg Loss = 500000 - 495000 = ` 5000
123.4 22. (d) CP of cycle for Raj = 1500 + 100 = ` 1600
Weight of 1 m long rod = kg
13
CP of cycle for Jai = æç1600 + ´ 1600ö÷ + 180 = ` 1940
10
23.4 è ø
Weight of 6 m long rod = ´ 6 = 10.8 kg 100
13
CP of cycle for Rama = æç1940 + ´ 1940ö÷ = ` 2231
15
5
15. (b) Milk = ´ 16 = 10 L è 100 ø
8
3 SP 25
and water = ´ 16 = 6 L 23. (b) 25 ´ CP = 20 ´ SP Þ =
8 CP 20
Þ Gain percentage = æç 1ö÷ ´ 100 = æç - 1ö÷ ´ 100 = 25%
Let x L of milk is added. SP 25
-
x + 10 2 è CP ø è 20 ø
So, =
6 1
24. (c) Total CP = 220 ´ (110) + (80) ´ 90 = ` 31400
Þ x + 10 = 12
\ x=2L CP ´ 18
SP = CP + = 31400 + 5652 = ` 37052
17 é 18 ì 19 100
ù
16. (d) í (1000000)üýú = 5814 37052
100 êë 100 î100 þû \ New rate = = ` 123.51
300
17. (a) Let cost of the pen = x 1
25. (a) Total cost price = 55000 + ´ 55000 = ` 55550
Case I Loss = CP–SP = x - 80 100
Case II Gain = SP–CP = 180 - x 3
Gain = ´ 55550 = ` 1666.50
180 - x = 4( x - 80) 100
Þ 180 + 320 = 5 x \ Selling price = CP+gain
\ x = ` 100 = 55550 + 1666.50 = ` 57216.50
18. (d) Let varieties are 3x kg and 4x kg, respectively. 26. (c) Let the value of scooter = ` x
\ Selling price = 18 (3 x + 4 x) = 126 x 13
CP of scooter = ` x
and cost price = 3 x ´ 16 + 4 x ´ 13 = 100 x 15
x ´ 10 11x
\ Profit = SP–CP = 126 x - 100 x = 26x SP of scooter = x + =`
100 10
Profit 11x 13 x
\ Profit percentage = ´ 100 Gain = SP - CP = -
CP 10 15
26 x 33 x - 26 x 7 x
= ´ 100 = 26% = -
100 x 30 30
7x
19. (d) Profit – Loss = Net gain Gain
Gain percentage = = 30 ´ 100
Let quantity be x. CP ´ 100 13 x
20 æ 7 x ö 20 æ 4 x ö = ` 120 15
ç ÷- ç ÷
100 è 10 ø 100 è 11 ø
=
7 x 15
´ ´ 100 = 29 %
3
30 13 x 13
\ x = ` 2200
20. (a) Let Ram paid ` x.
8 27. (a) Let number be x.
Then, CP for Amir = x + x
100 æ 40 x + 75ö 4 = 1200 Þ 2 x = 225
ç ÷
108 x 10 æ 108 x ö è 100 ø 5
After that, CP for Raj = + ç ÷
100 100 è 100 ø 225 ´ 5
x= = 668.5
2
According to the question,
108 x æ 1 ö 28. (c) Let the total votes are Q.
ç1 + ÷ = 297
100 è 10 ø Then, 60% of (Q - 85) - 40% of (Q - 85) = 78
297 ´ 100 ´ 10 6Q - 510 4Q - 340
\ x= = ` 250 Þ - = 78
108 ´ 11 10 10
\ Q = 475
404 Study Package for NTSE

x y 100 100 36. (c) CP of 200 kg of rice = (7.25 ´ 200) = ` 1450


29. (d) ´ 100 = z Þ = Þy= of x
y x z z
CP of 400 kg of rice = (575
. ´ 400) = ` 2300
10000
Þ y= % of x Total CP of 600 kg of mixture = (1450 + 2300) = ` 3750
z
Profit required on whole mixture = 20%
30. (c) Let x oranges were purchased. CP ´ (100 + Profit ) æ 120
\ SP must be = =ç ´ 3750ö÷ = ` 4500
6 100 è 100 ø
So, CP of x oranges = ´ x
8 1
Now, one-third of the mixture = ´ 600 = 200 kg
5 3
SP of x oranges = ´ x
3 SP of this 200 kg of mixture = (6 ´ 200) = 1200 kg
5 6x 5x 3 SP of remaining 400 kg = (4500 - 1200) = 3300 kg
Now, SP - CP =Gain x - = 286 Þ - x = 286
3 8 3 4
So, rate of selling the remaining mixture = æç
3300 ö
Þ 20 x - 9 x = 286 ´ 12 Þ 11x = 286 ´ 12 ÷
è 400 ø
\ x = 26 ´ 12 = 312
= ` 8 .25 per kg
31. (b) CP of milk = 5 ´ 20 = ` 100 S1 S2
37. (a) =
SP = CP + 32% of CP = 100 + 32 = ` 132 100 + x1 100 + x2
SP 132
SP = = = ` 5.5 per L 450 S2
20 + 4 24 Þ =
100 + (-25) 100 + 25
1200
32. (d) Price = = ` 12 per kg
100 \ S 2 = ` 750
As quantity remains same. 38. (b) Marked price = ` 150
10
So, profit = ´ 12 = ` 1.20 150 ´ 85
100 Selling price = = ` 127.5
100
SP = CP + Profit = (12 + 1.20)
= ` 13.20 per kg Ball price = ` 7.5
Net selling price = 127.5 - 7.5 = ` 120
33. (b) Let marked price = x
100
90 \ CP = ´ SP
Price after commission = x 100 + 20
100
100
Profit = 60% = ´ 120 = ` 100
100 9 9 120
CP = ´ x= x
100 + 60 10 16 39. (c) CP of 1 toy = `
10
When commission = 20% 11
80 4 11
SP = x= x SP of 1 toy = `
100 5 10
4 x 9 x 19 x Here, SP > CP
Profit = - =
5 16 80 11 10 121 - 100 21
Profit = SP - CP = - = =
Profit 10 11 110 110
Profit percentage = ´ 100
CP Profit
19 Profit percentage = ´ 100
x CP
19 x 16
= 80 ´ 100 = ´ ´ 100 21
9x 80 9x
110 21 11
16 = ´ 100 = ´ ´ 100 = 21%
10 110 10
2
= 42 % 11
9
12 22
30.8 40. (b) Let CP = ` x, SP = x - x= x
34. (d) ´ 745 = 0.308 ´ 745 100 25
100 15 x
3.08 If CP = x -
and option (d), ´ 7450 = 0.308 ´ 745 100
100 17 20 17 102 x
1000 500 1000 78 SP = x+ ´ x=
35. (b) 78 ´ + 69 ´ = x´ + ´ 500 20 100 20 1000
100 100 100 100 22 102 x
Þ 78 ´ 10 + 69 ´ 5 Now, x + 49 =
25 1000
= 10 x + 78 ´ 5
\ x = ` 350
\ x = 73.5
Algebra 405

Chapter

3
Algebra

Polynomial Value of a Polynomial f( x ) at x = a


An expression of the form a 0 x + a1 x n n -1
+ a2 x n -2 Let f ( x) = a 0 x n + a1 x n-1 + a2 x n-2 + ¼ + a n be a
+...+ a n-1 x + a n , where a 0 , a1 , a2 ,..., a n are real polynomial in x and a be a real number, then the real
numbers and n is a non-negative integer, is called a number a 0 a n + a1a n-1 + a2 a n-2 + ... + a n is called the
polynomial in the variable x. value of f ( x) at x = a.
Polynomials in the variable x are usually denoted by Thus, if f ( x) is a polynomial in x and a is a real number,
f ( x), g( x) and h( x) etc.
then the value obtained by replacing x by a in f ( x) is
Thus, f ( x) = a 0 x n + a1 x n - 1 + a2 x n-2 + ... + a n-1 x + a n called the value of f ( x) at x = a. It is denoted by f (a ).
● If a 0 ¹ 0, then n is called the degree of the e.g., Let f ( x) = x2 - 2x - 3 be a quadratic polynomial in
polynomial f ( x ); it is written as deg f ( x ) = n. x, then value of f (2) is
● a 0x n , a1x n -1, a 2x n - 2, ..., a n -1x, a n are called the terms of f (2) = 22 - 2 ´ 2 - 3 = - 3.
the polynomial f ( x ); a n is called the constant term.
● a 0, a1, a 2, ..., a n -1, a n are called the coefficients of the
Algebraical Identities
polynomial f ( x ). The following identities are used in factorisation and
simplification of algebraical expression
● If all the coefficients a 0, a1, a 2, ¼, a n -1, a n are zero,
then f ( x ) is called a zero polynomial. It is denoted by (i) ( a + b)2 = a 2 + 2ab + b2
the symbol 0. The degree of the zero polynomial is (ii) ( a - b)2 = a 2 - 2ab + b2
never defined. (iii) ( a + b + c )2 = a 2 + b2 + c 2 + 2ab
+ 2bc + 2ca
Degree of Polynomial
(iv) a 2 - b2 = ( a + b) ( a - b)
The degree of a polynomial is zero if and only if it is a (v) ( a + b)3 = a 3 + b3 + 3 ab ( a + b)
non-zero constant polynomial i.e., if f ( x) = c, c ¹ 0.
(vi) ( a - b)3 = a 3 - b3 - 3 ab ( a - b)
(i) A polynomial of degree one is called a linear
(vii) a 3 + b3 = ( a + b)3 - 3 ab ( a + b)
polynomial. It is of the form ax + b, where a and
(viii) a 3 - b3 = ( a - b)3 + 3 ab ( a - b)
b are real numbers and a ¹ 0.
(ix) a 3 + b3 = ( a + b)( a 2 - ab + b2 )
(ii) A polynomial of degree two is called a quadratic
(x) a 3 - b3 = ( a - b)( a 2 + ab + b2 )
polynomial. It is of the form ax 2 + bx + c, where
(xi) a 3 + b3 + c 3 - 3 abc
a, b and c are real numbers and a ¹ 0.
= ( a + b + c )( a 2 + b2 + c 2 - ab
(iii) A polynomial of degree three is called a cubic - bc - ca)
polynomial. It is of the form ax 3 + bx 2 + cx + d, 3 3 3
(xii) If a + b + c = 0, then a + b + c = 3 abc
where a, b, c and d are real numbers and a ¹ 0.
(xiii) ( x + a ) ( x + b)
(iv) A polynomial of degree four is called a
biquadratic polynomial etc. = x 2 + ( a + b) x + ab
406 Study Package for NTSE

Zeroes of a Polynomial f( x ) Coefficientof x2 -b


a +b+g =- =
A real number a is a zero of the polynomial f ( x) if and Coefficientof x 3 a
only if f (a ) = 0. Coefficientof x c
ab + bg + ga = =
e.g., If f ( x) = 2 x + 3, then Coefficientof x 3
a
-3 Constant term - d
f æç - ö÷ = 2 ´ æç ö÷ + 3 = -3 + 3 = 0.
3 and product of zeroes, abg = - =
è 2ø è 2 ø Coefficientof x 3 a
3
Therefore - is a zero of f ( x).
2 Ratio of Polynomial
● Ratio is defined as the comparison of two or more
To Find the Zeroes of a Polynomial f( x ) similar or identical variables. A ratio, being a
Put f ( x) = 0 and solve it for x. The values of x will be comparison is an abstract quantity i.e., it is dimension
the zeroes of the polynomial f ( x). less and has no units. The ratio 1 : 6 means that for
every 1 units of the first element there are 6 units of
● Remainder theorem If f ( x ) be any polynomial of second element.
degree 1 or greater than 1 and p be any real number ● The elements of a ratio may or may not be the actual
and f ( x ) is divided by ( x - p), then remainder is f ( p). values of the quantities that are being compared. The
● Factor theorem If f ( x ) be any polynomial of degree actual values of the quantities can be assumed to be ‘k’
1 or greater than 1 and p be any real number such that times the elements of the ratio (here, k ¹ 0).
f ( p) is 0, then ( x - p) is a factor of f ( x ). ● The meaning conveyed by the fraction a/b is the same as
Note A quadratic polynomial can have atmost two zeroes.
● the meaning conveyed by the ratio denoted as a : b.
● A cubic polynomial can have atmost three zeroes. ● Ratios are compared in the same way as fractions.
● A polynomial of degree n( > 1) can have atmost n zeroes.
● a 2 : b2 and a 3: b3 are called the duplicate and the
2
Example 1 Find remainder, if f (x ) = x - 5 x + 6 is divided triplicate ratios of a : b, while a : b and 3 a : 3 b are
by x - 5. called the subduplicate and subtriplicate ratios of a : b.
(a) 6 (b) 8 (c) 4 (d) 10 ● a : b is greater than unity, if a > b.
Solution (a) x - 5 = 0 Þ x = 5 ● a : b is less than unity, if a < b.
● a : b is equal to unity, if a = b.
So, remainder is f (5) = (5) 2 - 5 (5) + 6
● When a/b = c/d, then ( a + b)/ b = ( c + d )/d
= 25 - 25 + 6 = 6 (called componendo)
Example 2 If f (x ) = x 2 - 5 x + 6, then find whether (x - 2) ● When a/b = c/d, then ( a - b) / b = ( c - d ) / d
(called dividendo)
is a factor of f (x ) or not.
(a) yes (b) Not ● When a/b = c/d, then ( a + b) / ( a - b) = ( c + d ) /( c - d )
(called componendo and dividendo)
(c) Cannot be determined (d) None of these
Solution (a) f( 2) = ( 2) 2 - 5 ( 2) + 6 = 4 - 10 + 6 = 0 Proportion of Polynomial
As remainder is zero, so ( x - 2) is a factor of f( x). A proportion is formed when two ratios are equal.
If a : b = c : d, then it is said that a, b, c, d are in
Relation between Zeroes and
proportion.
Coefficients of Polynomial
● Continued proportion If three quantities x, y and z
For quadratic polynomial Let a and b be the zeroes are such that x : y = y : z, then x, y, z are said to be in
of the two degree polynomial continued proportion, y is called mean proportional of x
p( x) = ax2 + bx + c, a ¹ 0. and z and z is called the third proportional of x and y.
Coefficient of x b
Here, sum of zeroes, a + b = - 2
=- ● Rational expression If f ( x ) and g( x ) are two
Coefficient of x a
Constant term c polynomials and g( x ) ¹ 0, then quotient f ( x ) is called a
and product of zeroes, ab = = g( x )
Coefficient of x2 a
rational expression. Every polynomial is a rational
For cubic polynomial Let a , b and g be the zeroes of expression but every rational expression is not a
the three degree polynomial polynomial. f ( x ) is said to be in lowest form, if f ( x ) and
p( x) = ax 3 + bx2 + cx + d, a ¹ 0 g( x )
Here, sum of zeroes, g( x ) have no common factor.
Algebra 407
● Addition of rational expression ● Multiplication of rational expression
f (x) f (x) f (x) p(x)
Addition of and is defined as When and are multiplied, we get it as
g( x ) r( x ) g( x ) r (x)
f ( x ) p ( x ) f ( x )r ( x ) + p ( x ) g ( x ) f ( x) p( x)
+ = ´
g( x ) r ( x ) g(x) ´ r (x)
g( x) r ( x)
f ( x) ´ p( x)
● Subtraction of rational expression =
f (x) p(x) g ( x) ´ r ( x)
When we subtract from , we get it as
g( x ) r (x) f (x)
● Division of rational expression When is
g( x )
p( x) f ( x) p( x) g ( x) - f ( x) r ( x) p(x) f (x) p(x) f (x) r (x)
- = divided by , we get it as ¸ = ´
r ( x) g ( x) r ( x) g ( x) r (x) g( x ) r( x ) g( x ) p ( x )

Target Exercise
Elementary Level Questions
f (2) + f ( -2) 8. If a x = b 3, b y = c3, c z = a 3, then the value of xyz will be
1. If f ( x) = x4 + 3 x 3 + 2x2 - x + 1, then is
4 (a) 9 (b) 0
equal to (c) 27 (d) None of these
25 26 27 28
(a) (b) (c) (d) 1
2 2 2 2 9. If 2 x + y = 4 and 2 x - 2 y = , then the value of x will be
8
2. If x + 2 y = 8, 2x + 3 z = 16 and 4 y + 5 z = 32, then the (a) 3 (b) 1
value of x, y and z will be (c) 1/3 (d) 0
(a) 1, 2, 3 (b) -1, 2, 3
æ 1 1 ö
(c) 2, 3, 4 (d) -2, - 3, - 4 10. ç 2 2
- 2 ÷ is equal to
èx + y x - y2 ø
3. If a = - 1, b = - 2 and g = 3, then value of
2 x2 2 x2 2 y2 - 2 y2
a 3 + b 3 + g 3 - 3abg is (a) - 4 4
(b) 4 4
(c) 4 4
(d)
x - y x - y x - y x 4 - y4
(a) -1 (b) 0
(c) 1 (d) 2 1 1
11. If 7 x - 2
= 21 - 2 , then x will be
x -4 x -4
4. If x = 2, then the value of x + x + x + x +K is
(a) 3 (b) 4
(a) 1 or 2 (b) 2 (c) 1 or 4 (d) 4 (c) 5 (d) None of these
1 1 3 4 3
5. If x + y = p and xy = q, then the value of + will 12. If + = , then z will be
x3 y3 2 x 3 y 5z
be x+ y 27 xy 18 xy 9 xy
(a) (b) (c) (d)
p3 - 3 p3 - 3 p 3 xy 5 ( 9 y + 8 x) 5 ( 9 y + 8 x) 5 ( 8 y + 9 x)
(a) (b)
q2 q3
p3 - 3 pq 3
q - 3 pq 13. Eight years hence a man will be twice as he was 8 yr
(c) (d)
q 3
p2 ago, then his present age is
(a) 24 yr (b) 26 yr
1 1
6. If x + y = 5 and xy = 6, then the value of + 2 will be (c) 28 yr (d) 30 yr
x2 y
1 æ1ö
(a)
13
(b)
11
(c)
13
(d)
13 14. If f ( y) = y 3 - 2 y + 1, then value of f ç ÷ will be
24 30 32 36 3 è3 ø
27 1
(a) (b)
1 81 3
7. If x = 2, then value of x - is
1 10
x- (c) Both (a) and (b) (d)
1 81
x-
x
15. If 4 x +2 y = 2 4 x +10 y , then y is equal to
(a) 1 (b) 2
x -1 1
(c) 3 (d) Cannot be determined (a) (b) x (c) x (d) -3x
3 3 2
408 Study Package for NTSE

16. The zeroes of 2 y2 - 9 y + 4 are 22. Find the HCF and LCM of the polynomials
(a)
1
,2 (b) 2,
1
(c) 4,
1
(d) 2, 4 ( x2 - 5x + 6) and ( x2 - 7 x + 10).
4 2 2 (a) ( x - 2 ), ( x - 2 ) ( x - 3) ( x - 5)
17. The zeroes of x2 - 6x + 8 are (b) ( x - 2 ), ( x - 2 ) ( x - 3)
(c) ( x - 3), ( x - 2 ) ( x - 3) ( x - 5)
(a) 1, 2 (b) 2, 3 (c) 2, 4 (d) 3, 4
(d) None of the above
x+ y x- y 2 2 2
18. If 2 = 32 and 2 = 4 , then x + y is equal to
41 19 17 29 23. If the polynomial 2 x 3 - 9 x2 + 15 x + P, when divided
(a) (b) (c) (d)
2 2 2 2 by ( x - 2), leaves - P as remainder, then P is equal to
1 1 (a) - 16 (b) - 5 (c) 20 (d) 10
19. If y + = 12, then y 3 + 3 is equal to
y y 24. If ( x + a) is the HCF of x2 + p x + q and x2 + lx + m,
(a) 1680 (b) 1686 (c) 1692 (d) 1694 then the value of ‘a’ is given by
1 1 p- l q-m
20. If x + y = 12 and xy = 32, then + is equal to (a) (b)
x y q-m p- l
2 1 1 3 l+ p q+ m
(a) (b) (c) (d) (c) (d)
8 8 2 8 q+ m p+ l
x
21. If 2 ´ 0 .2 ´ x = 3 ´ 2 ´ 0 .2, then the value of is 25. If x2 - 3 x + 2 is a factor of x4 - px2 + q, then the
3
values of p and q respectively, will be
(a) 0.35 (b) 0.33
(a) - 5, 4 (b) - 5, - 4 (c) 5, 4 (d) 5, - 4
(c) 0.20 (d) None of these

High Skill Questions


1. If (7 x + 3 y) : (3 x + 8 y) = 9 : 17, then x : y will be 8. What should be added to x to make it equal to 1/ x ?
(a) 21 : 92 (b) 22 : 93 x2 -1 1- x 2 x x
(a) (b) (c) (d)
(c) 22 : 95 (d) 23 : 93 x x x 2 -1 1- x 2
2 ax - 25by x+5 x-1
2. If a : b = 2 : 3, x : y = 3 : 4 , then is 9. If = , then value of x is
3 ay + 4 bx x-6 x+2
(a) 24 : 5 (b) 5 : 24 3 2 -2 -3
(a) (b) (c) (d)
(c) -24 : 5 (d) None of these 7 7 7 7
3. A ratio is 7 : 9, if increment of 14 takes place in 10. A number is 27 times the square of its reciprocal. The
numerator, then the increment in denominator number is
should be (a) 2 (b) 2.5 (c) 3 (d) 4
(a) 16 (b) 18
11. The value of x satisfying the equation
(c) 20 (d) 22
x2 + a2 = ( x - b)2 is
27 m + 4 ´ 32 m - 3n ´ 55 m + 3n + 4 ´ 6 m +2 n - 3 a2 - b 2 b 2 - a2
4. The value of 2 m + 4 n+ 7 3m - n - 3 6 m -2 n-5
is (a) (b)
10 15 2 2b 2a
1 b 2 - a2
(a) 1 (b) 0 (c) 2 (d) (c) (d) None of these
2 2b
5. What must be added to ( x 3 + 5x2 + 7 x - 9) to obtain a 12. The value of n for which the expression
polynomial exactly divided by ( x - 6)? x4 + 4 x 3 + nx2 + 4 x + 1 becomes perfect square is
(a) 423 (b) 425 (a) 3 (b) 4
(c) 427 (d) 429 (c) 5 (d) 6
n2 - 8 2x 5
6. If 3 n = 81, then is equal to 13. If = , then x will be
n 36 6
(a) 8 (b) 7 23 25
(a) (b) 12 (c) (d) 24
(c) 10 (d) 11 2 2
2 -9x
7. If 32 x = (81) -1, then x is equal to 14. If 2 x + 3 y = 13 and xy = 6, then 8 x 3 + 27 y 3 should be
1 1 equal to which of the following?
(a) 4, (b) 2,
2 4 (a) 790 (b) 793
(c) 2, 1 (d) Both (a) and (b) (c) 783 (d) None to these
Algebra 409
15. The value of 8 x2 + 6 2 yzx - 4 y2 z2 is 17. If 2 a + b = 8 and 8 a - b = 2, then which of the following is
(a) ( x + 8 2 yz)( x - 2 2 yz) correct?
(b) (4 x + 2 yz)(2 x - 2 yz) 5 4 5 4
(a) a = ,b= (b) b = ,a=
4 3 4 3
(c) ( x + 2 yz)(8 x - 2 2 yz)
5 4 5 4
(d) None of these (c) a = ,b= (d) a = ,b=
3 5 3 3
16. The value of 8 + 2 15 is
(a) 3 + 2 18. If 49 x - 49 x -1 = 16464, then which of the following is
(b) 3 + 5 equivalent of (2x) x ?
(c) 2 + 5 (a) (5)5 / 2 (b) (7 )7/ 2
(d) None of the above (c) (3)3 / 2 (d) None of these

Hints and Solutions


Elementary Level Questions
1. (a) Given, f( x) = x4 + 3 x3 + 2 x2 - x + 1 5. (c) Use (a3 + b 3 ) = (a + b) (a2 + b 2 - ab)
\ f(2 ) = (2 )4 + 3(2 )3 + 2(2 )2 - 2 + 1 æ 1 1 ö æ 1 1ö æ 1 1 1ö
So, ç 3 + 3 ÷ = ç + ÷ ç 2 + 2 - ÷
èx y ø è x yø è x y xy ø
= 16 + 3 ´ 8 + 8 - 2 + 1 = 23 + 24 = 47
æ x + yö æ x 2 + y2 1 ö
and f(-2 ) = (-2 )4 + 3(-2 )3 + 2(-2 )2 - (-2 ) + 1 =ç ÷ çç - ÷÷
è xy ø è ( xy)
2
xy ø
= 16 - 24 + 8 + 2 + 1= 27 - 24 = 3
f(2 ) + f(-2 ) 47 + 3 æ x + yö æ x2 + y2 - xy ö x+ y
Now, = =ç ÷ çç ÷÷ = [( x + y)2 - 3 xy]
4 4 è xy ø è ( xy)2 ø ( xy)3
50 25 p
= = = ( p 2 - 3 q ) [Qput the values of( x + y) and ( xy)]
4 2 q3
2. (c) Given, x + 2 y = 8 …(i) p3 - 3 pq
=
2 x + 3 z = 16 …(ii) q3
and 4 y + 5 z = 32 …(iii) 1 1 y2 + x2 ( x + y) 2 - 2 xy
6. (d) 2
+ 2
= 2 2
=
Multiply by ‘2’ in Eq. (i) and subtracted from Eq. (iii), x y x y x 2y 2
5 z - 2 x = 16 …(iv) (5)2 - 2 ´ 6
= [Qput the values of( x + y) and ( xy)]
From Eqs. (ii) and (iv), we get (6)2
z=4 25 - 12 13
= =
From Eq. (ii), x = 2 36 36
From Eq. (i), y = 3 1 1
7. (a) Let x - = y Þ 2 - = y Þ 2 y - 1 = y2
1 y
3. (b) If a + b + c = 0, then a3 + b 3 + c 3 - 3abc = 0 x- ¼
x
Here, a + b + g = - 1 - 2 + 3 = 0 Þ y2 - 2 y + 1 = 0 Þ y = 1
So, a3 + b 3 + g 3 - 3abg = 0
8. (c) a x = b 3 Þ (a x ) y = (b 3 ) y
4. (c) Let x+ x+ x+ x+¼ = y Þ a x y = b3 y
Þ a xyz = [(c 3 )3 ]z [Q b y = c 3 ]
Þ 2+ 2 + 2 + 2 + 2 +¼ = y [Q given x = 2] Þ a xyz = (c z )9 = a27 [Qc z = a3 ]
Þ 2+ y= y By comparing, xyz = 27
Þ 2 2
( y ) = ( y - 2 ) [Q by squaring both sides] 9. (c) 2 x + y = 2 2 , 2 x - 2 y = (2 )-3 [given]
2
Þ y = y + 4 - 4y On comparing given Eqs. we get
Þ y2 - 5 y + 4 = 0 Þ ( y - 4)( y - 1) = 0 \ x+ y=2 ...(i)
and x - 2y = - 3 ...(ii)
\ y = 4 or 1
On multiplying Eq. (i) by 2 and then adding them, we get
Hence, x + x+ x+ x+ ... = 1or 4 3x = 1
\ x = 1/ 3
410 Study Package for NTSE

1 1 x2 - y2-x2 - y2 -2 y 2 20. (d) Given, x + y = 12 and xy = 32


10. (d) 2 2
- 2 2
= 4 4
=
x + y x - y x - y x 4 - y4 1 1 x+ y
Hence, + =
1 1 x y xy
11. (a) 7x - = 21 - Þ 7 x = 21
x2 - 4 x2 - 4 12 3
= =
\ x=3 32 8
3 4 3 9y + 8x 3 18 xy 21. (d) On squaring both sides, we get
12. (c) + = Þ = Þ z=
2 x 3y 5z 6 xy 5z 5 ( 9 y + 8 x) 2 ´ 0 . 2 ´ x = 3 ´ 4 ´ 0.04
x 016 . 2
13. (a) Let age of man at present be x yr. Þ = = = 0.4
3 0.4 5
Then, according to the question,
22. (a) x 2 - 5 x + 6 = x2 - 3 x - 2 x + 6
x + 8 = 2 ( x - 8)
Þ x + 8 = 2 x - 16 = x( x - 3) - 2( x - 3) = ( x - 3) ( x - 2 )

Þ 2 x - x = 8 + 16 Þ x = 24 yr x - 7 x + 10 = x2 - 5 x - 2 x + 10
2

14. (d) Given, f ( y) = y3 - 2 y + 1 = x( x - 5) - 2( x - 5)


3 = ( x - 5) ( x - 2 )
f æç ö÷ = æç ö÷ - 2 ´ + 1 =
1 1 1 1 2 10
- + 1=
è 3ø è 3ø 3 27 3 27 \ HCF = ( x - 2 )
1 æ 1 ö 1 10 10 and LCM = ( x - 2 ) ( x - 3) ( x - 5)
\ fç ÷ = ´ =
3 è 3 ø 3 27 81 23. (b) Given, f( x) = 2 x3 - 9 x2 + 15 x + P
15. (b) 4 x + 2 y = 2 4 x + 10 y = 42 x + 5 y So, x - 2 = 0 or x = 2
Þ x + 2 y = 2 x + 5 y [Q on comparing both sides] Þ f(2 ) = 2(2 )3 - 9(2 )2 + 15 ´ 2 + P
Þ - x = 3y
-1 Þ - P = 16 - 36 + 30 + P
\ y= x
3 Þ - 2 P = 10
16. (c) 2 y2 - 9 y + 4 = 2 y2 - 8 y - y + 4 Þ P=-5
= 2 y( y - 4) - 1( y - 4) = (2 y - 1)( y - 4) 24. (b) f ( x) = x 2 + p x + q
1
Þ y = ,4 Þ f ( - a) = a 2 - p a + q = 0
2
g ( x) = x2 + lx + m
17. (c) x2 - 6 x + 8 = x2 - 4 x - 2 x + 8
Þ g (- a) = a2 - la + m = 0
= x( x - 4) - 2( x - 4) = ( x - 4)( x - 2 )
Þ x = 4, 2 Þ a2 - p a + q = a2 - la + m = 0

18. (a) 2 x + y = 2 5 Þ x + y = 5 ¼ (i) q-m


\ a=
p- l
2 x-y = 2 4 Þ x - y = 4 …(ii)
25. (c) x2 - 3 x + 2 = x2 - 2 x - x + 2
On solving Eqs. (i) and (ii), we get
9 1 = x( x - 2 ) - 1( x - 2 ) = ( x - 2 ) ( x - 1)
x= , y=
2 2 f( x) = x4 - px2 + q
2 2
Hence, x2 + y2 = æç ö÷ + æç ö÷ =
9 1 81 1 41 Þ f(2 ) = (2 )4 - p(2 )2 + q
+ =
è2 ø è2 ø 4 4 2
3 = 16 - 4 p + q = 0
æ 1ö 3 1 æ 1ö
19. (c) çy + ÷ = y + 3 + 3 çy + ÷ Þ 4 p - q = 16 …(i)
è yø y è yø
and f(1) = 1 - p + q = 0
[Q(a + b )3 = a3 + b 3 + 3ab (a + b )]
Þ p-q =1 …(ii)
1 1
Þ (12 )3 = y3 + 3
+ 3 ´ 12 [Q given, y + = 12] On solving Eqs. (i) and (ii), we get
y y
1 p = 5 and q = 4
Þ y3 + 3
= (12 )3 - 3(12 ) = 1692
y
Algebra 411
High Skill Questions
7 x + 3y 9 x2 + a2 = x 2 + b 2 - 2 bx
1. (a) Given, = Þ119 x + 51y = 27 x + 72 y 11. (c)
3 x + 8 y 17
b 2 - a2
x 21 Þ 2 bx = b 2 - a2 Þ x =
Þ 92 x = 21y Þ = 2b
y 92
12. (d) Let f( x) = x4 + 4 x3 + nx2 + 4 x + 1
\ x : y = 21 : 92
2b 3 = ( x4 + 1) + 4 x ( x 2 + 1) + nx 2
2. (c) Put a = and x = y
3 4 é ù
= x 2 ê æç x 2 + 2 ö÷ + 4 æç x + ö÷ + nú
1 1
2b 3y ëè x ø è xø û
2´ ´ - 25by
2 ax - 25by 3 4 by - 25by
So, = = é 2 ù
= x2 ê æç x + ö÷ - 2 + 4 æç x + ö÷ + nú
3ay + 4bx 3 ´ 2 b ´ y + 4 ´ b ´ 3 y 2 by + 3by 1 1
3 4 êë è x ø è x ø úû
-24 é ù
= or - 24 : 5 2
= x2 ê æç x + ö÷ + 4 æç x + ö÷ + (n - 2 )ú
1 1
5
êë è x ø è x ø úû
3. (b) Let increment be y.
7 + 14 7 éì ü
2 ù
= x ê íæç x + ö÷ + 2 ý + (n - 6)ú.
1
Then, = Þ189 = 63 + 7 y
9+ y 9 êë îè xø þ úû
Þ 7 y = 126 Þ y = 18 Here, (n - 6) must be equal to zero, then f( x) becomes a perfect
2 7m + 4 ´ 32 m - 3 n ´ 55 m + 3 n + 4 ´ 6 m + 2 n - 3 square.
4. (d)
102 m + 4 n + 7 ´ 153 m - n - 3 ´ 2 6 m - 2 n - 5 i.e., n-6=0 Þ n=6
2 7m + 4 ´ 32 m - 3 n ´ 55 m + 3 n + 4 ´ (2 ´ 3)m + 2 n - 3 13. (c)
2 x 5 2 x 25
= Þ = Þ 2 x = 25 Þ x =
25
=
(5 ´ 2 )2 m + 4 n + 7 ´ (3 ´ 5)3 m - n - 3 ´ 2 6 m - 2 n - 5 36 6 36 36 2
2 8 m + 2 n + 1 ´ 33 m - n - 3 ´ 55 m + 3 n + 4 1 14. (b) (2 x + 3 y)3 = 8 x3 + 27 y3 + 18 xy (2 x + 3 y)
= =
2 8 m + 2 n + 2 ´ 33 m - n - 3 ´ 55 m + 3 n + 4 2 Þ (13)3 - 18 ´ 6 (13) = 8 x3 + 27 y3
5. (d) Put x = 6 in given polynomial [Q given, 2 x + 3 y = 13 and xy = 6 ]
f(6) = (6)3 + 5 ´ (6)2 + 7 ´ 6 - 9 = 216 + 180 + 42 - 9 = 429 Þ 2197 - 1404 = 8 x3 + 27 y3
6. (b) 3 n/ 2 = 34 Þ n = 8 [Q by comparing] Þ 8 x3 + 27 y3 = 793
n2 - 8 64 - 8 15. (c) 8 x2 + 6 2 xyz - 4 y2 z2
Þ = =7
n 8
2 -9 x = 8 x2 + 8 2 xyz - 2 2 xyz - 4 y2 z2
7. (a) 32 x = (81)-1
2 -9 x
= 8x ( x + 2 yz) - 2 2 yz ( x + 2 yz)
Þ 32 x = 3-4
= (x + 2 yz) (8 x - 2 2 yz)
Þ 2 x2 - 9 x = - 4 [Q by comparing]
16. (b) 8 + 2 15 = 5 + 3 + 2 15
2 2
Þ 2 x - 9x + 4 = 0 Þ 2 x - 8x - x + 4 = 0
= ( 5) 2 + ( 3) 2 + 2 5 3
Þ 2 x( x - 4) - 1( x - 4) = 0 Þ ( x - 4)((2 x - 1) = 0
1 = ( 5+ 3) 2 = 5+ 3
\ x = 4,
2 17. (d) 2 a + b = 2 3 Þ a + b = 3 ¼ (i)
1 1- x 2 1
8. (b) Required number = - x = and 8 a -b = 2 Þ 8 a -b = 83
x x
1
x+ 5 x-1 Þ a-b= ¼(ii)
9. (c) = Þ ( x + 5)( x + 2 ) = ( x - 1)( x - 6) 3
x-6 x+2
On solving Eqs. (i) and (ii), we get
Þ x2 + 7 x + 10 = x2 - 7 x + 6 5 4
a= ,b=
-4 -2 3 3
Þ 14 x = 6 - 10 Þ x = =
14 7 18. (a) 49 x - 49 x -1 = 16464 Þ (49) x-1 ´ 48 = 16464
10. (c) Let number be x. 5
Þ (49) x-1 = 343 Þ (7 )2 x- 2 = 7 3 Þ x =
1 2
Then, x = 27 × Þ x3 = (3)3 5/ 2
x2 \ (2 x) x = æç2 ´ ö÷
5
= (5)5/ 2
Þ x=3 [Q by comparing] è 2ø
412 Study Package for NTSE

Chapter

4
Linear Equations
Linear Equation in One Variable
If c < 0, a, b > 0 the graph will look like.
A linear equation in which the number of variables is e.g., The graphical representation of 3 x + 4 y = 12 will be
one, is known as linear equation in one variable.
for x = 0 ⇒ y = 3 and for y = 0 ⇒ x = 4 i. e., ( 4, 0) i. e., (0, 3)
e.g., 3 x + 5 = 10, y + 3 = 5 etc.
Y
Example 1 Solve the following equation 2 x + 6 = 18. (0, 3)
B
(a) 6 (b) 7 (c) 8 (d) 9
(4, 0)
Solution (a) Given, O A
X
2x + 6 = 18
2x = 18 − 6 ⇒ 2x = 12 ⇒ x = 6

Linear Equation in Two Variables Methods of Solving Linear Equations in Two


Equation ax + by + c = 0 has two variables x and y has Variables
the power of the variables 1 is a linear equation Two linear equations involving two variables
involving two variables.
a1 x + b1 y + c1 = 0 …(i)
Graphical representation of linear equation involving
and a2 x + b2 y + c2 = 0 …(ii)
two variables
ax + by + c = 0 (i) Elimination method We will eliminate all of
Put x =0 ⇒ y = − c/ b the variables x or y by making their coefficients
i. e., (0, − c / b) same and then subtracting or adding.
Put y =0 ⇒ x = − c/ a On multiplying Eq. (i) with a 2 and Eq. (ii) with a1 and
i.e., ( − c / a, 0) subtracting the equations, we get
Y a2 a1 x + a2 b1 y + c1 a2 = 0
B
(0, –c/b) a2 a1 x + a1 b2 y + c2 a1 = 0
− − −
(–c/a, 0)
X ( a2 b1 − a1 b2 ) y = ( c2 a1 − c1 a2 )
O A ax + by + c = 0 c a − c1 a2
i. e., y= 2 1
a2 b1 − a1 b2
On substituting this value in Eq. (i), we get
Depending upon the signs of a, b and c, we will plot the
points.
Linear Equations 413
 c a − c1 a2  (i) (ii) (iii)
a1 x + b1  2 1  + c1 = 0
 a2 b1 − a1 b2  x
=
y
=
1
b2 c1 − b1 c2 c2 a1 − a2 c1 a2 b1 − a1 b2
( a2 b1 − a1 b2 ) ⋅ a1 x + b1 c2 a1 − b1 c1 a2
b c − b1 c2
+ a2 b1 c1 − a1 b2 c1 Taking (i) and (iii), x = 2 1
⇒ =0 a2 b1 − a1 b2
( a2 b1 − a1 b2 )
c a − a2 c1
( b2 c1 − b1 c2 )  b c − b1 c2  and taking (ii) and (iii), y = 2 1
i. e., a1 x = a1 ⇒x =  2 1  a2 b1 − a1 b2
( a2 b1 − a1 b2 )  a2 b1 − a1 b2 
Example 2 Solve for x and y, y − 2 x = 10 and 3 x + 2 y = 6.
b2 c1 − b1 c2 c2 a1 − c1 a2
Thus, x = and y = (a) (– 2, 6) (b) (8, –2) (c) (– 6, 2) (d) (1, 2)
a2 b1 − a1 b2 a2 b1 − a1 b2
Solution (a) Let us solve the equations using all the
three methods
(ii) Substitution method
− 2x + y = 10 …(i)
a1 x + b1 y + c1 = 0 …(i)
and 3 x + 2y = 6 …(ii)
and a2 x + b2 y + c2 = 0 …(ii) (i) Elimination method On multiplying the Eq. (i) with
We calculate the value of x in terms of y from Eq. (i), 2 and subtraction both the equations, we get
( c + b1 y) − 4 x + 2y = 20
we get x=− 1 …(A)
a1 3 x + 2y = 6
− − −
On substituting this value in Eq. (ii), we get − 7x = 14 ⇒ x= −2

 ( c + b1 y)  Now, substituting the value of x in Eq. (i), we get


a2 − 1  + b2 y + c2 = 0 − 2 ( − 2) + y = 10 ⇒ y = 10 − 4 = 6
 a1 
Thus, x = − 2 and y = 6
i. e., − a2 c1 − a2 b1 y + a1 b2 y + c2 a1 = 0
(ii) Substitution method From Eq. (i), we get
c a − a2 c1
i. e., y= 2 1 y = 10 + 2x …(iii)
a2 b1 − a1 b2
On substituting the value of y in Eq. (ii), we get
On substituting this value in formula (A), we get 3 x + 2 (10 + 2x) = 6
the value of x i.e., 3 x + 4 x = 6 − 20
  c a − a2 c1   i.e., 7 x = − 14 ⇒ i.e., x = − 2
− c1 + b1  2 1  Now, from Eq. (iii), we get
− ( c1 + b1 y)   a2 b1 − a1 b2  
x= = y = 10 + 2x = 10 + 2 ( − 2) = 10 − 4 = 6
a1 a1 Thus, x = − 2 and y = 6
 a b c − a1 b2 c1 + b1 c2 a1 − a2 c1 b1 
=− 1 1 1 
(iii) Cross-multiplication method
 a2 b1 − a1 b2 
x
=
y
=
1
b2 c1 − b1 c2 c2a 1 − a 2 c1 a 2 b1 − a 1 b2
a1
On comparing by Eqs. (i) and (ii),
− a2 b1 c1 + a1 b2 c1 − b1 c2 a1 + a2 c1 b1
= −2x + y = 10 3 x + 2y = 6
a1 ( a2 b1 − a1 b2 ) a 1 b 1 c1
and
a b c
2 2 2
a ( b c − b1 c2 ) b c − b1 c2
= 1 2 1 ∴x= 2 1 x
=
y
=
1
a1 ( a2 b1 − a1 b2 ) a2 b1 − a1 b2 2 × 10 − 1 × 6 − 2 × 6 − 3 × 10 − 2 × 2 − 1 × 3
x y 1
(iii) Cross-multiplication method i.e., = =
a1 x + b1 y + c1 = 0 a1 x + b1 y + c1 = 0 14 − 42 − 7
14 − 42
⇒ x= = − 2 and y = =6
−7 −7
a2 x + b2 y + c2 = 0 a2 x + b2 y + c2 = 0 Thus, x = − 2 and y = 6
a1 x + b1 y + c1 = 0 Hence, we can follow any all of the three methods to solve
the equations.
î Elimination method is quicker to solve as compared with the other
a2 x + b2 y + c2 = 0 two methods.
414 Study Package for NTSE

Conditions of Solvability of Equations Example 3 A two-digit number has units digit and tens
digit sum 9 and the reverse of number subtracted from the
Thelinear equations a 1 x + b1 y + c1 = 0 number, the result obtained is 63. Find the number.
and a 2x + b2 y + c2 = 0 (a) 18 (b) 81 (c) 20 (d) 15
Solution (b) Let 10a + b be the number.
will have
a1 b1 Now, given a+ b=9 …(i)
c1
● infinite number of solutions, if = = and, then and (10 a + b) − (10 b + a ) = 63 ⇒ 9 (a − b) = 63
a2 b2 c2
⇒ a− b=7 ... (ii)
pair of linear equation is dependent and consistent. On adding Eqs. (i) and (ii), we get
a 1 b1 c1 a = 8 and b = 1
● no solution, if = ≠ and then pair of linear
a2 b2 c2 Thus, number = 10a + b = 81
equation is inconsistent. î A 2-digit number, ab = 10a + b and its reverse = ba = 10b + a
a 1 b1 ab + ba = 11( a + b) and ab − ba = 9 ( a − b)
● unique solution, if ≠ and then pair of linear Sometimes given pair of equations are not of linear form but they
a 2 b2 can be reduced to linear form by making some suitable
equation is consistent. substitutions.

Target Exercise
Elementary Level Questions
1. If the cost of 4 books and 5 notebooks is ` 32 and the 7. The sum of the two-digit number and its reverse is m
cost of 4 notebooks and 5 books is ` 31. The cost of each times the sum of its digits. If the subtraction of the
book is reverse from the number is n times the subtraction of
(a) 2 (b) 3 (c) 4 (d) 5 the digits, the value of m + n is
(a) 2 (b) 9 (c) 11 (d) 20
2. The digits of a 2-digit number, when written in
reversed form and subtract from the original number, 8. If the numerator and denominator of a fraction are
the result is 36. If the sum of the digits is 10, the increased by ‘1’ and ‘3’ respectively, the fraction
original number is becomes 1/2. What is the fraction, if the increase of 2 in
(a) 37 (b) 73 the numerator and 7 in denominator makes it 2/5?
(c) 36 (d) None of these 3 1 2 4
(a) (b) (c) (d)
2 2 3 5
3. 4 yr back, A’s age was 4 times that B’s age. What is A’s
1 9. The difference of the squares of two numbers is 8000.
present age, if after 3 yr, B’s age will be rd of A’s age?
3 What is the smaller of the two numbers, if their sum is
200?
(a) 56 yr (b) 60 yr (c) 63 yr (d) 66 yr (a) 80 (b) 100
4. The cost of 3 tables and 4 chairs is ` 2800 whereas the (c) 120 (d) 160
cost of 2 tables and 3 chairs is just ` 1950. What is the 10. The combined cost of 5 doors and 7 bells was ` 2350
cost of 1 table? while, the cost of 10 doors and 4 bells is ` 2200. How
(a) 250 (b) 300 much is the cost of 2 doors?
(c) 600 (d) None of these (a) ` 100 (b) ` 240
5. 10 yr ago, the age of a father was thrice the age of his (c) ` 80 (d) ` 120
son whereas, 10 yr later the son will be half the age of 11. 20 yr ago the combined age of Ram and Shyam was
the father. The difference between their present 5/ 9th the sum of their present ages. If Ram is 20 yr
ages is elder to Shyam, the present age of Ram is
(a) 30 yr (b) 40 yr (a) 25 yr (b) 35 yr (c) 55 yr (d) 90 yr
(c) 70 yr (d) 100 yr
12. Sum of the digits of a two-digit number is 24 times the
6. For what value of m, the equations 6x − 9 y = m and sum of their reciprocals. If the sum of there reciprocals
2 x − 3 y = 4 given will have infinite number of is 5/12, the number is
solution?
(a) 64 (b) 46
(a) 10 (b) − 12
(c) 36 (d) Cannot be determined
(c) 12 (d) None of these
Linear Equations 415
13. For what value of k, the equations kx − 9 y = 66 and 15. Kashish has coins only in 2 denominations i. e.,
2x − 3 y = 8 will have no solution? 10 paise and 25 paise. If the total number of coins and
(a) − 6 (b) 6 (c) 33/4 (d) None of these the total amount she has is ` 70 and ` 10 respectively,
14. The value of x − y, when the two equations are what is the number of 10 paise coins she has?
x + y = 50 and 3 x − 2 y = 0 is (a) 10 (b) 20
(a) − 10 (b) + 10 (c) 20 (d) 25 (c) 30 (d) 40

High Skill Questions


8 21 10 16 7
1. If + = and − = 1. 6. In an parking space, a total of 20 bikes and cars were
3 (2a + 3 b) 3 a + 2b 3 2a + 3 b 3 a + 2b parked. If the total number of tyres were found to be
The value of (3 a + 2b) is 70, the numbers of cars in the parking place was
13 3 47 49 (a) 5 (b) 10 (c) 15 (d) 20
(a) (b) (c) (d)
3 13 28 29 7. Amar has only 3 denominations of coins i. e.,
2. A 2-digit number has tens digit greater than the unit’s ` 1, ` 2 and ` 5. If he has a total of 40 coins and the total
digit. If the sum of its digits is equal to twice the amount of ` 130, the maximum number of ` 5 coins he
difference, how many such numbers are possible? can have, is
(a) 1 (b) 2 (c) 3 (d) 4 (a) 24 (b) 22 (c) 23 (d) 21
3. 4 friends went to fair. They found only 2 tickets 8. 4 yr hence, a father’s age will be 2 yr less than three
available at the counter. For the 2 more tickets they times the age of his son. Also, 6 yr back his age was
had to pay ` 50 more each ticket, then the actual cost. 2 yr more than five times the age of his son. After how
If at the end they found the per head cost to be ` 60, many years from now will their combined age be
what was the actual price of the ticket? 100 yr?
(a) 25 (b) 30 (a) 40 yr (b) 14 yr (c) 19 yr (d) 38 yr
(c) 35 (d) 40
9. A two-digit number ‘ab’ can be obtained either
4. Sum of the present ages of husband and wife is 90 yr. subtracting 12 from 4 times the sum of its digits or by
What is husband’s present age, if 27 yr back, men’s age adding 6 to twice the difference of its digits. What is
was half of his wife’s present age? the number equal to ab?
(a) 39 yr (b) 51 yr (a) 12 (b) 16
(c) 78 yr (d) None of these (c) 18 (d) Cannot be determined
5. The age of Sita is 8 yr more than twice the age of Sonu. 10. If abc − cba = 792, where abc is a 3-digit number, if
If after n yr, the age of Sita will be twice the age of a + b + c = 18 and a = 9 c, what is the number?
Sonu, then the value of n is (a) 990 (b) 981
(a) 6 (b) 8 (c) 918 (d) None of these
(c) 10 (d) Cannot be determined

Hints and Solutions


Elementary Level Questions
1. (b) Let the number of books and notebooks be a and b. 2. (b) Let the 2-digit number be ab = (10 a + b)
respectively. We have, a b − b a = 36 ⇒ 10a + b − (10b + a) = 36
4 a + 5 b = 32 …(i) ⇒ 9 (a − b ) = 36
and 5 a + 4 b = 31 …(ii) ⇒ a−b=4 …(i)
On multiplying Eq. (i) by 4 and Eq. (ii) by 5 and then subtracting,
Also, a + b = 10 …(ii)
we get
16 a + 20 b = 128 On solving Eqs. (i) and (ii), we get
_ a + 20 b = 155
25 we get a = 7, b = 3

Thus, the number
−9 a = − 27
⇒ a=3 a b = 10 × 7 + 3 = 73
416 Study Package for NTSE

3. (b) Let A and B’s present ages be a and b yr, respectively. 9. (a) a2 − b 2 = 8000 and a + b = 200 …(i)
So, (a − 4) = 4 (b − 4) Now, (a + b )(a − b ) = 8000
i .e., a − 4 b = − 12 …(i) 200 × (a − b ) = 8000
and (a + 3) = 3 (b + 3) 8000
⇒ (a − b) = = 40 …(ii)
i .e., a − 3b = 6 …(ii) 200
On solving Eqs. (i) and (ii), we get On solving Eqs. (i) and (ii), we get
b = 18 yr and a = 60yr a = 120, b = 80
A’s present age = 60 yr
10. (b) Let the cost of one door and bell be D and B respectively.
4. (c) We know that, 3 t + 4 c = 2800 …(i) According to the question,
and 2 t + 3 c = 1950 …(ii) 5 D + 7 B = 2350 …(i)
On multiplying Eq. (i) by 3 and Eq. (ii) by 4 and subtracting, then
and 10 D + 4 B = 2200 …(ii)
we get
On multiplying Eq. (i) by 2, we get
9t + 12c = 8400 10 D + 14 B = 4700
8t + 12c = 7800 10 D + 4 B = 2200
− − −
− − − ⇒ B = `250
t = 600
10 B = 2500
⇒ t = 600 From Eq. (i), we get
5. (b) (F − 10) = 3 (S − 10) i .e., F − 3 S = − 20 2350 − 7 × 250
…(i) D= = ` 120
and (F + 10) = 2 (S + 10) i .e., F − 2S = 10 …(ii) 5
On solving Eqs. (i) and (ii), we get S = 30 yr and F = 70 yr So, cost of two doors = 2 D = ` 240
Thus, the difference between their ages = 70 − 30 = 40 yr 11. (c) Let the age of Ram and Shyam be R and S respectively.
5
6. (c) The equations have infinite solutions, if [(R − 20) + (S − 20)] = (R + S )
a1 b c 9
= 1 = 1 i .e., 9(R + S − 40) = 5 (R + S )
a2 b2 c 2
⇒ 9 (R + S ) − 5(R + S ) = 360
⇒ a1 x + b1 y + c1 = 0
⇒ 4 (R + S ) = 360
and a2 x + b2 y + c 2 = 0
⇒ R + S = 90 …(i)
So, the equations 6 x − 9 y = m
We know that, R = S + 20
and 2 x − 3 y = 4,we have
i .e., R − S = 20 …(ii)
6 −9 m
= = On solving Eqs. (i) and (ii), we get
2 −3 4
R = 55yr and S = 35 yr
⇒ m = 12
12. (a) Let the number be 10x + y. Then,
7. (d) We know that,
1 1 5
+ = …(i)
ab + ba = 10a + b + 10b + a = 11(a + b ) x y 12
So , m = 11  1 1
and x + y = 24  + 
⇒ a b − b a = 10a + b − (10b − a) = 9 (a − b )  x y
So, n=9 5
= 24 × = 10 …(ii)
Thus, m + n = 11 + 9 = 20 12
N
8. (c) Let the fraction be . x+ y 5 10 5
From Eq. (i), = ⇒ =
D xy 12 x y 12
N+1 1
We know that, = ⇒ x y = 24
D+ 3 2
Now, ( x − y)2 = ( x + y)2 − 4 xy
⇒ 2N + 2 = D + 3
⇒ 2N − D = 1 …(i) = (10)2 − 4 × 24 = 100 − 96 = 4
N+2 2 x− y =2 …(iii)
and =
D+7 5 From Eqs. (ii) and (iii), x = 6 and y = 4
⇒ 5N + 10 = 2 D + 14 Hence, required number = 10 × 6 + 4 = 64
⇒ 5N − 2D = 4 …(ii) 13. (b) kx − 9 y = 66
On solving Eqs. (i) and (ii), we get 2 x − 3y = 8
N = 2 and D = 3 Condition of no solution,
2
Thus, fraction = k 9 66
= ≠ ⇒k = 6
3 2 3 8
Linear Equations 417
14. (a) x + y = 50 …(i) 15. (c) Let the number of 10 paise and 25 paise coins be x and y,
and 3x − 2 y = 0 …(ii) respectively.
On multiplying in Eq. (i) by 2 , we get So, x + y = 70 …(i)
2 x + 2 y = 100 and . x + 025
010 . y = 10
3x − 2 y = 0 ⇒ 2 x + 5 y = 20 …(ii)
⇒ x = 20 On solving Eqs. (i) and (ii), we get
5 x = 100
y = 40, x = 30
and as x + y = 50 ⇒ y = 30
Thus, the number of 10 paise coins = 30
So, x − y = 20 − 30 = − 10

High Skill Questions


8 21 10
1. (a) We have, + = 6. (c) Let the number of bikes and cars be B and C respectively,
3(2 a + 3 b ) 3 a + 2 b 3 B + C = 20 … (i)
16 7 2 B + 4 C = 70
and − =1 and
2a + 3b 3a + 2b [Q 2 tyres in bike and 4 tyres in car]
1
Taking = x i .e., B + 2 C = 35 ...(ii)
2a + 3b
1 On solving Eqs. (i) and (ii), we get
and = y, we get C = 15 and B = 5
3a + 2b
8 x + 42 y = 10 …(i) 7. (b) Let the number of ` 1, ` 2 and ` 5 coins be x, y and z
and 16 x − 7 y = 1 …(ii) respectively, x + 2 y + 5 z = 130
On multiplying Eq. (i) by 2 and subtracting from Eq. (ii), we get and x + y + z = 40 (subtracting)
− − −
13 47
y= and x= y + 4 z = 90
3 28
Now, for maximum value of z, we take options. If we take
1 13
∴Hence, = 3 a + 2b = option (a), z = 24
y 3
⇒ y = 90 − 4 × 24 = − 6 < 0 ( not possible)
2. (c) Given for the two digit number 10a + b, a > b Taking (c), z = 23
(a + b ) = 2(a − b ) ⇒ y = 90 − 4 × 23 = 90 − 92 = −2 < 0
⇒ a = 3b Taking (b), z = 22 ⇒ y = 90 − 4 × 22 = 2 > 0
b = 1, 2, 3 ⇒ a = 3, 6, 9 So, z = 22 is the maximum possible value.
So, the possible numbers are 31, 62 and 93. 8. (c) Let the age of father and son be F and S respectively,
3. (c) Actually cost per ticket = ` x F + 4 = 3 ( S + 4) − 2
and increased cost = `( x + 50) ⇒ F − 3S = 6 …(i)
i.e., ( x + x) + 2 ( x + 50) = Total amount paid and F − 6 = 5 (S − 6) + 2
4 x + 100 ⇒ F − 5 S = − 22 …(ii)
⇒ = ` 60 ⇒ x + 25 = 60
4 On solving Eqs. (i) and (ii), we get F = 48, S = 14
∴ x = 35 = Actual cost of ticket Let us assume after ‘x’ yr their combined age will be 100 yr.
4. (b) Let the present age of husband and wife be H and W i .e., (48 + 14 + x + x) = 100
respectively, ⇒ 2 x = 100 − 62 = 38, x = 19 yr
H + W = 90 …(i)
9. (d) a b = 4 (a + b ) − 12 …(i)
and 2(H − 27 ) = ( W − 27 )
and a b = 2(a − b ) + 6 …(ii)
⇒ 2 H − W = 27 …(ii)
On subtracting Eq. (ii) from Eq. (i), we get b = 2 a + 4
On solving Eqs. (i) and (ii), we get
W = 39 For a = 1, b = 6 and a = 2 ⇒ b = 8
and H = 51 Thus, the possible number, a b = 16 or 28

5. (d) Let Sita = x yr old and Sonu = y yr old 10. (b) abc − cba = 792 …(i)
100 a + 10 b + c − (100c + 10 b + a) = 792
So, x = (2 y + 8)
⇒ 100a + 10b + c − 100c − 10 b − a = 792
We know that, ( x + n) = 2 (2 y + 8 + n) and 99 (a − c ) = 792 …(ii)
⇒ x + n = 4 y + 16 + 2 n ⇒ a−c = 8
⇒ x = 2x + n On solving Eqs. (i) and (ii), we get c = 1and a = 9
Since, there are no value for any variable, we cannot determine Now, using a + b + c = 18, we get, 9 + b + 1 = 18 ⇒ b = 8
the years. Thus, the number = a bc = 981
Chapter

5
Quadratic Equation

Quadratic Equation We need to split the middle term 5 into two parts so that
there sum/subtraction is 5 and product is a × c = 1 × 6 = 6
Equation of the form ax + bx + c = 0, where a ≠ 0 is a
2
Those numbers will be 3 and 2 as 3 + 2 = 5 and 3 × 2 = 6.
quadratic equation. So, x2 + 5x + 6 = 0 is
e.g., x2 + 2 x + 5 = 0, 4 x2 − 5 = 0 and 3 x2 + 6 x − 4 = 0 x2 + 3 x + 2x + 6 = 0
are some examples of quadratic equations. ⇒ x( x + 3 ) + 2( x + 3 ) = 0
⇒ ( x + 3 ) ( x + 2) = 0
Roots of a Quadratic Equation
⇒ x+3=0
The value of variable which is satisfied the given or x+ 2= 0
equation, is called the roots of the equation. ∴ x = − 3 and − 2
Finding the Roots of a Quadratic Equation Hence, − 3 and − 2 are the roots of the equation.
ax + bx + c = 0
2
Nature of Roots of a Quadratic Equation
Method I By splitting the middle term i.e., b into
two parts whose addition/subtraction is equal to b and For quadratic equation ax2 + bx + c = 0, where a, b and
the product of two parts is equal to a × c. c are real,
l if D = 0, roots of the equation are real and equal.
Method II Sridharacharya’s Rule
l if D > 0, roots of the equation are real and distinct.
− b + b2 − 4 ac −b+ D if D < 0, roots of the equation are complex /imaginary.
α= = l

2a 2a Example 2 If the roots of the equation x 2 − 4 x + k = 0 are


−b− b2 − 4 ac − b− D real and equal, what is the value of k?
and β= = (a) 2 (b) 4
2a 2a
(c) 8 (d) 1
where, D = b2 − 4 ac is called the discriminant. Solution (b) We know that, roots of a quadratic equation
ax2 + bx + c = 0 are real and equal, if
Example 1 Find the roots of the equation x 2 + 5 x + 6 = 0.
D = b2 − 4 ac = 0
(a) ( −3, − 2 ) (b) ( 3, 2 )
So, for quadratic equation
(c) ( −3, 2 ) (d) ( 2, − 3 )
x2 − 4 x + k = 0 roots will be real and equal
Solution (a) x2 + 5x + 6 = 0 D = ( − 4)2 − 4 × k × 1 = 0
Comparing by quadratic equation, ax + bx + c 2
⇒ 16 − 4 k = 0 ⇒ k = 4
Here, a = 1, b = 5, c = 6
Quadratic Equation 419
Sum and Products of the Roots Example 3 Form a quadratic equation whose roots are − 4
and 2/3.
For quadratic equation ax + bx + c = 0, let the roots
2

are α and β, then (a) 3x 2 + 10x − 8 = 0


b (b) 10x 2 + 3x − 8 = 0
Sum of the roots = α + β = −
a (c) 8x 2 + 3x − 5 = 0
c
Product of the roots = αβ = (d) 5x 2 + 5x − 2 = 0
a
Solution (a) Sum of roots = α + β = − 4 + 2
Formation of a Quadratic Equation 3
− 12 + 2 −10
= =
Let the roots of an equation is α and β, then the 3 3
equation can be made as 2 −8
and product of roots = αβ = − 4 × =
3 3
x2 − (Sum of roots) x + (Product of roots) = 0
So, the quadratic equation is
⇒ x2 − (α + β) x + (αβ) = 0 x2 − (Sum of roots) x + (Product of roots) = 0
 b −10   −8 

c
x2 −  −  x + = 0 ⇒ x2 −   x+   =0
 3   3 
 a a
⇒ 3 x2 + 10 x − 8 = 0
⇒ ax + bx + c = 0
2

Target Exercise
Elementary Level Questions
1. The roots of the quadratic equation x2 − 14 x + 45 = 0 7. If the roots of the quadratic equation kx2 + 3 x + 2 = 0
are are real and unequal , which of the following is true
(a) − 5 and − 9 (b) 5 and −9 for k?
(c) 5 and 9 (d) −5 and 9 (a) k > 9 / 8 (b) k < 9 / 8
2. If α and β are the roots of equation 3 x + 5 x + 2 = 0,
2 (c) k < − 9 / 8 (d) k < 8 / 9
the value of α + β is 8. Which of the following equation have 3 and 4 as the
(a) 5 / 3 (b) 2 / 3 roots?
(c) − 5 / 3 (d) − 3 / 5 (a) x 2 + 7 x + 12 = 0
3. Which of the following is one of the roots of the (b) x 2 − x + 7 = 0
quadratic equation 5x2 − 12 x − 9 = 0? (c) x 2 − 7 x + 12 = 0
(a) −3 (b) 3 / 5 (d) x 2 + 7 x − 12 = 0
(c) −3 / 5 (d) −5
9. The product of two integers is 20. If one number is
4. Which of the following quadratic equations have real 8 less than the other, the smaller number is
and distinct roots? (a) 10 (b) −2
(a) x2 − 3 x + 9 = 0 (b) x2 − 4 x + 4 = 0 (c) 2 (d) None of these
(c) x2 + 2 x + 3 = 0 (d) x2 + 5 x + 1 = 0 10. There a number 3x is 10 less than the square of the
number x. If the number is a positive integer, its
5. The nature of the roots of the quadratic equation
value is
x − 2 x − 4 = 0 is
2
(a) 2 (b) 3 (c) 4 (d) 5
(a) real and equal (b) distinct and rational
(c) complex/imaginary (d) irrational 11. If the sum and the product of the roots of a quadratic
6. If α and β are the roots of the quadratic equation equation is −14 and −32, respectively, then the roots are
1 1 (a) 16, 2
x2 + 2 x − 3 = 0 , then the value of + is (b) − 16, 2
α β
(a) −2 / 3 (b) 2 / 3 (c) 16, − 2
(c) 3 / 2 (d) −3 / 2 (d) −16, − 2
420 Study Package for NTSE

12. If the product of the roots of the equation 14. For what values of n, the roots of the quadratic equation
x2 − ( k + 7) x + 2 (2 k − 2) = 0 is three times the sum of x2 + x (14 − n) − 14 n + 1 = 0 are equal integers?
the roots, what is the value of k? (a) −13, − 15 (b) −11, − 16
(a) 20 (b) 23 (c) 25 (d) 30 (c) −12, − 16 (d) −11, − 12

13. If one root of the equation x2 + 6 mx + 64 = 0 is the 15. The sum of the squares of two consecutive positive
square of the other, the value of m is integers is 365, the smaller of the two numbers is
(a) 5 / 3 (b) 8 / 3 (a) 12 (b) 13
(c) 10 / 3 (d) None of these (c) 14 (d) 15

High Skill Questions


1. The number of real roots of quadratic equation 8. If the root of the equation ( b − c) x2 +
x − 5 x − 6 = 0, is
2
( c − a) x + ( a − b) = 0 is 1 , the other root will be
(a) 4 (b) 3 (c) 2 (d) 1 b−a
(a) 1 (b)
b−c
2. The graph of quadratic equation ax2 + bx + c , if a > 0 a−b
has minimum value at what of following values? (c) (d) None of these
b −c
(a) b / a (b) −b / 2 a
−b 9. The values of x for the equation
(c) (d) Cannot be determined
a 1 3 1
x + 2 = 2 +  x −  is
2

3. If the sum of the reciprocal of two numbers, whose x 2 x


normal sum is 80 , is 4 / 75, then the smaller of the two (a) ± 1 (b) 0
numbers is (c) ± 2 (d) None of these
(a) 20 (b) 30
(c) 40 (d) 50
10. Roots of the equation 31− a + 3 a −1 = 2 is/are
(a) 0 (b) −1 (c) 1 (d) 2
4. If α and β are the roots of the quadratic equation
11. Which of the quadratic equation has roots 4 more
x2 − 5 x + 1 , then the value of α4 + β 4 is
than the roots of the equation x2 + 3 x + 1 = 0 ?
(a) 527 (b) −527
(a) x2 − 5 x + 5 = 0 (b) x2 − 5 x − 5 = 0
(c) 0 (d) None of these
(c) x2 + 5 x + 5 = 0 (d) None of these
5. What is the value of the common root of equations
( a − b) x2 + ( b − c) x + ( c − a) = 0 and a ( b − c) x2 + 12. If α and β are the roots of the equation
b ( c − a) x + c ( a − b) = 0 ? x2 − k ( x + 1) − c = 0, the value of (α + 1) (β + 1) is equal
(c − a) to
(a) 0 (b)
(a − b) (a) 1 (b) 0
(c) 1 (d) None of these (c) 1 − c (d) None of these

6. Which of the equations will have real and distinct 13. The roots of the quadratic equation
roots? ( a + b)2 x2 − 2 ( a2 − b2 ) x + ( a − b)2 = 0 are
(a) 5 x2 − 3 2 x + 2 5 = 0 (a) distinct (both positive) (b) equal
(c) distinct (1 −ve and 1 + ve) (d) complex
(b) 2 3 x2 + 2 x − 3 = 0
(c) x2 − 4 x + 5 = 0 14. The value m for which the equation
(d) None of the above
3 x2 + (7 + m) x + 2 − m = 0 has equal roots is
(a) −1 (b) 1
7. If the equations ax2 + bx + c = 0 and x2 + 2 x + 9 = 0 (c) 25 (d) None of these
have a common root and a, b and c are real numbers,
15. The nature of the roots of the quadratic equation
the ratio of a : b : c is
( b − c) x2 + 2 ( c − a) x + ( a − b) = 0
(a) 2 : 1 : 9 (b) 1 : 2 : 9
(a) real and equal (b) imaginary
(c) 9 : 2 : 1 (d) 1 : 9 : 2 (c) real and distinct (d) Cannot be determined
Hints and Solutions
Elementary Level Questions
1. (c) Given quadratic equation, 10. (d) Let the number be x.
x − 14 x + 45 = 0 ⇒ x − 9 x − 5 x + 45 = 0
2 2
Then, 3 x = x2 − 10 ⇒ x2 − 3 x − 10 = 0
⇒ x ( x − 9) − 5 ( x − 9) = 0 i.e., x2 − 5 x + 2 x − 10 = 0
⇒ ( x − 9) ( x − 5) = 0 ⇒ x = 9, 5 ⇒ x ( x − 5) + 2 ( x − 5) = 0
So, the roots of the equation are 9 and 5 . ⇒ ( x − 5) ( x + 2 ) = 0
∴ x = 5, − 2
2. (c)Q 3 x2 + 5 x + 2 = 0
11. (b) We know, α + β = − 14 and αβ = − 32
∴α + β = −b/a = − 5/3
Equation is x2 − ( − 14) x − 32 = 0
3. (c)Q 5 x − 12 x − 9 = 0 ⇒ 5 x − 15 x + 3 x − 9 = 0
2 2
⇒ x2 + 14 x − 32 = 0
⇒ 5 x ( x − 3) + 3 ( x − 3) = 0 ⇒ (5 x + 3) ( x − 3) = 0
⇒ x2 + 16 x − 2 x − 32 = 0
⇒ 5 x + 3 = 0 ⇒ x = − 3 /5
and ( x − 3) = 0 ⇒ x = 3 ⇒ x ( x + 16) − 2 ( x + 16) = 0
Thus, x = − 3 /5 and 3 are the roots. ⇒ ( x + 16) ( x − 2 ) = 0
4. (d) Roots are real and distinct, if D = b − 4 ac > 0
2
So, the roots are −16 and 2 .
(a) x2 − 3 x + 9 = 0 12. (c) x2 − (k + 7 ) x + 2 (2 k − 2 ) = 0
D = ( − 3)2 − 4 × 1 × 9 = 9 − 36 = − 27 < 0
Sum of roots = α + β = − b/a = (k + 7 )
(b) x − 4 x + 4 = 0 , D = ( − 4)2 − 4 × 1 × 4 = 16 − 16 = 0
2
2 (2 k − 2 )
and αβ = Product of roots = c/a =
(c) x2 + 2 x + 3 = 0 , D = (2 )2 − 4 × 1 × 3 = 4 − 12 = − 8 < 0 1
(d) x2 + 5 x + 1 = 0 , D = (5)2 − 4 × 1 × 1 = 25 − 4 = 21 > 0 We know that, 3 (α + β ) = αβ
⇒ 3 (k + 7 ) = 2 (2 k − 2 ) ⇒ 3 k + 21 = 4 k − 4
Thus, roots are real and distinct for option (d).
∴ k = 25
5. (d) x2 − 2 x − 4 = 0
13. (d) Let the roots be α and α 2 .
D = (−2 )2 − 4 × 1 × (−4) = 4 + 16 = 20 > 0
Then, x2 + 6 mx + 64 = 0
Thus, roots are irrational. c
Product of roots = = α × α 2 = α 3 = 64 ⇒ α = 4
6. (b) For x2 + 2 x − 3 = 0 a
b c So, the roots are 4 and 16.
α+β=− = 2 ⇒ αβ = = − 3
a a −b −20 10
Sum of the roots = = − 6 m = 4 + 16 ⇒ m = =−
1 1 α + β −2 2 a 6 3
Now, + = = =
α β αβ −3 3 14. (c) x2 + x (14 − n) − 14 n + 1 = 0
7. (b) Roots are real and unequal, if Roots are equal integers, when D = 0 = b 2 − 4 ac
D = b − 4 ac > 0
2
⇒ (14 − n)2 − 4 (1) ( − 14 n + 1) = 0
So, for equation kx + 3 x + 2 = 0 roots will be real and unequal,
2
⇒ 196 + n2 − 28 n + 56 n − 4 = 0
if D = (3)2 − 4 × 2 × k > 0 ⇒ n2 + 28 n + 192 = 0
9
i.e., 9− 8k> 0 ⇒ k< ⇒ n + 16 n + 12 n + 192 = 0
2
8
⇒ (n + 16) (n + 12 ) = 0
8. (c) We know quadratic equation is
∴ n = − 16 and −12
x2 − (Sum of roots )x + (Product of roots) = 0
15. (b) Let the two consecutive integers be x and ( x + 1.)
So, the equation having 3 and 4 as the roots is
We know that, x2 + ( x + 1)2 = 365
x2 − (3 + 4) x + 3 × 4 = 0
⇒ x2 + x2 + 2 x + 1 = 365
i.e., x2 − 7 x + 12 = 0
⇒ 2 x2 + 2 x − 364 = 0
9. (c) Since, the integer is 8 less than the other, let the integer be
x and x − 8. ⇒ x2 + x − 182 = 0
We have, x ( x − 8) = 20 i.e., x − 8 x − 20 = 0
2
⇒ x2 + 14 x − 13 x − 182 = 0
⇒ x − 10 x + 2 x − 20 = 0
2
⇒ ( x + 14) ( x − 13) = 0
⇒ x ( x − 10) + 2 ( x − 10) = 0 ⇒ ( x − 10) ( x + 2 ) = 0 ⇒ x = 13 and −14
∴ x = 10 and− 2 Since, the integers are positive, they are 13 and 14.
Thus, the smaller number is 10 − 8 = 2. Hence, the smaller number = 13.
422 Study Package for NTSE

High Skill Questions


1. (c) The equation x2 − 5 x − 6 = 0 1 3 1
9. (a) x2 + =2 + x− 
x2 2  x
Let x = y 2
⇒  x −  −  x − 1 = 0
1 3
y2 − 5 y − 6 = 0    
 x 2 x
y2 − 6 y + y − 6 = 0 [Q(a − b )2 = a2 + b 2 − 2 ab]
y ( y − 6) + 1 ( y − 6) = 0 ⇒ ( y − 6)( y + 1) = 0
2
1
Let x − = P ⇒ P 2 − 3 / 2 P = 0 ⇒ P = 0, 3 / 2
y = 6 and y = −1 x
x = 6 and x = −1 x2 − 1
If P=0 ⇒ = 0 ⇒x= ± 1
∴ x = 6 is the real roots of given quadratic equation. x
1 x2 − 1 3
−b If P = 3 /2 ⇒ x − = 3 /2 ⇒ =
2. (b) Since, the quadratic graph has minimum value at x = x x 2
2a
⇒ 2 x2 − 2 − 3 x = 0
when a > 0.
⇒ 2 x2 − 4 x + x − 2 = 0
⇒ 2 x( x − 2 ) + 1( x − 2 ) = 0
2
⇒  x − 1 − 3  x − 1 = 0
   
 x 2  x
– b/2a ⇒ ( x − 2 ) (2 x + 1) = 0 ⇒ x = 2 , x = − 1/2
1 1 4 a+ b 4 80 4 Thus, x = ± 1, 2, − 1/2
3. (b) a + b = 80 and + = ⇒ = ⇒ =
a b 75 ab 75 ab 75 3a 3 x 3
10. (c)Q + a =2 ⇒ + =2 (Q x = 3a )
⇒ a b = 20 × 75 = 1500 3 3 3 x
= 15 × 100 = 15 × 2 × 50 = 30 × 50 ⇒ x2 − 6 x + 9 = 0
∴ Smaller number = 30 ⇒ x = 3 = 3a ⇒ a = 1
4. (a) Since, α + β = 5 and αβ = 1 11. (a) For roots to be 4 more, we replace x with ( x − 4) in
∴ α 4 + β 4 = (α 2 + β 2 )2 −2α 2β 2 x 2 + 3x + 1 = 0
= [(α + β )2 − 2αβ ]2 − 2(αβ )2 So, the equation is
= (25 − 2 ) − 2 = 529 − 2 = 527
2 ( x − 4)2 + 3 ( x − 4) + 1 = 0
⇒ x 2 − 8 x + 16 + 3 x − 12 + 1 = 0
5. (c) Let x = 1be the common root of given equations,
⇒ x2 − 5 x + 5 = 0
(a − b ) x2 + (b − c ) x + (c − a) = 0 ...(i)
12. (c) The equation is x2 − kx − (c + k ) = 0
a (b − c ) x2 + b (c − a) x + c (a − b ) = 0 ...(ii)
∴ α + β = k and αβ = − ( c + k )
Put x = 1in Eqs. (i) and (ii), we get
∴ (α + 1) (β + 1) = αβ + (α + β ) + 1
a− b + b −c + c − a = 0 ⇒0 = 0 = − (c + k ) + k + 1 = 1 − c
and a(b − c ) + b(c − a) + c(a − b )
13. (b) D = b 2 − 4 ac = 4(a2 − b 2 )2 − 4(a + b)2 (a − b)2
⇒ ab − ac + bc − ab + ca − bc = 0 ⇒ 0 = 0
= 4(a2 − b 2 )2 − 4(a2 − b 2 )2 = 0
i.e., 1 satisties of both equation.
So, the roots are equal.
So, 1 is the common roots of given equation.
14. (a) 3 x2 + (7 + m) x − (2 − m) = 0
6. (b) (a) 5 x2 − 3 2 x + 2 5 = 0
Roots are equal, if D = 0
D = b 2 − 4 ac = 18 − 4 × 10 < 0 b 2 − 4 ac = (7 + m)2 − 12 (2 − m) = 0
(b) 2 3 x2 + 2 x − 3 = 0 ⇒ 49 + 14 m + m 2 − 24 + 12 m = 0
D = 4 + 24 > 0 ⇒ Real and distinct roots. ⇒ m2 + 26 m + 25 = 0
7. (b) The roots of x2 + 2 x + 9 are imaginary. ⇒ (m + 25) (m + 1) = 0
∴ m = − 25 and − 1
So, it will come in conjugate pairs.
So, both the roots should be common. 15. (d) (b − c ) x2 + 2 (c − a) x + (a − b) = 0
Thus, the equations D = b 2 − 4 ac = 4 (c − a)2 − 4(b − c ) (a − b )
x + 2 x + 9 = 0 and ax2 + bx + c = 0 are identical. = c 2 + a2 − 2 ac − a b + b 2 + ac − bc
a b c
= = ⇒ a : b : c = 1: 2 : 9 = a2 + b 2 + c 2 − ac − bc − a b
1 2 9
= 2 (a2 + b 2 + c 2 − a b − bc − c a)
8. (c) Let α = 1and β = ?
(a − b) (a − b) = (a − b )2 + (b − c )2 + (c − a)2 ≥ 0
∴ αβ = c/a = ⇒ β=
(b − c ) (b − c ) So, the roots are real and may or may not be equal.
Arithmetic Progression 423

Chapter

6
Arithmetic Progression
Definition e.g.,
Sum of 10 terms of an AP 12, 14, 16, 18, ... is
An arithmetic progression is a series in which the gap
between the terms is constant/fixed. Here, n = 10, a = 12, d = 2
10
An Arithmetic Progression (AP) is like So, S10 = [2 × 12 + (10 − 1) × 2]
2
a, a + d, a + 2d, a + 3 d, ...
= 5 (24 + 18) = 5 ( 42) = 210
where, a = first term and d = common difference
(ii) If in an AP first and last term and number of terms
e.g., 2, 5, 8, 11, 14, ... and 8, 3, − 2, − 7, ... are given, then
are two examples of AP with common differences of 3  a + l
and − 5, respectively. Sn =   × Number of terms
∴ nth term of an AP = Tn = a + ( n − 1) d  2 
where, a = First term where, a = First term
and l = Last term
n = Number of term
e. g., Sum of 9 terms of an AP whose first and last
and d = Common difference term are 1 and 17, respectively.
e.g., Here, n = 1, l = 17, number of terms = 9
(i) 10th term of an AP 3, 7, 11, 15, ... is 1 + 17 
∴ Sn =   × 9 = 81
T10 = 3 + (10 − 1) × 4 = 3 + 9 × 4  2 
[Qd = 7 − 3 = 4] Note ● Three terms a, b and c are in AP, if 2b = a + c

= 3 + 36 = 39 ● In an AP, Sum of 1st and last term = Sum of 2nd and the
2nd last term
(ii) 20th term of an AP having first term 8 and the = Sum of 3rd and 3rd last term and so on
common difference(−2), is ⇒ S n − S n −1 = an
⇒ Sum of 1st n terms − Sum of 1st (n − 1) terms = nth term
T20 = a + (20 − 1) d = 8 + 19 × ( − 2) = 8 − 38 = − 30
Example 1 If the first term and the common difference of
Sum of n Terms of an Arithmetic an arithmetic progression is 4 and 8 respectively, which is its
Progression 15th term?
(i) If in an AP number of terms and common difference (a) 116 (b) 120 (c) 125 (d) 115
are given, then Solution (a) First term, a = 4 and common difference, d = 8
n
Sn = [2 a + ( n − 1) d] We know nth term, Tn = a + ( n − 1)d
2
So, T15 = 4 + (15 − 1) 8 = 4 + 14 × 8
where, a = First term d = Common difference
= 4 + 112 = 116
and n = Number of terms
424 Study Package for NTSE

Example 2 If the first term and the last term of an arithmetic Example 3 If sum of 1st 5 terms of an arithmetic
progression are 3 and 153, respectively. If the sum of all the terms is progression is equal to the sum of first 6 terms, then find
2418, then find the number of terms in AP. the 6th term of an AP.
(a) 28 (b) 31 (c) 34 (d) 30 (a) 1 (b) 0
Solution (b) Here, first term, a = 3 and last term = 153 (c) 5 (d) 2

Let the last term of the series be the n th term, then Solution (b) We know that, S6 = S5
153 = 3 + ( n − 1) × d n
[usingSn = ( 2a + ( n − 1) d )]
2
⇒ ( n − 1) × d = 150 …(i) ⇒
6
[2a + ( 6 − 1) d ]
n 2
As the sum of these n terms = 2418 = [2a + ( n − 1)d ] 5
2 = [2a + (5 − 1) d ]
2
n
⇒ 2418 = [2 (3 ) + 150 ] [from Eq. (i)] ⇒ 12a + 30d = 10a + 20d
2
⇒ 2a = − 10 d
2418 × 2
∴ n= = 31 ⇒ a = − 5d
156
Now, a 6 = a + ( 6 − 1) d = a + 5d
Thus, the number of terms = 31 But a = − 5d
Alternate Method ∴ a6 = 0
We know, sum of terms of an AP Alternate Method
= 
First term + Last term We know that, S5 = S6
 × Number of terms
 2  ⇒ S6 − S5 = 0
3 + 153  S n − S n − 1 = an
2418 = 
and
⇒  × Number of terms
 2  ⇒ S 6 − S 5 = a6 = 0
2418 × 2 Thus, 6th term = 0
∴ Number of terms = = 31
156

Target Exercise
Elementary Level Questions
1. 10th term an AP whose first term and the common 8. The number of terms in the series 11, 15, 19, ...,
difference are 3 and 4, respectively is 211 is
(a) 39 (b) 42 (c) 45 (d) 50 (a) 49 (b) 50
(c) 51 (d) None of these
2. How many terms are there in AP 2, 6, 10, 14 ,......, 130?
(a) 32 (b) 33 (c) 34 (d) 35 9. If the sum and the product of first 3 terms of an AP
3. If the first term is equal to the common difference and the series is 42 and 2240 respectively, what are the
2nd term is 10. Then, the sum of first 20 terms is terms of the series?
(a) 1150 (b) 950 (c) 1050 (d) 1000 (a) 8, 12, 16 (b) 8, 14, 20
(c) 10, 16, 22 (d) None of these
4. If the sum of 1st n terms of an AP is 2n( n + 3). Then, what is
the value of its 6th term? 10. If the sum of 3 consecutive terms of an AP is zero.
(a) 24 (b) 28 Then, what will be their product?
(c) 32 (d) None of these (a) 15 (b) 3
(c) 27 (d) None of these
5. The sum of 3 terms of an AP is 39 and their product is 2080,
the terms are 11. If a, b and c terms an AP, which of the following
(a) 13, 15, 17 (b) 14, 18, 22 will also be an AP?
(c) 10, 13, 16 (d) None of these (a) a − 4, b + 4, c − 4 (b) a + 4, b + 4, c − 4
(c) a + 5, b + 6, c + 7 (d) None of these
6. The sum of first and the last term of an AP of 150 terms is
equal to the sum of which of the following two terms? 12. The sum of terms of two AP’s is an
(a) AP (b) GP
(a) 15th and 135th (b) 20th and 131th
(c) HP (d) None of these
(c) 50th and 51th (d) 90th and 60th
13. If the nth term of an AP is 4 n − 2. Then, the 10th
7. The 40 th term of the AP 2, 8, 14, 20, ... is
(a) 230 (b) 232
term will be
(a) 28 (b) 38 (c) 42 (d) 60
(c) 240 (d) 236
Arithmetic Progression 425
High Skill Questions
1. The first and the last term of an arithmetic
progression are 4 and 128, respectively. If the sum of 9. If a, b and c are in AP, then which of the following will
all the terms is 2970, what is the number of terms? also term an AP?
(a) 40 (b) 45 a b c
(a) a − b, b − c, c − a (b) , , +1
(c) 51 (d) Cannot be determined 2 2 2
2. What is the value of 11th term of an AP series, if the (c) a + b, b − c, c + a (d) None of these
sum of its first 11 terms is equal to the sum of 10. If the nth term of an AP is 5n − 2. Then, what is the
1st 10 terms? sum of first 10 terms of the same series?
(a) 11 (b) 1
(a) 250 (b) 255
(c) 0 (d) None of these
(c) 300 (d) None of these
3. What is the value of 9th term of an AP series whose
11. If the sum of first n terms of an AP is n(3 n + 7). Then,
6th term and 12th terms are 32 and 56, respectively? its 10th term is
(a) 42 (b) 44 (a) 63 (b) − 64 (c) 64 (d) 16
(c) 34 (d) None of these
12. In a AP series of 100 terms a1 + a50 + a51 + a100 = 400,
4. If the sum of first 9 terms of an AP is equal to the sum
so what is the sum of all the terms of the series?
of 1st 13 terms. Then, the sum of first 22 terms will be
(a) 10000 (b) 5000
(a) 0 (b) 1 (c) − 1 (d) 35
(c) 1000 (d) None of these
5. The ratio of the sum of first ‘P’ terms of two AP series is
13. The value of the 20th term for the AP series
( 4 P + 16) : (7 P − 17), what is the ratio of their 6th 1 2 1
terms? 10, 10 , 10 , 11, 11 , ... is
15 3 3 3
(a) 1 (b) 790
43 (a) (b) 260
59 3
(c) (d) None of these (c) 268 (d) None of these
160
6. If a7 = 13 a13, where a7 and a13 are the 7th and the 14. The sum of first 9 terms of an AP series is equal to the
13th terms respectively and the common difference is sum of first 11 terms of the same series what is the
6. Then, 29th term of the same AP is value of the sum of first and the last term of the series,
(a) 80 (b) 93 if there are 20 terms in it?
(c) − 93 (d) None of these (a) 0 (b) 1
(c) − 1 (d) None of these
7. An AP series which has 20 terms has the first term
and the last term 5 and 158, respectively. What is the 15. If x1, x2 , x3, …, xn are in AP. Then, the value of
sum of its 9th and the 12th term? 1 1 1 1
+ + + ... + is
(a) 213 (b) − 213 x1 x2 x2 x3 x3 x4 xn − 1 xn
(c) 200 (d) Cannot be determined
n−1 n−1
8. Two AP’s 45, 41, 37, ... and 5, 9, 13, ... have their kth (a) (b)
x1 xn x2 xn − 1
terms same. Find the value of k.
n
(a) 6 (b) 5 (c) (d) None of these
x1 xn
(c) 4 (d) None of these

Hints and Solutions


Elementary Level Questions
1. (a) a = 3, d = 4 ⇒ a10 = a + (n − 1) d 3. (c) a = d , a + d = 10 ⇒ a = d = 5
20
= 3 + (10 − 1) × 4 S 20 = [2 × 5 + (20 − 1) × 5]
2
= 3 + 36 = 39
= 10 (10 + 19 × 5)
2. (b) a = 2, d = 4, an = 130 = 10 (105) = 1050
an = a + (n − 1)d 4. (b) S n − S n − 1 = an
⇒ 130 = 2 + (n − 1) × 4
S 6 − S 5 = [2 × 6 (6 + 3)] − [2 × 5 (5 + 3)]
⇒ n = 33
= (108 − 80) = 28
426 Study Package for NTSE

5. (c) Let the terms be a − d , a and a + d . 9. (b) (a − d ) + a + (a + d ) = 42 ⇒ a = 14


then, (a − d ) + a + (a + d ) = 39 ⇒ a = 13 and (a − d ) × a × (a + d ) = 2240
⇒ (13 − d ) × 13 × (13 + d ) = 2080 (14 − d ) × 14 × (14 + d ) = 2240
⇒ (142 − d 2 ) × 14 = 2240
∴ 13 (169 − d 2 ) = 2080 ⇒ d = 3
∴ d=± 6
So, the terms are 13 − 3, 13, 13 + 3, i.e., 10, 13, 16. So, the terms are 8, 14 and 20.
6. (b) In a series of 150 terms, 10. (d) If (a − d ) + a + (a + d ) = 0
a1 + a150 = a2 + a149 = a3 + a148 = …
⇒ a=0
So, a20 + a131 = a1 + a150
Thus, product = (a − d ) a (a + d ) = 0
7. (d) an = a + (n − 1)d 11. (c) Since, a + 5, b + 6, c + 7 is an addition of two AP, so it
a40 = [2 + (40 − 1) × 6] = 2 + 39 × 6 should be an AP.
= 2 + 234 = 236
12. (a) Sum of two AP’s is an AP.
8. (c) an = a + (n − 1)d
13. (b) an = 4n − 2, a10 = 4 (10) − 2 = 38
211 = 11 + (n − 1) × 4 ⇒ n = 51

High Skill Questions


1. (b) We know that, sum of terms of an AP series Alternate Method
S p = Sq
= 
First term + Last term If
 × Number of terms = 2970
 2  ⇒ Sp =0
+q
4 + 128 So, if S 9 = S13
⇒ × n = 2970
2 ⇒ S 9 + 13 = S 22 = 0
2970 × 2
⇒ n= SP 4P + 16 P /2 [2 a + (P − 1) d ]
132 5. (a) Given, = =
S P′ 7 P − 17 P /2 [2 a′ + (P − 1) d ′ ]
⇒ n = 45
2 a + (P − 1) d
2. (c) Given, S11 = S10 =
2 a′ + (P − 1) d ′
⇒ S11 − S10 = 0
Put P = 2 × 6 − 1 = 11 [Q n = 6]
We know that, S n − S n − 1 = an ,
2 a + 10 d 4 × 11 + 16 60
So, S11 − S10 = 0 Given, = = =1
2 a′ + 10d ′ 7 × 11 − 17 60
⇒ a11 = 0 a + 5d a
⇒ = 6 =1
3. (b) Given that, 6th term (a6 ) = 32 a′ + 5 d ′ a6 ′
⇒ a + (6 − 1) d = 32 [Q an = a + (n − 1)d ] 6. (b) a7 = 13 a13
⇒ a + 5 d = 32 …(i)
⇒ a + 6d = 13(a + 12d ) [Q an = a + (n − 1)d ]
and 12th term (a12 ) = 56
⇒ a + (12 − 1) d = 56 ⇒ 12 a + 150 d = 0
⇒ a + 11 d = 56 …(ii) ⇒ 2 a = − 25d
On adding Eqs. (i) and (ii), we get = − 25 × 6 = − 150 ⇒ a = − 75 [Qd = 6]
⇒ 2 a + 16 d = 88 ∴ a29 = a + (28)d = − 75 + 28 × 6
⇒ a + 8 d = 44 …(iii) = − 75 + 168 = 93
∴ a9 = a + (9 − 1) d
= a + 8 d = 44 [from Eq. (iii)] 7. (a) Sum of first and last term of an AP = Sum of 2nd and 2nd last
term = Sum of 3rd and 3rd last term and so on
4. (a) Given, S 9 = S13 So, Sum of first and last term = a1 + a20 = 5 + 158
9 13
i .e., [2 a + (9 − 1) d ] = [2 a + (13 − 1) d ] = Sum of 9th and 12th term = a9 + a12 = 213
2 2
Q S = n [2 a + (n − 1)d ] 8. (a) We know that, 45 + (k − 1) × (− 4) = 5 + (k − 1) × 4
 
n
2 [Q an = a + (n − 1)d ]
⇒ 18 a + 72 d = 26 a + 156 d ⇒ 40 = 8 (k − 1) ⇒(k − 1) = 5
⇒ 8 a = − 84 d
⇒ 2 a = − 21 d …(i) ∴ k=6
22 Thus, 6th term is common.
Now, S 22 = [2 a + (22 − 1) d ]
2
9. (a) a − b, b − c, c − a each term has is ‘d’ it is an AP with all
= 11 (2 a + 21 d )
= 11 (2 a − 2 a) = 0 terms having ‘0’ gap.
Arithmetic Progression 427
10. (b) an = 5n − 2, a1 = 3, a2 = 8, a3 = 13 and so on 14. (a) Q S 9 = S11

S10 =
10
[2 × 3 + (10 − 1) × 5] Q S = n [2 a + (n − 1)d ] ∴ S11 − S 9 = 0 and a11 + a10 = 0
 
n
2 2 As there are 20 terms in AP,
= 5 (6 + 45) = 51 × 5 = 255 a1 + a20 = a2 + a19
11. (c) S n = n(3 n + 7 ) = ... = a10 + a11 = 0
We know that S n − S n − 1 = an Thus, a1 + a20 = 0
1 1 1 1
So, S10 − S 9 = a10 = 10 (3 × 10 + 7 ) − 9 (3 × 9 + 7 ) 15. (a) + + + ... + is
x1 x2 x2 x3 x3 x4 xn − 1 xn
= 370 − 306 = 64
12. (a) As AP has 100 terms, we know 1  d d d 
=  + + ... + 
a1 + a100 = a2 + a99 = a3 + a98 = ... and so on xx xn − 1 xn 
d  1 2 x2 x3
So, a1 + a50 + a51 + a100 = 400
x − x x − xn − 1 
1 =
1
 2 1 + x3 − x2 + K + n 
⇒ a1 + a100 = (400) = 200  x x
2 d  2 1 x3 x2 xn − 1 xn 
a + a100
Thus, S100 = 1 × 100 1 1 1 1 1 1 1 1
2 =  − + − + + K+ − 
d  x1 x2 x2 x3 x3 xn − 1 xn 
= 100 × 100 = 10000
13. (a) a = 10, d = , S 20 = 2 × 10 + (20 − 1) × 
1 20 1
1 1 1  1  xn − x1  1  (n − 1)d 
3 2  3  =  −  =   =  xx 
d  x1 xn  d  x1 xn  d  1 n 
= 10  20 +
19  790
 = (n − 1)
 3 3 =
x1 xn
428 Study Package for NTSE

Chapter

7
Introduction to
Euclid’s Geometry
● Axioms or postulates Axioms or postulates are the
assumptions which are obvious universal truths. They
● Euclid’s five postulates Euclid’s five postulates are
are not proved. given below
Mainly postulates are used for specially Geometry and (i) Postulate 1 A straight line may be drawn from
axioms are used for specially Algebra. any one point to any other point.
● Theorems Theorems are statements which are proved A B
using definitions, axioms, previously proved statements
Axiom Given two distinct points, there is a
and deductive reasoning.
unique line that passes through them.
● Euclid’s Definitions, Axioms or Postulates B
(i) A point is that which has no part. A
(ii) A line is breadth less length.
(iii) The ends of a line are points.
(ii) Postulate 2 A terminated line can be produced
(iv) A straight line is a line which lies evenly with the
indefinitely. Euclid’s terminated line is called a
points on itself.
line segment. So, according to the present day
(v) A surface is that which has length and breadth only.
terms, ‘A line segment can be extended on either
(vi) The edges of a surface are lines. side to form a line’.
(vii) A plane surface is a surface which lies evenly with
(iii) Postulate 3 A circle can be drawn with any
the straight lines on itself.
centre and any radius.
● Undefined terms There are some terms like point etc.,
A
which have no definitions, these terms are called
undefined terms.
In Geometry, a point, a line and a plane (in Euclid’s O
words, a plane surface) are undefined terms.
● Euclid’s Axioms
(iv) Postulate 4 All right angles are equal to one
(i) Things which are equal to the same things, are another.
equal to one another.
O X
(ii) If equals are added to equals, the wholes are equal.
(iii) If equals are subtracted from equals, the 2
remainders are equal. O Z
(iv) Things which coincide with one another are equal to 1
one another. A C B
(v) The whole is greater than the part. Y
(vi) Things which are double of the same things are
equal to one another. If Ð 1 = 90°
(vii) Things which are halves of the same things are and Ð 2 = 90°
equal to one another. Then, Ð1 = Ð2
Introduction to Euclid’s Geometry 429
(v) Postulate 5 If a straight line falling on two Solution. (a) Given, Ð1 = Ð4 and Ð3 = Ð2 ...(i)
straight lines makes the interior angles on the If Ð2 = Ð4
same side of it taken together less than two right Then, Ð2 = Ð1 [Q Ð1 = Ð4, by axiom 1] ...(ii)
angles, then the two straight lines, if produced
Þ Ð3 = Ð1 [Q Ð2 = Ð3]
indefinitely, meet on that side on which the sum of
angles is less than two right angles. Hence, Euclid’s axiom 1 is used to show the above result.

Example 2. In the given figure, if ÐDOB = 87° , ÐCOA = 82°


B
P and ÐBOA = 35°, then find ÐCOB and ÐCOD.
A
1
D C
2
Q B
C
D 35°
O A
e.g., The line PQ falls on lines AB and CD such (a) 45° and 40° (b) 47° and 40°
that the sum of the interior angles 1 and 2 is less (c) 50° and 50°(d) None of these
than 180° on the left side of PQ. Therefore, the
lines AB and CD will eventually intersect on the Solution (b) From figure, we have ÐDOB = ÐDOC + ÐCOB
left side of PQ. Given, ÐDOB = 87°
î Two distinct lines cannot have more than one point in \ ÐDOC + ÐCOB = 87° [by axiom 1] ...(i)
common. Similarly, ÐCOB + ÐBOA = ÐCOA
● Equivalent versions of Euclid’s fifth Þ ÐCOB + ÐBOA = 82° ...(ii)
postulate For every line l and for every point P not [QÐCOA = 82°, given]
lying on l, there exists a unique line m passing through
But ÐBOA = 35° [given]
P and parallel to l.
This result can also be stated in the following form. From Eq. (ii), ÐCOB + 35° = 82°
Two distinct intersecting lines cannot be parallel to the Þ ÐCOB + 35°-35° = 82°-35°
same line. [using axiom 3, subtracting equal thing, i.e. ÐBOA = 35° from
both sides]
Example 1. In the given figure, it is given that Ð 1 = Ð4 and
Þ ÐCOB = 47° ...(iii)
Ð 3 = Ð 2. By which Euclid’s axiom, it can be shown that, if
Ð 2 = Ð 4, then Ð 1 = Ð 3 ? On subtracting equal thing, ÐCOB from both sides from
Eq. (i), we get
D C
1 ÐDOC + ÐCOB - ÐCOB = 87° - ÐCOB [using axiom
3 3]
4
2 Þ ÐDOC = 87° - ÐCOB
A B
Þ ÐDOC = 87° - 47° [from Eq. (iii)]
(a) Axiom 1 (b) Axiom 2
\ ÐDOC = 40°
(c) Axiom 3 (d) Axiom 4
Hence, ÐCOB = 47° and ÐDOC or ÐCOD = 40°

Target Exercise
Elementary Level Questions
1. The number of dimensions, a solid has 4. A pyramid is a solid figure, the base of which is
(a) 1 (b) 2 (c) 3 (d) 0 (a) only a triangle (b) only a square
(c) only a rectangle (d) any polygon
2. Euclid divided his famous treatise ‘The Elements’
into 5. It is known that, if x + y = 10, then x + y + z = 10 + z.
(a) 13 chapters (b) 12 chapters (c) 11 chapters (d) 9 chapters The Euclid's axiom that illustrates this statement is
(a) first axiom (b) second axiom
3. If A, B and C are three points on a line such that A (c) third axiom (d) fourth axiom
lies in between B and C, then
1 BC 6. The number of interwoven isosceles triangles in
(a) BC = AC (b) AC = Sriyantra (in the Atharvaveda) is
2 4
(c) AC = BC (d) AB + AC = BC (a) seven (b) eight (c) nine (d) eleven
430 Study Package for NTSE

7. Greek’s emphasised on 9. If PQ is a line segment of length 12 cm and R is a point


(a) inductive reasoning in its interior,
(b) deductive reasoning then PR2 + QR2 + 2 PR × QR equals
(c) Both (a) and (b) (a) 12 (b) 13 (c) 144 (d) 169
(d) practical use of geometry
10. In the given figure, C is the mid-point of the segment
8. Which of the following needs a proof? AB. P and Q are mid-points of the segments AC and
(a) Theorems BC respectively, then AP is equal to
(b) Axiom
A P C Q B
(c) Definition
(d) Postulate AB AB AB
(a) (b) (c) (d) AB
4 2 3

High Skill Questions


1. Which of the following statements are false? 6. In the given figure, if Ð1 = Ð3, Ð2 = Ð4 and Ð3 = Ð4,
(a) The Euclidean Geometry is valid only for figures in the plane then the relation between Ð1 and Ð2.
(b) Euclid’s fourth axiom says that everything equals itself D C
(c) Two distinct intersecting lines cannot be parallel to the same 3
line 4
(d) There are an infinite number of lines which pass through two
distinct points 1
2
A B
2. Given four collinear points A, B, C and D. How many
all the line segments determined by these? (a) Ð1 > Ð2 (b) Ð1 < Ð2
(a) 2 (b) 6 (c) 5 (d) 4 (c) Ð1 = Ð2 (d) None of these

3. Solve the equation y - 25 = 40 and state which axiom 7. If P and Q are the centres of two intersecting circles,
will you use here? then
(a) Axiom 2 (b) Axiom 3 (c) Axiom 4 (d) Axiom 5 (a) PQ = QR + PR (b) PQ + QR = PR
(c) QR = PQ + PR (d) PQ = QR = PR
4. In the given figure, if AC = BD, then
(a) AB = BD (b) BC = AD (c) AB = CD (d) AC = CD 8. In the given figure, if AC = DC and CB = CE, then
A
5. In the given figure, if AB = BC and BX = BY , then E
C
B
D
B
X Y
(a) AB = DE (b) AD = CD (c) BE = CD (d) AD = AC

9. If AB = x + 3, BC = 2x and AC = ( 4 x - 5), then for


A C what value of x, B lines on AC?
(a) AX = CY (b) AX = BY (c) AX = BC (d) BY = AB (a) 6 (b) 7 (c) 8 (d) 9

Hints and Solutions


Elementary Level Questions
1. (c) A solid has shape, size, position and can be moved from one Now, by using Euclid axiom 4, things which coincide with one
place to another. So, solid has three dimensions, e.g., Cuboid. another are equal to one another, we get
2. (a) Euclid divided his famous treatise ‘The Elements’ into BC = BA + AC
13 chapters. 4. (d) A pyramid is a solid figure, the base of which is a triangle or
3. (d) Since, A lies between B and C, therefore BC coincides with square or some other polygon.
BA + AC. 5. (b) The Euclid’s axiom that illustrates the given statement is
B A C second axiom, according to which. If equals are added to
equals, the wholes are equal.
Introduction to Euclid’s Geometry 431
6. (c) The Sriyantra (in the Atharvaveda) consists of nine interwoven 10. (a) Since, C is mid-point of the line segment AB.
isosceles triangles. \ AB = AC + CB [by axiom 4]
7. (b) Greek’s emphasised on deductive reasoning. Þ AB = AC + AC [Q AC = CB]
1
8. (a) The statements that were proved are called propositions or Þ AC = AB ...(i)
theorems. 2
Similarly as, P is mid-point of line segment AC.
9. (c) Here, point R lies in the interior of PQ. 1
12 cm \ AP = AC ...(ii)
2
P R Q From Eqs. (i) and (ii), we get
AP = ìí ABüý = AB.
So, PR + RQ = PQ = 12 cm [by axiom 4] 1 1 1
Now, PR 2 + QR 2 + 2 PR × QR = (PR + QR )2 2 î2 þ 4
= (PQ )2 = (12 )2 = 144 cm

High Skill Questions


1. (d) (a) True, it fails on the curved surface. 6. (c) Given, Ð1 = Ð3,
(b) True, as it is the justification of the principle of superposition. Ð3 = Ð4 … (i)
(c) True, it is an equivalent version of Euclid’s fifth postulate. and Ð2 = Ð4 … (ii)
(d) False, because from two distinct points, only one straight line According to Euclid’s axiom 1, things which are equal to the
can pass. same things, are equal to one another.
2. (b) Here, are 6 lines segments. Therefore, from Eq. (i), Ð1 = Ð4 …(iii)
All the line segments determined by A, B, C and D are Now, from Eqs. (ii) and (iii), Ð1 = Ð2 [by axiom 1]
AB, BC, CD, AC, BD, AD 7. (d) In the given figure,
A B C D PR = PQ [radii of the same circle] … (i)
R
3. (a) There are use axiom 2.
y - 25 = 40
P Q
Add 25 both sides [by axiom 2, if equals are added to equals, the
wholes are equal]
y - 25 + 25 = 40 + 25 and QP = QR [radii of the same circle] … (ii)
Þ y = 65 From Eqs. (i) and (ii), PR = QR [by axiom 1] …(iii)
4. (c) Given, From Eqs. (i), (ii) and (iii), we get
AC = BD …(i) PQ = QR = PR [by axiom 1]
From figure, it is clear that 8. (a) Given, AC = DC
AC = AB + BC Þ CB = CE
and BD = BC + CD Þ AC + CB = DC + CE Þ AB = DE
On putting these values in Eq. (i), we get
9. (c) If B lies on AC, then
AB + BC = BC + CD
On subtracting BC from both sides, we get A B C
AB + BC - BC = BC + CD - BC AC = AB + BC
Þ AB = CD [by axiom 3] Þ 4x - 5 = x + 3 + 2 x
5. (a) Given, Þ 4x - 5 = 3x + 3
AB = BC … (i) Þ4 x - 5 + 5 + 3 x + 3 + 5 [adding 5 on both sides]
and BX = BY … (ii) Þ 4x = 3x + 8
On subtracting Eq. (ii) from Eq. (i), we get Þ 4x - 3x = 3x + 8 - 3x [subtracting 3x from both sides]
AB - BX = BC - BY Þx= 8
Þ AX = CY Hence, the value of x is 8.
432 Study Package for NTSE

Chapter

8
Lines and Angles
Point intersecting lines. When two lines intersect vertically,
then opposite angles are equal.
A fine dot on paper or a location on plane is called a
∠ 1 = ∠ 2 and ∠3 = ∠ 4
point. Point has no length, breadth or thickness. Point
is denoted by capital letters such as A, B or C etc. ● Line segment Line segment is a part of line. It has
two end points and a definite length.
Plane ● Ray If a line segment is extended to unlimited length
A sheet of paper or surface of table has a flat surface. on one of the end point, then we name it by a ray.
If we extend it in all directions to unlimited length, (i) A line contains infinite points.
then the extended flat surface is called a plane. (ii) Infinite lines can pass through a point.
(iii) Two distinct lines in a plane cannot have more than
Line one point common.

The basic idea of line is straightness. It has no breadth


or thickness. It can be extended indefinitely on both
Angle
directions. Line is denoted by small letters as l, m, n etc. Two rays having a common point, form an angle.
● Parallel lines Lines in the same plane that never e. g., ∠AOB
B
intersect and are always at equal distance from each
other are called parallel lines.
(i) l1 and l2 are parallel lines
(ii) line XY is called transversal. O
(iii) ∠ 1 = ∠ 7, ∠ 2 = ∠ 8, ∠ 3 = ∠ 5, ∠ 4 = ∠ 6, these A
angles are called alternate angles.
(iv) ∠ 1 = ∠ 5, ∠ 2 = ∠ 6, ∠ 3 = ∠ 7, ∠ 4 = ∠ 8, these Types of Angles
angles are called corresponding angles. ● Acute angle An angle which is less than 90° is called
X an acute angle.
2 1 ● Right angle An angle of 90° is called right angle.
I1
3 4 ● Obtuse angle An angle which is lying between 90°
6 5 and 180° is called obtuse angle.
I2
7 8 ● Straight angle An angle of 180° is called a straight
Y angle.
● Reflex angle An angle which is lying between 180°
● Intersecting lines When two and 360° is called a reflex angle.
1
lines or line segments cross each other
3 4 ● Complete angle An angle of 360° is called complete
at a single point, then they are called
2 angle.
Lines and Angles 433
● Complementary angles Two angles whose sum is 90° Example 2 In the figure shown, if PQ || RS, then find ∠QOS.
are called complementary angles.
P
● Supplementary angles Two angles whose sum is Q
35°
180° are called supplementary angles.
O
● Adjacent angles Two angles are said to be adjacent, if
they have a common arm and same vertex ∠COB and 75°
∠BOA are adjacent angles.
R S
C
B (a) 120° (b) 110° (c) 100° (d) 90°
Solution (b) Let us draw TU || PQ || RS.
O A Now, ∠PQO = ∠QOU = 35°
P
● Linear pair of angles Two adjacent angles form a Q
35°
linear pair of angle, if their non-common arms are two
opposite rays. Sum of angles of linear pair is always 180°. T O U

Angle Bisector 75°

A line which cuts an angle into two equal angles is R S


called an angle bisector. An angle bisector can be
internal or external. and ∠RSO = ∠SOU = 75° (alternate angles)
Thus, ∠QOS = ∠QOU + ∠SOU = 35° + 75° = 110 °
Internal Angle Bisector
Example 3 In the figure shown, if AF || CE , then find ∠FOP.
Here, two angles are formed ∠AOB and ∠BOC. Both
angles are equal (θ) because OB is the internal bisector. B

C A F
B (2x + 5) O
θ
θ (3x – 7)
O A
C P E
External Angle Bisector
D
Here, ∠A ′ OB and ∠BOC are equal and external
bisector is OB.
(a) 150° (b) 145° (c) 151° (d) 125°
A
Solution (c) We know, ∠AOP = ∠EPO (alternate angles)

O i.e., ( 2x + 5)° = (3 x − 7 )° ⇒ x = 12
θ C Thus, ∠AOP = 2x + 5 = 24 + 5 = 29
A′ θ
∴ ∠FOP = 180 ° − ∠AOP = 180 − 29 = 151°
B
Example 4 If ∠AOC = 3 x − 70 and ∠COB = 3 x + 40, what
Example 1 In the given figure, if AD ⊥ BC , then find y − x . value of x will make A, O and B, the collinear points?
A B
(a) 45° (b) 40° (c) 36° (d) 35°
y
5x x Solution (d) (3 x − 70 ) + (3 x + 40 ) = 180 °, i.e., ∠AOC and ∠BOC
E O F will form a linear pair for A , O and B to be in a straight line
i.e., 6 x − 30 = 180 °
D
C C
(a) 75° (b) 65° (c) 60° (d) 45°
(3x – 70)°
Solution (a) Since, BC ⊥ AD, we get y = 90° and ∠BOD = 90 ° (3x + 40)°
∴ x + 5x = 90 ° A O B
(Q ∠AOE = ∠FOD = 5x)
⇒ 6 x = 90 ° ⇒ x = 15° ⇒ 6 x = 210 °
Thus, y − x = 90 ° − 15 = 75° ⇒ x = 35°
Target Exercise
Elementary Level Questions
1. How many least number of distinct points determine a 8. In the figure shown, if l1 ||l2 , then y is
unique line?
(a) One (b) Two (c) Three (d) Infinite y
l1
2. Which one of the following determines a plane? 5x
x
(a) A line and a point on it l2
(b) Two points
(a) 30° (b) 45° (c) 50° (d) 60°
(c) Three non-collinear points
(d) None of the above 9. If an angle is increase 25% more than its
3. Which of the following statements is false? supplementary angle, the angles are
(a) A line segment can be produced to any desired length (a) 80° and 40° (b) 80° and 100°
(c) 25° and 75° (d) None of these
(b) Through a given point, only one straight line can be drawn
(c) Through two given points, it is possible to draw one and only 10. In the figure, if AB||CD and EG and GF are the angle
one straight line bisectors of ∠BEF and ∠CFE, respectively. What is
(d) Two straight lines can intersect in only one point the value of ∠EGF?
4. Number of pairs of vertical angles formed when two
lines intersect is/are E
(a) one pair (b) two pairs A B
40° G
(c) four pairs (d) None of these
F
5. Find the value of x in the figure shown. D C

(a) 60° (b) 90° (c) 120° (d) 150°


(3x + 7)°
(4x – 5)
11. If AB||CD and BC ||DE, what is the value of x?
A B E
45°
(a) 12° (b) 14°
(c) 16° (d) 18° x

6. In the figure given, find the value of a. C D


C (a) 35° (b) 105°
A (c) 145° (d) None of these
a
D 12. If PQ||ST, the value of x is
140°
B a
P Q
40°
E S
F T
°
15

x
(a) 60° (b) 70°
(c) 90° (d) None of these R
7. In the figure shown AB|| PQ, then ∠RSQ is (a) 25° (b) 55° (c) 155° (d) 105°
A 13. If ABCD is a parallelogram, then the value of x is
P A B
6x – 40
60°
B Q 20° R
5x + 15
S
D C
(a) 20° (b) 30°
(a) 55° (b) 135°
(c) 40° (d) 50°
(c) 85° (d) None of these
Lines and Angles 435
14. In the figure shown, the value of x is 15. If AD||BC, then ∠BAC is equal to
A D
35°

90° x
7x x
85°
(a) 10° B C
(b) 20°
(c) 30° (a) 30° (b) 60°
(d) 40° (c) 40° (d) 80°

High Skill Questions


1. In the figure shown, if m ||n and 5 ( ∠5) = 7 ( ∠8), what 4. If x = 2 ( ∠BEC) and AB||CG, what is the value of x?
is the value of ∠2 + ∠7?
A B
1 2 80°
m x°
3 4
F

5 6 C G
n D
8 7 110

(a) 120° (b) 105° E


(c) 150° (d) 180°
(a) 20° (b) 10° (c) 30° (d) 40°
2. In the figure shown, if BE ||DF , what is the value of
∠ABC? 5. If TU ⊥ ST, QP ||ST and SR ||TV , then the value of y
is
B Q P

20° C D 60°
A y
70° R
E
T
F S
35°

(a) 20° (b) 30° U


V
(c) 50° (d) 70°
(a) 10° (b) 20° (c) 30° (d) 115°
3. If in the figure, ∠PQR = 40° and ∠RSU = 70° with
6. If AB||CD||EF and GI ||JL, what is the value of
QT ||SU, what is the value of ∠PRQ?
∠JLE?
Q G
B
120°
A
H
J
R S D
P
K
C I
T
U
F
L
(a) 35°
(b) 70° E
(c) 85°
(a) 30° (b) 60°
(d) None of the above
(c) 120° (d) None of these
436 Study Package for NTSE

7. If AB||CD, ∠BHF = 40° , ∠BCI = 100° and 11. If AB||HG, what is the value of a?
∠CEF = 130°, then find ∠EFH. E
(6a – 5)°
B 40°
A H F
80° A B
C
30° E F
(10a + 5)° D
D H G
C
(a) 15
I (b) 2.5
(a) 50° (c) 7.5
(b) 80° (d) None of the above
(c) 110°
12. If AB||FE and AF = FC, then ∠FCD is equal to
(d) 150°
B
8. If PR ||TS and PU ||RS, then x is equal to
T D
C
P x° U E
40°
75°
70° G
A F
(a) 35° (b) 105°
40°
(c) 135° (d) 155°
Q R S
(a) 30° (b) 45° 13. In the figure, the value of y is
(c) 75° (d) None of these M
y
9. If m ||n, then the value of ∠QSR is 30°
P
m
60° 3a
50° x°
40°
R
S P Q
b
a b (a) 30° (b) 60°
n
Q R (c) 120° (d) None of these

(a) 60° 14. In the figure, the value of y is


(b) 120° T
(c) 160° O
(d) None of the above
90°
10. Which of the following two lines are parallel in the x°
figure shown? B A R

45°
l
115° P
m (a) 20° (b) 30°
n (c) 50° (d) Cannot be determined
65°
o 15. If AB||CD, then y is equal to
75°
p
45°
q
A B
4x° 3y°
(a) l || m 2x°
(b) m|| n D C
(c) n|| o .
(d) None of the above (a) 20° (b) 30° (c) 40° (d) 50°
Hints and Solutions
Elementary Level Questions
1. (b) In order to determine a unique line, atleast two distinct points 10. (b) Since, EG in an angle bisector, we get
are required. ∠BEG = ∠GEF = 40°
2. (c) Three non-collinear points are sufficient enough to determine
a plane. E
A B
3. (b) Since, an infinite number of straight lines can be drawn to 40° G
50°
pass through a given point. Hence, (b) is false statement. F 50°
4. (b) When two lines intersect, then four angles are formed. Thus, D C
there are two pairs of vertical angles.
5. (a) (4 x − 5)° = (3 x + 7 )° (vertically opposite
Also, as AB|| CD
angles)
⇒ ∠BEF + ∠CFE = 180°
⇒ 4 x − 3 x = 7 + 5 ⇒ x = 12 ° ⇒ (40° + 40° ) + ∠CFE = 180°
6. (b) 140° = 2 a (vertically opposite angles) ⇒ ∠CFE = 100°
⇒ a = 70° ∴ ∠EGF = 180° − (40° + 50° ) = 90°

7. (c) Given, AB|| PQ 11. (d) Given, AB|| CD Then, ∠ABC = ∠BCD = 45°(alternate angles)
Then, ∠ABQ = ∠PQR = 60° (corresponding angles) A B E
45°
A
P x°

60° 60° C D
B Q 20° R
Q BC || ED
S ∴ ∠BCD + ∠EDC = 180°
⇒ x = 180° − 45° = 135°
So, ∠PQR + ∠RQS = 180° (linear pairs)
⇒ 60° + ∠RQS = 180° 12. (c) ∠PQR = ∠QRU = 40° (alternate angles)

i.e., ∠RQS = 120° So, ∠SRU = 40° − 15° = 25°


P Q
Now, in ∆RQS, ∠RSQ = 180° − (120° + 20° ) = 40°
40°
[Q sum of all three angles of triangle = 180°] S
T
°
15

8. (a) Given, l1 || l2 x°
U
R
y° Also, as ST || RU
l1
5x° ⇒ x + ∠SRU = 180°
x° ∴ x = 180° − 25° = 155° [by property of alternate angles]
l2
13. (a) As, ABCD is a parallelogram.
y= x (corresponding angles)
A B
Also, 5 x + x = 180° (interior angles of same side)
(6x – 40)°
⇒ 6 x = 180°
∴ x = 30° = y (5x + 15)°
x × (100 + 25) 5x
9. (b) Supplementary angles are x and ⇒ . D C E
100 4
5x ∠BAD + ∠BCD = ∠BCD + ∠BCE = 180°
Then, x+ = 180°
4 ⇒ 6 x − 40 = 5 x + 15
⇒ 4 x + 5 x = 4 × 180° ⇒ x = 55°
⇒ 9 x = 4 × 180° [Q putting the values of ∠BAD and ∠BCE]
⇒ x = 80°, so the angles are 80° and 100°.
438 Study Package for NTSE

14. (a) Given, A D


35°

90° x
7x x
85°
7 x + 90° + x + x = 180° (angles on straight line) B C
9 x = 90° ⇒ x = 10°
Now, in ∆ABC,
15. (b) As, AD|| BC ∠BAC +∠ABC + ∠ACB = 180°
∠DAC = ∠ACB = 35° (alternate angles) ∴ ∠BAC = 180° − (85° + 35° ) = 60°

High Skill Questions


1. (d) Given, 5 (∠5) = 7 (∠8) ...(i) 4. (a) ∠ABD = ∠BDC = 80° (alternate angles)
∠5 + ∠8 = 180° (linear pairs) ∠EDC = 180° − (∠BDC ) = 180° − 80° = 100°
7
⇒ ∠8 + ∠8 = 180° [Q from Eq. (i)] A B
5
180° × 5
⇒ ∠8 = = 75°
80°

12 F
7 7
So, ∠5 = ∠7 = × ∠8 = × 75 = 105° 80°
C
5 5 G
D
and ∠2 = ∠6 = ∠8 = 75° 100° 110°
Thus, ∠2 +∠7 = 75°+105° = 180° x/2

Alternate Method E
Q ∠4 + ∠6 = 180° ⇒ ∠2 + ∠7 = 180°
Also, ∠CDE + ∠DEC = 110°
(angle between two parallel lines are supplementary) x x
i.e., + 100° = 110° [Q ∠DEC = ∠BEC = ]
2. (c) Given, BE || DF 2 2
⇒ x = 20°
∠BCD = ∠FDC = 70° (alternate angles)
B
5. (d) Given, TU ⊥ ST

70° D ⇒ ∠STV = 90° − (∠UTV ) = 90° − 35° = 55°


20°
A C Q P
70°
60°
E y
F Z R W

Now, in ∆ABC, T
S
∠BAC + ∠ABC = ∠BCD
3 5°

⇒ 20°+∠ABC = 70°
U
∴ ∠ABC = 70° − 20° = 50° V

3. (b) Given, QT || SU Q RS || TV

Then, ∠QPR = ∠RSU = 70° (alternate angles) We get, ∠RST = ∠STV = 55° (alternate angles)
Q Now, let us draw ZW passing through R is parallel to QP and ST.

40° So, ∠RQP = 60° = ∠QRZ


and ∠RST = ∠SRZ = 55° (alternate angles)
70° R S Thus, y = ∠QRZ + ∠ ZRS
P
70° = 60° + 55° = 115°
T
U 6. (c) Given, AB||CD,

Now, in ∆PRQ, ∠QPR + ∠PQR + ∠QRP = 180° Then, ∠GHA = ∠HIC = 120° (corresponding angles)
∠HIC + ∠HIK = 180°
⇒ 70° + 40° + ∠ QRP = 180°
120° + ∠HIK = 180°
∴ ∠QRP = 70° ⇒ ∠HIK = 60°
Lines and Angles 439
G 10. (b) For lines m and n, A
B 115° C
120° m
B
A
H
n
J D 65° E
120° D
F
K
C I ∠EDF + ∠EDB = 180° [Q straight line angle]
∠EDB = 180°− 65° = 115°
F
L m|| n as corresponding angles are equals.
E 11. (c) ∠HDC = ∠DCF = 10 a + 5 (alternate angles)
Q GI || JL
10 a + 5 = 40 + 6 a − 5 (exterior angle of ∆ECF = ∠DCF)
∠HIC = ∠JKI = 120° (corresponding angles)
E
Now, ∠JKI = ∠JLE = 120° (Q CD|| EF) (6a – 5)°
40°
7. (c) Given, EF || CD
A B
∴ ∠CEF + ∠ECD = 180° [Q interior angle] C F
⇒ ∠ECD = 180° − (130° ) = 50°
⇒ ∠HBC + ∠BCD = 180° (10a + 5)°
∴ ∠HBC = 100° [Q ∠ BCD = 80° ] H D G
Now, in quadrilateral BHFP, sum of all 4 angles is 360°. ⇒ 4a = 30 ⇒ a = 7.5
B
A H 12. (d) Given, ∠BAF = ∠EFG = 75° (corresponding angles)
80° 80° 40°
°
30 E
So, ∠CAF = 75° − 40° = 35°
P 1
F
30° 130° B
50°
D
C D
C

I E
40°
⇒ ∠EFH = 360° − (80° + 40° + 130° ) = 360° − 250° = 110°
75°
G
8. (d) Given, PU || RS and PR || TS A F
Since, PURS is a parallelogram. Q AF = FC ⇒ ∠FAC = ∠FCA = 35°
T Thus, ∠FCD = (180° − 35° ) = 155° [Q straight line angle]
13. (c) Given, ∠PMQ + ∠MPQ = ∠MQR
P x U x = 30° + 50° = 80° (exterior angle)
M
70° y
30°
40°
Q R S
∴ ∠UPR = ∠USR = 40°
50° x°
Now, ∠TPU + ∠UPR + ∠RPQ = 180° [Q straight line angle] 40°
R
P Q
⇒ x + ∠UPR + 70° = 180°
∴ x = 180° − (70° + 40° ) = 70° Now, y = x + 40° = 80° + 40° = 120° [Q similarly as above]
9. (b) Given, 60° + 3a = 180° [Q straight line angle] 14. (d) We know that, x + y = 180. The value of y cannot be
⇒ a = 40° determined because x is not known.
P T
M O
60° 3a
90°


S y
b
a b
n
Q R 15. (a) We know that,
Also, 3a + a + b = 180° [Q interior angle of same sides] 4 x + 2 x = 180° [Q interior angle of same sides]
⇒ b = 180° − 160° = 20° ⇒ x = 30° and 2 x = 3 y [Q corresponding angles]
∴ ∠QSR = 180° − (a + b ) 2
⇒ y = x = 20°
= 180° − (40° + 20° ) = 120° 3
440 Study Package for NTSE

Chapter

9
Triangles
A figure bounded by three line segments A
● Side opposite to the greatest angle is the longest side.
in a plane is called a triangle. It has ● Sides opposite to equal angles are equal.
three vertices, three sides and three ● The exterior angles of a triangle (at each vertex) is equal
angles. to the sum of the two opposite interior angles.
B C
∠D = ∠A + ∠B
Types of a Triangle
Triangle further divided into following types
Example 1 The angles of a triangle are x , y and z, such
that y − x = z − y = 15 ° , then find the value of y.
Based on Angle (a) 55° (b) 65° (c) 60° (d) 45°
● Acute angled triangle When all three angles of Solution (c) We know that, x + y + z = 180° ...(i)
triangle are acute, then it is an acute angled triangle.
A
● Right angled triangle When one angle of triangle is
90°, then it is called right angled triangle.
x
● Obtuse angled triangle When one angle of the
triangle is obtuse angle, then triangle is called obtuse
angled triangle. y z
B C
Based on sides and y − x = 15° ⇒ x = y − 15°
● Scalene triangle When all three sides of triangle are and z − y = 15° ⇒ z = 15° + y
of different length, then triangle is scalene triangle.
On substituting these in Eq. (i), we get
● Isosceles triangle When two sides of triangle are of ( y − 15° ) + y + (15° + y ) = 180 °
equal length, then triangle is called isosceles. Two
3 y = 180 ° ⇒ y = 60 °
angles of isosceles triangle are also equal.
● Equilateral Triangle It has all three sides of equal Congruent Triangles
length. It has all three angles as 60°.
Two triangles are said to be congruent, if all sides,
Properties of a Triangle angles of one triangle are equal to corresponding sides
● Sum of three angles of a triangle are always 180°.
and angles of another triangle.
● Sum of length of any two sides of a triangle is greater Condition for congruency have been given below
than the length of third side and difference of any two ● Side-Side-Side If all three sides of one triangle are
sides of a triangle is less than the third side. equal to the corresponding sides of another triangle,
then the two triangles are congruent.
A ● Side-Angle-Side If two sides and included angle of
one triangle are equal to the corresponding sides and
included angle of another triangle, then triangles are
B C D
congruent.
Triangles 441
● Angle-Side-Angle If two angles and included side of Results on Similar Triangles
one triangle are equal to two angles and included side of
Theorem 1 If line is drawn parallel to one side of a
another triangle, then triangles are congruent.
triangle intersecting the other two sides, then it
● Right-Hypotenuse-Side If hypotenuse and one side divides these sides in the same ratio.
of a right triangle is equal to hypotenuse and one side of
another right triangle, then triangles are congruent. A

Medians
The line segment joining a vertex of triangle to D E
mid-point of opposite side is called median.
● Median of triangle pass through common points which B C
divides each median in 2 : 1.
● If two medians of a triangle are equal, then triangle is Here, DE|| BC, then
isosceles. AD AE AD AE AB AC
= or = or =
DB EC AB AC BD EC
Altitude
Theorem 2 The ratio of areas of two similar triangles
A line segment from a vertex which is perpendicular to is equal to the ratio of the squares of any two
opposite side of triangle is called altitude. corresponding sides.
Example 2 In the given figure, find the measure of A P
∠B ′ A ′ C ′.
A A′
m

3x
m
6c

6c

2x+20° B C Q R

∆ABC ~ ∆PQR
60° 60°
B C Here,
B′ C′
Area ( ∆ABC) AB2
6 cm 6 cm
BC2 AC2
(a) 60° (b) 50° (c) 40° (d) 65° Then, = = =
Area ( ∆PQR) PQ2 QR2 PR2
Solution (a) In ∆ABC and ∆ A ′ B′C′,
AB = A ′ B′ = 6 cm [given] Theorem 3 The areas of two similar triangles are in
the ratio of the squares of corresponding altitudes.
∠ABC = ∠A ′ B′ C′ = 60 ° [given]
and BC = B′ C′ = 6 cm [given]
Here, ∆ABC ~ ∆PQR
A P
∴ ∆ABC ≅ ∆A ′ B′ C′ [by SAS congruence rule]
Then, ∠BAC = ∠B′ A ′C′
⇒ 3 x = 2x + 20 ° [from figure]
⇒ 3 x − 2x = 20 ° ⇒ x = 20 °
∴ ∠B′ A ′ C′ = 2x + 20 ° = 2 × 20 °+20 ° = 60 ° B D C Q S R

Area ( ∆ABC) AD 2
Similar Triangles Then, =
Area ( ∆PQR) PS2
Two triangles are said to be similar, if they are alike in
shapes only. The corresponding angles of two similar Theorem 4 The areas of two similar triangle are in
triangles are equal but the corresponding sides are the ratio of the squares of the corresponding medians.
A P
only proportional and not equal.
Condition for similarity of two triangles have been
given below.
● The three angles of one triangle are respectively equal to
the three angles of the second triangle. B D C Q S R
● Two sides of one triangle are proportional to two sides of
the other and the included angles are equal. Here, ∆ABC ~ ∆PQR
(i) Ratio of sides = Ratio of heights = Ratio of medians Area ( ∆ABC) AD2
Then, =
= Ratio of angular bisectors Area ( ∆PQR) PS2
(ii) Ratio of area = Ratio of square of corresponding sides
442 Study Package for NTSE

Theorem 5 The areas of two similar triangles are in A


the ratio of squares of the corresponding angle bisector
segments.
A P
O

B C

B X C Q Y R Also, AO = OC = OB = Circumradii
Here, ∆ABC ~ ∆PQR The position of circumcentre is as
Area ( ∆ABC) AX 2 ● Acute triangle is inside the triangle.
So, here =
Area ( ∆PQR) PY 2 ● Obtuse triangle is outside the triangle.
● Right triangle is the mid-point of the hypotenuse.
Theorem 6 If the areas of two similar triangles are
equal, then the triangles are congruent. Incentre
or
Equal and similar triangles are congruent. The point of intersection of the angle bisector of a
triangle is called the incentre.
Theorem 7 Mid-point theorem The line joining
the mid-points of any two sides of a triangle is parallel e.g., In the triangle shown AD, BE and CF are the angle
to the third side and is half of the third side. bisectors of ∠A, ∠B and ∠C, respectively and their point
A of intersection which is 'I' is the incentre of ∆ ABC.
A

P Q
F E
I

B C
B D C
Here, P and Q are mid-point of AB and AC.
1
1 î In ∆ABC, shown ∠BIC = 90 + ∠A,
So, PQ = BC 2
2 1 1
∠AIC = 90 + ∠B and ∠AIB = 90 + ∠C
2 2
Example 3 In a right ∆ABC, right angled at B, BD is
perpendicular on AC, then find the value of BD2 . Centroid
(a) AB × AD (b) AB × CD The point of intersection of medians in a triangle is
(c) AD × CD (d) None of these called the centroid.
Solution (c) We know that, ∠ ADB = ∠CDB e.g., In ∆ ABC, G the centroid, is the point of
∠ BAD = ∠ DBC A intersection of medians AD, BE and CF.
and ∠ABD = ∠DCB A

Thus, ∆ ABD ~ ∆ BCD D


AB BD AD
So, = =
BC CD BD F E
(i) (ii) (iii) G
Using ratios (ii) and (iii), C
B
we get B D C
BD 2 = AD × CD
Centroid divides the medians in 2 : 1 ratio,
Circumcentre i.e., AG : GD = BG : GE
The point of intersection of perpendicular bisectors of = CG : GF
sides of a triangle is called the circumcentre. = 2:1
e.g., O in ∆ABC is the point of intersection of The centroid divides the triangle into 6 triangles of
perpendicular bisector of sides AB, BC and CA. equal area.
Triangles 443
● In a right angled triangle, right angled at B, Example 4 Find the value of x in the figure shown, if BO
AB2 + BC 2 = AC 2 (by Pythagoras theorem) and CO are the bisectors of ∠B and ∠C, respectively.
A A

40°

O
C x
B
B C
● Appolonius theorem In a ∆ABC, if AD is the median,
then AB2 + AC 2 = 2 ( AD 2 + BD 2 ) (a) 100° (b) 90° (c) 110° (d) 95°
A
Solution (c) Since, BO and CO are the bisectors of ∠ B and
∠C, respectively.
1 1
We know that, ∠ BOC = 90 ° + ∠A = 90 ° + × 40 °
2 2
= 90 ° + 20 ° = 110 °
B C Thus, ∠ x = ∠ BOC = 110 °
D

Target Exercise
Elementary Level Questions
1. In the figure shown, the value of x is 5. In the figure shown, ∠XYO : ∠XZO is equal to
A X

O
65°
60°
B C D
(a) 5° (b) 6° (c) 7° (d) 8° Y Z

(a) 2 : 3 (b) 2 : 1
2. In the figure, if ∠ABC = ∠ACB = 48 ° and ∠D = 37 °,
(c) 1 : 2 (d) 1 : 1
then the value of ∠CAD is
A 6. In a ∆PQR, if ∠ R > ∠ Q, then
(a) PQ < PR (b) QR > PR
(c) QR < PR (d) PQ > PR

7. If ratio of the bases of two similar triangles is 2 : 5,


then the ratio of their areas is
(a) 2 : 25 (b) 4 : 5 (c) 4 : 25 (d) 4 : 9
B C D
(a) 10° (b) 11° 8. The length of two sides of a triangle are 15 cm and
(c) 12° (d) 13° 21 cm, then the third side will be
(a) 6 cm
3. If angles of a triangle are in ratio 3 : 1 : 5, the smallest
(b) 36 cm
angle is
(a) 20° (b) 60° (c) more than 6 but less than 36 cm
(c) 80° (d) 100° (d) more than 36 cm

4. If BO and CO are the bisectors of ∠ABC and ∠ACB of 9. In a ∆ABC, D, E and F are the mid-points of sides AB,
∆ABC, then ∠BOC is BC and CA, respectively. The area of ∆DEF is 24 cm2 ,
(a) less than 90° (b) more than 90° then area of ∆ABC is
1 (a) 48 cm 2 (b) 60 cm 2 (c) 72 cm 2 (d) 96 cm 2
(c) ∠ BAC (d) 90°
2
444 Study Package for NTSE

10. If the sides of a triangle are in 4 : 6 : 7 ratio, then the 15. The medians of a right angled triangle which are
triangle is drawn from the vertices of acute angles are 6 cm and
(a) acute angled 8 cm, then the length of the hypotenuse is
(b) obtuse angled
(a) 4 5 cm (b) 3 5 cm (c) 2 5 cm (d) 5 cm
(c) right angled
(d) None of the above 16. In ∆ABC, AB = 6 cm and DE ||BC, such that
1
11. In a right angled triangle ∆ABC, AD is perpendicular AE = AC, then AD is equal to
4
to the hypotenuse BC, then AB2 is equal to
(a) 1.2 cm (b) 1.5 cm (c) 2 cm (d) 4 cm
(a) AD × CD (b) DB × CB
(c) DB (d) None of these 17. If D is the point on the side BC of a ∆ABC, such that
AD bisects ∠BAC and perpendicular to BC, then
12. If G is the centroid of ∆ABC and the median AD is
(a) BD = CD (b) CD > CA (c) BA > BD (d) BD > BA
21 cm, then the value of DG is
(a) 7 cm (b) 14 cm 18. If RB||SC ||TD in the figure given, then the ratio of
(c) 15 cm (d) 18 cm
y : x is
13. ABC is an equilateral triangle. If straight line R
perpendicular to AB, BC and CA thought the points
T
B, C and A, respectively form a ∆PQR, then ratio of
∆PQR and ∆ABC is x S
(a) 2 : 1 (b) 1 : 4 z
y
(c) 3 : 2 (d) 3 : 1

14. ABC is a triangle in which AB = AC. The base BC is B


p
C
q
D
produced to D and ∠ACD = 130°, then ∠A is equal to p q p q
(a) 40° (b) 50° (a) (b) (c) (d)
q p p+ q p+ q
(c) 60° (d) 80°

High Skill Questions


1. In the figure shown, if AD = BD and AE = EC, what is 3. In ∆PQR shown PQ = 24 cm, PR = 30 cm and
y + z in terms of x? QR = 40 cm, if PS is the median, which of the
A following is true for the length of PS?
P
x

y z
B D E C

(a) 90 + x Q S R
(b) 90 − x (a) 23 < PS < 24
(c) 90 − x /2 (b) 21 < PS < 22
(d) 90 + x /2 (c) 20 < PS < 21
2. In the ∆ABC, DE is the perpendicular bisector of AC. (d) 18 < PS < 19
If ∠AED = 70°, what is the measure of ∠BCA? 4. In ∆ABC, AC = 3 cm, AB = 4 cm and AD is the bisector
(a) 80°
of ∠A, then the ratio BD : DC is
(b) 60°
(c) 40° (a) 9 : 16 (b) 16 : 9
(d) None of the above (c) 3 : 4 (d) 4 : 3
Triangles 445
P
5. The three sides of ∆PQR, the lengths of PQ, QR and
RS are 10 cm, 12 cm and x cm, respectively. If the S
U T
perimeter of the ∆PQR is ‘P’, which of the following is
true for P ? V
(a) 23 < P < 44 (b) 24 < P < 44
(c) 24 ≤ P ≤ 44 (d) None of these Q R
6. In ∆ABC shown in the figure, ∠A = 90°. D is a point (a) 20° (b) 30°
on BC such that DB : DC = 1 : 3. If DM and DL are (c) 45° (d) None of these
perpendiculars on AB and AC respectively, then
DM : LC is equal to 11. If O is the incentre of ∆PQR, where 120 < ( ∠QOR)
B < 130. Which of the following is a possible value of
∠P?
(a) 60° (b) 65° (c) 80° (d) 90°
D 12. If in the figure shown, U is the mid-point of SR and
M
PQ = 8 units, what is the length of TQ?
P
S
A L C U
(a) 1 : 3 (b) 1 : 2 (c) 1 : 1 (d) 4 : 1 20

7. ABC is an equilateral triangle in which AC is T Q 6 R


1
produced to R, such that CR = AC, then (a) 12 (b) 4 (c) 6 (d) 8
2
13. In the figure shown, DH : HG = 3 : 1, EG : GF = 1 : 2.
(a) BR 2 = 5 CR 2 (b) BR 2 = 3 CR 2
Find DI : IF.
(c) BR 2 = 7 CR 2 (d) None of these D

8. In the figure shown, AE ⊥ BC, D is the mid-point of


BC, then x is equal to I
A
H

a b
h d E G F
(a) 1 : 1 (b) 1 : 2 (c) 1 : 3 (d) 1 : 4

B E x D C 14. If D, E and F are the mid-points of the sides AB, AC


a and BC, respectively of an equilateral ∆ABC, the
a2 ratio of the perimeter of ∆ABC to that of ∆DEF is
b2 − d 2 −
a2 + b 2 + c 2 − d 2 4 A
(a) (b)
4 a
(h + d ) (c + d − h)
(c) (d) D E
3 2

9. In ∆ABC shown, ∠DAC is the exterior angle. If


AB = AE and AE = EC, how much is ∠ABE? B G C
D (a) 3 : 1 (b) 2 : 1 (c) 1 : 1 (d) 4 : 1
A
108° 15. If in the triangle shown, BD = AD and AE = EC, how
much is ∠DAE?
A

15°
B E C
(a) 18° (b) 36° (c) 48° (d) 72°

10. In ∆PQR, S and T are any points on PQ and PR such 40°


that ∆PST is isosceles with PS = PT the bisector of B D E C
∠R meets ST at U. If we know ∠PQR = 60°, what is (a) 65° (b) 95°
∠TUR ? (c) 70° (d) None of these
Hints and Solutions
Elementary Level Questions
1. (a) Given, 65° = ∠ACD is the exterior angle of ∆ABC. 6. (d) Since, ∠R > ∠Q, we get PQ > PR
Q Exterior angle of triangle is equal to the sum of two opposite (side opposite to the larger angle is longest)
interior angle P
So, ∠ ACD = ∠CAB + ∠ABC
A

65° Q R
60°
B C D 7. (c) We know that ratio of the area of two smaller triangles is equal
i.e., 65° = x + 60° ⇒ x = 5° to the ratio of the square of their corresponding sides.
2
2. (b) We have, ∠ABC = ∠ACB = 48° and ∠D = 37 ° So, ratio of area =   =
2 4
A  5 25
8. (c) We know that, subtraction of two sides < third side
< addition of two sides
So, 21−15 < Third side < 21 + 15
48° 48° 37°
i.e., 6 < Third side <36
B C D
∠ACB + ∠ACD = 180° (linear pairs)
9. (d) We know that, the line joining the mid-points of the side of a
triangle is parallel to the third side triangle is half of it. So, the
i.e., 48° + ∠ACD = 180°
figure is divided into 4 equal area triangles, if the area of
⇒ ∠ACD = 132 °
∆DEF = 24 cm2 , we get
Now, in ∆ACD ∠ DAC + ∠ ACD + ADC = 180°
A
⇒ ∠DAC + 132 ° + 37 ° = 180°
⇒ ∠DAC = 180° − 169° = 11°
3. (a) Since, angles of triangle are in the ratio 3 : 1 : 5. D F
Let the angles be 3x, x and 5x.
We know that, Sum of the angles of a triangle = 180°
3 x + x + 5 x = 180° ⇒ 9 x = 180° B E C

⇒ x = 20° (smallest angle) Area of ∆ABC = 4 × 24 = 96 cm 2

4. (b) We know that, BO and CO are the angle bisectors, so 10. (a) Let the sides of the triangle are 4 k, 6 k and 7k
1 Now, (7 k )2 = 49 k 2 and (6 k )2 + (4 k )2 = 36 k 2 + 16 k 2 = 52 k 2
∠BOC = 90 + ∠A
2 Since, (7 k )2 < (6 k )2 + (4 k )2
A So, the triangle is an acute angled triangle.
Alternate Method
O Let a, b, c be the sides of the triangle and c being the longest.
A
B C

i.e., more than 90°. a b


5. (d) Since, XY = XZ ⇒ ∠XYZ = ∠XZY …(i)
Also, OY = OZ ⇒ ∠OYZ = ∠OZY …(ii)
On subtracting Eq. (ii) from Eq. (i), we get X
B c C
∠XYZ − ∠OYZ = ∠XZY − ∠OZY If c 2 = a2 + b 2 ⇒ ∠A = 90°
i.e., ∠XYO = ∠XZO O If c 2 > c 2 + b 2 ⇒ ∠A > 90° and c 2 < a2 + b 2
Thus, ∠XYO : ∠XZO = 1 : 1 ⇒ ∠A < 90°
Y Z
Triangles 447
11. (b) We know that, in ∆ADB and ∆ABC, In ∆EBC, (2 x)2 + ( y)2 = 64 [Q by Pythagoras theorem]
∠B = ∠B (common) ⇒ 4 x + y = 64
2 2
…(i)
∠ADB = ∠BAC (90° each) In ∆ABD , (2 y)2 + ( x)2 = 36
and ∠DAB = ∠C [Q by AAA Congruent]
⇒ 4 y2 + x2 = 36 …(ii)
⇒ ∆ADB ~ ∆CAB
On solving Eqs. (i) and (ii), we get
C
64 176
4 x2 = and 4 y2 =
3 3
D
Now, in ∆ABC,
AC 2 = (2 x)2 + (2 y)2 = 4 x2 + 4 y2
64 176 240
A B ⇒ AC 2 = + = = 80
3 3 3
AD DB AB
So, = = ⇒ AB2 = DB × CB ∴ AC = 4 5 cm
CA AB CB
16. (b) Since, DE || BC ⇒ ∆ADE ~ ∆ABC
12. (a) We know centroid divides the medians in 2 : 1 ratio, so if
1 1 A
AD = 21, we get DG = × AD = × 21 = 7 cm
3 3
A
6D E

G B C
AD AE
= ⇒
AD 1
=  using AE = 1 
So,  
AB AC 6 4  AC 4 
B D C
∴ AD = 1.5 cm
13. (b) Since, it is an equilateral triangle P, Q and R will be the
mid-points of the respective sides, so arc of ∆ABC will be 4 times 17. (c) We know that,
the area of ∆PQR. Hypotenuse > (Base + Perpendicular)
A A

P Q

B R C
B D C
∴ Required ratio = 1 : 4
In ∆ABC, AB > (BD + DA)
14. (d) Given, A
i.e., AB > BD
18. (d) Given, RB|| SC
Then, ∆DSC ~ ∆DRB
1 1 130°
R
B C D T
We know, AB = AC
x S
⇒ ∠ABC = ∠ACB = ∠1 (say) z
Now, ∠1 + 130° = 180° (linear pair) y
⇒ ∠1 = 50°
Thus, ∠A = 180° − 2 ∠1 = 180° − 2 × 50° = 80° B C D
p q
15. (a) A DC SC q y
= ⇒ = …(i)
DB RB p+ q x
y
Also, SC || TD ⇒ ∆BSC ~ ∆BTD
6c

BC SC p y
m

E = ⇒ = …(ii)
BD TD p+ q z
8c
y m q
Thus, the ratio of y/x = [using Eq. (i)]
p+ q
B x D x C
448 Study Package for NTSE

High Skill Questions


1. (c) As AD = BD ⇒ ∠DBA = ∠DAB = y A 6. (a) We know that, DB : DC = 1 : 3 B
and AE = EC ⇒ ∠EAC = ∠ECA = z x Since, ∠BMD = ∠BAC = 90° 1
Thus, in ∆ABC, ∠A + ∠B + ∠C = 180° MD|| AC, ∆BMD ~ ∆BAC
⇒ ( y + x + z) + y + z = 180° D
BM MD BD 1 M
⇒ 2 ( y + z) = 180° − x ⇒ = = = ...(i)
y z BA AC BC 4 3
⇒ y + z = 90° − x / 2 B D E C
Using Eq. (i), we know 90°
2. (b) Since, DE is a perpendicular bisector of MD MD
AC, we get AD = DC = A L C
AC A L + LC
B
In ∆AED, MD 1
∠AED + ∠EDA + ∠EAD = 180° ⇒ = (using MD = AL)
100° E MD + LC 4
⇒ 70° + 90° + ∠EAD = 180°
⇒ ∠EAD = 20° 70° ⇒ 4MD = MD + LC
Now, in ∆ABC ⇒ 3 MD = LC
∠A + ∠B + ∠C = 180° A ⇒ MD : LC = 1 : 3
C D
⇒ 20° + 100° + ∠C = 180°
⇒ ∠C = 60° = ∠BCA 7. (b) Since, CR is the half of AC, we can place it on AC on the
∆ABC, so CR = CR ′ joining
3. (d) Since, PS is a median it bisects the base, QS = SR
3 A
P BR′ = Median of ∆ABC = side
2
Side R'
24 30 and CR =
2
In ∆ BR ′ R, BR 2 = (BR ′ ) 2 + (R ′ R )2
2
 3a  B C
Q S R =  + ( a)
2
R
40  2 

We know that, PQ 2 + PR 2 = 2 (PS 2 + QS 2 ) 3a2 7 a2


= + a2 =
(by Appolonius theorem) 4 4
⇒ 242 + 302 = 2(PS 2 + 202 ) a2
7 ×4
⇒ PS 2 = 338 = PS = 338 ⇒18 < PS < 19 ∴ BR 2 = 4
4
2
4. (d) Since, AD is an angle bisector using angle bisect theorem,
= 7   = 7 CR 2
a
we get 2
A [Q on squaring both sides]
8. (b) In ∆AED, d 2 = x2 + h2 …(i)
4 cm [Q by Pythagoras theorem]
3 cm 2
and in ∆AEC, b =  x +  + h2
2 a
…(ii)
 2
B D C From Eqs. (i) and (ii), we get
AB BD 4 a2
= = b 2 = x2 + + ax + (d 2 − x2 )
AC DC 3 4
BD 4 A
Thus, =
DC 3
5. (b) We know that, third side of a triangle is greater than the a b
subtraction of the other two sides and less than, the addition of h d
the other two sides. P
i.e., 12 − 10 < x < 10 + 12
i.e., 2 < x < 22 B E x D C
a/2 a/2
Since, x lies between 2 and 22. 10 x
The perimeter (P) would lie between ⇒ ax = b 2 − d 2 − a2 / 4
10 + 12 + 2 < P < 10 + 12 + 22 b 2 − d 2 − a2 / 4
∴ x=
i.e., 24 < P < 44 a
Q 12 R
Triangles 449
9. (d) Given, AB = AE UQ RU RQ
= =
ST RS RT
Then, ∠ABE = ∠AEB = α (let)
P
1 6
D ⇒ =
2 RT S
A ⇒ RT = 12 U 10
108° 8
(using RU = US)
20
α/2 So, RT − RQ = TQ
⇒ 12 − 6 = 6
T Q 6 R
α α α/2 13. (a) Let us drawn GJ || EI.
DH DI 3
B E C Q HI || GJ , = =
HG IJ 1
AE = EC Let DI = 3y and IJ = y
α Now, in ∆EIF, GJ || EI
⇒ ∠EAC = ∠ECA = (Q α = ∠EAC + ∠ECA)
2 D
α 3α EG FJ 2
= =
We know that, 108° = + α ⇒ = 108° 3y
2 2 GF JI 1 I
⇒ α = 72 ° = ∠ABE
3
Let FJ = 2 z and JI = z y=z
H J
10. (b) ∠QPR = 180° − (60 + 2 α) = 120° − 2α …(i) We have, IJ = z = y. 1 2z
P Thus, y = z
So, DI : IF = 3 : 3 = 1 : 1 E 1 G 2 F
S
U T 14. (b) Let each side of equilateral triangle be x m. Since, the side,
V DE, EF and DF are the side formed by joining the mid-points of
α the equilateral triangle, so ∆DEF as also equilateral triangle will
x
60°
α have side half of the sides of ∆ABC, i.e., each
2
Q R A
and ∠PTS = ∠PST (Q SP = TP, given)
Q ∠PTS = ∠TUR + α (∠PTS is an exterior angle of ∆UTR)
D E
⇒ ∠QPR = 180° − 2 ( ∠TUR + α ) …(ii)
From Eqs. (i) and (ii), we get
⇒ 120° − 2α = 180° − 2 ∠TUR − 2α B F C

⇒ 2 ∠TUR = 180°−120° Thus, the ratio of perimeter of ∆ABC : Perimeter of ∆DEF


⇒ ∠TUR = 30° 3x
= 3x : ⇒2 :1
2
11. (b) If O is the incircle of ∆PQR, we get
∠QOR = 90° +
1
∠P 15. (c) Given, BD = AD
2 ⇒ ∠DBA = ∠DAB = 15°
P
Since, A
120° < (∠QOR ) < 130°
i.e., 15°
1
120° < 90° + ∠P < 130°
2 O

1
120° − 90° < ∠P < 130° − 90° Q R
40°
2 B D E C
∠P
⇒ 30° < < 40°
2 Also, AE = EC
⇒ 60° < ∠P < 80° ⇒ ∠EAC = ∠ECA = 40°
By options, the possible value of ∠P = 65° Q ∠A + ∠B + ∠C = 180°
⇒ (15° + ∠DAE + 40° )
12. (c) QR = 6 and PQ = 8 and ∠Q = 90°
+ 15° + 40° = 180°
⇒ PR = PQ 2 + QR 2 = 10 (using Pythagoras theorem) [Q putting the value of angles A, Band C]
∆RUQ ~ ∆RST ⇒ ∠DAE + 110° = 180° ⇒ ∠DAE = 70°
(Q ∠R = ∠R common and ∠TQR = ∠STR = 90° )
450 Study Package for NTSE

Chapter

10
Quadrilaterals
Quadrilateral
Rectangle
A four sided closed figure is called a quadrilateral.
Sum of the interior angles of a quadrilateral is 360°. A rectangle is a parallelogram in which each angle is
90° and diagonals are equal.
Various Types of Quadrilaterals In the above figure, ABCD is a rectangle in which
AB||DC A B
Parallelogram and AB = DC
A quadrilateral in which opposite sides are parallel Also, BC||AD
and equal is called a parallelogram. and BC = AD
A D C
B Also, ∠A = ∠B = ∠C
= ∠D = 90°

Square
C
A parallelogram in which all sides are equal and each
D E
angle is 90°.
In the above figure, ABCD is a parallelogram. In which In the above figure, ABCD is a square, in which
AB|| CD and BC||AD. AB||DC, BC||AD A B
● Adjacent angles are supplementary. and AB = BC
● Opposite sides and opposite angles are equal. = CD = DA
● Diagonals bisect each other and each diagonal divide Also, ∠A = ∠B = ∠C
D
the triangle into two congruent triangles. = ∠D = 90° C

Rhombus Trapezium
A parallelogram in which all sides are equal and A quadrilateral in which one pair of opposite sides are
diagonals bisect each other at 90°. parallel is called a trapezium. If the non-parallel sides
A of a trapezium are equal, the trapezium is called an
isosceles trapezium.
B D A B
O

C
In the above figure, ABCD is a rhombus in which
AB = BC = CD = DA and AB || DC, BC|| AD C
D E
Quadrilaterals 451
Example 1 In the figure shown, if ABCD is a square, then C
find the value of y.
S R
D C
120° D B
y
O P Q
E
A
A B ● The quadrilateral formed by joining the mid-points of
the sides of a square, is also a square. The figure formed
(a) 75° (b) 85° (c) 90° (d) 65° by joining the mid-points of the pairs of consecutive
Solution (a) We know that, sides of a quadrilateral is a parallelogram.
∠DOC + ∠COE = 180 ° Here, ABCD is a quadrilateral while, EFGH is a
parallelogram.
⇒ ∠COE = 180 ° − 120 ° = 60 ° G
D C
As diagonals of a square bisect the angles ∠OCE = 45°
Now, in ∆COE, ∠OCE + ∠COE + ∠CEO = 180 °
H F
i.e., 45° + 60 ° + y = 180 °
⇒ y = 180 ° − 105° = 75°
A E B
Example 2 If the sides of a rhombus are 5 cm each and its
one diagonal is 6 cm, then find the length of the other diagonal. ● Two parallelograms on the same base and between
(a) 8 cm (b) 5 cm (c) 6 cm (d) 4 cm same parallel lines have equal areas.
● One parallelogram and one rectangle on the same base
Solution (a) Let ABCD be the rhombus in which sides are
and between same parallel lines have equal areas.
5 cm each and diagonal AC = 6 cm. Since, diagonals bisect
each other at 90°. Polygon
A B Any figure in a plane bounded by three or more line
segments is called a polygon. A regular polygon has
O all sides and angles equal. A regular polygon can be
D C inscribed in a circle. They are named according to their
AC number of sides as triangle, quadrilateral, pentagon,
AO = OC = = 3 cm
2 hexagon, for 3, 4, 5, 6, sides, respectively.
Now, in ∆AOD ∠AOD = 90 °
Applying Pythagoras theorem, we get (in ∆ADO)
● Sum of all the angles in a convex polygon is (2n − 4) 90°.
360°
AD 2 = AO 2 + OD 2 ● Exterior angle of a regular polygon is .
n
i.e., 52 = 3 2 + OD 2 ⇒OD = 25 − 9 = 4 360°
● Interior angle of a regular polygon is 180° − ⋅
Thus, the other diagonal, BD = 2 (OD) = 8 cm n
● Number of diagonals of a convex polygon with sides is
Some Facts about Quadrilaterals n( n − 3 )
.
● The quadrilateral formed by joining the mid-points of 2
the consecutive sides of a rectangle is rhombus. Here, Example 3 The ratio of the numbers of sides of two regular
E, F, G, H are mid-points of AB, BC, CD, DA polygons is 1 : 2. If each interior angle of the first polygon is
respectively, then EFGH is a rhombus. 120°, then the measure of each interior angle of the second
D G polygon is
C
(a) 140° (b) 135° (c) 150° (d) 160°
H F Solution (c) Given, interior angle of the first polygon = 120 °
Let number of sides in first polygon be n 1 .
A B n1 − 2
E Then, × 180 ° = 120 ° ⇒ 3 n 1 − 6 = 2n 1 ⇒ n 1 = 6
n1
● The quadrilateral formed by joining the mid-point of the
consecutive sides of a rhombus is a rectangle. Here, ∴ Sides of the second polygon = 2 n 2 = 6 × 2 = 12
PQRS will be a rectangle. 12 − 2
∴ Interior angle of the second polygon = × 180 ° = 150 °
12
452 Study Package for NTSE

Target Exercise
Elementary Level Questions
1. If a pair of opposite angles of a quadrilateral are 10. In the figure shown, if AB = 8 cm , BC = 6 cm and
supplementary, then the quadrilateral is a AD = 7 cm, what is the length of CD?
(a) rhombus (b) rectangle D
A
(c) parallelogram (d) square

2. ABCD is a rhombus. If ∠ACB = 56°, then ∠ADB is


equal to
(a) 32° (b) 34° (c) 124° (d) 134°

3. In a parallelogram ABCD, if AB = 2x + 5, CD = y + 1, B C
AD = y + 5 and BC = 3 x − 4, then ratio of AB : BC is (a) 7 (b) 51 (c) 2 7 (d) 42
(a) 4 : 7 (b) 12 : 11 (c) 31 : 35 (d) 71 : 21
11. ABCD is a rectangle and O is any point inside it, then
4. In a quadrilateral PQRS, side PQ is longest and RS is
which of the following is true?
the shortest , then (a) OB + OD = OA + OC
(a) ∠P < ∠R (b) ∠P > ∠R
(b) AO + OB + AB = CD + OC + OD
(c) ∠P = ∠R (d) ∠P = 2 ∠R
(c) BC + OC + OD = AD + OD + OA
5. If angles P, Q, R and S of the quadrilateral PQRS, (d) OA 2 + OC 2 = OB2 + OD2
taken in order, are in the ratio 3 : 7 : 6 : 4, then PQRS is
a 12. In a quadrilateral PQRS, the sides and diagonals are
(a) rhombus (b) parallelogram related as
(c) trapezuim (d) square (a) PQ + QR + RS + SP = 2(PR + QS )
(b) PQ + QR + RS + SP < PR + QS
6. If diagonals of a quadrilateral , with no angle equal to
(c) PQ + QR + RS + SP > 2(PR + QS )
90° bisect each other at right angles, then the
(d) PQ + QR + RS + SP > (PR + QS )
quadrilateral is a
(a) rhombus (b) trapezuim 13. The external angle of a regular polygon is 72°. Find
(c) rectangle (d) parallelogram the sum of all the internal angles of it.
7. In the figure, ABCD is a parallelogram, then (a) 360° (b) 480° (c) 352° (d) 540°
2∠ABC − ∠ADC is equal to 14. In a rhombus ABCD, if AC = 24 cm and BD = 32 cm,
A B what is the each side of ABCD?
(a) 36 cm (b) 20 cm (c) 80 cm (d) 87 cm

15. The difference between the interior and exterior


65°
angles of a regular polygon is 60°. Then, how many
sides are there in that polygon?
D C E (a) 5 (b) 6 (c) 7 (d) 8
(a) 50° (b) 60° (c) 65° (d) 75° 16. In the figure given below, PQRS is a parallelogram.
8. In a parallelogram ABCD, if the angle bisectors of ∠A PA bisects ∠P and SA bisects ∠S. What is ∠ PAS
and ∠B meet at a point P. Then, ∠APB is equal to equal to?
P Q
(a) 30° (b) 60°
(c) 90° (d) Cannot be determined A
S R
9. In the quadrilateral shown, if BE = BC and
(a) 60° (b) 75° (c) 90° (d) 100°
∠ABE = 60°, how much is ∠EBC?
A B 17. In quadrilateral ABCD shown in the figure
∠DAB = ∠DCX = 120°. If ∠ABC = 105°, then what is
130°
the value of ∠ADC?
X
D C
50°
D E C
120°
A B
(a) 40° (b) 60° (c) 90° (d) 30°
(a) 45° (b) 60° (c) 75° (d) 95°
Quadrilaterals 453
18. In the figure shown, if ABCD is a square and ∆AEB is 19. ABCD is a rectangle. The diagonals AC and BD
equilateral. The measure of ∠DEC is intersect at O. If AB = 32 cm and AD = 24 cm, then
E what is OD equal to?
(a) 22 cm (b) 20 cm (c) 18 cm (d) 16 cm

20. In the figure given below, PQRS is a parallelogram.


If AP, AQ, CR and CS are the bisectors of ∠P, ∠Q, ∠R
A B and ∠S respectively, then ABCD is a
P Q
C
D B

S A
C R
D
(a) 15° (b) 20° (a) square (b) rhombus
(c) 30° (d) 45° (c) rectangle (d) None of these

High Skill Questions


1. In a trapezium ABCD, AB is parallel to CD and the 6. A circle is inscribed in a square PQRS of side x units
diagonals intersect each other at O. What is the ratio each. In the gap remaining at each corner a square of
of OA to OC equal to? minimum possible area is drawn. Find the radius of
(a) Ratio of OB to OD (b) Ratio of BC to CD the circle that can be inscribed in each of the smaller
(c) Ratio of AD to AB (d) Ratio of AC to BD squares.
2. If ABCD is an isosceles trapezium inscribed in a ( 2 − 1) ( 2 − 1)
(a) x (b) x
semi-circle with diameter AD and AB = CD = 2 and 2 2 2
the radius of the semi-circle is 4 cm, what is the ( 2 − 1) ( 2 − 1)
length of BC? (c) x (d) x
B C 4 4 2

7. Two rectangular boards are joined to form a bigger


board. The length of each board is 6 cm. If the ratio of
A O D the length to width of the bigger piece is equal to the
(a) 6 cm (b) 7 cm length to width of each of the smaller pieces, then the
(c) 8 cm (d) None of these width of the bigger piece is
(a) 6 2 cm
3. In the figure given below, ABCD is a trapezium. If EF (b) 8 cm
is parallel to AD and BC. Then, ∠y is equal to (c) 5 cm
A D (d) More than 1 answer possible
x° y°

E θ 120° 8. ABCD is a rhombus. If AB is produced to F and BA is


F
Q z
produced to E such that AB = AE = BF , then
90° 50°
B C (a) ED2 + CF 2 = EF 2 (b) ED|| CF
(a) 30° (b) 45° (c) 60° (d) 65° (c) ED > CF (d) ED ⊥ CF
4. An equilateral ∆TQR is drawn inside a square PQRS. 9. A square is inscribed in a circle of radius ‘a’. Another
The value of ∠PTS is circle inscribed a square in it and another square
(a) 75° (b) 90° (c) 120° (d) 150° inscribed in this circle. The side of the square is
5. Let ABCD be a rectangle and P, Q, R, S be the (a) 2a (b) a / 2
mid-points of sides AB, BC, CD, DA respectively. (c) a / 2 (d) a
Then, the quadrilateral PQRS is a 10. ABCD is a quadrilateral such that BC = BA and
(a) square
CD > AD. Which one of the following is correct?
(b) rectangle but need not be a square
(a) ∠BAD = ∠BCD (b) ∠BAD < ∠BCD
(c) rhombus but need not be a square
(d) parallelogram but need not be a rhombus (c) ∠BAD > ∠BCD (d) 2∠BAD = ∠BCD
454 Study Package for NTSE

Hints and Solutions


Elementary Level Questions
1. (c) Pair of opposite angles of a quadrilateral is supplementary is a 6. (a) Diagonals bisect each other at 90° without each vertex angle
parallelogram. being 90° is in rhombus.
2. (b) ∠ACB = ∠CAD = 56° (alternate angles) 7. (c) We know that,
Now, in ∆AOD, ∠BCE = ∠ABC = 65° (alternate angle)
A and ∠ADC = ∠BCE = 65° (corresponding angle)

A B
56°
B D
O
56°
65°
C D C E
∠OAD + ∠AOD + ∠ODA = 180° So, 2 ∠ABC − ∠ADC = 2 × 65° − 65° = 65 °
i.e., 56° + 90° + ∠ODA = 180°
8. (c) We know that,
⇒ ∠ODA = 34°
A B
3. (c) We know that, AB = CD [Q for parallelogram]
⇒ 2x + 5 = y + 1
⇒ y − 2x = 4 ...(i) P
Also, y + 5 = 3x − 4 [Q AD = BC]
⇒ y − 3x = − 9 ...(ii)
D C
On solving Eqs. (i) and (ii), we get
x = 13 and y = 30 ∠A + ∠B = 180°
AB : BC = 2 x + 5 : 3 x − 4 = 31 : 35 (adjacent angles in a parallelogram are supplementary)
4. (a) By property of quadrilateral, we get 1 1 180°
⇒ ∠A + ∠B = = 90°
∠P < ∠R 2 2 2
S i.e., ∠PAB + ∠PBA = 90°
P Now, in ∆ABP,
∠BAP + ∠ABP + ∠APB = 180°
⇒ 90° + ∠APB = 180°
R ∴ ∠APB = 90°
9. (b) In quadrilateral ABED, we have
Q ∠BAD + ∠ADE + ∠ABE + ∠BED = 360°
5. (c) Let the angles of the quadrilateral PQRS be 3 k, 7 k, 6 k and A B
4k, respectively.
P Q 130° 60°
3k 7k
50°
D E C
4k 6k
R i.e., 130° + 50° + 60° + ∠BED = 360°
S
⇒ ∠BED = 120°
We know that,
Now, ∠BEC = 180° − ∠BED (linear pairs)
3 k + 7 k + 6 k + 4 k = 360°
= 180° − 120° = 60°
⇒ 20 k = 360°
⇒ k = 18° Q BE = BC
We get,
⇒ 3 k = 54°, 7 k = 126°, 6 k = 108° and 4 k = 72 °
∠BEC = ∠BCE = 60°
Since, the 2 pairs of adjacent angles are supplementary, PQRS ⇒ ∠EBC = 180° − (∠BEC + ∠BCE )
is a trapezium with PS||QR. = 180° − 120° = 60°
Quadrilaterals 455
10. (b) In ∆ABC, ∠B = 90° A

Using Pythagoras theorem,


12
AC 2 = AB2 + BC 2 16 16
B D
i.e., AC 2 = 82 + 62 = 100 ⇒ AC = 10 O
Now, in ∆ADC, AC 2 = AD2 + CD2
12

i.e., 102 = 7 2 + CD2 (by Pythagoras theorem) C


⇒ CD = 100 − 49 ⇒ CD =
2
51 cm
⇒ AD = 12 2 + 162
11. (c) Option (c) is true. (diagonals of rhombus bisect each other at 90°)
i.e., BC + OC + OD = AD + OD + OA = 144 + 256
A B = 400 = 20
Thus, the each side of the rhombus = 20 cm
15. (b) Here, (interior angle) − (exterior angle) = 60°
O (n − 2 ) × 180° 360°
⇒ − = 60°
n n
1
D C ⇒ [(n − 2 ) × 180° − 360°] = 60°
BC = AD and OC + OD n
= OD + OA = Diagonals of rectangle 1
⇒ [180° n − 360° − 360°] = 60°
If ‘O’ is the intersection point of diagonals, then ‘BC + OC + OD’ n
1
is always equal to ‘AD + OD + OA’. ⇒ [180° n − 720°] = 60°
n
12. (d) In ∆PQR, PQ + QR > PR ...(i) ⇒ 180° n − 720° = 60° n
and in ∆PSR, PS + SR > PR ...(ii) ⇒ 180° n − 60° n = 720°
P Q ⇒ 120° n = 720°
∴ n=6
Therefore, the polygon contains 6 sides.
16. (c) Since, PQRS is a parallelogram.
∴ PQ||SR
∴ ∠QPS + ∠RSP = 180° ([Q cointerior
S R
angles)
In ∆QSR, QR + SR > QS 1 1 1
...(iii) ⇒ ∠QPS + ∠RSP = × 180°
and in ∆PQS, 2 2 2
PQ + PS > QS ...(iv) ⇒ ∠ APS + ∠ASP = 90° ...(i)
On adding Eqs. (i), (ii), (iii) and (iv), we get (Q PA and SA are the angle bisectors of
∠P and ∠Q respectively)
2 (PQ + QR + SR + SP) > 2(PR + QS )
Now, in ∆APS,
PQ + QR + SR + SP > (PR + QS )
∠APS + ∠ASP + ∠PAS = 180°
13. (d) External angle of any polygon ⇒ 90° + ∠PAS = 180°
360°
= 72 ° ⇒ n = 5 ⇒ ∠PAS = 180° − 90° = 90°
n
∴ Given, polygon is regular pentagon. 17. (c) Given, ∠ABC = 105°
∴ Every interior angle of it ∠DAB = 120°, ∠DCX = 120°
= 180° − external angle = 180° − 72 ° = 108° X
120°
∴ Sum of interior angles of it D
C
= 5 × 108° = 540° 60°
14. (b) Given, AC = 24 cm
AC 120° 105°
Then, AO = OC = = 12 cm and BD = 32 cm
2 A B
⇒ BO = OD =
BD
= 16 cm ⇒ ∠DCB = 180° − 120° = 60°
2 We know that, sum of angles of a quadrilateral is equal to 360°.
In ∆AOD, AD2 = AO 2 + OD2 ∴ ∠ADC = 360° − (120° + 105° + 60° )
[Q by Pythagoras theorem] = 360° − 285°
= 75°
456 Study Package for NTSE

18. (c) We know that, AE = AD 19. (b) We have,


So, ∠AED = ∠ADE AB = 32 cm, AD = 24 cm
E
Since, ∠A = 90°
D C
∴ BD2 = AB2 + AD2
= (32 )2 + (24)2 24 cm
O

A B = 1024 + 576 = 1600


∴ BD = 40 cm A 32 cm B
Since, diagonals of a rectangle bisects
each other.
1 1
D C ∴ OD = BD = × 40 = 20 cm
2 2
180° − (90° + 60° )
∠EAD = = 15° 20. (c) Clearly, AB||DC and AD||BC. Therefore, ABCD is a
2
(Q ∠EAB + ∠DAB = ∠EAD) parallelogram but it is not necessary that AB = BC.
Now, ∠ DEC = 60° − (15° + 15° ) = 30° Thus, ABCD is a rectangle.

High Skill Questions


1. (a) Since, AB|| CD 3. (c) From the figure,
D C A D
x° y°
O θ 120º
E F
Q z°
A B
∴ ∠OAB = ∠OCD [alternate interior angles] …(i) 90º 50º
and ∠OBA = ∠ODC [alternate interior angles] …(ii) B C
Now, in ∆AOB and ∆COD, x° = z° = 50° [alternate interior angles
∠OAB = ∠OCD [from Eq. (i)] and corresponding angles]
∠OBA = ∠ODC [from Eq. (ii)] ∴ θ + z° = 180° [linear pair]
∠AOB = ∠COD
[vertically opposite angle] ⇒ θ = 180° − 50° = 130°
Hence, ∆AOB ~ ∆COD [by AA similarity] Now, in quadrilateral AQFD,

OA OB
= x° + y° + 120°+ θ = 360° ⇒ 50° + y° + 120° + 130° = 360°
OC OD ∴ y = 360° − 300° = 60°
2. (b) Let AE = x 4. (d) In ∆SRT,
B C
P S
∠SRT = 90° − 60° = 30°
2 2 T
1
∴ ∠RTS = (180° − 30° ) = 75°
2 60º
E Similarly, ∠PTQ = 75°
60º 60º
A 4 O F D Q ∠PTS + ∠PTQ + ∠QTR + ∠RTS = 360° Q R
8 ⇒ ∠PTS + 75° + 60° + 75° = 360°
⇒ EO = 4 − x ⇒ ∠PTS = 360° − 210° = 150°
In ∆ABE, BE 2 = A B2 − A E 2 = 4 − x2 ...(i)
5. (c) Given, ABCD is a rectangle.
[Q by Pythagoras theorem] ∴∠A = ∠B = ∠C = ∠D = 90°
In ∆BEO, BE 2 = BO 2 − EO 2
a/2 R a/2
D C
= 42 − (4 − x)2 = 16 − (4 − x)2 ...(ii)
b/2 b/2
From Eqs. (i) and (ii), we get
S Q
4 − x2 = 16 − (4 − x)2
b/2 b/2
⇒ 16 − 4 = (4 − x)2 − ( x)2
A B
⇒ 12 = (4 − x − x) × (4 − x + x) a/2 P a/2
⇒ 12 = 16 − 8 x ⇒ 4 − 2 x = 3 Let AB = CD = a and BC = AD = b
⇒ x = 1/ 2 Also, given P, Q, R and S are mid-points of AB, BC, CD and AD,
∴ BC = AD − 2 x = 8 − 1 = 7 cm respectively.
Quadrilaterals 457
Now, in ∆PBQ, Q ∠DAB + ∠CBA = 180°
PQ 2 = PB2 + QB2 ∴ ∠DAE + ∠CBF = 180°
2 2 2 2 In ∆AED,
PQ 2 =   +   ⇒ PQ =  a +  b 
a b
    ∠EAD + ∠AED + ∠ADE = 180° …(i)
2 2 2 2
In ∆BCF,
Similarly, in ∆QCR, ∆RDS and ∆SAP, ∠CBF + ∠CFB + ∠BCF = 180° …(ii)
2 2
PQ = RQ = SR = SP =  a +  b  On adding Eqs. (i) and (ii), we get
   
2 2 (∠EAD + ∠CBF ) + (∠AED + ∠ADE )
∴Quadrilateral PQRS can be a square or rhombus. + (∠CFB + ∠BCF ) = 360°
But, the diagonals of PQRS are not equal. ⇒ 180° + ∠ 2 AED + 2 ∠CFB = 360°
Hence, PQRS is a rhombus. [Q AE = AD and BC = BF ]
⇒ ∠AED + ∠CFB = 90° ⇒ 180° − ∠EMF = 90°
6. (d) Radius of the bigger circle = x /2
[angle sum property of a triangle]
P Q
⇒ ∠EMF = 90° ⇒ EM ⊥ FM
∴ ED ⊥ CF
9. (d) OA = a and AC = 2 a

S R D C
G H
Let the side of small square = a
Diagonal of smaller square = a 2
x 2 − x x ( 2 − 1) O
= a 2⇒ a =
2 2 2
y
1
Radius of the smaller circle = (length of the side of the F E
I
2 A B
smaller square) x

1  x( 2 − 1) x ( 2 − 1)
=   = Let AB = x, then in ∆ABC,
2  2 2  4 2
x 2 + x 2 = 4 a2 [Q by Pythagoras theorem]
7. (a) We know that, l = 6 cm ⇒ x = a 2, AE = a / 2
l
a2 a
In ∆OEA, OE = OA 2 + AE 2 = a2 − = = OF
b 2 2
2b (by Pythagoras theorem)
FH = 2 × a / 2 = a 2
Now, in ∆FHI, FI = y (let)
Ratio of length to width of the smaller rectangle equal to the Then, (HI) 2 + (FI) 2 = (FH) 2
bigger i.e., y2 + y2 = (a 2 )2 ⇒ 2 y2 = 2 a2 ⇒ y = a
l 2b
= ⇒ l 2 = 2 b2 ⇒ l = 2 b 10. (c) Join AC.
b l A B
(6)2 36
⇒ = b2 ⇒ = b ⇒b = 3 2
2 2
Thus, width of the bigger rectangle = 2 × 3 2 = 6 2 cm D C
8. (d) Produce ED and FC to meet at a point M. Now, in ∆ABC,
M AB = BC ⇒ ∠BAC = ∠BCA ...(i)
[angles opposite to equal sides]
D x C In ∆ADC,
CD > AD
x x ∴ ∠DAC > ∠DCA ...(ii)
E x x x F [since, in a triangle, angle opposite to greater side is greater
A B
than the angle opposite to smaller side]
Let each side of the rhombus be x. Then,
On adding Eqs. (i) and (ii), we get
AB = BC = CD = AD = BF = AE = x
∠BAD > ∠BCD
458 Study Package for NTSE

Chapter

11
Circles
A circle is a set of points which are equidistant from a ● Equal chords substand equal angles at the centre and
given point. The given point is known as the center of the chords which substand equal angle at the centre
circle. are equal in length.
ÐAOB = ÐCOD Û AB = CD
In the given figure ‘O’ is the centre and ‘r’ is radius of
the circle. ● Equal chords of circle are equidistance from the
centre and the chords equidistance from the centre
are equal in length.
AB = CD Û OM = ON
O
r
θ
r Sector
● The region between an arc and the two radii joining
A B
the centre to the end points of the arc is called a sector.

Basic Terms Related to Circle


Major sector
Chord O
We know that, radius ( r) = 1/ 2 (diameter) θ
Minor
● A line segment joining two points on the circumference of sector
A B
the circle is called a chord of the circle. In figure, PQ is a
chord of the circle. ● Sectors are of two types–minor sector and major
sector. The sector corresponding to minor arc is called
minor sector and the sector corresponding to major
arc is called major sector.
● When two arcs are semi-circles, then both segments
P Q and both sectors become same and each is known as a
semi-circular region.
● Perpendicular from the centre to the chord bisects it and ● The angle substanded by an arc at the centre of the
chord bisected by line from the centre is perpendicular on it. circle is twice the angle substanded by the same arc
M
at any other point on the circle.
A B
A

C 2θ
O
O
N
θ
D C
B
AM = MB Û OM ^ AB
Circles 459
● Angles inscribed in the same segment (or same arc) of a C
circle are equal.

D
B

80°
70°

E A F
Þ ÐDCB = 100°
● Angles in a semi-circle is a right angle.
Now, ÐDAE = ÐACD = 70°
● A quadrilateral inscribed in a circle is called a cyclic
quadrilateral and its each pair of opposite angles is 180° (angles are alternate segment)
and vice-versa. As AC bisects ÐDAB
● Exterior angle of a cyclic quadrilateral is equal to the ÐBAC = ÐDAC = 40°
interior opposite angle. In DADC, ÐADC = 180° - ( ÐDAC + ÐACD)
Example 1 In the figure shown, if AB = 12, DC = 16 and = 180° - (40° + 70° )
OM = 8, then what is the value of MN ? = 70°

12 Tangents
A B
M A line which touches the circle at any one point is
8
O called tangent.
N Q
D 16 C

P O
(a) 12 (b) 14
(c) 15 (d) 11
Solution (b) Let us join OB and OC. R

In DBOM, 2
OB = MB + OM 2 2 ● Radius is perpendicular on tangent i.e., OQ ^ PQ and
OR ^ PR.
Þ OB = 62 + 82 éQ MB = AB ù
ëê 2 úû ● Two tangents meeting at a common point outside the
circle arc equal in length i.e., PQ = PR.
= 10 = Radius
● Two circles touching each other externally have
Now, in DONC, OC2 = ON 2 + NC2
distance between the centre equal to the difference of
i.e., ON = 10 2 - 8 2 = 6 their radii and they can have only one tangent is
common.
Thus, OM + ON = MN = 8 + 6 = 14

Example 2 If AC bisects Ð DAB , what is the value of ÐADC?


C

D
B
● Two circles touch each other externally have distance
between their centres equal to the sum of their radii.
80° These circles can have 3 common tangents.
70°

E A F
(a) 65° (b) 75°
(c) 70° (d) 85°
Solution (c) We know that, ● Angle between the tangents and the chord drawn from
Ð DAB + Ð DCB = 180° the point of contact is equal to the angle in the alternate
(cyclic quadrilateral opposite angles are supplementary) segment.
460 Study Package for NTSE

(a) PT 2 = PA ´ PB = PC ´ PD
B C
(b) PT = PA ´ PB = PC ´ PD
(c) PT 3 = PA ´ PB = PC ´ PD
(d) None of the above

A
Solution (a) We know that, PT 2 = PA ´ PB
● If PCD and PAB are two secants, we have and PA ´ PB = PC ´ PD
PC ´ PD = PA ´ PB (tangent and a secant meeting at a common point)
D
Thus, PT 2 = PA ´ PB = PC ´ PD

Example 4 In the figure shown, find the value of PR.


C
P

P A B 45°
D
● Tangent and secant meeting at a common point outside 4
90°
the circle have, PT 2 = PA ´ PB.
R
T E 5 √2 Q

(a) 20 (b) 15
(c) 16 (d) 18
P A B Solution (d) Angles of DRPQ are 90°, 45° and 45° .
Þ PQ = RQ = a
Example 3 Find the relation between PT (a tangent), PAB and PR = a 2
and PCD (secants).
T Also, as RDP and REQ are secants.
B
RD ´ RP = RE ´ RQ
A i.e., 4 ´ a 2 = (a - 5 2 ) ´ a
P Þ 4 2=a-5 2
Þ a=9 2
C
D Thus, PR = ( 9 2 ) 2 = 18

Target Exercise
Elementary Level Questions
1. Number of tangents that can be drawn through a 4. What is the locus of a centre of circle which touch a
point on the circle, is given line at a given point?
(a) 3 (b) 2 (a) A line perpendicular to the given line, passing through the
(c)1 (d) 0 given point
(b) A line parallel to the given line
2. What is the number of circle passing through all the (c) A circle tangent to the given line at the given point
vertices of a given triangle? (d) A closed curved other than a circle
(a) One (b) Two
(c) Three (d) Infinite 5. In the figure shown, find the value of x.

3. What is the number of circle passing through a given


O
pair of points?
x
(a) One
(b) Two R 60° P
(c) More than two but finite Q
(d) Infinite
(a) 200° (b) 220° (c) 230° (d) 240°
Circles 461
6. In the figure shown, O is the centre of the circle and T
ÐAOC = 80°, what is the value of ÐMAO?
B

B A P
C O
M
(a) 16 cm (b) 10 cm (c) 4 5 cm (d) 18 cm

A 12. The length of AC in the figure shown, is


(a) 10° (b) 20° D
(c) 30° (d) None of these

7. In a circle of radius 4 cm with centre at O, AB and CD 6


are two diameters perpendicular to each other. The O
length of chord BD is P
B A
(a) 4 cm (b) 4 2 cm 8 3

(c) 4 3 cm (d) None of these


C
8. In the figure shown, AD is the diameter of the circle, (a) 4 (b) 5
then ÐCBD is (c) 6 (d) Cannot be determined
B C
13. An equilateral DABC is inscribed in a circle with
140°
centre O, then ÐBOC is equal to
A D A
O

O
(a) 45° (b) 50° (c) 55° (d) 60°

9. In the figure shown, ÐADC = 150°, what is the value of B C


ÐCDB ?
D C (a) 30° (b) 60° (c) 90° (d) 120°
150°
14. In the figure shown, O is the centre of the circle,
A B ÐABO = 25° and ÐACO = 30°. Then, ÐBOC is equal to
O
A

(a) 40° (b) 45°


(c) 60° (d) None of these
O
10. Find the value of y + x in the figure shown. B C

A

(a) 85° (b) 125° (c) 110° (d) 27
y 2
B
x 15. In the figure shown, O is the centre of the circle. If
D
O ÐOBC = 37° and ÐBAC = x°, what is the value of x?
A
C

(a) 110° (b) 120° (c) 130° (d) 140° O

11. In the figure shown, PT and PAB are the tangent and
B C
the secant drawn to a circle. If PT = 12 cm and
PB = 8 cm, then AB is equal to
(a) 53° (b) 74° (c) 111° (d) 148°
462 Study Package for NTSE

16. Find the value of ÐPQR, if it is known that PQRS is a 19. In the figure shown, the diameters AB and MN
parallelogram. intersect each other at 50°. Find the value of ÐMPB.
P A Q P

85°
M B
O
S B R 50°
(a) 80° (b) 85° (c) 90° (d) 95° A N
(a) 65°
17. Find the distance between two parallel chords AB
(b) 115°
and CD of lengths 32 units and 24 units, respectively, (c) 125°
which arc drawn in a circle of diameter 40 units. (d) None of the above
(a) 4 (b) 10 (c) 18 (d) 24
20. In the figure shown, if AB is the diameter of the circle,
18. In the figure shown, OC ^ AB , where O is the centre find the value of x.
of the circle. If the chord AB is 12 units long and PC is C
2 units, what is the value of the radius of the circle?
40°

A B
O
P x
A B D E
(a) 40°
C (b) 50°
(a) 5 (b) 10 (c) 60°
(c) 16 (d) 15 (d) 70°

High Skill Questions


1. If O is the centre of the circle and PA and PB are radius in terms of r? Given that the centres of all
tangents, what is the value of ÐAPB? 3 circles lie in the line with the centre of the bigger
A circle.
C

P C 120°
O

A B
B
O
(a) 40° (b) 60° (c) 80° (d) 100°

2. In the given figure, O is the centre of a circle, BOA is 1


(a) r/4 (b) r/3 (c) 2 r/3 (d) r
its diameter and the tangent at the point P meets BA 2
extended at T. If ÐPBO = 30 °, then ÐPTA is equal to
4. In the given figure, O is the centre of a circle, PQL and
P PRM are the tangents at the points Q and R
respectively and S is a point on the circle such that
B
30°
T ÐSQL = 50° and ÐSRM = 60°. Then, ÐQSR is equal
O A to
L
(a) 60° (b) 30° (c) 15° (d) 45° S 50° Q

3. In the figure shown, circle having centre O has radius O


r cm. Two identical circles with diameter AO and OB 60°
are drawn inside the circle. If a third circle centred at M R P
C is drawn, smaller that the other two, what is its (a) 40° (b) 50° (c) 60° (d) 70°
Circles 463
5. In the figure shown, find the value of ÐDEC. 10. If TA is a tangent to the circle at B, what is the value
A of x + y + z?
B T P
40°
E 20° P
x y
C B
D z O

(a) 100° (b) 120° (c) 80° (d) 60° 28°

6. If AB is the diameter of the circle, what is the value of A Q


ÐBED?
C (a) 56° (b) 112°
(c) 28° (d) 65°

11. If two circles of radius r intersect each other in a way


A B
20° E that their centres lie on the circumference of the other,
the length of their common chords will be
50°
(a) 2 r (b) 3/2 r
D (c) 3 r (d) 2/3 r
(a) 20° (b) 40° (c) 60° (d) 80°
12. If O is the centre of the circle as well as the incentre of
7. If O represents centre of the circle and PQ = PO = QR, ÐXYZ, then ÐYXZ is equal to
then find the value of x. X
P

x
O
Q O
Y Z

R
(a) 30° (b) 60°
(a) 20° (b) 25° (c) 90° (d) 120°
(c) 35° (d) Cannot be determined
13. In the figure shown, two identical circles of radius
8. In the figure shown, A and B are the centres of two
12 cm each are drawn with centres C and C ¢ with AB
circles. If P and Q are the points of intersection of the
and BD being their respective tangents at A and D. If
circles , what is the value of ÐPAQ + ÐPBQ?
BE is their common tangent and AB ^ BD, the length
P of BD is equal to

A B
A C E

Q
C'
(a) 120° (b) 60°
(c) 240° (d) None of these
B D
9. If a chord AB of a circle C1 of radius ( 3 + 1) cm
(a) 12 ( 2 - 1)
touches a circle C2 of radius ( 3 - 1) cm, then the
(b) 12 ( 2 + 1)
length of AB is (c) 12 2
(a) 8 3 cm (b) 4 4 3 cm (c) 4 3 cm (d) 2 4 3 cm (d) 24
464 Study Package for NTSE

14. In the figure shown, D and E are mid-points of lines 16. A circle placed against a right angled triangle centred
AB and AC, respectively and ÐABC = 100°. From C, a at O is the 14 cm radius. What is the radius of the
tangent CF is drawn such that ÐBCF is an acute smaller circle placed in the remaining gap?
AD AE
angle. DC bisects ÐACB and = . If the
AB AC
measure of ÐBCF is 30°, how much is ÐEDC?
O
A

D
E
(a) 7 ( 2 + 1) (b) 14 ( 2 - 1) (c) 7 ( 2 - 1)2 (d) 14 ( 2 - 1)2
B
17. Three identical circles of radius r cm each are placed
C inside an equilateral triangle. Find the length of the
(a) 25° (b) 30° side of the equilateral triangle in terms of r.
(c) 40° (d) None of these

15. P and Q are two points on a circle with centre at O. R is


a point on the minor arc of the circle between the
points P and Q. The tangents to the circle from the
point S are drawn which touch the circle at P and Q. If
ÐPSQ = 20°, then ÐPRQ is equal to
(a) 200° (b) 160°
(c) 100° (d) 80° (a) 2r (b) 3r (c) 2 r ( 2 + 1) (d) 2 r ( 3 + 1)

Hints and Solutions


Elementary Level Questions
1. (c) The number of tangents that can be drawn through a point on 6. (a) We know that, BA ^ CM B
the circle is one.
So, ÐCMB = ÐAMO = 90°
2. (a) The number of circles passing through all the vertices of a (vertically opposite angles)
given triangle is one.
Now, in DMAO C O
M 80°
3. (d) There are infinitely many circles passing through a given pair ÐOMA + ÐMAO + ÐAOM = 180°
of points.
\ ÐMAO = 180° - (90° + 80° )
4. (a) A line perpendicular to the given line, passing through the A
= 10°
given points is the required locus.
7. (b) DBOD is a right angled triangle as ÐBOD = 90°
5. (d) Let us take T, a point in the circumference and join RT and TP.
Now, RTPQ is a cyclic quadrilateral, so ÐRTP + ÐRQP = 180° So, DO = BO = 4 (radii)
T A

90°
4 O
O D C

x° 4
R P
60°
B
Q and in DBDO, BD2 = DO 2 + BO 2
Þ ÐRTP = 180° - 60° = 120° [Q by Pythagoras theorem]
So, x = 2 (120° ) = 240° Þ 2
BD = 4 + 4 2

(angle by an arc at the centre of the circle is twice the angle = 4 2 cm


made by the same arc at any other point on the circle.)
Circles 465
8. (b) As AD is the diameter, ÐABD = 90° 13. (d) We know that, ABC is an equilateral triangle.
(angle in a semi-circle is 90°) A

B C
50°
140° O

A D B C
O

So, ÐBAC = 60° [Q equilateral triangle has angles of 60°]


Hence, ÐBOC = 120° [Q by angle at the centre theorem]
So, ÐCBD = 140° - 90° = 50° 14. (c) We know, ÐABO = 25° and ÐOAC = 25°
9. (c) We know, ÐADB = 90° (as angle in a semi-circle is 90°) (Q OA = OB, so opposite angles are also A
equal)
D C
Similarly, ÐACO = 30° = ÐOAC
150° Thus, ÐBAC = ÐBAO + ÐOAC O
= 25° + 30° = 55°
A B Hence, ÐBOC = 2 (ÐBAC ) B C
O
= 2 (55° ) = 110°
15. (a) Given, Ð OBC = Ð OCB = 37° (Q OB = OC = radii)
Þ Ð BOC = 180° - (37° + 37° ) = 106°
Now, ÐCDB = 150° - 90° = 60° A

10. (b) Given, OA = OD = Radii


So, ÐOAD = ÐODA = y A
O
In DAOD, y + y + 60° = 180° y B
37°
Þ 2 y = 120° 60° x B C
O
\ y = 60° D
Also, y = x = 60° So, ÐBAC = x = 1/2 (ÐBOC ) = 1/ 2 (106° ) = 53°
(Q angle in the same arc) C
16. (b) We know, ÐABS = ÐQAB = 85° (alternate angles)
\ x + y = 60° + 60° = 120°
ÐAPS + ÐABS = 180°
11. (b) We know that, PT 2 = PA ´ PB (cyclic quadrilateral has opposite angles supplementary)
(relation between PT tangent and secant PAB) P A Q
Þ ÐAPS = 180° - 85° = 95°
T Now, ÐAPS + ÐPQR = 180° 85°
(as adjacent angles of a
parallelogram are supplementary)
B A P We get, ÐPQR = 85° S B R
17. (a) Given, AB = 32 and OM ^ AB
2 144 AB
12 = PA ´ 8 Þ PA = = 18 Þ AM = MB = = 16
8 2
Thus, AB = PB - PA = 18 - 8 = 10 cm Also, OB = Radius = 20
O
12. (b) Since, DC and AB are secant lines. In DBMO,
We know, A B
M
OM = OB2 - BM 2 = 202 - 162 16
DP ´ PC = BP ´ PA [Q by theorem] D N C
Þ 6 ´ PC = 8 ´ 3 D = 12 [Q by Pythagoras theorem]
Þ PC = 4 Now, in DONC
Also, DC ^ AB 6 NC = 12 , OC = 20 Þ ON = OC 2 - NC 2
In DAPC, AC = 2
AP + PC 2 O
B P
A = 202 - 12 2 = 16
[Q by Pythagoras theorem] 8 3
Thus, the distance between the chords
= 32 + 42
= 25 = 5 C i.e., MN = ON - OM = 16 - 12 = 4
466 Study Package for NTSE

18. (b) Given, OP ^ AB, we get AP = PB = 6 D As, MPBQ is a cyclic quadrilateral, we get
In DOPB, we have PB = 6, OB = r ÐMPB + ÐMQB = 180°
and OP = r - 2 Þ ÐMPB = 180° - 65° = 125°
So, OB2 = OP 2 + PB2 O 20. (b) Let us join AC.
[Q by Pythagoras theorem] As AB is the diameter.
Þ r 2 = 62 + (r - 2 )2 A B
P
C
Þ 36 + 4 - 4 r = 0
\ r = 10 units C
40°
50°
19. (c) Given, ÐMOA = 50° A B
50°
Þ ÐMOB = 180° - 50° = 130° [Q linear angles]
P x
D E
M B
\ ÐACD = 50°
50° ÐACD = ÐABD = 50°
O (angle in the same arc)
\ AB || DE
A N
\ ÐABD = ÐBDE
Q
= 50° (alternate angles)
1 130°
So, ÐMQB = ÐMOB = = 65°
2 2

High Skill Questions


1. (b) Quadrilateral AQBC is cyclic. 3. (b) We know that, radius of the biggest circle = r cm = BO
A
So, radius of circle with centre O¢ and C
O¢¢ is r/2.
P Q
Let the radius of the smallest circle = r ¢
C 120°
O
Now, in DO C O¢ ¢, A B
O' O''
OC = r - r ¢ O
B and OO ¢ ¢ = r /2
So, ÐACB + ÐAQB = 180° and CO ¢ ¢ = r ¢ + r/2
Þ ÐAQB = 180° - 120° = 60° Q CO ^ OO ¢ ¢
So, ÐAOB = 2 (60° ) = 120° We get, (CO ¢ ¢ )2 = (CO )2 + (OO ¢ ¢ )2
Now, in quadrilateral PAOB,
(by Pythagoras theorem)
ÐAPB + ÐPAO + ÐPBO + ÐAOB = 360°
Þ 120° + 90° + 90° + Ð APB = 360° i.e., (r ¢ + r/ 2 ) = (r - r ¢ ) + (r/ 2 )2
2 2

Þ ÐAPB = 60° r2 r2
Þ r¢ 2 + + rr ¢ = r 2 + r ¢ 2 - 2 rr ¢ +
(using OA ^ PA and OB ^ PB ) 4 4
2. (b) Join OP. Now, ÐBPA = 90° [angle in semi-circle] Þ r r¢ = r2 - 2 r r¢
In DPBA, ÐBPA + ÐPBA + ÐBAP = 180° Þ 3r ¢ = r
r
Þ 90° + 30° + ÐBAP = 180° \ r¢ =
3
\ ÐBAP = 60°
But Ð BAT is a straight angle. 4. (d) Since, PQL is a tangent and OQ is a radius, so ÐOQL = 90°
Þ ÐBAP + ÐPAT = 180° Þ ÐOQS = (90° - 50° ) = 40°
Þ 60° + ÐPAT = 180° Now, OQ = OS
Þ ÐPAT = 120° Þ ÐOSQ = ÐOQS = 40°
Now, OA = OP Þ ÐOPA = ÐOAP = 60° Similarly, ÐORS = (90° - 60° ) = 30°
Þ ÐOPT = 90° and OR = OS
Þ ÐOPA + ÐAPT = 90° Þ 60° + ÐAPT = 90° Þ ÐOSR = ÐORS = 30°
\ ÐAPT = 30° Þ ÐQSR = ÐOSQ + ÐOSR
In DPAT, we have = (40° + 30° )
ÐPAT + ÐAPT + ÐPTA = 180° = 70°
\ ÐPTA = 30°
Circles 467
5. (c) ÐACD = Exterior angle of DAPC = ÐCAP + ÐAPC AM 2 = OA 2 - OM 2 [Q by Pythagoras theorem]
= 40° + 20° = 60° = ( 3 + 1)2 - ( 3 - 1)2
A = (3 + 1 + 2 3 ) - (3 + 1 - 2 3 )
B Þ AM 2 = 4 3 Þ AM = 2 4 3
40°
E 20° P \ AB = 2 AM = 44 3
[Q perpendicular from centre to the chord bisects the chord]
C
D 10. (a) Given, x = ÐPQB = 28° (angle in alternate segment)
Also, ÐACD = ÐABE = 60° (angle in the same arc) Also, OB ^ TB
Now, in DABE, We get, ÐPBO = 90° - x = 90° - 28° = 62°
ÐBAE + ÐABE + ÐAEB = 180° Q OB = OP [Q radius of circle]
Þ 40° + 60° + ÐAEB = 180° We get, ÐOBP = ÐOPB = 62°
Thus, ÐAEB = ÐDEC = 80° (vertically opposite angles) Now, in DBPQ,
6. (c) Since, AB is the diameter. ÐADB = 90° T P
C
x y
B
E z O
A B
20° 28°
50°
A Q
D Ð BPQ + Ð PBQ + Ð PQB = 180°
Þ ÐADE = 90° - 50° = 40° Þ (62° + y) + (62° + z) + x = 180°
So, ÐBED = 20° + 40° = 60° (exterior angle of DAED)
\ x + y + z = 180° - 124° = 56°
7. (d) We have, PQ = PO = QR = Radii
11. (c) We know that,
Using PQ = QR, we get ÐQPR = ÐQRP = x
AP = PB = AB (radii)
P
Q DAPB is an equilateral triangle P
\ PD = Altitude of the equilateral triangle
x
3 3
= (Side of triangle) = ´r A B
2 2 D
Q O
3
Similarly, QD = r
2 Q
Thus, PQ = Common chord = PD + DQ = 3r
R
1
Also, but as no angle as given, we cannot determine the value of x. 12. (b) We know that, ÐYOZ = 90° + ÐYXZ ...(i)
2
8. (c) Let the radii of both the circles be r cm each. We know, (using O as the incentre)
PA = PB = AB = Radii of the circle. So, DPAB is an equilateral X
triangle. Similarly, ABQ is also an equilateral triangle.
P
So, ÐPAQ = 60° + 60° = 120° = ÐPBQ
O
Thus, A B
ÐPAQ + Ð PBQ = 120° + 120° = 240°
Y Z
9. (b) Let the chord AB of circle C1 touches
Q
the circle C 2 at point M. But as O is the centre of the circle.
A We know, ÐYOZ = 2 ÐYXZ ...(ii)
Ö3+1
C1 From Eqs. (i) and (ii), we get
1
M O 90° + ÐYXZ = 2 ÐYXZ
Ö3-1 C 2
2 1
Þ 90° = 2 Ð YXZ - Ð YXZ
2
B
Þ 180° = 3 ÐYXZ
Then, OA = 3 + 1and OM = 3 -1
Þ ÐYXZ = 60°
Now, in right angled DOAM,
468 Study Package for NTSE

13. (b) We know, AC = HP = BG = 12 cm 1 1


\ ÐPTQ = ÐPOQ = ´ 160° = 80°
2 2
and C ¢ D = PG = HB = 12 cm
Now, PTQR is a cyclic quadrilateral.
Let AH = x = GD (Q AB = BD = BE)
\ ÐPRQ = 180° - ÐPTQ
= 180° - 80° = 100°

A C E 16. (d) OA = BA = r = 14 Þ OB = OA 2 + AB2 = r 2 = 14 2


Let the radius of the smaller circle = r cm = CE = CD = BD
P C'
H

B G D
O
Now, in DCPC¢,
E
CP = PC ¢ = x C
= x2 + x2 B A
D
2 2
Þ CC ¢ = x + x = x 2
So, BD = CD = r
(by Pythagoras theorem) Þ BC = r 2, CE = r
x 2 = 24 cm Þ x = 12 2 Now, BO = r 2 + r + 14 = 14 2
Thus, AB = AH + HB = 12 2 + 12 Þ r ( 2 + 1) = 14 ( 2 - 1)
= 12( 2 + 1) = BD 14 ( 2 - 1) 2 -1
\ r= ´
14. (a) ÐBCF = ÐBAC = 30° (angle in alternate segment) ( 2 + 1) ( 2 - 1)
A
ÐACB = 180° - (30° + 100° ) = 50° = 14 ( 2 - 1)2
Þ ÐDCB = 25°
17. (d) Radius of each circle = r
AD AE
Also, = D
30°
We want to know the length of AB i.e., side of equilateral triangle
AB AC E
AD AE in terms of r .
i.e., = A
DB EC B 100°
Þ DE || BC 30°
So, ÐEDC = ÐDCB = 25° C 2r
F O
15. (c) Join P and Q with an another point,
say T, on the major arc. O'
Also, join PO and QO. B C
S In DAOD, we know OD ^ AD (radius on tangent)
and Ð DAO = 30° (equilateral triangle)
20°
Þ Ð AOD = ÐODA - ÐDAO
R = 90°-30° = 60°
P Q As we know, in triangle having angles 90° , 60° and 30° have
opposite sides 2 a, a 3 and a, respectively.
O Q OD = Radius = r
Þ AD = r 3 (opposite to 60°)
T Now, OO ' = r + r = 2 r
In quadrilateral POQS, Similarly, in DBO'E , we get
ÐPSQ = 20° EB = r 3
ÐOPS = ÐOQS = 90° Thus, AB = AD + DE + EB
[Q radius is perpendicular to tangent] = r 3 + 2 r + r 3 = 2 r ( 3 + 1)
\ ÐPOQ = 360° - (90° + 90° + 20° ) = 160°
Coordinate Geometry 469

Chapter

12
Coordinate Geometry
The coordinates of a point are written by using the
Cartesian System following conventions
The system used to describe the position of a point in a ● The x-coordinate of a point is its perpendicular
plane is called cartesian system. distance from the Y-axis measured along the X-axis
(positive along the positive direction of the X-axis and
In cartesian system, there are two perpendicular lines
negative along the negative direction of the X-axis).
and a point is located by refering them. The horizontal The x-coordinate is also called the abscissa. The
line XX ¢ is called X-axis and vertical line YY ¢ is called coordinates of a point on the X-axis are of the form
Y-axis. ( x, 0).
Y ● The y-coordinate of a point is its perpendicular
distance from the X-axis measured along the Y-axis
(positive along the positive direction of the Y-axis and
O
X' X negative along the negative direction of the Y-axis).
The y-coordinate is also called the ordinate. The
coordinates of a point on the Y-axis are of the form (0,
Y' y).
The point where these lines intersect each other is ● In stating the coordinates of a point in the coordinate
plane, the x-coordinate comes first and then the
called origin and is denoted by ‘ O’. OX and OY are
y-coordinate. We place the coordinates in brackets.
called positive directions while OX ¢ and OY' are
called negative directions. Sign Conventions of Coordinates in
These axes divide the plane into four parts. Various Quadrants
These four parts are called the quadrants (one to fourth ● If a point is in the Ist quadrant, then the point will be
part) numbered I, II, III and IV anti-clockwise from OX. in the form ( + , + ), since the Ist quadrant is enclosed by
So, the plane consists of these axes and these quadrants the positive X-axis and the positive Y-axis.
is called the cartesian plane or the coordinate plane or ● If a point is in the IInd quadrant, then the point will be
the xy-plane. These axes are called the coordinate axes. in the form (–, + ), since the IInd quadrant is enclosed
Y by the negative X-axis and the positive Y-axis.
● If a point is in the IIIrd quadrant, then the point will
Quadrant II Quadrant I be in the form (–, –), since the IIIrd quadrant is
enclosed by the negative X-axis and the negative
X' X Y-axis.
Quadrant III Quadrant IV
● If a point is in the IVth quadrant, then the point will
be in the form ( + , –), since the IVth quadrant is
enclosed by the positive X-axis and the negative Y-axis
Y'
(see figure).
470 Study Package for NTSE

Y Example 1 A point C divides the line AB, where A(1, 3) and


4 B(2, 7), in the ratio of 3 : 4. The coordinates of C are
II 3 I
æ5 ö æ3 ö æ 10 33 ö
(–, +) 2 (+, +) (a) ç , 5÷ (b) (–2, –9) (c) ç , 5÷ (d) ç , ÷
è3 ø è5 ø è7 7ø
1
X' X Solution (d) Given, m 1 = 3, m 2 = 4 , x1 = 1, x2 = 2, y 1 = 3
–4 –3–2–1 O 1 2 3 4
–1 and y 2 = 7
–2 æ m x + m 2 x1 m 1 y 2 + m 2 y 1 ö
III
–3
IV \ Coordinates of C = ç 1 2 , ÷
(–, –) (+, –) è m1 + m2 m1 + m2 ø
–4
Y' 3 ´ 2 + 4 ´ 1 3 ´ 7 + 4 ´ 3 ö æ 10 33 ö
= æç , ÷ =ç , ÷
The coordinates of origin O are (0, 0) because it has è 3+4 3+4 ø è7 7 ø
zero distance from both the axes, so its abscissa and
ordinate both are zero. Area of a Triangle
The area of the triangle formed by the points ( x1 , y1 ),
Distance Formula
( x2 , y2 ) and ( x 3 , y 3 ) is the numerical value of the
The distance between the points P ( x1 , y1 ) and Q ( x2 , y2 ) expression
is 1
= |[ x1 ( y2 - y 3 ) + x2 ( y 3 - y1 ) + x 3 ( y1 - y2 )]|.
PQ = ( x2 – x1 ) 2 + ( y2 – y1 ) 2 2
Since, distance is always non-negative, we take only ● Area of the triangle cannot be zero. If it comes out to be
the positive square root. zero, the points are collinear.

î The coordinate of the centroid of a triangle having vertices


Section Formula
(x1 , y1), (x2 , y2 ) and (x3 , y3 ) is
● The coordinates of the point P( x, y) which divides the æ x1 + x2 + x3 y1 + y2 + y3 ö
line segment joining the points A ( x1, y1 ) and B ( x2, y2 ) ç , ÷
è 3 3 ø
internally in the ratio m1 : m2 are
æ m1 x2 + m2 x1 m1 y2 + m2 y1 ö Example 2 Find the area of DABC, whose vertices are
ç , ÷
è m1 + m2 m1 + m2 ø A (8, –4), B(3, 6) and C(–2, 4).
● The coordinates of the point P( x, y) which divides the (a) 30 sq units (b) 40 sq units
line segment joining the points A ( x1, y1 ) and B ( x2, y2 ) (c) 50 sq units (d) 60 sq units
externally in the ratio m1 : m2 are Solution (a) Here, A (8, –4), then x1 = 8 , y 1 = -4
æ m1 x2 – m2 x1 m1 y2 – m2 y1 ö B (3, 6), then x2 = 3 , y 2 = 6
ç , ÷
è m1 – m2 m1 – m2 ø C (–2, 4), then x3 = –2, y 3 = 4
1
î The mid-point of a line segment divides the line segment in the \ Area of DABC = {x1 ( y 2 - y 3 ) + x2 ( y 3 - y 1 ) + x3 ( y 1 - y 2 ) }
ratio 1 : 1. Therefore, the coordinates of the mid-point P of the 2
join of the points A(x1, y1) and B (x 2 , y 2 ) is 1
= {8( 6 - 4 ) + 3(4 - ( -4 )) + ( -2)( -4 - 6 ) }
æ 1´ x1 + 1´ x 2 1´ y1 + 1´ y 2 ö 2
ç , ÷ 1
è 1+ 1 1+ 1 ø = [16 + 24 + 20 ]
2
æ x1 + x2 y1 + y2 ö 1
=ç , ÷ = ´ 60 = 30 sq units
è 2 2 ø 2
Target Exercise
Elementary Level Questions
1. The point of intersection of the lines x = 2 and y = 5 is 13. The coordinates of the point which divide the line
(a) (2, 5) (b) (5, 2 ) segment joining A(2, 1) and B (3, 5) internally in 2 : 3
(c) (2.5, 2.5) (d) None of these ratio, is
(b) æç , ö÷
12 13
(a) (6, 13)
2. A point which is 3 units away from the X-axis and è 5 5ø
6 units away from the Y-axis can be (c) (12, 13) (d) (6, 6.5)
(a) (3, 6) (b) (–3 , 6)
(c) (–3 , 3) (d) Both (a) and (b) 14. Which of the following is the coordinate of point C
that divides the line PQ with P(–1, 2) and Q(– 2 , – 1)
3. If a > 0 and b < 0, the point (a, – b) lies in which of the externally in 3 : 2 ratio?
following quadrants? (a) (4, 7) (b) (– 4, 7 ) (c) (– 4, – 7 ) (d) (4, – 7 )
(a) I (b) II (c) III (d) IV
15. A point Y (7, 17) divides the line segment joining the
4. If x < 0, 0 > y, which of the following point is in the points X (3, 5) and Z (1, – 1) externally in which of the
IIIrd quadrant?
following ratio?
(a) (– x, y) (b) ( x, y)
(a) 1 : 3 (b) 2 : 1
(c) (– x,– y) (d) ( x,– y)
(c) 2 : 3 (d) None of these
5. A line drawn perpendicular to X-axis is always
16. What is the area of the triangle formed by joining the
(a) perpendicular to Y-axis
points (0, 0), (2, 4) and (2, 0)?
(b) parallel to Y-axis
(a) 2 sq units (b) 4 sq units (c) 6 sq units (d) 8 sq units
(c) intersects Y-axis but not perpendicular
(d) Cannot form a relation 17. Which of the following is the distance between point
6. The distance of the point (4, 5) to the point (6, 9) is ( x, y) and the origin?
(a) 5 (b) 20 (a) ( x – y) 2 + ( x + y) 2 (b) ( x – 1)2 + ( y – 1)2
(c) 2 5 (d) None of these (c) x 2 + y2 (d) None of these
7. If the distance of the points (2, – 3) and (– 8, – 11) are 18. The coordinate of the centroid of a DABC formed by
equal from the point ( k, – 7). Then, what is the value points A( x1, y1), B ( x2 , y2 ) and C ( x3, y3), is
of k? x1 + x2 + x3 y1 + y2 + y3 ö
(a) 7 (b) – 7 (c) – 3 (d) 3 (a) æç , ÷
è 2 2 ø
+ + + +
(b) æç 1 3ö
8. For what value of m the points (– 3, – 5), (– 5, – 6) and x x2 x y
3, 1 y2 y
÷
( m, – 4) are collinear? è 3 3 ø
(a) 1 (b) 2 æ x1 x 2 x3 y1 y2 y3 ö
(c) ç , ÷
(c) –1 (d) None of these è 3 3 ø
(d) None of the above
9. If the end points of the diameter of a circle are (3, 4)
and (9, 8). Then, the coordinates of its centre will be 19. If the circle having end points of the diameter as
(a) (6, 12 ) (b) (6, 6) (c) (3, 6) (d) (12, 6) (3, – 4) and ( a, b) has its centre at (5, – 1), the value of
10. For what value of ‘ n’ the area of the triangle formed ( a, b) is
with points (1, n), (3, 5) and ( - 1, 2) will be 5 sq units? (a) ( 2, 7 ) (b) ( 7, 2 )
(a) 1 (b) – 1 (c) ( 2, 2 ) (d) None of these
(c) 2 (d) None of these
20. The point nearest to the origin among the points
11. The points (1, 7), (3, 3) and (7, – 5) form a given, is
(a) straight line (b) right triangle (a) (3, 4) (b) (– 1, - 1)
(c) isosceles triangle (d) scalene triangle (c) (0, 3) (d) (4, 0)

12. The area of the triangle formed by the points 21. AOBC is a rectangle whose three vertices are
( k – 1, k + 1) ( k + 3, k – 3) and ( k + 1, k + 2) is A (0, 3), O (0, 0) and B (5, 0). The length of its diagonal is
(a) 2 (b) 3 (a) 5 (b) 3
(c) 3 k (d) None of these (c) 34 (d) 4
472 Study Package for NTSE

22. The distance between the points A (0, 6 ) and B(0, – 2) 24. The distance between the points (0, 5) and (– 5, 0) is
is (a) 5 (b) 5 2
(a) 6 (b) 8 (c) 4 (d) 2 (c) 2 5 (d) 10

23. The distance of the point P (– 6, 8) from the origin is 25. The distance of the point P(2, 3) from the X-axis is
(a) 8 (b) 2 7 (a) 2 (b) 3
(c) 10 (d) 6 (c) 1 (d) 5

High Skill Questions


1. The distance between the points (2, k) and (– 4, 1) is 10. Which of the following points will be collinear with
2 10 units, then the value of k is (0, 1) and (– 2, 5)?
(a) – 1 (b) 1 (a) (2, - 3) (b) (3, 0) (c) (4, 5) (d) (2, 3)
(c) – 3 (d) None of these
11. If the coordinates of the vertices of a DPQR are P (3, 1),
2. If the end points of the diameter of a circle are ( 4, 6) Q (4, 5) and R (13, 9). Then, which of the following will
and (8, 4). Then, the radius of the circle is be the coordinates of the point S, if RS is a median from R
(a) 2 5 units (b) 5 units to PQ?
(c) 10 units (d) 2 20 units (a) (17, 7) (b) (17, 14) (c) (17/2, 7) (d) None of these

3. If the 3 consecutive vertices of a square are 12. For what value of p, the points (1, 1), (2, 3) and (p, 2)
(–1, 4), (– 3, 0) and (1, – 2). Then, which of the will form an isosceles right angled triangle?
following can be its fourth vertex? (a) 0 (b) – 4 (c) 4 (d) 2
(a) (3, 2 ) (b) ( 2, 3 )
(c) (– 3, 2 ) (d) (1, 2 )
13. The mid-point of the diagonal BD of a rhombus is
(3, 4). If the coordinate of point B is (1, 5), the
4. A triangle formed by the points A(2, 5), B (3, 8) and coordinate of point D will be
C ( x, y). If the centroid of the triangle is (3, 5), what is (a) (5, 3) (b) (3, 5)
the coordinate of point C? (c) (5, 1) (d) (3, 1)
(a) (2, 4) (b) (– 4, – 2 )
(c) (4, 2 ) (d) None of these
14. If the area of the equilateral DOAB shown in figure is
9 3 sq units, then what are the coordinates of point A?
5. A point on X-axis which is equidistance from the
A
points (3, 4) and (2, 5) is
(a) (2, 0) (b) (– 2, 0)
(c) (4, 0) (d) None of these
O C B
6. The coordinates of the mid-point of the sides of a
triangle are (3, 6), (4, 5) and (2, 1), then area of the (a) (3, 3 )
triangle is (b) (3, 3 / 2 )
(a) 3 sq units (b) 4 sq units (c) (3, 3 3 )
(c) 8 sq units (d) 12 sq units (d) (2, 3 )
7. Which of the following is the ratio in which the line 15. The quadrilateral formed by joining the points (3, 4),
segment joining the points (3, 7) and (2, 4) is divided (5, 6), (2, 8) and (0, 6) is a
by the X-axis? (a) parallelogram (b) rhombus
(a) 4 : 7 externally (b) 4 : 7 internally (c) square (d) rectangle
(c) 7 : 4 externally (d) None of these
16. The coordinates of the orthocentre of an equilateral
8. The triangle formed by joining the vertices (3, 2) triangle formed by the vertices A ( a1, b1), B ( a2 , b2 )
(5, – 3) and (– 5, 4) will be a and C ( a3, b3) is
a1 – a2 + a3 b1 – b2 + b3 ö
(a) æç
(a) acute triangle (b) obtuse triangle
, ÷
(c) right angled triangle (d) isosceles triangle è 2 2 ø
æ a1 + a2 + a3 b1 + b2 + b3 ö
9. If 3 consecutive vertices of a parallelogram are (5, 6), (b) ç , ÷
è 3 3 ø
(2, 8) and (0, 6). Then, which of the following can be the
fourth vertex? (c) çæ 2 a1a2 a3 2 b1b2 b3 ö
, ÷
è 3 3 ø
(a) (2, 5) (b) (4, 3) (c) (1, 2) (d) (3, 4)
(d) None of the above
Coordinate Geometry 473
17. If the mid-points of the sides of a triangle are (1, 5), A (1, 5)
(2, 6) and (3, 2). Then, the coordinates of the centroid of
the triangle is
(a) æç2, ö÷
13
(b) (3, 6)
è 3ø
O (2, 3)
(c) (6.5, 3) (d) (3.5, 6)

18. If A (3, 4), B( 4, 2) and C (–1, 2) are the vertices of the B D C


mid-points of the sides DE, DF and EF, respectively.
æ3 5ö
The coordinate of the vertex D will be (a) ç ÷ (b) 3 5 (c) 3 / 2 (d) None of these
(a) (6, 2 ) (b) (3, 4/3) è 2 ø
(c) (6, 4/3) (d) None of these 20. For what value of k, the vertices (2, 1), (3, 3) and (5, k)
19. If O is the centroid of the DABC, then what is the form an equilateral triangle?
length of median AD? (a) 4 (b) 2
(c) 3/4 (d) No such value exist

Hints and Solutions


Elementary Level Questions
1. (a) The point of intersection of x = 2 and y = 5 is (2, 5) 6. (c) Distance between (4, 5) and (6, 9) = ( x1 - x2 )2 + ( y1 - y2 )2
y=5 (2, 5)
= (6 – 4)2 + (9 – 5)2 = 2 2 + 42 = 4 + 16 = 2 5

7. (c) We know that, A(2,– 3), B(– 8,– 11) and C (k,– 7 )
O
X-axis AC = BC
x=2 Þ (k – 2 )2 + (–7 + 3)2 = (k + 8)2 + (–7 + 11)2
Y-axis i.e., k 2 – 4 k + 4 + 16 = k 2 + 16 k + 64 + 16
Þ 20 k = – 60
2. (d) (3, 6) and (– 3, 6) both.
\ k=–3
(– 3, 6) (3, 6) 8. (c) Points are collinear, if the area of the triangle formed by them
is zero.
So, for points (– 3,– 5),(– 5,– 6) and (m,– 4), the area of triangle is
X-axis to be zero.
–3 3 1
| x1 ( y2 - y3 ) + x2 ( y3 - y1 ) + x3 ( y1 - y2 )| = 0
2
Y-axis
1
i.e., |– 3 (– 6 + 4) – 5 (– 4 + 5) + m (– 5 + 6)| = 0
2
3. (a) If a > 0 and b < 0 in the point (a,– b) x-coordinate is +ve and i.e., |6 – 5 + m|= 0 Þ m = –1
y-coordinate is -ve, so it is in the Ist quadrant.
9. (b) Centre of a circle is a mid-point of the end points of its
diameter.
4. (c) Given, x < 0 and y < 0, so the point (- x,– y) is in the third
quadrant.
5. (b) A line perpendicular to X-axis is always parallel to Y-axis. (3, 4) (9, 8)

X-axis
x + x2 y1 + y2 ö
So, the coordinates of centre = æç 1 , ÷
è 2 2 ø
Y-axis 3 + 9 4 + 8ö
= æç , ÷ = (6, 6)
è 2 2 ø
474 Study Package for NTSE

10. (a) Area of triangle formed with points (1, n) (3, 5) and (–1, 2 ) is 15. (c) Let the point Y (7, 17 ) divides X (3, 5) and Z (1, – 1) externally in
1 m : n ratio, we get
|1 (5 – 2 ) + 3 (2 – n) – 1 (n – 5)|= 5
2 æ m ´ 1 – n ´ 3 , m ´ (–1) – n(5)ö = (7,17 )
ç ÷
1 è m– n m– n ø
[QArea of triangle = | x1( y2 - y3 ) + x2 ( y3 - y1 ) + x3 ( y1 - y2 )|]
2 On comparing x and y coordinates, we get
i.e., |3 + 6 – 3 n – n + 5|= 10 m– 3 n – m– 5 n
Þ = 7 and = 17
Þ |– 4 n + 14|= 10 Þ – 4 n + 14 = 10 m– n m– n
Þ 4n = 4 Þ m – 3 n = 7m – 7n
\ n=1 and – m – 5 n = 17 m – 17 n
Þ 6 m = 4n
11. (a) The points are A (1, 7 ), B(3, 3) and C(7,– 5).
i.e., m : n = 2 / 3 and 18 m = 12 n
AB = (3 – 1)2 + (3 – 7 )2 [Q Distance = ( x2 - x1 )2 + ( y2 - y1 )2 ] \ m : n = 2/3
= 2 2 + 42 = 20 = 2 5 Thus, m : n = 2 : 3

BC = ( 7 – 3)2 + (- 5 – 3)2 = 42 + 82 16. (b) Area of triangle


1
= | x1 ( y2 - y3 ) + x2 ( y3 - y1 ) + x3 ( y1 - y2 )|
= 16 + 64 = 80 = 4 5 2
and CA = ( 7 – 1)2 + (– 5 – 7 )2 1
= |0(4 – 0) + 2 (0 – 0) + 2(0 – 4)|
2
= 6 2 + 12 2 = 36 + 144 = 180 = 6 5 1
= ´ 8 = 4 sq units
As, AB + BC = CA, the points are collinear. So, the given points 2
form a straight line. 17. (c) Distance of a point ( x, y) from the origin is
12. (d) The points are (k – 1, k + 1), (k + 3, k – 3) and (k + 1, k + 2 ).
( x – 0)2 + ( y – 0)2 = x 2 + y2
1
So, area of triangle = | x1 ( y2 - y3 ) + x2 ( y3 - y1 ) + x3 ( y1 - y2 )|
x + x2 + x3 y1 + y2 + y3 ö
2 18. (b) Centroid of a triangle = æç 1 , ÷
1 è 3 3 ø
= |(k – 1)[(k – 3) – (k + 2 )] + (k + 3)
x + x2 y1 + y2 ö
19. (b) We know that, centre of circle = æç 1
2
, ÷
[( k + 2 ) - (k + 1)] + (k + 1) [(k + 1) – (k – 3)]| è 2 2 ø
1 æ 3 + a – 4 + bö
= |(k – 1)(–5) + (k + 3)(1) + (k + 1)(4)| =ç , ÷ = (5, – 1)
2 è 2 2 ø
1
= |– 5 k + 5 + k + 3 + 4 k + 4|
2
1
= ´ 12 = 6 sq units
2 (3, – 4) (a, b)
(5, –1)
13. (b) The coordinate of point which divides the line segment
joining A(2,1) and B(3, 5) internally in 2 : 3 ratio, is
2 3
On comparing x and y coordinates, we get
A (2, 1) P (x, y) B (3, 5) 3+ a – 4+ b
Þ = 5 and = -1
2 ´ 3 + 3 ´ 2 2 ´ 5 + 3 ´ 1ö 2 2
Coordinates of P( x, y) = æç , ÷ Þ 3 + a = 10 and -4 + b = -2
è 2+3 2+3 ø
æ \ a = 7 and b = 2
m x + m2 x1 m1 y2 + m2 y1 ö
çQ Section formula = 1 2 , ÷ Hence, the value of (a, b) is (7, 2).
è m1 + m2 m1 + m2 ø
20. (b) The distance of a point ( x, y) from the origin is x2 + y2 .
= æç , ö÷
12 13
è 5 5ø So, A (3, 4) = 32 + 42 = 5 unit
14. (c) The required coordinate B(–1,– 1) = 12 + 12 = 2
– 3 ´ 2 - (–1) ´ 2 3 (–1) – 2(2 )ö
= æç , ÷ = 1.414 unit
è 3–2 3–2 ø
C(0, 3) = 02 + 32
æ m x – m2 x1 m1 y2 – m2 y1 ö = 3 unit
çQ Section formula = 1 2 , ÷
è m1 – m2 m1 – m2 ø D,(4, 0) = 42 + 02 = 4 unit
= (– 4,– 7 ) Thus, point (–1,– 1) is the nearest to the origin.
Coordinate Geometry 475
21. (c) We know that, the adjacent of rectangle having angle 90°. 23. (c) Distance between the point P(– 6, 8) and origin O(0, 0)
Y
= ( x2 - x1 )2 + ( y2 - y1 )2
C (5, 3)
A = (0 + 6)2 + (0 – 8)2 = 36 + 64 = 100 = 10
(0, 3)
3 24. (b) Distance between the points (0, 5) and (–5, 0)
= ( x2 - x1 )2 + ( y2 - y1 )2
X' X
O 5 B (5, 0)
(0, 0) = (–5 – 0)2 + (0 – 5)2

Y' = 25 + 25 = 2 ´ 25 = 5 2
Use Pythagoras theorem in DOAB, 25. (b) The distance of the point P(2, 3) from the X-axis = Ordinate of
( AB)2 = (OB)2 + (OA)2 point P = 3
2 2
= (5) + (3) = 25 + 9 = 34 Y

\ AB = 34 P (2, 3)
Y
Which is the required length of diagonal of rectangle.
22. (b) Distance between the points A(0, 6) and B(0,– 2 ) 3 unit
2 2
= ( x2 - x1 ) + ( y2 - y1 ) X
2 2 2 2
= (0 – 0) + (–2 – 6) = (0) + (–8) = 0 + 64 = 8

High Skill Questions


x + x2 + x3 y1 + y2 + y3 ö
1. (a) Let the points be A (2, k ) and B (– 4,1.) 4. (c) Centroid of triangle = æç 1 , ÷
è 3 3 ø
We know that, AB = 2 10
æ 2 + 3+ x 5+ 8+ yö
Þ 2 10 = (4 + 2 )2 + (k – 1)2 (3, 5) = ç , ÷
è 3 3 ø
[Q Distance = ( x2 - x1 )2 + ( y2 - y1 )2 ] On comparing coordinates of x and y, we get
2 5+ x 13 + y
i.e., 40 = 36 + k – 2 k + 1 [Qsquaring both sides] Þ = 3 and =5
3 3
Þ k2 – 2 k – 3 = 0
i.e., x = 4 and y = 2
Þ (k – 3)(k + 1) = 0 Thus, ( x, y) = (4, 2 )
\ k = 3 or k = - 1
5. (b) Let the point on X-axis be P ( x, 0) which is equidistance from
2. (b) Diameter of the circle = Distance between (4, 6) and (8, 4) A(3, 4) and B(2, 5). i.e., PA = PB
= ( x2 - x1 )2 + ( y2 - y1 )2 = (8 - 4)2 + (4 - 6)2 Þ ( x – 3)2 + (4 – 0)2 = ( x – 2 )2 + (5 – 0)2
= 42 + 2 2 = 16 + 4 = 2 5 Þ x – 6 x + 9 + 16 = x2 - 4 x + 4 + 25 Þ 2 x = – 4 Þ x = –2
2

So, radius of the circle = æQ Radius = diameter ö Thus, point = (– 2, 0)


5 units ç ÷
è 2 ø
6. (d) Since, D, E and F are the mid-points of the sides of DABC.
3. (a) Let the points are A(- 1, 4), B(- 3, 0), C(1, - 2 ) and D( x, y). Area of DDEF = 1 / 4 area of DABC
x + x2 y1 + y2 ö æ –1 + 1 4 – 2 ö
The mid-point of AC = æç 1
A
, ÷= ç , ÷ = (0,1)
è 2 2 ø è 2 2 ø
A (–1, 4) B (– 3, 0)
(3, 6) D F (2, 1)

B E (4, 5) C
D (x, y) C (1, – 2)
So, area of DABC = 4 (area of DDEF)
Mid-point of BD = æç
x – 3 y – 0ö
, ÷ 1
\ Area of D DEF = | x1 ( y2 - y3 ) + x2 ( y3 - y1 ) + x3 ( y1 - y2 )|
è 2 2 ø
2
x– 3
Þ =0 1
= | 3 (5 - 1) + 4 (1 - 6) + 2 (6 - 5)|
2 2
y
Þ x = 3 and =1 Þ y=2 1 1
= | 12 - 20 + 2 | = ´ 6 = 3 sq units
2 2 2
Thus, ( x, y) = (3, 2 )
Thus, area of DABC = 4 ´ 3 = 12 sq units
476 Study Package for NTSE

7. (c) We have, A(3,7 ) and B(2, 4), let the point on X-axis be ( x, 0). 12. (c) The coordinates of the triangle are A(1, 1), B(2, 3), C ( p, 2 )
Also, let the division ratio of A and B by X-axis be m : n, we get AB = (2 - 1)2 + (3 - 1)2 = 12 + 2 2 = 5,
æ 2 m + 3 n , 4 m + 7 n ö = ( x, 0)
ç ÷ [Qby section formula] BC = ( p - 2 )2 + (2 - 3)2 = ( p – 2 )2 + 12
è m+ n m+ n ø
4 m + 7n and CA = ( p – 1)2 + (2 - 1)2 = ( p – 1)2 + 12
Here, =0
m+ n Let A B = BC
Þ 4m + 7 n = 0 Þ 5 = ( p - 2 )2 + 1
Þ 4m = - 7 n
Þ 5 = p2 – 4 p + 4 + 1
Þ m : n = –7 / 4
Þ p 2 – 4p = 0 Þ p = 0 or 4
i.e., It divides the points A and B in 7 : 4 ratio externally.
Taking p = 0, we get AB = 5, BC = 5 and CA = 2 which is
8. (c) We have , A (3,2 ), B (5,– 3) and C(– 5, 4)
not right triangle side combination, if we take p = 4, we get
AB = (5 – 3)2 + (–3 – 2 )2 AB= 5, BC = 5 and CA = 10. As, CA 2 = AB 2 + BC 2 and
= 2 2 + 52 = 29 AB = BC
For p = 4 the triangle is an isosceles right angled triangle.
BC = (–5 – 5)2 + (4 + 3)2 = 100 + 49 = 149
13. (a) Let the coordinates of point D be ( x, y). We know O is the
and CA = (- 5 - 3)2 + (4 - 2 )2 = 64 + 4 = 68 mid-point of diagonal BD.
2
BC > AB + CA2 2
So, (3, 4) = æç
1+ x 5 + yö é æ x1 + x2 y1 + y2 ö ù
è 2
,
2 ø
÷ êQ mid -point = çè 2 , 2 ÷ø ú
So, DABC is an right angled triangle. ë û
B (1, 5)
9. (d) Let the fourth vertex be ( x, y). Then,
A (5, 6) B (2, 8)

A C
O
(3, 4)
D (x, y) C (0, 6)
D (x, y)
The mid-point of diagonal AC = Mid-point of diagonal BD 1+ x 5+ y
Þ = 3 and =4
æ 5 + 0, 6 + 6ö = æ x + 2 , y + 8ö 2 2
i.e., ç ÷ ç ÷
è 2 2 ø è 2 2 ø Þ 1 + x = 6 and 5 + y = 8 Þ x = 5, y = 3
é æ x1 + x2 y1 + y2 ö ù Thus, D = (5, 3)
êQ mid -point = çè 2 , 2 ÷ø ú
ë û 3
14. (c) Area of equilateral triangle = (side) 2 = 9 3
Þ x + 2 = 5 and y + 8 = 12 4
\ x = 3 and y = 4
A
10. (a) From option (a), let point P (2, – 3) will be collinear with A (0, 1)
and B(-2, 5). Then,
1
x1( y2 - y3 ) + x2 ( y3 - y1 ) + x3 ( y1 - y2 ) = 0
2
O C B (6, 0)
1
Þ 0(5 + 3) + (-2 )(-3 - 1) + (2 )(1 - 5) = 0
2 3
1 1 Þ (side) 2 = 9 3
Þ 0´ 8+ 8- 8 = 0Þ 0+ 8- 8 = 0 Þ0 = 0 4
2 2 Þ (side) 2 = 36
11. (d) Coordinate of point S, as RS is a median = Mid-point of PQ Þ side = 36 Þ side = 6
x + x2 y1 + y2 ö æ 4 + 3 5 + 1ö æ 7 ö
= æç 1 , ÷= ç , ÷ = ç , 3÷ 3
è 2 2 ø è 2 2 ø è2 ø Length of altitude AC = (Side)
2
P (3, 1) 3
= ´6= 3 3
2
= y-coordinate of point A
S
C is the mid-point of OB.
x + x2 y1 + y2 ö æ 6 + 0 0 + 0 ö
\ Coordinate point ofC = æç 1 , ÷=ç , ÷ = (3, 0)
Q (4, 5) R (13, 9)
è 2 2 ø è 2 2 ø
So, the coordinate of point A = (3, 3 3 )
Coordinate Geometry 477
15. (a) AB = 2 2 + 2 2 = 8 [Q by distance formula] Similarly, æ x2 + x3 , y2 + y3 ö = (–1, 2 )
ç ÷
è 2 2 ø
DC = 2 2 + 2 2 = 8
Þ x2 + x3 = – 2, y2 + y3 = 4 ...(ii)
2 2
AD = 3 + 2 = 13 and x1 + x3 = 8, y1 + y3 = 4 ...(iii)
BC = 32 + 2 2 = 13
A (3, 4) B (5, 6) On adding Eqs. (i), (ii) and (iii), we get
x1 + x2 + x3 = 4 and y1 + y2 + y3 = 16 / 3
Using Eq. (ii) and above, we get
x1 = 4 – (–2 ) = 6 and y1 = 16 / 3 – 4 = 4 / 3
Thus, D( x1, y1 ) = (6, 4/ 3)
D (0, 6) C (2, 8) 19. (a) We know, AO = ( x2 - x1 )2 + ( y2 - y1 )2 = 12 + 2 2 = 5
The mid-point of diagonal, A (1, 5)
x + x2 y1 + y2 ö æ 3 + 2 4 +
AC = æç 1
8ö æ 5 ö
, ÷=ç , ÷ = ç , 6÷
è 2 2 ø è 2 2 ø è2 ø
= Mid-point of diagonal BD
But as AC = 12 + 42 = 17 O (2, 3)
¹ BD = 52 + 02 = 5
It is not a rectangle. B D C
Thus, ABCD is a parallelogram. 2
Þ AO = AD = 2 / 3 ´ AD
16. (b) In an equilateral triangle, orthocentre, incentre, centroid and 3
circumcentre all are at the same point and the point is (Q centroid divides medians in 2:1 ratio)
a + a2 + a3 b1 + b2 + b3 ö
= æç 1 , ÷ \ AD = 3 5 /2
è 3 3 ø
20. (d) Let the points be A (2, 1), B(3, 3) and C (5, k ).
17. (a) Coordinate of the centroid is same even when calculated with
the mid-point coordinates, so centroid \ AB = 12 + 2 2 = 5 [QDistance= ( x1 - x2 )2 + ( y1 - y2 )2
a + a2 + a3 b1 + b2 + b3 ö
= æç 1 , ÷ BC = 2 2 + (k – 3)2 = k 2 – 6 k + 13
è 3 3 ø
1 + 3 + 2 5 + 6 + 2 ö æ 13 ö and CA = 32 + (k – 1)2 = 10 + k 2 – 2 k
= æç , ÷ = ç2, ÷
è 3 3 ø è 3ø We should have AB = BC = CA for triangle to be equilateral.
x1 + x2 y1 + y2 ö Taking AB = BC, we get
18. (c) We know, æç , ÷ = (3, 4) Þ k 2 – 6 k + 13 = 5
è 2 2 ø
D (x1, y1) Þ k2 – 6 k + 8 = 0
Þ k 2 - 2 k - 4k + 8 = 0
Þ k(k - 2 ) - 4(k - 2 ) = 0 Þ (k - 2 )(k - 4) = 0
A (3, 4) B (4, 2) \ k = 4, 2
On taking BC = CA, we get
Þ k 2 – 2 k + 10 = k 2 – 6 k + 13 Þ 4 k = 3
E (x2, y2) C F (x 3 , y 3 ) \ k = 3/ 4
(–1, 2)
Since, there is no common value of k possible. The triangle
Þ x1 + x2 = 6, y1 + y2 = 8 ...(i) cannot be equilateral.
478 Study Package for NTSE

Chapter

13
Area and Perimeter

Area Example 1 The perimeter of an equilateral triangle is


Total space inside the boundary of a plane figure is 45 cm. Find its area.
called the area of that particular figure. 215 3 205 2
(a) sq cm (b) sq cm
In another words, the area of a plane figure is a 2 3
measure associated with the part of plane enclosed in 115 225 3
(c) sq cm (d) sq cm
the figure. Area is measured in square unit. 3 4
Solution (d) Given that, perimeter = 45 cm
Perimeter or 3 a = 45 (Q a = side)
It is the length of border around any enclosed plane. ∴ a=
45
= 15 cm
Therefore, sum of the sides of a plane figure is the 3
perimeter of that particular figure. Unit of perimeter 3 2 3
∴ Area = a = × 15 2
is same as the unit of sides of a given figure. 4 4
225 3
= sq cm
Types of Triangles 4

Isosceles Triangle
Triangle
It has any two sides and two angles equal and altitude
A figure enclosed by three sides is known as a triangle. bisects the base.
A triangle has three angles with total sum of 180° and
Let a = Each of two equal sides and b = Third side, then
sum of its any two sides is more than the third side.
b
Equilateral Triangle l Area = 4a 2 − b 2
4
It has all three sides equal and each angle is of 60°. b 2 1
l Height = a 2 −   = 4 a2 − b2
A  2 2

a l Perimeter = a + a + b = 2 a + b
a
h A

B a C
a a
3 3 2
l Area = (Side)2 = a h
4 4
3 3
l Height = (Side) = a B C
2 2 b/ 2 b/ 2
b
l Perimeter = 3 (Side) = 3 a
Area and Perimeter 479
Example 2 The perimeter of an isosceles triangle is 32 cm 1
Solution (a) Area of triangle = × Base × Height
while its equal sides together measure 18 cm. Find the 2
third side and each of the equal sides. 1
= × 6 × 8 = 24 sq cm
(a) 14 cm, 9 cm (b) 12 cm, 8 cm 2
(c) 18 cm, 6 cm (d) None of these
Quadrilaterals
Solution (a) Let the third side be x cm.
According to the question, x + 18 = 32
A figure enclosed by four sides is called a
quadrilateral. A quadrilateral has four angles and sum
∴ x = 32 − 18 = 14 cm
of these angles is equal to 360°.
18
Each equal side = = 9 cm
2 Parallelogram
Scalene Triangle It is a quadrilateral with opposite sides parallel and
equal. Let a and b be the sides of the parallelogram
It has three unequal sides.
and h be the height, then
Let a, b and c be the sides of the l Area = Base × Height = b × h l Perimeter = 2 ( a + b)
A
triangle, then
A b B
l Area = s ( s − a ) ( s − b) ( s − c )
a+ b+ c a c
Where, s = a h a
2
B C
l Perimeter = a + b + c b
D b C
Example 3 Find the area of a triangle whose sides are
26 cm, 28 cm and 30 cm. Example 5 The base of a parallelogram is twice its
(a) 336 sq cm (b) 106 sq cm height. If the area of the parallelogram is 144 sq cm, find
(c) 416 sq cm (d) 307 sq cm
its height.
Solution (a) Given that, a = 26 cm, b = 28 cm and c = 30 cm (a) 3 2 cm (b) 6 2 cm (c) 2 2 cm (d) 3 3
a + b + c 26 + 28 + 30 84
∴ s= = = = 42 cm Solution (b) As we know,
2 2 2
Area of parallelogram = Base × Height
∴ (s − a ) = 42 − 26 = 16 cm
According to the question, A B
(s − b) = 42 − 28 = 14 cm
2a × a = 144
(s − c) = 42 − 30 = 12 cm (a = Height, 2a = Base)
∴ Area = s (s − a ) (s − b) (s − c) 144 a
2
⇒ a = = 72
= 42 × 16 × 14 × 12 2
= (14 × 3 ) × 16 × 14 × (4 × 3 ) ∴ a = 72 = 6 2 cm D 2a C

= (14 × 4 × 2 × 3 ) = 336 sq cm Rhombus


Right Angled Triangle It is a parallelogram with all four sides equal. The
opposite angles in a rhombus are equal but they are
It is a triangle whose one angle is equal to 90°.
not equal to 90°.
Let p = Perpendicular, b = Base
A Let a = Side, d1 , d2 = diagonals, then
and h = Hypotenuse, then A B
a
1
l Area = × Perpendicular × Base d2
2 h
p a a
1
= × p× b
2 d1
B C
l Perimeter = p + b + h b D a C
Note Here, h 2 = p2 + b2 , which is known as 1
l Area = × d1 × d 2
Pythagoras theorem. 2
1 2 2 2 2
Example 4 The base and height of a right angular l Side (a) = d1 + d2 ⇒ 4a2 = d1 + d2
triangle are 6 cm and 8 cm respectively. What is its area? 2
(a) 24 sq cm (b) 26 sq cm l Perimeter = 4a
(c) 25 sq cm (d) 30 sq cm
480 Study Package for NTSE

Example 6 Find the area of a rhombus one side of which Solution (a) Given, area = 289 sq m
measures 10 cm and one diagonal 12 cm. ⇒ a 2 = 289 (a = side)
(a) 66 sq cm (b) 84 sq cm
∴ a= 289 = 17 cm
(c) 72 sq cm (d) 96 sq cm
∴ Perimeter = 4 a = 4 × 17 = 68 m
Solution (d) Given that, a = 10 cm, d 1 = 12 cm
Trapezium
According to the formula,
4 a 2 = d 21 + d 22 (d 1 and d 2 are diagonals and a is the side)
It is a quadrilateral, where only one pair of opposite
sides is parallel. Let a and b be the parallel sides and
⇒ 4 × 10 2 = 122 + d 22 ⇒ d 22 = 256 h be the height or perpendicular distance between a
∴ d2 =
256 = 16 cm and b, then
1 1
∴ Required area = × d 1 × d 2 Area = ( a + b) × h
2 2
A a B
1
= × 12 × 16 = 12 × 8 = 96 sq cm
2

Rectangle h

It is a parallelogram whose opposite sides are equal


and each angle is equal to 90°. D b C
Let l = Length, b = Breadth and h = Height, then
A l B Example 9 If the two parallel sides of a trapezium are
6 cm and 10 cm, respectively and distance between them
is 4 cm, find its area.
b d b
(a) 32 sq cm (b) 24 sq cm
l
(c) 26 sq cm (d) None of these
D C
Solution (a) Given that, a = 6 cm, b = 10 cm, h = 4 cm
l Area = Length × Breadth = l × b
According to the formula,
l Perimeter = 2 ( l + b) 1 1
Area = (a + b) × h = ( 6 + 10 ) × 4 = 32 sq cm
l Diagonal (d) = l2 + b2 2 2

Regular Polygon
Example 7 The perimeter of a rectangle is 50 cm. If its A polygon is a figure bounded by three or more
length is 15 cm, what will be its breadth? straight lines. If all sides and all the interior angles of
(a) 15 cm (b) 10 cm (c) 20 cm (d) 25 cm a polygon are equal, then it is called regular polygon.
Solution (b) According to the question, A polygon is called pentagon, hexagon, heptagon,
2 ( l + b) = 50 (l = length, b =breadth) octagon, nonagon and decagon according as it contains
⇒ l + b = 25 ⇒15 + b = 25 5, 6, 7, 8, 9 and 10 sides, respectively.
∴ b = 25 − 15 = 10 cm Let a = Each side of a regular polygon, then
3
Square l Area of regular pentagon = 5a 2
4
It is a parallelogram in which all the 3
sides are equal and perpendicular to A a B l Area of regular hexagon = 6a 2
4
each other i.e., each angle is equal to
l Area of regular octagon = 2a2 ( 2 + 1)
90°. a a n ( n − 1) n (n − 3 )
Number of diagonals =  − n  or 
d
Let a = Side, d = Diagonal, then l 
2 2
 2   2 
l Area = (Side) = a D a C
l Perimeter = 4 × Side = 4a Example 10 Find the area of a regular hexagon whose
perimeter is 18 cm (in sq cm).
l Diagonal (d) = a 2
12 27
Example 8 A square field has its area equal to 289 sq m. (a) (b) 9 3
5
Find its perimeter.
27 3 27 3
(a) 68 m (b) 72 m (c) (d)
2 4
(c) 70 m (d) None of these
Area and Perimeter 481
Solution (c) Given, Perimeter of regular hexagon = 6a Circular Ring
18 = 6 a Let R = Radius of bigger ring O
∴ a = 3 cm r = Radius of smaller ring r
R
Now, Area of regular hexagon Then,
2 2
l Area = π ( R − r )
3 3
= 6 a2 = 6 × (3)2 × l Difference in circumference of both the rings
4 4
3 27 3 = (2πR − 2πr )
=3×9× = sq cm
2 2 Example 12 The radius of a circular garden is 21 m. If a
path way of width 7m runs all around and inside the
Circle garden, then find its area.
It is a plane figure enclosed by a curved line on, which (a) 770 m2 (b) 700 m2
every point is equally distant from a fixed point
(c) 707 m2 (d) 777 m2
(centre) inside the curve.
Let radius = r and arc (AB) = l, then O
Solution (a) Required area of path
l Area = πr 2 r θ r = π(21)2 − π(21 − 7)2 [Q area of circle = π (radius)2]
l Circumference (perimeter) = 2 πr A B = π (21)2 − π(14)2
Diameter = 2 r l
= π [(21 + 14) (21 − 14)] [Q a2 − b2 = (a + b) (a − b)]
l
πr θ
l Length of the arc (l) = 22
180° = π [35 × 7 ] = × 35 × 7
7
πr 2θ 1
l Area of sector AOB = = lr = 770 m2
360° 2
l Area of a square inscribed in a circle of radius r is equal to Example 13 A wheel makes 2000 revolutions in covering
2r2 . a distance of 88 km. Find the radius of wheel.
l The area of the largest triangle inscribed in a (a) 6 m (b) 3 m
semi-circle of radius r is equal to r2 .
(c) 7 m (d) 4 m
l If area of a square is a sq unit, then the area of the circle
4a Sol. (c) Distance covered in 1 revolution
formed by the same perimeter is given by sq unit.
π 88 × 1000
= = 44 m
2000
Example 11 The radius of a circle is 2 cm. Find its According to the question, 2 πr = 44
diameter. 22
(a) 4 cm (b) 8 cm (c) 2 cm (d) 10 cm ⇒ 2× × r = 44
7
Solution (a) We know that, Diameter = 2r (r = radius) ∴ r = 44 ×
7
=7m
44
= 2 × 2 = 4 cm

Target Exercise
Elementary Level Questions
1. The diagonals of two squares are in the ratio of 3 : 2. 4. The side of a square is 5 cm which is 13 cm less than
Find the ratio of their areas. the diameter of a circle. What is the approximate
(a) 9 : 4 (b) 9 : 2 (c) 9 : 5 (d) 9 : 7 area of the circle?
2. If area of a regular pentagon is 125 3 sq cm, how (a) 245 sq cm (b) 235 sq cm
long is its each side? (c) 265 sq cm (d) 255 sq cm
(a) 10 cm (b) 15 cm (c) 16 cm (d) 25 cm 5. Find the area of a square inscribed in a circle of
radius 4 cm.
3. Find the number of diagonals of a regular polygon
having 12 sides. (a) 32 sq cm (b) 18 sq cm
(a) 56 (b) 54 (c) 60 (d) 40 (c) 64 sq cm (d) 25 sq cm
482 Study Package for NTSE

6. The area of a rectangle is 4 times the area of a 1 1 1


10. The sides of a triangle are in the ratio of : : and
square. The area of the square is 729 sq cm and the 3 4 5
length of the rectangle is 81 cm. What is the its perimeter is 94 cm. Find the length of the smallest
difference between the side of the square and the side of the triangle.
breadth of the rectangle? (a) 18 cm (b) 22.5 cm
(a) 18 cm (c) 24 cm (d) 27 cm
(b) 27 cm 11. Find the area of a triangle whose sides measure
(c) 24 cm 14 cm, 48 cm and 50 cm.
(d) 9 cm (a) 672 cm2 (b) 336 cm2
2
7. In the given figure, OP = PQ = 14 cm and semi-circles (c) 350 cm (d) 700 cm2
are drawn on OP, PQ and OQ as diameters. Then, the 12. ABC is an equilateral triangle inscribed in a circle. If
perimeter of the shaded area is the side of the triangle is 3 cm, what is the area of the
circle?
(a) 9 π / 3 cm2 (b) 3π cm2
(c) 3 3 π cm2 (d) 9 π / 4 cm2
O Q
P 13. D, E and F are mid-points of AB, AC and BC
respectively of triangle ABC. Find the area of the
(a) 88 cm (b) 176 cm quadrilateral DEFB, if the area of triangle ABC is 36
(c) 264 cm (d) 352 cm
sq cm.
(a) 12 sq cm (b) 18 sq cm
8. The perimeter of an isosceles triangle is 36 cm. If one
(c) 24 sq cm (d) 9 sq cm
of the equal sides is 13 cm, find the area of the
triangle. 14. If the perimeter of a rhombus is 52 cm and one of its
(a) 60 cm2 (b) 30 cm2 diagonals is 10 cm, then find its area (in sq cm).
(c) 30 2 cm 2
(d) 36 cm 2 (a) 154 (b) 168 (c) 120 (d) 175

9. If sides of a square are increased by 5%, by what 15. A trapezium has a height of 12 cm. Its longer and
per cent, its area will be increased? shorter parallel sides are 21 cm and 3 cm
respectively. Find its area (in sq cm).
(a) 4.44 (b) 19.26 (c) 10.25 (d) 16.16
(a) 72 (b) 108 (c) 216 (d) 144

High Skill Questions


1. The base of a parallelogram is thrice of its height. If 4. The largest triangle is inscribed in a semi-circle of
the area of the parallelogram is 2187 sq cm. Find its radius 4 cm. Find the area inside the semi-circle,
height. which is not occupied by the triangle?
(a) 27 cm (b) 35 cm (c) 29 cm (d) 26 cm (a) 8 ( π − 2) sq cm (b) 7 ( π − 1) sq cm
2. Find the area of the shaded portion in the figure (c) 8 ( π − 1)2 sq cm (d) 6 ( π − 2 ) sq cm
given below 5. The length of a rectangle is twice its breadth. If the
length is decreased by half of the 10 cm and the
4m breadth is increased by half of the 10 cm, the area of
4m the rectangle is increased by 5 sq cm more than
10 m 70 sq cm. Find the length of the rectangle.
(a) 30 cm (b) 40 cm (c) 21 cm (d) 45 cm
6. A rectangular lawn 80 m × 60 m has two roads each
30 m 10 m wide running in the middle of it, one parallel to
(a) 160 sq m (b) 144 sq m
the length and the other parallel to the breadth. Find
the cost of gravelling them at ` 20 per sq m.
(c) 169 sq m (d) 100 sq m
(a) ` 35000 (b) ` 39000 (c) ` 26000 (d) ` 41000
3. If area of a square is 64 sq cm, then find the area of
the circle formed by the same perimeter. 7. A 7 m wide road runs outside around a circular park
215 216 whose circumference is 176 m. Find the area of the
(a) sq cm (b) sq cm road.
π π
256 318 (a) 1386 sq m (b) 1472 sq m
(c) sq cm (d) sq cm
π π (c) 1512 sq m (d) 1760 sq m
Area and Perimeter 483
8. The perimeters of two squares are 68 cm and 60 cm. 15. The sides of a triangle are 25 m, 39 m and 56 m,
Find the perimeter of the third square whose area is respectively. Find the perpendicular from the
equal to the difference of the areas of these two opposite angle on the greatest sides.
squares. (a) 45 m (b) 90 m (c) 15 m (d) 30 m
(a) 64 cm (b) 60 cm (c) 32 cm (d) 8 cm 16. The length of the floor of a rectangular hall is 10 m
9. One diagonal of a rhombus is 60% of the other more than its breadth. If 34 carpets of size 6 m × 4 m
diagonal. Then, area of the rhombus is how many are required to cover the floor of the hall, then find
times the square of the length of the larger diagonal? the length and breadth of the hall.
1 2 6 3 (a) 34 m, 19 m (b) 34 m, 24 m
(a) (b) (c) (d)
5 5 7 10 (c) 19 m, 16 m (d) 14 m, 19 m
10. The hand of a clock are 10 cm and 7 cm, respectively. 17. The difference between two parallel sides of a
Find the difference between the distance travelled by trapezium is 8 cm. The perpendicular distance
their extremities in 3 days 5 h. between them is 38 cm. If the area of the trapezium is
(a) 4457.67 cm (b) 4557.67 cm 950 cm2, then find the length of the parallel sides.
(c) 4657.67 cm (d) 4575.67 cm (a) 29 cm, 21 cm
11. The area of a sector is 80 m 2. The radius of the circle (b) 30 cm, 21 cm
is 16 m. Find the angle of the sector. (c) 21 cm, 26 cm
(a) 34° (b) 34.8° (c) 35° (d) 35.8° (d) 21 cm, 80 cm
12. The radius of a circle is 75 cm. A zone of that circle 18. In a circle of radius 21 cm, an arc subtends an angle
has one of its parallel chords coinciding with the of 60° at the centre. Find the area of the minor and
diameter and the other equal to the radius. Find the major segment made by this arc.
area of that zone. (a) 48.1 cm 2, 1345.6 cm 2
2 2 2 2
(a) 8528 cm (b) 8328 cm (c) 8218 cm (d) 8238 cm (b) 14.6 cm 2, 1465.9cm 2
13. A took 15 s to cross a rectangular field diagonally (c) 16.1 cm 2, 1245.6 cm 2
walking at the rate of 52 m/min and B took the same (d) 40.1 cm 2, 1345.9 cm 2
time to cross the same field along its sides walking at 19. There is a garden of 140 m × 120 m and a gravel
the rate of 68 m/min. Find the area of the field. path is to be made of an equal width all around it, so
(a) 45 sq m (b) 90 sq m (c) 60 sq m (d) 30 sq m as to take up just one-fourth of the garden. What
14. Two sides of a triangle are 85 m and 154 m and the must be the breadth of the path?
perimeter is 324 m. Find the area of the triangle. (a) 8.65 m (b) 90 m
(a) 7216 m 2 (b) 1431 m 2 (c) 1350 m 2 (d) 2772 m 2 (c) 60 m (d) 121.3 m

Hints and Solutions


Elementary Level Questions
1. (a) Given, ratio = 3 : 2 3. (b) Given, number of sides,
Let the diagonals of two squares be 3x and 2 x. n = 12
1 Then, number of diagonals
( 3 x) 2
Then, ratio of their areas = 2 n(n − 3) 12(12 − 3)
= =
1
(2 x) 2 2 2
2 = 6 × 9 = 54
1
[Q Area of square = (diagonal) 2] 4. (d) Given, side of square = 5 cm
2
and diameter of circle = 13 + 5 = 18 cm
9
= =9:4 Diameter 18
4 ∴ Radius = = = 9 cm
2 2
2. (a) Let the side of the pentagon = a cm 22 22 × 81
Area of the circle = π r 2 = × (9)2 =
Given, area of regular pentagon = 125 3 sq cm 7 7
3 4 1782
⇒ 5a 2 = 125 3 ⇒ a 2 = 125 × = 100 =
4 5 7
∴ a = 100 = 10 cm = 254.57 ≈ 255 sq cm (approx.)
484 Study Package for NTSE

5. (a) Since, square is inscribed in a circle. 11. (b) Let a = 14 cm, b = 48 cm, c = 50 cm
∴ Diagonal of square = Diameter of circle = 2 × 4 = 8 cm This is a right angled triangle as
1
Now, area of square = × (Diagonal) 2 a2 + b 2 = c 2
2
1 64 ⇒ (14)2 + (48)2 = (50)2
= × (8)2 = = 32 sq cm
2 2 ⇒ 196 + 2304 = 2500
6. (d) Given, area of the square = 729 sq cm ⇒ 2500 = 2500
1 1
⇒ (Side) 2 = 729 ∴ Area = × Base × Height = × 14 × 48 = 336cm2
2 2
⇒ Side = 729 = 27 cm
12. (b) Given, A
Now, area of rectangle = 4 × 729
Side of triangle = 3 = r 3
⇒ 81 × Breadth = 4 × 729 [Q length = 81 cm] 3
⇒ r= 3 3
4 × 729 3
∴Breadth = = 4 × 9 = 36 cm r
O
81 ∴Area of circle = πr 2
Hence, required difference = 36 − 27 = 9 cm B C
9 3
= π× = 3 π cm2
7. (a) Given, figure has three semi-circles of radius 7 cm, 7 cm and 3
14 cm.
13. (b) As D, E and F are mid-points, areas of triangles ADE, EFC,
∴ Required perimeter = πr1 + πr2 + πr3
DEF and DFB are all equal and each is 1/4th of the triangle
22
= π [7 + 7 + 14 ] = × 28 = 88 cm ABC.
7
Area of quadrilateral DEFB
8. (a) Given,
A
Q h = 132 − 152
[Q by Pythagoras theorem]
D E
= 12 cm 13 13
1 h
Hence, Area = × base × height
2 B C
F
1
= × 12 × 10 = 60 cm2 5 1
2
10 =2 × (Area of ∆ABC)
4
9. (c) Let the original side of square = a
1
= × (36) = 18 sq cm
Then, the original area of square = (a)2 2
Now, after increment the new side of square, 14. Given, A
(100 + 5) 21 Q Perimeter = 52 cm = 4 (side)
=a× = a
100 20 ∴ Side = 13 cm 13 cm
2
441a2 Diagonal 1 = 10 cm
Again, new area = 
21a 
 = E
 20  400 = AD (say) B C

∴Required increment in percentage ∴ AE = 5 cm


441a2 In ∆ AEC
− a2
441 − 400 41 EC = AC 2 − AE 2
= 400 × 100 = × 100 = = 10.25%
a2 400 4 D
1 1 1
10. (c) Given, ratio = : : [Q by Pythagoras theorem]
3 4 5
60 60 60 = 132 − 52 = 12 cm
= : : [LCM of 3, 4, 5 =60]
3 4 5 ∴ Area of ABCD = 4 × Area of ∆AEC
1
= 20 : 15 : 12 = 4 × ( AE ) (EC )
2
Let lengths of the sides be 20 x, 15 x and 12 x. 1
= 4 × × 5 × 12 = 120 sq cm
Then, perimeter = a + b + c [where a, b, c are sides] 2
1
= 20 x + 15 x + 12 x = 94(given) 15. (d) We know that, Area of a trapezium =
2
⇒ 47 x = 94 (Height) (sum of the lengths of the parallel sides)
94 ∴Required area (in sq cm)
∴ x= =2
47 1
= (12 ) (21 + 3)
Hence, length of the smallest side = 12 x 2
= 12 × 2 = 24 cm = 144cm2
Area and Perimeter 485

High Skill Questions


1. (a) Let the height of parallelogram = a cm 5. (b) Given, length of rectangle,
then, base of parallelogram = 3a cm l = 2 × Breadth of rectangle (b)
Given, area of parallelogram = 2187 sq cm i .e., l = 2 b
⇒ a × 3 a = 2187 Decrease in length = Half of the 10 cm
1
⇒ a2 =
2187
= 729 = × 10 = 5 cm
3 2
∴ a = 729 = 27 cm Increase in breadth = Half of the 10 cm
1
Hence, height of the parallelogram is 27 cm. = × 10 = 5 cm
2
2. (b) Given, Increase in the area = (70 + 5) = 75 sq cm
4m
According to the question,
4m (l − 5) (b + 5) = lb + 75
10 m
⇒ (2 b − 5) (b + 5) = 2 b 2 + 75 [Q l = 2 b ]
2 2
⇒ 2 b + 10b − 5b − 25 = 2 b + 75
30 m ⇒ 5b = 75 + 25
100
Area of shaded portion, length wise ∴ b= = 20cm
5
= 30 × 4 [Q area of rectangle = length × breadth]
Hence, length of the rectangle
= 120 m 2 l = 2 × 20 = 40 cm
Area of shaded portion, breadth wise = 10 × 4 = 40 m2 6. (c) Area to be gravelled
Area of intersecting part = 4 × 4 = 16 m2 = x ( l + b − x)
[Q area of square = (side) ] 2 = 10 (80 + 60 − 10)
= 10 × 130 = 1300 sq m
Hence, total area of the shaded region
= 120 + 40 − 16 = 144 m2
10 m
A B
3. (c) Area of square = 64 sq cm
⇒ (Side) 2 = 64 ⇒ Side = 64 = 8 cm
10 m 10 m
Now, perimeter of square = 4 × 8 = 32 cm
According to the question,
Perimeter of circle = Perimeter of square D C
10 m
⇒ 2 πr = 32
32 16 ∴Cost of gravelling = ` (1300 × 20) = ` 26000
∴ r= = cm
2× π π 7. (a) Let r cm and R cm be the radii of park and road, respectively.
2 Given,
Hence, required area of circle = πr 2 = π   =
16 256
sq cm Circumference of the park = 176 m
 π π
176 × 7
4. (a) Given, radius of circle, ⇒ 2 π r = 176 ⇒ r = = 28 m
2 × 22
OA = OB = OC = 4 cm
∴ R = 28 + 7 = 35 m
πr 2 π(4)2
∴ Area of semi-circle = = = 8π sq cm
2 2 7m
B
R
r
4
O
A C
Now, area of the road = πR 2 − πr 2 = π [(35) 2 − (28) 2 ]
8 cm
1
∴Area of ∆ABC = × Base × Height =
22
[(35 + 28) (35 − 28)]
2 7
1
= × 8 × 4 = 16sq cm =
22
× 63 × 7
2 7
Now, required area (shaded region)
[Q a 2 − b 2 = (a + b ) (a − b )]
= 8 π − 16 = 8 ( π − 2 ) sq cm
= 1386 sq m
486 Study Package for NTSE

8. (c) Given, perimeter of first square = 68 cm 11. (d) Given, radius, OA = OB = 16 m


⇒ 4 × Side = 68 Let angle of the sector = θ
68
∴ Side, (a1) = = 17 cm (According to the question),
4
and perimeter of second square = 60 cm
60 B
∴ Side, (a 2 ) = = 15 cm O
16m
4 θ
According to the question,
Area of third square = (17 ) 2 − (15) 2
A
= (17 + 15) (17 − 15)
[Q a2 − b 2 = (a + b )(a − b )] πr 2θ
Area of sector = 80 ⇒ = 80
360°
= 32 × 2 = 64 sq cm
80 × 360° 80 × 360° × 7
Also, area of third square = (Side) 2 ⇒ θ= =
22 7 × (16)2 22 × 16 × 16
⇒ (Side) 2 = 64
∴ θ = 35.8°
∴ Side = 64 = 8 cm
12. (b) Given, radius of circle,
Hence, perimeter of third square
OA = OB = OC = OD = 75 cm
= 4 × 8 = 32 cm
Length of chord coinciding with diameter,
9. (d) Let the longer diagonal = x cm, CD = 75 × 2 = 150 cm
then other diagonal = 60% of first diagonal x
60 3x Minor
= × x= cm A B segment
100 5 60°
1
Now, area of rhombus = × product of diagonal C
O
D
2
1 3x
= × x×
2 5
3 and other chord AB = Radius = 75 cm
= × x2
10 Since, ∆OAB is an equilateral triangle
3 (each side =75 cm)
= × (square of longer diagonal)
10 So, angle of the sector, θ = 60°
3 3 3 2
Hence, area of rhombus is times the square of longer Area of ∆ AOB = (75) 2 [Q Area of triangle = ( a) ]
10 4 4
diagonal. 1732
. × 5625 9742.5
= = = 2435.6 sq cm
10. (b) Total time = 3 days 5 h = 77 h 4 4
Radius of minute hand, R = 10 cm πr 2θ
Area of the sector AOBX =
Radius of hour hand, r = 7 cm 360°
22 × (75)2 × 60° 22 × 5625
= =
Minute
circle 7 × 360° 7×6
10 = 2946.42 sq cm
∴Area of minor segment ABX
7 Hour
circle
= 2946.42 − 2435.6 = 510.82 sq cm
Hence, required area ABDC = Area of semi-circle − Area of
1 22
Total distance travelled by the extremity of hour hand minor segment = × × (75) 2 − 510.82
2 7
= Circumference × Number of revolution
77 = 8839.28 − 510.82 ≈ 8328 sq cm
= 2 πr ×
12 13. (c) Distance travelled by A in 15 s
[Q hour hand completes one revolution in 12 h] 15
= 52 × = 13 m l
C
22 77 60 D
=2 × ×7 × = 282.33 cm
7 12 ∴Diagonal of the field = 13 m
b
Total distance travelled by extremity of minute hand ⇒ l 2 + b 2 = (13) 2
= Circumference × Number of revolution = 2 πR × 77
⇒ l 2 + b 2 = 169 ...(i) A B
[Q minute hand completes one revolution in 1 h]
22 Distance travelled by B in 15 s
=2 × × 10 × 77 = 4840 cm
7 15
= 68 × ⇒ l + b = 17 m ...(ii)
Hence, required difference = 4840 − 282.33 = 4557.67 cm 60
Area and Perimeter 487
On squaring both sides, we get 17. (a) Let the two parallel sides of trapezium be a cm and b cm.
(l + b ) 2 = (17 ) 2 Given, a − b = 8 ...(i)
⇒ l 2
+ b 2
+ 2 lb = 289 Perpendicular distance, h = 38 cm
Area = 950 cm 2 b
⇒ 169 + 2 lb = 289 [from Eq. (i)]
1
289 − 169 ⇒ (a + b ) h = 950
⇒ lb = = 60 2
2 38 cm
950 × 2
Hence, area of the field = 60 sq cm ⇒ a+ b= = 50
38
14. (d) Let the sides of the triangle be a m, b m and c m .
∴ a + b = 50 ...(ii) a
Then, a = 85 m and b = 154 m
On adding Eqs. (i) and (ii), we get
Given, perimeter = a + b + c = 324
2 a = 58
⇒ 85 + 154 + c = 324 58
∴ a= = 29 cm
⇒ c = 324 − 239 2
⇒ c = 85 m On putting the value of a in Eq. (i), we get b = 29 − 8 = 21cm
Thus, given triangle is an isosceles triangle Hence, required lengths parallel sides are 29 cm and 21 cm.
b 18. (d) Given, radius of circle = 21cm
∴ Area = 4a 2 − b 2
4
An arc AXB subtends an angle of 60° at O.
154
= 4(85) 2 − (154) 2 Since, OA = OB = 21 cm (radius)
4
154 154 ∴ ∠ OAB = ∠ OBA = 60° = ∠ AOB
= × 5184 = × 72 Hence, ∆ABO is an equilateral triangle.
4 4
3 3 2
∴ Area = 2772 m2 Area of ∆ABO = × (21) 2 [Q Area of triangle = ( a) ]
4 4
15. (c) Given, sides of the triangle, 441 3 763.812
= = [Q 3 = 1732. ]
a = 39 m , b = 56 m and c = 25 m 4 4
Let h = perpendicular from ∠ B on greatest side. = 190.95 cm 2
Y
B
a+ b+c πr 2θ
Now, s = Area of sector AOB = O
2 360°
39 + 56 + 25 2
22 × (21) × 60° 21 21
= = 60 m 25 h 39 = = 231 cm 2 60°
2 7 × 360° A B
∴Area of triangle Now, area of minor segment AXB X
= s ( s − a) ( s − b ) ( s − c ) A D 56 C = Area of sector AOBX – Area of ∆ABO
1
⇒ × Base × Height = 231 − 190.95 = 401 . cm 2
2
22
= 60(60 − 39) (60 − 56) (60 − 25) Area of circle = πr 2 = × 21 × 21 = 1386 cm 2
1 7
⇒ × 56 × h Hence, area of major segment
2
= Area of circle − Area of minor segment
= 60 × 21 × 4 × 35 = 1386 − 401 . = 1345.9 cm 2
420 × 2
∴ h= = 15 m 19. (a) Given, length of the garden = 140 m
56
Hence, length of the perpendicular is 15 m. and breadth of the garden = 120 m
16. (b) Let the breadth of the hall = b m, ∴ Area of the garden = (140 × 120) m2
then length of the hall, l = (b + 10 ) m 1
Since, path covers th area of the garden, that means path is
Area of the hall = Length × Breadth 4
1
= b(b + 10) ...(i) inside the garden and area of path = × 140 × 120
4
Area of one carpet = (6 × 4) m 2
= 4200 cm2
Q 34 carpets are required to cover the floor. According to the formula,
∴ Area of floor of hall = 34 × area of one carpet Area of pathway = 2 x (l + b − 2 x)
= 34 × 6 × 4 ⇒ 2 x(140 + 120 − 2 x) = 4200
Now, b (b + 10 ) = 34 × 6 × 4 ⇒ x(260 − 2 x) = 2100
= 34 × 24 ⇒ x 2 − 130 x + 1050 = 0
∴ b = 24 m ∴ x = 8.65, 1213. [Q by Sridharacharya rule]
and l = 24 + 10 Since, 121.3 > 120, so it is not possible.
= 34 m Hence, required breadth of the path is 8.65 m.
488 Study Package for NTSE

Chapter

14
Volume and Surface Area

Solid Figures Cube


The objects which occupy space (i.e., they have three A box whose length, breadth and height all are equal,
dimensions) are called solids. The solid figures can be is called a cube.
derived from the plane figures.
e.g., In figure (i), we have a paper cut in the form as
shown. It is a plane figure but when we fold the paper
along the dotted lines, a box can be made as shown in
figure (ii).
● Edge of cube = Length = Breadth = Height
● Volume of a cube = (side)3
A B ● Total surface area of a cube = 6 × (side)2
A ● Covered surface area of cube = 4a 2
B
● Diagonal of a cube = 3 × side
D C Example 1 Find the surface area and the length of the
diagonal of the cube. If the volume of a cube is 2197 cu cm, are
D C (a) 1012 sq cm and 21.516 cm
(i) (ii) (b) 1024 sq cm and 24.516 cm
(c) 1014 sq cm and 22.516 cm
As the box in figure (ii) occupies some part of the (d) None of the above
space, it has more than two dimensions and therefore,
Solution (c) Volume of cube = (side) 3 = 2197 cu cm
it fulfils the criteria of being a solid figure.
∴ Side of cube = 13 cm
Volume ∴ Surface area of a cube = 6 (side) 2 = 6 (13) 2 = 1014 sq cm
The measure of part of space occupied by a solid is and length of the diagonal of the cube = 3 × side
called its volume. = 3 × 13
Surface Area = 22.516 cm

The sum of the areas of the plane figures making up Cuboid


the boundary of a solid figure is called its surface area.
A cuboid has six rectangular plane surfaces called
e.g., The area of paper in figure (i) is the surface area
faces. A cuboid has 8 corners called the vertices.
of box.
Volume and Surface Area 489
Example 3 The volume of a cylinder is 448 π cm3 and height
7 cm. Then, its lateral surface area and total surface area are
(a) 349 cm 2 and 753.286 cm 2
(b) 352 cm 2 and 754.286 cm 2
● Volume of cuboid = l × b × h (c) 353 cm 2 and 755.286 cm 2
(d) None of the above
where, l = length, b = breadth and h = height
Solution (b) Volume of the cylinder = 448 π cm 3
● Whole surface of cuboid = 2 ( lb + bh + lh )
Height of the cylinder = 7 cm
● Diagonal of the cuboid = l2 + b2 + h 2
Let radius be r, then πr2 h = 448 π
● Covered surface area of cuboid = 2 ( l + b) h 448 π 448
∴ r2 = = = 64 ⇒ r = 8 cm
Example 2 The areas of three adjacent faces of a cuboid are hπ 7
x , y and z. If its volume is V , then which is true? ∴ Lateral surface area of the cylinder
(a) V = x 3y 2z2 (b) V 2 = xyz = 2πrh = 2 ×
22
× 8 × 7 = 352 cm 2
x 2y 7
(c) V = 3 xyz (d) V =
z Total surface area of the cylinder = 2πr ( h + r)
22
Solution (b) Let the dimensions be l, b and h. = 2× × 8 (7 + 8 )
7
Then, volume of cuboid, V = lbh
22 5280
Also, x = lb, y = bh, z = hl = 2× × 8 × 15 = = 754.286 cm 2
7 7
∴ V = lbh
⇒ V 2 = l 2 b2 h 2 [Q on squaring both sides] Hollow Cylinder
The volume of material in a hollow r
π
= ( lb) ( bh) ( hl)
cylinder is the difference between
∴ V = xyz
2
the volume of a cylinder having the
external dimensions and the h
Right Circular Cylinder volume of a cylinder having the
A right circular cylinder is a solid generated by the internal dimensions.
revolution of a rectangle about one of its sides which R
remains fixed. A cylinder shown in figure (i) is Let R and r be the external and
described by revolving the rectangle ABCD about the internal radii of the hollow cylinder and h be its height.
side BC. The examples of right circular cylinder are ● Volume of material = π ( R2 − r 2 ) h
many such as water pipes, powder box, laboratory ● Total surface area = 2 π ( R + r ) ( h + R − r )
beakers etc. ● Curved surface area = 2 π Rh + 2 π rh = 2 π ( R + r ) h
B ● Total outer surface area = 2πrh + πR2 + π ( R2 − r 2 )
A
Where, R = External radius of base,
r = Internal radius of base,
and h = Height
h
Example 4 A hollow cylindrical tube, open at both ends is
made of iron 1 cm thick. The volume of iron used in making the
tube, if the external diameter is 12 cm and the length of tube is
D C 70 cm is
(i)
(a) 2420 cm 3 (b) 2520 cm 3
● Volume of the cylinder = (Area of base) × Height (c) 2720 cm 3 (d) 2900 cm 3
= πr 2h cu units
where, r is the radius of the base and h is the height of Solution (a) Here, external radius ( R ) = 6 cm
the cylinder. Internal radius ( r) = 6 − 1 = 5 cm and height ( h) = 70 cm
● Curved surface area = Circumference of the base ∴ Volume of iron used = External volume − Internal volume
× Height = 2πrh sq units
= πR 2 h − πr2 h
● Total surface area = Curved surface area + Area of two 22
circular ends = 2πrh + 2πr 2 = 2πr ( h + r ) = πh ( R 2 − r2 ) = × 70 × (36 − 25)
7
Where, r = Radius of base and h = Height
= 220 × 11 = 2420 cm 3
490 Study Package for NTSE

Right Circular Cone ● Lateral surface area of frustum of right circular cone
A right circular cone is a solid
A = πl ( R + r ) sq units
generated by the revolution of a where, slant height, l2 = h 2 + ( R − r )2
right angled triangle about one ● Total surface area of frustum of right circular cone
of its sides containing the right l = Area of base + Area of top + Lateral surface area
angle as axis. In the adjoining h = π [ R2 + r 2 + l ( R + r )]
figure a cone of height ‘h’ and ● Total surface Area of bucket = π [( R + r ) l + r 2 ]
radius ‘r’ is generated by (Q it is open at the bigger end)
revolving the right ∆AOB along B C
AO.
r O Example 6 The radii of the ends of a bucket of height 24 cm
are 15 cm and 5 cm. Then, its capacity is
The slant height of the cone is
(a) 8000 cm 3 (b) 8100 cm 3
l = AC = r2 + h2 (c) 8171.43 cm 3 (d) 8200.43 cm 3
1 2
● Volume of cone = πr h cu units Solution (c) Capacity of bucket
3
= Volume of frustum of a cone
● Curved surface area of cone = πrl sq units
πh 2 5 cm
● Total surface area of a cone = πr ( l + r ) sq units = ( R + r2 + Rr)
3
Where, r = Radius of base, 22 24
h = Height and l = Slant height = × [(15) 2 + 52 + 15 × 5] 24 cm
7 3
22
Example 5 The radius and the height of a right circular = × 8 (225 + 25 + 75)
7 15 cm
cone are in the ratio 5 : 12. If its volume is 314 m 3 , the slant
176
height and the radius are = (325)
7
(a) 12, 5 m (b) 13, 4 m (c) 1, 4 m (d) 13, 5 m
= 8171.43 cm 3
Solution (d) Let radius of cone = 5x
and the height of the cone = 12x Sphere
1 2
∴ Volume of cone = 314 = πr h A sphere is a solid generated by the revolution of a
3
semi-circle about its diameter. A sphere is the locus of
1
⇒ × 3.14 × (5 x) 2 × (12 x) = 314 x3 [here, π = 3.14] a point which moves in space such that its distance
3
from a fixed point in space remains constant. The fixed
⇒ 314 = 314 x3 ⇒ x3 = 1 ⇒ x = 1
point is called the centre of sphere and the constant
∴ Radius = 5 m and height = 12 m distance is called the radius of sphere.
∴ Slant height = r2 + h 2 = 25 + 144 = 169 = 13 m Here, ‘O’ is centre and r is radius. All radii are equal.
A
∴ Radius = 5 m and slant height = 13 m
r
Frustum of a Cone
If a cone is cut by a plane parallel to the base of the r
cone, then the portion between the plane and base is O
called the frustum of the cone.
Let R and r be the radii of base and top of the frustum
of a cone. Let h be the height, then
B
r ● The section of a sphere cut by any plane is a circle. If the
cutting plane passes through the centre of the sphere,
the section is called a great circle.
4
● Volume of sphere = πr 3 cu units
3
R ● Surface area of sphere = 4πr 2 sq units
Volume of frustum of right circular cone 4
Volume of a hollow sphere = π ( R3 − r 3 ) cu units


πh 2 2 3 3
= ( R + r + Rr ) cm where, r = inner radius and R = outer radius
3
Volume and Surface Area 491
Example 7 If the volume of a metal sphere is 38808 cm 3 . Example 9 The ratio of the volume of a cube to that of a
Then, its radius and its surface area are sphere, which will fit inside the sphere is
(a) 7 cm and 616 cm 2 (b) 21 cm and 5544 cm 2 (a) 3 π : 2
2
(c) 14 cm and 2464 cm (d) None of these (b) 2 : 3 π
Solution (b) Volume of the metal sphere = 38808 cm 3 (c) 3 : 2 π
4 3 (d) None of the above
But volume of sphere = πr
3 Solution (b) We know that,
4 3
∴ πr = 38808 Diagonal of cube = Diameter of sphere
3 2
i.e., a 3 = 2r ⇒ a = r
3 7 3
⇒ r3 = 38808 × × = 9261 ⇒ r = ( 9261) 1 / 3 = 21 cm
4 22 Volume of cube
∴ Require volume ratio =
∴ Surface area of the sphere = 4 πr2 Volume of sphere
22
=4 × × 21 × 21 = 5544 cm 2
7

Hemisphere
A plane passing through the centre cuts the sphere in
two equal parts each called a hemisphere.
r
(Side) 3 ( 2 / 3 r) 3
= =
4 3 4 / 3 π r3
πr
3
8 ×3
= = 2: 3 π
4 ×3 3 π
2 3
● Volume of hemisphere = πr cu units Example 10 A tent is cylindrical upto a height 3 m and
3
conical above. If the diameter of the base is 105 m and the slant
● Curved surface area of hemisphere = 2 πr 2 sq units height of the conical part is 53 m, total canvas used in making
● Total surface area = 2πr 2 + πr 2 = 3 πr 2 sq units for tent is
Example 8 The volume of two hemisphere are in the ratio (a) 9375 m 2
8 : 27. What is the ratio of their radii? (b) 8753 m 2
(a) 2 : 3 (b) 3 : 2 (c) 1 : 2 (d) 2 : 1 (c) 7550 m 2
Solution (a) Let volumes be V1 and V2 . (d) 2000 m 2
∴ V1 : V2 = 8 : 27 Solution (a) Total canvas = Curved surface area of the
2 3 2 3 figure
⇒ πr1 : πr2 = 8 : 27
3 3
⇒ r13 : r23 = 8 : 27 ⇒ r1 : r2 = 8 : 3 27 ⇒ r1 : r2 = 2 : 3
3 53 m

Surface Area and Volume of Combination


of Solids
In our day-to-day life we come across some complex
solid figures, e.g., a circus tent consisting of a
105 m
cylindrical base surmounted by a conical roof, a toy in
= 2π rh + π rl
the form of cone mounted on a hemisphere etc. All of
105 × 53 
= π  2 ×
105
these are combination of two or more basic solids. In × 3 + 
 2 2 
this topic, we will learn how to find the surface areas
22 105
and volumes of combination of two or more similar or = × [6 + 53 ] = 11 × 15 × 59 = 9375 m 2
7 2
different solid figures.
Target Exercise
Elementary Level Questions
1. The curved surface area of a cone of height equals to 8. What is the length of the longest rod which can be
4 times of radius and the radius equals to 6 cm, is placed in a cube of total surface area of 1176 cm 3?
3 (a) 7 cm (b) 14 cm (c) 7 3 cm (d) 14 3cm
(a) 188.5 cm2 (b) 180.6 cm2
(c) 184.8 cm2 (d) None of these 9. A cone, a hemisphere and a cylinder stand on equal
bases and have the same height the ratio of their
2. A sphere has the same volume as a cylinder whose
volumes is
height is 3 times the radius of its cross section. Then,
(a) 1 : 2 : 3 (b) 2 : 3 : 1 (c) 3 : 1 : 2 (d) 2 : 1 : 3
the ratio of their radii is
2
(a) 3 / 4 (b) (9 / 4)1/ 3 (c) (d) (4 / 9)1/ 3 10. If two cubes of sides 9 cm each are joined end by end,
3
then the surface area of the cuboid will be
3. A cone of radius 5 cm is filled with milk. If the milk is (a) 180 cm2 (b) 810 cm2 (c) 800 cm2 (d) 820 cm2
poured into a cylinder of radius 10cm, the height of
11. A solid is in the form of a cylinder surrounded by a
milk rises by 2 cm. Height of the cone is
cone. The heights of the cone and the cylinder are 6 cm
(a) 20 cm (b) 22 cm (c) 24 cm (d) 26 cm
and 12 cm respectively. If the radius of the cone is 7cm,
4. A sphere of diameter 6 cm is dropped in a right its volume will be
circular cylinder vessel partly filled with water. The (a) 2156 cm3 (b) 2100 cm3
radius of cylinder is 9 cm. If the sphere is completely (c) 2560 cm3 (d) None of these
submerged in water, the level of water rises by
(a) 0.44 cm (b) 0.64 cm 12. Find the volume of the largest right circular cylinder
(c) 1 cm (d) None of these which can be cut from a cube of side 7 cm.
(a) 260.5 cm3 (b) 269.5 cm3
5. A cone of height 12 cm and radius 14 cm is to be (c) 272.5 cm3 (d) None of these
divided into two parts by cutting through the
mid-point of vertical axis. Then, the volume of conical 13. The ratio of the volume of a cube to that of a sphere
part is which will fit inside the cube is
(a) 616 cm3 (b) 692 cm3 (a) 9 : π (b) 2 : π
(c) 308 cm3 (d) 346 cm3 (c) 3 : π (d) 6 : π

6. A toy is in the form of a right circular cone mounted 14. If S1 and S2 are the total surface area of a sphere and
on a hemisphere. If the radius of hemisphere is 3.5 cm the curved surface area of the circumscribed cylinder,
and the total height of the toy is 9.5 cm. The volume of then S1 : S2 is
(a) 1 : 2 (b) 1 : 1
toy is
(c) 1 : 4 (d) 1 : 3
(a) 164 cm 3 (b) 166.83 cm 3

(c) 169 cm3 (d) 172.88 cm3 15. A conical cap is filled with ice-cream. The ice-cream
forms a hemispherical shape on its top. The radius of
7. A vessel is 5 cm long and its base is triangular having the hemisphere is equal to the height of the cone. If
sides 2 cm, 3 cm and 4 cm, the curved surface area of the height of the hemispherical part is 7cm, the
vessel is volume of the ice-cream is
(a) 40 cm2 (b) 45 cm2 (a) 8701 cm3 (b) 1078 cm3
(c) 46 cm2 (d) 48 cm2 (c) 1087 cm3 (d) None of these

High Skill Questions


1. A cone is within the cylinder and cylinder is within a 2. A tent is cylindrical upto a 3 m height and conical
cube touch, by all vertical faces with same base and above. If the diameter of the base is 105 m and the
height, then the ratio of their volume will be slant height of the conical part is 53 m, total canvas
(a) 14 : 11 : 13 used in making the tent is
(b) 42 : 33 : 11 (a) 9007 m3 (b) 9875 m3
(c) 56 : 36 : 22 (c) 9573 m3
(d) None of the above (d) 9735 m3
Volume and Surface Area 493
3. A vessel is in the form of right circular cylinder (a) 132 π cm3 (b) 168 π cm3
mounted an a hemisphere. If the radius of (c) 100 π cm3 (d) 135 π cm3
hemisphere is 7 cm and the total height of vessel is 7. In the figure shown, if AD= 5, DG = 2 and GH= 4.
13cm. The surface area of vessel is
(a) 762 cm 2 (b) 768 cm 2 What is the shortest distance between E and C?
(c) 726 cm2 (d) 780 cm2 A B
4. A storage tank consists of circular cylinder with a
hemisphere adjoing at either end. If the diameter of D C
the cylinder be 1.4 m and its lenght be 8 m, the cost of
painting it on the outside at the rate of ` 10 per m2 E F
will be
(a) ` 413.6 (b) ` 41.36
(c) ` 4136 (d) ` 4140 G H
(a) 49 (b) 5 + 18
5. A cylinder is of height 31 cm and base radius 7 cm. A
(c) 65 (d) None of these
hemisphere of radius equal to base radius of cylinder
is cut off from one end and a cone of maximum height 8. The number of coins of base are 38.5 cm2 and
from remaining part is also cut off. The curved thickness 5 mm required to make a cylinder of
surface area of the remaining part is volume 462 cm3 are
(a) 506 cm2 (b) 508 cm2 (a) 12 (b) 18
(c) 510 cm2 (d) 512 cm2 (c) 20 (d) 24

6. The top and bottom radii of a frustum of a cone are 9. How many cubes each of edge 6 cm can be cut from a
6 cm and 3 cm, respectively. Its height is 8 cm. There cuboid of 42 cm × 36 cm × 24 cm ?
is a conical cavity of height 3cm and radius 6cm at the (a) 124 (b) 142
bottom. The amount of material in the solid is (c) 168 (d) 186

Hints and Solutions


Elementary Level Questions
1. (a) Curved surface area of cone = π rl where, H– h = Rise in water level
h = 4 / 3× 6 = 8 4 / 3 π × 27 4 27
We know that, ∴ H− h= = × = 0.44 cm
π × 81 3 81
r = 6, l = h2 + r 2 = 82 + 62 = 10
2
22 1  r h
∴ π rl = × 6 × 10 = 188.5 cm2 5. (c) Volume of conical part = π 
7 3 2 2
2. (b) Given, r

Volume (sphere) = Volume (cylinder)


4 3
⇒ π r = π R2h
3
4 3 r3 9 h
⇒ π r = πR 2 × 3R ⇒ = r/2
3 R3 4
r 39 h/2
∴ = = (9 / 4)1/ 3
R 4
3. (c) According to the question, 1 22
= × × 7 × 7 × 6 = 308 cm3
Volume (cone) = Volume/cylinder 3 7
1 2 6. (b) Volume of toy = Volume of cone + Volume of hemisphere
∴ π r h= π R2H
3 1 2 2 1 2
= π r h + π r 3 = π (3.5)2 × 6 + π (3.5)3 = 166.83 cm3
1 3 3 3 3
i.e., π (5)2 h= π (10)2 × 2 ⇒ h = 24 cm
3
7. (b) Perimeter of base (triangle) = Sum of all sides
4. (a) Volume of sphere = Change of volume in cylinder = 2 + 3 + 4 = 9 cm
4
⇒ π (3)3 = π (9)2 (H − h) Curved surface area = Perimeter of base × Height
3
= 9 × 5 = 45 cm2
494 Study Package for NTSE

8. (d) 6 (Side)2 = Total surface area 12. (b) The largest circular cylinder will have height and diameter both
i.e., 6 x = 1176
2
[Qside of cube = x (let)] same as the side of the cube.

⇒ x =14 ∴ Required volume = π r 2 h = π(7 / 2 )2 × 7 = 269.5 cm3

Thus, length of rod = x2 + x2 + x2 = 14 3 cm Vc x3 V


13. (d) = ⇒ c =6/ π
Vs 4 / 3 π x 3 / 8 Vs
9. (a) Let R be the radius of each figure.
∴ Height of hemisphere = R ∴ Vc : Vs = 6 : π
So, height of each figure = R S1 4π r2 4 πr 2 1
14. (b) = = =
∴ Ratio of volumes =  π R 2 × R  : π R 3 :( π R 2 × R )
1 2 S 2 2 π r(2 r ) 4 πr 2 1
3  3
∴ S1 : S 2 = 1 : 1
= 1: 2 : 3
15. (b) Since, the radius of the hemisphere part = Height of the cone
10. (b) We will have l = 18, b = 9 and h= 9
Radius of the part = Height of the part = Height of the cone
∴ Surface area = 2(lb + bh + hl )
= 2 (18 × 9 + 9 × 9 + 9 × 18 ) = 810 cm2 = 7 cm

11. (a) Volume = Volume of cylinder + Volume of cone ∴Volume of the ice-cream
= Volume of cone + Volume of hemisphere
= π × 7 2 × 12 + 1 / 3 π (7 )2 × 6
1 2
= πr 2 h + πr 3
= 49 π(14) = 49   (14)
22
3 3
7  1 2 22
= π(7 )2 × 7 + π(7 )3 = × (7 )3 = 1078 cm3
= 2156 cm3 3 3 7

High Skill Questions


1. (b) Given, radius and height of cone, cylinder and cube = r = 2 πrh − [ π rl + 2 πr 2 ]
= 2 × × 7 × 31 −  × 7 × 7 2 + (31− 7 )2 + 2 × × 7 2 
Then, volume of cube : volume of cylinder : volume of cone 22 22 22
7  7 7 
r2 1 r2
= V1 : V2 : V 3 = r 3 : π × r : π × r = 506 cm2
4 3 4
3
r 1 r 3
22 1 1 22 1 6. (a) We extend the frustum to form of the cone. Now, using
= r3 : π : π = 1 : × : × ×
4 3 4 7 4 3 7 4 similarty of triangle we get height of the smaller cone as 8 cm and
the height of the bigger cone as 16 cm.
= 84 : 66 : 22 = 42 : 33:11
2. (d) Total canvas = Curved surface area of figure
2 × 105 105 × 53 
= 2 π rh + π rl = π  × 3+ 
 2 2 
3
diameter 105
[Q radius = = m]
2 2
= 9735 m3 6
3. (c) Surface area of vessel
Thus, the required volume
= Surface area of cylinder + Surface area of hemisphere 1
= π(16 × 62 − 3 × 62 − 8 × 32 ) =132 π cm3
= 2 π r 2 + 2 π rh + π r 2 = 3 π r 2 + 2 π rh 3
3 × 22 2 22
= × 7 + 2 × × 7 × 6 = 22 (21 + 612 ) =726 cm2 7. (d) Thus, minimum length of EC
7 7
= (5)2 + (2 )2 + (4)2 = 25 + 4 + 16 = 45
4. (a) Cost of painting A B
= 10 × Curved surface area of tank 5
C
= 10 × [Curved surface area of cylinder + Curved D
surface area of hemisphere] 2 E F
= 10 × (2 π rh + π r 2 + 2 π r 2 )
G
= 10 [(2 π )(07
. ) 8 + π (07
. )2 + 2 π (07
. )2 ] 4 H
5
[Q radius =
diameter 14
=
.
=.07 m] 8. (d) Let the number of coins be k. 38.5 ×
× k = 462 ⇒ k =24
2 2 10
= ` 413.6 Volume of cuboid l × b × h
9. (c) Number of cubes = =
Volume of cube (a)3
5. (a) Curved surface area of the remaining solid
42 36 24
= Curved surface area of [cylinder − (cone + sphere)] = × × = 168
6 6 6
Trigonometry 495

Chapter

15
Trigonometry
P
Trigonometry is a branch of Mathematics in which we ( ) sin A =
H
study right angled triangles and relationship between
their sides and angles using trigonometric ratios. Using Pythagoras theorem,
H 2 = P2 + B2 ⇒ B = H 2 − P2 = 52 − 42 = 3
Trigonometric Ratios ∴
P 4
tan A = =
B 3
The ratios between different sides of a right angled
triangle with respect to its acute angles are called 24
Example 2 If sinθ = , then find the value of
trigonometric ratios. 25
cosec θ + cotθ.
Trigonometric ratios for right angled ∆ABC with respect
(a) 3/4 (b) 4/3 (c) 5/3 (d) 4/5
to angle A are given below
24 P
C ( ) sin θ = =
25 H
(called perpendicular, P)
Side opposite to ∠A

A
Using Pythagoras theorem, B = 7
) °
(H 9 0
se to

H 25
cosec θ = =
nu te

θ
te osi

P 24
po pp

H=5
Hy e o

B 7
and cot θ = = . B
d
Si

P 24
25 7
Thus, cosec θ + cot θ = + 90°
24 24
A Side adjacent B B C
32 4 P=4
to ∠A (called base, B) = =
24 3
BC P AB B
sin A = = , cos A = =
AC H AC H Trigonometric Table of Some Specific Angles
BC P AC H
tan A = = , cosec A = = 0° 30° 45° 60° 90°
AB B BC P
1
AC H AB B sin θ 0 1/ 2 3 /2 1
sec A = = , cot A = = 2
AB B BC P 3
cos θ 1 1/ 2 1/ 2 0
By Pythagoras theorem, H 2 = B2 + P2 2
1
4 tan θ 0 1 3 Not defined
Example 1 In a ∆ABC right angled at B, if sin A = , then find 3
5 cot θ Not defined 3 1 1/ 3 0
the value of tan A. sec θ 1 2/ 3 2 2 Not defined
(a) 4/3 (b) 3/4 cosec θ Not defined 2 2 2/ 3 1
(c) 5/3 (d) 4/5
496 Study Package NTSE

Sign of Trigonometric Ratios in Quadrants ( ) We know that, sin(90° − θ) = cosθ


and sec (90° − θ) = cosecθ
The two axes divide the plane into four sections is called
3 sin 32° sec 42° 3 sin (90° − 58° ) sec (90° − 48° )
‘quadrants’. ∴ − = −
cos 58° cosec 48° cos 58° cosec 48°
Quadrants
π 3 cos 58° cosec 48°
= − = 3 −1= 2
2
= 90° cos 58° cosec 48°
Quadrant II Quadrant I
sin/cosec All positive Example 4 Find the value of
positive sin35°⋅ cos 46°⋅ tan 45 ⋅ sec 55° ⋅ cosec 44°.
π = 180° 0° or 360° (a) 2 (b) 0 (c) 3 (d) 1
Quadrant III Quadrant IV ( ) We know that,
tan/cot positive cos/sec positive sec (90 ° − θ) = cosec θ and cosec (90 ° − θ) = sec θ
3π ∴ sin 35° × cos 46° × 1 × sec(90° − 35° ) cosec (90° − 46° )
(Q tan 45° = 1)
= 270°
2
Tabel for Complementary and Supplementary Angle = sin 35° × cos 46° × 1 × cosec 35° × sec 46°
of Trigonometric Ratios 1 1
= sin 35° × cos 46° × ×
sin 35° cos 46°
Terms/Angle 90° − θ 90° + θ 180° − θ 180°+ θ  1 1 
A = Angle  using sec θ = , cosec θ = 
 cos θ sin θ 
sin A cos A cos A sin A −sin A =1×1=1
cos A sin A − sin A − cos A − cos A
tan A cot A − cot A − tan A tan A Example 5 What is the value of
cosec A sec A sec A cosec A − cosec A (cos2 30 ° − sin2 45 ° + 2 tan2 30 ° + 1) ?
sec A cosec A − cosec A − sec A − sec A (a) 23/12 (b) 12/23 (c) 15/17 (d) 17/15
cot A tan A − tan A − cot A cot A ( ) By trigonometry table, we get
cos 30° − sin 2 45° + 2 tan 2 30° + 1
2

Trigonometric Identities 2
 3  1 
2
 1 
2

sin2 θ + cos2 θ = 1 =  −  + 2  +1
 2  3
l

 2 
l
sec2 θ − tan2 θ = 1 for 0 ° ≤ A < 90 °
3 1 2 9 − 6 + 8 + 12 23
l
cosec2 θ − cot2 θ = 1 for 0 ° < A ≤ 90 ° = − + + 1= =
4 2 3 12 12

Formulae to Remember Maximum and Minimum Value of the


sin θ Trigonometrical Functions
l
tan θ =
cos θ Function Minimum Maximum
l
cos 2 θ = cos2 θ − sin2 θ = 2 cos2 θ − 1 = 1 − 2 sin2 θ
sin θ −1 1
1 − tan2 θ
l
sin 2θ = 2 sin θ cos θ = cos θ −1 1
1 + tan2 θ |sin θ| 0 1
l
sin( A ± B) = sin A cos B ± cos A sin B |cos θ | 0 1
l
cos ( A ± B) = cos A cos B m sin A sin B |sin θ | 1 —
tan A ± tan B
l
tan( A ± B) = |cosec θ| 1 —
1 m tan A tan B
a sin θ ± b cos θ − a2 + b 2 a2 + b 2
2 tan A
l
tan 2 A = a sin θ ± b cos θ + c
1 − tan2 A − a2 + b 2 + c − a2 + b 2 + c
l
sin 3 θ = 3 sin θ − 4 sin3 θ
Example 6 Find the minimum value of 3 cos θ − 4 sin θ.
l
cos 3 θ = 4 cos3 θ − 3 cos θ
3 tan θ − tan3 θ (a) – 4 (b) – 3 (c) – 5 (d) 0
tan3 θ =
( ) We know that, the minimum value of a cos θ − b sin θ
l

1 − 3 tan 2 θ
is − a2 + b2 .
3 sin 32 ° sec 42 °
Example 3 Find the value of − .
cos 58 ° cosec 48 ° ∴ Minimum value of 3 cos θ − 4 sin θ

(a) 0 (b) 1 (c) 2 (d) 4 =− 32 + 42 = − 9 + 16 = − 5


Target Exercise
Elementary Level Questions
a a sin θ − b cos θ a2 − b2
1. If cotθ = , then the value of is 11. If cosec θ = , then cot2 θ is equal to
b a sin θ + b cos θ a2 + b2
a2 − b 2 a−b − 4 a2 b 2 4 a2 b 2
(a) (b) (a) (b)
a2 + b 2 a+ b (a + b )
2 2 2
(a + b 2 )2
2

(c) 0 (d) 1 2 ab
(c) (d) None of these
2. The value of tan 1° tan 2° … tan 89° is ( a2 + b 2 )
(a) 0 (b) 1
(c) − 1 (d) None of these
12. If a cos θ + b sin θ = m and b cos θ − a sin θ = n, then
what will be the value of a2 + b2 ?
3. 2 (sin 45° − cos 60° ) + (tan 30° × cot 45° ) is equal
4 3 2 2
(a) m2 + n2 (b) m2 − n2
to
48 − 19 (c) mn (d) m + n
(a) (b)
19 48 13. The value of
7
(c) 1 (d) sin 64 ° cosec 47 ° tan 25° × tan 65°
12 × + is
cos 26° sec 43 ° tan2 30°
sin A − 2 sin 3 A
4. is equal to (a) 1 (b) 3 (c) 4 (d) 2
2 cos 3 A − cos A
(b) tan A − cot A x sin 45° × cos 30° 3 sin 45° + 2 cos2 30°
2 2 2
(a) cot A 14. If = , then
(c) tan A (d) cos A 4 cos2 60° sin2 90° + 2 cos2 45°
what is the value of x?
5. (1 + cot θ − cosec θ) (1 + t an θ + sec θ) is equal to (a) 0 (b) 1
(a) 1 (b) − 1 (c) 3 (d) 4
(c) 2 (d) None of these
15. If 180° < θ < 270°, then the value of cos θ − sin θ is
6. sin(60° + θ) + sin(60° − θ) is equal to (a) sin θ − cos θ
(a) 0 (b) 3cosθ (b) sin θ + cos θ
(c) 3 (d)
3 (c) cos θ − sin θ
2 (d) cos 2θ
cos 42° + sin 42° 3
7. is equal to 16. If cos θ = , then the value of sin θ is
cos 42° − sin 42° 5
2 4
(a) s ec 84° + tan 84° (a) (b)
5 5
(b) s ec 84° − tan 84° 2
(c) − sec 84° − tan 84° (c) (d) Cannot be determined
3
(d) − s ec 84° + tan 84°
2
2 17. sin4 45° + sec2 45° − 2 tan2 60° is equal to
8. If cos( A − B) = and tan A tan B = 3, then 3
5 23 1 23 1
1 (a) (b) (c) − (d) −
(a) sin A sin B = 4 (b) cos A cos B = 6 2 6 2
10
(c) cos( A + B) =
3
(d) sin A cos B = 1 18. If tan2 θ = cos2 φ − sin2 φ, then which of the following
5 is true?
1
9. cos ( − θ) is equal to (a) cos 2 φ cos 2 θ = (b) cos 2 θcos 2 φ = 2
2
(a) cosθ (b) − cosθ
(c) cos 2 θ cos 2 φ = 1 (d) None of these
(c) cos2θ (d) None of these
5 19. sin 12° × cos 78 ° + sin 78 ° × cos 12° is equal to
10. If θ is acute and tan θ = , then the value of
12 (a) 1 (b) 0
cos θ + sin θ is (c) 2 (d) None of these
12 18
(a) (b) 20. If a right ∆ABC right angled at C and cos A = sin B,
13 5
17 then which of the following is true?
(c) (d) None of these (a) A = B (b) A > B (c) A < B (d) A = − B
13
498 Study Package NTSE

High Skill Questions


1. If cos (90° − θ) = ( 2 − 1) sin (90° − θ), then what is the 8. If sec 5 A = cosec ( A + 30° ), where 5A is an acute
value of cot θ ? angle, then find the value of A.
(a) 2 (b) 2 +1 (c) 2 −1 (d) 3 (a) 10° (b) 15°
1 (c) 5° (d) 30°
2. Find the value of (cos2 30° − sin2 45° )
3 9. If θ = 30°, then the value of
2
+ 2 (sin 60° − cos 45° ) + tan2 60°.
4 4
cosec 2 θ ⋅ cot θ − sin 2 θ ⋅ sec 2 θ is
3 (a) 0 (b) 1
65 4 24 26
(a) (b) (c) (d) (c) 2 − 3 (d) None of these
24 3 65 54
1 1
3. If cos θ − sin θ = 2 sin θ, then what is the value of 10. If cos θ = − and cotθ = − , then the value of sin θ is
2 3
cos θ + sin θ ? 1 − 3
θ cosec θ (a) (b)
(a) 2 cos (b) 3 2
2 2
(c) sin θ (d) cos θ −1 3
(c) (d)
3 2
4. What is the value of 1 − cos2 α − cos4 α, if

sin α + sin2 α = 1 ? 11. sec is equal to
(a) 0 (b) 1 3
(c) 2 (d) None of these 1
(a) 1 (b) (c) 0 (d) 2
m 2
5. If cotθ = , then what is the value of
n 12. What is the relation between x and y, if
m cos θ − n sin θ cot θ + cosec θ = x and cosecθ − cot θ = − y ?
?
m cos θ + n sin θ (a) xy = 0 (b) xy = 1
(c) xy2 = 1 (d) xy = − 1
m + n
2 2
m −n2 2
m+ n m− n
(a) (b) (c) (d)
m2 − n2 m2 + n2 m− n m+ n 13. If 2 cos2 α − 1 = 0 and α lies in III quadrant, then the
5π  3π value of sin α × tan α is
6. The value of sin   − cos   is 1 1
 4  4 (a) − 1 (b) − (c) (d) None of these
2 2
(a) 0 (b) 1 cos 2 A − 1 sin 2 A 1
(c) 2 (d) None of these 14. + × is equal to
sin A cos 2 A + 1 sec A
7. If tan ( A + B) = 3 and cos A = 1, then ∠B is (a) 2 sin2 A (b) cos A
(a) 0° (b) 30° (c) 45° (d) 60° (c) sin2 A (d) None of these

Hints and Solutions


Elementary Level Questions
a 3. (d) 2 (sin4 45° − cos 3 60° ) + (tan2 30° × cot 2 45° )
1. (c) If cot θ =
b
a sin θ − b cos θ a − b cot θ  1  4  1  3   1  2 2
∴ = [Qon dividing by cos θ] = 2   −  +   × (1) 
a sin θ + b cos θ a + b cot θ   2  2  
 3  

2 − 1 1
= 2  −  +  ×  = 2 × 
a 1 1 1 1
a−b× +
= b = a−a =0  4 8   3 1  8  3
a a+ a
a+ b× 1 1 7
b = + =
4 3 12
2. (b) tan1° tan2 ° tan 3° ... tan 89° sin A − 2 sin3 A sin A (1 − 2 sin2 A)
4. (c) =
= tan (90° − 89° ) tan (90° − 88° )... tan (90° − 46° ) 2 cos A − cos A
3
cos A (2 cos 2 A − 1)
× tan 45° × tan 46° × ......... × tan 89° (using identity, 1 − 2 sin2 A = 2 cos 2 A − 1 = cos 2 A)
= cot 89° cot 88° ....... cot 46° × tan 45° × tan 46° × ... × tan 89° sin A × cos 2 A
=
Now, using cot θ × tan θ = 1and tan 45° = 1, we get cos A × cos 2 A
tan 1° tan 2 ° ... tan 89° = 1 = tan A
Trigonometry 499
5. (c) (1 + cot θ − cosec θ)(1 + tan θ + sec θ) 5
10. (c) tan θ = ⇒ P = 5, B = 12
12
 cos θ 1   sin θ 1 
= 1 + −  1 + +  By Pythagoras theorem,
 sin θ sin θ   cos θ cos θ 
⇒ H= P 2 + B2 = (5)2 + (12 )2
 sin θ + cos θ − 1  cos θ + sin θ + 1
=   = 25 + 144 = 169 = 13
 sin θ  cos θ 
B 12 P 5
1 cosθ = = and sinθ = =
= (sin θ + cos θ − 1) × (cos θ + sin θ + 1) H 13 H 13
sin θ cos θ 12 5 17
Now, cos θ + sinθ = + =
(sin θ + cos θ)2 − (1)1 13 13 13
= [Q(a + b )(a − b ) = a2 − b 2 ]
sin θ cos θ
a2 − b 2
sin2 θ + cos 2 θ + 2 sin θ cos θ − 1 11. (a) cosec θ = ,
= a2 + b 2
sin θ cos θ
[Q(a + b )2 = a2 + 2 ab + b 2 ] Q cosec 2θ − 1 = cot 2 θ (identity)
2
2 sin θ cos θ  a2 − b 2   a2 − b 2 
= =2 [Qsin2 θ + cos 2 θ = 1] ⇒  2  − 1 = cot 2 θ
sin θ cos θ 2 Qgiven, cosec θ = 2 
a + b   a + b2 
6. (b) sin (60° + θ) + sin (60° − θ) a4 + b 4 − 2 a2 b 2 − ( a4 + b 4 + 2 a2 b 2 )
⇒ = cot 2 θ
[using sin ( A ± B) = sin A cos B ± sin B cos A] (a2 + b 2 )2

= sin 60° × cos θ + cos 60° × sin θ + sin 60° × cos θ − 4a2 b 2
i .e., = cot 2 θ
− sin θ cos 60° (a2 + b 2 )2
 3 
= 2 (sin 60° × cos θ) = 2  × cos θ = 3 cos θ 12. (a) acos θ + b sinθ = m …(i)
 2 
and b cos θ − asinθ = n …(ii)
cos 42 ° + sin 42 ° cos 42 ° + sin 42 ° cos 42 ° + sin 42 °
7. (a) = × On squaring and adding Eqs. (i) and (ii), we get
cos 42 ° − sin 42 ° cos 42 ° − sin 42° cos 42 ° + sin 42 °
(acos θ + b sinθ)2 + (b cos θ − asinθ)2 = m2 + n2
cos 2 42 ° + sin2 42 ° + 2 cos 42 ° × sin 42 °
=
cos 2 42 ° − sin2 42 ° i .e., a2 cos 2 θ + b 2 sin2 θ + 2 ab sin θ cos θ
[Q(a + b )(a − b ) = a − b ] 2 2
+ b 2 cos 2 θ + a2 sin2 θ − 2 ab sin θ cos θ = m2 + n2
1 + sin 84°
= = sec 84° + tan 84° (a2 + b 2 ) cos 2 θ + (a2 + b 2 ) sin2 θ = m2 + n2
cos 84°
⇒ (a2 + b 2 )(cos 2 θ + sin2 θ) = m2 + n2
(using sin2 θ + cos 2 θ = 1, 2 cos θ sin θ = sin 2θ
i .e., a2 + b 2 = m2 + n2 [Qcos 2 θ + sin2 θ = 1]
and cos 2 θ − sin2 θ = cos 2θ)
sin 64° cosec 47 ° tan 25° × tan 65°
2 13. (c) × +
8. (b) cos ( A − B) = and tan A tan B = 3 cos 26° sec 43° tan2 30°
5
2 sin (90° − 26° ) cosec (90° − 43° )
⇒ cos A cos B + sin A sin B = = ×
5 cos 26° sec 43°
[Qcos( A − B) = cos A cos B + sin A sin B] tan (90° − 65° ) × tan 65°
+ 2
2 1  1 
⇒1 + tan A tan B = ×  
5 cos A cos B  3
[Qon dividing by cos A cos B] cos 26° sec 43°
= × + 3 cot 65° × tan 65°
2 cos 26° sec 43°
⇒ 1+ 3 =
5 × cos A cos B Qcot θ = 1 
= 1+ 3 = 4
1  tanθ 
∴ cos A ⋅ cos B =
10 x sin2 45° × cos 2 30° 3 sin2 45° + 2 cos 2 30°
14. (d) =
9. (a) cosθ is an even function, so cos(− θ) = cos θ 4 cos 60°
2
sin2 90° + 2 cos 2 45°
2
whereas sinθ and tanθ are odd functions as  1 
2
 3
sin(− θ) = − sinθ x×  ×  x×
1 3
×
 2  2  2 4 ⇒ 3x × 1 = 3x
and tan (− θ) = − tanθ LHS = =
2 1 8 8
4 ×   4×
1
[A function f( x) is said to be even, if f (− x) = f ( x) and said to be 2 4
odd, if f(− x) = − f( x)]
500 Study Package NTSE

2
 1 
2
 3 18. (a) tan2 θ = cos 2 φ − sin2 φ (given)
3×  +2×  1
3× +2 ×
3 3 3
+
 2  2  2 4 =2 2 = 3 On adding 1 to both sides, we get
RHS = =
 1 
2 1 2 2
(1)2 + 2 ×  1+ 2 × tan2 θ + 1 = cos 2 φ − sin2 φ + 1
 2
 2
⇒ sec 2θ = (cos 2 φ − sin2 φ + cos 2 φ + sin2 φ)
3x 3
Now, LHS = RHS, gives = ∴ x=4
8 2 [Qsin2 θ + cos 2 θ = 1]

15. (a) 180° < θ < 270° i .e., third quadrant where sinθ and cosθ both = 2 cos 2 φ
are negative. 1 Qsec θ = 1 
∴ cos 2 φ × cos 2 θ =
cos θ − sinθ = − cos θ − (− sinθ) = sinθ − cos θ 2  cos θ 
3 B 19. (a) sin 12 ° × cos 78° + sin 78° × cos 12 °
16. (b) Given, cos θ = =
5 H
= sin 12 ° × cos (90° − 12 ° ) + sin (90° − 12 ° ) × cos 12 °
Now, by Pythagoras theorem,
= sin 12 ° × sin 12 ° + cos 12 ° × cos 12 °
P= H2 − B2 = (5)2 − (3)2 = 25 − 9 = 16 = 4
= sin2 12 ° + cos 2 12 °
P 4
Hence, sinθ = = =1
H 5
2 20. (a) As ∆ABC is right angled at C and ∠A = ∠B
17. (c) sin4 45° + sec 2 45° − 2 tan2 60°
3 ⇒ ∠A = ∠B = 45 ° A
4
2  1 
= ×  + ( 2 ) − 2 × ( 3)
2 2
So, sin 45° = cos 45° is the only possibility for
3  2
sin A = cos B
2 1
= × + 2 −2 × 3 or cos A = sin B
C B
3 4
1 23 Thus, A = B is true.
= −4=−
6 6

High Skill Questions


1. (b) cos(90° − θ) = ( 2 − 1) sin(90° − θ) 4. (a) Given, sin α + sin2 α = 1 …(i)
i .e., sin θ = ( 2 − 1) × cos θ i .e., 1 − sin2 α = sin α ⇒sin α = cos 2 α
cos θ 1 2 +1 2 +1 ⇒ cos 4 α = sin2 α [Qon squaring both sides]
⇒ = × = = 2 +1
sinθ 2 −1 2 + 1 ( 2 )2 − (1)2 So, 1− cos α − cos 4 α = 1 − sinα − sin2 α
2

which is equal to zero. [using Eq. (i)]


∴ cot θ = 2 + 1
m
1 2
2. (a) (cos 30° − sin2 45° ) + 2(sin4 60° − cos 4 45° ) + tan2 60°
2 5. (b) Given, cotθ =
3 3 n
m cos θ − n sin θ m cot θ − n
=
1  3   1    3  4  1  4  2
2 2 Now, [Qon dividing by sinθ]
=   −  + 2   −   + × ( 3)
2 m cos θ + n sin θ m cot θ + n
3  2   2   2   2  3 m
    m× −n
= n [Qput the value of cotθ]
1  3 1  9 − 1 + 2 × 3
=  −  + 2  m×
m
+ n
3 4 2  16 4  3
n
=
1  1  + 2  5  + 2 = 1 + 5 + 2 = 2 + 15 + 48 = 65 m2 − n2
    = 2
3  4  16  12 8 24 24 m + n2
3. (b) cos θ − sinθ = 2 sinθ 5π π π
= sin  π +  = − sin = − sin 45° = −
1
6. (a)Q sin
4  4 4 2
On squaring both sides, we get
3π π π
= cos  π −  = − cos = − cos 45° = −
cos 2 θ + sin2 θ − 2 sin θ cos θ = 2 sin2 θ 1
and cos
4  4 4 2
⇒ 1 = 2 sin2 θ + 2 sin θ cos θ
5π   3π  =  − 1  −  − 1  = 0
∴ sin   − cos      
⇒ 2 sin θ (cos θ + sin θ) = 1  4   4   2  2
1
cos θ + sinθ = 7. (d) tan( A + B) = 3 ⇒ tan( A + B) = tan 60° ⇒ A + B = 60°
2 sinθ
cosec θ Q 1 = cosec θ and cos A = 1⇒cos A = cos 0° ⇒ A = 0°
∴ cos θ + sin θ =
2  sinθ  Thus, B = 60°
Trigonometry 501
8. (a) Given, sec 5 A = cosec ( A + 30° ) 13. (b) We have, 2 cos 2 α − 1 = 0
1
⇒ sec 5 A = sec [90°−( A + 30° )] [Qsec (90°− θ) = cosecθ] cosα = −
2
⇒ sec 5 A = sec (60°− A)
As α is in III quadrant.
⇒ 5 A = 60° − A [Qon comparing]
Here, B = − 1, H = 2 , P = − 1
⇒ 6 A = 60°
P −1
∴ A = 10° sin α = =
H 2
9. (c) Given, θ = 30° P
and tan α = = 1
Then, cosec 2 θ ⋅ cot θ − sin 2 θ ⋅ sec 2 θ B
−1 −1
= cosec 60°⋅cot 30° − sin 60° ⋅ sec 60° So, sin α × tan α = × 1=
2 2
2 3 3 2
= × − × =2 − 3 cos 2 A − 1 sin 2 A Q 1 = cos θ
14. (d) + × cos A
3 1 2 1
sin A cos 2 A + 1  sec θ 
1 1
10. (d) cos θ = − and cot θ = −
2 3 cos 2 2 A − 1 2 sin A cos A
= + × cos A [Qsin2θ = 2 sinθcos θ]
Since, cos θ and cot θ both are negative θ is in the II quadrant. sin A cos 2 A + 1
⇒ sin θ ≥ 0 cos 2 2 A − 1 + 2 sin2 A cos 2 A
=
Here, P = 3, B = − 1 and H = 2 sin A(cos 2 A + 1)
P 3
So, sin θ = = (2 cos 2 A − 1)2 − 1 + 2 sin2 A cos 2 A
H 2 = [Qcos 2θ = 2 cos 2 θ − 1]
sin A(cos 2 A + 1)
7π π π
11. (d) sec = sec  2 π +  = sec = sec 60°
3  3 3 4 cos 4 A + 1 − 4 cos 2 A − 1 + 2 sin2 A cos 2 A
=
=
1
=
1
=2 sin A (cos 2 A + 1)
cos 60° 1 / 2
4 cos 4 A − 4 cos 2 A + 2 sin2 A cos 2 A
=
12. (d) cotθ + cosec θ = x and cosec θ − cot θ = − y sin A (cos 2 A − sin2 A + sin2 A + cos 2 A)
i .e., cotθ − cosec θ = y
2 cos 2 A (2 cos 2 A + sin2 A) − 4cos 2 A
cos θ 1 1 + cos θ =
i .e., + = = x sin A (2 cos 2 A)
sinθ sinθ sinθ
cos θ 1 cos θ − 1 2 cos 2 A(2 cos 2 A + sin2 A − 2 )
and − = = y =
sinθ sinθ sinθ 2 sin A cos 2 A
1 + cos θ 1 − cos θ  2 (cos 2 A − 1) + sin2 A
∴ x× y= × −   =
sinθ  sinθ  sin A
 1 − cos 2 θ  sin2 θ − 2 sin2 A + sin2 A −sin2 A
= −  
 = − = −1 = = = − sin A
 sin θ  sin2 θ
2
sin A sin A

⇒ xy = − 1
502 Study Package for NTSE

Chapter

16
Height and Distance

One of the important application of trigonometry is in P B P


Review sin q = , cos q = , tan q = ,
finding the height and distance of the point which are H H B
not directly measurable. This is done with the help of H H B
trigonometric ratios. cosec q = , sec q = , cot q =
P B P
● Line of sight A line of sight is drawn from the eye of
an observer to the point, where the object viewed by the

H
Perpendicular

yp
observer.

ot
en
=P
C'

us
A

e
β

=
H
θ
90°
t
gh
Si

Base = B
of
ne
Li

α
Example 1 From a point on the ground, Mohan observed
the angle of elevation of the 10th floor of a building to be 30°. If
C B
Mohan was 60 3 ft away from the bottom of the building, what
● Angle of elevation The angle of elevation of the was the height of the building?
point viewed is the angle formed by the line of sight (a) 60 ft (b) 30 ft (c) 25 ft (d) 35 ft
with the horizontal when the point being viewed is Solution (a) Let AB be the height of
above the horizontal level. the building upto 10th floor and C A
● Angle of depression When the line of sight is be the position of Mohan.
below the horizontal level, then the angle so formed by AB P
We know that, = = tan q
the line of sight with the horizontal is called the angle of BC B
depression. Here, q = 30°
90°
30°
B C
Now, Ð ACB = a is the angle of elevation of point A Þ
AB
= tan 30° =
1 60√3
when observed from point C. 60 3 3
and Ð C ¢AC = b is the angle of depression of point C \ AB =
60 3
= 60 ft = height of the building
when observed from point A. 3
Since, AC ¢ || BC, ÐC ¢AC = ÐACB Example 2 From a hot air balloon, a boy ‘ q ’ found the
i.e., a =b (alternate angles) angle of depression of a point on the ground to be 45°. If the
i.e., Angle of elevation = Angle of depression boy is 90 ft above the ground in a vertical line, at how need
As AB is perpendicular to BC, we apply trigonometric distance is he away from the point horizontally?
ratios to calculate the lengths of the missing sides. (a) 60 ft (b) 30 ft (c) 90 ft (d) 120 ft
Height and Distance 503
Solution (c) Let B be the position of the hot air balloon and 20 3 20 ( 3 + 1) 3
i.e., h= =
P be the point on the ground observed by the boy from the 3 -1 2
balloon, we know BA = 90 ft = distance of the balloon from = 10 ( 3 + 1) 3 m
the ground.
B Thus, the height of the hill = 10 ( 3 + 3 ) m
45°
Example 4 A ladder which rests against a vertical wall makes
30° angle with the ground. If the ladder is 20 ft long, what is
90 ft
the height of the wall?
90° (a) 10 2 m (b) 10 m (c) 5 2 m (d) 10 3 m
45°
A P Solution (b) Let PR be the ladder which is 20 ft long and PQ
is
We need to calculate the distance of the balloon from the
point of observation in horizontal line i.e., AP. P
BA P
In DBAP, = = tan q = tan 45° 20 ft
AP B h
90
i.e., = 1 Þ AP = 90 ft 90°
AP 30°
Q R
Thus, horizontal distance of the boy on balloon from a point
on the ground is 90 ft. \ PQ = h (let)
Example 3 An observer in a boat found the angle of elevation PR = 20 ft
of the top of the hill to be 60°. As he moved 20 m away from the In right angled triangle PQR.
hill, the angle of elevation changed to 45°. What is the height of
PQ
the hill? sin 30° =
PR
(a) 10 ( 3 + 3 ) m (b) 10 3 m
1 h
= Þ h = 10 m
(c) 10 m (d) None of these 2 20
Solution (a) Let AB be the height of the hill and D be the
Hence, the height of wall is 10 m.
lateral position of the boat, whereas C is the position after it
has travelled 20 m away from the hill. î Note Angle of elevation º Angle of depression triangle with
Let the height of hill = AB = h m 90°, 45° and 45° angles have opposite sides a 2, a and a,
A respectively.
AB P
In DABD, = = tan q
BD B
45°
h
i.e., = tan 60° = 3 a√2
BD a
h
Þ BD = ...(i) 90°
3 60° 45° 45°
AB P B D C a
Now, in DABC, = = tan45°
BC B 20 m Triangle with angle 90°, 60° and 30° have opposite sides 2a,
h a 3 and a, respectively.
Þ =1
BD + 20
Þ h = BD + 20
h 60° 2a
Now, using Eq. (i), we get h = + 20 a
3
90°
Þ h ( 3 - 1) = 20 3 30°
a√3
Target Exercise
Elementary Level Questions
1. A 15 m high pole has ‘x’ m long shadow on the ground. 8. From the mid-point of the line joining towers AB and
If the angle formed by the pole on the ground was 30°, CD the angle of elevation of the top of the towers is
what is the value of x? 30° and 60° respectively, then the ratio of the heights
(a) 15 m (b) 15 3 m of the towers AB and CD is
(c) 30 m (d) 15 2 m (a) 3 : 1 (b) 1 : 3 (c) 2 : 3 (d) 1 : 2
2. A building makes an angle ‘q’ degree with the point on 9. A balloon tied with a 12 m long rope make 45° angle
ground has its height equal to its distance from the with the ground. Find the height of the balloon.
point on the ground. What is the value of q? (a) 6 m (b) 6 2 m (c) 6 3 m (d) 18 m
(a) 30° (b) 60°
(c) 45° (d) 120° 10. What will be the angle of elevation of sun at the
instant, when the length of the shadow of a building
3. A man standing on sea shore observe the angle of is 1 / 3 times its height?
elevation of the top of a tree to be 60°, if the person is (a) 30° (b) 45° (c) 60° (d) 90°
16 3 m away from the base of tree. What is the height
of the tree? 11. From a point on the ground, the angle of elevation of
(a) 16 m (b) 32 m the 7th and the 12th floors of a building are 30° and
(c) 48 m (d) None of these 45°, respectively. What is the height of the building
from the 7th floor to the 12th floor, if upto 7th floor
4. Shadow of a 18 m high tree is h m long. If the top of the building is 150 m height?
the tree makes 45° angle with the ground, then the (a) 150 m (b) 150 ( 2 + 1) m
value of h is (c) 150 ( 3 + 1) m (d) 150 ( 3 - 1) m

12. From a point 15 m from the base of a building, the


angle of elevation of the top of the building found to
be 45°, if after moving ‘x’ m further the angle of
elevation became 30°, then what is the value of x?
(a) 15 ( 3 + 1) (b) 15 ( 3 - 1)
h
(c) 15 2 + 15 (d) None of these
(a) 6 m (b) 18 2 m
(c) 18 m (d) 36 m 13. An eagle flying 100 m above the ground, makes an
angle of depression ‘ q ’ at a point on the ground. If the
5. A person standing 30 m away from the base of a cliff
shortest distance between the eagle and the point on
observes the angle of elevation of the top of the cliff to
the ground is 200 m, then the value of q is
be 60°. The height of the cliff is
(a) 30° (b) 45° (c) 60° (d) 90°
(a) 15 m (b) 15 2 m
(c) 30 m (d) 30 3 m 14. From the top of a lighthouse, the angle of depression
6. A ladder leaning against a vertical wall is twice the of two opposite points were found to be q and 90° - q,
length of the wall. The angle made by the foot of the respectively. If the distance between the points is
ladder on the ground is 50 m, then the height of the lighthouse is
(a) 60° (b) 30° (a) 25 sin q (b) 50 cos q (c) 25 cos 2 q (d) 25 sin 2 q
(c) 45° (d) 90° 15. A flag which is at the top of 150 m high building has
7. If Mr Ram’s shadow at afternoon is 3 times his angles of depression of its top and bottom at a point
height, then the angle of elevation of the shadow at on the ground 60° and 30° respectively, what is the
the top of Mr Ram’s is height of the flag?
(a) 60° (b) 30° (a) 300 m (b) 500 m
(c) 45° (d) 90° (c) 300 3 m (d) None of these
Height and Distance 505
High Skill Questions
1. There are two poles of heights 15 3 m and 20 3 m, 6. A cat saw a rat when it was exactly at the middle of
respectively. If at one point on the ground, where both a vertical tower. At that moment the angle of
the poles subtend angle of 60° each, then the gap elevation as observed by the cat was 30°. The cat ran
between the bases of two points is a certain distance towards the tower to chase the
(a) 15 (b) 15 3 (c) 35 (d) 45 rat, which is to the top of the tower. If the cat, which
is now 20 m away from the foot of the tower, finds
2. A while walking towards a tower at a constant speed, the angle of elevation of the rat to be 60°, what
observed that the angle of elevation of the top of the
distance cat run?
tower changed to 60° from 45°. How much more than
(a) 10 m (b) 10 2 m
will he take to reach of the tower?
(c) 10 3 m (d) None of these
(a) 5 min (b) 5 ( 3 - 1) min
(c) 5 3 min (d) None of these 7. Two poles at points A and B stand on a horizontal
place. C and D are two points between them. The
3. A ladder which rests against a virile wall make 60° angle of elevation of the top of the poles as observed
angle with the ground and its top is 6 3 ft above the
from point C are 30°, 60° and from point D are 60°
ground. The foot of the ladder is moved away from the
and 45°. If CD = 30 m, then AB is equal to
wall along the ground will its angle with the ground
(a) 15 (4 + 3 ) (b) 15 ( 3 + 3)
becomes 45°. How much is the distance by which the
(c) 15 (4 - 3 ) (d) None of these
ladder to moved along the ground?
(a) 6 3 (b) 6 ( 2 + 1) (c) 6 ( 3 - 1) (d) 6 ( 2 - 1) 8. A man who is 6 ft tall founds the angle of elevation of
the top of a tower to be 45°. If the angle of depression
4. The angle of elevation of a bird flying above an
at the foot of the tower as observed by the man is 30°,
aeroplane, as observed front the aeroplane is 30°. At
what is the height of the tower?
the same time the angle of elevation of the aeroplane as
(a) 6 3 (b) 6 ( 3 - 1)
observed from a point on the ground and vertically
(c) 6 ( 3 + 1) (d) 6 ( 2 + 1)
below the bird is 60°. If the shortest distance between
the bird and the aeroplane at that moment is 1500 m, 9. A man standing on a light house, observes the angle
then the height of the aeroplane above the ground is of elevation of a boat coming towards the light house
(a) 1000 m (b) 1200 m to be 60° from 30° in 15 min. How much time will the
(c) 4500 m (d) None of these boat take to reach the foot of the light house?
(a) 10 min (b) 7.5 min
5. A building which is 30 m high was observed from a (c) 8 min (d) None of these
point on the ground. Observer found the angle of
elevation of a point on the second floor of the building 10. A 50 m high pole which is above a 250 m high
which is 10 m above the ground same as the angle building observed by an observer standing at a
subtended by the rest of the building above the point P. height of 300 m. If the top of building and the top of
If the height of the observer is to be ignored, the pole subtend equal angle i.e., q at the observer,
appoximate distance between the observer and the then the horizontal distance of the observer from the
foot of the building is (take 3 = 1732
. ) pole is
(a)17.32 m (b) 20 m (a) 25 6 m (b) 25 3 m
(c) 21.21 m (d) None of these (c) 25 m (d) None of these
Hints and Solutions
Elementary Level Questions
1. (b) Let AB = 15 m = pole 6. (b) Let the angle made by the foot of the ladder on the ground
A be ‘q’ degree.
We know that, ladder = 2 ´ wall (using of the wall)
15 m i.e., AC = 2 AB A
In DABC,
90° AB
30° = sinq
B C AC
x
AB 1
In DABC, the angles are 90°, 60°, 30° opposite side are 2a, a 3 Þ = sinq =
and a, respectively. 2 AB 2
90° θ
Since, opposite to 30° is a = 15 m, opposite to 60°is x \ q = 30°
B C
= a 3 = 15 3 7. (a) Let PQ = Ram and QR be his shadow.
Thus, x = 15 3 m We know that, PQ = 3 QR
Let f be the angle of elevation of Mr Ram’s shadow.
2. (c) Given, AB = BC P
A

Height

90° θ
B C
90° θ
Q R
AB AB a
In DABC, = tanq Þ =1 Shadow
BC AB PQ
In DPQR, = tanf
Þ tanq = 1 Þ q = 45° QR
3. (c) Let AB = tree and M be the position of the man. A 3 QR
i .e., = tanf
In DABM, QR
AB Þ tanf = 3 Þ f = 60°
= tan60°
MB Hence, the angle of elevation = 60°
AB AB C
Þ = 3 8. (b) In DABE, tan30° =
16 3 BE
60° 90° A 30°
\ AB = 16 ´ 3 = 48 m M 1 AB BE
B = Þ AB =
16√3 3 BE 3 60°
4. (c) AB = tree = 18 m and shadow of the tree = h m CD
A In DECD, tan60° = Þ
ED 30° 60°
CD B E D
3= Þ CD = 3ED
ED
18 m
Q E is the mid-point of BD. So, BE = ED

45° \ Required ratio = AB : CD


90°
C B BE 1
= : 3 ED = : 3 = 1: 3
h 3 3
AB = h = 18 m 9. (b) Let AB be the height of the balloon.
Using 90° : 45° : 45° ratio, we get A
Thus, the shadow AB = h = 18 m
5. (d) Let the length of cliff = AB A
12 m
In D ABC,
AB
= tanq
BC 45° 90°
AB C
i .e., = tan 60° B
30 90° AB AB 1 12
60° In DABC, = sin 45° Þ = Þ AB =
AB = 30 3 m B C AC 12 2 2
Thus, length of cliff = 30 3 m 30 m =6 2 m
Height and Distance 507
Alternate Method 13. (a) Since, E be the position of the eagle, we know
Using 90°, 45° and 45° ratio, we get sides a 2, a and a, EB = 100 m and EC = 200 m
respectively. Side opposite to 90° = a 2 = 12 E = Eagle
Þ a=6 2
\ AB = a = 6 2 m
10. (c) Let AB = building and BC be its shadow and ‘q’ be the 200 m
100 m
angle of elevation.

A B C

EB
In D EBC, = sinq
θ 90° EC
C B 100 1
Þ = sinq Þ sin q = i .e., q = 30°
200 2
1
We know that, BC = AB 14. (d) Let AD be the light house of height h m.
3
AB We know that, BC = 50 m
Now, in DABC, = tan q
BC A
3BC
i.e., = tanq
BC
Þ 3 = tanq = 60° h
11. (d) Using 90°, 60° and 30°, we get in DDBC, 90°– θ
θ
a = 150, so BC = a 3 = 150 3 B C
A D
50 m

D In DABD,
a√3
BD
150 = cot q Þ BD = hcot q
45° AD
C 30°
B DC
a√3 In DADC, = cot (90° - q) = tan q Þ DC = h tanq
AD
Now, in DABC, using 90°, 45° and 45° triangle ratio, we get We know that, BD + DC = BC
BC = AB = 150 3, so the height of the building from the 7th to Þ h cot q + h tan q = 50
12th floor = AD = AB - DB = 150 3 - 150 = 150 ( 3 - 1) m
50 50 tan q 50 ´ sin q
Þ h= = = ´ cos 2 q
12. (b) In DABD, cot q + tan q sec 2 q cos q
A 50
= 50 cos qsin q = ´ 2 sin qcos q
2
(using, sin 2A = 2sin A cos A)
= 25 sin 2 q
45° 30°
B D C 15. (a) Let AB be the flag.
15 m xm A
BC
In DBCD, = tan q
AB AB CD
= tanq Þ = tan 45° = 1
BD 15
150 1
Þ AB = 15 m i.e., = tan 30° =
CD 3
AB
Now, in DABC, = tan q Þ CD = 150 3 B
BC
AC 60°
15 Now, in DACD, = tan 60° 150 m
Þ = tan 30° CD 30°
15 + x C
D
AC
1 i.e., = 3
i.e., 15 = (15 + x ) 150 3
3
Þ AC = 150 ´ 3 = 450
Þ x = 15 3 - 15 = 15 ( 3 - 1) Thus, AB = AC - BC = 450 - 150 = 300 m
508 Study Package for NTSE

High Skill Questions


1. (c) In DABC, side opposite to 60° = 20 3 m = AB, so side 4. (d) Let B be the position of bird and A be the aeroplane want
opposite to 30° = BC = 20. Similarly in DDEC, side opposite to the value of DC.
B
60° = 15 3 m = DE, so opposite to 30° = CE = 15 In DBDA,
1500
A DA 3
D Þ = cos 30° = 30°
BA 2 D A
60°
30

3
20√3 Þ DA = 1500 ´ = 750 3 m
°

15√3 2
DC
90°
60° 60°
Now, in DDAC, = tan60°
DA
B C E
\ DC = (750 3 ) 3 = 2250 m
By the ratio of 90°, 60° and 30° angles in triangle, PB C
5. (a) In DPBC, = tan a
BC = a = 20 and CE = a = 15 BC
Thus, gap between the poles = BE = BC + CE = 20 + 15 = 35 10
Þ = tan a ...(i)
2. (d) In DABD, BC
BD = a and AB = a 3 A
(using 90°, 60° and 30°, triangle angle, side ratio)
Now, in DABC, angles are 90° : 45° : A
20
45°, so AB = BC a 3
Thus, distance covered in 10 min
P
= DC = a 3 - a a√3
Distance a 3 - a 10 α
\ Speed of A = =
Time 10 60° 45° α
B D C B C
Thus, the time A will take to cover a
(BD = a) is AB
10 min Now, in DABC, = tan 2a
a√3 BC
a
30 2 tan a éQ tan 2 q = 2 tan q ù
Distance a 3 - a 10 3+1 i.e., = êë
= = ´ BC 1 - tan a 1 – tan2 q úû
Speed 10 3 -1 3+1
= 5 ( 3 + 1) min 2 æç
10 ö
÷
3. (d) Let AC = DE = Length of ladder è BC ø
= [Q from Eq.
100
AB 1-
In DABC, = sin60° (BC )2
AC
(i)]
6 3 3 20 ´ BC
Þ = Þ
30
=
AC 2 BC BC 2 - 100
Þ AC = 2 ´ BC = 12 = Ladder = ED
Þ 30 BC 2 - 3000 = 20 BC 2 Þ BC 2 = 300
[by using 90°, 60° and 30° triangle ratio]
BC \ BC = 10 3 = 10 (1732
. ) = 17. 32 m
and = cos 60°
AC 6. (a) Let C be the last position of cat and A be the position of rat.
Þ
BC 1
= We want to know the value of DC = distance ran by cat
12 2 A
12
Þ BC = =6 a
2 A
°
30

Now, in DEDB, E
BD E
Þ = cos 45° 60°
ED a
1 6 √3
Þ BD = 12 ´ =6 2 30° 60° 90°
2 B
D C
Thus, CD = Distance by which ladder 90° 60°
20m
In DEBD, if EB = a (let), DB = a 3
45°
moved along the ground D
B C
= BD - BC = (6 2 - 6) 6 (using 90°, 60° , 30° triangle ratio)
= 6 ( 2 - 1) m Now, in DACB, AB = 2 a
Height and Distance 509
As, E to the mid-point = side opposite to 60° AB
Now, in DABC, = tan45° = 1
2a BC
So, side opposite to 30° = BC =
3 Þ AB = BC = 6 3 ft
2a Thus, height of the tower = AD
20 =
3 = AB + BD
\ a = 10 3 =6 3+ 6
So, DC = BD - BC = 10 3 ´ 3 – 20 = 30 - 20 = 10 m = 6 ( 3 + 1) ft
7. (a) In DAEC, AC = x, AE = x 3 9. (b) In DLMO,
(using 90°, 60° and 30° triangle side ratio) MO = x
E F LM
\ tan60° =
MO
LM
x√3 y√3 Þ 3= Þ LM = x 3
MO
60° 30° 45° 60° L
A C D B
x 30 y
x√3
x√3 90°
60° 30°
Now, in DEDA, if EA = x 3 = Side opposite to 45°
M O N
We have, AD = x 3 x
15 min
Thus, x 3 - x = 30
Now, in DLMN,
30 30 3+1
x= = ´
3 -1 3 -1 3+1 LM = x 3 = opposite to 30°
So, (x 3 ) ´ 3 = 3x = opposite to 60°
30 ´ ( 3 + 1)
= = 15 ( 3 + 1) Distance covered by the boat in 15 min
( 3 )2 - (1)2
= ON = MN - MO = 3x - x = 2 x
Similarly DB = y
Distance 2 x
Speed of the boat = =
In DFDC, FB = y 3 Speed 15
Now, in DFCB, Now, time taken by the boat to reach the foot of the light house
FB = y 3 = Side opposite 30° i.e., to cover MO = x distance
Distance x 15
\ CB = ( y 3 ) ´ 3 = 3 y = Side opposite to 60°. = = = = 7.5 min
Speed 2x 2
So, 3 y - y = 30 15
Þ y = 15 10. (d) As the angle subtended by the top of the building and the
Thus, AB = x + 30 + y = 15 ( 3 + 1) + 30 + 15 top of the pole is ‘q’ degree each, we get q = 45°
A B
= 60 + 15 3 = 15 ( 4 + 3 )
θ
CE 1 θ
8. (c) In DCED, = tan30° = 50
DE 3 90° 300
θ
6 1 C D
i.e., = Þ DE = 6 3 ft = BC
DE 3
250
A

E F
BD
Now, in DBCD , BD = 50 and = tan q = tan 45°
DC
90° 45° C
B Þ BD = DC = 50
Horizontal distance of observer from the pole = AB = CD = 50m
6 ft
30° 90°
D E
510 Study Package for NTSE

Chapter

17
Statistics

Data It is not easy to answer the question looking at the


In our day-to-day life, we might come across choices written haphazardly. So, we arrange the data
information, such as in table using tally marks.
● Runs made by a batsman in the last 10 test matches. Table
● Number of wickets taken by a bowler in the last 10 Subject Tally marks Number of students
ODIs. Arts |||| || 7
● Marks scored by the students of your class in the
Mathematics |||| 5
Mathematics unit test.
Science |||| | 6
● Number of story books read by each of your friends etc.
English |||| 4
The information collected in all such cases is called
data. Data is usually collected in the context of a Total 22
situation that we want to study. A data is a collection
of numbers gathered to give some information. The number of tallies before each subject gives the
e.g., A teacher may like to know the average height of number of students who like that particular subject.
students in her class. To find this, she will write the This is known as the frequency of that subject.
heights of all the students in her class, organise the Frequency gives the number of times that a particular
data in a systematic manner and then interpret it entry occurs. The table made is known as frequency
accordingly. Sometimes, data is represented distribution table as it gives the number of times an
graphically to give a clear idea of what it represents. entry occurs.
From table, frequency of students who like English is 4
Organising Data and frequency of students who like Mathematics is 5.
Usually, data available to us is in an unorganised form
Frequency of students who like Arts is 7 and frequency
called raw data. To draw meaningful inferences, we of students who like Science is 6.
need to organise the data systematically.
Clearly, we can observe that Arts is the most liked
e.g., A group of students was asked about their subject and English is the least liked subject.
favourite subject.
The results were as listed below Grouping of Data
Arts, Mathematics, Science, English, Mathematics, The data regarding choice of subjects showed the
Arts, English, Mathematics, English, Arts, Science, occurrence of each of the entries several times. e.g., Arts
Arts, Science, Science, Mathematics, Arts, English, is liked by 7 students, Mathematics is liked by 5
Arts, Science, Mathematics, Science, Arts. students and so on. This information can be displayed
graphically using a pictograph or a bar graph.
Which are the most liked and the least liked subject? Sometimes, however, we have to deal with a large data.
Statistics 511
e.g., Consider the following marks (out of 50) obtained Directions (Ex. 1 and 2) The following bar-graph
in Mathematics by 60 students of Class III shows the number of students in a particular class of
21, 10, 30, 22, 33, 5, 37, 12, 25, 42, 15, 39, 26, 32, 18, school. Then, answer the following questions.
27, 28, 19, 29, 35, 31, 24, 36, 18, 20, 38, 22, 44, 16, 24, 80
10, 27, 39, 28, 49, 29, 32, 23, 31, 21, 34, 22, 23, 36, 24, 70 1 Unit length = 10 students

Number of students
36, 33, 47, 48, 50, 39, 20, 7, 16, 36, 45, 47, 30, 22, 17. 60 – – – – – – – – – –
If we make a frequency distribution table for each 50 – – – – – – – – – –
40 – – – – – –
observation, then the table would be too long, so for 30
convenience, we make groups of observations say, 20
0-10, 10-20 and so on and obtain a frequency 10
distribution of the number of observations falling in
each group. 2000 2001 2002 2003
Years
Thus, the frequency distribution table for the above
data can be as follows
Table Example 1 What is the number of students in the year
2003?
Groups Tally marks Frequency
(a) 70 (b) 60 (c) 50 (d) 40
0-10 || 2 Solution (b) By observing the given bar graph carefully, we
10-20 |||| |||| 10 can say that the total number of students in the year 2003
was 60.
20-30 |||| |||| |||| |||| | 21
Example 2 Is the number of students in the year 2002 twice
30-40 |||| |||| |||| |||| 19 that in the year 2000?
40-50 |||| || 7 (a) Yes
50-60 | 1 (b) No
(c) Can’t be detain
Total 60 (d) More information required

Data presented in this manner is said to be grouped and Solution (b) The number of students in the year 2002 = 50
the distribution obtained is called grouped frequency The number of students in the year 2000 = 30
distribution. Thus, the difference between number of students in both the
It helps us to draw meaningful inferences like years = 50 - 30 = 20
● Most of the students have scored between 20 and 30. Which is not twice figure of 30.
● Eight students have scored more than 40 marks out of Thus, the number of students in the year 2002 was not twice
50 and so on. that of in the year 2000.
● Each of the groups 0-10, 10-20, 20-30 etc., is called a
class interval (or briefly a class). Histogram
In the class interval 10-20, 10 is called the lower class Consider the grouped frequency distribution of the
limit and 20 is called the upper class limit. marks obtained by 60 students in Mathematics test.
Table
Data Representation Class interval Frequency
Raw data can be represented in a number of ways to
draw the inferences. Some of the forms of data 0-10 2
representation are bar-graph, pie-chart, etc. 10-20 10
20-30 21
Bar-Graph
Bars of uniform width can be drawn horizontally or 30-40 19
vertically with equal spacing between them and then 40-50 7
the length of each bar represents the given number. 50-60 1
Such method of representing data is called a bar
diagram or a bar graph. Total 60
512 Study Package for NTSE

22 21 Example 4 How many students watched TV for


20 19 less than 4 h?
18 (a) 30 (b) 28 (c) 22 (d) 34
Number of students

16 Solution (d) Number of students who watched TV for less


14 than 4 h
12
10 = 22 + 8 + 4 = 34
10
8
6
7
Frequency Polygon
4 The mid-points of upper sides of rectangles of a
2
2 1 histogram are joined to obtain a frequency polygon.
0 10 20 30 40 50 60
8
Marks of students
7
This data has been displayed graphically as in the D
above bar-graph. 6
C
Is this graph, in any way different from the bar graphs 5
which you have drawn in previous classes. Observe E
4
that, here we have represented the groups of B
observations (i.e. , class intervals) on the horizontal 3
axis. The height of the bars show the frequency of the F
2
class interval. To draw the histogram, generally we
don’t take any gap between the class intervals and the 1
bars, so here we have taken the given class intervals
0
as 0-10, 10-20, 20-30, 30-40, 40-50 and 50-60. A 0 20 40 60 80 100 G
The graphical representation of data in this manner is In above figure, ABCDEFG is a frequency polygon.
called a histogram.
Directions (Ex. 3 and 4) Study the given histogram Pie-Chart (Circle Graph)
carefully and answer the questions given below. Have you ever come across data represented in
The number of hours for which students of a particular circular form as shown below?
class watched television during holidays has been Time spent by a Age groups of people
child during a day in a town
shown through the given graph. 30
8h th
ou
50 thousand

32 s an
32 Sleep d
28 0-14 yr
4h Home 15-60 yr
Number of students

24 22 work
rs

School 60 yr
he

20
Play

and d
Ot

6h
16 above san
3h o u
12 3h th
8 8 20
8 (i) (ii)
6
4
4 These are called circle graphs. A circle graph shows
1 2 3 4 5 6 7 the relationship between a whole and its parts. Here,
Number of hours of TV watched per day the whole circle is divided into sectors. The size of each
sector is proportional to the activity or information it
Example 3 For how many hours did the maximum number represents.
of students watch TV?
Directions (Ex. 5-7) Study the given pie-chart
(a) (3-4) (b) (5-6) carefully and answer the question based on it.
(c) (4-5) (d) (6-7) Pie-chart shown below gives the expenditure (in
Solution (c) The maximum number of students watched percentage) on various items and savings of a family
TV for (4-5) h. during a month.
Statistics 513
House rent
10%
Education for
children 15%
Arithmetic Mean (AM)
Transport The most common representative value of a group of
5% data is the arithmetic mean or the mean. To
understand this in a better way, let us look at the
Food
Others
25% following example
20%
Two vessels contain 20 L and 60 L of milk,
respectively. What is the amount that each vessel
Savings Clothes would have, if both share the milk equally? When we
15% 10%
ask this question, we are seeking the arithmetic mean.
Example 5 On which item, the expenditure was maximum? In the above case, the average or the arithmetic mean
(a) Clothes (b) Food Total quantity of milk 20 + 60 80
(c) House rent (d) Others = = L= L = 40 L
Number of vessels 2 2
Solution (b) Expenditure is maximum on food.
Thus, each vessels would have 40 L of milk.
Example 6 Expenditure on which item is equal to the total The average or Arithmetic Mean (AM) or simply mean
savings of the family? is defined as follows
(a) House rent (b) Education for children Sum of all observations
Mean =
(c) Food (d) Others Number of observations
Solution (b) Expenditure on education for children is the Note Above method is only for find the mean of ungrouped data.
same (i.e., 15%) as the savings of the family. Example 9 A batsman scored the following number of runs
in six innings 36, 35, 50, 46, 60, 55.
Example 7 If the monthly savings of the family is ` 3000,
Calculate the mean runs scored by him in an inning.
what is the monthly expenditure on clothes?
(a) ` 1500 (b) ` 3000 (a) 46 (b) 47 (c) 50 (d) 60
(c) ` 2000 (d) ` 1800 Solution (b) Total runs = 36 + 35 + 50 + 46 + 60 + 55 = 282
Solution (c) 15% represents ` 3000. To find the mean, we find the sum of all the observations
3000 and divide it by the number of observations.
Therefore, 10% represents = ` ´ 10 282
15 Therefore, in this case, mean = = 47. Thus, the mean runs
6
= ` 2000 scored by batsman in an inning are 47.
Mean of Grouped Data
Range Sfi xi
Now, if given data is grouped data, then mean ( X ) =
The difference between the highest and the lowest Sfi
where, f1 , f2 , f3 , … are frequencies of observations and
observation gives us an idea of the spread of the
x1 , x2 , x3 , … are mean value of observations.
observations. This can be found by subtracting the
lowest observation from the highest observation. We Example 10 Given data shows marks of 32 students in an
call the result the range of the observation. examination. Mean of the marks will be
Example 8 Find the range of the following set of scores 25, Class interval 0-10 10-20 20-30 30-40 40-50
15, 23, 40, 27, 25, 23 and 42. Frequency 12 6 8 4 2
(a) 37 (b) 27
(a) 18.125 (b) 21.875 (c) 22.425 (d) 22.545
(c) 32 (d) 25
Solution (a)
Solution (b) Range = Highest observation
- Lowest observation Class interval Mean value ( x ) Frequency ( f ) f ´ x

= 42 - 15 = 27 0-10 5 12 60
10-20 15 6 90
Measures of Central Tendency 20-30 25 8 200
In many frequency distribution, the average value 30-40 35 4 140
generally lies in the central part of the distributions. 40-50 45 2 90
Such values are called measures of central tendency. Total Sf = 32 Sfx = 580
Following things are known as the measures of central Sfx 580
\ Mean ( X ) = = = 18.125
tendency. Sf 32
514 Study Package for NTSE

Median Mode
To find median of an individual series, arrange the Mode is that term of the frequency distribution whose
given numbers in ascending or descending order. If the frequency is maximum.
number of observations is Now, given data is grouped data, then
n + 1ö
(i) odd, the median is æç ÷ th term. Mode = l +
( f1 - f 0 )
´i
è 2 ø
(2 f1 - f 0 - f2 )
éænö æn ö ù
ê çè 2 ÷ø th term + çè 2 + 1÷ø th term ú Where, l = Lower limit of mode group
(ii) even, the median is ê ú.
2 f1 = Frequency of mode group
ê ú
êë úû f 0 = Frequency of proceding group of mode group
Now, if given data is grouped data, then f2 = Frequency of next group of mode group
æ n - Cö i = Difference of class interval
ç ÷
è ø
Median = l + 2 ´i Example 13 Following are the margins of victory in the
f
football matches of a league.
where, l = Lower limit of median group
1, 3, 2, 5, 1, 4, 6, 2, 5, 2, 2, 2, 4, 1, 2, 3, 1, 1, 2, 3, 2, 6, 4, 3,
n = Sum of frequencies 2, 1, 1, 4, 2, 1, 5, 3, 3, 2, 3, 2, 4, 2, 1, 2
f = Frequency of median group Find the mode of this data.
C = Cumulative frequency of preceding group of (a) 1 (b) 2
median group (c) 3 (d) 4
i = Difference of class interval Solution (b) Let us put the data in a tabular form
Example 11 Find the median of the numbers 24, 36, 46, Margins of victory Tally marks Number of matches
17, 18, 25, 35.
1 |||| |||| 9
(a) 23 (b) 24
(c) 25 (d) 26 2 |||| |||| |||| 14
Solution (c) Arranging the data in ascending order, we get 3 |||| || 7
17, 18, 24, 25, 35, 36, 46.
Median is the middle observation. Therefore, 25 is the 4 |||| 5
median. 5 ||| 3
Example 12 The median from given grouped data would be 6 || 2

Class interval Frequency Cumulative frequency Total 40

10-20 5 5 Looking at the table, we can quickly say that 2 is the ‘mode’,
20-30 7 12 since 2 has occured the highest number of times. Thus, most
30-40 6 18 of the matches have been won with a victory margin of 2
goals.
40-50 8 26
50-60 4 30 Example 14 The mode from given grouped data would be
(a) 30 (b) 35 Class interval 10-20 20-30 30-40 40-50 50-60
(c) 38 (d) 40
Frequency 5 7 6 8 4
Solution (b) Here, n = 30 = 15 < 18
2 2
(a) 42 (b) 43
So, median group is (30-40).
(c) 44 (d) 45
Here, l = 30, n = 30, f = 6, C = 12 and i = 10
Solution (b) Here, highest frequency is 8.
æ n - Cö æ 30 - 12ö
ç ÷ ç ÷ Hence, mode group is (40-50).
è2 ø è ø
\ Median = l + ´ i = 30 + 2 ´ 10
f 6 Here, l = 40, f0 = 6, f1 = 8, f2 = 4 and i = 10
(15 - 12) ´ 5 ( f1 - f0 )
= 30 + \ Mode = l + ´i
3 ( 2f1 - f0 - f2 )
3 ´5 ( 8 - 6)
= 30 + = 30 + 5 = 35 = 40 + ´ 10
3 ( 2 ´ 8 - 6 - 4)
Statistics 515
2 Example 15 If mode of a grouped data is 10 and mean is
= 40 + ´ 10
(16 - 10 ) 4, then median will be
2 ´ 10 (a) 1 (b) 4 (c) 6 (d) 8
= 40 +
6
Solution (c) Here, Mode = 10, Mean = 4
10
= 40 + = 40 + 3.333 We know the formula,
3
= 43.333 » 43 Mode = 3 ´ Median - 2 ´ Mean
Þ 10 = 3 ´ Median - 2 ´ 4
Relation between Mean, Median and Mode
Þ 10 = 3 ´ Median - 8
There is a mathematical relation between mean,
median and mode. It is shown below Þ 3 ´ Median = 10 + 8 = 18
18
Þ Median = =6
Mode = 3 Median - 2 Mean 3

Target Exercise
Elementary Level Questions
Directions (Q. Nos. 1-3) Study the given bar chart Directions (Q.Nos. 4-6) Observe the histogram and
carefully and answer the questions based on it. answer the questions given below.
Bar chart represents the production
(in lakh tonne) of wheat from 2005 to 2010. 7
7
140
Number of Girls of Class VII

130 6
130
120 5
120
110 4
110
100 4
100
3
90 3
80
80 2 2
70 65 2
60 1 1
1
50
40
30 0 125 130 135 140 145 150 155 160
20 Heights (in cm)
10
0
2005 2006 2007 2008 2009 2010
Years
4. Which groups contain minimum girls?
(a) (130-135) and (155-160)
1. For which of the following year, wheat production is (b) (125-130) and (155-160)
maximum? (c) (125-130) and (135-140)
(a) 2005 (b) 2006 (d) (135-140) and (155-160)
(c) 2008 (d) 2010
5. Which group contain maximum girls?
2. For which of the following year, wheat production is (a) (140-145)
minimum?
(b) (135-140)
(a) 2005 (b) 2007
(c) (130-135)
(c) 2009 (d) 2008
(d) (145-150)
3. What is the mean production for all the years?
6. How many girls have a height of 145 cm and more?
(a) 100.83 lakh tonne (b) 110.5 lakh tonne
(a) 6 (b) 7
(c) 98.72 lakh tonne (d) None of the above (c) 5 (d) 2
516 Study Package for NTSE

Directions (Q. Nos. 7-9) Study the given information 7. If 20 people liked classical music, how many young
carefully and answer the questions based on it. people were surveyed?
(a) 100 (b) 250 (c) 150 (d) 200
A survey was made to find the type of music that a certain
group of young people liked in a city. Following pie-chart 8. Which type of music is liked by the maximum number
shows the findings of this survey. of people?
(a) Light (b) Classical (c) Folk (d) Semi-classical
Semi-
classical 9. If total number of young people in that group is 1000,
20% Light then how many of them like folk music?
40% (a) 120 (b) 90 (c) 100 (d) 300
Classical
10% 10. The enrollments in a school during six consecutive
Folk years are as follows
30% 1555, 1670, 1750, 2013, 2540, 2820
Find the mean enrollment of the school for the period.
(a) 2058 (b) 1058 (c) 2032 (d) 1120

High Skill Questions


Directions (Q. Nos. 1-3) Study the given information 7. The median and mean values of the marks obtained
carefully and answer the questions based on it. by the students of a class are 46.67 and 45.5,
respectively. Find out the mode of the marks.
The marks (out of 100) obtained by a group of students
(a) 60.00 (b) 58.58
in a science test are 85, 76, 90, 85, 39, 48, 56, 95, 81 and (c) 49.01 (d) 40
75.
8. If the mean and median are 25 and 28 respectively,
1. Find the highest and the lowest marks obtained by then find the value of the mode.
the students. (a) 35 (b) 34 (c) 36 (d) 134
(a) 48, 56 (b) 95, 39 (c) 56, 48 (d) 39, 95
9. Mean of median, mode and range of the data
2. Find the range of the marks obtained. 1, 2, 3, 3, 2, 5, 6, 2, 2, 1, 4, 6, 6, 5 is
(a) 52 (b) 51 (c) 56 (d) 48 (a) 7/3 (b) 10/3
3. Find the mean marks obtained by the group. (c) 11/3 (d) 5/3
(a) 73 (b) 71 (c) 57 (d) 74 10. The marks obtained by 30 students of Class X of a
Directions (Q. Nos. 4-6) Study the given information certain school in a Mathematics paper consisting of
carefully and answer the questions based on it. 100 marks are presented in table below
The rainfall (in mm) in a city on 7 days of a certain week was
recorded as follows. Class interval 10-25 25-40 40-55 55-70 70-85 85-100
Number of
Day Mon Tue Wed Thur Fri Sat Sun students 2 3 7 6 6 6
Rainfall (in mm) 0.0 12.2 2.1 0.0 20.5 5.5 1.0
Find the mean of the marks obtained by the students.
4. Find the range of the rainfall in the above data. (a) 62 (b) 60 (c) 63 (d) 65
(a) 10.5 (b) 12.5 11. An NGO working for welfare of cancer patients,
(c) 20.5 (d) 13.5 maintained its records as follows
5. Find the mean rainfall for the week. Age of patients (in years) 0-20 20-40 40-60 60-80
(a) 5.9 (b) 5.0
Number of patients 35 315 120 50
(c) 5.5 (d) 4.5
6. For how many days the rainfall was less than the Find mode.
mean rainfall? (a) 31 (b) 31.79
(a) 4 (b) 5 (c) 2 (d) 1 (c) 32 (d) 32.5
Hints and Solutions
Elementary Level
1. (c) Wheat production is maximum in the year 2008. 7. (d) Let the total number of people = x
2. (c) Wheat production is minimum in the year 2009. By given condition, 10% of x = 20
Sum of production 20 ´ 100
3. (a) Required mean production = Þ x= = 200
Sum of years 10
80 + 110 + 100 + 130 + 65 + 120 8. (a) Light music is liked by the maximum number of people 40%.
=
6 9. (d) The number of young people who like folk music = 30% of
605 30
= = 100.83 lakh tonne 1000 = ´ 1000 = 300
6 100
4. (b) Groups (125-130) and (155-160) contain minimum girls. 10. (a) Required mean
5. (a) The group (140-145) contain maximum girls. 1555 + 1670 + 1750 + 2013 + 2540 + 2820
=
6
6. (b) Number of girls having the height of 145 cm and more 12348
= = 2058
= 4+ 2 + 1= 7 6

High Skill Questions


1. (b) Highest marks = 95 10. (a) Let us make the following table for the given data.
and lowest marks = 39 Class interval Frequency (f ) x (Class marks) fx
2. (c) Range of marks = 95 - 39 = 56 10-25 2 10 + 25 35.0
= 17.5
Sum of marks 2
3. (a) Mean of marks = 25-40 3 25 + 40 97.5
Total students = 32.5
2
85 + 76 + 90 + 85 + 39 + 48 + 56 + 95 + 81 + 75 730 40 + 45
= = = 73 40-55 7 = 47.5 332.5
10 10 2
55-70 6 55 + 70 375.0
4. (c) Range = Highest rainfall - Lowest rainfall = 62.5
2
= 20. 5 - 0. 0 = 20. 5 70-85 6 70 + 85 465.0
= 77.5
5. (a) Mean rainfall 2
0.0 + 12.2 + 2.1 + 0.0 + 20.5 + 5.5 + 1.0 41.3 85-100 6 85 + 100 555.0
= = = 5.9 = 92.5
7 7 2

6. (b) Required number of days = 5 (Monday, Wednesday, Here, S f = 2 + 3 + 7 + 6 + 6 + 6 = 30


and S f x = 35.0 + 97 .5 + 332 .5 + 375.0 + 465.0 + 555.0 = 1860.0
Thursday, Saturday and Sunday)
On putting the values of S fx and S f in the formula,
7. (c) Median = 46.67 and Mean = 45.5
x = S fx , we get x = 1860.0 = 62
\ Mode = 3 Median - 2 Mean Sf 30
= 3 ´ 46.67 - 2 ´ 45. 5 = 49.01 Hence, the mean marks obtained by the students are 62.
8. (b) Mode = 3 ´ Median - 2 ´ Mean 11. (b) Here, maximum frequency is 315 and the class corresponding
to this frequency is 20-40. So, the modal class is 20-40.
= 3 ´ 28 - 2 ´ 25 = 34
Age of patients (in years) 0-20 20-40 40-60 60-80
9. (b) For median, 1, 1, 2, 2, 2, 2, 3, 3, 4, 5, 5, 6, 6, 6
3+ 3 Number of patients 35 = f0 315 = f1 120 = f2 50
\ Median = =3
2 \ l = 20, f1 = 315, f0 = 35, f2 = 120 and i = 20
Mode = 2 [Q 2 occurs most frequently] æ f -f ö
Range = Maximum value – Minimum value Now, mode = l + ç 1 0 ÷ ´ i
è 2 f1 - f0 - f2 ø
= 6 - 1= 5
315 - 35
Sum of terms = 20 + æç ö ´ 20
÷
\ Required mean = è 2 ´ 315 - 35 - 120 ø
Number of terms 280
= 20 + ´ 20 = 20 + 1179
. = 3179.
2 + 5 + 3 10 475
= =
3 3 Hence, average age of maximum number of patients is 31.79.
518 Study Package for NTSE

Chapter

18
Permutations and
Combinations
Fundamental Principle of Permutations
Counting (FPC) A permutation is an arrangement in a definite order of
a number of distinct objects taken some or all at a time.
If there are m different ways of occurrence of an event
and n different ways of occurrence of another event, The number of permutations of n different objects
then total number of ways of occurrence of the event is taken r at a time, when 0 ≤ r ≤ n and the objects do not
m × n. repeat is n Pr or P ( n, r).
n!
Factorial Notation Where, n Pr = P ( n, r) = ,0≤ r≤ n
( n − r) !
The notation ‘n ! ’ represents the product of first n n− 1
Example 2 Find n, if P3 : nP4 = 1 : 9.
natural numbers.
i.e., n ! = 1 ⋅ 2 ⋅ 3 ⋅ 4 K ( n − 1) n (n! is read as factorial n.) (a) 9 (b) 8 (c) 7 (d) 5

n ! = n( n − 1) ! = n( n − 1)( n − 2) ! and so on. Solution (a) Given, n − 1 P3 : n P4 = 1 : 9


0! = 1 ( n − 1 )! n! ( n − 1 )! n ( n − 1 )!
l ∴ : = 1: 9 ⇒ : = 1: 9
n! ( n − 1 − 3 )! ( n − 4 )! ( n − 4 )! ( n − 4 )!
l = n( n − 1)( n − 2) K ( r + 1)
r! ( n − 1 )! ( n − 4 )! 1 1 1
⇒ × = ⇒ = ⇒n = 9
n! ( n − 4 )! n ( n − 1 )! 9 n 9
l = n( n − 1)( n − 2) K ( n − r + 1)
( n − r )!
l Factorial of negative integers and fractions are not Formulae and Facts Related to
defined. Permutation
Example 1 How many 3-digits even numbers can be l The number of permutations of n different objects taken
r at a time, where repetition is allowed is nr .
formed from the digits 1, 2, 3, 4, 5, 6, if the digits can be
repeated? l The number of permutations of n objects, where p objects
n!
(a) 108 (b) 107 are of the same kind and rest are all different = .
p!
(c) 110 (d) None of these l The number of permutations of n objects, where p1
Solution (a) Even digits = 2, 4 , 6 objects are of one kind, p2 are of second kind, ..., pk are of
the kth kind and the rest, if any, are of different kind is
∴ Number of ways to fill the unit place = 3 n!
.
Number of ways to fill the ten place = 6 p1 ! p2 !K pk !
Number of ways to fill the hundred place = 6 Note l
When a particular object is always included is r ⋅n − 1 Pr − 1.
Hence, by FPC total number of ways = 3 × 6 × 6 = 108 l
When a particular object is never included is n −1
Pr .
Permutations and Combinations 519
Circular Permutations Formulae and Facts Related to Combination
If we arrange the objects along the circumference of a l The number of combinations of n different things taken r
curve, then permutations are known as circular at a time, where p particular thing never occur is n − pCr .
permutations. l The number of ways of selecting one or more than one in n
l The number of circular permutations of n distinct objects different objects is n C1 + nC2 + K + nCn = 2n − 1.
is ( n − 1)!. Here, anti-clockwise and clockwise order of Note l
n
C1 = n, n C n = 1 = nC 0
arrangements are considered as distinct permutations. l
n
C r = nC n − r for 0 ≤ r ≤ n.
l If anti-clockwise and clockwise order of arrangements l
If there are n persons in a room and they shake their hand
are not distinct. e.g., arrangement of beads is a necklace,
with others only once, then total number of shake hands
Arrangement of flowers in a garland etc., then number of
1 = nC 2.
circular permutations of n distinct items is ( n − 1)!.
2
Example 4 Find the number of 5 card combinations out of
Example 3 Three boys and three girls are to be seated a deck of 52 cards, if there is exactly one ace in each
around a round table, in a circle. Among them the boy X does combination.
not want any girl neighbour and the girl Y does not want any (a) 778315 (b) 778325
boy neighbour, then the number of such arrangements is
(c) 778320 (d) 778328
(a) 2 (b) 4
(c) 23 (d) None of these Solution (c) Number of ways selecting one ace from 4 aces
= 4C1
Solution (b) As shown in figure, 1, 2 and X are the three
boys and 3, 4 and Y are three girls. Boy X will have Number of ways selecting 4 cards out of 48 cards = 48
C4
neighbours as boys 1 and 2 and the girl Y will have Hence, by FPC, 5 cards out of 52 cards with exactly one ace
neighbours as girls 3 and 4.
48 × 47 × 46 × 45
X can be selected in 4 C1 × 48
C4 ways = 4 ×
24
1 2
 n n ( n − 1) ( n − 2) ( n − 3 ) 
Q C1 = n and n C4 =
3
 24 
4
48 × 47 × 46 × 45
Y = = 8 × 47 × 46 × 45
6
Number of ways in which 1 and 2 can be arranged
= 778320 ways
= 2 P2 = 2! = 2
Example 5 Four boys picked up 30 mangoes. In how many
Also, number of ways in which 3 and 4 can be arranged
ways can they divide them, if all mangoes be identical?
= 2 P2 = 2! = 2 33 33
(a) C4 (b) C2
Hence, required number of permutations = 2 × 2 = 4
(c) 5456 (d) None of these
Solution (c) Clearly, 30 mangoes can be distributed among
Combinations 4 boys such that each boy can receive any number of
Each of different selections made by taking some or all mangoes.
of a number objects, irrespective of their arrangements 33 !
Hence, total number of ways = 30 + 4 − 1C4 − 1 = 33C3 =
is called a combination. 3 ! 33 − 3 !
33 × 32 × 31 × 30 !
The number of all combinations of n distinct objects =
n! 3 × 2 × 1 × 30 !
taken r at a time given by n Cr = , 0 ≤ r ≤ n. 33 ⋅ 32 ⋅ 31
( n − r) ! r ! = = 5456
1⋅ 2⋅3
Target Exercise
Elementary Level Questions
1. The number of ways in which 9 identical balls can be 7. The total number of 9-digit numbers which have all
placed in three identical boxes is different digits is
9! 9! (a) 10! (b) 9! (c) 9 × 9! (d) 10 × 10!
(a) 55 (b) (c) (d) 12
(3!)4 (3!)3
8. The number of words which can be formed out of the
2. The total number of selections of atmost n things with letters of the word ARTICLE, so that vowels occupy
10 points as vertices, n of them being collinear, is 110. the even place is
Then, n is (a) 1440 (b) 144 (c) 7! (d) 4
C 4 × 3C 3
(a) 13 (b) 4 (c) 5 (d) 6
9. Eight chairs are numbered 1 to 8. Two women and
3. In a polygon, the number of diagonals is 54. The 3 men wish to occupy one chair each. First the women
number of sides of the polygon is choose the chairs from amongst the chairs 1 to 4 and
(a) 10 (b) 12 (c) 9 (d) 13 then men select from the remaining chairs. Find the
4. The letters of the word MODESTY are written in all total number of possible arrangements.
possible orders and these words are written out as in (a) 1440 (b) 1450
a dictionary, then the rank of the word MODESTY is (c) 1460 (d) 1470
(a) 5040 (b) 720 (c) 1681 (d) 2520 10. In how many ways can a student choose a program
5. Given 5 flags of different colours, how many different of 5 courses, if 9 courses are available and 2
signals can be generated, if each signal requires the specific courses are compulsory for every student?
use of 2 flags, one below the other. (a) 34 (b) 36 (c) 35 (d) 37
(a) 18 (b) 20 (c) 19 (d) 23 11. In an examination of 9 papers a candidate has to pass
in more papers, then the number of paper in which he
6. From a committee of 8 persons, in how many ways can
fails in order to be successful. The number of ways in
we choose a chairman and a vice-chairman assuming
which he can be unsuccessful, is
one person cannot hold more than one position?
(a) 255 (b) 256 (c) 193 (d) 319
(a) 54 (b) 55 (c) 52 (d) 56

High Skill Questions


1. If the different permutations of all the letters of the 5. In how many ways can the letters of the word
word EXAMINATION are listed as in a dictionary, PERMUTATIONS be arranged, if the words start
how many words are there in this list before the first with P and end with S?
word starting with E? (a) 1814400 (b) 1814405 (c) 1824050 (d) 1824060
(a) 908300 ways (b) 907200 ways 6. The number of ways in which seven persons can be
(c) 907300 ways (d) None of these arranged at a round table, if two particular persons
2. In how many ways can one select a cricket team of may not sit together is
eleven from 17 players in which only 5 players can (a) 480 (b) 120 (c) 80 (d) 100
bowl, if each cricket team of 11 must include exactly 4 7. How many numbers lying between 10 and 1000 can
bowlers? be formed from the digits 1, 2, 3, 4, 5, 6, 7, 8, 9
(a) 3960 (b) 3961 (c) 3962 (d) 3963 (repetition of digits is allowed)?
3. In how many different ways can five boys and five (a) 1024 (b) 810 (c) 2346 (d) 820
girls form a circle such that the boys and girls set 8. Everybody in a room shakes hands with everybody
alternate? else. The total number of hand shakes is 66. The total
(a) 4! (b) 2880 number of persons in the room is
(c) 5! (d) 6! (a) 11 (b) 12 (c) 13 (d) 14
4. In how many of the distinct permutations of the 9. A question paper is divided into two parts A and B
letters in the MISSISSIPPI do the four I’s not come and each part contains 5 questions. The number of
together? ways in which a candidate can answer 6 questions
(a) 35810 (b) 33810 selecting atleast two questions from each part is
(c) 33820 (d) 33840 (a) 80 (b) 100 (c) 200 (d) 3000
Hints and Solutions
Elementary Level Questions
1. (d ) ∴ Required number of ways = ( 3 C1 × 2C1 ×1 C1 ) × 2 7. (c) The total number of 9-digit numbers, having all digits
are different = 9 × 9 P8
= 3 × 2 × 1 × 2 = 12 9 × 9!
= = 9 × 9!
2. (c) Out of 10 points, let n points be collinear. Then, number 1!
of triangles are 10 C 3 − nC 3 = 110 (Q in first place we select anyone of nine numbers except
10 × 9 × 8 n(n − 1)(n − 2 ) zero. In rest of the eight places, we select any eight
∴ − = 110 numbers from remaining 9 numbers.)
3×2 3×2
⇒ 720 − 660 = n(n − 1)(n − 2 ) 8. (b) In a word ARTICLE, vowels are A, E, I and consonants
⇒ n(n − 1)(n − 2 ) = 60 are C, L, R, T.
⇒ n(n − 1)(n − 2 ) = 5 × 4 × 3 In a seven letter word, there are three even places in which
three vowels are placed in 3! way. In rest of the four
∴ n=5
places, four consonants are placed in 4! ways.
3. (b) Let number of sides of polygon is n. Since, number of ∴ Required number of ways = 3! × 4! = 6 × 24 = 144
sides of polygon is equal to number of vertices of polygon.
Q Number of diagonals of polygon = 54 9. (a ) Two women occupy the chair from 1 to 4 in 4 P2 ways
n(n − 1 ) and 3 men occupy the remaining chairs in 6 P3 ways.
∴ n
C 2 − n = 54 ⇒ − n = 54
2 4! 6!
∴ Required number of ways = 4 P2 × 6 P3 = ×
⇒ n − 3n = 108 ⇒ n − 3n − 108 = 0
2 2
4 − 2 ! 6 − 3!
⇒ n2 − 12 n + 9n − 108 = 0 ⇒ n(n − 12 ) + 9(n − 12 ) = 0 = 12 × 120 = 1440
⇒ (n + 9)(n − 12 ) = 0
10. (c) Total number of available courses = 9
∴ n = 12 (Q n cannot be negative.)
Out of these 5 courses have to be chosen. But it is given
4. (c) The number of words start with D = 6! = 720 that 2 courses are compulsory for every student i .e., you
The number of words start with E = 6! = 720 have to choose only 3 courses instead of 5, out of 7
The number of words start with MD = 5! = 120 instead of 9.
The number of words start with ME = 5! = 120 It can be done in 7C 3 ways
7! 7 ×6×5
Now, the first word start with MO is MODESTY. = = = 35 ways
Hence, rank of MODESTY = 720 + 720 + 120 + 120 = 1681 3! (7 − 3)! 6

5. (b) Number of ways choose the first flag = 5 11. (b) Q The candidate is unsuccessful, if he fails in 9 or 8 or 7
Number of ways to choose the second flag from the rest of or 6 or 5 papers.
four flags = 4 ∴ Numbers of ways to be unsuccessful
Hence, by FPC total number of ways = 5 × 4 = 20 = 9C 9 + 9C 8 + 9C 7 + 9C 6 + 9C 5
= 9C 0 + 9C1 + 9C 2 + 9C 3 + 9C 4
6. (d ) Out of 8 persons, one person can be chosen for 1
chairman in 8 ways. Now, a person cannot hold more than = ( 9 C 0 + 9C1 + ... + 9C 9 )
one position i .e., out of 7 persons, one person can be 2
1
chosen for vice-chairman in 7 ways. = (2 9 ) = 2 8 = 256
Hence, by FPC total number of ways = 8 × 7 = 56 2

High Skill Questions


1. (b) The letters of given word are A, A, E, I, I, M, N, N, O, T, X 2. (a) Cricket team
i .e., word starting with A are formed with the letters 2I’s,
2N’s, A, E, X, M, T, O (total 10 letters). Total 17 players
Hence, number of words formed by these letters
10!
= 5 Bowlers 12 Batsmen
2! 2!
We are to select 11 players including exactly 4 bowlers.
10 × 9 × 8 × 7 × 6 × 5 × 4 × 3 × 2 × 1
= Hence, 4 bowlers will be selected from 5 bowlers and
2 × 1× 2 × 1 remaining 7 players will be selected from 12 b
10 × 9 × 8 × 7 × 6 × 5 × 4 × 3 × 2 × 1 atsmen.
=
4 Now, 4 bowlers out of 5 bowlers can be selected in 5 C 4
= 10 × 9 × 8 × 7 × 6 × 5 × 3 × 2 × 1 ways.
= 907200 ways 7 players out of 12 players can be selected in 12 C 7 ways.
522 Study Package for NTSE

Hence, by FPC, total number of ways selecting the eleven O — 1 time


players = 5C 4 × 12C 7 = 5C1 × 12C 5 (Q nC r = nC n − r ) N — 1 time
5 × 12 × 11 × 10 × 9 × 8 S — 1 time
= (i) Word start with P and end with S i .e.,
120
Q nC = n(n − 1) (n − 2 ) (n − 3) (n − 4) P S
 5
120  First and last position are already filled by
= 5 × 11 × 9 × 8 = 55 × 72 = 3960 ways P and S, respectively.
10!
Remaining 10 positions can be filled = ways
3. (b) After fixing up one boy on the table the remaining can 2!
be arranged in 4! ways but boys and girls are to sit 10 × 9 × 8 × 7 × 6 × 5 × 4 × 3 × 2 !
=
alternate. 2!
B1 = 720 × 7 × 6 × 5 × 4 × 3 = 720 × 2520 = 1814400

B5 6. (a) Q Remaining 5 can be seated in 4! ways.


B2 5
Now, on cross marked five places 2 person can sit in P2
ways.
P4
B4
B3 P3
There will be 5 places, between two boys, these five places P5
can be filled by 5 girls in 5! ways.
The required number of ways = 4! × 5! = 2880
P2
4. (b) The word MISSISSIPPI has 11 letters in which
P1
M → 1 time
I → 4 times 5!
So, number of arrangements = 4! × = 24 × 20 = 480 ways
S → 4 times 3!
P → 2 times 7. (b) Case I When number in two digits.
The number of permutations of the word MISSISSIPPI in Total number of ways = 9C1 × 9C1 = 9 × 9 = 81
which 4 I’s and 4 S’s are alike
11! Case II When number in three digits
= …(i)
4! 4! 2 ! Total number of ways = 9C1 × 9C1 × 9C1 = 9 × 9 × 9 = 729
If all the I’s are together, then it will be considered as one ∴ Total number of ways = 81 + 729 = 810
letter and remaining 7 letters and 1 I’s letter (including 4 I’s) 8. (b) Let total number of persons be n.
will be considered as 8 letters. So, the number of Since, total number of hand shakes = 66
8!
permutations is . n(n − 1)
4! 2 ! ∴ n
C 2 = 66 ⇒ = 66
2
Hence, total number of arrangements ⇒ n2 − n − 132 = 0
11! 8!
= − ⇒ n − 12 n + 11n − 132 = 0
2
4! 4! 2 ! 4! 2 !
⇒ n(n − 12 ) + 11(n − 12 ) = 0
11 × 10 × 9 × 8 × 7 × 6 × 5 × 4! 8 × 7 × 6 × 5 × 4! ⇒ (n − 12 )(n + 11) = 0
= −
4! × 4 × 3 × 2 × 1 × 2 × 1 4! × 2 × 1 ∴ n = 12 (Q n cannot be negative.)
= 34650 − 840 = 33810
9. (c) The number of ways that the candidate may select
5. (a)The word PERMUTATIONS has (i) if 2 questions from A and 4 questions from B
P — 1 time = 5C 2 × 5C 4 = 50
E — 1 time (ii) 3 questions from A and 3 questions from B
R — 1 time = 5C 3 × 5C 3 = 100
M — 1 time (iii) 4 questions from A and 2 questions from B
U — 1 time = 5C 4 × 5C 2 = 50
T — 2 times
A — 1 time Hence, total number of ways = 50 + 100 + 50 = 200
I — 1 time
Probability 523

Chapter

19
Probability
Probability is nothing but chance of happening an Complement of an Event
event. Here, we shall study some definitions, formulae The complement of an event A is the set of all
and uses related to it. outcomes in the sample space that are not included in
the outcomes of event A. The complement of event A is
Definition represented by A (read as A bar).
The probability is defined as the chance of happening The probability of complement of an event can be found
or not happening of an event. by subtracting the given probability from 1.
Probability of happening of an event P ( A ) = 1 − P ( A)
Number of favourable outcomes Example 1 Probability of getting all head when three coins
= are rolled together is
Total number of outcomes
1 2 3 5
(a) (b) (c) (d)
Understanding of an Event 8 5 7 11
It is basically the state in which somebody tries to Solution (a) P (getting a head) = 1/ 2 or P (getting a tail)
know the all possibilities of an event. Some of the 1 1 1 1
examples are given below. Thus, P (HHH) = × × =
2 2 2 8
Tossing a coin When a coin is tossed, there are two
Example 2 From a pack of 52 cards, the probability of
possible outcomes i.e., Head (H) and Tail (T).
1 selecting an ace or a king is
So, we can say that the probability of getting H is or 1 2 3 4
2 (a) (b) (c) (d)
1 13 13 13 13
the probability of getting T is .
2 Solution (b) P (an ace or a king)
Throwing a die When a single die is thrown, there = P (an ace) + P (king) – P (ace and king)
are six possible outcomes : 1, 2, 3, 4, 5 and 6. 4 4 8 2
= + −0= =
1 52 52 52 13
The probability of getting any one of these numbers is .
6 î As there is no card common between king and ace card, they are
mutually exclusive events.
Certain and Impossible Events
Example 3 A dice is rolled twice, what is the probability of
A certain event is certain to occur, i.e., S (sample getting a 2 on the first roll and a prime number of the second
space) is a certain event. Probability of certain event is roll?
1, i.e., P (S ) = 1, and 5 7
(a) (b)
An impossible event has no chance of occurring, i.e., φ 12 12
is the impossible event. Probability of impossible event 1 2
is 0, i. e., P ( φ) = 0. (c) (d)
12 9
524 Study Package for NTSE

Solution (c) The numbers which can appear when a dice 1


Example 5 Probability of A giving a false statement is ,
is rolled are 1 or 2 or 3 or 4 or 5 or 6. Thus, P (getting 2 on first 6
1 3 1 3
roll and a prime number on the second roll) = × = whereas probability for B giving a true statement is . If A and B
6 6 12 5
Example 4 A bag contains 4 blue, 3 green and 5 yellow both make a statement, what is the probability of both lying?
coloured balls, a ball is picked at random from the bag, what is 1 2 4 7
(a) (b) (c) (d)
the probability that either yellow or green? 15 15 15 15
1 4 3 2
(a) (b) (c) (d) Solution (a) P (A giving false statement) = 1 and P (B giving
3 5 5 3 6
3 2
Solution (d) P (ball being either yellow or green) false statement) = 1 − =
5 5
(5 + 3 ) 8 2
= = = = 1 − P (selecting a blue ball) Thus, P (both A and B giving false statement)
12 12 3
1 2 1
4 1 2 = × =
= 1− = 1− = 6 5 15
12 3 3

Target Exercise
Elementary Level Questions
1. In a single throw of a dice, what is the probability of 10. The table below gives the feedback of 120 people
getting a prime number? about their likings of different brands of biscuits.
(a) 1/2 (b) 1/3 (c) 1/4 (d) 1/6
Britannia Parle-G Marie Gold Aviva Frontier
2. What is the probability of getting atleast one head 24 36 15 32 13
when two coins are tossed simultaneously?
1 1 3 2 If a candidate is selected at random, what is the
(a) (b) (c) (d)
4 2 4 3 probability that he likes Britannia?
3. If the probability of winning a game is 9/11, what is (a) 3/10 (b) 1/5 (c) 4/15 (d) 3/5
the probability of its losing? 11. In a factory 50 units are manufactured every day out
(a) 2/11 (b) 7/9 (c) 2/9 (d) 7/11 of which 10% get wasted. If a unit its selected at
random, what is the probability of it being a
4. In a bag containing 5 red, 4 black and 6 orange balls,
non-defective unit?
a ball is selected at random, what is the probability of (a) 1/10 (b) 1/5 (c) 1/2 (d) 9/10
getting a none red ball?
1 4 1 2 12. In a class of 80 students, 48 are boys and the rest of
(a) (b) (c) (d)
3 15 5 3 the students are girls. If 10 students shift to the other
class room and a student is selected at random from
5. A card is selected at random from a pack of 52 cards,
the remaining class, what is the probability that a
what is the probability of selecting a red colour king?
1 1 1 4 girl is selected?
(a) (b) (c) (d) (a) 11/35 (b) 27/70
2 26 13 13
(c) 11/40 (d) Cannot be determined
6. In a single throw of a dice, what is the probability of
13. Three coins when tossed 100 times, the frequency of
getting a number greater than 4?
tails was recorded as the table.
(a) 1/2 (b) 1/3 (c) 1/4 (d) 1/5
Number of tails Zero One Two Three
7. Probability that a number selected from 1, 2, 3, 4, ...,
35 is a prime number, is Frequency 22 36 24 18
(a) 13/35 (b) 12/35 (c) 9/35 (d) 11/35
If three coins are tossed again, what is the probability
8. Two coins are tossed simultaneously. The probability of getting atleast one head?
of getting one head is (a) 39/50 (b) 9/25 (c) 11/50 (d) 41/50
(a) 1/2 (b) 1/3 (c) 1/4 (d) 3/4
14. Out of the letters E, T and N, a three letter word is
9. What is the probability of selecting a consonant from formed at random using all the 3 letters. What is the
a set of 26 alphabets? probability that the word formed is TEN?
(a) 21/26 (b) 20/26 (c) 11/13 (d) 5/26 (a) 1/6 (b) 1/3 (c) 1/2 (d) 2/3
Probability 525
High Skill Questions
1. In a biased dice the probability of getting an odd 9. There are 100 letters and 100 corresponding
number 2/3 times the probability of an even number. envelopes. If letters are put in the envelopes, what is
What is the probability of getting a 5 in that dice? the probability that exactly one letter will go in the
(a) 2/5 (b) 4/5 wrong envelope?
(c) 8/25 (d) None of these (a) 1/00 (b) 1/50
(c) 1/25 (d) None of these
2. If 5 coins are tossed together, what is the probability
of getting atleast two heads? 10. Mohan lies on 3 days out of the 7 days in a week,
(a) 1/32 (b) 3/16 whereas Ram lies on 4 days out of 6 days of a week. If
(c) 13/16 (d) 5/8 both of them were asked a question, what is the
3. When three dice are rolled together, what is the probability that both gave a true reply?
probability of getting atleast two 5? (a) 1/7 (b) 4/21
(c) 5/21 (d) 2/7
(a) 1/9 (b) 2/27
(c) 4/27 (d) None of these 11. 400 people polled for two candidates A and B in
4. Probability of trains A, B and C arriving on times is election, their response was
1/3, 2/5 and 2/3, respectively. What is the probability Candidate/Response Against Favour
that atmost one of the trains arrive on time?
A 220 180
(a) 7/15 (b) 22/45
B 150 250
(c) 23/45 (d) None of these

5. In a group of 8 students, what is the probability of If a person was selected at random from the people
each one of them having birthday on the different day who polled, what is the probability he is in favour of A
of week? but against B?
(a) 1 (b) 1/7 (a) 5/16 (b) 1/3
(c) 1/8 (d) None of these (c) 11/32 (d) 13/32

6. When two dice are rolled together, the probability of 12. From the set of numbers from 1 to 250, one number is
getting an even number on one dice and a multiple of selected at random. What is the probabilty that it is
3 on the other is either a multiple of 5 or a multiple of 4?
(a) 1/6 (b) 7/36 (a) 1/5 (b) 1/4
(c) 4/9 (d) 5/36 (c) 2/5 (d) 3/5

7. A medical test machine gives the correct report in 13. A coins is tossed 5 times. What is the probability of
3 out of 5 times. Three patients A, B and C got their getting number of heads more than the number of
checkup done on the same machine, what is the tails?
(a) 1/3 (b) 1/32
probability that majority of them got the correct
(c) 5/32 (d) 1/2
report?
(a) 9/25 (b) 8/25 14. A bag which has got 5 pairs yellow socks, 6 pairs of
(c) 1/5 (d) None of these blue socks and 4 pairs of black socks. If 2 socks are
8. Two persons A and B appear in an interview for two selected at random, what is the probability that it will
vacancies. If the probabilities of their selections are form a white pair?
1 1 (a) 1/45 (b) 1/90
and respectively, then the probability that none (c) 1/18 (d) None of these
4 6
of them is selected, is 15. An unbiased dice is rolled twice, what is the
5 5
(a) (b) probability of not getting a 2 in anyone of the two
8 12 rolls?
1 1
(c) (d) (a) 1/3 (b) 5/12
12 24
(c) 25/36 (d) 35/36
Hints and Solutions
Elementary Level Questions
1. (a) Prime numbers in a dice = 2, 3 and 5 1 1 1
8. (a) P (getting one head) = P(HT ) + P(TH) = + =
4 4 2
Number of favourable outcomes
So, the P (prime number) =
Total number of outcomes 9. (a) Given, total alphabets = 26
3 1
= = Number of consonants = 21
6 2 Number of favourable outcomes
∴ Required probability =
2. (c) P (atleast one head) = 1 − P (no head) Total number of outcomes
= 21 / 26
= 1 −  1/2 ×  = 1 − =
1 1 3
 2 4 4 10. (b) P (person selected likes britannia)
3. (a) P (losing a game) = 1 − P (winning the game) Number of favourable outcomes
= = 24 / 120 = 1 / 5
= 1 − 9 / 11 = 2 / 11 Total number of outcomes

4. (d) P (non-red ball) = P (black or orange ball) = 1 − P (red ball) 11. (d) Number of defective units = 10% of 50 = 5

= 1 − 5 / 15 = 1 − 1 / 3 = 2 / 3 So, number of non-defective units = 50 − 5 = 45


Number of favourable outcomes
Thus, P (non-defective unit) =
5. (b) Number of red kings = 2 Total number of outcomes
So, P (red king) = 2 / 52 = 1 / 26 = 45 / 50 = 9 / 10
6. (b) Number greater than 4 are 5 and 6 . i.e., 2 numbers out of 6. î (Since, the number of defective units is 10%, the number of
So, P (getting more than 4 in a dice) non-defective units = 90% = its probability)
Number of favourable outcomes
= 12. (d) Since, we do not know the number of girls in class after
Total number of outcomes 10 students shifted, probability of selecting a girl from the
2 1 remaining class cannot be calculated.
= =
6 3
13. (d) P (atleast one head) = 1 − P (no head)
7. (d) Number of prime numbers in the first 35 natural numbers
= 1 − 18 / 100 = 82 / 100 = 41 / 50
= (2, 3, 5, 7, 11, 13, 17, 19, 23, 29, 31) = 11
14. (a) Total number of three letter words formed with three letters E,
∴ P(selecting a prime number)
Number of favourable outcomes T and N = 6 out of which TEN is one word, so P (TEN formed)
= = 11 / 35 = 1/ 6
Total number of outcomes

High Skill Questions


1. (d) Let the probability of an even number = 3x, where x is any 3. (b) Total number of possible outcomes when three dice are
positive real number, then the probability of an odd number rolled together
= 6 × 6 × 6 = 216
= 2 / 3 (3 x) = 2 x. Total probability = 3(3 x + 2 x) = 15 x (as there are
3 even and 3 odd numbers in dice). Thus, P (getting a 5) Number of favourable outcomes when atleast two 5 appears
= {( 5, 5, 1), (5, 5, 2 ), ..., (5, 5, 6), (5, 1, 5), (5, 2, 5,), (5, 3, 5),
= 2 x / 15 x = 2 / 15
(5, 4, 5), (5, 6, 5), (1, 5, 5), (2, 5, 5),(3, 5, 5), (4, 5, 5), (6, 5, 5)}
2. (c) P(atleast 2 heads) = 1 − P(less than two heads) = 16 outcomes
= 1 − [P (no head) + P (one head)] Thus, the required probability
= 1 − [(1 / 2 )5 + 5(1 / 2 5 )] =
Number of favourable outcomes
Total number of outcomes
= 1− 
1 5
+ = 16 / 216
 32 32 
6 = 2 / 27
= 1−
32 4. (b) P (atmost one train arrives on time)
32 − 6 26 =
1 2 1 1 3 2 2 2 2 2 3 1
× × + × × + × × + × ×
= =
32 32 3 5 3 3 5 3 3 5 3 3 5 3
22
= 13 / 16 =
45
Probability 527
5. (d) Days in a week are 7 and the number of students is 6. 11. (b) The letters which are same in the word Examination are aa, ii, nn.
So, it’s not possible to have their birthdays on different days on the So, the selecting a pair of letter which is same, we have a total of
week. So, probability is 0. 3 options out of where we can choose.
So, the probability (both letters being same)
6. (b) Total number of outcomes = 36 favourable outcomes
Number of favourable outcomes
= {(2, 2 ), (3, 2 ), (2, 6), (6,2 ), (4, 3), (3, 4), (6, 6)} = = 1/ 3
Total number of outcomes
Thus, P (even number on one and multiple of 3 on another)
Number of favourable outcomes 12. (c) Number of multiple of 5 = 50 and number of multiples of
= = 7 / 36
Total number of outcomes 4 = 62, number of numbers which are multiples of both as well
3 as 5 = multiples of 20 = 12
7. (a) P (test report to be correct) = = P(T ) Thus, P (selecting either a multiples of 5 or a multiples of 4)
5
3 2 50 62 12
and P (total report to be incorrect) = 1 − = = P(T ) = + −
5 5 250 250 250
100 2
For three prints A, B, C, majority to get the correct test report is = =
250 5
same as atleast 2 of them getting the correct report
3 3 2 3 3 3 13. (d) In 5 tossed of a coin, number of heads > of taills
= × × + × ×
5 5 5 5 5 5 10 cases 5 cases 1 case
= (HHHTT, HHHHT, HHHHH)
18 27 45 9
= + = = 5 5 5
Thus, P(heads > tails) = 10 ×   + 5   +  
125 125 125 25 1 1 1
2 2 2
8. (a) Required probability =  1 −   1 − 
1 1
1 1
 4  6 = (10 + 5 + 1) =
32 2
3 5 5
= × =
4 6 8 14. (b) We have 5 pairs of yellow socks, i.e., 10 yellow socks, 6 pairs
of blue socks ⇒ 12 blue socks and 4 pairs of black socks means
9. (d) Exactly 1 letter can never go in wrong envelope as the other black socks.
whose envelope is taken by the first letter will have no correct
P (selecting 2 socks and it form white pair) = 1 / 10 × 1 / 9 = 1 / 90
envelope left.
(we want 1 white socks from 10 white socks and next 1 more
So, P (exactly 1 letter going in wrong envelope) = 0 white socks from the remaining 9 white socks)
10. (b) P (Mohan lying) = P(M ) = 3/7 15. (c) P (getting a 2) = 1 / 6
and P(Mohan not lying) = P(M ) = 1 − 3 / 7 = 4 / 7 So, P (not getting a 2) = 1 − 1 / 6 = 5 / 6
P (Ram lying) = 4 / 6 = 2 / 3 = P(R ) Thus, P (not getting a 2 in 2 rolls of an unbiased dice)
and P (Ram not lying) = P(R ) = 1 − 2 / 3 = 1 / 3 Number of favourable outcomes
=
P (both giving true reply) = P(R ) × P(M ) Total number of outcomes
= 4 / 7 × 1 / 3 = 4 / 21 = 5 / 6 × 5 / 6 = 25 / 36
Part-V : History
Chapter

1
Ancient India
Indus Valley Civilisation Town Planning
(2500-1800 BC) The streets of major cities such as Mohenjodaro and
Harappa were laid out in perfect grid patterns.
The Indus Valley Civilisation was an ancient
Roads were well cut, dividing the town into large
civilisation thriving along the Indus River in
rectangular or square blocks. Lamp posts at intervals
Pakistan and North-Western India. Name of this indicate the existence of street lighting. Flanking the
civilisation was the Harappan Civilisation, in streets, lanes and by-lanes were well-planned houses.
reference to its first excavated city of Harappa.
The towns were divided into two parts
l RB Dayaram Sahni first discovered Harappa (on Ravi)
in 1921. RD Banerjee discovered Mohenjodaro or l Upper part or Citadel and
‘Mound of the Dead’ (on Indus) in 1922. l Lower part
l Sir John Marshal played a crucial role in excavacing In Mohenjodaro, a big public bath (Great Bath)
both these. Last site was found in IVC Dholavira and it measuring 12 m by 7 m and 2.4 m deep has been found.
was founded by RS Bist in 1991.
l Harappan Civilisation forms part of the proto-history of Economic Life
India and belongs to the Bronze Age. According to
The Indus people grew wheat, barley, rai, peas,
radio-carbon dating, it spread from the year
sesamum, mustard, rice (in Lothal), cotton, dates, melon,
2500-1750 BC.
etc. They also domesticated animals on large scale.
l In this civilisation copper, bronze, silver, gold and tin
l Weights and measures of accuracy existed in Harappan
were known.
culture (found at Lothal). Flint tool work, shell work,
l Cotton was grown at Mehrgarh from about 7000 years bangle making, pottery making, etc were practised.
ago. l The people were very fond of ornaments (of gold, silver,
ivory, copper, bronze, precious stones) and dressing up.
Geographical Extent Ornaments were worn by both men and women.
The Indus Valley Civilisation extended from l Potter’s wheel was in use. Their pottery was red or black
Baluchistan in Pakistan to Gujarat, with an upward pottery. They played dice games.
reach to Punjab from East of the river Jhelum to Ropar l Their favourite pastime game was Gambling.
on the upper Sutlej. l Made pots with beautiful black design.
Major sites in Pakistan are Harappa (on Ravi in West
Punjab), Mohenjodaro (on Indus), Chanhu-Daro
Religious Life
(Sindh), etc. In India, major sites are Lothal, Rangpur Main worship was the Mother Goddess. But the upper
and Surkotda (Gujarat), Kalibangan (Rajasthan), classes preferred a god, nude with two horns, much
Banwali (Hissar) and Alamgirpur (Western UP). similar to Pashupati Shiva. Phallus (lingam) and Yoni
worship was also prevalent. Many trees (pipal), animals
Ancient India 529
(bull), birds (dove, pigeon) and stones were worshipped. l Sapta Sindhu region had declined, where Ganges valley
Unicorn was also worshipped. However, no temple has became important.
been found, though idolatry was practised. At
Kalibangan and Lothal fire altars have been found. The Early Vedic Policy
l The Early Vedic Policy ‘Sabha’ and ‘Samiti’ continued to
The Vedic Age hold the ground but their character changed.
l Women were no longer permitted to sit in the ‘Sabha’
Early Vedic Age and it was now dominated by nobles and the
The Vedic Age began in India about 2000 BC with the Brahmanas.
coming of the Aryans. General opinion regarding the
original home of Aryans were somewhere in the area Social Organisation
East of the Alps, in the region of the Caspian Sea and The Later Vedic society came to be divided into four
the Southern Russian Steppes. varnas called the Brahmanas, Rajanyas or Kshatriyas,
Vaishyas and Sudras.
Coming of the Aryans to India
The earliest Aryan wave in India is represented by the
l

Rig Vedic people who appeared in the sub-continent


Buddhism
about 1500 BC.
Early Life of Buddha
l They came in conflict with the indigenous inhabitants
called the Dravidians mentioned as Dasas or Dasyus in Siddhartha, also known as Gautama (as well as
Rig Veda. Sakyamuni or Tathagata), the founder of Buddhism,
was born in 563 BC on the Vaishakha Poornima Day at
Vedic Literature Lumbini (near Kapilavastu) in Nepal.
Vedic literature gives us an elaborate knowledge over l Buddha belonged to a small gana known as the Sakya
the subject of socio-economic life of Aryans. gana and was a Kshatriya.
Vedic literature has been classified into four parts l His father Suddhodana was the Saka ruler.
(i) Vedas (ii) Brahmana l His mother Mahamaya (of Kosala Dynasty) died after 7
(iii) Aranyakas (iv) Upanishads days of his birth. So, he was brought up by his step
mother Gautami.
(i) Vedas There are 4 vedas
Rigveda It is the oldest Veda composed about 3500 l At the age of 16, he was married with Yoshodhara. He
years ago. It includes more than 1000 hymns. The remained in marital life for 13 years and had a son
tirta mandala contains the famous ‘Purushasukta’ named Rahul.
which explains the origin of four varnas.
Yajurveda It is a ritual veda. It describes the ritual Growth of Buddhism
for performing different sacrifices. Buddhism stands for three pillars
Samaveda It is a collection of melodic hymns. l The Buddha It’s Founder
Atharvaveda It describes the popular beliefs and
l The Dhamma His Teachings
superstitions of the humble folk.
l The Sangha Order of Buddhist monks and nuns
(ii) Brahmanas The Brahmanas explain the hymns of
the Vedas in an orthodox manner. They are written
in prose and are ritualistic in nature. The Dhamma
(iii) Aranyakas It means ‘‘the forest’ and these are called Buddha taught his disciple in Prakrit language. The
forest books because they were written mainly for the Dhamma as told by Buddha are
hermits and students living in the Jungles.
l The Four Noble Truths are
(iv) Upanishads It means to sit down near someone and
denotes a student sitting near his guru to learn. → The truth of dukkha (suffering, anxiety,
dissatisfaction)
Later Vedic Period → The truth of the origin of dukkha
l The later Vedic Period extends from 1000 BC to 600 BC. → The truth of the cessation of dukkha
l Evidence of iron was found from Gandhar which shows → The truth of the path leading to the cessation of
that iron was in use from 1000 BC. dukkha.
530 Study Package for NTSE

l The way to Nirvana (emancipation). In Buddhism or the l There is no historical basis for the first 22 Tirthankaras.
path that leads to cessation of suffering higher wisdom l Only the last two Tirthankaras are historical
and peace of mind is known as Noble Eight Fold path. personalities.
l The Noble Eight Fold path are l The 23rd Tirthankar Parshwanath was the son of king
→ Right View (Samma-ditthi) Ashvasena of Banaras.
Intentional action (Kamma). l The 24th and the last Tirthankar was Vardhman
Admirable friendship (Kalyanamittata). Mahavira. He was born in 540 BC. His father was
→ Right Resolve (Samma-Sankappo) Siddhartha and mother was Trishala.
→ Right Speech (Samma-Vaca)
Teachings of Mahavira
→ Right Action (Samma-Kammanto)
l The ultimate objective of Mahavira’s teaching was how
→ Right Livelihood (Samma-Ajivo)
one can attain the total freedom from the cycle of birth,
→ Right Affort (Samma-Vayamo) life, pain, misery and death and achieve the permanent
→ Right Mindfulness (Samma-Sati) blissful state of one’s self. This is also known as
→ Right Concentration (Samma-Samadhi) liberation, nirvana, absolute freedom or Moksha.
l Universal brotherhood (non-belief in caste system). In
Buddhist Philosophy
Jainism, three Ratnas (Triratnas) are given and they
The three characteristics of Buddhism are anitya are called the way to Nirvana. They are Right Faith,
(transiency), dukka (sorrow) and anatma (soullessness). Right Knowledge and Right Conduct.

The Sangha or Church Worshippers


Consists of monks (Bhikshus or Shramanas) and
Five Doctrines of Jainism
Bhikshus acted as a torch bearer of the Dhamma. (i) Non-violence (Ahimsa) not to cause harm to any living beings.
(ii) Truthfulness (Satya) to speak the harmless truth only.
Buddhist Literature (iii) Non-stealing (Asteya) not to take anything not properly given.
(iv) Chastity (Brahmacharya) not to indulge in sensual pleasure.
It is in Pali language. Buddhist scriptures in Pali are
(v) Non-possession/Non-attachment (Aparigraha) complete detachment
commonly referred to as Tripitakas, i.e., ‘Three-fold from people, places and material things.
Basket’.
l Vinaya Pitaka Rules of discipline in Buddhist
monasteries.
Rise of Janpadas,
l Sutta Pitaka Largest, contains collection of Buddha’s
sermons. Mahajanapadas and Kingdoms
l Abhidhamma Pitaka Explanation of the philosophical
principles of the Buddhist religion. Mahajanapadas
The word ‘Janapadas’ literally means ‘the land, where
Growth of Jainism the jana settled down’. Some more important Janapadas
Jainism is one of the oldest religions in the world. became Mahajanapadas.
Jainism doesn’t have a single founder and the truth is There were 16 Mahajanapadas which were also
said to have been revealed at different times by a mentioned Anguttar Nikay (Bodh text) and dominant
Tirthankara (Prophets or Gurus). A Tirthankara is during 600-321 BC. Out of these 16 Mahajanapadas,
considered omniscient, a role model, not a God. There Magadh was the most powerful and important one.
have been 24 Tirthankaras, all Kshatriyas. First
Tirthankar was Rishabhadev.
Ancient India 531
List of Mahajanapadas Dhananand
S.No. Mahajanapadas Capital He was the last ruler of Nand Dynasty. During his
reign, Alexander invaded India (in 326 BC). Nand
1. Kasi Varanasi
2. Kosala Sharaswati
Dynasty was followed by Mauryan Dynasty.
3. Malla Kushinagar
4. Chedi Shodivati The Mauryan Empire (322-185 BC)
5. Vatsa Kaushambi
6. Kuru Hastinapur, Indraprastha The Mauryan Empire (322-185 BC), was a geographically
7. Panchal Achicdhatra, Kampilya extensive and powerful political and military empire in
8. Matasya Viratnagar ancient India. Originating from the kingdom of Magadha
9. Shursena Mathura in the modern Bihar and Bengal, the empire had its
10. Ashmak Pathan (the only Mahajanapada in capital city at Patliputra (near modern Patna).
South India)
11. Avanti Mahishmati, Ujjain Chandragupta Maurya (322-297 BC)
12. Gandhar Taxila
13. Anga Champa
With the help of Chanakya (also known as Kautilya or
14. Magadha Rajgriha, Griviraj Vishnugupta), he overthrew the Nandas and
15. Vajji Varanshi established the rule of the Maurya Dynasty.
16. Kamboja Lumbini l Chandragupta built a vast empire, which included not
only good portions of Bihar and Bengal, but also
Western and North-Western India and the Deccan.
Rise of Magadha
l This account is given by Megasthenes (A Greek
It consists of modern district of Patna, Gaya, ambassador sent by Seleucus to the court of
Sahahabad. Magadha was ruled by Haryank Dynasty. Chandragupta Maurya) in his book ‘Indica’. We also get
the details from the Arthashastra of Kautilya.
Haryanka Dynasty (545-410 BC)
Bindusara (298-273 BC)
It was founded by Bimbisar. His capital was Rajgriha.
Chandragupta was succeeded by his son Bindusara in
He was the follower of Buddhism. He was followed by
297 BC. He is said to have conquered ‘the land between
his son Ajatshatru.
the two seas’ i. e., the Arabian Sea and Bay of Bengal. At
Ajatshatru the time of his death, almost the entire subcontinent
came under the Mauryan rule.
He murdered his father Bimbisar and became the ruler.
During his period, Ist Buddhist council was held. He Ashoka (269-232 BC)
was followed by his son Udaiyan.
Chandragupta’s grandson Ashokavardhan Maurya
Udaiyan is considered by contemporary historians to be perhaps
the greatest of Indian monarchs and perhaps the
He built city of Patliputra at the confluence of Sone and
world.
Ganga river.
As a young prince, Ashoka was a brilliant commander
The last Haryank Dynasty ruler was Nagdashak. He
who crushed revolts in Ujjain and Taxila. As a monarch,
was killed by Shishunag, thus Shishunag Dynasty he was ambitious and aggressive, re-asserting the
founded. Empire’s superiority in Southern and Western India.
But it was his conquest of Kalinga (261 BC) which
Shishunag Dynasty proved to be the turning point event of his life.
Kalashok After Kalinga war Ashoka converted to Buddhism.
Shishunag was succeeded by Kalashok. During his
region 2nd Buddhist council was held. Shishunag Post Mauryan Dynasties
Dynasty was followed by Nand Dynasty. (200 BC-AD 300)
Nand Dynasty
Shunga Dynasty
Mahapadmanand was the founder of Nand Dynasty, he
was also known as Sarv Kshtrantak. Pushyamitra Shunga was the founder of Shunga
Dynasty, Patanjali lived in his court.
532 Study Package for NTSE

Kanvas founded by Vasudeva and the four main rulers Samudragupta. His court poet Harisena wrote a
were Vasudeva, Bhumimitra, Narayana and Susumar. glowing account of the military exploits of his patron. In
a long inscription at the Prayag Prashasti pillar
Satvahana Dynasty (at Allahabad), the poet has enumerated the people and
Kanva Dynasty was followed Satvahana Dynasty, countries that were conquered by Samudragupta.
founded by Simuka. l Samudragupta believed in the policy of war and conquest
l Shaks were their contemporaries. of new lands. It is because of his bravery and generalship
that he is called the ‘Napoleon of India’ (by the historian
l Land grants to Brahmins was started by Satvahanas.
VA Smith).
l Nagarjun Konda Stupa at Amravati (Andhra) was built
l He performed Ashwamedha Yajna (horse sacrifice) to
by them.
delimit his territory.
l He assumed the title of Kaviraj.
Post Mauryan Invaders l He was succeeded by his son Ramagupta, who was
captured by the saka Kshatrapas and was soon suceeded
(200 BC-AD 300) by his brother Chandragupta II.
Indo-Greeks
The Greek ruler Demetrius-I invaded India but his son
Chandragupta II (AD 380-413)
Demetrius-II was able to establish Greek control over Chandragupta II, the Sun of Power (Vikramaditya),
Punjab, the whole of Sindh and Cutch. ruled until 413. His daughter Prabhavatigupta was
The Shakas married to Rudrasena II, the Vakataka king of Deccan.
Chandragupta II took the title of Vikramaditya by
The earliest Shaka ruler of India established Shaka defeating Rudrasimha III, a Kshatrap king of Ujjain. He
power in Gandhar. also assumed the title of Simhavikrama.
Pahlavas They were Iranian people Gondophares was Chandragupta’s court was adorned by celebrated nine
the greatest ruler. gems (navratnas) including Kalidasa, Amarsimha,
Indo—Greeks—Shakas—Pahalavas—Kushan Varahmihir and Dhanvantri.

Kushan Dynasty Sangam Age (300 BC-AD 600)


Founded Kujul Kadaphises. It belonged to Ychi tribe.
The age of three kingdoms of South Cheras, Chol,
Kanishka Pandyas is known as Sangam age.
The greatest Kushan ruler. He was a Bodh follower of Important
S.No. Kingdom Rule Capital
Buddhism. He started Shak era in AD 78. He earned King
more name and fame for his personal qualities than to 1. Cheras Kerala Mujris —
his association with Buddhism. 2. Pandyas South Tamil Nadu Madurai Nandeenfed-
dleyan
3. Cholas North Tamil Nadu Kaveripatnam Kanikala
The Guptas and New Kingdoms
The time of the Gupta Empire is referred to as the
Golden Age of India in science, mathematics, Epics
astronomy, religion and Indian philosophy.
Mahabharata
Chandragupta I (AD 319-335) l It was written in Sanskrit by Ved Vyas. It describes
about war between Kaurvas and Pandavas of 950 BC in
He was the first important king of Gupta Dynasty. He
Kurukshetra.
enhanced his power and prestige by marrying Kumara
l Translation of ‘Mahabharata’ in Persian is called
Devi, the princess of the Lichchavi clan of Nepal.
‘Rajmnama’ done by Badayuni, in Telgu done by
He acquired for himself the title of Maharajadhiraj. He Tikkanna.
started the Gupta era in AD 319-320.
Ramayana
Samudragupta (AD 335-375) It was written by Valmiki in Sanskrit. It gave
The Gupta kingdom was extended far and wide knowledge of the period from 500 BC-AD 4th century. It
enormously by Chandragupta’s son and successor has 2400 shlokas, also known as ‘Adikavya’.
Ancient India 533
Puranas leader. Then, came Dharmapala who won Kannauj,
l Puranas are 18 in number. Its writer is Ugrvaisha. It is founded Vikramsila University and revived
written during Gupta Age in AD 4th century. Matasya Nalanda University. Then, came Devapala who won
Puran is the oldest Puran. Orissa and Assam. He defeated Pratihara king Bhoja
and Rashtrakuta king Amoghvarsha. Then, came
Post Gupta Dynasties Mahipala who was attacked and defeated by
Rajendra Chola. The Palas were replaced by Senas in
Post Gupta Dynasties are
Bengal.
Pushyabhuti Dynasty Harsha
Vardhana (AD 606-647) Kingdoms of South
He belonged to the Pushyabhuti family and was the son l The Vakatakas The Vakatakas came to control parts
of Prabhakar Vardhan. of the Deccan and Central India till the rise of the
Chalukyas. The founder of this Brahmin Dynasty was
He originally belonged to Thaneshwar, but shifted to Vindhyasakti. Most important king was Pravarsen I
Kannauj. Chinese pilgrim, Hieun Tsang (Prince of who performed four Ashwamedha Yajna.
Travellers) visited during his reign. He spent about eight
l Chalukyas of Vatapi (Badami) The founder of this
years (635-643) in the dominions of Harsha and earned Dynasty was Pulakesin I. They established their
his friendship. kingdom at Vatapi (modern Badami, Karnataka).
l He was defeated by Pulakesin II, the great Chalukya Pulakesin II was their most famous king, who was a
king, on the banks of Narmada in AD 620. contemporary of Harsha. Hieun Tsang visited his
l Harsha was a great patron of learning. kingdom.
l He established a large monastery at Nalanda. Much of the paintings and sculptures of the Ajanta and
Ellora caves were completed during the Chalukyan
l Banabhatta, who adorned his court, wrote ‘Harshacharita’
reign.
and ‘Kadambari’. Harsha himself wrote three plays–
‘Priyadarshika’, ‘Ratnavali’ and ‘Nagananda’. l The Rashtrakutas The founder of this Dynasty was
Dantidurga. They were originally district officers
After the death of Harsha, in AD 647, the empire once
under Chalukyas of Badami. Their king, Krishna I
again broke up into petty states.
is remembered for constructing the famous rock-cut
Kailasha temple at Ellora. Their king,
The Ganges
Amoghvarsha, is compared to Vikramaditya in
They are also called Chedagangas of Orissa. Their king
giving patronage to men of letters. He wrote the first
Narsimhadeva constructed the Sun Temple at Konark.
Kannada poetry named Kaviraj Marg and
King Anantvarman Ganga built the famous Jagannath
Prashnottar Mallika. He built the city of
temple at Puri. Kesaris, who used to rule Orissa before Manyakheta as his capital.
Ganges built the Lingaraja temple at Bhubhaneshwar.
l The Pallavas The founder of this dynasty was
The Palas Simhavishnu. They set up their capital at Kanchi
In the middle of the 8th century, the Pala Dynasty came (South of Chennai). Narsimhavarman was their
into power. Their capital was Monghyr in Bengal. Its greatest king. He founded the town of Mamalapuram
founder was Gopala (AD 750) who was elected to the (Mahabali- puram) which he adorned with beautiful
throne as he had proved his valor and capability as a rock-cut Raths or Seven Pagoras.
Target Exercise
Elementary Level
1. Which was the first metal used by man? 15. Kautilya was the Prime Minister of which of the
(a) Copper (b) Silver following Indian rulers?
(c) Bronze (d) Brass (a) Chandragupta Maurya (b) Chandragupta I
2. The Indus Valley Civilisation flourished during (c) Ashoka (d) Harsha Vardhana
(a) 5000-3500 BC (b) 3000-1500 BC 16. Sarnath’s Lion capital is attributed to
(c) 2500-1750 BC (d) 1500-500 BC (a) Kanishka (b) Harsha Vardhana
(c) Ashoka (d) Chandragupta
3. The Indus Valley Civilisation was discovered in
(a) 1902 (b) 1921 17. The external trade of the Sangam people was carried
(c) 1922 (d) 1932 on mainly with
(a) Romans (b) Arabs
4. The main characteristic of the Indus Valley (c) Chinese (d) South East Asian nations
Civilisation was
(a) town planning (b) drainage system 18. ‘Panchatantra’ was written by
(c) well laid out roads (d) pucca houses (a) Kalidasa (b) Vishnu Sharma
(c) Tulsidas (d) Banabhatta
5. The main occupation of the people of the Indus Valley
Civilisation was 19. Dhanvantari was
(a) agriculture (b) cattle rearing (a) a famous general of Chandragupta Maurya
(c) hunting (d) trade (b) a noted physician in the court of Chandragupta Vikramaditya
(c) a famous dramatist who lived during the time of Harsha
6. The Indus Valley people had trade relations with (d) a musician in the court of Ashoka
(a) Egypt (b) Greece 20. Who among the following is credited with the
(c) Ceylon (d) Mesopotamia
invention of Algebra?
7. The meaning of ‘Buddha’ is (a) Aryabhatta (b) Bhaskara
(a) anything beyond God (b) soul after death (c) Aapstamba (d) Medhatithi
(c) the enlightened one (d) the ultimate path for moksha 21. Kalidasa lived during the reign of
8. Jainism was divided into two sects known as (a) Samudragupta (b) Chandragupta Maurya
(a) Kapalika and Kalamukha (b) Mahayana and Hinayana (c) Ashoka (d) Chandragupta II
(c) Ajivika and Nyaya Vaisesika (d) Svetambara and Digambara 22. The Iron Pillar at Mehrauli was built by
9. Ashoka was much influenced by a Buddhist monk (a) Mauryas (b) Nandas (c) Khalijis (d) Guptas
called 23. The large Shiva Temple at Thanjavur was built by
(a) Upagupta (b) Vasubandhu (a) Rajendra Chola I
(c) Asvaghosha (d) Ambhi (b) Rajaraja Chola I
(c) Chandellas
10. The Fourth Buddhist Council was held by (d) Rashtrakutas
(a) Ashoka (b) Kanishka
(c) Chandragupta (d) Harsha Vardhana 24. Which of the following is the oldest Dynasty?
(a) Satvahanas (b) Pallavas (c) Chalukyas (d) Cheras
11. The last of the 24th Jain Tirthankaras was
(a) Parsvanath (b) Mahavira 25. The temple of Konark was built by Narsimhdeva of
(c) Rishabhadev (d) Arishtanemi the
12. Buddhism and Jainism both gave stress on (a) Hoysalas (b) Kakatiyas
(c) Gangas (d) Chalukayas
(a) worship of God (b) non-violence
(c) self-mortification (d) None of these 26. The Council of the ‘Nine Gems’ is associated with
13. Who among the following is considered as the first (a) Ballala Sena (b) Harsha Vardhana
(c) Chandragupta II (d) Devapala
national ruler of India?
(a) Chandragupta Maurya (b) Ashoka 27. Which one of the following Gupta rulers was related
(c) Chandragupta I (d) Kanishka to the Lichachavis on his mother's side?
14. Ashoka, the great, conquered Kalinga in the year (a) Srigupta (b) Chandragupta I
(a) 261 BC (b) 58 BC (c) AD 261 (d) AD 78 (c) Ghatotkachagupta (d) Samudragupta
Ancient India 535
28. The achievements of Samudragupta are described 39. The practice of image worship began among the
in the followers of Brahmanism with the emergence of
(a) Hathigumpha inscription (b) Allahabad pillar inscription (a) Shaivism (b) Ajivikas
(c) Girnar inscription (d) Sarnath inscription (c) Bhagavatism (d) Atharvaveda

29. The coins of which of the following reveal their love 40. Which of the following Gupta kings is famous for his
for music? courageous stand against the Hunas?
(a) Mauryas (b) Guptas (c) Chalukyas (b) Cholas (a) Chandragupta II (b) Kumaragupta
(c) Skandagupta (d) Buddhagupta
30. The Panchatantra was written during the
(a) Later Vedic period (b) Sultanate period 41. Which one amongst the following is the oldest
(c) Maurya period (d) Gupta period Dynasty?
31. The two colossal images of the Buddha at Bamiyan (a) Pallava Dynasty (b) Chola Dynasty
were works of (c) Maurya Dynasty (d) Gupta Dynasty
(a) the Maurya Art 42. Hieroglyphic script meant
(b) the Gupta Art (a) Sacred script (b) Religious script
(c) the Gandhara Art (c) Sumerian script (d) Iraqi script
(d) the early Mathura school of Art
43. Egyptians were highly skilled in the art of
32. The earliest extent (still standing) temples date from
(a) science (b) medicine
(a) the Vedic period (b) the Maurya period (c) surgery (d) astronomy
(c) the Gupta period (d) the Shunga period
44. Oracles foretold the
33. Most of the Ajanta paintings were done during the
(a) past (b) present
period of (c) future (d) death
(a) Harsha Vardhana (b) Late Guptas
(c) Mauryas (d) Kushanas 45. Copan of the Mayan civilisation was famous for a
(a) museum (b) library
34. Who among the following has been called the (c) observatory (d) hospitals
‘Napoleon of India’?
(a) Samudragupta (b) Chandragupta 46. Mayans worshipped the
(c) Ashoka (d) Harsha Vardhana (a) Maize goddess (b) Rain god
(c) Sun god (d) Wind god
35. Which of the following Gupta rulers was equally
great in the art of peace, wars and personal 47. The Aztec empire had a unique feature
accomplishment? (a) gardens (b) chinampas
(a) Chandragupta I (b) Chandragupta II (c) pools (d) stadiums
(c) Kumaragupta I (d) Skandagupta 48. The Aztecs were also known as
36. Raja Gotami-Putra Siri Satakshani was a ___ ruler. (a) Tenochtitlan (b) Toltecs
(c) Mexicans (d) Incas
(a) Satavahana (b) Pallava
(c) Chalukya (d) Kushan 49. The Inca civilisation was famous for an unique device
37. Amongst the following, who contributed the to remember called
(a) memory aid (b) quipu
Chalukya in Western India was that of the
(c) codices (d) tables
(a) Parthians (b) Indo-Greeks
(c) Kushanas (d) Guptas 50. Macchu Picchu was a fortress city in the
(a) Inca’s civilisation
38. During the Gupta period which of the following (b) Maya’s civilisation
places was not a seat of learning? (c) Aztec’s civilisation
(a) Vaishali (b) Mathura (c) Pataliputra (d) Kanchi (d) African civilisation

High Skill Questions


1. Which of the following is the correct chronological 2. Arrange the following in chronological order
order? 1. Skandagupta 2. Rajendra Choladeva I
(a) Kushanas-Guptas-Sungas-Mauryas 3. Chandragupta I 4. Harsha Vardhana
(b) Mauryas-Sungas-Kushanas-Guptas (a) 1 3 2 4 (b) 2 3 1 4
(c) Guptas-Kushanas-Mauryas-Sungas
(d) Sungas-Mauryas-Guptas-Kushanas (c) 3 1 4 2 (d) 4 1 3 2
536 Study Package for NTSE

3. Which one of the following ports handled the North 18. Which of the following groups of animals is depicted
Indian trade during the Gupta period? on the Pasupati seal from Mohenjodaro?
(a) Tamaralipti (b) Broach (a) Lion, Elephant, Buffalo and Bull
(c) Kalyan (d) Cambay (b) Tiger, Elephant, Rhinoceros and Buffalo
(c) Lion, Rhinoceros, Buffalo and Horse
4. The silver coins issued by the Guptas were called (d) Bull, Buffalo, Elephant and Rhinoceros
(a) rupayaka (b) karshapana
(c) dinara (d) pana 19. Which of the following Harappan sites has yielded
evidence of rice?
5. Which of the following Gupta rulers defeated the
(a) Kalibangan (b) Lothal
Huna ruler, Mihirakula?
(c) Harappa (d) Rakhigarhi
(a) Skandagupta (b) Buddhagupta
(c) Narasimhagupta (d) Yasodharman 20. The word Mesopotamia means the land
6. Who among the following literary figures of the Gupta (a) between two rivers (b) between two oceans
(c) between two mountains (d) between two islands
Age is given the title of ‘Indian Shakespeare’?
(a) Harisena (b) Kalidasa 21. In the courtyard of the Quwwatul Islam Mosque of
(c) Vishakhadatta (d) Bharavi Delhi stands the famous iron pillar in the memory
7. Carbon dating is of
(a) study of ancient culture (a) Ashoka (b) Chandragupta Vikramaditya
(b) study of religion (c) Harsha (d) Anangapala
(c) study of age of some object 22. The first Huna invasion took place during the reign of
(d) study of floods
(a) Skandagupta (b) Samudragupta
8. Mohenjodaro is famous for (c) Kanishka (d) Kumaragupta
(a) ash mounds (b) fire altars 23. Zero was invented by
(c) great bath (d) None of these
(a) Aryabhatta (b) Varahmihir
9. Harappan used weights in multiples of (c) Bhaskar I (d) an unknown Indian
(a) 10 (b) 12 (c) 14 (d) 16
24. The sacred books of the Jaina are called
10. The Indus Valley Civilisation was (a) Agama-Sidhanta (b) Angas
(a) rural civilisation (b) urban civilisation (c) Parvas (d) Upangas
(c) semi-urban civilisation (d) None of these
25. Who was the founder of Sishunaga dynasty?
11. Script of Indus Valley (a) Kalasoka (b) Udayin
(a) is written from left to right (b) has 98 letters (c) Sishunaga (d) None of these
(c) is pictograpic (d) None of these
26. Who wrote Milandapanho?
12. Ashtadhyayi of Panini is a work on (a) Buddhadutta (b) Nagasena
(a) Sanskrit grammar (b) Sanskrit literature (c) Buddhaghosha (d) Kautilya
(c) Sanskrit play (d) Early buddhist text
27. Jatakas are
13. Sumerians were first in developing of (a) poems (b) stories (c) speeches (d) a caste
(a) iron (b) writing (c) wheel (d) mettallurgy
28. Buddha allowed women to enter the Sangha to
14. Which place denotes that Harappa had connection accommodate the wishes of
with sea? (a) Maya (b) Amrapali
(a) Kalibangan (b) Lothal (c) Yasodhara (d) Gautami Prajapati
(c) Mohenjodaro (d) Ropar
29. Which one of the following Buddhist texts contains
15. Which one of the following sites is famous for the rules of monastic life?
pre-historic paintings? (a) Abhidhamma Pitaka (b) Digha Nikaya
(a) Bagh (b) Ajanta (c) Vinaya Pitaka (d) Vibhasha Sastra
(c) Bhimbetka (d) Amrawati
30. In which of the following texts the rules of conduct for
16. The pyramid of Gizeh was built by the Buddhist orders are collected?
(a) Chepos (b) Tutankhamen (a) Vinaya Pitaka (b) Sutta Pitaka
(c) Rameses II (d) Thebes (c) Abhidhamma Pitaka (d) Anguttara Nikaya

17. The sphinx was carved out of 31. Buddhist literature is mainly written in language
(a) rocks (b) piece of stone (a) Prakrit (b) Pali
(c) minerals (d) gold (c) Sanskrit (d) All of these
Ancient India 537
32. Buddhism was totally against 42. Who was court physician of Kanishka?
(a) the caste system (b) the Vedas (a) Nagasena (b) Sushruta
(c) old rituals (d) supremacy of Brahmins (c) Charaka (d) Ashwaghosha
33. Nandas had their capital in 43. Patanjali, the great grammarian was a contemporary of
(a) Patliputra (b) Magadha (a) Chandragupta (b) Vasumitra
(c) Vaishali (d) Gaya (c) Pushyamitra (d) Samudragupta
34. Who of the following were the first Non-Kshatriya 44. Satvahana Dynasty was founded by
ruler? (a) Simuka (b) Satakarni (c) Hall (d) Pulumavi
(a) Mauryas (b) Shishunags
45. Who deciphered the Brahmi script on Ashokan
(c) Haryankas (d) Nandas
edicts?
35. Bimbisara, the ruler of Magadh belonged to (a) Metcalf (b) Wilson Jones
(a) Haryanka Dynasty (b) Shishunag Dynasty (c) James Princep (d) None of these
(c) Nanda Dynasty (d) Maurya Dynasty
46. Which of the following animals doesn’t appear on
36. Kharoshthi script was Sarnath pillar of Ashoka?
(a) an Indian development (a) Horse (b) Lion
(b) an impact of Persian invasion (c) Tiger (d) Bull
(c) developed independently
(d) None of the above 47. Who among the following mentioned that slavery was
absent in ancient India?
37. ‘Indica’ the book was written by (a) Herodotus (b) Plutarch
(a) Panini (b) Herodotus (c) Megasthenese (d) Plalemy
(c) Dimechus (d) Megasthenese
48. Who among the following rulers was the first to
38. Which script is mainly used in Ashokan edicts? construct the Sudarsana lake in Saurashtra?
(a) Sanskrit (b) Prakrit (c) Pali (d) Brahmi (a) Chandragupta Maurya (b) Ashoka
39. Alaxender’s invasion India took place in (c) Rudradaman (d) Skandagupta
(a) 320 BC (b) 326 BC 49. Who among the following Indo-Greek rulers has been
(c) 336 BC (d) None of these mentioned in the Indian Buddhist literature?
(a) Demetrius (b) Menander
40. The rock edict of Ashoka were discovered in
(c) Strato (d) Antialcidas
(a) 13th century (b) 18th century
(c) 19th century (d) 20th century 50. After Mauryas who were the first ruler of
North-Western India?
41. ‘Arthashashtra’ written by Kautilya is a book on
(a) Bactrian Greek (b) Sakas
(a) economy (b) polity
(c) Satvahana (d) Prathiharas
(c) society (d) None of these

Answers
Elementary Level
1. (a) 2. (c) 3. (b) 4. (a) 5. (a) 6. (d) 7. (c) 8. (d) 9. (a) 10. (b)
11. (b) 12. (b) 13. (a) 14. (a) 15. (a) 16. (c) 17. (a) 18. (b) 19. (b) 20. (a)
21. (d) 22. (d) 23. (b) 24. (d) 25. (c) 26. (c) 27. (d) 28. (b) 29. (b) 30. (d)
31. (b) 32. (c) 33. (b) 34. (a) 35. (b) 36. (a) 37. (d) 38. (a) 39. (c) 40. (c)
41. (c) 42. (c) 43. (b) 44. (c) 45. (c) 46. (a) 47. (b) 48. (a) 49. (b) 50. (a)

High Skill Questions


1. (a) 2. (c) 3. (b) 4. (a) 5. (d) 6. (b) 7. (c) 8. (c) 9. (d) 10. (b)
11. (c) 12. (a) 13. (a) 14. (b) 15. (c) 16. (a) 17. (a) 18. (b) 19. (b) 20. (a)
21. (a) 22. (d) 23. (d) 24. (b) 25. (c) 26. (b) 27. (b) 28. (d) 29. (c) 30. (a)
31. (b) 32. (b) 33. (b) 34. (d) 35. (a) 36. (b) 37. (d) 38. (c) 39. (b) 40. (c)
41. (b) 42. (c) 43. (a) 44. (a) 45. (c) 46. (c) 47. (c) 48. (c) 49. (b) 50. (a)
Chapter

2
Medieval India
The Cholas (AD 846-1279) Arab Conquest of Sind
Chola Dynasty Sind was then ruled by Dahir. Muhammad-bin-Qasim
(of Iraq) defeated Dahir in AD 712. This is considered to
Founder Vijayalaya, their capital was Tanjore. Cholas be the first invasion of India by the Arabs.
were founder of Pallavas.
Raj Raj I It was a Chola king. He built Vrihideshwar Muhammad Ghori
temple at Tajore devoted to God Vishnu. Chundmoney The real founder of the Muslim Empire in India was
Vihar was constructed at Nagpattnam by Shailandra Shihab-ud-Din Muhammad Ghori or Muhammad of
(king of SE) on his permission. Ghazni Dynasty.
Rajendra I Raj Raj I was followed by his son Rajendra I. l He was a ruler of a small kingdom in Afghanistan. But
He constructed a temple called Gangaikond Cholpuram he was interested in conquering Northern India and
at the mouth of Ganga river. adding it to his kingdom and not merely in getting gold
and jewellery like Mahmud. His first invasion was
The Rajputs directed against Multan in AD 1175, which was
successful. By 1182, Sindh was also captured. Punjab
Rajputs rose to political importance in AD 9th and 10th was captured by AD 1186.
centuries. It is thought that they were of foreign origin l Prithviraj Chauhan, who was the king of Delhi at that
that came as conquerors and settled in West India. time, received contingents from other Rajput kings and
Of all the Rajput clans, four clans – Pratihara or defeated him in the First Battle of Tarain (1191). But
Pariharas of South Rajasthan, Chauhans of East he defeated Prithviraj in the Second Battle of Tarain
Rajasthan, Chalukyas or Solankis of Kathiawar and in 1192. He defeated Jaichand (Gadwal Rule) in Battle
Parmaras or Pawars of Malwa – claimed descent from a of Chhandawar was in 1194. He captured Delhi and
mythical figure that arose out of a sacrificial fire pit Ajmer and thus laid the foundation of Muslim Rule in
India.
near Mt Abu. So, they were called Agnikula or fire
family. l He died in AD 1206, leaving Qutab-ud-Din Aibak, his
slave the charge of the empire.
Two main clans of Rajputs are : Suryavansha (Sun
family) and Chandra Vansha (Moon family).
Delhi Sultanate (AD1206-1526)
The Chauhans Delhi Sultanate refers to the many Muslim Dynasties
Ruled over Shakambri (Modern Sambhar-Rajasthan). that ruled in India from 1206 to 1526. Several Turkish
and Afghan dynasties ruled from Delhi
Important king Prithviraj Chauhan. ‘Prithviraj Raso’
was composed by his court poet Chandbardai. l Ilbari Dynasty (1206-1290)
l Khilji Dynasty (1290-1320)
First Battle of Tarain (1191) Prithviraj Chauhan
l Tughlaq Dynasty (1320-1413)
defeated Muhammad Ghori.
l Sayyid Dynasty (1414-1451)
Second Battle of Tarain (1192) Muhammad Ghori
l Lodhi Dynasty (1451-1526)
defeated Prithviraj Chauhan.
Medieval India 539
Ilbari Dynasty Also called as Slave Dynasty. Iltutmish whole country on a stable basis, he died. Humayun did a
was the real founder of Delhi Sultanate. He made Delhi blunder by dividing his empire among his three
as his capital in place of Lohore. Illtutmish was brothers – Kamran, Hindal and Askari.
succeeded by his daughter Razia, the first and the last l Sher Shah Suri gradually gained power during his time.
Muslim ruler of Medieval India. Humayun was attacked by Sher Shah at Chausa
The Khilji’s Jalaluddin Khilji founded this dynasty. It is (Battle of Chausa) in 1539, but escaped. But in the
greatest ruler was Alauddin Khilji. Under Ala-ud-din Battle of Kannauj (also called Battle of Bilgrama) in
Khilji, the Khiljis became known for having repeatedly 1540, he was defeated by Sher Shah and had to flee. He
defended South Asia against the Mongel Envasions. had to pass nearly 15 yr (1540-1555) in exile.
l Humayun had the chance to return in 1555. Sher Shah,
The Tughlaqs Ghiyasuddin Tughlaq is the founder of
the victor of Kannauj, died in 1545. He was succeeded by
this dynasty. Mohammadbin Tughlaq was the most his son Islam Shah, who ruled upto 1553. He was
remarkable personality among the sultans of Delhi who succeeded by Muhammad Adil Shah. He was very fond
was known for his failed experiments. of pleasures and left the affairs of his Government in the
The Sayyids Khizr Khan founded this dynasty and hands of Hemu, his minister.
ruled as a deputy of Timur’s son and successor, l His authority was challenged by Ibrahim Shah and
Shah Rukh. Sikandar Shah. There were a large number of bloody
battles among the various rivals. The net result of all
The Lodhis Bahlol Lodhi was the founder of this this was that the Suri Empire was broken up. Bairam
dynasty. Sikandar Lodhi and Ibrahim Lodhi were two Khan, Humayun’s most faithful officer, helped
famous ruler of this dynasty. Humayun in this. Humayun died in 1556.
In 1526, the Delhi Sultanate was absorbed by the
emerging Mughal Empire.
Akbar (1556-1605)
Jalaluddin Muhammad Akbar was the son of Humayun
The Mughals and Hamida Banu Begam. He was born at Amarkot in
1542. Bairam Khan coronated him at Kalanaur when he
was 14 yr old.
Babur (1526-1530)
l Bairam Khan represented him in the Second Battle of
He was a descended of Timur on his father’s side and Panipat in 1556 against Hemu Vikramaditya. Hemu,
Chengiz Khan on his mother’s side. His family belonged the PM of Muhammad Shah Adil of Bengal, was defeated.
to the Chaghtai section of the Turkish race and were Hemu is considered the last Hindu king of Delhi.
commonly known as Mughals. He originally ruled over l Between 1556 and 1560, Akbar ruled under Bairam
Ferghana (Afghanistan). Khan’s regency. (Bairam Khan was assassinated at
l Babur was invited to attack India by Daulat Khan Lodhi Patan, Gujarat, on his way to Mecca by one of his old
(Subedar of Punjab), Alam Khan (uncle of Ibrahim Afghani enemies).
Lodhi) and Rana Sanga. He defeated Ibrahim Lodhi in
the First Battle of Panipat in 1526. Important Aspects of Akbar’s Rule
l He then defeated Sangram Singh (Rana Sanga) of l
Ralph Fitch (1585) was the first Englishman to visit Akbar’s
Mewar in the Battle of Khanwa in 1527. This court.
weakened the Rajput confederacy and strengthened l
Abolished jaziyah in 1564 and the tax on pilgrims. The use of
Babur’s position. Babur took the title of ‘Ghazi’ after beef was also forbidden.
that. He defeated another Rajput ruler, Medini Rai (of l
Regularly visited the shrine of Sheikh Muinuddin Chishti at
Chanderi) in the Battle of Chanderi in 1528. Finally, Ajmer.
he defeated the Afghan chiefs (under Mahmud Lodhi l
Believed in Sulh-i-Kul or peace to all.
brother of Ibrahim Lodhi) in the Battle of Ghagra in l
Built Ibadatkhana (Hall of prayers) at Fatehpur Sikri.
1529. He died in 1530. l
Formulated an order called Din-i-Ilahi or Tauhid-i-Ilahi in 1581.
Birbal, Abul Fazl and Faizi joined the order.
l His memoir, the Tuzuk-i-Baburi in Turki language is a l
His Land Revenue System was known as Todar Mal Bandobast
classic of world literature. It shows his humane outlook
or Zabti System.
and sensitivity to the beauty of nature. l
Also introduced the Mansabdary System to organise the
nobility as well as the army. Mansabdar meant holder of a rank.
Humayun (1530-40 and 1555-56)
The throne inherited by Humayun was not a bed of l Akbar conquered Malwa in 1561 defeating Baz
roses. Babur had practically got no time to consolidate Bahadur. He was later made the Mansabdar to honour
his position and authority. Before Babur could put the his skill as a musician. Then, defeated Garh-Katanga
540 Study Package for NTSE

(ruled by Rani Durgawati) followed by Chittor and l Painting reached its zenith in India under Jahangir.
Ranthambhore. Special progress was made in portrait painting and
l Akbar followed a policy of reconciliation with the painting of animals. Use of ‘Halo’ or Divine Lights
Rajputs. Some of the Rajput princes entered into started under Jahangir.
matrimonial alliances with him. In 1562, he married the l Captain Hawkins (1608-11) and Sir Thomas Roe
eldest daughter of Raja Bharmal of Jaipur. In 1570, he (1615-1619) visited his court.
married princesses from Bikaner and Jaisalmer. Later in
1584, Prince Salim was married to the daughter of Raja Shah Jahan (1627-1658)
Bhagwan Das.
Shah Jahan was an able general and administrator. His
All these activities paved the way for friendship between
Rajputs and Mughals (except Mewar).
Childhood name was Khuman. His policy of annexing
the Deccan was quite successful. He made his son,
l Won Gujarat in 1572. It was in order to commemorate
his victory of Gujarat that Akbar got the Buland
Aurangzeb, the Viceroy of Deccan in 1636.
Darwaza constructed at Fatehpur Sikri. Last 8 years of Shahjahan’s life were very painful, as
l Akbar defeated Mewar’s Rana Pratap Singh in the there was a brutal war of succession among his four sons
Battle of Haldighati in 1576. Later, Raja Maan Singh – Dara, Shuja, Aurangzeb and Murad. Dara was his
conquered Bihar, Bengal and Orissa for Akbar. favourite but Aurangzeb was the ablest. Ultimately,
At his death in 1605, Akbar’s empire included Kashmir, Aurangzeb took control and he was made him prisoner
Sind, Kandhar and extended as far as the Godavary in in the Agra Fort, being looked after by his daughter,
the Deccan. Jahan Ara, till his death in 1666.
Jahangir (1605-1627) Aurangzeb (1658-1707)
Akbar’s eldest son Salim assumed the title of At the time of Shahjahan’s illness, Dara was in Delhi
Nur-ud-din-Muhammad Jahangir and ascended the and the other brothers were in different places – Shuja
throne. He mostly lived in Lahore which he adorned in Bengal, Murad in Gujarat and Aurangzeb in Deccan.
with gardens and buildings. But soon, his eldest son Aurangzeb first defeated the Imperial army in the
Khusro revolted, which was suppressed. The fifth Sikh Battle of Dharmatt and then defeated a force led by
guru, Guru Arjun Dev, had helped Khusro. So, he was Dara in the Battle of Samugarh. Thereafter, he entered
executed after 5 days of torture. Agra and crowned himself with the title of ‘Alamgir’
l Jahangir’s greatest failure was the loss of Kandahar to (conqueror of the world).
Persia in 1622.
l Under Aurangzeb, the Mughal Empire reached its
l The most important event in Jahangir’s life was his greatest extent. The Mughal conquests reached the
marriage to Mehr-un-Nisa, the widow of Sher Afghan, in territorial climax during his reign, as Bijapur (1686) and
1611. The title of Nur Jahan was conferred on her. She Golconda (1687) were annexed to the Mughal empire.
had great influence on Jahangir’s life. His reign can be broadly divided into two 25 yr periods:
l Jahangir had a chain of justice outside his palace in first in the affairs of North India when the Maratha
Agra (called Zanzir-i-Adil). He also laid a number of power under Shivaji emerged and second marked by his
gardens, such as the Shalimar and Nishat gardens in preoccupations about the affairs of Deccan.
Kashmir.

Later Mughals/Fall of the Mughals


The empire lost power after Aurangzeb’s rule. His successors were weak and incapable rulers.
Some of the important name were Bahadur Shah → Jahandar Shah → Farrukh Siyar → Muhammad Shah → Ahmad Shah →
Alamgir II → Shah Alam II → Akbar II → Bahadur Shah II → Akbar II → Bahadur Shah II

Contemporary Provincial Kingdoms


Bahmani Kingdom
Hasan Gangu, who assumed the title of Abul Muzaffar Alauddin Bahman Shah with the approval of the nobility
(1347) founded Bahmani kingdom.
Medieval India 541
Bahmani Kingdom
Kingdom Year Founder Dynasty
Berar 1484 Fataullah Imad Imad Shahi
Bijapur 1489 Yusuf Adil Khan Adil Shahi
Ahmadnagar 1490 Malik Ahmad Nizam Shahi
Golkonda 1512/1581 Quli Shah Qutab Shahi
Bidar 1526/1527 Amir Ali Barid Barid Shahi

Vijayanagar Empire
Vijayanagar Empire referred to as the kingdom of Bisnagar by the portuguese, was an empire based in South India,
in the Deccan, plateau region. It was established in 1336 by Harilara I and his brother Bukka Raya I of Sangam
Dynasty.
Vijayanagar Dynasties
Dynasty Founder Period
Sangam Harihar and Bukka 1336-1485
Saluva Saluva Narasimha 1485-1505
Tuluv Vira Narasimha 1505-1570

Important Dynasties of Medieval India


Dynasty Period Founder Important Ruler
Later Chola 1070-1279 Koluthunga • Rajaraja Chola
Slave 1206-1290 Qutb-ud-din Aibak • IItutmish, Razia Sultan, Balban
Khilji 1290-1320 Jalal-ud-din-Khilji • Ala-ud-din Khilji Mohammad-bin-Tughlaq
Tughlaq 1320-1414 Ghiyas-ud-din-Tughlaq • Muhammad-bin-Tughalq, Firoz Shah Tughlaq
Sayyid 1414-1450 Khizr Khan • Farid Khan
Lodi 1451-1526 Bahlol Lodi • Sikander Lodi, Ibrahim Lodi
Vijayanagar 1336-1565 Harihara and Bukka • Deva Raya-I, Deva Raya-II, Krishna Deva Raya
Bahmani 1347 Alauddin Hasan • Feroz Shah
Mughal 1526-1857 Babur • Humayun, Akbar, Jahangir, Shah Jahan, Aurangzeb
Sur 1540-1556 Sher Shah Suri • Salim Shah, Adil Shah

Sikhs l Guru Arjundev He completed the construction of


Amritsar. He compelled Adi Granth. He was exacuted
Sikhs were contemporary of Mughals. on the order of Jahangir.
l Guru Nanak He is the founder and first Guru of Sikhs. l Guru Hargobind He is called as saccha padshah. He
He was born in Nankana Sahib Talwandi (Pak) Father – constructed Aakal Takht. He converted Sikhs into
Kulu Mehta, Mother – Trupta. fighter caste.
l Guru Angad He is second Sikh Guru. He was l Guru Har Rai He is the seventh of ten living Guru Ji of
contemporary of Humayun. He invented Gurumukhi the Sikhs who became Guru Ji on 8 March, 1644.
script. l Guru Harkrishan He is the eight of the eleven Sikh
l Guru Amardas He became Sikh Guru at the age of 72 Gurus. He became Guru Ji on 7 October, 1661.
following is the footsteps of his teacher. Sri Guru Angad l Guru Tegh Bahadur Is hanged on the order of
Dev Sahib. Aurangzeb.
l Guru Ramdas Akbar gave him land for establishment l Guru Gobind Singh Last Sikh Guru, and founder of
of Amritsar. Khalsa. After him, Banda Bahadur took the command of
Sikhs.
Target Exercise
Elementary Level
1. Prithviraj Chauhan belonged to the Rajput Dynasty of 14. Akbar defeated Rana Pratap in the battle of
(a) Chandellas (b) Gahadavalas Haldighati in the year
(c) Tomaras (d) Guhilots (a) 1570 (b) 1576
(c) 1588 (d) 1596
2. Who was the first and the last woman ruler of Delhi?
(a) Chand Bibi (b) Noor Jahan 15. Buland Darwaza is situated in
(c) Razia Sultan (d) Mumtaz Mahal (a) Delhi (b) Fatehpur Sikri
(c) Lucknow (d) Meerut
3. Which of the following generals of Alauddin Khilji
conquered Deccan a number of times? 16. The capital of the Mughal Empire was shifted from
(a) Ulugh Khan (b) Nusrat Khan Agra to Delhi by
(c) Malik Kafur (d) None of these (a) Jahangir (b) Aurangzeb
(c) Humayun (d) Shahjahan
4. Mughal Dynasty was to Bahadur Shah Zafar as
Lodhi Dynasty was to 17. The Chola empire reached its zenith under
(a) Bahlol Lodhi (b) Daulat Khan Lodhi (a) Parantaka I (b) Raj Raj
(c) Ibrahim Lodhi (d) Sikandar Lodhi (c) Rajendra Chola I (d) Rajendra Chola II

5. The Delhi sultanate virtually ended due to the 18. The titles of ‘Mummadi Chola’, ‘Cholamartanda’ and
invasion of ‘Raja Kesari Aromoli’ were assumed by which of the
(a) Chengiz Khan (b) Babur following Chola rulers?
(c) Nadir Shah (d) Timur (a) Rajendra I (b) Vijayalaya
(c) Rajaraj I (d) Rajadhiraja
6. The city of Agra was founded in 1504 by
19. Which of the following is similar to the Taj Mahal in
(a) Rana Sanga (b) Ibrahim Lodhi
(c) Firoz Tughlaq (d) Sikandar Lodhi
construction ?
(a) Akbar’s tomb (b) Jahangir’s tomb
7. Who amongst the following was the ruler of Chittor, (c) Humayun’s tomb (d) Bibi-ka-maqbara
when Alauddin Khilji attacked and conquered it in 20. The most famous musician at the court of Akbar was
AD 1303 ?
Tansen. His original name was
(a) Rana Sanga (b) Rana Kumbha
(a) Baz Bahadur (b) Makaranda Pande
(c) Rana Ratan Singh (d) Rana Hammir
(c) Lal Kalwant (d) Ramtanu Pande
8. The official language of the Delhi sultanate was 21. The mausoleum of Sher Shah is at
(a) Urdu (b) Arabic (a) Delhi (b) Sasaram
(c) Persian (d) Hindi (c) Agra (d) Lahore
9. Babur came to India originally from 22. Buland Darwaza, built by Akbar, is to commemorate
(a) Ferghana (b) Khiva
the victory of
(c) Khorasan (d) Seistan
(a) Orissa (b) Bengal
10. When Akbar was young, his guardian was (c) Delhi (d) Gujarat
(a) Hemu (b) Faizi 23. The tomb of Babur is at
(c) Abul Fazl (d) Bairam Khan (a) Lahore (b) Kabul
11. Sher Shah is well known for his administrative skill, (c) Sasaram (d) Sikandra
especially his 24. When did Vasco da Gama come to India?
(a) market control steps (b) land revenue system (a) 1492 (b) 1498 (c) 1398 (d) 1542
(c) mansabdari system (d) law and order
25. Sher Shah’s real name was
12. Sher Shah died while fighting in (a) Hemu (b) Faizi
(a) Chausa (b) Kalinga (c) Bahadur (d) Farid
(c) Kalinjar (d) None of these
26. Which Mughal emperor gave land for the construction
13. The coin rupia was first issued by of the Golden Temple at Amritsar?
(a) Sher Shah (b) Alauddin Khilji
(a) Akbar (b) Jahangir
(c) Akbar (d) Muhammad-bin-Tughlaq
(c) Shahjahan (d) Aurangzeb
Medieval India 543
27. Sher Shah is credited for introducing for the first 34. Which of the following is wrongly matched with his
time the following measures except contemporary Mughal king?
(a) postal system (a) Rana Sanga — Babur
(b) spy system (b) Prithviraj Chauhan — Akbar
(c) currency system (c) Jujhar Singh — Shahjahan
(d) land revenue system based on scientific principles (d) Jaswant Singh — Aurangzeb

28. The ‘Jaziya’ was abolished by 35. Chand Bibi, who conceded Berar to Akbar, was the
(a) Muhammad-bin-Tughlaq (b) Babur ruler of
(c) Akbar (d) Sher Shah (a) Golconda (b) Bahmani
(c) Bijapur (d) Ahmednagar
29. Which of the following contemporary sources of the
Mughal period is especially useful for obtaining the 36. In the Mughal administration, military recruitment
information on the agrarian condition? was being looked after by
(a) Akbaranamah (b) Ain-i-Akbari (a) Diwan (b) Wazir (c) Bakshi (d) Kotwal
(c) Tariksh-i-Firishta (d) Tuzuk-i-Baburi
37. The king generally considered to be the greatest ruler
30. Who among the following Mughal rulers was of Kashmir in the 15th century was
ignorant of the art of reading and writing? (a) Yusuf Shah (b) Zain-ul-Abidin
(a) Babur (b) Humayun (c) Haider Shah (d) Shah Mirza
(c) Akbar (d) Jahangir
38. Who among the following was regarded as ‘Zinda pir’?
31. Revenue system during Akbar’s reign was in the (a) Babur (b) Akbar
hands of (c) Jahangir (d) Aurangzeb
(a) Bairam Khan (b) Man Singh
(c) Birbal (d) Todar Mal
39. Bernier visited India during the reign of
(a) Akbar (b) Jahangir
32. The East India Company was founded in London (c) Shahjahan (d) Aurangzeb
during the reign of
40. The foreign traveller, who visited and described
(a) Jahangir (b) Akbar
Vijayanagar soon after its fall in the battle of
(c) Shahjahan (d) Aurangzeb
Talikota, was
33. Which of the following Rajput Dynasties did not (a) Abdur Razzak
surrender to Akbar? (b) Ceasar Frederick
(a) Parmar (b) Pratihara (c) Nicolo Conti
(c) Rathor (d) Sisodiya (d) Nuniz

High Skill Questions


1. The dynasty of Harihara and Bukka, the founders of 5. Which of the following pairs is not matching correct?
the Vijayanagar Empire is known as Sangam (a) Abolition of Jaziya — AD 1564
Dynasty, because (b) Abolition of the practice of enslavement of prisoners of
(a) Vijayanagar was situated on the confluence of two rivers war—AD 1562
(b) a mixture of races resided in Vijayanagar (c) Abolition of Sati—AD 1560
(c) the name of their father was Sangam (d) Abolition of pilgrim tax—AD 1563
(d) Sangam literature was produced during their rule
6. Tho noble who introduced important reforms in the
2. The battle of Tarain was fought between landrevenue administration of Mughals in deccan
(a) Prithviraj and Ghori (b) Jaichand and Ghori during the reign of Shahjahan was
(c) Prithviraj and Jaichand (d) Babur and Rana Sanga (a) Murshid Quli Khan (b) Asaf Khan
(c) Mahabat Khan (d) Khan-i-Zama
3. Sher Shah defeated Humayun in battle of
(a) Panipat (b) Chausa 7. When was India invaded by Nadir Shah?
(c) Agra (d) Karnal (a) AD 1779 (b) AD 1739
(c) AD 1749 (d) AD 1735
4. Mahzar of 1579 was drafted by
(a) Sheikh Mubarak 8. Which of the following was the court language during
(b) Sheikh Abdul Nabi the reign of Akbar?
(c) Abul Fazl (a) Hindi (b) Urdu
(d) Faizi (c) Persian (d) Arabic
544 Study Package for NTSE

9. Akbar was born at 24. Who translated Ramayana into Persian?


(a) Delhi (b) Lahore (a) Amir Khusrau (b) Malik Muhammad Jaisi
(c) Agra (d) Amarkot (c) Abdul Qadir Badayuni (d) Mulla Sheereen

10. With which Mughal general did Shivaji sign the 25. Who among the following was not involved in the
Treaty of Purandhar? tripartite struggle for the possession of the imperial
(a) Jaswant Singh (b) Jai Singh city of Kannauj?
(c) Shaista Khan (d) Diler Khan (a) The Cholas (b) The Rashtrakutas
(c) The Palas (d) The Gurjara Pratiharas
11. Which of the following commodities constituted the
main item of exports in Mughal India? 26. City of Vijayanagar was located on the river
(a) Opium and indigo (b) Cotton and silk fabrics (a) Krishna (b) Cauveri
(c) Bullion and horses (d) Sugar and salt petre (c) Tungabhadra (d) Pennar

12. Tax free villages granted to the learned brahmins in 27. Who among the following Sultans crushed
ancient India were known as Turkan-i-Chihalgani?
(a) Agrahara (b) Nagastora (a) Iltutmish (b) Balban
(c) Agrasseni (d) Bhukti (c) Alauddin Khalji (d) Mohammad Tughlaq

13. Who among the following rulers was known by the 28. In which of the following places Ibadatkhana was
title of ‘Andhra Bhoja’? constructed by Akbar?
(a) Devaraya II (b) Krishnadeva Raya (a) Agra (b) Sikandar
(c) Harihara (d) Vidyaranay (c) Delhi (d) Fatehpur Sikri

14. Who among the following foreign travellers visited 29. Arrange the following rulers of Delhi Sultanate in
India during AD 7th century? chronological order of their reign and choose the correct
(a) Alberni (b) Fa-Hien Answer
(c) Hieun-Tsang (d) Suleman 1. Qutubuddin Aibak 2. Balban
15. Ibn Batuta was traveller from 3. Illutmish 4. Nasiruddin
(a) Africa (b) Arab (c) Iran (d) Morocco (a) 4 1 3 2 (b) 1 3 4 2
(c) 1 2 3 4 (d) 3 1 4 2
16. Jaipal belonged to
(a) the Chandela Dynasty (b) the Hindushahi Dynasty 30. The Mahzar of 1579, conferred on Akbar
(c) the Pratihara Dynasty (d) the Paramara Dynasty (a) infallibility degree
(b) supremacy over religion
17. Who was founder of Vijayanagar Empire? (c) right to choose one of the conflicting juristic opinions
(a) Krishnadev Rai (b) Harihar and Bukka (d) divine right of kingship
(c) Dev Rai II (d) Sadashiv
31. Who among the following Mughal emperors has the
18. Who of the Sultans transferred his capital to Delhi? largest number of Hindu nobles?
(a) Alauddin Khilji (b) Balban (a) Akbar (b) Jahangir
(c) Gayasuddin Tughlaq (d) Iltutmish (c) Shahjahan (d) Aurangzeb
19. Which language was used by the ruler Krishnadeva 32. The Sikh Guru who founded the city Taran Taran
Rai in his work ‘Amuktamalyada’? (a) Guru Ramdas (b) Guru Arjun
(a) Sanskrit (b) Tamil (c) Guru Gobind Singh (d) Guru Tegh Bahadur
(c) Telugu (d) Kannada
33. The famous Kohinoor diamond presented to
20. Timur invaded India and sacked Delhi during the Shahjahan by
reign of (a) Asad Khan (b) Mir Jumla
(a) Iltutmish (b) Muhammad-bin-Tughluq (c) Muhammad Amin (d) Sadulla Khan
(c) Nasiruddin Mahmud (d) Bahlol Lodhi
34. The Third Battle of Panipat was fought in the reign of
21. The city of Agra was founded by (a) Bajirao I (b) Bajirao II
(a) Sikandar Lodhi (b) Khizr Khan (c) Balaji Bajirao (d) Balaji Vishwanath
(c) Bahlol Lodhi (d) Firoz Tughlaq
35. The transformation of the Sikhs into a military,
22. ‘Hunuz Delhi Durasth’ (Delhi is yet far off ). Who has fighting community was begun by
said this? (a) Guru Hargovind (b) Guru Gobind Singh
(a) Ghazi Malik (b) Shaikh Salim Chisti (c) Guru Tegh Bahadur (d) Guru Arjun Dev
(c) Nizamuddin Aulia (d) Amir Khusrau
36. Who was the first emperor to permit the East India
23. Assignment of ‘Iqta’ in lieu of salary was stopped by Company to start trade in India?
(a) Iltutmish (b) Jalaluddin Khilji (a) Akbar (b) Jahangir
(c) Alauddin Khilji (d) Firoz Tughlaq (c) Shahjahan (d) Farrukh Siyar
Medieval India 545
37. The practice of ‘jaharokha darshan’ to the public was 39. Match the following
stopped by
Rulers Place of Tomb
(a) Jahangir
(b) Shahjahan A. Akbar 1. Sikandra
(c) Aurangzeb B. Shahjahan 2. Lahore
(d) None of the above C. Jahangir 3. Sasram
38. In which order did the following dynasties rule Delhi? D. Sher Shah 4. Agra

I. Salve II. Khilji Codes


III. Lodi IV. Saiyyad A B C D A B C D
V. Tughlaq (a) 3 4 1 2 (b) 4 3 2 1
(c) 1 2 4 3 (d) 1 2 3 4
(a) I, II, V, IV, III
(b) I, II, III, IV, V 40. Who wrote ‘Humayun-nama’?
(c) II, III, IV, V, I (a) Gulbadan Begum (b) Abul Fazi
(d) IV, V, III, II, I (c) Nur Jahan (d) Todar Mal

Answers
Elementary Level
1. (b) 2. (c) 3. (c) 4. (c) 5. (b) 6. (d) 7. (c) 8. (c) 9. (a) 10. (d)
11. (b) 12. (c) 13. (a) 14. (b) 15. (b) 16. (b) 17. (c) 18. (c) 19. (c) 20. (d)
21. (b) 22. (d) 23. (b) 24. (b) 25. (d) 26. (a) 27. (c) 28. (c) 29. (b) 30. (c)
31. (d) 32. (b) 33. (d) 34. (b) 35. (d) 36. (c) 37. (b) 38. (d) 39. (c) 40. (b)

High Skill Questions


1. (c) 2. (a) 3. (b) 4. (a) 5. (c) 6. (a) 7. (b) 8. (c) 9. (d) 10. (a)
11. (b) 12. (a) 13. (b) 14. (c) 15. (d) 16. (b) 17. (b) 18. (d) 19. (c) 20. (c)
21. (a) 22. (c) 23. (c) 24. (c) 25. (a) 26. (c) 27. (b) 28. (d) 29. (b) 30. (c)
31. (d) 32. (b) 33. (b) 34. (c) 35. (a) 36. (b) 37. (c) 38. (a) 39. (c) 40. (a)
Chapter

3
Events and Process
The Rise of Nationalism in Europe l In 1848, food shortage and wide spread unemployment
brought the population of Paris out on the road. Lonis
The idea of nationalism emerged as a force during the Philippe was forced to flee and the National Assembly
nineteenth century, which brought about a tremendous proclaimed France, a republic based on universal male
suffrage.
change in the political scenario of Europe. As a result,
nation-state emerged in place of the multi-national
The Making of Germany
dynastic empires of Europe.
In Germany, nationalist feelings were wide spread
l A nation is culmination of a long past of endeavours,
sacrifice and devotion. In a nation-state, not only its among middle class, who in 1848 tried to unite the
rulers, but the majority of its citizens shared a common different regions of the German confederation into a
identity which was gained by long struggle. nation-state governed by elected Parliament. The
l In the mid-eighteenth century, Europe, there were no liberal initiative to nation-building was repressed by
‘nation-state’ as we know them today. Europe were the combined forces of the monarchy and the military,
divided into kingdoms, Duchies and Cantons, whose supported by the large land owners of Prussia.
rulers had their autonomous territories. l The first step towards unification of Germany was
l After the industrialisation in the second half of the 18th unconsciously taken by Prussia, when it established the
century, new social classes came into being, viz., a Zolverian or custom union of all German states except
working class population and middle classes made of Austria and Hanover.
industrialists, businessman and professionals. l Prussia’s Chief Minister Otto Von Bismarck was the
architect of this process with the help of Prussian army
l In early 19th century, ideas of national unity emerged.
and bureaucracy.
The idea of nationalism is closely related with the
ideology of liberalism. The concept of liberalism emerged l William I, the Prussian emperor, though deeply
after the French Revolution. ‘Liberalism’ stood for conservative, believed in the national mission of Prussia
freedom for the individual and equality of all before to unify Germany.
the law. l Bismarck set the diplomatic stage for challenging
Austria as well as trying out his new enemy Denmark
The Age of Revolutions (1830-1848) over the question of the control of two Duchies of
Schleswig and Holstein. In 1864, Business fought a war
Liberalism and nationalism came to be increasingly against Denmark and took control of Schleswig.
associated with revolution in many regions of Europe l Bismarck followed aggressive foreign policy, the ‘Blood
like, Italian and German states, the provinces of the and Iron’ policy and relatively subdued domestic one,
Ottoman Empire, Ireland and Poland. playing with the nationalistic sentiments of the people of
l The first upheaval took place in France in July, 1830. Germany.
l The July Revolution sparked an uprising in Brussels, l Bismarck made Russia ‘neutral’ by helping her at the
which led to Belgium breaking away from the United time of polish insurrection. He also secured the
Kingdom of the Netherlands. neutrality to France and Italy.
Events and Process 547
l In 1870, Prussia defeated France and occupied Alsace In 1854, he supported Victor Emmanuel II in his efforts
and Lorraine. to unify the Italian states. In 1860, his ‘Red Short’
l On 18 January, 1871 an assembly comprising the troops defeated Austria to secure the independence of
princes of the German states, representatives of the Naples and Sicily, which later on joined Sardinia. In
army, important Prussian ministers including the Chief
Minister Bismarck gathered in the Hall of Mirrors at
1867, Garibaldi led an army of volunteers to Rome to
Versailles to proclaim the new German Empire headed fight the last obstacle to the unification of Italy. In 1870,
by Kaiser William I of Prussia. during the war with Prussia, France withdrew its troops
from Rome and the Papal states were finally joined to
The Unification of Italy Italy. In this way, Garibaldi had a great contribution to
The unification of Italy owed much to the diplomacy of achieve the unification of Italy.
Cavour, the Prime Minister of Piedmont from 1852 to
1860. The radicals, mostly republicans emphasised Colonialism and Imperialism
more on the contribution of Mazzini and Garibaldi. Colonialism as a system gives emphasis over the fact
l The other factors of the unification of Italy are the that a colony exists for the benefit of the metropolitan
concept of nationalism, role of Napoleon’s war, state. From fifteenth century onwards, the way for
Revolutions of 1820, 1830 and 1848. colonisation was started with the geographical
l Italy was politically divided into 11 states, though discovery and commercial revolution.
economic independence was always present. l The colonialism continued upto the end of the 18th
l The states of Tuscany, Modena, Parma and the Papal century.
joined hands with Sardinia l In the beginning of the 19th century, the colonies of
l By March, 1860 the entire central Italian states were Spain in Latin America got independence. In 1822,
unanimously united with Piedmont. Brazil became free from Portugal.
l Rome became a part of Sardinia and in this way the final l America War of Independence gave a serious jolt to
unification of Italy, was also achieved in 1871. colonialism because it proved that European powers
could not rely on the colony for their benefit for an
unlimited period.
Contribution of Mazzini and
l Some thinkers like Adam Smith, James Mill started to
Garibaldi criticise colonialism and mercantilism.
From 1870s onwards colonialism reappeared in altered from and it
Mazzini came to be known as neo-colonialism and imperialism.
He was a dedicated republicanist. He is known for his l In order to compensate the loss of the home market, the
noble efforts to achieve the unification of Italy. He is capitalists are looking for the market abroad. This
regarded as the spiritual force behind the unification phenomena explained imperialist expansion by
of Italy. He had formed a secret society called Young European powers.
Italy. On several occasions, he tried to organise the l Colonialism was associated with commercial capitalism
movement in favour of unification. He inspired the but neo-colonialism and imperialism was associated
with industrial and financial capitalism.
youths of not only Italy, but of the whole Europe to fight
for their motherland. But, he was basically an idealist
and was not ready to compromise with the reality. His The French Revolution
ideas had filled the minds of the people with such French Revolution 14 July, 1789, proved a landmark in
sentiments that Cavour was able to achieve the the world history. Thinker like Karl Marx tried to prove
unification of Italy without much difficulty. that it was the economic decline and stagnation which
resulted into a revolution in France. Liberal thinkers
Garibaldi regarded it as a legitimate political protests against the
He is perhaps the most celebrated of Italian freedom tyrannies.
fighters, who is knowns as ‘Physical Force’ or the ‘Sword
Different Sections in French Society
of Italy’. He met Mazzini in 1833 and joined the Young
l French society in the 18th century was divided into
Italy Movement for the unification of Italy. The uprising
privileged and non-privileged sections.
in Piedmont (1834) was suppressed and Garibaldi had
l There were three estates in the society, of which only the
to flee to South America. members of the third estate paid taxes.
548 Study Package for NTSE

l The first and the second estate belonged to the The Revolution Began
privileged section. The first estate consisted of the clergy
and the second estate comprised of the nobility. They l On 5 May, 1789, Louis XVI called together an assembly
enjoyed certain privileges by birth. The most important of the Estates General to pass proposals for new taxes.
of these was exemption from paying taxes to the state. l Voting was conducted in the past, according to the
l All members of the third estate had to pay taxes to the principle that each state had one vote. But now,
members of the third estate demanded that voting now
state. These include a direct tax, called taille and
be conducted by the assembly as a whole, where
number of indirect taxes.
each/member would have one vote. But, the King
rejected this proposal and the members of the third
Emergence of Middle Class estate boycotted the session.
The third estate consisted of the lower class and the l On 17 June, 1789, the third estate declared itself to be
middle class. The middle class consisted of merchants, National Assembly of France. The members of National
capitalists, intellectuals and different professional Assembly was busy at Versailles drafting a
groups like, lawyers or administrative officials. Constitution.
l The 18th century witnessed the emergence of this social l On July 14, the agitated crowd stormed and destroyed
groups ‘middle class’. the Bastile. Agitation spread from village to village.
l All of these were educated and believed that no group in l Peasants looted hoarded grain and burnt down
society should be privileged by birth. documents containing records of manorial dues. A large
number of nobles fled from their homes and migrated to
l A person’s social position must depend on his merit.
neighbouring countries.
These ideas of freedom and equal laws and opportunities
for all were put forward by thinkers like John Locke and
Jean Jacques Rousseau. France became a Constitutional Monarchy
l Monarch Louis XVI finally accepted that his powers
The Role of Philosophers would from now on be checked by a Constitution.
In French Revolution, the philosophers are supposed to l On 4 August, 1789, the assembly passed a decree
abolishing the feudal system of obligations and taxes.
have played a significant role. They made an attack over
ancient regime. Philosopher like Voltaire, who was a l The clergy were forced to give up their privileges.
‘Tithes’ (tax from the peasants) were abolished and
great exponent of individual freedom made severe
lands owned by the church were confiscated.
attack on autocratic nature of French monarchy,
l According to the Constitution of the National Assembly,
stupidity of nobility and superstition of Church.
the power insteading of being concentrated in the hands
l In his Two Treatises of Government Locke refuted of one person were now separated and divided among
the doctrine of the divine and absolute right of the the legislature, executive and judiciary.
monarch.
l The Constitution began with a declaration of the rights
l Rousseau proposed a form of government based on a of man and citizen. It was the duty of the state to protect
social contract between people and their each citizen’s natural rights.
representatives.
l In The Spirit of Law, Montesquieu tried to establish France Abolished Monarchy and became a
that in order to ensure individual freedom, the Republic
separation of power among executive, legislative body
and the judiciary is essential. The ideas of these l Although, Louis XVI had accepted the Constitution, he
philosophers spread among people through books and entered into secret negotiations with the King of
newspapers. Prussia. France continued to threaten the old order in
Europe. Neighbouring countries planned to send troops.
Economic Crisis Before this could happen, the National Assembly
declared war against Prussia and Austria.
The financial system of France reached up to the stage
l The National Assembly voted to imprison the royal
of total collapse. The population of France rose from family. Elections were held.
about 23 million in 1715 to 28 million in 1789.
l The newly elected assembly was called the Convention.
Most workers were given a fixed wage by their owner. On 21 September, 1792 it abolished the monarchy and
But, their wage did not keep pace with rising prices of declared France, a Republic.
the commodities. So, the gap between the poor and the l Louis XVI was sentenced to death by a court on the
rich widened. To overcome the economic crisis, Monarch charge of treason.
Louis XVI had to increase taxes.
Events and Process 549
The Reign of Terror l His imperialist policy resulted into the vast expenses on
army, which thrusted on the common people. This made
l The period from 1793 to 1794 is referred to as the Reign the people against the Tsar and his inefficient
of Terror. government.
l Robespierre followed a policy of severe control and l At his time, the main religions were Russian Orthodox
punishment. All those ‘enemies’ of the republic were Christianity, Catholicism, Protesantism, Islam and
arrested, imprisoned and then tried by a revolutionary Buddhism.
tribunal.
l If the court found the concerned person guilty, he was
guillotined.
The Views of Karl Marx and Friedrich Engels
Karl Marx wrote his principle in his famous book ‘Das Capital’. Marx said
l Finally, Robespierre, himself was convicted by the court
that industrial society was ‘capitalist’. He was deadly against capitalism and
and was guillotined.
private property. Capitalists owned the capital invested in factories and the
profit of capitalists was produced by workers. The conditions of workers
Socialism and Russian Revolution could not improve as long as this profit was accumulated by the capitalists.
According to Marx, the whole society should have common control over
The ideas of Socialism the means of production. All the units of production should be
nationalised.
l By the mid-nineteenth century, socialist ideas spread
Marx and Engels suggested that workers had to overthrow capitalism and
through Europe.
the rule of private property. This could be possible only through revolution
l Socialists were against private property and claimed and by establishing the ‘Dictatorship of the Proletariat’. Marx believed that
that it was the root of all social ills. to free themselves from the capitalist exploitation, workers had to
l The propertied class only concerned with their personal construct a radically socialist society, where all property was socially
gain and not with the welfare of those, who made the controlled. This would be a Communist society.
property productive. l A Communist society was the natural society of the future, according
to Karl Marx.
l Socialists suggested that instead of individual control, l
The state will be withered away in Communist society.
property should be controlled collectively. Different l
The teachings of Karl Marx made a great contribution to the rise of
socialists had different visions regarding the future of revolution in Russia.
the society.
l Some thinkers, like Robert Owen believed in the idea of
cooperatives. He sought to build a cooperative Bloody Sunday and its Impact
community called New Harmony in Indiana. In 1905, on one Sunday, the workers of Russia, led by
l Other socialists claimed that cooperatives could not be Father Gapon reached the Winter Palace of Tsar in
built on a wide scale, only through individual initiative. order to present a petition. But they were attacked by
They demanded that Government should encourage police and the Cossacks. They were fired at
cooperatives and replace capitalist enterprises.
indiscriminately resulting in the death of more than 100
l These cooperatives were to be associations of people, workers and about 300 wounded. This incident started a
who produced goods together and divided the profits
series of events that became known as the 1905
according to the work done by each members.
revolution. The incident is known as ‘Blood Sunday’ in
The Rule of Tsar Nicholas II Russian history.
l Tsar Nicholas II was an autocratic ruler. He believed in
the Divine Right of the King. So, there was no check on The Revolution of 1905
his powers and rights.
There was the growing discontentment against the
l The privileged classes had got special rights while the existing system and the government in Russia. In 1904,
general public had no say in the government.
a war broke out between Russia and Japan. When
l Tsar Nicholas II ruled Russia and its empire. Russia lost this war with Japan in 1905, the popular
l Russian empire included Finland, Latvia, Lithunia, discontentment reached up to a boiling point.
Estonia, parts of Poland, Ukraine, Belarus.
l It stretched also to the pacific and comprised of central
After the incident of ‘Bloody Sunday’, the revolution
Asian states, Georgia Armenia and Azerbaijan. started throughout Russia. In this revolution, the
peasants took the larger participation. Even some
l Tsar Nicholas II was an inefficient and weak ruler.
sections of the army and navy revolted. Initially,
People became hostile to the Tsar and his corrupt
bureaucracy. government came under a pressure and introduced a
550 Study Package for NTSE

representative organisation Duma. But when forces of l Soldiers and striking workers had gathered to form a
revolution subsided, government once again suppressed ‘Soviet’ or ‘Council’ in the same building, where Duma
the Duma. The Revolution of 1905 proved a dress met. It was known as Petrograd Soviet.
rehearsal of the Revolution of 1917. l Nicholas II abdicated the throne on 15 March, 1917.
Soviet leaders and Duma leaders formed a Provisional
The First World War and the Russian Government to run the country.
A Provisional Government was setup under the
Empire l
leadership of Prime Minister Kerensky.
l In 1914, the First World War started between two
European alliances- Germany, Austria and Turkey (the The Revolution of October, 1917
central powers) and France, Britain and Russia (later
Italy and Romania). l The Bolshevik leader, Vladimir Lenin returned from his
exile to Russia in April, 1917.
l Each country had a global empire. So, the war was
fought outside Europe as well as in Europe. l Lenin and the Bolsheviks had opposed the war, since
1914. He felt that it was time for Soviets to take over
l Initially, the war was popular in Russia and the people
power.
supported Tsar Nicholas II.
l Lenin gave ‘April Theses’, which suggested three
l Tsar Nicholas II decided to take the command of
demands-the war be brought to a close, land be
Russian army into his own hand. So, he left his court and
transferred to the peasants and the banks be
went to the battle field.
nationalised.
l The activities of Tsarina Alexandra and her advisor
l Lenin argued that the Bolshevik Party rename itself the
‘Rasputin’ contributed to the declining prestige of the Communist Party to indicate its new radical aims.
monarchy. Russia gained some success in the beginning
of the war, but Russia lost badly in Germany and l Trade unions grew in number. Soldiers’ committees
were formed in the army.
Austria between 1914 to 1916.
l The power of Provisional Government reduced and the
l It became difficult for the Russians to fight due to the
Bolshevik influence grew.
shortage of war material and food stuffs.
l Popular demonstrations staged by the Bolsheviks in
l The untrained farmers were sent to the battle field in large July, 1917 were sternly repressed.
numbers. There were over 7 million causalities by 1917.
l Lenin persuaded the Petrograd Soviet and the
l As they retreated, the Russian army destroyed crops Bolshevik Party to agree to a socialist seizure of power
and building to prevent the enemy from being able to on 16 October, 1917.
live off the land.
l Soviet appointed a Military Revolutionary Committee
l It led to over 3 million refugees in Russia. By 1916, under the leadership of Leon Trotskii to organise the
railway lines began to break down. seizure.
l Small workshops, producing essentials were shut down l The uprising began on 24 October, 1917. Prime Minister
due to shortage of labour. Kerensky had left the city to summon troops.

February Revolution in 1917 Important Changes After October


l In February, 1917, there was acute shortage of food and Revolution
other commodities in Russia. Riots at bread shops were l The Bolsheviks Party was renamed the Russian
common. Communist Party (Bolshevik).
l There was a severe famine in the country. The prices l The Bolsheviks entered into a treaty with Germany at
shot up. Brest Litovsk and ended the war with Germany.
l The workers of fifty factories went on strike. l The Bolsheviks became the only party to participate in
l In many factories, women led the way to strikes on the elections to the all Russian Congress of Soviets.
23 February, 1917. This came to be called the l The secret police punished those, who criticised the
International Women’s Day. Bolsheviks.
l Lawlessness prevailed everywhere. Workers l The Bolsheviks setup a government on the basis of the
demonstrated in the streets raising slogans about bread, principles of Karl Marx.
wages and democracy.
l All private property was confiscated.
l On 27 February, 1917 the Police headquarters were
ransacked. l Land was given to the peasants. All loans were remitted.
l The government tried to control the situation and called l All the factories were nationalised and their
out the cavalry, but the cavalry refused to fire on the management was handed over to the workers.
demonstrators. l The property of the church was confiscated.
Events and Process 551
l All persons were required to render labour. and implements were transferred to the ownership of
l The Bolsheviks created Soviet Union (USSR) from the collective firms.
Russian empire in December, 1922. l Peasants worked on the land and the profit of the
collective firm was shared among them.
Bolsheviks l But, some enraged peasants resisted the authorities and
destroyed their livestock. Between 1929 to 1931, the
Bolsheviks were the majority group of the workers of number of cattle fell by one-third.
Russia. Under the leadership of Vladimir Lenin, they l Those who opposed collectivisation were severely
believed in the revolutionary methods for bringing punished. Many were deported and exiled.
about changes in the society and the state. l Stalin allowed some independent cultivation, but
After October Revolution, Russia became the one-party treated these cultivators unsympathetically.
state i.e, the Bolshevik Party, which was renamed as l Production did not increase immediately, inspite of
the Russian Communist Party (Bolshevik). The accepting the policy of collectivisation.
Bolsheviks were able to bring about the successful l The bad harvests of 1930-1933 led to one of the most
revolution in Russia in 1917 and they changed the devastating famine in Soviet Russia when over 4 million
people died.
whole structure of Russia and the Russian society.
Influence of Russian Revolution in India
Mensheviks l By the mid-1920s, the Communist Party was formed in
India.
Mensheviks were the minority group of the workers of
Russia. They believed in parliamentary methods and l Important Indian politicians and intellectuals took an
interest in the Soviet experiment and visited Russia.
participation in elections. They thought that the party
should be open to all. They favoured parties as were l Many received education in the USSR’s Communist
University.
existing in France and Germany, which took part in
elections to the legislature of their countries. l Jawaharlal Nehru, Rabindranath Tagore visited Russia
and wrote about Soviet Socialism.
Stalin and Collectivisation l There was much that was written in Hindi, Bengali,
Marathi, Malayalam, Tamil and Telegu about
Collectivism is a principle of living and working communist ideology.
together as a group, diametrically opposed to
individualism. In Russia, this idea was accepted in the Nazism and the Rise of Hitler
period of the early planned economy.
l Russia were facing an acute problem of grain supplies Rise of Nazism
during 1927-28. l The collapse of the Weimar Republic and the consequent
l Peasants refused to sell their grain to government rise of Nazis was a complex historical phenomena into
buyers at the fixed rates. which, some separate and many inter-related stand
were interwoven.
l Stalin became the leader of the Soviet Communist
Party, after the death of Lenin in January, 1924. l Nazism was not one or two isolated acts. It was a system,
a structure of ideas about the world and politics.
l Stalin believed that rich peasants and traders in the
countryside were holding stocks in the hope of higher l Some of the main factors behind the rise of Nazism were
prices. It created acute problem of grain supplies. viz. the shock of the great depression, the fear from
Bolsheviks, the dynamism of Hitler, the failure of
l Stalin introduced firm emergency measure. political parties to tackle Germany’s problem, the
Stalin introduced the ‘collectivisation’ of Soviet agriculture, which humiliating Peace Treaty of Versailles etc.
brought the great majority of peasants into collective and state l Germany was forced to sign the Peace Treaty of
firm. Versailles after its defeat in the First World War.
Germany lost its overseas colonies, a tenth of its
l This policy of Stalin met with fierce resistance in the
population, 13% of its territories, 75% of its iron and 26%
countryside and is carried out with ruthless
of its coal to France, Poland, Denmark and Lithuania.
determination and at terrible human and material cost.
l The Allied Powers demilitarised Germany to weaken its
l In 1928, party members toured the grain producing
power.
areas, supervising enforced grain collections and raiding
‘Kulaks’ the name for well to do peasants. l The War Guilt Clause held Germany responsible for the
war and damages that the Allied Powers suffered. Thus,
l From 1929, the Communist Party forced all peasants to
Germany was forced to pay compensation amounting to
cultivate in collective firms (Kolkhoz). The bulk of lands
6 million.
552 Study Package for NTSE

The Great Economic Depression l In 1932, Reichstag election, Nazi Party got 37% votes
and became the largest party.
l After the First World War, several countries of Europe
were hit by economic depressions. Many countries had l Hitler was a good orator. His passionate lectures about
already suffered a lot due to the war. motherland and promises moved people.
l Demand of goods fell sharply after the war. Thus l Hitler promised to build a strong nation, undo the
farmers, traders, industrialists were hit due to steep fall injustice of the Versailles Treaty, restore the dignity of
in demand. It rendered workers jobless. Even America the German people, make opportunity for the
could not escape from this crisis. German investments unemployed people.
and industrial recovery were totally dependent on l Nazi propaganda projected Hitler as a saviour of
short-term loans, largely from the USA. German people.
l The wall street exchange (stock exchange of USA) l On 30 January, 1933, Hitler became Chancellor, the
crashed in 1929 and the support from it, was withdrawn. highest authority in the Cabinet of ministers.
Fearing a fall in prices, people sold 13 million shares in l The Fire Decree of 28, February 28, 1933 indefinitely
one day, which triggered off the great economic suspended freedom of speech, press and assembly that
depression. had been guaranteed by the Weimar Constitution.
l Over the next three years i.e., between 1929 to 1932, the l On 3 March, 1933, the famous Enabling Act was passed.
national income of USA fell by half. Agriculture was This act established the dictatorship of Hitler.
affected factories shut down, exports fell, speculators
l Except Nazi Party and its affiliates, all political parties
withdrew their money from the market. This recession
and trade unions were banned.
in the US economy were felt worldwise.
l The state established complete control over the
The Condition of Germany in the Year economy, media, army and judiciary.

1929-1932 Nature of Nazi Rule in Germany


The Economic crisis created deep anxieties and fears in Hitler’s regime is considered as the most barbaric
German people. The middle classes, especially salaried regime in world history. Hitler believed in the racial
employees and pensioners suffered a lot, as the currency purity and supremacy of the Aryan race of the Germans.
lost its value. Small businessman, self employed and He wanted only a German society of ‘Pure and Healthy
retailers suffered as their business got ruined. Nordic Aryans’.
They were filled with the fear of proletarianisation, an They were considered as ‘desirable’, other communities
anxiety to become impoverished to the level of working were classified as ‘undesirable’. Jews, Gypsies and
classes. Only the condition of organised workers were Blacks living in Germany were considered as racial
slightly better, but unemployment began to lessen their inferior and were widely persecuted. A large number of
bargaining power. these people were killed in gas chambers by the use of
The farmers were affected by the sharp fall in agricultural poisonous gas.
prices. The economic crisis created large scale of Hitler aimed at increasing the German empire and
unemployment. There were 4400000 out of work at the acquiring all the colonies. Germany invaded Poland in
end of 1931, 5 million in September, 1932 and 6 million in September, 1939 and it led a war with France and
the winter of 1932-33. In 1932, the Nazis polled England. Hitler attacked Soviet Russia in June, 1941.
13.7 million votes and obtained 230 seats, which was a But the Soviet Red Army inflicted a crushing and
clear indication of their popularity. From then on Hitler’s humiliating defeat on Germany.
final assumption of power as Chancellor on 30 January, A large part of the urban population of the occupied
1933 was the logical extension in the course of events. Europe were killed by the Nazis. Concentration camps
were setup by the Nazis in Poland, Austria, Holland,
The Rise of Hitler France, Czechoslovakia etc., for mass killing.
l After the First World War, Germany was forced to sign a
humiliating Peace Treaty of Versailles. The German Role of Youth in Nazi Germany
defeat horrified Hitler and the Versailles Treaty made l Hitler believed that a strong Nazi society could be
him furious. established only by teaching children Nazi ideology.
l In 1919, Hitler joined the German Worker’s Party and l All schools in Germany were ‘cleansed’ and ‘purified’,
renamed it the Nationalist Socialist German Worker’s teachers who were Jews, ‘politically unreliable’ were
Party. This party came to be known as the Nazi Party. dismissed. Children were also segregated. ‘Undesirable
l During the Great Depression, Nazism became a mass children’ - Jews, the physically handicapped, Gypsies
movement. were thrown out of the schools.
Events and Process 553
l School textbooks were rewritten. Racial science was l While boys were taught to be aggressive, masculine and
introduced to justify Nazi ideas of race. hard-working and girls were told to become good
l Children were taught to be loyal, submissive, hate Jews mothers and bring up pure-blooded Aryan children.
and worship Hitler. l Women had to maintain the purity of the race, look after
l Even sports was introduced to nurture a spirit of home and teach their children Nazi ideology.
violence and aggression among children. Hitler thought l Hitler said that “In my state the mother is the most
that boxing could make children iron hearted, strong important citizen”.
and masculine. l Those mothers, who produced racially desirable
l Ten years old children had to enter Jungvolk. At 14, all children were awarded, given favoured treatment in
boys had to join the Nazi Youth organisation, ‘Hitler hospitals, concessions in shops, theatres and even in
Youth’. After a period of rigorous ideological and railways.
physical training, they joined the labour service, at the l Honours were given to encourage women to produce
age of 18. Then, they had to serve in the armed forces many children. A bronze cross for four children, a silver
and enter one of the Nazi organisation. for six and a gold for eight or more were given to
l The Youth League of the Nazi was renamed ‘Hitler women.
Youth’. All other youth organisations were banned.

The Foreign Policy of Hitler The Nationalist Movement in


l The foreign policy of Hitler can be divided in three Indo-China
stages.
l In the first stage (1933-36), the objective of the foreign Indo-China Region and French Domination
policy of Hitler was to break or contravene the over it
provisions of the Treaty of Versailles. He also tried to
Indo-China comprises of Vietnam, Laos and Cambodia.
convince the allied powers that he was not having any
People lived in these areas under the shadow of Chinese
bad intentions. In 1934, he concluded a no aggression
pact with Poland and in 1935, he became ready to limit empire. Like the English and the Dutch, the French
German Navy in relation to Britain. wanted to spread their colony in Asia.
l In the second stage (1936-38), the first target of After defeating China in 1860, France occupied
Germany was Austria. In 1934, a Nazi Party was formed Vietnam and Cambodia. France again defeated China
in Austria and in 1938 with the support from Nazi Party in Franco-Chinese War (1883-85) and occupied Tonkin
of Austria. and Annam. In 1887, France formed the Union of
l In 1939, the foreign policy of Hitler reached upto its China comprising all these colonies. Later France
third stage. In September, 1939, Germany invaded occupied Thailand and Laos and established a strong
Poland and this started a war with France and England. large empire in Asia.
In September, 1940, a Tripartite Pact was signed
between Germany, Italy and Japan. Importance of Colonies
l Hitler attacked Soviet Union in June, 1941. It was a
Like other colonial powers, the French thought that
historic blunder of Hitler. The Soviet Red Army defeated
colonies were necessary because
Germany at Stalingrad. In the beginning, US remained
l Colonies were considered essential to supply natural
neutral in the Second World War.
resources and other essential commodities.
l When Japan extended its support to Hitler and bombed
l It was duty of the advanced European countries to
the US base at Pearl Harbour, the US entered the
bring the benefits of civilisation to backward people.
Second World War.
l By increasing the agricultural production, colonial
l The Second World War ended in May, 1945 with Hitler’s
power can export the products in the international
defeat and the US dropping of the atom bomb on
market. Vietnam exported two-thirds of its rice
Hiroshima in Japan.
production and by 1931, it had become the third largest
exporter of rice in the world.
Role of Women in Nazi Society
l In Nazi Germany, young people, even children were Steps Taken by France to Exploit the
repeatedly told that women were radically different Natural Resources of Vietnam
from men. According to the Nazi ideology, the fight for
equal rights for men and women that had become a part Like all other colonial powers, France exploited the
of democratic struggles was quite wrong and it would natural resources of Vietnam in numerous ways.
destroy the society.
554 Study Package for NTSE

l All available raw materials of Vietnam like rubber, rice The Incident of Saigon Native Girls School
and coal were exported to France at a very low price.
An incident which happened in Saigon Native Girls
l Infrastructure, like road, rail and waterways were
developed for trade, move military garrisons and control
School in 1926, created much agitation among the
the entire region. Vietnamese. A Vietnamese student sitting in one of the
front seats was asked to vacate it for her fellow French
l Rubber plantation in Vietnam was mostly owned by the
French elite. They exploited the local people in all
student. But as the girl refused to do so, the principal
possible ways. expelled her. When other angry students protested,
they too were expelled.
Paul Bernard’s View Regarding the This incident led to a further spread of agitation and
Economy of Vietnam protests among students. See the situation getting out
of control, the government ordered the school authority
An eminent writer, Paul Bernard gave his own view to take the students back. The principal reluctantly
regarding the economy of Vietnam. agreed, but warned the students in a very rude manner.
l The economy of Vietnam should be developed because This incident made a mark on the nationalist feelings of
the colonial powers like France wanted to make profit the Vietnamese.
from its colonies.
l If the economy of the colonies were developed, then the The Ideas behind the Tonkin Free Schools
standard of living of the people would develop, as a l The French colonisation was not only based on economic
result demand of commodities will be increased which exploitation, but it was also driven by the idea ‘to civilise
means more profit for the colonial power. the native people’.
l Several barriers to economic growth like high l Education is the most important means to civilise the
population levels, low agricultural productivity and native.
extensive indebtedness among the peasants should be l The French authority established many French school
removed. in Vietnam.
l Land reforms and industrialisation were essential to l Of them, one is Tonkin Free School, which was
boost up the economy. started in 1907 to provide Western education.
l In this school, Science, Hygiene and French language
The Dilemma of Colonial Education in were taught.
Vietnam
l France believed that they were bringing modern Plague and the ‘Rat Hunt Scheme’
civilisation to the Vietnamese. Education was the most l To create a modern Vietnam, the French authority
important way to civilise the native. decided to rebuild a part of Hanoi with modern
l Education was a requisite for employment in the architecture. But the beautiful part of Hanoi was struck
colonial civil service and for other white-collar, high by bubonic plague.
status jobs. l The French part of Hanoi had wide avenues and well
l While introducing the modern education system in laid out sewer system.
Vietnam, the French authorities were themselves in a l The other ‘native’ part of the city remained filthy, as a
dilemma. result the refuse from the old city overflowed the whole
area during rains or floods.
l The colonial power needed an educated local labour
force. But, education might create problems. Educated l The large sewers of the modern part of the city became
Vietnam might raise questions about the French the breeding grounds for the rats.
authority. l There was a wide spread plague which threatened the
l French citizens living in Vietnam because afraid that very lives of the French.
they might loose their jobs to the educated natives. l In order to check this menance, the French authority
l To resolve this dilemma, the French authority started the ‘Rat Hunt Scheme’ in 1902.
introduced a deliberate policy of failing students, l The French hired Vietnamese workers and paid them
particularly in the final year of their education, so that for each rat they caught.
they could not get a better-paid jobs. l As many as 20000 rats were caught in a single day (30
l In 1925, in a population of 17 million, there were less May, 1902). Rat hunting became a profitable business
than 400 who passed the examination. for the natives.
Events and Process 555
Religion of Vietnam The Difference between the Ideas of Phan
l The earliest established religions in Vietnam are Boi Chau and Phan Chu Trinh
Mahayana Buddhism, Confucianism and Taoism. Other l Phan Boi Chau and Phan Chu Trinh were both
minorities are Roman Catholicism, Cao Dai, Hoa Hao, nationalist leaders, but they differed in their approach
Hinduism, Islamism, Protestanism and Theravada to Vietnamese nationalism.
Buddhism etc.
l Phan Boi Chau formed the ‘Revolutionary society’ with
l The majority of Vietnamese people claimed themselves Prince Cuong De as its head.
as non-religious, although they visit religious temples
l But, Phan Chu Trinh was totally against monarchy and
several times every year.
preferred to establish a democratic republic.
l According to a 1999 census, most Vietnamese list
l Phan Chu Trinh was influenced by the democratic ideals
themselves as having no religious affiliation.
of the West, thus he did not want a wholesale rejection of
l A 2002 Pew Research Centre Report claimed that 24% of Western civilisation.
the population of Vietnam believed religion as ‘very
important’.
Go East Movement
l One of the most universal spiritual practices common to
Vietnamese is ancestor veneration, a practice shared l ‘Go East Movement’ became popular in the first decade
with Chinese and most Asian culture. of the 20th century.

l All Vietnamese regardless of formal religious affiliation l Some Vietnamese students went to Japan and looked for
have an altar in their home or business, where prayers Japanese help and appealed to the Japanese as fellow
were offered to their ancestors. These practices are done Asians.
frequently during important traditional or religious l Japan had improved itself and resisted colonisation by
celebrations. the West.
l Beliefs in ghosts and spirits is common. l Vietnamese students established a branch of the
Restoration Society in Tokyo.
Hoa Hao Movement l Japan has proved its military capability by defeating
Russia in 1907.
l Hoa Hao is a religious tradition based on Buddhism.
l After 1908, many Vietnamese including Phan Boi Chau
l It is founded in 1939 by Huynh Phu So.
were deported and forced to seek exile in China and
l It spread in the fertile Mekong Delta area. Thailand.
l It claimed approximately 2 million followers throughout l Development in China also inspired Vietnamese
Vietnam. nationalists.
l Aid to poor was favoured over Pagoda building as l In 1911, under the leadership of Sun Yat-Sen, a
expensive rituals. Republic was setup in China by overthrowing the
l Religious and social ceremonies must be simple and monarchy.
modest and are not to include offerings. l Inspired by these development Vietnamese students
l Elaborate wedding or funeral customs were viewed as established association for the restoration of
waste of money, which would be better spent helping the Vietnam.
needy.
l Adherents are expected to attend communal services on The Communist Movement in Vietnam
first and fifth of each lunar month and on other Under the Leadership of Ho Chi Minh
Buddhist holidays.
l Inspired by the European communist parties Ho Chi
l Huynh Phu So opposed the sale of child brides, gambling Minh established Vietnamese Communist Party (1930).
and use of alcohol and opium.
l Very little is known about the early life of Ho Chi Minh.
l French authority tried to suppress the movement
inspired by Huynh Phu So. l He studied in French schools which produced great
leaders like Ngo Dinh Diem, Vo Nguyen Giap and Pham
l French authority declared him mad and put him in a Van Dong.
mental asylum.
l He had chance to meet Russian leaders including Lenin.
l Doctor who had to prove him insane became his follower.
l After spending about 30 yr abroad in America, Europe,
l In 1941, the French doctors declared he was sane. China and Thailand, Ho Chi Minh returned to Vietnam
l The French authority exiled him to Laos and sent many in May 1941.
of his followers to concentration camp.
556 Study Package for NTSE

l When Japan occupied Vietnam in 1941, he resumed l This trail was an immense network of footpaths and
contact with Indochinese Communist Party leaders roads and was used to transport men and material from
and helped to found a new communist dominated the North to South Vietnam.
independent movement popularly known as l The trail had support bases and hospitals.
‘Vietminh’ that fought the Japanese. In 1945, Japan
l The Vietnamese porters, which were mostly women,
surrendered.
deserve all praise for their great sacrifice in carrying
l Ho Chi Minh is the key figure in the foundation of the 25 kg weight on their backs or about 70 kg on their
Democratic Republic of Vietnam (1945). bicycles.
l Ho Chi Minh became the President. l Most of the trail was outside Vietnam in neighbouring
l The French were unwilling to grant independence to countries like Laos and Cambodia with branch lines
their colonial subjects and in 1946 war broke out. extending into South Vietnam.
l After eight years battle Vietminh guerrillas defeated l US regularly bombed this trail trying to disrupt
French troops in the Battle of Dien Bien Phu in 1954. supplies.
l He officially stepped down from power in 1955 due to ill l If any line to trail was broken, it was repaired
health but remained the inspirational figure for immediately and the supply line maintained as ever.
Vietnamese for a united communist Vietnam.
l Ho Chi Minh died on 3 September, 1969. The Role of Women in the Anti-Imperialist
Struggle in Vietnam
The Entry of the US into Vietnam War l The Vietnamese women showed great heroism and
l Vietnam War was occured in Vietnam, Laos and patriotism in nationalist movement.
Cambodia from 1 November, 1955 to the fall of Saigon on l They worked as porters and carried 25 kg of food or war
30 April, 1975. material on their back and 70 kg on their bicycles.
l This war was fought between North Vietnam, supported l Women whether young or old began to work and fight for
by its communist allies and government of South the country.
Vietnam, supported by United States, South Korea,
l Many women joined the movement, they nursed the
Australia, New Zealand, Thailand etc.
wounded, constructed underground room and tunnels to
l American military advisor arrived in what was then save their army.
French Indonesia beginning in 1950.
l Between 1965 and 1975, of the 17000 youth who worked
l US combat units were deployed beginning in 1965. on the trail, 70 to 80% were women.
l Operations spanned international borders with Laos, l One military historian suggested that there were
Cambodia heavily bombed. 1.5 million in the regular army, the militia, the local
l American involvement in the war peaked in 1968. forces and professional teams.
Inspite of its advanced technology and good medical
supplies US casualities were high. About 47244 died in
battle and 303704 were wounded.
Nationalism in India
l The phase of struggle with US was brutal as it widely Formation of Indian National
used chemical weapons like Napalam, Agent Orange
and phosphorous bombs.
Congress (INC)
l US military involvement ended on 15 August, 1973 as a l Formed in 1885 by AO Hume, an Englishman and a
retired civil servant.
result of the ‘Case-church Amendment’ passed by the
US Congress. l Hume’s main purpose was probably to provide a ‘safety
valve’ to the growing discontent among the educated
l The capture of Saigon by the Vietnam People’s Army in
Indians.
April 1975, marked the end of the war.
l Other important leaders during foundation were
l The North and South Vietnam were reunified in 1976. Dadabhai Naoroji, Badruddin Tayabji, Anand Mohan
l US regarded the spread of communism a great danger to Bose, RC Dutt, Ferozshah Mehta, Gopal Krishna
the other countries. Gokhale, G Subramaniyam Aiyyar, Dinshaw Vacha, Bal
Gangadhar Tilak, Mahadev Govind Ranade, Madan
The Ho Chi Minh Trail Mohan Malaviya.
l The Vietnamese had to carry out a long struggle for l First session of INC held in Bombay under WC
independence - first against France, then against Japan Bonnerjea in 1885 (72 delegates attended it).
and lastly against USA. In this long way of struggle, l In the first two decades (1885-1905), quite moderate in
they had to follow long routes and paths to avoid enemy its approach and confided in British justice and
attack. generosity.
Events and Process 557
l But the repressive measures of the British gave rise to Demand for Swaraj (1906)
extremists within Congress like Bipin Chandra Pal, Bal
Gangadhar Tilak and Lala Lajpat Rai (Lal, Bal, Pal), In December, 1906 at Calcutta, the INC under
along with Aurobindo Ghosh. Dadabhai Naoroji adopted ‘Swaraj’ (Self Government)
as the goal of Indian people.
l Badruddin Tayabji was the first Muslim President of
INC in 1887 in Madras Session.
Surat Split (Session) of INC (1907)
l George Yule was the first English President of 1888 INC
Session held at Allahabad. l The INC split into two groups; the extremists and the
moderates, at the Surat Session in 1907. Extremists
l Annie Beasant was the first woman President of INC were led by Bal, Pal, Lal while the moderates by GK
preceded Calcutta Session in 1917. Gokhale.
l Sarojini Naidu was the first Indian woman President of l Controversy rose over the elected President, Ras Bihari
INC preceded Kanpur Session of 1925. Ghosh, as extremists didn’t accept him. Extremists
wanted Lala Lajpat Rai to be chosen.
Partition of Bengal l The government after this launched a massive attack on
l By Lord Curzon on 16 October, 1905, through a royal extremists by suppressing their newspapers and
proclamation, reducing the old province of Bengal in arresting their leaders.
size by creating East Bengal and Assam out of rest of
Bengal. Minto-Morley Reforms (1909)
l The government said that it was done to stimulate l It enlarge the legislative councils and at the same time
growth in Eastern region. make him more representative and effective.
l Actually, the objective was to set up a communal gulf l Besides other constitutional measures, it envisaged a
between Hindus and Muslims. separate electorate for Muslims.
l A mighty upsurge swept the country against the l Aimed at dividing the nationalist ranks and at rallying
partition. National movement found real expression in the moderates and the Muslims to the government’s side.
the movement against the partition of Bengal in 1905.
Ghadar Party (1913)
Swadeshi Movement (1905) l Formed by Lala Hardayal, Taraknath Das and Sohan
l It had its origin in the anti-partition movement of Singh Bhakna.
Bengal. The leaders of Bengal felt that mere l The name was taken from a weekly paper, Ghadar,
demonstrations, public meetings and resolutions were which had been started on November 1, 1913 to
not enough and something more concrete was needed commemorate the 1857 revolt.
and the answer felt was Swadeshi and Boycott. l Headquarters was at San Francisco.
l An important aspect of the Swadeshi Movement was
emphasis placed on self-reliance.
Home Rule Movement (1916)
l Lal, Bal, Pal and Aurobindo Ghosh played an important l After Tilak’s return, having served sentence of six years
role. in Mondalay (in Burma), he tried securing the
readmission of himself and other extremists into the
l INC took the Swadeshi call first at the Banaras Session, INC. With the need being felt for popular pressure to
1905 presided over by GK Gokhale. attain concessions, disillusionment with Minto-Morley
l A resolution to boycott British goods was adopted on reforms and war-time miseries, Tilak and Annie
7 August, 1905 at a meeting of INC at Calcutta. Beasant readied to assume leadership.
l Bonfires of foreign goods were conducted at various l Started by BG Tilak (April, 1916) at Poona and Annie
places. Beasant and S Subramania Iyer at Adyar, near Madras
(September, 1916).
Formation of Muslim League (1906) l Tilak’s league was to work in Maharashtra, Karnataka,
Central Province and Berar and Annie Beasant’s in the
l Setup in 1906 under the leadership of Aga Khan, Nawab
rest of India.
Salimullah of Dhaka and Nawab Mohsin-ul-Mulk.
l Objective Self Government for India in the British
l It was a loyalist, communal and conservative political
Empire.
organisation which supported the partition of Bengal,
opposed the Swadeshi Movement, demanded special l Tilak linked up the question of Swaraj with the demand
for the formation of linguistic states and education in
safeguards of its community and a separate electorate
vernacular language. He gave the slogan, ‘‘Swaraj is my
for Muslims.
birth right and I will have it.’’
558 Study Package for NTSE

Lucknow Pact (1916) put pressure on mill owners and they agreed to increase
l It was happened following a war between Britain and the wages.
Turkey leading to anti-British feelings among Muslims.
l Both INC and Muslim League concluded this (Congress
Government of India Act, 1919 or
accepted the separate electorate and both jointly Montague-Chelmsford Reforms, 1919
demanded for a representative government and Dyarchy system introduced in the provinces.
dominion status for the country).
Subjects of administration were divided into two
August Declaration (1917) categories
(i) Central (ii) Provincial.
l After the Lucknow Pact, a British policy was announced
l The Provincial Legislature was to consist of one house
which aimed at ‘increasing association of Indians in
only (Legislative Council).
every branch of the administration for progressive
realisation of responsible government in India as an l The number of Indians in the Governor-General’s
integral part of the British empire’. This came to be Executive Council was raised to three in a Council of
called the August Declaration. eight. The Indian members were entrusted with
departments such as law, education, labour, health and
l Because of Hindu-Muslim unity exhibited in Lucknow
industries.
Pact.
l The centre was now to have a Bicameral Legislature
l The Montague-Chelmsford reforms or the Act of 1919
for the first time. It actually happened after 1935 Act.
was based on this declaration.
l Communal representation extended to Sikhs,
Emergence of Gandhiji and the Idea of Christians, Anglo-Indians etc.
Satyagraha Rowlatt Act (18 March,1919)
Gandhiji arrived in India in 1915 at the age of 46 from l It gave unbridled powers to the government to arrest
South Africa. Having led by Indians in that country in and imprison suspects without trial for two years
non-voilent marches against racist restrictions, he was maximum. This law enabled the government to suspend
already a respected leader known internationally. He the right of Habeas Corpus, which had been the
was popularised by his method of fighting called foundation of civil liberties in Britain.
Satyagraha based on truth and non-voilence. l Caused a wave of anger in all sections. As a protest,
Gandhiji formed a Satyagraha society to evoke mass
Champaran Satyagraha support. 6 April, 1919 was observed as ‘Satyagraha Day’
l It was the first movement led by Gandhiji in India in when people all over the country observed fast and
Champaran district of Bihar. hartal. In the protest demonstration, Dr Satyapal and
l Here, indigo planters were oppressed to grow indigo on Dr Kitchlew were arrested. It was the first country-wide
at least 3/20 of their land part and sell it at price fixed by agitation by Gandhiji and marked the foundation of the
European owners. This movement was fixed success of Non-cooperation Movement.
Gandhiji in India. l During March and April 1919, the country witnessed a
remarkable political awakening in India. There were
Kheda Satyagraha hartals, processions and demonstrations everywhere.
In 1917, crops failed in Kheda district of Gujarat but the
government refreshed to remit land revenue and Jallianwala Bagh Massacre (13 April, 1919)
insisted on its full collection. Gandhiji supported the l People were agitated over the arrest of Dr Kitchlew and
peasants and advised them to withold payments of Dr Satyapal on 10 April, 1919. They assembled in
revenue till their demand of its remission was met. It Jallianwala Bagh in agitation.
was a success, government issued instructions that l General O’ Dyer fired at people who assembled in the
revenue should be recovered only from those peasants Jallianwala Bagh, Amritsar. As a result hundreds of men,
who could afford to pay. Sardar Vallabhbhai Patel women and children were killed and thousands injured.
accompanied Gandhiji in this movement.
l Rabindranath Tagore returned his ‘Knighthood’ in
protest. Sir Shankaran Nair resigned from Viceroy’s
Ahmedabad Mill Strike
Executive Council after this.
In 1918, Mahatma Gandhi intervened in a dispute l Hunter Commission was appointed to enquire into it.
between workers and mill owners of Ahmedabad. He l On 13 March, 1940, Sardar Udham Singh killed O’ Dyer
advised the workers to go on strike and to demand a 35% when the latter was addressing a meeting in Caxton
increase in wages. He undertook a fast unto death which Hall, London.
Events and Process 559
Khilafat Movement (1920) Swaraj Party (1923)
l Muslims were agitated by the treatment done with l Motilal Nehru, CR Das and NC Kelkar (called
Turkey by the British in the treaty that followed the Pro-changers) demanded that the nationalists should
First World War. end the boycott of the Legislative Councils, enter them
l Two brothers, Mohammad Ali and Shaukat Ali started and expose them. But the no-changers like Rajendra
this movement, along with Maulana Azad, Hakim Ajmal Prasad and Rajgopalachari adhered to the Gandhian
Khan and Hasrat Mohani. It was jointly led by the programme of boycott of legislatures. The pro-changers
Khilafat leaders and the Congress. formed the Swaraj Party on 1 January, 1923, contested
the elections and embarrassed the Government by
l Gandhiji viewed the Khilafat agitation as a golden
opposing its measures.
opportunity for bringing the Hindus and Muslims
together. l In the 1923 elections, the Swarajists got a majority in
Bengal and Central Province.
l On August 31, 1920, the Khilafat Committee launched a
Non-cooperation Movement. l The two sections were reunited in 1930 after the Lahore
Session.
l Gandhiji now pressed the Congress to adopt a similar
plan of action. Although, it was initially opposed by CR
Das, but was later passed unopposed.
Simon Commission (1927)
l Constituted under John Simon, to review the political
Non-cooperation Movement (1920) situation in India and to introduce further reforms and
extension of parliamentary democracy.
l It was the first mass based political movement under
Gandhiji. In his famous book ‘Hind Swaraj’ (1909) l Indian leaders opposed the Commission, as there were
Mahatma Gandhi declared that British rule was no Indians in it.
established in India with the cooperation of Indians and l Congress turned the boycott into the movement.
had survived only because of this cooperation. If Indians l Simon and his colleagues landed in Bombay on
refused to cooperate, British rule in India would collapse 3 February, 1928 and were greeted with hartals and
within one year and Swaraj would come. black-flag demonstrations.
l Congress passed the resolution in its Calcutta Session in l The government used brutal repression and police
September, 1920. attacks to break the popular opposition. At Lahore, Lala
l The movement envisaged Lajpat Rai was severely beaten in a lathi-charge. He
(i) Surrender of titles and honorary offices. succumbed to his injuries on 17 November, 1928.
(ii) Resignation from nominated offices and posts in the
local bodies. Revolutionary Activities
(iii) Refusal to attend Government darbars and boycott of l The first political murder of a European was committed
British courts by the lawyers. in 1897 at Poona by the Chapekar brothers, Damodar and
(iv) Refusal of general public to offer themselves for Balkishan. Their target was Mr Rand, President of the
military and other Government jobs and boycott of Plague Commission, but Lt Ayerst was accidentally shot.
foreign goods etc.
l In 1907, Madam Bhikaiji Cama, a Parsi revolutionary
l CR Das and Motilal Nehru gave up their legal practice unfurled the flag of India at Stuttgart Congress (of
and Subhash Chandra Bose resigned from the Civil Second International).
Service.
l In 1908, Khudiram Bose and Prafulla Chaki threw a
l The Prince of Wales visited India during this period. But bomb on the carriage of Kingsford, the unpopular judge
he was greeted with empty streets and downed shutters of Muzaffarpur. Khudiram, Kanhaiyalal Dutt and
when he came (17 November, 1921). Satyendranath Bose were hanged (Alipur Case).
l In 1909, ML Dhingra shot dead Col William Curzon
Chauri-Chaura Incident (1922) Whyllie, the political advisor of ‘India Office’ in London.
l The Congress Session at Allahabad in December 1921, l In 1912, Rasbihari Bose and Sachindra Nath Sanyal
decided to launch a Civil Disobedience Movement. threw a bomb at Lord Hardinge at Delhi. (Delhi
Gandhiji was appointed its leader. Conspiracy Case).
l But before it could be launched, a peaceful l In October, 1924, a meeting of revolutionaries from all
demonstration in a bazar at Chauri-Chaura (Near parts of India was called at Kanpur.
Gorakhpur) turned into a violent clash with people and
l The meeting was attended by old revolutionary leaders
as a result 22 policemen died on 5 February, 1922.
like Sachindra Nath Sanyal, Jogesh Chandra
l This compelled Gandhiji to withdraw the Chatterjee and Ram Prasad Bismil and some young
Non-cooperation Movement on 12 February, 1922. revolutionaries like Bhagat Singh, Shiv Verma,
560 Study Package for NTSE

Sukhdev, Bhagwati Charan Vohra and Civil Disobedience


Chandrashekhar Azad. l Thousands people in different parts of our country broke
l They setup Hindustan Socialist Republic like salt, the manufactured salt and demonstrated in
Association/Army (HSRA). Their three objectives were front of government salt factories.
(i) To raise the consciousness of people against the l As the movement spread, foreign cloth was boycotted
futility of Gandhian movement of non-violence. and liquor shops were picketed.
(ii) To perform direct action and revolution to attain l Peasants refused to pay revenues, chaukidari taxes and
complete independence.
village officials resigned.
(iii) To set up a republic of the United States of India on
the federal structure. l Many places people lived in forests, violated forest laws
by collecting woods and grazing cattle in reserved areas.
l They carried out a dacoity on the Kakori bound train on
the Saharanpur-Lucknow railway line on 9 August, l One of the important features of Civil Disobedience
1925. Movement was the large scale participation of women.
l Bhagat Singh, with his colleagues shot dead Saunders Women participated in protest marches, manufactured
(Assistant SP of Lahore, who ordered lathi-charge on salt and picketed foreign cloth and liquor shops. In
Lala Lajpat Rai) on 17 December, 1928. urban areas they were from high caste families and in
rural areas they come from rich peasant households.
l Then, Bhagat Singh and Batukeshwar Dutt threw a
bomb in the Central Assembly on 8 April, 1929. Thus, l The Salt Satyagraha sparked off other forms of defiance.
Bhagat Singh, Rajguru and Sukhdev were hanged on In the North-West, Frontier provinces. Khan Abdul
23 March 23, 1931 at Lahore Jail (Lahore Conspiracy Ghaffar Khan, popularly known as the Frontier Gandhi,
Case) and their bodies cremated at Hussainiwala near organised the society of ‘Khudai Khidmadgars’
Ferozpur. (sequants of God), also known as Red Shirts.
l In 1929, only Jatin Das died in Lahore jail after 63 days
l When Abdul Ghaffar Khan was arrested in April 1930,
fast to protest against horrible conditions in jail.
angry crowds demonstrated in the streets of Peshwar
l Meerut Conspiracy Case started in 1929 and continued facing police firing.
for four years against 31 communists accused of
conspiracy against the British Sovereign. In this,
Muzaffar Ahmed received transportation for life.
The Concept of ‘Harijan’
l
All social groups were not moved by the concept of ‘Swaraj’.
l Surya Sen, a revolutionary of Bengal, formed the Indian l
One such group was the ‘untouchables’ who from 1930’s onwards
Republic Army in Bengal. In 1930, he masterminded the began to call themselves as ‘Dalit’ or oppressed.
raid on Chittagong armoury. He was hanged in 1933. l
The congress ignored the ‘Dalit’ class, for fear of offending the
l In 1931, Chandrashekhar Azad shot himself at Alfred conservative upper class Hindus.
Park in Allahabad. l
Gandhiji called untouchables Harijan or the children of God.
l
Gandhiji believed that Swaraj would not come for hundred years, if
Lahore Session (1929) untouchability was not eliminated.
l
Gandhiji organised Satyagraha for the ‘Harijans’ to secure their entry
l On 19 December, 1929, under the Presidentship of JL into temples and access to public wells, tanks, roads and schools.
Nehru, the INC, at its Lahore Session, declared Poorna l
Gandhiji himself cleaned the toilets to dignify the work of bhangi (the
Swaraj (Complete Independence) as its ultimate goal. sweepers).
l On 3 December, 1929, the newly adopted tri-colour flag l
Gandhiji urged the upper class to change their mind set regarding the
was unfurled and 26 January, 1930 was fixed as the untouchables.
First Independence Day, which was to be celebrated
l
Dalit leaders wanted a different political solutions to the problems of
the community.
every year.
l
They demanded reserved seats in educational institutions and a
separate electorate that would choose dalit members for legislative
Dandi March (1930) and Civil Disobedience councils.
Movement
First Round Table Conference (1930)
Dandi March
l It was the first conference arranged between the British
l Also called the Salt Satyagraha. Along with and Indians as equals. It was held on 12 November, 1930
78 followers, Gandhiji started his March from in London to discuss Simon Commission.
Sabarmati Ashram on 12 March, 1930 for the small
village Dandi to break the salt law. He reached the sea l Boycotted by INC Muslim League, Hindu Mahasabha,
shore on 6 April, 1930. Liberals and some others were there.

l He picked a handful of salt and inaugurated the Civil l Postponed to 2 January, 1931, in the absence of any
Disobedience Movement. major political party.
Events and Process 561
Gandhi-Irwin Pact Poona Pact (25 September, 1932)
l Moderate statesman, Sapru, Jaikar and Srinivas l After the announcement of communal award and
Shastri initiated efforts to break the ice between subsequent fast of Gandhiji, mass meeting took place
Gandhiji and the government. almost everywhere.
l The two (Government represented by Irwin) signed a l Political leaders like Madan Mohan Malaviya, Dr BR
pact on 5 March, 1931. Ambedkar and TSS Rajan became active.
l In this, the INC called off the Civil Disobedience l Eventually Poona Pact was reached and Gandhiji broke
Movement and agreed to join the Second Round Table his fast on the sixth day (25 September, 1932).
Conference. l In this, the idea of separate electorate for the depressed
l The Government on its part released the political classes was abandoned, but seats reserved to them in
prisoner and conceded the right to make salt for the provincial legislature were increased.
consumption of villages along the coast. l Thus, the Poona Pact agreed upon a joint electorate for
l The Karachi Session of 1931 endorsed the Gandhi-Irwin upper and lower castes.
Pact. The Karachi Session is also memorable for its l Harijan upliftment now became the principal concern of
resolution on Fundamental Rights and the National Gandhiji. An All India Antiuntouchability League was
Economic Programme. started in September 1932 and a weekly ‘Harijan’ in
January 1933. On 8 May, 1933, Gandhiji decided to
Second Round Table Conference (1931) begin a 21 days fast for the purification of himself and
l Gandhiji represented the INC and went to London to his associates for the Harijan cause.
meet British PM Ramsay Macdonald. l He started the Individual Civil Disobedience on
l In the conference, Gandhiji demanded immediate August 1, 1933.
establishment of a full responsible government at the
centre as well as in the provinces with complete control Third Round Table Conference (1932)
over defence, external affairs and finance. Proved fruitless as most of the national leaders were in
l However, the session was soon deadlocked on the prison. The discussions led to the passing of the
minorities issue and this time separate electorates was Government of India Act, 1935. Famous for its white
demanded not only by Muslims but also by Depressed paper issue.
Classes, Indian Christians and Anglo-Indians.
l MacDonald ended the session with an address which The Government of India Act (1935)
announced the creation of two new Muslim majority l Based on the Simon Commission report. It provided for
provinces, North-West Frontier Province and Sindh, set the establishment of All India Federation consisting
up a committee on Franchise, finance and states and of the British provinces and the Princely States. The
held out the humiliating and the dangerous prospect of a joining of Princely States was voluntary and as a result,
unilateral British Communal Award if the Indians the federation did not come into existence.
failed to agree among themselves.
l Dyarchy was introduced at the Centre (e.g.,
l On Gandhiji’s arrival in Bombay, the Congress Working
Department of Foreign Affairs and Defence were
Committee decided to resume the Civil Disobedience
reserved for the Governor-General).
Movement. In 1932, INC was declared an illegal
organisation and all its leaders were arrested. Gandhiji l The Federal Legislature (Central Legislature) was to
was sent to Yeravada jail in Poona. have two chambers (bicameral); the Council of State and
the Federal Assembly.
l The Civil Disobedience Movement was withdrawn in
1934, because after that Gandhiji decided to make l Provincial autonomy replaced dyarchy in provinces
Harijan work the central plank of his new rural i.e., the distinction between reserved and transferred
constructive programme. subjects was abolished and full responsible government
was established, subject to certain safeguards. They
The Communal Award (16 August,1932) were granted separate legal identity.

l Announced by Ramsay McDonald. It showed Divide and l The Congress rejected the 1935 Act and demanded the
Rule Policy of the British. convening of a Constituent Assembly elected on the
basis of adult franchise to frame a Constitution for an
l Envisaged representation of Muslims, Sikhs, Indian
independent India. Jawaharlal Nehru described it as ‘we
Christians, Anglo-Indians, women and even Backward
are provided with a car, all breaks and no engine’.
Classes.
l Yet, INC fought the election in 1937, when the
l Gandhiji, who was in Yeravada jail (Poona) at that time,
Constitution was introduced and formed ministries in
started a fast unto death against it.
seven out of eleven provinces. Later, Congress formed
562 Study Package for NTSE

coalition governments in two others, only Bengal and act of the underground movement was the
Punjab had non-Congress ministries. Punjab was under establishment of Congress Radio with Usha Mehta as
the Unionist Party and Bengal under the Krishak Praja its announcer.
Party-Muslim League coalition. l Parallel Governments were setup at various places. The
first one was in Ballia in Eastern UP under the
Demand for Pakistan leadership of Chittu Pande. Others were in Satara,
l In 1930, Iqbal suggested that the Frontier Province, Talcher, parts of Eastern UP and Bihar.
Baluchistan, Sindh and Kashmir be made the Muslims l The movement was however crushed.
state within the federation.
l Chaudhary Rehmat Ali gave the term Pakistan in 1933. Wavell Plan ( June-July 1945)
l Mohammad Ali Jinnah of Bombay gave it practicality. l This plan given by Lord Wavell.
l Muslim League first passed the proposal of separate l The plan was that the Viceroy’s Executive Council
Pakistan in its Lahore Session in 1940 (called Jinnah’s should be so reconstituted that its members, except the
Two-Nation Theory). It was drafted by Sikandar Governor-General and the Commander-in-Chief, were
Hayat Khan, moved by Fazlul Haq and seconded by to be Indians (from various parties). Simla
Khaliquzzaman. Conference was convened in this regard, but was
l In December, 1943, the Karachi Session of the Muslim rejected by Jinnah.
League adopted the slogan, ‘Divide and Quit’.
The Cabinet Mission Plan (1946)
August Offer (8 August, 1940) The struggle for freedom entered a decisive phase in the
l It offered year 1945-46. The new Labour Party PM, Lord Attlee,
(i) Dominion status in the unspecified future; made a declaration on 15 March, 1946, that British
(ii) A post-war body to enact the Constitution; Cabinet Mission (comprising of Lord Pethick Lawrence
(iii) To expand the Governor-General’s Executive Council as Chairman, Sir Stafford Cripps and AV Alexander)
to give full weightage to minority opinion. will visit India.
l Rejected by the INC because there was no suggestion of
l The mission held talks with the Indian National
the National Government and because the demand for Congress and Muslim League to bring about acceptance
the dominion status was already discarded in favour of of their proposals.
Poorna Swaraj. It was accepted by the Muslim League. l On May 16, 1946, the mission put forward its proposals.
l Rejected the demand for separate Pakistan and instead
The Cripps Mission (1942) a federal union consisting Princely States was
l In December 1941, Japan entered the World War II and suggested.
advanced towards Indian borders. By 7 March, 1942, l The union would deal into the following subjects;
Rangoon fell and Japan occupied the entire South-East Foreign affairs, Defence and Communication and would
Asia. have the powers to raise the finances required for the
l The British Government with a view to get cooperation above subjects.
from Indians sent Sir Stafford Cripps, leader of the House l There were to be three groups of Provinces. Group A
of Commons to settle terms with the Indian leaders. with six Hindu majority provinces (Bombay, United
l He offered a draft which contained the following Province, Bihar, Central Province, Orissa, Madras);
proposals. Dominion status to be granted after the war. Group B with three Muslim majority provinces (Sind,
NWFP, Punjab) and Group C (Assam and Bengal). After
The Revolt of 1942 and The Quit India Movement the first general elections, a province could come out of
l Also called the Vardha Proposal and Leaderless the group and after 10 years, a province could call for
Revolt. reconsideration of the group or union constitution.
l The resolution was passed on 8 August, 1942, at l Both Congress and Muslim League accepted it.
Bombay. Gandhiji gave the slogan ‘Do or die’. l The elections of the Constituent Assembly were held in
l On August 9, the Congress was banned and its July 1946. The Congress got 209 of the total 273 seats.
important leaders were arrested. Gandhiji was kept at
the Aga Khan Palace, Pune. Interim Government
l The trend of underground revolutionary activities also Based on Cabinet Mission Plan, an Interim Government
started during the phase Jaya Prakash Narayan, Ram consisting of Congress nominees was formed on
Manohor Lohia and Aruna Asaf Ali started
2 September, 1946. Jawalarlal Nehru was its
consolidating underground networks. The most daring
Vice-President and the Governor-General remained as
Events and Process 563
its President. Muslim League did not join it initially Partition and Independence
but finally Wavell succeeded in having five members l All political parties accepted the Mountbatten Plan.
of the League join the government on 26 October,
l Two Commissions were appointed by the British
1946. Government with Sir Cyril Redcliffe as Chairman of both to
see through the partition and fix the international
Jinnah’s Direct Action Resolution boundaries of the two nations to be.
l Jinnah was alarmed at the results of the elections l At the time of independence, there were 562 small and big
because the Muslim League was in danger of being Princely States in India.
totally eclipsed in the Constituent Assembly. l Sardar VallabhBhai Patel, the first Home Minister, used
l Therefore, Muslim League withdrew its acceptance iron hand in this regard. By 15 August, 1947, all the States,
of the Cabinet Mission Plan on 29 July, 1946. with a few exceptions like Kashmir, Hyderabad and
l It passed a ‘Direct Action’ resolution, which Junagarh had signed the Instrument of Accession. Goa was
condemned both the British Government and the with the Portuguese and Pondicherry with the French.
Congress (16 August, 1946).
l It resulted in heavy communal riots. The Sense of Collective Belongingness in
l Jinnah celebrated Pakistan Day on 27 March, 1947. Nationalist Movement
Nationalist movement spreads when people belonging to
Formation of Constituent Assembly different regions and communities begin to develop a sense
The Constituent Assembly met on 9 December, of collective belongingness.
1946 and Dr Rajendra Prasad was elected as its Undoubtedly this sense of collective belongingness
President. develops through the experience of united struggles. But
many cultural process also contribute to it as was seen in
Mountbatten Plan India during its freedom movement.
l The identity of a nation is more often symbolised in a figure
l On 3 June, 1947, Lord Mountbatten put forward his
or image. In the 20th century, the identity of India came to
plan which outlined the steps for the solution of
be visually associated with the image of ‘Bharat Mata’.
India’s political problems. The outlines of the Plan
were l This image of ‘Bharat Mata’ was first created by Bankim
(i) India to be divided into India and Pakistan. Chandra Chattopadhyay in 1870 when he wrote ‘Vande
(ii) Bengal and Punjab will be partitioned and a Mataram’ for our motherland. Later this song included in
referendum in NEFP and Sylhet district of Assam his Novel Anandmath and widely sung by the freedom
would be held. fighters.
(iii) There would be a separate Constitutional l The image of Bharat Mata was first painted by
Assembly for Pakistan to frame its Constitution. Rabindranath Tagore. Later the image acquired many
(iv) The Princely States would enjoy the liberty to join different forms as it was drawn by different artists.
either India or Pakistan or even remain
independent. l Indian folksongs and folktales sung by bards played an
important role for making the ideas of nationalism. These
(v) 15 August, 1947 was the date fixed for handing
over power to India and Pakistan. gave true picture of traditional culture which was corrupted
by foreigners.
l The British Government passed the Indian
Independence Act of 1947 in July 1947, which l In Bengal, Rabindranath Tagore and in Madras Natesa
contained the major provisions put forward by the Sastri collected massive collection of folktales and songs
Mountbatten Plan. which led the movement for folk revival.
Target Exercise
Elementary Level
1. Who is the author of the book Two Treatises of 12. When did the Greek struggle for independence begin?
Government? (a) 1821 (b)1850 (c) 1870 (d) 1890
(a) John Locke (b) Jean Jacques Rousseau
13. Industrialisation began in England in
(c) Arthur Young (d) George Danton
(a) the first half of the 18th century
2. The ______ century witnessed the emergence of social (b) the second half of the 18th century
groups viz., the middle class. (c) the first half of the 19th century
(a) 19th (b) 18th (d) the second half of the 17th century
(c) 20th (d) 17th 14. Germany faced the crisis known as hyperinflation in
3. Who proposed the idea of social contract between the year of
people and their representatives? (a) 1950 (b) 1923 (c) 1914 (d) 1940
(a) Jean Paul Marat (b) Jean Jacques Rousseau 15. Which of the following factors are responsible for the
(c) CL Montesquieu (d) FMA Voltaire
rise of Hitler?
4. In his book, ‘The Spirit of Laws’, ______ proposed a (a) Crisis in economy (b) Crisis in polity
division of power within the Government between the (c) Crisis in society (d) All of these
legislative, the executive and the judiciary. 16. Adolf Hitler was born in _____ in Austria.
(a) Voltaire (b) Rousseau (a) 1871 (b) 1889 (c) 1875 (d) 1885
(c) Montesquieu (d) Descartes
17. In ______, the famous Enabling Act, was passed,
5. When did Louis XVI become the king of France?
which gave Hitler all powers to sideline Parliament
(a) 1774 (b) 1789
and rule by decree.
(c) 1777 (d) 1791
(a) 1931 (b) 1933 (c) 1940 (d) 1924
6. Which groups of French society would have gained
18. Ngo Dinh Diem, Vo Nguyen Giap, Pham Van Dong
from the Constitution of 1791?
and Ho Chi Minh were the famous leaders of
(a) Middle class (b) Richer section of society
(c) Both ‘a’ and ‘b’ (d) None of these (a) Vietnam (b) China
(c) South Korea (d) North Korea
7. In 18th century, political clubs became an important
19. Who is the founder of Vietnamese Communist
rallying point for people who wished to discuss
(Vietnam Cong San Dang) party?
government policies and plan their own form of
(a) Pham Van Dong (b) Ngo Dinh Diem
action. Of them which club was most successful?
(c) Ho Chi Minh (d) None of these
(a) The Jacobins (b) The Marseillaise
(c) The Bolsheviks (d) None of these 20. Who was the first President of the Vietnam
Democratic Republic?
8. The National Anthem of France the Marseillaise was
(a) Ngo Dinh Diem (b) Henry Navarre
composed by
(c) Ho Chi Minh (d) None of these
(a) John Locke (b) Jean Paul Marat
(c) Roget de L’Isle (d) None of these 21. Subhash Chandra Bose escaped from India in
(a) 1940 (b) 1941
9. Name the socialist, who sought to build a cooperative
(c) 1942 (d) 1943
community called New Harmony in Indiana.
(a) Louis Blance (b) Robert Owen 22. Who organised Abhinav Bharat?
(c) Henry Mayhew (d) None of these (a) Damodar Chapekar
(b) Vinayak Damodar Savarkar
10. At the beginning of the 20th century, the vast (c) Rash Bihari Bose
majority of Russian were (d) Vasant Kumar
(a) workers (b) money lenders
(c) traders (d) agriculturists 23. Who among the following was hanged by the British
Government in the Kakori Conspiracy?
11. The Russian Social Democratic Workers party was (a) Bhagat Singh
founded in _______ by socialists. (b) Chandrashekhar Azad
(a) 1871 (b) 1914 (c) Ram Prasad Bismil
(c) 1898 (d) 1901 (d) Batukeshwar Dutt
Events and Process 565
24. Who among the following composed the famous 35. Who founded the Banaras Hindu University?
flagsong ‘Jhanda Uncha Rahe Hamara’? (a) Mahatma Gandhi
(a) Maithili Sharan Gupta (b) Madan Mohan Malaviya
(b) Suryakant Tripathi Nirala (c) Jawaharlal Nehru
(c) Makhan Lal Chaturvedi (d) None of the above
(d) Shyam Lal Gupta ‘Prasad’
36. The first woman President of the Indian National
25. When did JawaharLal Nehru become the President of Congress was
the Indian National Congress for the first time? (a) Kasturba Gandhi
(a) 1929 (b) 1927 (b) Annie Beasant
(c) 1921 (d) 1932 (c) Sarojini Naidu
(d) Vijayalakshmi Pandit
26. The leader of the peasants’ movement in Bihar in
1930’s was 37. The Quit India Movement started at
(a) CR Das (b) Swami Sahajanand (a) Delhi on 15 August, 1942
(c) Muzaffar Ahamad (d) Rajendra Prasad (b) Bombay on 8 August, 1942
(c) Lahore on 7 July, 1942
27. Who led the Bardoli Satyagraha of 1928? (d) Wardha on 7 August, 1942
(a) Followers of Gandhiji
(b) Congress Socialist Party 38. Mahatma Gandhi was assassinated on
(c) Leftists (a) 26 January, 1948
(d) Kisan Sabha (b) 30 January, 1948
(c) 30 January, 1949
28. The strategy of ‘Divide and Rule’ was adopted by (d) 26 January, 1950
(a) Lord Curzon (b) Lord Wellesley
(c) Lord Minto (d) Both ‘a’ and ‘c’ 39. Who initiated the greeting of ‘Jai Hind’?
(a) Mahatma Gandhi
29. Delhi became the capital of India in (b) BG Tilak
(a) 1910 (b) 1911 (c) 1916 (d) 1923 (c) JawaharLal Nehru
(d) Subhash Chandra Bose
30. Consider the following events
1. Swadeshi Movement 40. Of the following, whose ideas had influenced
2. Home Rule Movement Mahatma Gandhi’s thought?
3. Anti-Rowlatt Act Movement (a) Karl Marx (b) Hitler
4. Khilafat Movement (c) Tolstoy (d) Lenin
Their correct chronological order is 41. Which of the following statements regarding Quit
(a) 1 2 3 4 (b) 1 2 4 3 India Movement is correct?
(c) 2 1 4 3 (d) 3 1 2 4
(a) It was entirely a non-violent movement
31. Who was the Viceroy of India at the time of (b) The upper middle class did not show interest in this
Jallianwala Bagh Massacre? movement
(c) The Muslims did not participate in this movement
(a) Lord Curzon (b) Lord Hardinge
(d) The movement paved the way for British seriously
(c) Lord Chelmsford (d) Lord Irwin
considering granting freedom to India
32. The first movement launched against the British in
42. The famous song ‘‘Raghupati Raghav Raja Ram’’ was
India was
sung during Dandi March by the renowned musician
(a) Khilafat Movement
(b) Swadeshi Movement (a) Mallikarjun Mansur
(c) Non-cooperation Movement (b) Krishna Rao Shankar Pandit
(c) Onkar Nath Thakur
(d) Quit India Movement
(d) Digambar Vishnu Paluskar
33. Who established Hindustan Socialist Republican
43. The capital of India was shifted to Delhi during the
Army?
time of
(a) SC Bose
(a) Minto
(b) Ras Bihari Bose
(b) Chelmsford
(c) Chandrashekhar Azad
(c) Curzon
(d) Sardar Bhagat Singh
(d) Hardinge
34. Who was called by the British rulers as ‘the leader of
44. Before Delhi, where was the capital of India during
Indian unrest’?
the British India?
(a) Gopal Krishna Gokhale (b) Subhash Chandra Bose
(c) Bal Gangadhar Tilak (d) MK Gandhi (a) Lucknow (b) Bombay
(c) Calcutta (d) Patna
566 Study Package for NTSE

45. Whom did Mahatma Gandhi describe ‘as pure as 48. According to Nazi ideology, there was no equality
crystal, as brave as a lion and the most perfect man in between people, but only a
the political field’? (a) racial hierarchy
(a) BG Tilak (b) Motilal Nehru (b) economical hierarchy
(c) GK Gokhale (d) CR Das (c) political hierarchy
(d) None of the above
46. Which of the following revolutionary leaders
organised an attack on the armoury of Chittagong? 49. Nazis wanted only a society of pure and healthy
(a) Jatin Das ‘Nordic Aryans’. This means
(b) Chandra Shekhar Azad (a) Aryans who lived in North European countries
(c) CR Das (b) Aryans who had German or related origin
(d) Surya Sen (c) Both ‘a’ and ‘b’
(d) None of the above
47. ‘One people, one empire, and one leader’ is the famous
slogan of 50. Hitler becomes Chancellor of Germany on
(a) Mazzini (a) January, 1933
(b) Bismarck (b) March, 1930
(c) Adolf Hitler (c) January, 1935
(d) Lenin (d) February, 1938

High Skill Questions


1. Who was the leader of the Jacobin club? 9. Olympe de Gouges was one of the most important of
(a) Voltaire (b) Mantesquieu the politically active women in revolutionary
(c) Robespierre (d) Rousseau (a) France (b) Spain (c) Prussia (d) Italy
2. In which year all men of 21 years and above in France 10. Napoleon Bonaparte crowned himself ‘Emperor of
regardless of their wealth did get the right to vote? France’ in
(a) In 1791 (b) In 1789 (a) 1798 (b) 1791
(c) In 1792 (d) In 1795 (c) 1804 (d) 1811
3. On 21 September, 1792 the newly elected assembly
abolished the monarchy and declared France as a
11. In which year, Napoleon Bonaparte was defeated at
(a) republic (b) democracy Waterloo?
(c) sovereign state (d) socialist state (a) 1810 (b) 1815
(c) 1797 (d) 1812
4. What was the charge against Louis XVI by the court?
(a) Autocracy 12. What do you mean by Suffragette Movement?
(b) Treason (a) A movement to give women the right to vote
(c) Misusing state’s wealth (b) A movement to give men the right to vote
(d) None of the above (c) A movement to give both men and women the right to vote
(d) A movement to give women the right to participate in the
5. Which period in France was referred to as the reign of Government
Terror?
(a) 1793 to 1794 (b) 1789 to 1791 13. ______ were against private property and saw it as
(c) 1770 to 1774 (d) 1772 to 1795 the root of all social ills of the time.
(a) Liberals (b) Radicals
6. Who was the inventor of Guillotine? (c) Nationalists (d) Socialists
(a) Robespierre (b) Louis XVI
(c) Dr Guillotine (d) Dr Danton 14. _______ believed that to free themselves from
capitalist exploitation, workers had to construct a
7. ‘‘Terror is nothing but justice, swift, severe and
radically socialist society, where all property was
inflexible; ______ and is used to meet the most urgent
needs of the fatherland’’. Who is the speaker? socially controlled.
(a) Rousseau (b) Desmoulins (a) Marx (b) Robert Owen
(c) Robespierre (d) Montesquieu (c) Locke (d) None of these

8. In which year, the women in France did win the Right 15. What does the term ‘Jadidists’ mean?
to Vote? (a) Socialist reformers within the Russian empire
(b) Muslim reformers within the Russian empire
(a) 1790 (b) 1871
(c) Nationalists within the Russian empire
(c) 1850 (d) 1946
(d) None of the above
Events and Process 567
16. At the time of the First World War, who was the Tsar 28. Synagogue is the place of worship for people of
in Russia? (a) Nazi faith (b) Jewish faith
(a) Nicholas IV (b) Nicholas II (c) Parsi faith (d) Buddhist faith
(c) Louis VI (d) Nicholas X
29. The first Indian to denounce the ‘Economic Drain
17. _______ is a political philosophy that stressed the Theory’ in the House of Commons of British.
importance of tradition, established institutions and (a) Dadabhai Naoroji (b) Gopal Krishan Gokhale
customs, and preferred gradual development to quick (c) Mohammad Ali Jinnah (d) Shyamji Krishna Verma
change. 30. Match the following
(a) Liberalism (b) Conservatism
(c) Radicalism (d) None of these List I List II
18. In ______, representatives of the European powers, A. Lord Wellesley 1. Fort William College
Britain, Russia, Prussia and Austria, who had B. Lord Cornwallis 2. Sanskrit College for Hindus
collectively defeated Napoleon, met at Vienna to C. Sir Thomas Munro 3. Ryotwari Systems in South
draw up a settlement for Europe and signed the India
D. Lord Minto 4. Indian Press Act, 1910
Treaty of Vienna.
(a) 1817 (b) 1815 Codes
(c) 1818 (d) 1820 A B C D
(a) 1 2 3 4
19. American Constitution came into force in (b) 2 3 4 1
(a) 1790 (b) 1789 (c) 4 3 1 2
(c) 1850 (d) 1950 (d) 3 4 2 1
20. Mazzini, Garibaldi and Cavour, all made their 31. Match the following
contribution in their own way in the unification of.
(a) Germany (b) Italy List I List II
(c) Russia (d) Britain
A. Brahmo Samaj 1. Swami Vivekanand
21. ‘April Thesis’ was given by the Bolshevik leader B. Ramkrishna Mission 2. Dayanand Saraswati
(a) Vladimir Lenin (b) A Shlyapnikov C. Arya Samaj 3. Raja Ram Mohan Roy
(c) Plekhanov (d) None of these D. Satyshodhak Samaj 4. Jyotiba Phule

22. The _______ party was renamed as the Russian Codes


Communist Party. A B C D
(a) 2 3 4 1
(a) Mensheviks (b) Bolsheviks
(b) 3 2 1 4
(c) People’s (d) None of these
(c) 3 1 2 4
23. _______ was an Indian revolutionary and the founder (d) 1 2 3 4
of the Mexican Communist Party.
32. Match the following
(a) Rajani Palme Dutta (b) MN Roy
(c) PC Joshi (d) None of these Newspapers Editors
24. In Russia ‘Kulaks’ means A. Samachar Darpan 1. Ram Mohan Roy
(a) the name for well-to-do peasants B. Mirat-ul-Akhbar 2. BG Tilak
(b) the small sized peasant C. Kesari 3. Mahatma Gandhi
(c) the traders D. Young India 4. John Clark Marshman
(d) the factory workers
Codes
25. The deadly poison ‘Agent Orange’ was sprayed over A B C D
Vietnam to destroy forests and fields by ______ forces. (a) 2 1 4 3
(a) Japanese (b) French (c) German (d) US (b) 2 3 4 1
(c) 4 3 2 1
26. US used _____, agent orange and phosphorous bombs (d) 4 1 2 3
to destroy villages and jungles of Vietnam.
33. Which Indian revolutionary helped Subhash
(a) Napalam (b) Hydrogen bomb
Chandra Bose in raising ‘Indian National Army’?
(c) Both ‘a’ and ‘b’ (d) None of these
(a) Batukeshwar Dutt (b) Rash Bihari Bose
27. After the Versailles Treaty ______ lost its overseas (c) Ram Prasad Bismil (d) Surya Sen
colonies, population, territories, iron and coal 34. Which Act of the Government of India introduced the
reserves to France, Poland, Denmark and Lithuania. separated electorate for Muslims?
(a) Germany (b) Canada (a) 1909 (b) 1919
(c) Britain (d) None of these (c) 1935 (d) None of these
568 Study Package for NTSE

35. Who wrote ‘Poverty and Un-British rule in India’? 40. In which year the Muslim League was founded?
(a) RC Dutt (b) Dadabhai Naoroji (a) 1904 (b) 1906 (c) 1908 (d) 1911
(c) DR Gadgil (d) BN Ganguli
41. Integration of states was done by
36. Who was the President of the Indian National (a) Sardar Vallabh Bhai Patel
Congress at the time of Indian Independence? (b) Sheikh Abdullah
(a) JB Kriplani (c) Maulana Azad
(b) Jawahar Lal Nehru (d) Motilal Nehru
(c) Maulana Abul Kalam Azad
(d) Rajendra Prasad 42. Who was the first English President of the Indian
37. Who was the Commander-in-Chief of INA before National Congress?
Subhash Chandra Bose? (a) AO Hume (b) CF Andrews
(a) Giani Pritam Singh (c) George Yule (d) Hamilton
(b) Capt Mohan Singh
43. When did Cripps Mission come to India?
(c) Major Fuzihara
(d) Capt Suraj Mal (a) 1939 (b) 1942
(c) 1945 (d) 1946
38. Who was not associated with the Ghadar Party?
(a) Lala Har Dayal 44. Who did not support the Non-cooperation Movement?
(b) Kartar Singh (a) Motilal Nehru (b) Rajendra Prasad
(c) Ganesh Vishnu Pingle (c) Saukat Ali (d) Annie Beasant
(d) Shyamji Krishna Verma
45. Which paper was published by early leaders of Indian
39. Which one of the following faced trial in Kakori Case? Congress in England?
(a) Asfaquallah Khan (b) Rajendra Lahiri (a) India (b) Indian
(c) Roshan Singh (d) All of these (c) Indian Opinion (d) The Leader

Answers
Elementary Level
1. (a) 2. (b) 3. (b) 4. (c) 5. (a) 6. (b) 7. (a) 8. (c) 9. (b) 10. (d)
11. (c) 12. (a) 13. (b) 14. (b) 15. (d) 16. (b) 17. (b) 18. (a) 19. (c) 20. (c)
21. (b) 22. (b) 23. (c) 24. (d) 25. (a) 26. (b) 27. (a) 28. (a) 29. (b) 30. (a)
31. (c) 32. (b) 33. (c) 34. (c) 35. (b) 36. (b) 37. (b) 38. (b) 39. (d) 40. (c)
41. (c) 42. (d) 43. (d) 44. (c) 45. (c) 46. (d) 47. (c) 48. (a) 49. (c) 50. (a)

High Skill Questions


1. (c) 2. (c) 3. (a) 4. (b) 5. (a) 6. (c) 7. (c) 8. (d) 9. (a) 10. (c)
11. (b) 12. (a) 13. (d) 14. (a) 15. (b) 16. (b) 17. (b) 18. (b) 19. (b) 20. (b)
21. (a) 22. (b) 23. (b) 24. (a) 25. (d) 26. (a) 27. (a) 28. (b) 29. (a) 30. (a)
31. (c) 32. (d) 33. (b) 34. (a) 35. (b) 36. (a) 37. (b) 38. (d) 39. (d) 40. (b)
41. (a) 42. (c) 43. (b) 44. (d) 45. (c)
Chapter

4
Livelihoods, Economics
and Societies
Forest Society and Colonialism to run locomotives and sleepers were required to hold
tracks together.
Forest Resources l Each mile of railway track required between 1760 to
2000, sleepers.
Forests are renewable resources and contribute l By 1890, about 25500 km of track had been laid and in
substantially to economic development of a country. 1946, the length of the tracks had increased to over
They also play a major role in enhancing the quality of 765000 km.
environment. l As the railway tracks spread through India, a large and
l Forests provide us bamboo wood for fuel, grass, charcoal, large number of trees were required.
packaging, fruits, flowers, animals, birds and many
other things. Rise of Commercial Forestry
l Between 1700 to 1995, the period of industrialisation, The British Government appointed German expert
13.9 million sq km of forest or 9.3% of the world’s total Dietrich Brandis, the first Inspector General of Forests
area was cleared for industrial use, cultivation, pastures
in India. Brandis realised that a proper system had to be
and fuel wood.
introduce to manage forests and people had to be
l Under colonial rule, deforestation became more trained in the science of conservation. Brandis set-up
systematic and extensive.
the Indian Forest Service in 1864 and helped to
formulate the Indian Forest Act of 1865.
Land to be Improved
l The Imperial Forest Research Institute was set up at
In the colonial period, cultivation expanded rapidly. The Dehradun in 1906.
British encouraged the production of commercial crops l The Forest Act of 1865, was amended twice, once in 1878
like jute, sugar, wheat and cotton. The demand for these and then in 1927.
crops increased for the growing need of urban l The headquarters of Forest Survey of India is at
population and raw materials for industries. Dehradun and four regional offices are at Shimla,
Kolkata, Nagpur and Bengaluru.
l The colonial powers thought that forests were
unproductive, so land should be brought under
cultivation, so that it could enhance the income of the
Scientific Forestry
state. Between 1880 to 1920, cultivated area rose by A system of cutting trees controlled by the forest department in which old
6.7 million hectares. trees are cut and new ones are planted.

Decline in Forest Cover due to Railways How the Lives of People are Affected by
The spread of railways played an important role in the
the Forest Laws?
decline of forests cover in India. Wood was used as fuel The Forest Act of 1878, divided forest into three
categories, viz reserved, protected and village forests.
570 Study Package for NTSE

The best forests were called ‘reserved forests’ from l The British sent troops to suppress the rebellion and it
where villagers could not take anything for their own took three months for the British to regain control.
use. For house building or fuel, villagers could take
wood from only protected and village forests. Forest Transformation in Java
l Villagers wanted forests to satisfy different needs–fuel, Java is an island of Indonesia. It was mostly covered
fodder, leaves. The Forest Act meant severe hardship for with forests. The colonial power in Indonesia were the
them. Dutch, and they wanted timber from Java to build ship.
l It became quite difficult for them to meet their everyday The Kalanga of Java were a Community of skilled forest
needs like, getting wood for their houses, collecting cutters and shifting cultivators.
fruits and roots, hunting and fishing, grazing their
cattle etc. l In the 18th century, Dutch tried to make the Kalangas
work under them.
l To meet their needs, people were forced to steal woods.
l In 1770, the Kalangas revolted and attacked the Dutch
l If they were caught, they were given harsh punishment fort at Joana but the uprising was easily suppressed by
or sometimes were harassed by the police and forests the Dutch.
staff.
l In the 19th century, the Dutch enacted Forest laws in
l Women, who collected fuel wood had to face much Java. According to this law, villager’s access to forests
harassment. were restricted. Wood could only be cut for making
houses and river boats.
How did Forest Rules Affect Cultivation? l Villagers were punished for grazing cattle in the forests,
Shifting cultivation is a traditional agricultural practice transporting wood without a permit, or travelling on
in many parts of Asia, Africa and South America. It was forest roads with horse carts or cattle.
one of the major impacts of European colonialism. l Around 1890, a widespread movement under the
l In shifting cultivation, cultivators used to cut certain leadership of Surontiko Samin developed. People
parts of the forest in rotation, burn the trees and sow protected by lying down on their land when the Dutch
seeds in the ashes, after the first monsoon rains. came to survey it, while others refused to pay taxes,
fines or perform labour.
l The British Government decided to ban shifting
cultivation.
l As a result, many communities dependent on shifting
Pastoralists in the Modern World
cultivation were forced to displace from their own home. Pastoral Nomads and their Movements
l Some had to change their occupations, while some
revolted against the law. Gujjar Bakarwals of Jammu and Kashmir
l Gujjar Bakarwals of Jammu and Kashmir are great
The Effects of Colonial Management herders of goat and sheep. Many of them migrated to
in Bastar this region in search of pastures for their animals.
l In winter, when the mountains were covered with snow,
Bastar is located in the Southern most part of they lived with their herds in the low hills of Siwalik
Chhattisgarh and the borders of Andhra Pradesh, range. Here, the dry scrub forests provided pasture for
Orissa and Maharashtra. A number of different their herds.
communities live in Bastar, they are Maria and Muria l In summer, when the snow melted in the mountains and
Gonds, Dhurwas, Bhatras, Halbas etc. mountainsides were lush green, they moved onto high
l In 1905, the British Government proposed to reserve levels.
two-thirds of the forests in Bastar. The Colonial l The variety of sprouted grass provided rich nutritious
government ordered to stop shifting cultivation, hunting forage for their animal.
and collection of forest products. l By the end of September, they were on the move again
l A large number of people were displaced without any for their downward journey. They continued this
notice or compensation. practice every year.
l Some villagers were allowed to stay on in the reserved
forests on the condition that they would work free for the Gaddi Shepherds of Himachal Pradesh
Government. l Gaddi Shepherds of Himachal Pradesh had a similar
l The people of Bastar organised themselves and revolted cycle of seasonal movements like Gujjar Bakarwals of
against the British. Jammu and Kashmir.
l Markets were looted, the houses of officials and traders, l During winter, they grazed their flocks in scrub forests
schools and police stations were burnt and robbed and of the low hills of Siwalik range.
grains were redistributed.
Livelihoods, Economics and Societies 571
l By April, they moved North and spent the summer in l It was the alteration of the monsoon and dry season that
Lahul and Spiti. When the snow melted on the high defined the seasonal rhythms of their movement.
mountains, they moved on to higher mountain l They lived near the woods, cultivated small patches of
meadows. land, sold woven blankets and took care of their herds.
l By September, they began their return movement. On l In the summer, they moved to the coastal tracts and left
the way they stopped once again in the villages of Lahul when the monsoon came.
and Spiti, where they reaped the summer harvest and
sowed their winter crop. The Life Style of Banjaras
l They further descended to their winter base – the
The Banjaras were an important group of graziers,
Shiwalik hills. Next April, once again the began their
which were found in the villages of Uttar Pradesh,
march with their cattle to the summer meadows. Every
year, they went through the similar cycle of seasonal Punjab, Rajasthan, Madhya Pradesh and Maharashtra.
movement. l In search of good pasture land for their cattle, they
moved over long distances.
Other Pastoral Communities l In exchange for grain and fodder, they sold plough cattle
l In Garhwal and Kumaon, the Gujjar cattle herders came and other goods to villagers.
down to the dry forests of the bhabar in the winter and
went up to the high meadows the bugyals in summer. Pastoral Community in Desert
l Many pastoral communities like Bhotiyas, Sherpas and Raikas lived in the deserts of Rajasthan. Before 1947,
Kinnauris followed the cyclical movement between they used to migrate into Sindh and grazed their
summer and winter pastures. animals on the banks of the Indus. But after partition,
l All of them had to adjust to seasonal changes and make when Sindh became a part of Pakistan, this activity was
effective use of available pastures in different places. restricted.
Bhabar Dry forested area below the foothills of garhwal and kumaun. During the monsoon, the Raikas of Barmer, Jaisalmer,
Bugyal Vast meadows in the high mountains. Jodhpur and Bikaner stayed in their home villages,
where pasture was available. By October, when these
grazing grounds were exhausted, they moved out in
Pastoral Nomads in Plateaus, search of new pasture.
Plains and Deserts of India l They returned again in their home villages during the
next monsoon.
Dhangar Community of Maharashtra l In recent years, Raikas have been migrating to Haryana
Dhangars were an important pastoral community of instead of Sindh region.
Maharashtra. They were mainly shepherds, blanket l One group of Raikas, known as Maru (Desert) Raikas,
weavers and buffalo herders. In the early twentieth herded camel and another group reared sheep and goat.
century, their population in this region was estimated
to be 467000. They stayed in central plateau of Effects of Colonial Rule on
Maharashtra during monsoon, which was a semi-arid
area with poor soil and low rainfall. Here they
Pastoral Life
cultivated only bajra. The Colonial government made different laws from time
l The thorny scrub of this plateau provided forage for
to time, which severely affected the lives of the
their cattle. pastoralists.
l In October, when they had reaped bajra crop, they
started their journey Westwards to Konkan, where Effects of the Wasteland Rules
they and their flock was welcomed by the Konkan l The colonial power believed all grazing land as
peasants. ‘Wasteland’ because these were unproductive.
l The Dhangar flocks were fed on the stubble (or green l These lands did not produce revenue or agricultural
stalks left after the Kharif harvest) and in return their product.
flocks manured the fields. l From the mid-nineteenth century, the Colonial
Government introduced Wasteland Rules, which were
Other Pastoral Communities enacted in various parts of our country.
The Gollas, Kurumas and Kurubas were important l By implementing this rule, the government took over
pastoral communities of the dry central plateau of uncultivated lands and gave these to selected
Karnataka and Andhra Pradesh. individuals.
572 Study Package for NTSE

l In most areas, the lands taken over by the government Tanzania. The Massai cattle herders live primarily in
were actually grazing lands used regularly by the East Africa, 300000 in Southern Kenya and another
pastoralist communities. 150000 in Tanzania.
l Thus, the expansion of cultivation created a lot of
problems for pastoralists. After colonial rule, best grazing lands of Massai
community were taken for settlement and gradually
Effects of the Forest Act Massai were pushed into small area in South Kenya and
North Tanzania. The Massai community lost about 60%
l The Forest Acts made by the British Government
changed the lives of pastoralists.
of their land and were confined to an arid zone with
insufficient rainfall and poor pastures.
l In the reserved forests, no pastoral activity was allowed
and in the protected forests, their activity was strictly
restricted. Peasants and Farmers
l In the protected forests, they needed a permit for entry.
l The permit specified the timing of their entry and The Time of Open Fields and Commons in
departure. England
l If they overstayed there, they were liable to fines.
The agriculture system in England changed
Effects of Criminal Tribes Act dramatically over the late 18th and early 19th century.
Before this time, a large part of England was open. The
In 1871, the British Government passed the Criminal land was not enclosed or partitioned by the landlords. At
Tribes Act. By this Act, many communities of the beginning of each year, people was allocated a
craftsmen, traders and pastoralists were classified as number of strips to cultivate. These strips were of
Criminal Tribes. varying quality, not next to each other.
l They were stated to be criminal by nature and birth. l Beyond these strips of land, there was also common
l As a result of this Act, they were not allowed to move land.
without a permit. l Everybody had access to the common land. Here they
l The village police also kept a strict watch on them. grazed their cattle, collected berries and fruits for food
and fuel wood for fire.
Imposition of Grazing Tax and its Effect l They also fished in the rivers and ponds and hunted
In the mid-nineteenth century, Grazing Tax was rabbit in common forests.
introduced by the British government in most pastoral l For the poor, the common land was essential for survival
lands of India. as it helped them to overcome bad times.
l In order to increase its income, the government imposed
tax even on animals. The Factors Led to the Enclosures
l The pastoralists had to pay tax on every animal they in England
grazed in the pastures. After the mid-eighteenth century, the enclosure
l They were issued passes, pay the tax and only then they movement swept through the countryside in England.
were allowed to enter a grazing tract. In the 16th century, the price of wool went up in the
world market. Rich farmers wanted to expand wool
Pastoralism in Africa production for profit. So, they began dividing and
enclosing common land to separate their property from
In Africa, over 22 million people depend on some form of that of others.
pastoral activity for their livelihood. The important From the mid-eighteenth century, the population of
pastoral communities of Africa are Bedouins, Britain expanded rapidly. Between 1750 to 1900, it
Berbers, Massai, Somali, Baran and Turkana. multiplied over four times, mounting from 7 million in
Most of them lived in semi-arid grassland where rainfed 1750 to 30 million in 1900. Thus, the need of different
food crops greatly increased, which forced the landlords
agriculture is difficult. They raise cattle, camels, goat,
to enclose more lands for cultivation.
sheep and donkeys. They sell milk, meat, animal skin
l Britain was industrialising at that time. More and more
and wool. Some earn through trade and transport,
people migrated to towns in search of jobs. As the urban
others combine pastoral activity with agriculture. population increased, the market for foodgrains
expanded, which encouraged the land owners to enclose
Changes in the Life of Massai Community more lands.
Before colonial rules, Massailand stretched over a vast l During the last decade of the 18th century, France was
area from North Kenya to the steppes of Northern at war with England. Then, it became quite difficult for
Livelihoods, Economics and Societies 573
England to import foodgrains from Europe. Rich land The Impact of the Westward Expansion of
owners of England were encouraged to produce more
and more foodgrains by enclosing the common village
the Settlers in the USA
land. From 1775 to 1783, after the American War of
l The greed of the land owners was as much responsible Independence and the formation of the United States of
for the extension of enclosures as the need of the hour. America, the Americans began to move Westward.
As profits flowed in, the land owners pressurised the l As a result, the American Indians had to be cleared from
Parliament to pass the Enclosure Acts. the land. After 1800, the US Government took a policy of
driving the American Indian Westward, first beyond the
The Conditions of the Poor after the river Mississippi and then further West.
Enclosure Movement l To displace local tribe was not an easy task. Many wars
Enclosures allowed only the landlords to make more were waged in which Indians were massacred and many
of their villages burnt.
profit. But for the poor, life became hard. They could no
longer collect firewood, fruit and berries, or graze their l The American Indians were forced to sign treatise and
move Westward.
cattle or hunt small animals for meat.
l The settlers slowly and slowly kept on their march
l Enclosure happened on an extensive scale in the
towards the West. They settled on the Appalachian
Midlands and the countries around which led the poor
plateau by the first decade of the 18th century and then
displaced from their own land. From the Midlands, they moved into the Mississippi valley between 1820 and
moved to the Southern countries of England in search of 1850.
job. But nowhere, they could find secure jobs.
Wherever the settlers went, they slashed and burnt the
l Earlier the labourer lived with their landlords. They
forests and cleared the land for cultivation. The settlers
helped their master and worked for them throughout
the year. But, as landowners tried to make more profits, cleared larger areas and started cultivation. They
they cut the amount they had to spend on their labourer. ploughed the land and sowed corn and wheat. After
Thus, the labourers were employed only during harvest 1860, the Great Plains across the river Mississippi
time. became a major wheat producing area of America.

The Introduction of Threshing Machines The Conversion of the Countryside in the


Enclosures allowed the richer landlords to expand the USA from the Bread Basket to a Dust Bowl
land under their control, but it affected the poor In the early 20th century, wheat cultivation in USA had
severely. The poor were not only starved but they expanded dramatically. Farmers slashed and burnt
became jobless.
forests indiscriminately, uprooted all vegetation, which
l Fearing a shortage of labour, the landlord began buying
had deep roots in the Earth. Because of the cutting of
threshing machines. They thought these machines
would help them to reduce their dependence on trees and grasses, there were no rains year after year
labourers. and the temperatures soared.
l A single threshing machine could do the work of more l The tractors had broken the soil into dust. The whole
than twenty labourers. region had become a dust bowl.
l Thus, the loss of their livelihood, forced the poor to l In the 1930s, terrifying dust storms began to blow over
oppose the introduction of threshing machine. the Southern Plains.
l At that time farmers received threatening letters, l The wind blew with ferocious speed. Black blizzards
urging them to stop using these machines. Most of rolled in sometimes 7000 to 8000 ft high. It looked like
these letters were signed in the name of Captain monstrous waves of muddy water.
Swing. l People were blinded and choked, cattle were suffocated
l The Captain Swing riots spread in the countryside at to death, sand covered fields and rivers and destroyed
that time. the tractors and machines.
l The American dream of a land of plenty had turned into
Sod Pieces of Earth with grass.
a nightmare.
l The Government took strong action. In 1976 prisoners l In their greed for more profit, the Americans played
were tried, nine men were hanged, 505 transported, over havoc with the natural ecology. They completely
450 of them to Australia and 644 put behind bars. destroyed the ecological balance of the land.
574 Study Package for NTSE

l After 1930, the Americans realised that they had to The Making of a Global World
maintain the ecological conditions of each region.

The Wheat Farmers of USA Silk Routes Link the World


From the late 19th century, there was a dramatic The silk routes were regarded as the most important
expansion of wheat production in the USA. The urban route linking the distant parts of the world. Historians
population increased and the export market became have identified several silk routes, over land and by sea
bigger. which are linking Asia with Europe and Northern
l The spread of railways made it easy to transport the Africa.
grain from the wheat growing regions to the Eastern l These routes are known to have existed even before the
coast for export. Christian Era and flourished till the 15th century.
l During First World War, the world market boomed. l These ancient routes were called silk routes because
Russian supplies of wheat were cut off, thus USA had to along these routes were mainly carried the silk cargoes
feed Europe. from China to different regions of Asia.
l In 1910, about 45 million acres of land in the USA was l Chinese pottery, Indian spices and textiles were also
under wheat. After nine years, the area had expanded to carried by these routes.
74 million acres, an increase of about 65%. l Along these routes, the Buddhist preachers, Christian
l In USA, a new class was emerged - ‘the wheat barons’ missionaries and later on Muslim preachers travelled.
who controlled as much as 2000 to 3000 acres of land l These routes proved to be a great source of trade and
individually. cultural links between distant parts of the world.

The Great Agrarian Depression Food Travels


During First World War and post-war years, the Usually, traders and travellers introduced new crops to
production of agriculture expanded very rapidly. But the lands they travelled. It is a common belief that
the boom of late 19th and early 20th century seemed to ‘noodles’ travelled West from China and became
have come to an end. ‘spaghetti’.
l By the 1920s, unsold stocks of grains piled up, storehouses
l Common foods like potatoes, soya, groundnuts, maize,
overflowed. Vast amount of foodgrains became fodder.
tomatoes, chillies, sweet potatoes etc were introduced
l Wheat prices fell and export markets collapsed. in Europe and Asia after Columbus discovered
l All these factors created the Great Agrarian America.
Depression of the 1930s that ruined wheat farmers l These foods came from America’s original inhabitants-
everywhere. American Indians.
The Indian Farmer and Opium Production l Europe’s poor began to eat better and live longer with
the introduction of potato.
In the late 18th century, like East India Company was
buying tea and silk from China for sale in England. The The Global Transfer of Disease in the Pre-
major question before the company was how to pay back
modern Period Helped in the Colonisation
the cost of the tea brought from China.
of America
l To solve this problem, the British merchants began
smuggling opium into China as the Chinese ruler was The Spanish and Portuguese were the first Europeans
not prepared to legally allow the trade of opium within to conquer America in mid-sixteenth century.
his country. They were aware of the dangers of opium The conquest was not only as a result of superior fire
addiction. power.
l By the early 1820s, about 10000 crates were being l It was the germs of disease like small pox which they
annually smuggled into China, after fifteen years it took with them.
became 35000 crates every year.
l America’s original inhabitants had no immunity against
l By 1773, the East India Company in Bengal had germs particularly of small pox.
established a monopoly to trade in opium.
l The germs of small pox brought by the Europeans to
l By 1820s, the British Government found that the opium America spread largely in America.
production in their territory was declining but its
production outside British territories, i.e., princely l This disease killed and decimated whole communities,
states in central India and Rajasthan was increasing. paving the way for colonisation.
Livelihoods, Economics and Societies 575
The Economical Change in 19th Century l Completely impoverished African were forced into the
labour market. They had no option left.
l Economists identify three types of movement or flows for
economical changes in 19th century. l Under such devastation the European colonisers were
able to easily conquer and subdue Africa.
l The first is the flow of trade which referred largely to
trade in goods, specially cloth and wheat. Indentured Labour Migration from India
l The second is the flow of labour due to migration of l Indentured labour means a bonded labourer under
people in search of employment. contract to work for an employer for a specific amount of
l The third is the movement of capital for short-term or time.
long-term investments over long distances. l In the 19th century, hundred of thousands of Indian and
Chinese labourer went to work on plantations, in mines
The Meaning of ‘Corn Laws’ and in different construction projects around the world.
l Industrialisation and population growth had increased l Most Indian indentured workers came from the
the demand for foodgrains in Britain in late 18th present-day regions of Eastern Uttar Pradesh, Bihar,
century. Central India and Tamil Nadu.
l This situation pushed up foodgrains prices. l In mid-nineteenth century these regions of India
l Under pressure from landlord class, the government experienced many social changes which forced the poor
restricted the import of corn. These laws were commonly to migrate in search of work.
known as ‘Corn Laws’.
l After the introduction of ‘Corn Laws’ food prices became Indian Trade and Colonialism
very high.
Fine cottons of India exported all over Europe. With the
l Unhappy with high food prices, industrialists, urban industrialisation, British cotton manufacturers began
dwellers forced the British Government to abolish the
to expand and the industrialists pressurised the
Corn Laws.
governments to restrict cotton imports.
l After the abolition of Corn Laws, food could be imported
l The British Government introduced high tariffs on
into Britain more cheaply than it could be produced
import of cotton cloth.
within the country.
l Indian textile industry faced stiff competition and
l British agriculture was unable to compete with imports.
export of cotton began to decline.
The Impact of Technology on Food Availability l While exports of manufactures declined rapidly, export of
raw materials, like raw cotton, indigo increased very fast.
l Technology or new inventions like railways, steamships,
telegraph, refrigerated ship had a great impact on food l Britain grew opium in India and exported it to China
availability. and with the earned money Britain financed tea and
other imports from China.
l Faster railways, lighter wagons and larger ships helped
to move food more cheaply and quickly from far away l Over the 19th century, British manufactures flooded the
farms in America, Australia or New Zealand to final Indian market.
markets of Europe. l Foodgrain and raw material exports from India to
Britain and the other countries of the world increased.
l The refrigerated ships greatly helped to transport the
perishable food items over long distance. Especially, it l The value of British exports to India was much higher
greatly facilitated the shipment of frozen meat from than the value of British imports from India.
America to different European countries. l Britain had a ‘trade surplus’ with India.
l Earlier time meat was considered as an expensive The Death of Men of Working-Age in Europe
luxury beyond the reach of the European poor. But as a
because of the World War
result of the refrigerated ship, meat became a daily diet
for the poor. The First World War was the first modern industrial
war. For the first time modern weapons like machine
The Effects of Rinderpest in Africa guns, tanks, aircraft, chemical weapons etc., are used on
l Rinderpest was a fatal animal disease which arrived in a massive scale.
Africa in the late 1880s. l To fight the war, millions of soldiers had to be recruited
l This disease which came from British Asia along with from around the world and most of them were men of
the Asian animals spread like forest fire in the whole working age.
Africa and killed 90% of the cattle. l During the war, 9 million people were dead and
l The loss of cattle destroyed livelihood of many Africans. 20 million were injured.
l Planters, mine owners and colonial Government l These deaths and injuries greatly affected the workforce
successfully monopolisted the remained cattle resources. in Europe. As a result industries were adversely
affected.
576 Study Package for NTSE
The First World War Changed the Economic Life of Bretton Woods Institutions
the People in Britain Bretton Woods Conference established the
The First World War changed the life of the people of International Monetary Fund (IMF) and the
Britain in a number of ways. While Britain was International Bank of Reconstruction and Development
preoccupied with war, industries had developed in India (popularly known as the World Bank) to maintain
and Japan. After war, it was impossible for Britain to economic stability and full employment in the industrial
recapture Indian market and compete with Japan world.
internationally.
l Both these institutions commenced financial operations
l As a result of the war, most of cities, factories, fields and in 1947, which they are continuing upto this date.
trades of Britain were ruined.
l Decision making in these institutions is controlled by
l When war ended, production contracted and
the Western Industrial Powers.
unemployment increased in Britain. In 1921, one in
every five British workers was out of work. l The US has an effective right of veto over key IMF and
l Britain had borrowed heavily from US to finance war World Bank decisions.
expenditure. At the end of the war, Britain was l Under this system, the national currencies followed the
burdened with huge external debts. fixed exchange rates and were pegged to the US dollars.
l As a result of the lack of financial resources, the British
Government had to impose heavy taxes on its people. l This system ushered an era of unprecedented growth of
trade and incomes for the Western industrial nations
l The prices of almost all things greatly rose due to the
and Japan. There was also world wide spread of
unavailability of daily use items.
technology and enterprise.
The Great Depression The Decision of MNCs to Relocate Production to
The Great Depression began around 1929 and lasted till Asian Countries
the mid 1930s. During this period, most of the countries l Multinational Corporations (MNCs) are large
of the world experienced catastrophic declines in companies that operate in several countries at the same
production, employment, incomes and trades. time. The first MNCs were established in the 1920s and
the worldwide spread of MNC was a notable feature of
The First World War gave a boost to the agriculture
the 1950s and 1960s.
(abundance) and industry of USA. But after the war, there
was a glut in the market and it pushed down the prices of l From the late 1970s, MNCs began to shift production
operations to low wage Asian countries.
industrial goods and agricultural product. As a result USA
was caught in a severe economic crisis. American l Such a decision increased the job opportunities in the
capitalists stopped all loans to the European countries. Asian countries.
The countries depended crucially on US loans now faced l The coming of new MNCs gave the people the
an acute crises. opportunity to enjoy new varieties of things.
l These relocation of industry stimulated world trade and
The Effects of the Second World War capital flow.
l The Second World War broke out after two decades of l These MNCs helped the Asian countries to come out of
the First World War (1939-1945). the former colonial powers which still held the main
l It was fought between the Axis powers (Nazi, Germany, controls of economic power of the new independent
Japan, Italy) and the Allies (Britain, France, Soviet countries.
Union and USA). l MNCs proved a great help to the true beginning of
l The world saw once again death and destruction on a globalisation.
wider scale. At least 60 million people were killed and
million were injured. The Age of Industrialisation
l Vast parts of Europe and Asia were devastated and
destroyed. The Meaning of Proto-Industrialisation
l The war caused an immense amount of economic
devastation and social disruption. Proto-industrialisation means large scale industrial
l The USA and Soviet Union emerged as dominant super production for an international market even before the
power in the world. emergence of factory system. Such production was not
l The rise of two world powers led to the race for based on factories. This phase of industrialisation
armaments in the world of war. before the appearance of factories is referred to as
l Germany was weakened and was soon divided into two Proto-industrialisation by many historians. In the
parts of East Germany and West Germany. 17th and 18th centuries, the demand for goods began
l Japan and Italy, being defeated nations became weak. growing.
Livelihoods, Economics and Societies 577
The Pace of Industrial Change supervise weavers, collect supplies and examine the
quality of cloth.
The Industrial Revolution took place in Europe in the
l The East India Company tried to eliminate the existing
18th century. The most dynamic industries in Britain
traders and brokers connected with the cloth trade and
were cotton and metals. Cotton was the leading sector in establish more direct control over the weavers by
the first phase of industrialisation. Then iron and steel appointing gomasthas.
industry led the way. l This system prevented company weavers from dealing
l The new industries could not easily displace traditional with other buyers. Weavers who took loan from the
industries like domestic units. gomastha had to hand over the cloth to the gomastha.
l The pace of change in the traditional industries was not They could not give it to any other trader.
set by steam-powered cotton or metal industries. Small l The new gomasthas were outsiders and their treatment
innovations led the growth in many non-mechanised with the weavers was very harsh.
sectors.
l The weavers lost the space to bargain for prices.
l Technological changes occurred slowly. Industrialists
accepted the most powerful technology gradually. l The price they received was miserably low and the loans
they had accepted tied them to the company.
Women Workers in Britain Attacked the Spinning l Many weavers deserted their villages and set up their
Jenny looms elsewhere.
Spinning Jenny was invented by James Hargreaves in
1764. This machine speeded up spinning process and Manchester Comes to India
reduced labour demand. As cotton industries developed in England, the
l By the use of this machine, a single worker could make a industrialists pressurised the government to impose
number of spindles and spin several threads at a time. import duties on cotton textiles so that Manchester good
l It meant that as a result of this machine, many weavers could sell in Britain without any competition from other
became unemployed. countries.
l Spinning Jenny was introduced in the woolen industry. l At the same time, industrialists persuaded the East
l It was the fear of unemployment which made women India Company to sell British manufactures in Indian
workers, who survived on hand spinning, began market.
attacking the new machines. l In the early 19th century, the exports of British cotton
l This conflict continued for a long time. good increased largely.
l Cotton weavers in India faced two problems at the same
The Port of Surat Declined by the End of the 18th
time. Their export market collapsed and the local market
Century shrank being captured with Manchester imports.
Before the age of machines the port of Surat on the West l By the 1850s, weaving industry of India began to decline.
coast of India was the major port to handle the Indian l By the 1860s, Indian weavers faced a new problem. As
foreign trade with the West. It connected India to the civil war broke out in America, cotton supplies from US
Gulf and Red Sea Ports. By the 1750s, export trade were cut off.
controlled by Indian merchants was breaking down. l Raw cotton exports from India increased and the price of
l Various European companies, especially the East India raw cotton shot up.
Company gradually gained the monopoly rights to trade. l Weavers in India were starved of supplies.
l They set up and patronised their own ports like Bombay l Factories in India began production. Weaving industry
situated on the West coast of India. faced hard competition.
l All this resulted in the decline of old ports like Surat.
The Early Entrepreneurs of India
l The exports from this port fell dramatically.
l In Bengal, Dwarkanath Tagore invested in shipping,
l In the last years of the 17th century, the gross value of
shipbuilding, mining, banking, plantations and
trade that passed through Surat was ` 16 million, by
insurance.
1740s, it slumped to ` 3 million.
l In Bombay, Parsis like Dinshaw Petit and Jamsetjee
The East India Company Appointed Gomastha to Nusserwanjee Tata built huge industrial empires in
Supervise Weavers in India India, accumulated their wealth partly from exports to
China and partly from raw cotton shipments to England.
After establishing political power, the East India
l Seth Hukumchand, a Marwari businessman set up
Company wanted to control the monopoly right to trade.
the first Indian jute mill in Calcutta in 1917. He also
They appointed paid servant called gomastha to traded with China.
578 Study Package for NTSE
Industrial Production in India Increased During the products, such as surgical instruments, watches and
First World War objects of precious metal.
Before First World War (1917-1918), the industrial l During the First World War the number of large
factories increased in London which attracted large
growth in British India was quite slow. The war created
number of people to settle here.
a dramatically new situation. Almost all the British
mills switched over to the production of war materials The Changes in the Kind of Work
for the British army.
Available to Women in London between
l Manchester imports to India declined.
the 19th and the 20th Century
l Indian mills had a golden opportunity to supply for the
home market. l Women in the late 18th and early 19th centuries in
Britain were employed in large numbers in the factories
l As the war prolonged, the Indian industry was asked to
as they were available at cheaper rates than men.
cater to the war needs of the British.
l Indian industry became busy in manufacturing jute l In the 19th century, due to technological developments,
bags, cloth for army uniforms, tents, leather boots, horse women lost their industrial jobs and forced to work
and mule saddles etc. within households.
l Many new workers were employed and hours of work l According to 1861 census, there were a quarter of a
were also extended. million servants in London, of whom the vast majority
were women.
l The First World War proved a blessing in disguise for
the Indian industry. l A large number of women were engaged in tailoring,
washing, making matchboxes etc.
The Role of Advertisement in Expanding the
l In 20th century, there was change once again. At that
Markets for Products time, women got employment in wartime industries and
In the late 19th century, the British manufacturers offices. Then they withdrew from domestic services.
used calenders to popularise their products, unlike
Housing Problems of Poor in London in the
newspapers and magazines, calenders were used even
19th Century
by people who could not read.
l Like the images of Gods, figures of important After the industrial revolution, a large number of people
personages, of emperors and nawabs were also used in began to migrate in London. But, the factory or
advertisements and calenders. when the products were workshop owners did not house the migrant workers.
being used by kings, their quality could not be Individual landowners put up cheap and unhealthy
questioned. tenements for the workers.
l Indian manufacturers used nationalist message of
Swadeshi for their products. In 1887, a survey revealed that 1 million Londoners
were very poor and were expected to live only up to an
average age of 29 (compared to the average life
Work, Life and Liesure expectancy of 55 among the gentry and the middle
Reasons behind the Expansion of class). The condition of slums was very unhealthy.
Gradually, a large number of people began to recognise
Population in London from the Middle of
the need for housing for the poor.
the 18th Century
l The one-room houses occupied by the poor were a
The population of London was about 675000 in 1750. It serious threat to public health as they were
multiplied four fold from 1 million in 1810 to four overcrowded, badly ventilated and lacked sanitation.
million in 1880. l Poor housing system could create fire hazards which
l According to Historian Gareth Stedman Jones, ‘‘19th could engulf other areas in the fire disaster.
century London was a city of clerks and shopkeepers, of l After the Russian Revolution of 1917, there was a
small masters and skilled artisans, of a growing number
widespread fear of social disorder. Poor section of society
of semi-skilled and sweated out workers, of soldiers and
could revolt against the social disparity.
servants, of casual labourers, street sellers and beggars.
Thus, worker’s mass housing schemes were
l Five major types of industries in London employed a
accepted by the British Government. A million of
large number of people. These industries were clothing
and footwear, wood and furniture, metals and houses, mostly single-family cottage were built by the
engineering, printing and stationery and precision British State.
Livelihoods, Economics and Societies 579
Underground Railway System in London l The new Paris city soon got converted into a civic pride
as the new capital became the toast of all Europe.
The underground railway in the world opened on 10th
l Paris became the hub of many new architectures, social
January, 1863 in London. and intellectual developments that were very
l The underground railway solved the transport problem influential through the 20th century in many parts of
to a great extent. the world.
l The London tube railway led to a massive displacement
of the London poor. The Expansion of Bombay’s Population
As a result the population in the city became more in the Mid-nineteenth Century
dispersed. Now the people began to settle in
better-planned suburbs and travel to work. Bombay is the biggest sea-port on the West coast of
l The expansion of transport network wore down social India. Bombay became an important administrative
distinctions and also created new ones. centre in Western India and by the end of the 19th
century it became a major industrial centre. When in
Haussmanisation of Paris 1819, Bombay became the capital of Bombay
In 1852, Louis Napaleon II crowned himself emperor. Presidency, it attracted more and more people
After taking over he undertook the rebuilding of Paris towards it.
with vigour. The chief architect of the new Paris was Baron With the growth of trade in cotton and opium, a large
Haussman who rebuilt Paris for continuous 17 years. number of traders and bankers as well as artisans and
l He designed straight, broad avenues or boulevards and shopkeepers came to settle in Bombay.
open spaces transplanted full-grown trees. l The establishment of textile mills also led to a fresh
l This new reconstruction displaced upto 350000 people surge in migration.
from the centre of Paris. l By 1921, there were 85 cotton mills with about 146000
l Some said that the city of Paris had been monstrously workers.
transformed. l Large numbers of people from the nearby district of
l Some lamented the passing of an earlier way of life, and Ratnagiri to work in the Bombay mills.
the development of an upper-class culture. l When Bombay became the hub of Indian films many
l Some believed that Haussman had ‘killed the street’ new people-artists, dramatists, playwriters, poets,
and its life, to produce an empty, boring city, full of singers, script writers etc. came to settle here.
similar-looking boulevards and facades.

Target Exercise
Elementary Level
1. When was the Indian Forest Service set-up? 6. How many tigers were killed between 1875-1925?
(a) 1864 (b) 1664 (c) 1564 (d) 1764 (a) 200000 (b) 150000 (c) 80000 (d) 160000
2. How much India’s landmass was under cultivation in 7. Which colonial power ruled Java?
1600? (a) The English (b) The Portuguese
(a) One-third (b) One-sixth (c) One-fourth (d) One-fifth (c) The Dutch (d) The French
3. Which Act was passed in 1865? 8. Where is Bastar located?
(a) Indian Forest Act (b) The Factory Act (a) Madhya Pradesh (b) Bihar
(c) The Regulatory Act (d) The Land Acquisition Act (c) Jharkhand (d) Chhattisgarh
4. What do you mean by deforestation? 9. Which country occupied Indonesia during the Second
(a) Not planting new trees (b) Selling trees to traders World War?
(c) Disappearance of forests (d) None of the above (a) Japan (b) Germany (c) France (d) England
5. Where was the Imperial Forest Research Institute 10. Which one of the following is not a pastoral
setup in 1906? community?
(a) Bombay (b) Shimla (a) The Rajputs (b) The Sherpas
(c) Dehradun (d) Sunderban (c) The Kinnauris (d) The Bhotiyas
580 Study Package for NTSE
11. What are Dhars? 26. The American Indians settled on the Appalachian
(a) Deep valleys (b) Desert land plateau by the first decade of
(c) Fertile plains (d) High meadows (a) the 16th century (b) the 17th century
12. Which one of the following Acts was passed in 1871? (c) the 18th century (d) the 15th century
(a) Wasteland Rules (b) The Forest Act 27. How much land the wheat barons possess
(c) Grazing Tax (d) The Criminal Tribes Act individually?
13. In which areas, the Raikas lived? (a) Between 600 –1200 acres (b) Between 2000 –3000 acres
(a) Himachal Pradesh (b) Uttarakhand (c) Between 700 –1500 acres (d) Between 4000 –5000 acres
(c) Rajasthan (d) Punjab 28. Which country came to be known as the bread basket
14. Dhangars were an important pastoral community of of the world?
(a) Himachal Pradesh (b) Rajasthan (a) India (b) Japan (c) France (d) USA
(c) Maharashtra (d) Bihar 29. Silk routes are known to have existed, since before
15. Massai tribe is one of the pastoral tribes of the Christian era and thrived almost till the
(a) Asia (b) America (c) Africa (d) Australia (a) 15th century (b) 16th century
(c) 17th century (d) 18th century
16. In which of the continent of the world, about half of
the population lives as pastoralists? 30. Who were the first Europeans to conquer America?
(a) Asia (b) Europe (a) The French (b) The Portuguese
(c) Africa (d) America (c) The Spanish (d) Both ‘b’ and ‘c’
17. In which part of India Gaddi Shepherd live? 31. What was the ‘Corn Laws’ passed by the British
(a) Himachal Pradesh (b) Jammu and Kashmir Government?
(c) Karnataka (d) Jharkhand (a) Restriction on the export of corn
(b) Restriction on the import of corn
18. Which one of the following is not a pastoral
(c) Import corn from other countries
community of Africa? (d) Import corn of good quality
(a) The Bedouins (b) The Massai
(c) The Dhangars (d) The Barbers 32. In the 19th century, nearly ______ million people
emigrated from Europe to America and Australia.
19. Which one of the following is the game reserve of
(a) 40 (b) 10 (c) 50 (d) 60
Tanzania?
(a) The Massai Mara Park (b) The Serengeti Park 33. What is Rinderpest?
(c) The Samburu National Park (d) None of these (a) A cattle disease in Britain (b) A cattle disease in Africa
(c) A cattle disease in Russia (d) A cattle disease in America
20. In which type of forests, no pastoral activity was
allowed? 34. When did the Great Depression begin?
(a) Reserved forests (b) Protected forests (a) 1925 (b) 1750 (c) 1850 (d) 1929
(c) Village forests (d) Wild forests 35. Which one of the country was not a part of the Allies
21. Why did the peasants protest against the threshing powers in the Second World War?
machines in the 19th century? (a) Britain (b) France
(a) These were too difficult to operate (c) Soviet Union (d) Japan
(b) These were not safe 36. Axis powers in the Second World War were Germany,
(c) These deprived the workers of their livelihood Japan and
(d) These were very expensive (a) China (b) Italy (c) Australia (d) Denmark
22. When was the land enclosed for food production in 37. ______ created the cotton mill.
England? (a) Richard Arkwright (b) James Watt
(a) In the 16th century (b) In the 17th century (c) Samuel Crompton (d) None of these
(c) In the 18th century (d) In the 19th century
38. Which port connected India to the Gulf and Red Sea
23. Who was Captain Swing? Ports?
(a) A mythic name to threaten the farmers
(a) Calcutta (b) Bombay (c) Hooghly (d) Surat
(b) A captain in the British army
(c) A leader of the factory workers 39. The first cotton mill in Bombay came up in ______
(d) A leader of the cultivators (a) 1754 (b) 1854 (c) 1870 (d) 1910
24. Name the crop which improved the fertility of the soil. 40. Who were the first Europeans to come to India?
(a) Wheat (b) Opium (a) The British (b) The French
(c) Turnip (d) Sugarcane (c) The Patuguese (d) The Spanish
25. After the American War of Independence, the white 41. Name the paid servant appointed by the British to
Americans began to move deal with the Indian weavers.
(a) Northward (b) Southward (a) Seth (b) Gomastha
(c) Westward (d) Eastward (c) Sepoy (d) Lambardar
Livelihoods, Economics and Societies 581
42. Handicrafts production expanded in the ______ 47. What was the expected life of poor workers in London
century due to some technological changes. in 1887, according to Charles Booth?
(a) 17th (b) 18th (c) 19th (d) 20th (a) 29 years (b) 39 years
(c) 49 years (d) 59 years
43. ______ was a colossal city with a population of about
675000 in mid-eighteenth century. 48. Between the two World Wars (1919-1939), who
(a) Paris (b) London (c) Moscow (d) Bombay accepted the responsibility of housing of the poor
44. Who built tenement for the migrant workers in class in England?
(a) The British state (b) The Industrialists
London?
(c) The Social groups (d) The Private landowners
(a) The British Government (b) The Nobles
(c) Individual landowners (d) The factory owners 49. Name one factor which changed the form of
45. The novel ‘Debganer Martye Aagaman’ was written urbanisation in the modern world.
(a) Capitalism (b) Industrialisation
by
(c) Secularism (d) Unemployment
(a) Premchand
(b) Bankim Chandra Chattopadhaya 50. Who was the first Municipal Commissioner of
(c) Durgacharan Ray Bombay?
(d) Rabindranath Tagore (a) Arthus Crawford
46. The first underground railways in the world opened (b) James Simpson
in 1863 in (c) Kessowjee Naik
(a) London (b) Paris (c) Moscow (d) Berlin (d) Dinshaw Petit

High Skill Questions


1. Between 1700 to 1995, how much area of forest cover 7. A system of cutting trees controlled by the forest
in the world was cleared industrial uses cultivation, department in which old trees are cut and new ones
pastures and fuel wood? planted is known as
(a) 13.9 million sq km (b) 14 million sq km (a) modern forestry (b) planned forestry
(c) 20 million sq km (d) 25 million sq km (c) scientific forestry (d) None of these

2. How many sleepers were required for each mile of 8. Match the following
railway track?
List I List II
(a) 1700 to 2500 (b) 1760 to 2000
(Traditional Agricultural Practice) (Nome of the Country)
(c) 1000 to 2000 (d) 1360 to 2600
A. Lading 1. Sri Lanka
3. The ______ Forest Act divided forests into three B. Milpa 2. Africa
categories, reserved, protected and village forests. C. Chitemene 3. Central America
(a) 1868 (b) 1878 D. Chena 4. South-East Asia
(c) 1906 (d) 1865
Codes
4. Over 80000 tigers, 150000 leopards and 200000 A B C D
wolves were killed for reward in the period of (a) 4 3 2 1
(a) 1820-1920 (b) 1775-1875 (b) 4 2 1 3
(c) 1875-1925 (d) 1880-1980 (c) 1 2 4 3
(d) 2 4 1 3
5. Match the following
9. The Kalangs of ______ were a community of skilled
List I List II forest cutters and shifting cultivators.
A. Banjaras 1. Chhattisgarh (a) Sri Lanka (b) Bastar
B. Karava 2. Madras Presidency (c) Java (d) Sumatra
C. Santhals 3. Rajasthan
D. Gonds 4. Jharkhand 10. What are Bugyals?
(a) Vast meadows in the high mountains
Codes (b) Waste lands
A B C D A B C D (c) Natural pastures in the low mountains
(a) 3 1 2 4 (b) 3 2 4 1 (d) Deserts
(c) 4 2 1 3 (d) 1 3 2 4
11. ______ is a dry forested area below the foothills of
6. Baigas are a forest community of ______ India. Garhwal and Kumaun.
(a) North (b) South (a) Bhabar (b) Kafila
(c) Central (d) South-West (c) Bugyal (d) None of these
582 Study Package for NTSE

12. Who are nomads? 25. In the early nineteenth century, ______ and ______
(a) Poor farmers in the valleys were two of the major commercial crops.
(b) Poor farmers in the mountains (a) indigo, opium (b) rice, wheat
(c) People who move from one area to another (c) jute, wheat (d) cotton, jute
(d) Thieves and dacoits
26. Which ruler did not allow the British traders to trade
13. In the deserts of Rajasthan lived the in opium with his country?
(a) Raikas (b) Kurumas (c) Kurubas (d) Gollas (a) Rulers of Afghanistan (b) Rulers of Saudi Arabia
14. The autumn crop, usually harvested between (c) Rulers of China (d) Rulers of Japan
September and October is known as 27. Who had introduced opium into China in the early
(a) Rabi (b) Kharif 16th century?
(c) Stubble (d) None of these (a) The Spanish (b) The Portuguese
15. By ______ the government began collecting taxes (c) The British (d) The Americans
directly from the pastoralists. 28. Who was the inventor of the first mechanical reaper
(a) 1880s (b) 1790s (c) 1920s (d) 1650s in USA?
(a) Cyrus Mc Cormick (b) James Watt
16. Which one of the following factors affect the seasonal
(c) John Kay (d) None of the above
movements of the Dhangars of Maharashtra?
(a) Cold and snow 29. Between 1750 and 1850, ______ million acres of land
(b) Alternate monsoon and dry seasons was enclosed in Britain.
(c) Climatic disturbance (a) 5 (b) 6 (c) 8 (d) 10
(d) Drought and flood
30. By the early ______ century white Americans began
17. Even today, over ______ million Africans depend on to move Westward.
some form of pastoral activity for their livelihood. (a) 16th (b) 17th (c) 18th (d) 20th
(a) 40 (b) 22 (c) 25 (d) 30

18. Before colonial times Massailand stretched over a Direction (Q. No. 31) In the question given below are two
statements labelled as Assertion (A) and Reason (R). In the context
vast area from North Kenya to the steppes of
at the two statements which one of the following is correct?
Northern
(a) Brazil (b) Tanzania (c) Somalia (d) Ethopia 31. Assertion (A) The cultivators were unwilling to turn
19. Name the people who are still great herders of goats their fields over to poppy.
and sheep. Reason (R) The crop had to be grown on the best land
(a) The Gaddi shepherds of Himachal Pradesh where peasants usually produced pulses.
(b) The Gujjar Bakarwals of Jammu and Kashmir
(c) The Bhotiyas Codes
(d) The Kinnauris (a) A is true but B is false (b) A is false but B is true
(c) B is the reason of A (d) B is not the reason of A
20. Why was the land enclosed in the 16th century?
(a) To build new factories 32. The countries known as the ‘central power’ in Europe
(b) To build new houses were
(c) To provide more area for sheep farming (a) Germany, Austria-Hungary, Ottoman-Turkey
(d) To stock foodgrains (b) Germany, Russia, France
(c) Russia, Germany, Britain
21. Why the enclosure of land was opposed by the poor (d) None of the above
people?
(a) It took away their job 33. Nearly 50 million people emigrated from Europe to
(b) It deprived them of their common lands America and ______ in the 19th century.
(c) It destroyed their homes and agricultural land (a) Africa (b) Asia (c) Australia (d) Oceania
(d) It formed them to migrate to urban areas
34. Who is the author of ‘Man of Destiny’ (1898)?
22. From the late ______ century, there was a dramatic (a) Charles Dickens (b) Thomas Hardy
expansion of wheat production in the USA. (c) George Bernard Shaw (d) William Shakespeare
(a) 16th (b) 17th (c) 18th (d) 19th
35. ______ stimulated new investments and
23. The Great Agrarian Depression of the ______ ruined improvements in transport.
wheat farmers everywhere. (a) Capitalism (b) Socialism
(a) 1930s (b) 1940s (c) 1920s (d) 1950s (c) Colonisation (d) Imperialism

24. What is meant by Black Blizzards? 36. Rinderpest disease killed ______ % of the cattle of
(a) High rising sea weaves (b) Terrifying dust storms Africa.
(c) Devastating earthquakes (d) Devastating floods (a) 50 (b) 70 (c) 80 (d) 90
Livelihoods, Economics and Societies 583
37. Upto the 18th century, which two countries among 42. ‘Chutney music’ is popular in
the following were the richest countries of the world? (a) Trinidad and Guyana (b) Peru
(a) India, China (b) China, Japan (c) New Zealand (d) Brazil
(c) Japan, Russia (d) Russia, France
43. Match the following
38. Great Britain had a ‘trade surplus’ with
List I List II
(a) India (b) China (c) Japan (d) Russia
A. Sicily 1. East Africa
39. Which were the two most dynamic industries in
B. Eritrea 2. Britain
Britain?
(a) Cotton and metal industries C. Surinam 3. Italy
(b) Silk and cotton industries D. Leeds 4. Caribbean Islands
(c) Electronic and metal industries
(d) None of the above Codes
A B C D A B C D
40. Who is the author of ‘Dombey and Son’ ? (a) 3 1 4 2 (b) 3 4 2 1
(a) Charles Dickens (b) Leo Tolstoy (c) 2 3 4 1 (d) 3 2 1 4
(c) Victor Hugs (d) Thomas Hardy
44. Match the following
41. Match the following
List I List II
List I List II
A. Aeroplane 1. James Watt
A. Dadasaheb Phalke 1. Actor
B. Spinny Jenny 2. Orville and Wilbur Wright
B. Saadat Hasan Manto 2. Music Director
C. Steam Engine 3. Graham Bell
C. Sachin Dev Burman 3. Film Director
D. Telephone 4. James Hargreaves
D. Prithvi Raj Kapoor 4. Writer

Codes Codes
A B C D A B C D A B C D A B C D
(a) 2 4 1 3 (b) 4 3 2 1 (a) 2 3 1 4 (b) 2 4 1 3
(c) 3 4 2 1 (d) 3 2 1 4 (c) 3 4 1 2 (d) 4 3 2 1

Answers
Elementary Level
1. (a) 2. (b) 3. (a) 4. (c) 5. (c) 6. (c) 7. (c) 8. (d) 9. (a) 10. (a)
11. (d) 12. (d) 13. (c) 14. (c) 15. (c) 16. (c) 17. (a) 18. (c) 19. (b) 20. (a)
21. (c) 22. (c) 23. (a) 24. (c) 25. (c) 26. (c) 27. (b) 28. (d) 29. (a) 30. (d)
31. (b) 32. (c) 33. (b) 34. (d) 35. (d) 36. (b) 37. (a) 38. (d) 39. (b) 40. (c)
41. (b) 42. (d) 43. (b) 44. (c) 45. (c) 46. (a) 47. (a) 48. (a) 49. (b) 50. (a)

High Skill Questions


1. (a) 2. (b) 3. (b) 4. (c) 5. (b) 6. (c) 7. (c) 8. (a) 9. (c) 10. (a)
11. (a) 12. (c) 13. (a) 14. (b) 15. (a) 16. (b) 17. (b) 18. (b) 19. (b) 20. (c)
21. (b) 22. (d) 23. (a) 24. (b) 25. (a) 26. (c) 27. (b) 28. (a) 29. (b) 30. (d)
31. (c) 32. (a) 33. (c) 34. (c) 35. (c) 36. (d) 37. (a) 38. (a) 39. (a) 40. (a)
41. (c) 42. (a) 43. (a) 44. (b)
Chapter

5
Everyday Life, Culture
and Politics
The Story of Cricket l Till the middle of the 18th century, bats were roughly
the same shape as hockey sticks, curving outwards at
History is not just about the dramatic events in the the bottom. There was a simple reason for this, the ball
world. It is equally about the small things in our lives. was bowled underarm, along the ground and the curve
Everything around us has a history-the clothes we at the end of the bat gave the batsman the best chance of
wear, the food we eat, the music we hear, the medicines making contact.
we use, the literature we read, the games we play.
Since, we relate to them in our daily lives, their history Laws of Cricket
escapes us. l
The cricket gave itself rules and regulations so that it could be played
in a uniform and standardised way.
At one time, a century and half ago, cricket was an l
The first written ‘Laws of Cricket’ were drawn up in 1744. They stated,
English game. It had been invented in England and the principles shall choose from amongst the gentleman present two
became intricately linked to the culture of 19th century. umpires who shall absolutely decide all disputes.
l
The stumps must be 22 inches high and the bail across them six inches.
Victorian Society The ball must be 5 and 6 ounces and two sets of stumps 22 yards apart.
There was no limit on the shape or size of the bat.
The game was expected to represent all that the England l
The world’s first cricket club was formed in Hambledon in the 1760s
valued fair play, discipline and gentlemanliness. and the Marylebone Cricket Club (MCC) was founded in 1787. In
1788, the MCC published its first revision of the laws and became the
l With the British, cricket spread to the colonies. There guardian of cricket regulations.
again it was supposed to uphold the values of l
The MCC revision of the laws brought in a series of changes that
Englishness. occured in the second half of the 18th century. It became common to
l The colonial masters assumed that only they could play pitch the ball through the air, rather than roll it along the ground.
the game as it ought to be played, in its spirit. l
The weight of the ball was limited to between 5 12 to 5 3 4 ounces and
width of the bat to four inches. In 1774, the first Leg-Before law was
l The game of cricket thus got linked up closely with the
published and three days had become the length of a major match.
politics of colonialism and nationalism.

Development of Cricket as a Game in Cricket and Victorian England


England l The organisation of cricket in England reflected the
Cricket grew out of the many stick and ball games played nature of English society. The rich who could afford to
in England 500 years ago, under a variety of different play it for pleasure were called amateurs and the poor
who played it for living were called professionals.
rules. The word ‘Bat’ is an old English word that simply
means stick or club. By the 17th century, cricket had l The social superiority of amateurs was built into the
customs of cricket. Amateurs were called gentlemen
evolved enough to be recognisable as a distinct game and
while professionals had to be content with being
it was popular enough for its fans to be fined for playing it described as players. They even entered the ground from
on Sunday, instead of going to church. different entrances.
Everyday Life, Culture and Politics 585
l Amateurs tended to be batsman, leaving the l Similarly, the small countries of the Caribbean that
energetic, hardworking aspects of the game, like fast together make up the West Indies team were British
bowling to the professionals. That is partly why the Colonies till well after the Second World War.
laws of the game always give the benefit of doubt to the l Decolonisation or the process through which different
batsman. parts of European empires became independent nations,
l Cricket is a batman’s game because its rules were made began with the independence of India in 1947 and
to favour ‘gentleman’. The social superiority of the continued for the next half of century.
amateur was also the reason the captain of a cricket l Even after the independence of colonies, the
team was traditionally a batsman. disappearance of the British empire, the regulation of
l Its often said that the ‘Battle of Waterloo was won on the international cricket remained the business of the
fields of Eton’. This means that Britain’s Military success Imperial Cricket Conference (ICC).
was based on the values taught to school boys in its public
schools. Eton was the most famous of these schools. Commerce, Media and Cricket Today
l Till the last part of the 19th century, sports and vigorous exercise for The 1970s were the decade in which cricket was
girls was not a part of their education. transformed, it was notable for the exclusion of South
l Dorothea Beale, principal of Cheltenham Ladies College from 1858 to Africa from international cricket. In 1971, the first
1906 began acquiring play grounds and allowing girls to play some of
the games earlier considered male preserves. one-day international was played between England and
Australia in Melbourne.
l Kerry Packer, an Australian television tycoon who saw
Cricket Race and Religion the money-making potential of cricket as a televised
Cricket in colonial India was organised on the principle sport signed up fifty one of the world’s leading cricketers
against the wishes of the National Cricket Boards and for
of race and religion. The first record of cricket being
about two years staged unofficial tests and one-day
played in India is ‘1721’, an account of recreational internationals under the name of World Series Cricket.
cricket played by English sailors in Cambay. The first
l Kerry Packer made cricket more attractive to television
Indian club, the Calcutta Cricket Club was established audiences which endured and changed the nature of the
in 1792. game.
l The origins of Indian cricket, that is cricket played by
Indians are to be found in Bombay and the first Indian Mahatma Gandhi and Colonial Sport
community to start playing the game was the small l
Mahatma Gandhi believed that sport was essential for creating a
community of Zoroastrians, the Parsis. balance between the body and mind. However, he often emphasised
l The Oriental Club of Bombay established in 1848, Parsi that games like cricket and hockey were imported into India by the
clubs were funded and sponsored by Parsi businessmen British and were replacing traditional games.
like Tatas and Wadias. l
Such games as cricket, hockey, football and tennis were for the
l The establishment of the Parsi gymkhana became privileged.
precedent for other Indians who in turn established l
They showed a colonial mindset and were a less effective education
clubs based on the idea of religious community. than the simple exercise of those who worked on the land.
l By 1890, Hindus and Muslims were busy gathering
funds and support for Hindu gymkhana and an Islam l Television channels made money by selling television
gymkhana. spots to companies and continuous television coverage
l The National Cricket Championship (later named the made cricketers celebrities, who made even larger sums
Ranji Trophy) was established but not until of money by making commercials for a wide range of
independence did it properly replace the Pentangular. products, from tyres to colas, on television.
l Television coverage changed cricket, it expanded the
The Modern Transformation of the Game audience for the game by beaming cricket into small
towns and villages.
Modern cricket is dominated by tests and one day
internationals, played between national teams. India l The technology of satellite television and the worldwide
reach of multi-national television companies created a
entered the world of test cricket in 1932, a decade and
global market for cricket. The game’s centre of gravity
half before it became an independent nation. shifted to South Asia. This shift was symbolised by
l The first test was played between England and shifting of the ICC headquarters from London to
Australia, when Australia was still a white settler tax-free Dubai.
colony, not even a self-governing dominion.
586 Study Package for NTSE

Hockey, India’s National Game l From childhood, girls were tightly laced up and dressed
in stays. The effort was to restrict the growth of their
Modern Hockey evolved from traditional games once bodies, contain them within small moulds.
current in Britain. Amongst its sporting ancestors,
l When slightly older, girls had to wear tight fitting
hockey can count the Scottish game called Shinty, the corsets. Tightly laced, small-waisted women were
English and Welsh game called Bandy and Irish Hurling. admired as attractive, elegant and graceful. Clothing,
l Hockey, like many other modern games, was thus played a part in creating the image of frail,
introduced into India by the British army in colonial submissive Victorian women.
times. The first hockey club in India was started in l Over the 19th century, ideas changed. As the suffarage
Calcutta in 1885-86. movement developed, many women’s began agitating
for democratic rights and compaigning for dress reform.
l India was represented in hockey competition of the
Olympic Games for the first time in 1928. In the finals, l Women’s magazines described how tight dress and
corsets caused deformities and illness among young
India defeated Holland by three goals to nil.
girls, such clothing restricted body growth and
l The brilliance and skill of players like great Dhyan hampered blood circulation.
Chand bought India a string of Olympic Gold medals,
between 1928 and 1956. Important Terms
l The two other Gold medals for India came in the 1964 l Stays Support as part of women’s dress to hold the body straight.
Tokyo Olympics and the 1980 Moscow Olympics. l Corset A closely fitting and stiff inner bodice, worn by women to
give shape and support to the figure.
Suffarage The right to vote. The suffragettes wanted the right for
Clothing : A Social History l

women to vote.
All societies observe certain rules, some of them quite
strict, about the way in which men, women and children l In America, a similar movement developed amongst the
should dress or how different social classes and groups white settlers on the East coast. Traditional feminine
clothes were criticised on a variety of grounds.
should present themselves.
l Long skirts swept the grounds and collected filth and
l These norms define the identity of people. They shape dirt, which caused illness and were voluminous and
notions of grace and beauty, ideas of modesty and difficult to handle.
shame.
l As times change and societies are transformed, these New Values
notions also alter. Modifications in clothing come to Many changes were made possible in Britain due to the
reflect these changes. introduction of new materials and technologies.
l Before the age of democratic revolutions and
Other changes came about because of the two World
development of capitalist markets in 18th century
Wars and the new working conditions for women.
Europe, most people dressed according to their regional
codes and cost of materials that were available in their
New Materials
region. Clothing styles were also strictly regulated by
class, gender or status in the social hierarchy. Before the 17th century, most ordinary women in
l The colonisation of most of the world by Europe, after Britain possessed very few clothes made of flex, linen or
the 18th century, the spread of democratic ideals and wool, which were difficult to clean.
the growth of an industrial society, completely changed During industrial revolution, Britain began the mass
the ways in which people thought about dress and its manufacture of cotton textiles exported to many parts of
meaning and Western dress styles for men were adopted the world. Cotton clothes became more accessible to a
worldwide.
wider section of people in Europe.
Clothing and Notion of Beauty The War
The French Revolution had raised the question of Many European women stopped wearing jewellery and
equality and ended aristocratic privileges, as well as the luxurious clothes. As upper class women mixed with
laws that maintained these privileges. However, other classes, social barriers were eroded and women
differences between social strata remained. The notion began to dress in similar ways.
of what was beautiful or ugly, proper or improper, l During the World War women in Britain were
decent or vulgar, differed. employed in ammunition factories. They wore a
l Style of clothing also emphasised differences between working uniform, bright colours faded from sight and
men and women. only sober colours were worn as the war dragged on.
Everyday Life, Culture and Politics 587
Most important, women took to cutting their hair short Christian or Hindu to wear a jacket or cover their upper
for convenience. bodies, but not like the women of high caste.
l By the 20th century, a plain and austere style came to
reflect seriousness and professionalism. Gymnastics British Rule and Dress Codes
and games entered the school curriculum for women.
l In different cultures, specific items of clothing often
They had to wear clothes that did not hamper
convey contrary meanings. This frequently leads to
movement. They needed clothes that were comfortable
and convenient. misunderstanding and conflict.
l Another such conflict related to the wearing of shoes.
Earlier British officials to follow Indian Etiquette and
Transformations in Colonial India remove their footwear in the courts of ruling kings or
During the colonial period there were significant chiefs.
changes in male and female clothing in India. On the l Lord Dalhouise, made it mandatory that Indians were
one hand this was a consequence of the influence of made to take off their shoes when entering any
government institution, only those who wore European
Western dress and missionary activity and on the other
clothes were exempted from this rule.
it was due to the effort by Indians to fashion clothing
l It took many years before shoes were permitted into the
styles that embodied an indigenous tradition and
courtroom.
culture.
Indians reacts to Western style clothing in three different The National Dress
ways l As nationalist feelings swept across India by the late
l Especially men, began incorporating some elements of 19th century, Indians began devising cultural symbols
Western style clothing in their dress. The wealthy that would express the unity of the nation.
Parsis of Western India were among the first to adapt l Artists looked for a national style of art. Poets wrote
Western-style clothing. Baggy trousers and phenta (or national songs. Then, a debate began over the design of
hat) added to long collarless coats with boots and a the National Flag.
walking stick.
Western-style clothing was also especially attractive to l Rabindranath Tagore, suggested that instead of
groups of Dalit converts to Christianity who now found combining Indian and European dress, India’s national
it liberating. dress should combine elements of Hindu and Muslim
dress. Thus, the Chapkan (A long buttoned coat) was
l There were others who were convinced that Western
culture would lead to a loss of traditional culture considered the most suitable dress for men.
identity. l There were also attempts to develop a dress style that
l Some men resolved this dilemma by wearing Western would draw on the traditions of different regions.
clothes without giving up their Indian ones. Many l Jnanadanandini Devi, wife of Satyendranath Tagore
Bengali bureaucrats began stocking Western style adopted the Parsi style of wearing the sari. However,
clothes for work outside the home and changed into these attempts at devising a Pan-Indian style did not
more comfortable Indian clothes at home. fully succeed. Women of Gujarat, Kodagu, Kerala and
Asam continue to wear different types of sari.
Caste Conflict and Dress Change
l Though there were no formal Sumptuary Laws as in The Swadeshi Movement
Europe, India had its own strict social codes of food and l The British first came to trade in Indian textiles that
dress. were in great demand all over the world.
l The caste system clearly defind what subordinate and l The industrial revolution in Britain which mechanised
dominant caste Hindus should wear, eat etc., and these spinning and weaving and greatly increased the demand
codes had the force of law. for raw materials such as cotton and indigo, changed
l The Shanars (also called Nadars) were a community of India’s status in the world economy.
toddy tappers who migrated to Southern Travancore to l Indian peasants forced to grow crops such as indigo and
work under Nair landlords. cheap British manufacture easily replaced coarser
l The Government of Travancore issued a proclamation in Indian one.
1829 ordering Shanars women to abstain in future from l Large number of Indian weavers and spinners were left
covering the upper parts of their body. without work and important textile weaving centres
l The abolition of slavery in Travancore in 1855 led to such as Murshidabad, Machilipatnam and Surat
declined as demand fell.
even more frustration among the upper castes who felt
they were losing control. In 1859 another Government l In 1905, Lord Curzon decided to partition Bengal to
proclaimation permitted shanar women, whether control the growing opposition to British rule.
588 Study Package for NTSE

l The Swadeshi Movement developed in reaction to this Print in Europe


measure.
For centuries, silk and spices from China flowed into
l People were urged to boycott British goods of all kinds Europe through the silk route. In the 11th century,
and start their own industries for the manufacture of
Chinese paper reached Europe via same route. In 1295,
goods such as match boxes and cigarettes.
Marco Polo, a great explorer, returned to Italy after
l The use of khadi became the patriotic symbol.
many years of exploration in China. He brought the
l The change of dress appealed largely to the upper castes technology of woodblock printing back with him.
and classes rather than to those who had to make do
with less and could not afford the new product. l Italians began producing books with woodblocks, luxury
editions were still handwritten on very expensive
l Despite its limitation, the experiment with swadeshi vellum, meant for aristocratic circles.
gave Mahatma Gandhi important ideas about using
cloth as a symbolic weapon against British Rule. l The Olive press provided the model for printing press
and moulds were used for casting the metal types for the
letters of the alphabet.
Print Culture and the Modern l By 1448, Gutenberg perfected the system. The first book
World he printed was Bible. About 180 copies were printed and
it took three years to produce them.
History l In fact, printed books at first closely resembled the written
manuscripts in appearance and layout. The metal letters
The earliest kind of print technology was developed in
imitated the ornamental handwritten styles.
China, Japan and Korea. In China, books were printed by
l By 1550, printing presses were set up in most countries
rubbing paper against the inked surface of woodblocks.
of Europe. Printers from Germany travelled to other
l The traditional Chinese Book Accordion Book was countries, seeking work and helping start new presses.
folded and stitched at the side. Superbly skilled
l The shift from hand printing to mechanical printing led
craftsman could duplicate, with remarkable accuracy,
to the print revolution.
the beauty of calligraphy.
l By the 17th century, as urban culture bloomed in China, Print Revolution and its Impact
the uses of print diversified. Print was no longer used
just by scholars, officials. Merchants used print in their Print revolution was not just a development, a new way
everyday life. of producing books, it transformed the lives of people,
l Reading increasingly became a leisure activity. The new changing their relationship to information and
readership preferred fictional, narratives, poetry, knowledge and with institutions and authorities, it
auto-biographies and romantic plays. Shanghai became influenced population perceptions and opened up new
the hub of the new print culture. ways of looking at things.
Print in Japan A New Reading Public
l Buddhist Missionaries from China introduced l With the printing press, a new reading public emerged,
hand-printing technology in Japan around AD 768-770. printing reduced the cost of books. Time and labour
The oldest Japanese book printed in AD 868, is the required to produce each book came down. Books flooded
Buddhist. ‘Diamond sutra’ containing six sheets of text the market, reaching out to an ever-growing readership.
and woodcut illustrations.
l Access to books created a new culture of reading. Earlier
l Pictures were printed on textiles, playing cards, and reading was restricted to the elites, common people lived
paper money. in a world of oral culture.
l In medieval Japan, poets and prose writers were l But the transition was not so simple, books could be read
regularly published and books were cheap and only by the literate and the rate of literacy in most
abundant. European countries were very low till the 20th century.
l Printing of visual material led to interesting publishing l Printers began publishing popular ballads and folk tales
practices. In the late 18th century, in the flourishing and such books profusely illustrated with pictures.
urban circles at Edo (later to be known as Tokyo)
illustrated collections of paintings depicted an elegant Print and Dissent
urban culture, involving artists and tea house
gatherings. l Print and popular religious literature stimulated many
distinctive individual interpretations of faith even
l Libraries and book shops were packed with hand printed
among little-educated working people.
material of various types—books on women, musical
instruments, calculations, flower arrangements, cooking l In the 17th century, Menocchio, a miller in Italy, began
and famous places. to read books that were available in his locality.
Everyday Life, Culture and Politics 589
l He reinterpreted the message of the Bible and Print : The French Revolution
formulated a view of God and creation that enraged the
l Many Historians have argued that print culture created
Roman Catholic church.
the conditions within which French Revolution occured.
l When the Roman church began its inquisition to repress
l Print popularised the ideas of the enlightenment
heretical ideas. Menocchio was hauled up twice and thinkers collectively. Their writings produced a critical
ultimately executed. evaluation on tradition, superstition and depotism.
l Roman church, troubled by such effects of popular They argued for the rule of reason rather than custom
readings and questionings of faith imposed severe and every thing be judged through the application of
controls over publishers and booksellers and began to reason and rationality.
maintain an index of prohibited books. l Print created a new culture of dialogue and debate. All
values, norms and institutions were re-evaluated.
l Inquisition A former Roman Catholic court for identifying and l By the 1780s, there was an outpouring of literature that
punishing heretics. mocked the royality and criticised their morality. In the
l Heretical Beliefs which do not follow the accepted teachings of the process, they raised questions about the existing social
church.
order.

Reading-Changes Trends Print Culture made Changes in Society


l As primary education became compulsory from the late
In the 17th and 18th centuries literacy rate went up in
19th century, children became an important category of
most parts of Europe. As literacy and schools spread in readers. Anything that was considered unsuitable for
European countries, there was a vital reading mania. children or would appear vulgar to the elites, was not
People wanted books to read and printers produced included in the published version.
books in ever increasing numbers. l Women became important as readers as well as writers,
New forms of popular literature appeared in print, penny magazines were especially meant for women were
mannals teaching proper behaviour and house keeping.
targeting new audiences. There were almanacs or ritual
l Some of the best known novelists were women Jane
calenders, along with ballads and folktales and penny
Austen, the Bronte Sisters Anne, Emily and Charlotte
chapbooks were carried by petty pedlars known as Bronle George Eliot (Mary Anne Evans). Their writings
chapman and sold for a penny, so that even the poor became important in defining a new type of women –A
could buy them. person with will, strength of personality, determination
l In France, the Biliotheque ‘Blue’ which were low priced and power to think.
books printed on poor quality paper and bound in cheap l Libraries in England became instruments for educating
blue covers. white collar workers, artisans and lower middle class
people.
l Newspapers and journals carried information about
wars and trade as well as news of developments in other l By mid-nineteenth century Richard M Hoe of New York
places. had perfected the power driven cylindrical press.

l The ideas of scientists and philosophers now became l Printers and publishers continuously developed new
strategies to sell their product. Periodicals serialised
more accessible to the common people.
important novels, which gave birth to a particular way
l The discoveries of ISAAC Newton, the writings of of writing novels.
thinkers such as Thomas Paine, Voltaire and Jean
Jacques Rousseau were also widely printed and read. India and the World of Print
l Thus, there ideas about science, reason and nationality
India had a very rich and old tradition of handwritten
found their way into popular literature.
manuscripts in Sanskrit, Arabic, Persian as well as in
l By the end of 18th century, there was a common various vernacular languages, copied on palm leaves or
conviction that books were a means of spreading on handmade paper. Manuscripts were highly
progress and enlightenment.
expensive and fragile. They had to handle carefully and
l Louise-Sebastien Mercier, a novelist in 18th century were not widely used in everyday life.
France declared ‘‘The printing press is the most
The printing press first came to Goa with the
powerful engine of progress and public opinion is the
force that will sweep despotism away’’.
Portuguese missionaries in the mid-sixteenth century.
The Jesuit priests learnt Knokani and printed several
Despotism A system of government in which absolute books. The Catholic priests printed the first Tamil book
power is exercised by an individual, unregulated by legal
and constitutional checks. in 1579 and in 1713, the first Malyalam book was
printed.
590 Study Package for NTSE

l In 1780, James Augustus Hickey began to edit the l In 1836, a notable event took place when Charles
Bengal Gazette, a weekly magazine that described itself Dicken’s ‘Pickwik Papers’ was serialised in a magazine.
as a commercial paper open to all but influenced by Magazines were attractive, since they were illustrated
none. and cheap.
l There were Indians too, who began to publish Indian l Serialisation allowed readers to relish the suspense.
Newspapers. The first to appear was the weekly Bengal
Gazette, brought out by Gangadhar Bhattarcharya who The World of the Novel
was close to Raja Rammohan Roy.
l In the 19th century, the everyday life of people was the
l The 19th century, was a time of intense controversies central theme of novel writers.
between social and religious reformers and the Hindu
l The growth of industry was accompanied by an economic
orthodoxy over matters like widow immolation,
philosophy which celebrated the pursuit of profit and
monotheism, Brahmanical priesthood and idolatry.
undervalued the lives of workers.
l Raja Rammohan Roy published the Sambad Kaumudi
l Deeply critical of these developments novelists such as
from 1821 and Hindu orthodoxy commissioned the
‘Charles Dickens’ wrote about the terrible effects of
‘Samachar Chandrika’ to oppose his opinion.
industrialisation on people’s lives and characters in his
l From 1822, two Persian newspapers were published, novel ‘Hard Times.’ In other novel too, Dickens focused
‘Jam-I-Jahannama’ and ‘Shamsul Akhbar’. In the same on the terrible conditions of urban life under industrial
year a Gujarati newspaper, the ‘Bombay Samachar’ capitalism. His ‘Oliver Twist’ (1838) is the tale of a poor
made its appearance. orphan who lived in a world of petty criminals and
l The Deoband seminary, founded in 1867, published beggars. Emile Zola’s ‘Germinal’ (1885) on the life of a
thousands of Fatwas telling Muslim readers how to young miner in France explores in harsh detail the grim
conduct themselves in their everyday life and explaining conditions of miner’s lives.
the meanings of Islamic doctrines. l 19th century British Novelist ‘Thomas Hardy’ wrote
l The first printed edition of the Ramcharitmanas of about traditional rural communities of England that
Tulsidas, a 16th century text, came out from Calcutta in were fast vanishing. ‘Mayor of Caster Bridge (1886) is
1810. about a successful grain merchant, who became the
l Other new literary forms also entered the world of mayor of the farming town of Caster Bridge.
reading lyrics, short stories, essays about social and l The novel uses the vernacular, the language that is
political matters. spoken by common people. By coming closer to the
l By the end of 19th century, a new visual culture was different spoken languages of the people, the novel
taking shape. Painters like Raja Ravi Verma produced produces the sense of a shared world between diverse
images for mass circulation. people in a nation.
l By 1870s, caricatures and cartoons were being l Like the nation, the novel brings together many cultures.
published in journals and newspapers.
Novel : The New Women
Novel, Society and History The most exiciting element of the novel was the
involvement of women. 18th century saw women get more
The novel is modern form of literature. It is born from leisure to read as well as write. Novels began to exploring
print, a mechanical invention. Novels produced a the world of women–their emotions and identities, their
number of common interests among their scattered and experiences and problems. Many novels were about
varied readers. As readers were drawn into the story domestic life. They drew upon their experience, wrote
and identified with the lives of fictitious characters, about family life and earned public recognition.
they could think about issues such as the relationship l Amar Jiban written by Rashsundari Debi (published in
between love and marriage, the proper conduct for men 1876) was the first full-lenght autobiography published
and women and so on. in the Bengali language. In Maharashtra, Tarabai
Shimla and Pandit Rambai wrote about the miserable
l The novel first took firm root in England and France. lives of upper caste Hindu women.
Novels began to be written from the 17th century. But
l The Jane Austen’s ‘Pride and Prejudice’ gives us a
they really flowered from the 18th century.
glimpse of the world of women in genteel rural society in
l New groups of lower-middle class people such as early 19th century Britain.
shopkeepers and clerks, along with the traditional
l But women novelists did not simply popularise the
aristocratic and gentlemanly classes in England and domestic role of women. Often their novels dealt with
France now formed the new readership for novels. women who broke established norms of society before
l In France, publishers found that they could make super adjusting to them.
profits by hiring out novels by the hour. l Charlotte Bronte’s ‘Jane Eyre’ Published in 1847.
Everyday Life, Culture and Politics 591
Novels for the Young l Srinivas Das’s novel published in 1882 was titled
l Novels for young boys idealised a new type of man, ‘Pariksha-Guru’ reflects the inner and outer world of
powerful, assertive, independent and daring. Most of newly emerging middle classes.
these novels were full of adventure set in places remote l The writings of Devaki Nandan Khatri created a
from Europe. novel-reading public in Hindi. His best-seller
l Novels like RL Stevenson’s ‘Treasure Island 1883’. ‘Chandrakanta’ a romance with dazzling element of
Rudyard Kipling's ‘Jungle Book’ (1894) became very fantasy is believed to have a contributing immensely in
famous. popularising the Hindi language.
l Love stories written for adolescent girls also became l It was with the writing of Premchand that the Hindi
popular in this period. Especially in the USA notably novel achieved excellence. He began writing in Urdu
‘Ramona’ by Helen Hunt Jackson, ‘What Katy Did’ by and then shifted to Hindi. Many critics think that his
Sarah Chauncey Woolsey who wrote under the novel ‘Sewasadan’ published in 1916, lifted the Hindi
pen-name Susan Coolidge. novel from the realm of fantasy, moralising and simple
Novels in Indian Context entertainment to a serious reflection on the lives of
ordinary people and social issues.
The modern novel form developed in India in the
19th century as Indians became familiar with the Novels in Bengal
Western novel. The development of the vernaculars,
In the 19th century, the early Bengali novels lived in
print and a reading public helped in the process.
two worlds, many of them were based on past, their
l The earliest Indian novels were written in Bengali and
character’s, events and love stories based on historical
Marathi. The earliest novel in Marathi was Baba
Padmajis ‘Yamuna Paryatan (1857) followed by events. Another group of novels depicted the inner
Lakshman Moreshwar Halbe’s ‘Muktamala’ (1861). world of domestic life in contemporary settings.
l The delightful novel called ‘Indulekha’ published in l Bankim Chandra Chattopadhyay’s ‘Durgeshnandini’
1889 was the first modern novel in Malayalam. (1865) is the first novel written in Bengali.
l Kandukuri Viresalingam began translating Oliver l In Bengal, many historical novels were about Marathas
Goldsmith’s ‘‘Vicar of Wake Field’’ into Telugu. He and Rajputs. These novels produced a sense of a
abandoned this plan and instead wrote an original Pan-India belonging. They imagined the nation to be full
Telugu novel called ‘Rajasekhara Caritamu’ in 1878. of adventure, heroism, romance and sacrifice.
l Bhudeb Mukhopadhya’s ‘Anguriya Binimay’ was the
Novels in Hindi first historical novel written in Bengal.
In the North, Bharatendu Harishchandra, the pioneer l Bankim Chandra Chattopadhyas ‘Anandamath’ (1882),
Rabindranath Tagore’s ‘Gora’ (1910), Surat Chandra
of modern Hindi literature, encouraged many members
Chattopadhyay’s ‘Pather Dabi’ (1926) are same of the
of his circle of poets and writers to recreate and marker pieces in Bengal, which inspired the freedom
translate novels from other languages. fighters.

Target Exercise
Elementary Level
1. The word ‘Bat’ is an old English word that simply 3. The game of cricket was invented in
means (a) England (b) Australia
(a) stick (b) club (c) South-Eastern England (d) Caribbean
(c) Both ‘a’ and ‘b’ (d) roud
4. The length of the pitch for cricket game is specified
2. Till the middle of the 18th century bats was roughly as
the same shape as hockey sticks, curving outwards at (a) 22 yards (b) 22 meters
the bottom because (c) 24 yards (d) None of these
(a) it is easier to play with curved bat
5. Initially the weight of the cricket ball was specific as
(b) the flat bat was not invented
(a) 10 ounces (b) 5 ounces
(c) the ball was bowled underarm along the ground
(d) None of the above (c) 6 ounces (d) Both ‘b’ and ‘c’
592 Study Package for NTSE

6. The person who absolutely decides all disputes in the 16. The game of cricket after becoming international
game of cricket known as game, spread to which of the following countries?
(a) refree (b) umpire (a) Only in countries that the British conquered and ruled
(c) coach (d) None of these (b) To the maximum countries of the world
(c) To the countries of Latin America
7. Which of the following was the world’s first cricket (d) None of the above
club?
(a) Hambledon Cricket Club (b) Marylebone Cricket Club 17. Despite the exclusiveness of white cricket elite in the
(c) Sydney Cricket Club (d) None of these West Indies, the cricket became hugely popular in the
Caribbean because
8. In 1788, the MCC published its first revision of the (a) they find it very easy to play
laws and became the guardian of cricket’s regulations, (b) Caribbeans saw in the game a chance for self-respect and
which of the following statements is/are true about international standing
changes introduced by MCC in the game of cricket? (c) they were trained and guided by the English players
(a) To pitch the ball through, the air rather than roll it along the (d) None of the above
ground 18. Name of the first black player who captained the
(b) Replacement of the curved bat with the straight one
West Indian Team?
(c) Reduced the influence of rough ground and brute force
(a) Garry Sobbers (b) Lawance Gibbs
(d) All of the above
(c) Frank Worrell (d) None of these
9. After the new cricket regulations published by MCC,
the weight of the cricket ball reduced to 19. Cricket in colonial India organised on the principle of
(a) equality (b) sportsmanship
(a) 5 ounces (b) 51/2 ounces
(c) 5 12 to 5 3 4 ounces (d) None of these (c) race and religion (d) None of these

10. Originally, the time-limit for cricket matches was 20. Which of the following communities was the first to
(a) three days start playing cricket in India?
(b) five days (a) Hindus (b) Muslims
(c) six days (c) Indian Christians (d) Zoroastrians, the Parsis
(d) no time limit as to bowl out a side twice
21. Which of the following was the first Indian Cricket
11. The rich who could afford to play cricket for pleasure Club?
were known as (a) Bombay Cricket Club (b) The Oriental Cricket Club
(a) amateurs (b) professionals (c) Calcutta Cricket Club (d) Gymkhana Cricket Club
(c) players (d) None of these
22. In India, the cricket tournament was initially called
12. The poor people who played the game of cricket for the quadrangular, because it was played by four
living were called teams. Which of the following was not a part of it?
(a) amateurs (b) professionals (a) Europeans (b) Parsis
(c) players (d) None of these (c) Muslims (d) Sikhs
13. Which of the following persons consider sports like 23. The tournament became pentangular with the
cricket and rugby as an organised way of teaching addition of fifth team. Which of the following was the
discipline, importance of hierarchy, the skills and fifth team?
leadership qualities that helped to build and run the (a) Hindus (b) Portgueese
British empire? (c) Sikhs (d) The ‘rest’
(a) Thomas Arnold (b) Andrew James 24. Popular and famous cricketer, Vijay Hazare, played
(c) B Mccrone (d) None of these for which of the following team?
14. Till the last of the 19th century sports and vigorous (a) Parsis (b) Hindus
exercise for girls was not a part of their education. (c) Muslims (d) The rest
Which of the following is the exponent of sports for 25. Which of following leaders of Indian National
girls in Britain? Movement condemned the Pentangular Tournament
(a) Dorothea Beale as communally divisive competition?
(b) Thomas Arnold (a) Dadabhai Naoroji (b) Mahatma Gandhi
(c) Both ‘a’ and ‘b’ (c) BG Tilak (d) Subash Chandra Bose
(d) None of the above
26. The famous Ranji Trophy was earlier known as
15. Which of the following games was usually played by (a) National Tournament
girls in Britain at the end of the 19th century? (b) National Cricket Championship
(a) Cricket (b) Hockey (c) National Cricket Tournament
(c) Croquet (d) None of these (d) None of the above
Everyday Life, Culture and Politics 593
27. The International Cricket Council, International 39. The earliest kind of print technology was first
Cricket Regulation Body, earlier known as developed in which of the following countries?
(a) Imperial Cricket Conference (a) China (b) Japan
(b) International Cricket Committee (c) Korea (d) None of these
(c) Imperial Cricket Committee
(d) None of the above 40. Books in China were printed by rubbing paper
against the inked surface of
28. Which of following is non-cricket playing country? (a) metal blocks (b) rubber blocks
(a) South Africa (b) Kenya (c) wood blocks (d) None of these
(c) Zimbabwe (d) None of these
41. In Japan, hand printing technology was introduced
29. Which of the following countries was excluded from by China’s
international cricket for policy of racial segregation? (a) merchants (b) sailors
(a) South Africa (b) West Indies (c) buddhist missionaries (d) None of these
(c) Sri Lanka (d) Zimbabwe
42. Printing of visual material led to interesting
30. The laws which control the behaviour of those publishing practices in the late 15th century. In the
considered social inferiors, preventing them from flourishing urban circles at Edo, later to be known as
wearing certain clothes, consuming certain food (a) Tokyo (b) Hiroshima
known as (c) Nagasaki (d) None of these
(a) Sumptuary laws (b) Hierarchical laws
43. The art of printing on paper brought to Europe by
(c) Aristocratic laws (d) None of these
(a) Gutenberg (b) Marco Polo
31. Which of the following sections of society could wear (c) Voltaire (d) None of these
clothes made of expensive materials like ermine, silk,
44. By 1448, Gutenberg perfected the printing system.
velvet and brocade?
The first book he printed was the
(a) Merchants (b) Middle class
(a) Bible (b) novel
(c) Royalty (d) Common people
(c) book on French Revolution (d) None of these
32. Which of following revolutions abolishes the
45. Which of the following countries is responsible for
Sumptuary Laws?
spreading the printing technology to other European
(a) French Revolution (b) American Revolution
countries?
(c) Russian Revolution (d) All of these
(a) Italy (b) Britain (c) Germany (d) France
33. The material which became more popular in Europe
for clothing, in contrast to the material used, in the 46. Which of the following is/are main content of print
17th century, after the Industrial revolution in Britain books in their early stage?
(a) Popular Ballads (b) Folk Tales
(a) silk (b) cotton (c) fur (d) linen
(c) Both ‘a’ and ‘b’ (d) None of these
34. Which of the following factors transforms the clothing
47. A 16th century movement to reform the catholic
pattern in European countries in the 19th century?
church dominated by Rome known as Protestant
(a) Industrial revolution (b) War
Reformation. The movement is lead by
(c) Christianity (d) None of these
(a) Erasmus (b) Martin Luther
35. Which of the following communities in India were (c) Rousseau (d) Voltaire
among the first to adopt Western-style clothing in 48. The novel is a modern form of literature was the
their dress? result of
(a) Hindus (b) Parsis (c) Muslims (d) None of these (a) Print revolution
36. Which section of Indian Society wearing western (b) French revolution
clothes without giving up their Indian? (c) American War of Independence
(d) None of the above
(a) Political leaders (b) Bengali bureaucrats
(c) Educated Middle class (d) None of these 49. In which of following countries novel was first
flourished?
37. In Travancore women’s of which community were not
(a) England (b) France
allowed to wear a cloth across their upper bodies
(c) Both ‘a’ and ‘b’ (d) None of these
(a) Shanars (b) Nair (c) Christian (d) None of these
50. Which of the following novel of Charles Dicken’s was
38. In traditional Indian clothing pattern, wearing of serialised in a magazine as magazines were
Turban symbolises attractive since they were illustrated and cheap?
(a) protection form the heat (b) respectability (a) Hard Times (b) Pickwick Papers
(c) fashion (d) ceremonial necessity (c) Oliver Twist (d) None of these
594 Study Package for NTSE

High Skills Questions


1. Many important changes occured during the 19th 7. Match the following
century. Which of the following is not a part of these
List I List II
changes?
(a) The rule about the no ball was introduced A. Adelaide 1. Delhi
(b) The exact circumference of the ball was specified B. Chepauk 2. Kolkata
(c) Boundaries were introduced, where previously all shots had C. Eden Garden 3. Chennai
to run D. Feroz Shah Kotla 4. Oval
(d) None of the above
Codes
2. Which of the following statements is/are correct A B C D A B C D
about the game of cricket? (a) 4 3 1 2 (b) 4 2 1 3
(c) 1 2 3 4 (d) 2 1 3 4
(a) One of the peculiarities of test cricket is that a match can go
on for five days and still end in a draw 8. Match the following
(b) The laws simply lay down that the umpire shall agree with
both captains on the boundaries of the playing area List I List II
(c) The cricket remained a colonial game, limited to countries A. Eton 1. Self-governing areas under the
that had once been part of the British Empire control of British crown
(d) All of the above B. Hierarchy 2. First black West Indies captain
C. Frank Worrell 3. Organised by rank and status
3. The wood of ______ tree is used in making of bats.
D. Dominion 4. Famous Military School in England
(a) willow
(b) sandal Codes
(c) teak A B C D A B C D
(d) None of the above (a) 1 2 3 4 (b) 2 1 4 3
(c) 4 3 2 1 (d) 3 4 1 2
4. Match the following
9. Match the following
List I List II
A. Quadrangular 1. Playes for the rest, the fifth team, List I List II
comprised of all communities A. Amateurs 1. Collected financial contribution for
B. Pentangular 2. Editor of newspaper the Bombay specific purpose
Chronicle B. Patronage 2. The poor who played for a living
C. Vijay Hazare 3. Tournament played by five teams C. Professionals 3. Agreement by wealthy supported to
D. S A Brelvi 4. Tournament played by four teams give financial support for a specific
cause
Codes D. Subscription 4. The rich people who could afford to
A B C D play for pleasure
(a) 4 3 1 2
(b) 3 4 2 1 Codes
(c) 1 2 3 4 A B C D A B C D
(d) 2 1 3 4 (a) 1 2 3 4 (b) 2 1 3 4
(c) 3 4 2 1 (d) 4 3 2 1
5. The areas which are self-governing but under the
control of British crown known as 10. The Novel ‘Tom Brown’s Schooldays’, was written by
(a) Thomas Hughes (b) Thomas Arnold
(a) sovereign
(c) Frank Worrell (d) None of these
(b) independent
(c) dominion 11. Which of the following was the first dress reformer in
(d) None of the above Europe?
(a) Mrs Amella Bloomer (b) Mrs Stanton
6. ‘The cricket’s most important tool are all made of
(c) Mary Somerville (d) None of these
natural pre-industrial materials.’ In the light of this
statement which of the following statements is/are 12. Which of the following statements is/are true about
true? suffrage movement developed in Europe in the
(a) The bat is made of wood as are the stumps 19th century?
(b) The ball is made with leather, twine and cork (a) Women in Europe began agitating for democratic rights
(c) The material of bat changed slightly, over time (b) Suffrage movement also calls for dress reforms
(d) All of the above (c) The suffrage movement was headed by Mrs Stanton
(d) All of the above
Everyday Life, Culture and Politics 595
13. Match the following 17. Match the following
List I List II List I List II

A. Chapkan 1. First to adapt Western-style clothing A. Protestant 1. A system of Governance in which


Reformation absolute power is exercised by an
B. Khadi 2. A symbol of patriotism individual
C. Parsis 3. Those belonging to the Brahmo Samaj B. Inquisition 2. A former Roman Catholic court for
D. Brahmo 4. The most suited national dress for men identifying and punishing heretics
C. Heretical 3. Martin Luther king
Codes D. Despotism 4. Beliefs which do not follow the
A B C D accepted teachings of the church
(a) 4 2 1 3
Codes
(b) 4 3 1 2
A B C D A B C D
(c) 3 4 2 1 (a) 3 2 4 1 (b) 2 3 1 4
(d) 1 2 3 4 (c) 1 2 3 4 (d) 2 1 3 4
14. Which of the following statements is/are true about 18. Match the following
clothing as special aspect of social and political life?
(a) All societies observe certain rules about the way in which List I List II
men, women and children should dress A. Ulama 1. First Gujarati Newspaper
(b) Social classes and groups follow certain norms which define B. Fatwa 2. Printing press first came to Goa
the identity of people C. Portuguese 3. Legal scholars of Islam and the
(c) Clothing shape our notions of grace and beauty Missionaries Sharia (a body of Islamic law)
(d) All of the above D. Bombay 4. A legal pronouncement on Islamic
Samachar law usually given by a Mufti (legal
15. Match the following scholar) to clarify issues on which
the law is uncertain
List I List II
A. Ermine 1. The right to vote Codes
A B C D A B C D
B. Blue, white and 2. Cotton cloth printed with designs (a) 4 3 2 1 (b) 1 2 3 4
red colours and flowers
(c) 2 1 4 3 (d) 3 1 2 4
C. Suffrage 3. Sign of the patriotic citizen
D. Chintz 4. Type of fur, which only royalty
19. Which of the following statements is/are correct
could wear about the print/urban culture bloomed in China?
(a) Print was no longer used just by scholars, officials
Codes (b) Merchants used to print their everyday life as they collected
A B C D trade information
(a) 4 3 2 1 (c) The new readership preferred fictional Narratives, poetry,
(b) 4 3 1 2 autobiographies
(c) 1 2 3 4 (d) All of the above
(d) 2 1 4 3
20. Which of the following statements is/are true about
16. Match the following the print revolution in the world?
List I List II (a) The first book Gutenberg printed was the Bible
(b) The time and labour required to produce each book came
A. Calligraphy 1. A term used to describe pocket size down and multiple copies could be produced with greater
books that are sold by travelling pedlars ease
B. Vellum 2. A historical account or folk tale in verse (c) Access to books created a new culture of reading
usually sung or recited (d) All of the above
C. Ballad 3. A parchment made from the skin of
animals 21. Which of the following statements is/are true about
the development of novel in India?
D. Chapbook 4. The art of beautiful and stylised writing
(a) The Indian Novelists wrote to develop a modern literature of
Codes the country that could produce a sense of national belonging
A B C D and cultural equality
(a) 4 3 2 1 (b) Translation of novels into different languages helped to
(b) 4 3 1 2 spread the popularity of the novel
(c) 1 2 3 4 (c) Novels helped in establishing a relationship with the past
(d) 2 1 4 3 (d) All of the above
596 Study Package for NTSE

22. Match the following 30. Who was country’s first test captain?
(a) CK Nayudu (b) Vijay Hazare
List I List II (c) Palwankar Baloo (d) None of these
A. Yamuna Paryatan 1. Rabindranath Tagore
31. Learie Constantine, famous cricketer, halls from
B. Muktamala 2. Prem Chand
which country?
C. Ghare Biare 3. Baba Padmanjiis (a) England (b) South Africa
D. Godan 4. Moreshwar Halbe (c) West Indies (d) Australia
Codes 32. Khadi or wearing of Khadi became a symbol of
A B C D A B C D (a) purity (b) self-reliance
(a) 4 3 2 1 (b) 4 3 1 2 (c) patriotic duty (d) All of these
(c) 4 3 1 2 (d) 1 2 3 4
33. The partition of Bengal was done in 1905 by which of
23. Match the following following Viceroys?
List I List II (a) Lord Curzon (b) Lord Canning
(c) Lord Harding (d) None of these
A. Epistolary 1. Charlotte Bronte
B. Germinal 2. The normal spoken form of a language 34. Which of the following is not an textile weaving
rather than formal literary form centre in colonial India?
C. Vernacular 3. Novel written in the form of series of letters (a) Murshidabad (b) Machilipatnam
D. Jane Eyre 4. Emile Zola (c) Surat (d) None of these
Codes 35. Rabindranath Tagore advocated for India’s dress
A B C D should be combination of which two elements?
(a) 3 1 2 4 (a) Simplicity, self-reliance
(b) 3 4 2 1 (b) Indian and European dress
(c) 1 2 3 4 (c) Hindu and Muslim dress
(d) 2 1 3 4 (d) None of the above
24. Which of the following is/are features of the novel 36. As a national dress, which of the following was
written by Prem Chand? considered the most suitable dress for men?
(a) He depicted the poor condition of women in society (a) Turban (b) Chapkan
(b) Social issues like child marriage and dowry are woven into (c) Dhoti (d) None of these
the novel
(c) Opportunities provided by colonial authorities to upper 37. Which of the following factors played a part in creating
classes to govern themselves the image of frail, submissive victorian women?
(d) All of the above (a) Education (b) Clothing
(c) Right to vote (d) None of these
25. Which of the following factor serves as theme for
novel writing in Bangla in the 19th century? 38. Which of the following is/are part of the popular
(a) Stories based on historical events literature printed in the 18th century?
(b) The inner world of domestic life in contemporary society (a) Almanacs (b) Ballads and Folk Tales
(c) Romantic relationships between men and women (c) Chapbooks (d) All of these
(d) All of the above
39. Many historians have argued that print culture
26. The first one-day international match was played created the conditions, the application of reason and
between Australia-England in 1917 at
rationality, within which ______ revolution occurred.
(a) Lords (b) Sydney
(a) French (b) Industrial
(c) Melbourne (d) None of these
(c) American (d) None of these
27. Which of the following person is responsible for
40. The printing press first came to India in the
transforming cricket into marketable game?
mid-sixteenth century with
(a) Kerry Packer (b) David Richardson
(a) Chinese travellers
(c) Bill Gates (d) None of these
(b) Portuguese missionaries
28. The headquarter of ICC shifted from London to (c) British merchants
(a) Sri Lanka (b) South Africa (d) French merchants
(c) Dubai (d) None of these
41. The first English weekly magazine that described
29. India played their first test against England in the itself as ‘A commercial paper open to all, but
year influenced by none’ published in India was
(a) 1930 (b) 1931 (a) Bengal Gazette (b) National Herald
(c) 1932 (d) None of these (c) Kesari (d) None of these
Everyday Life, Culture and Politics 597
42. Which of the following act put censorship on Indian 44. Which of the following is known as the Pioneer of
press in the 19th century? Modern Hindi literature?
(a) Rawlatt act (a) Bharatendu Harishchandra (b) Devki Nandan Khatri
(b) Vernacular act (c) Prem Chand (d) None of these
(c) Citizen’s charter
45. The best seller Hindi Novel ‘Chandrakanta — A
(d) None of the above
romance with dazzling elements of fantasy, is written
43. The Deoband Seminary, founded in 1567, published by?
thousands of prouncement telling Muslims readers (a) Bharatendu Harishchandra
how to conduct themselves in their daily life and (b) Devaki Nandan Khatri
explaining the meanings of Islamic doctrines’ known as (c) Prem Chand
(a) Fatwas (b) Ulema (d) None of the above
(c) Shariat (d) None of these

Answers
Elementary Level
1. (c) 2. (c) 3. (c) 4. (a) 5. (d) 6. (b) 7. (a) 8. (d) 9. (c) 10. (d)
11. (a) 12. (b) 13. (a) 14. (a) 15. (c) 16. (a) 17. (b) 18. (c) 19. (c) 20. (d)
21. (b) 22. (d) 23. (d) 24. (d) 25. (b) 26. (b) 27. (a) 28. (d) 29. (a) 30. (a)
31. (c) 32. (a) 33. (b) 34. (a) 35. (b) 36. (b) 37. (a) 38. (b) 39. (a) 40. (c)
41. (c) 42. (a) 43. (b) 44. (a) 45. (c) 46. (c) 47. (b) 48. (a) 49. (c) 50. (b)

High Skill Questions


1. (d ) 2. (d ) 3. (a ) 4. (a ) 5. ( c) 6. (d ) 7. ( c) 8. ( c) 9. (d ) 10. (a )
11. (a ) 12. (d ) 13. (a ) 14. (d ) 15. (b) 16. (a ) 17. (a ) 18. (a ) 19. (d ) 20. (d )
21. (d ) 22. (b) 23. (b) 24. (d ) 25. (d ) 26. ( c) 27. (a ) 28. ( c) 29. ( c) 30. (a )
31. ( c) 32. (d ) 33. (a ) 34. (d ) 35. ( c) 36. (b) 37. (b) 38. (d ) 39. (b) 40. (b)
41. (a) 42. (b) 43. (a) 44. (a) 45. (b)
Part-VI : Geography
Chapter

1
India : Size and Location
Location l In the North and North-East, it shares land boundaries
with China, Nepal and Bhutan.
l Mainland of the Indian subcontinent comprises of India,
l In the East, it shares boundaries with Myanmar and
Pakistan, Bangladesh, Nepal and Bhutan.
Bangladesh.
l It extends between 8°4′ N and 37 °6′ N latitude and
l Our Southern neighbours across the sea consist of the two
between 68°7′ E and 97°25′ E longitude.
island nations i.e., Sri Lanka and Maldives.
l The Tropic of Cancer (23 1/2° N) passes almost through
l Sri Lanka is separated from India by the Palk Strait.
the middle of India.
l Andaman and Nicobar Islands lie to the South-East of
the mainland while Lakshadweep Islands lie to its India : Political
Jammu
South-West. and
Kashmir

l Lakshadweep Islands lie in the Arabian Sea while Srinagar

Andaman and Nicobar Islands are in the Bay of Bengal.


Himachalh
jab

Prades
Pun

Shimla

Size C ha nd igarh
Dheradun
Uttarakhand
Haryana
l The landmass of India has an area of 3.28 million sq km Delhi N
E
ha l
nac esh
PA Aru Prad
and about 2.4% of the total geographical area of the
Sikkim
L g a r
Lucknow tan a
E
world. Rajasthan Jaipur
Gangtok BHUTAN
Asom Nagaland
Uttar Pradesh Dispur
Kohima
India is the seventh largest country in the world.
Bang

Patna
l Shillong
Bihar Manipur
lade

l India ranks second in the world in terms of population Gujarat


Meghalaya
Agartala Imphal
sh

after China. Gandhinagar


Bhopal
Jharkhand
Paschim Aizol
rh

Ranchi Bangal Mizoram


ga

l India measures 3214 km from North to South and Madhya Pradesh Kolkata Tripura
tis
at
hh

2933 km from East to West. Donah


C

Odisha Myanmar
Daman and Diu Silvas
Raipur Bhubaneshwar
l It has a land frontier of 15200 km and coastline of 7517 km. Maharashtra BAY
Mumbai
l 82°30′ E meridian helps in calculating the Indian OF
BENGAL
standard time which is 5 h 30 min ahead of Greenwich Hyderabad
Mean Time (GMT). Arabian Sea
Andhra
Yanam (Puducherry)
Panaji Pradesh
l From Gujarat to Arunachal Pradesh, there is time lag of Goa Karnataka
two hours. Hence, time along the standard meridian of
India passing through Mirzapur (in Uttar Pradesh) is Bengaluru
Andam

Chennai
taken as standard time for the whole country. Mahe Puducherry
an and Nicobar Isl

(Puducherry)
Ker

Tamil Nadu Karakal

India and Its Neighbours


a la

(Puducherry)

l India has 28 states and 8 union territories. Thiruvananthapuram


a nd

Sri Scale1 : 18,000,000


l In the North-West, it shares land boundaries with
s (I

Lanka Km 100 0 100 200 300 400Km


nd

Pakistan and Afghanistan.


i a)
India : Size and Location 599
Position and Extent of India and Its l Ranking of the countries of the world in terms of
area is as follows
Location
(i) Russia (ii) Canada (iii) USA
l India is located on one of the peninsulas of Southern (iv) China (v) Brazil (vi) Australia
Asia. The country extends from Kashmir in the North to (vii) India
Kanyakumari in the South.
l The Southernmost tip of the Republic of India-Indira
l The Arabian Sea and the Bay of Bengal are situated on Point (on great Nicobar Island). This has now been
either side of peninsular India. submerged due to the Tsunami of December 2004.
l The importance of the location of India is that it is l The Kanyakumari is the Southernmost tip of the
located on the world’s major sea routes. mainland of the Indian subcontinent.
l The Indian Ocean is named after the name of our l Tropic of Cancer passes through the states of Gujarat,
country India. Madhya Pradesh, Rajasthan, Chhattisgarh, Jharkhand,
Paschim Banga, Tripura and Mizoram.

Target Exercise
Elementary Level
1. The Southernmost tip of the Republic of India is 9. From Gujarat to Arunachal Pradesh there is a time
(a) Kanyakumari (b) Tamil Nadu lag of
(c) Kerala (d) Indira Point (a) 5 h and 30 min (b) 2 h
(c) 5 h (d) 4 h
2. In terms of geographical area, India ranks
(a) 2nd (b) 5 th 10. India’s total area accounts for about
(c) 6 th (d) 7 th (a) 2.8% of total geographical area of world
(b) 2.5% of total geographical area of world
3. India extends between (c) 2.4% of total geographical area of world
(a) 8°4′ N to 37°6′ N latitude (d) 2.6% of total geographical area of world
(b) 37°6′ N to 42° 5′ N latitude
(c) 8°4′ N to 35°5′ N latitude 11. The easternmost longitude of India is
(d) None of the above (a) 97°25′ E (b) 77°6′ E
(c) 68°7′ E (d) 82° 32′ E
4. North to South extension of India is
(a) 2933 km (b) 3214 km 12. Which one of the following countries does not share
(c) 15200 km (d) 7517 km an international land boundary with India?
5. Which of the following oceans gets its name from the (a) Pakistan (b) Nepal
(c) Bhutan (d) Sri Lanka
name of any country?
(a) Pacific Ocean (b) Atlantic Ocean 13. The mainland of India extends between
(c) Indian Ocean (d) None of these (a) 68° 7′ E longitude to 97°25′ E longitude
(b) 8° 4′ E longitude to 37°6′ E longitude
6. India comprises of (c) 65°7′ longitude to 97°25′ E longitude
(a) 25 states and 7 union territories (d) None of the above
(b) 28 states and 6 union territories
(c) 28 states and 7 union territories 14. Greenwich Mean Time (GMT) is the standard time of
(d) 25 states and 8 union territories which of the following countries?
7. Sri Lanka is separated from India by (a) Japan (b) USA
(c) Germany (d) England
(a) Palk Strait (b) Indian Ocean
(c) Hudson Bay (d) Bay of Bengal 15. Which of the following countries has the largest area
8. India shares international land boundaries with in the world?
(a) Canada (b) USA
(a) 5 countries (b) 6 countries
(c) Russia (d) China
(c) 7 countries (d) 8 countries
600 Study Package for NTSE

High Skill Questions


1. Match the following 3. Which among the following is false?
Country Rank in Terms of Area 1. India lies entirely in the Northern hemisphere.
A. Brazil 1. 4th
2. The southernmost point of Indian Union Indira Point
B. USA 2. 7th got submerged under the sea water in 2004 during
C. China 3. 5th Tsunami.
D. India 4. 3rd 3. 82°30′ E has been selected as the standard meridian of
India.
Codes
A B C D
4. India is the 8th largest country.
(a) 3 1 4 2 (a) 1 and 2 (b) 2 and 3
(b) 1 2 3 4 (c) 3 and 4 (d) Only 4
(c) 4 3 2 1
4. Mainland of the Indian subcontinent comprises of
(d) 3 2 4 1
(a) India, Pakistan, Bangladesh, Nepal
2. Which among the following is/are correct for India? (b) India, Nepal, Bhutan, Bangladesh
1. Landmass of India has an area of 3.28 million (c) Nepal, Bhutan, Bangladesh, India, Pakistan
(d) Bhutan, Bangladesh, India, Nepal, Myanmar
sq km.
2. Palk Strait separates India from Bangladesh. 5. Uttarakhand, Uttar Pradesh, Bihar, Paschim Banga
3. North to South extension of India is 3214 km. and Sikkim have common frontiers with
4. India shares international boundaries with (a) China
7 countries. (b) Nepal
(c) Bhutan
(a) Only 1 (b) 1 and 4
(d) Myanmar
(c) 1, 2 and 3 (d) 1, 3 and 4

Answers
Elementary Level
1. (d) 2. (d) 3. (a) 4. (b) 5. (c) 6. (c) 7. (a) 8. (c) 9. (b) 10. (c)
11. (a) 12. (d) 13. (a) 14. (d) 15. (c)

High Skill Questions


1. (a) 2. (d) 3. (d) 4. (c) 5. (b)
Chapter

2
Physical Features of India
India is a large landmass formed during different l The collision of Indo-Australian plate with Eurasian
geological periods. Weathering, erosion and deposition plate, the sedimentary rocks accumulated in the
have also created and modified the earth’s landmass to geosyncline, known as Tethys were folded to form the
its present form. mountain systems of West Asia and Himalayas.

Formation of Northern Plains of India


Theory of Plate Tectonics
l A large basin was formed because of Himalayan uplift
According to this theory, the crust (upper part) of the from the Tethys sea and peninsular plateau.
earth has been formed out of seven major and some l In due course of time this depression gradually got filled
minor plates. with deposition of sediments by the rivers flowing from
The movement of the plates results in stress within the the mountains in the North and the peninsular plateau
in the South.
plates and the continental rocks, leading to folding,
faulting and volcanic activity. l These extensive alluvial deposits led to the formation of
the Northern plains of India.
The plate movements are divided into three types
l Plates come towards each other and form convergent
Points to Remember
boundary.
l
Geologically, the peninsular plateau constitutes one of the ancient
l Plates move away from each other and from divergent land masses on the earth’s surface and one of the most stable land
boundary. In the event of two plates coming together blocks.
plates may either collide and crumble or one may slide l
The Himalayas and the Northern plains are the most recent land
under the other. forms.
l Sometimes plates also move horizontally each other and l
From the viewpoint of geology, the Himalayan mountains form an
form transform boundary. unstable zone.
l
The whole mountain system of Himalayas represents a very youthful
Formation of Mountain Systems of West topography with high peaks, deep valleys and fast flowing rivers.
Asia and Himalaya l
The Northern plains are formed because of alluvial deposits.
The crust of the earth is divided into a number of pieces
because of the conventional currents.
This resulted in separation of Indo-Australian plate Physiographic Divisions of India
from the Gondwana land towards North. The physical features of India can be divided into six
l Gondwana land includes the oldest landmass and the physiographic divisions.
peninsular part. Gondwana land included India, 1. The Himalayan Mountains
Australia, South Africa, South America and Antarctica.
2. The Northern Plains
l The drift towards North resulted in collision of this plate
3. The Peninsular Plateau
with the Eurasian plate.
602 Study Package for NTSE

4. The Indian Desert Some Highest Peaks of the Himalayas


5. The Coastal Plains S.No. Peak Country Height in Metres
6. The Islands
1. Mt Everest Nepal 8848
The Himalayan Mountains 2. Kanchenjunga India 8598
3. Makalu Nepal 8481
The Himalayas are geologically young and structurally
4. Dhaulagiri Nepal 8172
folded mountains. They are present in the Northern
borders of India. The mountain ranges run in a 5. Nanga Parbat India 8126
West-East direction from the Indus to the 6. Annapurna Nepal 8078
Brahmaputra. 7. Nanda Devi India 7817
l Himalayas form an arc and cover a distance of about 8. Kamet India 7756
2400 km. 9. Namcha Barwa India 7756
l Himalaya mountains’ width varies from 400 km in 10. Gurla Mandhata Nepal 7728
Kashmir to 150 km in Arunachal Pradesh.
l The altitudinal variations are greater in the Eastern Himachal or Lesser Himalayas
half than in the Western half. l Himachal or lesser Himalayas are lying to the South of
the great Himalayas, and their altitude varies between
Great (or) Inner Himalayas (or) the ‘Himadri’ 3700 and 4500 metres with average width of 50 km.
l The Himalayas consist of three parallel ranges in their l Pir Panjal range is the longest range in Himachal.
longitudinal extent. The Northernmost range is known l Dhaula Dhar and the Mahabharat ranges are also
as the Great (or) Inner Himalayas or the ‘Himadri’. prominent ones.
l It contains all the prominent Himalayan peaks with an l Kashmir, Kangra and Kullu valleys are also present in
average height of 6000 metres. these mountains.
l The folds of the Himalayas are asymmetrical in nature, l Himachal or lesser Himalayas are well known for their
composed of granite and perennially snowbound. hill stations.

Shiwaliks
India Shiwaliks are the outermost range of the Himalayas.
KA

RA Physical
G
R
KO
RA
M
They extend over a width of 10-50 km and have an
RA

us
E N
altitude varying between 900 and 1100 metres.
G

E
d
In A
R.
m
Duns
T
elu n a b
Jh e China
R. . C h SH
R
vi IV
Ra AL
n

.
Duns are the longitudinal valleys lying between lesser
ista

R IK
j RA
tle H Tibet
Su NG
Pak

d us
R. E I
M R. Tsangpo Himalayas and Shiwaliks. Dehradun, Kotli Dun and
In NE A
R. R.
Ga
PAL L A Y A S
Patli Dun are some of well known Duns.
GE

R. n ga
Great Indian a
AN

Ya BHUTAN putr
ma
R.

m R. G
IR

Deserts ba
l un rah
Points to Remember
ha g
R. B
Ga
AL

m a h ar
ha a
AV

C a R. Kosi KHASI JAINTIA


da

R. tw
AR

Be
k

on RANGE
R.
R Bangladesh .S l
The part of the Himalayas lying between Sutlej and Indus has been
R. D TROPIC OF
MALWA PLATEAU
VCHHOTANAGPUR a mo
da r CANCER
traditionally known as Punjab Himalaya but it is also known regionally
i

E
DE
ah

YA RANG PLATEAU
A
as Kashmir and Himachal Himalayas.
M

H
VIND H
MA LS
R.

mada HI
L
GIR R. Nar
HILLS l
The part of the Himalayas lying between Sutlej and Kali rivers is known
H

R. T
R. Ma
AR

api Mayanmar
han ad
i as Kumaon Himalayas.
ISG

DECCAN
ATT

R. Goodavari
l
The Kali and Tista rivers demarcate the Nepal Himalayas.
CH

PLATEAU
The part of the Himalayas lying between Tista and Dihang rivers is
S

l
AT
GH

known as Assam Himalayas.


WE

R. Krishna
N
ER
S

R.
TER

ST

Pe
Andaman and Nicobar Islands (India)
EA

nn
T

eru
EL COAS
NG

Purvanchal or Eastern Hills


HA
TS

ND

NILGIRI
l Purvanchal mountains or Eastern hills bend sharply to
COROMA

HILLS
CAR

veri the South and spread along the Eastern boundary of


MAL

R.Cau
OM

India beyond the Dihang gorge.


ABA

ANNAMALAI
RA
R CO

LAKSHADWEEP
These hills run through the North-Eastern states and
HILL

l
AST

(INDIA)
S

Sri
Scale1 : 18,000,000
Kilometres 100 0 100 200 300 400 Kilometres
mostly composed of sandstones which are sedimentary rocks.
Lanka
l The Purvanchal comprises of the Patkai hills, Naga
hills, Manipur hills and Mizo hills.
Physical Features of India 603
The Northern Plain l The Eastward extensions of the plateau are locally
known as Bundelkhand and Baghelkhand.
l The Northern plain was formed by the interplay of the
three major river systems. They are the Indus, the l The Chota Nagpur plateau marks the further Eastward
Ganga and the Brahmaputra. extension and is drained by the Damodar river.
l The Northern plain spreads over an area of 7 lakh sq km. 2. The Deccan Plateau The Deccan Plateau is a triangular
l It is about 2400 km long and 240 to 320 km broad and is landmass that lies to the South of the river Narmada.
also a densely populated physiographic division. l The Satpura ranges are on the North side of the plateau.
l The Western part of the Northern plain is called as the l Mahadev and Kaimur hills and the Maikal range form
Punjab plains. It was formed by the Indus and its its Eastern extensions.
tributaries. This section of the plain is dominated by the l The Deccan plateau is separated by faults from the
doabs. Doab is made up of two words ‘do’ meaning two Chota Nagpur plateau.
and ‘ab’ meaning water.
l In the North-East, the extension of Deccan plateau is
l The plain which extends between Ghaggar and Teesta
also known locally as Meghalaya plateau, Karbi-
rivers is known as Ganga plain. The Ganga plain
Anglong plateau and North Cachar hills.
spreads over the states of North India including
Haryana, Delhi, Uttar Pradesh, Bihar, partly l The Garo, the Khasi and the Jaintia hills are the
Jharkhand and Paschim Banga. In Asom lies the prominent hills present from West to East.
Brahmaputra plain.
The Western Ghats The Western Ghats are the
The Northern plains can be divided into four regions Western edges of the Deccan plateau. Western Ghats lie
based on the variations in relief features. parallel to the Western coast.
(i) Bhabar The rivers deposit pebbles in a narrow belt of
l The Western Ghats are continuous and can be crossed
about 8 to 16 km in width lying parallel to the slopes of
through passes only. e.g., The Thal, Bhor the Pal Ghats
the Shiwaliks. It is known as bhabar.
and Sankotra Gap.
(ii) Terai All the streams disappear in the bhabar belt.
l The Western Ghats average elevation is 900-1600
South of the bhabar belt, the streams and rivers
metres and they are higher than the Eastern Ghats.
re-emerge and create a wet, swampy and marshy
region known as Terai. Terai was a thickly forested l The Western Ghats cause orographic rain by forcing the
region full of wildlife. rain bearing moist winds to rise along the Western
(iii) Bhangar The largest part of the Northern plain is slopes of the ghats.
formed of older alluvium. This lies above the flood l The height of the Western Ghats progressively increase
plains of the rivers and presents a terrace like feature. from North to South.
This part is known as bhangar. l Anai Mudi (2695 metres) and the Doda Betta (2637
(iv) Khadar The newer, younger deposits of the flood metres) are the highest peaks in the Western Ghats.
plains are called khadar. These lands are fertile and
The Eastern Ghats The Eastern Ghats are the
ideal for intensive agriculture.
Eastern edges of the Deccan Plateau. Their average
The Peninsular Plateau elevation is 600 metres.
l The Eastern Ghats are discontinuous and irregular.
The peninsular plateau was formed due to the breaking
They are dissected by rivers draining into the Bay of
and drifting of Gondwana land. The peninsular plateau Bengal.
is made up of old crystalline, igneous and metamorphic
l The Eastern Ghats stretch from the Mahanadi valley to
rocks. the Nilgiris in the South.
The peninsular plateau consists of two broad l Mahendragiri (1501 metres) located in Odisha & A.P.
divisions. border is the highest peak in the Eastern Ghats.
1. Central Highlands Central Highlands lie to the North of l Shevroy Hills and the Javadi Hills are located to the
theNarmada river. They cover a major part of the Malwa South-East of the Eastern Ghats.
plateau.
l The Vindhya range is bounded by the central highlands
The Indian Desert
on the South and Aravalls on the North-West. The Indian desert lies towards the Western margins of
l The Westward extension gradually merges with the the Aravalli hills. This region receives very low rainfall
sandy and rocky desert of Rajasthan. (below 150 mm per year).
l The cental highlands are wider in the West but l It has an arid climate with low vegetation cover.
narrower in the East. l Luni is the only large river in this region.
604 Study Package for NTSE

l Barchans (crescent shaped dunes) cover larger areas l Earlier Lakshadweep Islands were known as Laccadive,
and become more prominent near the Indo-Pakistan Minicoy and Amindive.
boundary. e.g., in Jaisalmer a group of barchans are l In 1973, these were renamed as Lakshadweep.
present. l The Island covers a small area of 32 sq km.
The Coastal Plains l Kavaratti is the administrative headquarters of
Lakshadweep.
The peninsular plateau consists of narrow coastal l The Pitti Island, which is uninhabited, has a bird
strips. They are running along the Arabian Sea on the sanctuary.
West and the Bay of Bengal on the East.
l The Western coast is present between the Western Andaman and Nicobar Islands
Ghats and the Arabian Sea. The Western Coast is a They are a chain of islands located in the Bay of Bengal
narrow plain. extending from North to South.
l The Western coast consists of three sections They are divided into two broad categories
(i) The Konkan coast (Mumbai-Goa) is the Northern part. (i) The Andaman in the North
(ii) The Kannad plain is the central stretch. (ii) The Nicobar in the South
(iii) The Malabar coast is Southern stretch.
l These islands lie close to the equator and experience
l The Eastern coastal plains along the Bay of Bengal are equatorial climate. They have a thick forest cover.
wide and level.
l India’s only active volcano is found on Barren Island in
l In the Northern part, the plain is referred to as the the Andaman group.
Northern Circar and the Southern part is known as the
Coromandel coast. Corals
l Rivers such as the Mahanadi, the Godavari, the Krishna Coral polyps are short-lived microscopic organisms which live in colonies.
and the Kaveri have formed extensive deltas on the East They flourish in shallow, mud free and warm waters. They secrete calcium
coast of India. Lake Chilka is an important feature along carbonate. The coral secretion and their skeletons form coral deposits in the
the Eastern coast. form of reefs.
Reefs
The Islands There are three kinds of reefs. They are barrier reef, fringing reef and atolls.
Lakshadweep Islands The Great Barrier Reef of Australian is an example of barrier reef. Atolls are
circular or horse shoe shaped coral reefs.
The Lakshadweep Islands group is lying close to the
Malabar coast of Kerala and they are composed of small
coral islands.

Target Exercise
Elementary Level
1. The Himalayan mountain system belongs to which 4. The Himalayas are generally divided into three
one of the following? ranges which do not include
(a) Volcanic mountains (b) Block mountains (a) Shiwalik (b) Greater Himalaya
(c) Folded mountains (d) None of these (c) Karakoram (d) Lesser Himalaya

2. Shiwaliks are 5. The largest inhabited riverine island in the world is


(a) to the North of Himadri (a) Bhola Island (b) Great Braila Island
(b) foot hills of Himalayan ranges (c) Majuli (d) Manpura Island
(c) the plateaus between Pamir and Myanmar
(d) in Meghalaya 6. Luni river flows in
(a) Rajasthan
3. The highest peak of the Himalayas in India is in (b) Madhya Pradesh
(a) Uttar Pradesh (b) Kashmir (c) Gujarat
(c) Sikkim (d) Himachal Pradesh (d) Asom
Physical Features of India 605
7. Which among the following is the largest salt water 12. Which of the following is the administrative
lake in India? headquarter of Lakshadweep?
(a) Lake Superior (b) Lake Victoria (a) Kavaratti Island (b) Pitti Island
(c) Lake Chilka (d) None of these (c) Nicobar Island (d) None of these

8. Kangra and Kullu valleys are in 13. What does the term Doab mean?
(a) Tamil Nadu (b) Maharashtra (a) Where the delta of river begins
(c) Andhra Pradesh (d) Himachal Pradesh (b) Where two or more rivers meet
(c) Land between two separate river systems
9. The highest peak in the Eastern Ghats is (d) Land between two tributaries of a river
(a) Anai Mudi (b) Mahendragiri
(c) Kanchenjunga (d) Khasi 14. The central highland and the Deccan plateau are two
broad divisions of
10. The other name of Great Himalayas is (a) the Western Ghats (b) the Eastern Ghats
(a) Himachal (b) Himadri
(c) Bundelkhands (d) The peninsular plateau
(c) Shiwalik (d) None of these
15. The central highland lies to the
11. Anai Mudi and Doda Betta are the highest peak of (a) South of Narmada river (b) East of Narmada river
(a) Greater Himalaya (b) Middle Himalaya (c) North of Narmada river (d) None of these
(c) Eastern Ghats (d) Western Ghats

High Skill Questions


1. The Purvanchal comprises of 5. Match the following
(a) Patkai hills, Naga hills, Manipur hills and Nilgiri hills
(b) Naga hills, Manipur hills, Anamalai hills, Mizo hills List I List II
(c) Patkai hills, Manipur hills, Naga hills, Mizo hills A. The Himalayas are formed of 1. Nilgiri Hills
(d) Mizo hills, Manipur hills, Cardamom hills, Anamalai hills parallel fold ranges of which the
oldest range is
2. Which one of the following statements is not correct?
B. The highest peak of South India is 2. Mt K2
(a) The Western Ghats are relatively higher in their Northern
region C. Junction point of Eastern Ghats 3. The Great
(b) Anai Mudi is the highest peak in the Western Ghats and Western Ghats Himalayan Range
(c) Tapi river lies to the South of Satpura D. Highest mountain peak of India is 4. Anai Mudi
(d) Western Ghats cause orographic rainfall
Codes
3. Gondwana land included A B C D A B C D
(a) India, Australia, South Africa, South America (a) 1 2 3 4 (b) 4 3 1 2
(b) India, Australia, South America, South Africa, Antarctica (c) 3 4 1 2 (d) 4 2 1 3
(c) Australia, India, Antarctica, South Africa, North America
(d) None of the above 6. Arrange the following Himalayan peaks in order as
they occur from West to East
4. Match the following 1. Kanchenjunga 2. Mt Everest
List I List II 3. Nanda Devi 4. Dhaulagiri
(a) 1, 2, 3, 4 (b) 1, 2, 4, 3 (c) 3, 2, 1, 4 (d) 3, 4, 2, 1
A. An area consisting of marshy 1. Khadar
underground seepage 7. Match the following
B. The new alluvium soil is called 2. Bhangar Peaks Mountain Range
C. The old alluvium is termed as 3. Island A. Gurushikhar 1. Zanskar
D. Any area of land smaller than a continent 4. Terai B. Dhupgarh 2. Satpura
and entirely surrounded by water C. Doda Betta 3. Nilgiri
D. Nanga Parbat 4. Vindhya
Codes 5. Aravalli
A B C D
(a) 2 3 4 1 Codes
(b) 4 1 2 3 A B C D A B C D
(c) 4 1 3 2 (a) 4 2 5 1 (b) 3 4 1 5
(d) 1 2 3 4 (c) 4 5 2 1 (d) 5 2 3 1
606 Study Package for NTSE

8. India’s only active volcano is found on Barren Island in 10. Match the following lists
(a) Lakshadweep
List I List II
(b) Andaman and Nicobar group of islands
(c) Pitti Island A. Greater Himalaya 1. Himachal or lesser Himalaya
(d) Kavaratti island B. Middle Himalaya 2. Himadri or Inner Himalaya
C. Outer Himalaya 3. Shiwalik
9. The Western coastal strip, South of Goa is referred to
as Codes
(a) Coromandel (b) Kannad A B C A B C
(c) Konkan (d) Northern Circars (a) 2 1 3 (b) 3 2 1
(c) 1 2 3 (d) 3 1 2

Answers
Elementary Level
1. (c) 2. (b) 3. (c) 4. (c) 5. (c) 6. (a) 7. (c) 8. (d) 9. (b) 10. (b)
11. (d) 12. (a) 13. (d) 14. (d) 15. (c)

High Skill Questions


1. (c) 2. (a) 3. (b) 4. (b) 5. (c) 6. (d) 7. (d) 8. (b) 9. (b) 10. (a)
Chapter

3
Climate
(iv) Distance from the sea (continentality)
Climate (v) Ocean currents
(vi) Relief features
Climate refers to the sum total of weather conditions l The amount of solar energy received varies according to
and variations over a large area for a long period of time the latitude because of the curvature of the earth e.g., air
(more than thirty years). temperature generally decreases from the equator
towards the poles.
Weather l When one goes from the surface of the earth to higher
altitudes, the atmosphere becomes less dense and
Weather refers to the state of the atmosphere over an temperature decreases. e.g., the hills are cooler during
area at any point of time. summers.
l The elements of weather and climate are the same, that l The pressure and wind system of any area depend on the
is temperature, atmospheric pressure, wind, humidity latitude and altitude of the place. Thus, it influences the
and precipitation. temperature and rainfall pattern.
l On the basis of the generalised monthly atmospheric l Continentality means very hot during summers and
conditions, the year is divided into seasons such as very cold during winters. As the distance from the sea
winter, summer and rainy season. increases, the influence of the sea decreases and the
people experience extreme weather conditions.
Temperature and Precipitation l Ocean Currents along with onshore winds affect the
l In summer, the temperature touches 50° C in some parts climate of the coastal areas. e.g., any coastal area with
of the Rajasthan desert and 20° C in Pahalgam in warm (or) cold currents flowing past it will be warmed or
Jammu and Kashmir. cooled if the winds are onshore.
l In winter, the temperature at Drass in Jammu l Relief also determines the climate of a place. e.g., high
and Kashmir is around minus 45° C and 22° C in mountains act as barriers for cold (or) hot winds and
Thiruvananthapuram, Kerala. may also cause precipitation if they are high and lie in
l The annual precipitation varies from over 400 cm in the path of rain-bearing winds.
Meghalaya to less than 10 cm in Ladakh and Western
Rajasthan. Factors Affecting India’s Climate
Factors affecting India’s climate are as follows
Climatic Controls
There are six major controls of the climate of a place. Latitude
They are The area lying South of the Tropic of Cancer belongs to
(i) Latitude tropical climate area and the area lying North of the
(ii) Altitude Tropic of Cancer belongs to the sub-tropical climate
(iii) Pressure and winds area.
608 Study Package for NTSE

The Tropic of Cancer passes almost through the Coriolis Force Force caused by the earth’s rotation. The Coriolis force is
middle of the country from the Rann of Kuchchh in responsible for deflecting winds towards the right in the Northern
the West to Mizoram in the East and it passes hemisphere and towards the left in the Southern hemisphere.
through several states. They are Gujarat, Rajasthan, Jetstream A narrow belt of high altitude (above 12000 m) westerly
wind in the troposphere.
Madhya Pradesh, Chhattisgarh, Jharkhand,
Paschim Banga, Tripura and Mizoram. Western Cyclonic Disturbances It is a weather phenomena which
occurs in winter months and is caused by the westerly flow from the
Mediterranean region.
Altitude
The Himalayas on the North side of India which have
l Western cyclonic disturbances are caused by the westerly
an average height of about 6000 prevent the cold
flow in North and North-Western part of India.
winds from Central Asia entering the subcontinent,
l In summer season, the subtropical westerly jetstream
i.e., the subcontinent experiences milder winters as
moves North of the Himalayas, with the apparent
compared to Central Asia because of the Himalayas.
movement of the sun.

Pressure and Winds l An easterly jetstream called as the subtropical easterly


Jetstream blows over India during the summer months.
The climate and weather condition in India are
governed by the following atmospheric conditions
The Indian Monsoon
l Pressure and surface winds
l Upper air circulation The climate of India is strongly influenced by monsoon
l Western cyclonic disturbances and tropical cyclones winds. The Arabs, which also came to India as traders,
named this seasonal reversal of the wind system ‘monsoon’.
Pressure and Wind Conditions over India
The monsoons are experienced in the tropical area roughly
In the winter season, high pressure area develops between 20° North and 20° South.
North of the Himalayas and cold dry winds blow
from this region to the low-pressure areas over the The mechanism of the monsoons depends on the following
oceans to the South. factors
l The differential heating and cooling of land and water
South-West Monsoon Winds creates low pressure on the land mass of India while the
l In summer season, a low pressure area develops over seas around experience comparatively high pressure.
North-Western India. l The shift of the position of Inter Tropical Convergence Zone
l Air moves from the high-pressure area over the (ITCZ) in summer over the Ganga plain (this is the
Southern Indian Ocean in a South-Easterly equatorial trough normally positioned about 5° North of the
direction, crosses the equator and finally turns right equator. It is also known as the monsoon trough during the
towards the low pressure areas over the Indian monsoon season).
sub-continent. l The presence of a high-pressure area East of Madagascar,
l These winds blow over warm oceans, gather approximately at 20° South over the Indian Ocean. The
moisture and bring widespread rainfall over the intensity and position of this high-pressure area affects the
mainland of India. Indian Monsoon.
l The Tibetan plateau gets intensely heated during summer,
which results in strong vertical air currents and the
Upper Air Circulation formation of low pressure over the plateau at about 9 km
The upper air circulation in Indian subcontinent is above sea level.
dominated by a westerly flow. In important l The movement of the westerly jetstream to the North of the
componant of this flow is the Jetstream. Himalayas and the presence of the tropical easterly
jetstream over the Indian peninsula during summer.
Jetstream Southern Oscillation (SO)
l The Jetstreams are located approximately over the
The changes in the pressure conditions over the Southern
27°-30° North latitude and they are known as
oceans also affect the monsoons. Normally when the
subtropical westerly jetstreams.
tropical Eastern South Pacific Ocean experiences high
l Over India, these Jetstreams blow South of the
pressure, the tropical eastern Indian Ocean experiences
Himalayas all through the year except in summer
season.
low pressure. But in certain years, there is a reversal in the
pressure conditions and the Eastern Pacific has lower
Climate 609
pressure in comparison to the Eastern Indian Ocean. Stages of the Monsoon
This periodic change in pressure conditions is known as
The monsoon arrives at the Southern tip of the Indian
the Southern Oscillation or SO.
peninsula generally by the first week of June.
ENSO (El Nino Southern Oscillations) Subsequently, it proceeds in two directions- the Arabian
A feature connected with the SO is the El Nino Sea branch and the Bay of Bengal branch.
phenomenon in which a warm ocean current that flows l The Arabian Sea branch reaches Mumbai about ten
past the Peruvian coast, in place of the cold Peruvian days later on approximately the 10th of June. Thus, it is
current, every 2 to 5 years. a fairly rapid advance.
l The Bay of Bengal branch also advances rapidly and
The changes in pressure conditions are connected to El
arrives in Asom in the first week of June. The lofty
Nino. Hence, the phenomenon is referred to as ENSO
mountains cause the monsoon winds to deflect towards
(El Nino Southern Oscillations). the West over the Ganga plains.
Intensity of the Monsoon l By mid-June, the Arabian Sea branch of the monsoon
arrives over Saurashtra-Kuchchh and the central part
The difference in pressure over Tahiti (Pacific Ocean,
of the country.
18° South/149° West) and Darwin in Northern Australia
l The Arabian Sea and the Bay of Bengal branches of the
(Indian Ocean, 12°30 South/131° East) is computed to
monsoon merge over the North-Western part of the
predict the intensity of the monsoons. If the pressure
Ganga plains.
difference is negative, it would mean below average and
l Delhi generally receives the monsoon showers from the
late monsoons.
Bay of Bengal branch by the end of June (tentative date
El-Nino This is a name given to the warm ocean is 29th of June). By the first week of July, Western Uttar
current along the coast of Peru. ‘El-Nino’ is a Spanish Pradesh, Punjab, Haryana and Eastern Rajasthan
word meaning ‘the child’ and refers to the baby Christ, experience the monsoon. By mid-July, the monsoon
as this current starts flowing during Christmas. reaches Himachal Pradesh and the rest of the country.
La-Nino This is just opposite situation of EL-Nino. In
this situation a cooling of the surface water of the Retreating of the Monsoon
eastern and central pacific ocean, occurring somewhat Withdrawal or the retreat of the monsoon is a more
less frequently then EL-Nino events but causing gradual process. The withdrawal of the monsoon begins
similar, generally opposite disruptions to global in the North-Western states of India by early
weather patterns. La-Nina conditions occur when the September. By mid-October, it withdraws completely
pacific trade winds blow more strongly them usual, from the Northern half of the peninsula.
pushing the sun-warmed surface water farther West
The withdrawal from the Southern half of the peninsula
and increasing the upwelling of cold water in the
is fairly rapid. By early December, the monsoon has
Eastern regions. Together with the atmospheric effects
withdrawn from the rest of the country.
of the related Southern oscillation, the cooler water
brings drought to Western South America and heavy
rains to Eastern Australia and Indonesia. The Seasons
There are four main seasons in India.
The Onset of the Monsoon and
Withdrawal The Cold Weather Season (Winter)
The monsoon winds are not steady winds but are The cold weather season begins from mid-November in
pulsating in nature. The monsoons will be affected by Northern India and stays till February. December and
different atmospheric conditions over the warm tropical January are the coldest months in the Northern part of
seas. The duration of the monsoon is between 100-120 India.
days from early June to mid-September. l The temperature decreases from the South to the North.
The average temperature of Chennai, on the Eastern
Burst of the Monsoon coast, is between 24° - 25° C, while in the Northern
Around the time of its arrival, the normal rainfall plains, it ranges between 10° - 15° C. Days are warm and
increases suddenly and continues constantly for several nights are cold.
days. This is known as the ‘burst’ of the monsoon and l Frost is common in the North and the higher slopes of
can be distinguished from the pre-monsoon showers. the Himalayas experience snowfall.
610 Study Package for NTSE

l The North-East trade winds prevail over the country l The Deccan plateau and parts of Madhya Pradesh also
during this season. They blow from land to sea and it is a receive some amount of rain inspite of lying in the rain
dry season for most parts of the country. shadow area.
l Some amount of rainfall occurs on the Tamil Nadu coast l For various reasons, the trough and its axis keep on
from these winds, as here they blow from sea to land. moving Northward or Southward, which determines the
l A characteristic feature of the cold weather season over spatial distribution of rainfall. When the axis of the
the Northern plains is the inflow of cyclonic monsoon trough lies over the plains, rainfall is good in
disturbances from the West and the North-West. these parts.

The Hot Weather Season (Summer) Retreating Monsoons


It is hot weather season in India from March to May. In During October-November the monsoon trough or the
March, the highest temperature is about 38° C, recorded low-pressure trough over the Northern plains becomes
on the Deccan plateau. In April, temperature in Gujarat weaker. This is gradually replaced by a high-pressure
and Madhya Pradesh is around 42° C. system.
l In May, temperature of 45° C is common in the The South-West Monsoon winds weaken and start
North-Western parts of the country. In peninsular withdrawing gradually. By the beginning of October,
India, temperature remain lower due to the moderating the monsoon withdraws from the Northern plains.
influence of the oceans. l The months of October-November form a period of
l The summer months experience rising temperature and transition from the hot rainy season to dry winter
falling air pressure in the Northern part of the country. conditions.
Towards the end of May, an elongated low pressure area l The deltas of the Godavari, the Krishna and the Kaveri
develops in the region extending from the Thar Desert in are frequently struck by cyclones, which cause great
the North-West to Patna and Chhotanagpur plateau in damage to life and property.
the East and South-East. Circulation of air begins to set l Sometimes these cyclones arrive at the coasts of Odisha,
in around this trough. Paschim Banga and Bangladesh. The bulk of the rainfall
l A striking feature of the hot weather season is the ‘loo’. on the Coromandel coast is derived from depressions
These are strong, gusty, hot, dry winds blowing and cyclones.
during the day over North and North-Western India.
Sometimes they even continue until late in the evening. Distribution of Rainfall
Advancing Monsoon ( The Rainy Season) Parts of the Western Coast and North-Eastern India
The low-pressure condition over the Northern plains receive over 400 cm of rainfall annually. However, it is
intensifies by early June. It attracts the trade winds of less than 60 cm in Western Rajasthan and the adjoining
part of Gujarat, Haryana and Punjab. Rainfall is
the Southern hemisphere. These South-East trade
equally low in the interior of the Deccan Plateau, and
winds originate over the warm sub-tropical areas of the
East of the Sahyadris.
Southern oceans. They cross the equator and blow in a
l The area of low precipitation is also around Leh in
South Westerly direction, entering the Indian peninsula
Jammu and Kashmir. The rest of the country receives
as the South-West Monsoon. moderate rainfall.
As these winds blow over warm oceans, they bring l The maximum rainfall of this season is received in the
abundant moisture to the subcontinent. These winds are North-Eastern part of the country. Mawsynram in the
strong and blow at an average velocity of 30 km per hour. Southern ranges of the Khasi Hills receives the highest
average rainfall in the world. Rainfall in the Ganga
l Early in the season, the windward side of the Western
valley decreases from East to West. Rajasthan and parts
Ghats receives very heavy rainfall which is more than
of Gujarat get scanty rainfall.
250 cm.
Target Exercise
Elementary Level
1. The word monsoon is derived from the Arabic word 8. The retreating monsoon withdraws from the
‘Mausim’ which means (a) West coast to the East coast
(a) climate (b) season (c) wind (d) stream (b) North to the South
(c) North-East India to the West coast
2. Which among the following is the wettest place on the (d) North-West India to Bengal
earth?
(a) Cherapunji (b) Mawsynram 9. Which of the following is incorrectly matched?
(c) Kerala (d) None of these A. Aandhi : Uttar Pradesh
B. Loo : North-West India
3. Which one of the following characterises the cold C. Kal Baisakhi : Paschim Banga
weather season in India? D. Mango shower : Odisha
(a) Warm days and warm nights
(b) Warm days and cold nights
10. Which among the following is not a factor affecting
(c) Cold days and cold nights India’s climate?
(d) Cold days and warm nights (a) Latitude (b) Altitude
(c) Pressure and winds (d) Snowfall
4. Which one of the following states receives the highest
rainfall during winter months? 11. Jetstreams blow South of the Himalayas all through
the year except in
(a) Punjab (b) Rajasthan (c) Meghalaya (d) Tamil Nadu
(a) winter (b) rainy season
5. Winter rains in North and North-West India are (c) summer (d) None of these
generally associalted with the phenomenon of 12. Pre-monsoon showers are also called cherry blossom
(a) retreating monsoon (b) temperate cyclone
in
(c) local thunderstorm (d) shift in jetstream movement
(a) Kerala (b) Karnataka
6. During the monsoon season rainfall decreases from (c) Tamil Nadu (d) Andhra Pradesh
the Ganga delta to the Punjab plains. It is because 13. October heat is marked by
(a) monsoon current moves Westward along the Ganga plain
1. clean cloudless sky
and becomes drier
(b) the Western region are warmer than the Eastern region 2. high temperature
(c) hills do not form barriers for winds 3. high humidity
(d) the area is far away from the sea (a) 1 and 2 (b) 2 and 3
(c) 3 and 1 (d) All of these
7. In May-June each year the monsoon winds approach
the Southern tip of India from 14. The driest place in India is
(a) monthly direction (a) Leh (b) Barmer (c) Jaisalmer (d) Bikaner
(b) North-Easterly direction 15. The climate of India is
(c) North-Westerly direction (a) tropical climate (b) subtropical climate
(d) South-Westerly direction (c) savanna type of climate (d) subtropical monsoon

High Skill Questions


1. Which of the following are the main reasons for the (c) North-Western India receives heavy rainfall by retreating
origin of the South-West Monsoon in India? monsoon
1. Development of a low pressure in North-West (d) North-Eastern India receives heavy rainfall by North-East
monsoon
India.
2. Deflection of trade winds. 3. Arrange the following cities in decreasing order of
3. Arrival of cyclonic stream from the Pacific Ocean. the rainfall they receive from South-West monsoon in
4. Heating of the Arabian Sea and the Bay of Bengal. summer months
(a) 1, 2 and 3 (b) 1 and 2 1. Allahabad 2. Kolkata
(c) 1 and 4 (d) 3 and 4 3. Delhi 4. Patna
(a) 2, 4, 1, 3 (b) 3, 1, 4, 2
2. Which of the following statements is correct?
(c) 1, 4, 3, 2 (d) 2, 4, 3, 1
(a) Eastern Ghats receive more rainfall than Western Ghats
(b) Aravalli mountains help heavy rainfall in the adjoining areas
612 Study Package for NTSE
4. Match the following lists 7. Which of the following is the main cause of heavy
rainfall in Tamil Nadu during winters?
List I List II
(a) Blowing of trade winds
A. Cold weather 1. June to September (b) Eastern Ghats provide a barrier for North-West monsoon
B. Hot weather 2. October to November winds
C. South-West monsoon 3. December to February (c) Nearness to equator
D. Retreating monsoon 4. March to May (d) Tropical cyclones which originate when the North-West
Monsoon winds cross Bay of Bengal
Codes
A B C D
8. Punjab and Haryana receive rainfall in winter from
(a) 3 4 1 2 (a) Westerly disturbances
(b) 4 3 2 1 (b) cyclones in Bay of Bengal
(c) 1 2 3 4 (c) anticyclones
(d) 3 2 4 1 (d) None of the above

5. The extremes of temperature between summer and 9. The Western disturbances which cause rain in
winter are quite low in the Southern part of North-West India originate in
peninsular India mainly because (a) Black Sea
(b) Caspian Sea
(a) the adjoining oceans moderate the temperature
(c) Aral Sea
(b) the sky is generally cloudy
(d) Mediterranean Sea
(c) the sun’s rays are almost vertical throughout
(d) strong winds blow throughout the year 10. The pre-monsoon mango showers occur predomi-
6. Retreating monsoon causes rainfall during the nantly in
month of (a) Paschim Banga and Asom
(a) August and September (b) Goa and Maharashtra
(b) September and October (c) Malwa plateau
(c) October and November (d) Kerala
(d) November and December

Answers
Elementary Level
1. (b) 2. (b) 3. (b) 4. (d) 5. (b) 6. (a) 7. (d) 8. (d) 9. (d) 10. (d)
11. (c) 12. (b) 13. (d) 14. (c) 15. (d)

High Skill Questions


1. (c) 2. (d) 3. (a) 4. (a) 5. (a) 6. (c) 7. (d) 8. (a) 9. (d) 10. (b)
Chapter

4
Drainage
Drainage system is defined as the river system of an most river system in the sub-continent. It enters India in
area. e.g., Ganga River System etc. the Ladakh district of Jammu and Kashmir. In Jammu
and Kashmir, its Himalayan tributaries are Zaskar,
On the basis of area drained, the river basins of India Dras, Gartang, Shyok, Shigar, Nubra, Gilgit etc.
have been classified into three categories
l The Sutlej (Satudri), Beas (Vipasa), Jhelum (Vitasta),
(i) Major River basins with catchment area of more than Chenab (Asikni) and Ravi (Parushni or Iravati) are the
20000 sq km and above. important tributaries of Indus which join together to
(ii) Medium River basins having a catchment area enter the Indus near Mithankot in Pakistan.
between 2000-20000 sq km.
l The Indus make several deep gorges. The deepest of all
(iii) Minor River basins with catchment area below 2000
is at Gilgit, which is 5200 m in height above the level of
sq km.
the water at its bed. Indus falls into Arabian Sea. In
l India has 113 river basins, of which 14 major, 44
Tibet, Indus is known as ‘Singi Khamban’ Or Lion's
medium and 55 are minor river basins. Mouth.
l The top 5 major river basins of India in descending l According to the Indus water treaty signed between
order of area are the Ganga (861404 sq km), the Indus India and Pakistan in 1960, India can utilize only 20% of
(321284 sq km) the Godavari (312812 sq km), the the total discharge of Indus, Jhelum and Chenab.
Krishna (258848 sq km) and the Brahmaputra
(258008 sq km). The Ganga River System
The Ganga River System is the second major drainage
The Himalayan Rivers system of India. It rises in the Gangotri glacier near
The major Himalayan rivers are the Indus, the Ganga Gaumukh (3900 m) in Uttarakhand, here, it is known as
and the Brahmaputra. the Bhagirathi. At Devprayag, the Bhagirathi meets the
Himalayan River System is divided into three major river Alaknanda, hereafter, it is known as Ganga.
systems l The Alaknanda has its source in the Satopanth glacier
above Badrinath. It consists of the Dhauli and Vishnu
(i) The Indus River System
Ganga which meet at Joshimath or Vishnu Prayag. The
(ii) The Ganga River System
other tributaries of Alaknanda such as the Pindar joins
(iii) The Brahmaputra River System
it at Karnaprayag while Mandakini or Kali Ganga
meets it at Rudraprayag.
The Indus (Sindhu) River System l It is 2525 km long of which 1450 km is in Uttarakhand
The Indus is one of the most important drainage and UP, 445 km in Bihar and 520 km in Paschim Banga.
systems of the subcontinent of India. It has a length of l The left bank tributaries of Ganga are: Ramganga,
2880 km, of which 709 km lies in India. The catchment Gomti, Kali or Sarda, Gandak, Kosi, Mahanadi.
area of the Indus is about 1165000 sq km, out of which l The right bank tributaries of Ganga are: Yamuna and
about 321284 sq km is in India. Son. Yamuna joins the Ganga at Allahabad.
l Indus originates from the Bokhar Chu (glacier) in the l Hooghly is a distributory of Ganga flowing through
Tibetan region near Mansarovar lake. It is the Western Kolkata.
614 Study Package for NTSE

l The Ganga river is an international river as it passes tributaries of Godavari are: Manjra, Penganga,
through Bangladesh before merging into the Bay of Wardha, Wainganga, Indravati, Sabari Pranhita.
Bengal. Kosi is called as ‘Sorrow of Bihar’ while
Damodar is called as ‘Sorrow of Bengal’ as these cause Krishna (Length 1290 km, Basin 259000 sq km)
floods in these regions.
The river Krishna has its origin near Mahabaleshwar in
The Brahmaputra River System the vertical faces of the Western Ghats. Its important
tributaries are Koyna, Ghataprabha, Malprabha,
The Brahmaputra rises in Tibet, East of Mansarovar
Bhima, Tungbhadra, Musi and Muneru.
lake. It is one of the largest river of the world. It is
volume-wise largest river of India whereas lengthwise Kaveri (Length 765 km, Basin 87900 sq km)
Ganga river is the longest in India. It flows Eastwards
parallel to the Himalayas. On reaching Namcha Barwa, The Kaveri is a sacred river like the Ganga. Hence, it is
it takes a ‘U’ turn and enters India in Arunachal called the Ganga of South India. It originates from Taal
Pradesh through a gorge. Kaveri. Its important tributaries are: Herangi,
Hemavati, Lokpavni, Shimsa, Arkavati, Kabani,
l Brahmaputra is known as Tsangpo in Tibet, Dihang or
Siang in Arunachal Pradesh, Brahmaputra in Asom and Bhavani and Amravati. The river Kaveri makes the
Jamuna in Bangladesh. second biggest waterfall in India known as
l The right bank tributaries of Barahmaputra are: Sivasamudram.
Subansiri, Dhareli, Manas, Sankos, Tista and Raidak
etc. West Flowing Rivers
l The left bank tributaries of Brahmaputra are: Dihang, (or Estuaries Forming Rivers)
Lohit, Burhi Dihang, Dhansiri, Kalang and Kapili etc.
West flowing rivers do not form delta and these fall into
l Brahmaputra forms largest number of riverine island.
the Arabian Sea. Example of West flowing rivers are:
Majuli is the largest riverine island in the world.
Sabarmati, Mahi, Narmada, Tapi and Luni etc. All
these rivers form estuaries and fall into the Arabian
The Peninsular Rivers Sea.
The Peninsular River System can be divided in two
groups Narmada (Length 1300 km, Drainage Basin 98800 sq km)

(i) East flowing rivers (or delta forming rivers) The river Narmada rises from the plateau of
(ii) West flowing rivers (or estuaries forming rivers) Amarkantak of the Maikal Hills of Chhattisgarh.
Moving North-Eastwards, it passes through a complex
East Flowing Rivers course near Jabalpur, through some impressive marbel
(or Delta Forming Rivers) gorges, the most spectacular being Dhuandhar
waterfalls (10 m high) near Jabalpur. Its important
East flowing rivers do not form estuaries and it fall into tributaries are: Hiran, Orsang, Barna, Kolar, Burhnar,
the Bay of Bengal. Example of East flowing rivers are Tawa and Kundi etc.
the Mahanadi, Godavari, Krishna and Kaveri etc. All
these rivers form delta and fall into the Bay of Bengal. Tapi (Length 700 km, Basin 66900 sq km)
Mahanadi (Length 885 km, Basin 141600 sq km) The river Tapi rises in the Satpura ranges in the Betul
district of Madhya Pradesh. Its basin covers parts of
It is the most important river of Odisha as well as that of
MP, Gujarat and Maharashtra. Its main tributaries are:
Chhattisgarh. It originates from Dandakaranya plateau
Purna, Betul, Patki, Ganjal, Dhatranj, Bokad and
near Raipur. The important tributaries of Mahanadi are
Amravati.
Mand, Hasdeo, Seonath, Ong, Jonk, Tel. Its drainage
basin is shared by Maharashtra, Karnataka and
Andhra Pradesh. Drainage System in India
Godavari (Length 1465 km, Basin 312800 sq km) On the basis of mode of origin, Indian rivers are divided
into two major groups
It is the largest river of peninsular India. It originates (i) The Himalayan Rivers
from Trimbak plateau near Nasik (Maharashtra). It is
(ii) The Peninsular Rivers
also known as Dakshin Ganga. The important
Drainage 615
Difference between Himalayan Rivers and humanity. They also help in the prevention of sudden
Peninsular Rivers floods and droughts.

Himalayan Rivers Peninsular Rivers River Pollution


These rivers are perennial in These rivers are seasonal in nature The growing domestic, municipal, industrial and
nature i.e., the water flows and receive water only from rainfall. agricultural demand for water from rivers naturally
throughout the year.
These have large basins and These have small basins and
affects the quality of water.
catchment areas. catchment areas.
These rivers are much These rivers are much older, some of National River Conservation Plan (NRCP)
younger, excepting a few them are as old as the pre-cambrian l The activities of Ganga Action Plan (GAP) Phase-I, was initiated in 1985
antecedent (older than period. and was closed on 31st March, 2000.
Himalaya) rivers like; Indus, l The Ganga Action Plan (GAP) Phase-II, has been merged with the NRCP.
Sutlej, Ganga, Brahmaputra The expanded NRCP now covers 152 towns located along 27 interstate
etc. rivers in 16 states.
These rivers make only deltas. These rivers make deltas (Godavari, l A total of 215 schemes of pollution abatement have been sanctioned.
The Sunderban delta is the Krishna, Kaveri) and estuaries like; So far, 69 schemes have been completed under this action plan. A
largest delta in the world. Narmada and Tapi. million litres of sewage is targeted to be intercepted, diverted
In the plains, these make These make shallow meanders. and treated.
numerous sharp meanders
and ox-bow lakes.
Most of them originate from the Most of them originate from the Role of Rivers in the Economy
Himalayan glaciers. Western Ghats and plateau.
Rivers have been of fundamental importance
throughout the human history. Using rivers for
The Lakes irrigation, navigation, hydro-power generation is of
A lake is a natural depression filled with water. In this special significance-particularly to a country like India,
period of population explosion and scarcity of water, where agriculture is the major source of livelihood of the
lakes are an important source of water supply to majority of its pollution.

Target Exercise
Elementary Level
1. Which of the following river is known as Jamuna in 6. By which name does the Brahmaputra enter into
Bangladesh? India?
(a) Ganga (b) Brahmaputra (a) Dihang (b) Manas (c) Tsangpo (d) Dhansiri
(c) Narmada (d) Tapi
7. River Damodar is called ‘Sorrow of Bengal’ because it
2. Famous Dal Lake is in (a) gets flooded after causing havoc
(a) Uttarakhand (b) Punjab (b) causes maximum soil erosion
(c) Himachal Pradesh (d) Kashmir (c) forms numbers of dangerous waterfalls
3. Which of the following is the longest river in India? (d) is not a perennial river
(a) Ganga (b) Narmada 8. Shyok is tributary of
(c) Brahmaputra (d) Godavari (a) Brahmaputra (b) Indus
4. Which of the following river is known as India’s river (c) Chenab (d) Sutlej
of sorrow? 9. By what name is the Ganga known in Bangladesh?
(a) Hooghly (b) Damodar (a) Padma (b) Bhagirathi
(c) Ghaghara (d) Kosi (c) Rupnarayan (d) Nubra
5. In which state is Jog falls located? 10. Which of the following peninsular rivers is Westward
(a) Maharashtra (b) Karnataka flowing?
(c) Tamil Nadu (d) Kerala (a) Mahanadi (b) Godavari (c) Tapi (d) Kaveri
616 Study Package for NTSE

11. Which of the following pairs are correctly matched? Select the correct answer using the codes given below
1. Narmada : Jabalpur (a) 1 and 4 (b) 1 and 3
(c) 1 and 2 (d) 3 and 4
2. Godavari : Nasik
3. Mahanadi : Cuttack 14. According to which treaty, India only use 20% of total
(a) 2 and 3 (b) 1 and 3 water carried by Indus River System
(c) 1 and 2 (d) 1, 2 and 3 (a) Indus Water Treaty of 1960
(b) India-Nepal Water Treaty of 1965
12. The Himalayan rivers are (c) India-China Water Treaty of 1960
(a) monsoon fed (b) seasonal (d) None of the above
(c) snow fed (d) ephemeral
15. Which of the following river is known as ‘Vridha
13. Which among these build a common delta? Ganga’?
1. Ganga 2. Brahmaputra (a) Godavari (b) Krishna
3. Godavari 4. Krishna (c) Mahanadi (d) Kaveri

High Skill Questions


1. The Bhakra Nangal Multipurpose Project is a joint Codes
verture of A B C D A B C D
(a) Punjab and Haryana (a) 2 1 4 3 (b) 4 3 2 1
(b) Punjab, Haryana and Rajasthan (c) 1 2 3 4 (d) 3 4 1 2
(c) Punjab, Haryana, Rajasthan and Himachal Pradesh
(d) Punjab and Himachal Pradesh 5. Which one of the following pairs is not correctly
matched?
2. What is correct sequence of rivers - Godavari, Mahanadi,
Narmada and Tapi in descending order of their length? River City
(a) Godavari–Mahanadi–Narmada–Tapi (a) Gomti Lucknow
(b) Godavari–Narmada–Mahanadi–Tapi Saryu Ayodhya
(b)
(c) Narmada–Godavari–Tapi–Mananadi
(c) Alaknanda Badrinath
(d) Narmada–Tapi–Godavari–Mahanadi
(d) Narmada Satna
3. Which of the following is true with regard to
characteristics of the Himalayan rivers? 6. Which one of the following rivers flows between
I. Many of them have their sources in Inner Himalaya. Vindhayan and Satpura range?
II. They have a perennial flow. (a) Narmada (b) Son (c) Mahe (d) Netravati
III. They have a great capacity for erosion. 7. Which river termed as ‘Ganga of South’?
IV. They do not form gorges. (a) Narmada (b) Tapi (c) Kaveri (d) None of these
(a) I and II (b) I, II and III (c) III and IV (d) II and IV
8. Wular lake of Kashmir is a/an
4. Match the following pairs (a) ox-bow lake
(b) lake formed by blocking of ice
River Origin (c) lake formed by terminol morainic dam
A. Indus 1. Uttarakhand (d) lake formed by deposition of silt
B. Ganga 2. Tibet
9. Most of the rivers flowing Westward from the
C. Godavari 3. Madhya Pradesh
D. Narmada 4. Maharashtra
Western Ghats do not form delta because
(a) of the high gradient (b) they are too slow
5. Karnataka
(c) they are not perennial (d) there is no vegetation

Answers
Elementary Level
1. (b) 2. (d) 3. (a) 4. (d) 5. (b) 6. (a) 7. (a) 8. (b) 9. (a) 10. (c)
11. (d) 12. (c) 13. (c) 14. (a) 15. (a)

High Skill Questions


1. (b) 2. (b) 3. (b) 4. (a) 5. (d) 6. (a) 7. (c) 8. (a) 9. (a)
Chapter

5
Resources and
Development
Resources examples of such resources. These resources cannot be
recycled and get exhausted with their use.
Everything available in our environment which can be
used to satisfy our needs, provided it is technology Individual Resources These are owned privately by
accessible, economically feasible and culturally individual. Land, plot, houses, other property etc., are
accepted can be termed as resource. The process of examples of resources ownership by individuals.
transformation of things available in our environment Community Owned Resources There are resources
involves an inter-dependent relationship between which are accessible to all the members of the
nature, technology and institutions. Resources are a community. Public parks, picnic spots etc., are de facto
function of human activities. Human beings themselves accessible to all people living there.
are essential components of resources. Resources can be National Resources Technically, all the resources,
classified in the following ways belong to the nation. The country has legal powers to
Resources acquire even private property for public good. All the
On the basis of origin Biotic and abiotic
minerals, water resources, forests, wildlife, land within
On the basis of Renewable and non-renewable 12 nautical miles (19.2 km) from the coast termed as
exhaustibility territorial water and resources therein belong to the
On the basis of ownership Individual, national and international nation.
On the basis of status of Potential, developed, stock and
development reserves
International Resources There are international
institutions which regulate some resources. The oceanic
Biotic Resources Obtained from biosphere and have resources beyond 200 nautical miles of the exclusive
life such as human beings, flora and fauna. fisheries, economic zone belong to open ocean and no individual
livestock etc. country can utilize these without the concurrence of
Abiotic Resources All those things which are international institutions.
composed of non-living things are called abiotic Potential Resources Resources which are found in a
resources, rocks and metals. region, but have not been utilized. The Western part of
Renewable Resources Resources which can be India particularly Rajasthan and Gujarat have
renewed or reproduced by physical, chemical or enormous potential for the development of wind and
mechanical processes are known as renewable or solar energy.
replenishable resources. Such as solar, wind, water, Developed Resources Resources which are surveyed
forest and wildlife. and their quality and quantity have been determined
Non-renewable Resources These occur over a very for utilization. The development of resources depends
long geological time. Minerals and fossil fuels are on technology and level of their feasibility.
618 Study Package for NTSE

Stock l Land resources are used for the following purposes


(i) Forests.
Material in environment which have the potential to (ii) Land not available for cultivation- Barren and waste
satisfy human needs but human beings do not have the land, for non-agricultural purposes - roads, factories
appropriate technology to access them. etc.
(iii) Other uncultivated land-permanent pastures and
Reserves grazing land.

These are the subset of the stock, which can be put into Rio de Janeiro Earth Summit, 1992
use with the help of existing technical knowledge, but In June 1992, more than 100 heads of states met in Rio de Janeiro in Brazil, for
their use has not been started. The water in the dams, the first International Earth Summit. The Summit was convened for
forests etc., is a reserve which can be used in the future. addressing urgent problems of environmental protection and
socio-economic development at the global level. The assembled leaders
Sustainable Development signed the Declaration on Global Climatic Change and Biological Diversity.
Sustainable development means ‘development should take place The Rio Convention endorsed the global Forest Principles and adopted
without damaging the environment’, and development in the present Agenda 21 for achieving sustainable development in the 21st century.
should not compromise with the needs of the future generations.
Agenda 21
It is the declaration signed by world leaders in 1992 at the United Nations
Conference on Environmement and Development (UNCED), which took
Resource Planning in India place at Rio de Janeiro, Brazil. It aims at achieving global sustainable
development. It is an agenda to combat environmental damage, poverty,
l Resource planning is a complex process which involves
disease though global cooperation on common interests, mutual needs and
(i) identification and inventory of resources across the shared responsibilities. One major objective of the Agenda 21 is that every
regions of the country. local government should draw its own local Agenda 21.
(ii) evolving a planning structure endowed with
appropriate technology, skill and institutional set up
for implementing resource development plans.
(iii) matching the resource development plans with Land Degradation and Conservation
overall national development plans.
l Our basic needs for food, shelter and clothing are
l The availability of resources is a necessary condition for obtained from land.
the development of any region, but mere availability of l Human activities have not only brought about
resources in the absence of corresponding changes in degradation of land have also aggravated the pace of
technology and institutions may hinder development. natural forces to cause damage to land.
l There are many regions in our country that are rich in l Human activities such as deforestation, over grazing,
resources, but these are included in economically mining and quarrying to have contributed significantly
backward regions. in land degradation.
l In India, development in general and resource l There are many ways to solve the problems of land
development in particular does not only involve the degradation. Afforestation and proper management of
availability or resources, but also the technology, quality grazing can help to some extent.
of human resources and historical experience of the l Planting of shelter belts of plants, control on over
people. grazing, stabilization of sand dunes by growing thorny
bushes are some of the methods to check land
Land Resources degradation.
l Land is a natural resource of utmost importance. It l Proper management of waste land, proper discharge and
supports natural vegetation, wildlife, human life, disposal of industrial effluents and wastes after
economic activities, transport and communication treatment can reduce land and water degradation in
system. India has land under a variety of relief features industrial and sub-urban areas.
namely; mountains, plateaus, plains and islands. About
43% of the land area is plain, which provides facilities Soil As a Resource
for agriculture and industry.
Soil is the most important renewable natural resource.
l
Fallow Land Land left without cultivation for one or less than one It is the medium of plant growth and supports different
agricultural year. types of living organisms on the earth. Thus, soil is a
l
Net Sown Area Area sown more than once in an agricultural year. living system. Relief, parent rock or bed rock, climate,
Plus net sown area is known as Gross Cropped Area.
vegetation and other forms of life and time are
Resources and Development 619
important factors in the formation of soil. Various formation and erosion go on simultaneously and
factors of nature such as change in temperature, actions generally there is a balance between the two.
of running water, wind and glaciers, activities of l The balance is disturbed due to human activities like
decomposers etc contribute to the formation of soil. deforestation, over-grazing, construction and mining
etc., while natural forces like; wind, glacier and water
Classification of Soils lead to soil erosion.
Alluvial Soil Black Soil l In such process the top soil is washed away which is
known as’ Sheet Erosion’.
Most widely spread and important It is also known as regur soil.
soil. l The soil conservation can be done by contour ploughing,
terrace cultivation, strip farming, shelter belts.
These have been deposited by the Climatic condition along with
Indus, the Ganga and the parent rock materials are Contour Ploughing Ploughing along the contour lines
Brahmaputra river systems. responsible for the formation. which decelerate the flow of water down the slopes.
Also found in Eastern coastal plains, Soil is typical of the Deccan trap
deltas of Mahanadi, the Godavari, (basalt) region spread over
Terrace Cultivation Cut out on the slopes making
the Krishna amd the Kaveri rivers. North-West Deccan plateau and terraces (Conservation).
made of lava flows. Shelter Belts Planting lines of trees to create shelter.
To their age, alluvial soil can be Spread over Maharashtra,
classified as old (Bangar) and new Saurashtra, Malwa, Madhya Strip Cropping Strip of grass are left to grow between
alluvial (Khadar). Pradesh and Chhattisgarh. the crops which breaks up the force of the wind
Soil contains adequate proportion Rich in soil nutrients such as (Conservation).
of potash, phosphoric acid and calcium carbonate, potash,
lime. magnesium and lime. Water as Resource
Red and Yellow Soils Laterite Soil Three-fourth of the earth’s surface is covered with
Red soil develops on crystalline Found in areas of high water, but only a small proportion of it accounts for
igneous rocks. temperature and heavy rainfall. fresh water that can be put to use. The fresh water is
Areas of low rainfall in the Eastern Result of intense leaching due to mainly obtained from surface run off and ground water
and Southern parts of the Deccan heavy rain. that is continually being renewed and recharged
plateau. through the hydrological cycle ensuing that water is a
Red and Yellow soils found in parts Mainly found in Karnataka, re-newable resource.
of Odisha, Chhattisgarh and Kerala, Tamil Nadu, Odisha and
Western Ghats. Asom. l The abundance and renewability of water, it is difficult
to imagine that we may suffer from water scarcity. The
The soil develop reddish colour Suitable for plantation of tea and
movement we speak of water shortages, we immediately
due to diffusion of iron in crystalline coffee.
and metaphorphic rocks.
associate it with the region having low rain fall or those
that are drought prone.
Arid Soil Forest Soil l The availability of water resources varies over space and
Soils range from Red to Brown in Found in hilly and mountainous time, mainly due to the variations in seasonal and
colour. regions. annual precipitation, but water scarcity in most cases
Sandy in texture, and saline in They are loamy and silty in valley
caused by over-exploitation, excessive use and unequal
nature. sides, and coarse grained in the access to water among different social groups.
upper slopes. l The water scarcity may be an outcome of large and
Soil lacks humus and moisture. In the snow covered areas of growing population and consequent greater demands for
Himalayas, these soils water and unequal access to it.
experience denudation and are l The scarcity of water may be due to bad quality of water
acidic with low humus content.
which is polluted by domestic and industrial wastes,
The lower horizons of the soil are chemicals, pesticides and fertilizers used in agriculture,
occupied by Kankar because of thus making it hazardous for human use.
the increasing calcium content
downwards. After proper irrigation, Water Resource Management
these soils became cultivable as in
the case of Western Rajasthan. Archaeological and historical records show that from
ancient times, we have been constructing sophisticated
hydraulic structure like; dams, reservoirs or lakes,
Soil Erosion and Conservation embankments and canals for irrigation. Dams were
The denudation of the soil cover and subsequent traditionally built to impound rivers and rainwater that
washing down is described as soil erosion. The soil could be used later to irrigate agricultural fields.
620 Study Package for NTSE

l Today, dams are built not just for irrigation, but for Madhya Pradesh has the largest area under permanent forests, constituting
electricity generation, water supply for domestic and 75% of its total forest area. Jammu and Kashmir, Andhra Pradesh,
industrial uses, flood control, recreation, inland Uttarakhand, Kerala, Tamil Nadu, Paschim Banga and Maharashtra have
navigation and fish breeding. large percentages of reserved forests of its total forest area whereas Bihar,
Harayana, Punjab, Himachal Pradesh, Odisha and Rajasthan have a bulk of it
l Multi-purpose projects and large dams have also been
under protected forests. All North-Eastern states and parts of Gujarat have
the cause of many new social movements like; the
a very high percentage of their forests as unclassed forests managed by local
Narmada Bachao Andolan and Tehri Dam Andolan etc.
communities.
l Resistance to these projects has primarily been due to
the large scale displacement of local communities.
Unclassed Forests There are other forests and
Rainwater Harvesting wastelands belonging to both government and private
In ancient India, along with the sophisticated hydraulic individuals and communities.
structures, there existed an extraordinary tradition of
water harvesting system. Community and Conservation
l People had in-depth knowledge of rainfall regimes and l Conservation strategies are not new in our country.
soil types and developed wide ranging techniques to Enactment of wildlife acts, formation of wildlife
harvest rainwater, ground water, river water and flood centuries and national parks are such examples of forest
water in keeping with the ecological conditions and their conservation.
water needs.
l Certain societies revere a particular tree which they
l In hills and Mountaneous regions people built diversion have preserved from time immemorial. The Mundas and
channels like ‘guls’ and ‘kuls’. Santhal of Chhota Nagpur region worship Mahua
l Rooftop rainwater harvesting was commonly practised (Bassia latifolia) and Kadamba (Anthocaphalus
to store drinking water, particularly in Rajasthan, cadamba) trees, and the tribals of Odisha and Bihar
Shillong and Meghalaya. worship the tamarind and Mango trees during
l In Meghalaya, old system of tapping stream and spring weedings.
water by using Bamboo pipes known as bamboo drip l The famous Chipko Movement in the Himalayas has
irrigation system. not only successfully resisted deforestation in several
l In arid and semi-arid regions, agricultural fields were areas but has also shown that community afforestation
converted into rainfed storage structures that allowed with indogenous species can be enormously successful.
water to moister the soil like ‘Khadins’ in Jaisalmer. l The Indian Wildlife Act (Protection) was implement in
1972.
Land Degradation in Different States
Mining Jharkhand, Chhattisgarh, Madhya Pradesh and Odisha Natural Vegetation and Wildlife Resources
Overgrazing Gujarat, Rajasthan, Madhya Pradesh and Maharashtra. Types of Vegetation
Over Irrigation Punjab, Haryana, Western Uttar Pradesh.
The major types of vegetation can be classified into
following
Forest and Wildlife Resources Tropical Evergreen Forests These forests are
In India much of its forest and wildlife resources are restricted to heavy rainfall areas of Western Ghats and
either owned or managed by the government through Island groups of Lakshadweep, Andaman and Nicobar,
the Forest Department. upper parts of Asom and Tamil Nadu coast.
These are classified under the following categories l There is no definite time for trees to shed their leaves
and these forests appear green all the year round.
Reserved Forests Reserved Forests are regarded as
l Ebony, Mahogany, Rosewood, Rubber and Cinchona are
the most valuable as far as the conservation of forest commercially important trees of these forests.
and wildlife resources are concerned. More tha half of
Tropical Deciduous Forests These are the most
the total forest land. Reserved forests are also referred
widespread forests of India and also called the
as permanent forest estates.
Monsoon forests and spread over the region receiving
Protected Forests Almost one-third of the total forest
rainfall between 200 cm and 70 cm.
area is protected forest, as declared by the forest
l Trees of this forest type shed their leaves in dry summer.
department.
Resources and Development 621
l These forests exit mostly in the Eastern parts of the (iv) Rare Species Species with small population may
North-Eastern states along the foothills of Himalayas, move into the endangered or vulnerable category, if
Jharkhand, West Odisha and Chhattisgarh and on the negative factors affecting them continue to operate.
Eastern slopes of Western Ghats. Himalayan bear, wild Asiatic Buffalo, Desert Fox,
and Horn Bill etc are such examples.
l Teak, Bamboos, Sal, Shisham, Sandalwood, Mulbury
are commercially important trees of these forests. (v) Endemic Species These are species which are only
found in some particular areas usually isolated by
The Thorn Forests and Scrubs Region with less than Natural or Geographical barriers. Examples of such
70 cm of rainfall, the natural vegetation consists of species are: the Andaman Teal, Nicobar Pigeon,
thorny trees and bushes. Andaman wild Pig, Mithun in Andhra Pradesh.
l This type of forests found in semi-arid areas of Gujarat, (vi) Extinct Species These are species which are not
Rajasthan, Madhya Pradesh. found after searches of known or likely areas where
l Acacias, Palms, Euphorbias and Cacti, are the main they may occur. Asiatic Cheetah, Pink Head Duck
plant species of these forests. are examples of such species.
e.g., Asiatic Cheetah The world’s fastest land
Montane Forests These forests found between a Mammal is a unique and specialised member of the
height of 1000 and 2000 metres. cat family can move at the speed of 112 km/h declared
l Evergreen broad-leaf trees such as Oaks and Chestnuts extinct in India.
are trees of these forests.
Mangrove Forests The Mangrove tidal forests are Points to Remember
found in the areas of coasts influlenced by tides. l A chemical compound called ‘Taxol’ is extracted from the bark,
l The deltas of the Ganga, the Mahanadi, the Krishna, the needles, twings and roots of the tree named Taxus Wallachiane
Godavari and the Kaveri are covered by such vegetation. (Himalayan Yew) used to treat cancers—The drug is now the biggest
selling anti-cancer drug in the world.
l In the Ganga-Brahamaputra delta ‘Sundari’ trees are l
The species is under great threat due to over exploitation.
found, which provide durable hard timber. l
The Major threats to tiger population are such as poaching for trade,
l There are different categories of existing plants and shrinking habitat, growing human population etc.
animal species. l
‘Project tiger’ one of the well publicised wildlife compaigns in the
Classification Based on the International Union for world was launched in 1973.
l
Beej Bachao Andolan and Navdanya programmes by formers and
Conservation of Nature and Natural Resources (IUCN). citizens groups have shown that adequate levels of crop production
We can classify existing plants and animals as can be done without using synthetic chemicals.
(i) Normal Species Species whose population levels
are considered to be normal for their survival such as Depletion of Forests and Wild Life
Cattle, Sal, Pine, Rodents etc. l
Expansion of railways, agriculture, commercial and scientific forestry.
(ii) Endangered Species These are species which are
l
Mining The Buxa Tiger Reserve in West Bengal is the extened by
in danger of extinction. The examples of such species dolomile mining.
are black buck, Indian wild ass, Indian rhino, Lion
l
Wrong methods of farming especially due to ‘slash and burh’ practice
tailled Macaque etc. (shifting cultivation).
l
Large scale development projects e.g., Narmada Sagar Project in
(iii) Vulnerable Species These are species whose Madhya Pradesh.
population has declined to levels from where it is l
Habitat destruction, hunting, environmental polution are ether
likely to move into the Endangered category in the factors.
near future. The examples of such species are Blue
Sheep, Asiatic Elephant, Gangetic Dolphin etc.
Target Exercise
Elementary Level
1. Which one of the following types of resources is iron 9. On the basis of ownership, which of the following
ore? resources is a part of it?
(a) Renewable (b) Flow (a) Individual (b) Community
(c) Biotic (d) Non-renewable (c) National and international (d) All of these

2. Under which of the following types of resource can 10. Materials in the environment which have the potential
tidal energy be put? to satisfy human needs, but human being do not have
(a) Replenishable (b) Abiotic the appropriate technology to access known as
(c) Human-made (d) Non-recyclable (a) reserve (b) stock
(c) potential (d) sustainability
3. Which one of the following is the main cause of land
degradation in Punjab? 11. The oceanic resources beyond 200 nautical miles of
(a) Intensive cultivation (b) Overgrazing the exclusive economic zone is an example of
(c) Over irrigation (d) Deforestation (a) abiotic resources (b) national resources
(c) international resources (d) oceanic resources
4. In which of the following states is terrace cultivation
practised? 12. All the minerals, water resources, forests, wildlife
(a) Punjab (b) Haryana land within the political boundaries and oceanic area
(c) Uttar Pradesh (d) Uttarakhand up to ....... nautical miles from the coast termed as
5. In which of the following states is black soil found? territorial water.
(a) 12 (b) 200 (c) 15 (d) 100
(a) Jammu and Kashmir (b) Rajasthan
(c) Gujarat (d) Jharkhand 13. Which of the following factor is responsible for land
6. Which of the following does not make substance a degradation in India?
resource? (a) Mining (b) Quarrying
(c) Overgrazing (d) All of these
(a) Utility (b) Value
(c) Quantity (d) None of these 14. Mining caused land degradation in which of the
7. Which of the following is a human made resource? following states?
(a) Medicines to treat patient (b) Spring water (a) Jharkhand (b) Madhya Pradesh
(c) Tropical forests (d) None of these (c) Odisha (d) All of these

8. Biotic resources are 15. The soil which contains potash, phospheric acid and
(a) derived from living thing (b) made by human being lime is an example of
(c) derived from non-living (d) None of these (a) alluvail soil (b) black soil
(c) arid soil (d) laterite soil

High Skill Questions


1. Which of the following statements is not an argumnt (b) Evolving a planning structure endowed with appropriate
in favour of multi-purpose river projects? technology
(a) Multi-purpose projects bring water to those areas which (c) Matching the resource development plans with overall
suffer from water scarcity national development plans
(b) Multi-purpose projects by regulating water flow helps to (d) None of the above
control floods
3. The National Forest Policy (1952) considered
(c) Multi-purpose projects lead to large scale displacements
and loss of livelihood
essential for maintenance of
(d) Multi-purpose projects generate electricity for our industries (a) land resources
and our homes (b) natural vegetation
(c) ecological balance
2. Which of the following features is not a part of (d) forest resources
resource planning in India?
4. ‘Soil’ as resource is an example of
(a) Identification and inventory of resources across the regions
(a) renewable (b) non-renewable
of the country
(c) biotic (d) abiotic
Resources and Development 623
5. Match the following lists 8. Match the following lists
List I List I List II
List II
(Activities Responsible A. Terrace Farming 1. Rows of tree plantation
(Effected States)
for Land Degrading) B. Strip Cropping 2. Top soil washed away
A. Mining 1. Maharashtra C. Shelter Belts 3. Western and Central
B. Deforestation 2. Jharkhand Himalyas
C. Overgrazing 3. Punjab D. Sheet Erosion 4. Grass are left to grow
D. 4. Madhya Pradesh between the crops
Overirrigation
Codes
Codes
A B C D A B C D
A B C D A B C D
(a) 3 4 1 2 (b) 4 3 2 1
(a) 2 4 1 3 (b) 1 2 3 4
(c) 1 2 3 4 (d) 2 1 4 3
(c) 3 2 1 4 (d) 2 3 1 4
9. Development should take place without damaging
6. Match the following lists
the environment and development in the present
List I List II should not compromise with the needs of the future
(Type of Soil) (Found in)
generations known as
A. Alluvial soil 1. Rajasthan (a) economic growth
B. Black soil 2. Karnataka (b) economic development
C. Laterite soil 3. Deccan plateau (c) sustainable development
D. Arid soil 4. Himalayan river system (d) resource development
Codes 10. Which of the following statements is/are true about
A B C D A B C D development of resources?
(a) 3 4 2 1 (b) 4 3 2 1
(a) Sustainable existence of all forms of life is the basic requirement
(c) 1 2 3 4 (d) 2 1 4 3
for resource planning
7. Ploughing along the contour lines can decelerate the (b) Sustainable existence is a component of sustainable
flow of water down the slop is known as development
(a) agriculture (b) cultivation (c) Equitable distribution of resources has became essential for
(c) contour ploughing (d) None of these a sustained quality of life and global peace
(d) All of the above

Answers
Elementary Level
1. (d) 2. (a) 3. (b) 4. (d) 5. (c) 6. (c) 7. (c) 8. (a) 9. (d) 10. (b)
11. (c) 12. (a) 13. (d) 14. (d) 15. (a)

High Skill Questions


1. (c) 2. (d) 3. (c) 4. (a) 5. (a) 6. (b) 7. (c) 8. (a) 9. (c) 10. (d)
Chapter

6
Agriculture
In India, two-thirds of the population is engaged in Jhumming
agricultural activities. Agriculture is a primary activity, The ‘slash and burn’ is an agricultural technique which involves cutting
which produces most of the food that we consume. and burning of forests or woodlands to create fields.
Besides food grains, it also produces raw materials for The ‘slash and burn’ practice is known as Milpa in Mexico and Central
various industries. America, Conuco in Venezuela, Roca in Brazil, Masole in Central Africa,
Ladang in Indonesia and Ray in Vietnam. In India, this primitive form of
Types of Farming cultivation is called Bewar or Dahiya in Madhya Pradesh, Podi or Penda
in Andhra Pradesh ‘Pama Dabi’ or ‘Koman’ in Brmga’ in Odisha, Kuniari
Farming varies from subsistence to commercial type. At in Western Ghats, ‘Valre' or 'Waltre' in South-Eastern Rajasthan, ‘Khil’ in
the Himalayan belt, 'Kuruwa' in Jharkhand, and ‘Jhumming’ in the
present, in different parts of India, the following farming North-Eastern regions.
systems are practised

Primitive Subsistence Farming Intensive Subsistence Farming


Primitive subsistence agriculture is practised on small This type of farming is practised in areas of high
patches of land with the help of primitive tools like hoe, population pressure on land. It is labour intensive
dao and digging sticks and family/community labour. farming, where high dose of biochemical inputs and
l This type of farming depends upon monsoon, natural irrigation are used for obtaining higher production.
fertility of the soil and suitability of other
environmental conditions to the crops grown. Commercial Farming
l The practice of ‘slash and burn’ is used in this type of The main characteristic of this type of farming is the use
farming. of higher dose of modern inputs, e.g., High Yielding
l When the soil fertility decreases, the farmers shift and Variety (HYV) seeds, chemical fertilizers,
clear a fresh patch of land for cultivation. insecticides and pesticides in order to obtain higher
l This type of shifting allows nature to replenish the productivity.
fertility of the soil through natural processes; land
l The degree of commercialisation of agriculture varies
productivity in this type of agriculture is low as the
from one region to another. e.g., rice is a commercial crop
farmers do not use fertilizers or other modern inputs. It
in Haryana and Punjab, but in Odisha, it is a
is known by different names in different parts of the
country. subsistence crop.

l It is Jhumming in North-Eastern states like; Asom, l Plantation is also a type of commercial farming. In this
Meghalaya, Mizoram and Nagaland, Pamlou in type of farming, a single crop is grown on a large area.
Manipur, Dipa in Bastar district of Chhattisgarh, and The plantation has an interface of agriculture and
in Andaman and Nicobar Islands. industry.
Agriculture 625
l In India—tea, coffee, rubber, sugarcane, banana etc., variations in soil, climate and cultivation practices. Major
are important plantation crops. Tea in Asom and North crops grown in India are: rice, wheat, millets, pulses, tea,
Bengal, coffee in Karnataka are some of the important coffee, sugarcane, oil seeds, cotton and jute, etc.
plantation crops grown in these states.
Rice
Cropping Pattern l Rice is the staple food crop of a majority of the people
India has three cropping seasons — Rabi, Kharif and in India. Our country is the second largest producer of
Zaid. rice in the world after China.
l It is a kharif crop which requires high temperature,
Rabi Crops (above 25°C) and high humidity with annual rainfall
above 100 cm. In the areas of less rainfall, it grows with
l Rabi crops are sown in winter from October to December
the help of irrigation.
and harvested in summer from April to June. Some of
the important rabi crops are: wheat, barley, peas, gram l Rice is grown in the plains of North and North-Eastern
and mustard. India, coastal areas and the deltaic regions.
l States from the North and North-Western parts such as l Development of dense network of canal irrigation and
Punjab, Haryana, Himachal Pradesh, Jammu and tubewells have made it possible to grow rice in areas of
Kashmir, Uttarakhand and Uttar Pradesh are important less rainfall such as Punjab, Haryana and Western
for the production of wheat and other rabi crops. Uttar Pradesh and parts of Rajasthan.
l Availability of precipitation during winter months due
to the Western temperate cyclones and the success of the Wheat
green revolution in Punjab, Haryana, Western Uttar Wheat is the second most important cereal crop. It is the
Pradesh and parts of Rajasthan has also been an main food crop, in North and North-Western parts of the
important factor in the growth of the above mentioned country.
rabi crops.
l This rabi crop requires a cool growing season and the
Kharif Crops bright sunshine at the time of ripening. It requires 50 to
75 cm of annual rainfall evenly distributed over the
l Kharif crops are grown with the onset of monsoon in
growing season.
different parts of the country and these are harvested in
September-October. l There are two important wheat-growing zones in the
country - the Ganga-Sutlej plains in the North-West and
l Important crops grown during this season are: paddy,
black soil region of the Deccan. The major
maize, jowar, bajra, tur (arhar), moong, urad, cotton,
wheat-producing states are Punjab, Haryana, Uttar
jute, groundnut and soyabean.
Pradesh, Bihar, Rajasthan and parts of Madhya
l The most important rice-growing regions are: Asom, Pradesh.
Paschim Banga, coastal regions of Odisha, Andhra
Pradesh, Tamil Nadu, Kerala and Maharashtra, Millets
particularly the Konkan coast along with Uttar Pradesh
and Bihar. Jowar, bajra and ragi are the important millets grown
l Recently, paddy has also become an important crop of in India. Though, these are known as coarse grains, they
Punjab and Haryana. have very high nutritional value. e.g., ragi is very rich in
iron, calcium, other micro nutrients and roughage.
l In states like; Asom, Paschim Banga and Odisha, three
crops of paddy are grown in a year. These are Aus, Aman l Jowar is the third most important food crop with respect
and Bora. to area and production. It is a rain-fed crop mostly grown
in the moist areas which hardly needs irrigation.
Zaid Crops l Maharashtra is the largest producer of jowar followed by
l In between the rabi and the kharif seasons, there is a Karnataka, Andhra Pradesh and Madhya Pradesh.
short season during the summer months known as the l Bajra grows well on sandy soils and shallow black soil.
Zaid season. Rajasthan is the largest producer of bajra followed by
l Some of the crops produced during zaid are: Uttar Pradesh, Maharashtra, Gujarat and Haryana.
watermelon, muskmelon, cucumber, vegetables and l Ragi is a crop of dry regions and grows well on red, black,
fodder crops, sugarcane takes almost a year to grow. sandy, loamy and shallow black soils. Karnataka is the
largest producer of ragi followed by Tamil Nadu. Apart
Major Crops from these states, Himachal Pradesh, Uttarakhand,
Sikkim, Jharkhand and Arunachal Pradesh are also
A variety of food and non-food crops are grown in important for the production of ragi.
different parts of the country depending upon the
626 Study Package for NTSE

Maize l Linseed and mustard are rabi crops. Sesame is a kharif


crop in North, and rabi crop in South India. Castor seed
Maize is a kharif crop which requires temperature is grown both as rabi and kharif crops.
between 21°C to 27°C and grows well in old alluvial soil.
It is a crop which is used both as food and fodder. Tea
l In some states like; Bihar maize is grown in rabi season Tea cultivation is an example of plantation agriculture. It
also. Use of modern inputs such as HYV seeds,
is also an important beverage crop in India.
fertilizers and irrigation have contributed to the
increasing production of maize. l The tea plantation is owned by Indians. The tea plant
l Major maize-producing states are Karnataka, Uttar grows well in tropical and sub-tropical climates endowed
Pradesh, Bihar, Andhra Pradesh and Madhya Pradesh. with deep and fertile well-drained soil, rich in humus
and organic matter.
Pulses l Tea bushes require warm and moist frost-free climate
all through the year. Frequent showers evenly
India is the largest producer as well as the distributed over the year ensure continuous growth of
consumer of pulses in the world. These are the major tender leaves.
source of protein in a vegetarian diet. l Tea is a labour intensive industry. It requires abundant,
l Major pulses that are grown in India are tur (arhar), cheap and skilled labour. Tea is processed within the tea
urad, moong, masur, peas and gram. Pulses need less garden to restore its freshness.
moisture and survive even in dry conditions. l Major tea producing states are Asom, hills of Darjeeling
l All these crops except arhar help in restoring soil and Jalpaiguri districts, Paschim Banga, Tamil Nadu
fertility by fixing nitrogen from the air. Therefore, these and Kerala.
are mostly grown in rotation with other crops.
l Apart from these, Himachal Pradesh, Uttarakhand,
l Major pulse-producing states in India are Madhya Meghalaya, Andhra Pradesh and Tripura are also
Pradesh, Uttar Pradesh, Rajasthan, Maharashtra and tea-producing states in the country. India is the leading
Karnataka. producer as well as exporter of tea in the world.

Food Crops other than Grains Coffee


India produces about 4% of the world’s coffee
Sugarcane production. Indian coffee is known in the world for its
Sugarcane is a tropical as well as a subtropical crop. It good quality.
grows well in hot and humid climate with a temperature The Arabica variety initially brought from Yemen is
of 21°C to 27°C and an annual rainfall between 75 cm produced in the country. This variety is in great demand
and 100 cm. all over the world. Initially its cultivation was
l It can be grown on a variety of soils and needs manual introduced on the Baba Budan Hills and even today its
labour from sowing to harvesting. cultivation is confined to the Nilgiri in Karnataka,
l India is the second largest producer of sugarcane only Kerala and Tamil Nadu.
after Brazil. It is the main source of sugar, gur (jaggary),
khandsari and molasses.
l The major sugarcane producing states are Uttar
Horticulture Crops
Pradesh, Maharashtra, Karnataka, Tamil Nadu, India is the largest producer of fruits and vegetables in
Andhra Pradesh, Bihar, Punjab and Haryana. the world. India is a producer of tropical as well as
temperate fruits.
Oil Seeds l Mangoes of Maharashtra, Andhra Pradesh, Uttar
Pradesh and Paschim Banga; oranges of Nagpur and
India is the largest producer of oil seeds in the world. Cherrapunjee (Meghalaya); bananas of Kerala,
Different oil seeds are grown covering approximately Mizoram, Maharashtra and Tamil Nadu.
12% of the total cropped area of the country. l Lichi and guava of Uttar Pradesh and Bihar;
l Main oil seeds produced in India are groundnut, pineapples of Meghalaya; grapes of Andhra Pradesh
mustard, coconut, sesame (til), soyabean, castor seeds, and Maharashtra; apples, pears, apricots and walnuts of
cotton seeds, linseed and sunflower. Jammu and Kashmir and Himachal Pradesh are in
l Groundnut is a kharif crop and accounts for about half of great demand the world over.
the major oilseeds produced in the country. l India produces about 13% of the world’s vegetables.
l Andhra Pradesh is the largest producer of groundnut India is a prominent country to produce pea,
followed by Tamil Nadu, Karnataka, Gujarat and cauliflower, onion, cabbage, tomato, brinjal and potato
Maharashtra. in the world.
Agriculture 627
Non-Food Crops fragmentation of land holdings necessitating
consolidation of holdings. The laws of land reforms were
Rubber enacted but the laws of implementation was lacking or
l Rubber is an equatorial crop, but under special
lukewarm.
conditions, it is also grown in tropical and sub-tropical l The Green Revolution (1970s) based on the use of
areas. It requires moist and humid climate with rainfall package technology and the White Revolution
of more than 200 cm and temperature above 25°C. (Operation Flood) were some of the strategies initiated
to improve the lot of Indian agriculture. But, this too led
l Rubber is an important industrial raw material. It is
to the concentration of development in few selected
mainly grown in Kerala, Tamil Nadu, Karnataka and
areas. High yielding wheat was first introduced to India
Andaman and Niooobar Islands and Garo hills of
in 1968 by Norman Borlaug. MS Swaminathan is known
Meghalaya. India ranks 5th among the world’s natural
as ‘Father of Green Revolution in India’.
rubber producers.
l In the 1980s and 1990s, a comprehensive land
development programme was initiated, which included
Fibre Crops both institutional and technical reforms.
Cotton, jute, hemp and natural silk are the four major
l Provisions for crop insurance against drought, flood,
fibre crops grown in India. The first three are derived cyclone, fire and disease, establishment of Grameen
from the crops grown in the soil, the latter is obtained banks, cooperative societies and banks for providing
from cocoons of the silkworms fed on green leaves loan facilities to the farmers at lower rates of interest
specially mulberry. were some important steps in this direction.
l Kissan Credit Card (KCC), Personal Accident Insurance
Cotton Scheme (PAIS) are some other schemes introduced by
l India is the third-largest producer of cotton in the world. the Government of India for the benefit of the farmers.
Cotton grows well in drier parts of the black cotton soil of l Moreover, special weather bulletins and agricultural
the Deccan plateau. lt requires high temperature, light programmes for farmers were introduced on the radio
rainfall or irrigation, 210 frost-free days and bright and television.
sunshine for its growth. l The government also announces Minimum Support
l It is a kharif crop and requires 6 to 8 months to mature. Price (MSP), remunerative and procurement prices for
Major cotton-producing states are Maharashtra, important crops to check the exploitation of farmers by
Gujarat, Madhya Pradesh, Karnataka, Andhra speculators and middlemen.
Pradesh, Tamil Nadu, Punjab, Haryana and Uttar
Pradesh. Bhoodan – Gramdan
Mahatma Gandhi declared Vinoba Bhave as his spiritual heir. He also
Jute participated in Satyagraha as one of he foremost satyagrahis. He was one
l Jute is known as the golden fibre. Jute grows well on of he votaries of Gandhi’s concept of gram swarajya. After Gandhiji’s
well-drained fertile soils in the flood plains where soils martyrdom, Vinoba Bhave undertook padyatra to spread Gandhiji’s
are renewed every year. High temperature is required message covered almost the entire country. Once, when he was
during the time of growth. delivering a lecture of Pochampalli in Andhra Pradesh, some poor landless
villagers demanded some land for their economic well-being. Vinoba
l Paschim Banga, Bihar, Asom, Odisha and Meghalaya
Bhave could not promise it to them immediately but assured them to
are the major jute producing states. It is used in making
talk to the Government of India regarding provision of land for them, if
gunny bags, mats, ropes, yarn, carpets and other they undertook cooperative farming.
artefacts. Due to its high cost, it is losing market to
Suddenly, Shri Ram Chandra Reddy stood up and offered 80 acres of land
synthetic fibres and packing materials, particularly the
to be distributed among 80 landless villagers. This act was known as
nylon.
‘Bhoodan’. Later he travelled and introduced his ideas widely all over India.
Some zamindars, owners of many villages offered to distribute some
Technological and Institutional villages among the landless. It was known as Gramdan. However, many
land-owners choose to provide some part of their land to the poor
Reforms farmers due to the fear of land ceiling act. This Bhoodan-Gramdan
‘Land reform’ was the main focus of our First Five Year movement initiated by Vinoba Bhave is also known as the Blood-less
Plan. The right of inheritance had already lead to Revolution.
628 Study Package for NTSE

Contribution of Agriculture to the National Economy : Employment and Output


India : Growth of GDP in Major Sector (in %)
2002-07
Sector 1980-90 1992-2001
(Tenth plan projected)
Agriculture 3.6 3.3 4.0
Industries 7.1 6.5 9.5
Services 6.7 8.2 9.1
GDP 5.6 6.4 8.0
Source : Tenth Five Year Plan. 2002-07

From the table, it is clear that though the GDP growth rate is increasing over the years, it is not generating
sufficient employment opportunities in the country. The growth rate in agriculture is decelerating which is an
alarming situation.

Target Exercise
Elementary Level
1. Agriculture activities falls under which of the (c) These crops are grown in large parts of India
following activities? (d) None of the above
(a) Primary (b) Secondary
7. Which of the following is not an crop of Kharif
(c) Tertiary (d) None of these
seasons?
2. Which of the following is an feature of primitive (a) Rice (b) Cotton (c) Soyabean (d) Mustard
subsistence farming?
8. The crop sown in between the rabi and kharif seasons
(a) Farming depends on monsoon
known as
(b) Natural fertility of the soil
(a) Jhoom farming (b) Horticulture
(c) Suitability of other environmental conditions to the crops
(c) Zaid (d) None of these
grown
(d) All of the above 9. Which of the following are the crops produced during
3. The system of agriculture in which farmer clear a ‘Zaid’ season?
patch of land and produce crops. When the soil (a) Watermelon (b) Muskmelon
(c) Cucumber (d) All of these
fertility decreases, farmer shift and clear a fresh
patch of land, known as 10. Which of the following countries is the largest producer
(a) Jhum farming (b) Alternate farming of rice in the world?
(c) Commercial farming (d) None of these (a) India (b) China
(c) Bangladesh (d) Brazil
4. Which of the following is the feature of commercial
farming 11. Which of the following is the important wheat
(a) Use of High Yielding Variety (HYV) seeds growing zones in the country?
(b) use of chemical fertilizers, insecticides, and pesticides (a) Ganga-Sutlej plains
(c) The degree of commercialisation of agriculture varies from
(b) Black soil regions of Deccan
one region to another
(c) Both ‘a’ and ‘b’
(d) All of the above
(d) None of the above
5. Which one of the following is not an plantation crop?
(a) Tea (b) Coffee
12. Rajasthan is the largest producer of which of the
(c) Rubber (d) Pulses
following crops?
(a) Bajra (b) Ragi (c) Maize (d) Millets
6. Which of the following statements is not true about
13. India is the largest producer as well as the consumer
rabi crops?
of
(a) Rabi crops are sown in July-August and harvested in
(a) pulses (b) millets
September-October
(c) ragi (d) maize
(b) Wheat, barley, gram, mustard are important rabi crops
Agriculture 629
14. Which of the following is the tropical as well as 15. Which of the following is the largest producer of oil
subtropical crop? seeds in the world?
(a) Sugarcane (b) Oil seeds (a) India (b) Brazil
(c) Tea (d) Coffee (c) USA (d) Bangladesh

High Skill Questions


1. Which of the following is not an oil seed crop? 8. Match the following lists
(a) Groundnut List I List II
(b) Soyabean
(c) Sesame A. Sericulture 1. Breeding of fish
(d) None of the above B. Pisciculture 2. Rearing of silk worms
C. Viticulture 3. Growing of fruits, vegetables for
2. Rubber is an important industrial raw material commercial use
grown in which of the following states in India? D. Horticulture 4. Cultivation of grapes
(a) Kerala (b) Tamil Nadu
(c) Karnataka (d) All of these Codes
A B C D A B C D
3. Which of the following is not an fibre crop? (a) 2 1 4 3 (b) 1 2 3 4
(a) Cotton (b) Jute (c) 2 1 3 4 (d) 4 3 1 2
(c) Hemp (d) None of these
9. Match the following lists
4. Which of the following crops is known as ‘Golden
List I List II
Fibre’?
(a) Jute (b) Silk A. Rubber 1. Karnataka
(c) Hemp (d) Cotton B. Jute 2. Paschim Banga
C. Tea 3. Kerala
5. Which of the following is the components of food D. Coffee 4. Asom
security system in India?
(a) Buffer stock Codes
(b) Public Distribution System A B C D A B C D
(c) Both ‘a’ and ‘b’ (a) 1 2 3 4 (b) 3 2 4 1
(c) 2 3 4 1 (d) 3 2 1 4
(d) None of the above
10. Match the following lists
6. Which of the following describes a system of
agriculture, where a single crop is grown on a large List I List II
area? A. Jhumming 1. Mandya Pradesh
(a) Shifting cultivation B. Bewar 2. Odisha
(b) Plantation agriculture C. Penda 3. North-Eastern States
(c) Horticulture D. Koman 4. Andhra Pradesh
(d) Intensive agriculture
Codes
7. Which of the following is a leguminious crop? A B C D A B C D
(a) Pulses (b) Jowar (a) 3 1 4 2 (b) 3 1 2 4
(c) Millets (d) Sesame (c) 1 2 3 4 (d) 2 1 4 3

Answers
Elementary Level
1. (a) 2. (d) 3. (a) 4. (d) 5. (d) 6. (a) 7. (d) 8. (c) 9. (d) 10. (b)
11. (c) 12. (a) 13. (a) 14. (a) 15. (a)

High Skill Questions


1. (d) 2. (d) 3. (d) 4. (a) 5. (c) 6. (d) 7. (b) 8. (a) 9. (c) 10. (a)
Chapter

7
Manufacturing Industries
Production of goods in large quantities after processing Inter-dependence of Agriculture
from raw materials to more valuable products is called and Industries
manufacturing. e.g., paper is manufactured from wood,
l Agriculture and industries are not exclusive of each
sugar from sugarcane, iron and steel from iron ore and other.
aluminium from bauxite.
They move hand in hand. For instance, the agro-
People employed in the secondary activities industries in India have given a major boost to
manufacture the primary materials into finished goods. agriculture by raising its productivity.
The workers employed in steel factories, car, breweries, l Industrial sector depends on the agriculture sector for
textile industries, bakeries etc, fall into this category. raw materials and sell their products such as irrigation
The economic strength of a country is measured by the pumps, fertilizers, insecticides, pesticides, plastic and
development of manufacturing industries. PVC pipes, machines and tools etc., to the farmers.
l Thus, development and competitiveness of
Importance of Manufacturing manufacturing industry has not only assisted
agriculturists in increasing their production, but also
Manufacturing sector is considered the backbone of made the production processes very efficient.
development in general and economic development in
particular mainly because Contribution of Industry to National
l Manufacturing industries not only help in modernising Economy
agriculture, which forms the backbone of our economy, l The share of manufacturing sector has stagnated at 17%
they also reduce the heavy dependence of people on of GDP out of a total of 27% for the industry which
agricultural income by providing them jobs in secondary includes 10% for mining, quarrying, electricity and gas
and tertiary sectors. in the last two decades.
l Industrial development is a pre-condition for l The trend of growth rate in manufacturing over the last
eradication of unemployment and poverty from our decade has been around 7% per-annum. This is much
country. This was the main philosophy behind public lower in comparison to some East Asian economies,
sector industries and joint sector ventures in India. It where it is 25 to 35%.
was also aimed at bringing down regional disparities by
l The National Manufacturing Competitiveness Council
establishing industries in tribal and backward areas.
(NMCC) has been set up with the objective of increasing
l Export of manufactured goods expands trade and competitiveness and growth in the manufacturing sector.
commerce, and brings in much needed foreign exchange.
Manufacturing Industries 631
Industrialisation and Urbanisation (iv) Co-operative sector industries are owned and
operated by the producers or suppliers of raw
materials, workers or both.
l Industrialisation and urbanisation go hand in hand.
e.g., The co-operatives pool in the resources and
Cities provide markets and also provide services such as share the profits or losses proportionately such as
banking, insurance, transport, labour, consultants and the sugar industry in Maharashtra, the coir industry
financial advice etc., to the industry. in Kerala.
l Many industries tend to come together to make use of l Based on the bulk and weight of raw material and

the advantages offered by the urban centres known as finished goods


agglomeration economies. Gradually, a large industrial (i) Heavy industries such as iron and steel.
agglomeration takes place. (ii) Light industries that use light raw materials and
l In the pre-independence period, most manufacturing produce light goods such as electrical industries.
units were located in places from the point of view of
overseas trade such as Mumbai, Kolkata, Chennai, etc. Agro Based Industries
l Consequently, there emerged certain pockets of Cotton, jute, silk, woollen textiles, sugar and edible oil,
industrially developed urban centres surrounded by a etc are the industries based on agricultural raw
huge agricultural rural hinterland. materials.

Classification of Industries Textile Industry


l The textile industry occupies unique position in the
Industries may be classified as follows Indian Economy, because it contributes significantly to
l There are two types of industries based on the source of industrial product 14%), employment generation (35
raw materials. They are million persons directly the second largest after
agriculture) and foreign exchange earnings (about 24.6%).
(i) Agro based industries
e.g., cotton, woollen, jute, silk textile, rubber and l Textile industries contributes 4% towards GDP. It is the
sugar, tea, coffee, edible oil. only industry in the country, which is self-reliant and
(ii) Mineral Based Industries complete in the value chain i.e., from raw material to the
e.g., iron and steel, cement, aluminium, machine highest value added products.
tools, petrochemicals.
l According to main role played by the industries Cotton Textiles
(i) Basic or key industries which supply their products There are nearly 1600 cotton and human made fibre
or raw materials to manufacture other goods. textile mills in the country. About 80% of these are in
e.g., iron and steel and copper smelting, aluminium the private sector and the rest in the public and
smelting.
co-operative sectors. In the early years, the cotton
(ii) Consumer industries that produce goods for direct
textile industry was concentrated in the cotton growing
use by consumers.
e.g., sugar, toothpaste, paper, sewing machines, fans belt of Maharashtra and Gujarat.
etc. l Availability of raw cotton, market, transport including
l On the basis of capital investment accessible port facilities, labour, moist climate, etc had
A small scale industry is defined with reference to contributed towards its localisation.
the maximum investment allowed on the assets of a l This industry has close links with agriculture and
unit. This limit has changed over a period of time. At provides a living to farmers, cotton boll pluckers and
present, the maximum investment allowed is rupees workers engaged in ginning, spinning, weaving, dyeing,
one crore. If investment is more than one crore on designing, packaging, tailoring and sewing.
any industry then it is known as a large scale
industry. l The industry by creating demands supports many other
l On the basis of ownership of industries industries, such as, chemicals and dyes, mill stores,
packaging materials and engineering works.
(i) Public sector, owned and operated by government
agencies. l India has world class production in spinning, but
e.g., BHEL, SAIL etc. weaving supplies low quality of fabric as it cannot use
(ii) Private sector industries owned and operated by much of the high quality yarn produced in the country.
individuals or a group of individuals. Weaving is done by handloom, powerloom and in mills.
e.g., TISCO, Bajaj Auto Limited, Dabur Industries. l India exports yarn to Japan. Other importers of cotton
(iii) Joint sector industries which are jointly run by the goods from India are USA, UK, Russia, France, East
state and individuals or a group of individuals. European countries, Nepal, Singapore, Sri Lanka, and
e.g., Oil India Limited (OIL) is jointly owned by African countries.
public and private sectors.
632 Study Package for NTSE

l India has the second largest installed capacity of Karnataka, Tamil Nadu, Andhra Pradesh and Gujarat
spindles in the world, next to China, at around 34 along with Punjab, Haryana and Madhya Pradesh.
million (2003-04). l Sugar industry is seasonal in nature. In recent years,
l The trade of cotton yarn, accounting for one-fourth of the there is a tendency for the mills to shift and concentrate
total trade in the world. However, our trade in garments in the Southern and Western states, especially in
is only 4% of the world’s total. Maharashtra. This is because the cane produced here
has a higher sucrose content.
Jute Textiles l The cooler climate also ensures a longer crushing
India is the largest producer of raw jute and jute goods season. Moreover, the co-operatives are more successful
in these states.
and stands at second place as an exporter after
Bangladesh. l Major challenges include the seasonal nature of the
industry, old and inefficient methods of production,
l There are about 70 jute mills in India. Most of these are
transport delay in reaching cane to factories and the
located in Paschim Banga, mainly along the banks of the
need to maximise the use of baggase.
Hooghli river, in a narrow belt (98 km long and 3 km
wide).
Mineral Based Industries
l The first jute mill was set up near Kolkata in 1859 at
Rishra. Industries that use minerals and metals as raw
materials are called mineral based industries.
Factors Responsible for their Location in the
Hooghli Basin Iron and Steel Industry
l Inexpensive water transport, and was supported by a The iron and steel industry is the basic industry. All
good network of railways, roadways and waterways to other industries such as heavy, medium and light,
facilitate movement of raw material to the mills. depends on it for their machinery. Steel is needed to
l Abundant water for processing raw jute, cheap labour manufacture a variety of engineering goods, construction
from Paschim Banga and adjoining states of Bihar, material, defence, medical, telephonic, scientific equipment
Odisha and Uttar Pradesh.
and a variety of consumer goods.
l Kolkata as a large urban centre provides banking,
l The total production of the steel in is 32.8 million fans.
insurance and port facilities for export of jute goods.
India ranks ninth among the world crude steel
l The jute industry supports 2.61 lakh workers directly producers. India is the largest producer of sponge iron.
and another 40 lakhs small and marginal farmers who
l Most of the public sector undertakings market their
are engaged in cultivation of jute and mesta
steel through Steel Authority of India Limited (SAIL)
Challenges for the jute Industries.
while TISCO markets its produce by itself in the name of
l Stiff competition in the international market from Tata Steel.
synthetic substitutes and from other competitors like
l In the 1950s, China and India produced almost the
Bangladesh, Brazil, Philippines, Egypt and Thailand.
same quantity of steel. Today, China is the largest
l The internal demand has been on the increase due to the producer of steel. China is also the world’s largest
Government policy of mandatory use of jute packaging. consumer of steel.
l In 2005, National Jute Policy was formulated with the l In 2004, India was the largest exporter of steel which
objective of increasing productivity, improving quality, accounted for 2.25% of the global steel trade.
ensuring good prices to the jute farmers and enhancing
the yield per hectare. Challenges
l The main markets are USA, Canada, Russia, United Though, India is an important iron and steel producing
Arab Republic, UK and Australia. The growing global country in the world yet, we are not able to perform to
concern for environment friendly, bio-degradable
materials, has once again opened the opportunity for
our full potential largely due to
jute products. l High costs and limited vailability of coking coal.
l Lower productivity of labour.
Sugar Industry
l Irregular supply of energy.
l India stands second as a world producer of sugar, but
l Poor infrastructure.
occupies the first place in the production of gur and
khandsari. India also import good quality steel from other
l Sixty per cent Mills are in Uttar Pradesh and countries. However, the overall production of steel is
Bihar. There are over 460 sugar mills in the country sufficient to meet our domestic demand.
spread over Uttar Pradesh, Bihar, Maharashtra,
Manufacturing Industries 633
Aluminium Smelting l Gujarat, Tamil Nadu, Uttar Pradesh, Punjab and
Kerala contribute towards half the fertilizer production.
Aluminium smelting is the second most important
metallurgical industry in India. It is light, resistant to Cement Industry
corrosion, a good conductor of heat, mallable and
The cement industry requires bulky and heavy raw
becomes strong when it is mixed with other metals. It is
materials like limestone, silica, alumina and gypsum.
used to manufacture aircraft, utensils and wires. It has
gained popularity as a substitute of steel, copper, zinc Coal and electric power are needed apart from rail
and lead in a number of industries. transportation. The industry has strategically located
plants in Gujarat that have suitable access to the
l There are 8 aluminium smelting plants in the country
market in the gulf countries.
located in Odisha (Nalco and Balco), Paschim Banga,
Kerala, Uttar Pradesh, Chhattisgarh, Maharashtra and The first cement plant was set up in Chennai in 1904.
Tamil Nadu. In 2004, India produced over 600 million After independence, the industry expanded. Decontrol
tonnes of aluminium. of price and distribution since 1989 and other policy
l Bauxite, the raw material used in the smelters is a very reforms led the cement industry to make rapid strides in
bulky, dark reddish coloured rock. capacity, process, technology and production. India
produces a variety of cement. There are 128 large plants
Chemical Industries and 332 mini cement plants in the country.
The chemical industry in India is fast growing and
diversifying. It contributes approximately 3% of the Automobile Industry
GDP. It is the third largest in Asia and occupies the Automobiles provide vehicle for quick transport of good
twelfth place in the world in term of its size. It comprises services and passengers. At present, there are 15
both large and small scale manufacturing units. Rapid manufacturers of passenger cars and multiutility
growth has been recorded in both inorganic and organic vehicles, 9 of commercial vehicles, 14 of the two and
sectors. three-wheelers. The industry is located around Delhi,
l Inorganic chemicals include sulphuric acid (used to Gurgaon, Mumbai, Pune, Chennai, Kolkata, Lucknow,
manufacture fertilizers, synthetic fibres, plastics,
Indore, Hyderabad, Jamshedpur and Bengaluru.
adhesives, paints, dye stuffs), nitric acid, alkalies, soda
ash (used to make glass, soaps and detergents, paper)
and caustic soda. These industries are widely spread Information Technology and Electronics
over the country. Industry
l Organic chemicals include petrochemicals, which are l Bengaluru has emerged as the electronic capital of
used for manufacturing of synthetic fibers, synthetic
India. Other important centres for electronic goods are
rubber, plastics, dye-stuffs, drugs and pharmaceuticals.
Mumbai, Delhi, Hyderabad, Pune, Chennai, Kolkata,
l Organic chemical plants are located near oil refineries or Lucknow and Coimbatore.
petrochemical plants.
l A major impact of this industry has been on employment
Fertilizer Industry generation. Upto 31 March, 2005 the IT industry
employed over one million persons.
The fertilizer industry is centred around the production l It is encouraging to know that 30% of the people
of nitrogenous fertilizers (mainly urea), phosphatic employed in this sector are women.
fertilizers and ammonium phosphate (DAP) and
l This industry has been a major foreign exchange earner
complex fertilizers which have a combination of
in the last two or three years because of its fast growing
nitrogen (N), phosphate (P), and potash (K). Business Processes Outsourcing (BPO) sector.
l The potash is entirely imported as the country does not
have any reserves of commercially usable potash or
potassium compounds in any form. Industrial Pollution and
India is the third largest producer of nitrogenous
l

fertilizers. There are 57 fertilizer units manufacturing Environmental Degradation


nitrogenous and complex nitrogenous fertilizers, 29 for
urea and 9 for producing ammonium sulphate as a Industries are responsible for four types of pollution
byproduct and 63 other small units produce single (i) Air, (ii) Water
superphosphate. (iii) Land and (iv) Noise
l At present, there are 10 public sector undertakings and
one in co-operative sector at Hazira in Gujarat under the The polluting industries also include thermal power
Fertilizer Corporation of India. plants.
634 Study Package for NTSE

Air Pollution Noise Pollution


Air Pollution is caused by the presence of high Noise Pollution not only results in irritation and anger,
proportion of undesirable gases, such as sulphur dioxide it can also cause hearing impairment, increased heart
and carbon monoxide. rate and blood pressure among other physiological
l Airborne particulate materials contain both solid and effects.
liquid particles like dust, sprays mist and smoke. l Unwanted sound is an irritant and a source of stress.
Industrial and construction activities, machinery,
l Smoke is emitted by chemical and paper factories, brick
factory equipment, generators, saws and pneumatic and
kilns, refineries and smelting plants, and burning of
electric drills also make a lot of noise.
fossil fuels in big and small factories that ignore
pollution norms.
l Toxic gas leaks can be very hazardous with long-term
Control of Environmental
effects. e.g., the Bhopal Gas tragedy that occurred
have air pollution adversely affects human health,
Degradation
animals, plants, buildings and the atmosphere as a l Reduction of Industrial Production
whole.
(i) minimising use water for processing by reusing and
Water Pollution recycling it in two or more successive stages.
(ii) harvesting of rainwater to meet water requirements.
Water Pollution is caused by organic and inorganic
(iii) treating hot water and effluents before releasing
industrial wastes and affluents discharged into rivers.
them in rivers and ponds.
l The main industries in this regard are paper, pulp,
l Treatment of industrial effluents can be done in three
chemical, textile and dyeing, petroleum refineries,
tanneries and electroplating industries that let out phases.
dyes, detergents, acids, salts and heavy metal like lead l Primary treatment by mechanical means. This involves
and mercury pesticides, fertilizers, synthetic chemicals screening, grinding, flocculation and sedimentation.
with carbon, plastics and rubber, etc into the water l Secondary treatment by biological process.
bodies.
l Tertiary treatment by biological, chemical and physical
l Fly ash, phospo-gypsum and iron and steel slags are the
processes. This involves recycling of waste water.
major solid wastes in India.
(i) Overdrawing of ground water reserves by industry
Thermal Pollution where there is a threat to ground water resources
also needs to be regulated legally.
Thermal Pollution of water occurs when hot water from
(ii) Participate matter in the air can be reduced by fitting
factories and thermal plants is drained into rivers and smoke stacks to factories with electrostatic
ponds before cooling. precipitators fabric filters, scrubbers and inertial
l Dumping of wastes specially glass, harmful chemicals, separators.
industrial effluents, packaging, salts and garbage
(iii) Smoke can be reduced by using of or gas instead of
renders the soil useless. Rain water percolates to the soil coal in factories.
carrying the pollutants to the ground and the ground
water also gets contaminated. Machinery and equipment can be used and generators
should be fitted with silencers.

Target Exercise
Elementary Level
1. Production of goods in large quantities after processing 3. Which of the following factors influenced the
from raw material to more valuable products is called industrial locations?
(a) manufacturing (b) processing (a) Availability of raw material
(c) production (d) None of these (b) Labour and capital
(c) Power and market
2. Manufacturing activities are part of which of the (d) All of these
following sector?
(a) Primary (b) Secondary 4. Which of the following is not an agro based industry?
(c) Tertiary (d) None of the above (a) Silk (b) Tea
(c) Coffee (d) Sewing machines
Manufacturing Industries 635
5. The industries on the basis of ownership are 8. Which of the following countries has the largest
(a) public sector (b) private sector installed capacity of spindles in the world?
(c) co-operative sector (d) All of these (a) India (b) China
(c) Britain (d) USA
6. Basic or key industries means
(a) the industries based on agro-raw materials 9. India is the largest producer of raw jute and jute
(b) an industries where maximum investment allowed on the goods and stands at second place as an exporter after?
assets of a unit (a) Bangladesh
(c) An industry which supply their products or raw materials to (b) China
manufacture other goods (c) Myanmar
(d) None of the above (d) Japan
7. Arrange the following sectors according to their 10. Which of the following is not an mineral based
contribution to the annual GDP of Indian Economy. industry?
(descending order) (a) Iron and steel
(a) Agriculture, industry, tertiary sector (b) Aluminium
(b) Tertiary, industry, agriculture (c) Sugar
(c) Industry, tertiary, agriculture (d) Cement
(d) None of the above

High Skill Questions


1. Which of the following is/are basic or key industries? 7. Noise pollution not only results in irritation and
(a) Iron or steel (b) Copper smelting anger, it can also cause
(c) Aluminium smelting (d) All of these (a) hearing impairment (b) increased heart rate
(c) blood pressure (d) All of the above
2. Most of the public sector undertakings market their
steel through 8. Iron ore, cooking coal and limestone are the chief raw
(a) individually materials of which industry?
(b) Steel Authority of India Limited (SAIL) (a) Steel (b) Aluminium
(c) TISCO (c) Automobile (d) Fertilizer
(d) Steel Industries Corporation
9. The industry has been a major foreign exchange
3. Which of the following industries uses limestone as a earner in the last decade or so because of its fast
raw material?
growing?
(a) Aluminium (b) Cement (c) Sugar (d) Jute
(a) Technology
4. Which of the following industries uses bauxite as a (b) Foreign investment
raw material? (c) Business processes outsourcing
(a) Aluminium (b) Cement (c) Jute (d) Steel (d) None of the above

5. Which of the following industries manufactures 10. Which of the following statement is/are true about
telephones, computer etc. thermal pollution?
(a) Steel (b) Aluminium (a) It occurs when hot water from factories and thermal plants
(c) Electronic (d) Information Technology is drained into rivers and ponds before cooling
(b) Wastes from nuclear power plants causes cancer birth
6. Water pollution is caused by organic and inorganic defects and miscarriages
wastes and affluents discharged into the rivers. The (c) Dumping of wastes specially glass, harmful chemicals,
organic content involves industrial efficient renders the soil useless
(a) paper (b) pulp (d) All of the above
(c) textile and dyeing (d) All of these

Answers
Elementary Level
1. (a) 2. (b) 3. (d) 4. (d) 5. (d) 6. (c) 7. (b) 8. (b) 9. (a) 10. (c)

High Skill Questions


1. (d) 2. (b) 3. (b) 4. (a) 5. (d) 6. (d) 7. (d) 8. (a) 9. (c) 10. (a)
Chapter

8
Minerals and Energy
Resources
A naturally occurring substance that has a definite chemical composition is called a mineral. Minerals are formed
from a certain combination of elements depending on the physical and chemical conditions under which the
mineral forms. This, in turn, results in a wide variety of colours, hardness, crystal forms, lusture and density that a
particular mineral possesses. Geologists use these properties to classify minerals. Some of the minerals occur in
igneous and metamorphic rocks, in cracks, crevices, faults or joints. The smaller occurrences are called veins and
the larger ones are called lodes.
l Major metallic minerals like tin, copper, zinc, lead, etc. are obtained from veins and lodes.
l In sedimentary rocks a number of minerals occur in beds and layers. They have been formed as a result of deposition,
accumulation and concentration in horizontal strata.
l Another mode of formation involves the decomposition of soluble constituents, leaving a residual mass of weathered
material containing ores. Bauxite is formed this way.
l Certain minerals may occur as alluvial deposits in sands of valley floors and the base of hills. Gold, silver, tin and platinum
are most important among such minerals.
l Oean waters contain vast quantities of minerals. Common salt, magnesium and bromine are largely derived from ocean
waters.
l Metallic minerals contain metal in raw form. Metals are hard substances that conduct heat and electricity and have a
characteristic lustre or shine.
l Ferrous minerals contain iron while non-ferrous minerals do not contain iron.

Classification of Minerals
Mineral are classified as

Minerals

Metallic Non-metallic

Mica, Salt, Potash,


Sulphur, Granite,
Ferrous Non-ferrous Precious Limestone, Marble,
Sandstone, and
Iron, ore, Nickel Copper, Lead, Gold, Silver Mineral Fuels
Manganese, Cobalt Tin, Bauxite Platinum
Minerals and Energy Resources 637
Ferrous Minerals
Iron Ore Manganese
— Iron ore is a basic mineral and the backbone of industrial — Mainly used in manufacturing of steel and ferro-manganese alloy.
development.
— Odisha-Jharkhand belt hematite ore is found in Badampahar — Also used in manufacturing bleaching powder, paints and
mines in the Mayurbhanj and Keonjhar districts. insecticides.
— Durg-Bastar-Chandrapur belt lies in Chhattisgarh and — Odisha is the largest producer of managenese ore in India.
Maharashtra. Exported to South Korea and Japan.
— Bellary-Chitradurga-Chikmaglur-Tumkur belt in Karnataka.
Kudremukh mines located in the Western ghats of Karnataka.
— Maharashtra-Goa belt the state of Goa and Ratnagiri district of
Maharashtra exported through Marmagao Port.

Non-Ferrous Minerals

Copper Aluminium
— Being malleable, ductile and a good conductor, copper is mainly — It is from bauxite, a clay-like substance that alumina and later
used in electrical cables, electronics and chemical industries. alluminium is obtained.
— The Balaghat mines-Madhya Pradesh, The Singhbhum — Combines the strength of a metal such as iron, with extreme
-Jharkhand, Khetri mines- Rajasthan are famous mine of copper lightness and also with good conductivity and great malleability.
in India.
— Mainly found in Amarkantak plateau, Maikal hills and the plateau
region of Bilaspur-Katni.
— Odisha is the largest bauxite producing state in India i.e.,
Panchpatmali in Koraput district.

Conservation of Minerals
Non-Metallic Minerals
l The total volume of workable minerals deposits is an
Mica insignificant fraction, about one percent, of the earth’s
crust.
l Mica is a mineral made up of a series of plates or leaves.
It splits into thin plates easily. l We are rapidly consuming mineral resources that
required millions of years to be created and concentrated.
l It can be clear, black, green, red, yellow or brown.
l The geological processes of mineral formation are much
l Due to its di-electric strength, low power loss factor, slower then the rates of their consumption. Mineral
insulating properties and resistance to high voltage, it is resources are finite and non-renewable.
one of the most indispensable minerals used in electrical
and electronic industries. l Continued extraction of ores leads to increasing costs, as
mineral extraction is done from greater depths along
l Its deposits are found in the northern edge of the
with decrease in quality.
Chhotanagpur plateau. Koderma-Gaya- Hazaribagh
belt of Jharkhand is the leading producer of mica. l Improved technologies need to be constantly evolved to
allow refining of low grade ores at low costs.
l In Rajasthan, the major mica producing area is around
Ajmer. Nellore mica belt of Andhra Pradesh is also an l Recycling of metals, using scrap metals and other
important producer in the country. subtitutes are steps in conserving our mineral resources
for the future.
Rock Minerals
l Limestone is found in association with rocks composed of Energy Resources
calcium carbonates or calcium and magnesium carbonates.
l Energy can be generated from fuel minerals like coal,
l It is found in sedimentary rocks of most geological petroleum, natural gas, uranium and converted to
formations. electricity.
l Limestone is the basic raw material for the cement l Energy resources can be classified into conventional and
industry and essential for smelting iron ore in the blast non-conventional sources.
furnace.
638 Study Package for NTSE

l Conventional sources of energy are those which have l Power and fertilizer industries are the key users of
been in common use for a long time. Firewood and fossil natural gas.
fuels are the two main conventional energy sources.
These are non renewable. Electricity
l Non-conventional sources include solar, wind, tidal, l Electricity is generated mainly by running water which
geothermal, biogas and atomic energy. These are drives turbines and by burning other fuels such as coal,
renewable sources of energy. petroleum and natural gas to drive turbines.
l Bhakra Nangal, Damodar Valley Corporation and the
Coal Tehri hydel project are producing hydroelectric power.
l In India, coal is the most abundantly available fossil fuel.
l It has been formed due to the compression of plant
material over millions of years.
Non-Conventional Sources of
l Coal is found in a variety of forms depending on the Energy
degree of compression, depth and time of burial.
l The increasing use of fossil fuels is leading to its
l Lignite is low grade brown coal, which is soft with high
shortage. It is estimated that if the present rate of
moisture content.
consumption continues, the reserves of these fuels will
l Principle lignite reserves are found in Neyveli in Tamil get exhausted. Moreover, their use also causes
Nadu. environmental pollution.
l Major sources of coal are in the Damodar valley and l There is a need for using non-conventional sources such
nearby area, particularly Jharia, Raniganj and Bokaro. as solar energy, wind energy, tidal energy and other
non-conventional sources which are renewable.
Petroleum
l Petroleum or mineral oil is the next major energy source Solar Energy
in India after coal. l India is a tropical country. It has enormous possibilities
l Petroleum refineries act as a ‘Nodal Industry’ for for tapping solar energy.
synthetic textiles, fertilizer and numerious chemical l Photovoltaic technology converts sunlight directly into
industries. electricity.The largest solar plant of India is located at
l About 63% of India’s petroleum production is from Madhopur, near Bhuj in Gujarat, where solar energy is
Bombay High oilfield in the Arabian Sea, 18% from used to sterilize milk cans.
Gujarat and 16% from Asom.
l Ankaleshwar is a major oil field in Gujarat. Wind Energy
l Digboi is the oldest oil producing well in India. Digboi, l India now ranks as a ‘wind super power’ in the world.
Naharkatiya and Moran-Hugrijan are the important oil The largest wind farm cluster is located in Tamil Nadu
fields in Assam. in between Nagercoil and Madurai.
l Petroleum and its derivatives are called black gold, as l Apart from these, Andhra Pradesh, Karnataka, Gujarat,
they are very valuable. Kerala, Maharashtra and Lakshadweep have important
wind farms.
Natural Gas l Nagarcoil and Jaisalmer are well known for effective use
of wind energy for generation of electricity.
l Natural gas is an important clean energy resource found
in association with or without petroleum. It is used as a
source of energy as well as an industrial raw material in Biogas
the petrochemical industry. l Shrubs, farm waste, animal and human waste are used to
l Natural gas is considered an environment friendly fuel produce biogas for domestic consumption in rural areas.
because of low carbon dioxide emissions and is, l Decomposition of organic matter yields biogas, which
therefore, the fuel of the present and future. has higher thermal efficiency in comparison to kerosene,
l Large reserves of natural gas have been discovered in dung cake or charcoal.
the Krishna- Godavari basin. Along the West coast the
reserves of the Bombay High and allied fields are Tidal Energy
supplemented by finds in the Gulf of Cambay.
l Andaman and Nicobar Islands are important areas l Oceanic tides can be used to generate electricity.
having large reserves of natural gas. l Flood gate dams are built across inlets. During high
l The 1700 km long Hazira- Vijaipur-Jagdishpur cross- tide, water flows into the inlet and gets trapped when
country gas pipeline links Bombay High and Bassein the gate is closed.
with the fertilizer, power and industrial complexes in l In India, the Gulf of Kuchchh provides ideal conditions
western and northern India. for utilising tidal energy.
Minerals and Energy Resources 639
Geothermal Energy
l Geothermal energy refers to heat and electricity produced by using the heat from the interior of the earth.
l Geothermal energy exists because the earth grows progressively hotter with increasing depth.
l Two experimental projects have been set up in India to harness geothermal energy. One is located in the Parvati valley
near Manikaran in Himachal Pradesh and the other is in the Puga valley, Ladakh.

Target Exercise
Elementary Level
1. Which of the following minerals is formed by 8. Which of the following practices will not conserve
decomposition of rocks, leaving a residual mass of LPG in your kitchen?
weathered material? (a) Soaking the dal for sometime before cooking it
(a) Coal (b) Bauxite (b) Cooking food in a pressure cooker
(c) Gold (d) Zinc (c) Keeping the vegetable chopped before lighting the gas for
cooking
2. Koderma, in Jharkhand is the leading producer of (d) Cooking food in an open pan kept on low flame
which one of the following minerals?
9. Which of the following is not a metallic mineral?
(a) Bauxite (b) Mica
(a) Iron ore (b) Bauxite
(c) Iron (d) Copper
(c) Gold (d) Mica
3. Minerals are deposited and accumulated in the
10. Minerals that lie near the surface are simply dug out,
stratas of which of the following rocks?
by the process known as
(a) Sedimentary rocks
(a) mining (b) shafts
(b) Igneous rocks
(c) drilling (d) quarrying
(c) Metamorphic rocks
(d) None of these 11. Which of the following is not an example of
4. Which one of the following is contained in monazite non-metallic mineral?
sand? (a) Limestone (b) Mica
(c) Silver (d) Gypsum
(a) Oil (b) Uranium
(c) Thorium (d) Coal 12. Which of the following countries has no known
material deposit in it?
5. Which of the following is not a characteristic of (a) South Africa (b) Brazil
minerals? (c) Ghana (d) Switzerland
(a) They are created by natural process
(b) They have a definite chemical composition 13. The resources made of remains of plants and animals
(c) They are inexhaustible which were buried under the earth for million of years
(d) Their distribution is uneven got converted by heat and pressure are known as
(a) petroleum (b) fossil fuel
6. Which one the following states is not a producer of (c) coal (d) None of these
mica?
(a) Jharkhand 14. Which of the following resources is also known as
(b) Rajasthan black gold?
(c) Karnataka (a) Coal (b) Fire wood
(d) Andhra Pradesh (c) Petroleum (d) None of these
7. Which of the following is a leading producer of copper 15. Which of the following is not a non-conventional
in the world? source of energy?
(a) Bolivia (b) Chile (a) Solar energy (b) Wind energy
(c) Ghana (d) Zimbabwe (c) Natural gas (d) Tidal energy
640 Study Package for NTSE

High Skill Questions


1. Which of the following statements is/are true about 6. Match the following
minerals? List I (Mines) List II (Minerals)
(a) Minerals are homogenous, naturally occurring substances
A. Singhbhum 1. Copper
with a definable internal structure
B. Khetri 2. Bauxite
(b) Minerals are found in varied forms in nature, ranging from
C. Amarkantak Plateau 3. Iron-ore
the hardest diamond to the softest talc
D. Raniganj 4. Coal
(c) A mineral is formed from a certain combination of elements
depending upon the physical and chemical conditions Codes
under which the material forms A B C D A B C D
(d) All of the above (a) 3 1 2 4 (b) 1 2 3 4
(c) 3 2 1 4 (d) 4 1 2 3
2. The group of sedimentary minerals including,
gypsum, potash salt and sodium salt are formed as a 7. Match the following
result of which process?
List I List II
(a) Decomposition
(b) Evaporation A. Nodal Industry 1. Electricity generation by burning
(c) Both ‘a’ and ‘b’ of coal
(d) None of the above B. Thermal Electricity 2. Thorium
C. Monazite Sands 3. Petroleum
3. Which of the following minerals are called ‘placer D. Geothermal Energy 4. Generation of electricity by using
deposits’ and are not corroded by water? the heat from the interior of the earth
(a) Gold (b) Silver
(c) Tin (d) All of these Codes
A B C D
4. Which of the following is the finest form of iron ore? (a) 3 1 2 4
(a) Magnetite (b) Hematite (b) 3 1 4 2
(c) Both ‘a’ and ‘b’ (d) None of these (c) 1 2 3 4
(d) 4 3 1 2
5. Match the following
8. Which of the following factors led to the
List I List II
popularisation and adoption of non-conventional
A. Nickel 1. Non-metallic sources of energy?
B. Copper 2. Energy resource (a) Rising prices of fossil fuels and their potential shortages,
C. Natural gas 3. Non-ferrous raising uncertainties about the security of energy supply in
D. Mica 4. Ferrous future
Codes (b) Increasing use of fossil fuels, causing serious
A B C D A B C D environmental problems
(a) 4 3 2 1 (b) 3 4 2 1 (c) These resources are renewable
(c) 1 2 3 4 (d) 2 1 4 3 (d) All of the above

Answers
Elementary Level
1. (b) 2. (b) 3. (a) 4. (c) 5. (c) 6. (c) 7. (b) 8. (d) 9. (d) 10. (d)
11. (c) 12. (d) 13. (b) 14. (c) 15. (c)

High Skill Questions


1. (d) 2. (b) 3. (d) 4. (a) 5. (a) 6. (a) 7. (a) 8. (d)
Chapter

9
Population
People are important to develop the economy and l The absolute numbers added each year or decade is the
society. The people make and use resources and are magnitude of increase. It is obtained by simply subtracting
themselves resources with varying quality. Human the earlier population, from the later population.
beings are producers and consumers of earth’s l The rate or the pace of population increase is the other
resources. The Census of India provides us with important aspect. It is studied in per cent per annum.
information regarding the population of our country. A e.g., a rate of increase of 2 % per annum means that in a
census is an official enumeration of population done given year, there was an increase of two persons for
every 100 persons in the base population. This is
periodically i.e., after a gap of ten years. The first
referred to as the annual growth rate.
complete census was taken in the year 1881.
The Indian census is the most comprehensive source of
l

demographic, social and economic data. Processes of Population-


l The population of India on 1st March, 2011 was Change/Growth
1,21,01,93,422. India added 181 million to its population
Since 2001, slightly lower than the current population of l There are three main processes of change of population:
Brazil. India, with 2.4% of the world’s surface area, birth rate, death rate and migration.
accounts for 17.5% of its population. l The natural increase of population is the difference
l Many religions have originated in the country and the between birth rate and death rate.
all major religions of foreign origin have flourished here. l Birth Rate is the number of live births per thousand
India has the distinction of being the land from where persons in a year.
important religions.
l Death Rate is the number of deaths per thousand
Ever since its inception, the Census of India has been persons in a year. The main cause of the rate of growth of
collecting and publishing information about the the Indian population has been the rapid decline in
religious affiliations as expressed by the people of India. death rates.
In fact, population census has the rare distinction of l Migration is the movement of people across regions and
being the only instrument that collects the information. territories. Migration can be internal or international.
On this diverse and important characteristic of the
Indian population. Age Composition
l The age composition of a population refers to the
Population Growth number of people in different age groups in a country.
l It is one of the most basic characteristics of a population.
Growth of population refers to the change in the number
l A person’s age influences what he needs, buys and his
of inhabitants of a country/territory during a specific capacity to perform.
period of time, say during the last ten years. Such a
change can be expressed in two ways in terms of The population of a nation is generally grouped into
absolute numbers and in terms of percentage change three broad categories
per year.
642 Study Package for NTSE

(i) Children (Below 15 Years) They are economically Primary activities include agriculture, animal
unproductive and need to be provided with food, husbandry, forestry, fishing, mining etc.
clothing, education and medical care.
Secondary activities include construction, building
(ii) Working Age (15-59 Years) They are economically
productive and biologically reproductive. They and manufacturing industry.
comprise the working population. Tertiary activities include transportation, communication,
(iii) Aged (Above 59 Years) They can be economically banking, insurance, administration and services.
productive though they may have retired. They may
be voluntarily working but they are not available for
employment through recruitment.
National Population Policy
l The percentage of children and the aged affect the l Recognising that the planning of families would improve
dependency ratio because these groups are not individual health and welfare, the Government of India
producers. initiated the comprehensive family planning
programme in 1952.
Sex Ratio l The family welfare programme has sought to promote
l The Sex Ratio is the number of females per 1000 males responsible and planned parenthood on a voluntary basis.
in the population. The National Population Policy, 2000 is the culmination of
l This information is an important social indicator to years of planned efforts.
measure the extent of equality between males and l The National Population Policy, 2000 provides a policy
females in a society at a given time. framework for imparting free and compulsory school
education up to 14 years of age, reducing infant morality
Literacy Rates rate to below 30 per 1000 live births, universal
l Literacy is a very important quality of a population. immunization of children against all vaccine
Obviously, only an informed and educated citizen can preventable diseases, promoting delayed marriage for
make intelligent choices and undertake research and girls and family welfare a people-centred programme.
development.
Some Excerpts from the Census of India, 2011
l Low levels of literacy are a serious obstacle for economic
Population Total 1210193422
improvement.
Males 623724248
Females 586469174
Occupational Structure
Literacy Total 74.04 %
l The percentage of population that is economically active Males 82.14 %
is an important index of development. Females 65.46 %
l The distribution of the population according to different Density of Population per km 2 382
types of occupation is referred to as occupational structure. Sex Ratio per 1000 males 940 females
Child Sex Ratio per 1000 males 914 females
l Occupations are generally classified as primary,
(0-6 age group)
secondary and tertiary.

Target Exercise
Elementary Level
1. Migrations change the number, distribution and 3. The magnitude of population growth refers to
composition of the population in (a) total population of an area
(a) the area of departure (b) the area of arrival (b) the number of persons added each year
(c) Both ‘a’ and ‘b’ (d) None of these (c) the rate at which the population increases
(d) the number of females per thousand males
2. A large proportion of children in a population is a
result of 4. According to the census, a literate person is one who
(a) high birth rate (a) can read and write his/her name
(b) high death rate (b) can read and write any language
(c) high life expectancy (c) is 7 years old or above and can read and write any language
(d) more married couples with understanding
(d) knows reading, writing along with arithmetic
Population 643
5. The number of females per 1000 males in the 8. Which of the following is/are factors affecting the
population is known as distribution of population?
(a) population ratio (b) sex-ratio (a) Topography (b) Climate
(c) gender ratio (d) None of these (c) Both ‘a’ and ‘b’ (d) Nither ‘a’ nor ‘b’
6. The difference between the birth rate and the death 9. In which of the following conditions will there will be
rate of a country is called an increase in population?
(a) population growth (b) growth rate (a) Death rate more, than birth rate
(c) natural growth rate (d) None of these (b) Death rate and birth rate same
7. Population density is the number of people living in a (c) Birth rate more than death rate
unit area of the earth’s surface. It is normally (d) None of the above
expressed as 10. The first complete census was held in India in the
(a) per square km (b) per 1000
year
(c) per square metre (d) None of these (a) 1881 (b) 1872 (c) 1891 (d) 1882

High Skill Questions


1. Census in India is held after a gap of 7. Which of the following statements is/are correct?
(a) six years (b) eleven years (a) The proportion of people working in different activities varies
(c) ten years (d) at the will of the government in developed and developing countries.
(b) Developed countries have a high proportion of people in
2. The total population of an area at the beginning of a
secondary and tertiary activities.
given time period is known as
(c) Developing countries tend to have a higher proportion of their
(a) average population (b) base population workforce engaged in primary activities.
(c) annual population (d) None of the above (d) All of the above
3. The movement of people from one place to another 8. A large proportion of Indian population is engaged in
within the country is called which of the following activities of the primary
(a) immigration (b) migration
sector?
(c) internal migration (d) None of these
(a) Agriculture (b) Mining
4. The adolescent are the most important resource for (c) Animal husbandry (d) Quarrying
the future. Which of the following age groups
9. Match the following (as per 2001 census) lists
represents them?
(a) Generally below 15 years List I List II
(b) The age group of 10-19 years
(c) The age group of 7-19 years A. Maximum density of population 1. Uttar Pradesh
(d) None of the above B. Maximum sex ratio 2. Sikkim
5. The distribution of population according to different C. Maximum populated state 3. NCR Delhi
types of occupation is referred to as D. Least populated state 4. Kerala
(a) profession Codes
(b) economic activities A B C D A B C D
(c) occupational structure (a) 3 4 1 2 (b) 1 2 3 4
(d) None of the above (c) 2 1 4 3 (d) 4 3 1 2
6. Which of the following activities will be included in 10. The population of India is what per cent of the world’s
primary activities? total population?
(a) Agriculture (b) Forestry (a) 21% (b) 17.5% (c) 21% (d) 18%
(c) Mining (d) All of these

Answers
Elementary Level
1. (c) 2. (a) 3. (c) 4. (c) 5. (b) 6. (c) 7. (a) 8. (c) 9. (c) 10. (a)

High Skill Questions


1. (c) 2. (b) 3. (c) 4. (b) 5. (c) 6. (d) 7. (d) 8. (a) 9. (a) 10. (b)
Chapter

10
Transport-Lifelines of
National Economy
The movement of goods and services from their supply l The major objective of these super highways is to
locations to necessitates the need of transport. The pace reduce the time and distance between the mega cities of
of development of a country depends upon the India.
production of goods and services as well as their l These projects are being implemented by the National
movement over space. Therefore, efficient means of Highway Authority of India (NHAI).
transport are pre-requisites for fast development. Golden Quadrilateral Super Highway
Movement of these goods and services can be over three The government has launched a major road development project
important domains of our earth. i.e., land, water and air. linking Delhi-Kolkata-Chennai-Mumbai by six lane super highways.
Based on these, transport can also be classified into
land, water and air transport. National Highways
National Highways link the extreme parts of the
Roadways country. These are the primary road systems and are laid
India has one of the largest road networks in the world. and maintained by the Central Public Works
The growing importance of road transport vis-a-vis rail Department. The historical Sher Shah Suri Marg is called
transport is rooted in the following reasons National Highway No 1 Between Delhi and Amritsar.
l Construction cost of roads is much lower than that of
railway lines. State Highways
l Roads can traverse comparatively more dissected and Roads linking a state capital with different district
undulating topography. headquarters are known as state highways. These are
l Roads can negotiate higher gradients of slopes and as such constructed and maintained by the state Public Works
can traverse mountains such as the Himalayas. Department.
l Road transport is economical in transportation of a
small number of persons and relatively smaller amount District Roads
of goods over short distances.
These roads connect the district headquarters with
l It also provide door-to-door service.
other places of the district and are maintained by Zila
l Road transport is also used as a feeder to other modes of Parishad.
transport such as rails, air and sea.
l The North-South corridor linking Srinagar and Other Roads
Kanyakumari and the East-West corridor connecting
Silchar and Porbandar are parts of this porject. Rural roads which link areas and villages with towns
are classified under this category.
Transport-Lifelines of National Economy 645
National Highways
Waterways
Development Project
Srinagar l India is one of the seafaring countries.
Legends
17
Jammu Golden Quadrilateral l Waterways are the cheapest means of transport. They
North-South Corridor
Pakistan
17 Jalandhar East-West Corridor are most suitable for carrying heavy and bulky goods.
National Highway Number

1
l These are a fuel-efficient and environment friendly
Delhi Sonipat
China
Tibet mode of transport.
Gurgaon Agra
Jaipur
8 Lucknow
Etawah 28
Gorakhpur Siliguri l The Ganga river between Allahabad and Haldia, the
Muzaffarpur
Kishangarh
Gwalior
3 Kanpur
57 31 37 30
Brahmaputra river between Sadiya and Dhubri and the
Palanpur
76 Kota
75
25
Fatehpur
2 Varanasi Islampur
Guwahati 54 West coast canals in Kerala are declared as national
waterways.
Sikandara Purnea
Shivpuri Silchar
NA 14 Jhansi Panagarh
15 Chittorgarh 26
Samakhiali NA Udaipur Sagar
7 2
Ahmedabad Bypass Kolkata

Major Sea Ports


8B 6
Rajkot Vadodara Lakhnadon
Porbandar
60 Kharagpur
Surat Nagpur 5
Diu Island 8

Mumbai
Bhubaneshwar Kandla This is a tidal port in Kuchchh. It caters to the
Pune
7 5 convenient handling of exports and imports of granary
Satara Vishakhapatnam
Hyderabad
and industrial belt products stretching across the states
And a

Eluru
Arabian sea Belgaum
of Jammu and Kashmir, Himachal Pradesh, Punjab,
5
man and Nicobar Is

Kurnool Vijayawada
4 Chilakaluripet

Sira
7
Nellore Haryana, Rajasthan and Gujarat.
L ak

Tumkur
sha

Chennai
Mumbai Mumbai is the biggest port in India with a
Bengaluru 4
d we

Hosur Poonamallee
46
land

Krishnagiri
ep

Ranipet Bay of Bengal


spacious natural and well-sheltered harbour.
Coimbatore Salem
Ind

N
(Ind

47
ia

Kochi 7
ia)

Madurai W E
7
S
Murmagao It is the premier iron ore exporting port of
Indian Kanyakumari Ocean Root to Scale the country. It is located in Goa.
Mangalore port located in Karnataka caters to export of
Railways iron ore cencentrates from Kudremukh mines.
l Railways are the principal mode of transportation for Kochi Kochi is a port located in Kerala at the entrance
freight and passengers in India.
of a lagoon with a natural harbour.
l Railways also make it possible to conduct multifarious
activities like business, sightseeing, pilgrimage along Tuticorin This is a port located in Tamil Nadu and has
with transportation of goods over longer distances. a natural harbour with a rich hinterland.
l Railways in India bind the economic life of the country
Chennai One of the oldest artificial ports of the
as well as accelerate the development of industry and
agriculture.
country it ranks next to Mumbai in terms of volume of
trade and cargo.
l The distribution pattern of the railway network in the
country has been largely influenced by physiographic, Vishakhapatnam It is the deepest port in India.
economic and administrative factors.
Paradip It is located in Odisha and specialises in the
n National Highway No. 44 between Srinagar and Kanyakumari export of iron ore.
is the longest higway in India.
Kolkata It is an inland riverine port. The port serves a
n Delhi and Mumbai are connected by National Highway No. 8.
very large and rich hinterland of Ganga-Brahmputra
n National Highway No. 15 covers most of Rajasthan.
basin. Being a tidal port, It requires constant dredging
of the Hooghly.
Pipeline Haldia It was developed as a subsidiary port in order to
l Pipeline transport network is a new arrival on the release growing pressure on the Kolkata port.
transportation map of India.
l Pipelines are used for transporting crude oil, petroleum
products and natural gas from oil and natural gas fields
Airways
to refineries, fertilizer factories and big power plants. l Air travel today is the fastest, most comfortable and
l There are three important networks of pipeline prestigious mode of transport.
transportation in the country. l It can cover very different and difficult terrains like high
(i) From oilfields in Asom to Kanpur. mountains, dreary deserts, dense forests and also long
(ii) From Salaya in Gujarat to Jalandhar in Punjab. oceanic stretches with great ease.
(iii) From Hazira, Gujarat to Jagdishpur in Uttar Pradesh. l Air travel has made access to remote areas easier.
646 Study Package for NTSE

Communication l As resources are space bound, no country can survive


without international trade.
l Ever since humans appeared on the earth, they have l Export and import are the components of international
used different means of communication. trade.
l Long distance communication is far easier without
l The difference between the export and import is known
physical movement of the communicator or receiver.
as the balance of trade of a country.
l Personal communication and Mass Communication
l When the value of export exceeds the value of imports, it
include television, radio, press, films, etc.
is called a favourable balance of trade. On the contrary,
l Mass communication provides entertainment and if the value of imports exceeds the value of exports, this
creates awareness among people about various national is termed as an unfavorable balance of trade.
programmes and policies.
l India is the largest producer of feature films in the
world. The Central Board of Film Certification is the Tourism As a Trade
authority to certify both Indian and foreign films.
l Tourism in India has grown substantially over the last
three decades.
International Trade l Tourism promotes national integration as well as
l The exchange of goods among people, states, and providing support to local handicrafts and cultural
countries is referred to as trade. pursuits.
l The market is the place where such exchanges take l It also helps in the development of international
place. Trade between two countries is called internatio- understanding about our culture and heritage.
nal trade. It make place through sea, air and land
l Rajasthan, Goa, Jammu and Kashmir and temple towns
routes.
of South India are important destinations of foreign
l The volume of international trade of a country is an tourists in India.
index to its economic prosperity. It is considered the
economic barometer for a country.

Target Exercise
Elementary Level
1. Which two of the following extreme locations of India 5. Which one of the following is the most important
are connected by the East-West corridor? modes to transport in India?
(a) Mumbai-Nagpur (b) Mumbai-Kolkata (a) Pipeline (b) Roadways
(c) Silchar-Porbandar (d) Nagpur-Siliguri (c) Railways (d) Airways
2. Which mode of transportation reduces 6. Which one of the following terms is used to describe
trans-shipment losses and delays? trade between two or more countries?
(a) Railways (b) Pipeline (a) Internal Trade (b) External Trade
(c) Roadways (d) Waterways (c) International Trade (d) Local Trade
3. Which of the following states is not connected with 7. The organisation of the Indian Government which
HVJ pipeline? undertakes construction and maintenance of roads in
(a) Madhya Pradesh the border areas of the country is
(b) Gujarat (a) Border Road Organisation
(c) Maharashtra (b) NHAI
(d) Uttar Pradesh (c) CPWD
(d) PWD
4. Which one of the following ports is the deepest
land-locked and well-protected port along the East 8. Which of the following is the largest public sector
undertaking in the country?
Coast?
(a) Roadways
(a) Chennai (b) Indian Railways
(b) Tuticorin (c) Shipping Corporation of India
(c) Paradwip (d) None of the above
(d) Vishakhapatnam
Transport-Lifelines of National Economy 647
9. The first train steamed off from Mumbai to Thane in 12. Which of the following is the premier iron ore
(a) 1853 (b) 1863 exporting port of the country?
(c) 1873 (d) 1885 (a) Murmagao (b) Mumbai (c) Kandla (d) Kochi
10. Pipeline transport network is a new arrival on the 13. Which of the following is not a part of Mass
transportation map of India. This is used for Communication?
transportation of (a) Radio (b) Television
(a) crude oil (c) Newspaper (d) None of these
(b) petroleum products 14. Bulk imports of India as a group includes
(c) natural gas
(a) fertilizers (b) newsprint
(d) All of the above
(c) edible oil (d) All of these
11. The difference between the export and import of a
15. Which of the following commodities accounts for the
country is known as
largest import of India?
(a) trade value (b) bilateral trade
(a) Petroleum products (b) Edible oils
(c) balance of trade (d) trade defecit
(c) Precious stones (d) Inorganic chemicals

High Skill Questions


1. The growing importance of road transport vis-a-vis 3. Match the following lists
rail transport is rooted in which of the following
List I List II
statements?
(a) Construction cost of roads is much lower than that of railway A. National Highway-7 1. Delhi-Amritsar
lines B. National Highway-1 2. Delhi-Mumbai
(b) It provides door-to-door service C. National Highway-8 3. Varanasi-Kanyakumari
(c) Roads can negotiate higher gradients of slopes, and as such
D. National Highway-15 4. Covers most of Rajasthan
can traverse mountains
(d) All of the above
Codes
2. Match the following A B C D A B C D
(a) 3 1 2 4 (b) 3 2 1 4
List I List II (c) 1 2 3 4 (d) 2 1 4 3
A. Vishakhapatnam 1. Inland riverine port 4. International trade is regulated and monitored by
B. Kolkata 2. Natural harbour (a) United Nations Organisations
(b) GATT
C. Tuticorin 3. Deepest landlocked port
(c) World Trade Organisation
D. Mumbai 4. Biggest in terms of trade (d) UNCTAD
and cargo
5. The roads which the district headquarters connects
Codes with the other places of the district are maintained by
A B C D (a) Central Government
(a) 3 1 2 4 (b) State Government
(b) 3 1 4 2 (c) Zilla Parishad
(c) 1 2 3 4 (d) Panchayats of Villages
(d) 1 2 4 3

Answers
Elementary Level
1. (c) 2. (d) 3. (c) 4. (d) 5. (c) 6. (c) 7. (a) 8. (b) 9. (a) 10. (d)
11. (c) 12. (a) 13. (d) 14. (d) 15. (a)

High Skill Questions


1. (d) 2. (a) 3. (a) 4. (c) 5. (c)
Part-VII : Civics
Chapter

1
Framing of Indian Constitution
and its Basic Features
l Sovereign It emphasises that there is no authority
The Preamble outside India on which the country is in anyway
The ‘Preamble’ means the preface or introduction of the dependent. People have supreme right to make
decisions on internal as well as external matters.
Constitution.
l Socialist It means the achievements of socialist
The famous ‘objective Resolution’ of Nehru is base for
pattern of society through democratic means.
Preamble
Government should regulate the ownership of land and
The Preamble to the Indian Constitution declares its industry to reduce socio-economic in equalities.
aims, principles and ideals. It contains the philosophy l Secular It means that state shall not discriminate
on which the entire Constitution has been built. It against the citizens in any way on the basis of religion.
also embodies the ideals and aspiration of people of There is no official religion and citizens have complete
India. freedom to follow any religion.
l The Preamble declares that it is the people of India who l Democratic It means that the rulers are elected by the
had enacted, adopted and given the Constitution to people only, have the authority to run the government.
themselves. The Constitution declares India as a Sovereign, Socialist,
l The Constitution proclaiming India as Sovereign, Secular, Democratic, Republic.
Socialist, Secular, Democratic, Republic, the latter l Republic The word republic means that there is no
include Justice, Liberty, Equality and Fraternity. hereditary ruler in India and the head of the institution is
directly or indirectly elected by the people.
l Justice Social inequalities have to be reduced and the
Preamble of India government should work for the welfare of all, especially
We, the people of India, having solemnly resolved to constitute India into a the disadvantaged section of society.
SOVEREIGN, SOCIALIST, SECULAR, DEMOCRATIC, REPUBLIC and to l Liberty Liberty of thought, expression, belief, faith and
secure to all its citizens.
worship. Liberty is an essential attribute of a free society.
l JUSTICE—social, economic and political;
l Equality All are equal before law. The government
l LIBERTY—of thought, expression, belief, faith and worship;
ensure equal opportunities for all.
l EQUALITY—of status and of opportunity, and to promote among them all,
l Fraternity No one should treat a fellow citizen as
l FRATERNITY—assuring the dignity of the individual and the unity and
inferior. We must behave like a member of same family.
integrity of the Nation;
In our Constituent Assembly, on this twenty-sixth day of November, 1949, The Constitution (42nd Amendment) Act, 1976
do hereby adopt, enact and give to ourselves this Constitution. amended the Preamble and added the word socialist,
secular and integrity to the Preamble.
Framing of Indian Constitution and its Basic Features 649
The Constituent Assembly formed 13 important
Indian Constitution and its Basic l

committees for framing the Constitution, amongst them


Features Drafting Committee was most important which
consisted of Chairman (Dr BR Ambedkar) with six other
l It is the lengthiest Constitution in the world. It
members.
originally contained 395 Articles and 8 Schedules.
l The Indian Constitution is partly rigid and partly l The other important commitees were Union Power
flexible. Committee, Fundamental Rights and Minority Committe,
Provincial Constitution Committee, Union Constitution
l The Constitution provides for a Parliamentary system of
Committee, Flag Committee, Steering Committee etc. All
government.
these committees submitted their reports, which were
l The Constitution makes India a secular state. widely discussed by the Constituent Assembly.
l The Indian Constitution combines the features of both l The draft Constitution of India prepared by the Drafting
federal and unitary systems of government. Committee was submitted to the President of the
l The Constitution contains an elaborate list of Assembly on 21st February, 1948.
Fundamental Rights and Duties. l Finally, on 26th November, 1949, the people of India
l The Constitution outlines certain Directive Principles of through the Constituent Assembly adopted, gave to
State Policy which the government has to keep in mind themselves the Constitution of India. On 24th
while framing laws and formulating policies. January, 1950, which was the last working day of the
l The Constitution provides for a bicameral legislature at Assembly finally Constitution were signed. The
the centre consisting of Lok Sabha and Rajya Sabha. Constitution came into full operation with effect from
l The Constitution vests extraordinary powers in the 26th January, 1950.
President dirung emergencies. l Our Constitution is the lengthiest Constitution of the
l It provides for an independent judiciary. world which comprises of originally 22 Parts, 395
Articles and 8 Schedules. Constitution Assembly took 2
l The Constitution introduces Universal Adult Franchise.
Years 11 Month and 18 Days to frame the Constitution
It gives the right to vote to all citizens above 18 years of
of India. Our Constitution was constituted of different
age without discrimination.
provisions from different sources of the constitutions of
l The Constitution makes special provisions for minorities, the world.
Scheduled Castes, Scheduled Trribes, etc.
From the Constitution of USA we have taken
l The Constitution draws its authority from the people
and has been promulgated in the name of the people. (i) Independence of Judiciary, Judicial Review, President
as the Executive Head and President as the Supreme
This is evident from the Preamble which states, ‘‘We the
Commandar of the Armed Force.
people of India .... do hereby adopt, enact and give to
ourselves this Constitution’’. (ii) Rule of Law and Law making Procedures, System of
Single Citizenship etc.

The Constituent Assembly Irish Constitution was the source of Directive Principles,
l To form the Constitution an Assembly of elected Method of Presidential Elections and Nomination of
representatives termed as Constituent Assembly Members of Rajya Sabha by President.
formed. The Constituent Assembly was set up under Idea of federation with a strong centre taken from
Cabinet Plan of 1946. The demand of Constituent Canadian Constitution. While emergency provision taken
Assembly to draft the Constitution was raised in 1935.
from Germany. Concurrent List taken from Australia,
But the British Government accepted it in 1940.
Fundamental Duties from Japanese Constitution. While
l Initially, there was a total number of 389 members in
Constitutional Amendments from South Africa.
the Constituent Assembly out of which 93 were
representatives from Indian States, 292, from British
India and rest from Chief Commissioners’ Provinces.
Features of the Constitution
l The Mountbatten Plan of 3rd June, 1947, announced l Adult Suffrage The uniform adult suffrage system has
the partition of the country, and a separate Constituent been adopted in India. Under this, every men and
Assembly for the State of Pakistan was proposed. Hence, women who attain the age of 18 years or above shall
after partition of India members of Assembly decreased have the right to elect representative for the legislature.
to 299. l Single Citizenship It is true that our Constitution of
l First meeting of Constituent Assembly was held on 9th India provides dual polity i.e., centres and states but it
December, 1946 and Sachidanand Sinha was elected as provides single citizenship for the whole India. There is
Interim President. But on 11th December, 1946, no state citizenship in India, on the other hand
Dr Rajendra Prasad was elected as the President of American Constitution follows dual citizenship i.e.,
Constituent Assembly. citizenship of America and a state citizenship.
650 Study Package for NTSE
l Parliamentary Form of Government Our Constitution Abolition of Untouchability
establishes Parliamentary form of government both at
Article 17 abolishes ‘Untouchability’ and forbids its
the centre and the states. At the centre President is the
head of the Constitution and real executive power is practice in any form. The enforcement of any disability
vested in Council of Ministers headed by Prime arising out of untouchability is to be an offence
Minister. punishable in the accordance with law.
l Federation with Strong Centre It is one of the most Thus, on the grounds of untouchability no person can be
remarkable features of the Indian Constitution where denied access to shops, public restaurant, hotels and
Constitution acquires a unitary character during the places of public entertainment or the use of wells, tanks,
time of emergency. This also shows that our bathing ghats, road and places of public resort
Constitution is rigid but at the demand of circumstances
maintained wholly or partly out of State fund or
it becomes flexible too.
dedicated to the use of general public.

Abolition of Titles
Fundamental Rights and Duties in
Article 18 prohibits the state to confer titles on
the Indian Constitution anybody whether a non-citizen. Military and academic
In India, like most other democracies in the world, some distinctions are however, exempted.
rights are given in our Constitution.
l Some rights which are fundamental to our life are
Right to Freedom (Articles 19-22)
given a special status and are termed as Fundamental l Personal liberty is the most important of all the
Rights. Fundamental Rights.
l Fundamental Rights enshrined in Part III of the l Article 19 of the Constitution provides six fundamental
Constitution contain a long list of Fundamental Rights. freedom to the citizens of India i.e.,
Our Constitution provides us six major Fundamental (i) Freedom of Speech and Expression
Rights, these are (ii) Freedom of Assembly
(i) Right to Equality (iii) Freedom to Form Association
(ii) Right to Freedom (iv) Freedom of Movement
(iii) Right Against Exploitation (v) Freedom of Resident and Settlement
(iv) Right to Freedom of Religion (vi) Freedom of Profession, Occupation, Trade or Business.
(v) Cultural and Educational Rights
Freedom of Speech and Expression
(vi) Right to Constitutional Remedies
In our democracy Freedom of Speech and Expression is
Right to Equality very essential. It means the right to express one's own
belief or opinion freely by words of mouth, writing,
(Articles 14-18), Article 14 guarantees to all people
printing, pictures or any other mode. It includes the
equality before law or the equal protection of the laws
expression of one’s ideas through any communicable
within the territory of India. Article 14 confers on all
medium or visible representation such as banners, signs
individuals two types of right to equality viz, Equality
etc. There may be one's own views or right to propagate
before law, Equal protection of law.
or publish the views of other people.
Right Against Discrimination Freedom of speech and expression also includes the
Article 15 directs that state shall not discriminate freedom of press, as well. In our Constitution no where
against a citizen on grounds only of religion, race, caste the term press is used but it is implicit in the freedom of
sex or place of birth or any of them. speech and expression.

Equality of Opportunity Freedom of Assembly


Article 16 deals with equality of opportunity in public Freedom of Assembly includes right to hold meeting.
employment. It guarantees equality of opportunity to all This right like other individual rights is not absolute
citizens in the matter of appointment to any office or of but restricted. The Assembly must be non-violent and it
any other employment under the state. No citizen can be does not give right to strike.
discriminated against any employment of office under Freedom of Movement
the state on grounds only of religion, race, caste, sex, It guarantees to citizen of India the right to move freely
descent, place of birth on residence. throughout the territory of India.
Framing of Indian Constitution and its Basic Features 651
Freedom of Resident Freedom as to Payment of Taxes for promotion of
The right to move freely throughout the territory of any particular religion
India and right to reside and settle in any part of the Article 27 Under this no person shall be compelled to pay
territory which shows the integrity and unity of India. any tax for religious purposes. The government cannot
India is one integrated country and its citizens have the pressurise to pay any tax to the individual in respect of
freedom to move throughout the territory of India and to particular religion and religious activity. Prohibition of
reside and settle anywhere in India. religious instruction in state aided institutions
Freedom of Profession and Occupation Article 18 says that no religious instruction shall be
Constitution of India guarantees that all citizens have given in any educational institution. In educational
the right to practise any procession or to carry on any instuitutions managed by private bodies no person shall
occupation or trade or business, besides this legislation be compelled to take part in any religious instruction or
may impose reasonable restrictions on this right in the to attend any religious worship.
interest of general public.
Cultural and Educational Rights
Right Against Exploitation (Articles 23-24) (Articles 29-30)
Article 23 Prohibition of traffic in human beings and Article 29 According to this Article protection of
forced labour interest of minorities, language, culture and religion of
l Traffic in human beings here means selling and buying minorities needs special protection otherwise, they may
of human beings normally women and children for get neglected under the majority of religion, culture and
immoral and other purposes. language group.
l Our Constitution also prohibits practice of bonded labour Right of Minorities to establish and administer
on ‘begar’ in any form where workers is forced to given
educational institutions.
service to the master without fee or at nominal charges.
l When this practice takes place on a life-long basis, it is Article 30 According to this all minorities whether
called the practice of bonded labour. based on religion or language shall have the right to
l Prohibition of Employment of Children in Factories etc. establish and adinister educational institution of their
own choice.
Article 24 Prohibits the employment of children below
14 years of age in hazardous jobs, such as railways and Institution run by government or receiving government
ports, beedi making, fire crackers and matches, printing aid cannot deny any individual to get admission or the
and dyeing. ground on religion or language.

Right to Freedom of Religion (Articles 25-28) Right to Constitutional Remedies (Article 32)
Article 25 Freedom of Conscience, Profession, Practice This right guarantees the right to move the law court for
and Propagation of Religion— Every person has a right the enforcement of the rights. Both the Supreme Court
to profess, practice and propagate the religion he or she and the High Courts can issue writs of habeas corpus
believes in. Propagate one’s religion does not mean that madamus prohibition, quo warranto and certiorari.
any person put pressure to another to convert into his
religion by force, fraud, inducement or allurement.
Suspension of Fundamental Rights
l Anyone is free to accept any religion according to choice
it does not mean that he can do any thing whatever he When the President makes a proclamation of emergency
wants in the name of religion. under Article 352, the six freedoms guaranteed under
e.g., Sacrifice animal or human beings as offering to Article 19, are automatically suspended. The President
supernatural forces or God. can suspend other Fundamental Rights through specific
Freedom to Manage Religious Affairs orders. These orders must be approved by the
Parliament. But he can not suspend the freedoms under
Article 26 Religions have the following rights, viz.
Articles 20 and 21 under any circumstances.
(a) To establish or maintain institution for religious and
charitable purposes
Writs
(b) To manage its own affair in matters of religion
(c) To acquire movable and immovable property For the enforcement of any of the Fundamental Right, the
(d) To administer such property in accordance to law Supreme Court can issue writs of the following nature.
652 Study Package for NTSE
1. Habeas Corpus Habeas corpus means ‘to have a
body’. By issuing such a writ the court requires that a Some Important Terminology
person who has been imprisoned, be brought before it l Amnesty International An international organisation of
in order that the court may know on what ground he volunteers who campaign for human rights. This
has been confined and to set him free if there is no organisation brings out independent reports on the
lawful justification for the imprisonment. violation of human rights all over the world.
2. Mandamus Mandamus means a command. It l Apartheid The official policy of racial separation and ill
commands the person to whom it is addressed, to treatment of blacks followed by the government of South
perform some public or quasi-public legal duty which Africa between 1948 and 1989.
he has refused to perform. l Claim Demand for legal or moral entitlements a person
3. Prohibition It is a kind of command from a superior makes on fellow citizens, society or the government.
court to an inferior court forbidding it to do certain l Covenant Promise made to individuals, groups or countries
things which in the opinion of the higher court, are in to uphold a rule or principle. It is legally binding on the
excess of its jurisdiction. signatories to the agreement or statement.
4. Certiorari It is a kind of command which can be l Dalit A person who belongs to the castes which were
issued to any judicial and quasi-judicial authority considered low and not touchable by others. Dalits are also
quashing an order, which has already been passed known by other names such as the scheduled castes,
by it, but which in the opinion of the higher court, is depressed classes etc.
in excess of its jurisdiction.
l Ethnic Group An ethnic group is a human population
5. Quo Warranto It gives the right to a court to enquire whose members usually identify with each other on the
into the legality of the claim which an individual basis of a common ancestry. People of an ethnic group are
asserts to a public office and to oust him from that united by cultural practices, religious beliefs and historical
office if it finds that his claim is not well-founded.

Target Exercise
Elementary Level
1. Legally sanctified document which incorporates 6. Apart from BR Ambedkar (Chairman of Drafting
certain rules that the citizen and government have to Committee), how many other members were in
follow is/are called Drafting Committee?
(a) Fundamental Rights (b) Fundamental Duties (a) 4 (b) 6 (c) 7 (d) 5
(c) The Constitution (d) None of these
7. Constitution of India was adopted on
2. The Constituent Assembly was set up under (a) 26 January, 1950 (b) 15 August, 1947
(a) Government of India Act, 1935 (c) 26 November, 1949 (d) 26 November, 1950
(b) Cabinet Mission Plan of 1946
8. Who among the following was not a member of the
(c) Cripps Mission
Constituent Assembly?
(d) None of the above
(a) Sardar Vallabh Bhai Patel (b) Raja Gopalachari
3. Who was the Chairman of Drafting Committee of the (c) Mahatma Gandhi (d) Lord Mount Batten
Indian Constitution?
9. First meeting of Constituent Assembly was held in
(a) Rajendra Prasad (b) Tej Bahadur Sapru
(a) 10th December, 1946 (b) 9th December, 1947
(c) C Raja Gopalachari (d) Dr BR Ambedkar
(c) 9th December, 1948 (d) 9th December, 1946
4. Who was elected as permanent President of the 10. Who was elected as interim President of the
Constituent Assembly?
Constituent Assembly?
(a) Sachidanand Sinha (b) BR Ambedkar (a) BR Ambedkar (b) BN Rao
(c) Pandit Nehru (d) Dr Rajendra Prasad (c) Rajendra Prasad (d) Dr Sachidanand Sinha
5. How many committees were formed for framing of the 11. Who proposed the Preamble before the Drafting
Constitution? Committee of the Constitution?
(a) 11 (b) 10 (a) BN Rao (b) Mahatma Gandhi
(c) 13 (d) 14 (c) Jawahar Lal Nehru (d) BR Ambedkar
Framing of Indian Constitution and its Basic Features 653
12. The demand of Constituent Assembly to Draft the 17. Who was the viceroy of India when India became
Constitution was first time raised in the year free?
(a) 1931 (b) 1932 (a) Lord Wavell
(c) 1933 (d) 1935 (b) Lord Irwin
(c) Lord Willington
13. Initially how many members were in Constituent
(d) Lord Mount Batten
Assembly?
(a) 380 (b) 385 (c) 389 (d) 292 18. After partition from Pakistan the member of
Constituent Assembly decreased from 389 to
14. Abolition of untouchability lies in which Article
(a) 384 (b) 292
(a) Article 17 (b) Article 18 (c) 380 (d) 299
(c) Article 16 (d) Article 14
19. Directive Principles of State Policy are taken from
15. Amongst all 389 member of Constituent Assembly
(a) Britain
how many member were representatives from Indian (b) USA
States? (c) UK
(a) 292 (b) 384 (d) Ireland
(c) 93 (d) None of these
20. How many Fundamental Rights exist in Indian
16. How many members were from Chief Commissioner Constitution?
provinces? (a) 5 (b) 7
(a) 4 (b) 5 (c) 2 (d) 6 (c) 6 (d) 8

High Skill Questions


1. Who among the following was not the member of 6. Match List I (Articles of the Constitution of India)
Drafting Committee? with List II (Provision) and select the correct answer
(a) N Gopal Swamy Ayyangar using the codes given below the lists (2004)
(b) Alladi Krishnaswami Iyer
(c) Mohammad Sadullah List I List II
(d) Pandit Jawaharlal Nehru
A. Article 14 1. The State shall not discriminate against any
2. In the Indian Constitution, the Right to Equality is citizen on grounds only of religion, race,
granted by Five Articles. They are caste, sex, place of birth or any of them.
(a) Article 16 to 20 B. Article 15 2. The State shall not deny to any person
(b) Article 15 to 19 equality before the law or the equal
(c) Article 14 to 18 protection of laws within the territory of India.
(d) Article 13 to 17
C. Article 16 3. ‘Untouchability’ is abolished and its
3. Prohibition of discrimination on grounds of religion practice in any form is forbidden.
etc. (Article 15 of the Constitution of India) is a D. Article 17 4. There shall be equality of opportunity for all
Fundamental Right classifiable under (1995) citizens in matters relating to employment
(a) The Right of Freedom of Religion on appointment to any office under the
(b) The Right against Exploitation state.
(c) The Cultural and Educational Rights
(d) The Right to Equality Codes
A B C D
4. The Constitution of India recognises (a) 2 4 1 3
(a) only religious minorities (b) 3 1 4 2
(b) only linguistic minorities (c) 2 1 4 3
(c) religious and linguistic minorities (d) 3 4 1 2
(d) religious, linguistic and ethnic minorities
7. The Cabinet Mission of India was headed by
5. The concept of Public Interest Litigation originated in (a) Stafford Cripps (b) A V Alexander
(a) the United Kingdom (c) Lord Pethick Lawrence (d) Hugh Gaits Kell
(b) Australia 8. Right to Constitutional remedies comes under
(c) the United States
(a) Fundamental Rights (b) Legal Rights
(d) Canada (c) Constitutional Rights (d) Natural Rights
654 Study Package for NTSE
9. Which Article of Indian Constitution is called “the 16. The Indian Constitution is regarded as
heart and soul of the Constitution”? (a) Federal
(a) Article 18 (b) Article 22 (b) Unitary
(c) Article 32 (d) Article 14 (c) Parliamentary
(d) Federal in form and Unitary in Spirit
10. Right to freedom of religion includes Article
(a) Article 14 to 18 (b) Article 25 to 28 17. India became a Sovereign Democratic Republic on
(c) Article 24 to 28 (d) Article 30 to 32 (a) 15 August , 1947
(b) 30 January , 1948
11. Which Part of Indian Constitution is described as
(c) 26 January , 1950
Magna Carte of India? (d) 26 November, 1929
(a) Part I (b) Part II
(c) Part III (d) Part IV 18. How long did the Constituent Assembly take to
finally pass the Constitution?
12. Freedom of press implicit in which Article of the (a) About 6 months in 1949
Constitution? (b) Exactly 1 years since 26 November , 1948
(a) 19 (b) 20 (c) 21 (d) 22 (c) About 2 years since 15 August , 1947
13. Abolition of titles lies in which Article of the (d) About 3 years since 9 December , 1946
Constitution? 19. The Constitution names our country as
(a) 16 (b) 17 (c) 18 (d) 19 (a) Bharat
14. Protection in respect of conviction for offences lies in (b) India, i.e., Bharat
one of the Six Fundamental Rights, it is (c) Hindustan
(a) Right to Equality (d) Arya Yrata
(b) Right to Freedom 20. What is the single biggest source of Indian
(c) Right Against Exploitation Constitution?
(d) Cultural and Education Rights (a) Constitution of Britain
15. Fundamental Rights are taken from (b) Constitution of USA
(a) UK (b) US (c) Constitution of USSR
(c) Ireland (d) Japanese (d) Government of India Act 1935

Answers
Elementary Level
1. (c) 2. (b) 3. (d) 4. (d) 5. (c) 6. (b) 7. (c) 8. (c) 9. (d) 10. (d)
11. (c) 12. (d) 13. (c) 14. (a) 15. (c) 16. (a) 17. (d) 18. (d) 19. (d) 20. (c)

High Skill Questions


1. (d) 2. (c) 3. (d) 4. (c) 5. (c) 6. (c) 7. (c) 8. (a) 9. (c) 10. (b)
11. (c) 12. (a) 13. (c) 14. (d) 15. (b) 16. (d) 17. (c) 18. (d) 19. (b) 20. (d)
Chapter

2
Working of Institutions Organs
of Government
For smooth running of a country there is a need of The electoral college includes all the elected members of
channel to run the country properly. Hence in our both the Houses of Parliament and the elected members
democratic form, the whole country runs on three of Legislative Assemblies of states. The vote is cast by
different heads i.e., Legislative, Executive and secret ballot.
Judiciary. All three have own area of working under
Qualifications of President
which no one can interfere but having indirect control
over each other. Being elected for the post of President some
qualifications are required
l He must be a citizen of India.
Union Executive l He must have completed the age of 35 years.
l He must be qualified for election as a member of the
President House of People i.e., Lok Sabha.
The union executive includes President, Vice President l He must not hold any office of profit under the
and the Council of Ministers. The executive power vests Government of India or government of any state.
to the President according to the Constitution who shall
Oath and Tenure
exercise his power directly or through officers
subordinate to him. The oath of President is administered by Chief Justice
of Supreme Court. The President holds office for a term
He is the Supreme Commander of armed forces. He is
of five years from the date on which he enters his office.
the first citizen of India and occupies the first position
He is eligible for re-election. He may resign from his
under the warrant of precedence. Warrant of
office before the expiry of his term by writing to the Vice
precedence means hierarchy of position occupied by
President. The office of President can not remain vacant
various dignitaries attending a state function.
for more than 6 month.
All the executive actions are taken in his name and the
President is to exercise them on the advice of Council of Impeachment
Ministers responsible to Parliament. There is a special procedure for the removal of the
President. He can be removed by impeachment for
Election of President violation of the Constitution.
The President of India is elected by indirect election. The power of impeachment is vested in Parliament. The
He is elected by an electoral college in accordance with impeachment process can be initiated in any House of
the system of proportional representation by means of Parliament. If one House supports the impeachment
a single transferable vote. with two-third majority then it moves to Second House.
656 Study Package for NTSE

Then the other House investigates the charges. If the Term and Office of Vice President
other House approveds the charges by two-third He is elected for a term of 5 years and continues until
majority again, the President is removed from his office. his successor comes.
Powers He may resign by addressing his resignation to the
President. He can be removed from his office by a
Power of President can be discussed under following
resolution of the Council of States passed by a majority
heads
of all the then members of the Council of the States and
l Administrative Power The President appoints Prime agreed by the House of People. The procedure for
Minister, Attorney General, Comptroller and Auditor
removal cannot be initiated in Lok Sabha.
General, Judges of the Supreme Court and High Court
of the States, Governor of State, members of the Election The list of both Presidents and the Vice Presidents are
Commission.
The Presidents of India
l Military Power He has the power to declare war and
peace but under the control of Parliament. 1. Dr Rajendra Prasad January 26, 1950 to May 13, 1962
2. Dr S Radhakrishnan May 13, 1962 to May 13, 1967
l Legislative Power The President can dissolve the Lok
3. Dr Zakir Hussain May 13, 1967 to May 3, 1969
Sabha at the recommendation of Prime Minister he
nominates 12 Members of Rajya Sabha and may 4. Varahgiri Venkata May 3, 1969 to July 20, 1969
nominate two members of Anglo-Indian Community in Giri (acting)
Lok Sabha. Bill passed by the Parliament must receive 5. Justice M July 20, 1969 to August 24, 1969
President’s as approval before it becomes an Act. Hidayatullah (acting)
6. V V Giri August 24, 1969 to August 24,
l Emergency Power Article 352 (National Emergency)
1974
On the grounds of war, external aggression or armed
7. Dr Fakhruddin Ali August 24, 1974 to February 11,
rebellion, the President can proclaim emergency after
Ahmed 1979
receiving the written recommendation of the cabinet. He
8. BD Jatti February 11, 1977 to July 25, 1977
also recommends the financial emergency if he thinks
(acting)
that the financial stability of the country is threatened.
9. Neelam Sanjiva July 25, 1977 to July 25, 1982
î Fundamental Rights can be suspended during emergency Reddy
except Article 20 and 21. July 25, 1982 to July 25, 1987
10. Giani Zail Singh
11. R Venkataraman July 25, 1987 to July 25, 1992
Vice President 12. Dr Shankar Dayal July 25, 1992 to July 25, 1997
Sharma
According to Article 63 there shall be a Vice President
13. KR Narayanan July 25, 1997 to July 25, 2002
of India. He is the ex-offico chairman of Rajya Sabha. 14. APJ Abdul Kalam July 25, 2002 to July 25, 2007
He is elected jointly by the Houses of the Parliament. 15. Ms Pratibha July 25, 2007 to 13 July 2012
The procedures for his election consists of a secret Devisingh
ballot, proportional representation and single Patil
16. Pranab Mukherjee July 25, 2012- July 25, 2017
transferable vote. An absolute majority of the votes
polled is also required for the election of the Vice 17. Ram Nath Kovind July 25, 2017-till date
President.
Vice-President of India
Being a Vice President he is entitled to get the salary of
1. Dr S Radhakrishnan 1952-1966
` 125000 per month.
2. Dr Zakir Hussain 1962-1967
He acts as the President of India if the office of the 3. Varahagiri Venkata Giri 1967-1969
President falls vacant due to his resignation, death, 4. Gopal Swarup Pathak 1969-1974
impeachment, illness or absence from the country. The 5. BD Jatti 1974-1979
vice President holds the office till a new President is 6. Justice Mohammed Hidayatullah 1979-1984
elected. 7. R Venkataraman 1984-1987
8. Dr Shankar Dayal Sharma 1987-1992
Qualification of the Vice President
9. KR Narayanan 1992-1997
l He must be a citizen of India.
10. Krishan Kant 1997-2002
l Must have completed the age of 35. 11. Bhairon Singh Shekhawat 2002-2007
l Qualified to be elected as a member of Rajya Sabha and 12. Mohammad Hamid Ansari 2007-2017
not hold any office of profit.
13. Venkaiah Naidu 2017-till date
Working of Institutions Organs of Government 657
Council of Ministers Parliament
Council of Ministers comes into existence as soon as the Parliament consists of the President, Council of States
Prime Ministers sworn in. The Prime Minister shall be (Rajya Sabha) and House of the People (Lok Sabha).
appointed by the President and other Ministers shall be
Functions of the Parliament
appointed by the President on the advice of the Prime
Minister. Most important function of Parliament is to make laws
for the benefit of the country. It makes laws on all
The Council of Ministers is a composite body which may
subjects of Union List, State List and Concurrent List.
contain three categories of ministers viz., Cabinet
(i) Union List Subject of national interest in which only
Ministers, Ministers of State, Deputy Ministers.
Union Government can make laws. It contains 99
(i) Cabinet Ministers Prime Minister of India appoints subjects.
the Cabinet Ministers (ii) State List It contains the matter of local importance
They are assisted by minister of state and deputy on which only the State Government can make laws.
minister. It contains 61 subjects.
(ii) Ministers of State A minister of state may be (iii) Concurrent List Powers of the Parliament on which
given independent charge of a ministry but he both State and Union Government make laws.
generally assists a Cabinet Minister in running
the ministry. A Minister of State cannot attend the It puts controls on financial matter over the executive. It
meeting of cabinet on his own right but can attend enjoys the powers of removing certain high officials from
if invited. their office e.g.,- Impeachment of President, removal of
(iii) Deputy Minister He is the junior member of the Vice President, Judges of Supreme Court and High Court.
Council of Minister. He always serves under Cabinet
Minister or a minister of state.
Lok Sabha or the Lower House
Prime Minister and Council of Lok Sabha is popular house of the Parliament as its
members are directly elected by the people. It consists of
Ministers not more than 552 members. According to Constitution
Constitution lays down that Prime Minister shall be only 530 members are the representatives of the states
appointed by the President and other ministers shall be while 20 representatives belong to union territories and
appointed by the President on the advice of Prime not more than 2 member of the Anglo-Indian
Minister. Prime Minister is basically amongst the community nominated by the President.
member of majority party. The normal tenure of Lok Sabha is 5 years but it may
(i) As the heads of the government, the Prime Minister dissolve earlier.
has wide rangin of powers. He chairs cabinet
meetings. He coordinates the work of different Qualifications for the Membership of Lok Sabha
departments. His decisions are final in case l To be a citizen of India.
disagreement arise between departments.
l Not less than 25 years of age.
(ii) All ministers work under his leadership. The Prime
l To be registered voter in any of the parliamentary
Minister distributes and redistributes work to the
constituencies in India.
ministers.
(iii) The Prime Minister has the power to dismiss l Should not hold any office of profit.
ministers. When the PM quits, the entire ministery Special Power of the Lok Sabha Lok Sabha enjoys
quits. some special powers which are not available to Rajya
Sabha i.e.,
Legislature l Money bill and finance bill can be introduced in Lok
Sabha.
Legislature refers to that branch of the government l Confidence and no confidence motion can be initiated
which is concerned with enactment of laws. The and passed only in Lok Sabha.
Constitution of India has provided a legislature largely
on the pattern of Britain. The legislature is known in Rajya Sabha or Upper House of
various political systems with different names.
Parliament
The two Houses of Union Legislature in India are
It consists of two classes of members i.e., nominated
known as Lok Sabha and Rajya Sabha. While the two
members and representatives of the states and union
Houses of legislature in state are named as Legislative
territories. It consists of not more than 250 members. At
Assembly and Legislative Council.
present the total membership of Rajya Sabha is 245.
658 Study Package for NTSE

Rajya Sabha is a permanent body and not subject to Supreme Court


dissolution. One-third of its members retire in every 2
The Supreme Court is the apex court of India. It includes
years. Representatives of each state are elected by the
at present one chief justice and 33 other judges. The
elected member of the legislative Assembly of the state senior most judge of Supreme Court is appointed as the
in accordance with the system of Proportional Chief Justice of India.
representation by means of single transferable vote. Chief Justice of Supreme Court is appointed by President
Speaker and Deputy Speaker with the consultation of such Judges of Supreme Court
and High Court as he may think necessary. But in
Speaker is the presiding officer of the Lok Sabha. Both
appointing other judges the President shall consult the
the Speaker and Deputy Speaker are elected by the Chief Justice of India.
members of Lok Sabha, (STET) amongst themselves. If
No minimum age is prescribed for appointment as Judge
Speaker is absent the Deputy Speaker performs the
of the Supreme Court, nor any fixed period of office. Once
duties of the Speaker.
appointed, the Chief Justice and other Judges of the
l The Speaker and the Deputy Speaker remain in office so Supreme Court hold office till they attain the age of 65
long as they are member of the House. The Speaker or
years. However, a Judge may resign his office by writing
Deputy Speaker may resign his office by writing each
other. to the President. He may also be removed from this office
by the President on the ground of proved misbehaviour
l The Speaker enjoys some special powers i.e.,
or incapacity.
(i) When joint sitting of the Parliament takes place it is
the Speaker who presides over a joint meeting. The Judge of the Supreme Court must be
(ii) Whether a bill is a money bill or not, is certified only
l a citizen of India and
by the Speaker and his decision is final and binding.
l either a distinguished jurist in the opinion of the
(iii) If the Speaker is a member of any committee he is the
President, or has been a Judge of a High Court for 5
ex-offico chairman of such a committee.
years, or has been an Advocate of High Court (or two or
Chairman and Deputy Chairman of Rajya Sabha more such courts in successions) for at least 10 years.

The Vice President of India is the Ex-offico Chairman of


The High Court
Rajya Sabha. In his absence Deputy Chairman of Rajya
At the State level, the highest court is the High Court.
Sabha presides over the Rajya Sabha. The Deputy
The Parliament can establish a common High Court for
Chairman is elected by the members of Rajya Sabha
two or more States. At present, there are 25 High Courts
among themselves.
in the country. Among the Union Territories, only Delhi
Special Power of Rajya Sabha has a High Court of its own.
It includes l The High Court consists of Chief Justice and such other
Judges as the President of India may from time to time
l A resolution seeking the removal of the Vice President appoint.
can originate only in Rajya Sabha where the resolution
l A Judge of a High Court holds office until he attains the
must be passed by a majority of all the members of Rajya
age of 62 years. Every Judge—permanent, additional or
Sabha and agreed to by the Lok Sabha.
acting—may vacate his office earlier in any of the
l It can create one or more All India Services which is following ways.
supported by two-third members of Rajya Sabha.
(i) by resignation in writing addressed to the President
l It can pass resolution as it is necessary or enpedient in
(ii) by being appointed a Judge of the Supreme Court or
the natural interest that parliament should make a law
being transferred to any other High Court, by the
on a matter enumerated is state list. President.
(iii) by removal by the President on an address of both
Judiciary Houses of Parliament, on the ground of proved
misbehaviour or incapacity.
The judiciary has always been recognised as one of the
essential organs of the government of any state. All the The Judge of a High Court
courts at different levels in a country put together are l must be a citizen of India
called judiciary. Indian judiciary consists of a Supreme l must have held a judicial office for at least 10 years in the
territory of India; or must have been an advocate of a
Court for the entire nation, High courts in the States,
High Court or of two or more such courts in succession for
district courts and the courts at local level. The decision of at least 10 years, or a distinguished jurist in the opinion
Supreme Court is binding to all other courts of the country. of the president.
Working of Institutions Organs of Government 659
Subordinate Courts
The organisation of subordinate courts throughout the country is generally uniform. There are two types of law
courts in every district—civil and criminal courts. They function under the superintendence and control of the High
Court of the State. The Court of the District Judge is the highest Civil Court in a district. Besides the District Court
there are Courts of sub judges, Munsiff Courts and Courts of small causes. In a district, criminal cases are heard by
the Court of Sessions Judge. Below the District and the Sessions Courts are the Courts of First Class Magistrate

Target Exercise
Elementary Level
1. The eligible age to be appointed as the President of 8. Number of member nominated by President in Rajya
India is Sabha is
(a) 25 years (b) 30 years (a) 10 (b) 2 (c) 6 (d) 12
(c) 35 years (d) 21 years
9. Which among the following is the apex court of India?
2. Oath of the President is administered by (a) High Court (b) Supreme Court
(a) Vice President (c) District Court (d) None of these
(b) Lok Sabha Speaker
10. Judges of Supreme Court is appointed by
(c) Judge of Supreme Court
(d) Chief Justice of India (a) The retired Chief Justice of Supreme Court
(b) Lok Sabha Speaker
3. Who among the following is 13th President of India? (c) Vice President
(a) Pratibha Patil (d) President
(b) Sonia Gandhi
11. Judges of Supreme Court hold the office till the age of
(c) Man Mohan Singh
(d) Pranab Mukherjee (a) 62 (b) 65 (c) 60 (d) 64

4. How many members is presently in Rajya Sabha? 12. The Parliament of India consist of
(a) 545 (b) 238 (c) 345 (d) 245 (a) President (b) Rajya Sabha
(c) Lok Sabha (d) Vice President
5. The term of Rajya Sabha member was fixed by the
Select the correct answer using the codes given below
(a) President (b) Parliament
(c) Cabinet (d) Constitution (a) a, b and c (b) b and c
(c) b, c and d (d) a, b, c and d
6. Speaker of Lok Sabha sends his resignation to
13. The members of the Rajya Sabha are
(a) Prime Minister (b) Deputy Speaker
(c) President (d) Parliament Secretariat (a) elected indirectly
(b) mostly nominated
7. Who presides over the joint meeting of Parliament? (c) elected directly as well as indirectly
(a) Prime Minister (b) Vice President (d) elected by the members of the Legislative Assemblies and
(c) Lok Sabha Speaker (d) None of these Legislative Council of States

High Skill Questions


1. The Deputy Chairman of the Rajya Sabha is (c) 2 and 12 (d) None of the above
(a) nominated by the chairman 3. A joint meeting of Lok Sabha and Rajya Sabha is
(b) elected by elected members of the Rajya Sabha presided by
(c) nominated by the President
(a) Deputy Speaker of the Lok Sabha
(d) elected by the Rajya Sabha from amongst its members (b) President
2. How many members are nominated in Lok Sabha and (c) Vice President
Rajya Sabha respectively? (d) Speaker of the Lok Sabha
(a) 2 and 10 (b) 12 and 2
660 Study Package for NTSE
4. The annual financial statement is caused to be laid before 10. Which among the following is not the
both Houses of Parliament by qualification to be elected as President of India?
(a) President (b) Speaker (a) Completed the age of 35 years
(c) Vice President (d) Finance Minister (b) Qualified for election as member of the house of people
5. Which House is better placed with regard to control over (c) Must be citizen of India
the executive? (d) Qualified for election as a member of the Rajya Sabha
(a) Lok Sabha (b) Rajya Sabha 11. Match the following
(c) Both are equally placed
(d) It depends from which house the Prime Minister comes List I List II
6. A bill which certifies as money bill can be introduced in A. Presidents 1. Deputy Speaker
(a) Lok Sabha only
B. Lok Sabha Speaker 2. Vice President
(b) Rajya Sabha only
(c) Both in Lok Sabha and Rajya Sabha one by one C. Vice President 3. Lok Sabha Speaker
(d) None of the above D. Deputy Speaker 4. President
7. Name the Upper House of the National Legislature of Codes
Great Britain A B C D A B C D
(a) The House of Commons (b) The House of Lords (a) 1 4 2 3 (b) 2 1 4 3
(c) The Rajya Sabha (d) The Senate (c) 4 2 3 1 (d) 1 2 3 4
8. Who among the followings do not take part in election of 12. The lower house of the Indian Parliament is
President of India? called
(a) Nominated members of Lok Sabha and Rajya Sabha and
Legislative Assemblies of states (a) The Lok Sabha
(b) Elected members of both House of Parliament (b) The Rajya Sabha
(c) Elected members of Legislative Assembly (c) The Legislative Council
(d) Both ‘b' and ‘c’ (d) The Legislative Assembly

9. President of India is elected by 13. New High Court is created on which following
(a) elected member of Lok Sabha only state
(b) elected member of Rajya Sabha only (a) Tripura (b) Goa
(c) elected member of Lok Sabha and Rajya Sabha (c) Nagaland (d) Mizoram
(d) elected member of Lok Sabha, Rajya Sabha and State Assemblies

Answers
Elementary Level
1. (c) 2. (d) 3. (d) 4. (d) 5. (d) 6. (b) 7. (c) 8. (d) 9. (b) 10. (d)
11. (b) 12. (a) 13. (a)

High Skill Questions


1. (d) 2. (c) 3. (d) 4. (a) 5. (a) 6. (a) 7. (b) 8. (a) 9. (d) 10. (d)
11. (b) 12. (a) 13. (a)
Chapter

3
Democracy and Related
Phenomena
Democracy comes from Greek word Demokratic. In l Social division takes place when some social differences
Greek demos means people and krati means ‘Power’. overlaps with other differences. The difference between
the blacks and whites in USA becomes a social division
In the world scenario we have seen that different
where blacks tends to be poor, homeless and
countries of world struggle a lot to have a good discriminated. As in our country dalits tend to be poor
government like Chile, a ‘country in South America and landless.
where Salvador Allende (then President) over thrown
by military through conspiracy and violence. Thus, Social Decision into Politics
military dictatorship was established in Chile. Later on l Democracy involves competition among various political
in 2006 election, Michelle Bachelet was elected as parties. Their competition tends to divide any society.
President of Chile. Whenever democracy exists these divisions are reflected
In 1980’ Poland was ruled by polish United worker’s in politics.
party, was one of the communist parties that ruled in l Social divisions affect the system of voting in most of the
several countries of East Europe at that time. Later on countries people from one community tend to prefer the
candidate of their own community to others party.
with verge of time in 1990, Presidential election takes
place and people choose Walese as the President of
Factors Deciding the Outcome of Politics
Poland.
and Social Division
The government of Poland was supported and controlled
by the government of the Soviet Union. l It depends upon how people perceive their identity. It is
difficult when people see their identities in singular and
exclusive terms. It is easier if people see that their
Democracy and Diversity identities are multiple and are complementary with the
National identity.
Origin of Social Differences l It depends upon how political leaders raise the demands
Social differences are based on mostly on accident of of any community. If demands of any community are
within the Constitutional Framework then it is easy to
birth. Normally we don’t choose to belong to our
accommodate but if it is not, these could not be
community, we belong to it simply because we were born accommodated.
into it.
l It depends upon how the government reacts to demands
l All kinds of social differences are not based on accident of different groups. If rulers are willing to share power
of birth but some are based on our choices like some and accommodate the reasonable demands of minority
people are atheist i.e. they don’t believe in God, some community, social divisions become less threatening for
people choose more than one religion, some of us choose the country but if they supress demand then it leads to
different occupation of choice. agitation.
662 Study Package for NTSE

There are three kinds of social differences that can take Communalism
the form of inequalities. These are l Communalism happens when beliefs of one religion are
presented as superior to those of other religions.
Social Differences Based on Gender
l It happens when the demands of one religions group are
l While talking gender, it is not based on biology but on formed in opposition to another and when state power is
social expectation. We often see that main responsibility used to establish domination of one particular religions
of woman is house-hold work and bringing up children. group over the rest.
This leads to sexual division of labour. While men do l This manner of using religion in politics is known as
work outside the home. communal politics.
l These differences can be easily seen in politics, l Political mobilisation on religion lines is another form
participation in public affair etc. Women have minimal at communnalism. This involves the use of sacred
envolvement and it leads to agitation for equal rights. symbols, religions leaders, emotional appeal and plain
fear in order to bring the followers of one religion
Feminist Movement together in the political arena.
l Some times majority community controls the political
l More radical women’s movements aimed at equality in
power in a country.
personal and family life as well. These movements are
l Communalism takes its most ugly form in communal
called feminist movement.
violence, riots and massacre.
l In our country we follow patriarchal society hence,
women are still far behind men despite some Secular State
improvement. Woman face disadvantage, discrimination. There is no official religion for the Indian State. Unlike
In India, only 65% women are literate while men
the states of Buddhism in Sri Lanka that of Islam in
constitute nearly 82%. Parents prefer resources for their
Pakistan while Christanity in England on constitution
boy’s education rather than spending equally on their
sons and daughters. Almost in all the areas women are
does not give a special status to any religion.
paid less than men even both do exactly some work. Caste Inequalities
l In India less then 10% of women elected in Lok Sabha
In most societies occupation are passed on from one
while in State Assemblies less than 5% hold the seats,
one way to solve this problem is to make it legally
generation to another which led to hereditary
binding to have a fair representation of women in the occupational division. Member of the same caste group
elected bodies. There are 59 woman MPs in 15th form a social community, married within the caste group.
Loksasha. l Caste system based on exclusion of and discrimination
l Ferminist a man or a woman who belives in equal right against the out caste groups.
and opportunities for women and men. Castes and Caste System in modern India have under
l In Panchayat Raj one third of seats in Panchayats gone great changes with
and Municipalities are now reserved for women (i) Urbanisation
which led to the 10 lakh elected women (ii) Occupational Mobility
representatives in rural and urban bodies. A bill (iii) Breaking down of Caste Hierarchy
which demand the reservation of at least one third of Caste can also take various forms in politics like
seats in Lok Sabha and State Assemblies has been (i) To win election parties choose candidate to keep in
pending before the Parliament. mind the caste composition of electorate.
(ii) Political party tries to get support of specific caste by
Division Based on Religious Differences gain their sentiment.
l Religious diversity is widely spread in the world. Many (iii) When parties choose candidates in elections, they
countries along with India, have followers of different keep in mind the caste composition of the electorate.
religions. The religions differences are often expressed
in the field of politics. Popular Struggles and Movements to
l Human right group always demands the special Establish Democracy
safeguard to protect religious minorities who are victim The expansion of democracy is not a smooth and
of communal riots in a country. straight way. There are many countries of world who
l Women’s movements emphasises to make better family struggle a lot to get the status of democratic country.
laws where women get equitable benefit. Amongst them some get full fledge status of Democratic
All these instances involve a relationship between country, while some of them may be called as
religion and politics. Semi-democratic where government has formed but
controlling power is in the hand of king or queen.
Democracy and Related Phenomena 663
Movement for Democracy leader of this Andolan. This movement started with
specific issue of people displaced by creation of the Sardar
in Nepal Sarovar Dam on Narmada River.

Nepal witnessed a popular movement in April 2006, to Influence of Pressure Group and
restoring democracy. Nepal had won democracy in 1990
although the king remain the head of the state the real Movements in Politics
power was exercised by elected representatives.
They try to gain public support for their goals lead by
In Nepal king Birendra, accepted the absolute monarchy campaign, organising meeting. Filing petition they try to
to Constitutional monarchy. King Birendra was killed in influence India to giving more attention to their issues.
2001 after the death of Birendra, King Gyanendra
Pressure groups are organisation that attempt to
dissolved the Parliament.
influence government policies.
Later on in April 2006, movement started, where people
In some instances, the pressure groups are either
give their full support to all major political parties hence
formed or lead by the leaders of political parties or act as
lead to the formation of new government where Girija
extended arms political parties.
Prasad Koirala elected as the new Prime Minister.
Some times political parties grow out of movements.
The struggle of Nepali people is a source of inspiration
to democrats all over the world. Outcome of Democracy is one of major aspect which
shall be point out carefully because if any country
Bolivia Water War wants democratic form of government then the mass
Bolivia is a poor country in Latin America. In Bolivia of the people shall be benefited by the different
Government give up its control of municipal water policies of the government which led to the upliftment
supply multinational company Cochabamba the of society.
company increased the price of water by four times. In democracy people have sole right to choose their
By the effort of new alliance of labour, human rights, a representative and having indirectly control over it.
community leaders and through mass support, forced People of democratic country should be able to
the officials of MNC to flee the city and made participate in decision making that affects them all.
government concede to all the demands of protestors. l If we compare democratic and non-democratic
Again water supply was restored to the municipality of government then in democratic form people have right
old rates. This came to known as Bolivia’s War. to examine the process of decision making which termed
l Interest Group or Pressure Group Forming an as transparency which is often missing form non-
organisation and undertaking activities to promote their democratic government.
interest on their view points are called as interest group l With the change of time democracies may frustrate as
or pressure group. well as unsatisfy the demand of majority of its population.
It may seen that corruption prevail most of the major
Pressure group are organisation that attempt to
segment of democracy, at the same time most of the
influence government policies. These organisation are people think that non democracies are less corrupt.
formed when people having common occupation,
l If we talk about the economic growth and development
interest, opinion come together to achieve common in the perspective of outcome of democracy then it is
objective. expected to produce good governments but not fair to
l Sectional Interest Group and Public Interest expect that they would also produce development.
Group Interest group seek to promote the interest of a l If we look between 1950 to 2000 then we will found that
particular section or group of society e.g., Trade Union, democracies have slightly lower economic growth in
professional (lawyer, doctor, teacher). They termed comparisons to all dictatorship country. Slow rate of
sectional because they represent section of society. development may frustrate us but this alone cannot be
l Promotional Group or Public Interest Group It is a the reason to reject the democracy.
group of individuals who come together in order to l Countries like South Africa and Brazil approx 60% of the
benefit people beyond their membership in the group. national income taken away by top 20% of people. While
They promotes issue of general public concern other countries like Denmark and Hungary have better
e.g.,human rights, consumer right etc. record.
l Movement Group It is a group of people with a l Democracy stands much superior to any other form of
particular set of aims. In India Narmada Bachao Andolan government in promoting diguity and freedom of the
is a good example of this kind. Medha Patekar is chief individual.
Target Exercise
Elementary Level
1. The form of government in which representatives are 8. In Panchayati Raj seats reserved for women is
elected by people is called (a) 30% (b) One third
(a) Autocracy (b) Democracy (c) 33% (d) None of these
(c) Plutocracy (d) None of these
9. How much percent of seats demanded to be reserved
2. Which among the following became the first country in both Lok Sabha and State Assemblies in Women
in Africa to gain independence? Reservation bill?
(a) Ghana (b) Egypt (a) One third (b) Two third (c) One fourth (d) Only 15 %
(c) Zambia (d) Nigeria
10. Which among the following is not a cause behind the
3. Aung San Suu Kyi belongs to upgradation in caste system of India
(a) South Korea (b) North Korea (a) Urbanisation
(c) Myanmar (d) China (b) Occupational mobility
(c) Caste hierarchy
4. Which among the following is not a permanent
(d) None of the above
member of United Nation Security Council?
(a) Russia (b) UK (c) France (d) Germany 11. In India seats are reserved for women in
(a) Lok Sabha
5. How many numbers are there in the Security Council (b) State Legislative Assemblies
of the United Nation? (c) Cabinet
(a) 5 (b) 10 (c) 15 (d) 20 (d) Panchayati Raj Bodies
6. 10 non-permanent member of Security Council 12. Shift of population from rural areas to urban areas
elected for the term of termed as
(a) 5 years (b) 6 years (c) 3 years (d) 2 years (a) occupational mobility (b) urbanisation
7. The term patriarchal society means (c) migration (d) caste hierarchy
(a) When women are the decision makers and hold positions of 13. When was universal adult franchise granted in India
power and prestige (a) 1952 (b) 1947 (c) 1950 (d) 1958
(b) When men are the decision makers and hold position and
power and prestige 14. Whom among the following is the present Secretary
(c) Society where both man and woman having equal status in General of United Nation Organisation?
decision making (a) Kofi Annan (b) Ban Ki Moon
(d) None of the above (c) Trygveli (d) Uthant

High Skill Questions


1. Match the following 2. Match the following
List I List II List I List II
A. Homogeneous 1. A woman or a man who believes in A. Veto 1. A condition under which the freedom of
Society equal rights and opportunities for expression is taken away citizens have to
women and men. take prior permission from censor
B. Migrant 2. Those laws that deals with family authorities of government for making
related matters such as marriage, speech or publishing news
adoption etc. B. Martial law 2. The right of a person, party or nation to
C. Family Laws 3. Anybody who shifts from one region stop a certain decision or law
or country to another region within a C. Communist 3. When a military authority takes control of
country or to another country. State the normal administration of justice.
D. Feminist 4. A society that has similar kinds of D. Censorship 4. A state governed by communist party
people especially where there are no without allowing other parties to compete
significant ethnic differences. for power.
Codes Codes
A B C D A B C D A B C D A B C D
(a) 3 2 1 4 (b) 4 3 2 1 (a) 2 3 4 1 (b) 1 2 3 4
(c) 2 3 4 1 (d) 3 1 2 4
(c) 4 2 3 1 (d) 3 2 4 1
Democracy and Related Phenomena 665
3. Which of the following statements is/are true? Codes
A B C D
1. Promotes equality among peoples.
(a) 2 4 1 3
2. Improve the quality of decision making. (b) 2 1 4 3
3. Provide a method to resolve conflict. (c) 1 2 3 4
Select the correct answer using the codes given below (d) 4 3 2 1
(a) 1 and 2 (b) 2 and 3 (c) 1 and 3 (d) All of these 9. The green belt movement is related with which
4. Which of the following international organisation country?
maintaining peace and security among countries? (a) Zimbabwe
(a) Gerneral Assembly (b) International Monetary Fund (b) Kenya
(c) UN Security Council (d) None of these (c) Nigeria
(d) Sri Lanka
5. Democracy is a form of government in which
10. Match the following
1. rulers elected by the people take all the major
decision List I List II
2. election offer a choice and fair opportunity to the
people to change the current rulers A. Organisation that seek to 1. Movement
promote the enterest of a
3. the choice and opportunity is available to all the particular section or group
people on an equal basis
B. Organisation that mobilise 2. Public interest Group
Select the correct answer using the codes given below people with a view to win
(a) 1 and 2 (b) 2 and 3 (c) 1 and 3 (d) All of these political power
6. Which among the following countries has no official C. Organisation that seek to 3. Sectional interest Group
promote common interest
religion?
(a) Sri Lanka (b) Pakistan (c) England (d) India D. Struggled launched for the 4. Political Parties
resolution of a social problem
7. In dealing with social division which one of the with or without an organisa-
following statements is not correct about democracy? tional structure
(a) Democracy is the best way to accommodate social diversity
Codes
(b) Democracy always leads to disintegration of society on the
A B C D
basis of social division
(a) 2 4 3 1
(c) Due to political competition in a democracy, social division get
(b) 3 4 1 2
reflected in politics
(c) 2 3 4 1
(d) In a democracy it is possible for communities to voice their
(d) 3 4 2 1
grievances in a peaceful manner
11. Wangari Mathai is a famous
8. Match the following
(a) politician of Kenya
List I List II (b) leader of Kenya
(c) environmental and poltical activist of Kenya
A. Power shared among different 1. Community Government.
(d) None of the above
organs of government
B. Power shared among 2. Separation of Power 12. Who among was the leader of Chinese revolution?
governments of different (a) Huzintao
levels (b) Aung San Suu Kyi
C. Power shared by different 3. Coalition Government (c) Mao Tse-Tung
social groups (d) Zambegi
D. Power shared by two or more 4. Federal Government.
political parties

Answers
Elementary Level
1. (b) 2. (a) 3. (c) 4. (d) 5. (c) 6. (d) 7. (b) 8. (b) 9. (a) 10. (d)
11. (d) 12. (b) 13. (c) 14. (b)

High Skill Questions


1. (b) 2. (a) 3. (d) 4. (c) 5. (d) 6. (b) 7. (a) 8. (a) 9. (b) 10. (d)
11. (c) 12. (c)
Chapter

4
Electoral Process in India
and Political Parties
In a democracy it is not possible for people to govern Election in India
directly. If we talk about India on in perspective of India
because of its large size due to its geographical area and Lok Sabha and Vidhan Sabha (Assembly) election are
due to its population size it is quite easy and beneficial, held regularly after every five year. After the five years
if we work in a channel through elected people. the term of all the elected representatives comes
to an end. The Lok Sabha or Vidhan Sabha stands
There are more than hundred countries in the world in
‘dissolved’.
which election occurs to choose people's representative.
We also know that in most of the countries election General Election
occurs but they are not democratic.
Elections are held in all constituencies at the same time,
either on the same day or within a few days. This called
Election a general election.
Election is a process through which people can choose
By Election
their representative at regular interval and make a
change whenever it is required hence this mechanism is If any vacancy occurs due to death or resignation of a
called election. member, then election is held only for one constituency
to fill the vacancy.
Condition of a Democratic Election In India we follow area based system of representation
(i) Everyone should be able to choose their candidate our country divided into different areas for purposes of
and every one should have one vote and every vote elections, are called electoral constituencies.
have equal value. MP In Lok Sabha election if representative elected
(ii) Political parties and candidates should be free to from their Constituencies are called member of
contest elections and offer real choice to the voters. Parliament or an MP.
(iii) Election must be held regularly at regular interval.
MLA Each state is divided into a specific number of
Election are all about political competition and it is good
Assembly constituencies. In this case the elected
to have political competition because this is the way representative is called the Member of Legislative
that we can select our future leaders. Regular electoral Assembly or MLA.
competition good for both political parties and leaders.
They know that if they can raise issues that people want Nomination of Candidates
to be raised, their popularity and chances of victory will
There is no any restriction on any one to contest an
increase in the next elections but if they fail to satisfy
election. Every one who wishes to contest an election
the voters with their work then they will not able to win
has to fill a nomination form and deposit some security.
again.
Electoral Process in India and Political Parties 667
Recently Supreme Court of India declared that every Obstacles of Free Fair Election
candidate has to make a legal declaration giving full l Because of having criminal background on criminal
details of connection, candidates put others out of the electoral race
(i) Any serious criminal cases pending against him and to secure a ticket from major parties.
(ii) Details of assets and liabilities of the candidate and l Smaller parties and independent candidates suffer a
his/her family huge disadvantage compared to bigger parties.
(iii) Education qualification of the candidate l Some families tend to dominate political parties, tickets
This all information has to be made public which are distributed to relatives from these families.
provides opportunity to the voters to make their decision
on the basis of information provided by the candidate. Political Parties
A political party is a group of people wanting to hold and
Election Campaign maintain power within the government. Political parties
In our country such compaigns take place for a two week are one of the most visible institution in a democracy.
period between the announcement of the final list of l It will advertise a common commitment by its leaders
candidates and the date of polling. The candidates and its membership to a set of political, social, economic
contact their voters, through election meeting and and cultural values that distinguish it from other
political parties.
political parties mobilise their supporters to make
people aware of their coming plan. l It includes three components.

During this election campaign according to our election (i) the leader (ii) the active members (iii) the followers
law no party or candidate can Functions
(i) give bribe or threaten voters
(ii) appeal to vote in the name of caste or religion l They contest elections by putting up their selected
candidates. Candidates are either selected by member
Code of Conduct According to this no party can and supporters of a party or top leaders chase
(i) use any place of worship for election propaganda candidates for contesting election.
(ii) use government vehicles, aircrafts etc l Political parties offers different policies and programmers
Ballot Paper It is a sheet of paper in which name of and voters have the right to choose from them. They
candidate and his party name with symbol are listed but reduce a vast multitude opinion into a few basic positions
now a day EVM (Electronic Voting Machine) are used to which it supports. It is the ruling party that decides lines
and directions of ruling the government.
record votes.
l Political parties form and run governments and political
In our country election is conducted by an independent
executives from such party takes big policy decisions.
and powerful body termed as Election Commission.
l They play a decisive role in making laws for a country.
Election Commission l The losing political party plays the role of opposition to
the parties in power. They raise their voice on
In our country elections are conducted by an government failures and wrong policies.
independent and very powerful Election commission. It
l Political parties provide access to government machinery
consist of one chief election commissioner and two other and welfare scheme implemented by governments.
election commissioner.
Party System Existed in India
Term Appointment
In a democracy any one can form political party hence in
Appointed by president of India but once appointed, he India more than 750 parties are registered with the
is not answerable to the president or the government. election commission of India. But few amongst them
Powers effectively in the race to win.
l Conduct election from announcement to declaration of One Party System Countries where only one party is
results. allowed to control and run the government are called
one party system e.g., China, only communist party is
l It implement code of conduct and punishes any
candidates or party who violates it. allowed to rule.
l He can give certain guideline with regard to election and Two Party System It is a system where two major
every party or candidate has to follow it whether they political parties dominate voting in nearly all elections at
like it or not. every level of government and as a result nearly all elected
l On election duty government officers work under the offices are members of one of the two major parties.
control of EC and not the government e.g., UK and USA
668 Study Package for NTSE

l Multi Party System If is condition where more than (iii) BSP (Bahujan Samaj Party) It is the IInd largest
two political parties are in serious contention for power, party having 10 member in 16th Lok Sabha election.
alone or in coalition. Thus in India we have multiparty l Establish by Kanshi Ram in 1984
system.
l Succeeded by Mayawati in 2003, symbol is Elephant
l Alliance When several parties in a multi-party system (iv) CPIM (Communist Party of India Marxist) It
join hands for the purpose of contesting elections and emerged out of a dicision within the communist party
winning power it is called an alliance. of India.
l Establish in 1964
Types of Political Parties in India
l It has strong presence in the States of Kerala, West
India has a multi party system with predominance of Bangal, and Tripura. As of 2011, CPI (M) is leading
small regional parties. India have two kinds of political the State government in Tripura.
parties l It followes the principles of Marxist philosophy.
(i) National Party Party secures at least 6% of total (v) CPI (Communist Party of India)
votes in Lok Sabha election or Assembly election in l It believes in Marxism-Leninism and supports
four statar and winus at least four seats in Lok Sabha socialism, secularism and democracy and opposes
termed as national party. imperialism and communalism.
(ii) State Party Party secures at least 6% of total votes
l Founded in 1925
in an election of the Legislative Assembly of state
and wins at least two seats is recognised as a state l It has significant presence in the States of Kerala,
party. Over the last three decade the number and West Bengal, Punjab, Andhra Pradesh and Tamil
strength of these parties has expended. Nadu.
Now-a-days no singal national party is able to secure (vi) NCP (National Congress Party)
on its own a majority in Lok Sabha. Hence it led to l After Split in the Congress Party, it formed in 1999
form alliances with state parties. by Sharad Pawar, PA Sangma and Tariq Anwar.
Some of the example of state parties in different
l Its significant presence in Maharashtra, Meghalaya,
stater are as
Manipur and Assam.
Major Political Parties in India (vii) AITC (All India Trinamool Congress)
l Its formed in 1998 by its founder Mamta Banerjee.
(i) INC (Indian National Congress) One of the oldest
party of world founded in 1885. INC emerges as the l Its most active in West Bengal.
largest party in 2009 election and secure 206 seat in l It is the fifth largest party in 16th Loksabha
15th Lok Sabha. Election.
(ii) BJP (Bhartiya Janta Party) It is the second largest (viii) NPP (National People’s Perty)
party in terms of representation in Parliament by
l It is founded by P.A. Sangma after his expulsion
securing 116 seat
from the NCP in July 2012.
l establish : in 1980
l It is most concentrated in the state of Meghalya.
l BJP headed the national govt. along with NDA
l It is the first political party from North-East State.
from 1998 to 2004 while defeated by INC in 2004
election

Target Exercise
Elementary Level
1. Election held after every 5 years is termed as 4. The salary and allowances of Chief Election
(a) by election (b) general election Commission and other commissioner of India is
(c) electoral constituencies (d) None of these changed upon
2. Chief Election Commission of India appointed by (a) Contingency Fund of India
(b) Consolidated Fund of India
(a) Prime Minister (b) Vice President
(c) According to desire of President
(c) President (d) Lok Sabha Speaker
(d) Fixed by the Parliament
3. The Election Commission of India is a
5. Chief Election Commissioner removed from his office
(a) advisory body (b) non permanent body
(c) independent body in same manner and on the grounds as judges of
(d) permanent and an independent body (a) High Court (b) Supreme Court
(c) Civil Court (d) None of these
Electoral Process in India and Political Parties 669
6. Who among the following revise electoral rolls and 11. Code of conduct means
register of all eligible voters (a) A guidelines given to Chief Election Commission of India
(a) Prime Minister under which he has to work
(b) President (b) A set of norms and guidelines to be followed by political
(c) Vice President parties and contesting candidate during election time.
(d) Election Commission of India (c) Both a and b
(d) None of the above
7. At the State level, the Election Commission is
assisted by 12. Which among the following is not a challenge to
(a) District Magistrate political parties in democratic country
(b) Chief Minister (a) Lack of internal democracy
(c) Governor
(b) Dynastic succession
(d) Chief Electoral Officer
(c) Money and muscle power
8. Chief electoral Officer is appointed by (d) None of these
(a) District Collector of respective district 13. Who among the following is the founder of Bahujan
(b) CM of respective state
Samaj Party?
(c) Chief Election Commission
(d) Chief Election Commission in consultation with state (a) Sonu Mahraj
government (b) Moti Lal Nehru
(c) BR Ambedkar
9. On which day voters day celebrated (d) Kanshi Ram
(a) 26 January
14. A signed document submitted to an officer where a
(b) 24 January
person makes a sworn statement regarding her
(c) 25 January
personal information is called
(d) 21 January
(a) Rigging (b) Affidavit
10. The term rigging means (c) Formal paper (d) None of these
(a) Fraud and malpractices indulged by a party on candidate to
increase its votes 15. Changing party allegiance from the party on which a
(b) It is a system through which any person having criminal person got elected to a different party called
cases can contest election (a) Defection
(c) Under this any political party without declaring its assets (b) Rigging
contest election (c) Affidavit
(d) None of the above (d) None of these

High Skill Questions


1. Match the following 2. Match the following
List I List II List I List II

A. A person who is strongly committed 1. Rigging A. INC 1. 1980


to a party, group of function B. BJP 2. 1984
B. Changing party allegiance from 2. Code of C. BSP (Bahujan Samajwadi Party) 3. 1885
the party on which a person Conduct D. CPI 4. 1925
got elected to a different party
C. Frand and Malpractices indulged 3. Defection Codes
by a party or candidate to increase A B C D
its votes (a) 3 2 4 1 (b) 3 1 2 4
(c) 4 3 2 1 (d) 2 3 4 1
D. A set of norms and guidelines to be 4. Partisans
followed by political parties and 3. What makes an election democratic?
contesting candidates during (a) Every one should have one vote and every vote have
election time
equal value
Codes (b) Election must be held after every 5 years
A B C D (c) Candidate preferred by the people should get elected
(a) 4 3 1 2
(d) Free and fair election
(b) 1 2 3 4
(c) 4 3 2 1 Select the correct answer using the codes given below
(d) 3 2 4 1 (a) a and b (b) a, b and c
(c) a and c (d) All of these
670 Study Package for NTSE

4. Which among the following is true 6. Which among the following statement is correct in
(a) Supreme Court made if mandatory to file an affidavit giving context of election commissioner of India?
details of property those who are contesting election (a) Election commissioner holds the office until he attain the
(b) The constitution do not restricts to MLA's and MP's from age of 65
changing parties in order to become minister or for cash
(b) He is appointed by chief justice of India
reward
(c) Salary and allowances charged upon Consolidated Fund of
(c) Both a and b
India.
(d) Only a
(d) Both a and c
5. Which among the following is condition of a
democratic election? 7. By election occurs when
(a) if any vacancy occurs due to foreign tour of leader/minister
(a) Politicle parties and candidates should be free to contest
election (b) if any vacancy occurs due to death or resignation
(b) Election must be held regularly at every few year (c) Both a and b
(c) Every one should be able to choose their candidate (d) Only b
(d) All of the above

Answers
Elementary Level
1. (b) 2. (c) 3. (d) 4. (b) 5. (a) 6. (d) 7. (d) 8. (d) 9. (c) 10. (a)
11. (b) 12. (d) 13. (d) 14. (b) 15. (a)

High Skill Questions


1. (a) (b) (d) (a) (a) (d) (d)
Chapter

5
Power Sharing and
Federalism
Power Sharing l Many powers of the Central Government given to State
Government of two region hence State Governments are
not subordinate to the Central Government.
The idea of power sharing has emerged in opposition to
the notions of undivided political power. For a long time l Brussels has a separate government in which both
communities enjoy equal representation.
it was believed that all power of government must
l Apart from the central and state government. There is a
reside in one person or group of persons located at one
third kind of Government i.e., community government
place. which is elected by people belonging to one language
community- Dutch, French and German—speaking.
Belgium and Sri Lanka î European Union Headquarters is in Brussels.
l Belgium is a small country in Europe. It has borders
with France, the Netherland, Germany and Accommodation in Sri Lanka
Luxembourg.
l Sri Lanka is an island nation, just a few km off the
l The ethnic composition of this small country is very Southern coast of Tamil Nadu.
complex. 59% of countries population lives in Flemish
l It has a diverse population in which 74% speaks Sinhala
region and speaks Dutch language. while 18% enjoy to speak Tamil.
l Another 40% of people live in Wallonia region and speak l Among Tamils there are two-subgroup
French and remaining 1% speak German.
(i) Sri Lankan Tamil Tamil natives of the country are
l Capital city Brussels where 80% people speak French called Sri Lankan Tamil.
and 20% are Dutch speaking. (ii) Indian Tamil Whose fore fathers came from India as
l The minority French speaking community was plantation workers during colonial period are called
relatively rich and powerful which led to dissatisfaction Indian Tamil.
Most of the Sinhala speaking are Buddhist while
between Dutch speaking and French speaking because
most of the Tamils are Hindus or Muslims.
Dutch get benefit of economic development much later.
l There are about 7% Christian who are both Tamil and
Sinhala.
Accommodation in Belgium
l Due to vast majority, Sinhala speakers dominance over
l To lessen the conflict between Dutch speaking and government by virtue of their majority. Which, led to the
French speaking the Belgium leader work out an series of majoriterian measures to establish Sinhala
arrangement that would enable everyone to live supremacy.
together within the same country. l In 1956, Sinhala adopted as official language, thus

l Constitution make an arrangement of equal disregarding Tamil.


representation of both Dutch and French speaking in l The Government policies which always focus on Sinhala
Central Government. Thus, no single community can group get beneficial more than Tamil led to the increased
make decision unilaterally. the feeling of alienation among Sri Lankan Tamils.
672 Study Package for NTSE

l As a result relations between Sinhala and Tamil In federal system the Central Government cannot order
communities strained over time. the State Government to do something. State
l The Sri Lankan Tamils launched parties and struggles Government has its own power which is not answerable
for the recognition of Tamil as an official language, to the Central Government. Both these government are
equality of opportunities. In securing education and jobs
separately answerable to the people.
but their demands were repeatedly denied.
l In 1980, several political parties demanding Tamil Eelam Key Features of Federalism
(State) in Northern and Eastern part of Sri Lanka.
l Distrust between two communities turns into Following are the key features of federalism
widesprede conflict which turned into a civil war. As a l There are two or more levels of Government.
result many people of both communities have been l Different tiers of government govern the same citizens,
killed and lost their livelihoods. but each tier has its own jurisdiction in specific matters.
The fundamental provision of Constitution can’t be
Benefits of Power Sharing l
unilaterally changed by one level of government.
Power sharing is good because it reduce the possibility l If any disputes arises between different levels then court
of conflict between social groups. has the power to interpret.
It is good to ensure the stability of political order. l The federal system safeguard as well as promote unity of
country.
Forms of Power Sharing l Sources of revenue for each level of government are
In modern democracies power sharing take many forms clearly specified to ensure its financial autonomy.
(i) Powered is shared among different organs of government, There are two kinds of route through which federation
such a legislature, executive, judiciary. Such a separation have been formed. The first route involves independent
ensures that none of the organs can exercise unlimited states coming together to form a bigger unit e.g., USA,
power. Each organs check the others and this
Switzerland and Australia.
arrangement is called a system of check and balances.
(ii) It can be shared among government at different levels
What makes India a Federal Country
such a general government for the entire country is
usually called federal government and at the regional or The Constitution declared India as Union of States
provincial level they are termed to different names in although it didnot use the word federation, the Indian
India, we call it as State Government. Same principles Union based on the principles of federalism.
can be extended to levels of government lower than State Constitution originally provided for a two tier system of
Government i.e. municipality and panchayat. government i.e., Central Government and State
(iii) Power may be shared among different social groups such Government.
as religious and linguistic groups. e.g., community l Constitution provided them a separate jurisdiction to
government in Belgium. Central Government, State Government and Panchayat.
(iv) Power sharing arrangements can also be seen in the l All states in India are not having not identical powers
way political parties, pressure groups and movements like Jammu and Kashmir as it enjoy special status.
control or influence those in power.
l There are some units of Indian Union which holds least
Some countries provide constitutional and legal powers like Chandigarh, Lakshadweep, Delhi are the
arrangement for socially weaker section and women are union territories. These do not enjoy power of state.
represented in the legislature and administration to l On the provision of power sharing there is need of two
give space in government and administration to diverse third majority passed by each house of Parliament then
social group who otherwise would feel alienated from it has to be ratified by the legislatures of at least half of
government. the states.
l In case any dispute arises on division of powers High
Court and Supreme Court make a decision.
Fedralism
It is a system in which the power is divided between the How Federalism Practiced
central authority and various constituent units of the The real success of federalism in India can be attributed
country. A federation consist two forms of government to the nature of democratic polities in our country. It
i.e, one is government of entire country and other ensured that the sprit of federalism respect for diversity
governments of states that look after day-to-day and desire for living together shared ideal in our
administering of their state or Province Both level of country. Some of the ways in that the sprit of federalism
government enjoy independent power. which this happened
Power Sharing and Federalism 673
Linguistic State Decentralisation in India
Creation of linguistic states was the first and major test
for democratic politics in our country. Demand of state on When power is taken from Central and State
linguistic basis, made the country more united. government and given to local Government, it is called
Language Policy decentralisation. A major step towards decentralisation
was taken in 1992. Under 73rd Amendment Act of 1992
A Second test for Indian federation is language policy.
Constitution give status to the Panchayati Raj
Our Constitution did not give status of national institution.
language to any one language.
l Rural local government is popularly known as
l Hindi was identified as official language But hindi is the Panchayati Raj
mother tongue of only about 40 % of indians.
l Each village, or group of villages in some state has a
l Apart from Hindi there are 22 language recognised as Gram Panchayat. In this, several ward members often
scheduled languages by Constitution. called Panch and President as Sarpanch.
l According to the Constitution, the use of english for l The Panchayat work under Gram Sabha.
official purposes was to stop in 1965. However, many
l A few Gram Panchayats are grouped together to form
non-Hindi speaking states demanded that the use of
Panchayat Samiti or block or mandal. The members of
english continue.
Panchayat Samiti were elected by all Panchayat
members in that area.
Centre-State Relation l All the Panchayat Samities on mandals in a district
Restructuring the centre state-relation is one more way together constitute Zilla Parishad.
in which federalism has been strengthened in practice.
l The Constitutional arrangements for sharing power Municipality
work in reality depends to a large extent on how the l Similarly local government bodies exist for urban areas as well as it set
ruling parties and leaders follow these arrangements. up in towns.
l
Municipalities and municipal corporations are controlled by elected
l All this changed significantly after 1990.
bodies consisting of peoples’s representatives.
l This was the era of the coalition Governments at the l
Municipal chairperson is the poltical head of the muncipality,
center. generally termed as Mayor.
l This led to a new culture of power sharing and respect
for the autonomy of State Government.

Target Exercise
Elementary Level
1. Sinhala is an official language of 6. Rural local government is popularly known as
(a) Belgium (b) Pakistan (c) Nepal (d) Sri Lanka (a) municipality (b) panchayati Raj
(c) local Government (d) rural government
2. Headquarters of European Union is in
(a) New York (b) Sri Lanka 7. ‘Panch and Sarpanch’ is important part of
(c) Brussels (d) None of these (a) municipality (b) urban government
(c) rural government (d) panchayati raj
3. Violent conflict between opposing groups within a
country that becomes so intense that it appears like a 8. Which among the following states get special status
war called and having its own constitution
(a) World War (b) Regional War (a) Delhi (b) Chandigarh
(c) Civil War (d) State War (c) Shimla (d) Jammu and Kashmir
9. English for official purpose was stop in
4. Dutch speaking community belongs to
(a) 1948 (b) 1960 (c) 1965 (d) 1970
(a) Sri Lanka (b) Brussels
(c) Nepal (d) None of these 10. How many language officially incorporated in Indian
Constitution?
5. When power is taken away from Central and State
(a) 18 (b) 20 (c) 22 (d) 24
Government and given to local government, is called
(a) decentralisation (b) coalition 11. Sri Lanka emerged as an independent country in
(c) centralisation (d) None of these (a) 1946 (b) 1947 (c) 1948 (d) 1949
674 Study Package for NTSE

High Skill Questions


1. Power sharing is desirable because 6. Consider the following statements
(a) it reduces the conflict between social groups 1. Panchayat works under the overall supervision of
(b) if ensure the stability of political order the Gram Sabha
(c) Only a
(d) Both ‘a’ and ‘b’
2. Gram Sabha meet atleast twice or thrice in a year
3. All the Panchayat Samiti or Mandals in a district
2. Community government is elected by together constitutes Zilla Parishad
(a) people belonging to one language community
4. Few Gram Panchayats are grouped together to form
(b) people belonging to more than one language community
called Panchayat Samiti or Block Mandal
(c) there is no any community government in the world
(d) None of the above Which of the above statements are correct?
(a) 1 and 2 (b) 1 and 4 (c) 1, 2 and 3 (d) All of these
3. In the context of Belgium which one of the following is
true? 7. Distinguishing feature of a federal Government is
(a) Number of Dutch and French speaking minister be equal in (a) national government gives some powers to the provincial
strength in Central Government government
(b) Many power of government given to state (b) power is distributed among the legislature, executive and
(c) In Brussels 80% people speak French judiciary
(d) All of the above (c) elected officials exercise supreme power in government
4. According to which Act Sinhala recognise as the only (d) government power is divided between different levels of
official language of Sri Lanka? government
(a) Act of 1948 8. Match the following
(b) Act of 1949
(c) Act of 1956 List I List II
(d) Act of 1950 A. State Government 1. Union List
5. Consider the following statements about power B. Central Government 2. State List
sharing arrangement in Belgium and Sri Lanka. C. Central and State Government 3. Concurrent List
1. Iin Belgium, the Dutch speaking majority people tried
Codes
to impose their domination on the minority French A B C A B C
speaking community (a) 3 2 1 (b) 2 1 3
2. In Sri Lanka the policy, of government generally (b) 3 1 2 (d) 1 2 3
benefit the dominance of Sinhala speaking majority 9. Match List I with List II and select correct answer
3. In Sri Lanka, Tamil demanded federal arrangement using the codes given below
of power sharing
List I List II
4. The transformation of Belgium from unitary
A. State 1. Prime Minister
government to a federal one prevented a possible
B. Union of India 2. Mayor
division of the country on linguistic line
C. Gram Panchayat 3. Governor
Which of the statements given above are correct?
D. Municipal Corporation 4. Sarpanch
(a) 2, 3 and 4
(b) 1, 2 and 4 Codes
(c) 3 and 4 A B C D A B C D
(d) All of the above (a) 1 2 3 4 (b) 4 3 1 2
(c) 3 1 4 2 (d) 2 3 1 4

Answers
Elementary Level
1. (d) 2. (c) 3. (c) 4. (b) 5. (a) 6. (b) 7. (d) 8. (d) 9. (c) 10. (c) 11. (c)

High Skill Questions


1. (d) 2. (a) 3. (d) 4. (c) 5. (d) 6. (d) 7. (d) 8. (b) 9. (c)
Part-VIII : Economics
Chapter

1
Agriculture and Allied
Activities
Farming-Base of Village Economy l The fourth requirement is the knowledge and
enterprise to be able to put together land, labour and
Farming is the main activity in the rural areas and physical capital and produce an output either to use for
villages. Whereas several other activities such as dairy, themselves or to sell in the market. The fourth
transport, small scale manufacturing etc are carried out requirement can be termed as Human Capital.
on a limited scale. These production activities need
various types of resources-natural resources, man made Role of Electricity and Green Revolution
items, human effort, money etc. The availability of electricity has transformed the
system of irrigation from drawing the water from wells
Factors Required For Production and irrigate small fields to electric-run tube wells which
The aim of production is to produce the goods and irrigate much larger areas of land more effectively.
services than the people required. There are four l The large use of electric-run tube wells facilitates the
requirements for production of goods and services. farmers to adopt multiple cropping pattern and results
l The first requirement is Land and other natural in increasing the production.
resources such as water, forests and minerals. l Farmers of Punjab, Haryana, Western Uttar Pradesh
l The second requirement is Labour. i. e., the people who were the first to try out the modern method green
do the work some production activities require highly revolution in India. In the field of wheat production.
educated worker to perform the necessary task. Other l In many areas, the green revolution is responsible for
activities require workers who can do mannual work. the loss of soil fertility due to increased use of chemical
Each worker is providing the labour necessary for fertilizers, reduction in the level of ground water and
production. cause many environmental hazards.
l The third requirement is Physical Capital, the variety l Chemical fertilizers provide minerals which dissolve in
of inputs required at every stage during production. The water and are immediately available to plants. But
physical capital consists of fixed capital and working these may not be retained in the soil for long.
capital.
(i) Fixed Capital Tools, machinery, and building are Landless Labourers
termed as fixed capital as these can be used for many l The main cause of the farmers poor economic conditions
years.
is the fragmentation of land the division of one plot of
(ii) Working Capital Raw material and money in land among male members of the family. Cultivation of
hand termed as working capital. Some money is such plots does not bring adequate income to the
always required during production to make farmer’s family which leads to the creation of landless
payments and buy other necessary items.
labourers in the large number.
676 Study Package for NTSE

l These landless labourers which in the fields of farmers of grass and the Jowar and Bajra that grows during the
on the daily wages, work must regularly look for work. rainy season. The milk is sold in the nearby market
Due to heavy competition for work among the labourers, place or to far away towns and cities.
they agree to work on lower wages.
l The minimum floor level wages is ` 150 fixed by the Small Scale Manufacturing
Government of India
Another from of non-farm activity is small scale
manufacturing which involves very simple production
Capital-Need and Mobilisation methods and are done on a small scale keeping in view
l The modern farming methods require a great deal of the needs of the people of village. They are carried out
of capital, now the farmer require more money than mostly at home or in the fields with the help of family
before. labour rarely are labourer hired.
l Most of the small farmers have to borrow money to
arrange for the capital. They borrow from large farmers The Shopkeepers
or moneylenders or the traders who supply various
inputs for cultivation. The rate of interest on such loans is The traders of village are shopkeeper who buy various
very high. They are put to great distress to repay the loan. goods from wholesale markets in the cities and sell them
l In contrast to the small farmers, the medium and large in the village. A wide range of items such as grossery -
farmers have their own saving from farming. They are Sugar, Tea, Oil, Rice, Wheat etc.
thus able to arrange for the capital needed.
Transport A Fast Growing Sector
Non-Farm Activities There are variety of vehicles on the road connecting
villages to cities e.g., ricksha, tonga, jeep, tractor and
As agricultural is a seasonal work and for earning extra
money as well as earning throughout the year. People of people driving the traditional bullock cart are people in
villages engaged in non farm activities such as diary, the transport services. They carry people and goods
transport, small scale manufacturing shops etc. from one place to another and in return get paid for it.
The number of people involved in transport has grown
Dairy over the last several years.
Dairy is a common activity in many families of the
villages people feed their buffalos/cows on various kinds

Target Exercise
Elementary Level
1. Which of the following is the main activity of the 5. Tools, machines, buildings can be used in production
people in village or rural areas? over many years. All of these are known as which
(a) Agricultural (b) Dairy capital ?
(c) Small manufacturing (d) None of these (a) Physical capital (b) Fixed capital
2. Apart from the agriculture the people of village (c) Working capital (d) None of these
engaged in which of the following activities 6. Raw materials and money in hand are form of which
economically? capital?
(a) Transport (b) Dairy (a) Fixed capital (b) Physical capital
(c) Small manufacturing (d) All of these (c) Working capital (d) None of these
3. Which of following resources required for production 7. The standard unit of measure in land is ?
activities in village or rural area? (a) Hectare (b) Bigha
(a) Natural resources (b) Money (c) Guintha (d) Square
(c) Human efforts (d) All of these 8. Which of the following is not an Kharif crop?
4. Which of the following is/are required for the (a) Rice
production of goods and services in the village? (b) Jowar
(a) Land (b) Labour (c) Bajra
(c) Physical capital (d) All of these (d) Wheat
Agriculture and ALlied Activities 677
9. Which of the following is not an Rabi Crop ? 10. Multiple cropping refers to?
(a) Wheat (a) To grow more than one crop on a piece of land during the
(b) Barley year
(c) Rapeseed (b) To grow different crops on a piece of land at the same time
(c) To grow different crops on a different piece of land
(d) Potato
(d) None of the above

High Skill Questions


1. Which of the following factor is most important in 7. The use of the chemical fertilizers or consumption of
production of goods and services and able to put chemical fertilizers is highest in which of the
together land, labour and capital? following states in the country
(a) Physical capital (b) Knowledge and enterprise (a) Punjab (b) Haryana
(c) Working capital (d) None of these (c) Uttar Pradesh (d) Maharashtra
2. Which of the following is/are true about agricultural 8. The cost of cultivation is rising very fast. Which of the
activities and production in rural area of village ? following factors are responsible for it?
(a) Farmers are able to grow multiple crops due to well (a) The use of chemical fertilizers and pesticides
developed system of irrigation (b) Increasing cost of electricity
(b) Electricity has transformed the system of irrigation (c) Rising price of agricultural implements
(c) Green revolution made it possible to produce far larger (d) All of the above
quantities of food grains from the same piece of land 9. Match the following
(d) All of the above are true
List I List II
3. Which of the following is the feature of green
revolution? A. Land 1. Knowledge and enterprise
(a) Use of high yielding varieties of seeds B. Fixed capital 2. Raw material and money in hand
C. Working capital 3. Tools, machinery, building
(b) Plenty of water for irrigation
D. Human capital 4. Natural resources such as water,
(c) Use of chemical fertilizers and pesticides
forests, minerals
(d) All of the above
Codes
4. Farmers adopt multiple cropping system as it?
A B C D A B C D
(a) Increase the income
(b) Keep the farmers employed throughout the year (a) 4 3 2 1 (b) 1 2 3 4
(c) It maintains the fertility of soil (c) 4 3 1 2 (d) 2 1 4 3
(d) None of the above 10. Match the following
5. Second green revolution is related to the production
List I List II
of which crop?
(a) Wheat (b) Rice A. Green revolution 1. Dairy Products
(c) Oilseeds (d) Sugarcane B. Non-farm activity 2. Natural resources, money,
human effort
6. Which of the following is/are drawbacks of the green C. Production of goods 3. Dairy, transport, small
revolution? and services manufacture
(a) Loss of soil fertility due to increased use of chemical and D. White revolution 4. Have Seeds, chemical
fertilizers fertilizers and pesdicities
(b) Continuous use of ground water for tubewell irrigation has Codes
reduced the water level below the ground A B C D A B C D
(c) Chemical fertilizers and pesticides escapes from the soil
(a) 4 3 2 1 (b) 1 2 3 4
and pollute ground water, river and lakes
(c) 3 4 1 2 (d) 2 1 4 3
(d) All of the above

Answers
Elementary Level
1. (a) 2. (d) 3. (d) 4. (d) 5. (b) 6. (c) 7. (a) 8. (d) 9. (d) 10. (a)

High Skill Questions


1. (b) (d) (d) (c) (b) (d) (a) (d) (a) (a)
Chapter

2
People as Resource
Anything that can be used to satisfy a need is a Primary Sector Includes production of raw material or
resource. Water, electricity, a vegetable and textbook making direct use of nature resources.
have something in common. They have all been used by Primary sector includes agriculture, forestry, animal
you, so they have utility. A utility or usability is what husbandry, fishing, poultry farming, mining and
makes an object or substance a resource. quarrying.
l Things became resources only when they have a value.
Secondary Sector Includes producing processed and
All resources have some value. Value means worth.
Some resources have economic value, some do not. Metal
manufactored goods.
may have an economic value, a beautiful landscape may Manufacturing is included in the secondary sector.
not. But both are important and satisfy human needs. Tertiary Sector It includes producing services.
l Some resources can become economically valuable with Trade, transport, communication, banking, education,
time. Your grandmother’s home remedies have no
health, tourism, insurance etc are included in the
commercial value today. But, if they are patented and
sold by a medical firm tomorrow, they could became tertiary sector.
economically valuable. l Activities in these sectors result in the production of
goods and services. These activities add value to the
l Time and technology are two important factors that can
national income.
change substances into resources.
l These activities are called economic activities as they
l People can make the best use of nature to create more involve remuneration to any one who performs, i.e.,
resources when they have the knowledge, skill and the activity performed for pay or profit.
technology to do so. That is why human beings are a
special resource. People are human resources.
Quality of Population
l People as Resource is a way of referring to a
country’s working people in terms of their existing l The quality of population depends upon the literacy
productive skills and abilities. Education and health rate, health of a person indicated by life expectancy and
help in making people a valuable resource. Improving skill formation acquired by the people of the country.
the quality of people’s skill so that, they are able to l The quality of the population ultimately decides the
create more resources is known as Human Resource growth rate of the country. Illiterate and unhealthy
Development. population are a liability for the economy. Literate and
healthy population are an asset for the country.

Economic Activities : Primary, Education


Secondary and Tertiary Sectors Education in the initial years of life bears fruits in the
later years. It is an important input for the growth,
The various economic activities performed by the people
opens new horizons, and creates new aspirations and
have been classified into three main sectors
values.
People as Resource 679
l It enhances national income, cultural richness and the areas. In case of rural areas there is seasonal and
efficiency of governance. disguised unemployment. Urban areas have mostly
l Literacy is not only a right, it is also needed if the educated unemployment.
citizens are to perform their duties and enjoy their l Seasonal unemployment happens when people are
rights properly. not able to find jobs during some months of the year.
l ‘Sarva Siksha Abhiyan’ is a significant step towards People dependent upon agriculture usually face such
providing elementary education to all children in the kind of unemployment.
age group of six to fourteen years by the Government. It l Disguised unemployment occurs when people appear
is a time-bound initiative of the central Government and to be employed but are not fully employed. This usually
the community for achieving the goal of universalisation happens among family members engaged in agricultural
of elementary education. activity. The work requires the services of five people
l Mid-Day Meal This scheme has been implemented to but engages eight people. The contribution made by
encourage attendance and retention of children in three extra people does not add to the contribution made
schools and improve their nutritional status. by the five people. If three people are removed, the
productivity will not decline.
Health Effects of Unemployment
The health of a person helps him or her to realise his or l Unemployment leads to wastage of manpower
her potential and the ability to fight illness. An resources. It also tends to increase economic overload.
unhealthy person becomes a liability for an l It effects the quality of life of people.
organisation or the nation. l It has detrimental impact on the overall growth of an
Improvement in the health status of the population has economy.
been the priority of the country. Over the last five l It is an indicator of a depressed economy.
decades India has built up a vast health infrastructure l Agriculture is the most labour absorbing sector of the
and has developed manifold power required at Primary, economy. There has been a decline in dependence of
Secondary and Tertiary sector in Government as well as population on agriculture partly because of disguised
in the private sector. unemployment. Some of the surplus labour in
agriculture has moved to either the secondary or the
Unemployment tertiary sector.
Unemployment is said to exist when people who are l In the secondary sector, small scale manufacturing is
willing to work at the current wages cannot find jobs. In the most labour absorbing.
India, we have unemployment in rural and urban areas. l In the tertiary sector various new services are now
The nature of unemployment differs in rural and urban appearing like biotechnology, information technology etc.

Target Exercise
Elementary Level
1. When investment is made in the form of education, 3. The total value of goods and services produced in one
training and medical care on people this is known as year within the territory of a country is known as
(a) fixed capital (a) Gross Domestic Product (GDP)
(b) human capital (b) NNP
(c) working capital (c) Industrial Production
(d) None of the above (d) None of the above

2. People as a resource is a way of referring to a 4. The population of a country without food, education
country’s working people in terms of their existing and health facilities will be an
productive (a) asset (b) liability
(c) non capital asset (d) None of these
(a) skills and abilities
(b) working capital 5. Economic activities can be classified into
(c) national income (a) primary (b) secondary
(d) None of the above (c) tertiary (d) All of these
680 Study Package for NTSE

6. Which of the following is not a primary activity? 9. Activities involve remuneration to any one who
(a) Agriculture (b) Fishing (c) Mining (d) Tourism performs; activity performed for pay or profit is called
(a) market activity (b) national activity
7. Which of the following is not a tertiary activity?
(c) primary activity (d) None of these
(a) Banking (b) Education (c) Health (d) Forestry
10. Non-market activities are production for
8. Manufacturing is included in which activity?
(a) society (b) nation
(a) Primary (b) Secondary
(c) self-consumption (d) None of these
(c) Tertiary (d) None of these

High Skill Questions


1. Which of the following is an example of how the input (b) An unhealthy person became a liability for the organisation/
of greater knowledge in the form of improved nation
production technologies can rapidly increase (c) Health is indispensable for realising one’s well being
productivity? (d) None of the above
(a) Green revolution (b) Multiple cropping system
(c) Landless labourers (d) None of these 8. If any work requires the services of five people but
engages eight people, the contribution made by the
2. Development in which sector is an example of how three extra people does not add to the contribution
the importance of human capital has come to acquire made by five people. If these three were removed, the
a higher position than that of material, plant and productivity of work will not decline. This is an
machinery? example of which kind of unemployment?
(a) Service Sector (b) Information Technology Sector
(a) Seasonal (b) Disguised
(c) Agriculture (d) None of these
(c) Urban employment (d) None of these
3. Human capital is in one way superior to other
9. Match the following
resources like land and physical capital because
(a) human resource can make use of land and capital List I List II
(b) land and capital are more useful
(c) they have easy access A. Market Activities 1. Tertiary service
(d) None of the above B. Manufacturing 2. Rural areas
C. Disguised Employment 3. Secondary activities
4. Primary, secondary and tertiary activities result in the D. Banking 4. Activity performed for pay or
production of goods and services which add value to profit
(a) national income (b) national wealth Codes
(c) national capital (d) None of these
A B C D A B C D
5. The household work done by women is an example of (a) 4 3 2 1 (b) 4 1 3 2
which form of economic activity? (c) 1 2 3 4 (d) 2 1 4 3
(a) Market activity (b) Non-market
10. Match the following
(c) Consumpti

You might also like